You are on page 1of 843

100%

OiuU^i

India's No.1
Selling Book

w
F lo
ee
Fr
for
ur
s
ook
Yo

Chemistry
eB

Vol. II
our
ad
Y

1
Dr. S.N. Dhawan Dr. S.C. Kheterpal
Re
nd

^ According to new syllabus and rationalised content in the NCERT Book.


Fi

^ Multiple Choice Questions and Assertion-Reason Type Questions.


♦ Case-Based Very Short/Short Questions and Case-Based MCQs & Assertion-Reason Questions.
♦ Articlewise Conceptual Problems, Very Short Answer, Short Answer and Long Answer
Questions, all with answers.
^ NCERT Textbook Questions and Exercises with solutions.
♦ Questions of NCERT Exemplar Problems with solutions.
♦ Competition Focus for NEET, JEE (Main) and JEE (Advanced).

Pradeep Publications India


Wadeep fubUjcaUoni^frotuU^aniwunceAthe
launch (fail new Aeries:
"Pradeep's Stellar Series"
for the students seriously aspiring for Medical

w
or Engineering career. Pradeep's Stellar Series has

F lo
been scientifically designed and revised, separately
for MEET and JEE (Main), to allay the fear and

ee
Fr
pressure factors ofstudents. So, have a copy of
each book of the series and soothe your nerves

for
ur
for confident preparation.
s
Books in Pradeep's Stellar Series :
ook
Yo
MCQs in PHYSICS Vol. I for MEET
eB

MCQs in PHYSICS Vol. II for NEET


MCQs in CHEMISTRY Vol. I for NEET
r
ou
ad

MCQs in CHEMISTRY Vol. II for NEET


MCQs in BIOLOGY Vol. I for NEET
Y

MCQs in BIOLOGY Vol. II for NEET


Re
nd

MCQs in PHYSICS Vol. I for JEE (Main)


Fi

MCQs in PHYSICS Vol. II for JEE (Main)


MCQs in CHEMISTRY Vol. I for JEE (Main)
MCQs in CHEMISTRY Vol. II for JEE (Main)
MCQs in MATHEMATICS Vol. I for JEE (Main)
MCQs in MATHEMATICS Vol. II for JEE (Main)
Uie name tifat inspl^es^
con/fidence 'fh-% HTThefuviatwn !
Pradeep's Advanced Learning Series
Pradeep's Advanced Learning Physics Vol. I

w
(Mechanics-I)

F lo
Pradeep's Advanced Learning Physics Vol. II
(Mechanics-II)

ee
Pradeep's Advanced Learning Physics Vol. Ill

Fr
(Waves and Thermodynamics)
Pradeep's Advanced Learning Physics Vol. IV

for
(Electrostatics and Current Electricity)
ur
Pradeep's Advanced Learning Physics Vol. V
s
(Magnetism and EMI)
ook
Yo
Pradeep's Advanced Learning Physics Vol. VI
eB

(Optics and Modern Physics)

Pradeeps Advanced Learning Series


our
ad

has been exclusively designed for


Y

JEE (Advanced) Examination.


Re
nd

This series is a perfect source for exhaustive


Fi

preparation of JEE (Advanced) in a


systematic, meticulous and creative manner,
guiding students to solve problems with
speed and accuracy.
ow
New Course

Chemistry

e
Fl
re
Vol. II

F
ur
r
Strictly according to new syllabus & examination pattern of CBSE, ISCE and state boards of Punjab, Haryana,

fo
H.P., J & K, Odisha, Karnataka. Kerala, Nagaland, Manipur, Rajasthan, Jharkhand. Bihar, U.R,
Uttarakhand, M.P, Chhatisgarh, Assam, Gujarat, West Bengal, A.P. etc.
ks
for class XI ot-f2 stage.
Yo
oo
Dr. S.N. DHAWAN Dr. S.C. KHETERPAL
B

M.Sz.. Ph.a M.Sc., Ph.D.


Department of Chemistry Department of Chemistry
re

Kurukshetra University Institute of integrated and Honors Studies (IIHS)


KURUKSHETRA
Kurukshetra University
KURUKSHETRA
u
ad
Yo
d
Re
in
F

2023-2024

Pradeep Publications
JALANDHAR (INDIA)
Published by:
PRADEEP PUBLICATIONS
Head Office ; Circular Road, 0pp. Sitla Mandir, Jalandhar-144 008 (Pb.).
Tele: 0181-2283736, 2280164

w
e-mail: pradeeppublications@gmail.com
Branch Office: 95, Shyam Lai Marg, opp. Hindi Park, Daryaganj, New Delhi - 110 002,
Tele: 011-23273595, 23281566

F lo
ee
All rights reserved
® Publisher

Fr
All rights reserved. No part of this publication may be reproduced, stored in a retrieval system or transmitted in
any form or by any means, electronic, mechanical, photocopying, recording, scanning, web or otherwise
without the prior written permission of the publisher. Pradeep Publications has obtained all the information in
this book from the sources believed to be true and reliable. However, Pradeep Publications or its authors or

for
editors or illustrators don't take any responsibility for the absolute accuracy of any information published and
ur
the damages or loss suffered thereupon.

All disputes subject to Jalandhar Jurisdiction only.


ks
Yo
oo
eB

Edition : Fortieth

ISBN: 978-93-95649-25-4
r
ou
ad

PRICE : ? 1598.00 [For Vol. 1 & 11]


Y
Re
nd
Fi

Every genuine copy ofthis book has the hologram of PRADEEP PUBLICATIONS on its title cover which art work is shown
above. It contains the trade mark of Pradeep Publications. Above the trade mark, there are two circles. One circle moves
from centre to comer while the other circle moves in reverse direction, when the hologram is tilted. Readers are cautioned
not to buy any such book which does not bear the said hologram.
The book without this hologram maybe spurious one and anyone having thisbookcanbe prosecuted under law.

Printed at: New Verma Printers, Jalandhar.


w
Units
8. Redox Reactions
Pages
8/1—8/134
8.1. General Introduction
8/1

o
8.2. Oxidation-Reduction-Classical Concept 8/1
8.2.1. Oxidation

e
8.2.2. OxidisingAgentorOxidant

Fl
re
8.2.3. Reduction
8.2.4. Reducing Agent or Reductant

F
8.2.5, Oxidation-Reduction Reactions are Complementary
8.3. Oxidation and Reduction - Electron Transfer Concept 8/3
ur
8.3.1. Competitive Electron Transfer Reactions

r
8.4. Oxidation Number
8/8

fo
8.4.1. Rules for assigning Oxidation Numbers
8.4.2. Oxidationand Reductionin terms of OxidationNumber
8.4.3. ks
Oxidising and Reducing Agents in terms of Oxidation Number
Yo
8.4.4. Distinction between Valency and Oxidation Number
oo
8.4.5. Oxidation Number and Nomenclature
8.4.6. Types ofRedox Reactions
8.4.7. The Paradox of Fractional Oxidation States
eB

8.5. Balancing of Chemical Equations ofRedox Reactions 8/22


8.6. Redox Reactions as the basis for Titrations 8/30
8,6.1. Types ofRedox Titrations
ur

8.6.2. Stoichiometry ofRedox Reactions in Solutions


8.7. Redox Reactions and Electrode Processes
ad

8/37
Yo

8.7.1. Direct and Indirect Redox Reactions


8.7.2. Construction of an Electrochemical Cell
8.7.3. Salt Bridge and its Functions
d

8.7.4. Salient Features of an Electrochemical Cell


8.7.5. Representation of an Electrochemical Cell
Re
in

8.7.6. Electrode Potential


8.7.7.
Effect of Concentration of Metal Ions and Temperature on
F

the Electrode Potential - Standard Electrode Potential


Measurement of Standard Electrode Potential
8.7.9, Electromotive Series
8.7.10. Applications of the Electromotive Series
8.8. Applications ofRedox Reactions 8/50
8.9. Some Additional Useful Information About Redox Reactions 8/51
8.10. Typical Problems 8/52
Revision At A Glance 8/54
Multiple Choice Questions 8/57
Assertion-Reason Type Questions 8/60
8/65
Conceptual Questions
8/71
Very Short Answer Questions [7 mark]
8/73
Short Answer Questions [2 or 2 marks]
8/74
LongAnswer Questions [5 or more marks]
8/74
Case-Based Very Short/Short Questions
8/76
Case-Based MCQs andAssertion-Reason Questions
8/79
NCERT Questions and Exercises with Answers
8/100
NCERT Exemplar Problems with Answers, Hints and Solutions
8/115
Competition Focus [NEET/JEE Special]

w
■ Hydrogen 9/1—9/60
9.1. General Introduction 9/1
9/1
9.2. Unique Position of Hydrogen in the Periodic Table

F lo
9/4
9.3. Occurrence of Hydrogen
9/5
9.4. Isotopes of Hydrogen
9/7
9.5. Preparation of Dihydrogen

ee
9.5.1. Laboratory Preparation of Dihydrogen

Fr
9.5.2. Commercial Preparation or Manufacture of Dihydrogen
9/10
9.6. Properties of Dihydrogen
9/13
9.7. Uses ofDihydrogen

for
9/14
9.8. Hydrides
9.9. Water 9/18
ur
9.9.1. Structure ofWater Molecule and its Aggregates
9.9.2. Properties of Water
s
9.10. Hard and Soft Water mi
ook
Yo
9.10.1. Types of Hardness of Water
9.10.2. Methods of Removing Hardness of Water—Softening of Hard Water
eB

mi
9.11. Heavy Water
9.11.1. Preparation of Heavy Water
9.11.2. Properties of Heavy Water
our

9.11.3. Uses of Heavy Water


ad

9/29
9.12. Hydrogen Peroxide
9.12.1. Preparation of Hydrogen Peroxide
9.12.2. Manufacture of Hydrogen Peroxide
Y

9.12.3. Concentration of Hydrogen Peroxide Solution


Re

9.12.4. Strength ofHydrogen Peroxide Solution


nd

9.12.5. Properties ofHydrogen Peroxide


9.12.6. Uses of Hydrogen Peroxide
Fi

9.12.7. Tests of Hydrogen Peroxide


9.12.8. Structure of Hydrogen Peroxide
9/41
9.13. Hydrogen Economy
9/42
9.14. Some Additional Useful Information About Hydrogen
9/44
9.15. Typical Problems
Revision At A Glance 9/45
9/48
Competition Focus [NEET/JEE Special]

* Not included in CBSE syllabus. This chapter has been given onlyfor the preparation of competitive examinations.
*10. s-Block Elements (Alkali and Alkaline Earth Metals) 10/1—10/70
10.1. General Introduction 10/1

10.1.1. Occurrence and Abundance

10.1.2. Anomalous Behaviour of First Element of a Group


10.1.3. Diagonal Relationship
SECTION-I. ALKALI METALS
10.2. Occurrence 10/3

10.3. Electronic Configuration 10/3

10.4. Trends in Atomic and Physical Properties 10/4

w
10.5. Chemical Properties 10/7

10.5.1. Reactivity and Electrode Potential


10.5.2. Reactivity towards Water

F lo
10.5.3. Reactivity towards Oxygen
10.5.4. Reactivity towards Air and Moisture
10.5.5. Reactivity towards Hydrogen

ee
10.5.6. Reactivity towards Halogens
10.5.7. Solutions in Liquid Ammonia

Fr
10.6. General Characteristics of the Compounds of the Alkali Metals 10/11

10.6.1. Oxides and Hydroxides

for
10.6.2. Halides
ur
10.6.3. Salts ofOxoacids
10.7. Anomalous Behaviour of Lithium 10/15

10.8. Diagonal Relationship ofLithium with Magnesium 10/16


s
ook
10.9. Extraction of Alkali Metals 10/18
Yo
10.9.1, Difficulties Encountered during Extraction ofAlkali Metals
10.9.2. Extraction of Lithiu
eB

10.9.3. Uses of Lithium


10.9.4. Extraction of Sodium
10.9.5. UsesofSodium
r

10.9.6. UsesofotherAlkaliMetals
ad
ou

10.10. Some Important Compounds of Sodium 10/20

10.10.1. Sodium Carbonate (Washing Soda), Na^COj.lOHjO


Y

10.10.2. Sodium Chloride (Common Salt),NaCl


10.10.3. Sodium Hydroxide (Caustic Soda), NaOH
Re

10.10.4. Sodium Hydrogen Carbonate (Baking Soda), NaHCO,


nd

10.10.5. Sodium Thiosulphate Na2Sj03.5Hj0 (Hypo)


Fi

10.11. Biological Importance of Sodium and Potassium 10/27

SECTION-II. ALKALINE EARTH METALS

10.12. Occurrence 10/28

10.13. Electronic Configuration 10/28

10.14. Trends in Atomic and Physical Properties 10/29

10.15. Reactivity and Electrode Potential 10/31

10.15.1. Reducing Character


10.15.2. Reactivity towards Water—Formation of Hydroxides

* Not included in CBSE syllabus. This chapter has been given only for the preparation of competitive examinations.
10.15.3. Reactivity towards Air (Nitrogen and Oxygen)
10.15.4. Reactivity towards Halogens—Formation of Halides
10.15.5. Reactivity towards Hydrogen—Formation of Hydrides
10.15.6. Reactivity towards Carbon—Formation of Carbides
10.15.7. Reactivity towards Acids
10.15.8. Solutions in Liquid Ammonia
10.15.9. Tendency to form Complexes
10.16. Genera! Characteristics ofCompoundsofthe Alkaline Earth Metals 10/35
10.16.1. Oxides and Hydroxides
10.16.2. Halides

w
10.16.3. Solubility and Thermal Stability of Oxo Salts
10.17. Anomalous Behaviour of Beryllium 10/40
10.18. Diagonal Relationship of Berrylium with Aluminium 10/41

F lo
10.19. Uses 10/42
10.20. Some Important Compounds ofMagnesium and Calcium 10/42

10.20.1. Magnesium Chloride Hexahydrate, MgCIj.6 HjO


10.20.2. Calcium Oxide, (Quick Lime), CaO

ee
10.20.3. Calcium Hydroxide, (Slaked Lime), Ca(OH)j

Fr
10.20.4. Calcium Carbonate, CaCOj
10.20.5. Plaster of Paris, CaS04.1/2 H^O or (CaSOJj.HjO
10.21. Cement

for
10/46
10.21.1. Average Composition of Portland Cement
ur
10.21.2. Raw Materials
10.21.3. Manufacture of Cement
10.21.4. Setting of Cement
s
ook
10.21.5. Cement Substitutes
Yo
10.21.6. Cement Industry in India
10.22. Biological Importance ofMagnesium and Calcium 10/48
eB

10.23. Typical Problems 10/49


RevisionAtA Glance 10/49
Competition Focus [NEET/JEE Special] 10/55
our
ad

*11, Some p-Block Elements 11/1—11/64


11.1. General Introduction 11/1
11.2. General Characteristics ofp-BIock Elements 11/2
Y

11.2.1 Difference in Chemical Behaviour of First Element of each group


Re

as compared to the subsequent members of the same group


nd

PART-I. GROUP 13 ELEMENTS : BORON FAMILY


11.3. Introduction 11/4
Fi

11.4. Occurrence 11/4


11.5. Electronic Configuration 11/5
11.6. Atomic and Physical Properties 11/5
11.7. Chemical Properties 11/8
11.7.1. Oxidation States and Trends in Chemical Reactivity
11.7.2. Reactivity towards Dioxygen or Air
11.7.3. Reactivity towardsAcids and Bases
11.7.4. Reactivitytowards Halogens

* Not included in CBSE syllabus. This chapter has been given onlyfor the preparation of competitive examinations.
11.8. Properties of Boron 11/13

11.9. Anomalous Properties of Boron 11/13

11.10. Some Important Compounds of Boron and Aluminium 11/14

11.10.1. Borax,Naj[B,Oj(OH)J.8 orNOiBA-10H^O


11.10.2. Orthoboric acid (Boric acid), H3BO3 or B(OH)3
11.10.3. Boron Hydrides
11.10.4. Alums

11.11. Uses of Boron, Aluminium and their Compounds 11/21

lo w
PART-II. GROUP 14 ELEMENTS : CARBON FAMILY
11.12. Introduction 11/22
11.13. Occurrence 11/23

11.14. Electronic Configuration 11/23

11.15. Atomic and Physical Properties 11/24

11.16. Oxidation States and Trends in Chemical Reactivity 11/25

e
11.16.1. Reactivity towards Oxygen

re
11.16.2. Reactivity towards Water

rF
rF
11.16.3. Reactivity towards Halogens
11.17. AnomalousBehaviourofCarbon 11/29
11.18. Carbon 11/30

fo
11.18.1. Allotropes of Carbon
u
11.18.2. Atomic and Physical Properties
11.18.3. Chemical Properties ks
11.18.4. UsesofCarbon
Yo
11.19. Some Important Compounds ofCarbon and Silicon 11/35
oo
11.19.1. Oxides ofCarbon
11.19.2. Silicon Tetrachloride
eB

11.19.3. Silicon Dioxide, Silica (SiOj)


11.19.4. Silicon Carbide, Carborundum, SiC
11.19.5. Silicones
ur

11.19.6. Silicates
ad

11.19.7. Zeolite45
Yo

11.20. Typical Problems 11/45


RevisionAtA Glance 11/46

Competition Focus [NEET/JEE Special] 11/51


d
Re
in

12. Organic Chemistry-Some Basic Principles and Techniques 12/1—12/222


F

PART-I. GENERAL INTRODUCTION TO ORGANIC COMPOUNDS

12.1. Distinction between Organic and Inorganic Compounds 12/1

12.1.1. Vital Force Theory


12.1.2. Modem Definition of Organic Compounds
12.2. Tetracovalency ofCarbon 12/3

12.3. Shapes of Simple Organic Molecules 12/3

12.3.1. Some Characteristic Features of n-Bonds

12.3.2. Effect of Hybridization on Bond Length and Bond Strengths


12.4. Structural Representation ofOrganic Compounds 12/7

12.4.1. Complete and Condensed Formulae


12.4.2. Bond-line Structural Formulae

12.4.3. Polygon Formulae


12.5. Three-Dimensional (3-D) Representation of Organic Molecules 12/9
PART-n. CLASSIFICATION AND NOMENCLATURE OF ORGANIC COMPOUNDS
12.6. Classification ofOrganic Compounds 12/10

12.7. Functional or Characteristic Groups 12/13

ow
12.8. Homologous Series 12/15

12.9. Nomenclature ofOrganic Compounds 12/16

12.9.1. General Rules for lUPAC NomenclatureofOrganic Compounds


12.9.2. Nomenclature ofdifferent Classes of Organic Compoimds
12.9.3. Rules for lUPAC Nomenclature for Branched Chain Alkanes
12.9.4. Rules for lUPAC Nomenclature ofUnsaturated Hydrocarbons (Alkenes and Alkynes)

e
Fl
12.9.5. Rules for lUPAC Nomenclature of Compounds containing one Functional Group, Multiple

re
Bonds and Substituents

F
12.9.6. Rules for lUPAC Nomenclature ofPolyfunctional Compounds
12.9.7. Rules for lUPAC Nomenclature ofCompounds containing more than two like carbon
containing Functional Groups
ur
or
12.9.8. Rules for Naming Alicyclic Compounds
12.9.9. Writing Structural Formulae from the lUPAC Name ofthe Compound

sf
12.9.10. Nomenclature of Simple Aromatic Compoimds
12.9.11. Systematic Nomenclature for Di- and Polyfunctional Aromatic Compounds
k
Yo
12.10. Isomerism 12/59
oo
12.10.1. Structural Isomerism
12.10.2. Stereoisomerism
B

12.10.2.1. Optical Isomerism


12.10.2.2. Some General Concepts about Optical Isomerism
re

12.10.2.3. OpticalActivity-Discovery
12.10.2.4. MolecularAsymmetry and Chirality-Cause of Optical Activity
12.10.2.5. Enantiomersl2.10.2.6. Racemic mixture
u
ad

12.10.2.7. Diastereomers
Yo

12.10.2.8. Meso compounds


PART-m. FUNDAMENTAL CONCEPTS OF ORGANIC REACTION MECHANISM
12.11. Electrophiles and Nucleophiles 12/75
d

12.12. Fission ofa Covalent Bond 12/75


Re
in

12.13. Electronic Displacements in a Covalent Bond 12/76


12.13.1. Inductive Effect
F

12.13.2. Electromeric Effect


12.13.3. Resonance or Mesomerism
12.13.4. Resonance Effect or Mesomeric Effect
12.13.5. Hyperconjugation Effect
12.14. Reactive Intermediates 12/83
12.14.1. Carbocations
12.14.2. Carbanions
12.14.3. Free Radicals

12.15. Common Types ofOrganic Reactions 12/90


PART-IV. PURIFICATION OF ORGANIC COMPOUNDS
12.16. General Introduction 12/97
12.16,1. Filtration

12.16.2. Crystallisation or Recrystallisation


12.16.3. Fractional Crystallisation
12.16.4. Sublimation
12.16.5. Simple Distillation
12.16.6. Fractional Distillation

w
12.16.7. Distillation under Reduced Pressure or Vacuum Distillation
12.16.8. Steam Distillation
12.16.9. Differential Extraction
12.16.10. Chromatography

Flo
PART-V. QUALITATIVE ANALYSIS
12.17. General Analysis 12/108
12.17.1. Detection of Carbon and Hydrogen

e
12.17.2. Detection ofNitrogen

re
12.17.3. Detection of Halogens
12.17.4. Detection of Sulphur

F
12.17.5. Detection of Phosphorus
12.17.6. Detection of Oxygen ur
PART-VI. QUANTITATIVE ANALYSIS

r
12.18. General Introduction 12/113

fo
12.18.1. Estimation of Carbon and Hydrogen
12.18.2. Estimation ofNitrogen
12.18.3. Estimation of Halogens (Carius method)
ks
Yo
12.18.4. Estimation of Sulphur (Carius method)
oo
12.18.5. Estimation of Phosphorus
12.18.6. EstimationofOxygen
B

12.19. SomeAdditional Useful InformationAbout 12/126

Organic Chemistry- Some Basic Principles and Techniques


e

12.20. Typical Problems 12/128


Revision At A Glance 12/128
ur

Multiple Choice Questions 12/137


Assertion-Reason Type Questions
ad

12/143
Yo

Conceptual Questions 12/149


Very Short Answer Questions [I mark] 12/156
Short Answer Questions [2 or 3 marks] 12/161
Long Answer Questions [5 or more marks] 12/164
d

Case-Based Very Short/Short Questions ,


Re

12/164
in

Case-Based MCQs andAssertion-Reason Questions 12/166


NCERT Questions and Exercises with Answers 12/169
F

NCERT Exemplar Problems with Answers. Hints and Solutions 12/178

Competition Focus [NEET/JEE Special] 12/194

13. Hydrocarbons 13/1—13/208


13.1. introduction 13/1

13.2. ClassificationofHydrocarbons 13/1


PARTI. ALKANES
13.3. Alkanes 13/3
13.3.1. Nomenclature of Alkanes
13.3.2. Isomerism of Alkanes
13.3.3. Structure of Alkanes
13.3.4. Methods ofPreparation ofAlkanes
13.3.5. Physical Properties pfAlkanes
13.3.6. Chemical Reactions ofAlkanes
13.3.7. Uses ofAlkanes
13.4. Stereoisomerism 13/19
13.5. Conformations ofAlkanes 13/19
13.5.1. Conformations of Ethane

w
PARXn.ALKENES
13.6. Alkenes 13/21
13.6.1. Structure of Double Bond
13.6.2. Nomenclature ofAlkenes

Flo
13.6.3. Isomerism ofAlkenes

13.6.4. General Methods ofPreparation ofAlkenes


13.6.5. Physical Properties ofAlkenes

e
re
13.6.6. Why do Alkenes undergo Electrophilic Addition Reactions ?
13.6.7. Mechanism ofElectrophilic Addition Reactions to Alkenes

F
13.6.8. Chemical Reactions ofAlkenes
13.6.9. Uses ofAlkenes ur PARTm.ALKYNES

or
13.7. Alkynes 13/47

13.7.1. Structure ofTriple Bond

f
13.7.2. Nomenclature ofAlkynes ks
13.7.3. Isomerism in Alkynes
Yo
13.7.4. Classification ofAlkynes
oo
13.7.5. Methods of Preparation ofAlkynes
13.7.5.1. Manufacture ofAcetylene
B

13.7.6. Physical Properties ofAlkynes


13.7.7. Reactivity ofAlkynes versus Alkenes
re

13.7.8. Chemical Reactions ofAlkynes


13.7.9. DistinctionbetweenanAlkane,AlkeneandanAlkyne
u

13.7.10. Uses ofAlkynes


ad
Yo

PARTIV.ARENES

13.8. Aromatic Hydrocarbons or Arenes 13/65


13.8.1. Classification and Nomenclature ofArenes
13.8.2. Isomerism in Arenes
d
Re

13.8.3. Structure ofBenzene


in

13.8.4. Resonance

13.8.5. Calculation ofResonance Energy ofBenzene


F

13.8.6. Aromaticity—^HuckelRule
13.8.7. Methods ofpreparation ofArenes
13.8.8. Physical Properties ofArenes
13.8.9. Chemical Reactions ofArenes
13.8.10. Uses ofArenes

13.8.11. Mechanism ofElectrophilic Substitution Reactions


13.8.12. Directive Influence of Substituents and their Effect on Reactivity
13.8.13. Directive Influence ofElectron-withdrawing Groups in Nucleophilic Substitution Reactions
13.8.14. Polynuclear Aromatic Hydrocarbons—Carcinogenicity and Toxicity
13.9. Typical Problems 13/92
RevisionAtA Glance 13/93
Multiple Choice Questions 13/101
Assertion-Reason Type Questions 13/109
Conceptual Questions 13/116
Very Short Answer Questions [/ mark] 13/124

ow
Short Answer Questions \2or3marks] 13/129
LongAnswer Questions \ or more marks]
[5 13/132
Case-Based Very Short/Short Questions 13/132
Case-Based MCQs andAssertion-Reason Questions 13/136
NCER T Questions and Exercises with Answers 13/138
NCERT Exemplar Problems with Answers, Hints and Solutions 13/151

e
Competition Focus [NEET/JEE Special] 13/168

re
Frl
*14. Environmental Chemistry 14/1—14/50
SECTION iV ENVIRONMENT AND ENVIRONMENTAL POLLUTANTS

F
14.1. Introduction
14/1
14.2. Environmental Pollution and Environmental Pollutants 14/3
14.3. Types of Pollutants 14/5
ou
or
14.4. Types ofPolIution 14/5
SECTION II. AIR OR ATMOSPHERIC POLLUTION

kfs
14.5. Introduction
14/6
14.6. Sources ofAir/Atmospheric Pollution 14/6
14.7. TroposphericPollution 14/7
oo
14.8. Chemical Reactions Occurring in the Atmosphere 14/14
14.9. Ozone Layer-Earth’s Protective Umbrella 14/16
Y
14.9.1. Formation of ozone layer
B

14.9.2, Depletion ofozone layer (Creation of “ozone hole”)


14.9.3. Effects/Consequences of depletion ofozone layer
re

SECTION III. SOME EFFECTS OF AIR POLLUTION


14.10. Smog and Its Kinds 14/18
oYu

14.11. Acid Rain


14/20
14.12. Greenhouse Effect and Global Warming
ad

14/21
SECTION IV. WATER POLLUTION
14.13. Introduction
14/23
d

14.14. TypesofWaterPoIlution 14/23


14.15. Sources/Causes of Water Pollution 14/25
in
Re

14.16. Classification of Water Pollutants 14/25


14.17. International Standards for Drinking Water 14/28
F

SECTION V. SOIL/LAND POLLUTION


14.18. Introduction
14/29
14.19. Composition of the Soil 14/29
14.20. Sources of Soil Pollution 14/30
SECTION VI. STRATEGY FOR CONTROL OF ENVIRONMENTAL POLLUTION
14.21. Management of Waste 14/31
14.21.1. Waste management of Household Waste
14.21,2. Management of Industrial Waste
14.21.3. Some Recent Developments About Waste Recycling
Not included in GBSE syllabus, t his chapter has been given only Jor the preparation oj competitive examinations.
14.22. Green Chemistry as an Alternative Tool for Reducing Pollution 14/32
14.22.1. Introduction

14.22.2. Technique Used


14.22.3. Green Chemistry in Day-to-Day Life
14.23, Some Additional Useful Information About ‘Environmental Chemistry’ 14/34

14.24. Typical Problems 14/35

RevisionAtA Glance 14/36

ow
Competition Focus [NEET/JEE Special] 14/41

Appendix A/1—A/22
i. ImportantName Reactions and Processes 7JT

2. Distinction between Pairs of Compounds A/6


A/10
3. Some Typical Conversions

e
4. Identification of Unknown Organic Compounds/Reagents A/18

re
rFl
A/21
5. What Happens When?

F
r
ou
fo
ks
oo
Y
r eB
ou
ad
Y
d
Re
in
F
low
REDOX REACTIONS

ee
rF
Fr
8.1. GENERAL INTRODUCTION

Chemistry deals with the study of composition, structure and properties of varieties of matter and the
change of one kind of matter into another. This transformation of one kind of matter into another occurs

for
through a number of different types of reactions. One important type of such reactions is reduction- oxidation
or simply redox reactions {red from reduction and ox from oxidation). All these reactions are always
u
accompanied by energy changes in form of heat, light or electricity.
ks
A number of phenomena both physical as well as biological fall in this category of reactions. These
reactions are widely used in biological, phiirmaceutical, industrial, metallurgical and agricultural areas. The
Yo
oo
importanceof these reactions is evident from the fact that burning of different types of fuels such as wood,
coal, kerosene, LPG (Liquefied Petroleum Gas), petrol, diesel, CNG (Compressed Natural Gas), etc. for
eB

obtaining energy for domestic, transport and other commercial purposes, electrochemical processes like
manufacture of chlorine and caustic soda, corrosion of metals, and operation of dry and wet batteries are
diverse examples of redox reactions. Recently, environmental issues like hydrogen economy (use of liquid
hydrogen as fuel) and development of ozone hole are also been regarded as redox phenomena.
r
ou
ad

8.2. OXIDATION-REDUCTION-CLASSICAL CONCEPT


Y

8.2.1. Oxidation
According to the classical concept.
nd
Re

Oxidation may be defined as a process which involves the addition of oxygen or any other
electronegative element, or as a process which involves the removal of hydrogen or any other
Fi

electropositive element.

For example, (/) 2 Mg(.9) + O, (g) ^ 2 MgO(s) {Addition of oxygen)


(lO ^ is) + CI2C?) — MgCl2(.9) {Addition of electronegative element, chlorine)

{in) 2T^(g) +0.(5) 2S(.9) + 2H20(0 {Removal of hydrogen)

(iv) ^ {aq) + H20(/) + 03(g) ^ 2K0H {aq) + ^(j) + 02(g)


{Removal of electropositive element, potassium)
In all these reactions, the compound underlined has undergoneoxidation.
8/1
8/2 New Course Chemistry (XI)EZalHn

8.2.2. Oxidising Agent or Oxidant


According to the classical concept,

An oxidising agent or oxidant is a substance which supplies oxygen or any other electronegative
element, or removes hydrogen or any other electropositive element. An oxidising agent after
carrying out oxidation is itself reduced in a chemical reaction.
For example, oxygen, chlorine and ozone in reactions (0 to (iV) listed above in art 8.2.1. are oxidising
agents. Some odier important examples of oxidising agents are fluorine, manganese dioxide, hydrogen peroxide,
potassium permanganate, potassium dichromate, nitric acid. etc. The.se are illustrated by the following equations :
Mg is) + F2 (g) ^ MgFo (s)
A\iC\{aq) + Mn02(.v) ^ MnCl2(a^) + Cl2(g)+2H20(/)

w
\i2O2iaq) + lYAiaq) ^ 2KOH (aq) + U (s)
2KMn04(fli?) + 10 FeS04 {aq) + 8 H2S04(«f?) -> 2MnS04(fl^) + K2S04(a^) + 5Fe2(S04>3(a^)

F lo
+ 8H2O ([)
K2Cr202(fj^) + 3802(g) + H2S04(fli^) 4 Cr2(S04)3(ii^) + K2S04(o^) + H20(0
i0HNO3(a^) + l2(i') ^ 10NO2(g) + 2HIO3 iaq) + 4H20(/)
8.2.3. Reduction

ree
According to the classical concept.

for F
Reduction may be deifned as a process which involves the addition of hydrogen or any other
electropositive element, or removal of oxygen or any other electronegative element.

For example, (/) Br2 (g) + H2S (g) ^2HBr(g) + S(^) {Addition of hydrogen)
Your
(11) 2HgC\2 (aq) + SnCl2(fl^) > Hg2Cl2(i) + SnCl4 (aq)
ks

(Addition of electropositive element, mercury)


eBoo

(iU) 0^(s) +H2(g) ^ Cu (s) + H,0(/) (Removal of oxygen)

(IV) 2FeCl3 (aq) + 502(g) + 2 H2O(0 ■> 2FeC\2(aq) + H2S04(fl^) + 2HCl(a^)


(Removal of electronegative element, chlorine)
ad
our

In all these reactions, the compound underlined has undergone reduction.


8.2.4. Reducing Agent or Reductant
According to the classical concept.
Re

A reducingagent or reductantmay be deifned as a substance which supplies hydrogen or any


other electropositiveelement, or removes oxygen or any other electronegative element. A reducing
Y

agent after carrying out reduction is itself oxidi.sed in a chemical reaction.


Find

For example, hydrogen sulphide, stannous chloride, hydrogen and sulphur dioxide respectively in
reactions (/) to (/v) listed above in Art, 8.2.3 are reducing agents. Some other important examples of reducing
agents are carbon, carbon monoxide, aluminium, nitrous acid, etc, These are illustrated by the following
equations
CuO (^) + C(5)- ^ Cu(^) + CO(g)
F02O3 (j) + 3CO(g) -^2Fe(i) + 3C02(g)
FC203(5') + 2A1(5) 2Fe(j) + Al203(j)
2KMn04 (aq) + 3H2SO4 (aq) + 5HNO2 (aq)
K2SO4 (aq) + 2MnS04 (aq) + SHNOj (aq) + 3H2O (1)
REDOX REACTIONS 8/3

8.2.5. Oxidation-Reduction Reactions are Complementary


Whenever any substance is oxidised, another substance is always reduced at the same time, and vice-
versa. In other words, oxidation and reduction reactions are complementary, i.e., they always go hand in
hand or side by side. This is illustrated by the following examples :
(/) Reaction between hydrogen sulphide and chlorine

ow
H2S (g) + CI2 (g) ^ 2HC1 (g) + S is)
Here, H2S is oxidised to S while Cl^ is reduced to HCl.
iii) Reaction between stannous chloride and mercuric chloride.
SnCl2 (aq) + 2HgCl2 (aq) SnCl4 (aq) + Hg2Cl2 (5)
Here, SnCl2 is oxidised to SnCl4 while HgCl^ is reduced to Hg^CI-*.
(Hi) Reaction between manganese dioxide and hydrochloric acid.

e
Mn02 is) + 4HC1 iaq) ^ MnCU iaq) + CU ig) + 2H2O (/)

re
Here, HCl is oxidised to C/2 while M11O2 is reduced to MnCl2-

rFl
F
8.3. OXIDATION AND REDUCTION - ELECTRON TRANSFER CONCEPT
It is a well known fact that the various chemical reactions occur through redistribution of electrons
among the reacting substances. Any substance that loses electrons is said to be oxidised and the one wliich

r
gains electrons is said to be reduced. Thus, according to the electronic concept, oxidation and reduction may
be defined as follows :
ou
fo
ks
Oxidation may be defined as a process in which an atom or an ion loses one or more electrons.
That is why oxidation is also called de-electronation.
This loss of electrons either increases the positive charge or decreases the negative charge of the atom
oo
or the ion. For example,
Y
(/) Loss of electrons results in increase in positive charge :
B

Na ^ Na"^ + e~ Mg- ^ Mg2+ + 2e~


Fe-+ -> Fe^'*' + e~ Sn^'*' Sn'^ + 2 e-
re

(//) Loss of electrons results in decrease in negative charge :

MnOf-
4-
MnO^ +e [Fe(CN)g] 4 [Fe(CN)(,f + e'
ou
Y
ad

2C\- ■> CI2 + 2 e” S-- S + 2 e~

Reduction may be defined as a process in which an atom or an ion gains one or more electrons.
That is why reduction is also called electronation.
d

This gain of electrons either decreases the positive charge or increases the negative charge of the atom
in

or the ion. For example,


Re

(/) Gain of electrons results in decrease in positive charge :


F

Fe^+ + e“ Fe--^ 2Hg-*+ 2 e~- > Hg^


2+

Sn4+ + 2e“- ^Sn^+ Sb^+ + 2


(/i) Gain of electrons results in increase in negative charge :
C\2 + 2e~- ^2CI-
; MnO^ +e > MnO^
S + 2e~ -^S2- ; [FeCCN)^]-^- + > [FeCCN)^]
4-

Oxidation-reduction as an electron-transfer process. We have discussed above that oxidation involves


loss of electrons and reduction involves gain of electrons. Since there cannot be a net gain or loss of electrons
in a chemical reaction, therefore, all chemical reactions involving loss or gain of electrons must occur
simultaneously. In other words, in a chemical reaction, a substance can lose electrons only if there is present
another substance which can gain electrons. Conversely, a substance can gain electrons only if another substance
which can lose electrons is also present in the system. This implies that oxidation can take place only if
8/4 ‘Pnadeefr'4, New Course Chemistry (XI)IS

reduction also occurs at the same time or vice-versa. In other words, oxidation-reduction reactions are
complementary, i.e., they always go side by side or hand in .hand. This may be illustrated by the following
reactions :

4Na(^) + 02(g) ^ 2 Na20 is) 2Na(j) + Cl2(g) ^ 2 NaCl (s)

w
2 Na is) + S (i) ^ Na2S (j')
All these reactions are redox reactions because in each of these reactions, sodium is oxidised due to the
addition of either oxygen or more electronegative element such as chlorine or sulphur. Simultaneously, oxygen,
chlorine and sulphur are reduced because to each of these, the electropositive element sodium has been
added. Further, from our knowledge of chemical bonding, we know that sodium oxide, sodium chloride and

o
sodium sulphide are all ionic compounds and may be better written as (Na"‘')20^” (s), Na'*' Cl” (j) and (Na'*‘)2S^"

e
respectively. In view of the ionic nature of these compounds, the above erdox reactions may be rewritten as

re
under:

Frl
Loss of Ae~, oxidised Loss of 2e“, oxidised'

F
(0 4Na(^) + O2te) ^ 2 (Na‘")202- is) Hi) 2 Na is) + Clj ig) ►2Na‘^cr(5)
_r
A

Gain of 4e .reduced
ou Gain of 2e , reduced

r
Loss of 2e”, oxidised

so
HU) 2Na(5) + S(5) ► (Na*)2S2-(i)

kf
i k

Gain of 2e“, reduced


oo
For sake of convenience, each of the above redox reactions can be considered as a sum of two half
reactions - one involving oxidation called oxidation half reaction and the other involving reduction called
Y
reduction half reaction. To explain these half reactions, let us consider the oxidation of sodium by chlorine
B

to form sodium chloride.

Na(5) ■> Na"^ + e ioxidation)


re

CI2 + 2 e~ ^2C1- ireduction)


oY

In order to get the overall equation for the redox reaction, the two half reactions are simply added if the
u

number of electrons lost during the oxidation half reaction is equal to the number of electrons gained during
ad

the reduction half equation. But if the number of electrons lost during oxidation half reaction are different
from the number of electrons gained during reduction half reaction, the two half reactions are multiplied by
d

suitable integers so that when the two half equations are added, the electrons cancel out of the final redox
equation. For example, during oxidation of sodium by chlorine, the number of electrons lost by each sodium
in

atom is one while those gained by each chlorine molecule, are two, therefore, the oxidation half equation is
Re

multiplied by 2 and then added to the reduction half equation to get the equation for the overall redox reaction
as shown below :
F

Na is) ^ Na'*’ + e~]x2 ioxidation)


CI2 ig) + 2e- ■>2C1" ireduction)

Overall redox equation : 2 Na (5) + CI2 ig) ^ 2 Na+ Cl- is) or 2 NaCl (5)
Here, the suffix, (5) simply denotes that the ultimate product formed is in the solid state.
Similarly, in the formation of sodium oxide or sulphide, sodium loses one eleetron and thus gets oxidised
to sodium ion while each atom of oxygen molecule or sulphur atom accepts two electrons and thus get
reduced to oxide or sulphide ions.
H) Na is) Na"*" + e“] X 4 ioxidation)
O2 ig) + 4 e~ ireduction)
Overall redox reaction : 4 Na is) + O2 ig) 4
2 (Na'*')202- is) or 2 Na20 is)
REDOX REACTIONS 8/5

{//) Na (^) ■> Na'*' + e \ x2 (oxidation)


S {5) + 2 e~ ^S2- (reduction)

Overall redox reaction : 2 Na (s) + S (,y) ■> (Na'^)-iS“ (a-) or Na2$ (.9)
In all the three reactions described above, electrons are transferred from one substance, i.e., Na to the
other, i.e., CI2, O2 or S. Thus, oxidation-reduction or redox reactions may be regarded as electron-transfer
reactions in which the electrons are transferred from one reactant to the other. The substance which loses

ow
electrons is called a reducing agent while the other which accepts the electrons is called an oxidising agent.
Thus,

A substance (atom, ion or molecule) which can readily lose electrons to other substances is
called a reducing agent or a reductant while a substance (atom, ion or molecule) which can
readily accept electrons from other substances is called an oxidising agent or an oxidant.

e
Fl
Further since, reducing agents donate electrons to other substances while oxidising agents accept electrons

re
from other substances, therefore, reducing agents are electron donors while oxidising agents are electron

F
acceptors. In other words, reducing agents after reducing other substances themselves get oxidised while
oxidising agents after oxidising other substances themselves get reduced in the process.
Let us now reconsider the three reactions (between Na & CL, O, and S) discussed above in the light of
ur
or
reducing and oxidising agents.
In the reaction of Na with CI2, Na gives electrons to chlorine which gets reduced to Cr ion while Na

sf
gets oxidised to Na'*' ions. Conversely, CL accepts electrons from Na and gets reduced to Cl“ while Na gets
oxidised to Na"'’. Therefore, Na is a reducing agent while CI2 is an oxidising agent. Similarly, in the reaction
k
Yo
of Na with O2 or S. Na acts as a reducing agent while O2 or S behaves as an o.tidising agent. For further
oo
illustration, consider the following reactions :
Oxidised
B

(/) H2S +
2 FeCL > 2 FeCL + 2 HCl + S
re

(Reducing agent) (Oxidising agent)


I Reduced

Here, H2S reduces FeCl3 to FeCl2 while itself gets oxidised to S. Conversely, FeCl3 oxidises H2S to S
u
ad

while itself gets reduced to FeCL- Therefore, H2S acts as a reducing agent while FeCl^ acts as an oxidising
Yo

agent .
Oxidised
d

(ii) 2A1 +
Fe203 > ALO3 + 2Fe
Re
in

(Reducing agent) (Oxidising agent)


* Reduced
F

Here, A1 reduces Fe203 to Fe while itself gets oxidised to AI2O3. Conversely, Fe203 oxidises AI to
AI2O3 while itself gets reduced to Fe. Therefore, Al acts as a reducing agent while Fc20j acts as an oxidising
agent.
From the above discussion, we conclude

Oxidation is a process in which one or more electrons are lost.


Reduction is a process in which one or more electrons are gained.
Oxidant is a substance which can accept one or more electrons.
Reducant is a substance which can donate one or more electrons.

In a redox reaction, oxidant is reduced by accepting electrons and reductant is oxidised by losing
electrons.
8/6 7^'ietdecfr New Course Chemistry (XI)

I. In the reactions given below, identify the species undergoing oxidation and reaction :
(0 CH4 (g) + 2 O2 (g) > CO2 (g) + 2 H2O (/) (/O2H2S (g)-H02 (g) ^2 Sis)+ 2 H2O (0

m CH2 = CH2 (g) + H, (g) > H3C — CH3 (g) ((V) 2 HgO is) ^ 2 Hg (/) + O2 (g)

ow
(V) Mg (5) + S(.v) >MgS (s)
2. Using electron-transfer concept, identify the oxidant and rcductant in the following redox reactions,
(o) Zn is) + 2 H+ iaq) ^ Zn^+ (aq) + H2 (g)
ib) 2 [Fe(CN)fi]‘^ iaq) + H2O2 iaq) + 2 iaq) > 2 [FeCCN)^]'-*- iaq) + 2 H2O (/)
(c) 2 [Fe(CN)(ij3- iaq) + 20H" iaq) + H2O2 iaq) > 2 [Fe(CN)fi]‘*- iaq) + O2 (g) + 2 H2O (/)

e
Fl
re
id) BrO^(a^) + F2 (g) + 2 OH“ iaq) > BiO“(fl£/) + 2 F^ iaq) + H^O (/)
ie) 2NaC103 iaq) + I2 iaq) > 2Nal03 iaq) + CI2 (g)

F
3. Write the half reactions for the following redox reactions ;

ia) 2Fe^+ iaq) + 21“ iaq) > 2 Fe“+ iaq) + Ij iaq) ib) Zn is) + 2 H+ iaq)
ur > Zn^+ iaq) + Hj (g)

r
(c) A1 (.9) + 3 Ag'*’ iaq) > Ap'*' iaq) + 3 Ag (i)

fo
4. Split the following redox reactions into the oxidation and reduction half reactions.
ia) 2 K is)+ CI2 ig) > 2 KCl is) ib) 2 A1 is) + 3 iaq) > 2 Al^-^ iaq) + 3 Cu is)
ANSWERS
ks
Yo
1. (0 CH4 is oxidised to CO2 while O2 is reduced to H,0. (n) H2S is oxidised to S while O2 is reduced to H^O.
oo
iiii) CH2 = CHo is reduced to CH3—CH3 while H2 is oxidised to CH3—CH3.
(iv) Hg^'*' is reduced to Hg while 0^“ is oxidised to O2. (v) Mg is oxidised to Mg-"*" while S is reduced to S^".
eB

2. Oxidants ; («) H"'' (b) H2O2 (c) [Fe(CN)g]^“ id) Fj (e) NaC103
Reductants : (a) Zn ib) [FeCCN)g]'*“ (c) H2O2 id) BrOg (e) I2.
ur

...Oxidation
3. (a) 2 I iaq) I2 iaq) + 2 e'
ad

Fe^'*’ iaq) + e ^ Fe-+ iaq) ] x 2 ...Reduction


Yo

2 Fe3+ iaq) + 2l~ iaq) ■> 2 Fe2+ iaq) + I2 iaq) ...Overall redox reaction

ib)Znis) ^ ZtP-* iaq) + 2 e ...Oxidation


d

2 H+ iaq) + 2e- ^H2 ig) ...Reduction


Re
in

Zn is) + 2H-^ iaq) >Zn2+(a,) + H2(«) ...Overall redox reaction

ic) Al is) aP"*" iaq) + 3 e ...Oxidation


F

Ag"^ iaq) + e ^ Ag is)] X 3 ...Reduction

Al (5) + 3 Ag"*" iaq) AP'*’ iaq) + 3 Ag is) ...Overall redox reaction

4. ia)Kis) ●>K+{g)-he'[x2 ...Oxidation

CI2 ig) + 2e- ^2CI-(g) ...Reduction

K^ig) + cr ig) ^ KCl (5) J X 2

2K{j) + Cl2(g) ^ 2 KCl (^) ...Overall redox reaction

ib) Al is) ■> aP+ iaq) + 3 e- [x 2 ...Oxidation

Cu2+ iaq) + 2e~ -> Cu (j) ] X 3 ...Reduction

2 Al is) + 3 Cu^+ iaq) ^ 2 aP+ (g) -H 3 Cu is) ...Overall redox reaction


REDOX REACTIONS 8/7

83.1. Competitive Electron TVansfer Reactions


Take a rod of metallic zinc and clean it well with a sand paper. Now place it in a solution of copper
nitrate in a beaker as shown in Fig. 8.1. for about one hour. The following changes are observed :
FIGURE 8.1

Zn rod
Copper nitrate Cu deposited
solution over Zn rod

low
Intensity of ^ Finally ^
blue colour Ions are
decreases
deposited as
Cu A-

Initial stage Intermediate stage Final stage

Redox reaction between zinc and copper nitrate occurring in a beaker

ee
F
(/) Zinc rod gradually starts dissolving,

Fr
(ii) Reddish brown copper metal starts depositing on the zinc rod.
(Hi) The blue colour of copper nitrate solution starts fading and ultimately becomes colourless. The
formation of Zn^"*" ions, among the products, can be easily detected when the colour of the solution due to

for
Cu^'*’ ions starts fading. If hydrogen sulphide (H2S) is passed through the solution containing Zn^'*’ ions,
ur
formation of white zinc sulphide, ZnS can be seen.
(iv) The reaction is exothermicand the solution becomes hot. ks
(v) The solution remains electrically neutral.
The reaction between metallic zinc and the aqueous solution of copper nitrate is
Yo
oo
Zn (^) + Cu^'*' (aq) > Zn^’*' (aq) + Cu (5) ...(0
In this reaction, zinc loses electrons to form Zn-'*’ ions. Therefore, zinc gets oxidised. Evidently, if zinc
eB

is oxidised, losing electrons, there must be some species present in the solution which must get reduced by
accepting the electrons lost by zinc. This species is Cu^'*’ ions provided by dissociation of copper nitrate.
Thus, the above reaction may be rewritten as
r

Oxidation (loss of 2e")


ou
ad

Zn (s) +
Cu^'*’ (aq) *■ Zn~* (aq) Cu (s)
Y

Reduction (gain of 2c”)


Let us now investigate the slate of equilibrium of reaction (/). For this purpose, place a rod of copper in
a zinc sulphate solution. No visible reaction is noticed and all attempts to detect the presence of Cu^'*' ions in
nd
Re

solution by passing H2S gas fail since no black ppt. of cupric sulphide (CuS) is formed. We, therefore,
conclude that the state of equilibrium or reaction {/) greatly favours the products over the reactants.
Fi

Let us now extend FIGURE 8.2


the electron transfer Cu rod

reaction to copper metal Silver nitrate Ag deposited


over Cu rod
and silver nitrate sol solution

ution. For this purpose,


place a copper rod in
silver nitrate solution as Blue colour
shown in Fig. 8.2. starts
Intensity of ^
blue colour
The appearing increases
following
reaction occurs and the
solution acquires blue Initial stage Intermediate stage Final stage
colour due to the
Redox reaction between copper and silver nitrate occurring in a beaker
formation of Cu^'*' ions.
8/8 New Course Chemistry (XI)CZsl9D

Oxidation (loss of 2t? )

Cu (5) +
(aq) > Cu""^ (fl^) +
2Ag(5)

Reduction (gain of 2e ) *
Here, copper metal, Cu (i-) is oxidised to Cu""^ (aq) while {aq) is reduced to .silver metal, Ag (s). The
equilibrium greatly lies in favour of Cu"'*’ {aq) and Ag (s).
For purpose of comparison, let us now place a rod of cobalt metal in nickel sulphate solution. The

w
following reaction occurs :
Oxidation (loss of 2e )

F lo
Co (i-) + {aq) Co^'*’ (aq) Ni {s)

Reduction (gain of 2e ) '

ee
At equilibrium, chemical tests reveal that both Ni^+ {aq) and Co^"^ {aq) are present in moderate
concentration. In other words, in this case, neither the reactants, Co (.s) and Ni^"^ {aq) nor the products, Co^"^

Fr
{aq) and Ni (.y) are greatly favoured.
This competition for loss or gain of electrons by metals or their metal ions is similar to the competition
between various acids to lose a in water. Therefore, we can develop a table in which metals and their ions

for
are listed according to their tendency to lose or gain electrons in the same way as we have already developed
ur
a table (unit 7) in which the various acids are listed according to their acid strength.
The results of experiments discussed above reveal that zinc releases electrons to copper and copper
releases electrons to silver. Therefore, the electron-releasing tendency of these three metals is in the order ;
s
ook
Zn > Cu > Ag.
Yo
By conducting more experiments between various other metals and their combinations with suitable
metal ions, we have developed a metal activity series. With the help of this activity series, we can construct
eB

a class of cells in which chemical reactions are used as a source of electrical energy. These cells are called
Galvanic cells and would be discussed in detail in class XII.

8.4. OXIDATION NUMBER


r
ou
ad

As already discussed, oxidation-reduction reactions involve the transfer of electrons from one atom or
very easy to determine the total number of electrons transferred
ion to the other. In case of ionic reactions, it is
from one atom or ion to the other. However, in many redox reactions involving covalent compounds, it is not
Y

so easy to determine (/) direction of transfer of electrons and {ii) number of electrons transferred from one
reactant to the other simply by looking at the chemical equations. For example, in the following redox reaction,
Re

2H2 (g) + 02 (g) >2H20 (/)


nd

both the reactants and the products are covalent compounds. From our knowledge of chemical bonding, we
know that during the formation of H2O molecule, an electron pair is shared between each hydrogen and
Fi

oxygen atom and that electron is not completely transferred from hydrogen to oxygen atom. Yet in H2O
molecule, O atom has higher electron density and thus there is a partial transfer of electronic charge better
called as electron shift from hydrogen to oxygen. In other words, we can easily say that hydrogen is oxidised
and oxygen is reduced. What is said about the reaction between H2 and O2 may be true for a number of other
reactions involving covalent compounds. Two such reactions are :
H2 (g) + Cl2 (g) >2HC1 (g)
CH4 (g) + 4 CI2 (g) > CCI4 (0 + 4 HCl (g)
In these reactions, H2 and CH4 act as the reducing agents while CI2 acts as the oxidising agent.
With a view to identify oxidising and reducing agents by keeping track of number of electrons transferred
from one reactant to the other both in ionic and covalent compounds, and to help in balancing of equations,
the concept of oxidation number was introduced. Thus,
REDOX REACTIONS 8/9

Oxidation number of an element may be defined as the charge which an atom of the element
has in its ion or appears to have when present in the combined state with other atoms. Oxidation
numbers are also called oxidation states.

8.4.1. Rules for assigning Oxidation Numbers


The following rules are applied to determine the oxidation number of an atom in an ion or a molecule.
1. The oxidation number of all the atoms of different elements in their respective elementary states and
allotropic forms is taken to be zero. For example, in N2, CI2, H2, He, P4, Sg, O^, O3, C (diamond or graphite),

w
Br2, Na, Fe, Ag etc., the oxidation number of each atom is zero.
2. The oxidation number of a monoatomic ion is the same as the charge on it. For example, oxidation
numbers of Na'*’, Mg^"*" and ions are +1, +2 and +3 respectively while those of Cr, S^“ and N^“ ions are

F lo
-1,-2 and - 3 respectively.
3. The o.xidation number of hydrogen is +1 when combined with non-metals and is - 1 when combined
with active metals called metal hydrides such as LiH, KH, MgH2> CaU2, etc.

ee
4. The oxidation number of oxygen is -2 in most of its compounds, except in peroxides like H2O2,
5flC>2» ^here it is - }. Other interesting exceptions are found in the compounds : OF2 (oxygen difhioride)

Fr
and O2F2 (dioxygen difhioride) where the oxidation number of oxygen are +2 and +1 respectively. This is
due to the fact that fluorine being the most electronegative element known has always an oxidation number of

for
5. In compoundsformed by union of metals with non-metals, the metal atoms will have positive oxidation
ur
numbers and the non-metals will have negative oxidation numbers. For example,
(a) The oxidation number of alkali metals (Li, Na, K etc.) is always +I and those of alkaline earth
metals (Be. Mg, Ca, etc) is +2.
s
ok
(b) The oxidation number of halogens (,F Cl, B,r I) is always - I in metal halides such as K,F AlCl^,
Yo
MgBr2< Cdl2< etc.
o

6. In compounds formed by the union of different elements, the more electronegativeatom will have
eB

negative oxidation number whereas the less electronegative atom will have positive oxidation numbe.r For
example,
(a) N is given an oxidation number of - 3 when it is bonded to less electronegative atom as in NH3 and
r

NI3, but is given an oxidation number of +3 when it is bonded to more electionegative atoms as in NF3.
ad
ou

(b) Since fluorine is the most electronegative element known so its oxidation number is always - 1 in its
compounds, i.e., oxides, interhalogen compounds, etc.
(c) In interhalogen compounds of F, Cl, Br, and I; the more electronegativeof the two halogens gets the
Y

oxidation number of - 1. For example, in IF7, the oxidation number of F is - 1 while that of I is +7. Similarly,
in BrCl3, the oxidation number of Cl is -1 while that of Br is +3.
Re
nd

7. In neutral compounds, the sum of the oxidation numbers of all the atoms is zero.
8. In complex ions, the sum of the oxidation numbers of all the atoms in the ion is equal to the charge
Fi

on the ion.

9. In polyatomic ions, the algebraic sum of the oxidation number of all the atoms in the ion is equal to
the charge on the ion. For example, the sum of the oxidation numbers of all the three oxygen atoms and one

carbon atom in the carbonate ion COj" is -2.


A term which is more oftenly used for oxidation number is oxidation state. For example in CO->, the oxidation
state as well as the oxidation number of carbon is + 4 while the oxidation state or oxidadon number of oxygen is
- 2. This means that the oxidation number denotes the oxidation state of an element in a compound.

Metals invariably have positive oxidation states while non-metals may have positive or negative oxidation
states. Transition metals usually display several oxidation states. The highest positive oxidation state for
5-block elements is equal to its group number but for p-block elements it is equal to group number minus 10
8/10 T^n^ndeefr'^ New Course Chemistry (XI)ESC

(except for noble gases). However, the highest negative oxidation state for p-block elements is equal to eight
minus the number of electrons in the valence shell.
In other words, the highest positive oxidation state increases
across a period in the periodic table. For example, in the third period, the highest positive oxidation state/
number increases from +1 to +7 as shown below :

Group 1 2 13 14 15 16 17

Element Na Mg A1 Si P S Cl

Gompo^d/s NaCl ^04 ^3 SiCl4 PP5.E4O10 SFg,S03 HO04,


Oddafiop staite of the + 1 + 2 + 3 + 4 + 5 + 6 + 7

w
i^derlihed. dement hi;
decompound

F lo
We shall now illustrate the use of above rules in determining the oxidation numbers and also for identifying
the reductants and oxidants in redox reactions.

Calculate the oxidation number of (i) S in H2S, (ii) C in CO2, (ui) C in

e
CH2Q2, (iv) N in and (v) P in Na3P04.

Fre
Solution, (i) S in H2S. Let the oxidation number of S in H2S be x. Writing the oxidation number of each
+1 ;c
atom above its symbol. H2 S (●.* Oxidation number of H is +1)

for
Sum of oxidation numbers of various atoms in H2S = 2(+1) + jc = 2 + jc
But the sum of the oxidation numbers of various atoms in H2S {neutral) is zero (Rule 7).
r
.*. 2 + X = 0 or X = - 2
You
Thus, the oxidation number of S in is - 2.
oks
(ii) C in CO2. Let the oxidation number of C in CO2 be x. Writing the oxidation number of each atom above
;c -2
eBo

its symbol. (Oxidation number of O is - 2)


CO2
.*. Sum of the oxidation numbers of various atoms in CO2 = jc + 2(-2) = jc-4
But the sum of oxidation numbers of various atoms in CO2 {neutral) is zero (Rule 7).
x-4 = 0 jc = + 4
our
ad

or

Thus, the oxidation number of C in CO2 is +4


(iii) C in CH2CI2. Let the oxidation number of C in CH2CI2 be x. Writing the oxidation number of each
X +1 -1
atom above its symbol. CH2CI2 (●.* Oxidation number of H is +1 and that of
dY
Re

Cl is - 1)
.*. Sum of the oxidation numbers of various atoms in CH2CI2 = x + 2 (+ 1) + 2 (- 1) = x
Fin

But the sum of the oxidation numbers of various atoms in CH2CI2 {neutral) is zero (Rule 7).
x=0

Thus, the oxidation number of C in CH2CI2 is zero


(iv) N in (NH4)2S04. Let the oxidation number of nitrogen in (NH4)2S04 be x. Writing the oxidation
number of hydrogen above its symbol and that of SO^” ion above its formula.
;c +1

(●.● Oxidation number of SO4 is - 2)


.-. Sum of oxidation numbers of all the atoms in (NH4)2S04 = 2x+2(+lx4) + (-2) = 2x + 6
But the sum of oxidation numbers of all the atoms in (NH4)2S04 {neutral) is zero (Rule 7).
.●. 2 X + 6 = 0 or X = - 3.

Thus, the oxidation number of nitrogen in (NH4)2S04 is - 3


REDOX REACTIONS 8/11

(v) P in Na3P04. Let the oxidation number of P in Na3P04 be x. Writing the oxidation number of each atom
+1 JC -2
above its symbol, Naj P O4 (●/ Oxidation number of Na is +1 and that of O is - 2)

Sum of the oxidation numbers of various atoms in Na3P04 = 3(+1)+a: + 4(-2) = x- 5


But the sum of oxidation numbers of all the atoms in Na3P04 {neutral) in zero (Rule 7).
X - 5 = 0 or JC = + 5

Thus, the oxidation number of P in Na3P04 is + 5

Calculate the oxidation number of (i) N in NO3 ; (ii) P in HjPjO^ ;

w
(iii) C in CO|- ; (iv) Cl in CIO4 ; (v) Cr in Cvfi]- ; (vi) Mn in MnO“ and (vii) Fe in [Fe(CN)g]'*-.
Solution, (i) N in NO3 . Let the oxidation number of N in NO3 be x. Writing the oxidation number of

F lo
-2

each atom above its symbol. N O3

Sum of the oxidation number of all the atoms in NO J ion = jc + 3 (- 2) = x - 6

ee
But the sum of oxidation numbers of all the atoms in NO3 ion is equal to the charge present on it, i.e. - 1

Fr
(Rule 8) jc-6 = -lorjc = + 5

Thus, the oxidation number ofN in NOJ is + 5

for
(ii) P in H3P2O7 . Let the oxidation number of P in H3P2O7 be x. Writing the oxidation number of each
ur
atom above its symbol.
+1 -2

H3 P3 07 (●.* Oxidation number of H is +1 and that of O is - 2)


s
ook
Yo
Sum of the oxidation numbers of all the atoms in H3P2O2 = 3 (+ 1) + 2 (a:) + 7 (- 2) or 2 x - 11
But the sum of oxidation numbers of all the atoms in H3P2O2 is equal to the charge present on it Le. - 1
eB

(Rule 8) 2x-11 =-1 orx = + 5

Thus, the oxidation number of P in H3P2O2 is + 5


(iii) C in COj". Let the oxidation number of C in COj" be x. Writing the oxidation number of each atom
r

X -2
ou
ad

above its symbol, C O3 Sum of the oxidation numbers of all the atoms in CO3 ion = x + 3 (- 2) = x - 6
But the sum of oxidation numbers of all the atoms in
COj" ion is - 2 (Rule 8) x - 6 = - 2 or x = + 4
Y

Thus, the oxidation state of C in COl" is + 4


(iv) Cl in CIO4. Let the oxidation number of Cl in CIO4 be x. Writing the oxidation numbers of each atom
Re
nd

above its symbol. Cl O4 Sum of oxidation numbers of all the atoms in CIO4 ion = x + 4 (- 2) = x - 8
Fi

But the sum of oxidation numbers of all the atoms in CIO4 ion is equal to the charge present on it, i.e., - 1
(Rule 8) .*. x-8 = -lorx- + 7

Thus, the oxidation number of Cl in CIO4 is + 7


(v) Cr in CrjO^. Let the oxidation number of Cr in CejO^" be x. Writing the oxidation number of each
-2
atom above its symbol. Cr2 O.J
Sum of the oxidation numbers of all the atoms in Cr202~ ion = 2 (x) + 7 (- 2) = 2 x - 14
But the sum of oxidation numbers of all the atoms in Cr^O^" is equal to the charge on it, i.e., - 2 (Rule 8)
2 X - 14 = 2 or X = 6

Thus, the oxidation number of Cr in ion is + 6


8/12 T‘>tadcefa.'4. New Course Chemistry (XI)CEIHD

(vi) Mn in MnO^. Let the oxidation number of Mn in MnO^ be .r. Writing oxidation number of each atom
-2
above its symbol, we get Mn O4
Sum of the oxidation numbers of all the atoms in MnO^ = a: + 4(-2)=j:-8

ow
But the sum of oxidation numbers of all the atoms in MnO^ is - I (Rule 8) /. x - 8 = - I or x = + 7
Thus, the oxidation number of Mn in MnO^ is+7
(vii) Fe in [Fe(CN)g]‘*“. Let the oxidation number of Fe in [Fe(CN)g]‘^ be x. Writing the oxidation number
of each atom above its symbol and that of cyanide ion above its formula, we get
-I

Fe (CN)g (●.● Oxidation number of CN" is - 1)

e
Sum of oxidation numbers of all the atoms in [FefCN)^]'^ = x + 6 (- 1)=j:-6
Fe(CN)g]'*“ is equal to - 4. (Rule 8}

re
But the sum of oxidation numbers of all the atoms in

rFl
X - 6 = - 4 or .V = + 2
4-
Thus, the oxidation number of Fe in lFe(CN )^J is +2.

F
r
ou
fo
1. Find the oxidation number of the element in bold in the following species :

ks
(i) SiH4, BH3. BF3, S2O2-, BrO“ and HPOj^ (ii) PbS04, U2O^-,B40^",Cr02-, K2Mn04.
2. Determine the oxidation number of C in the following : C^H^, C4HJQ, CO, CO^ and HCO3 .
oo
3. Determine the oxidation number of O in the following : OF2, Na202, Na20, KO2, KO3 and O2F2.
4. Find out tlie oxidation number of Cl in HCl, HCIO, CIO4 and CIO2.
Y
B

5. Find out the oxidation number of sulphur in the following species :

H2SO4, S205-.S202-,HSO- and HSO“.


re

6. Determine the oxidation number of all the atomsin the following well known oxidants KMn04, K2Ct207
and LiAlH4.
ou
Y

7. Determine the change in the oxidation number of S in H2S and SO2 in the following industrial reaction :
ad

2H2S (g) + S02 (g) 4 3S (.v) + 2H20 (g)


8. What is the oxidation number of S in 8202
d

ANSWERS

1. (0 Si = - 4 in SiH4, B = - 3 in BH3, B = + 3 in BF3, S = + 2 in 82©^ , Br = + 7 in 6104


in
Re

and P + 5 in HPO^" ,
F

(ii) S = + 6 in PbS04, U = + 5 in U2O7", B = + 3 in 8407“, Cr = + 6 in CrO^" and Mn = + 6 in K2Mn04-


2. O.N. of C = - 3 in CjH^, - 2-5 in C4H10, + 2 in CO. + 4 in CO2 and + 4 in HCOj .
3. O.N. of O = + 2 in OF2, - 1 in Na202 - 2 in Na-iO, - 1/2 in KO2, - 1/3 in KO3 and + 1 in 0'>F2-
4. O.N. of Cl = - 1 in HCl. + 1 in HCIO, + 7 in CIO" and + 4 in CIO2.

5. O.N. of S = + 6 in H2SO4 and HSO4 and S20|“, + 4 in HSO^ and + 3 in S2O4”*,


6. K = +I,Mn = + 7,0 = -2;K = +l,Cr = + 6,0 = -2;Li = +I,Al = + 3,H = -l.
7. O.N. of S changes from - 2 in H2S and + 4 in SO2 to zero in elemental sulphur.
-I +1 +1 -1
8. The O.N. of S in S2CI2 is + 1 as shown : Cl— S — S —Cl
REDOX REACTIONS 8/13

Retain in Memory
o
1. Oxidation number of metals in amalgams (Na-Hg, Zn-Hg, etc.) and in metal
O. .0
carbonyls [Ni(CO)4, Fe(CO)5, Cr(CO)g, etc.] is usually zero since they
exist in the elemental state. :cr:
O O
2. The oxidation number of Cr is + 6 and not + 10 in CrOj because four j: + 4(-l)+l(-2) = 0
oxygen atoms are involved in peroxide linkages. (0-0 (Cr = 0
bond) bond)
X = + 6

w
8.4.2. Oxidation and Reduction In terms of Oxidation Number
in terms of oxidation number,

F lo
Oxidation may be defined as a chemical change in which there occurs an increase in the oxidation
number of an atom or atoms while reduction may be defined as a chemical change in which there
occurs a decrease in the oxidation number of an atom or atoms. A redox reaction may then be
defined as a reaction in which the oxidation number of atoms undergoes a change.

ee
For example, consider the reaction between zinc and hydrochloric acid. Writing the oxidation number

Fr
of all the atoms above their respective symbols, we have,
I Oxidised 1

for
0 +1 -1 2+ -] 0
Zji + 2 H Cl Zn CI2 + H2
ur
— Reduced

In this reaction, the oxidation number (O.N.) of zinc increases from 0 to +2, that of hydrogen
s
ook
decreases from +1 to 0 while that of chlorine remains unchanged. Thus, zinc is oxidised while hydrogen
Yo
is reduced.

For further illustration, consider the following examples.


eB

I Oxidised

+ 1 -I 0 0 +I -I
(0 2H I + CI2 l2(^) + 2H Cl
our
ad

' Reduced -
Here, the oxidation number of iodine increases from 1 to 0, that of chlorine decreases from 0 to - I
while that of hydrogen remains unchanged. Therefore, HI is oxidised to I2 while CI2 is reduced to Cl~ ions.
Y

Oxidised
Re

+4 -2 0 0 +3 -2
nd

Hi) 3 Mn O2 + 4A1 3 Mn +
2AI2 O3
Fi

* Reduced
Here, the oxidation number of Mn decreases from +4 to 0, that of A1 increa.ses from 0 to +3 while that
of oxygen remains unchanged. Thus, Mn02 is reduced to Mn while Al is oxidised to A/2O3.
I Oxidised

+7 -2 -1 +1 +2 0 -2

Hii) 2Mn O4 + 10 Cl + 16 H 2Mn + 5CI2 + 8H2O


Reduced
t'
In this reaction, the oxidation number of manganese decreases from +7 in MnO^ to +2 in Mn-'*’ ions,
that of chlorine increases from — I in Cl“ ion to 0 in CI2 gas while that of oxygen and hydrogen remain
unchanged, Therefore, MnO^f is reduced while Cr is oxidised.
8/14 ‘P'uxdeap- 'a New Course Chemistry (XI) KRimn

8.43. Oxidising and Reducing agents in terms of Oxidation Number


Oxidising agents or Oxidants. Since oxidising agents are electron acceptors and acceptance of electrons
causes a corresponding decrease in the oxidation number, therefore, in terms of oxidation number,
i4/i oxidising agent or an oxidant may he defined as a substance the oxidi ion number of whose
atom (or atoms) decreases.
For instance, in the above examples, i.e., (/) to (Hi), the oxidation number of chlorine decreases from 0

low
in O', to - ) in Cl" ion ; that of managanese decreases from +4 in MnO^ to 0 in Mn and that of manganese
decreases from +7 in MnO“ to +2 in Mn^"^ ion. Therefore, all the three, i.e., C/2, Mn02 and MnO^ are
oxidising agents or oxidants. Similarly, the oxidation number of N decreases from +5 in KNO3 to +3 in
KNO2, therefore, KNO3 is an oxidising agent.
O.N. decreases-

e
+5 +3

2 K NO3 >
2K NO2 +
O2

re
rF
Likewise, KCIO3 is an oxidising agent since the oxidation number of Cl decreases from +5 in KCIO3 to

F
- I in Cl" ion.
O.N. decreases

or
+5 -I

2 K CIO3 > 2KC1 + 3 02


ou
sf
Reducing agents or Reductants. Since reducing agents arc electron donors and donation of electrons
causes a conesponding increase in the oxidation number, therefore, according to the oxidation number concept.
k
A reducing agent or a reluctant may be defined as a substance, the oxidation number of whose
atom (or atoms) increases.
oo
For instance, in the examples (/) to (Hi) discussed under Art. 8.4.2, page 8/14, the oxidation number of
Y
B

iodine increases from - 1 in HI to 0 to I2 ; that of aluminium increases from 0 in A1 to +3 in AI2O3 and that of
chlorine increases from - 1 in Cl” ion to 0 in CI2. Therefore, all the three, i.e., HI, Al and Ct ion are reducing
re

agents. Further consider the following redox reaction between H^S and HNO3.
I O.N. increases
u
ad

-2 +5 +2 0
Yo

H2S +
HNO3 ► NO + S + H2O
'— O.N. decreases 1
Here, the oxidation number of S increases from - 2 in H2S to 0 in elemental sulphur, while that of N
d

decreases from +5 in HNO3 to +2 in NO, therefore, H2S is a reducing agent while HNO^ in an oxidising
Re
in

agent.

Sample Prot^lem il Identify the oxidant and rcductant in the following reactions :
F

(a) 10 H'*’ (ag) + 4 Zn (s) + NO^iag) 4Zn^+ (ag) + NHj(fl^) + 3 H2O (/)
(b) I2 (5) + H2S (g) ^ 2 HI (g) + S is).
Solution, (a) Writing the O.N. of all the atoms above their symbols, we have.
+I 0 +5 +2 -3 + I -2

10H+(a^)+ 4Zn(5) +N03(a<?) > 4Zn^*(aq)+ NHj(o^) +3H2 0(/)


Here, there is no change in the O.N. of H and 0-atoms. But the O.N. of Zn changes from zero in Zn to +2 in
Zn^'*', therefore, it is oxidised and hence Zn acts as a reductant.
The O.N. of N decreases from +5 in NOJ to - 3 in NH| and, therefore, it is reduced and hence NO^ acts
as the oxidant.
REDOX REACTIONS 8/15

(/’] Writing the O.N. of all the atoms above their symbols, we have,
0 + ! -2

^ 2H l\g) + S(.v)
0

U(g)+H^Sig)
Here, O.N. of H does not change. The O.N. of I2 decreases from zero in to - 1 in HI, therefore, I2 is
reduced and hence it acts as an oxidant. The O.N. of
S increases from - 2 in H2S to zero in S, therefore, H^S is
oxidised and hence it acts as the reductant.

ow
SUPPLEMENT YOUR
KNOWLEDGE FOR COMPETITIONS

Equivalent weights of oxidising and reducing agents. The equivalent weights of oxidising and reducing
agents can be calculated by the number of electrons gained or lost. Thus, the equivalent weight of an
oxidising/reducing agent is equal to its molecular weight divided by the number of electrons gainedAost
as represented in the balanced chemical equation.
Further, we know that during oxidation, the O.N. of a particular atom of the oxidising agent decreases

e
while during reduction, the O.N. of a particular atom of the reducing agent increases.

Fl
re
Therefore, equivalent weight of an oxidising/reducing agent is equal to its molecular weight divided by
the total change in the O.N. of a particular atom of the oxidising/reducing agent. For example,

F
1. Equivalent weight of KMn04
+7 +2

(i) in acidic medium Mn O4 + 8 H'*’ + 5 - Mn^+ + 4H.,0


ur
r
Total number of electrons gained = 5

fo
Total change in the O.N. of Mn = 7 - 2 = 5
Mol.wt.of MnOT 55+4x16 119
Eq. wt. of MnO^ = 5
^ 5
ks
5
= 23-8
Yo
Mol. wt.of KMnO^ 39 + 55 + 4x16 158
oo
or
Eq. wt. of KMn04 = 5 5 5
= 31-6

(ii) In neutral or alkaline medium


B

+7 +4

MnO^+2H.,0 + 3e" 4 Mn0.,+40H-


re

Total number of electrons gained = 3


Total change in O.N. of Mn = 7-4 = 3
u
ad

Mol. wt.of MnO^ 55 + 4x16 119


Yo

Eq. wt. of MnO^ = 3 3 3


= 39*66

Mol. wt. of KMnO^ _ 39 + 55 + 64 158


or
Eq. wt. of KMn04 “ - =52*66
d

3 3
Re

2. Equivalent weight of K2Cr207


in

+6 +3

Cr2 02- + I4H-^+6e" — > 2Cr^++7H20


F

Total number of electrons gained = 6


Total change in O.N. of Cr = 2 (+ 6) - 2 (+ 3) = 6
Mol.wl.of Cr207~ 2x52+7x16 216
Eq. wt. of C.r,07 = 6 6
= 36
6

MoI.wt.of K^Cr^O^ 2x39 + 2x52 + 7x16 294


or
Eq. wt. of K2Cr207 = = 49
6 6 6
3. Equivalent weight of H2O2
-1 0

H2O2 > 2H'*' +02 + 2e~ .(acidic medium)


H2O2 + 2 OH- + 2 H2O + O2 + 2 (basic medium)
8/16 ‘P’uide^'4- New Course Chemistry (XI)K2SMD

Total number of electrons lost = 2


Total change in the O.N. of 0 = 0 - 2 (- 1) = 2
Mol. wt. of H2O2 _ 2 + 32 34
— = 17
Eq. wt. of H2O2 = 2 2 2
4. Equivalent weight of oxalic acid, (COOH>2
+3 +4

C2H2O4 ^ 2CO, + 2H++2e'

w
Oxalic acid
Total number of electrons lost = 2

Total change in the O.N. ofC = 2x4-2x3 = 2


Mol.wt.ofC.,H204 _ 24+2 + 64 90

Flo
Eq. wt. of oxalic acid = — =45
2 2 2

5. Equivalent weight of FeS04

ee
+2 +3

FeS04+l/2H2S04 - +
1/2 Fc2(804)3+ H++C- or Fe2+ ■> Fe^"^ + e~

Fr
Total number of electrons lost = 1
Total change in O.N. of Fe = 3 - 2 = 1
Mol. wt. of FeS04 _ 56 + 32 + 4 x 16 = 152
Eq. wt. of FeS04 =

for
1 1
ur
8.4.4. Distinction between Valency and Oxidation Number
The term oxidation number and valency have different meanings as discussed below :
ks
Yo
Valency Oxidation Number
oo
1. Valency is the combining capacity of an element. 1. Oxidation number is the charge which an atom has
It is defined as the number of hydrogen atoms or or appears to have when present in the combined
eB

double the number of oxygen atoms with which state.

an atom of the element combines.

2. Valency is only a number. As such it does not 2. Since the oxidation number refers to the charge,,
it can be positive or negative. For example, in H2O,
r

have plus or minus signs attached to it. For


the oxidation number of oxygen is - 2 and that of
ou

example, in H2O the valency of oxygen is two


ad

and that of hydrogen is one. hydrogen is +1.


3. Valency of an element cannot be zero. 3. Oxidation number of an element can be zero. For
Y

example, the oxidation number of carbon in


CH2CI2 is zero.
d

4. Since atoms always combine in simple whole 4. In reality no element can ha\'e a fractional oxidation
Re

numbers, therefore, valency of an element is number. But if in a compound, the same elementexists
n

always a whole number. in two different oxidation states, then its average
Fi

oxidation number may have fractional value. For


example, the average oxidation number of Fe in Fe304
is + 8/3 and that of S in Na2S40(^ is + 2-5.
5. In some cases, the valency of an element is fixed 5. The oxidation number of an element may be
in all its compounds. For example, the valency different in different compounds. For example, the
of C is 4 in all its compounds viz. CH4, C2H^, oxidation number of carbon is - 4 in CH4 - 3 in
CH3CI, C2H4C2H2, CH2CI2, CHC13 and CC^. C2H6, - 2 in CH3CI, - 1 in C^H,, zero in CH2CI2,
+ 2 in CHCI3, + 3 in C2CI6 and + 4 in CCI4.
6. The valency of an element may be variable but 6. The oxidation number of an element may also be
the variation is limited to only two values. For variable but the variation is too large. For example
example, the valency of nitrogen is either 3 or 5 oxidation number of N is +1 in N2O, +2 in NO, +3
in all its compounds, viz., N2O, NO, N2O3, NO2, in N-,03, +4 in NO2, +5 in N2O5, 0 in N2, - 1 in
N2O5, N,. N2H2, N2H4 and NH3. N2H2, - 2 in N2H4 and - 3 in NH3.
REDOX REACTIONS 8/17

8.4.5. Oxidation Number and Nomenclature


The compounds of metals which show more than one oxidation slates are distinguished from one another
by placing a Roman numeral such as I, II, HI, IV, V, VI, VII, etc. indicating the oxidation state of the metal
within parenthesis after the symbol or name of the metal. For example, copper forms two oxides, i.e., CU2O
and CuO. In CU2O, the oxidation number of copper is +1 while that in CuO, it is +2. Therefore, these two
oxides are distinguished as Cu2(I)0 and Cu(II)0 oxide respectively. This system of nomenclature was
introduced by Alfred Stock, a German chemist, and is commonly known as Stock notation after his name. As
an illustration, consider the stock notations for the following compounds:

Formula and chemical name of the compound Stock notation

w
CU2CI2, Cuprous chloride Cu2(I)Cl2
CUCI2, Cupric chloride Cu(II)Cl2

F lo
FeS04, Ferrous sulphate Fe(II)S04
Fe2(S04)3. Ferric sulphate Fe2(ni)(S04)3
Cr203, Chromium trioxide Cr2(III)03
Na2Cr04, Sodium chromate Na,Cr(VI)04
V2O5 Vanadium pentoxide V,(V)05

e
Fre
K2Cr20-7, Potassium dichromate K2Cr2(VI)07
K2Mn04, Potassium manganate K2Mn(VI)04
KMn04, Potassium permanganate KMn{Vn)04

for
Mn207, Manganese heptoxide Mn2(VII)07
However, in case of compounds of non-metals which show variable oxidation states, stock notation is
r
not generally used. For example, two halides of phosphorus such as PCI3 and PCI5 having phosphorous in i:_
oxidation state of -»-3 and +5 respectively are distinguished by names only, i.e., phosphorus trichoride and
You
ks
phosphorus pentachloride respectively.
o

8.4.6. Types of Redox Reactions


eBo

Some important types of redox reactions are discussed below :


1. Combination reactions. A reaction in which two atoms or molecules combine together to form a
third molecule is called a combination reaction. For example, A + B >C
our
ad

For this reaction to be a redox reaction, either one or both A and B must be in the elemental form. All
combustion reactions in which elemental oxygen is used and all other reactions which involve elements other
than oxygen are redox reactions. Some important examples of this category are ;
0 0 +4 -2 -4 +1 0 +4 -2 + 1 -2
(0 C(s) + 0^ (g) ^ C 02(g) 07) C H^(g) + 202(g) —>C 02(g) + 2H2 0(/)
dY
Re

0 +2 -3 0 0 +2 -2

(m) 3Mg(^) + N2(g) 4 Mg3 N2(.s) (iV) 2 Mg (5) +02(g) 2MgO(5)


Fin

0 0 +2 -2
(V) Fe(j) + S(5) > Fe S (3)
2. Decomposition reactions. A reaction in which a molecule breaks down to form two or more
components is called a decomposition reaction. All decomposition reactions are reverse of combination
reactions. Further, all decomposition reactions are not redox reactions. For a decomposition reaction to be a
redox reaction, it is essential that one of the products of decomposition must be in the elemental state. For
example, consider the following reactions :
+1 -2 A 0 0
+1-1 A 0 0
(0 2H2 0(0 > 2H2(g) + 02(g) Hi) 2NaH(5) > 2Na(5)-hH2 (g)
+ 1 +5 -2 ^ +1 _| 0
(Hi) 2K Cl 0^(s) » 2K Cl (^) +3 02(g)
8/18 P>tadeep,*^ New Course Chemistiy (Xl)nadil

In all these reactions, either both [H2 and O2 in reaction (0, Na and H2 in reaction (n)] or one [O2 in
reaction {Hi)] are in the elemental state and hence all these decomposition reactions are redox reactions.
Now let us consider decomposition of calcium carbonate to give calcium oxide and carbon dioxide

CaCOj (5) CaO (s) + CO2 (g)


Since in this reaction, both the products are not in the elemental form, therefore, it is not a redox reaction.
3. Displacement reactions. A reaction in which an atom or ion in a compound is replaced by an atom

w
or ion of some other element, is called a displacement reaction. In general, it is represented by the equation,
X + YZ >XZ + Y
Here, from the compound YZ, the atom Y has been displaced by another atom X.
Types of displacement reactions. Displacement reactions are of the following two types :

Flo
(a) Metal-displacement reactions (b) Non-metal displacement reactions
(a) Metal displacement reactions. In these reactions, a metal in the compound is displaced by some

e
other metal in the elemental state. For example,

re
+2 +6 -2 0 0 +2 +6 -2

(i) Cu S O4 (aq) + Zn (s) Cu (5) + Zn S O4

rF
Copper sulphate Zinc Copper Zinc sulphate
+5 -2 . +2-2
2V(^) + 5CaO(5)
(,i) VjOj + 5Ca(i) —
ur
fo
Vanadium pentoxide Calcium Vanadium Calcium oxide
0 +2-1
+ 2 Mg (5) > Ti(^) + 2MgCl2(/)
(«0
ks
Utanium tetrachloride Magnesium Titanium Magnesium chloride
Yo
oo
+3 -2 0 A 0 +3-2
(iv) Ct^ O3 (^) + 2Al(s) ^ 2Cr(5) -t- Al2 03(5)
B

Chromium trioxide Aluminium Chromium Aluminium oxide

In each of these reactions, the reducing metal [i.e., Zn, Ca, Mg and Al] are better reducing agents than
re

the metal being reduced [i.e., Cu, V, Ti and Cr respectively]. In other words, the reducing metal has greater
ability to lose electrons as compared to the one being reduced.
(b) Non-metal displacement reactions. In these reactions, a metal or a non-metal displaces another
u
ad

non-metal from its compound. In most of these reactions, the non-metal displaced is hydrogen. However,
Yo

some reactions involving displacement of oxygen or halogens are also known. Depending upon the capability

of the reducing metal or non-metal, the following cases arise:


(0 All alkali metals and some alkaline earth metals (Ca, Sr and Ba) which are very good reducing
nd

agents displace dihydrogen from cold wate.r


Re

0 +1 -2+1 0

2Na(j)-^2H2 0(/) 2NaOH(a^)-hH2(g)


Fi

0 +1 -2 2+ +2 +1 0

Ca (s) + 2H2 O (/) ^ Ca (O H)2 (aq) + H^ig)


(it) Less active metals such as magnesium and iron react with steam to produce dihydrogen gas :
0 0

Mg(5) + 2H2 0(g) 4 Mg(0 H)2(a^) + H2(g)


+1 -2 +2,+3 -2 0

3Fe(5)-i-4H2 0(g) » Fe3 04(^) + 4H2(g)


(Hi) Many metals, including those which do not react with cold wate,r are capable of displacing
dihydrogen from acids. For example.
0

+ ^C\2(aq) + U^(g)
0 +1-1

Zn(j) + 2HCl(a^)
I

REDOX REACTIONS
8/19

0 +2 -1 0
Mg(5) + 2HCl(a^) > MgCl2(fl^) + H2(^)
0 +1-1 +2 -1 0

Fe(j) + 2HCl(fl^) > FeCl2 {aq) + Yi^ig)


The reactions of Zn and Mg with HCl are used to prepare dihydrogen in the laboratory.
Metals like cadmium and itn which do not eract with steam also eract with acids ot displace dihydiogen gas.
0 +1-1 0
Cd(5) + 2HCl(fl^) »CdCl2(a^) + H2(g)

w
0 +1-1 +2 -1 0
Sn (j) + 2HCl(a^) ^ SnCl2(a^) + H2(g)
(iv) Very less reactive metals such as silver (Ag) and gold (Au) which may occur in the native state do

F lo
not react even with dilute hydrochloric acid.
Reactivity of metals. From the above discussion, it follows that the rate of evolution of H2 by metals
from water and aqueous acids can be used to determine the order of reactivity of metals. For example, sodium
(Na) reacts with water at the fastest rate, magnesium (Mg) reacts slowly, iron (Fe) reacts at the slowest rate

ee
while silver (Ag) and gold (Au) do not react at all.

Fr
Further, it has also been discussed in Art. 8.3.1 on page 8/8 that the reducing ability of metals such as
zinc (Zn), copper (Cu) and silver (Ag) follows the order : Zn > Cu > Ag.
Reactivity of non-metals. Like metals, activity series also exists for non-metals. Since non-metals

for
have a tendency to accept electrons, therefore, this reactivity depends upon their oxidising power. The oxidising
ur
power of halogens decreases as we move down the group 17 from fluorine to iodine. Thus, fluorine (Fj) is the
strongest oxidising agent. It displaces CI2, Br2 and I2 from the solution of chloride, bromide and iodide ions
respectively. In fact, F2 is so reactive that it even displaces oxygen from water.
s
ook
0
Yo
0

(0 2F2(g) + 2H2 0(/) + 4H F(a^) + 02(g)


It is because of this reason that the displacement reactions of chlorine, bromine and iodine using fluorine
eB

are not earned out in aqueous solutions. On the other hand, chlorine can displace bromine from bromide ions
and iodine from iodide ions.
0

(/0Cl2(g) + 2KBl-i (aq)


+1 -1 0
our

» 2K Cl(a^) + Bi^(/)
ad

0 +1 -1 +1 -1 0
(«0Cl2 (g) + 2KrHaq) > 2K Cl(a^) + l2(j)
Their corresponding ionic equations are :
Y

0 -1 -1
Re

(iv) Cl2(g) + 2Br-(a^) > 201" (a^) + Br2(/)


nd

0 -1

(V) Cl2(^) + 2I-(a^) > 2Cl-+l2(5)


Fi

Reactions (iv) and (v) form the basis of identifying Br“ and I~ ions in the laboratory by carbon tetrachloride
(CCI4) or carbon disulphide (CS2) test popularly known as ‘Layer Test’.
Similarly, bromine can displace iodine from iodide ions.
0 -1 -1 0

(Vi) Br2(/) + 2r(a^) > 2Br"(a^) +12(5)


The above halogen displacement reactions, i.e., (iv), (v) and (vi) have been widely used in industry.
Instead of using one halogen to oxidise another halide ion, halogens can also be prepared by oxidation of
their corresponding halide ions using suitable chemical oxidising agents.
Oxidising agent
2X-
X2 + 2 ^
8/20 T^ftauUc^’4. New Course Chemistry (XI)EZ5IHD

Although a number of oxidising agents such as KM11O4, K2^r207, Mn02, etc. are available to oxidise
Cr, Br" and 1” ions to form CI2, Br2 and I2 respectively no oxidising agent is available to oxidise F' ions to
F2 because F2 itself is the strongest oxidising agent. Therefore, the only way to prepare F2 is to oxidise F“
ions electrolytically.
4. Disproportionation reactions. A reaction in which the same species is simultaneously oxidised as
well as reduced is called a disproportionation reaction. For such redox reactions to occur, the reacting
species must contain an element which has at least three oxidation states. The element in the reacting species
is present in the intermediate oxidation state while the higher and lower oxidations states are available for
oxidation and reduction to occur. For example, the decomposition of hydrogen peroxide {H2O2) is a

w
disproportionation reaction where oxygen atom undergoes disproportionation.
I Reduced

+1 -2 0

F lo
+1 -1

2 H2 O2 (ag) ► 2 H2 O (/) + 02(^)


Oxidised '
In this reaction, the O.N. of oxygen which is -1 in peroxide ion increases to zero in O2 and decreases to

ee
-2 in HoO.
Phosphorus, sulphur and chlorine also undergo disproportionation reactions in alkaline medium as shown

Fr
below :
Reduced

for
0 -3 +1
ur
(0 P4(^) + 3 0F[ (ag) +
3 H2O (/) ► PH3 (g) + 3 H2PO2 (ag)
●Oxidised
t
s
Reduced
ook
Yo
0 +2.5

(ii) S8(^-) + 12 0H~ (ag) ► 4 (ag) + 2 $2 O3 (ag) +


6H2O (1)
eB

Oxidised
Reduced
1
0 +1 -1
Cold
► CIO (ag) + Cl (ag) + H2O (1)
r

(Hi) C\2(g) +
2 OH" (ag)
ad
ou

Oxidised —
t
— Reduced
I
Y

0 +5

(/V) 3 CI2 (g) 4-


6 0H"(fl^) ^—*'5Cr(ag) +
ClOJ(a^) + 3H20(/)
Re
nd

Oxidised ^

The reaction (Hi) describes the preparation of household bleaching agents. The hypochlorite ion
Fi

(CIO") acts as the bleaching agent by oxidising the colouring matter to colourless compounds.
Like chlorine, bromine and iodine also undergo similar disproportionations reactions, i.e., (Hi) and (iv) but
fluorine does not. The reason for this anomalous behaviour is that fluorine being the strongest oxidising agent does
not show positive oxidation states. It, therefore, reacts in a different way forming oxygen difluoride (OF2)
-1 +2 -I
0

2F, (g) + 2 0H (ag) 4 2F"(a^)+0 F^(g) +H20(/)

8.4.7. Th? Paradox of Fractional Oxidation States


During our study of redox reactions, we come across many compounds in which the oxidation number
of a particular element in the compound is fractional. For example, the oxidation number of C is C3O2
(carbon suboxide) is 4/3 that of Br in BrjOg (tribromooctaoxide) is 16/3 and that of S in Na2S406 (sodium
tetrathionate) is 2.5.
REDOX REACTIONS 8/21

Since electrons are never transferred or shared in fractions, therefore, the very idea of fractional oxidation
states is unconvincing. In fact, fractional oxidation state is only the average oxidation state of an element
when two or more of its atoms are present in different oxidation states in a given compound.
In such cases, the actual oxidation states of the element can be determined by knowing its structure. For
, , -2 +2 0 +2 -2
example, the structure of C3O2 (carbon suboxide) isO = C= C= C = 0
Since a double bond is formed by mutual sharing of two electrons and since oxygen is more electronegative
than carbon, therefore, each of the two terminal oxygen atoms have an oxidation state of
- 2 and the carbons to which they are attached have an oxidation state of + 2. Further, since whenever a
covalent bond is formed between two similar atoms (or between two similar atoms which are further attached

w
to similar species) each of the atoms is given an oxidation state of zero, therefore, the oxidation state of
central carbon atom in C3O2 is zero.
Thus, the two terminal carbon atoms are present in + 2 oxidation state each, whereas the third one is present

F lo
in zero oxidation state. Therefore, the average oxidation state of carbon in C3O2 = (2 x 2 + 1 x 0)/3 = 4/3.
However, in reality, the terminal carbon atoms have an oxidation state of+2 while the middle carbon
has an oxidation state of zero.
Let us now consider the structure of tribromooctaoxide (Br30g).

ee
O

Fr
O O
^.+6 +4 +6^
0=Br—Br—Br==0

for
O O
o

Following arguments discussed under C3O2, we can easily find out that the terminal bromine atoms are
ur
present in + 6 oxidation state while the middle one has an oxidation state of + 4. However, the average
oxidation number of Br in Br30g turns out to be [2 (+ 6) + 1 (+ 4)]/3 = 16/3.
s
Lastly, let us consider the structure of tetrathionate ion.
ook
Yo
O O
+5l 0 0 I +5
eB

“O—S—S—S—S—O"

o o
Since each of the two terminal sulphur atoms is cormected to two oxygen atoms by a double bond and
our

one oxygen atom by a single bond, therefore, the oxidation state of each of these terminal sulphur atoms is
ad

+ 5. Since two central sulphur atoms are linked to each other by a single bond and each sulphur is further
attached to similar species on either side, the electron pair forming the S—S bond remain in the centre and
hence each of the two central sulphur atoms has an oxidation state of zero. However, the average oxidation
Y

state of the four sulphur atoms is (2 x 5 + 2 x 0)/4 = 2-5.


Re

From the above discussion, we conclude that the idea of fractional oxidation state is only a myth while
nd

the actual oxidation states are revealed by the structures only. Further, whenever we come across with fractional
oxidation state of any particular element in any species, we must understand that it is the average oxidation
Fi

number only. In reahty (revealed by structures only), the element in that particular species is present in two or
more whole number oxidation states. For example, Fe304 (magnetic oxide) is known to be a mixture of FeO
and Fe203 having the composition, FeO.Fc203. Therefore, oxidation number of Fe in FeO is + 2 while in
Fe203 it is + 3. The average value, however, is [1 (+ 2) + 2 (+ 3)]/3 = 8/3.
Likewise Mu304 is regarded as a mixture of MnO and Mn02 having the composition, 2 Mn0.Mn02-
Therefore, the oxidation state of Mn in MnO is + 2
while that in Mn02 it is + 4. The average, however, is
[2(+2)+l(+4)]/3 = 8/3.
Similarly, red lead (Pb304) is considered to be a mixture with the composition, 2 PbO.Pb02. Therefore,
the oxidation state of Pb in PbO is + 2 while in Pb02, it is + 4. Here, again the average value is [2 (+ 2) +
l(+4)]/3 = 8/3.
8/22 New Course Chemistry (XI)EEIHn

Curiosity Question
r Q. Usually in a redox reaction, one substance is oxidised and the other is reduced. Can you
think of an inorganic compound which undergoes Intramolecular redox reaction ?
Ans. Intramolecular redox reaction can occur if the cation is oxidised and the anion is reduced. For
example, decomposition of ammonium dichromate may be regarded as an intramolecular redox
reaction in which O.N. of N increases from - 3 to 0 and that of Cr decreases from + 6 to + 3, i.e.,

NH^ is oxidised to N2 whiie Cr20j~ is reduced to Cr203

w
-3 +6
Heat 0 +3 . u /-»
(NH4)2Cr207 N2+Cr203 ^ ^2®
J

Flo
8.5. BALANCING OF CHEMICAL EQUATIONS OF REDOX REACTIONS
According to the ‘Law of Conversation of Mass’, each chemical equation must be arithmetically balanced,
i.e., the number of atoms of each element on both sides of the chemical equation must be equal. Two methods

ee
which have been used to balance all types of chemical equations are
(i) Hit and Trial Method {ii) Partial Equation Method

Fr
However, chemical equations which involve oxidation and reduction {i.e., redox reactions) can more
easily be balanced with the help of the following two methods :
I. Oxidation Number Method

for
ur
II. Half Equation Method or Ion-Electron Method
I. Oxidation Number Method
The various steps involved in the balancing of redox equations by oxidation number method are :
s
Step 1. Write the skeletal equation of all the reactants and products of the reaction.
ok
Yo
Step 2. Indicate the oxidation number of each element above its symbol and identify the elements which
undergo a change in the oxidation number (O.N.).
Bo

Step 3. Calculate the increase or decrease in oxidation number per atom and identify the oxidising and
reducing agents. If more than one atom of the same element is involved, find out the total increase or decrease
in O.N. by multiplying this increase or decrease in O.N. per atom by the number of atoms undergoing that
re

change.
Step 4. Multiply the formulae of the oxidising and the reducing agents by suitable integers so as to
ou

equalise the total increase or decrease in o.xidation number as calculated in step 3.


ad

Step 5. Balance all atoms other than H and O.


Y

Step 6. Finally balance H and O atoms by adding HjO molecules using hit and trial method.
Step 7. In case of ionic reactions,
(a) For acidic medium. First balance O atoms by adding H2O molecules to whatever side deficient in
nd
Re

O atoms and then balance H atoms by adding H'*' ions to whatever side deifcient in H atoms,
(b) For basic medium. First balance O atoms by adding H2O molecules to whatever side deficient in O
Fi

atoms. The H atoms are then balanced by adding H2O molecules equal in number to the deficiency of H atoms
and an equal number of OH~ ions are added to the opposite side of the equation. Remove the duplication, if any.
These rules are illustrated by the following examples.
Sample Problem □ Balance the equation,
Mg (aq) + HNO3 (aq) > Mg (N03)2 {aq) + N^O (g) + H2O (/)
Solution. Step 1. Find out the elements which undergo a change in oxidation number (O.N.)
I O.N. increases by 2 per Mg atom
0 + 1 +5 -2 +2 +5 -2 + I -2 + 1 -2

Mg +
H N O3 > Mg (N 03)2 +
N2 O +
H2O ...(/)

O.N. increases by 4 per N atom


REDOX REACTIONS 8/23

Here, O.N. of Mg increases from 0 in Mg metal to +2 in MgCNO,)^ and that of N decreases from +5 m
L
HNO3 to +1 in NjO.
Step 2. Find out the total increase and decrease in O.N.

Since there is only one Mg atom on either side of Eq. (/), therefore, total increase in O.N. of Mg is 2. Furthe,r
since there are two N atoms in N2O on R.H.S. and only one in HNO^ on L.H.S. of Eq. (i), therefore, multiply
HNO3 on L.H.S. of Eq. (/) by 2 and thus the total decrease in O.N. of N is 2 x 4 = 8.
Step 3. Balance increase/decrease in O.N.
Since the total increase in O.N. is 2 and decrease is 8, therefore, multiply Mg on the L.H.S. of Eq. (/) by 4.
Combining steps 2 and 3, we have.
4 Mg (f) + 2 HNO3 (aq) > Mg(N03)2 (aq) + N2O (g) + H2O (/) .(«●)

w
Step 4. Balance all atoms other than O and H.
To balance Mg on either side of Eq. («), multiply Mg(N03)2 by 4, we have,

F lo
4 Mg (j) + 2 HNO3 {aq) > 4 Mg(N03>2 {aq) + N2O (g) + H2O (/) .{Ui)
Now, there are 10 nitrogen atoms on R.H.S. of Eq. {Hi) and only 2 on L.H.S., therefore, to balance N-atoms,
change the coefficient of HNO3 from 2 to 10 on L.H.S. of Eq. (»7), we have,
4 Mg {s) + 10 HNO3 {aq) > 4 Mg(N03>2 + N2O (g) + H2O (/) m

ee
Step 5. Balance O and H atoms by hit and trial method.

Fr
Since there are 30 oxygen atoms on L.H.S. but only 26 oxygen atoms on R.H.S. of Eq. (iv), therefore, to
balance O atoms, change the coefficient of H2O from 1 to 5, we have,

for
4 Mg {s) + 10 HNO3 {aq) > 4 Mg(N03)2 {aq) + N2O (g) + 5 H2O (/) {V)
The H atoms get automatically balanced. Thus, Eq. (v) represents the correct balanced equation.
r
i.ampIS
Balance the oxidation reduction reaction, FeS2 + >^0203 + SO2
You
Solution. This is an example of a reaction which occurs in absence of acids and bases and hence balancing
s
ook
of O atoms cannot be done by addition of H2O molecules but has to be done on the basis of gain or loss of
electrons. To balance such reactions, following steps are followed :
eB

Step 1. Identify atoms whose oxidation numbers undergo a change. +2 -1 0 +3 -2 44 -2


Writing the oxidation number of each atom above its symbol, we have, FeSj + 62 4 Fcj O3 + S O2
Here, the oxidation number of Fe has increased from +2 to +3 and that of S has increased from -1 to +4
while that of O has decreased from 0 to -2. In other words, both Fe and S of FeS2 have been oxidised. Since Fe
our
ad

and S must maintain their atomic ratio of 1:2, therefore, the change ofoxidation number of these two atoms must
be considered together.
Step 2. Determine the total increase and decrease in oxidation numbers.
Indicating the increase and decrease in oxidation numbers in each case, we have.
dY
Re

+2 43
Fe > Fe It
= 11 T
Fin

-1 44 .(«■)
S2 »2S lot
2-
and
O2 4 20 4i .{Ui)
Step 3. Balance the total increase and decrease in oxidation numbers.
To balance the total increase in oxidation number of Fe and S and decrease in oxidation number of O,
multiply Eq. (li) by 4 and Eq. {Hi) by 11, and adding, we have.
42 -1 43 44 2-
4[Fe4-S.]4-110, 4 4Fe4-8 S + 220 (iv)
2- ^ ^ 2-
Since O does not occur independently so the 22 O must be factorized in such a manner that they become
parts of Fc203 and SO2. Rearranging, Eq. {iv), we have.
T^ftadecfi. New Course Chemistry (XI) ESZ

u
8/24

+2 -1 +3 2- +4 2-

4[Fe +821+1102 4 [4Fe + 6 0]+[8S +16 0]

o
or 4FeS2+1102 >2Fe203 + 8S02
This represents the required balanced equation,
n. Half-Reaction Method or Ion Electron Method
This method of balancing redox equations is based upon the principle that electrons lost during oxidation
half reaction of any redox reaction are equal to the electrons gained during reduction half reaction. The various

Fo
steps involved in this method are :

k
Step 1. Write the skeletal equation and indicate the oxidation number (O.N.) of all the elements which
appear in the skeletal equation above their respective symbols.

l
r
Step 2. Find out the species which are oxidised and which are reduced.
Step 3. Split the skeletal equation into two half reactions, i.e., oxidation half reaction and reduction half

o
reaction.

o
Step 4. Balance the two half reaction equations separately by the rules described below :

Y
(0 In each half reaction, first balance the atoms of the elements which have undergone a change in oxidation
number.

o
Y
f
(h) Add electrons to whatever side is necessary to make up the difference in oxidation number in each half
reaction.
(ill) Balance charge by adding ions if the reaction occurs in the acidic medium and by adding OH“ ions
if the reaction occurs in the basic medium, B
(iv) Balance oxygen atoms by adding required number of H2O molecules to the side deficient in O atoms.
(v) In the acidic mpHinm, H atoms are balanced by adding ions to the side deficient in H atoms. Howeve,r in
r
the basic niedinm, H atoms are balanced by adding H2O molecules equal in number to the deficiency of H atoms and
d
an equal number OH~ ions are included in the opposite side of the equation. Remove the duplication, if any.
Step 5. The two half reactions are then multiplied by suitable integers so that the total number of electrons
u
e

gained in one half reaction is equal to the number of electrons lost in the other half reaction. The two half
reactions are then added up. These rules are illustrated by the following examples.
n

Step 6. Verification. To verify whether the equation thus obtained is balanced or not, the total charge on
either side of the equation must be equal.
io

tsamDi&lRrogr^iriiin Dichromate ion in aqueous acidic medium reacts with ferrous ion to give
ad

ferric and chromium ions. Write the balanced chemical equation corresponding to the reaction.
Solution. Step 1. Write the skeletal equation of the given reaction.
Ctplj (fl^) + Fe2+(a^) C?*(.aq) + Ft^*iaq) .(0
F

Step 2. Write the O.N. of all the elements above their respective symbols.
Re

O.N. decreases by 3 per Cr atom

+6 -2 +3

Cx20Y{aq) + Fe^* {aq) > C?*{aq) +

O.N. increases by 1 per Fe atom

Step 3. Find out the species which have been oxidised and reduced and split the given skeletal equation into
two half reactions.
Since the O.N. of Cr decreases from + 6 in Cr^+ and that of Fe increases from + 2 in Fe^+
to + 3 in

to + 3 in Fe^+ therefore, Ci^O^” gets reduced while Fe^+ gets oxidised. Thus, the above skeletal equation (i) can
be divided into the following two half reaction equations :
Oxidation half equation : Fe^"*" {aq) > Fe^"*" {aq) .(«)

Reduction half equation : CtpYiaq) » C^(aq) m


REDOX REACTIONS 8/25

Step 4. To balance oxidation half equation (ii).


(a) Balance all atoms other than O and H.
Not needed because Fe is already balanced.
(b) Balance the O.N. by adding electrons.
The O.N. of Fe on L.H.S. of Eq. (//) in Fe^"^ is + 2 while on the R.H.S. in Fe^+, it is + 3. Therefore, add one

e~ to R.H.S. of Eq. {ii), we have.


■» (fiq) + e~ ..(iV)
(c) Balance charge by adding ions. Not needed since the charge is already balanced. Thus, Eq. (iv)
represents balanced oxidation half reaction.
Step 5. To balance the reduction half equation (Hi),

w
(a) Balance all atoms other than H and O.

Since there are two Cr atoms in Crp^~ on the L.H.S. of Eq. (Hi) and only one on R.H.S., therefore,

F lo
multiply Cr^+ by 2, we have, Ctplj~{aq) > 2 Cr^+ (aq) .(V)
(b) Balance the O.N. by adding electrons.

The total O.N. of two Cr atoms in Crplf on the L.H.S. of Eq.

ee
(v) is 2 x 6 = 12 while on the R.H.S., it is

Fr
2x3 = 6. Therefore, add 6 e~ to L.H.S. of Eq. (v), we have, (aq) + 6e~ > 2 Cr^'*’ (aq) .(VI)
(c) Balance charge by adding ions since the reaction occurs in the acidic medium.

for
The total charge on L.H.S. of Eq. (vi) is - 8 while on the R.H.S., it is + 6. Therefore, add 14 H+ to L.H.S. of
Eq. (vi), we have.

Ctplf (aq) + 14 H+ (aq) + 6 e~ > 2 Cr^^ (aq)


r
..(vii)
You
(d) Balance O atoms by adding H2O molecules. Since there are seven O atoms on the L.H.S. of Eq. (vii) but
s
no O atom on the R.H.S., therefore, add 7 H2O to the R.H.S. of Eq. (vii), we have,
ook

Crplj- (aq) + 14 H+ (aq) + 6 e~ > 2 Cr^+ (aq) + 7 H2O (/) (viii)


eB

The H atoms get automatically balanced.


Thus, Eq. (viii) represents the balanced reduction half equation.
Step 6. To balance the electrons lost in Eq. (iv) and gained in Eq. (viii), multiply Eq. (iv) by 6 and add to Eq.
(viii), we have.
our
ad

6Fe^‘*‘(a^) ^6Fe3+ + 6e-

Cr202- (aq) + 14 H+ (aq) + 6 e" 2 Cr3+ (aq) + 7 H2O (0


dY

Crjdlj- (aq) + 6 Fe^^ (aq) + 14 H+ (aq) ^ 2 Cr3+ (aq) + 6 Fe^^ (aq) + 7 H2O (/) ,(ix)
Re

This gives the final balanced redox equation.


Fin

Step 7. Verification. Total charge on L.H.S. of Eq. (ix) = - 2 + 6 (+ 2) + 14 (+ 1) = + 24


Total charge on R.H.S. of Eq. (ix) = 2 (+ 3) + 6 (+ 3) = + 24
Since the magnitude of charge on both sides of Eq. (tx) is equal, therefore, Eq. (ix) represents the correct
balanced redox equation.
iami
Balance the equation,

AS2S3 (s) + N03(fl^) + H+ (aq) AsOj-(a^) + S (s) + NO (g) + HjO (/)


Solution. Step 1. To identify the atoms whose oxidation numbers have undergone a change.
Writing the oxidation number of each atom above its symbol, we have,
+3 -2 +5 +5 0 +2

ASz 83(5) + N O3 (aq) + H"*" (aq) > As05-(fl^) + S(^)+N02(g) + H20(/)


8/26 New Course Chemistry (XI)IS!

Here, the oxidation number of As has increased from +3 to +5 and that of S has increased from —2 to 0 while
that of N has decreased from +5 to +2. In other words, both As and S have been oxidised while NOj has been
reduced. Since As and S must maintain their atomic ratio of 2:3 (as in As2S^) therefore, the change in oxidation
numbers of these two atoms must be considered togethe.r Keeping in view these points, the above redox reaction
can be split up into the following two half reactions :
+3 -2 +5 , 0
Oxidation half equation : As2 ■>
AsC^ (aq) + S(s) .(0

low
+5 +2
Reduction half equation : NOj(fl9) > NO(g) .(«*)

Step 2. To balance the oxidation half Eq. (/)

(a) Balance all the atoms other than H and O. Multiply AsO^ by 2 and S by 3 on R.H.S. of Eq. (i), we
have.

e
re
+3 -2

rF
As2 Sj (j) 2AsOl-(aq) + 3S(s) ,(iU)

F
(b) Balance the oxidation number by adding electrons.
Since each As atom loses two electrons and there are two As atoms, therefore, due to the oxidation of As
alone add 4 e~ to R.H.S. of Eq. (m). Further, since each S atom loses two electrons and there are three S atoms,

r
therefore, due to the oxidation of S alone, add 6 e~ to R.H.S. of Eq. (Hi). Combining these two oxidation steps

fo
u
together, add 10 e~ to R.H.S. of Eq. (Hi), we have,
AS2S2 (s) 2 AsOj" (aq)+ 3 S (j) + 10 e~
ks m
Yo
(c) Balance charge by adding ions.
oo
The total charge on R.H.S. of Eq. (iv) is -16 and zero on the L.H.S., therefore, add 16 H"*" to R.H.S. of Eq.
(iv) we have.
B

AS2S3 (s) 4
2 AsOj- (fl^) + 3 S (s) + 16 H+ (aq) + 10 .(V)
re

(d) Balance O atoms by adding H2O molecules. Since there are eight O atoms on the R.H.S. of Eq. (v) but
none on the L.H.S., therefore to balance 0-atoms, add 8 H2O to the L.H.S. of Eq. (v), we have,
u
ad

AS2S3 (s) + 8 H2O (/) 2 As03-(a^) + 3 S (s) + 16 H+ + 10 e" .(vO


Yo

The H-atoms get automatically balanced. Thus Eq. (vi) represents the balanced oxidation half equation.
Step 3. To balance the reduction half Eq. (ii)
d
Re

(a) Balance oxidation number by adding electrons. Oxidation number of N in NOJ is +5 on L.H.S. while
in

it is +2 in NO on R.H.S., therefore, add 3 e~ to L.H.S. of Eq. («). we have,


.(vw)
F

NO3 (a^) + 3e” ^NO(g)


(b) Balance charge by adding ions.
The total charge on L.H.S. is - 4 while it is zero on R.H.S., therefore, add 4 H'*’ to L.H.S. of Eq. (v«). We
have.

NO3 (09) + 4 H+(o^) + 3 e- ^ NO (g) .(viu)

(c) Balance O atoms by adding H2O molecules. Since there are three O atoms on the L.H.S. of Eq. (viU) but
only one on the R.H.S., therefore, add 2 H2O to the R.H.S. of Eq. (viii), we have.

NO3 (aq) + 4 H"^ (aq) + 3 e ■>N0(g) + 2H20(/) ...(«)

The H-atoms are automatically balanced. Thus, Eq. (ix) represents the balanced reduction half equation.
REDOX REACTIONS 8/27

Step 4. To balance the electrons lost in Eq. (vi) and gained in Eq. (ix), multiply Eq. (ix) by 10 and Eq. (vi) by
3 and add. We have,

3 AS2S3 (5) + 24 H2O (/) 6 AsOj- (fl^) + 9 S (5) + 48 H+ {aq) + 30 e"


lONOj {aq) + 40 H+ {aq) + 30 e~- 4 10 NO (g) + 20 H2O (/)
3 AS2S3 {s) + IONO3 + 4 H2O (0 6 AsQ3- {aq) + 9 S (j) + 10 NO (g) + 8 H+ {aq)
This gives the final balanced redox equation.
ESamjpre^robiem In passing chlorine gas through a concentrated solution of alkali, we get
chloride and chlorate ions. Obtain balanced chemical equation for this reaction.
Solution. Step 1. Write the skeletal equation for the given

w
reaction.

CI2 {g) + OH- {aq) > Cl- {aq) + CIOJ {aq) (I)
Step 2. Write the O.N. of all the elements above their respective symbols.

F lo
O.N. increases by 5 per Cl atom
0 -1 +5
Cl2(g) +
OH"(a^) ► C\-{aq) + C\02 (aq)
i

e
Fre
O.N. decreases by 1 per Cl atom
Total increase = 2x5 = 10
Total decrease = 2x-l=-2

for
Step 3. Find out the oxidant and the reductant and split the skeletal Eq. (i) into two half reactions.
Here, O.N. of Cl decreases from 0 in CI2 to - 1 in Cl” therefore, CI2 acts as an oxidant. Further, the O.N. of
Cl increases from 0 in CI2 to +5 in CIOJ, therefore, CI2 acts as a reductant. In other words, CI2 acts both as an
r
oxidant as well as a reductant. Therefore, the two half reactions
You
are :
oks
Reduction half equation ; CI2 (g) > Cl“ {aq) (ii)
Oxidation half equation : CI2 (g) > CIOJ {aq) ...(/«)
eBo

Step 4. To balance the reduction half Equation (ii).


(a) Balance all atoms other than O and H.
Since there are 2 Cl atoms on L.H.S. of Eq. {ii) but only one on the R.H.S., therefore, multiply Cl" ion by 2,
we have.
our
ad

CI2 (aq) > 2 Cl (aq)


(b) Balance oxidation number by adding electrons. The O.N. of Cl in CI2 on L.H.S. of Eq. (tv) is 0 while on
the R.H.S. it is -1 in Cl ion. Thus, each Cl atom accepts one electron. Since there are two Cl atoms on the R.H.S.,
therefore, add 2 e~ to L.H.S. of Eq. (tv), we have.
dY
Re

Cl2(g) + 2e- > 2 Cl- (aq) (V)


(c) Balance charge. Not needed since charge on either side of Eq. (v) is balanced. Thus, Eq. (v) represents
the balanced reduction half reaction.
Fin

Step 5. To balance the oxidation half Equation (Hi)


(a) Balance all atoms other than O and H.
Since there are 2 Cl atoms on L.H.S. of Eq. (Hi) and only one on the R.H.S., therefore, multiply ClOj ion by
2, we have.

CI2 (g) 2CIO3 {aq) ..(vO


{b) Balance oxidation number by adding electrons. The O.N. of Cl in CI2 on L.H.S. of Eq. {vi) is zero while
on the R.H.S. in CIO3 , it is +5. Thus, each Cl atom loses five electrons. Since there are two Cl atoms on R.H.S.,
therefore, add 10 e~ to R.H.S. of Eq. {vi), we have.
CI2 (g) > 2CIO3 {aq) + 10 e- ,(vii)
8/28 "pfutdeep.'^ New Course Chemistry (XI)GSSISS

(c) Balance charge by adding OH~ ions since the reaction occurs in the basic medium.
The total charge on the R.H.S. of Eq. (vii) is - 12 and zero on the R.H.S. Therefore, add 12 OH" ions to the
L.H.S., we have,
,(vi«)
CI2 (g)+ 120H-(fl^) -> 2C10J {aq)+ 10 e-
(d) Balance O atoms.

w
The R.H.S. of Eq. (vi/i) contains six O atoms but on the L.H.S., there are 12. Therefore, add 6 H2O to the
R.H.S.. we have.
..m
CI2 (g) + 12 OH- (aq) > 2C10J (aq) + 6 H2O (/) + 10 e"
By doing so, H atoms are automatically balanced, therefore, Eq. (tx) represents the balanced oxidation half

o
equation.
Step 6. To balance the electrons gained in Eq. (v) and lost in Eq. (ix), multiply Eq. (v) by 5 and add to Eq.

e
(ix), we have, 5CI2 (g)+\0e- > lOCl-(fl^)

re
rFl
CI2 (g) + \20H-(aq) -) 2C10J (aq) + 6H2O (1) + \0 e

F
6CI2 (g)+ 120H-(fl^) lOCr (aq) + 2C10J (aq) + 6H2O (/)
or 3CI2 {g) + 60H-(fl^)- 5Cr (aq) + CIO3 (aq) + 3H2O (!)
This represents the final balanced redox equation.

r
ou
fo
● I ■S’ p wmm
ks
k ■>

1. Balance the following equations in acidic medium by both oxidation number and ion electron methods and
identify the oxidants and the reductants :
oo
(i) MnO^ (aq) + C2H2O4 (aq) Mn^"^ (aq) + CO2 (g) + H2O (/)
Y
(ii) H2S (aq) + CI2 (g) > S (s) + Cr (aq)
B

(iU) MnO" (aq) + C2H5OH (aq) 4 Mn^"^ (aq) + CH3COOH (aq)


(iv) Bi (s) + NOJ (aq) ^ Bi^’’’ (aq) + NO2 (g)
re

(v) Cr20^- (aq) + C2H4O (aq) Cr^'*' (aq) + C2H4O2 (aq)


ou

(vO MnO^ (aq) + Br" (aq) — Mn^'*’ (aq) + Bt2 (aq)


Y
ad

(vii) Cu (aq) + NOJ (aq) -> Cu2+ (aq) + NO2 (g)


(viii) H2S (g) + Fe^’’' (aq) ■ > Fe^'*' (aq) + S (5) + H'*’ (aq)
d

(ix) r (aq) + IO3 (aq) + H’’" (aq) > I2 (aq) + H2O (!)
in

(x) r (aq) + O2 (g) + H2O (!) > I2 (aq) + OH" (aq)


Re

(xi) Cu + Au’’’ > Au + Cu^’’’


Sn2+ (aq) + NHJ (aq) + H2O (/)
F

(xii) Sn (5) + NOj (aq) + H'^ (aq) -


(xiii) Cu (s) + NOJ (aq) + H’’’ (aq) —> Cu^'*' (aq) + NO (g) + H2O (!)
(xiv) Zn (s) + NOJ (aq) + H+ (aq) —> Zn2+ (aq) + N2O (g) + H2O (!)
(xv) Sn (aq) + NOJ (aq) + H’’’ (aq) > Sn02- (aq) + NO2 (g) + H^O (!)
(xv/> As (5) + NOj (aq) + H^ (aq) ■ AsOl~ (aq) + NO2 (g) + H2O (/)
(xvii) CrjO^- (aq) + Fe^’’’ (aq) Cr^-’- (aq) + Fe^'’’ (aq)
(jcvjVi) MnOj (aq) + Fe^’’’ (aq) > Mn (aq) + Fe^'’’ (aq)
2+

(xix) S + HNO3 > SO2 + NO2 + H2O


REDOX REACTIONS

rw
8/29

2. Balance the following equations in basic medium by both oxidation number and ion electron methods and
identify the reductants and the oxidants.
(/) P4 + NaOH + H2O > PH3 + 3 NaH2P02
(ii) N2H4 (g) + CIOJ (aq) > NO (g) + Cl" (aq)
(Hi) CI2O7 (g) + H2O2 (aq) > CIO2 (aq) + O2 (g)
(iv) Cr(OH)4 (aq) + H2O2 (aq) > CrO^' (aq) + H2O (/)

ulo
r
(v)Zn(i)+ NO3 (aq) >Zn^-^ (aq)+ NHJ (aq)

F
(v/)AI(5)+ NO3 (aq) > AI(OH)“ (ag) + NH3(g)
(vii) Pb02 (j) + Cl~ (aq) Pb(OH)J (a^) + CIO-(fl^)

oF
(viii) Fe(OH)2 (aq) + H2O2 (aq) ^ 2Fe(OH)3 (s)
(u:) Bi(OH)3 U) + Sn02- (aq)- ^ Bi (^) + SnO^“ (aq)

rs
U) Cr (s) + CIO4 (aq) ^ Cr(OH)3 (^) + CIOJ (aq)

ko
3. Balance the following redox reactions :
(i) Sn02 + C — ^ Sn + CO

of
(ii) Fe304 + C - —> Fe + CO
(Hi) I2 + HNO3 ^ HIO3 + NO2 + H2O
(iv) FeS04 + HNO3 + H2SO4
o
^ Fe2(S04>3 + NO + H2O
Y
(v) Fe + HNO3 - ^ Fe(N03)2 + NH4NO3 + H2O
B
(vi) Sb + HNO3 - H3Sb04 + NO2 + H2O
Y

(vH) Hg + HNO3 ^ Hg2(F^03)2 NO + H2O


ANSWERS
er

1. (1) 2MnO“ (aq) + 6 R-*- (aq) + 5 C2H2O4 (aq) >2 (aq) + 10 CO2 (g) + 8 H2O (0
u

(ii) H2S (aq) + Cl2 (g) > S (^) + 2 CF (aq) + 2 (aq)


d

(iii) 4MnO" (aq) + 5 C2H5OH (aq) + 12 H+ (aq) 4 Mn^+ + 5 CH3COOH (aq) + 11 H2O (!)
o
ad

(iv) Bi (s) + 3NOJ (aq) + 6H* (aq) ^ Bi3+ (aq) + 3 NO2 (g) + 3 H2O (/)
in

(v) Crp‘^~ (aq) + 3 C2H4O (aq) + 8 H+ (aq) ^ 2 Cr3+ (aq) + 3 C2H4O2 (aq) + 4 H^O (/)
(vi) 2Mn04 (a^) + 10 Br" (aq) + 16 (aq) ^2 (aq) + 5 B12 (aq) + 8 H2O (/)
Re

(vii)C\i(s)+ 2NO3 (aq) + 4H'^(aq) ^ Cu^+ (aq) + 2 NO2 (g) + 2 H2O (/)
F

(viii) H2S (g) + 2Fe^-^ (aq) 2Fe2+ (aq) + S (5) + 2 R-" (aq)
(ix) 51- (aq) + IO3 (aq) + 6H^ (aq) > 3I2 (aq) + 3R2O (/)
(x) 41- (aq) + O2 (g) + 2R2O (/) ^ 212 + 40R- (aq)
(xi) Cu (s) + 2Au+ (aq) > 2Au (i) + Cu^+ (aq)
(xii) 4 Sn (j) + NO" (aq) + 10 (aq) ^ 4 Sn2+ (aq) + NHJ (aq) + 3 R2O (/)
(xiii) 3 Cu (5) + 2 NOJ (aq) + 8 R-^ (aq) ■ 3 Cu2+ (aq) + 2 NO (g) + 4 H2O (/)
(xiv) 4 Zn (s) + 2NO3 (aq) + 10 R-^ (aq) —> 4 Zn^-*- (aq) + N2O (g) + 5 R2O (0
(xv) Sn (s) + 4 NO3 (aq) + 2 R+ (aq) SnO|- (aq) + 4 NO2 (g) + R2O (/)
(rvi) As (x) + 5 NOJ (aq) + 2H^ (aq) — AsO^- (aq) + 5 NO2 (g) + H2O (/)
8/30 'pxeuUcft'^ New Course Chemistry (XI)S!E33D]

(xv/i) Cr20^ {aq) + 6 Fe^"^ (at?) + 14 {a<^ 2 H) + 6 Fe^-" {aq) + 7 H2O (/)
(xviiV) MnO^ {aq) + 5 Fe^"^ (at/) + 8 (aq) - 4 Mn^'*' {aq) + 5 Fe^'^ (fl^) + 4 H2O (/)
(xix) S + 4 HNO3 > SO2 + 4 NO2 + 2 HoO
2. (() P4 (5) + 3 NaOH (aq) + 3 H2O (/) > PH3 (g) + 3 NaH2?02 (ay)
(«) 3 N2H4 (g) + 4CIO3 (aq) ^ 6 NO (g) + 4 Cl" (ay) + 6 H2O (/)

(in) C1207 (g) + 4 H2O2 (at/) + 2 OH" (at/) ^ 2C10; (ay) + 4 O2 (g) + 5 H2O (/)
(/V) 2Cr(OH)- (aq) + 2 OH* (aq) + 3 H2O2 (ay) ^ 2CrO|- (ay) + 8 H2O (/)

w
(v) 4 Zn (s) + NO3 (aq) + 7 H2O (/) ^ 4 Zn^+ (ay) + NHJ (ay) + 10 OH" (ay)
(v/) 8 A1 (5) + 3 NO3 (ay) + 18 H^O (/) + 5 OH“ (oy) ^ 8AKOH)- (ay) + 3NH3(g)

F lo
(vU) PbO, (5) + Cr (ay) + H2O (/) + OH" (ay) -> Pb(OH)' (ay) + CIO* (ay)
(vm) 2 Fe(OH)2 (ay) + H2O2 (ay) > 2 Fe(OH)3 (s)
(la:) 2 Bi(OH)3 (5) + 3SnO““ (ay)- ^ 2 Bi (s) + 3SnO| (ay) + 3 H2O (/)

e
Fre
(X) 2 Cr (5) + 3CIO4 (ay) + 3 H2O -> 2 Cr(OH)3 (5) + 3 CIO" (ay)
3. (i) Sn02 + 2 C > Sn + 2 CO (ii) Fe304 + 4 C - ■> 3 Fe + 4 CO

for
(Hi) I2 + 10 HNO3 > 2 ffl03 + 10 NO2 + 4 H2O
(iv) 6 FeS04 + 2 HNO3 + 3 H2SO4 >3 + 2 NO + 4 H.O
(v) 4 Fe + 10 HNO3 ^ 4 Fe(N03)2 + NH4NO3 + 3 H2O
(vi) Sb + 5 HNO3 > H3Sb04 + 5 NO2 + H^O
r
You
(vii) 6 Hg + 8 HNO3 3 Hg2(N03)2 + 2 NO + 4 H2O
oks
eBo

8.6. REDOX REACTIONS AS THE BASIS FOR TITRATIONS


Titration is a process in which the solutions of two reagents are allowed to react. It is carried out by
adding the standard* solution of one reagent taken in a burette to the known volume (10 or 20 cm^ measured
by a pipette) of the solution of the other reagent taken in a flask called the titration flask. The solution Uiken
ad
our

in the burette is called the titrant while that taken in the titration flask is called the analyte. The titration is
carried out till the reagents present in both the solutions react completely. The stage at which the two reagents
react completely is called the equivalence point or the end point. The end point is detected by adding drop
of a suitable indicator.**
In acid-base titrations, an acid is titrated against a base using phenolphthalein or methyl orange as the
Re
dY

indicator. Phenolphthalein is colourless in the acid solution but turns pink in the basic solution. Methyl
orange, on the other hand, is red in the acid solution but turns yellow in the basic solution.
Fin

The objective of acid-base titrations is to find out the exact amount of an acid (or the base) present in a
given solution by reacting it against the solution of a standard base (or the acid).
In a similar way, redox titrations can be used to determine the exact amount of an oxidising (or a
reducing agent) in a given solution by titrating it against the standard solution of a suitable reducing agent (or
the oxidising agent).
However, acid-base and redox titrations differ from each other in the choice of indicators.
Choice of indicators. The main difference between the two types of titrations is that the indicators used
in acid-base titrations to detect the changes in pH are not useful for redox titrations. This difficulty can be
overcome in the following three ways.
*A solution whose normality or molarity is known is called a standard solution.
**An indicator is an organic dye which changes colour with the pH of the solution at the equivalence point.
REDOX REACTIONS
8/31

1. One the reagents, which is itself intensely coloured may be used as an indicator. For example, KMn04
or MnO“ acts as the self-indicator.

In KMn04 titrations, when whole of the reductant (Fe^^or Cpl~) is oxidised, end point is reached. At
the end point, the violet colour of Mn04 solution disappears and a lasting (permanent) tinge of pink colour
appears. This colour change is so sensitive that it can easily be detected even at very low concentrations
(10-6 mol dm-3 or lQ-6 mol L"’) of MnO^.
2. If there is no dramatic auto-colour change (as with KMn04 titrations), there are indicators which are

w
immediately oxidised after the last drop of the reactant has reacted producing a dramatic colour change.
Unlike MnO^ (or KMn04), (or K2Cr207) is not a self-indicator. Therefore, in K2Cr202 titrations,
some indicator is used to detect the end point. The following two types of indicators are usually used ;

F lo
(/) Internal indicators. A few drops of these indicators are added to the reactant taken in the titration
flask prior to the start of the titration. Examples are : diphenylamine and N- phenylanthranilic acid. With
diphenylamine at the end point, colour changes from bluish green or purple to blue-violet. However, with N-
phenylanthranilic acid, at the end point, colour changes from green to violet-red.

ee
(») External indicator. Potassium ferricyanide, K3lFe(CN)^] is used as an external indicator during

Fr
the titration of ferrous salts against K2Cr207 solutions. Several drops of the indicator are placed side by side
on a glazed white tile. A drop of the oxidised solution is taken out with a glass rod out of titration flask and
touched with the drop of the indicator from time to time during the course of titration. In the beginning, the

for
colour is blue which changes to green and finally to brownish yellow. Thus, at the end point, colour is brownish
ur
yellow or there is no change in the colour of the indicator,
(m) Another indicator which has been used in redox reactions is starch which produces intense blue
colour with molecular iodine. The use of this indicator is, however, limited to only those reagents which can
s
ook
either oxidise I ions such as Cu“'*‘ ions or reduce U such as thiosulphate ion (S^O^-) ions.
Yo
2 Cu^"^ {aq) + 4l~ (aq) CU2I2 (^) + I2 (.r); I2 (ciq) + 2 S.,Oj (aq)^ 2 I (aq) + (aq)
eB

Thus, when I ions are oxidised to molecular U, intense blue colour appears at the end point. However,
when I2 is reduced to I ions, the intense blue colour disappears at the end point.
It may be noted here that although I2 is insoluble in water, it dissolves readily in water in presence of KI
r

due to the formation of KI3.


ad
ou

KI (aq) + I2 (5) V - Kl3(a^)


8.6.1. Types of Redox Titrations
Y

Depending upon the nature of the oxidising agent, these are divided into the following categories :
1. Potassium permanganate titrations. In these titrations, reducing agents like FeS04, Mohr’s salt
Re
nd

[(NH4)2S04.FeS04.6 H2O], H2O2, AS2O3, oxalic acid (COOH)2, oxalates (COONa)2, etc. are directly titrated
against KMn04 as the oxidising agent in acidic medium. For example,
Fi

(/) Oxidation offerrous salts :


5 Fe2+ (aq) + MnO^ (aq) + 8 H"*" (aq) ^ 5 Fe^+ (aq) + Mn^+ (aq) + 4 H2O (/)
Ferrous ion Permaganate ion Ferric ion

(ii) Oxidation of oxalates :


COO"
5 I (aq) + 2MnO" (aq) (aq) ^ 2 Mn^+ (aq) + 10 CO2 (g) + 8 H2O (/)
COO"
Oxalate ion

2. Potassium dichromate titrations. In these titrations, the above listed reducing agents are directly
titrated against K2Cr207 as the oxidising agent in acidic medium. For example.
8/34
'4. New Course Chemistry (XI) orsTTi
Applying molarity equation to the balanced redox equation, we have,
M,V M2V,
(KMn04) = (FeSO^) or IX V, _ 1 ^ 1x25x8
= lO-Ocm^
8x1 ~ 4 5 or V, =
"2 4x5

Thus, the volume of M/8 KMnO^ solution required - 10-0 mL.

Sample Problem Q Calculate the concentration of hypo (Na2S203-5 H2O) solution in


g dnr^ if lO'O niL of this solution decolourised 15 mL of M/40 iodine solution.

Solution. The balanced equation for the redox reaction is 2S202~+l2


From the balanced equation, it is evident that 2 moles of Na2S203 s 1 mole of h
Applying molarity equation, we have,

w
M.V M,V M.xlO 15x1 15x1x2 3
I 1 2 2
(Na2S203) = -a.) or or M

F lo
0
«2 1x40 10x40 40

Thus, the molarity of the hypo solution = 3/40 M


-1
Mol. mass of Na2$203-5 H,0 = 248 g mol
248x3
Concentration of Na2S^03-5 H2O = = 18*6 g dm ^

e
40

Fre
Sample
Problem Q 0‘5 g of an impure sample of oxalate was dissolved in water and the solution
made to 100 mL. On titration, 10 mL of this solution reqired 15 mL of N/20 KMn04 solution. Calculate the

for
percentage of pure oxalate in the sample.
Solution. The reduction half reaction is C 0“"
'-2^4 ^ 2CO2 + 2 e-
r
Oxalate ion
You
oks
2x12 + 4x16
Eq. wt. of = = 44
2
eBo

Let N| be the normality of the oxalate solution. Applying normality equation, we have,
15 3
15 X 1/20 (KMn04) = Nj x 10 (C20“~) N1 “
10x20
= —N
40
ad
our

3
Strength of oxalate solution = Normality x Eq. wt.
3-3
Amount of pure C2O4 present in 100 mL = xl00 = 0-33g
1000
Re
Y

But amount of €204" present in impure sample = 0-5 g (Given)


d

0-33
Fin

Percentage of pure oxalate = X100 = 66


0-50

1. Calculate the volume of 0-05 M KMn04 solution required to oxidise completely 2-70 g of oxalic acid
(H2C2O4) in acidic medium.
2. How many grams of K2CV2O1 are required to oxidise Fe^^ present in 15-2 g of FeS04 to Fe^* if the reaction
is carried out in an acidic medium ?

3- 15-0 mL of 0T2 M KMnO^ solution are required to oxidise 20-0 mL of FeSO^ solution in acidic medium.
What is the concentration of FeS04 solution ?
8/35
REDOX REACTIONS

4. Calculate the percentage of oxalate ions in a given sample of oxalate salt, 3-0 g of which has been dissolved
per litre of the solution. 10 mL of the oxalate salt solution required 8 niL of 0-01 M KMn04 solution for
complete oxidation.
by dissolving 3-6 g L“^ Calculate the volume of 0-01 M
5. A solution of ferrous oxalate has been prepared
KMn04 solution required for complete oxidation of 100 mL of ferrous oxalate solution in acidic medium.
6. Metallic tin in presence of HCl is oxidised by K2Cr207 to stannic chloride. What volume of decinormal
dichromate solution will be reduced by 1 g of tin ? A*t. wt. of Sn = 118-7.
7. How many millimoles of potassium dichromale is required to oxidise 24 mL of 0-5 M Mohr’s salt solution

w
in acidic medium ?

8. 2-48 g of Na2S203..r H^O was dissolved per litre of the solution. 20 mL of this solution required 10 mL of
0 01 M iodine solution. Find out the value of .t.

9. 50 mL of an aqueous solution of HiOi was reacted with excess of KI solution and dilute H2SO4. The

Flo
liberated iodine required 20 mL 0-1 N Na2$->03 solution for complete interaction. Calculate the concentration
of H2O2 in gL“"*.

ee
10. Both Cr.,0^" (aq) and MnO;^ (aq) can be used to titrate Fe^"^ (aq). If in a given titration, 24-50 mL of

Fr
0-1 M Cr20^~ were used, then what volume of 0-1 M MnO;^ solution would have been used for the same
titration ?

ANSWERS

for
1. 240 mL
r
2. 4-9 g
7. 2 milimoles
3. 0-45 M
8.5
4. 58-67%

9. 0-68 g
5. 150 mL
10. 29-4 mL
ou
6. 336-9 mL

HINTS FOR DIFFICULT PROBLEMS


ks
1. Balanced equation for the redox reaction is ;
oo
2KMn04 + 5(C00H)2 + 3 H2SO4 > K2SO4 + 2 MnS04 + 10 CO^ + 8 H2O
Y
B

No. of moles of oxalic acid = 2-70/90 = 0-03 mole

From the balanced equation, 5 moles of (COOH)2 = 2 moles of KMn04


re

0-03 mole of (COOH)2 = 2/5 x 0-03 = 0-012 mole of KMn04


Now 0-05 mole of KMn04 is present in solution = 1000 mL
ou
ad

1000x0012
.●. 0-012 mole of KMn04 is present in solution = 0-05
= 240mL
Y

2. The balanced chemical equation for the redox reaction is :


^■2^1-207 + 6 FeS04 + 7 H2SO4 —> K2SO4 + Cr2(S04)3 + 7 H2O
nd
Re

From the balanced equation, it is clear that


6 moles of FeS04 s 1 mole of K2Cr,07 or 6 x 152 g of FeS04 are oxidised by K2Cr207 = 294 g
Fi

294
or
15-2 g of FeS04 are oxidised by K2Cr207 = xl5-2 = 4-9g
6x152

3. The balanced chemical equation for the redox reaction is :


2 KMn04 + 10 FCSO4 + 8 H2SO4 ●>
K2SO4 + 2 MnS04 + 5 Fe2(S04)3 + 8 H2O
Applying molarity equation to the above redox reaction,
20xM 15x0-12x10

= l^^^(KMn04) =
1 = 0-45 M
10
(FeS04) or Mj = 2x20

4. Balanced chemical equation for the redox reaction is :

2Mn0“-fI6H++5C20“ ^2Mn^++ IOCO2 + 8H2O


8/36
^ New Course Chemistry (XI) prtii

r.2-x t>01x8/x^
Applying molarity equation.
^—(C2O2 ) = ■ ^ (MnO^) or M|=0-02M
Mol. wt. of C2O2- = 88
Cone, of in g L“‘ = 88 x 0 02 = 1-76 g
1-76x100
%C20f = 3
= 58-67

-1
Mass in gL 3-6
5. Molarity of FeC204 solution = = 0-025 M
Mol. massing mol * 144

w
The balanced chemical equation for the redox reaction is :
5FeC204 + 3MnO- + 24 H+ +
5 Fe3+ + 3 Mn^+ + 10 CO2 + 12 H2O

F lo
Applying molarity equation, we have,
0-01 xV,,, 0-025x100,^ ^ ^ ,
—-—(Mn04> = ^ (FeC204) or ^^0025x100x3 = ISOmL
5x001

6. Balanced chemical equation is :

e
Fre
2 K2Cr207 + 3 Sn + 28 HCl > 4 KCl -f 4 C1CI3 + 3 SnC^ + 14 H2O
2(2x39 + 2x52 + 7x 16) 3x 118-7

for
= 2 X 294 = 588 g =356-1 g
Now 356-1 g Sn react with K2Cr207 = 588 g
588
r
1 g Sn will react with ¥i.2Ct20-j = 356-1
g = 1-651 g
You
oks
Mol. wt. 294
— = 49.
Eq. wt. of K2Cr207 = —g
eBo

Decinormal K2Cr207 solution means 1000 mL solution contains 4-9 g K2Ct207, i.e., 4-9 g K2Cr207 are
present in 1000 mL solution
1000
1-651 g K2Cr207 will be present in = xl-651= 336-9mL
ad
our

4-9

7. No. of millimoles of K2Cr207 present in 24 mL of 0-5 M solution = 24 x 0-5 = 12


The balanced chemical equation for the redox reaction is :
K2Cr207 + 6 (NH4)2S04.FeS04.6 H2O + 7 H2SO4 > K2SO4 + 6 (NH4)2S04 + 3 Fe2(S04)3
dY
Re

+ Cr2(S04)3 + 43 H2O
From the balanced equation, 6 moles Mohr’s salt are oxidised by K2Cr207 = 1 mole

“ ^ ^ =2 millimoles
Fin

12 millimoles of Mohr’s salt will be oxidised by K2Cr207


8. The balanced equation for the redox reaction is : 2 Na2S203 +12 > Na2S405 + 2 Nal
Let the molarity of Na2S203.x H2O solution = M 1
M,x20 10x01
Applying molarity equation to the above redox reaction, we have,
-l^(NajS203) = 1
(I2)
M, = 0-01 M
Mol. wt. of Na2S203Jc H2O = 2 x 23 + 2 x 32 + 3 x 16 + x x 18 = 158 + 18 x
.-. Amount of Na2S203. x H2O present per litre = (158 + 18 jc) x 0-01 g
But the actual amount dissolved = 2-48 g
Equating these values, we have, (158 + 18 x) x 0-01 = 2-48 or x = 5
8/37
REDOX REACTIONS

-1

9. H2O2 +2KI + H2SO4 -> K2SO4 + I2 + 2 H2 0


Na2S202 + It Na2S40^ + 2 Nal
2 + 2x16
= 17
Total change in O.N. of O = 2 (- I) - 2 (- 2) = 2 /. Eq. wt. of H2O2 “ 2

Let N] be the normality of I2 solution.


Since one equivalent of H2O2 produces I equivalent of l2-
50 mL of N| I2 solution = 50 mL of N [H2O2 solution.
Applying normality equation, we have, 20 x 01 (Na2S203) = 50 x N[ (H2O2)

low
20x01 -1
N
1 “
50
= 0-04N or Strength of HtOt solution = 0 04 x 17 = 0’68 gL

10. The balanced redox reactions involving oxidation of Fe"'*' ions by Cr207 and MnO^ respectively are .
Cr20^-+6Fe2+ + 14 ■> 2 + 6 Fe^+ + 7 HtO
and Mn04 + 5 Fe“+ + 8 R-" ■> Mn2+ + 5 Fe^-' + 4 H.O

ee
Suppose V2 mL of M2 Fe“"^ is titrated against 24-50 mL of 0-1 M Cr^O^ and V[ mL of 0-1 M MnO^

rF
Fr
solutions, then.
24-5 X 0-1
1 (Cr,0^^) = ^-^(Fe2+) ...(0

for
V.1 X 0-1 MtV,
and
1
(MnOp = (Fe^-^) ...(»)
u
24-5 X- = 29 4 mL
ks
Equating (i) and {ii), Vj = 5
Yo
oo
8.7. REDOX REACTIONS AND ELECTRODE PROCESSES
eB

8.7.1. Direct and Indirect Redox Reactions


We have discussed in Art. 8.3.1, page 8/7 that when a zinc rod is placed in Cu(N03)2 solution, a redox
reaction occurs. During this reaction, Zn is oxidised to Zn^"^ ions while Cu”'*' ions are reduced to Cu by direct
transfer of electrons from Zn to Cu^'*’ ions. Since here transfer of electrons occurs directly over short-distances
r

(generally within molecular diameters) no useful electrical work can be obtained. Instead, the chemical energy
ou
ad

of the reaction appears as heat. Such redox reactions in which the oxidation and reduction reactions occur in
the same vessel are called direct redox reactions. If, now the same redox reaction is carried out in such a
Y

manner that the transfer of electrons is allowed to take place only indirectly over large distances through

metallic wires so that chemical energy is converted into electrical energy. This is possible only if zinc rod is
separated from Cu(N03)2 solution, i.e., oxidation and reduction reactions are carried out in different vessels.
Re
nd

Such redox reactions in which the oxidation and reduction reactions take place in dijferent vessels are called
indirect redox reactions and the device used for carrying out an indirect redox reaction is called an
Fi

electrochemical cell. Thus,

An electrochemical cell or simply a chemical cell may be defined as a device to convert chemical
energy produced in an indirect redox reaction into electrical energy. Electrochemical cells are
also called galvanic cells or voltaic cells after the names of the scientists, Luigi Galvani (1780)
and Alessandro Volta (1800) who were the First to perform experiments on the conversion of
chemical energy into electrical energy.
8.7.2. Construction of an Electrochemical Cell
In order to carry out the above redox reaction indirectly, place zinc rod in 1 M ZnS04 solution in the left
beaker and a copper rod in a 1 M CUSO4 solution in the right beaker. No reaction takes place in either of the

t
8/38
7^k<uUe^'^ New Course Chemistry (XI)CZ5Ian
two beakers, and at the interlace of the metal and its salt solution in each beaker where both the reduced and
oxidized forms of the same species are present. Each beaker constitutes a half cell. It is also called as an
electrode or a redox couple.
A redox couple may be defined as combination of the oxidised and reduced forms of th e same
substance taking part in an oxidation or reduction half reaction.
A redox couple is usually
FIGURE 8.3
represented by separating the

ow
ELECTRON
oxidised form and the reduced V FLOW
form by a vertical line or a slash
which represents the interface
0-^ CURRENT

(solid/solution). For example, in FLOW I

the above experiment, two redox e soj- SALT BRIDGE


K i e'

couples are represented as Zn^*fZn

e
and Cu^'*'/Cu. In both cases. CONTAINING K2SO4
X

re
rFl
oxidised form is put before the /
reduced form. ANODE
CATHODE

F
Now connect the zinc and the ZINC COPPER
copper rods by a copper wire ROD ROD
through a voltmeteror ammeterand ZnS04 soj C1JSO4

r
SOL.
an on-off switch (Fig. 8.3). Also SOL.
ou
fo
connect the solutions in the two
beakers by a salt bridge which An electrochemical or galvanic cell
provides electric contact between the
ks
two solutions without allowing them to mix with each other. A salt bridge is a U-tube containing solutions of an
inert electrolyte* such as KCl, KNO3, K2SO4, NH4NO3, etc. in agar-agar or gelatine.
oo
When the switch is in the off position, no reaction takes place in either of the beakers and no current
Y
flows through the metallic wire. As soon as the switch is in the on position, the voltmeter shows a deflection
eB

thereby indicating the flow of current through the circuit.


Let us investigate as to why this current flows. This current is due to chemical reaction taking place in
two beakers.
ur

Left beaker. Zinc rod .starts dissolving and zinc sulphate solution becomes more concentrated, i.e.,
each zinc atom loses two electrons to form Zn-'*' {aq) ion. The zinc ions then migrate away from the rod into
ad

solution while the electrons move through the wire, via voltmeter and ultimately reach the copper rod. The
Yo

change taking place may be represented as :


Zn (i) ■> Zn^'*’ [aq) + 2 c ...(0
d

Right beaker. Copper sulphate solution .starts getting more diluted and metallic copper starts depositing,
Re

i.e., the electrons released by zinc atoms in the left beaker on reaching the copper rod combine with Cu^+ {aq)
in

ions present in the solution to form neutral copper atoms which, in turn, gel deposited on the copper rod. The
change taking place may be represented as :
F

Cu^'*' {aq) + 2 e~ ^ Cu (.V) ...(«)


The overall reaction taking place in two beakers is Zn {s) + Cu^'*’ {aq) 4
Zn^'*' {aq) + Cu (5)
It may be noted here that this is the same reaction which takes place when zinc rod is placed in copper
sulphate solution. The only difference is that by this arrangement we have separated the two half reactions,
i.e., oxidation of zinc occurs in one beaker and reduction of Cu2+ ions takes place in another beaker. Further,
the transfer of electrons now doe.s not take place directly from Zn to Cu^+ but through the metallic wire
connecting the two rods as shown by the deflection in the voltmeter which shows the flow of current.
*An inert electrolyte is one whose ions do not take part in the redox reaction and also do not react with the
solutions of the electrolytes taken in the two beakers. For example. KCl cannot be used if one of the electrodes is
silver electrode containing AgN03 solution becau.se KCl reacts with AgN03 solution to form white ppt. of AgCl.
8/39
REDOX REACTIONS

FIGURE 8.4
The cell based on the above reaction is commonly called Daniell
VOLTMETER
cell. A popular form of this cell is shown in Fig. 8.4. Here, salt bridge
is replaced by a porous pot through which SO^“ ions migrate from e

CUSO4 solution to the ZnS04 solution.


COPPER
8.73. Salt bridge and its Functions VESSEL

The two main functions of the salt bridge are as follows : POROUS POT

SO4; CUSO4
(j) It allows the movement of ions from one solution to the SOL.
SO
other without mixing of the two solutions. Thus, whereas electrons r4 ZINC ROD

ZnS04
flow in the outer circuit in the wire, the inner circuit is completed by SOL.

the flow of ions from one solution to the other and hence the current
A popular form of Daniell cell
flows.

w
(ii) It helps to maintain the electrical neutrality of the solutions in the two half cells due to flow of ions

F lo
(e.g., SO|" ions may shift and combine with the Zn-'*' ions in the first half cell).
If the salt bridge is not employed, the flow of electric current will not occur as explained below :
The moment the two rods are connected by a copper wire, the flow of electrons from zinc to copper rod
occurs for a short while. As a result of this transference of electrons, a net positive charge is developed near

the zinc rod. On the other hand, Cu2+ (aq) ions accept the electrons released by zinc rod and get deposited on

ree
the copper rod as Cu(s). This leads to an excess of SO““ ions in the solution and hence a net negative charge

for F
is developed around copper rod. The positive charge accumulated around zinc rod will prevent electrons to
flow from zinc rod towards copper rod while the negative charge developed around copper rod will not allow
Cu^'^(ag) ions to accept the electrons released by zinc rod. Consequently, the reaction will stop and no current
will flow. The use of salt bridge, on the other hand, avoids this situation as explained below :

number of SO^" ions will move from the salt


Your
As Zn^"*" ions are produced near the zinc rod, an equal
ks

bridge towards zinc rod thereby neutralizing the positive charge around zinc rod. Similarly, ions will
eBoo

move from the salt bridge towards copper rod thereby neutralizing the negative charge (due to an excess of

S02- ions around copper rod). As a result, the solutions in the two beakers remain electrically neutral and
thus allow the electric current to flow.
ad
our

Curiosity Questions
Q. 1. Why does the electrochemical cell stop working after some time ?
Ans. The electrode potential of any electrode depends upon the concentration of ions. With time, as
Re

fhe cell operates, the concentration of the metal ions (e.g., Zn^"*" ions) in the anodic compartment
increases and hence electrode potential of anode also increases. At the same time, the
Y

concentration of metal ions {e.g., ions) decreases in the cathodic compartment and hence
Find

electrode potential of the cathode also decreases. Ultimately, the electrode potentials of the
two electrodes become equal. At this stage, the cell stops working since E^en (which is equal to
the difference of the electrode potentials of the cathode and anode) becomes zero.
Q. 2. What would happen if no salt bridge were used In the electrochemical cell (e.g., Zn - Cu
cell)?
Ans. The positive ions (/.e., Zn^^) formed by loss of electrons will accumulate around zinc electrode
and negative ions (/.e., SO^“) left after reduction of Cu^* ions will accumulate around copper
electrode. Thus, the solution will develop charges and the current stops flowing. Further, since
the inner circuit is not complete, therefore, the current stops flowing.
J

t
8/40
j New Course Chemistry fxnrosrwm
8.7.4. Salient Features of an Electrochemical Cell
Some important features of an electrochemical cell are sumarized below :
(0 The zinc rod at which the oxidation occurs is called the anode while the copper rod at which the
reduction takes place is called the cathode.

ow
iii) The overall reaction taking place in an electrochemcial cell is due to two half reaction ; one occurring
in each beaker. Each beaker constitutes a half cell also called the electrode or the redox couple. Thus, a half
cell or an electrode consists of a metal rod dipped in the solution of its own ions,
(iii) The reaction taking place in a half cell is called a half ceil reaction. Thus, equations (/) and (ii)
represent two half cell reactions.
(iv) The half-cell reaction occurring at anode is called oxidation half cell reaction while that occurring

e
at the cathode is called reduction half cell reaction. For example, equation (/) represents oxidation half cell

re
reaction whereas equation (//) represents reduction half cell reaction. The overall redox reaction is then
obtained by adding the two half cell reactions,
(v) The two half reactions always take place simultaneously, i.e., half cell reactions cannot take place

F
Frl
independently.
(v/) Sine electrons arc produced at the zinc electrode, this electrode is irch in electrons and pushes these
electrons into the external circuit and hence acts as the negative pole or electrode. The copper electrode, on

ou
the other hand, is deficient in electrons (since it needs electrons to reduce Cu^"^ (aq) ions into Cu) and thus
pulls the electrons from the external circuit. As a result, it acts as a positive pole or electrode.

osr
(v/0 The electrons flow from the negative pole to the positive pole in the external circuit. However,
conventionally, the current is said to flow in the opposite direction,
(viii) As copper from copper sulphate solution is deposited on the copper electrode and sulphate ions

kf
migrate to the other side, the concentration of copper sulphate solution decreases as the cell operates.
Consequently, the current falls with the passage of time.
oo
(ix) As a result of redox reaction, the weight of copper rod increases while that of zinc rod decreases.
This gain and loss in weights of the metal rods is in the ratio of their equivalent weights. For example,
Y
B
Loss in weight of zinc rod _ Eq. wt. of Zn _ 32-50
Gain in weight of copper rod Eq. wt. of Cu 31-75
Some important generalizations about an electrochemical cell may be summed up as follows :
re
Y

● Oxidation occurs at the anode while reduction occurs at the cathode


u

● Anode acts as the negative pole while cathode acts as the positive pole
● Electrons flow from anode to cathode in the external circuit while current flows from cathode
ad
do

to anode.
● Chemical energy ofthe redox reaction occurring in the galvanic cell is converted into electrical
energy.
in
Re

SUPPLEMENT YOUR
KNOWLEDGE FOR COMPETITIONS
1. Only those electrolytes for which cations and anions have nearly the same ionic mobilities (i.e., distance
F

travelled by an ion per second under a potential gradient of one volt/metre) are used as electrolytes in the
salt bridge. Thus, KCl, KNO3, K2SO4 and NH4NO3 are preferred over NaCl, NaN03 and Na2S04.
2.
Among cations, ion has the highest ionic mobility and among anions, OH~ has the highest ionic
mobility. The ionic mobility of some common cations and anions follows the order:
Cations : H"*- > NH+ = > Ag-^ > Ca^+ > Mg^+ > Na+ > Li+
Anions : OH~ > SOf- > Cl" > NOJ > CO|“ > > CH3COO“.
3. Agar-agar is a seaweed colloid. It is a mixture of two polysaccharides, i.e., agarose (main) andagaropectin.
It dissolves in hot water and sets, on cooling, to a jelly at concentration as low as 0-5%. Its chief uses are
as
a solid medium for cultivating micro-organisms, as a thickner, as an emulsion stabilizer in food industry
and as a laxative.
REDOX REACTIONS
8/41

8.7.5. Representation of an Electrochemical Cell


An electrochemical cell is represented in a manner as illustrated below for the Daniell cell:
Zn I Zn2+ (ci) I [ Cu2+ (03) I Cu
By convention, the electrode on which oxidation occurs is written on the left hand side and the electrode
on which reduction occurs is written on the right hand side. The electrode on the left hand side is written by
wilting the symbol of the metal (or the gas) first followed by the symbol of the ion with its concentration in
brackets. The electrode on the right hand side is written by first writing the ion along with its concentration in
brackets followed by the symbol of the metal (or the gas). Single vertical lines represent the interface between
the two phases (e.g., sohd/liquid or liquid/gas) and the double line represents the salt bridge ● Cj and C2

w
represent the concentration of Zn2+ (aq) ions and Cu2+ (aq) ions respectively.
In a similar manner, Cu - AgNO-; cell may be repre.sented as Cu I Cu^"^ (Cf) 11 Ag+ (C2) I Ag
Instead of writing single vertical lines, sometimes semicolons

F lo
represented as
are used. Thus, the above cells may be
Zn;Zn2+ (c,)IICu2+ (03); Cu
Cu ; Cu2+(c^) 11 Ag+ (C2);Ag
Further, when the ions are in direct contact, e.g., for Daniell cell using a porous pot, a single vertical

ee
line
instead of a double vertical line is used. Thus, in such a case, we have : Zn ; (c,) I Cu-"^ (C2); Cu

Fr
8.7.6. Electrode Potential

n It has been discussed above that each electrochemical cell consists of two electrodes or half

for
cells. Each electrode, m turn, consists of a metal dipped in the solution of its own ions. At one of these
electrodes oxidation occurs while at the other, reduction takes place. In other words, one electrode has a
our
tendency to lose electrons while the other has a tendency to gain electrons.
The tendency ofan electrode to lose or gain electrons is called electrode potential. Since each electrode
s
represents a half cell, therefore, electrode potential is also called potential for half cell
ok
rr .r, electrode potential of any electrode depends upon the concentrations of the ions and temperature,
o

e concentration is taken as unity, i.e., I mol L ‘ (if any gas appears in the electrode reaction, it is taken at
Y
eB

1 atmosphere pressure) and temperature is taken as 298 K, the electrode potentials thus determined are called
standard electrode potentials and denoted by E°.
Fiirther the electrode potential is termed as oxidation potential if the electrode loses electrons and ISi-
r

called the reduction potential if the electrode gains electrons. Oxidation and reduction potentials are just
ad
ou

reverse of each other. For example, if the oxidation potential of an electrode is x volts then its reduction
potential is - x volts.
FIGURE 8.5
8.7,7. Effect of Concentration of Metal Ions and
Y

SOLUTION SOLUTION
Temperature on the Electrode Potential - 3 © 3 ©
Standard Electrode Potential
Re

3 © 3 0
nd

When an electrode say zinc rod is dipped in the solution 3 © 3 0


of its own ions, the following two opposing tendencies may 2e- 2e-
Fi

occur: 3 © 3 0
(0 The zinc atoms of the rod may lose electrons to form 3 © © 0
Zn Zn2+ Cu' 1^ Cu2+
Zn ions which pass into the solution and the electrons thus 9 © IONS 0 0 IONS
released accumulate on the rod (Fig. 8.5 a). 3 ©
IN SOLN.
3 ©
IN SOLN.

Zn (5) ■> Zn^"*" (aq) + 2 e~ (oxidation) 3 © 3 0


(ii) These accumulated electrons may attract the Zn^+
ions from the solution to form atoms of zinc metal which get OXIDATIONTENDENCY REDUCTIONTENDENCY
o o
deposited on the zinc rod.
Zn^"*" (aq) + 2 e~ Zn (j') (reduction) Development of a potential difference
between (a) Zn and ions and
These two opposing tendencies will continue and (b) Cu and Cu^'*’ ions
eventually the following equilibrium is reached.
8/42 p>uidec^ 4 New Course Chemistry (XI) BZsI9D

Zn (5) Zn^'*’ {aq) + 2 e


Now if the metal has a higher tendency to get oxidised, then at equilibrium the metal rod will acquire a
net negative charge w..rt. the solution. If, however, the metal ions have a higher tendency to get reduced (Fig.
8.5 b), a net positive charge will develop on the rod vv.r.f. the solution. This separation of charges, in turn,
creates a potential difference between the metal rod and the solution. This potential dijference which is set up
between the metal and its own ions in the solution is called the electrode potential. The magnitude of this
potential difference, however depends upon the following factors,
(i) the nature of the metal and its ions (ii) the concentration of the ions in solution (Hi) temperature
Thus, we conclude that electrode potentials depend upon the concentration of the metal ions in solution

w
and the temperature. Therefore, electrode potentials are generally measured under standard conditions, i.e., l
molar concentration of metal ions (1 mol L"') and a temperature of298 K and are ar/M standard electrode
potentials and are denoted by E®.

F lo
Thus, to define a standard electrode potential for a half cell or an electrode, we write
Cu^+ (1 mol L-*, aq) + 2e^ > Cu {s)
The equation for the half cell is written as a reduction reaction and the electrode potential is called

ee
standard reduction potential.
8.7.8. Measurement of Standard Electrode Potential

Fr
The absolute value of potential for a single electrode cannot be measured directly because of the following
three reasons : (i) A half cell reaction cannot take place independently,

for
(ii) It is a relative tendency to lose or gain electrons.
(Hi) For purpose of measurement, as soon as another metal conductor is put into the solution, it will set
ur
up its own potential.
In view of these difficulties, the electrode potential has to be measured against some reference electrode.
s
The reference electrode used is the standard or normal hydrogen electrode (NHE).
ook
Yo
It consists of a platinized platinum electrode (platinum electrode having a coating of black platinum)
dipped in 1 M solution of H+ ions (1 M HCl) at 298 K and pure hydrogen gas maintained at a pressure ot 1
atmosphere, is bubbled through the solution containing platinized platinum electrode (Fig. 8.6).
eB

The finely divided black platinum coated over platinum FIGURE 8.6
absorbs H2 and thus helps to establish a rapid equilibrium
between H-, and ions :
r

H2 gas at
1 ATM PRESSURE
When in a cell, this electrode acts as the anode i.e..
ad
ou

oxidation takes place, the following reaction occurs, i.e.,


some hydrogen gas changes into H'*’ ions which go into the COPPER WIRE
Y

solution.

H2 (8) ^ 2H+ (aq) + 2 e~ BUBBLES- Hg


Re
nd

When this electrode acts as the cathode, i.e.. reduction OF H2 gas


IM HCl SOL.
takes place, the following reaction occurs : ●●Q4T -

PLATINUM FOIL
Fi

2H-*- (aq) + 2e~ ^ H2 (g) 0-


COATED WITH

i.e., some ions from the solution change into H2 Pt BLACK

gas. Thus, the electrode is reversible with respect to IF' ions. Standard Hydrogen Electrode
This electrode is usually represented as :
Pt, H2 (g, 1 atm), (aq, cone = c)
The electrode potential of the standard hydrogen electrode is taken as zero.
this electrode as one of the
To determine the electrode potential of any electrode, a cell is set up using
electrodes and the second electrode is the standard hydrogen electrode. The EMF of the cell is rneasured. As
the EMF of the cell is the difference in the electrode potentials of the two h^f cells
potential of the standard hydrogen electrode is taken to be zero, therefore the EMF of such a cell will directly
give the electrode potential of the cell under investigation.
REDOX REACTIONS
8/43

The direction of flow of current further indicates whether oxidation or reduction takes place at the
e ectrode under investigation with respect to the hydrogen electrode. According to lUPAC convention, the
e ectrode potential is given a posiUve sign if reduction occurs and a negative sign if oxidation occurs at the
electrode under investigation with respect to the hydrogen electrode.
8.7.9. Electromotive Series
The standard electrode potentials of a large number of electrodes have been determined using standard
hydrogen electrode as the reference electrode, for which the electrode potential has been arbitrarily taken as
zero. By convention, standard electrode potentials refer to reduction reactions. It is because of this reason
that earlier they were referred to as standard reduction electrode potentials. If. however, reactions are written
m the opposite way, t.e. , as an oxidation reaction, the electrode potentials are referred to as standard oxidation

w
deeWe potentials. Since reduction half reaction is just the reverse of oxidation half reaction, the oxidation
potential of any electrode is obtained from the reduction potential just by changing the sign. For example, the
standard reduction potential of the electrode Zn^+ZZn (aq) for the reaction,

F lo
Zn--*- (aq) + 2 e" > Zn (s) is - 0.76 V.
then the standard oxidation potential of the electrode Zn/Zn^^ (aq) for the reaction,
Zn (^) ^ Zn2+ (aq) + 2 e" is + 0.76 V.

ee
nnH ‘‘dopted by lUPAC. the terms standard oxidation electrode potential

Fr
and standard reduction electrode potential are not used but only the term standard electrode potential IS
i used
and the halt cell reactions are always written as reduction reactions.

for
, electrode potential is given a positive sign if reduction occurs at that electrode w.rt the
Tnfri electrode and is given a negative sign if oxidation occurs at that electrode w.rt. the
Tndeni
tendency JfZTlf 1 to occur' ' m the forward direction, i.e., inelectrode potential is a measure of the
our
of the half reaction the direction of reduction.
at 70 Jk n™ber of electrodes at 1 M concentration of the dissolved ions
s
ook

streZh^ H O’^'dising agents are written in decreasing order of their


ngth i.e , *e tendency of the oxidation half reaction goes on decreasing as we move from top to bottom
Y
eB

or

8.7.10. Applications of the Electromotive Series


Some of the important applications are given below
our
ad

mthe .substances
K T” compare the relative .strength of oxidising and reducing agents.
are airanged m decreasing order of
In the electrochemical series
their electrode potentials. This means that as we move
DOwer of llieT h T -substance for reduction goes on decreasing. In other words, the oxidising
Z7d7nvf ot IT goes on decreasing. Therefore, the substances at the top of the table have the maximum
dY

bn fnm n/Y hi “r T oxidising agents. On the other hand, substances at the


Re

bottom of the table have lower electrode potentials, therefore, they have lower tendency to get reduced
at the bottom of the table can be easily oxidised and hence act as strong reducing
Fin

From the Table 8.1., page 8/44 it is evident that standard electrode potential of F, is the highest and
DomntSs H ^ is “ very poor reducing agent. Since thi standard electrode
potentials of ha ogens decreases m the order F, > Cl^ > Br, > I^, therefore, their oxidising power decreases
decreV^™'" ^ ^ ^ standard electrode potentials for halide ions
nrwem
powers of'Ltrf
of the halides ■'hdecreases m the ^same order,
' i.e., T > Br“ > Cl“ > F“^ ^ V), therefore, reducing^
Dotenh-rz "“’Z" ittis the lowest while Li+ (+ 3 05 V) has the highest electrode
potential, therefore, Li metal is the strongest reducing agent while Li+ is the weakest oxidising agent Since
(y) n + {bv) ^.n
so-e - (jus3b SupnpaJ isaSuoJis) <■

(O')! <r + {bD)


£6-2 -
(y) «a f _3l + {bD) ^2^8
06*2 -
(y)Bo ■e _3l + {bD) ^^0
28-2 -
(y) BM <- _3 + {bD) ^BN
\L-Z -
(y) §IM f _3l + {bv)
L£-Z -
(y) IV <■ _a£ + {bo) ^.giv
991 -
+ (/)
£80 (bD) _h02 + (^) <■

(y) uz f _3l + {bD)


92 0 -
(y) JO + (^») +e^3

w
<■
PLQ~
(y)3d f _9Z + (to)
tt-0 -
(y) po <r + {b^^) +zPD
01^-0 -
_3z + (,v) t'osqd
1£0- (to) 4os + (y) qd <-

+ {i»») +zOD

Flo
82 0 -
(y) 03 <r

(y) !N <■ _9Z + {bo)


£2-0 -
(y) us _3Z + (to)

e
<-
t^l-0 -
_3Z + (to)

re
£1-0 -
(y) qd <■

_3£ + {bD)
O

9£0 0 - (y) 3d <-


(D

F
+ (^'^) +UZ 0}

000 503 (apoipap pJBpuBis) (^) <-

_3 + (s) ja^v
CO
D

010 + CO _iQ + (S) 3y f CQ


_a + (s) dSv CO
-13 + (y) Sv
D
ur
r
<■
22-0 + CO
_9 + (to) <t>

fo
81-0 +
CO
(to) +H3 f
CO
CD
(y) U3 + (bo)
P£-0 + 3
(D
(y) U3 <■
ks _9 + (^») +no O
2S-0 + ZT
o (bo) -12 <- -33 + (^) o
X
Yo
i?e-o +
(/) <r -K + +HZ + (^) a.
oo
CD
89-0 + CO
Q.
C (/»D) <r _a + {bD) 3
22-0 + o CO

-9Z + (^») +|^H


B

3
62-0 + CO (/)2h 2 i- 0)
CO
(D CD
CO
(y) §V _a + {bv) ^Sy 3
re

<r
08-0 + CD CO

S8-0 +
3
CO
(/) 3h f -32 + (bty) +j3h
(/) O^H 2 + (^)ON f _3£ + 4.H^ + W -ON
u

26-0 +
ad

(^) -J82 <- -32 + {^) ^Jg


Yo

80-1 +
O^H2 + +^u^^^ <■ -32 + +Ht^ + (^) ^OUI^
£^●1 +

(;) O^H£ <■ _32 + (to)+o%3+ (^)^O-


£2-1 + 1
d

-39 + -jO^JO ▼
Re

££●1 + (y) O^H2 + +i~^OZ <r


in

(bo) _I32 <r -32 + (^) ^13


9£'l +
+ (to) ^o^H8 + (M -Om
F

6V1 + 0)O^H2I +(bo)^^m <r

(y) nv _3Z + (to)


OS-1 +
82-1 + (?) 0"H2 «● -32 + +H 2 + (^J')
18-1 + (to) +2«0 f (1U3§B -3 + (bv) +^03
(jUagB Supnpsj JS32jB9/^) (^J’) -d3 <- Suisipixo jsaSuoJis) -32 + (S) ^d
28-3 +

U0IpB3H 3pOJp3(3
(SJIOA UI) IBRuajod apojjDaja pjBpuejs
H S6Z siBpuajod apojjoap pjepue^s 1-8 31gVX

HifaUCT (DO XJ[JS|UI3xi3 9Sjno3 m3N ^


REDOX REACTIONS
8/45

From Ae above discussion, it foUows that all substances which have positive electrode potentials are
stronger oxidising agent than H* while all substance which have negative electrode potentials are stronger
reducing agents than H2 gas. *
In other words.
(0 A negative E° means that the redox couple is a stronger reducing agent than H^/H2 couple.
(ii) A positive
. E° means that the redox couple is ^ stronger oxidising agent than W-/H2 couple.
of metals. In general, metals having low electrode potentials are easily
Tn r® electrons can reduce the ions of other metals having higher electrotte

low
Sts? " potentials (f.e., lying lower in the ekctrochemical
J potenliais (U, lying higher up in the electrochemical
■ ‘u- ^ 0/their salts. For example, Zn having lower electrode potential (- 0-76 V) can
ha^ttt ‘^“^4 ttverse never
Zn (s) + CUSO4 (aq) ^ZnSO^(aq) + Cu(s)

ee
f+
(+ 00 80V1
80 V) from the solution of AgN03 potentialreverse
(+ 0-34does
V) can displace Ag with higher electrode potential

rF
Fr
not occur.
Cu (j) + 2 AgNOj (aq) ^Cu(NOj)2(aq) + 2Ag(s)
whether a metal will liberate hydrogen from the aqueous solution of acids or not All
etals wiA negative electrode potentials are stronger reducing agents than hydrogen and hence would liberate

for
hydrogen from the aqueous solutions of acids. Convemely, all me^ with positive values ofdS^e SS
ou
acms. inus, metals like Mg, Al, Zn, Fe, Sn, etc. will liberate hydrogen but
metals like Cu, Hg, Ag etc. do not
liberate hydrogen from the aqueous solutions of acids.
s
ok
D0int's‘.f^„rirHiffr“'“— ^ flowa throngh two

S ^ difference IS said to exist between them. The potential difference generated by a cell when
o

th? «nStoi7al"° " conditions of zero electron flow) is caUed electromotive tom (EMF)
Y
eB

or

m.t,i ? T «>““ochemical cell consists of two half cells, i.e„ electrodes. One of these electrodes
mtentifd'ff ® TT than the other electrode. As a ersult
tow^renrisf *c c'cctrode at a higher potential to the electrode at a
r
ou
ad

*’ ” ftom less positive


to more posmve electrode. However, cuiient flow is in a direction opposite to the electron flow
Y

EMF of a ceUmay be defined as the difference in the electrode potendais of the two had cells
When the cell is not sending current through the circuit.
nd
Re

the an^eCm ftafrd,°e“,^Sr^ '"“®


Fi

of

E“.cell
- [Standard electrode potential of cathode] - [Standard electrode potential of anode] (/)
or E“cell = F»
cathode -E°
anode ,„(ii)
an “ P'““* *c left and Cathode is placed on the irght while erpresenting
an elecdxichemical cell, therefore, the EMF of the cell is given by the expression
E“«U = E” R .(Hi)
where E“r and E°l refer to the standard electrode potentials of the cathode and anode erspectively.
In order to determine the reaction taking place in the cell, the following steps are followed,
(i) Write reduction
othe.r
equations for both the electrodes along with their electrode potentials, one after the
8/46 ‘P'Kidee^'^ New Course Chemistry (XI)BEnQl

(//) Balance the electrical charges and the number of atoms of each element on either side of each of
the above two reduction equations,
m Multiply each reduction equation by a suitable integer so that the number of electrons involved in
both the half reactions are equal.
It may be noted that while multiplying halfreacHons with suitable integers, their E" values are not
multiplied by any such integers because the.se values simply convey the tendency for reduction.
(iv) Subtract the equation with lower electrode (reduction) potentialfrom the one having higher electrode
potential. This difference gives the EMF of the cell.
The electrode with higher reduction potential has a strong tendency to gain electrons and hence acts as
the cathode while the electrode with lower reduction has a strong tendency to get oxidised potential acts as

w
the anode.

sample Problem Q \ cell IS prepared by dipping a chromium rod in 1 M Cr2{S04>3 solution and

F lo
an iron rod in 1 M FeS04 solution. The standard reduction potentials of chromium and iron electrodes are
- 0*75 V and - 0*45 V respectively.
(a) What will be the cell reaction ? (b) What will be the standard EMF of the cell ?
(c) Which electrode will act as anode ? (d) Which electrode will act as cathode ?

ree
Solution. The two half cell reduction equations are :
...(/)
> Fe is); E° = - 045 V

for F
(aq) + 2e~
...(h)
(aq) + 3 e~ > Cr (s) ;E^ = - 0-75 V
the anode while Fe^"^/Fe electrode
Since Cr^'*'/Cr electrode has lower electrode potential, therefore, it acts
as

with higher electrode potential acts as the cathode.


by 3 and Eq. (ii) by 2. But do not mulhply the.r E
. . co

To equalise the number of eleetrons, multiply Eq, (0


values. Thus,
Your
> 3 Fe (5); E° = - 0 45 V ...(Hi)
3 Fe-+ (aq) + 6e'
ks

^ 2 Cr (s); E“ = - 0-75 V ...(iv)


eBoo

2 (aq) + 6e^
To obtain equation for the cell reaction, subtract Eq. (iv) from Eq. (Hi), we have,
2 Cr (s) + 3 Fe2+ (aq) >2 (aq) + 3 Fe (s); E^^cu = " - (- 0-75 V) = + 0-30 V
Thus, the EMF of the cell = + 0*30 V
ad
our

Sample Problem 0 The half cell reactions with their oxidation potentials are
Pb (s) > Pb--^ (aq) + 2 e- ; = + 0-13 V, Ag (s) ^Ag^(H^) + ^-;E”„^=-0*80V
Write the cell reaction and calculate its EMF.
Solution. Rewrite the two equations in the reduction form by reversing the signs of oxidation potentials. Thus,
Re

...(i)
Pb^+ (aq) + 2e~ >Pb(5); E° = -013 V
Y

...(iO
Ag^ (aq) + e > Ag (s); E° = + 0-80 V
Find

To obtain the equation for the cell reaction, multiply Eq. (ii) with 2 and subtract Eq. (.) from it, we have,
Pb (5) + 2 Ag"^ (aq) »Pb
2+
(aq) + 2 Ag (s); E”„„ = + 0-80 - (- 0 13) = + 0-93 V

1. A cell is prepared by dipping copper rod in 1M copper sulphate solution and zinc rod in IM ZnS04 solution.
The standard reduction potentials of copper and zinc are + 0-34 and - 0-76 V respectively.
(i) What is the cell reaction ?
(ii) What will be the standard electromotive force (EMF) of the cell ?
(ill) Which electrode will be positive ? (iv) How will the cell be represented ?
REDOX REACTIONS
8/47

2. Following cell is set up between copper and silver electrodes : Cu I Cu^+ {aq) 11 Ag+ {aq) I Ag
If its two half cells work under standard conditions, calculate the e.m.f. of the cell
[Given E‘ = + 0-34 volt, E'
Cu2+/Cu = + 0-80 volt]
Ag-^/Ag
3. Wnte the cell reaction and calculate the standard E" of the cell : Zn I Zn^+ (1 M) 11 Cd“+ (1 M) I Cd
Given E® = 0-763 volt, E' = 0-403 volt
Zn,Zn2+ Cd.Cd2+

4. The standard EMF of the cell : Ni I Ni"+ 11 Cu^+ I Cu

low
standard electrode potential (reduction potential) of copper electrode is 0-34 volt Calculate
the standard electrode potential of nickel electrode. '-aicuiate
5. The e.m.f. (E“) of the following cells are
Ag I Ag- (1 M) II Cu2- (1 M) Cu ; E» = - 046 V, Zn I Zn^- (1 M) 11 Cu^- (1 M) I Cu : E» = + MO V
Calculate the e.m.f. of the cell Zn I Zn^+ (I M) 11 Ag"^ (1 M) I Ag
6. Two half cells are AI-^+ (a^)/Al and Mg^-^ {aq)M%

ee
The reduction potentials of these half cells are - 1 -66 V and - 2-36 V respectively. Calculate
potential. Write the cell reaction al.so.

rF the cell

Fr
7. Calculate E® for the cell : A11 Al '’+ (1 M) 11 Cu~+ (1 M) I Cu.
Given : E° and E®
ai^Vai Cu^+ZCu - 1-66 V and + 0-34 V respectively.
as

for
ANSWERS

1- (0 Zn (s) + CuSO^ (aq)


u
^ ZnS04 (aq) + Cu (s); (//) E“,^„ = M V ; (Hi) Cu electrode
s
(/V) Zn I ZnS04 (1 M) 11 CUSO4 0 I Cu
ok
Yo
2. E® cell = E® -E® = E® - E®
Cu2+/Cu =+0-80-(+0-34) = 0-46 V
cathode anode
Ag'*'/Ag
o
eB

3.E cell = 0-360 volt, Zn -1- Cd^"^ Zii2+ + Cd 4. E® - - 0-25 volt


Ni-+.Ni
5. 1-56 V
7. 2-0 V
6. 0-70 V, 3 Mg + 2 A13+ ^ 3 Mg~* + 2A\
r
ou
ad

HINTS FOR DIFFICULT PROBLEMS


3. Reverse the signs of oxidation potentials to get the values of the reduction or electrode potentials. Thus,
Y

^Zn,Zn2^ " ^ ■■ V and ■.■ e® Cd.Cd2+


, ^ = 0-403 V .-. E® Cd^+ ,,
Cd
= -0-403 V
Re
nd

with

In other words, Zn loses electrons and Cd^^ ion accepts them.


Fi

Therefore, cell reaction IS


i
Zn -*● Cd2+ ■> Zn^+ + Cd
and E“ = E® -E®
Zn^'^.Zn = - 0-403 - (- 0-763) = + 0-360 V
cell
Cd^'^.Cd
5. - 0-46 = E® -E“
Cu-'^,Ci! Ag'^.Ag ...(0
1-10 = E® — F°
Cu2+.Cu Zn2+.Zn ...(«)
Subtracting Eq. (i) from Eq. {i7), we have,

cell = 1-10-(-0-46) = E® — E° or 1-56 V=E® — E®
Ag'^.Ag Zn+,Zn Ag+,Ag Zn^+.Zn
In other words, E“ of the cell, Zn I Zn^^ (M) I Ag+ (M) I Ag, is 1-56 V.
8/48 pna.decp^'4. New Course Chemistry (XI)lffilHI]
6. Since, {aq)M% electrode = - 2-36 V is at a lower potential than (a^)/Al electrode = - 1-66 V,
therefore, {aq)(U% electrode acts as the anode and Ap"^ (fl^)/AI acts
as the cathode. In other words,
Mg loses electrons and AP"^ ion accepts electrons. Thus, the cell reaction is
3+
>3Mg--^ + 2Al and -E° = _ 1-66-{-2-36) = +0'70V
3Mg + 2Al

w
7. E“ celt = E° E° = 0-34-(- 1-66) = 2-0 V
Cu^+.Cii Al^'^.Al

5 To predict the Feasibility or Spontaneity of a Redox Reaction


The EMF of a cell is an important property. It can be used to predict the spontaneity of a redox reaction as

o
discussed below.

e
To predict whether a given redox reaction is feasible or not. the EMF of the cel! based upon the given redox

re
reaction is calculated. // the EMF comes out to be positive, the reaction mkes t^e reTrL

rFl
out to be negative, the forward reaction as shown by the redox equation does not occur , instead, the reverse
reaction occurs. The following examples will illustrate this point.

F
Sample Problem Q Predict whether zinc and silver react with
PROBLEMS 1 M sulphuric acid to give out hydrogen or not. Given that the standard
potentials of zinc and silver are - 0.76 volt and + 0.80 volt respectively.

r
ou BASED
Solution, (a) To predict reaction of zinc with sulphuric acid :

fo
ON
If Zn reacts, the following reaction should take place

ks
Zn^-*- + H,
Predicting
Zn + H2SO4 > ZnS04 + H2, Le., Zn + 2H+ ^
4

Spontaneity By convention, the cell may be represented as Zn 1 Zn*"^ 11 I H2


of a Reaction
oo
Standard EMF of the cell, - E° H+ZHj
Y
= + 0.76 volt
B

Thus, the EMF of the cell comes out to be positive. Hence, the reaction takes place.
re

ib) To predict the reaction of silver with sulphuric acid .


If Ag reacts, the following reaction should take place :
> Ag2S04 + H2, i.e. 2Ag + 2H+ 4 2Ag+ + H2
ou

2Ag + H2SO4
Y
ad

By convention, the cell may be represented as Ag 1 Ag"^ 11 I H2


E° cell = £'■ -E® = 0 - 0.80 = - 0.80 volt
Ag+,Ag

negative. Hence, this reaction does not take place.


d

Thus, the EMF of the supposed cell comes out to be


= 0-0 V, therefore, Zn is a stronger reducing
in

= - 0-76 V is lower than E,


Re

Alternatively, since E Zn^-^/Zn


= 0-80 V is higher than
agent than H2 and hence can reduce ions to evolve H2 In contrast, since Ag'^/Ag
F

that of E° . = 0 0 V, therefore. Ag is a weaker reducing agent than H2 . and hence cannot reduce H* ions to
evolve H-) gas.
Sample Problem 0 Can a solution of 1 M copper sulphate be stored in a vessel made of nickel
= + 0.25 volt, E" = - 0.34 volt.
metal ? Given that E® 2+ Cu,Cu
2+ “
Ni.Ni
or Is it possible to store copper sulphate solution in a nickel vessel ?
Solution. In this problem, we want to see whether the following reaction takes place or not
2+
> Ni^+ + Cu
Ni + CUSO4 > NiS04 + Cu, i.e., Ni + Cu
By convention, the cell may be represented as Ni I Ni^'*' 11 Cu^+ I Cu
REDOX REACTIONS 8/49

We are given the oxidation potentials as E“ Ni. = +0-25 volt and E® 2+


= -0-34 volt
Cu,Cu

Hence, by reversing the sign of oxidation potentials, the reduction potentials will be
E" = -E° = -0-25 volt and E = -E° = +0-34 volt
Ni,Ni
2+
Qi^'^.Cu Cu,Cu
2+

Now E° = E° E° = + 0.34 - (- 0.25) = + 0-59 volt


cell Cu^'^/Cu Ni-'*'/Ni
Thus, EMF conies out to be positive. This implies that CUSO4 reacts with nickel. Hence, CuSO^ solution
cannot be stored in a nickel vessel.

w
Alternatively, Since E° = - 0-25 V is lower than E° = + 0-34 V, therefore, Ni can easily lose
Ni-^.Ni Cu^-^.Cu
electrons to Cu^-*- ions. In other words, the following reaction, Ni + Cu^+ Ni--' + Cu

will occur. Thus, CuSO^ solution cannot be stored in a nickel vessel.

Flo
e
re
1. Predict reaction of IN sulphuric acid with the following metals : (i) copper (ii) lead {Hi) iron

F
Given E° = +0-34 volt ; E° = - 0.13 volt; E‘ = - 0.44 volt
Cu^+.Cu Pb^^.Pb Fe^-'.Fe
2. Can a solution of I M ZnS04 be stored in a vessel made of copper ? Given that
ur
r
E‘ = + 0.76 volt, and E° = 0.34 volt

fo
Zn.Zn
2-»-
Cu^+.Cu
3. Is it safe to stir 1 M AgN03 solution with a copper spoon ? Given

Ag-'.Ag
= 0.80 volt, and E°
Cu,Cu
2+
ks
= - 0.34 volt. Explain
Yo
4. Can we use a copper vessel to .store 1 M AgN03 solution ? Given that
oo
E° = + 0-34 V and E° = + 0-80 V.
Cu^-^/Cu Ag-'/Ag
B

5. Why blue colour of copper sulphate solution gets discharged when zinc rod is dipped in it ?
(Given E* = 0-34 V and E° = 0-76 V)
Cu2+/C u Zn/Zn 2+
re

ANSWERS
u

1. (i) No (ii) Yes (iii) Yes 2. Yes 3. No 4. No


ad
Yo

HINTS FOR DIFFICULT PROBLEMS

1. If the metal is to react with dil. H^S04 (i.e., H-*" ions) to produce H2 gas, the metal should have a lower
d

electrode potential than that of standard hydrogen electrode, i.e., 0 0 V.


Re
in

(0 Since, E‘
Cu^+.Cu
= 0-34 V is higher than E° = 0-0 V, therefore, Cu will not react with 1 N H2SO4
H+.H^
to produce H2 gas.
F

{ii) Since, E° = -0-13 Vand E’’ = 0-0, therefore, lead will react with IN H2SO^ to produce
Pb^-'.Pb H+.H2
H2 gas.
{Hi) Since E° 0-44Vand E°
H+.Ho
= 0-0 V, therefore, iron will react with IN H2SO4 to produce
Fe^-'.Fc
H2 gas.
2. Since, E = -E° = - 0-76 V is at a lower potential than E‘ = 0-34 V, therefore, Zn
Zn^^.Zn Zn.Zn
2+
Cu^+.Cu
can only lose electrons to Cu^'' ions. Conversely, Zn^* cannot accept electrons from Cu and hence the
following reaction will not occur.
Zn^+ + Cu Zn + Cu^+
In other words, ZnSO^ .solution can be safely stored in a copper vessel.
8/50 i New Course Chemistry (XI) orsTMn

3. Since, E‘ = + 0-80 V is higher than E° = -E' = - (- 0-34) = + 0-34 V, therefore,


Ag+,Ag Cu-+,Cu Cu^+.Cu
Ag"^ ions can easily accept electrons from Cu. In other words, the following reaction will occur.
Cu + 2 Ag+ > Cu^+ + 2 Ag
Hence, 1 M solution cannot be stirred with a copper spoon.
4. Since, Cu will react with Ag"*" ions, as discussed in Ans to Q. 3 above, therefore, AgN03 solution cannot be
stored in copper vessel.
5. Since, E“ Zn.Ziv^^ ^
Zn'^+.Zn = “ ^^'26 V is lower than E‘ Cu2+,Cu , therefore, Zn will lose electrons and
copper will accept them. In other words, the following reaction will occur, Zn + Cu^"*" Zn2+ + Cu

w
Since, blue Cu^'*' ions are consumed and colourless Zn-'*' ions are produced during the above reaction,
therefore, colour of CuSO^ solution gets discharged when zinc rod is dipped in it.

F lo
8.8. APPLICATIONS OF REDOX REACTIONS

Oxidation-reduction reactions are the basis of many applications of chemistry in industry and in our
daily life as discussed under ‘General Introduction in Art. 8.1, page 8/1. Some additional applications are

ee
discussed below :

Fr
1. Extraction of metals. By using a suitable reducing agent, metal oxides can be reduced to metals. For
example, Fe203 is reduced to iron in the blast furnace using coke as the reducing agent.
Fe203 (5) + 3 C is) 2 Fe is) + 3 CO (g)

for
Similarly, Al-jO^ is reduced to aluminium by cathodic reduction in an electrolytic cell.
ur
Other metals such as lithium, sodium, potassium, magnesium, calcium, etc. are also obtained commercially
by electrolytic methods.
s
2. Electrochemical cells or batteries. Electrochemical cells or batteries based on redox reactions are
ook
widely used in our day today life to run a number of small and big gadgets and equipments. For example
Yo
storage cells are used to supply all the electrical needs of our cars, trucks, buses, trains, aeroplanes, etc.
Similarly, electrical energy needed in the space capsule is obtained by the reaction of hydrogen and oxygen in
eB

fuel cells which are electrochemical cells using oxygen and hydrogen electrodes.
3. Photosynthe.sis. Green plants convert carbon dioxide and water into carbohydrates in presence of
sunlight. This reaction is called photosynthesis and is sensitized by chlorophyll.
r

Sunlight
ad
ou

6C02(g) + 6H20(/) Chlorophyll


■> + 602(g)
Carbohydrate
Y

During this reaction, CO2 is reduced to carbohydrates while water is oxidised to oxygen. The energy
needed for the reaction is provided by sunlight.
Re

This reaction is a source of food for plants and animals. It also maintains a constant supply of 21% of O2
nd

by volume in the atmosphere needed for combustion of fuels and breathing of all living creatures in the
world.
Fi

4. Supply of energy . The energy required for our daily needs is obtained by oxidation of fuels. For
example, oxidation of fuels such as wood, gas, kerosene, petrol, etc. produces a large amount of energy which
we need for various purposes in our daily life.
Fuels (wood, petrol, kerosene, gas) + O2 ■> CO2 + H2O -I- Other products + Energy
Human body also needs energy for proper functioning. This is obtained by the oxidation of glucose in
our body to CO2 and water.
CgHi206(o(?) + 602(g) 6 CO^ (g) + 6 H2O (/) + Energy
Glucose

5, Production of chemicals. Many chemicals of our daily needs such as caustic soda, chlorine,fluorine,
etc. are produced by electrolysis which is based on redox reactions.
REDOX REACTIONS 8/51

6. Quantitative analysis. Redox reactions arc very useful in quantitative analysis by redox titrations.
These titrations involve the reactions between oxidising and reducing agents and help in estimating the amount
of unknown substances in solutions.

8.9. SOME ADDITIONAL USEFUL INFORMATION ABOUT REDOX REACTIONS


I. Useful Tips for Calculating Oxidation Numbers when the compound contains covalent and
coordinate covalent bonds. To calculate the oxidation number of atoms in such molecules, the following
rules are generally used.
1. For each covalent bond between two dissimilar atoms, assign an oxidation number of+1 to the less

w
electronegative atom and - I to the more electronegative element.
For example, O.N. of the two carbon atoms in CH3COOH may be calculated as follows:

Flo
H O

H—C—C—O—H
1
H

ee
C2 is attached to three H-atoms (less electronegative than carbon) and one —COOH group (more

Fr
electronegative than carbon), therefore, O.N. of C2 is 3 x (+ 1) + .r + 1 x (- I) = 0 or at = - 2
C] is, however, attached to one oxygen atom by a double bond, one OH (O.N. = - 1) and one CH3
(O.N. = + 1) group, therefore, O.N. of C [ is + i +X + 1 x (- 2) + 1 x (- I) = 0 or = + 2

for
ur
If, however, the covalent bond is between two similar atoms or between two similar atoms which are
further attached to similar species, each atom is given an O.N. of zero. For example, the central C atom in
carbon suboxide (030-5) has an O.N. of zero while each terminal carbon has an oxidation state of +2.
ks
+2 0 +2
Yo
0=0 =0=0 =0
oo
2. In case of coordinate covalent bonds, two cases arise : (/) If a coordinate bond is formed between
less electronegative than the acceptor atom, assign
same atoms or dissimilar atoms but the donor atom is
eB

an
oxidation number of +2 to the donor atom and -2 to the acceptor atom,
(ii) Conversely, if the donor atom is more electronegative than the acceptor atom, neglect the contribution
of the coordinate bond.
r
ou

The following examples will illustrate the above rules


ad

(a) Oxidation number of C in H - C = N and H - N ^ 0


Y

(/) By conventional method. Since there are no standard rules for determining the oxidation numbers of
0 and N, therefore, conventional method cannot be used to calculate the O.N. of O in HON or HNO.
(ii) By chemical bonding method. Since N is more electronegative than 0, therefore, each covalent
Re
nd

bond gives an O.N. of - 1 to N. Now since there are three covalent bonds, therefore, the O.N. of N is HON
is -3.
Fi

+1 X -3
Now, H C N +1 + .r - 3 = 0 or .v = +2

Thus, the oxidation number of C in HCN = + 2.


Oxidation number of C in H—N = C. Here, the contribution of coordinate bond is neglected because
the donor atom, i.e., N is more electronegative than the acceptor atom, i.e., 0. The O.N. of N in
H—N = O remains to be - 3 since it has three covalent bonds. Thus,
lx(+l) + lx (-3) + X =0 or +1 -3 + X = 0 or Jt = +2

(for H) (for N) (for C)

Thus, the oxidation number ofC in HNC = + 2.


8/52 New Course Chemistry (XI)SSEan]

aiO. TYPICAL PROBLEMS

R 1. M g of a sample of copper ore is dissolved and Cu^'*’ {aq) is treated with KI. The iodine thus
liberated required 12*12 mL of 0*1 M Na2S203 solution for titration. What is the percentage of
copper in the ore ?
Sol. The complete balanced equation for the redox reaction is

2 Cu2+ + 4 r + 2 $20^- Cu2l-> *f" S-^Og +21


12-12
No. of moles of S2O5" used = xO-1 = 1-212 X 10 ^ moles
1000

w
From the balanced equation,

2 moles of 820^“ reduce Cu^'*’ = 2 moles

F lo
1-212 X 10"^ moles of 820^“ will reduce = 1-212 x 10“^ moles
Wt. of pure Cu present in the ore = 1-212 x 10”^ x 63-5 = 0-077 g [v At. wt. of Cu = 63-5]
0-077

ee
Thus, %age of Cu in the ore = xIOO =7%
I-l

Fr
F. 2. An aqueous solution of 0*10 g KIO3 (formula weight = 214*0) was treated with an excess of KI
solution. The solution was acidified with HCL The liberated I2 consumed 45*0 mL of thiosulphate
solution to decolourise the blue starch-iodine complex. Calculate the molarity of sodium thiosulphate

for
solution.
ur
Sol. The reactions involved are :

2 IO3 + 12H++ lor > 6 I2 + 6 H9O


s
2S2O5- +I2- S4O2- + 2 n X 6
ook
Yo
2IO3 + 12H++ I2S2O2- ^ 6 S4OI- + 21- + 6 H2O
eB

0-1
No. of moles of KIO3 = 214

Now 2 moles of KIO3 react with Nao$203 = 12 moles


our
ad

0-1 12 0-1
mole
214 mole of KIO3 Nu2S203 214

12 0-1
Now — X— mole of Na2S203 is present in 45 mL of the solution.
Y

214
Re

12 0-1 1000
nd

Molarity of Na2S203 solution =—X 214


X —
45
= 0*0623 M

mtk 2*68 X 10 ^ moles of a solution containing an ion A"'*' require 1*61 x 10 ^ moles Mn04 for the
Fi

oxidation of A”'*' to AO3 in acid medium. What is the value of n ?


Sol. Step 1. To write the reduction and oxidation half reactions.
1+
Reduction : MnO^ + 8 H'*’ + 5 » Mn + 4 H2O -(0
-t-5
Oxidation : A'’-^ + 3 H2O ■> AO3 + 6 H'*’ + (5 - «) e ...(H)
Step 2. To find out the value of n.
Since in a redox reaction, number of electrons lost = number of electrons gained, therefore, multiply
oxidant of Eq. (/) i.e., MnO^ by (5-n) and reductani of Eq. (ii) i.e., A""'’ by 5 and equate, we have,
(5 - n) Mn04 = 5 A""^, i.e., (5-n) moles of MnO^ will oxidise A"'*'= 5 moles
REDOX REACTIONS 8/53

or
1-61 X 10"^ moles of MnO^ will oxidise A"'*’ = —
5-/J
X 1-61 X 10 ^ moles ...(///)

But the number of moles of A"'*' actually oxidised = 2-68 x 10"^ moles ...(/V)

5 , 1
Equating the values of Eq. (Hi) and (/v), we have, j — - n
X 1-61 X 10"^ = 2-68 X 10--^

^-0
or 5 X 1-61 =(5-n) X 2-68 or 2-68 n = 5 {2-68 - 1-61) = 5 x 107 = 5-35 or = 2-68

P, 4. 12-S3 mL of 0*051M Se02 reacts exactly with 25*5 mL of 0*1M CrS04 which is oxidised to Cr2(S04)3.
To what oxidation state is the selenium converted during the reaction ?

w
Sol. Let O.N. of Se in the new compound = x
I Reduced 1

F lo
4+
Se02 -i-
(4-x) Cr^'’ > Se^ +

Oxidised

Now 12-53 mL of 0-051 M SeO, = 12-53 x 0-051 = 0-64 millimoles of Se02

e
Fre
and 25-5 mL of 0-1 M CrS04 = 25-5 x 0-1 = 2-55 millimoles of CrS04
But according to balanced redox equation, (4 - jr) moles of CrS04 reduce 1 mole ot Se02

for
2-55
millimoles
2-55 millimoles of CrS04 will reduce Se02 =
(4-x)
r
But SeO-> actually reduced = 0-64 millimoles
You
2-55
s
Equating these two values, we have ’ 4-x- = 0-64 orjc = 0
ook

P, 5. 25-0 mL of an aqueous solution of H2O2 was treated with excess of K1 solution in acidic medium
eB

and the liberated iodine required 10*0 mL of 0*01 M thiosulphte solution. Find out the concentration
of H2O2 in grams per litre ?
Sol. Step 1. To write the balanced chemical equation of the redox reaction.
2 KI + H2SO4 + H2O2 ^ ^2
our
ad

2 Na2S203 + I2 “ ■ ^ Na->S40g + 2 Nal


2 KI + H2SO4 + 2 Na2S203 + H2O2 ■> K2SO4 + Na-)S40g -t- 2 Nal 2 H2O
From the above equation,
dY

1 mole of H2O2 = 2 moles of Na2S203


Re

Step 2. To find out the concentration ofH202-


Fin

Let the molarity of H2O2 solution = Mj


Applying molarity equation,
MiV, M2V2
(H2O2) = (Na2S203)
n
1 "2

M,x25
1 10x0-1 10x0-1
or or M
1 “
= 0-02 M
1 2 2x25

Mol. wt. of H2O2 = 2x1+2x16 = 34

Concentration of H2O2 = 0-02 x 34 = 0-68 g L“^


8/54 ‘Pn^xdee^'4i. New Course Chemistry fxnresm

ANNIG^H

1. Oxidation is a process which (/) involves addition of oxygen or any other electronegative element, or
removal of hydrogen or any other electropositive element, or
(//) involves loss of electrons. That is why oxidation is also culled de-electrunation.
2. Reduction is a process which involves addition of hydrogen or any other electropositive element, or

low
involves removal of oxygen or any other electronegative element, or
(ii) involves gain of electrons. That is why reduction is also called electronation.
3. An oxidising agent or an oxidant is a substance which (/) supplies oxygen or any other electronegative
element, or removes hydrogen or any other electropositive element, or
{ii) can readily accept electrons from other substances. That is why oxidising agents are electron acceptors.
Examples : KMn04. K2Cr20y, HNO3. O,, O3, H2O2. KCIO3. F2, CI2, Bro. I2. HgCl2, FeCl3, etc.

e
4. An oxidising agent after carrying out oxidation itself gels reduced.

re
5. A reducing agent or a reductant is a substance which

rF
(0 supplies hydrogen or any other electropositive element or removes oxygen or any other electronegative

rF
element, or (ii) can readily donate electrons. That is why reducing agents are electron donors.
Examples : H2, C, CO, Al, SO2, HNO2. H^S, SnCl,, etc.
6. A reducing agent after carrying out reduction itself gets oxidised.

fo
7. Redox reactions are reactions in which oxidationand reduction reactions occur simultaneously.
u
8. Oxidation and reductions always occur simultaneously, i.e., wheneverany substanceis oxidised, some
other substance is always reduced. For example. 2 HgCl2 + SnCl2-
ks > Hg2Cl2 + SnCl4
Here, HgCh is reduced to Hg2Cl2 while SnCl-^ is oxidised to SnCl4. Thus, HgCl2 acts as an oxidising
Yo
agent while SnCl2 acts as a reducing agent.
oo
9. Oxidation number (O.N.) is the charge which an atom has in its ion or appears to have when present in
the combined state.
B

10. Rules for assigning Oxidation Number.


re

(0 The O.N. of atoms of all elements in their elementary state is zero.


(ii) The O.N. of hydrogen is usually + 1 except in metal hydrides where it is - 1.
u

(Hi) The O.N. of oxygen is usually - 2 except in case of peroxides where it is - 1 and in OF^ where it is + 2.
ad

(iv) For each covalent bond between two different atoms, assign, an O.N. of + 1 to the less electronegative
Yo

atom and - 1 to the more electronegative atom,


(v) If a coordinate bond is formed between two dissimilar atoms but the donor atom is less electronegative than
the acceptor atom, assign an O.N. of + 2 to the donor atom and - 2 to the acceptor atom. However, if the donor
d

atom is more electronegative than the acceptor titora, neglect the contribution of the coordinate bond.
Re
in

(vi) In neutral compounds, the sum of the oxidation numbers of all the atoms is zero.
(vU) In monoatomic ions, the O.N. of the atom is equal to its charge.
F

(v/f7) In complex ions, the sum of oxidation numbers of all the atoms in the ion is equal to the charge on the ion.
(ix) The O.N. of metal in amalgams and metal carbonyls, i.e., NilCOlj, FefCO)5, Cr(CO)g, etc. is zero.
11. In terms of oxidation number, oxidation involves increase in the oxidation number while reduction involves
decrease in the oxidation number of the element in the given sub.stance. On the other hand, oxidising
agent is a substance, the oxidation number of its atom or atoms decreases while reducing agent is a
substance, the oxidation number of its atom or atoms increases and redox reactions involve simultaneous
increase and decrea.se of oxidation number of interacting species.
12. Equivalent weight of an oxidising/reducing agent is equal to its moleeular weight divided by the number
of electrons gained/losi as represented in the balanced chemical equation. For example,
(0 Mn04 +8H''' + 5 e~ > Mn-'*' + 4 H,0
REDOX REACTIONS 8/55

Eq. wt. of Mn04 = 119/5 = 23*8 and that of KMn04 = 158/5 = 31*6
[ Mol. mass of Mn04 = 55 + 4 x 16 = 119]

(ii) Cr,07~ + 14 + 6 e" ^ 2 + 7 H-iO

Eq. wt. of 0207“ = 216/6 = 36 and that of K2Cf207 = 294/6 = 49


[●.● Mol. mass of Cr207 =2x52 + 7x 16 = 216]
13. Types of redox reactions. There are four types of redox reactions as described below :
(i) Combination redox reactions are those in which either one or both the leacting species are in the
elemental form. For example,

w
C is) + O2 ig) ^ CO2 ig) ; CH4(^) + 2 02(g) ^ CO2 ig) + 2 H.O {/)
(ii) Decompositionredox reactions are those reactions in either one or both the decomposition products

F lo
are in the elemental form. For example.
A A

2 H2O (/) 4 2H2(g) + 02(fi) : 2 KCIO3 is) ■> 2KC1U) + 3 02(5)


{/») Displacementredox reactionsare those in which an atom or ion in a compound is replaced by an

ee
atom or ion of some other element. For example,
CUSO4 iacj) + Zn (5) > Cu (s) + ZnS04 ; Zn (s) + 2 HCl ig) > Z11CI2 iaq) + H2 (g)

Fr
(iv) Disproportionation reactions. A reaction in which the same species is simultaneously oxidised as
well as reduced is called a disproportionation reaction. For example,

for
cold

CI2 (g) + 2 0H-(^j^) ^ CIO- iaq) + Cr iaq) + H2O (/)


ur
P4 is) + 3 OH- iaq) + 3 H2O (/) ^ PH3 ig) + 3 H2PO2 iaq)
14. Fractional oxidation states. In reality, no element can have a fractional oxidation state. In fact, it is only
s
ook
the average oxidation state of an element when two or more of its atoms are present in different oxidation
Yo
states in a given compound. For example,
ii) The average O.S. of C in C3O2 is 4/3 while the actual oxidation slates of three carbons are + 2, 0, + 2
eB

-2 +2 0 +2 -2
as shown 0 = C= C= C = 0.

iii) The average O.S. of Br in Br30g is 16/3 while the actual oxidation states of three Br atoms are : + 6,
our

+ 4 and + 6 as shown.
ad

O
O O
%+6 +4 +6.^
0=Br — Br—Br O
Y

O O
O
Re
nd

iiii) The average O.S. of the four S atoms in S^Og is 2-5 while the actual O.S. of the four S atoms are +5,
0, 0 and +5 as shown :
Fi

O O
+5 0 0 +5
"O—S—S—S—S—O"

o o
15. Electrode. A metal dipped in the solution of its own ions is called an electrode or a redox couple.
16. Electrode potential is the tendency of an electrode to gain or lose electrons.
17. Standard electrode potential (E“). Since electrode potential depends upon the nature of the metal,
concentration of ions in solution and the temperature, therefore, electrode potentials are usually measured
under standard conditions, i.e., one molar concentration of metal ions and a temperature of 298 K. The
electrode potentials measured under these conditions are called standard electrode potentials.
8/56 New Course Chemistry (XI)BZ>19D

18. Standard or Normal hydrogen electrode (SHE or NHE). The absolute value of potential of a single
electrode cannot be measured since a half-cell reaction cannot take place independently. Therefore, electrode
potential has to be measured against a reference electrode called SHE or NHE.
A NHE consists of a platinum wire sealed is a glass tube and has a platinum foil attached to it. The foil is
coated with finely divided platinum and acts as the platinum electrode. It is dipped in 1 M solution of
ions (t.e., 1 M HCl) at 298 K and pure H2 gas maintained at a pressure of 1 atmosphere is bubbled over

ow
platinum electrode.
The NHE is a reversible electrode, H2 (g) s ^ {aq) + 1 e~
It acts as anode if the other electrode can accept the electrons, i.e., H^ (g) ^ 2 H+ {aq) + 2e~
It may act as a cathode if the other electrode donates electrons, i.e., 2 H"^ (aq) + 2 e' ^ H2 (g)
The electrode potential of a standard hydrogen electrode is taken to he zero.

e
If. Electromotive or Electrochemical or Activity series is a table in which the standard electrode potentials
of various electrodes are arranged in their decreasing order.

re
rFl
Applications (/) To predict reducing and oxidising power of elements, {a) Lower the electrode potential,
stronger is the reducing agent:

F
Li > Mg > Al > Zn > Fe > Co > Ni > Sn > Pb > Cu > I2 > Fe^* > Hg > Ag > Br2 > Cl-y > F2.
{b) Higher the electrode potential, .stronger is the oxidising agent: F2 > Cl-y > Br2 > I2
(/{) To predict whether a metal will react with dilute mineral acids to liberate hydrogen gas or not.

or
ou
All metals {i.e.. Mg, Al, Zn, Fe. etc.) with -ve electrode potentials are stronger reducing agents than
hydrogen and hence react with dilute mineral acids to liberate H2 gas.

ksf
Conversely, metals {i.e., Cu, Ag, Hg, Au, etc.) with positive electrode potentials are weaker reducing
agents than hydrogen and hence do not react with dilute mineral acids to liberate H^ gas.
20. Electrochemical or Galvanic or Voltaic cell is a device in which chemical energy of a redox reaction is
oo
converted into electrical energy.
21. Daniel cell is the most common example of an electrochemical cell. It consists of two electrodes, i.e., Zn
Y
plate dipped in a concentrated solution of ZnS04 (say 1-0 M) and Cu plate dipped in a concentrated
B

solution of CUSO4 (say 1 -0 M). The two solutions are connected by a salt bridge containing a concentrated
solution of an inert electrolyte such as KCl, KNO3, etc.
re

On connecting the two electrodes by a wire, the redox reaction occurs and the electrons flow from anode
to cathode, i.e.. Zn to Cu electrode in the external circuit while current flows in the reverse direction from
oYu

cathode to anode, i.e., Cu to Zn.


ad

22. The function of the salt bridge is (0 to complete the inner circuit by allowing the flow of ions from one
solution to the other without mixing and («) to maintain electrical neutrality.
23. Oxidation occurs at the anode while reduction occurs at the cathode.
d

24. Anode acts as a negative pole while cathode acts as the positive pole.
in
Re

25. Construction of an electrochemical cell. Compare the electrode potentials of the two electrodes. The
electrode having lower electrode potential is made the anode while the other having higher electrode
potential is made the cathode.
F

For example, the electrode potentials of Zn^'''/Zn electrode and Cu^‘*'/Cu electrodes are - 0-76 V and
+ 0-34 V respectively. Therefore, Zn^‘*'/Zn electrode is made the anode while Cu-'''/Cu electrode is made
the cathode.

Representation of an electrochemical cell. By convention, anode is written on the left and cathode is
written on the right. The complete cell may be represented as : Zn I Zn^'*’ (cj) I Cu^'*' (C2) I Cu
where C| and C2 are the concentrations of Zn^"*" and Cu^* ions respectively.
27. EMF of a cell is given by the expression, E‘ cell = E° cathode -E“anode

28. Predicting feasibility of a redox reaction. In general, a redox reaction is feasible only if the EMF of the
galvanic cell it forms is +ve, i.e., the species which has lower electrode potential is oxidised and the
species with higher electrode potential is reduced.
REDOX REACTIONS 8/57

QUESriONS

Based on NCERT Book

I. Multiple Choke Questions (<?) Oxidation state of carbon in HCN is + 4


{d) All statements are correct.
1. In the reaction, SO-> + 2 H^S — ^ 3 S + 2 HoO the
substance that is oxidised is 8. Oxidation states of P in H4P2O5, H4P20(;, H4P2O7
are respectively
(a) SO2 (b) H2O
(a) +3, +5, +4 {b) +5, +3, +4
(c)S (d) H2S
(c) +5, +4, +3 (d) +3, +4, +5
2. In which of the following reactions, the underlined
substance has been reduced 7 9. The oxidation states of iodine in HIO4, H3IO5 and
H^10<^ arc, respectively
(a) Br- + H2S > 2 HBr + S
(fl)+ l. + 3, + 7 (b) + 7. + 7, + 3
ib) 2 HgCl2 + SnCU > Hg2Cl2 + SnC^

F low
(c) + 7, + 7, + 7 (</) + 7, + 5, + 3
(c) CI2 + 2ia > 2 KCI + I2
10. Which of the following have been arranged in
(d) 2 Cu^"^ + > Cu9l2 + I-) decreasing order of oxidation number of sulphur ?
3. Bromine water reacts with SO2 to form (a) N32S406 > H2S2O7 > NU2S203 > Sj,
(a) HBr and S (b) H20andHBr (h) H2SO4 > SO2 > H2S > H2S2OH
(c) S and H->0 id) H2S04andHBr
(c) SO2 > SO5- > SOf- > USO4
4. Which of the following chemical reactions depicts

for Fre
the oxidising behaviour of H2SO4 ? (d) H2S04>H2S03>SCl2>H2S
11. In which of the following compounds, nitrogen
(a) NaCl + H2SO4 > NaHS04 + HCi
exhibits highest oxidation state?
(h) 2PCI5 + H2SO4 >
(«) N2H4 (h) NH3
2 POCI3 + 2 HCI + SO2CI2
(c) N3H (d) NH2OH
(c) 2HI + H2SO4 ^ I2 + SO2 + 2 H2O 12. Which is the strongest acid in the following?
(d) Ca(OH)2 + H2SO4 > CaS04 + 2 H2O
Your
(a) HCIO4 (b) H2SO3
eBo ks

5. In which of the following redox reactions, the (c) H-,SO 4 (d) HCIO3
underlined substance acts as a reducing agent ?
13. The oxidation numbers of the sulphur atoms in
(a) 2KMnO^ + 10 FeS04 + 8 H2SO4 4
peroxomonosulphuric acid (H2SOg) and pero-
ad

2 MnS04 + K2SO4 + 5 Fe2(S04)3 + 8 H2O xodisulphuric acid (H2S20g) are respectively


our

(fl) + 8 and + 7 (b) + 3 and + 3


(b) 2 K4[Fe(CN)6l + H2O2 + H2SO4 ●»

(c) + 6 and + 6 (d) + 4 and + 6


KjrFefCN)^] + K2SO4 + 2 H2O 14. The oxidation state of chromium in the final
(c) 2 KjfFefCNlg] + 2 KOH + H2O2
Re

product formed by the reaction between K1 and


acidified potassium dichromaie solution is
K4 [Fe(CN)e] + O2 + 2 H2O
(«) + 3 ib) + 2
(d) 2 KBr03 + E2 + 2 OH" ^ KBr04 + 21^
Find Y

(c) + 6 id)+ 4
+ H2O
6. A metal ion loses 3 electrons, its oxidation 15. Acidified K2Cr207 solution turns green when
number will be Na2S03 is added to it. This is due to the formation
of
ia) +3 ib) + 6
ic)0 (d)-3 ia) 0-2(504)3 ib) CtO^-
7. Which of the following statements is correct ? (c) Cr2(S03)3 id) CrS04
ia) Oxidation number of Fe in [Fe(H20)5N01S04 16. A mixture of potassium chlorate, oxalic acid and
is + 1
sulphuric acid is heated. During the reaction, which
(/?) Oxidation number of sodium in sodium element undergoes maximum change in the
amalgam is - 1 oxidation number?
8/58 New Course Chemistry (XI)BE

(fl)S (b)H (c)CH^(g) + 2 02(g) ^C02(«) + 2H20(/)


(c) Cl (d)C (d) All are combination redox reactions.
17. The number of electrons that are involved in the 23. Which of the following is not a decomposition
reduction of permanganate to manganese (II) salt, redox reaction ?
manganate and manganese dioxide respectively are A

ia) 5,1, 3 (b) 5, 3, 1 (a) CaCOj is) - CaO (i) + CO2 (g)
A
(c) 2,7,1 (d) 5,2,3 (b) 2 KCIO3 is) ■> 2 KCl (s) + 3 O2 (g)
A
18. What is the oxidation state of Co in
(c) 2 NaH is) - ^ 2 Na (s) + H2 (g)
[Co(H20)5C1]2+? A

(fl)+2 (fc)+3 (d)2H2O(0 ^ 2H2(g) + 02(g)

w
(c)+l id)+4 24. Which of the following is a metal displacement
reaction ?
19. Oxidation state of each Cl in CaOCl2 is/are
ia) 2 Na is) + 2 H2O (0 ^ 2 NaOH iaq)

F lo
ia)0 (/>) + !
(^0 + l,-l
+ H2(g)
ic)-l
ib) CUSO4 iaq) + Zn is) Cu (j)
20. The oxidation states of S atoms in 840^" from + ZnS04 iaq)
left to right respectively are (c) Fe is) + S is)

e
■» FeS is)

Fre
O O id) Fe is) + 2 HCl iaq) FeCl2(o9) + H2(g)
no—S—S—S—S—O" 25. Which of the following is not a non-metal

for
displacement reaction ?
O O
(a)2F2(g) + 2H2O(0 ^ 4 HF iaq) + O2 (g)
ia) + 6,0,0, + 6 ib) + 3, + 1, + 1, + 3 (/;)3Fe(5) + 4H20(g) -»Fe304(5) + 4H2(g)
r
(c) + 5,0, 0, + 5 (d) + 4, + 1, + 1, + 4 ic)Cl2ig) + 2l-iaq)- ^ 2 Cr iaq) +12 is)
You
21. The correct structure of tribromooctaoxide is
id)l2is) + 2Briaq)- ^ 21~ iaq) + Br2 (/)
oks
O"
O ,0 26. Which of the following is a disproportionation
% /●
eBo

reaction ?
(a) 0=Br — Br—Br—O”
O
ia) 2 KMn04 is) K2Mn04 (5) + Mn02 is)
O
O" + 02(g)
O ib) 2 NaBr iaq) + CI2 (g) ^ 2 NaCl iaq)
our
ad

O ,0
% + Bt2 (0
ib) 0==Br Br—Br=0
ic) 2 CuBr iaq)
% ■> CuBr2 iaq) + Cu is)
O O
o id) Mn02 is) + 4 HCl iaq) —> MnCl2 iaq)
+ 2 H2O (/) + CI2 (g)
dY
Re

O-
o o
% 27. In acidic medium, H2O2 changes Cr20| to C1O5
(c) O Br — Br—Br o-
which has (-0-0-) bonds. Oxidation state of Cr
Fin

o O in Cr05 is
O"
ia) +5 ib) +3
O ic) +6 id) -10
O O"
% 28. When CI2 gas reacts with hot and concentrated
id) ~0—Br — Br Br O
\ sodium hydroxide solution, the oxidation number
-o
o
o- of chlorine changes from
22. Which of the following is/are combination redox ia) zero to + 1 and zero to - 5
reaction/s ? ■ ib) zero to - 1 and zero to + 5
(a) C (5)+ 02(5) ^C02 (g) ic) zero to - 1 and zero to + 3
ib) 3 Mg is)+ N2ig) > Mg3N2 (5) id) zero to + 1 and zero to - 3
REDOX REACTIONS 8/59

29. The reaction, P4 + 3 NaOH + 3 H2O > 35. In the given reaction,
3 NaH2p02+ PH3 K2Cr207 + XH2SO4 + YSO2 >
is an example of K2SO4 + Cr2(S04>3 + ZH2O
(a) disproportionation reaction X, Y,Zare
{b) neutralization reaction (a) 1. 3, 1 (b) 4, 1, 4
(c) double decomposition reaction (c) 3, 2, 3 (d) 2, 1, 2.
(d) pyrolytic reaction

low
36. The values of x and y in the following redox
30. Choose the disproportionation reaction among the reaction,
following redox reactions,
XCI2 + 6OH- ■> CIO J + y cr + 3 H2O are
(fl) 3 Mg (s) + N2 (8) > Mg3N2 (s)
ia) x = 2,y = 4 ib) x = 5,y = 3
(b) P4 (j) + 3 NaOH (aq) + 3 H2O (/)
(c) x = 3,y = 5 id) x = 4,y = 2
PH3 ig) + 3 NaH2P02 (aq)
37. The equivalent mass of potassium permanganate

e
(c) CI2 (g) + 2 KI (aq) > 2 KCl (aq) +12 (s) in alkaline medium is

re
id) Cr203 is)+ 2 M is) > AI2O3 is) + 2 Cr is) ia) Molar mass/5 ib) Molar mass/3

rF
ie) 2 NaH is) > 2 Na is) + H2 ig) (c) Molar mass/2 id) Molar mass itself

F
31. Which of the following are disproportionation 38. a K2Cr207 + /; KCl + c H2SO4 X C1O2CI2
reactions ?
+ y KHSO4 + ZH2O

r
(0 2Cu+ >Cu2+ + Cu°
The above equation balances when

fo
Hi) 3Mn02-+4H+ >
ou
ia)a = 2, b = 4, c = 6 and x = 2, y = 6, z = 3
2Mn04 Mn02 + 2 H2O ib)a = 4,
ks b = 2, c = 6 and jc = 6, y = 2, z = 3
ic) a = 6, b = 4, c = 2 and x = 6, y = 3, z = 2
iiii) 2 KMn04 K2Mn04 + Mn02 + O2
id) a=l, b = 4,c = 6 and j: = 2, y = 6, z = 3
oo
iiv) 2MnO“ + 3 Mn^+ + 2 H2O > 39. In the redox reaction,
Y
5Mn02+4H+ A:KMn04 + NH3 4 y KNO3 + Mn02 + KOH
B

Select the correct option from the following : + H2O


re

id) (i) and iiv) only ib) (i) and (//) only ia)x = 4,y = 6 ib)x = 3,y = S
(c) (i), (h) and iiii) id) (ii), (i/7) and (iv) ic)x = ^,y = 6 id)x = S,y = 3
u

32. In which of the following compounds, carbon 40. In the balanced chemical reaction,
ad

exhibits a valency of 4 but oxidation state of - 2 ?


Yo

IO3 +al-+blV- ^ CH2O + dl2


ia) CH3CI ib) CHCI3 a, b, c and d respectively correspond to
(c) CH2CI2 id) HCHO
(«) 5, 6, 3, 3 ib) 5, 3, 6, 3
33. In the redox reaction,
d

ic) 3, 5, 3, 6 id) 5, 6, 5, 5
Re

Mn04 + 8 H+ + 5 Br-
in

4
41. The following redox equation,
Mn2+ + 4 H2O + 5/2 Br2
x: UQ2+ + Cr20^- + y H+
F

Which one is the reducing agent ?


ia) H+ ib) MnO- aU0^+ + zCr3+ + /,H20
the value of the coefficients, x, y and z are respectively
ic) Br (d) Ivln2+
ia) 3, 8, 2 ib) 3, 8, 7
34. When KMn04 oxidising agent and (c)3, 2,4 id) 3, 1, 8
ultimately forms MnO^^, Mn02, Mu203 and 42. Consider the following redox equation,
Mn+2, then the number of electrons transferred in Zn + x HNO3 > y Zn(N03)2 + ^ NH4NO3
each case respectively, is
+ H2O
ia) 4, 3, 1, 5 ib) 1,5, 3, 7
The coefficients x, y and z in the balanced equation
ic) 1,3, 4, 5 id) 3, 5,7,1 respectively, are
8/60 New Course Chemistry (XI)

(a) 10 : 1 : 4 (/>) 10 : 4 : 1 51. When acidified K2Cr207 solution is added to Sn^^


(c)4: 10: 1 (c/)4: 1 : 10 salts, then Sn-'*' changes to
43. The coefficients p, q, r and s in the following (a) Sn {b) Sn^-"
P4 (s) + p OH" {aq) + q H2O (/) ^rPH3 (g) (c) Sn-*-" {d) Sn*
52. In the standardization of Na2S203 using K2CF207
+ s H^PO.) (0(?)
by iodometry, the equivalent weight of K2Cf207 is

ow
balanced equation respectively, are (a) (molecular weight)/2
(n) 3, 3, 1,3 (b) 3, 1,3,3 (b) (molecular weight)/6
(c) 3, 1, 1,3 (d) 3, 3, 1. 1 (c) (molecular weight)/3
44. When copper a treated with a certain concen (d) same as molecular weight.
tration of nitric acid, nitric oxide and nitrogen
53. KMn04 acts as an oxidising agent in alkaline

e
dioxide are liberated in equal amounts according
medium. When alkaline KMn04 is treated with KI,

re
to the equation,

rFl
iodide ion is oxidised to
xCu + y HNO3 ^ Cu(N03)2 + no + NO2 ib) 10-
(a) I2

F
+ H2O
The coefficients x and y are (c) IO3 (d) 104
(a) 2 and 3 (b) 1 and 3 54. Fordecolourizationof 1 mole ofKMn04, the moles

r
(c)2 and 6 (d) 3 and 8 of required is
ou
fo
(a) 1/2 {h) 3/2
45. Mn04 oxidises (i) C20j“ in acidic medium at
333 K and (ii) HCl. For balanced chemical
equations, the ratios [MnO^ :C204~] in (/) and
(c) 5/2

ks (d) 7/2
55. The number of moles of KMn04 reduced by one
mole of KI in alkaline medium is
oo
[Mn04 : HCl] in (ii) respectively, are (fl) one (b) two
Y
(a)2:5 and 1 :5 (/?) 1 : 5 and 2 : 5 (c) five id) one fifth
eB

(c)5: 2 and 1 : 8 (i/) 2 : 5 and I : 8 56. The number of moles of KMn04 needed to react
46. The ratio of number of moles of KMn04 required with one mole of SO3" in acidic solution is
to oxidise one mole each of FeC204, Fc2(C204)3
r

and FeS04 respectively, is (a) 4/5 (b) 2/5


ou

(a) 3 : 6 : 1 (6) 3 : 1 : 6 (c) 1 (d) 3/5


Y
ad

(c) 6 : 3 : 1 (J) 6 : 1 : 3 II. Assertion-Reason Questions


47. Which of the following is a set of reducing agents ?
For questions below, two statements are given
d

(fl) HNO3, Fe“+, F2 (b) F", Cl", Mn04 one labelled Assertion (A) and the other labelled
Reason (R). Select the correct answer to these
Re
in

(c) r, Na, Fe^-^ id) CrOj-, Na questions from the codes (a), (fr), (c) and (d)
48. Which of the following species can function both given below :
F

as oxidising as well as reducing agent ? («) Both A and R are correct and R is the correct
(a) Gi¬ (h) CIO4 explanation of A.
(/?) Both A and R are correct but R is not the correct
(c) CIO" lt/) Mn04 explanation of A.
49. The pair of compounds that can exist together is (c) A is correct but R is wrong.
(a) FeCl3, SnCl2 ib) HgCl2, SnCl2 (d) A is wrong but R is correct.
(c) FeCl2, SnCl2 id) FeCl3, KI 57. Assertion. In the reaction between KMn04 and
50. Which of these will not be oxidised by ozone ? KI, Mn04 ions act as an oxidant.
(a) KI ib) FeS04 Reason. Oxidation state of Mn changes from + 2
(c) KMn04 id) K2Mn04 to + 7.
REDOX REACTIONS 8/61

58. Assertion. In the reaction between H-,8 and FeCl3 Reason. It is redox reaction in which I2 acts as a
to give FeCl2, HCI and S, M2S acts as a reductant. reducing agent and NaCl03 as an oxidising agent.
Reason. The oxidation state of increases from
65. Assertion. Decomposition of H2O2 to give H2O
- 2 in H2S to zero in elemental sulphur. and O2 is a disproportionation reaction.
59. Assertion. HgCl2 and SnCl2 cannot stay together. Reason. The O.S. of O decreases from - I in
Reason. HgCl2 is an oxidising agent but SnCU i.s to - 2 in H.,0 and increases from - I in H2O2 to
a reducing agent. zero in O2.
60. Assertion. Both the sulphur atoms in NaoS203 66. Assertion. The reaction, 2 F2 (g) + 2 OH“ (aq)
have the same oxidation number.
> 2 F~ (aq) + OF2 (g) + H,0 (/) is a

w
Reason. When Na2S203 reacts with dilute HCI, disproportionation reaction.
SO2 is evolved with precipitation of sulphur. Rea.son. Here, F2 is reduced to F^ ions and OF2.
61. Assertion. 2 CuCl > CUCI2+ Cu is a 67. Assertion. The thermal decomposition of CaC03

F lo
disproportionation reaction.
to CaO and CO2 is a redox reaction.
Reason. All transition metals show dispropor Reason. It is simply a decomposition reaction.
tionation reactions.
68. Assertion. Fe304 contains iron atoms in two

ee
62. Assertion. Silver can be precipitated by adding zinc different oxidation numbers.
powder to AgN03 solution.

Fr
Reason. Fe^"*" ions decolourize KM11O4 solution.
Reason. ZnS04 solution can be stirred with a silver
spoon. 69. Assertion. Pb304 contains lead atoms in two
O different oxidation slates.

for
O 0
Reason. Pb02 is good oxidising agent.
63. Assertion. The oxidation slate of Cr in | :cr;
ur
is + 10. O 0 70. Assertion. Br2 can oxidise 1“ ions to I2 but I2
cannot oxidise Br“ ions to 613.
Reason. The oxidation state of Cr cannot be
s
Reason. I2 is a weaker oxidising agent than Br2.
more than 6 since its electronic configuration is
ook
Yo
[Ar] 4sK 71. Assertion. All the carbon atoms in C3O2 have the
same oxidation state.
64. Assertion. The reaction,
eB

Reason. The middle carbon atom has an oxidation


2 NaC103 (aq) + I2 (aq) > NaI03 + CI2 (aq) state of zero while the other two have an oxidation
is a non-metal replacement reaction. state of + 2.
our
ad

ANSWERS

I. Multiple Choice Questions


Y

l.(d) 2.(0) 3.(J) 4. (c) 5. (c) 6. (b) 7. (fl) 8. (d) 9. (c) 10. (d)
Re
nd

11. (c) 12.(0) 13. (c) 14. (o) 15.(a) 16. (c) 17. (a) 18. (b) 19. (d) 20. (c)
21. (b) 22. (i[) 23.(0) 24. (b) 25. (d) 26. (c) 27. (c) 28.(h) 29.(a) 30. (b)
Fi

31. (b) 32.(0) 33. (c) 34. (c) 35.(a) 36.(c) 37. (b) 38. (d) 39. (d) 40.(a)
41. (o) 42. (h) 43.(0) 44. (c) 45. (d) 46. (o) 47.(c) 48. (c) 49. (d) 50.(c)
51. (c) 52. (b) 53. (c) 54. (c) 55. (b) 56. (b)

II. Assertion-Reason Type Questions


57. (c) 58.(a) 59. (a) 60. (d) 61. (c) 62. (b) 63. id) 64.(b) 65. (a) 66. (d)
67. (d) 68. (b) 69. (b) 70. (a) 71. (d)
8/62 New Course Chemistry (XI)S!EIHD

For Difficult Questions

o o
I. Multiple Choice Questions
1. The electropositive element hydrogen has been HO— S —0—0— S —OH

w
removed for H2S, i.e., H2S is oxidised. O O
2. Br2 is reduced to HBr.
Peroxodisulphuric acid
3. Bf2 + SO2 + 2 H2O > 2 HBr + H2SO4 1 + 1 (-2)+x + 2(-2) + 2(- l)+x + 2(-2)
5. H2O2 acts both as an oxidising agent as well as a

lo
+ 1 (- 2) + 1 = 0
reducing agent. In option (c), it acts as a reducing or X + 6
agent since it reduces K3[Fe(CN)(^] to K4[Fe(CN)6].

e
14. K2Cr-)07/H2SO4 oxidises KI to I2 and itself gets

re
6. M-’+ +
reduced to Cr2(S04)3

rF
7. Since the charge on SO^” is - 2, therefore, the

F
+6 +3

total charge on the complex ion, [Fe(H20)5N0] is K^Cr^O, ^ Cr2(S04>3


+ 2. Since NO is a paramagnetic molecule, its
15. K2Cr207/H2S04 oxidises Na2S03 to Na2S04 and
unpaired electron is transferred to iron. As a result,

r
itself gets reduced to Cr2(S04)3.
NO carries + 1 charge. Further since H2O is a

fo
u
neutral molecule, sum of its O.N. is zero. Putting
the values of all the oxidation numbers, the O.N.
+5

16. KC103 + 3(C00H)2


ks
+3 H2SO4

-I +4
of Fe in the complex ion = jc + 0x5 + l= + 2
Yo
j: = + 1.
KCl +6CO2 +3H2O
oo
Thus, the O.N. of Fe is + I and the correct structure Maximum change in O.N. of Cl = + 5 - (- 1) = + 6
+1 + +7 2+
eB

of [Fe(H20)5N0]S04 is [Fe(H20)5 NOJSOj- 17. MnO" +5e~ Mn

8. O.N. of P in H4P2O5 = 4 (+1) + 2 jc + 5 (-2) = 0 +7 +6


or a: = + 3
Mn04 +e Mn04
ur

O.N. of P in H4P2O6 = 4 (+1) + 2 X + 6 (-2) = 0 Manganate ion.


or X = + 4
ad

+4
Yo

+7
O.N. of P in H4P2O7 = 4 (+1) + 2 X + 7 (-2) = 0 Mn04 +3e MnO 2
or X = + 5

+7 +7 +7
Thus, option (n) is correct.
d

9. HIO4, H3l05,HgI06 18. Since H2O is a neuti-al molecule, sum of its O.N.
Re

is zero while O.N. of Cl is - 1. If now x is the O.N.


in

-rf) +4 +2 -2
of Co, then,
10. H2SO4 > H2SO3 > SCI2 > H2S x + 5xO + (-l) = + 2
F

-3 -1/3 -I
x = +2+l= + 3
11. N2H4, NH3, N3H, NH2OH +1 -1

12. The acid which has the highest O.N. of the element, 19. CaOCl2 is actually Ca^'*’ (OCr)Cl“ . Therefore,
i.e., +7 of Cl in HCIO4 is the strongest acid. the O.N. of the two Cl atoms are + 1 and - 1.
O 20. Refer to Art. 8.4.7, page 8/21.
13. H —0 —O —S —OH 21. Refer to Art. 8.4.7, page 8/21.
25. A weaker oxidising agent (I2) cannot displace a
O
stronger oxidising agent (Br2).
Peroxomonosulphuric acid +\ +2 0

1 + 2 (- 1) + X + 2 (- 2) + (- 2) + 1 = 0 or X = + 6 26. 2CuBr(a?) ■> CuBr2(fl^) + Cu(j)


REDOX REACTIONS 8/63

27. Refer to ‘Retain in Memory’, page 8/13. 37. In the alkaline medium, conversion of MnO^ to
0 -1 +5 . +7 +4
28. 3Cl2 + 6NaOH >5NaCl + NaC103 + 3H20 MnO; +2H20 + 3e- ^ Mn02 + 4 0H-
0 +1 -3 Mn02 is a 3e change and hence
29. P4 + 3 NaOH + 3 H2O > 3 NaH2 PO2 + PH3 Eq. wt. of KMn04 “ Molar mass/3
30. In reaction (/>), P4 is both reduced to PH3 and 38. K2Cr207 + 2 H2SO4
oxidised to NaH2P02. Thus, it is a disproportio 2 KHSO4 + 2 Cr03 + H2O
nation reaction. But in all other redox reactions, 2 KCl + H2SO4 ^2KHS04 + 2HC1]x2
different species are oxidised/reduced, i.e., in

w
Cr03 + 2HC1 ■> er02Cl2 + H2O] X 2
reaction (a). Mg is oxidised but N2 is reduced ; in
reaction, (c) CI2 is reduced but KI is oxidised to I2 K2Cr207 + 4 KCl + 6 H2SO4 ■>

in reaction (d). Cr203 is reduced but A1 is oxidised. 2 Cr02Cl2 + 6 KHSO4 + 3 H2O

F lo
31. (/) 2Cu
+1 2+
> Cu+Cu
0
39. Mn04 + 2 H2O + 3 e" ■>Mn02 + 4 0H"[x8
+6 +7 +4 NH3 + 9 OH" ^ NOJ + 6 H2O + 8 [X 3

ee
(ii) 3Mn02- +4H+ 2Mn04 Mn02
8Mn04 + 3 NH3 > 8 Mn02 + 3NOj

Fr
+ 2H2O
+ 5 OH" + 2 H2O
Options (/) and (ii) represent disproportionations
but others are not. or 8 KMn04 + 3 NH3 > 8 Mn02 + 3 KNO3

for
+ 5 KOH + 2 H2O
32. The valency of C in all the four compounds is 4
Thus, coefficient of KMn04 = ^ of KNO3
our
but O.N.ofCinCH3Cl is: = 3.
j: + 3x1 + 1x(-1) = 0 or x = -2.
40. IOJ+5I-+6H+ ^ 3 H2O + 3 I2
s
33. Br" is the reducing agent since it reduces MnO^
ook
41.
to Mn^"*" and itself gets oxidised to Br2. f
O.N. increases by 2 per U atom
+7 1
Y
+6 2
eB

+4 2+ +6 3+
34. Mn04 + e ■> MnO|- ; UO + Cr207 Uof +Cr
+7 +4
O.N. decreases by 3 per Cr atom 1
Mn04 +3e ■> Mn02
Balanced oxidation half reaction is :
our

+7 +3
ad

2Mn04 +4e ^ Mn203 ; UO2+ + H2O > U02+ + 2H+ + 2e- ...(0
+7 +2 Balanced reduction half reaction is :
Mn04 +5e“ » Mn-^2
Cr20|- +14H+ + 6e" > 2 Cr^^ + 7 H2O ...(«)
Y

Since number of electrons transferred in each case


Multiply Eq. (i) by 3 and add to Eq. (ii), we have,
Re

are 1, 3,4 and 5 respectively, therefore, option (c)


nd

is correct. 3U02+ + Cr202- + 8H+


35. Balanced equation is 3U02+ + 2Cr3+ + 4H20 ...(iiO
Fi

K2Cr207 + H2SO4 + 3 SO2 ■> Thus, Eq. (i70 represents balanced redox reaction
K2SO4 + 012(804)3 + H2O equation. Therefore, coefficients are: x: = 3 ; y = 8
and z = 2, i.e., option (a) is conect.
X = 1, Y = 3 and Z = 1 and hence option (a) is 0 +2
correct.
42. (0 Zn Zn(N03>2 ^ 2 ...(oxidation)
36. The balanced equation, +5 -3

3 CI2 + 6 OH"’ > C102 + 5 cr + 3 H2O (ii) HNO3 > NH4NO3 is ...(reduction)

X = 3, y = 5. To balance increase/decrease in O.N., multiply Eq.


(i) by 4 and add to Eq, (ii), we have,
Thus, option (c) is correct.
4 Zn + HNO3 > 4 Zn(N03>2 + NH4NO3
8/64 “Pnadee^'^ New Course Chemistry (XI)E

To balance N atoms on both side, multiply HNO3 46. From Ans. to Q. 30, page 8/128. Oxidation of
by 10, we have, 3
1 mole of FeC204 need KMn04 = -
4 Zn + 10 HNO3 ^ 4 Zn(N03)2
+ NH4NO3 ...(Hi) Oxidation of 1 mole of Fe2(C204)3 need KMn04
6
To balance O atoms, add 3 H2O to R.H.S of Eq.
(Hi), we have, 5

^ 4 Zn(N03>2 + NH4NO3 1
4 Zn + 10 HNO3
Oxidation of 1 mole of FeS04 need KMn04 = -
+ 3H2O

w
Thus, coefficients x, / and z are, 10 : 4 : I, i.e., 3 6 1
Ratio is = -: or 3:6:1
option (b) is correct. 5 5 5

43. The balanced Eq. is Thus, option (a) is correct.

F lo
47. I~ (- 1), Na (0) and Fe2+ (+ 2) all the three have
P4(5) + 30H"(fl^)+3H20(/)
■>

P 9
already their minimum O.N. and hence their O.N.
can only increase by losing electrons and hence

ee
PH3(g) + 3H2PO"(a^) act as reducing agents.

Fr
48. Since Cl in CIO" has an O.N. of+1 which is higher
Thus, the ratio ofp : q : r: S is 3:3:1: 3. than its lowest O.N. of-1 and lower than its highest
O.N. of +5, therefore, CIO" can act as an oxidising
44. With dil. HNO3, NO is produced and with cone.

for
as well as a reducing agent.
HNO3, NO2 is produced. The two balanced
ur
equations are : 49. One oxidising agent (FeCl3 or HgCl2) and one
reducing agent (SnCl2 or KI) cannot stay together
3 Cu + 8 HNO3 ^ 3 Cu(N03)2 + 2 no but two reducing agents (FeCl2, SnCl2) can stay
+ 4 H2O ...(/)
s
together. Thus, option (c) is correct.
ook
Yo
Cu + 4 HNO3 ^ Cu(N03>2 + 2 NO2 50. KMn04 cannot be oxidized further because Mn is
+ 2 H2O ...(h) already in its maximum oxidation state of + 7.
eB

3+
Since NO and NO2 are produced in equal amounts, 52. Cr20^ +14H‘‘' + 6e ^ ICr+lUp
therefore, adding Eq. (/) and (//), we have,
Since accepts 6 electrons for its reduction
4 Cu + 12 HNO3 > 4 Cu(N03)2 + 2 NO2 to Cr^'’" ions.
r

+ 2 NO + 6 H2O Eq. wt. = Mol. wt./6.


ou
ad

or 2Cu + 6HNO3 ^ 2 Cu(N03)2 + NO2 53. 2 KMn04 + H2O + KI ■>

+ NO + 3 H2O 2Mn02 + 2K0H+ KIO3


Y

Pot.iodate
Thus, coefficients x and y of Cu and HNO3 or I" + 6 OH" ^103+3 H2O + 6 e"
respectively are 2 and 6, i.e., option (c) is correct.
Re
nd

54. Red. half reaction :


45. The balanced equations are :
Mn04 + 8H+ + 5e~ ^ Mn^+ + 4H2O
Fi

2Mn04+5C202"+16H+ ^ 2 Mn2+
Oxid. half reaction :

+ 10 CO2 + 8 H2O ...(0 H2O2 ■>02 + 2H+ + 2e~


Since 1 mole of H2O2 gives 2e~ and reduction of
Mn04 : C2OJ" = 2:5
1 mole of Mn04 requires Se~, therefore, reduction
2KMn04+ 16HC1 ^ 2 KCl + 2 MnCl2 of 1 mole of KMn04 will require 5/2 moles of H2O2.
+ 5CI2 + 8H2O ...(h) 55. 2 KMn04 + H2O > 2 KOH + 2 Mn02 + 3 [O]
Kl + 3 [O] >KI03
.-. Mn04 : HCl = 2 : 16 or 1 : 8.
2 KMn04 + H2O + KI ■> 2 Mn02 + 2 KOH
Thus, option (d) is correct. + KIO3
REDOX REACTIONS 8/65

2K.Mn04 = KI 62. Correct explanation. Zn is more reactive than Ag


2 moles one mole
and hence precipitates Ag from AgN03 solution.
Thus, 2 moles of KMn04 are reduced by 1 mole of 0
KI, i.e., option (b) is correct. O 0
56. Reduction half reaction : 63. Correct A. The O.S. of Cr in |
O O
+ 6 is
MnO-+8H++5e- — ^ Mn-+ + 4 HoO
.r+ 1 (-2) + 4(- 1) = 0
Oxidation half reaction : or ,v = + 6

64. Correct explanation. It is a non-metal


SO2-+H2O — SOj-+2H+ + 2e-

w
displacement redox reaction since in this reaction
Since 1 mole of SO|" gives 2 e~ but reduction of I2 acts as a reducing agent and thus decreases the
1 mole of KMn04 requires 5 e~, therefore, no. of O.N. of Cl from + 5 in NaC!03 to zero in CI2.

F lo
moles of KMn04 required to react with 1 mole of 66. Correct A. The given reaction is not a
SQ2- = 2/5. disproportionation reaction since F2 can only be
reduced but not oxidised.

II. Assertion-Reason lype Questions 67. Correct A. It is simply a decomposition reaction

ee
57. Correct R. The O.S. of Mn decreases from 4- 7 in since O.N. of neither Ca nor that of C and O

Fr
change,
Mn04 to + 2 in
68. Correct explanation. The composition of Fe304
60. Correct A. The oxidation number of the two

for
is Fe0.Fe203 having iron in + 2 and + 3 oxidation
sulphur atoms in Na2S203 are - 2 and + 6 that is states respectively. Fe^'*' ion pre.sent in FeO being
our
why Na2S203 on treatment with dil. HCl gives SO2 a reducing agent decolourise KMn04 solution.
from the sulphur in + 6 oxidation number while
69. Correct explanation. The composition of Pb304
the other with - 2 oxidation number is precipitated
s
is : 2 PbO. PbO^ having Pb in -1- 2 and + 4 oxidation
ook
as sulphur.
states respectively. Whereas PbO^ is a strong
61. Correct R. Only those transition metals which oxidising agent but PhO is basic in nature.
Y
show three different oxidation states undergo
eB

71. Correct A. All the carbon atoms in C3O2 do not


disproportionationreactions. have the same oxidation state.
our
ad

i&i
ir PROBLEMS

(li) 2Fe'^'^ {aq) +21 (aq) —> I2 {^)+2 Fe”'*^ (aq)


Y

I. Oxidation-Reduction-Electronic
and Oxidation Number Concept (Hi) 2Na (s) + CI2 (g) — ^ 2 NaCl (s)
Re
nd

1. What is the name of the reaction, (iv) Mg(s) + Cl2(g) — 4MgCl2 (s)
(v) Zn (.v) + 2H-*- (aq) — (aq) + H2(g)
2 CH3CH2CH2SH
Fi

■>

In each of the reactionsgiven above, mention


CH3CH2CH2—S—S—CH2CH2CH3
(a) which reactant is oxidized ? To what ?
(Whether condensation, oxidation, reduction
or polymerization) (b) which reactant is the oxidiser ?
Ans. (c) which reactant is reduced ? To what ?
This is an example of oxidation reaction since
two H-atoms have been removed. (d) which reactant is the reducer ?
2. Write the following redox reactions using half Ans. (0 Zn(.y) ■> Zn^'*' + 2e~ (oxidation) ;
equations. Pb2+(aq) + 2e- ●> Pb (5) (reduction)
(/) Zn (s) + PbCl2 (aq) Zn is oxidised to Zn”'*', Pb""^ is reduced to Pb ;
Pb (s) + ZnCl2 (aq) Pb^'*' is the oxidiser and Zn is tlie reducer.
8/66 “P>uuiteefi,'4i New Course Chemistry (XI)ESC

(ii) 2Fe^+ + 2g- ■> 2Fe^’*' (reduction); 5. What is the maximum and mininuim oxidation
21- -> I2 + 2e~ (oxidation) states for Na, Mg, Al, Sn and Mn ?
Fe^'*' is reduced to Fe^'*', 1“ is oxidised to I2 ; Ans. The minimum oxidation state for all metals is 0.

r is the reducer and Fe^"*" is the oxidiser. The maximum oxidation state for s- and p-block
elements is equal to the number of electrons
(iii) 2Na 2Na‘^ + 2e~ (oxidation);
present in the valence shell. Thus, maximum
CI2 + 2e- -> 2 Cr (reduction) oxidation state of
Na is oxidised to Na'*’ and CI2 is reduced to Cl"; Na = + 1, that of Mg is + 2, that of A1 is + 3 and

low
Na is the reducer and CI2 is the oxidiser. that of Sn is + 4.
(iv) Mg > Mg^"*" + 2e (oxidation); However, for f/-block elements, the maximum
CI2 + 2e- ^ 2 Cl (reduction) oxidation state is equal to the sum of (n - 1)
and ns electrons. Thus, maximum oxidation state
Mg is oxidised to Mg^"*' while CI2 is reduced to
Cl- ; ofMn (3d^4s^) = 5 + 2 = + 7.
6. What are the maximum and minimum
Mg is the reducer and CI2 is the oxidiser.

ee
Zn » Zn^"*" + 2e- (oxidation); oxidation numbers of N, S and O ?
(V)
Ans. (0 The Iiighest oxidation number (O.N.) of N is
2H+ + 2e- ¥ H2 (reduction)

rF
Fr
+5 since it has five electrons in the valence shell
Zn is oxidised to Zn^"*" while H'*' is reduced to H2 ;
(2 2 p^) and its minimum O.N. is -3 since it
Zn is the reducer and H"*" is the oxidiser.] can accept three more electrons to acquire the
3. Photosynthesis involves the following overall nearest inert gas (Ne) configuration.

for
reaction.
(ii) Similarly, the highest O.N. of S is +6 since it
Sunlight has six electrons in the valence shell (3 5^ 3 p^)
ou
6CO2 + 6H2O ■>
and its minimum O.N. is -2 since it needs two
Chlorophyll
s
more electrons to acquire the nearest inert gas
ok
C«Hu04 + 6O2 (Ar) configuration.
Glucose
(Hi) Likewise the maximum O.N. of Cl is +7 since
Bo

Identify the species oxidised and the species it has seven electrons in the valence shell (3 ^
Y
reduced. 3 p^) and its minimum O.N. is -1 since it needs
Ans. Photosynthesis involves a series of complex only one more electron to acquire the nearest (Ar)
re

reactions but the overall reaction involves gas configuration.


reduction of CO2 to C6H12O6 and oxidation of 7. Nitric acid acts only as an oxidising agent
H2O to O2 as explained below : while nitrous acid acts both as an oxidising as
ou
ad

Reduced well as a reducing agent Why ?


Ans. (i) HNO3 : Oxidation number of N is HNO3
Y

+4 -2 0 0 0

6CO2 + 6H2O ^6^1206 6 O2 = +5

Oxidised
Maximum oxidation number of N = + 5,
nd
Re

Minimum oxidation number of N = -3


4. Chlorine dioxide (QO2) is used to kill bacteria
in meat, soft drinks and dairy products. Being Since the oxidation number of N in HNO-^ is
Fi

an unstable compound, it can be synthesized maximum (+5), therefore, it can only decrease
by the following reaction : by accepting electrons. Hence, HNO^ acts only
as an oxidising agent,
CI2 + NaC102 ^ 2CIO2 + 2NaCl
Identify the substance oxidised and reduced. (ii) HNO2 : Oxidation number of N in HNO2
= +3
Ans. The given redox reaction is
Oxidised Maximum oxidation number of N = + 5,

0 +1 +3-2
\
+4 -2 +1 -I
Minimum oxidation number of N = - 3

CI2 + NaCl O2 > 2CIO2 + 2NaCl Thus, the oxidation number of N can either
increase by losing electrons or can decrease its
Reduced
O.N by accepting electrons. Therefore, HNO2
Here, CI2 is reduced to NaCl while NaC102 is acts both as an oxidising as well as a reducing
oxidised to CIO2 agent.
REDOX REACTIONS 8/67

8. How does CU2O act as both oxidrnt and +1 X -2


reductant ? Explain with proper reactions (a) By conventional method : Na^ S, O3
showing the change of oxidation numbers in or2x(+ l) + 2.v + 3x(-2) = 0
each example. (West Bengal JEE 2004) or X - + 2 (wrong)
Ans. Cu'*’ undergoes disproportionation to form Cu"'*' But this is wrong because both the sulphur atoms
and Cu, cannot be in the same oxidation state as is evident
Reduced from the fact that when Na^S203 is treated with
dil. H2SO4, one S atom gets precipitated while

low
+ 2 + 0

Cu(aq)
the other gets converted into SO^. The oxidation
+
Cu(aq) Cu(aq) + Cu(.y)
numbers of these two S atoms can, however, be
Oxidised 1 determined by the chemical bonding method.
Here, Cu"^ oxidises Cu'*' to Cu^'*’, therefore, Cu* (h) By chemical bonding method : The structure
acts as an oxidant. of Na2$203 is
Again, Cu'*' reduces Cu'*' to Cu, therefore, Cu + S s

ee
acts as a reductant. t
Na-^ “0—S—Q-Na-^ or Na'*-0—S—0"Na+

F
Thus, Cu* or CU2O acts both os an oxidant as

Fr
well as reductant. 0 O

Specific examples where Cu-,0 acts both a Since there is a coordinate bond between the two
reductant as well as an oxidant are discussed S atoms, therefore, the acceptor S atom has an

for
below :
ur O.N. of - 2. The O.N. of the other S atom can be
calculated as follows :
Thus, Cu* or C112O acts both os an oxidant as
well as a reductant 2x(+l) + 3x(-2) + ji+lx(-2) = 0
ks
(for Na) (forO atoms) (for coordinate S)
(0 When heated in air, CU2O is oxidised to CuO,
Yo
or +2-6 + x--2 = 0 or .v= + 6
+ 1 -2 +2 -2
oo
0

CU2O +I/2O2 > 2CuO Thus, the twoSatoms inNa2S20^have oxidation


numbers of - 2 and + 6.
eB

i.e., CU2O acts as a reductant and reduces O2 to (ii) Oxidation number of chlorine in bleaching
0^-.
powder, CaOCl2
(ii) When heated with Cu^S, it oxidises S^~ to 2+ -2 X
r

SO2 and hence CU2O acts as an oxidant Average O.N. of Cl in CaOCl2 is Ca O CI2
ou
ad

+1 -2 4l -2 0 44 or2.v + 2- 2 = 0orx = 0
2CU2 O +CU2 S 6CU + SO2 (a) By stoichiometry. The composition of
Y

9. Can the reaction, bleaching powder is Ca^'*' (OCl”) Cl". Here, O.N.
of Cl in OCr is +1 while that in Cl" is - 1 and
Cr20^“ + H2O <■ t 2CrOj' + 2H+ the average of two oxidation numbers = I x
nd
Re

be regarded as a redox reaction ? (+ |) + 1 x(- 1) = 0.


11. Find out the oxidation states of two types of
Fi

Ans. Oxidation number of Cr in Cr20^" = + 6,


Fe atoms in Fe4[Fe(CN)g]3 and rewrite the
formula in stock notation form.
Oxidation number of Cr in CrO^” = + 6
Ans. From our knowledge of coordinate complexes,
Since during this reaction, the oxidation number
we know that the species Fc4 which lies outside
of Cr has neither decreased nor increased,
the complex ion, i.e., [Fe(CN)^3 is the +ve part
therefore, the above reaction cannot be regarded
as a redox reaction.
while the complex ion itself is the -ve part. In
other words, the +ve charge on 4 Fe atoms outside
10. Find out the oxidation numbers of (i) S atoms
the coordination sphere is balanced by the -ve
in Na2S203 and Cl atoms in bleaching powder, charge on the complex ion. Since Fe has two
CaOCl2. oxidation states, i.e., +2 and +3 and oxidation
Ans. (0 Oxidation numbers of S atoms in Na2S203 number of CN" = - 1, therefore.
8/68 ‘PxaxCee^'A N. w Course Chemistry fXIiPgTWn

Total -ve charge on the complex ion, (ii) Since E® of Fe is more negative than that of
[Fe(CN)(5l3 = 3 [(+ 2 X 1) + 6 (- 1)] = - 12 Ni, therefore, Fe will be oxidized to Fe^'*’ ions
Total -ve charge on 4 Fe atoms = 4 x (+ 3) while Ni^'*' ions will be reduced to Ni. Thus. Fe
reduces ions.
= +12

Fe in the complex ion has an O.N. of + 2 while 15. The electrode potential of four metallic
the Fe atoms outside the coordination sphere have elements (A, B, C and D) are + 0*80, -0*76,
an O.N. of + 3. Thus, the stock notation for
+0*12 and +0*34 V respectively. Arrange them
Fe4[Fe(CN>6]3 is FeJ" [Fe^ (CN)^]3 . in order of decreasing electropositive
character.
12. Consider the following two decomposition

w
reactions : Ans. Higher the electrode potential (E“) lower is the
tendency of the metal to lose electrons and hence
(i) 2 H2O >2 + O2
lower is the electropositive character of the metal.
(h) CaC03 > CaO + CO2

F lo
Since the electrode potentials increase in the
Which of the two is a redox reaction ? Explain. order:
Ans. For a decomposition reaction to be a redox
- 0-76 (B), + 0-12 (C). + 0-34 (D) and +0-80 (A),
reaction, one or both the products of

ee
therefore, electropositive character decreases in
decomposition must be in the elemental state.
the same order: B > C > D > A.
Therefore, reaction (/) is a redox reaction since

Fr
both the products, i.e., H2 and O2 are in the 16. I2 and Br2 are added to a solution containing
elemental state. Br“ and I" ions. What reaction will occur if,

for
0 0
I2 + 2 e- + 2 I- ; E" = +0*54 V and
2H^O(l) >2U^(g) + 0^ig) Bc2 + 2e~ -> 2 Br" ; E” = + 1-09 V ?
ur
But reaction (//) is not a redox reaction since
Ans. Since E° of Bt2 is higher than that of I2, therefore,
neither of the two products is in the elemental
Br2 has a higher tendency to accept electrons that
s
state
I^. Conversely, 1“ ion has a higher tendency to
ook
Yo
+2 +4 -2 +2-2 +4 -2
lose electrons than Br” ion. Therefore, the
Ca C O3 (i) ^ CaO(5) + C02(g) following reaction will occur :
eB

13. A mixture of FeCl2 and SnCl2 can exist 21- + I2 + 2 e-


together but that of FeCl3 and SnCl2 cannot. Bt2 + 2 e 4 2Br-
Explain. Why ?
21 + Br2 >l2+2Br
Ans. Since both FeCl2 and SnCl2 being reducing
our

In other words, I~ ion will be oxidised to I2 while


ad

agents can stay together but an oxidising agent


(FeCl3) and reducing agent (SnC^) cannot stay Br2 will be reduced to Br~ ions.
together. 17. Explain the following :
(/) It is not possible to determinethe
Y

II. Electrode Potential


absolute value of single electrode potential.
Re

14. An iron rod is immersed in a solution


(ii) Iron undergoes oxidation more readily
nd

containing 1*0 M NiS04 and 1*0 M ZnS04.


than copper.
Predict giving reasons which of the following
Fi

reactions is likely to proceed ? (Hi) In an electrochemical cell, an electrode with


(/) Fe reduces Zn^'*’ ions, (ii) Iron reduces Ni^'*’ lower electrode potential acts as the reducing
ions. Given : agent,
(iv) When a copper rod is placed in silver
E' = -0'76V, E' Fe^^/Fe = -0-44V nitrate solution, the solution becomes hot but
Zn^^/Zn
the reverse is not true,
and E = -0*25V
(v) Iron reacts with dilute H2SO4 to evolve H2
Ans. (/) Since E° of Zn is more negative than that of gas but Ag does not.
Fe, therefore. Zn will be oxidised to Zn^'*’ ions (vi) The colour of KI solution containing starch
while Fe-'*' ions will be reduced to Fe. In other
turns blue when it is shaken with cold CI2
words. Fe will not reduce Zrr'^ ions. water. Explain why ?
REDOX REACTIONS 8/69

Ans. (/) The potential difference between two reducing agent than H2 and hence reduces H*
electrodes can be determined by connecting them ions to produce H2 gas
to a voltmeter. Therefore, it is not possible to Fe is) + 2 H+ (aq) > Fe2+ (aq) + H2 (g)
determine the potential of a single electrode
In contrast, Ag has higher electrode potential (/>.,
because a single electrode constitutes a half cell
and a half cell reaction cannot take place = 0-80 V) than hydrogen. Therefore,
Ag+/Ag
independently.
H2 is a better reducing agent than Ag. In other
An electrode in a half cell cannot lose or gain words, Ag cannot reduce ions to produce H2

w
electrons by itself. For transfer of electrons, one gas.
half cell has to be connected to some other half
2 Ag (j) + 2 2 Ag-^ ,(aq) + (g)
cell. Thus, we cannot determine the absolute
value ofelectrode potential of a single electrode. (vi) Since CI2 (P = + 1 -36 V) is a stronger

Flo
cij/cr
In other words, only relative value of electrode
potential can be determined by connecting the oxidising agent than I2 (E“ = + 0-54 V),
half cell with some standard electrode as the I2/1-

ee
reference electrode. therefore, when CI2 is passed through KI

Fr
(«) The electrode potential of iron, (i.e., solution, CI2 gets reduced to colourless Cl“ ions
while 1“ ions get oxidised to violet coloured
E®_F© 2+m
/Fc
~~ lower than that of copper iodine

for
(/.e., E® = + 0-34 V) and hence Fe has CI2 {aq) + 21" {aq) > 2 Cl {aq) +12 (5)
Cu2+/Cu
r
greater tendency to get converted into Fe^'*’ ions
The I2 thus produced forms a blue coloured
ou
inclusion complex with starch and hence the
than Cu. In other words, iron undergoes oxidation solution turns blue.
ks
more readily than copper.
18. Copper dissolves in dilute nitric acid but not
{iiO An electrode with lower electrode potential
oo
in dilute HCl. Explain.
has more tendency to get oxidised. In other words,
Ans. Since E® of Cu^'^/Cu electrode (+ 0-34 V) is
Y
it has more tendency to release electrons and
eB

hence acts as a reducing agent. higher than that of H‘^/H2 electrode (0-0 V),
therefore, H"*" ions cannot oxidise Cu to Cu^'*’ ions
(/v) Copper {i.e., E®^2+/q, = + V) has and hence Cu does not dissolve in dil. HCl. In
r

lower electrode potential than silver {i.e., contrast, the electrode potential of NOj ion, i.e.,
ou
ad

E® = + 0-80 V). Therefore, Cu releases


Ag+/Ag NOJ /NO electrode (+ 0-97 V) is higher than
Y

electrons and gets oxidised to Cu^'*' ions while that of copper electrode and hence it can oxidise
Ag"^ ions accept these electrons and get reduced Cu to Cu^'*’ ions and hence Cu dissolves, in dil.
to Ag metal.
nd

HNO3. dissolves in dil. HNO^ due to


Re

Cu {s) + 2 Ag*" {aq) > Cu^'*’ {aq) + 2 Ag (jr)


The chemical eneigy of this reaction appears as oxidation of Cu by NOj ions and not by H*
Fi

heat and hence the solution becomes hot. ions.

Since the reverse reaction cannot occur (because 19. The standard electrode potential corres
the E® of Ag is higher than that of Cu), ponding to the reaction Au^"*^ (aq) + 3e“ >
2 Ag (5) + Cu2+ {aq) 2 Ag+ (5) + Cu {s) Au (s) is 1*50 V. Predict if gold can be dissolved
therefore, no heat is liberated and hence the
in IM HCl solution and on passing hydrogen
solution does not become hot. gas through gold salt solution, metallic gold
will be precipitated or not.
(v) Fe has lower electrode potential {i.e.,
Ans. Consider the half reactions,
Fe2+/Fe = - 0-44 V) than that of hydrogen (/. e.,

2 H+ {aq) +2e~ > H2 (g) ; E® = 0-0 V

H+/H2
= 0-0 V), therefore, Fe is a better Au^+ {aq) + 3 e~ > Au {s) ; E“ = 1-50 V
8/70 New Course Chemistry fXI1 orsTMn

Since E° (I -50 V) for Au^'''/Au is higher than that Since the EMF of the cell is -ve, therefore,

I
CUSO4 does not react with silver. In other words,
H'*'/ — H-j (0 0 V), therefore, Au^"*" can be more O/.VO4 solution can be stored in a silver vessel.

ow
{Hi) We cannot store, CUSO4 solution in a gold
easily reduced than H"*" ions. This implies that vessel if the following redox reaction occurs ;
Au^"*" ions can be reduced to metallic gold by H2 2 Au + 3 Cu2+ 4 2 Au^-' + 3 Cu
gas but H'*’ ions cannot oxidise metallic gold to
Au^"*" ions. In other words, metallic i^old does not The cell corresponding to the above redox
dissolve in I M HCl. Instead II, gas can teduce reaction may be represented as

e
gold salt to metallic gold. Au I Au^+ 11 Cu-+ I Cu and

re
III. Electrochemical Ceils E° = E° -E°
cell
Cu^'^.Cu Au^'*',Au
20. Is it possible to store :
= 0-34- 1-50 =-1-26 V

F
Frl
(i) Copper sulphate solution in a zinc vessel ?
Since the EMF of the above cell reaction is -ve,
(ii) Copper sulphate solution in a silver vessel ?
therefore, CUSO4 solution does not react with
(Hi) Copper sulphate solution in a gold vessel ?
Given: E' = +0*34V ; ou gold. In other words, CuSO^ solution can be
stored in a gold vessel.

sr
Cu2+,Cu
21. The standard electrode potentials at 298 K are

kfo
E" = +0*80V and given below :
Ag+,Ag
E" = -0-76V, E° Fe^+ZFe = -0*44V,
E® =r + l-50V. Zn^'^/Zn
Au^''',Au oo E' = (h0V and E” = +0-34V.
H^/Hj Cu^*/Cu
Ans. (i) We cannot place CUSO4 solution in a zinc
Y
vessel, if the following redox reaction occurs : Which of the two electrodes should be
reB

Zn + CUSO4 > ZnS04 + Cu combined to form a cell having highest EMF ?


or Zn + Cu2+ >Zn^+ + Cu Identify the cathode and the anode and write
the cell reaction. Also mention the direction
By convention, the cell may be represented as
uY

of flow of electrons in the external as well as


Zn I Zn^-*-11 Cu^-^ I Cu
the internal circuit.
= E‘ -E° Ans. To have maximum EMF, the anode should have
cell Cu^'^.Cu Zn-'^,Zn
ad

the minimum and cathode should have the


do

= 0-34 - (- 0-76) = + MO V maximum E®. Therefore, Zn^'*'/Zn couple should


Since EMF comes out to be positive, therefore, be made as the anode while Cu^’^/Cu couple
CUSO4 reacts with zinc. In other words, CuSO^ should be made as the cathode. The cell reaction
in

.solution cannot be stored in a zinc vessel, IS ;


Re

(ii) We cannot store CUSO4 solution in a silver Zn (i) + Cu^'*' {aq) ^ Zn-"*" (aq) + Cu (s)
vessel if the following redox reaction occurs :
F

EMF = E'^ 2+
-E'’
Cu /Cu Zt?* I Zn
2 Ag + Cu^^ 2 Ag'^ + Cu
= + 0-34 - (- 0-76) = + M V.
By convention, the cell of the above redox
The direction of flow of electrons is from Zn to
reaction may be represented as
Cu in the external circuit and from Cu to Zn in
Ag I Ag+ 11 Cu2+ I Cu and the internal circuit. The direction of flow of
= E' -E° current is, however, in the reverse direction, i.e.,
cell
Cu^'^.Cu Ag'^.Ag from Cu to Zn in the external circuit and from
= 0-34-0-80 = -0-56 V Zn to Cu in the internal circuit.
REDOX REACTIONS 8/71

● Very Short Answer


● Short Answer

● Long Answer

VERY SHORT ANSWER QUESTIONS Carrying 1 mark

1. Oxidation-Reduction-Electronic 7. Name a compound of carbon which have both


valency and oxidation number of 4.
and Oxidation Number Concept
Ans. In CCI4. caibon is aiiached to four monovalent
1. Deilne oxidation and reduction in terms of
chlorine atoms and hence its valency is 4. Its

w
electrons. oxidation number is also 4 as calculated below :
Ans. Oxidation involves loss and reduction involves
^ + 4(- 1)=0 or x = 4
gain of electrons.

F lo
8. What is valency and oxidation number of
2. What is a redox reaction ? Give one example.
carbon atoms in C3O2 ?
Ans. Oxidation—reduction reactions are called redox
Ans. The structure of C30t is
reactions, Mg + F2 > Mg-"*" (F")2-
-2 +2 0 +2 -2
Here, Mg is oxidised to Mg-"^ while F2 is reduced

e
o = c =c= c = c

Fre
to F~ ions.

3. Show that the formation of sodium chloride from Since each double bond is formed by sharing of
two electrons, and each carbon has two double
gaseous sodium and gaseous chlorine is a redox

for
reaction. bonds, therefore, all the three carbon atoms have

Ans.
a valency of 2 x 2 = 4.
Na(g) ■> Na"^ (g) + e ]x2
For calculation of oxidation number, refer to page
our
...oxidation
8/21.
Cl2(g) + 2e- ^2C|-(^) ...reduction
s
9. Define oxidation and reduction in terms of
ook

2 Na-" Cl- (s) oxidation number.


2NaU) + Cl2(5)
...overall redox reaction Ans. Oxidation involves increase in O.N. while
eB
Y
4. Define oxidising and reducing agents in terms reduction involves decrease in O.N.
of electrons.
Sn2+ + 2Hg2+ ■>
Sn-^+ + Hg^+
Ans. Oxidising agents are electron acceptors while
reducing agents are electron donors. For example. Here, Sn^'*' gets oxidised to Sn'^'*' because its O.N.
our
ad

Oxidising agent: increases from +2 to +4 while Hg^"^ gets reduced


MnO^ + 8 H+ + 5 <?- to Hg2’*’ because its O.N. decreases from + 2 to
Pemianganate ion + 1.
2+
dY

Mn + 4HoO Further since O.N. of Sn^"*" increases from +2 to


Re

2+
Reducing agent: Fc2+ ●> Fe^-^ + e +4, it acts a reducing agent while O.N. of Hg
Ferrous ion
decreases from +2 to +1, it acts as an oxidising
Fin

5. What is the oxidation state of Ni in Ni(CO)4 ? agent.


Ans. Zero. Oxidation state of metal in metal carbonyls 10. When magnesium ribbon is burnt in air, two
is zero.
products are formed, magnesium oxide and
6. Name a compound of carbon whose valency magnesium nitride ? Point out the oxidising
is four but its oxidation number is zero ? and reducing agents.
Ans. In CH2CI2, the valency of carbon is four, since it Ans. Reducing Oxidising
is attached to four monovalent atoms but its agent agent
oxidation number is zero as calculated below : 0 +2 -2

.r +1 -I 2Mg(.r) + Ojig) ^ 2 Mg O is)


CH2CI2 ●●.^ + 2(+l) + 2(-1) = 0 or .r = 0 Mag. oxide
8/72 New Course Chemistry (X1)BS2SD

0 +2 -3 concentration of Ag'*’ ions decreases while that


3 Mg (s) + No (g) Mg3 N2 is)
of NO3 ions remains unchanged.
Mag. nitride
2 Ag-*- iaq) + Fe (j) > 2 Ag (_s) + Fe"-^ (aq)
11. In the reaction, MnO, + 4 HCl > MnCl2 +
CI2 + 2H2O which species is oxidised ? 16. Arrange the following metals in which they
Ans. HCl is oxidised to Cl-, since the O.N. of Cl displace each other from the solution of their
increases from - 1 in HCl to 0 in CI2. salts : Al, Cu, Fe, Mg, Ag and Zn.
12. What are displacement redox reaction ? Give Ans. Metals which have lower electrode potentials can
two examples. displace others which have higher electrode

w
Ans. A redox reaction in which an atom or ion in a potentials from the solutions of their salts. Thus,
the order is :
compound is replaced by an atom or ion of some
other clement is called a displacement reaction. Mg (- 2-37 V), A1 (- i -66 V), Zn (- 0-76 V),

F lo
These are of two types : Fe (- 044 V), Cu (+ 0-34 V), Ag (+ 0-80 V).
(/) Metal displacement redox reaction :
111. Electrolysis and
+2 +6 -2 0 0 +2 -2 Electrochemical Cells

ee
Cu S 0^{aq) + 7si(s) ■> Cu(^) + 21n S
Metal
17. Give anodic and cathodic reactions occurring

Fr
(i7) Non-metal displacement redox reaction : during electrolysis of acidified water.
+1-1
Anodic reaction :

for
Zn(i)+ 2HCl(fl^) +
2H20(/) > 02(g) + 4H*(aq) + 4e-
Cathodic reaction :
our
+2 +6 -2 0

Zn S O^(aq) + H2 (g) 2 H2O (0 + 2 e~ > H2 (g) + 2 OH- (aq)


Non-Mctal
18. The following two reactions can occur during
s
ook
n. Electrode Potential electrolysis of aqueous sodium chloride
solution.
13. The standard electrode potentials of a few
Y
eB

Na+ iag) + e~ > Na (s), E" = - 2*71 V


metals are given below :
2 H2O (0 + 2 e~ > H2 (g) + 2 OH" (aq),
A1 (- 1-66 V), Cu (+ 0-34 V), Li (- 3-05 V),
E“ = - 0-83 V
Ag (+ 0-80 V) and Zn (- 0-76 V).
our
ad

Which reaction takes place preferentially and


Which of these will behave as the strongest why ?
oxidising agent and which as the strongest
Ans. Since E° of H2O is higher than that of Na'*’ ions,
reducing agent ?
therefore, reduction of H2O takes place
Y

Ans. Since E® value of Li (-3-05 V) is the lowest while preferentially to evolve H2 gas at the cathode.
Re

that of Ag (+ 0-80 V) is the highest, therefore, Li 19. 2 M (s) + H2SO4 (aq) > M2SO4 (aq) + H2 (g)
nd

is the strongest reducing agent while Ag"*" is the Give the representationof the cell which
strongest oxidising agent. involves the above redox reaction.
Fi

14. At what concentration of Zn^^ (aq) will its Ans. M (s) I M2SO4 (aq) 11 H+ (aq) I H2 (g), Pi.
electrode potential become equal to its 20. A cell is constructed using Cu^"^/Cu and Al^/Al
standard electrode potential ? electrodes. What is the net cell reaction ?
Ans. At 1 M concentration.
Ans. From Table 8.1, page 8/44, we know that E“ of
15. A solution of silver nitrate was stirred with Al^'^/Al (- 1-66 V) is lower than that of Cu^'^/Cu
iron rod. Will it cause any change in the (+0-34 V). Therefore, oxidation occurs at the
concentration of silver and nitrate ions ? A1^'*’/A1 electrode and reduction occurs at the
Cu^'’’/Cu electrode. Consequently, the net cell
Ans. Since E° of Fe^'^^/Fe (- 0-44 V) is lower than that reaction is
of Ag'*'/Ag (+ 0-80 V) electrode, therefore, Ag"*"
gets reduced and Fe gets oxidised. As a result,
2 A1 (s) + 3 Cu^+ (aq) > 2 Al^-^ (aq) + 3 Cu (s).
REDOX REACTIONS 8/73

21. Define EMF of the cell. oxidation or reduction half reaction. For example,
Ans. EMF of a cell is defined as the difference in the (aq)/Zn (s), CI2 (g)/Cr (ag), etc.
electrode potentials of the two half cells when 23. Can we use KCl as electrolytein the salt bridge
the cell is not sending current through the circuit. of the cell, Cu (s) I Cu^"'' (ag) I [ Ag'*’ (aq) I Ag (^) ?
22. What is a redox couple ? Ans. KCl cannot be used as electrolyte in the salt
Ans. A redox couple consists of oxidised and reduced bridge because Cr ions will combine with Ag"^
form of the same substance taking part in an ions to form white precipitates of AgCl.

w
SHORT ANSWER QUESTIONS Carrying 2 or 3 marks

1. Oxidation-Reduction-Electroiiic 13. Discuss the following redox reactions :


and Oxidation Number Concept (i) Combination reactions

F lo
(//) Decomposition reactions
1. Explain the terms: oxidation and reduction in terms
(Hi) Displacement reactions
of electrons. Give one example in each case.
(tv) Disproportionation reactions.

ee
[Art. 83]
Give one example in each case. [Art. 8.4.6]
2. Define the terms : oxidising agent and reducing

Fr
agents according to the electronic concept. Give 14. Comment upon the statement. “Paradox of
fractional oxidation numbers.' [Art. 8.4.7]
one example in each case. [Art. 8.3]
15. Starting with the correctly balanced half

for
3. Comment upon the statement : oxidation and
reactions, write the overall net ionic reaction in
reduction reactions go side by side.
our
the following changes :
[Art. 8.2-8.3]
(0 Chloride ion is oxidised to Clo by MnO^ (in
4. Explain what happens when a zinc rod is dipped acid solution)
s
in CUSO4 solution ? [Art. 8.3.1.]
ook
5. What are half reactions ? Explain with examples.
(//) Nitrous acid (HNO2) reduces MnO^ (in
acid solution)
[Art 8.3.1.]
Y
(in) Nitrous acid (HNO2) oxidises 1“ to I2 (in
eB

6. In the following redox reactions, identify the


acid solution)
oxidising agent and the reducing agent:
(0 H3PO2 (aq) + 2 HgO 2 (aq) + 2 H2O (aq) ■> (/v) Chlorate ion (QO3) oxidises to Mn02 (-0
(in acid solution)
r

H3PO4 (aq) + 2 Hg (/) + 4 HCl (aq)


ad
ou

(v) Chromite ion (OOj) is oxidised by H2O2


(//■) O2 (g) + PtFg (g) ^ 0+ [PtF^r(s)
(in strongly basic medium)
[Hint. Oxidising agents : (/) HgCl2, (») Ptp5 ; Also find out the change in the oxidation number
Y

Reducing agents : (/) H3PO2, (ii) O2] of the underlined atoms. [Art. 8.5]
7. Define oxidation and reduction in terms of
[Ans. (0 2Mn04 + 16 + lOCl"
Re
nd

oxidation number. Give examples in each case


to illustrate your answer. [Art 8.4.2] 5CI2 + 2Mn^* + 8H2O
Oxidation number of Mn changes from +7 in
Fi

8. Define oxidising and reducing agents in terms


of oxidation number. Cite two examples in each MnOT4 to +2 in Mn^'*'
case to support your answer. [Art 8.4.3]
(ii) 2Mn04 + 6H-^ + SNO^
9. H2S acts only as a reductant whereas SO2 acts
oxidant and reductant both. Why ? [Art 8.4.2] 5NOJ +3H20 + 2Mn2+
10. H2O2 acts as reductant as well as oxidant. Oxidation number of N changes from +3 in NO2
Explain. [Art 8.4.2]
ion to +5 in NOJ ion
11. Explain the difference between valence and
oxidation number. [Art 8.4.4] (Hi) 21- + 4H‘*-+ 2NO2 ^ I2 + 2NO + 2H2O
12. Discuss briefly types of redox reactions. Give one Oxidation number of N changes fi om +3 is NO2
example in each case. [Art 8.4.6] to +2 in NO
8/74 New Course Chemistry (XI)S23HD

17. Arrange the following metals in increasing order


(/V) + ClOj + 6H-^
of reactivity. Which one will be the strongest
SMn'^'*' + Cl" + 3H2O reducing agent and which is the weakest ?
Oxidation number of Cl changes from + 5 in
Mg, Na, Ag, Cu, Fe, Zn [Art. 8.7.10]
ClOj to -1 in Cr
lAns. Ag < Cu < Fe < Zn < Mg < Na ; Na is
(v) 2 CrOj + H2O2 + 20H- ->
the strongest reducing agent while

low
2Cr02- +2H2O Ag'*' is the strongest oxidising agent]
«,':iidation number of Cr changes from +5 in ill. Electrochemical Cells
CrOj to +6 in CrO^“ ] 18. Draw a labelled diagram for the Daniell cell.
Discuss its working. [Art. 8.7.2]
II. Electrode Potential
19. Discuss briefly the function of the salt bridge in
16. What is an electrochemical series ? How can this an electrochemical cell. [Art. 8.7.3]

e
be used to explain the oxidising and reducing 20. Discuss the applications of redox reactions.

re
rF
abilities of elements? [Art 8.7.9-8.7.10] [Art. 8.8]

F
LONG ANSWER QUESTIONS Carrying 5 or more marks

r
1. Explain the terms : (t) oxidation, («) reduction, terms of oxidation number. Give two examples

fo
(Hi) oxidising agent and (rV) reducing agent in in each case to illustrate your answer.
ou
terms of electrons. Give two examples in ech case [Art. 8.2-8.41
to justify your answer. [Art. 8.3]
2. Brielly discuss some redox reactions occurring
[Art. 8.3.1]
ks
5. What is an electrochemical cell ? Briefly discuss
its construction and working. How is it
in aqueous solutions.
oo
represented ? [Art. 8.7.2]
3. Define oxidation number. How does it differ from
6. What is electrochemical series ? How does it help
Y
valency 7 Write the general rules of assigning
eB

oxidation numbers to various atoms in ions and us (0 in comparing the oxidising and reducing
molecules. [Art. 8.4.1] power of different elements and (//) predicting
whether a metal will react with a mineral acid to
4. Explain the terms ; (i) oxidation, (i7) reduction, [Art. 8.7.5]
give hydrogen gas or not ?
ur

(Hi) oxidising agent and (iv) reducing agent in


ad
Yo

CASE-BASED VERY SHORT/SHORT QUESTIONS

CASE 1. Redox reactions involve transfer of Based on the above paragraph, answer
d

electrons from one atom or ion to the other. In case of questions no. 1 to 4 :
Re

ionic reactions, it is very easy to determine the total


in

1. Find out oxidation state of iron in compounds (A)


number of electrons transferred from one atom or ion
and (B).
to the other. However, in many redox reactions involving
F

covalent compounds, it is not so easy to determine Na4[Fe(CN)5NOS] [Fe2(CO)9l


(i) the direction of transfer of electrons and («) number (A) (B)
of electrons transferred from one reactant to the other
simply by looking at the chemical reaction. In such 2. A compound contains atoms of three elements A,
cases, to know which atom is oxidised and which atom B and C. If the oxidation number of A is + 2, B is
is reduced, the concept of oxidation number has been + 5 and 6 is - 2, then what is the possible formula
introduced. The oxidation number or oxidation state of of the compound 7
an element is defined as the charge which an atom of 3. Out of the following, pick out compounds which
the element has in its ion or appears to have when
act both as oxidising and reducing agents: HNO3,
present in the combined state with other atoms. To
determine the exact oxidation number of an atom in an
HNO^, CIO", Fe-^'*' Give plausible explanation for
your answer.
ion or molecule, certain rules have been formulated.
REDOX REACTIONS 8/75

4. For the reaction, (0 cuprous chloride


+ MnO" 2+ (ii) potassium manganate
> Mn03 + I/2O2
8. The formula of sodium thiosulphate, is Na2S203.
If one mole of MnO^ oxidises 1-67 moles of Do the two sulphur atoms have the same or
to MnOj , find out of value of x. different oxidation state/s. Explain.

ow
CASE 2. Although the term valency and oxidation CASE 3. Oxidation-reduction reactions, (i.e..
state or number have different meanings yet these terms Redox reactions) are complementary, i. e., whenever any
are quite useful in understanding the formation of substance is oxidised another substance is always
compounds from different elements. Further, reduced at the same time and vice-versa. These redox
compounds of metals which show more than one reactions are classified into four types. These are :

e
oxidation states are distinguished from one another by combination reactions, decomposition reactions, metal
placing a Roman numeral such as I, II, III, IV, V, VI, displacement/non-metal displacement reactions and

re
VII, etc. indicating the oxidation state of the metal disproportionation reactions.

Frl
within parenthesis after the symbol or name of the metal. Based on the above paragraph, answer

F
This system of nomenclature is called Stock notation. questions no. 9 to 12 :
Based on the above paragraph, answer 9. Give an example of a combi nation reaction redox
questions no. 5 to 8 : reaction.

5. With a suitable example define the terms valency


ou 10. Give an example of a decomposition redox

or
and oxidation number. reaction.

6. Discuss two main points of difference between 11. Are all decomposition reactions are redox

kfs
valency and oxidation number. reactions ? Justify.
7. Write the formula of the following compounds 12. Define disproportionation reaction ? Give one
with stock notation.
oo
example.
Y
ANSWERS
B

1. Na4[Fe(CN)gNOS] ^ the O.N. of N in HNO2 is + 3 which is higher


(A) than its minimum O.N. of - 3 and lower than its
re

A~ highest O.N. of + 5. Likewise, O.N. of Cl in CIO"


4 Na+ + [Fe(CN)5(NOS)l is + 1 which is higher than its lowest O.N. of - 1
Let theO.S of Fe be.x, O.S. of CN =- 1 and O.S.
and lower than its highest O.N. of + 7. Thus,
oYu

of NOS=-l.
HNO2, CIO" can function both as an oxidising
ad

x + 5{-l)+l (- 1) = 4 or x = + 2
as well as a reducing agent.
+7
[Fe2(CO)9]
●> Mn^'*'
d

(B) 4. MnO^ +5e


Let the O.S. of Fe be x. Since 1 mole of MnO^ accepts 5 moles of
in
Re

electrons, therefore, 5 moles of electrons are lost


2(.r) + 9(0)orx = 0 (V O.S of CO = 0) by 1-67 moles of
F

2. Sum of the oxidation numbers of all the elements .●. 1 mole of M-'"*’ will lo.se electrons = 5/1 -67
in the M.F. of any compound is zero. = 3 moles (approx)
If we assume that A forms the cation and B and
Since M-''*' changes to MOJ {where O.N. of M
C together form the anion, then by hit and uial = + 5) by accepting 3 electrons
method, we find that A = B = 2 and C = 8. we
.●. Oxidation state of M, i.e., x = + 5- 3 = + 2
have,
5. Valency is the combining capacity of an element.
3x(+2)+ 2 (+5-4x2) = 6- 6 = 0
For example, carbon combines with four hydrogen
Thus, formula of the compound is A3(BC4)2. atoms to form CH4, so valency of carbon is 4.
3. Any species whose O.N. is lower than the highest On the other hand, oxidation number is the charge
and higher than the lowest O.N. can act both as which an atom has or appears to have when
an oxidising as well as the reducing agent ? Now present in the combined state. For example, Mg
2+
8/76 “Pxudee^'^ New Course Chemistry (XI)CSISD

and have oxidation numbers of + 2 and - 3 0

respectively. Further, in CO2, since the O.N. of O CH^U) + 202(g) 4

is - 2, therefore, O.N. of C is + 4, x.e., x + 2(-2) +1 -2


= 0.
CO^(g) + 2H^Oil)
6. (/) Valency cannot be zero but O.N. can be zero. 10. A redox reaction in which a molecule breaks
For example, O.N. of C in CH2CI2 is zero. down to form two or more components is called
(«) Valency is only a number, no +ve or -ve sign a decomposition reaction. For example,
attached to it. For example, valency of O is 2 but
+1 -2 0 0
its O.N. is - 2. Electrolysis
2H2 Oil) ^ 2H2(g) + 02(g)

w
7. Stock notation for cuprous chloride is Cu (I) Cl
or Cu2 (I) CI2 and that of potassium manganatc is 11. No. All decomposition reactions arc not redox
K2Mn (VI) O4. reactions. For decomposition reaction to be a

F lo
redox reaction, it is essential that one of the
8. Structure of sodium thiosulphate is
S S
products of decomposition must be in the
t elemental state. For example,
Na'^O—S—O'Na"^ or Na'^'^O—S—O Na+

ee
+l +5 -2 A +1-1 0
O 2 K Cl 03(5) ^ 2KCl(j) + 302(g)

Fr
o

Since there is a coordinate bond between the two In contrast, decomposition of CaC03 to form
S atoms, therefore, the acceptor S atom has an CaO and CO2 is not a redox reaction, since none

for
O.N. of - 2. The O.N. of the other S atom can be of products of decomposition, i.e., CaO and CO2
calculated as follows : are in the elemental stale
our
2x(+l) + 3x(-2) +X + lx (-2) = 0 A
CaCOj is) ^ CaO (5) + CO2 (g)
(forNa) (forOatoms) (for coordinates)
s
12. A reaction in which the same species is both
or +2-6+a:-2 = 0 or Jt = + 6
ook
oxidised as well as reduced is called a
Thus, the two S atoms have O.N. of - 2 and + 6.
disproportionation reaction. For example.
9. Redox reactions in which two atoms or molecules
Y
eB

Reduced
combine together to form a third molecule are I
called combination reaction. For example, 0 ●3 + 1

P4 + 3 NaOH + 3 H2O ► PH3 + 3NaH2P02


0 0 +2 -3
r

3Mg(^)+N2(g) ^ Mgj N2 (s) Oxidised


ad
ou

CASE-BASED MCQs AND ASSERTION-REASON QUESTIONS


Y

Based on the above paragraph, answer


Re

CASE 1. Electrochemical cells or batteries based


nd

on redox reactions are widely used in our daily life to questions no. 1 to 4 :
run a number of small or big gadgets and equipments. 1. Which of the following is not a redox reaction ?
Fi

Further, the energy required for our daily needs is (a) H2 + Br2“ ●>2HBr

obtained by burning of fuels. So much so, human body ib) CI2 + H2O ^ HCl + HOCl
also needs energy for its proper functioning which is
(c) NaCl + AgN03 ■> NaN03 + AgCl
obtained by burning of glucose. Besides these, redox
reactions are also used in pharmaceutical, industrial, id) 2 Na2S203 + I2 ■> Na2S40g + 2 Nal
metallurgical and agricultural areas. 2. In which of the following redox reactions the
underlined substance acts as an oxidising agent ?
Besides the classical concept, redox reactions have
been explained in terms of electrons and oxidation ia) NaCl03 + I2 — ^ 2 NaI03 + CI2
numbers. ib) H2S + 2 FeCl3 2 FeCl2 + 2 HCl + S
REDOX REACTIONS 8/77

(c) Cu + 2 AgNOj ●> Cu(N03).^ + 2 Ag (c) Terminal Br atoms have an O.S. of + 4 while
id) Mn02 + 4HC1 the central Br atom has an O.S. of + 6
MnCl2 + CI2 + 2 H2O
Choose the correct option out of the four options id) Terminal Br atoms have an oxidation state of
given below: + 6 while the central Br atom has an O.S. of+4

(a) Both Assertion (A) and Reason (R) are true 6. The pair of compounds having metals in their
and Reason (R) is the correct explanation of highest oxidation state is
Assertion (A).
{a) Mn02, FeCl3
{h) Both Assertion (A) and Reason (R) are true
{b) MnO^, CTO2O2

w
but reason (R) is not the correct explanation
of Assertion (A), (c) [FelCN)^]^-, [Co(CN)6l
3-

(c) Assertion (A) is true but Reason (R) is false. id) [NiCl4]2-, [C0CI4J-

F lo
{d) Assertion (A) is false but Reason (R) is true.
Choose the correct option out of the four options
3. Assertion. 2 CU2O (5) + CU2S (i) given below:
6 Cu (j) + SO2 ig) is a redox reaction. (fl) Both Assertion (A) and Reason (R) are true

ee
Reason. In this reaction, CU2O is an oxidant and and Reason (R) is the correct explanation of
CU2S is the reductant.

Fr
As.sertion (A).
4. Assertion. HNO3 is a powerful oxidising agent. ib) Both Assertion (A) and Reason (R) are true
Reason. HNO3 and Pb02 do not react. but reason (R) is not the correct explanation

for
of Assertion (A),
CASE 2. In some compounds, the application of
our
simple rules of determining the oxidation number of a (c) Assertion (A) is true but Reason (R) is false.
particular atom of an element gives fractional oxidation id) Assertion (A) is false but Reason (R) is true.
numbers. Since electrons are never transferred or shared
s
7. Assertion. The structure of letrathionate (840^“)
ook
in fractions, therefore, the very idea of fractional ion is
oxidation numbers is unconvincing. In fact, fractional
oxidation number is only the average oxidation number 0 O
Y
eB

of an element when two or more of its atoms are present


"O—S—S —S—S—O"
in different oxidation states in a given compound. In
such cases, the actual oxidation numbers of the element
0 o
can be determined by knowing its structure.
r

Reason. It is obtained by oxidation of Na2S203


ad
ou

Based on the above paragraph, answer


questions no. 5 to 8 :
with I2.

5. Which one of the following statement about the 8. Assertion. Thiosulphate ion (8203“) contains
Y

oxidation state (O.S.) of Br in Br303 is correct ? two sulphur atoms with oxidation states of - 2
{a) All Br atoms have the same O.S. of + 4 and + 6.
Re
nd

ib) All Br atoms have the same O.S. of + 6 Reason, it contains one 8 = 8 bond.
Fi

ANSWERS

1. ic) 2. ib) 3. ia) 4. ib) 5. id) 6.ih) 7. (a) 8.(6)

HINTS/EXPLANATIONS For Difficult Questions

1. NaCl + AgN03 > AgCl i + NaN03 is a 3. Both A and R are correct and R is the correct
precipitation reaction. explanation of A.
2. FeCl3 acts as an oxidising agent since it oxidises 4. Correct explanation. HNO3 and PbO, both are
H2S to S. oxidising agents.
8A78 New Course Chemistry (XI)ESE

5. Correct A. The terminal two Br atoms have an


O O
oxidation number of + 6 while the central Br atom
has an oxidation number of + 4. +5l 0 0 1+5

ow
“O—s—s—s—S—O"
o
o, T ,0 O O
'^+6 1+4
0=Br—Br—Br=0
8. Correct explanation. The structure of
O O
o thiosulphate ion is

6. Highest O.S. of Mn is + 7 in MnO^ and that of -2


s

re
Cr is + 6 in Cr02Cl2.
7. Correct explanation. The two terminal S atoms ■^S
have an oxidation number of + 5 while the two

Flr
internal S atoms have an oxidation number of zero.

F
o

ou
sr
fo
k
oo
Y
reB
uY
ad
do
in
Re
F
REDOX REACTIONS 8/79

WITH
ANSWERS

1
I

w
NCERT INTEXT SOLVED QUESTIONS

F lo
Q. 8.1. In the reactions given below, identify the species undergoing oxidation and rccuction :
(i) HjS (g) + a^ig) > 2 HCl (5) + S is) (ii) 3 Fc304 (s) + 8 AI (s) > 9 Fe is) + 4 AI2O3 (s)

ee
(iU) Na (s) + Hj ig) > 2 NaH (s).
Ans. (;) Here, H2S is oxidised to S because hydrogen has been removed from it while CI2 has been reduced to

Fr
HCl because hydrogen has been added to it.
in) Here, aluminium is oxidised to aluminium oxide because oxygen has been added to it while ferrous
ferric oxide (or magnetic oxide) has been reduced to iron because oxygen has been removed from it.

for
iiii) As shown in Ans. to Q. 8.2 below, sodium is oxidised to sodium ion while hydrogen is reduced to
ur
hydride ion.
Q. 8.2. Justify that the reaction : 2 Na (s) + Hj ig) 2 NaH (s) is a redox reaction.
Ans. Since NaH is an ionic compound, it may be represented as Na'*’ H” is). Thus,
s
ok
2 Na (s) + H2 (g) 2 Na-*- H- (^)
Yo
This reaction can be split up into the following two half reactions :
o

Na is) ■ ■> Na’*' + e~]x2 ioxidation)


eB

H2 (g) + 2e~ ^2H- ireduction)

Overall redox reaction : 2 Na (j) + H2 (g) > 2 Na’*' H" (j)


In the first half reaction, Na is oxidised to Na’*’ ion while in the second half reaction, H-, is reduced to H"
r

ions. Therefore, the overall reaction is a redox change.


ou
ad

Q. 8.3. Using stock notation, represent the following compounds : HAUCI2, TUO, FeO, Fe20„ Cul, CuO,
MnO and Mn02.
Y

Ans. By applying the various rules for determining the oxidation number of different atoms in a compound,
the oxidation number of each metallic element in the given compounds is as follows :
In HAUCI4, + 3 ; in TUO. T1 has + 1 ; in FeO, Fe has + 2 ; in Fc203, Fe has + 3 ; in Cul, Cu has
Re
nd

+ 1 ; in CuO, Cu has + 2 ; in MnO, Mn has + 2 while in Mn02, Mn has + 4 oxidation state. Therefore,
these compounds may be represented as :
Fi

HAu (IU) Cl4, Tl2(I) O, Fe(II) O, Fe2(UI) O3, Cu(I)I, Cu(U)0, Mn(U)0 and Mn(IV)02.
Q. 8.4. Justify that the reaction : 2 CujO (s) + CU2S (s) > 6 Cu (s) + SO2 (g)
is a redox reaction. Identify the species oxidised/reduced, which acts as an oxidant and which acts
as a reductant.

Ans. Writing the oxidation number of each atom above its symbol, we have,
+1 -2 +1 -2 0 +4 -2

2Cu2 0(^) + Cu2S 4 6Cu (^)+S 02(g)


Here, in this reaction, the oxidation number of copper decreases from + 1 in CU2O or Cu^S to 0 in copper
metal, therefore, Cu* is reduced. Further, the oxidation number of S increases from - 2 in CU2S to
+ 4 in SO2, therefore, S^~ is oxidised. Thus, the above reaction is a redox reaction.
Further, oxygen in CU2O helps sulphur in CU2S to increase its oxidation number from - 2 to + 4, therefore,
Cu-)0 is an oxidant.
8/80 'pfuxcCec^ ^ New Course Chemistry (XI)BaSIHD

Conversely, sulphur in CU2S helps copper both in CU2O and CU2S to decrease its oxidation number from
+ I to zero in copper metaJ, therefore, CU2S is the reductant.
Q. 8.5. Which of the following species, do not show disproportionation reaction and why ?
C10-,a02,a03 and a04
Also write reaction for each of the species that disproportionat es.

w
+1 +3 +5 +7

Ans. The oxidation state of Cl in all the given species are : C10“, ClOj, ClOj, CIO4

Out of above species, CIO4 does not undergo disproportionation since in this oxoanion, chlorine is
already present in the highest oxidation state of + 7 and hence cannot be further oxidised.

o
All the remaining oxoanions have oxidation states lower than the highest (i.e., + 7) and higher than the

e
lowest (/.e., -1), therefore, all these species undergo disproportionation reactions as shown below :

re
+1 -1 +5 +3 -1 +5 +5 -1 +7

3C10- >2a-+C10j ; 3CIO2 >C1- + 2C10J ; 4C10J >C1- + 3C104

Frl
F
Q. 8.6. Suggest a scheme of classification of the following redox reactions,
(a) Nj (g) + O2 (g) > 2 NO (g) ; (6) Pb(N03)2 (s) > PbO (s) + 2 NO2 (g) + ^ <>2 (ff)
(c) NaH is) + H2O (/) > NaOH (aq) + H2 (g)
ou
or
(d) 2 NO2 (g) + 2 OH- (aq) > NOj (aq) + NO3 (aq) + H2O (/)

kfs
0 0 +2-2

Ans. (a) N2 (g) + ©2 (g) ^ 2 N O (g)


In this reaction, the compound nitric oxide is formed by combination of elemental substances like nitrogen
oo
and oxygen. Therefore, it is a combination reaction. Since in this reaction, the oxidation number of
nitrogen increases from 0 in N2 to + 2 in NO and that of oxygen decreases from 0 in O2 gas to - 2, in NO
Y
therefore, it is a combination redox reaction.
B

+2 +5 -2 ^ +2 -2 +4-2 1 0
ib)Ph(NO^)2is) > Pb0(s) + 2N02(g) + -02(g)
re

In this reaction, lead nitrate decomposes to form three products, viz., lead oxide, nitrogen dioxide and
oxygen. Therefore, it is a decomposition reaction. Since in this reaction, the oxidation number of nitrogen
oYu

decreases from + 5 in lead nitrate to + 4 in NO2 and that of oxygen increases from - 2 in lead nitrate to
ad

0 in O2, therefore, it is a decomposition redox reaction.


+1 -1 +1 -2 +1 -2+1 0

(c) NaH(s) + H2 0 (/) NaOH(a^) + H2 (g)


d

In this reaction, hydrogen of water has been displaced by hydride ion to form dihydrogen gas. Therefore,
it is a displacement reaction. Since in this reaction, the oxidation number of hydrogen increases from
in
Re

- 1 in hydride ion to 0 in dihydrogen gas and that of hydrogen decreases from + 1 in water to 0 in
dihydrogen, therefore, it is a displacement redox reaction.
F

I Reduced
1 +5
+4 +3

(d) 2N02(g) + 2 0H-(fl^) ► NO2 (aq) + NO3 (aq) + H2O (/)


Oxidised
t
This is a disproportionation reaction since here the oxidation state of nitrogen decreases from + 4 in NO2
to + 3 in NO2 ion, as well as increases from + 4 in NO2 to + 5 in NOj ion.
Q. 8.7. Why do the following reactions proceed differently ?
Pb3©4 + 8 HCl ^3PbCl2 + Cl2 + 4H20
and Pb304 + 4HN03 ^ 2 Pb(N03)2 + Pb©2 + 2 H2O
REDOX REACTIONS 8/81

Ans. Pb304 is actually a stoichiometric mixture of 2 moles of PbO and one mole of Pb02, i.e., 2 Pb0.Pb02-
In Pb02, lead is present in + 4 oxidation state, whereas in PbO, lead is present in + 2 oxidation state.
Since + 2 oxidation state of Pb is more stable, therefore, Pb02 acts as an oxidant (oxidising agent) and
hence oxidises Cl~ ions of HCl into CI2. Furthermore, PbO is a basic oxide and hence reacts with HCl to
form PbCl2 and H2O. Thus, the reaction of Pb304 with HCl can be split into two reactions, namely, acid-
base and redox reaction as shown below :
2PbO+ 4 HCl ->2PbCl2 + 2H20 ...(acid-base reaction)
Pb02 + 4HC1 ^PbCl2+Cl2 + 2H20 ...(redox reaction)

2 PbO + Pb02 + 8 HCl » 3 PbCl2 + CI2 + 4 H2O ...(overall reaction)


or
Pb304 -I- 8 HCl »3PbCl2 + Cl2 + 4H20
Since HNO3 is an oxidising agent, and in Pb02, Pb is already present in its highest oxidation state of

w
+ 4, therefore, no redox reaction occurs between Pb02 and HNO3. However, acid-base reaction between
PbO and HNO3 occurs as follows :

F lo
2 PbO -I- 4 HNO3 > 2 Pb(N03>2 + 2 H2O
The overall reaction of Pb304 with HNO3 can then be written as
2 PbO + Pb02 + 4 HNO3 > 2 Pb(N03>2 + Pb02 + 2 H2O
or
Pb304 -I- 4 HNO3 > 2 Pb(N03>2 + Pb02 + 2 H2O
Thus, it is the passive nature of PbO2 against HNO^ that makes the reaction of PbjO^ with HNO^

e
Fre
different fiom that with HCl.
Q. 8.8. Write the net ionic equation for the reaction of potassium dichromate (VI), K2Cr207 with sodium
sulphite, Na2S03, in acid solution to give chromium (HI) ion and sulphate ion.

for
Ans. Step 1. Write the skeleton equation for the given reaction
CtfO^ (aq) + SO| (aq) Cr3+ (a^)-i-S02- (aq) .(/)
r
Step 2. Find out the elements which undergo a change in oxidation number (O.N.).
You
O.N. decreases by 3 per Cr atom
oks
+6 -2 +4-2
1
+3 +6-2
...(&●)
Cr2 O7” (aq) + S Of" (aq) ► Cr^'" (aq) + S Of(aq)
eBo

O.N. increases by 2 per S atom ^


Here, O.N. of Cr decreases from + 6 in Cr20^“ to + 3 in Cr^'*’ while that of S increases from + 4 in
our
ad

SO3 to + 6 in SO4
Step 3. Find out the total increase and decrease in O.N.
Since there are two Cr atoms on L.H.S. and only one on R.H.S., therefore, multiply Cr^'*' on R.H.S. of
Eq. (0 by 2 and thus the total decrease in O.N. of Cr is 2 x 3 = 6.
dY

Further, since there is only one S atom on either side of Eq. («), therefore, total increase in O.N. of S is 2.
Re

Step 4. Balance increase/decrease in O.N.

is 6, therefore, multiply SO^” on L.H.S. and SO^“ on


Fin

Since the total increase in O.N. is 2 and decrease

R.H.S. of Eq. (ii) by 3. Combining steps 2 and 3, we have,


Cr207 (aq) + 3 SO|~ (aq) > 2 Cr^^ (aq) + 3 SOj" (aq) .(Hi)
Step 5. Balance all atoms other than H and O.
Not needed, since both Cr and S atoms are already balanced.
Step 6. Balance O atoms by adding H2O molecules.
Since there are seven O atoms in Cr207~ and nine in 3 SOj” on L.H.S. and only 12 on the R.H.S. of Eq.
(Hi), therefore, add 4 molecules of H2O to R.H.S. of Eq. (Hi), we have,
Cr20?- (aq) + 3 SOf" (aq) > 2 Cr^^ (aq) + 3 SOj" (aq) + 4 H2O (/) ...(iv)
8/82 7>fuidee^ 4- New Course Chemistry (XI) ES[

Step 7. Balance H atoms by adding ions since the reaction occurs in the acidic medium.
Since there are 8 H on R.H.S. and none on the L.H.S., therefore, add 8 H'*' to the L.H.S. of Eq. (fv), we have,

Cr20^- {aq) + 3 SO^" {aq) + 8 H+ {aq) > 2 Cr^^ {aq) + 3 SOj" {aq) + 4 H2O (/) (V)
Thus, Eq. (v) represents the correct balanced equation.
Q. 8.9. Permanganate ion reacts with bromide ion in basic medium to give manganese dioxide and bromate
ion. Write the balanced chemical equation for the reaction.
Ans. Step 1. Write the skeletal equation. The skeletal equation for the given reaction is :

MnO^ {aq) + Br" {aq) > Mn02 (5) + BrOj {aq)


Step 2. Find out the elements which undergo a change in oxidation number (O.N.)

w
O.N. increases by 6 per Br atom

+7 -1 +4
1
+5

F lo
Mn04 {aq) +
Br {aq) ► Mn02 {s) + B1O3 {aq) ...(I)
O.N. decreases by 3 per Mn atom

Here, O.N. of Br increases from -1 in Br“ to +5 in BrOj , therefore, Br acts as reductant. Further, O.N.

e
Fre
of Mn decreases from +7 in MnO^ to +4 in Mn02, therefore, MnO^ acts as oxidant.
Step 3. Find out total increase/decrease in O.N.

for
Since there is only one Br atom on either side, therefore, total increase in O.N. ofBr is 6. Further, since
there is only one Mn atom on either side, therefore, total decrease in O.N. of Mn is 3.
Step 4. Balance increase/decrease in O.N.
r
Since the total increase in O.N. is 6 and decrease in O.N. is 3, therefore, multiply MnO^ by 2. Combining
You
ks
steps 2 and 3, we have.
o
eBo

2Mn04 (fl^) + Br~(fl^) — ^Mn02(5)+ BrOj {aq) m

Step 5. Balance all atoms other than O and H.


To balance Mn on either side of Eq. (li), multiply Mn02 by 2, we have,
ad
our

2Mn04 {aq) + Bt~ {aq) ^ 2 Mn02 (5) + BrOj {aq) (i«)

Step 6. Balance O atoms by adding H2O molecules. Since there are 8 oxygen atoms on L.H.S. of Eq. (iii)
and only 7 on the R.H.S., therefore, add one H2O to the R.H.S. of Eq. {Hi), we have.
2Mn04 {aq) + Br~ {aq) ^ 2Mn02 {s) + BrOj {aq) + H2O (/) m
Re
dY

Step 7. Balance H atoms by adding H2O and OIF ions since the reaction occurs in basic medium. Since
there are two H atoms on R.H.S. and none on L.H.S. of Eq. (/v), therefore, add 2H2O to L.H.S. and
Fin

20H" to R.H.S. of Eq. (iV), we have,

2Mn04 (a?) + Br" {aq) + 2 H2O (/) ^ 2Mn02 {s) + BrOj {aq) + H2O (/) + 20H" {aq)
or
2Mn04 {aq) + Br" {aq) + H2O (/) ^ 2Mn02 (j) + BrOj {aq) + 2 OH" {aq) (V)
Thus, Eq. (v) represents the correct balanced equation.
Q. 8.10. Permanganate (YH) ion, in basic solution oxidises iodide ion I~ to produce molecular iodine (I2)
and manganese (IV) oxide (Mn02). Write a balanced ionic equation to represent this redox reaction.
Ans. Step 1. Write the skeletal equation for the given reaction.

Mn04 .iaq) + l {aq) ■> Mn02 {s) +12 (5) .(/)


REDOX REACTIONS 8/83

Step 2. Write the O.N. of all the elements above their respective symbols.
O.N. decreases by 3 per Mn atom
1
+7 -2 -1 +4 0

Mn04 (aq) + I (aq) ► Mn O2 (.s) + I2 (-s)


O.N. increases by 1 per I atom
t
Step 3. Find out the species which have been oxidised and reduced and split the given skeletal equation
into two half reactions.

Here, O.N. of Mn decreases from + 7 in MnO^ to + 4 in Mn02 and that of I increases from - 1 in 1" ion

w
to 0 in I2 molecule. Therefore, MnO^ acts as the oxidant while 1“ ion acts as the reductant. Thus, the
above skeletal Eq. (/) can be divided into the following two half reaction equations :

F lo
Oxidation half equation : l-(aq) .(«)

Reduction half equation : Mn04 (aq) ■¥ Mn02 (5) .(Hi)

Step 4. To balance oxidation halfEq. (ii).

ee
(a) Balance all atoms other than O and H atoms. Since there are two 1 atoms on the R.H.S. of Eq. (ii) but

Fr
only one on the L.H.S., therefore, multiply I" ion by 2, we have,
21-(aq) ^l2(^) ...(/V)
(b) Balance O.N. by adding electrons.

for
The O.N. of I in I" ion is - 1 while that in I2 is 0. Thus, each 1“ ion loses one electron. Since there are two
r ions on the L.H.S., therefore, add 2 e~ to the R.H.S. of Eq. (/v), we have,
our
21-(aq) 4 I2 (s) + 2 e" .(V)
(c) Balance charge.
oks
Not needed since charge on either side of Eq. (v) is balanced. Thus, Eq. (v) represents the balanced
oxidation half equation.
o
Y
Step 5. Balance the reduction half equation (Hi),
eB

(a) Balance all atoms other than O and H.


Not needed because Mn is already balanced.
(b) Balance O.N. by adding electrons.
our
ad

The O.N. of Mn in Mn04 on the L.H.S. of Eq. (Hi) is + 7 while the O.N. of Mn in Mn02 is + 4 on the
R.H.S. Therefore, add 3 e~ to L.H.S. of Eq. (Hi), we have,
.(vO
Y

MnOj (aq) + 3e~ > Mn02 (s)


Re

(c) Balance charge by adding OH~ ions since the reaction occurs in the basic medium.
nd

The total charge on L.H.S. of Eq. (vi) is - 4 while on the R.H.S., it is zero. Therefore, add 4 OH“ to the
R.H.S. of Eq. (vO, we have.
Fi

Mn04 (aq) + 3 e ^Mn02(j) + 4 0H-(ag) (viO

(d) Balance O atoms.


The R.H.S. of Eq. (vH) has six O atoms while the L.H.S. has only four. Therefore, add 2 H2O to the
L.H.S. of Eq. (vH), we have,

MnOj (aq) + 2 H2O (/) + 3 ^ Mn02 (5) + 4 OH- (aq) .(viu)

By doing so, H atoms are automatically balanced, therefore, Eq. (viii) represents the balanced reduction
h^f equation.
Step 6. To balance the electrons lost in Eq. (v) and gained in Eq. (viii), multiply Eq. (v) by 3 and Eq. (viii)
by 2 and add, we have.
8/84 Course Chemistry rxnPPTWtn

6I-(a^) ■> 3 I2 (j) + 6 e


2Mn04 {aq) + 4 H2O (/) + 6 e~ ^ 2 Mn02 (i) + 8 OH" (aq)

2 MnO^ (aq) + 6l~ (aq) + 4 H2O (/) — ^ 2 Mn02 (s) + 3 I2 (i) + 8 OH" (aq) ■Ox)
This represents the final balanced redox equation.
Step 7. Verification. Total charge on L.H.S. of Eq. (lx) = 2 (- l) + 6(-l)=-8

ow
Total charge on R.H.S. of Eq. (ix) = 8 (- 1) = - 8
Since the magnitude of charge on either side of Eq. (ix) is equal, therefore, Eq. (ix) represents the correct
balanced redox equation.
NCERT EXERCISE

Q. 8.1. Assign oxidation number to the underlined elements in each of the following species :

e
(a) NaHjPO^ (b) NaHS04 (c) (rf) K2M11O4 (e) CaOj (f) NaBH4 (g) H2S2O7

re
rFl
(h) KAI (804)2-12 H2O

F
Ans. (a) Let the oxidation number of P be x. Writing the oxidation number of each atom above its symbol, we
+ 1 +1 j -2
have, Na H2 P O4
Sum of oxidation numbers of various atoms in NaH2P04 = 1 (+ 1) + 2 (+ 1) + 1 (a:) + 4 (- 2) = x - 5

r
ou
But the sum of oxidation numbers of various atoms in NaH2P04 (neutral) is zero (Rule 7)

fo
X - 5 = 0 or X = + 5

ks
Thus, the oxidation number of P in NaH2PO^ = 5.
+1 +1 .T -2

(b) Na H SO4 1 (+ I)+ 1 (+ I)+x + 4(-2) = 0orx= + 6


oo
Thus, the oxidation number ofS in NaHSO^ = + 6.
+I -2
Y
(c) H4 P, O-j .●. 4 {+ 1) + 2 (x) + 7 (- 2) = 0 or X = + 5
B

Thus, the oxidation number of P in H^P20-j = + 5.


+1 -●>
re

(i/) K2 Mn O4 2(+1)+l(x) + 4(-2) = 0orx = + 6


Thus, the oxidation number of Mn in K-,MnO^ = + 7.
ou
Y

(e) Let the oxidation number of O be x. Since Ca is an alkaline earth metal, therefore, its oxidation
ad

number is + 2. Thus,
+2 JT

Ca O2 + 2 + 2 (x) = 0 or X = - 1
d

Thus, oxidation number of O in CaO-y = - 1.


(/) In NaBH4, H is present as hydride ion. Therefore, its oxidation number is - 1. Thus.
in
Re

+1 X -1

Na B H4 1 (+ l)+x + 4(- l) = 0orx = + 3


F

Thus, the oxidation number of B in NaBH^ = + 3.


+ 1 .r -2
(g) H2 S2 O7 2 (+ 1) + 2 (x) + 7 (- 2) = 0 or X = + 6

Thus, the oxidation number of S in //2S2O7 = + 6.


+I +3 .r -2 +1 -2

(/i) K A1 (804)2-12 (H2 O) or + 1+3 + 2x+8(-2)+12(2x 1-2)orx = + 6


Alternatively, since H2O is a neutral molecule, therefore, sum of oxidation numbers of all the atoms in
H2O may be taken as zero. As such water molecules may be ignored white computing the oxidation
number of S.
+]+3+2x-16 = 0orx = + 6

Thus, the oxidation number ofS in KAI (SO^)2- 12 H2O = + 6.


REDOX REACTIONS 8/85

Q. 8.2. What are the oxidation numbers of the underlined elements in each of the following and how do
you rationalize your results ?
(a) KI3 ib) (c) ^04 id) OHaCHjOH (e) CH3OOOH
Ans. (a) In KI3, since the oxidation number of K is + 1, therefore, the average oxidation number of iodine
= - 1/3. But the oxidation number cannot be fractional. Therefore, we must consider its structure,
K'*' [I—<-1]“. Here, a coordinate bond is formed between I2 molecule and 1“ ion. The oxidation number
of two iodine atoms forming the I2 molecule is zero while that of iodine forming the coordinate bond is
- 1. Thus, the O.N. of three I atoms, atoms in KI3 are 0,0 and - 1 respectively.

low
+1 jc -2

(b) By conventional method. O.N. of S in H2S4O6 = H2 S4


or 2 (+ 1) + 4 X + 6 (- 2) = 0 or X = + 2-5 {wrong)
But it is wrong because all the four S atoms cannot be in the same oxidation state.
By chemical bonding method. The structure of H2S40g is shown below :
O O

ee
+5 II 0 0 II +5
H—O—S—S—S—S—OH

rF
Fr
O O

The O.N. of each of the S-atoms linked with each other in the middle is zero while that of each of the
remaining two S-atoms is + 5. Refer to page 8/21.

for
X -2

(c) By conventional method. O.N. of Fe in Fej O4 or 3x + 4 (- 2) = 0 or x = 8/3


ou
+2 -2 +3 -2

By stoichiometry. Fe304 s Fe O .Fe2 O3 O.N. of Fe in Fe304 + 2 and + 3


s
ok
X +1 -2
(d) By conventional method. O.N. of C in CH3CH2OH = C2 H^ O
or2x + 6(+ 1)+ 1 (-2) = 0orx = -2
o
Y
By chemical bonding, C2 is attached to three H-atoms (less electronegative than carbon)
eB

H H
2I ll
H—C—C—OH
and one CH2OH group (more electronegative than carbon), therefore.
I I
r

H H
ou
ad

O.N. of €2= 3 (+ 1) +x + 1 (- 1) = 0 orx = - 2


Cj is, however, attached to one OH (O.N. = -1) and one CH3 (O.N. = +1) group,
Y

therefore, O.N. of Cj = + 1 + 2 (+ 1) + x + 1 (-1) = 0 or x = - 2


;c +1 -2
nd
Re

(e) By conventional method. CH3COOH = C2 H4 O2 or 2 x + 4 - 4 = 0 or x = 0


By chemical bonding method, C2 is attached to three H-atoms (less electronegative than carbon)
Fi

H O
2I ill
and one —COOH group (more electronegative than carbon). H—C—C—OH

therefore, O.N. of C2 = 3 (+ 1) + x + 1 (-1) = 0 or x = - 2


Cj is, however, attached to one oxygen atom by a double bond, one OH (O.N. = - 1) and one CH3(O.N.
= + 1) group, therefore, O.N. of C| = + 1 + x + 1 (-2) + 1 (-1) = 0 or x = + 2
Q. 8.3. Try all possible approaches to justify that the following reactions are redox reactions,
(a) CuO (s) + Hj (g) > Cu (s) + HjO (g) (b) Fc203 (s) + 3 CO (g) » 2 Fe (s) + 3 CO2 (g)
(c) 4 BCI3 (g) + 3 L1A1H4 (s) > 2 B2H^ (g) + 3 LiCl (s) + 3 AICI3 (s)
(d)2K(s) + F2(g) >2K+J^(s) (e)4NH3(g) + 5O2(g) >4NO(g) + 6H2O(g)
8/86
7^ncuCee^’<i New Course Chemistry (XI)|S3I9D
+2 -2 0 0

Ans. (a) CuO (s) + H2 (g) > Cu (5) + H2 0(g)


Here, O is removed from CuO, therefore, it is reduced to Cu while O is added to H2 to form H2O,
therefore, it is oxidised. Further, O.N. of Cu decreases from + 2 in CuO to 0 in Cu but that of H increases
from 0 in H2 to + 1 in H2O. Therefore, CuO is reduced to Cu but H2 is oxidised to H2O. Thus, this is a
redox reaction.
+3 -2 +2 0 44

{b) Fc2 O3 is) + 3 CO {g) > 2 Fe (5) + 3 CO2 (^)


Here, O.N. of Fe decreases from +3 is Fe203 to 0 in Fe while that of C increases from + 2 in CO to + 4
in CO2. Further, oxygen is removed from Fe203 and added to CO, therefore, Fe203 is reduced while CO
is oxidised. Thus, this is a redox reaction.

w
+3 -1 +1 +3 -I -3 +1 +1 -1 +3 -1

(c) 4 B CI3 (g) + U A1H4 (j) > 2B2 Hg (g) + 3 Li Cl (s) + 3 A1Q3 (s)
Here, O.N. of B decreases from + 3 in BrCl3 to - 3 in B2Hg while that of H increases from -1 in LLAIH4

F lo
to + 1 in B2Hg. Therefore, BCI3 is reduced while LLAIH4 is oxidised. Further, H is added to BCI3 but is
removed from LiAlH4, therefore, BCI3 is reduced while LiAlH4 is oxidised. Thus, it is a redox reaction.
(d)2K(j) + F2(g) >2K+F-(5)
Here, each K atom has lost one electron to form K'*' while F2 has gained two electrons to form two F~

e
ions. Therefore, K is oxidised while F2 is reduced. Thus, it is a redox reaction,

Fre
-3 +1 0 +2-2

(e) 4 N H3 («) + 5 O2 («) > 4 N d (g) + 6 Hj O (s)


Here, O.N. of N increases from -3 is NH3 to +2 is NO while that O decreases from 0 is O2 to -2 in NO

for
or H2O. Therefore, NH3 is oxidised while O2 is reduced.
Further H has been removed from NH3 but added to O2. Therefore, NH3 has been oxidised while O2 is
r
reduced. Thus, this is a redox reaction.
You
Q. 8.4. Fluorine reacts vrith ice and results in the change : H2O (s) + F2 (g) » HF (g) + HOF (g)
s
Justify that this reaction is a redox reaction.
ook

+1 -2 0 +1 -1 +1 -2 +1
Ans. Writing the O.N. of each atom above its symbol, we have, H2 O + F2 4 H F + H O F
eB

Here, the O.N. of F decreases from 0 in F2 to - 1 in HF and increases from 0 in F2 to + 1 in HOF.


Therefore, F2 is both reduced as well as oxidised. Thus, it is a redox reaction and more specifically, it is
a disproportionation reaction.
our
ad

Q. 8.5. Calculate the oxidation number of sulphur, chromium and nitrogen in H2SO5, C1O5 and NO3.
Suggest structures of these three compounds. Count for the fallacy.
Ans. (i) O.N. of S is H2SO5.
By conventional method, the O.N. of S in H2SO5 is : 2 (+1) + .x + 5 (-2) = 0 or x = + 8
dY

This is impossiblebecausethe maximumO.N. of S cannotbe more than six since it has only six electrons
Re

in the valence shell. This fallacy is overcome if we calculate the O.N. of S by chemical bonding method.
Fin

The structure of H2SO5 is H—O— S —O—O—H


O

2x(+l) + j: + 2(-l) + 3x(-2)=0 or x = +6


(for H) (for S) for (O—O) (for other O atoms)
(ii) O.N of Cr in C1O5
O
According to conventional method, O.N. of Cr is : x + 5 (- 2) = 0 or x = + 10. This is
.0
impossible because the maximum O.N. of Cr cannot be more than six since it has
:cr:
3 d^ 4 outer orbital configuration. This fallacy is removed if we calculate O.N. of Cr by O' o
chemical bonding method. The structure of C1O5 is

k
REDOX REACTIONS 8/87

From this structure, the O.N. of Cr can be calculated as follows :


x + 4(- 1) + l(-2) = 0 or jc = + 6

(for 0-0) (for = O)

(iii)O.N.ofNis NOj
According to conventional method, O.N. of N is NOJ = .t + 3 (- 2) = - 1 or =+5
O

According to chemical bonding method, “0—N.^ O


x+ 1(-1) + K-2) + l(-2) = 0 or x + 5

w
(for O") (for = O) for O

Thus, there is no fallacy about the O.N. of N in NOj whether one calculates by conventional method or

F lo
by chemical bonding method.
Q. 8.6. Write formulas for the following compounds : (a) Mercury (II) chloride, (b) Nickel (II) sulphate,
(c) Tin (TV) oxide, (d) Thallium(I) sulphate,(e) Iron (HI) sulphate, (f) Chromium (HI) oxide.
Ans. (a) Hg(n)Cl2, (b) Ni(H)S04, (c) Sn(IV)02, {d) Th2(I)S04, (e) Fe2(m)(S04)3, (/) Cr2(m)03.

ee
Q. 8.7. Suggest a list of substances where carbon can exhibit oxidation states from -4 to +4 and nitrogen

Fr
from -3 to +5.
T-r?

Ans. Compound O of Carbon CpmpQUiid

for
CH4 -4 NH3 -3

NH2— NH2 -2
CH3CH3 -3
ur
CH2 = CH2orCH3Cl -2 NH = NH -1
CH = CH - 1 N = N 0
oks
CH2CI2 or CgHj20g 0 N2O + 1
Yo
C2CI2 or CgClg + 1 NO + 2
o

CO or CHCI3 + 2 N2O3 + 3
eB

C2Cl6or(COOH)2 + 3 N2O4 + 4

CO2 or CCI4 + 4 N2O5 + 5

Q. 8.8. While sulphur dioxide and hydrogen peroxide can act as an oxidising as well as reducing agents in
our
ad

their reactions, ozone and nitric acid act only as oxidants. Why ?
Ans. (0 In SO2, O.N. of S is 44. In principle, S can have a minimum O.N. of -2 and maximum of 4-6.
Therefore, S in SO2 can either decrease or increase its O.N. and hence can act both as an oxidising as
well as a reducing agent.
dY

(//) In H2O2, the O.N. of O is -1. In principle, O can have a minimum O.N. of -2 and maximum of zero
Re

(+1 is possible in O2F2 and 4 2 in OF2). Therefore, O in H2O2 can either decrease its O.N. from -1 to
-2 or can increase its O.N. from -1 to zero. Therefore, H2O2 acts both as an oxidising as well as a
Fin

reducing agent.
(Hi) In O3, the O.N. of O is zero. It can only decrease its O.N. from zero to -1 or -2, but cannot increase
to 4-2. Therefore, O3 acts only as an oxidant.
(iv) In HNO3, O.N. of N is 45 which is maximum. Therefore, it can only decrease its O.N. and hence it
acts as an oxidant only.
Q. 8.9. Consider the reactions : (a) 6 CO2 (g) 6 H2O (/) » C^Hi20g (s) + 6 O2 (?)
(b) O3 (g) + H2O2 (0 » H2O (/) 4 2 O2 (?)
Why it is more appropriateto write these reactionsas :
(a) 6 CO2 (?) 4-12 H2O (/) > C6H12O6 (s) + 6 H2O (/) 4- 6 O2 (?)
(b) O3 (?) 4- H2O2 (/) > H2O (/) 4- O2 (?) 4- O2 (?)
Also suggesta techniqueto investigatethe path of the above (a) and (b) redox reactions.

i
8/88
New Course Chemistry (XI)BZaI9D

Ans. (a) Although the mechanism of photosynthesis is very complex but broadly speaking it may be visualized
to occur in two steps. In the first step, H2O decomposes to give H2 and O2 in presence of chlorophyll and
the H2 thus produced reduces CO2 to CgHj20g in the second step. During the second step, some H2O
molecules are also produced as shown below :
12 H2O (/) > 12 H2 (g) + 6 O2 (g) .(0
6 CO2 (8) + 12 H2 (g) > Cg H12O6 (s) + 6 H2O (/) .(*0

6 CO2 (g) + 12 H2O (/) > C6H12O6 (5) + 6 H2O (/) + 6 O2 (g) .(*«)
Therefore, it is more appropriate to write the equation for photosynthesis as (t/i) because it emphasises
that 12 H2O are used per molecule of carbohydrate formed and 6 H2O are produced during the process,
(b) The purpose of writing O2 two times suggests that O2 is being obtained from each of the two reactants.
O3 (^) >02 (g) + O (g)

w
H2O2 + O (^) > H2O (/) + O2 (g)
03(g)+ H202(/)

F lo
»H20(/) + 02(g) +02(g)
The path of reactions(a) and (b) can be determinedby using H20^* or D2O in reaction (a) or by using
H20^^ or O^* in reaction (b).
Q. 8.10. The compound AgF2 is unstable. However, if formed, the compound acts as a very strong oxidising

e
agent Why ?

Fre
Ans. In Agp2, oxidation state of Ag is +2 which is highly unstable. Therefore, it quickly accepts an electron to
form the more stable +1 oxidation state. Ag2+ + e~ > Ag+
Therefore, Agp2, if formed, will act as a strong oxidising agent.

for
Q. 8.11. Whenever a reaction between an oxidising agent and a reducing agent is carried out, a compound
of lower oxidation state is formed if the reducing agent is in excess and a compound of higher
oxidation state is formed if oxidising agent is in excess. Justify this statement giving three
our
illustrations.

Ans. (1) C is a reducing agent while O2 is an oxidising agent. If excess of carbon is burnt in a limited supply
s
ook

of O2, CO is formed in which the oxidation state of C is +2. If, however, excess of O2 is used, the
initially formed CO gets oxidised to CO2 in which oxidation state of C is + 4.
eB
Y
+2 +4
2C(s) + 02(g) ^ 2CO(g) ; C(s) + 02(g)- ^ CO2 (g)
(Excess) (£xceM)
(ii) P4 is a reducing agent while CI2 is an oxidising agent. When excess of P4 is used, PCI3 is formed in
our
ad

which the oxidation state of P is +3. If, however,excess of CI2 is used, the initially formed PCI3 reacts
further to form PCI5 in which the oxidation state of P is +5
+3 +5
P4(^)-f-6Cl2(g) ^ 4 PQ3 ; P^(s) + IOCI2 ^ 4PQ5
dY

(Excess) (Excess)
Re

(Hi) Na is a reducing agent while O2 is an oxidising agent. When excess of Na is used, sodium oxide is
formed in which the oxidation state of O is -2. If, however, excess of O2 is used, Na202 is formed in
Fin

which the oxidation state of O is -1 which is higher than -2.


-2 -1
4 Na (s) + O2 (g) ■> Na20 (s) ; 2Na(5) +202(g) ■¥
Na202 (^)
(Excess) (Excess)
Q. 8.12. How do you count for the following observations ?
(a) Though alkaline potassium permanganate and acidic potassium permanganate both are used
as oxidants, yet in the manufacture of benzoic acid from toluene we use alcoholic potassium
permanganate as an oxidant Why ? Write a balanced redox equation for the reaction,
(h) When concentrated sulphuric acid is added to an inorganic mixture containing chloride, we
get pungent smelling gas HCl, but if the mixture contains bromide then we get red vapour of
bromine. Why ?
REDOX REACTIONS 8/89

Ans. (a) Toluene can be oxidised to benzoic acid in acidic, basic and neutral media according to the following
redox equations :
(i) Acidic medium

Mn04 iaq) + SH*iaq) + 5e- >Mn2+(a^) + 4H20(/)]x6


CH3 COOH

Q] (/) + 2H20(/) ► [Qj (^) + 6 H"'(fl^) + 6 e"] X 5

low
CH3 COOH

5
A (0 + 6 MnO^iag) + 18 (aq) > 5 +6Mn2+(a^)+14H20(/)

e
Toluene Benzoic acid
(ii) Basic and neutral media

re
rF
Mn04 (aq) + 2 H2O (/) + 3 c“ ^Mn02(5) + 4 0H-(a^)]x2

F
CH3 COO“

or
(/) + 7 0H”(a^) ► (fl^) + 5 H2O (0 + 6 e"
ou
f
CH3 COO- ks
A JL
oo
+ 2 Mn04 (aq)
► rQj +2Mn02(5) + H20(/) + 0H-(fl^)
Y
B

Tolueme Benzoate ion

In the laboratory, benzoic acid is usually prepared by alkaline KMn04 oxidation of toluene. However, in
re

industry alcoholic KMn04 is preferred to acidic or alkaline KMn04 because of the following reasons :
(0 The cost of adding an acid or the base is avoided because in the neutral medium, the base (OH~ ions)
are produced in the reaction itself.
u
ad

(ii) Since reactions occur faster in homogeneous medium than in heterogeneous medium, therefore,
Yo

alcohol helps in mixing the two reactants, i.e., KMn04 (due to its polar nature) and toluene (because of
its being an organic compound).
(b) When cone. H2SO4 is added to an inorganic mixture containing chloride, a pungent smelling gas HCl
d

is produced because a stronger acid displaces a weaker acid from its salt.
Re

2NaCl + 2H2SO4 - ■> 2NaHS04 + 2 HCl


in

Stronger acid Weaker acid

2 HCl + H2SO4 ^f-> CI2 + SO2 + 2H2O


F

Since HCl is a very weak reducing agent, it cannot reduce H2SO4 to SO2 and hence HCl is not oxidised
to CI2
However, when the mixture contains bromide ion, the initially produced HBr being a stronger reducing
agent than HCl, reduces H2SO4 to SO2 and is itself oxidised to produce red vapour of Br2.
2 NaBr + 2 H2SO4 ^ 2 NaHS04 + 2 HBr
2 HBr + H2SO4 ■> Br2 + SO2 + 2 H2O
Q. 8.13. Identify the substance oxidised, reduced, oxidising agent and reducing agent for each of the following
reactions,

(a) 2 AgBr (s) + CgHg02 (aq) > 2 Ag (s) + 2 HBr (aq) + C5H4O2 (aq)
8/90 New Course Chemistry (XQESsiaD

(b) HCHO a) + 2 [Ag(NH3)2l+ (aq) + 3 OH" (aq) ■>

2 Ag (s) + HCOO- (aq) + 4 NHj (aq) + 2 HjO (/)


(c) HCHO (I) + 2 Cu2+ (aq) + 5 OH" (aq) > CujO (s) + HCOO" (aq) + 3 HjO (I)
(d) N2H4 0) + 2 H2O2 G) > N2 (g) + 4 H2O 0)
(e) Pb (s) + Pb02 (s) + 2 H2SO4 (aq) » 2 PbS04 (s) + 2 H2O 0)
Ans. Substance oxidised Substance reduced Oxidising agent Reducing agent
fa) AgBr(5) AgBr(5)

w
(b) HCHO (aq) [Ag (NH3)2r [Ag (NH3)2l+ HCHO (aq)
(c) HCHO (aq) Cu2+ (aq) Cu^'*' {aq) HCHO (aq)
(d) N2H4(/) H202(/) H202(/) N2H4(0
ie) Pb(5) Pb02 (s) Pb02 is) Pb(5)

o
Q. 8.14. Consider the reactions : 2 S203~ (aq) +12 (s) ^ S4OJ- (aq) + 2l-(aq)

e
S2OI- (aq) + 2 Br2 (/) + 5 H2O (1) > 2 SOj" (aq) + 4 Br" (aq) + 10 H+ (aq)

re
rFl
Why does the same reductant, thiosulphate react differently with iodine and bromine ?

F
Ans. The average O.N. of S in 820^“ is + 2 while in 840!“ it is + 2-5. The O.N. of S in 80^“ is +6. Since
Bf2 is a stronger oxidising agent that I2, it oxidises S of 820|“ to a higher oxidation state of +6 and

r
hence forms 804“ ion. I2, however, being a weaker oxidising agent oxidises S of 8203“ ion to a lower
ou
fo
oxidation of +2-5 in 840^“ ion. It is because of this reason that thiosulphate reacts differently with Br2
and I2.
ks
Q. 8.15. Justify giving reactions that among halogens, fluorine is the best oxidant and among hydrohalic
compounds, hydriodic acid is the best reductant
oo
Ans. Halogens have a strong tendency to accept electrons. Therefore, they are strong oxidising agents. Their
relative oxidising power is, however, measured in terms of their electrode potentials. Since the electrode
Y
potentials of halogens decrease in the order: F2 (+ 2-87 V) > CI2 (+ 1-36 V) > Br2 (+ 1-09 V) > I2 (+ 0-54
eB

V), therefore,their oxidisingpower decreasesin the same order.


This is evident from the observation that F2 oxidises Cl“ to CI2, Br~ to Br2,1” to I2 ; CI2 oxidises Br~ to
Br2 and I" to I2 but not F" to F2. Br2, however, oxidises I" to I2 but not F" to F2, and Cl“ to CI2.
ur

F2 (g) + 2 Cl- (aq) >2F- (aq) + CI2 (g); F2 (g) + 2 Br" (aq) > 2 F~ (aq) + BT2 (I)
F2 (g) + 21“ (aq) > 2 F“ (aq) +12 (5) ; CI2 (g) + 2 Br“ (aq) > 2 Cl" (aq) + BT2 (/)
ad
Yo

CI2 (g) + 21- (aq) > 2 Cl" (aq) +12 (s) and Br2 (0 + 21" > 2 Br~ (aq) +12 (5)
Thus, F2 is the best oxidant
Conversely,halide ions havea tendencyto lose electronsand hence can act as reducing agents. Since the
d

elecUx)de potentials of halide ions decreases in the order: I" (- 0-54 V) > Br" (- 1 -09 V) > Cl" (- 1 -36 V)
> F" (- 2-87 V), therefore, the reducing power of the halide ions or their corresponding hydrohalic acids
Re
in

decreases in the same order: HI > HBr > HCl > HF. Thus, hydriodic acid is the best reductant. This is
supported by the following reactions. For example, HI and HBr reduce H2SO4 to SO2 while HCl and HF
F

do not.

2HBr + H2S04 > Br2 + SO2 + 2 H2O ; 2HI + H2SO4 >I2+S02 + 2H20
Further 1“ reduces Cu^"*" to Cu'*’ but Br“ does not.
2 Cu^'*' (aq) +41“ (aq) > CU2I2 (^) +12 (●*) ; Cu^'*’ (aq) + 2 Br > No reaction.
Thus, HI is a stronger reductant than HB.r
Further among HCl and HF, HCl is a stronger reducing agent than HF because HCl reduces Mn02 to
Mn^"*" but HF does not.
Mn02 (f) + 4 HCl (aq) »MnCl2 (aq) + CI2 (g) + 2 H2O
Mn02 (s) + 4 HF (Z) X > F2 formed.
Thus, the reducing character of hydrohalic acids decreases in the order : HI > HBr > HCl > HF
REDOX REACTIONS 8/91

Q. 8.16. Why does the following reaction occur ?


XeOj- {aq) + 2 F" {aq) + 6 H+ {aq) > XeOj (s) + Fj (g) + 3 HjO (/)
What conclusion about the compound Na4Xe05 (of which XeO^ is a part) can be drawn from
the reaction ?
+8 -1 46 0

Ans. XeOj- (a?) + 2 («?) + 6 H+ {aq) > Xe03 (j) + Fj {q) + 3 H2O (/)
Here, O.N. of Xe decreases from +8 in XeOg“ to +6 in Xe03 while that of F increases from -I in F~ to
0 in F2. Therefore, XeO^“ is erduced while F" is oxidised. This eraction occurs because Na2XeO^“
(or XeOj“) is a stronger oxidising agent than F2.
Q. 8.17. Consider the reactions :

w
(a) H3PO2 {aq) + 4 AgNOj {aq) + 2 H2O (/) > H3PO4 {aq) + 4 Ag (s) + 4 HNO3 {aq)

F lo
(b) H3PO2 {aq) + 2 C11SO4 {aq) + 2 H2O (/) » H3PO4 {aq) + 2 Cu (s) + H2SO4 {aq)
(c) CgHjCHO (0 + 2 [Ag(NH3)2r iaq) + 3 OH" {aq) +

CgHgCOO- {aq) + 2 Ag (s) + 4 NH3 {aq) + 2 H2O (0


(d) CgHgCHO (0 + 2 Cu2+ {aq) + 5 OH" {aq) —> No change observed
What inference do you draw about the behaviour of Ag'*' and Cu^'*' from these reactions ?

ree
Ans. Reactions {a) and {b) indicate that H3PO2 (hypophosphorous acid) is a reducing agent and thus reduces

F
both AgN03 and CUSO4 to Ag and Cu respectively. Conversely, both AgN03 and CUSO4 act as oxidising

for
agent and thus oxidise H3PO2 to H3PO4 (orthophosphoric acid)
Reaction (c) suggests that [Ag(NH3)2]‘^ oxidises C6H5CHO (benzaldehyde) to C6H5COO" (benzoate
ion) but reaction {d) indicates that Cu^+ ions cannot oxidise C6H5CHO to CgH5COO“. Therefore, from
the above reactions, we conclude that Ag^ ion is a stronger oxidising agent than ion.
r
Q. 8.18. Balance the following redox reactions by ion-electron method.
You
ks
(a) Mn04 {aq) +1" {aq) + Mn02 (5) +12 (s) (in basic medium)
o
eBo

(b) Mn04 {aq) + SOj (g) - -> Mn^'*' {aq) + HSO4 (in acidic solution)
(c)H202(flg) + Fe2+(og)- + Fe^ {aq) + H2O (/) (in acidic solution)
(d) Cr20^- (og) + S02(g) —» Cr^ {aq) + SO^“ {aq) (in acidic solution)
ad

Ans. (a) Refer to Ans. to Q. 8.10, pages 8/79-8/80.


our

{b) Following the general procedure for ion electron method, detailed on page 8/26, the balanced half
reaction equations are :
Oxidation half equation : SO2 (g) + 2 H2O (/) > HSO4 {aq) + 3 H+ {aq) + 2e~ ...(i)
Reduction half equation : MnO^ {aq) + 8 H'*’ {aq) + 5 e~ > Mn^'*’ {aq) + 4 H2O (/)
Re

.(«)
dY

Multiply Eq. (/) by 5 and Eq. (//) by 2 and add, we have,


Fin

2 Mn04 {aq) + 5 SO2 (g) + 2 H2O (0 + H+ {aq) > 2 Mn^+ {aq) + 5 HSO4 {aq)
(c) Oxidation half equation : Fe^'*' {aq) > Fe^'*’ {aq) + e~ .(0
Reduction half equatin’' : H2O2 {aq) + 2 H+ (a^) + 2 e" > 2 H2O (/) .(//)
Multiply Eq. (i) by 2 J add it to Eq. (/i), we have,
H2O2 {aq) + 2 Fe^+ {aq) + 2 H+ {aq) > 2 Fe^^ {aq) + 2 H2O (/)
{d) Following the general procedure for ion electron method detailed on page 8/24, the balanced half
reaction equations are ;
Oxidation half equation: SO2(g) + 2H2O(0 — SO^ {aq) + 4 H'*' {aq) + 2 e .(/)

Reduction half equation : {aq) + 14 {aq) + 6 > 2 Cr3+ {aq) + 7 H2O (0 >(«*)
8/92 "Pn^iuieefr’^ New Course Chemistry (XI)^Z9D

Multiply Eq. (0 by 3 and add it to Eq. (/'O, we have,

CrjO^- (aq) + 3 SO2 (q) + 2 H+ (aq) 2 Cr3+ (aq) + 3 SOj" (aq) + H2O (0
Q. 8.19. Balance the following equations in basic medium by ion electron method and oxidation number
method and identify the oxidising agent and the reducing agent

(a)P4(s) + OH-(og) ^ PH3 (g) + HjPOj (aq) (b) N2H4 (D + CIO5 (aq) ^NO(g) + Cl-(ag)

(c) CI2O7 (g) + H2O2 (aq) ■» ClOj (fl«) + 02(g) + H+


O.N. decreases by 3 per P atom

0 -3 +1 +1 +1 -2

w
Ans. (a) P4(-s) + OH” (aqr) >p H3(g) + H2 P O2

O.N. increases by 1 per P atom

F lo
P4 acts both as an oxidising as well as a reducing agent,
(^dation number method:
Total decrease in O.N. of P4 in PH3 = 3 x 4 = 12
Total increase in O.N. of P4 in H2PO2 =1x4 = 4

e
Fre
Therefore, to balance increase/decreases in O.N. multiply PH3 by 1 and H2PO2 by 3, we have.

^PH3(g)+ 3H2PO2 (aq)

for
?^(s) + OU-(aq)
To balance O atoms, multiply OH“ by 6, we have.
r
?4(s) + 6 0U-(aq) ^PH3(g)+ 3H2PO2 (aq)
You
To balance H atoms, add 3 H2O to L.H.S. and 3 OH” to the R.H.S., we have.
s
ook

P4 (s) + 6 OH- (aq) + 3 H2O (/) ^ PH3 (g) + 3 H2PO2 (aq) + 3 OH" (aq)
eB

or
P4 (s) + 3 OH- (aq) + 3 H2O (/) ^PH3(g)+ 3H2PO2 (aq) .(i)

Thus, Eq. (i) represents the correct balanced equation.


Ion electron method. Split the given redox reaction into two half reactions (U) and (Hi) and balance
our

them as described below :


ad

Oxidation half reaction : P4(^) ^ H2PO2 (aq)


0 +1

Balancing P atoms, we have. P4(5) An^?Ol(aq)


dY
Re

Balance O.N. by adding electrons. Pd(^) ^ 4H2PO2 (fl^) + 4e-


Fin

Balance charge by adding 8 OH- ions, P4 (s) + 8 OH- (aq) 4H2PO2 (aq) + Ae~ ,(tii)
O and H get automatically balanced. Thus, Eq. (Hi) represents the balanced oxidation half reaction.
0 -3
Reduction half reaction : P4 is) > PH3 (g)
Balancing P atoms, we have. P4(^) ^ 4PH3(g)
Balance O.N. by adding electrons. P4(s) + 12e- ■>4PH3(g)
Balance charge by adding 12 OH- ions, P4 (s) + 12 e~ -»4PH3(g)+12 0H-(a^)
Balance O atoms, by adding 12 H2O to L.H.S. of above equation.
P4 (j) + 12 H2O (t) + 12 e~ > 4 PH3 (g) + 12 OH- (aq) .(V)
To cancel out electrons, multiply Eq. (Hi) by 3 and add it to Eq. (v), we have.
REDOX REACTIONS 8/93

4 P4 (^) + 24 OH- {aq) + 12 H2O (/) ■> 4 PH3 {aq) + 12 H2PO2 (aq) + 12 OH" {aq)

or P4(g) + 3 0H-(a^) + 3H20(/) 4PH3(fl^)+ 3H2PO2 {aq) .(Vi)


Thus, Eq. (vO represents the correct balanced equation.
O.N. increases by 4 per N atom

-2 +5 +2 -1

(6) N2H4(/) + ClOKa^) > NO(g) + C\-{aq)


O.N. decreases by 6 per Cl atom

w
Therefore, N2H4 acts as the reducing agent while CIO3 acts as the oxidising agent.
Oxidation number method

F lo
Total increase in O.N. of N = 2 x 4 = 8, Total decrease in O.N. of Cl = 1 x 6 = 6

Therefore, to balance increase/decrease in O.N. multiply N2H4 by 3 and ClOj by 4, we have,

3 N2H4 (0+4 CIOJ {aq) > NO (g) + Cl" {aq)

ee
To balance N and Cl atoms, multiply NO by 6 and Cl" by 4, we have,

Fr
3 N2H4 (0+4 CIOJ {aq) > 6 NO (g) + 4 Cl" {aq)
Balance O atoms by adding 6 H2O,

for
3 N2H4 (0+4 ClOj {aq) > 6 NO (g) + 4 Cl" {aq) + 6 H2O (0 .(0
ur
H atoms get automatically balanced and thus Eq. (0 representsthe correct balanced equation.
Ion electron method.
oks
-2 +2
Yo
Oxidation half reaction : N2H4 (/) > NO (g)
o

Balance N atoms. N2H4(0 >2NO(g)


eB

Balance O.N. by adding electrons, N2H4 (0 >2NO(g) + 8e-


Balance charge by adding 8 OH“ ions, N2H4 (0 + 8 OH“ {aq) > 2 NO (g) + 8 e“
Balance O atoms by adding 6 H2O, N2H4 (0 + 8 OH“ {aq) > 2 NO (g) + 6 H2O (0 + 8 ».(«)
our

Thus, Eq. (iO represents the correct balanced oxidation half equation.
ad

+5 -1

Reduction half reaction : ClOj {aq) > Cl- {aq)


Balance O.N. by adding electrons, ClOj {aq) + 6 e~ > Cl" {aq)
Y
Re

Balance charge by adding OH" ions, CIOJ {aq) + 6 > Cl" {aq) + 6 OH“ {aq)
nd

Balance O atoms by adding 3 H2O, ClOj {aq) + 3 H2O (0 + 6 e~ » Cl" {aq) + 6 OH~ {aq) ...(/«)
Fi

Thus, Eq. (m) represents the correct balanced reduction half equation.
To cancel out electrons gained and lost, multiply Eq. (i7) by 3 and Eq. {Hi) by 4 and add, we have,
3 N2H4 (0+4 ClOj {aq) > 6 NO (g) + 4 Cl" {aq) + 6 H2O (0 m
Thus, Eq. (/v) represents the correct balanced equation
O.N. decreases by 4 per Cl atom

+7 -1 +3 0
(C) Cl207(g) +
H202(«^) > CIOi(g) +
92(g) +
O.N. increases by I per O atom t
Thus, CI2O7 (g) acts an oxidising agent while H2O2 {aq) as the reducing agent.
8/94 T^n^euUe^ New Course Chemistry (XI) El
Oxidation number method

Total decrease in O.N. of CI2O7 = 4x2 = 8, Total increase in O.N. of H2O2 = 2x1=2
To balance increase/decrease in O.N. multiply H2O2 and O2 by 4, we have,

ow
CI2O7 (g) + 4 H2O2 (aq) > CIO2 (aq) + 4 O2 (g)
To balance Cl atoms, multiply CIO2 by 2, we have,
CI2O7 ig) + 4 H2O2 (aq) > 2 CIO2 (aq) + 4 O2 (g)
To balance O atoms, add 3 H2O to R.H.S., we have,

e
CI2O7 (g) + 4 H2O2 (aq) > 2 CIO2 (aq) + 4 O2 (g) + 3 H2O (/)

re
To balance H atoms, add 2 H2O to R.H.S. and 2 OH“ to L.H.S., we have,
CI2O7 (g) + 4 H2O2 (g) + 2 OH- (aq) > 2 CIO2 (aq) + 4 O2 (g) + 5 H2O

Frl
F
This represents the balanced redox equation.
Ion electron method
-1 0
Oxidation half reaction :
ou
H2O2 (aq) ^ O2 (g)

sor
Balance O.N. by adding electrons, H2O2 (aq) > O2 (g) + 2e~
Balance charge by adding 2 OH" ions, H2O2 (aq) + 2 OH“ (aq) > O2 (g) + 2e~
Balance O atoms by adding 2 H2O, H2O2 (aq) + 2 OH“ (aq) > O2 (g) + 2 H2O (l) + 2e~

kf
.(/)
+7 +3

Reduction half reaction : CI2O7 (g) ^ CIO2 (aq)


oo
Y
Balance Cl atoms ; Cl207(g) > 2CIO2 (aq)
B

Balance O.N. by adding electrons, CI2O7 (g) + 8 e~ > 2 CIO2 (aq)


Add 6 OH" ions to balance charge : CI2O7 (g) + 8 e~ > 2 ClOj (aq) + 6 OH“
re

Balance O atoms by adding 3 H2O to L.H.S., we have,


oY
u

CI2O7 (g) + 3 H2O (0 + 8 e- > 2 CIO2 (aq) + 6 OH" (aq) m

To cancel out electrons, multiply Eq. (i) by 4 and add it to Eq. (ii), we have,
ad

4 H2O2 (aq) + 8 OH" (aq) + CI2O7 (g) + 3 H2O (0


d

2 a02 (aq) + 6 OH" (aq) + 4 O2 (g) + 8 H2O (0


in

or
CI2O7 (g) + 4 H2O2 (aq) + 2 OH" (aq) » 2 ClOj (aq) + 4 O2 (g) + 5 H2O (0
Re

Q. 8.20. What sorts of informations can you draw from the following reaction :
F

(CN)2 (g) + 2 OH- (aq) > CN" (aq) + CNO“ (aq) + HjO (0
Ans. Let x be the O.N. of C. O.N. of C in cyanogen, (CN)2 = 2(r-3) = 0 orr = + 3
O.N. of C in cyanide ion, CN“ = Jc-3 = -l orx = + 2
O.N. of C in cyanate ion, CNO" = x- 3- 2 = -l orjc = + 4
The following four informations can be drawn from the above reaction :
(i) The reaction involves decomposition of cyanogen, (CN)2 in the alkaline medium to cyanide ion, CN~
and cyanate ion, CNO".
(ii) The O.N. of C decreases from + 3 in (CN)2 to + 2 in CN" ion and increases from + 3 in (CN>2 to + 4
is CNO~ ion. Thus, cyanogen is simultaneously reduced to cyanide ion and oxidised to cyanate ion.
(Hi) It is an example of a redox reaction in general and a disproportionation reaction in particular.
(iv) Cyanogen is a pseudohalogen (behaves like halogens) while cyanide ion is a pseudohalide ion (behaves
like halide ion).
REDOX REACTIONS 8/95

Q. 8.21. The Mn^ ion is unstable in solution and undergoes disproportiona tion to give Mn^, Mn02 and
H'*' ion. Write a balanced ionic equation for the reaction.
Ans. The skeletal equation is : {aq) > {aq) + Mn02 (5) + H"*" {aq)
+3 +4

Oxidation half equation : {aq) > Mn02(5)


Balance O.N. by adding electrons, Mn^'*' {aq) > Mn02 (5) + e~
3+

Balance charge by adding 4 ions, Mn {aq) > Mn02 {s) + 4 H'*' {aq) + e~
3+

Balance O atoms by adding 2 H2O : Mn {aq) + 2 H2O (/) > Mn02 (^) + 4 {aq) + e~ .(0

w
+3 +2

Reduction half equation : Mn^"*" > Mn 2+


Balance O.N. by adding electrons : Mn^"^ {aq) + e~ > Mn^'*' {aq) .(«●)

F lo
Adding Eq. (/) and Eq. («), the balanced equation for the disproportionation reaction is
2 Mn^+ {aq) + 2 H2O (/) > Mn02 (j) + Mn^+ {aq) + 4 H+ {aq)
Q. 8.22. Consider the elements : Cs, Ne, I, F

ee
(a) Identify the element that exhibits -ve oxidation state,
(b) Identify the element that exhibits +ve oxidation state

Fr
(c) Identify the element that exhibits both +ve and -ve oxidation states
(d) Identify the element which neither exhibits -ve nor +ve oxidation state,
Ans.

for
(a) F. Fluorine being the most electronegative element shows only a -ve oxidation state of -1.
{b) Cs. Alkali metals because of the presence of a single electron in the valence shell, exhibit an oxidation
our
state of +1.
(c) I. Because of the presence of seven electrons in the valence shell, I shows an oxidation state of - 1 (in
compounds of I with more electropositive elements such as H, Na, K, Ca, etc.) or an oxidation state of
oks
+1 (in compounds of I with more electronegative elements, Le., O, F, etc.) and because of the presence
of fr-orbitals it also exhibits +ve oxidation states of +3, +5 and +7.
o

{d) Ne. It is an inert gas (with high ionization enthalpy and high positive electron gain enthalpy) and
Y
eB

hence it neither exhibits -ve nor +ve oxidation states.


Q. 8.23. Chlorine is used to purify drinking water. Excess of chlorine is harmful. The excess chlorine is
removed by treating with sulphur dioxide. Present a balanced equation for the reaction for this
redox change taking place in water.
our
ad

Ans.
The skeletal equation is : CI2 {aq) + SO2 {aq) + H2O (/) > Cl" {aq) + SO {aq)
Reduction half equation : CI2 {aq) > Cl" {aq)
0 -1
Y

Balance Cl atoms. CI2 {aq) ^ 2 Cl" {aq)


Re

Balance O.N. by adding electrons : CI2 {aq) + 2 e" ^ 2 Cl" {aq) .(0
nd

+4 +6

Oxidation half equation SO2 {aq) > SOj {aq)


Fi

Balance O.N. by adding electrons : SO2 {aq) > SOj" {aq) + 2e~
Balance charge by adding 4 ions : SO2 {aq) > SOj" {aq) + 4 {aq) + 2e~
Balance O atoms by adding 2 H2O,
SO2(o^) + 2H2O(0 4S02- (fl^)+4H+(09) + 2e" ,{U)
Adding Eq. {i) and Eq. {ii), we have.

CI2 {aq) + SO2 {aq) + 2 H2O (/) 4 2 Cl" {aq) + SOj" {aq) + 4 H+ {aq)
This represents the balanced redox reaction.
8/96 7>>uuUefr'^ New Course Chemistiy (XI)ESian

Q. SJA, Refer to the periodic table given in your book and now answer the following questions,
(a) Select the possible non-metals that can show disproportionat ion reaction,
(b) Select three metals that show disproportionation reaction.

ow
Ans. (a) The non-metals are : P4, CI2 and Sg.
(0 P4 (s) + 3 OH- (aq) + 3 H2O (/) > PH3 (g) + 3 H2PO2 (ag)
cold
(ii) CI2 (g) +2 OH-(aq) » Cl- (aq) + CIO- (aq) + H2O (1)
hot
or 3 CI2 (g) +6 OH-(aq) ^ 5 cr (aq) + ClOj (aq) + 3 H2O (/)

re
(iii)Sg(s) + 12 OH- 4 S2- (aq) + 2 $20^- (aq) + 6 H2O (f)
(b) The metals are : Cu'*', Ga'^, In\ Mn^'*', etc.

Flr
F
2 Cu+ (aq) -> Cu^+ (aq) + Cu (s)
3 Ga+ (aq) -> Ga3+ (aq) + 2 Ga (s)
3In+ (aq)
ou ^In^+ (aq)+ 2^ (s)

sr
3+ 4+ 2+

2 Mn (aq) + 2 H2O (/) > Mn02 (s) + Mn (aq) + 4 H+ (aq)

fo
Q. 8.25. In Ostwald’s process for the manufacture of nitric acid, the flrst step involves the oxidation of

k
ammonia gas by oxygen gas to give nitric oxide gas and steam. What is the maximum weight of
nitric oxide that can be obtained starting only with 10*0 g of ammonia and 20*0 g of oxygen ?
oo
Ans. The balanced equation for the reaction is :
HOOK
Y
4NH3(g) + 502(g) Pt
» 4NO(g) +
6H20(g)
4x 17 5x32 4x30
reB

= 6Sg = 160g = l20g


Here, 68 g of NH3 will react with O2 = 160 g
uY

160 g
10 g of NH3 will react with O2 = x20g =23-6g
10 g
But the amount of O2 which is actually available is 20-0 g which is less than the amount which is
needed. Therefore, O2 is the limiting reagent and hence calculations must be based upon the amount of
ad
do

O2 taken and not on ±e amount of NH3 taken.


From the equation, 160 g of O2 produce NO = 120 g
120
in

20 g of O2 will produce NO = 160


x20- =l5g
Re

Q. 8.26. Using the standard electrode potentials given in the Table 8.1, Page 8/44 predict if the reaction
between the following is feasible :
F

(a) Fe^ (aq) and 1“ (aq\ (b) Ag"*" (aq) and Cu (s), (c) Fe^"^ (aq) and Cu (s)
(d) Ag (s) and Fe^ (aq), (e) Br2 (aq) and Fe^ (aq).
Ans. (a) The possible reaction between Fe^'*' (aq) and t~ (aq) is
2 Fe^+ (aq) + 2 T (aq) > 2 Fe^+ (aq) +12 (s)
The above redox reaction can be split into the following two half reactions, one involving oxidation and the
other reduction. Writing electrode potential for each half reaction from Table 8.1, Page 8/44, we have,
Oxidation : 2\-(aq) ^ I2 (s) + 2 e~ ; E“ = -0-54V ...(0
Reduction : 2 Fe^+ (aq) + e“ Fe^*(aq)]x2 ; E“ = + 0-77 V

It may be noted that for oxidqtion reactions, i.e., Eq. (i), the sign of the electrode potential as given in
Table 8.1 is reversed. To get the equation for the overall reaction, the number of electrons lost in Eq. (0
and gained in Eq. (ii) must be cancelled. To do so, Eq. (/O is multiplied by 2 and added to Eq. (0- Further,
REDOX REACTIONS 8/97

it may be noted that whenever any half reaction equation is multiplied by any integer, its electrode
potential is not multiplied by that integer. Thus,
OveraU reaction: 2 Fe^+ (,aq) + 21" (aq) > 2 Fe^+ (aq) +12 (s) ; E” = + 0*23 V
Since the EMF for the above reaction is positive, therefore, the above reaction is feasible,
(b) The possible reaction between Ag"^ (aq) and Cu (^) is
Cu (s) + 2 Ag'*’ (aq) > Cu^'*’ (aq) + 2 Ag (s)
The above redox reaction can be split into the following two half reactions. Writing electrode potential
for each half reaction from Table 8.1, page 8/44, we have,
Oxidation : Cu(s) ●» Cu^'*’ (aq) + 2 e~ E® = -0-34 V
Reduction : Ag'*’ (aq) + e~ 4 Ag (j)] X 2 E“ = + 0*80 V

w
Overall reaction : Cu (s) + 2 Ag+ (aq) > Cu^'*’ (aq) + 2 Ag (5); E® = + 0-46 V
Since the EMF of the above reaction comes out to be positive, therefore, the above reaction is feasible,

F lo
(c) Suppose the reaction between Fe^'*’ (aq) and Cu (s) occurs according to the following equation.
Cu (s) + 2 Fe^+ (aq) > 3 Cu^+ (aq) + 2 Fe^-^ (aq)
The above reaction can be split into the following two half reactions. Writing electrode potential for
each half reaction from Table 8.1, page 8/44, we have,

ee
Oxidation : Cu (s) > Cu^"*" (aq) + 2e~ E® = -0-34 V

Fr
Reduction : Fe^'*' (aq) + e~ Fe^'*' (aq)\ x 2 E® = + 0*77 V

OveraU reaction : Cu (s) + 2 Fe^-*- (aq) > Cu^-^ (aq) + 2 Fe2+ (aq);W = + 0-43 V

for
Since the EMF of the reaction is positive, therefore, the above reaction is feasible.
Alternatively, if the reaction between Fe^"*" (aq) and Cu (s) occurs according to the following equation.
ur
3 Cu (s) + 2 Fe3+ (aq) > 3 Cu^+ (aq) + 2 Fe (5)
The EMF of the reaction comes out to be -ve, i.e., - 0-376 V (- 0-34 V - 0-036 V) and hence this
s
reaction is not feasible.
ook
(●■● E® = - 0-036 V)
Yo
Fe3+,Fe
(d) Suppose the reaction between Ag (j) and Fe^'*' (aq) occurs according to the following equation :
eB

Ag (5) + Fe^'*' (aq) > Ag'*' (aq) + Fe^'*' (aq)


The above reaction can be split into the following two half reactions. Writing electrode potential for
each half reaction from Table 8.1, page 8/44, we have.
Oxidation:
our

Ag(s) >Ag* (aq) + e ; E® = - 0-80 V


ad

Reduction : Fe^'*' (aq) -f e~ Fe2-(04) : E® = -F 0-77 V

OveraU reaction : Ag (j) + Fe^'*' (aq) > Ag+ (aq) + Fe2+ (aq) ; E® = - 0*03 V
Y

Since the EMF of the reaction is negative, therefore, the above reaction is not feasible.
Alternatively, the reaction between Ag (s) and Fe^'*’ (aq) may occur according to the following equation
Re
nd

3 Ag (s) + Fe^"*" (aq) > 3 Ag'*’ (aq) -F Fe (s)


On similar lines, we can calculate the EMF of this reaction comes to be even more negative, i.e.,
Fi

- 0-836 V, and hence this redox reaction is also not feasible,


(e) Suppose the eraction between Br2 (aq) and Fe^'*' (aq) occurs according to the following equation :
Br2 (aq) + 2 Fe^'*' (aq) > 2 Br“ (aq) + 2 Fe^'*’ (aq)
The above eraction can be split into the following two half reactions. Writing electrode potential for
each half reaction from the *^ble 8.1, page 8/44, we have,
Oxidation : Fe^-^ (aq) > Fe^-*- (aq) + e“] x 2 ; E® = - 0-77 V
Reduction : Bf2 (aq) + 2 e~ > 2 Br" (aq) ; E® = -F 1 -09 V
OveraU reaction: 2 Fe2+(a^) + Br2(o9) >2Fe^* (aq) +2 BT-(aq) ; E® = -fO-32V
Since the EMF for the above reaction is positive, therefore, this reaction is feasible.
8/98 7^n<zdee^’4i, New Course Chemistry (XI)QE

Q. 8.27. Predict the products of electrolysis in each of the following :


(i) An aqueous solution of AgN03 silver electrodes,
(ii) An aqueous solution of silver nitrate with platinum electrodes,
(iii) A dilute solution of H2SO4 with platinum electrodes,
(iv) An aqueous solution of CUCI2 with platinum electrodes.
Ans. (0 In aqueous solution, AgNO^ ionises to give Ag"*" {aq) and NO3 {aq) ions.
AgN03 {aq) ^ Ag+(fl^) + NO3 {aq)
Thus, when electricity is passed, Ag'*' {aq) ions move towards the cathode while NO3 ions move towards
the anode. In other words, at the cathode, either Ag'*' {aq) ions or H2O molecules may be reduced. Which
ofthesewill actuallyget dischargedwoulddependupon their electrode potentials which are given below :
Ag+ {aq) + e~ > Ag {s); E° = + 0-80 V .(*●)
2H20(0 + 2e~ > H2 (g) + 2 OH- ; E° = -0-83V ...(«■)
Since the electrode potential {i.e., reduction potential of Ag'*’ {aq) ions is higher than that of H2O molecules,
therefore, at the cathode, it is the Ag"^ {aq) ions (rather than H2O molecules) which are reduced.

F low
Similarly, at the anode, either Ag metal of the anode or H2O molecules may be oxidised. Their electrode
potentials are : Ag(s) > Ag'''(fl^) + e-;E“ = -0-80 V (m)
2 H2O (0 O2 {g) + 4 H-*- {aq) + 4 e' ; E" = - 1-23 V ...(IV)
Since the oxidation potential of Ag is much higher than that of H2O, therefore, at the anode, it is the Ag
of the silveranode which gets oxidisedand not the H2O molecules. It may, however, be mentioned here
that the oxidation potential of NO3 ions is even lower than that of H2O since more bonds are to broken

for Fre
during reduction of NO3 ions than those in H2O.
Thus, when an aqueous solution of AgNO^ is electrolysed, Ag from Ag anode dissolves while Ag* {aq)
ions present in the solution get reduced and get deposited on the cathode,
{ii) If, however, electrolysis of AgN03 solution is carried out using platinum electrodes, instead of silver
electrodes, oxidation of water occurs at the anode since Pt being a noble metal does not undergo oxidation
easily. As a result, O2 is liberated at the anode according to equation (iV).
Thus, when an aqueous solution ofAgNO^ is electrolysed using platinum electrodes, Ag* ions from the
Your
eBo ks

solution get deposited on the cathode while O2 is liberated at the anode.


{Hi) In aqueous solution, H2SO4 ionises to give H'*' {aq) and SO^" {aq) ions.
ad

H2SO4 {aq) > 2 H+ {aq) + SOj" {aq)


our

Thus, when electricity is passed, H'*' {aq) ions move towards cathode while SO^” {aq) ions move towards
anode. In other words, at cathode either H'*' {aq) ions or H2O molecules are reduced. Their electrode
potentials are : 2 H-*- {aq) + 2 > H2 (g); E* = 0 0 V
Re

H2O {aq) + 2e- » H2 {g) + 2 OH" {aq) ;E> = - 0-83 V


Since the electron potential {i.e., reduction potential) of H"*" {aq) ions is higher ±an that of H2O, therefore,
at the cathode, it is H* (aq) ions (rather than H2O molecules) which are reduced to evolve H2 gas.
Find Y

Similarly, at the anode, either SO^“ {aq) ions or H2O molecules are oxidised. Since the oxidation
potential of SO|“ is expected to be much lower (since it involves the cleavage of many bonds as
compared to those in H2O) than that of H2O molecules, therefore, at the anode, it is H2O molecules
(rather than SO^~ ions) which are oxidised to evolve O2 gas.
From the above discussion, it follows that during electrolysis of an aqueous solution ofH2SO/^ only the
electrolysis of H2O occurs liberating H2 at the cathode and O2 at the anode.
(tv) In aqueous solution, CUCI2 ionises as follows : CUCI2 {aq) > Cu^* {aq) + 2 Cl” {aq)
On passing electricity, Cu^'*' {aq) ions move towards cathode and Cl" {aq) ions move towards anode.
REDOX REACTIONS 8/99

Thus, at cathode, either {aq) or H2O molecules are reduced. Their electrode potentials are :
Cu2+(a^) + 2e" > Cu (^) E° = + 0-34 V

H2O (/) + 2 e- > H2 (g) + 2 OH- E" = - 0-83 V

Since the electrode potential of Cu^'*’ {aq) ions is much higher than that of H2O, therefore, at the cathode,
it is Cu^'*' {aq) ions which are reduced and not H2O molecules.
Similarly, at the anode, either Cl“ {aq) ions or H2O molecules are oxidised. Their oxidation potentials
are:
2Cl-(a^) > Cl2(g) + 2e- ; E® = -1-36V
2H20(0 > 02{g)+4U-^{aq) + 4e- ; E" = -1-23V
Although oxidation potential of H2O molecules is higher than that of Cl" ions, nevertheless, oxidation

w
of Cr {aq) ions occurs in preference to H2O since due to overvoltage* of O2 {i.e., water needs greater
voltage for oxidation to O2 as it is a kinetically slow process) much lower potential than - 1-36 V is
needed for the oxidation of H2O molecules.

F lo
Thus, when an aqueous solution of CUCI2 is electrolysed, Cu metal is liberated at the cathode while CI2
gas is evolvedat the anode.
Q. 8.28. Arrange the following metals in the order in which they displace each other from the solution of
their salts. Al, Cu, Fe, Mg and Zn.

ee
Ans. From the Table 8.1, Page 8/44, the E\,3+,,, = - 1-66 V, E“ = + 0-34 V,

Fr
Al /Al /C'U

E"
Fe^H-ZFe =-0-44V, = - 2-36 V and E“ Zn2+/Zn = - 0-76 V.

for
Since a metal with more -ve electrode potential is a stronger reducing agent than the one having less
negative or +ve electrode potential, therefore. Mg can displace all the above metals from their aqueous
ur
solutions, Al can displace all metals except Mg from the aqueous solutions of their salts. Working on
similar lines, we can show that Zn can displace all metals except Mg and Al from the aqueous solutions
s
of their salts while Fe can displace only Cu from the aqueous solution of its salts. Thus, the order in
ok
which they can displace each other from the solution of their salts is Mg, Al, Zn, Fe, Cu.
Yo
Q. 8.29. Given the standard electrode potentials, K+/K = - 2*93 V, Ag+/Ag = 0*80 V, Hg^/Hg = 0*79 V, Mg^/Mg
o

= - 2*37 V, Cr^/Cr = - 0*74 V. Arrange these metals in increasing order of their reducing power.
eB

Ans. Lower the electrode potential, better is the reducing agent. Since the electrode potentials increase in the
order; K+/K (- 2-93 V), Mg^+/Mg (- 2-37 V), Cr^+ZCr (- 0-74 V), Hg2+/Hg (0-79 V), Ag+/Ag (0-80 V),
therefore, reducing power of metals decreases in the same order, i.e., K, Mg, Cr, Hg, Ag.
r

Q. 8.30. Depict the galvanic cell in which the reaction, Zn (s) + 2 Ag*^ {aq) > Zn^ {aq) + 2 Ag (s) takes
ad
ou

place. Further show ; (i) which of the electrode is negatively charged (ii) the carriers of current in
the cell and (iii) individual reaction at each electrode.
Ans. The given redox reaction is
Y

Za{s) + 2 Ag"*" {aq) Zn2+ {aq) + 2 Ag {s)


Since Zn gets oxidised to Zn^"*" ions, and Ag'*' gets reduced to Ag metal, therefore, oxidation occurs at the
Re
nd

zinc electrode and reduction occurs at the silver electrode. Thus, galvanic cell corresponding to the
above redox reaction may be depicted as : Zn I Zn^'*' {aq 11 Ag+ {aq) I Ag
Fi

(0 Since oxidation occurs at the zinc electrode, therefore, electrons accumulate on the zinc electrode and
hence, zinc electrode is negatively charged.
(ii) The ions carry current. The electrons flow from Zn to Ag electrode while the current flows from Ag
to Zn electrode.

(ill) The reactions occurring at the two electrodes are :


Zn(5) ■> Zn2+ {aq) + 2 e"
Ag'*' {aq) + e~ 4 Ag (s)

♦For metal ions to be deposited on the cathode during electrolysis, the voltage required is the same as the
standard electrode potential. However, for liberation of gases, some extra voltage is required than the theoretical
value of the standard electrode potential. This extra voltage is called overvoltage or bubble voltage.
8/100 ‘p>uidccfr''A New Course Chemistry ('Xl'^rosTun

WITH ANSWERS,
pj HINTS AND SOLUTIONS

w
MULTIPLE CHOICE QUESTIONS-I

1. Which of the following is not an example of (a) Bromine is a stronger oxidant than iodine.
redox reaction ? (b) Bromine is a weaker oxidant than iodine,

Flo
(a) CuO + H2 — 4 Cu + H2O (c) Thiosulphate undergoes oxidation by bromine
(b) F6203 + 3 CO > 2 Fe + 3 CO2 and reduction by iodine in these reactions.
(c) 2 K + F2 ^2KF (d) Bromine undergoes oxidation and iodine

ee
(d) BaCl2 + H2SO4 > BaS04 + 2 HCl undergoes reduction in these reactions.

Fr
2. The more positive the value of E®, the greater 6. The oxidation number of an element in a

is the tendency of the species to get reduced. compound is evaluated on the basis of certain
Using the standard electrode potential of redox rules. Which of the following rules is not correct
couples given below find out which of the in this respect ?

for
ur
following is the strongest oxidising agent.
E® values : Fe^'*’/Fe2'^ = + 0*77 ;
(a) The oxidation number of hydrogen is always
+ I.

I2 (s)/I- = + 0*54 ; Cu2+/Cu = + 0-34 ; (b) The algebraic sum of all the oxidation numbers
in a compound is zero,
ks
Ag+/Ag = + 0-80 V
3+ (c) An element in the free or the uncombined state
Yo
(a) Fe (b) I2 is)
bears oxidation number zero,
oo
(c) Cu2+ (d)Ag^
3. E® values of some redox couples are given (d) In all its compounds, the oxidation number of
eB

fluorine is - 1.
below. On the basis of these values, choose the
correct option. 7. In which of the following compounds, an
element exhibits two different oxidation states.
E® values : Br2/Br = + 1-09 ;
Ag+/Ag (s) = + 0-80 ; Cu^+ZCu (s) =: + 0-34 ; (a) NH2OH ib) NH4NO3
r

(c) N2H4 id) N3H


ou

I2 (5)/I- = + 0-54
ad

8. Which of the following arrangements represent


(a) Cu will reduce Br" (b) Cu will reduce Ag
increasing oxidation number of the central
(c) Cu will reduce I" (d) Cu will reduce Br2
Y

atom ?
4. Using the standard electrode potential, find out
the pair between which redox reaction is not (a) C1O2 , CIOj , CrO^" , MnO^
d
Re

feasible.
E® values : = + 0-77 ; = + 0-54 ; (b) ClOj , CrO^” , MnO^ , CrO^
n

Cu2+/Cu = + 0-34 ; Ag+/Ag = + 0-80 V (c) C1O2 , CIO3 , MnO^ , CrOj-


Fi

(a) Fe^-*' and 1“ ib) Ag'*’ and Cu


(c) Fe^-" and Cu id) Ag and Fe^‘*‘
(d) CiO^” , Mn04 , C1O2 , ClOj
9. The largest oxidation number exhibited by an
5. Thiosulphate reacts differently with iodine and
element depends on its outer electronic
bromine in the reactions given below :
configuration. With which of the following
^2^l~ + h ^ S4O2- + 2I- outer electronic conHgurations the element will
exhibit largest oxidation number ?
Sjpl- + 2Br2 + 5H2p ■>
ia) 3d^ 4s^ (b) 3d^ 4^2
2SO|- + 2Br- + 10H+ (c) 3d^ 45^ (d) 3d^ 4^2
10. Identify disproportionation reaction
Which of the following statements justifies the
above dual behaviour of thiosulphate ? ia) CH4 + 2 O2 > CO2 + 2 H2O
REDOX REACTIONS 8/101

(b) CH4 + 4CI2 — ^ CCI4 + 4 HCl 11. Which of the following elements does not show
(c)2F2 + 2 0H-~ 2 F* + OF2 + H2O disproportionation tendency ?
ia) C! (b) Br
(d) 2 NO2 + 2 OH- > NO2 + NOj + H2O (c)F (^/)I

MULTIPLE CHOICE QUESTIONS-II


In the following questions two or more options electronic configurations will exhibit more than
may be correct: one oxidation state in its compounds.
12. Which of the following statement(s) is/are not {a) 3s
1
(b) 3d^ 4s-

w
true about the following decomposition (c) 3</2 4i'2 (d) 3s^ 3p^
reaction: 2 KCIO3 2 KCl + 3 O2 15. Identify the correct statements with reference
{a) Potassium is undergoing oxidation to the given reaction
{b) Chlorine is undergoing oxidation

F lo
P4 + 3 OH" + 3 HjO > PH3 + 3 H2PO2
(c) Oxygen is reduced
(a) Phosphorus is undergoing reduction only.
{d) None of the species are undergoing oxidation
or reduction {b) Phosphorus is undergoing oxidation only,

ee
13. Identify the correct statement (s) in relation to (c) Phosphorus is undergoing oxidation as well as
reduction.
the following reaction :

Fr
Zn + 2 HCl ^ ZnCl2 + H2 {d) Hydrogen is undergoing neither oxidation nor
reduction.
(a) Zinc is acting as an oxidant
16. Which of the following electrodes will act as

for
(b) Chlorine is acting as a reductant
anodes, when connected to Standard Hydrogen
(c) Hydrogen ion is acting as an oxidant
our
Electrode ?
(d) Zinc is acting as a reductant.
(a) A1^+/A1 E® = - 1-66
14. The exhibition of various oxidation states by
ib) Fe2+/Fe E© = - 0-44
s
an element is also related to the outer orbital
(c) Cu^+ZCu E® = + 0-34
ook
electronic configuration of its atom. Atom(s)
having which of the following outermost (J)2F-(ag)/F2(g) E® = + 2-87
Y
eB

ANSWERS

Multiple Choice Questions -1


l.(d) 2.(d) 3.(d) 4.(d) 5. (a) 6. (a) 7. (i) 8. (a) 9.(.d) 10. (t/) 11. (c)
r
ad
ou

Multiple Choice Questions - II


12. (a, b, c, d) 13. (c. d) 14. {b, c, d) 15. (c, d) 16. {a, b)
Y

HINTS FOR DIFFICULT MULTIPLE CHOICE QUESTIONS


Re
nd

Multiple Choice Questions -1


Fi

2. Since E® of the redox couple, AgVAg is the most positive, i.e., 0-80 V, therefore, Ag'*’ is the strongest
oxidising agent.
3. Copper will reduce Br2, if the E® of the redox reaction, 2 Cu + Br2 ■> CuBr2 is +ve.
Now Cu ^ Cu2+ + 2 c- ; E® = - 0-34 V*
Br2 + 2 e ^ 2 Br- ; E® = + 1-09 V

Cu + Br2 ■> CuBr2 ; E® = + 0-75 V

Since E® of this reaction is +ve, therefore, Cu can reduce Br2 and hence option (</) is correct,
4. Calculate the E“ cell of the four redox reactions. If E° cell of a reaction is -ve, that reaction will not occur.

*PIease note that to get oxidation potential of Cu, the sign of the reduction potential has been reversed.
8/102 New Course Chemistry (XI)BZSI9D

(a) 2 Fe3+ + 2 e~ ^ 2 Fe2+ ; E° = + 0-77 V


21- I2 + 2 ; E° = - 0-54 V (sign of E“ is reversed)
2 Fe3+ + 21- ^ 2 Fe2+ +12 ; E"^^,, = + 0-23 V
This reaction is feasible since E°j.eii = +ve.
ib) Cu Cu^'*’ + 2 g-; E“ = - 0*34 V (sign of E“ is reversed)

ow
2 Ag+ + 2 e- ^2Ag;E“ = + 0-80V

Cu + 2 Ag+ > 2 Cu2+ + 2 Ag ; E“ = + 0-46 V


This reaction is feasible since E^^gu is +ve.
(c) 2 Fe3+ + 2 e- > 2 Fe^+ ; E° = + 0-77 V
Cu > Cu^"*" + 2 g-; E® = - 0-34 V (sign of E“ is reversed)

e
re
2 Fe^+ + Cu > 2 Fe^+ + Cu^+ ; E® = + 043 V

Frl
This reaction is feasible since E®ggj| is +ve.

F
Ag > Ag"*" + g-; E® = - 0-80 V (sign of E® is reversed)
Fe3+ + e- > Fe^+ ; E® = + 0-77 V
Ag + Fe^ > Ag+ + Fe^+ ; E“ = - 0-03 V

or
ou
This reaction is not feasible since E®ggj, is -ve.
Thus, option (d) is correct.

kfs
5. Br2 is a stronger oxidising agent than I2. For details, refer to Ans. to Q. 8.14, page 8/87.
6. O.N. of H is always + 1 is a wrong statement since it is +1 in hydrogen halides, - 1 in hydrides and zero in the
H2 molecule.
oo
7. Ammonium nitrate is an ionic compound. It consists of two ions, i.g., NH| and NOj .
The O.N. of N in the two species is different as shown below :
Y
B

O.N.ofNin NHJ = jc + (4 x 1) = +1 orx = -3, O.N. of N in NOj =x + 3 (-2) = -1 orx = + 5.


8. Writing the O.N. of Cr, Cl and Mn on each species in the four set of ions, we have,
re

+3 +5 +6 +7 +5 +6 +7 +3
(a) Cr02 , ClOj , CrOj- , MnO^ (b) CIO3 , CrOj- , Mn04 , CrO^
oYu

+3 +5 +7 +6 +6 +7 +3 +5

(c) Cr02 , ClOj , Mn04 , CrOj- (^0 Cr02- , Mn04 , CrO^ , CIO^-
ad

Only in arrangement (a), the O.N. of central atom increases from left to right, therefore, option (a) is correct.
9. Highest O.N. of any transition element = (n- 1) d electrons + ns electrons,
d

therefore, larger the number of electrons in the 3d orbitals, higher is the maximum O.N.
(a) 4d^ 4s^ = 3; (b) 3d^ 4s^ = 3 + 2 = 5 ; (c) 3d^ 4s^ = 5 + 1 = 6 and (d) 3d^ 4s^ = 5 + 2 = 7.
in
Re

Thus, option (d) is correct.


10. Reactions in which the same substance is oxidised as well as reduced are called disproportionation reactions.
F

Writing the O.N. of each element above its symbol in the following reactions, we have,
-4 +1 0 +4 -2 +1 -2
(«) C H4 +2O2 ^ C O2 +2H2O
-4 +1 0 +4 -1 +1 -1

(b) C H4 +4CI4 C CI4 +4H Cl


0 -2+1 -1 +2 -1 +1 -2
(c) 2F2 +20 H- » 2F- + O F2 + H2 O
+4 -2 -2 +1 + 3 -2 +5 -2 +1 -2
(r/)2N O2 +20 H- > N O2 + N OJ +H2O
Thus, in reaction (d), N is both oxidised as well as reduced since the O.N. of N increases ft^om + 4 in NO2
to + 5 in NOj and decreases from + 4 in NO2 to +3 in NOj . Thus, option (d) is correct.
REDOX REACTIONS 8/103

11. Being the most electronegative element, F can only be reduced and hence it always shows an oxidation
number of — 1. Further, due to the absence of J-orbitals, it cannot be oxidised and hence it does not show
+ve oxidation numbers. In other words, F cannot be simultaneously oxidised as well as reduced and hence
does not show disproportionation reactions. Thus, option (c) is correct.
Multiple Choice Questions - II

w
12. Writing the O.N. of each element above its symbol, we have,
+1 +5 -2 +1 -1 0

2KC1 O3 ^ 2KC1 + 302


(a) Since O.N. of K does not change, therefore, K neither undergoes reduction nor oxidation. Thus, option
(a) is not correct,

o
{b) Since O.N. of Cl decreases from + 5 in KCIO3 to - 1 in KCl, therefore. Cl undergoes reduction, i.e.,

e
option (b) is not correct.

re
(c) O.N. of O increases from - 2 in KCIO3 to 0 in O2, therefore, O is oxidised, i.e., option (c) not correct,

rFl
(d) This statement is not correct because Cl is undergoing reduction and O is undergoing oxidation.

F
Thus, all options are incorrect.
13. Writing the O.N. of each element above its symbol, we have.
0 +i -1 +2 -1 0

-> Zn CI2 + H2

or
Zn +2H Cl
ou
(a) The O.N. of Zn increases from 0 in Zn to + 2 in ZnCl2, therefore, Zn acts as a reductant. Thus, option
(c) is not correct.

ksf
{b) The O.N. of Cl does not change, therefore, it neither acts as a reductant nor an oxidant. Therefore,
option (b) is not correct,
oo
(c) The O.N. of H decreases from +1 in to 0 in H2, therefore, acts an oxidant. Thus, option (c) is
correct.
Y
(^0 As explained in option (a), Zn acts as a reductant, therefore, it cannot act as an oxidant.
B

Thus, options (c) and (d) are correct.


14. Elements which have only ^-electrons in the valence shell do not show more than one oxidation state.
re

Thus, element with 35* as outer electronic configuration shows only one oxidation state of + I.
Transition elements, i.e., elements (b, c) having incompletely filled fr-orbitals in the penultimate shell
oYu

show variable oxidation states. Thus, element with outer electronic configuration as 3d* 45^ shows variable
ad

oxidation states of +2 and +3 and the element with outer electronic configuration as 3d^ 4sr^ shows variable
oxidation states of +2, +3 and +4.
p-Block elements also show variable oxidation states due to a number of reasons such as involvement of
d

d-orbitals and inert pair effect. For example, element (d) with 3s^ 3p^ as (i.e., P) as the outer electronic
configuration shows variable oxidation states of +3 and +5 due to involvement of d-orbitals.
in
Re

Thus, options (b), (c) and (d) are correct.


IS. Writing the O.N. of each element above its symbol, we have.
F

0 -2 +1 +1 -2 -3 +1 +1 +1

P4 + 3O H~ + 3H2O ^ P H3 +3H2 PO2

In this reaction, O.N. of P increases from 0 in P4 to +1 in H2PO2 and decreases to - 3 in PH3, therefore,
P has undergone both oxidation as well as reduction. Thus, option (a) and (b) are wrong and option (c) is
correct. Further, O.N. of H remains +1 in all the compounds, i.e., H neither undergoes oxidation nor
reduction. Thus, option (d) is correct. In other words, options (c) and (d) are correct.
16. All electrodes which have negative electrode potentials are stronger reducing agents than H2 gas and hence
act as anodes when connected to SHE. Thus, Ap'^/Al (E° = - 166 V) and
Fe^'^/Fe (E® = - 0*44 V) act as anodes, i.e., options (a) and (b) are correct.
8/104 "Pnade^’a. New Course Chemistry fxnrosTwm

SHORT ANSWER QUESTIONS


17. The reaction a2(g) + 2 0H-(fl^) ■> CIO- (fiq) + C|- {aq) + HjO (/)
represents the process of bleaching. Identify and name the species that bleaches the substances due
to its oxidising action.
Ans. Writing the O.N. of each element above its symbol, we have,
0 -2 +1 +1 -2 -! +1

Ci2(g) + 20 Y\-{aq) > Cl 0"(ai?)+Cr (a^) + H20“(/)


In this reaction, O.N. of Cl increases from 0 in Cl^ to +1 in CIO" and decreases to - 1 in Cl". Therefore, CI2

low
is both oxidised to CIO" and reduced to Cl". Since Cl" ion cannot act as an oxidising agent (because it can
not decrease its O.N. lower than - 1), therefore. CI2 bleaches substances due to oxidising action of CIO"
(hypochlorite)ion.
18. MnO^" undergoes disproportionation reaction in acidic medium but MnO^ does not. Give reason.
Ans. In MnO^ , Mn is in the highest oxidation state of + 7 (/.«., cannot be oxidised further) and hence it cannot

ee
undergo disproportionation.

F
In contrast, the O.N. of Mn in MnO|" is + 6. Therefore, it can increase its O.N. to + 7 or decrease its O.N.

Fr
to some lower value. Thus, MnO^" undergoes disproportionation according to the following reaction.

for
+6 -2 +7 -2 44 -2 +1 -2
3Mn 0\- + A}\*
ur
■> 2Mn O4 + Mn O2+ 2H2 0 j
Here, the O.N. of Mn increases from + 6 in MnO|" to +7 in MnO^ and decreases to +4 in Mn02- Thus,
s
M/1O4" undergoes disproportionation in acidic medium.
ook
Yo
19.
PbO and Pb02 react with HCl according to following chemical equations :
2 PbO + 4 HCI > 2 PbCl2 + 2 HjO
eB

Pb02 + 4 HCI > PbCl2 + CI2 + 2 H2O


Why do these compounds dilTer in their reactivity ?
Ans. Writing the O.N. of each element above its symbol in the following reactions, we have,
r

4-2 -2 +1 -I 4-2 -1 4-1 -2


ou
ad

(0 2Pb O -H 4H Cl ^ 2Pb CI2 + 2H, O


44 -2 +1 -1 +2 -I 0 4-1 -2
(») Pb O2 + 4H Cl ^ Pb CI2 + CI2 + 2H2 O
Y

In reaction (i), O.N. of none of the atoms undergo a change, therefore, it is not a redox reaction. It is, in
fact, an acid-base reaction since PbO being a basic oxide reacts with HCI to form PbCl2 and H2O.
Re
nd

PbO + 2 HCI > PbCl2 + H2O ...(acid-base reaction)


In reaction (n). Ph in Pb02 is present in +4 oxidation state. Since due to inert pair effect, +2 oxidation
Fi

state of Pb is more stable, therefore, Pb02 acts as an oxidising agent. As a result, it oxidises Cl" to CI-, and
itself gets reduced to Pb-'*'. In other words, reaction of Pb02 with HCI is a redox reaction.
Pb02 + 4 HCI > PbCl2 + CI2 + 2 H2O ...(redox reaction)
Thus, the two compounds differ in their reactivity because they undergo different types of reactions.
20. Nitric acid is an oxidising agent and reacts with PbO but it does not react with Pb02. Explain why ?
Ans. Since in Pb02, Pb is in the maximum oxidation state of +4 and in HNO3, N is in the maximum oxidation
state of +5, therefore, none of them can oxidise each other.
In other words, Pb02 is passive towards HNO3 and hence no reaction occurs. In contrast, PbO being basic
undergoes an acid-base reaction to form Pb(N03)2 and H2O
PbO + 2 HNO3 > Pb(N03)2 + H2O ...(acid base reaction)
REDOX REACTIONS 8/105

21. Write balanced chemical equation for the following reactions :

({) Permanganate ion (MnO^) reacts with sulphur dioxide gas in acidic medium to produce Mn^
and hydrogen sulphate ion. (Balance by ion electron method)

(ii) Reaction of liquid hydrazine (N2H4) with chlorate ion (OOp in basic medium produces nitric
oxide gas and chloride ion in gaseous state. (Balance by oxidation number method)
(m) Dichlorine heptaoxide (CI2O7) in gaseous state combines with an aqueous solution of hydrogen
peroxide in acidic medium to give chlorite ion (OOp and oxygen gas. (Balance by ion electron
method)

w
Ans. (i) Step 1. Write the skeleton equation for the given reaction.

Mn04 {aq) + SOj (g) Mn2+(fl^) + HSOpfl^) ..(a)

F lo
Step 2. Find out the elements which undergo a change in O.N.
O.N. of Mn decreases by 5
+7 +4
;
+2 +6

►Mn2+(flqr) + HSOi(fl^)

ee
Mn04(a^) + S02(g)
1

Fr
O.N. of S increases by 2

Here, O.N. of Mn decreases from +7 in Mn04 to +2 in Mn^'*', therefore, Mn04 undergoes reduction.

for
Further, O.N. of SO2 increases from +4 in SO2 to +6 in HSO4 , therefore, SO2 undergoes oxidation.
our
Step 3. Divide the given skeleton Eq. (a) into two half equations.
Reduction half equation : Mn04 iaq) ^ Mn2+ iaq) ib)
s
ook

Oxidation of half equation : S02(g) > HSOpfl^) .(c)


Step 4. To balance reduction half Eq. (b). Since Mn atoms on either side of Eq. (b) are already balanced,
Y
eB

balance O.N. by adding electrons. The O.N. of Mn in MnO^ is +7 while that in Mn^'*’ is +2. Therefore,
each Mn atom loses 5 electrons, and hence add 5 e~ to L.H.S. of Eq. (/>), we have,

MnOpo^)+5e“ Mn2+ iaq) id)


our
ad

Since the reaction occurs in acidic medium, therefore, balance charge by adding H* ions.
Now total charge on the L.H.S. of Eq. id) is -6 while on the R.H.S., it is +2. Therefore, add 8 H'*' to the
L.H.S. of Eq. id), we have.
Y

MnOpfl^) + 8H+(«9) + 5e- ^ Mn^'*' iaq) ..(c)


Re

7b balance H and O atoms, add 4 H2O to the R.H.S. of Eq. (e), we have.
d
Fin

Mn04 iaq) + 8H'*' iaq) + 5e ^Mn2+(o9) + 4H20(/) .(T)


Thus, Eq. (f) represents the balanced reduction half equation.
Step 5. To balance oxidation half Eq. (c).
Since S atoms on either side of equation (c) are already balanced, balance O.N. by adding electrons. Since
O.N. of S on L.H.S. of Eq.(c) is +4 while on the R.H.S., it is +6, therefore, add 2 e~ to the R.H.S. of Eq. (c).
We have.

S02(g) 4 HSOpfl<?) + 2e >(g)


Now balance charge by adding ions. Since the total charge on R.H.S. of Eq. (g) is -3 and zero on
L.H.S. add 3 H+ to R.H.S. of Eq. (g). We have,
SO2 (g) > HSO4 iaq) + 3 H+ (o^) + 2 e” .(*)
8/106 New Course Chemistry (XI)CBiaD

Novi balance O atoms by adding H2O molecules. Since there are two O atoms on the L.H.S. and four on
the R.H.S. of Eq. (ft), add 2 H2O to the L.H.S. of Eq. (ft). We have.
S02(fl?) + 2H20 (/) 4 HS04(a^) + 3H+(fl^) + 2c- .(/)
The number of H atoms are automatically balanced. Thus, Eq. (i) represents the balanced oxidation half
equation.
Step 6. To cancel the number of electrons gained in reduction half Eq. (f) and number of electrons lost in
;he oxidation half Eq. (0, multiply Eq. (/") by 2 and Eq. (/) by 5 and add them together, we have.
2Mn04 {aq) + 16H+ {aq) + lOe" ■» 2 Mn^+ {aq) + 8 H2O (/)
5SO2(g)+10H2O(0 ^ 5 HSO4 +15 H+ {aq) + lOe"

w
2Mn04 {aq) + 5SO2 {g) + (/) + H+ {aq) > 2Mn2" {aq) + 5HSO4 {aq)

F lo
This represents the final balanced redox equation.
(I'O Refer to Ans. to Q. 8.19 (ft), page 8/93.
(ill) Step 1. Write the skeleton equation for the given reaction.

CI2O7 {g) + H2O2 {aq) > ClOj {aq) + O2 (g) .(«)

ee
Step 2. Find out the elements which undergo a change in O.N.

Fr
O.N. of Cl decreases by 4 per Cl atom

for
+7-2 +1 -1 +3-2 0

CI2O7 (g) + H202(a^) ► Cl 02(0?) + 02(g)


1
r
O.N. of O increases by 1 per O atom
You
Here, O.N. of CI2 decreases from +7 in CI2O7 to +3 in CIO2 , therefore, CI2O7 undergoes reduction.
s
ook

Further, O.N. of O inc reases from -1 in H2O2 to 0 in O2 (g), therefore, H2O2 undergoes oxidation.
Step 3. Divide the given skeleton equation (a) into two half equations.
eB

Reduction half equation: CI2O7 ^ CIO2 >{b)


Oxidation half equation : H2O2 ^02 .(C)
Step 4. Balance reduction halfEq. (ft).
our
ad

To balance Cl on either side of Eq. (ft), multiply CIO2 on R.H.S. of Eq. (ft), by 2, we have,

Cl207(g) > 2C102(a^) ...(d)


dY

Since O.N. of Cl is +7 is CI2O7 and +3 in ClOj, therefore, decrease in O.N. of Cl per atom = 7-3 = 4.
Re

Further, since there are two Cl atoms on the R.H.S. of Eq. (d), therefore, total decrease in O.N. of Cl
= 2x4 = 8.
Fin

Therefore, to balance O.N. of Cl, add 8 e~ to L.H.S. of Eq. {d), we have,

CI2O7 (g) + 8 e > 2CIO2 {uq) Xe)


Now the total charge on the L.H.S. of Eq. {e) is -8 while on R.H.S. it is -2. Therefore, to balance charge,
add 6 H'*’ to L.H.S. of Eq. (e), we have.
CI2O7 (g) + 6 H"*" {aq) + S e > 2 CIO2 {aq) ..(O
To balance O and H atoms, add 3 H2O to R.H.S. of Eq. {f), we have,
CI2O7 (g) + 6 H+ {aq) + Se~ > 2C10^ {aq) + 3H2O (/) ...(g)
This represents the balanced reduction half equation.
REDOX REACTIONS 8/107

Step 5. To balance oxidation half Eq. (c).


The O.N. of O in H2O2 is -1 while that in O2 is zero. Since there are two oxygen atoms, therefore, total
increase in O.N. of O is 2. Therefore, to balance O.N., add 2 e~ to R.H.S. of Eq. (c), we have,
H2O2 {aq) > O2 (g) + 2 e~ ...w
To balance charge, add 2 to R.H.S. of Eq. (h), we have,
H2O2 (aq) > O2 (g) + 2 H+ + 2 e" ...(0
This represents the balanced oxidation half equation.
Step 6. To cancel out the number of electrons gained in reduction half reaction, and lost in oxidation half
reaction, multiply Eq. (i) by 4 and add to Eq. (g), we have,

CI2O7 (g) + 6 H+ (04?) + 8 e" » 2 CIO2 (aq) + 3 H2O (1)

w
4 H2O2 (aq) > 4 O2 (g) + 8 H+ (a^) + 8 c"

F lo
Cl207(g)+4H202(o^) ^ 2CIO2 (aq) + 3H2O(1) + 4 O2 (g) + 2 H+ (aq)
This represents the Erst balanced redox equation.
22. Calculate the oxidation number of phosphorus in the following species.

ee
(a) HPO|- and (b) PO|-

Fr
jc = + 3.
Ans. (a) Let the O.N. of P in HPO3 be jc. .*. 1 + ;c + 3 (- 2) = - 2 or
(/;)LettheO.N.ofPin PO5-bex. .-. x + 4(-2) = -3 or x = + S.

for
23. Calculate the oxidation number of each sulphur atom in the following compounds :
(a) NajSjOj (b) NajSPg
ur
(c) NajSOj (d) Na2S04
Ans. (a) - 2 and + 6. Refer to Q. 10(i) page 8/67 for details.
(b) + 5,0,0, + 5. Refer to page 8/21 for details.
s
ook
jc = + 4.
(c) Let the O.N. of S in Na2S03 be x. .-. 2 (+ 1) + x + 3 (- 2) = 0 or
Yo
(d)LettheO.N.ofSinNa2S04bex. 2 (+1)+ x + 4 (-2) = 0 or jc = + 6.

24. Balance the following equations by the oxidation number method,


eB

(i) Fe2+ + H+ + CrjOf- > Ci^ + Fe^ + HjO (ii) + NOj > NOj + IO3
(«i)Ij + SjO|- (iv) Mn02 + C20j“ > Mn^ + CO2
our
ad

Ans. (i) Step 1. Find out the elements which undergo a change in O.N.
O.N. increases by 1 per atom-
+6 +3 I
► Cr^'^ + Fe^’^ + H2O
2+
Fe + + Cr207~
.(«)
Y

●- O.N. decreases by 3 per atom 1


Re
nd

Step 2. Find out the total increase and decrease in O.N.


Since there is only one Fe atom on either side of Eq. (a), therefore, total increase in O.N. is 1. Further, since
Fi

there are two Cr atoms in on L.H.S. and only one in Cr^'*’ on L.H.S. of Eq. (a), therefore, total
decrease in O.N. = 2x3 = 6.
Step 3. Balance increase/decrease in O.N.
Since the total increase in ).N. is 1 and decrease is 6, therefore, multiply Fe^"^ on the L.H.S. of Eq. (a) by
6, we have.

6Fe2++H++Cr20^- — ^Cr3+ + 6Fe3+ + H20 m


Step 4. Balance all atoms other than O and H.
To balance Cr on either side of Eq. (/?), multiply Cr^'*' by 2 on R.H.S. of Eq. (b), we have,
6Fe2++H++Cr20^- ^2Cr3+ + 6Fe3+ + H20 >(c)

t
8/108 ‘P^eicCeefi,'^, New Course Chemistry (XI)iSS2aD

Step 5. Balance O and H atoms by hit and trial method.


Since there are 7 oxygen atoms on L.H.S. of Eq. (c) and only 1 in H2O on multiply H2O by 7, we have,
6Fe2+ + H++Cr20|- >6 + 2 Cf3+ + 7 H2O
To balance H, multiply on L.H.S. of Eq. (d) by 14, we have,
6Fe2+ + 14H++Ci20f- >6 +2 + 7 H2O ie)
Thus, Eq. (e) represents the correct balanced equation,
(ii) Step 1. Find out the elements which undergo a change in O.N.
O.N. increases by 5 per 1 atom
Ir
0 +5 +4 +5
h + NO5 > NO2 + lOJ
1 .(a)

w
’ O.N. decreases-
by 1 per N atom

F lo
Step 2. Find out the total increase/decrease in O.N.

Since there are two I atoms on the L.H.S. of Eq. (a) and only 1 on the R.H.S., therefore, multiply lOj on
the R.H.S. of Eq. (a) by 2 and thus total increase in O.N. is2x 5 = 10. Further, since there is only one N
atom on either side of Eq. (a), therefore, total decrease in O.N. = 1x1 = 1.

ree
Step 3. Balance increases/decrease in O.N.

for F
Since total increase in O.N. is 10 and decrease is only 1, therefore, multiply NOJ on L.H.S. of Eq. (a) by
10, we have.

I2+IONOJ > NO2+IOJ ...(ft)


Step 4. Balance all atoms other than O and H.
Your
Since there are two I atoms on L.H.S. and only 1 on R.H.S. of Eq. (b), therefore, multiply IO3 by 2 on
ks
eBoo

R.H.S. of Eq. (b). Further, since there are 10 N atoms on L.H.S. and only 1 on R.H.S. of Eq. (b), therefore,
multiply NO2 by 10 on R.H.S. of Eq. (b). Combining these two steps, we have,
I2+IONOJ > IONO2+2IO3 (c)
Step 5. Balance O and H atoms by hit and trial method.
ad
our

To balance O atoms on either side of Eq. (c), add 4 H2O to the R.H.S. and to balance H atoms add 8 to
the L.H.S. of Eq. (c), we have.

I2+IONOJ + 8H+ > IONO2+2IOJ + 4H2O id)


Thus, Eq. (d) erpresents the correct balanced equation,
Re

(m) Step 1. Find out the elements which undergo a change in O.N.
Y

O.N. decreases by 1 per I atom-


Find

0 +2 -I 2.5
12 + S20i- + S4OI- (a)
O.N. increases by 1/2 per S atom i
Step 2. Find out the total increase/decrease in O.N.
Since there are two I atoms on L.H.S. of Eq. (a) and only 1 on R.H.S., therefore, the total decrease in O.N.
= 2x1=2. Further, since there are two S atoms on L.H.S. and four S atoms on the R.H.S. of Eq. (a),
therefore, total increase in O.N. = 1/2 x 4 = 2.
Step 3. Balance increase/decrease in O.N.
The total increase/decrease in O.N. is already balanced.

it
REDOX REACTIONS 8/109

Step 4. Balance all atoms other and O and H atoms.


Since there are two I atoms on the L.H.S. and only one on the R.H.S. of Eq. (a), multiply I~ on R.H.S. of
Eq. (a) by 2. Further since, there are two S atoms on the L.H.S. and four S atoms on the R.H.S. of Eq. (a),
therefore, multiply 8203" on L.H.S. of Eq. (a), by 2, we have.

^ 2I-+S4O2- .(*)

w
Thus, Eq. {b) represents the correct balanced equation.
(iV) Step 1. Find out the elements which undergo a change in O.N.
O.N. decreases by 2 per Mn atom
+4 +3
\
+2 +4

Mn02 + c,o|- > Mn2+ + CO2

o
(a)

e
O.N. increases by 1 per C atom

re
Step 2. Find out the total increase/decrease in O.N.

rFl
Since there is only one Mn atom on either side of Eq. (a), therefore, total decrease in O.N. = 1x2-2.

F
Further, since there are two C atoms on the L.H.S. and only one on the R.H.S. of Eq. (a), therefore, total
increase in O.N. = 2x1=2.
Step 3. Balance increase/decrease in O.N.

r
The total increase or decrease in O.N. is already balanced.
ou
fo
Step 4. Balance all atoms other than O and H atoms.
Since there are two C atoms on the L.H.S. and only 1 on the R.H.S. of Eq. (a), therefore, multiply CO2 on
R.H.S. of Eq. (a) by 2, we have,
Mn02+C20j- ^Mn2+ + 2C02
ks m
oo
Step 5. Balance all atoms other than O and H atoms.
Since there are six O atoms on L.H.S. and only four on the R.H.S. of Eq. {b), add 2 H2O to the R.H.S. of
Y
eB

Eq. (/>), we have.


Mn02 + C2O5- > Mn2+ + 2 CO2 + 2 H2O .(c)
Now, there are four H atoms on the R.H.S. and none on the L.H.S. of Eq. (c), add 4 H*^ ions to the L.H.S.
r

of Eq. (c), we have.


You

Mn02 + C2O5- + 4 H+ > Mn2+ + 2 CO2 + 2 H2O id)


ad

Thus, Eq. (d) represents the correct balanced equation.


25. Identify the redox reactions out of the following reactions and identify the oxidising and reducing
agents in them.
d

(0 3 HQ iaq) + HNO3 {aq) > Clj (g) + NOCl (g) +2 HjO (/)
Re
in

(*●) HgQj iaq) + 2KI (aq) > Hglj (s) + 2 KCl (aq)

FejOj (s) + 3CO(g) 2 Fe (s) + 3 COj (g)


F

(*«)
(/V) PCI3 (0 + 3 HjO (0 > 3 HCl (aq) + H3PO3 (aq)
(V) 4NH3 + 3 02(g) >2N2(g) + 6H20(g)
Ans. (0 Writing the O.N. on each atom above its symbol, we have.
+1 -1 0 +3 -2 -1 +1 -2

3HCl(a9)+HN 03(09) ■»Cl2(g) + N0a + 2H20(/)


Here, the O.N. of Cl increases from - 1 in HCl to 0 in CI2, therefore, Cl~ is oxidised and hence HCl acts as
the reducing agent.
The O.N. of N decreases from +5 in HNO3 to +3 in NOCl, therefore, HNO^ acts as the oxidising agent.
Thus, reaction (0 is a redox reaction.
8/110 it New Course Chemistry (XI) EZ

(I'O Writing the O.N. of each atom above its symbol, we have,
4-2 —1 4-1 -1 +2 -1 4-1 -1

HgCl2(ag) + 2K \{aq) 4 Hg I2(s) + 2K+Cr(fl^)


Here, the O.N. of none of the atoms undergo a change, therefore, reaction (U) is not a redox reaction.
4-3 -2 +2 -2
^ 0 44 -2
iiii) Fe2 03(j) + 3C 0(g) ^ 2Fe(5) + 3C 02(g)
Here, O.N. of Fe decreases from +3 in Fe203 to 0 in Fe, therefore, Fe203 acts as an oxidising agent.
Further, O.N. of C increases from +2 in CO to +4 in CO2, therefore, CO acts as a reducing agent.

w
Thus, reaction (lii) is a redox reaction.
(iv) Writing the O.N. of each atom above its symbol, we have,
4-3 -1 4-1 -2 4-1 -1 4-1 4-3-2
P Cl3(/)-{-3H2 0(0 4 3H Cl(a^) + H3 P 03(0^)

Flo
Here, O.N. of none of the atoms undergo a change, therefore, reaction (iv) is not a redox reaction.
(v) Writing the O.N. of each atom above its symbol, we have.

e
re
0 0

4N H3+302(g) -»2N2(g)+6H2 0(g)

F
Here, O.N. of N increases from - 3 to 0 in N2, therefore, NH3 acts as a an reducing agent.
Further, O.N. of O decreases from 0 in O2 to - 2 in H2O, therefore, O2 acts as a oxidising agent.
Thus, reaction (v) is a redox reaction.
ur
r
26. Balance the following ionic equations

fo
(1) CrjOf- + H+ +1- > Cr3+ + Ij + HjO («) CrjOf" + Fe^+ + H+
ks > Cr^ + Fe^ + HjO
(«0 Mn04+S0f- + H+ > Mn2+ + S0j- + H20
Yo
(iV) MnOj + H+ + Br" > Mn^ + Bfj + HjO
oo
Ans. (0 Step 1. Write the O.N. of all atoms above their respective symbols.
B

O.N. decreases by 3 per Cr atom


re

+6 —2 -1 34 0 41 -2

Cr207 + H^ + r Cr + I2 + H2O .(a)


●—O.N. increases by 1 per I atom J
u
ad

Step 2. Divide the given skeleton Eq. (a) into two half reaction equations.
Yo

Reduction half equation: ■»Cr3+ >{b)


Oxidation half equation : I" 4I2 .(c)
Step 3. To balance reduction half equation (b).
d
Re

Following the method discussed under Sample Problem 3, page 8/24, the balanced reduction half equation
in

is C^O|-+14H++6e" 4 2 Cr3+ + 7 H2O id)


F

Step 4. To balance the oxidation half equation (c).


Following the method discussed under Ans. to Q. 8.10, pages 8/82-8/83, the balanced oxidation half equation is
21- 412 + 2 ie)
Step 5. To balance the electrons gained in Eq. (d) and lost in Eq. (e), multiply Eq. (c) by 3 and add it to Eq.
(d), we have.

Ci20f-+14H++6e- 42Cr3+ + 7H20


61- 4 3 I2 + 6 e"

Ci20^-+14H+ + 6I- - 42Cr3+ + 3l2 + 7H20


This gives the final balanced ionic equation.
REDOX REACTIONS 8/111

(«) Refer to Sample Problem 3, pages 8/24-8/25.


(Hi) Step 1. Write the O.N. of all atoms above their respective symbols.
O.N. decreases by 5 per Mn atom
+7 -2 +4-2 +2 +6-2,
Mn04 + S O3 + SOi" + H2O ...(a)

O.N. increases by 2 pe" S atom


t
Step 2. Divide the skeleton Eq. (a) into two half-reaction equations.
Reduction half equation : Mn04 > Mn^+ ...(*)

Oxidation half equation : so|- —> S05- ..(c)

w
Step 3. To balance reduction half Eq. (b).
Following the method discussed under Ans. to Q. 21 (/), page 8/102, the balanced reduction half equation

F lo
IS

Mn04+8H++5e- ^Mn2+ + 4H20 .(rf)


Step 4. To balance oxidation half Eq. (c).

e
The O.N. of S in SO^“ is +4 while in SO^“, it is +6. Since there is only one S atom on either side of Eq.

Fre
(c), therefore, total increase in O.N. = 2. Thus, to balance the O.N., add 2 e~ to R.H.S. of Eq. (c), we have.

SO|- > SOj- + 2e- (c)

for
Balance charge by adding ions. The total charge on R.H.S. of Eq. (e) is - 4 while on the L.H.S., it is
-2. Therefore, to balance charge, add 2 !!■*■ ions to R.H.S. of Eq. (e), we have,
r
SO|- > SO5- + 2H+ + 2e- (f)
You
oks
Balance O atoms by adding H2O molecules. Since there are three O atoms on the L.H.S. of Eq. (f) and four
on the R.H.S., add 1 H2O to the L.H.S. of Eq. (f), we have,
eBo

SO|- + H2O > SO|- + 2H+ + 2c- ...(&)


Step 5. To balance electrons, multiply Eq. (d) by 2 and Eq. (g) by 5 and add, we have,
2Mn04 + 16H+ + lOe" > 2 Mn2+ + 8 H2O
our
ad

5S0^- + 5H20 > 5SOj- + 10H++10c-


2Mn04+5S0^-+6H+ > 2Mn2++5S03" + 3H20
This represents the correct balanced redox equation.
dY
Re

(iV) Step 1. Write the O.N. of all the atoms above their respective symbols.
O.N. decreases by 5 per Mn atom
1
Fin

+7 -2 +● . -1 +2 ' +2 -1
Mn O4 + + Br" ►Mn 2+ + B
Br2 + H2O
.(a)
‘-O.N. increases by 1 per Br atom-^
Step 2. Divide skeleton Eq. (a) into two half reaction equations.
Reduction half equation : Mn04 .>Mn2+ .(*)
Oxidation half equation : Br- ^Br2 .(c)
Step 3. To balance reduction half equation (b).
8/112
‘Pxetdeejbr 4 New Course Chemistry (XI)
Following the method discussed under Ans. to Q. 21 (/), page 8/105, the balanced reduction half equation
IS

Mn04+8H++5e- ^ Mn^-" + 4 HjO 4d)


Step 4. To balance oxidation half equation (c).
Balance atoms other than O and H. Since there are two Br atoms on R.H.S. and only one on L.H.S. of Eq.
c), therefore, multiply, Br‘' on L.H.S. of Fx>. (c) by 2, we have,
2Br" Br2 ...(e)
Balance O.N. by adding electrons. O.N. of Br in Br" is - 1 while in Br2 it is 0. Since there are two Br atoms
on L.H.S., therefore, total increase in O.N. = 2x1=2. Thus, to balance O.N., add 2 e~ to R.H.S. of Eq. (e),

w
we have.
2Br-^ Br2 + 2 e ...(/●)
This represents balanced oxidation half equation.

F lo
Step 5. To balance the number of electrons gained in Eq. (d) and lost in Eq. (/), multiply Eq. (d) by 2 and
Eq. (/■) by 5 and add together, we have,
2MnO4+l6H++10e" ^ 2 Mn2+ + 8 H2O

ee
10 Br 5 Br2 + 10 e"

Fr
2MnO4+10Br"+16H+ ●> 2 Mn2+ + 5 Br2 + 8 H2O
This represents the correct balanced ionic equation.

for
our
MATCHING TYPE QUESTIONS
27. Match Column I with Column II for the oxidation states of the central atoms.
Column I Column II
s
ook
(0 Cr20?" (a) +3

(«) Mn04 (b) + 4


Y
eB

m VO3 ic) + 5

(IV) FeFj- id) + 6

ie) + 7
r
ou
ad

Ans. (0 O.N. of Cr in Cr207 = +6.

(«) O.N. of Mn in MnO^ = +7 .


Y

(Hi) O.N. of V in VO3 = jc + 3 {- 2) = -I or = + 5.


Re

(/v) O.N. of Fin FeF^ = jc + 6 (-1) =-3 or r = + 3.


nd

Thus, correct matching is : (i)^(d) ; (»)->(e); (m)->(c) ; (iv)->(«3).


Fi

28. Match the items in Column I with relevant items in Column II.
Column I Column II

0) Ions having positive charge (a) + 1

in) The sum of oxidation number of (b) -1


all atoms in a neutral molecule

m Oxidation number of hydrogen ion (H"*") (c) + 1

(/V) Oxidation number of fluorine in NaF id) 0

(V) Ions having negative charge ie) Cation

if) Anion

Ans. (0^(c); (»)-»(</); ini)~^ic); iiv)^(b); (v)->(/).


REDOX REACTIONS 8/113

ASSERTION AND REASON TYPE QUESTIONS

In each of the following questions, a statement of Assertion (A) is given followed by a corresponding
statement of Reason (R) just below it. Of the following options, mark the correct choice as
(fl) Both A and R are true and R is the correct explanation of A.
(b) Both A and R are true but R is not the correct explanation of A.
(c) A is true but R is false.
(d) Both A and R are false.

low
29. Assertion (A) : Among halogens fluorine is the best oxidant.
Reason (R) : Fluorine is the most electronegative atom.
Ans. (b) Correct explanation. Among halogens F2 is the best oxidant because it has the highest F® value.
30. Assertion (A) : In the reaction between potassium permanganate and potassium iodide, permanganate
ions act as oxidising agent.
Reason (R) : Oxidation state of manganese changes from +2 to +7 during the reaction.

e
Ans. (c) Correct R. Oxidation state of Mn decreases from +7 to +2.

re
rF
31. Assertion (A) : The decomposition of hydrogen peroxide to form water and oxygen is an example of a

F
disproportionationreaction.
Reason (R): The oxygen of peroxide is in - 1 oxidation state and it is converted to zero oxidation state in
O2 and - 2 oxidation stale in H2O.

r
Ans. (a) R is the correct explanationof A.

fo
32. Assertion (A): Redox couple is the combinationof oxidised and reduced fonn of a substance involved in
ou
an oxidation or reduction half cell. ks
I

Reason (R): In the representation e® and E®


Cu^+ZCu ’
Fe^'*'/Fe^^ and Cu^‘‘'/Cu are redox couples.
Fe^'^/Fe2+
oo
Ans. (a) R is the correct explanation of A.
Y
eB

LONG ANSWER QUESTIONS


33. Explain redox reactions on the basis of electron transfer. Give suitable examples.
Ans. Refer to Art. 8.3, pages 8/3-875.
ur

34. On the basis of standard electrode potential values, suggest which of the following reactions would
take place ? (Consult the book for E® value).
ad
Yo

(i)Cu + Zn2+ — ^ Ca^* + Zn


(h) Mg + Fe^'*' — Mg^"*" + Fe
(m) Brj + 2 Cl" CI2 + 2 Br
(iV) Fe + Cd2+- ^ Cd -H Fe^+
d
Re

Ans. (i) From the Table 8.1, page 8/44.


in

E° = +0-34V and E“ Zn‘^*IZn = -0-76V


Cu2+/Cu
F

The reaction, Cu + Zn^''’ > Cu^"^ + Zn is feasible if E%gjj is +ve. In this cell, Cu gels oxidised to Cu^'*',
therefore, Cu^^/Cu redox couple acts as the anode. Further, since Zn^'*' gets reduced to Zn, therefore,
Zn^'''^n acts as the cathode.
-F® = E®
cathode anode Cu^'^/Cu ^ Zn^+/Zn = _ 0-76 - 0-34 = - MOV

Since of the reaction, Cu + Zn^'*’ > Cu^'*' + Zn is -ve, therefore, this reaction does not occur.

(I'O From Table 8.1, page 8/44, E® Mg2+/Mg = -2-37V and E®


Fe^'*'/Fe
= -0-74V.

The reaction, Mg + Fe^'*’ > Mg“'*' + Fe is possible, if is +ve. Here, Mg“'*’/Mg redox couple acts as
the anode while Fe^'*'/Fe acts as the cathode.
8/114 7>n4zdee^’^ New Course Chemistry (Xl)BBXaD

E“ceU = E“ -E" anode = E“ -E° = _ 0-74-(-2-37) = + 1-63 V


cathode Fe^+ZFe
Since E“ cell is +ve, therefore, the reaction, Mg + Fe^"^ —> Mg^'*’ + Fe will take place.
(in) From Table 8.1, page 8/44, E“ = +108V and E" = +l-36V.

ow
Br-ZBrj cr/cij
The reaction, Br2 + 2 Cl“ > CI2 + 2 Br“ will occur if E°j,gj| is +ve. In this cell, Br2/Br~ redox couple will
act as the cathode and Cl2/Cr will act as the anode.
E“ cell = E° -E“ anode = E° -E“ = + 1 08- 1-36 = -0-28 V
cathode
Br-jZBr CI2ZCI-
Since E®j.gn is -ve, therefore, the redox reaction Br2 + Ct~ » C/2 + 2 Br will not occur.

e
(iv) From the Table 8.1, page 8/44, E® Fe^+ZFe =

-0-74V and E®
Cd^+zCd
= -044V

re
The reaction, Fe + Cd^"^ > Cd + Fe^"*" will occur if E®^cell is +ve. Here, in this reaction, Fe^‘*‘/Fe electrode
will act as the anode while Cd^'*'/Cd electrode will act as the cathode.

Frl
F
E® cell = E®
^ Fe^+ZFe = _ 0-44 - (- 0-74) = + 0-30 V
—170 — 170 _ T70
cathode ^ anode ^ Cd^"** /Cd
Since E® cell is +ve, therefore, the redox reaction, Fe + Cd^'*’- > Cd + Fe^'*’ will occur.
ou
35. Why does fluorine not show disporportionation reaction ?

sor
Ans. Refer to Art. 8.4.6, page 8/20.
36. Write redox couples involved in the reactions (i) to (iv) given in question 34.
Ans. (/) Cu2+/Cu and Zn^+ZZn ; («) Mg^+/Mg and Fe^+ZFe; (Hi) Br^/Br and CI2/CI-; (iv) Fe^+ZFe and Cd^+ZCd.

kf
37. Find out the oxidation number of chlorine in the following compounds and arrange them in increasing
order of oxidation number of chlorine.
oo
NaC104, NaClOj, NaClO, KCIO3, CljOy, CIO3, CI2O, NaCl, Cl2, ClOj.
Which oxidation state is not present in any of the above compounds?
Y
Ans. Let the O.N. of Cl in these compounds be x.
B

O.N. ofClinNaC104 + 1 + jc + 4(-2) = 0 or x = + 7

O.N.ofClinNaC103 + 1 +jc + 3(-2) =0 or x = + S


re

O.N.ofClinNaC10 + 1 +X+ 1 (-2) =0 or JC = +1


oY

O.N.ofClinNaC102 + 1 +x + 2(-2) =0 or jc = + 3
u

O.N.ofClinCl207 + 2jc + 7(-2) = 0 or jcs + 7


ad

O.N. ofClinC103 X + 3 (- 2) = 0 or x= + 6

O.N.ofClinCl20 2x+l (-2) = 0 or x = + l


d

O.N. of Cl in NaCl +1+x=0 or x = -l

O.N. of Cl in CI2 2x = 0 x= 0
in

or

O.N.ofClinC102 jc + 2 (- 2) = 0 x = + 4
Re

or

None of the compounds have an O.N. of + 2.


F

Increasing order of O.N. of Cl is : - 1, 0, + 1, + 3, + 4, + 5, + 6, + 7.


38. Which method can be used to And out strength of reductant/oxidant in a solution ? Explain with an
example.
Ans. The method used to find out the strength of reductant/oxidant in solution is to connect the redox couple of
that species to normal hydrogen electrode and measure its electrode potential. If it is +ve, the electrode acts
as the reductant and if it is negative it acts an oxidant Compare the values and determine their comparative
strength as an oxidant or reductant.
REDOX REACTIONS 8/115

■ IJi NEET/JEE
SPECIAL

w
For ultimate preparation of this unit for competitive examinations, students should refer to

F lo
● MCQs in Chemistry for NEET
Pradeep's Stellar Series.... ● MCQs in Chemistry for JEE (Main) .

separately available for these examinations.

ee
Multiple Choice Questions correct Answer)

Fr
I. Oxidation-Reduction-Electronic (b) XeFg + 2 H2O —> XeO^Fj + 4 HF

for
and Oxidation Number Concept (c) XeF4 + 02p2- XeF^ + O2
ur
1. Iodine reacts with cone. HNO3 to yield Y along (J)XeF2 + PF5 — ^ [XeF]+ PF^
with other products. The oxidation state of iodine (JEE Main 2017)
s
in Y, is
ook
5. The redox reaction among the following is
Yo
(a) 5 ib) 1
(a) reaction of [Co(H20)g]Cl3 with AgN03
(c) 7 id) 3 (JEE Main 2019)
(b) formation of ozone from atmospheric oxygen
eB

2. The correct order of N compounds in its decreasing in the presence of sunlight


order of oxidation states is
(c) combination of dinitrogen with dioxygen at
(a) HNO3, NO, NH4CI, N2 2000 K
r

(b) HNO3, NO, N2, NH4CI (d) reaction of H2SO4 with NaOH
ou
ad

(c) HNO3, NH4CI, NO, N2 (JEE Main 2020)


(d) NO, HNO3, NH4CI, N2 6. Hot concentrated sulphuric acid is a moderately
Y

(IIT Paper I, 2012 ; NEET 2018) strong oxidizing agent. Which of the following
3. Which of the following processes does nor involve reactions does not show oxidizing behaviour ?
Re
nd

oxidation of iron ?
(a) Cu + 2H2SO4 ^ CUSO4 + SO2 + 2H2O
(a) Formation of Fe(CO)5 from Fc (b) S + 2H2SO4-> 3SO2 + 2H2O
Fi

(b) Liberation of H2 from steam by iron at high (c) C + 2H2SO4 -> CO2 + 2SO2 + 2H2O
temperature
(c) Rusting of iron sheets id) Cap2 + H2SO4 ^ CaS04 + 2HF
(NEET Phase - U 2016)
id) Decolourisation of blue CUSO4 solution by
iron. {AIPMT 2015) 7. In the neutral or faintly alkaline medium, KMn04
4. Which of the following is an example of redox oxidizes iodide into iodate. The change in
reaction ? oxidation state of manganese in this reaction is
from
(a) XeFg + H2O XeOF4 + 2 HF
ANSWERS

1. (a) 2.(b) 3. (a) 4. (c) 5. (c) 6.(^0


8/116
Nf.vi Course Chemistry rXT1 orgm

(a) + 7 to + 4 (/j) + 6 to + 4 (c) disproportionation reaction ; - 3 and + I


(c) + 7 to + 3 (<f) + 6 to + 5 {d) disproportionation reaction ; - 3 and + 3
(NEET 2022) {IIT Paper II, 2012)
8. Oxidation number of potassium in K-,0, K2O2 and 15. Which of the given reaction is not an example of
KO2. respectively, is disproportionation reaction ?
(fl) + 1, + 1 and + 1 {b) + 2,+ I and + 1/2
(c) + 1, + 2 and + 4 (d) + 1, + 4 and + 2 (a) 2 H2O2 > 2 H2O + O2
(JEE Main 2020) (b) 2 NO2 + H2O > HNO3 + HNO2
9. The dark purple colour of KMn04 disappears in (c) MnO-+4H-^+3e" ^ Mn02 + 2 H2O
the titration with oxalic acid in acetic medium.
(d) 3Mn02“+4H+ —

w
■>
The overall change in the oxidation number of
manganese in the reaction is
2Mn0"+Mn02+2H20
(a) 5 (b) 1 (JEE Main 2022)

F lo
(c)7 (d) 2 (JEE Main 2022) 16. Which of the following reaction is metal
10. What is the change in oxidation number of carbon displacement reaction ? Choose the right option.
in the following reaction ? (a) 2 Pb(N03)2 > 2 PbO + 4 NO2 + O2

ee
CH4 (g) + 4Cl2 (g) -^CCl4 (/) + 4HCl (g)
(b) 2 KCIO3 ^ 2 KCl + 3 O2

Fr
(a) 0 to + 4 (i7) - 4 to + 4
(c) 0 to - 4 (</) + 4 to + 4 (c) Cr,03 + 2 A1 AI2O3 + 2 Cr
(NEET 2020) (^Fe + 2HC1- > FeCl2 + H2

for
11. The oxidation number of the underlined atom in (NEET 2021)
the following species, is
ur
17. (A) HOCl + H2O2 ^ H3O+ + cr + O2
(a) H^CI^ is+ 3 (b) Cu2 O is - 1 (B) I2 + H2O2 + 2OH- — ■> 2 r + 2 H2O + O2
s
(c) aoj is+ 5 Choose the correct option.
ook
(d) Kj Cr^ Oj is + 6
Yo
(NEET 2020)
(a) H2O2 acts as reducing and oxidising agent
respectivelyin equations (A) and (B)
eB

12. Oxidation number of the product when Cr^O^” (b) H2O2 acts as reducing agent in equations (A)
is reacted with OH“ is
and (B)
(«) +4 ib) + ^
(c) H2O2 acts as an oxidising agent in equations
our

(c) + 6 id)0 (JEE Main 2021)


ad

(A) and (B)


13. 3 CIO- (aq) ■> CIOj + 2 cr is an example of id) H2O2 acts as oxidising and reducing agent
(a) oxidation reaction respectively in equations (A) and (B)
Y

(b) reduction reaction (JEE Main 2021)


Re

(c) disproportionation reaction 18. Small quantities of solution of compounds TX,


nd

id) decomposition reaction TY and TZ are put into separate test tubes
(Karnataka CET 2017) containing X, Y and Z solution. TX does not react
Fi

14. The reaction of white phosphorus with aqueous with any of these. TY reacts both with X and Z.
NaOH gives phosphine along with another TZ reacts with X. The decreasing order of ease of
phosphorus containing compound. The reaction oxidation of the anions X“, Y' and Z" is
type ; the oxidation states of phosphorus in (a) Y", Z“, X- ib) Z-, X", Y-
phosphine and the other product are resi>ectively (c) Y-, X", Z- id) X". Z-, Y-
ia) redox reaction ; - 3 and - 5
(DCE 2009)
ib) redox reaction ; + 3 and + 5
ANSWERS
7. ia) 8.ia) 9.(a) 10. ib) 11. ib) 12.(c) 13.(c) 14.ic) 15.(c) 16.(r)
17. ib) 18. ia)
REDOX REACTIONS 8/117

19. In aqueous alkaline solution, two electron 26. If the molecular wt. of Na2S203 and I2 are Mj
reduction of HO^ gives and M2 respectively, then what will be the equiva
lent wt. of Na2S203 and I2 in the following reaction ?
(fl) HO- ib) H2O
(c) O2 id) O" 2S2O5-+I2 ^ S4O2- + 2I
(West Bengal JEE 2015) {<3)M,,M2 (/?)M],M2/2
20. In neutral or alkaline solution, MnO^ oxidises {c)2M,,M2 (rf)Mi,2M2
thiosulphate to 27. Consider the following reaction,

w
(a) Sp^~ ib) S2O2 xUnO^ + yCpj-+ :
2+
+ 2 y CO2 + 4/2 H2O
.rMn
(c) S02- id) soj- The values of x, y and z in the reaction are,

Flo
(JEE Main 2022) respectively
21. On reaction with stronger oxidising agent like (a) 5, 2 and 8 ib) 5, 2 and 16
KIO4, hydrogen peroxide oxidises with evolution (c) 2, 5 and 8 id) 2, 5 and 16

ee
of O2. The oxidation number of I in KIO4 changes (JEE Main 2013)
to

Fr
28. For the redox reaction,
ia)-l (^) + 3 2+

(c) + 5 (^+1
Mn04+C20j"+H-^ > Mil + CO2 + H2O
The correct coefficients of the reactants for the
(JEE Main 2022)

for
ur
22. The compound that cannot act both as oxidising
balanced equation are

and reducing agent is MnO~ Cpl~ H"-


ia) HNO2 ib) H3PO4 (a) 16
s 5 2

(c) H2SO3 id) HjOj


ok
ib) 2 5 16
Yo
(JEE Main 2020) ic) 2 16 5
o
23. The species given below that does not show (d) 5 16 2 (NEET 2018)
eB

disproportionation is 29. Consider the titration of potassium dichromate


(a) BrO ib) BtQ- solution with acidfied Mohr’s salt solution using
dimethylamine as indicator. The number of moles
ic) BrOj (d) BrO-
of Mohr’s salt required per mole of dichromate is
r
ou
ad

(JEE Main 2021) (a) 3 (b)4


24. The standardization of Na2S203 using K2Cr202 ic)5 id) 6
Y

by iodometry, the equivalent weight of K2Cr20y (IIT Paper II, 2007)


IS
30. In order to oxidise a mixture of one mole of each
ia) (molecular weight)/2
nd

of FeC204, Fc2(C204)3, FeS04 and Fe2(S04)3 in


Re

ib) (molecular weight)/6


acidic medium, the number moles of KMn04
ic) (molecular weight)/3 required is
Fi

id) None of these ia) 2 ib) 1-5


25. 1 mole of FeS04 (atomic weight of Fe is 55.84 g ic) 3 id) 1 (JEE Main 2019)
mol"^) is oxidized to Fe2(S04)3- Calculate the 31. 3-92 g of ferrous ammonium sulphate are
equivalent weight of ferrous ion dissolved in 100 ml water. 20 mL of this solution
ia) 55.84 ib) 27.92
requires 18 mL of potassium permanganate during
ic) 18.61 (f/) 111.68 titration for complete oxidation. The weight of
ie) 83.76 (Kerala PET 2018) KMn04 present in one litre of the solution is
ANSWERS
19.ia) 20.id) 21.(c) 22. ib) 23.{d) 24.(*) 25.(a) 26.ib) 27.id) 28. ib)
29. id) 30.ia)
8/118
New Course Chemistry (XI)SSEIHD

(a) 34-76 g (h) 12-38 g II. Electrode Potential


(c) 1-238 g (d) 3-476 g
37. A standard hydrogen electrode has zero electrode
32. MnO^ ions are reduced in acidic condition to potential because
Mn^"*" ions whereas they are reduced in neutral ((/) hydrogen is easiest to oxidize
condition to Mn02- The oxidation of 25 arL of a (b) this electrode potential is assumed to be zero
solution X containing Fe~ ions required in acidic
(c) hydrogen atom has only one electron
condition 20 mL of a solution Y containing
id) hydrogen is the lighest element.
MnO;^ ions. What volume of solution Y would
38. Zinc can be coated on iron to produce galvanized
be required to oxidise 25 mL of solution X iron but the reverse is not possible. It is because

w
containing Fe-'*’ ions in neutral condition ? (r/) zinc is lighter than iron
(a) 11 -4 mL (b) 12-0 mL (b) zinc has lower melting point than iron
(c) 33-3 niL (d) 35-0 mL (c) zinc has lower negative electrode potential

F lo
(e) 25-0 mL than iron
(Kerala PET 2008)

33. Ceric ammonium sulphate and potassium (d) zinc has higher negative electrode potential
than iron (MEET Phase - II 2016)
permanganate are used as oxidising agents in

ee
acidic medium for oxidation of ferrous ammonium 39. If a half cell, A -f e~ ■¥ A has a large negative
potential, it follows that

Fr
sulphate to ferric sulphate. The ratio of number of
moles of cerium ammonium sulphate required per (a) A is easily reduced {b) A is readily oxidised
mole of ferrous ammonium sulphate to the number (c) A" is readily reduced (d) A~ is readily oxidised

for
of moles of KMn04 required per mole of ferrous 40. A gas X at 1 atm is bubbled through a solution
ammonium sulphate is containing a mixture of IM Y“ and IM Z" ions at
ur
ia)5:\ (/^) 1:1 25°C. If the reduction potentials of Z > Y > X, then
(c)l:2 (a) Y will oxidize X but not Z
(d)2-A (DCE 2009)
s
(b) Y will oxidise both X and Z
ook
34. How many tnL of 0-125 M must be reacted
Yo
(c) Y will oxidise Z but not X
with 12-00 mL of 0-200 M MnO^ if the redox
(d) Y will reduce both X and Z
eB

products are Cr207~ and ? (AMU Med. 2009)


(a) 8 mL (b) 16 mL 41. Given E°
C'r^'^/Cr =-0-74V; e! 2+ = 1-51 V;
(c) 24 niL (d) 32 mL (AHMS 2013) MnO^ /Mn
our
ad

35. Assuming complete ionization, same moles of the E” = 1-33 V ; e" = 1-36 V
ci2/cr
following compounds will require the least amount
of acidified KMn04 for complete oxidation ? Based upon the data given above, strongest
oxidising agent will be
(a) FeSOg (b) FeC204
dY

(c) Fe(N02)2 (d) FeS04 (a) Mn04 (b) CI-


Re

(RE-AIPMT 2015) (c) Cr^+ (d) Mn2+


(JEE Main 2013)
Fin

36. N^(g) + 3H^Jg) ^ 2NH3(g)


20g 6g 42. Given that e: = + 123 V
O^/HjO
Consider the above reaction, the limiting reagent
= 2-05V ;
of the reaction and the number of moles of NH3 S20I-/S05-
formed respectively, are :
E° = + i-09v ;
(a) H2, 1-42 moles (b) H2, 0-71 moles Br,/Br"

(c) N2, 1-42 moles (d) N2, 0-71 moles E“ =I-4V


(JEE Main 2022) Au^'''/Au

ANSWERS

31. id) 32. (c) 33. (fl) 34. id) 35. id) 36. (c) 37. ih) 38. id) 39. (^0 40. (a) 41. (a)
REDOX REACTIONS 8/119

The strongest oxidising agent is (fl) X > Y > Z {b)Y>Z>X


(c) Y > X > Z (({)Z>X>Y
(a) s^ol" ib) 0,
(AIPMT Prelira 2011)
3+
(c) Br2 id) Au 48. On the basis of the following E“ values, the
(JEE Main 2019) strongest oxidizing agent is
43. The standard reduction potential at 298 K for the LFe(CN)^' [Fe(CN)6]^- +; E° = -0-35 V
following half cell reactions are : Fe
2+
E° = -0-77 V
Zn^-^ + 2e~ ^ Zn is) ; E" = - 0-762 V (fl) Fe^-^ ib) [Fe(CN)6]3-
Cr^+ ia(j) + 3e- -> Cr (x); E" = - 0-740 V ic) [Fe(CN)6l
4_
id) Fe“+ (AIPMT 2008)
2 H+ iaq) + 2 e~ H2 (g); E" = 0-0 V 49. Standard reduction potentials of the half reactions

w
F2 + 2e^- ^21^ iag) = 2-87 V are given below :
which of the following is the strongest reducing F2 (g) + 2e ~ 2 F* iac/) ;E° = + 2-85 V
agent Cl2ig)+2e- 2 cr iaq) ; E'^ = + 1-36 V

F lo
ia) Cr (i) ib) Zn is) Br, (g) + 2 e" 2 Br-(a^) ; E“ = + 1-06 V
(c)H2 ig) id) F2 (g) i2(g)+2e-- ^2 1- (fl^) : E° = + 0-53 V
(Karnataka CET 2017) The strongest oxidising and reducing agents
respectivelyare

ee
44. Given :
ia) F, and I ib) Br-> and Cl

Fr
O

E. = l-36V, E Cr^-^/Cr = -0-74 V


a2/cr (c) CI2 and Br~ id) CI2 and 12
(AIPMT Main 2012)
V?-/c,3^=1-33V, e;Mn04/Mn2+ = 1-51 V

for
III. Electrolysis and
Among the following, the strongest reducing agent is Electrochemical Cells
ur
(a) Cr^-^ (b) Cl-
(c) Cr id) Mn^+ 50. The products formed when an aqueous solution
oks
of NaBr is electrolysed in a cell having inert
(JEE Main 2017, AIIMS 2013)
Yo
electrodes are
45. The correct order of reduction potentials of the
o

ia) Na and Br2 ib) Na and O2


eB

following pairs is
ic) H2, Br2, NaOH id) H2 and O2
I.CI2/CI- 11.12/1“ III. Ag-^/Ag
IV. Na+/Na V. Li+/Li
51. If electrolysis of aqueous CUSO4 is carried out
using Cu-electrodes, the reaction taking place at
our

ia) 1 > III > II > IV > V (/?) I > II > III > IV > V
ad

the anode is
ic) I > III > II > V > IV ((/) I > II > III > V > IV (u) + e~ ■> H

(JEE Main 2022) <b) Cu--^ iaq)-h 2 e~ ^ Cu (i)


46. Standard electrode potential values :
dY

Fe^+ZFe, E" = - 0-44 V, Fe^VFc-\ E“ = 0-77 V


(c) S02-(a^)-2^- >S04
Re

If Fe-'*', Fe^'*' and Fe blocks are kept together, it id) Cuis)-2e^ — ^ Cu^+ iaq)
will lead to (West Bengal JEE 2019)
Fin

ia) increase in Fe^'*' 52. Which of the following statements is/are true for
an electrochemical cell ?
ib) decrease in Fe^'*'
ic) Fe^'*'/Fe^'*' remains unchanged ia) Oxidation occurs at the anode only
id) Fe^'*' decreases (Bihar CECE 2015) ib) Reduction occurs at the anode only
(c) Oxidation occurs at both the anode and cathode
47. Standard electrode potential of three metals X, Y
and Z are - 1 -2 V, + 0-5 V and - 3-0 V respectively. id) Reduction occurs at both the anode and catliode
The reducing power of these metals will be (J & K CET 2018)

ANSWERS

42. (a) 43. ib) 44. (c) 45. (a) 46. (b) 47. id) 48. (o) 49. ia) SO. (c) 51. id) 52. ia)
8/120
New Course Chemistry (XI)BZs2SI]

53. Consider the following relations for EMF of an 56. The emf of the cell involving the following
electrochemical cell reaction

(0 EMF of cell = (Oxidation potential of anode) 2Ag-^ + H2 >2Ag + 2H+


- (Reduction potential of cathode) is 0*80 volt. The standard oxidation potential of
(//) EMF of cell = (Oxidation potential of anode) silver electrode is
+ (Reductionpotential of cathode) (a) - 0-80 volt (b) 0-80 volt
(Hi) EMF of cell = (Reduction potential of anode) (c) 040 volt (J) - 0 40 volt
+ (Reduction potential of cathode) (AMU Engg. 2011)
(/v) EMF of cell = (Oxidation potential of anode) 57. The standard oxidation potentials of Zn, Cu, Ag
- (Oxidation potential of cathode) and Ni electrodes are + 0-76, - 0-34, - 0-80 and

w
Which of the above relations are correct + 0-25 V respectively. Which of the following
(a) (///) and (/) (b) (0 and (i7) reaction will provide maximum voltage ?

F lo
(c) (m) and (iv) id) (ii) and (/V) (a) Cu + 2 Ag+ (aq) -> Cu-+ iaq) + 2 Ag
(AIPMT Main 2010) (b) Zn + 2Ag+(a^) —> Zn"''' iaq) + 2 Ag
54. If the half cell reactions are given as (c) H2 + Ni2+(fl^)- ^ 2 H''' iaq) + Ni
(0 Fe-+ iaq) + 2 e~ ^ Fe (s); E“ - - 0-44 V (d) Zn + Cu^"*" (aq)- Zn"'*' iaq) + Cu

ee
(i7) 2 iaq) + 1/2 O2 (g) + 2e~ > H2O (/) ; (e) Zn + 2W- (aq)- -> Zn^'*' iaq) + H2.

Fr
E° = + 1-23 V
(Kerala PET 2007)
The E® for the reaction.
58. At 298 K, the standard electrode potentials of

for
Fe is) + 2 m (aq) + 1/2 O2 (g) > Cu^'*'/Cu, Zn^'*'/Zn, Fe^'*'/Fe and Ag'^'/Ag are

Fe2+(fl^) + H2O (/) is 0-34 V, - 0-76 V, - 0-44 V and 0-80 V respectively


ur
(a)+ I-67 V ib)- 1-67 V On the basis of standard electrode potentials,
(c) + 0-79 V id) - 0-79 V predict which of the following reaction cannot
s
55. Two electrochemical cells, occur ?
ook
Yo
Zn I Zn2+ 11 Cu^* I Cu and Fe I Fe^''' I! Cu^''' I Cu
(fl) CUSO4 (aq) + Zn is) ZnS04 iaq) + Cu (5)
are connected in series. What will be the net e.m.f.
ib) CUSO4 iaq) + Fe (j)-
eB

of the cell at 25°C ?


FeS04 (aq) + Cu (j)
Given : Zn^''' I Zn = - 0-76 V, (c) FeS04 iaq) + Zn (5) -> ZnS04 (aq) + Fe (^)
Cu^-^ I Cu = + 0-34 V. Fe2+ I Fe = - 041 V (^0 2 CUSO4 iaq) + 2 Ag (s)
our

(a) + 1-85 V (fc)- 1-85 V 2 Cu (5) + Ag2S04 (aq)


ad

(c) + 0-83 V (d) - 0-83 V (NEET 2022)

m[ Multiple Choice Questions (with One or More than One Correct Answers);
Y
Re

59. Consider the redox reaction, (a) Both CU2O and CU2S are reduced
nd

2820^- +I2 > 840^- +21" (b) Only CU2S is reduced


(c) CU2S is the oxidant
Fi

(a) 820^“ gets reduced to S40g“ (d) Only CU2O is reduced


61. For the reaction :
(b) S2O3 gets oxidised to S40g“
(c) I2 gets reduced to I” (d) I2 gets oxidised to I”. r+C10j+H2S04 + CI- + HSO4+I2
60. Which of the following statements about the The correct statement(s) in the balanced equation
following reactions are wrong ? is/are

2 CU2O (s) + CU2S (s) ^ 6 Cu (s) + SO2 (g) (a) stoichiometric coefficient of HSO^ is 6
ANSWERS
53. (d) 54. ia) 55. (a) 56. (a) 57.(b) 58. id)
59. (b.c) 60. (b.f.d)
REDOX REACTIONS 8/121

(b) iodide is oxidized


(a) HNO2 ib) H2O2
(c) sulphur is reduced
(c) H2S (d) SO2.
(d) H2O is one of the products
63. Which of the following substances undergo
(JEE Advanced 2014) disproportionation reactions under basic medium ?
62. Which of the following act both as oxidising as (a) F2 (b) P4
well as reducing agents ? (c) S $ id) Bt2

il Multiple C&ioice Questions (Based on the given Passage/Comprehen


i;it h)

The comprehension given below is followed by some multiple choice questions. Each question has

w
one
correct option. Choose the correct option.
[compreiT^nsionlfl The coefficients of x and y are

F lo
The concept of oxidation
number (O.N.) is very important in under (a) 2 and 3 (b) 2 and 6
standing redox reactions. It helps to identify (c) 1 and 3 (d) 3 and 8.
the oxidant/reductant in a redox reaction. It 67. For the reaction,
also helps to (1) find out the possible molecular

ee
formula of any neutral compound and (h) to + Mn04 - ^ MOJ + Mn2+ + 1/2 O2

Fr
balance redox reactions.
if one mole of MnO^ oxidises T67 moles of
to MO^ , then the value of x in the reaction is
64. A mole of hydrazine (N2H4) loses ten moles of

for
ia)5 ib)3
electrons to form a new compound X. Assuming (c)2 id) 1.
that all the nitrogen appears in the new compound,
r
what is the oxidation state of nitrogen in X ? (there (^^omprerier^iSnlH Bleaching
You
powder and
is no change in the oxidation number of hydrogen
s
in the reaction.)
bleach solution are produced on a large scale
ook

and used in several household products. The


(a)~\ ib)~3 effectiveness of bleach solution is often
measured iodometry.
eB

(c)+3 id)+5
(^)+l. (Kerala PET 2016) 68. Bleaching powder contains a salt of an oxoacid as
65. A compound contains atoms of three elements A, one of its components. The anhydride of that
our
ad

B and C. If the oxidation number of A is + 2, B is oxoacid is


+ 5 and that of C is - 2, the possible formula of ia) CI2O
the compound is ib) CI2O7
(c) CIO2 id) CUOg
ia) A3(BC4)2
dY

69. 25 mL of household bleach solution was mixed


ib) A3(B4C)2
Re

with 30 mL of 0-50 M KI and 10 mL of 4 N acetic


(c) ABC2
acid. In the titration of the liberated iodine,
Fin

id) A3(BC3)2- 48 mL of 0-25 N Na2S203 was used to reach the


66. When copper is treated with a certain end point. The molarity of the h lusehold bleach
concentrationof nitric acid, nitric oxide and solution is
nitrogen dioxide are liberated in equal volumes ia) 0-48 M ib) 0-96 M
according to the equation,
(c) 0-24 M id) 0-024 M
xCu+y HNO3 > Cu(N03)2 + no + NO2
(UT Paper n, 2012)
+ H2O.

ANSWERS
61. ia,h,d) 62. ia.b.d) 63. (h,c,d) 64. (r) 65. (n) 66. (h) 67. (c)
68. (rt) 69.(c)
8/122 New Course Chemistry (XI)EZsIHII

m Matching Type Questions


Match the entries of column I with appropriate entries of column II and choose the correct option out
of the four options (a), (b), (c), (d) given at the end of each question.
70. Column 1 Column II

(A) Device in which chemical energy is converted into (P) Electrolysis


electrical energy.
(?) Redox titrations
(L) Device which keeps electrical neutrality in two half
reactions in an electrochemical cell.

w
ir) Electrochemical cell
(C) A process in which electric energy is used to bring about
decomposition of an electrolyte.
Process which involves reactions between oxidising and reducing is) Salt bridge

F lo
(D)
agents to estimate the amount of unknown substances in solution,
(a) A-r, Bs, C-p, D-q {b) A-r, B-5, C-q, D-p (c) As, B-p, C-r, D~c{ {d) A-p, B-r, Cs, D-q

ee
71. Column I (Compound) 1 olumn II (Oxidation numbers of S atoms)

Fr
ip) + 1,+ I
(A) N32S2
(?) - 1,- 1
(B) Na2S203

for
ir) + 6, + 6
(C) Na2S207
is) -2,+ 6
(D) S2C12
ur
(a) A-r, B-/?, C-5, D-^ {b) A-p, B-j, C-q, D-r (c) A-^, Bs, C-r, D-p id) As, B-r, C-p, D-q
oks
72. Column I ('oiumn II
Yo
(A) KMn04 ip) Used in salt bridge
o

Used as an oxidising agent


eB

(B) SnCl2 (?)


(C) ZnS04 ir) Used as a reducing agent

(D) K2SO4 is) Used as electrolyte in Daniel! cell


our

(c) A-r, B-^, C-p, D-5 (d) A-r, B-s, C-q, D-p
ad

(a) A-q, B-r, C-p, D-s ib) A-q, B-r, C-s, D-p
p q

K9 Matrix-Match Type Questions A'


© O©
Y

B
0® ©
Re

Match the entries of column I with appropriate entries of column II. Each
nd

entry in column I may have one or more than one correct option from column II. ^ © O0
If the correct matches are A-p, s ; B-r ; C-p, q ; D-s, then the correctly bubbled
Fi

4x4 matrix should be as follows : D


0 0IO ©
Column II
73. Column I

(A) Metal which reacts with dilute acids to give H2 gas (p) Zinc

(B) Metal whose container can be used to store cone. HNO3 {q) Silver

(C) Metal which is used as an electrode in Daniell cell (r) Aluminium

(D) Metal which does not react with dilute acids to give H2 gas (a-) Copper
ANSWERS
70.(0) 71. (c) 72. (i?) 73. iA-p.r ; B-r ; C-p.s ; D-q,s)
REDOX REACTIONS 8/123

74. Column I Column II


(A) CUSO4 + Zn > Cu + ZnS04 (P) Non-metal displacement reaction
(B) 2 KCIO3 — > 2 KCl + 3 O2 Disproportionation reaction
(C) 3 CI2 + 6 OH- > 5 C|-+ CIOJ + 3 H2O ir) Decomposition reaction
(D) CI2+2KI » 2 KCI + I2 is) Redox reaction

75. Column I
Column II
(A)

w
Electrode on which reduction occurs ip) Anode
(B) Electrode on which oxidation occurs iq) Cathode
(C) Electrode connected to positive pole of the battery ir) Negative pole
(D) Electrode connected to negative pole of the battery

Flo
is) Positive pole

VI. A B C 0
Integer Type Questions

ee
DIRECTIONS. The answer to each of the following questions is
@ ®®©

Fr
a single digit
integer, ranging from 0 to 9. If the correct answers to the question numbers ® ®®®
A, B, C and D (say) are 4,0, 9 and 2 respectively, then the correct darkening
of bubbles should be as follows : ® ®®®

for
ur
76. The value of n in the molecular formula Be„ Al2SigO,g is (IIT Paper I, 2010) ® ®®®
77. Among the following, the number of elements showing only one non-zero ® 00©
oxidation state is O, Cl, F, N, P, Sn, TI, Na, Ti.
ks
(IIT Paper II, 2010)
78. The difference in the oxidation numbers of the two types of sulphur atoms in ® ®©®
Yo
Na2S405 is
oo
(IIT Paper I, 2011)
79. How many moles of electrons are involved in the conversion of 1 mole of
00®®
eB

Crp2- ions to Cr^'*' ions ?


©®©©
80. The oxidation number of Mn in the product of alkaline oxidative fusion of Mn02
IS. © ®©©
r

81. How many moles of iodine are liberated when 2 moles of potassium permanganate react with potassium
ou
ad

iodide ?

82. Reaction of Br2 with Na2C03 in aqueous solution gives sodium bromide and sodium bromate with evolution
Y

of CO2 gas. The number of sodium bromide molecules involved in the balanced chemical equation is
(IIT Paper I, 2011)
nd
Re

83. Consider the following list of reagents : Acidified K2CT2OJ, alkaline KMn04, CUSO4, H2O2, CI2, O3, FeCl3,
HNO3 and Na2S203. The total number of reagents that can oxidise aqueous iodide to iodine is
Fi

(JEE Advanced 2014)


84. In the titration of KMn04 and oxalic acid in acidic medium, the change in oxidation state of carbon on the
end point is 2022)
85. The oxidation state of man anese in the product obtained in a reaction of potassium permanganate and
hydrogen peroxide in basic .ledium is

ANSWERS
74. (A-j ; B-r,s ; C-q,s : D-p,s) 75. {A-q.s ; B-p.r; C-p ; D-q) 76. (3) 77. (2) 78. (5)
79. (6) 80. (7) 81. (5) 82.(5) 83.(7) 84.(1) 85. (4)
8/124 New Course Chemistry (XI)E!S19n

VII. Numerical Value Type Questions (■" Decimal Notation)

For the following question, enter the correct numerical value, (in decimal-notation, truncated/rounded-off
to the second decimal place, e.g., 6*25, 7-00, - 0-33, - -30, 30-27, - 127-30) using the mouse and the on
screen virtual numeric keypad in the place designated to enter the answer.

in FeS04 to Fe^'*’ if the


2+
86. How many grams of potassium dichromate are required to oxidise 20-0 g of Fe
reaction is carried out in an acidic medium ? (Molar mass of K2Cr20-7 and FeS04 are 294 and 152 respectively

w
(AMU Engg. 2015)

87. A 2 0 g sample containing Mn02 is treated with HCl liberating Cl2- The CI2 gas is passed into a solution of
KI and 60 mL of 0-1 M Na,S,0-, is required to titrate the liberated iodine. The percentage of Mn02 m the
sa.i>ple is . .

Flo
88 20 mL of 0-02 M K.5Cr-,07 solution is used for titration of 10 mL of Fe^'*’ solution in tlie acidic medium. The
* molarity of Fe^- sofution is Main 2022)

e
,V!II. Assertion-Reason Type Questions

re
F
TYPE I

DIRECTIONS. The questions given below contain statement-1 (Assertion) and statement-2 (Reason).
Each question has four options (a), (b), (c) and (d) out of which ONLY ONE is correct. Choose the correct
ur
r
fo
option as under:
(a) Statement-1 is True, Statement-2 is True ; Statement-2 is a correct explanation for Statement-1,
(b) Statement-1 is True, Statement-2 is True ; Statement-2 is not a correct explanation for Statement-1,
ks
(c) Statement-1 is True, Statement-2 is False. (d) Statement-1 Is False, Statement-2 is True.
Yo
89. Slatement-l. HNO2 acts both as an oxidising as well as a reducing agent.
oo
Statement-2. The O.N. of N can increase above +3 and can also decrease below +3.
90. Statement-1. A substance which gets reduced can act as an oxidising agent.
B

Statement-2. In the reaction, 3 CIO" CIO3 + 2 Cr, Cl atom is oxidised as well as reduced.
re

91. Statement-1. All decomposition reactions are redox reactions.


Statement-2. H2O on decomposition gives H2 and O2.
u
ad

92. Statement-1. A solution of FeS04 can be stored in a copper vessel.


Yo

Statement - 2. E" of Cu < E“ of Fe.


93. Statement-1. Copper liberates hydrogen from a dilute solution of hydrochloric acid.
Statement-2. E" of Cu is higher than that of H2.
d

94. Statement-1. The electrochemical cell stops working after sometime.


Re
in

Statement-2. The electrode potential of both the electrodes become zero.


TYPE n
F

DIRECTIONS. In each of the following questions, a statement of Assertion (A) is given followed by a
CO rresponding statement of Reason (R) just below it. Of the statements, mark the correct answer as
(a) If both assertion and reason are true, and reason is the true explanation of the assertion,
(ft) If both assertion and reason are true, but reason is not the true explanation of the assertion,
(c) If assertion is true, but reason is false. (d) If both assertion and reason are false.
95. Assertion. Fe reacts with HCl to produce H2 gas.
Reason. Fe is a better reducing agent than H2
ANSWERS
89. (fl) 90. (ft) 91. (d) 92. (c)
86. (6-45) 87. (13 05) 88. (0-24)
93. (d) 94. (c) 95.(a)
REDOX REACTIONS
8/125

96. Assertion. Sulphur dioxide and chlorine are both bleaching agents.
Reason. Both are reducing agents.
97. Assertion. Copper sulphate solution is not stored in a zinc vessel.
Reason. Zinc forms complex with CUSO4. (AlIMS 2007)
98. Assertion. ?2 does not undergo disproportionation reactions.
Reason. Fluorine shows only 0 and -I oxidation states.
99. AsserUon. In the iodometric titration, starch is used as an indicator.
Reason. Starch is a polysaccharide.
lOft A ^ . (ARMS 2006)
100. Assert,on. For the Dan.ell cell, Zn I 11 1 Cu with = 1 ● I V, the application of opposite potential
greater than MV results into flow of electron from cathode to anode.
Reason. Zn is deposited at anode and Cu is dissolved at cathode. (ARMS 2006)
101. Assertion. The Daniel cell becomes dead after some time.

w
Reason. Oxidation potential of zinc anode decreases and that of copper increases. (ARMS 2013)
102. Assertion. Permanganate titrations are not performed

F lo
in presence of hydrochloric acid.
Reason. Chlorine is formed as a
consequence of oxidation of hydrochloric acid. (JEE Main 2022)

e
Fre
For Difficult Questions

for
Multiple Choice Questions (with One Correct Answer)
1. I2+ IOHNO3 2HIO3 ION2O + 4H2O -1 +1 -2 -2 -1 +1 -I
r
Y
ib) + 2H2O ^ Xe02p2 + 4H F
You
+1 X -2
oks

H I O3 , i.e.,l+x-6 = 0 or X = 5. Reduced
eBo

+5 +2
, - +4 -1 +1 -1 +6 -1
2. HNO3, NO. N2, NH4CI
0
(c) Xep4 + O2F2 > XeFfi + O2
3. (a) Oxidation does not occur during formation of Oxidised
Fe(CO)5 from Fe since O.N. of iron remains to be
ad
our

zero +2-1 +5-1 +2-1 +5-1


0 0 (d) XeF2 + PF5 > [XeFf PI^
Fe + SCO ^ Fe(CO)5 In options (a), (b) and (d) O.N. of the atoms remain
In all other cases oxidation of Fe occurs since its the same. But in option (c), O.N. of Xe increases
O.N. increases.
from +4 in Xep4 to + 6 in XeFg while the O.N. of
Re
Y

0
O decreases from + I in O2F2 to 0 in O2. Thus, it
(b) 3 Fe is) + 4 H2O (g) ^ Fe3 O4 (s) + 4 H2 (g) is a redox reaction.
d
Fin

3+
0 +3
(c) 4Fe + 3 O2 + X H2O -+ 2F62 O3 .XH2O 5. [Co(H20)g]Cl3+3AgN03 +

3+
0 +2

(d)CuSO^+ Fe ^ FeS04 + Cu i [Co(H20)g](N03)3 + 3AgClT


hv
+6 -1 +1 -2 +6 -2-1 hv
+1 -1
O2 + 0 + 0; O + O2 O3
4. (a) XeFg + H2O XeOp4 + 2HF

ANSWERS

96.(c) 97. ic) 98. id) 99. (h) 100. (a) 101. (a) 102. (a)
8/126 'Piuxde€p-'A New Course Chemistry (XI)EEIHD
+6
na ●●●'■:
For Difficult Questions
12. Cr20^’ + 0H“ > 2Cr0j“ + H
+2 Thus, option (c) is correct.
2000 K
» 2 NO (Redox reaction)

w
N2+O2 +1 +5 -1

+ 6 + 6
13. 3C10" 4 ClOj + 2 Cl-
H2 S O4 +2 NaOH 4 Naj S O4 + 2HoO Here, O.N. of Cl increases from + 1 in CIO" to +
Thus, option (c) is correct. 5 in CIO3 while that of Cl decreases from +1 in
+2-1 + l +6-2 +2+«-2 +1-1
CIO" to 1 in Cr, therefore, it is a
^ CaS04 +2HF

e
o
6. Cap2+H2 S O4 disproportionation reaction.

re
Here, the O.S. of all the atoms do not change and 0 -3 +1

hence it is not a redox reaction. In other words, 14. p4 + 3NaOH » PH3 + NaH2 PO2
H2SO4 does not act as an oxidising agent in this O.N. of P in P4 decreases from 0 to - 3 in PH3 and

Frl
F
reaction while in all the remaining reactions it acts increases from 0 to + 1 in NaH2P02- Therefore, it
as an oxidising agent. is a disproportionation reaction.
7. In neutral or faintly alkaline solution, +7 +4

+7
-1 ou 15. Mn04 + 4H"-+3e" >Mn02+2H20

sor
2MnO" +H2O+ I This is not a disproporation reaction but is only a
+5
+4 redox reaction.
2Mn 02+20H" + I03

kf
16. Cr203 + 2 A1 AI2O3 + 2 Cr
Here, the O.S. of Mn changes from + 7 in MnO^ Thus, option (c) is correct.
oo -1 0
to + 4 in Mn02. +1 -1

17. (A) HOCl + H2O2 ^ + cr + O2


Thus, option (a) is correct.
Y
-1 0
8. For K2O :lr-2 = 0 or x = +\
B

(B) I2 + H2 O2 +2 OH- .>21 +2 H2O + O2


For K2O2 :2x-2 = 0orA:=+I Since in both the reactions, O.N. of O increases
(In peroxide, O.N of 0 is - 1) from - 1 in H2O2 to zero in O2, therefore, in both
re

these reactions. H2O2 acts as a reducing agent.


oY

ForK02::c-2x1/2 or x = +\
u

(In superoxides, O.N. of 0 is - 1/2) 18. Since TX does not react with any of the solutions,
it means that X" is least easily oxidized.
ad

9. The balanced redox equation is


+7
Since TY reacts with both X and Z, i.e.,
d

^ Z' + Y,
2Mn04 + 5 (COO")2 + 16 H-" Y" + X + X" + Yand Y"+Z
+2 therefore, Y" is most easily oxidized.
in

2Mn2-^+10CO2+8H2O Since TZ reacts with only X, i.e.,


Re

The overall change in O.S. of Mn = + 7-2 = 5 Z" + X + X" + Z, therefore,


F

Thus, option (a) is correct. Z" is more easily oxidized than X"
+ 4
Combining all the results, the decreasing order of
10. CH4(^) + 4Cl2(g) ^ CCl4(/) + 4HCl(g) oxidation of anions follows the order
.r = + 3 Y" > Z" > X"
11. HMCI4 =(+l) + .« + 4(-l) = 0
—> OH"
x = -2
19. HO2
CU2O =2(+ l) + .r = 0
Balancing 0 atoms on both sides, we have,
;c = + 5 ...(0
003 =x + 3(-2)=-l HO" 4 2 OH'

K^Cr^O^ =2i+l) + 2x+’n-2) = 0 x = + 6 O.N. of each 0 atom in HO2 is - 1 while that in


OH" is - 2. In other words, total O.N. of two 0 atoms
Thus, option (b) is correct.
REDOX REACTIONS
8/127

.-. Eq. wt. of K2Cr207 = Mol. wt./6


For Difficult Quostions
Thus, option (h) is correct.
in HO2 is - 2 while that of two O atoms in OH" is 25. Since oxidation of F-'*' ●» Fe^'*' is a one electron
- 4. Therefore, to balance O.N. of O atoms on either change, therefore, equivalent wt. of Fe-'*’ = atomic
wt. of Fe^+ = 55*84.
side, add 2e~ to L.H.S. of Eq. (0. On doing so we
have. +2 +2-5
26 2Na2S203=Na2S40fi
HO; + 2e- ^ 2 0H" ...(//)
Total increase is O.N. of S for two molecules of
Since the reaction is occuiring in basic medium, Na2S203

w
therefore, to balance H atoms, add one H^O = 4 X 2-5 - 2 [2 (+ 2)] = 2
molecule to L.H.S. and one OH" ion to R.H.S.”of
Total increase in O.N. of S per molecule of
Eqn. (//). Tlius, the balanced equation is : Na2S203 = 1

F lo
HO" + H,0 + 2eT ^ 30H Eq. mass = M,/! = M,
20. In neutral or alkaline solution, MnO^ oxidises Now
0

h
-1
^ 21

thiosulphate (S^O-") to sulphate (SOj")

ee
Total decrease is O.N. per molecule of
8Mn0"+3S20f+H2O = 0-2 (-1) = 2“

Fr
Eq. mass of I2 = M2/2
8MnO2+6S0j-+2OH" 27. Reduction half reaction - Refer to 0. 21 (i\ i>aee
8/105.

for
+5

21. KIO4 + H2O2 ^ KIO3+H2O+O2 MnO" +&H*+ 5 e


^Mn2+ + 4H20 ...(/)
ur
22. O.N. of P in H3PO4 = 3 (+ 1) + ^ + 4 (- 2) +3 +4

.r = + 5 Oxidation half reaction : €20^" 4 CO2


s
Since P has a maximum O.N. of + 5, therefore, it
ook
Balance C atoms: €20^“ 42CO2
Yo
...(«)
cannot increase its O.N. beyond + 5 and hence it
cannot act as an oxidising agent. Of course, it can Balance O.N. by adding 2 e" to R.H.S. of Eq. (ii),
eB

we have.
decrease its O.N. of + 3, + 1, - 3, etc. and hence
can act only as a reducing agent. 4 2 CO-) + 2 e ...(Hi)

23. The O.N. of Br in BrO^ is Charge is automatically balanced.


r

To cancel electrons, multiply Eq. (0 by 2 and Eq.


ad
ou

jt + 4(-2) = -1 or x = + l {Hi) by 5 and adding, we have.


Since O.N. of Br in BrOj is + 7 which is the 2Mn0;^+5C202- + l6H+ ■>

maximum. Therefore, it cannot increase its O.N.


Y

beyond + 7 but, of course, it can decrease its O.N. 2 Mn^-*- + 10 CO2 + 8 H^O
to + 5,+ 3, + 1 or - I. 28. 2M11O-+5C2O2-+I6H+
Re

■>
nd

Thus, BrO^ cannot show disproportionation 2Mn2++10CO2 + 8H2O


Thus, option (b) is correct.
Fi

reaction.
29.
24. Ionic equations involved in this redox titration are: Mohr’s salt [FeS04 ● (NH4)2S04.6 H2O] i: IS a

Oxidation : reducing agent involving 1 e~ change while


oxidising agent involving 6 e~
Cr^O^- + 14 H+ 4 2 Cr^+ +J^2 + 1 H2O change, i.e.,
Fe2+ -4 Fe^+ + e"J X 6
Reduction ; 2S2OI" +.4^ -4 S4O2- +>r ] X 3
CrjOl" + 14H+ + 6e"- 4 2 Cr3+ + 7 H2O
Cr202- +6S2O2- +14H+ 4 2Cr3++ 3S4OI
+ 7H2O 6 Fe2+ + CrjO^- + 14 h+ ■>

Thus,
= 6Na2S20|- 6 Fe^-*- + 2 Cr^-^ + 7 H2O
8/128 New Course Chemistry (XI)EEnEl

3 6 1
= - + -+ - = — = 2 moles.
10 ^ ,
For Difficult Questions
5 5 5 5
2+
31. The redox reaction involving the oxidation of Fe
Thus, 1 mole of Cr20^“ ion will oxidise 6 moles (from ferrous ammonium sulphate) is ;
of Mohr’s salt. > 5 Fe^-^ + Mn^+
Mn04 + 5 Fe2+ + 8
■ InO' + 8 H+ + 5 e' > Mn^"^ + 4 H2O + 4H2O
Mol. wt. of ferrous ammonium sulphate.
FeC204 - Fe2+ + C2OJ-
(NH4)2S04-FeS04.6 H2O = 392

w
Fe^"^ > Fe^+ + e~ Molarity of (NH4)2S04.FeS04-6 H2O
C2O2-
■> 2CO2 +2e Wt.
X
1000

Mol.wt. Volume

Flo
Thus, oxidation of I mole of Fe^+ loses 1 mole of
3-92 1000
electrons, oxidation of 1 mole of C2O4 loses X = 0-lM
392 100
two moles of electrons but reduction of 1 mole of

ee
KMn04 accepts 5 moles of electrons, Applying molarity equation.

Fr
(i) Oxidation ofFeC204 : M,Vi
(Ferrous amm. sulphate)
Oxidation of 1 mole of Fe^'*’ need KMn04 n
1

1 M2V2

for
= - mole (KMn04)
5
ur «2

and oxidation of 1 mole of C2O4 need KMn04 or


01x20 _ M2XI8
5 1
ks
= — mole
01x20 _ M
Yo
5
or
M2 = 5x18^ ~ 45
oo
In other words, oxidation of 1 mole of FeC204
Amount of KMn04 present in one litre
eB

_1 2^3 = Molarity x Mol. wt.


need KMn04 "5'^5 5
1
(//) Oxidation of Fe2(C204)2 ■ = _Lxl58 = 3'51g
45
r

Now Fe2(C204>3 = 2 Fe^^ + 3C2OI"


ou

Since 3-476 g is close to 3-51 g, therefore, option


ad

Fe
3+
is not affected by KMn04 (d) is correct.
32. Redox equation for oxidation of Fe^"^ ions in acidic
Y

For 3 moles of C20j“, moles of KMn04 needed and neutral media respectively are :
2+
Mn04 +8H-^ + 5Fe
Re
nd

2 6
=3x-=-
5 5 Mn2+ + 5 Fe^^ + 4 H2O ...(0
Fi

2+
Thus, oxidation of 1 mole of Fc2(C204)3 need Mn04 + 2 H2O + 3 Fe
Mn02 + 3 Fe^-*- + 4 OH"...(«)
KMn04 =~mole. Since under acidic conditions, 1 mole of MnO^
(iii) Oxidation of 1 mole of FeSO^ as discussed oxidises 5 moles of Fe^'*’ ions but only 3 moles
1
under neutral conditions, therefore, volume of
above need KMn04 - ^ mole . Mn04 used for oxidation of 25 mL of the same
Fe^"^ solution under neutral conditions
(iv) Fe2(S04)3 does not undergo oxidation.
Thus, total moles of KMn04 needed to oxidise 20x5
= 33-3 mL
1 mole of FeC204, Fe2(C204)3 and FeS04 3
REDOX REACTIONS 8/129

{b) Oxidation of FeC20^


For Difficult Questions
> Fe^'*’ + e~ Total no. of e

33. Mn04 + 8 H-" + 5 e- ■>


+ 4 H2O C,Q2- ^ 2CO2 + 2 e~ involved = 3

(NH4)2Ce(S04)2 = Ce‘*+ (c) Oxidation of Fe(N02)2


and (NH4)2FeS04-6H20 = Fe-+ Fe-* Fg3+ Total no. ofe
Now + e~ Ce^+
and Fe^-*- > Fe^-*- + e- 2 no; + 2H2O ^ 2 NOJ + 4 + 4t?-J involved = 5
Since reduction of KMn04 is a 5 e" change and (cf) Oxidation of FeSO^

w
oxidation of (NH4)2S04-FeSO4*6 H2O is a 1 Fe
2+
4 Fe-^"^ + e~
change, therefore, oxidation of 1 mole of
(No. of electrons involved = 1)
(NH4)^S04-FeS04.6H-,0 will require 1/5 mole of

F lo
KMnd4 Since FeS04 least number of electrons,
therefore, it will require least amount of KMn04
Further since reduction of (NH4)2S04-Ce(S04)2 for its oxidation.
is a W change and that of oxidation of

ee
(NH4)2S04-FeS04-6 H2O is also a 1 e~ change, 36. N2(g) + 3H2(g) ●> 2 NH3 (g)
therefore, oxidation of 1 mole of 20 g 5g

Fr
(NH4)2S04'FeS04-6 H2O will require one mole 20 5

of (NH4)2S04-Ce(S04)2. Thus, the ratio of the 28 2

number of moles of (NH4)2S04-Cc2(S04)2 and

for
0'83 moles 2-5 mole.s

KMn04 required to oxidise I mole of Now 3 mole of H2 require N2 = 1 mole


ur
(NH4)2S04-FeS04-6 H2O is 1 : 1/5 or 5 : 1.
1
34. Reduction equation :
.●. 2-5 mole of will require N2 = ~ ^ 2-5
s
MnOj+8H+ +5e~ Mn^+ + 4 H2O ] X 6
ok
Yo
= 0-81 mole
Oxidation equation : But N2 actually available = 20/28 = 0-71 mole
o
eB

2 Cr^+ + 7 H2O — Cr202- + 14H+ +6e~]x5 Since amount ofN2 required is less than required
by the chemical reaction, therefore, N2 is the
Balanced redox equation : limiting reagent.
6MnO; + 10Cr^-"+ 11 H2O 4
Now 28 g of N2 will give NH3 = 2 moles
r
ad
ou

6Mn2-"+ 5Cr20^“ + 22 R-*- ...(/) .●. 20 g of N2 will give NH3 = —^20


28
Applying molarity equation.
= 1-42 mules
Y

M,V,
(Cr3+) = M2V2 (MnOp Thus, option (c) is correct.
n
'^2
Re

I
38. A metal with more negative electrode potential is
nd

0125 XV i 0-200 X1200


_ more easily oxidised than a metal with less
10 6 negative electrode potential. Thus, zinc with
Fi

0-200 X12-00 10 E° = - 0-76 V is more easily oxidised by O2 and


or V,1 = = 32mL
6 0-125 moisture of the atmosphere than iron with
E® = - 0-44 V and thus protects iron from
35. Reduction half reaction :
corrosion. The reverse, however, is not beneficial
MnO;-i-8H++5e- - -> Mn^-^ + 4H^O since Fe with less -ve electrode potential cannot
No. of electrons involved in the reaction = 5 protect zinc with more - ve electrode potential
(a) Oxidation ofFeSO^ to Fe2(SO^)j from corrosion.

Fe^+ Fe^+ + e- Total no. of e' 39. Large negative potential for the reaction,
A + e A“ implies that the reverse reaction
so|- + H2O ^ SO^“-f 2H‘*‘+ 2e“ involved = 3 occurs, i.e.. A" is readily oxidised.
8/130 ‘^%cuUcp.'4- New Course Chemistry (XI)CEaQ]

49. F2 with the highest reduction potential (+ 2-85 V)


For Difficult Questions
is the strongest oxidising agent and 1“ with the

ow
highest oxidation potential (- 0-53 V) is the
40. Higher the reduction potential, stronger the oxidi
strongest reducing agent.
sing agent. Since reduction potentials decrea.se in Ionises
the order Z > Y > X, therefore, their oxidising 50. NaBr ¥ Na"*" (aq) + Br (aq)
powers also decrease in the same order. Hence, Y At cathode :
is a stronger oxidising agent than X but weaker
than Z. Therefore, Y can oxidise X but not Z.
2 H2O (/) -t- 2 c- ¥H2 (g) + 2 0H- iaq)
Since the electrode potential of H2O (- 0-83 V) is
41. Higher the E" (i.e., reduction potential), stronger
higher than that of, Na'*' (- 2-71), therefore, H2O

re
is the oxidising agent. Thus, M11O4 with is reduced in preference to Na"*" ions.
E“ = 1 -51 V is the strongest oxidising agent.

Flr
At anode :

F
+7 +6 2 Br- (aq) ¥ B12 (g) + 2 e
42. S2 0|-+2e" > sol (Reduction)
Thus, H2 is liberated at the cathode, Br2 at the

ou
Higher reduction potential stronger is the oxidising
anode and NaOH is left in the solution.

51. At anode, oxidation occurs. The possible reactions

sr
agent. Since the redox couple E which may occur at anode are :
S20|-/SO|-
Cu2+ + 2 e- ;

fo
Cu (s) E” = - 0-80 V
= + 2-05 V has the highest reduction potential,

k
therefore, it is the strongest oxidising agent. 2 H^O (/) 4 O2 (g) + 4 H+ (aq) + 4 ;
E° = - 1-23 V
43. Zinc has the most -ve electrode potential.
Therefore, it is easily oxidised and hence it acts
as the strongest reducing agent.
oo
Since oxidation potential of Cu is higher than that
of H2O, therefore, at the anode, it is the Cu of the
Y
copper anode which gets oxidised and not the H2O
44. Since = 0 - 0-74 has the most negative
reB

’Cr3+/Cr molecules. Thus, option (b) is correct.


E“, therefore, it is the strongest reducing agent. 52. Oxidation occurs at anode.

45. Refer to page Table 8.1, page 8/44. 53. We know that, oxidation potential = - Reduction
uY

The correct order of the reduction potentials of potential and vice-versa.


the following pairs is : EMF of the cell = Reduction potential of cathode
1 (CI2/CI- = ! -36 V) > III (Ag+/Ag = 0-80 V) > - Reduction potential of anode ...(/)
ad
do

II (12/r = 0-52 V) > IV (Na+/Na = - 2-71 V) > = Reduction potential of cathode + Oxidation
V (Li-^/Li = - 3-05 V). potential of anode ...(«)
in

Thus, option (a) is correct.


= Oxidation potential of anode - Oxidation
46. Higher is the E°, stronger is the oxidising agent,
potential of cathode ...{iv)
Re

3+
Thus Fe with E° = 0-77 V is the strongest
Thus, option (d) is correct.
F

oxidising agent.
Fe^+ -t- e" ^Fe^-" 54. Since electrode potential of Fe^'^'/Fe - - 0-44 V is
Thus, the amount of Fe^'*' will decrease. lower than the electrode potential of
47. Lower the E°, stronger is the reducing agent. Thus, = + 1-23 V. therefore, Fe^'*'/Fe electrode acts as
the reducing power decreases in the order : O

the anode and Er = + 1 -23 V acts as the


Z (- 3 0 V) > X (- 1-2 V) > Y (+ 0-5 V). Oj/HjO
48. [FelCN)^]^ -i- ^ [Fe(CN)6l‘^; cathode.

E° = + 0-35 V
.-. EMF of cell =e!cathode -e!anode
Fe^"- + ^ Fe2+ ; E° = + 0-77 V
= 1-23-(-0-44)
Higher -i-ve electrode potential, stronger is the
= 1-67 V
oxidising agent. Thus, option («) is correct.
REDOX REACTIONS 8/131

O 57. The reduction poieiUial of Zn, Cu, Ag and Ni


For Difficult Questions (obtained by reversing the sign of oxidation
potentials) are : - 0-76 , + 0-34 , + 0-80 and
55. EMF for the cell, Zn I Zn^+ 11 Cu^+ I Cu - 0-25 respectively. The maximum e.m.f. of the
= 0-34 - (- 0-76) cell is obtained when the electrode potential of
the anode is minimum and that of the cathode is
= MOV
maximum, i.e. Zn and Ag respectively
EMF for the cell, Fe I Fe^^ 11 Cu--^ I Cu E.M.F. = E° cathode -E'’anode
= 0-34 - (- 0-41 V) = + 0-80 - (- 0-76)
= 0-75 V = + 1-56 V

Since the two cells are connected in series, Thus, option {h) is correct.

w
therefore, the net EMF of the two cells
58. For a reaction to be spontaneous, cell
must be
= 1-10 -f 0-75
positive

F lo
=: F85 V O

Ecell=E cathode -E
anode
0
56. Since E =0 V For (o), Zn is anode (on which oxidation occurs)
2H^/H2
and Cu is cathode (on which reduction occurs)

0

and = +0-80 V, E.cell = 0-34-(-0-76)= 1-1 V

e
Ag'^/Ag

Fre
For{b) Ecell = 0-34 - (- 0-44) = 0-78 V
therefore, ... acts as the anode and
2H'^/H2

for
For(c) Ecell = -0 44-(- 0-76) = 0-32 V
Ag'*'/Ag acts as the cathode and the
E° cell = E‘ cathode -E‘
anode For (cf) E^^„ = 0-34 - (0-80) = - 0-46 V
= 0 - (+ 0-80)
r
Reaction (d) has -ve E cel! and hence this reaction
You
= - 0-80 V
ks
cannot occur.
o
eBo

m> Multiple Choice Questions (with One or More than One Correct Answers)
60. Refer to Q. 8.4, pages 8/79-8/80. H2SO4 and HSO^ , therefore, S is not reduced.
ad
our

The remaining three statements (a, b, d) are


61. Balanced equation of the reaction is correct.

-1 ■*■6 0 +6

6I-+C10J-I-6H2S04 ► Cr -I- 3I2 + 6HSO4 + 3H2O 62. H2S acts only as a reducing agent while all others
act both as oxidising as well as reducing agents.
Oxidised
63. Only F2 does not undergo disproportionation
Re
Y

Since O.N. of S remains the same, i.e. + 6 in reaction.


d
Fin

QD! Multiple Choice Questions (Based on the given Passage/Comprehens ion)

64. Total O.N. of 2 nitrogen atoms in N2H4 is - 4. compounds, we find that in M.F. A3(BC4)2 the sum
Since it loses 10 moles of electrons, therefore, the of O.N. of all the elements comes out to zero.
total O.N. of two N atoms in X increases by 10, 3x(-t-2)-+-2x(-t-5-i-4x-2) = 0
i.e., the total O.N. of two N atoms in X = - 4 -f 10
Thus, option (a) is correct.
= + 6.
O.N. of each N in X = + 6/2 = -i- 3.
66. Balanced equations for evolution of NO and NO2
respectively are ;
65. Sum of oxidation numbers of all the elements in
3 Cu + 8 HNO3
the M.F. of any compound is zero. Putting the O.N.
of all the elements in the M.F. of all the 3 Cu(N03)2 + 2 NO + 4 H2O ...(0
8/132 New Course Chemistry fXT~)OTSTwm

therefore, its corresponding acid is HOCl and the


For Difficult Questions
anhydride of this acid must be CI2O, i.e.,
2 HOC! > H2O + CI2O
Cu + 4 HNO3 4
Cu(N03>2 + 2 NO2 + 2 H2O
69. The chemical equations involved in this problem
...(//) are :

Adding Eqns. (1) and (i7), we have,


Ca(OCl)-, + 4 + 2 e- > Ca“-*- + 2 H2O + CI2
4Cu+ I2HNO3 ...(0

ow
4 Cu(N03)2 + 2 NO2 + 2 NO + 6 H2O 2 KI + CI2 ^ 2 KCl + I2
or 2 Cu + 6 HNO3 2 Na2S-j03 +1-5 ■> Na,S40g + 2 Nal
2 Cu(N03)2 + NO2 + no + 3 H2O Ca(OCl)2 + 4 H+ + 2 + 2 KI + 2 Na2S203
Thus, coefficients x and y of Cu and HNO3 Ca2+ + 2 H2O + 2 KCl + Na2S40g + 2 Nal
respectively are 2 and 6. ...(«)
+7
Applying normality equation to Eq. (n), we have

e
Mn-'*'
67. Mn04 +5e N,Vi [Ca(OCl)2l = N2V2 [Na2S203]

Fl
re
N,1 X 25 = 0-25x48
Since 1 mole of MnO^ accepts 5 moles of

F
electrons, therefore, 5 moles of electrons are lost 0-25x48.
or N = 0-48N
1 “
by 1-67 moles of j^.v+ 25

/. 1 mole of
ur
will lose electrons = 5/1 -67 Thus, the normality of Ca(OCl)-> solution

or
= 0-48 N.
= 3 moles (approx.)
Now, since Eq. (0, is 2 e~ change.

f
Since changes to MO3 (where O.N. of M Eq. wt. of Ca(OCl)2 = Mol. wt./2
= + 5) by accepting 3 electrons ks
In other words, molarity = normality/2
Yo
Oxidation state of M, = 0-48/2
oo
i.e.. ,t = + 5- 3 = + 2 = 0-24M

68. The active constituent of bleaching powder is Thus, the molarity of the hou.iehold bleach
eB

Ca(OCl)2. Since the anion of this salt is OCl~, = 0-24 M

VI.
Integer Type Questions
ur

76. Since net charge on the neutral molecule is zero, 80. The balanced equation is
ad
Yo

therefore, for the given molecule, Be„Al2SigO|g +4 +1 +7 -2

II (+ 2) + 2 (+ 3) + 6 (+ 4) + 18 (- 2) = 0 4Mn02+4K0H+302 -> 4K Mn 04-I-2H20


or 2 II + 6 + 24 - 36 = 0 or 2 n = 6 Thus, O.N. of Mn in KMn04 is 7.
d

or n =3 81. Balanced redox reaction is


Re
in

77. Non zero oxidation states of O are - 1, - 2 ; that 2Mn04 + 101- + 16H+ > 2 Mn“+ + 5 I2 + 8 H2O
of Cl are - I. -f 1. -f 3, + 5 and + 7 ; that of F is
Thus 2 moles of KMn04 react with KI to liberate
F

- 1 ; that of P are - 3, - 2, + 1, + 3 and 5 ; that of


5 moles of I2.
Sn are -f 2 and + 4 ; that of T1 are + 1 and + 3 ; that
of Na is + 1 and that of Ti is + 2, + 3 and + 4. 82. 3 Br2 4- 3 Na2C03 > 5 NaBr -1- NaBrOj + 3 CO2
Thus, 5 moles of NaBr are formed.
Thus, only F and Na can exist in one non-zero
oxidation state. 83. Alkaline KMn04 oxidises I to 10^
78. 5-0 = 5. Refer to Art. 8.4.7, page 8/21. 2 KMn04 + H2O + KI ■>

79. The balanced equation is


2 Mn02 + 2 KOH + KIO3
Cf20^- + I4H++6f- 4 2 Cr^-' + 7 H2O Pot.iodate

Thus, 6 moles of electrons are needed in the above Na2So03 being a reducing agent cannot oxidise
conversion. r to I2.
REDOX REACTIONS 8/133

84. During KMn04 titration with oxalic acid in acidic


For Oifficult Questions
medium, oxalic acid is oxidized to CO2. The
All the remaining seven reagents can oxidise oxidation state of C in oxalic acid + 3 and in CO2
is + 4
aqueous iodide ion to iodine, i.e.,
^ -2-2+1
6 KI + K2Cr207 + 7 H2SO4 ■>
COOH
2^-2x4+1x2=0
I
4 K2SO4 + Cr2(S04)3 + 3 I2 + 7 HjO COOH a: = + 3

ow
2 CUSO4 + 4 KI > CU2I2 + 2 K2SO4 + l2 Thus, change in oxidation slate of carbon
H2O2 + 2 KI > I2 + 2 KOH = 4-3 =+1.

C12 + 2KI >2KCI + l2 85. Mn04 oxidises H20-) to 0-> forming Mn02
O3 + 2 KI + H2O >2 KOH + I2 + O2 +7 +4

2 FeClg + 2 KI >2 FeCl^ + 2 KCl + I2 2M11O4+3H2O2 4 2M11O2 +3O2 + 2H2O

e
4 HNO3 + 2 KI > 2 KNO3 + 2 NO2 + I2 + 2 0H-

re
Fl
+ 2H,0 In Mn02* oxidation state of Mn is 4.

F
VII.
Numerical Value Type Questions Decimal Notation)
ur
r
86. ^^7804 —> K2SO4 + Cr7(S04)3 Now 2 moles of Na2S203 react with Mn02

fo
= ] mole
+ 4H2O + 3 [O]
2FeS04 + H2SO4 + O ^ Fe2(S04)3 + H2O] x 3 6 X 10"-^ M of Na-5S903 will react with Mn02
ks 6x10“^
Yo
Balanced redox reaction : = 3x 10-^ M
oo
2
K2Cr207 + 6FeS04 + 7H2SO4 -> K7SO4
Mol. mas.s of Mn02 = 55 + 2 x 16 = 87 g mol"'
+ Cr2(S04)3 + 3Fe2(S04)3 + 7H7O
eB

Mass of MoOt reacted = 3 x 10”-^ x 87 = 0-261 g


It is evident from balanced equation that
But mass of sample of MnO^ actually taken = 2 g
6 moles of FeS04 = 1 mole of K2Cr707
0-261
or 6 X 152 g of FeS04 = 294 g of K2Cr207
ur

.-. % MnOj = 2
xlOO = 13-05

20 g of FeS04 will require K2Cr707


ad

88. The redox reaction are


Yo

294 Reduction :
X 20= 6-45 g
6x152
Cr^O^- +14H-^ +6e~ 4 2 Cr^-^ + 7 H7O
d

87. The balanced redox equations in this reaction are


Oxidation : 6Fe2+ 4 6 Fe^'*' + 6 e
Re

MhOt + 4 HCl > MnCl2 + 2 H^O + CU


in

2 KI + CI2 > 2 KCl + I2


Redox reaction : Cr,0^ + 6Fe-"^ + 14H'^ ●>

2 Na2S203 + I2 > Na2S40g + 2 Nal


F

2 Cr^+ + 6 Fe^+ + 7 H7O


The overall equation :
Applying molarity equation
Mn02 + 4 HCl + 2 KI + 2 Na2S203
= MnCl2 + 2 KCl + Na2S406 + 2 Nal + 2 H2O M,V,(Cr,0^-)_M2V2(Fe^'^)
1 6
60xfrl
No. of moles of Na2S203 taken = 1000 2xa02 M2 xlO
= 3 X 10“^ M 1 6

From the overall equation, 2x0-02x6


M., = = 0-24
Mn02 = 2 Na2S203 10
8/134 “P^eidee^'^. New Course Chemistry (XI)EEIHD

For Difficult Questions

VIII. Assertion-Reason Type Questions

90. Correct explanation. Oxidising agents are 96. Correct reason. CI2 bleaches by oxidation while
electron acceptors and hence get reduced. SO2 bleaches by reduction.
91. Correct statement-1. Only those decomposition 97. Correct reason. Zinc dissolves while copper gets
reactions which undergo oxidation-reduction are deposited.
called redox reactions. 98. Correct assertion. F2 undergoes disproportio

w
nation reaction.
92. Correct .statcment-2. E° of Fe is lower than E" of
0 -1 +1
Cu.
F^+H^OOce) ^ HF-fHOF

F lo
93. Correct statement-1. Copper does not liberate H2 Correct reason. F2 gets reduced to HF and
from dil. HCl. oxidised to HOF.

94. Correct statcment-2. Electrodes potential of both 99. Correct explanation. In iodometric titration, the

ee
the electrodes become equal. liberated iodine produces blue colour with starch.

Fr
for
ur
s
ook
Yo
eB
our
ad
Y
Re
nd
Fi
w
HYDROGEN

Flo
e
re
9.1. GENERAL INTRODUCTION

Hydrogen is the first element in the periodic table and is also the lightest element known. Its atomic

F
form exists only at high temperatures. In the normal elemental form, it exists as a diatomic molecule, i.e. H2.
That is why it is also called dihydrogen. It forms more compounds than any other element. It may be mentioned
ur
or
here that the global energy crisis imminent in the near further can be overcome to a great extent by the use of
hydrogen as a source of energy.

f
It was first discovered by Henry Cavendish in 1766. He prepared it by the action of dil. H^S04 on iron.
Its name hydrogen was proposed by Lavoisier because it produces water on burning with oxygen (Greek :
ks
hydro means water and gene means producer).
Yo
oo
The various forms, i.e., isotopes, allotropes, molecular and ionic forms of hydrogen are given in the box
below :
B

Symbol H
re

Atomic number, Z
1
Electronic configuration Is

Isotopes Protium, ; Deuterium, “H or D ; Tritium, or T


u
ad

Allotropes Ortho hydrogen, o-H-y; Para hydrogen, p-H-,


Yo

[Also 0-D2. p-L>2, o-T'j. p-Tt1


Molecular forms H2, “H2 (or D2), ^H2 (or T2), HD, HT, etc.
Ionic forms
H-, H^,H+ H7n
d
Re
in

[Also corresponding ionic forms of D and T]


F

9.2. UNIQUE POSITION OF HYDROGEN IN THE PERIODIC TABLE


Hydrogen is the first element in the periodic table. However, a proper position could not be assigned to
it either in the Mendeleev’s periodic table or Long form of the periodic table because of the following two
reasons :

(/) In some properties, it resembles alkali metals. As such, it can be placed in group 1 of the periodic table
along with alkali metals.

* Not included in CBSE syllabus. This chapter has been given only for the preparation of competitive examinations.
9/1
9/2 ‘pKoidee^'A New Course Chemistry (XI)BSlsn

(ii) In some other properties, hydrogen resembles halogens. As such, it can be placed in group 17 along
with halogen.s.
(//■/) In some yet another properties, hydrogen differs from both alkali metals and halogens.
Thus, the position of hydrogen in the periodic table is controversial or anomalous. The main points of
resemblance of hydrogen with alkali metals and halogens are briefly discussed below :
I. Resemblance with Alkali metals. Hydrogen resembles alkali metals, i.e., Li, Na, K, Rb, Cs and Fr of
group I of the periodic table in the following respects :
1. Electronic configuration. Like alkali metals, hydrogen also contains one electron in its outermost
(valence) shell :
Hydrogen, H Lithium, Li Sodium, Na Potassium, K
\s
1
Is- 2.9* Is- 2s-2p^ 3^‘ 1^^ 2s^ 2p^ 3^2 4^'
2. Electropositive character. Like alkali metals, hydrogen also loses its only electron to form hydrogen
ion, i.e., H"*" (proton).

F low
H + e

Is^ I5O
Na Na+ + e

h-2 2^2 2p^ 3^* Is 2 2s2 2p^


Thus, hydrogen like alkali metals exhibits electropositive character.

e
3. Oxidation state. Like alkali metals, hydrogen exhibits an oxidation state of + 1 in its compounds.

e
for Fr
H+ cr Na-^ Cr K-"Br-
Hydrogen chloride Sodium chloride Potas,sium bromide

4. Combination with electronegative elements or non-metals. Like alkali metals, hydrogen combines
with electronegative elements (non-metals) such as oxygen, halogens and sulphur forming their oxides, halides
and sulphides respectively. For example.
Your
Oxide H2O like Na^O, K2O, etc.
s
eBo k

Halides HCi like NaCl, KCl, etc.


Sulphide H2S like Na2S, K2S, etc.
5. Liberation at the cathode. When an aqueous solution of HCI is electrolysed, H2 is liberated at the
ad

cathode in the same way as alkali metals are liberated at the cathode during the electrolysis of their fused
our

halides.
At cathode At anode
Electrolysis
2HC1 (aq) ■> H2(5) +
Cl2(g)
Re

Electrolysis
2NaCl (/) 2Na(0 + Cl2(g)
Find Y

6. Reducing character. Like alkali metals, hydrogen also acts as a strong reducing agent. For example,
Heat
Fe304 + 4 H2 ^ 3Fe + 4H20
Heat
B/^Oj -I- 6 K 4 2 B + 3 K2O
n. Resemblance with Halogens. Hydrogen resembles halogens, i.e. F, Cl, Br and I of group 17 of the
periodic table in the following respects ;
1. Electronic configuration. All the halogens have seven electrons in their respective outermost shells
and thus have one electron less than the stable configuration of the nearest inert gas. Hydrogen, on the other
hand, has one electron in the outermost shell and thus has one electron less than the stable configuration of
the nearest inert gas, i.e., He.

J
HYDROGEN 9/3

H One electron less than He

F One electron less than Ne

1^2 2^2p^
Cl One electron less than Ar
1^22^22/763523/75 1^2 2^2 2/76 3^2 3/?6
2. Electronegative character. Halogens have a strong tendency to gain one electron to form halide
ions. In a similar way, hydrogen shows some tendency to gain one electron to form hydride (H~) ion.

w
H + e~ H" (He gas configuration)
l5^ 152
Cl + e Cl-
■>
(Ar gas configuration)

F lo
1522522^63^23^5 I52 252 2/76 352 3/76
3. Ionization enthalpy. The ionization enthalpy of hydrogen is quite comparable with those of halogens
but much higher than those of alkali metals.

ee
Element
Ionization enthalpy (A,- H) Element
Ionization enthalpy (A^. H)
H 1312kJmol-l 520 kJ mol“^

Fr
Li
F 1681 kJ mol-i Na 496 kJ mol“*
Cl 1255 kJ mol“^ K 419 kJ mol-*

for
4. Oxidation state. Just like halogens, hydrogen shows an oxidation state of-1. For example,
ur
With sodium : Na'*’ H~ like Na'*' Cl"
iWth calcium ; Ca2+ (H")2 like Ca2+ (Cr>2
5. Liberation at the anode. When fused alkali metal hydrides such as lithium or sodium hydride is
s
ook
subjected to electrolysis, hydrogen is liberated at the anode. Similarly, halogens are liberated at the
Yo
anode
when fused alkali metal halides are electrolysed.
At anode At cathode
eB

Electrolysis
2NaH(/) ^ H2(g) + 2Na (0
Electrolysis
2NaCl(/) ^ Cl2(g) + 2Na (0
r

6. Atomicity and non-metallic character. Just like halogens, hydrogen also exists as a diatomic
ou
ad

molecule : H2, F2, CI2, Br2, etc. Further, like halogens, hydrogen is also a typical non-metal.
7. Combination with metals. Hydrogen combines with highly electropositive alkali and alkaline earth
metals to form metallic hydrides. In a similar way, halogens combine with these metals to form metallic
Y

halides.
With alkali metals
2Na + H2 >2NaH ; 2Na + Cl2 > 2NaCl
Re
nd

With alkaline earth metals Ca + H2 > CaH2 ; Ca + CI2 > CaCl2


8. Formation of covalent compounds. Like halogens, hydrogen readily combines with non-metals
Fi

such as carbon, silicon, nitrogen, etc. to form covalent compounds.


With hydrogen : CH4 SiH4 NH3
Methane Silane Ammonia

With halogens: CCI4 SiCl4 NCI3


Carbon tetrachloride Silicon tetrachloride Nitrogen trichloride
9. Replacement or Substitution reactions. In many compounds of carbon, hydrogen can be replaced
by halogens and halogens can be replaced by hydrogen. For example,
Diffused sunlight
CH4 + CI2 CH3CI + HCl
Methane Methyl chloride
9/4 New Course Chemistry (XI)sasiHE

Sn/HCl
CH3CI + 2 [H] ■>
CH4 + HCl
or Zn/HCl
Methane
Methyl chloride
III. Difference from Alkali metals and Halogens. Hydrogen differs from both alkali metals and halogens
in the following respects :
1. Hydrogen is comparatively less electropositive than alkali metals and less electronegative than
halogens. For example, hydrogen has less tendency to form H"*" ions as compared to alkali metals which
readil” form monovalent cations. Similarly, hydrogen has less tendency to form H ions as compared to
halogens which readily form halide (X“) ions.
2. Unlike halogens and alkali metals, hydrogen contains only one proton (but no neutrons) in its nucleus

w
and only one electron in the extranuclear part.
3. Nature of oxides. The nature of oxides of alkali metals, halogens and hydrogen is quite different. The
oxides of alkali metals are basic while those of halogens are acidic hut the oxide of hydrogen, i.e., H2O is

F lo
neutral. Thus,
H2O Li20, Na20, K2O, etc CI2O7
Neutral Acidic
Basic

ee
4. Absence of unshared pairs of electrons. The hydrogen atoms in hydrogen molecule do not possess
atom in halogen molecule possesses three unshared

Fr
any unshared pairs of electrons whereas each halogen
pairs of electrons as shown below :
H:H :C1 : Cl:

for
5. Nature of compounds. The compounds of hydrogen with halogens, i.e. hydrogen halides (HF, HCl,
ur
HBr, HI) are low boiling covalent compounds whereas alkali metal halides (LiF, NaCl, KBr, Csl) are high
melting ionic solids.
oks
6. Size of ions. The size of ion (*- 1 -5 x pm) is much smaller than those of alkali metal cadons
larger than any of the halide ions (p- = 136,
Yo
(72-180 pm) but the size of H“ ion {208 pm) is much
Cr = 181, Br" = 191 pm) except 1“ ion (219 pm).
o

7. Structure of hydrates. Being very small in size, H'*' does not exist freely and is always associated
eB

with other atoms or molecules. For example, H^ ion exists in aqueous solution as hydrated proton having the
formula, [Ho04]+ in which four H2O molecules are letrahedrally arranged around the H+ ion. However, it is
generally written as [H30'^] and called hydronium ion. Alkali metal cations also exist as hydrates but the
our

extent of hydration decreases as the size of alkali metal cation increases. Thus
ad

H H
;0—►H'" or :0 H [Li(H20)6]-^
Y

H H Lithium hexahydrate cation


Re

Hydronium ion
nd

Halide ions, also exist in aqueous solutions as hydrated ions but their structures are not well established.
That is why, they are usually represented as X" (aq) where X = F , Cl, Br or I.
Fi

Conclusion, From the above discussion, it follows that hydrogen is unique in its behaviour because it
not only resembles alkali metals and halogens in certain properties but at the same time it differs from both of
them in certain other properties. Therefore, it is not justified to include hydrogen either along wdh alkali
metals of group 1 or halogens of group 17. In other words, the position of hydrogen in the periodic table is
anomalous or controversial. That is why hydrogen is sometimes referred to as a rogue element .
9.3. OCCURRENCE OF HYDROGEN
Hydrogen is the most abundant element in the universe (70% of the total mass of the universe). The
giant planets such as Jupiter and Saturn contain mostly hydrogen. Hydrogen constitutes about half of the
mass of the sun and stars.
HYDROGEN
9/5

^ The extremely high temperature of the sun brings about fusion of hydrogen atoms liberating large amount
4}H
1 ^He +
0
2+je^ + Energy
Helium Positron

,c r abundant element on the surface of the globe. In the combined form, it constitutes
154% of the earth’s crust and the oceans. However, on the earth, it, is the ninth element in order of abundance
r, ^ocks, it is 10th in order of abundance approx.
0- 5% by weight. Hydrogen is also the essential constituent of water, coal, petroleum, clay and all animal and
vegetable matter.

w
Earth does not possess enough gravitational pull to retain the light H. molecules, therefore it ISi not
round in our atmosphere. ^
9.4. ISOTOPES OF HYDROGEN

Flo
Isotopes are the different atoms of the same element which have the same atomic number but
different mass numbers.

ee
Hydrogen has three isotopes. These are called protium, deuterium and tritium.

Fr
Representation. The isotopes of an element are usually represented by putting the atomic number as
the subscript and their mass number as the superscript on the symbol of the element. Thus, the three isotopes
of hydrogen are represented as | H, (or D) and J H

for
(or T).
ur
Occurrence. The most abundant isotope of hydrogen is protium. This occurs in natural hydrogen to an
extent of 99-9844% ; the remaining 0-0156% being deuterium mostly in form of HD. Tritium being unstable
because of its radioactive nature occurs only in traces (one in 10*^ parts), Le.. the natural abundances of
ks
jH, fH and f H are in the ratio of 1 : 1-56 x lO'^ : 1 x
Yo
oo
Some important characteristics of these isotopes are given in Table 9.1.
eB

TABLE 9.1.
Atomic Properties of Isotopes of Hydrogen
Property H D T

Relative abundance (%)


r

99-985 0-0156 10-15


ou

Relative atomic mass (g mol"^)


ad

1-007825 2-014102 3-016049


Nuclear spin quantum number 1/2 I 1/2
Y

Radioactive stability Non-radioactive, Non-radioactive, Radioactive, emits low


stable stable
energy p-articles, ty2= 12-33 y
nd
Re

Electronic structure. All the three isotopes of


hydrogen have the same atomic number (Z = 1), FIGURE 9.1

therefore, all have only one electron in their only shell


Fi

(i.e., K-shell) and one proton in the nucleus. Since the


mass numbers of the three isotopes are different,
therefore, they differ from one ''nothcr in the number
of neutrons in the nucleus
© IP
1N
IP
2N

as s
wn in Fig. 9.1.
(0 Protium or ordinary hydrogen [JH] . This HYDROGEN
OR PROTIUM
DEUTERIUM TRITIUM

is the most abundant isotope of hydrogen. Since its (^Hor D) (3HorT)


atomic number is 1 and mass number is also 1, its (jH)
nucleus has only one proton (but no neutron) and one Isotopes of Hydrogen
electron is revolving around the nucleus in its only shell,
i.e. K-shell.
9/6 T^’uideefi.’A New Course Chemistry (XI)BZsI9D

(ii) Deuterium or heavy hydrogen [ j H or D]. Its natural abundance is 0.0156%. It is usually prepared
by the electrolysis of heavy water (D2O). Since its atomic number is 1 and its mass number is 2, therefore, its
nucleus has one proton and one neutron while one electron is present in the K-shell. It is generally denoted by
the symbol D.

{/■») TVitium [ or T]. It is the least abundant (10“‘^%) of all the isotopes of hydrogen and is formed
in the upper atmosphere by reactions induced by cosmic rays. It is radioactive with a short half-life of 12-33
years. It decays by p-emission with no y-radiations
3He + -1
1

It is prepared artificially by the bombardment of nitrogen or an isotope of lithium with neutrons :

w
^He 1
1
+
1
0
n
6^
+
I ^Li + 0
n +

F lo
Since its mass number is 3 and atomic number is 1, its nucleus has one proton and two neutrons while
one electron is present in the K-shell. It is usually represented by the symbol T.
Thus, the three isotopes of hydrogen differ from one another only in the number of neutrons in the
nucleus.

ee
Properties— Isotope Effect, Since the three isotopes of hydrogen have the same atomic number and

Fr
electronic configuration (Is^), they have similar chemical properties. But owing to their different masses, the
rates or equilibrium constants of these reactions are different. For example, reaction between proUum and
chlorine is 13.4 times faster than between deuterium and chlorine ; protium is adsorbed more rapidly than

for
deuterium on activated charcoal. Similarly, electrolysis of ordinary water {H2O) occurs more rapidly than
that of heavy water (D2O).
r
You
This type of deference in properties due to difference in atomic masses is called isotope effect.
oks
Further, due to different masses, the physical properties of these isotopes are quite different.
D2 (dideuterium) and T2 (ditritium) are given i
o

in
Some common physical properties of H2 (dihydrogen),
eB

Table 9.2.
TABLE 9.2. Physical constants of H2, D2 and T2
Property H2 I>2 T2
our
ad

2-016 4-028 6-03


Molecular mass (g mor^)
13-96 18-73 20-62
Melting point (K)
20-39 23-67 25-00
Boiling point (K)
dY

-1 0-09 0-18 0-27


Density in gL
Re

0-197 0-250
Enthalpy of fusion (kJ mor‘) 0-117
1-393
Fin

1-226
Enthalpy of vaporization (kJ mof') 0-904
446-9
443-35
Enthalpy of dissociation (kJ mol"' at 298-2 K) 435-88
74-14 74-14
Intemuclear distance (pm)
-1 1312
Ionization enthalpy/kJ mol
Electron gain enthalpy (kJ mol”^) -73
37
Covalent radius (pm)
208
Ionic radius, H“ (pm)

Uses of Tyitium. Tritium is used to make thermonuclear devices and for carrying out researches in
fusion reactions as a means of producing energy. Tritium gas is usually stored by making UT3, which on
heating to 673 K releases T2.
HYDROGEN
9/7

673 K
2UT3 — ) 2U + 3T2
It is widely used as a radioactive tracer since it is relatively cheap and easy to work with.

9.5. PREPARATION OF Dl HYDROGEN


The important sources of dihydrogen are water, alkalies and acids.
I. From water.
Dihydrogen may be obtained from water by reduction either with active metals or by
electricity.
1. By the action of water on active metals
(/) Cold wate.r Very active metals, i.e., alkali and certain alkaline earth metals like Na, K, Ca ,

w
etc. react
with water at room temperature evolving dihydroge n.

2Na + 2HoO > 2NaOH + H2 2K + 2H2O > 2KOH + H2

F lo
Ca +2H2O >Ca(OH)2 + H2
The reaction with alkali metals is so vigorous and exothermic that the hydrogen evolved catches fire In
order to slow down the reaction, amalgams (alloys with mercury) of these metals are generally used. In these
amalgams, only a small surface area of the metal comes in contact with water and, therefore, the reaction is

ee
slowed down.

Fr
With calcium, the reaction is not so vigorous and hence can be easily controlled.
(ii) Boiling wate.r Less active metals like Zn, Mg, Al, etc. decompose boiling water liberating dihydrogen.

for
Heat
Heat
Zn +H2O ZnO + H2 Mg + H2O ^ MgO + H2
ur
Heat
2A1 + 3H2O > AI2O3 + 3H2
dihydrogen^^^^” metals like Fe, Sn, Ni, etc. decompose steam at high temperatures evolving
s
ook
Yo
102.3-I073K
3 Fe + 4 H2O ■>
Fe304 + 4H2
(Steam) Magnetic oxide
eB

2. By electrolysis of water. Dihydrogen of high purity is usually obtained by the electrolysis of water i in

presence of small amount of an acid or a base. During electrolysis, dihydrogen is collected at cathode while
dioxygen is liberated at anode as shown below :
our
ad

Electrolysis
2 H2O (/) Traces of acid/ba.se ^ 2H2(g) + 02(g)
Pure water is only weakly ionized and hence is a poor conductor of electricity but presence of an acid or
a base makes it a better conductor of electricity.
Y

H2O (/) ^ H+(<7^) + OH-(n^)


Re
nd

At cathode : H'*’(a^) + e~ -> H- ; H- + H ^«2 (g)


At anode : OH-(ii^) ■^OU + e- ; 4 OH
■> 2H2O (/) + O2 (g)
Fi

Although dihydrogen prepared by this method is of high purity, yet this method is not commercially
viable since it is quite expensive. However, where electricity is cheap, this method can be used,
beryllium, zinc, tin and aluminium, etc. react with boiling alkali solutions
liberating dihydrogen.
Heat
Heat
Be + 2NaOH
Na2Be02 + H2 Zn + 2NaOH 4
Na2Zn02 + H2
Sod. beryllate Sod. zincaie
Heat
Sn + 2NaOH + H-,0 Na2Sn03 + 2 H.^
Sod. stannate
Heal
2 Al + 2 NaOH + 2 H2O ■>
2 NaA!02 + 3 H2
Sod. meta-aluminate
9/8 4. New Course Chemistry (XI)

III. From acids. Metals which are more electropositive than hydrogen i.e., lie above hydrogen in the
electrochemical series) such as zinc, iron, magnesium, etc. react with dilute mineral acids (dil. HCl and dil.
H2SO4) to liberate dihydrogen gas.
Zn + H2SO4 -> ZnS04 + H2 Fe + 2 HCl FeCl2 + H2
Mg + 2 HCl — ^ MgCl2 + H2
Metals like copper, silver, mercury etc. which are FIGURE 9.2
less electropositive than hydrogen (/.€.. lie below CONC. H2SO4
hydrogen in the electrochemical series) do not liberate
dihydrogen from acids.

w
9.5.1. Laboratory Preparation of DIhydrogen HoGAS
In the laboratory, dihydrogen is prepared by action W

F lo
LU uj
of dil. H-,S04 on granulated zinc. BEEHIVE
HELP
Zn + H2S04(di7.) ^ ZnS04 + H2
5
Granulated pieces of zinc are placed in a Woulfe’s . DILUTE H2O

ee
bottle and are covered with water. The bottle is fitted with a H2SO4
GRANULATED ZINC
thistle funnel and a delivery tube as shown in Fig. 9.2.

Fr
Cone. H2SO4 is poured slowly through the thistle Preparation of dihydrogen gas in the laboratory
funnel. As the acid falls in the Woulfe’s bottle, it gets

for
diluted and then reacts with zinc evolving dihydrogen gas. It is collected by downward displacement of water.
It has been observed that sometimes the bubbles of dihydrogen produced stick to the surface of the zinc
ur
metal preventing the further reaction of the acid on the metal. Such a situation can, however, be avoided by
adding a few crystals of copper sulphate to the reaction mixture.
s
Preparation of pure dihydrogen gas. In case veo" dihydrogen gas is required it may be obtained
ook
Yo
as follows;
(i) By the action of pure dilute sulphuric acid on magnesium ribbon.
eB

Mg + H2SO4 {dil.) > MgS04 + H2


(ii) High purity (> 99-95%) dihydrogen is obtained by the electrolysis of a warm aqueous solution of
barium hydroxide using platinum or nickel electrodes.
{in) By the action of water on sodium hydride.
r
ad
ou

NaH + H2O > NaOH + H2


(iv) By the action of KOH on scrap aluminium (Uyeno’s method).
2A1 + 2 KOH + 2H2O » 2 KAIO2 + 3 H2
Y

This metliod is used for military purposes.


Re

9.5.2. Commercial Preparation or Manufacture of DIhydrogen


nd

FIGURE 9.3
1. By the electroysis of water. This is the best
+ ^ WATER u-
Fi

method of manufacturing dihydrogen where electricity


is cheap. A small quantity (15—25%) of acid or alkali DiOXYGEN-^ : /-^-►DIHYDROGEN
is added to water to make it a good conductor and
electrolysed in a cell as shown in Fig. 9.3.
In this cell, iron sheet is used as a cathode while
nickel plated iron sheet acts as anode.
The two electrodes are separated from each other c :

by an asbestos diaphragm which prevents mixing of 20% NaOH


ASBESTOS

dihydrogen and dioxygen. On passing electric current, SOLUTION


DIAPHRAGM

dihydrogen gas is collected at cathode while dioxygen


at anode. When 20% NaOH solution is used for Electrolysis of water for commercial
electrolysis, the decomposition of water takes place as preparation of dihydrogen
follows ;
HYDROGEN
9/9

H2O ^ ^ H+ + OH-
At cathode, + e~ ^ H- ; H- + H- ^ H2
At anode, 4 O H~ 4 4 OH + 4 e" ; 4 OH > 2 H2O + O2
2. From syngas-Bosch process. When super heated steam is passed over red hot coke or coal at 1270
K in presence of Ni as catalyst, a mixture of carbon monoxide (CO) and dihydrogen ( H2) is produced.
1270 K
C(s) + H2O (g) » CO(g) + Hj(g) - 121-3 kj
Ni ^ *
Coke Steam
Water gas
Earlier a 1:1 mixture of CO and H2 was called water gas l;ut now all mixtures of CO and H, irrespective

w
o eir composition are called synthesis gas or syngas. The name synthesis gas or syngas is derived from the
fact that a mixture of CO and H2 is used for the synthesis of methanol and a number of hydrocarbons.
This process ofproducing syngas from coke or coal is called ‘coal gasification*

F lo
Now a days, syngas is produced from sewage, saw-dust, scrap wood, newspapers, etc.
It is ^fficult to obtain pure H2 from water gas or syngas, since CO is difficult to remove. Therefore, to
remove CO^d increase the production of dihydrogen from syngas, CO of the syngas is oxidised to CO-, by

ee
mixmg It with more steam at 673 K in presence of iron chromate as catalyst.

Fr
673 K
COCgl-HHj (g) -hHjO(g) co^ ig)+m^ig)

for
FeCrO
Steam
Syngas

This chemical reaction in which carbon monoxide of the syngas reacts with steam to form carbon
ur
dioxide and more dihydrogen is called water gas shift reaction.
CO2 thus produced is removed either by scrubbing the mixture with sodium arsenite solution or by
s
ook
passing the mixture through water under 30 atm pressure when CO2 dissolves leaving behind H^ which is
Yo
collected. » 2
eB

3. From hydrocarbons. One of the most recent methods developed for manufacture of dihydrogen
involves either partial oxidation of hydrocarbons obtained from petroleum or thermal cracking of natural gas.
(/) Partial oxidation of hydrocarbons. A mixture of hydrocarbons (say crude naphtha) is mixed with
our

steam and passed over a heated Ni catalyst at 1270 K.


ad

1270
^n^2n+2 + /2H2O ^ /iCO + (2/1 + 1) H2
Ni
Naphtha Steam
dY

In this process, instead of crude naphtha, natural gas may also be used.
Re

1270 K
CH4(g) HjOte)
Fin

Ni CO(g)-i-3H3(g)
Nalural gas Steam Syngas
H2 is then obtained from the syngas by the process described above. This whole process of obtaining H-,
from natural gas is called steam reforming process. ^
(li) Thermal cracking of natural gas. Dihydrogen may also be obtained by thermal cracking of natural
gas.

1270 K
CH4 ^ C + 2H2
Catalyst

4. Lane’s process. Dihydtogen can also be manufactured by passing alternate currents of steam and
water gas over red hot iron. The method, in fact, consists of two stages.
9/10 ‘P’undceft-’A New Course Chemistry (XDGSBCm

(/) Oxidation stage. Super heated steam is FIGURE 9.4

passed over iron filings heated to about 1023 - 1073


DIHYDROGEN GAS
K when hydrogen is formed and magnetic oxide of STEAM

iron is left behind {Fig. 9.4). POROUS IRON


1023-1073 K
IHt
“ 1
I
3 Fe + 4 H^O ■>
I
I

Steam rzE n
III

w
Fe304 + 4 H2 + 160.7 kJ
Magnetic oxide I
I FURNACE \ fHl
(n) Reduction stage. When the whole of iron IHt
ha., been oxidised, the steam supply is cut off and a

Flo
CO2 H2O
stream of water gas (H2 + CO) is passed to reduce WATER GAS

Fe304 back to iron.

e
Manufacture of dihydrogen
Fe304 + 4H2 ^ 3 Fe + 4 H2O ;

re
by Lane's process
F6304 + 4CO ^ 3 Fe + 4 CO,

F
Thus, by passing steam and water gas alternately over heated iron, dihydrogen gas can be manufactured
from a small quantity of iron,
by-product. Large quantities of dihydrogen are obtained as a by product in various industries.
ur
r
5. As a

fo
For example.
(0 From petroleum cracking plants (it) in the manufacture of sodium hydroxide
ks and chlorine by electrolysis
of brine solution (see unit 10). During electrolysis, the following reactions take place.
Yo
At anode: 2 Cr (aq) ^ CU(g) + 2e'-
oo
At cathode : 2 H,0 (/) + 2 e" ^ H2 (g) + 2 OH" (fl^)
B

Overall reaction : 2 Cl (aq) + 2 H2O (/) ^ CI2 (g) + H2 (g) + 2 OH (aq)


re

Adding spectator Na"^ ions, we have.


2 Na+ (aq) + 2 Cl" (aq) + 2 H2O (/) > CI2 (g) + H2 (g) + 2 Na-" (aq) + 2 OH (aq)
Presently about 77% of the industrial hydrogen is produced from petrochemicals, 18% from coal, 4%
u
ad
Yo

from electrolysis of aqueous solutions and 1% from other sources.


9.6. PROPERTIES OF DIHYDROGEN
(a) Physical properties. 1. It is a colourless, tasteless and odourless gas
d
Re

the lightest substance known. For example, one litre of dihydrogen at N.T.P. weighs 0-0899 g. Its
in

2. It is

density is approx. 1/14 th of that of air.


F

3. It is slightly soluble in water since its molecules are non-polar.


4. It can be liquefied under low temperature and high pressure.
Some important physical constants of dihydrogen along with those of dideuterium and ditritium are
given in Table 9.2.
(b) Chemical properties. The chemical properties of dihydrogen to a large extent depend upon its bond
dissociation enthalpy. The H—H bond dissociation enthalpy (435-88 kJ mol ^) is the highest for a single
bond between two atoms of any element. It dissociates into its atoms in only 0-081% at 2000 K which
increases to 95-5% at 5000 K. 5000 K
H2(g) 4 2H(g)
95-5%
Dihydrogen Atomic hydrogen
HYDROGEN 9/11

Thus, dihydrogen is quite stable and relatively inert at room temperature due to its high H—H bond
dissociation enthalpy. Therefore, most of the reactions of dihydrogen occur at high temperatures.
Atomic hydrogen is, however, produced by dissociation of H2 at high temperatures in an electric arc
(Fig. 9.13, page 9/66) or under ultraviolet radiations. Since the electronic configuration of atomic hydrogen
is 1j^, it needs one more electron to complete its orbital. Therefore, atomic hydrogen is very reactive and
combines with almost all the elements. It, however, reacts in three different ways, i.e., (0 by loss of its single
electron to form H'*', (i7) by gain of one electron to form H“, and {Hi) by sharing its electron with other atoms
to form single covalent bonds,

w
Some important chemical properties of dihydrogen are discussed below :
1. Neutral character. It is neutral to litmus.
2. CombustibiUty. It is a highly combustible gas and bums in air or dioxygen with a pale biae flame to

Flo
form water. However, it is not a supporter of combustion.
2H2(g) + 02(g) 4 2H20(/)
3. Reaction with metals. Dihydrogen reacts with strongly electropositive metals like sodium, potassium,

ee
calciu^ etc. at high temperatures to form salt-like {electrovalent or ionic) metal hydrides, in which the

Fr
oxidation state of hydrogen is -1. For example,
573 K 573 K
2Na + H2 ^ 2Na+H-
Ca + H2 ■»
Ca2+(H->2

for
ur
Sod. hydride Cal. hydride
4. Reaction with non-metals. Dihydrogen combines with many non-metals at high temperatures in
presence of catalysts to form covalent or molecular hydrides. For example*,
ks
(0 With dioxygen, H2 forms H2O. The reaction is strongly exothermic
Yo
oo
970 K
2H2(g) + 02(g) 2 H2O (/); AH = - 285-9 kJ mol"i
eB

or Electric discharge

(«) With halogens. Dihydrogen combines with halogens (X2) to form halogen halides
H2(g) + X2(g) » 2HX(g) (X = F,Cl,Br,I)
r

The reactivity of halogens towards dihydrogen decreases in the order: F2 > CI2 > Br2 > Ii-
ou
ad

Thus, flourine reacts in dark, chlorine in presence of diffused sunlight, while the reactions with bromine
and iodine occur on heating in presence of a catalyst.
Y

Dark 673 K
H2(g) + F2(g) 2HF(g) H2(g) + Cl2(g) ^ 2 HCl ig)
Sunlight
nd
Re

Hydrogen fluoride Hydrogen chloride


673, Pt 673. Pt
H2(g) + Br2(g)
Fi

2HBr(g) ; ^2(8) + 12(5) 2HI(g)


Catalyst
Hydrogen bromide Hydrogen iodide
{Hi) With dinitrogen, it forms ammonia.
673 K, 200 atm
Njte) + 3H2(«) Fe,Mo
■>
2NH3(*) ; AH = -92-6kJmol-'
Ammonia

Here, Fe acts as a catalyst while Mo acts as a promoter.


This reaction is used for the manufacture of ammonia by the Haber’s process,
(iv) With sulphur, it forms hydrogen sulphide.
700 K
^2(8) + S(0 ^ H2S(g)
9/12 New Course Chemistiy (XI)K2SX9D

(v) iWth carbon. Depending upon the conditions, it forms methane or acetylene.
1375 K Electric arc
2C(5) + H2(g) ^ HCsCH
C(s) + 2H2(g) > CH4(g) 3300 K
Methane Acetylene
5. Reduction of metal oxides and ions. Dihydrogen acts as a reducing agent and hence reduces
oxides of certain metals less electropositive than zinc such as those of Cu, Pb, Fe, etc. to the corresponding
metals.

MAW + yHjte) ^ xM is) + y H2O (/)


Heat Heat

For example, CuO + H2 ■> Cu + H2O ZnO + H2 Zn + H2O

w
Heat Heat

PbO + H2 ^ Pb + H2O Fe304 + 4H2 ^ 3Fe + 4H20


The oxides of strongly electropositive metals such as those of alkali and alkaline earth metals are not

F lo
reduced by this method.
Dihydrogen also reduces some metal ions in aqueous solutions. For example,
Pd2+(a^) + H2(g) ^ Pd (s) + 2

ee
CvP-^iaq) + H2(g) » Cu (5) + 2 H'*’ (aq)

Fr
6. Hydrogenation of unsaturated hydrocarbons. Unsaturated hydrocarbons such as alkenes and alkynes
add dihydrogen in presence of a catalyst to form saturated hydrocarbons.

for
Ni or Pt or Pd
Ni or Pt or Pd
CH2 = CH2 + H2 ^ CH3—CH3 ; HCsCH + 2H2 473 K ^ CH3—CH3
473 K
Ethane
r
Ethene Ethane Acetylene
Hydrogenation of unsaturated organic compounds in presence of heterogeneous and homogeneous
You
s
catalysts is used in many industrial processes. For example,
ook

(i) Hydroformylation of olefins. Olefins react with carbon monoxide and dihydrogen in presence of
octacarbonyl dicobalt as catalyst under high temperature and pressure to form aldehydes.
eB

[Co(CO)4l2
RCH = CH2 + H2 + CO ^ RCH2CH2CHO
High temp., pressure
Alkene Aldehyde
our
ad

This reaction is called hydroformylation or the oxo-process. The aldehydes thus obtained on subsequent
catalytic reduction give alcohols.
Ni
RCH2CH2CHO + H2 ^ RCH2CH2CH2OH
dY

Alcohol
Aldehyde
Re

(«) Hydrogenation of oils. The vegetable oils such as soyabean oil, cotton seed oil, groundnut oil, etc.
are called polyunsaturated oils since they contain many C = C bonds. When these oils are exposed to air for
Fin

prolonged periods, the double bonds present in them undergo oxidation and the oils become rancid, f.e,
develop unpleasant odour and taste due to formation of lower aldehydes and carboxylic acids. To avoid this,
double bonds are hydrogenated. For this purpose, dihydrogen is bubbled through edible oils in presence of
finely divided nickel at 473 K when the oils are converted into solid fats.
Ni. 473 K
Vegetable oil + H2 4 Vegetable fat
(Liquid) (Solid)

This process is called hydrogenation or hardening of oils and is used in the manufacture of vegetable
ghee like Dalda, Gagan, Rath, etc. from vegetable oils. It may, however, be noted that hydrogenation reduces
the number of double bonds but does not completely eliminate them.
HYDROGEN 9/13

Cunosity Questions
f Q. 1. Normally pure substrates and reagents are used in chemical reactions. Explain why In
the preparation of dIhydrogen by action of dilute sulphuric acid on zinc metal, impure
zinc is preferred ?
Ans. Pure zinc is not used in the preparation of dihydrogen because its reaction with dilute H2SO4 is
slow. The reason being that initially formed gaseous hydrogen forms an extremely thin film on
the surface of zinc which prevents further action of zinc with acid.
However, the presence of impurities in zinc increases the rate of reaction due to the formation
of electrochemical cells.

w
Q. 2. Concentrated sulphuric acid is an excellent dehydrating agent and is widely used for
drying such gases which do not react with It. But concentrated sulphuric acid cannot be
used for drying dihydrogen gas. Why ?

F lo
Ans. Cone. H2SO4 on absorbing HgO from moist H2 produces so much heat that hydrogen catches
fire.

ree
9.7. USES OF DIHYDROGEN
Dihydrogen is used : (/) in the manufacture of ammonia (by Haber’s process) which is used for the

F
production of various fertilizers like urea, ammonium sulphate, calcium ammonium nitrate (CAN), etc.
(ii) in the hydrogenation of vegetable oils to form solid fats, /.<?., Vanaspati Ghee.

for
(Hi) in the manufacture of bulk chemicals such as methanol
ur
700 K, 200 atm.
CO is) + 2 H2 (g) ●> CH30H(/)
Cobalt catalyst
s
ook
Yo
(/v) in the manufacture of hydrogen chloride.
hv
H2(^) + Cl2(g)
eB

> 2 HCl ig)


(v) in the manufacture of metal hydrides (refer to Art. 9.8).
(vr) in metallurgy to reduce heavy metal oxides to metals.
(v/7) in the atomic hydrogen (produces temperatures around 4000 K) and oxy-hydrogen torches (produces
r
ad
ou

temperatures between 2270-2770 K) for cutting and welding. Dihydrogen is dissociated with the help of an
electric arc and the hydrogen atoms thus obtained are allowed to recombine on the surface to be welded. As
a result, temperature of around 4000 K is generated.
Y

(viil) Liquid hydrogen (mixed with liquid oxygen) is used as a rocket fuel in space research. It is also
used in fuel cells for generating electrical energy. It has many advantages over the conventional fuels. It does
Re
nd

not cause any pollution and produces greater energy per unit mass of fuel in comparison to gasoline and other
fuels.
Fi

SUPPLEMENT YOUR
KNOWLEDGE FOR COMPETITIONS

1. Hydrogen under very high pressure is expected to behave like a metal.


2. The name dihydrogen is commonly used for H2 molecule, but while referring to the isotopic mixture
with natural abundance for H and D. the name diprotium is preferred.
3. The name proton is used for but while referring to the isotopic mixture of proton (H"^) and deutron
(D"^) w.r.t. to their natural abundance, the name hydron is used.
4. Heavy hydrogen or deuterium was separated from liquid hydrogen by fractional evaporation by H.C.
Urey. For this great contribution, he was awarded the Nobel Prize in Chemistry in 1934.
9/14 N*'W Course Chemistry fXl^prsTwm

9.8. HYDRIDES
Dihydrogen combines with a number of elements to form binary compounds called hydrides. Their
general formula being MH,. where M represents the element and a: the number of hydrogen atoms. The various
elements which form hydrides are :
(/) All the main group elements except those of noble gases and probably indium and thallium.
(//) All lanthanides and actinides.
ini) Transition metals~-Sc, Y, La, Ac, Tc, Zr, Hf, Pd and to a lesser extent V, Nb, Ta, Cr, Cu and Zn.

w
Depending upon the physical and chemical properties, the hydrides have been divided into the following
three broad categories :
1. Ionic or Salt-like or Saline hydrides 2. Metallic or Interstitial hydrides
3. Molecular or Covalent hydrides

Flo
1. Ionic hydrides. These are formed by those metals whose electronegativity values are appreciably
lower than that of hydrogen (2-1). Thus, all elements of group 1, i.e., alkali metals and group 2, i.e., alkaline
earth metals on heating at high temperatures (595—973 K) form ionic hydrides. However, BeH2 and MgH2

e
have covalent polymeric structures. (Refer to Supplement Your Knowledge on pages 9/17—9/18)

re
Properties. Some important properties of ionic hydrides are listed below :

F
(/) These are formed by transfer of electrons from the metals to the hydrogen atoms and thus contain
hydride ion, H~. For example, lithium hydride, Li* H~ and calcium hydride, Ca^* (H~)2-
(ii) These are white crystalline solids and their crystal structures consist of ions.
ur
r
Alkali metal hydrides (i.e., LiH, NaH, KH, RbH and CsH) have rock salt structures. Their thermal

fo
stability, however, decreases from LiH to CsH due to the reason that the lattice energies of these hydrides
decrease progressively as the size of tlie metal cation increases from Li"*" to CsL In a similar way, the stabilities
ks
of hydrides of alkaline earth metals decrease in the order : CaH2 > SrH2 > BaH2.
Yo
(///) The density of these hydrides is higher than those of the metals from which they are formed. This is
oo
because the H~ ions occupy holes in the lattice of the metal without distorting the metal lattice,
(iv) They have high melting and boiling points and conduct electricity in the fused state, liberating
B

dihydrogen at the anode.


At anode : 2 H" (melt) ^ H2(g) + 2e-
re

At cathode : Na'*' (melt) + e~ Na(/)


(v) They have high heats offormation and are always stoichiometric,
u
ad

(vi) Except LiH, they burn in air on strong heating (675—775 K) due to their decomposition into
Yo

hydrogen which is inflammable


675-775 K
CaH2 (5) ^ Ca(j)+ H2(g)
(v/0 They react violently with water to form the corresponding metal hydroxides with the liberation of
d
Re

dihydrogen. Thus, they act as strong bases.


in

NaH (j) + HoO (/) NaOH(fl^) + H2(g) ; CaH2(j) + 2H20(/) > Ca(OH)2 (aq) + 2 H2 (g)
Becau.se of the exothermic nature of the reaction, the evolved H2 catches fire. The fires so produced
F

cannot be extinguished by CO-, because it gets reduced by the hot metal hydride. However, sand is useful
since it a highly stable solid.
(viii) They are powerful reducing agents, especially at high temperatures
Heal Heat
HCOONa
2 CO + NaH ^ HCOONa + C CO2 + NaH
Sodium formate Sodium formate

450 K Heat

2 BF3 + 6 NaH SiCL4 + 4 NaH SiH4 + 4 NaCl


Boron Diborane Silicon Silane

trifluoride tetrachloride
Heat
PbS04 + 2 CaH2 ^ PbS + 2Ca(OH)2
HYDROGEN 9/15

Howeve,r their reactivity towards H2O limits their usefulness as reducing agents in aqueous solutions,
(ix) Lithium hydride is rather unstable at moderate temperatures. It is, therefore, used in the synthesis
of other useful complex metal hydrides such as lithium aluminium hydride (LiAlH^) and lithium borohydride
(UBH4).
Dry ether Dry ether
8 LiH + AI2CI6 2 UAIH4 + 6 LiCl ; 2 LiH + B2Hg 2UBH4
Diborane

Similarly with NaH, B-,Hg gives sodium borohydride (NaBH4)


Dry ether
2 NaH + B2H6 ^ 2NaBH4

w
These complex metal hydrides are widely used as reducing agents in organic synthesi.s.
Uses. Ionic hydrides and their complexes (LiAlH4, NaBH4, etc.) are used as reducing agents. Since on
heating ionic hydrides decompose to evolve dihydrogen which ignites spontaneously, therefore, they are used

F lo
as solid fuels.

2. Metallic or Interstitial hydride.s. ^/-Block elements of groups 3.4,5 {Sc. Ti. V. Y, Zr, Nb, La, Hf. Ta,
Ac, etc.) lO, 11,12 (Pd, Cu, Zn, etc.) and/-block elements (Ce, Eu, Yb, Th, U, etc.) on heating with H-, under

ee
pressure form hydrides. In group 6, Cr alone forms the hydride, CrH. The metals of group 7, 8 and 9'do not
form hydrides. In fact, the region of the periodic table from groups 7—9 which does not form hydrides is

Fr
referred to as the hydride gap. These hydrides generally have properties similar to those of the parent metals
and hence are called metallic hydrides. Earlier, it was thought that in these hydrides, hydrogen atoms being
small in size, occupy some (but not all the interstitial sites) in the metallic lattice producing distortion without

for
any change in its type. That is why these hydrides are also called interstitial hydrides. However, recent
studies have shown that except for hydrides of Ni, Pd, Ce and Ac, other hydrides of this class have lattice
ur
different from that of the parent metal.
These interstitial hydrides may also be regarded either as alloys or interstitial solid solutions of hydrogen
s
in metals. However, it is not certain whether the hydrogen is present in the interstitial sites as hydrogen atoms
ook
Yo
or H^ ions with delocalized electrons.
Properties. Some important properties of these hydrides are listed below :
eB

(/) They are hard, have a metallic lustre, conduct electricity and have magnetic properties,
(ii) The density of these hydrides is lower than those of metals from which they are formed since the
crystal lattice expands due to inclusion of H2-
{Hi) These hydrides are often non stoichiometric, i.e., in these hydrides law of constant composition
r
ad

does not hold good. For example, LaHj.g,, ¥63.55, TiH,.5^1.8, ZrH,.3^,.75, VHo.;,,, NiH„,^„.7.
ou

etc. Further the ratio of H-atoms to the metal atoms in these hydrides is notfixed but varies with the temperature
and the pressure.
Uses. Due to interstitial hydride formation, these metals adsorb large volumes of hydrogen on their
Y

surface. This property of adsorption of a gas by a metal is known as occlusion.


Re

The amount of hydrogen occluded depends upon the nature and physical state of the metal, i.e., colloidal
nd

palladium > palladium > platinum > gold > nickel.


If red hot Pd is cooled in H2, it adsorbs or occludes about 935 times its own volume of Ht gas. This may
Fi

be used to separate H2 or D2 from He and other gases. However, on strong heating, the occluded hydrogen is
liberated. Thus, metallic hydrides can be used as hydrogen storage media. This property has high potential
for hydrogen storage and as a source of energy.
Further, metals such as Ni, Pd, Pt, etc. which can adsorb large volumes of hydrogen are widely used in
catalytic reduction/hydrogenation reactions for preparation of large number of compounds.
3. Molecular or Covalent hydrides. These are mainly formed by p-block elements and some .v-block
elements (Be and Mg) primarily due to the reason that the electronegativity difference between these elements
and the hydrogen atom is quite small. Their general formula is either XH,, (for A-block elements) or XHg.„
(forp-block elements) where n is the number of electrons in valence shell. These hydrides usually consist of
discrete covalent molecules which are held together by weak van der Waals forces of attraction and hence
are called covalent or molecular hydrides.
9/16 New Course Chemistry (XI)ESian

Nomenclature. The systematic names of molecular hydrides are usually derived from the name of the
element and the suffix—am. For example, phosphane for PH3, oxidane for H2O and azane for NH3. However,
common names like phosphine, water and ammonia are more commonly used.
Preparation. These are prepared by a number of diiterent methods :
(/) By direct combination of elements with dihydrogen. For example,
673 K. 200 atm Electric discharge
N2(g) + 3H2(g) ^ 2NH^(g) ; 2H2(g) + 02(g) ^ 2H20(g)

w
Fe,Mo

713K Dark, little moisture


H2(g) + S(/) ^ H2S(g) ; H2(g) + F2(g) ^ 2HF(/)
23 K

(//) By reduction of a suitable halide with LiAlH^ in dry ether

o
Ether
4 BCI3 + 3 UAIH4 2 B2H6 + 3 AICI3 + 3 LiCl

e
Boron trichloride Diborane

Fl
re
Ether
SiCl4 + LiAlH4 ■> SiH4 + LiCl + AICI3
Silicon tetrachloride Silane

F
Ether
SnCl4 + LiAlH4 ■» SnH4 + LiCl + AICI3
Tin tetrachloride
ur Stannane

r
Ether

fo
GeCl4 + LiAlH4 GeH4 + LiCl + AICI3
Germanium tetrachloride Germane

ks
(«7) By hydrolysis ofmetal borides, carbides, nitrides, phosphides, etc. Refer to page 9/22 for reactions,
Yo
(iv) By action of acids on suitable binary compounds. For example,
oo
2Mg3B2 + 4H3PO4 B4H,o + 2Mg3(P04)2 + H2
Mag. boride Tetraborane-10
B

AI4C3 + 12 HCl ^ 3CH4 + 4AICI3


Aluminium carbide Methane
e

FeS + H2SO4 ^ H2S + FeS04 ; ^^3^2 ^ H2SO4 ^ 2 PH3 + 3 CaS04


ur

Iron sulphide Cal. phosphide Phosphine


ad

(v) By reduction of an oxoacid with NaBH4 in aqueous solution


Yo

4 H3ASO3 + 3 NaBH4 ■> 4 ASH3 + 3 H3BO3 + 3 NaOH


Arsenious acid Arsine

Classification. Depending upon the number of electrons and bonds present in their Lewis structures,
d

molecular hydrides have been classified as electron-deficient, electron-exact or electron-precise and electron
Re
in

rich hydrides.
Properties. Some important properties of covalent hydrides are discussed below :
F

(/) Covalent hydrides are usually volatile compounds having low melting and boiling points and also
do not conduct electricity,
(ii) Hydrides of group 13 (i.e. BH^, AlH^, etc.) do not have sufficient number of electrons to form
normal covalent bonds and hence are called electron-deficient hydrides. They generally exist in polymeric
forms such as B2Hg, B4H10, (A1H3)„, etc.
iiii) Hydrides of group 14 (i.e. CH4, SiH^, GeH^, SnH^, PbH^) have exact number of electrons to form
normal covalent bonds and hence are called electron-exact or electron-precise hydrides. All these hydWdes
have tetrahedral shapes. Their bond lengths increase from CH4 to PbH4 as the size of the element increases
from C to Pb.
(iv) Hydrides of group 15, 16 and 17 (i.e., NHj. PH3, H2O, H2S, H,F HCl, etc.) have more electrons
than required to form normal covalent bonds and hence are called electron-rich hydrides. The excess
HYDROGEN 9/17

electrons in these hydrides are present as lone pairs of electrons. For example, group 15 hydrides have otie
lone pair, group 16 hydrides have two lone pairs while group 17 hydrides have three lone pairs of electrons.
The presence of these lone pairs of electrons on the highly electronegative O, N and F atoms result in the
formation of intermolecular H-bonds. As a result of these intermolecular H-bonds, these hydrides exist as
associated molecules as shown below :

8- s-
6* N 6" 5-^ 6" 5^ \ 5- 5-"
N—H N—H N—H-9- ”0—H -O—H -b—H-

8- 8-

§x!>s
Fs

w
140° ’H

V''

F lo
8-
Consequently, the hydrides offirst elements of groups 15, 16 and 17 (i.e., NHy H^O, HF) have abnormally
high boiling points as compared to the boiling points of the hydrides of the second element of each group
(i.e., PHj, H2S, HCl). The boiling points of the hydrides of the rest of the elements of each group, howeve,r

e
increase as the atomic number of the element or the molecular mass of the hydride increases down the group

Fre
as shown in the following table.

Group 15 Group 16 Group 17

for
Hydride b.p. (K) Hydride b.p. (K) Hydride b.p. (K)
r
NH3 238-5 H2O 373 HF 292-4
You
PH3 185-5 H^S 213 HCl 188-0
s
ook

AsHj 210-6 H^Se 231 HBr 205-9


SbH3 254-6 H2Te 270-8 HI 237-5
eB

(v) The lighter elements of group 14, 15 and 16 form polynuclear hydrides in which two or more atoms
of the same element are linked togethe.r This property of self-linking of atoms is called catenation and is
maximum for carbon, (C2H6, C3H8, C4H10 .... etc.), followed by nitrogen (N2H4, NH3), oxygen (H2O2,
our
ad

H2O3) and sulphur (H2S^ where = 2 - 6).


SUPPLEMENT YOUR
KNOWLEDGE FOR COMPETITIONS

Besides three main categories of hydrides, i.e., ionic, interstitial and covalent, certain hydrides are better
dY

known as polymeric hydrides and complex hydrides. For example,


Re

(0 Polymeric hydrides. The.se are formed by elements having electronegativity in the range 1-40 - 2-0.
These usually exist is the polymeric form in which the monomer molecules are held together in two or
Fin

three dimensions by hydrogen bridges. Some common examples are :


(BeH2)„, (AIH3),,, (InH3)„, (GaH3)„, (SiH4)„, etc.
These are amorphous solids which decompose above 523 K to evolve H2 gas.
Structure. Beryllium hydride exists in the polymeric form involving three-centre-two electron bonds as

shown below :

H H H

'^Be^
9/18 New Course Chemistry (XI)CEISIl

(/■/) Complex hydrides. In these hydrides, the hydride ion (H") acts as the ligand and is attached to the
central metal atom by coordinate bonds. These are formed both by transition and non-transition elements.
Among the non-transition elements, the most important complex hydrides are formed by elements of
group 13. These are sodium borohydride (NaBH4), lithium borohydride (UBH4) and lithium aluminium
hydride {UAIH4).
These are versatile reducing agents and are widely used for reduction of organic compounds.

9.9. WATER
TABLE 9.3. Estimated World
Occurrence. Water is essential to all forms of life but

w
distribution of water over the earth’s surface is not uniform Water Supply
(Table 9.3). Human body has about 65% water while some Source % of Total

plants have as high as 95% water.

F lo
Oceans 97-33
In nature, water is found is all the three phases, Le.,
Saline lakes and inland seas 0-008
solid, liquid and gas. 2-04
Polar ice and glaciers
9.9.1. Structure of Water Molecule and Its Aggregates

ee
Ground water 0-61

In H2O molecule, the oxygen is .vp^-hybridized and 0-009

Fr
Fresh water Lake.s
hence has four sp^-hybridized orbitals. Two of these si[y‘- Soil moisture 0-005
orbitals are half-filled and hence overlap with h-orbitals of Atmospheric water vapour 0-001

for
hydrogen to form two sjr’-s, O—H, <T-bonds while the other Rivers 00001
two contain a lone pair of electrons each. Since the
ur
oxygen atom is jp^-hybridized, therefore, the geometry of H2O molecule should be tetrahedral and the ZHOH
bond angle should be 109.5°. But experimentally, it has been determined that the actual ZHOH bond angle is
s
104.5°. This may be explained as follows :
ook
Yo
In H2O, the oxygen atom is surrounded by two shared pairs, and two lone pairs of electrons. But according
to VSEPR (Valence Shell Electron Pair Repulsion) theory, lone pair—lone pair repulsions are stronger than
eB

bond pair-bond pair repulsions. As a result, the ZHOH bond angle in water is slightly smaller than the
regular tetrahedral angle of 109.5°. The actual bond angle is 104-5° and each O—H bond has a bond length
of 95.7 pm.
r

In the gaseous state, water exists as discrete molecules. It is a bent molecule with bond angle of 104-5°
ad
ou

and bond length of 95-7 pm as shown in Fig. 9.5(a).


The electronegativity of oxygen (3.5) is
Y

LONE PAIRS
much higher (next only to fluorine) than that
of hydrogen (2.1) and hence the shared pairs
Re
nd

of electrons in the O—H bonds are attracted


slightly more towards the oxygen atom. As a
Fi

result, oxygen carries a partial negative charge


(6-) and hydrogen atoms carry a partial
positive charge (6+) as shown in Fig. 9.5(b).
Since these two dipoles are inclined to each
other at an angle of 104.5, therefore, H2O is (a) Structure of H2O in the gas phase
a highly polar molecule. Its actual dipole (b) Polar nature of H2O molecule
(c) Resultant dipole moment of water molecule
moment is 1.84 D (Fig. 9.5(c)).
Hydrogen bonding in water and ice. In the liquid state, the H2O molecules are held together by
iniermolecular H-bonds. Each oxygen atom can fomi two H-bonds by using each of its two lone pairs of
electrons. In other words, each water molecule is generally H-bonded to four other water molecules as shown
HYDROGEN 9/19

in Fig. 9.6(a). Thus, in the liquid state, H2O exists as an associated liquid. Experimental studies suggest
that liquid water consists of aggregates of varying number of water molecules held together by H-bonds and
free water molecules in a dynamic equilibrium with the aggregates continually forming, collapsing and
reforming.
When water freezes, it forms ice which is the crystalline FIGURE 9.6

form of water. Depending upon the conditions employed for


freezing of water, nine crystalline forms of ice are known. '9
At atmospheric pressure, ice crystallizes in the normal
H
hexagonal form (I„) but at very low temperatures, it
177pm
condenses in the cubic form. In the normal hexagonal ice,

w
each oxygen atom is tetrahedrally surrounded by four other 95.7prTW®\'"‘-v
oxygen atoms, i.e., there is a hydrogen atom between each
pair of oxygen atoms. This gives ice an open cage like

F lo
structure as shown in Fig. 9.6{b). From the Fig. 9.6(b), it is
clear that each oxygen is surrounded by four hydrogen atoms,
two by strong covalent bonds (shown by solid lines) and
two by weak hydrogen bonds (shown by dotted lines). Since

e
Fre
the H-bonds are longer (177 pm) than covalent bonds (95.7 VACANT SPACES

pm), the molecules of water are not closely packed in the


crystal lattice. In other words, there exists a number of vacant

for
spaces in the crystal lattice and hence the density of ice is
lower than that of liquid water. When ice melts, some of the
H-bonds are broken and the cage like structure partially
r
collapses. In other words, some of the vacant spaces
You
oks
disappear which are now occupied by some of the water
molecules. Therefore, for the same mass of water, the volume
eBo

decreases iuid hence the density increases. In other words,


the density of liquid water at 273 K is higher than that of ice
and hence ice floats over liquid water.
As the temperature is raised gradually above 273 K,
our
ad

more and more of H-bonds break consequently more and (a) Structure of water in the liquid state
more H2O molecules start coming closer resulting in decrease (b) Tetrahedral arrangement of
in volume and hence increase in density. This goes on till oxygen atoms in ice
277 K (precisely 276-98 K). As the temperature Is raised
dY

above 277K, the increase in volume due to expansion of liquid water becomes much more than the decrease
Re

in volume due to breaking of H-bonds. Therefore, above 277 K, there is a net increase in volume on heating
thereby resulting in decrease in density. Hence, density of water is maximum at 277 K.
Fin

This property of maximum density at 277 K helps aquatic animals to survive during v. nter months. In
severe cold, the upper layer of sea water freezes while the water (with maximum density at 277 K) remains as
liquid under the surface of ice. As a result, aquatic animals can live safely in water even during severe cold
weather.

9.9.2. Properties of Water


(a) Physical properties. Most of the unique properties of water are due to the presence of H-bonding
between its molecules. Some of the physical constants of liquid water (H2O) and its deuterium analogue, i.e.,
heavy water (D2O) are listed in Table 9.4.
9/20 "PnAdcep-'^, New Course Chemistry (XI)ESsISD

TABLE 9.4. Some physical constants of H2O and D2O


Property H2O D2O

Molecular mass (g mol"’) 18-0151 20.0276


273-0 276-8
Melting point (K)
373-0 374-4
Boiling point (K)
Temperature of maximum density (g/cm^) 276-98 284-2

Maximum density (298 K) (g/cm^)

w
1-000 1-106

Density (298 K) (g/cm^) 1 -0000 1-1059

Enthalpy of formation, AHy’ (kJ mol"’) -285-9 - 294-6

Enthalpy of vaporisation (373 K) (kJ mol"') 40-66 41-61

Flo
Enthalpy of fusion (kJ mol"') 6-01 6-28

Specific heat (Jg"' K"‘) 4-177

e
Ionisation constant [H'*'] [OH"J (mol^ L"^) 1-008 X 10"''* 1-95 X 10"'5

re
Dielectric constant 78-39 78-06

Viscosity at 293 K (in centipoise) 0-8903 1-107

F
Electrical conductivity (293 K/ohm"' cm"')
ur 5-7 X 10"*

Some of the important physical properties are discussed below :

r
fo
(/) The freezing point, boiling point, heat of fusion and heat of vaporization of water are abnormally
higher than those of the hydrides of the other elements of the same group 16 such as H2S, H2Se, H2Te, etc.
This is due to the presence of intermolecular hydrogen bonding in H2O molecules which is, howeve,r absent
ks
among the molecules of H2S, H2Se, H2Te, etc.
Yo
(ii) Water has a higher specific heat, thermal conductivity and surface tension than most other liquids.
oo
These properties allow water to play a vital role in the biosphere. For example, the high heat of vaporization
and the high heat capacity of water are responsible for moderation of the climate and body temperature of
B

living organisms.
{Hi) Water because of its high dielectric constant (78-39) has the ability to dissolve most of the inorganic
e

(ionic) compounds and is, therefore, regarded as a universal solvent. Whereas solubility of ionic compounds
takes place due to ion-dipoleinteractions{i.e. solvationof ions), the solubility of covalent compounds such
ur

as alcohols, amines, urea, glucose, sugar, etc. takes place due to the tendency of these molecules to form
ad

hydrogen bonds with water.


Yo

(iv) Heavy water (D2O) has slightly higher values of physical constants because of its higher molecular
mass as compared to that of ordinary water (H2O).
(b) Chemical propertiej.. Water displays a wide range of reactions. It behaves as an acid, a base, an
d

oxidant, a reductant and as a ligand to metal ions. Some of these properties are discussed below ;
Re

1. Stability. Due to high negative heat of formation (AH^° = 285.9 kJ mol"'), water is quite stable at
in

ordinary temperature but decomposes into its elements at very high temperature. The extent of decomposition
is, however, not appreciable.Even at 1500 K, it is only 2% .
F

1500 K
2H2O (g) > 2H2 (g) + O2 (g)
Water also decomposes into its elements, when electricity is passed through it in presence of a small
amount of an acid or an alkali.

Electric current
2H2O (/) ^ 2H2(g) 02(g)
(Ai cathode) (At anode)
2. Acid-base character—Amphoteric character. Water is a weak electrolyte, i.e., it undergoes ionization
to a small extent to give H30'*‘ (aq) and OH" (aq) ions. As a result, pure water has very low electrical
conductivity.
HYDROGEN 9/21

H2O (/) + H2O (/) > H3O+ {aq) + OH- (o^); = 1.0 x mol^ L"2 at 298 K.
Thus, water acts both as an acid and as a base and hence is said to be amphoteric in characte.r For
example, it can act as an acid towards NH3 and as a base towards H2S :
H2O (/) + NH3 (aq) > NHJ (aq) + OH“ (aq)
Acid Basc2
1 Acid2 Base 1

H2O (0 + H2S iaq) > Hp^iaq) + HS-(aq)


Base Acid2 Acid 1
1 Base2
In general, water can act as a base towards acids stronger than itself and as an acid towards bases
stronger than it. Thus, in terms of its amphoteric nature, auto-protolysis, i.e., self ionization of water may be

w
represented as follows :
H20(/) + H20(0 H3O+ {aq) + OH-(a^)

F lo
Acid 1 Base2 Acid2 Base 1

{Acid) {Base) {Conjugate acid) {Conjugate base)


3. Oxidation-reduction or Redox reactions. Besides, acid-base reactions, water also undergoes
oxidation-reduction reactions.

ee
(i) As an oxidising agent Water reacts with number of active metals like Na, Ca, etc. whose electrode

Fr
potential is less than - 0.83 V liberating dihydrpgen gas.
[2H20(/) + 2e- ^ 20U-{aq) + H2(g); E‘» = -0-83V]

for
2Na(j) + 2H20(0 > 2NaOH(fl^) + H2(g)
{Reductant) {Oxidant)
ur
Similarly, when steam is passed over red hot coke (1273 K), syngas is formed.
1273 K
s
cw + HjOCj) —^ CO(g) + H,(g)
ook
Yo
{Reductant) {Oxidant)
In all these reactions, water acts as an oxidising agent and hence itself gets reduced to dihydrogen gas.
eB

(ii) As a reducing agent. Water reacts with highly electronegative elements like fluorine whose electrode
potential is higher than +1-23 V liberating dioxygen and trioxygen.
[02(g) + 4H+(a^) + 4e- > 2H20(/); E« = +1-23V]
our

2F2(g) + 2H20(/) > 02{g) + 4K^{aq) + 4F-{aq)


ad

{Oxidant) {Reductant)
or
3F2(g) + 3H20(0 > 03(g) + 6F-(a^) + 6H+(o^)
In these reactions, water acts as a reducing agent and hence itself gets oxidised to either or O3.
Y

Similarly, during photosynthesis, water is oxidised to dioxygen.


Re
nd

Sunlight
XCO2 + yH20 C^(H20)y + XO2
Chlorophyll
Fi

4. Hydrolytic reactions. Water can hydrolyse many oxides (metallic and non-metallic), hydrides,
carbides, nitrides, phosphides and other salts. In these reactions, and OH“ ions of water interact with the
anions and cations of the compound respectively leading to the formation of an acid or a base or both as
shown below;

CaO {s) + H2O (/) > Ca(OH)2 {aq) ; SO2 (g) + H2O (/) > H2SO3 {aq)
Sulphurous acid
P4O10 (5) + 6 H2O (0 >4H3P04(o^) ; CaH2(j) + 2H2O (/) > Ca(OH)2 (a^) + 2 H2 (g)
Phosphoric acid Calcium hydride
SiCl4(/) +4H20(0 » Si02.2H20(j) or Si(OH)4 + 4 HCl (a^)

I
9/22 New Course Chemistry (X1)DS]SD

AICI3 (s) + 6 H2O (0 [A1(0H2)6]^-" («^) + 3Cl-(a^)


CaC2 (s) + 2 H2O (/) Ca(OH >2 {aq) + HC = CH (g)
Calcium carbide Acetylene

Al4C3(5) + 12H20(/) > 4 A1(0H)3 (aq) + 3 CH4 ig)


Aluminium carbide Methane

Ca3N2 (s) + 6 H2O (/) ^ 3Ca(OH)2(flg) + 2NH3(g)


Calcium nitride Ammonia

w
A!N (5) + 3 H2O (0 ^ Al{OH)3{or/) + NR^(g)
Aluminium nitride Ammonia

Ca3P2(5) + 6H20(0 ^ 3Ca(OH)2(«^) + 2PH3(^)


Phosphine

Flo
Calcium phosphide
Na2C03 (s) + 2 H2O (/) ^ 2 NaOH (aq) + H2CO3 (aq)
Since the hydrolysis of Na2C03 produces a strong base (NaOH) and a weak acid, i.e., carbonic acid

e
(H2CO3), therefore, an aqueous solution ofNa2CO-^ is alkaline in nature.

re
In contrast, the hydrolysis of CUSO4 produces a weak base i.e., Cu(OH)2 and a strong acid (H2SO4),

F
therefore, an aqueous solution 0/CUSO4 is acidic in nature.
CuS04(j) + 2H20(/) ^ Cu(OH)2(a^) + H2S04(fl^)
5. Hydrate formation. Many ionic compounds crystallise from water with one or more molecules of
ur
r
water associated with them. For example, BaCl2.2H20, CUSO4.5H2O, FeS04.7H20, Na2SO4.10H2O, etc.

fo
This water in combination with ionic salts is called water of crystallisation and such crystals are called
hydrated salts or simply hydrates. These hydrates can be classified into the following three categories :
ks
(i) Coordinated water. Water molecules are coordinated to the central metal ion to form complex
Yo
ions. For example, in nickel nitrate hexahydrate, ferric chloride hexahydrate and chromium chloride
oo
hexahydrate, the six water molecules are linked to the central ion Ni^"^, or Cr^ ion by coordinate
bonds.
B

[Ni(OH2)6]2+ (N0J>2, [Fe(OH2)6l^* 3 Cr, [Cr(OH2)6]3+ 3 Cl"


re

(//) Hydrogen-bonded water. Water molecules are linked to some oxygen containing anions by hydrogen
bonding. For example, in copper sulphate pentahydrate, CUSO4.5H2O, four H2O molecules are linked to the
u

central Cu^'*’ ion by coordinate bonds while the fifth H2O molecule is linked to the sulphate (SOj“) ion by
ad
Yo

H“bonding. Thus, CUSO4.5 H2O may be represented as [Cu(H20)4]^'^ . H2O (Refer to Ans. to Q. 9.4,
page 9/53 for detailed structure).
(Hi) Interstitial water. Water molecules are present in interstitial sites or voids in the crystal lattice.
d
Re

For example, in barium chloride dihydrate, BaCl2.2H20, the two H2O molecules occupy tlie voids in the
in

crystal lattice.
F

9.10. HARD AND SOFT WATER

Natural water contains dissolved salts. Depending upon its behaviour towards soap solution w..rt. lather
formation, water may be classified as soft water and hard water.
Soft water. Water that produces lather with soap readily is called soft wate.r Examples of soft water
are : rain water, distilled water and demineralised water.
Hard water. Water which does not produce lather with soap readily is called hard water. A few examples
of hard water are : sea water, river water, spring water, lake water and well water.
Cause of Hardness of Water. It has been established that hardness of water is due to the presence of
bicarbonates, chlorides and sulphates of calcium and magnesium in it. Hard water does not produce lather
with soap readily. This may be explained as under :

t
HYDROGEN 9/23

Soap is sodium or potassium salt of certain higher fatty acids such as stearic acid, palmitic acid, oleic
acid, etc. When hard water is treated with soap solution, Ca"''’ and Mg^'*’ ions present in hard water react with
the anions of fatty acids present in soaps to form scum or curdy white precipitates. As a result, hard water
does not produce lather with soap immediately.
2C,7H35COO"Na+ + CaCl2 ■>
(C,7H35COO)2Ca i + 2NaCl
Sodium stearate (Soap) Calcium sletu’ate (Curdy whiteppt.)
2C,7H3gCOO“Na+ + MgSO^ (C|7H35COO)2Mg i + N32S04
Sodium stearate (Soap) Magnesium stearate (Curdy white ppt.)

w
However, when all the and Mg-'*’ ions present in hard water have been precipitated by addition of
sufficient amount of soap, the resulting water becomes soft and thus readily produces lather with soap. Thus,
hard water is not suitable for washing purposes since lot of soap is wasted in precipitating out Ca^* and
Mg^* ions present in hard wate.r

Flo
9.10.1. ‘TVPss of Hardness of Water
Hardness of water is of two types : (a) Temporary hardness and (b) Permanent hardness

ee
(a) Temporary hardness. Temporary hardness of water is due to the presence of bicarbonates of calcium
and magnesium, i.e., Ca(HC03)2 and Mg(HC03)2. It is called temporary hardness since it can be easily removed

Fr
by simply boiling and filtering the water. Temporary hardness is also called carbonate hardness,
(b) Permanent hardness. Permanent hardness of water is due to the presence of soluble chlorides and
sulphates of calcium and magnesium, i.e., CaCl2, CaSO^, MgCl2 and MgS04. This type of hardness is called

for
ur
permanent hardness since it cannot be removed simply by boiling the water. Pennanent hardness is aLso
called non-carbonate hardness.

Curiosity Question
ks
r
Yo
Q. What is blue baby syndrome ?
oo
Ans. In many agricultural areas, tube well water is contaminated with nitrate (NOJ) ions. Excessive
eB

nitrates are especially dangerous to infants. In the infant’s digestive system, nitrate ion is reduced
to nitrite ion. This results in methemogtobinenia or blue baby syndrome. The baby turns blue
and can die after drinking the water if not treated.
J
r

9.10.2. Methods of Removing Hardness of Water-Softening of Hard Water


ou
ad

The process of removing hardness, i.e., soluble bicarbonates, chlorides and sulphates of calcium and
magnesium from hard water is called softening of wate.r Depending upon the nature of the dissolved salts,
Y

hard water can be softened by the following methods.


I. For temporary hard water. The temporary hardness of water is due to the presence of bicarbonates
of calcium and magnesiam. It can be removed by any one of the following methods:
Re
nd

1. By boiling. When temporary hard water is boiled, bicarbonates of calcium and magnesium decompose
to form insoluble calcium and magnesium carbonates respectively.
Fi

Ca(HC03)2 —> CaCOg 'L + CO2 + H2O


(Temporary hardness) Cal. carbonate

A
Mg(HC03)2 MgCOg i + CO2 + H2O
(Temporary hardness) Mag. carbonate
These insoluble carbonates are removed by filtration and the water is rendered soft.
2. By Clark’s proc^. This process is used on a commercial scale. In this process, calculated quantity
of quick lime is added. The bicarbonates present in the temporary hard water react with lime to form insoluble
calcium carbonate and magnesium hydroxide which can be easily filtered off. The reason being that MgCOg
is more soluble in aqueous solution than Mg(OH)2 and hence Mg(OH)2 is formed instead of MgCOg.
9/24 New Course Chemistry (XI)CEIHIl

Ca0(5) + H2O ^ Ca(0H)2


Quick lime Slaked lime

Ca(HC03)2 + Ca(0H)2 ■>


2 CaCOj i + 2H2O
Mg(HC03)2 + 2 Ca(0H)2 2 CaC03 I + Mg(OH)2 J- + 2H2O
Howeve,r if excess of lime is added, water will again become hard due to absorption of C02from the
atmosphere by unused slaked lime to form soluble calcium bicarbonate.
Ca(OH)2 + 2 CO2 ^ Ca(HC03)2
II. For permanent hard water. The following methods are used for the removal of permanent hardness

w
of water.

Washing soda process. In this process, hard water is treated with a calculated amount of washing soda
(Na2C03) when chlorides and sulphates of calcium and magnesium present in hard water get precipitated as
insoluble calcium and magnesium carbonates which can be easily filtered off. The water thus becomes soft.

F lo
CaCl2 +Na2C03 >CaC03i + 2NaCi ; MgS04 + Na2CO3 MgC03 i + Na2S04
III. For both temporary and permanent hard waters. The following methods are used.

ee
1. Lime-soda process. This method is employed for the softening of hard water which contains both
temporary and permanent hardness. In this method, calculated quantities of slaked lime, Ca(OH)2 and sodium

Fr
carbonate are used. Whereas, slaked lime removes temporary hardness, sodium carbonate removes permanent
hardness.

Ca(HC03)2 + Ca (OH)2 ^ ICaCOj i + 2H2O ; CaCl2 + Na2C03 ■>


CaC03 I + 2NaCl

for
■> MgC03 i + N32S04
MgS04 + Na2C03
ur
2. Ion exchange method. This is the most modem method for softening of hard water. In this method,
the Ca-'*’ and Mg^'*’ions present in hard water are exchanged by those present in complex inorganic and
organic compounds called ion exchangers. Ion exchangers are mainly of two types :
k s
(a) Inorganic ion exchangers—Permutit process. Complex inorganic salts like hydrated sodium
Yo
property of exchanging Ca^'^ and Mg-"^ ions
oo
aluminium silicates, Na2Al2Si20g. XH2O have the interesting
present in hard water with sodium present in complex salts. The naturally occurring complex salts are called
zeolites. These may also be prepared artificially by fusing strongly a mixture of washing soda (Na2C03>,
eB

alumina (AI2O3) and silica (Si02). The fused mass is washed with water to remove soluble impurities and the
porous mass thus obtained is called permutit. Both zeolite and permutit can be represented by the general
formula, Na2Z where Z = Al2Si20g . XH2O. FIGURE 9.7
r

The zeolite or permutit is loosely packed over layers


ou

9b * NaCI
ad

of gravel and sand in a big tank (Fig. 9.7). Hard water is ^SOLUTION

introduced from the top into the base of the tank. From
Y

HARD
WATER
the bottom, water rises up through the gravel and sand
layers and finally percolates through the bed of the
nd

I
Re

permutit. During the process, the Ca^’*’ and ions ZEOLITE SOFT
are exchanged by sodium ions in the permutit. WATER
.4?
Fi

N&2Z + CaCl2 4
GRAVEL. SAND
Sodium zeolite {From hard water)
+-●
CaZ + 2NaC!
w
Calcium zeolite {left in the tank) WASTE

where Z = Al-,Si20g . XH2O. Permutit process for water softening


Similarly,
MgZ + 2NaCl
Na2Z +
MgCl2
Sod. zeolite {From hard water) Calcium zeolite {left in the tank)
As a result, Ca^'*’ and Mg^'*' ions get attached to the zeolite and the water which rises above the permutit
layer is reasonably soft. The softened water still contains sodium salts. But these sodium salts do not precipitate
soap and hence do not prevent the lather formation.
HYDROGEN 9/25

After some time, the whole of permutit gets exhausted due to the conversion of sodium zeolite into
calcium and magnesium zeolite. It can, however, be regenerated by passing a 10% solution of NaCl through it.
CaZ + 2NaCl
Na2Z + CaCl2
{Exhausted permutit) {Regenerated permutit)
MgZ + 2NaCI Na,Z + MgCl2
{Exhausted permutit) {Regenerated permutit)
The soluble calcium and magnesium salts thus formed are washed away by water and the regenera ed
permutit can be used again.
Advantages of the Permutit process

w
(/) It is an efficient process.
{it) This is a very cheap process since during the process only sodium chloride is consumed which is quite
cheap.

F lo
(///) It can be used to remove both temporary and permanent hardness completely,
(b) Organic ion exchangers. One of the major drawbacks of the inorganic ion exchangers (permutit
process) is that these can remove only Ca^'*' and Mg"'*’ ions present in hard water. However, recently it has

ee
been found that certain synthetic organic exchangers also called ion exchange resins are even superior to

Fr
zeolites since they can remove all types of cations (Na'*', Ca-'*', Mg-"*-, etc.) and anions (Cl", SO^" , HCOJ,
etc.) present in hard water. The resulting water is called demineralised water or deionised water and is as
good as distilled water.

for
Ion exchange resins are giant organic molecules of high molecular masses. These are essentially of two
ur
types:
(i) Cation exchange resins. These resins consist of giant hydrocarbon framework attached to acidic
groups such as - COOH {carboxyl) or -SO2OH {sulphonic acid) groups. They may be represented by the
s
ook
general formula R—COOH or R—SO^OH where R represents the giant hydrocarbon framework.
Yo
Since these resins can exchange ions with cations such as Ca^* and ions present in hard
wate,r they are called cation exchange resins or simply cation exchangers.
eB

(«) Anion exchange resins. FIGURE 9.8


These resins consist of giant
hydrocarbon framework attached HARD WATER
H*. Cl'; so^"
r

to basic groups such as OH"


ad
ou

(hydroxide) ions usually in form of


substituted ammonium hydro DIL HCI OR
DIL. NaOH
xides. These may be represented H2SOi_^
Y

=♦ FOR
by the general formula. FOR
REGENERATION
REGENERATION
+
Re

R—NHjOH where R denotes


nd

CATION*- ANION
EXCHANGE EXCHANGE
the giant hydrocarbon framework. RESIN RESIN
Fi

Since these resins can exchange GRAVEL- GRAVEL

OH~ ions with anions such as Cl~ (|SE I SOFT WATER


and SO^ ions present in hard WASTE WASTE

water, they are called anion Ion exchange process for water softening using organic resins
exchange resins or simply anion
exchangers.
Let us now explain the working of these resins in demineralizati on of hard water.
First of all, hard water is passed though a tank packed with cation exchange resins supported over gravel
(Fig. 9.8). All the cations present in hard water will exchange with H'*' ions furni.shed by resins as shown
below :
9/26 “pnadeefr New Course Chemistry (XI) ESC
2R—COO-H+ + CaCl2 - ■» (RC00)2Ca + 2H+ + 2Cr
(Cation exchange resin) (From hard water) {Exhausted resin)

2R—COO-H+ + MgS04 (RC00)2Mg + 2H+ + SOj-


(Cation exchange resin) (From hard water) (Exhausted resin)
Thus, the water which comes out from the bottom of the first tank is richer in
ions. This water is then
passe ’ through a second tank which is packed with anion exchange resins supported over gravel
(F;g. 9.8). Here, the Cl“ and SOj" ions present in hard water exchange with OH ions of the ersins.
+ +

R—NH3OH- + ci- R—NH3C1“ + OH“

w
(Anion exchange resin) (From hard water) (Exhausted resin)
+ +

2R—NH3OH- + SOj- (R—NH3)2SQ2- + 20H-

F lo
(Anion exchange resin) (From hard water) (Exhausted resin)
Simultaneously, the H'*’ ions coming from the first tank combine with OH" ions liberated in the second
tank to form water.

ee
H+ + OH" > H2O
Thus, the water obtained by this method is free from all types of cations as well as anions. Therefore,

Fr
this method is particularly useful for obtaining drinking water from sea water and also for obtaining pure
water for laboratory purposes.

for
Regeneration of resins. The exhausted ersin in the first tank is regenerated by treatment with moderately
concentrated H2SO4 or HCl.
ur
Ca(0(X:R)2 + 2H+C1" ■» 2RCOO-H+ + Ca^+ + 2C1"
(Exhausted resin) (Regenerated resin)
s
Similarly, the exchausted ersin in the second tank is ergenerated by treatment with moderately strong
ook
Yo
NaOH solution.
+
eB

R—NH3CI" +Na+OH" R—NH3OH- + Na+ + Cl"


(Exhausted resin) (Regenerated resin)
Thus, the cation and anion exchangers can be used over again and again. It makes this process both
efficient and cheap.
our
ad

3. Calgon process (sequestration). In this process, Ca^"^ and Mg^'*’ ions present in hard water are erndered
ineffective (sequestrated) by treatment with sodium poly metaphosphate, (NaP03)jp where x is as high as 100,
or more commonly with sodium hexametaphosphate. Na2[Na4(P03)g]. The trade name for sodium
Y

hexametaphosphate is calgon (which means calcium gone).


Re

When calgon is added to hard water, the Ca^'*' and Mg^"*" ions present in it combine with sodium
nd

hexametaphosphate to form soluble complex of calcium and magnesium salts.


2CaCl2 + Na2[Na4(P03)g] - ^ Na2 [Ca2(P03)g] + 4NaCl
Fi

(From hard water) Sod. hexametaphosphate Complex salt (solubie)


2MgS04 + Na2[Na4(P03)6l - 4 Na2[Mg2(P03)6l + 2Na2S04
(From hard water) Sod. hexametaphosphate Complex salt (soluble)
The complex calcium and magnesium ions do not form any precipitate with soap and hence readily
produce lather with soap solution.
The water softened by the above process can be used for laundry and household washing purposes and
also for raising steam in boilers. Calgon is also used in synthetic detergents to check the wastage of soap
during washing of clothes and also for ermoving boiler scales.
HYDROGEN 9/27

SUPPLEMENT YOUR
KNOWLEDGE FOR COMPETITIONS

1. Degree of hardness is defined as the number of parts of calcium carbonate or equivalent to various
calcium and magnesium salts present in a million parts of water by tnass. It is expressed in ppm.
Example. One kilogram of a sample of hard water contains 1 mg ofCaCl2 and I mg ofMgCl2. Find out
the total hardness in terms of parts of CaCO^, per 10^ parts of water by mass.
Solution. (0 Mol. mass of CaCl2= 111 g mor'
100
Now, lUgofCaCl2=100gofCaC03 lmgofCaCl2 = 111 X 1 mg of CaC03= 0-9 mg of CaC03
-I
(ii) Mol. mass of MgCl2 = 95 g mol

w
100
Now 95 g of MgCl2 = 100 g of CaC03 1 mg of MgCl2 = 95 X 1 mg of CaC03 = 1 05 mg of CaCOj
Thus, 1 kg of hard water contains = 0-90 + 1 -05 = 1 -95 mg of CaC03

F lo
Since 1 kg of water = lO^g = 10^ mg
10^ mg of hard water contains CaC03 = 1 -95 mg or Degree of hardness = 1-95 ppm.

ee
2. Estimation of hardness of water. The hardness of water due to and Mg-'*' ions is usually estimated
volumetrically. A known volume of hard water containing buffer solution of pH 10 is titrated against a

Fr
standard solution of EDTA (ethylenediamine tetraacetic acid disodium salt) using Eriochrome Black T as
indicator. Under these conditions, Ca^'*' and Mg^'*' ions form complexes with EDTA. When all the Ca^'*'
and Mg-'*' ions are consumed, the next drop of EDTA changes the colour of the indicator from wine red to

for
blue.
ur
9.11. HEAVY WATER

Chemically heavy water is deuterium oxide (D2O). It was discovered by Harold C. Urey, an American
s
Chemist in 1932. He showed that ordinary water contains traces (1 part in 6000 parts) of D^O.
ook
Yo
9.11.1. Preparation of Heavy Water
Heavy water is mainly prepared from ordinary water by prolonged electrolysis.
eB

1. By prolonged electrolysis. This method is based upon the principle that when ordinary water is
electrolysed, protium (or hydrogen) is liberated much more readily than deuterium (Do) because of the following
reasons :
r

(/) Being smaller in size, /f*" ions have greater mobility (or speed) as compared to D* ions,
ad
ou

(ii) Because of lower discharge potential, H'^ ions are discharged at the cathode more easily than D*
ions.

(in) Hydrogen atoms combine much more rapidly to form molecular hydrogen than do deuterium atoms
Y

to form £>2-
FIGURE 9.9
In view of the above reasons, as the electrolysis continues,
Re
nd

the concentration of heavy water in ordinary water gradually


increases. If electrolysis is continued till only a small volume
Fi

remains, then almost pure D2O is obtained. About 29,000 litres


of water must be electrolysed to gel one litre of 99% pure D2O.
The electrolytic cell designed by Brown, Degget and Urey GASES

for the preparation of heavy water is shown in Fig. 9.9. STEEL


CELL '
It consists of a steel cell (45 cm. long and 10 cm. in
diameter) which acts as a cathode. The anode is a cylindrical WATER
sheet of nickel with a number of holes punched in it. In actual NICKEL
ANODE' 00 o O 0

practice a large number of such cells are employed for 0°

electrolysis of water in a number of stages.


Ooo o

In India, heavy water is manufactured at Bhabha Atomic


Research Centre (BARC), Trombay in Bombay (Maharashtra) Electrolytic cell for the preparation of
by electrolysis of ordinary water. heavy water
9/28 New Course Chemistry (XI)CS19D

2. By fractional distillation. At normal atmospheric pressure, the boiling points of ordinary and heavy
water are 373 K and 37442 K respectively. This small difference in their boiling points forms the basis of
preparation of heavy water by fractional distillation of ordinary water. In actual practice, fractionating columns
of the order of about 12 metres height are employed.
9.11.2. Properties of Heavy Water
(a) Physical properties. It is a colourless, odourless and tasteless mobile liquid heavier than water.
Because of higher molecular mass, there is a marked difference in physical properties of ordinary water and
heavy water as shown in Table 9.4, page 9/20.
Further since dielectric constant of D2O is lower than that of H2O, therefore, ionic compounds are less
soluble in D-,0 than in H2O.
{b) Chemical properties, the chemical properties of heavy water are quite similar to those of ordinary
water. However, the reactions of heavy water with most of the substances proceed at a much slower rate as
compared to ordinary water. Some important chemical properties of heavy water are described below :

F low
1. Electrolysis. When heavy water is electrolysed, dideuterium is obtained at the cathode.
2D2O ■> 2 D2 + O2
Deuterium oxide Dideuterium

2. Action of alkali and alkaline earth metals. Heavy water reacts slowly with alkali and alkaline earth
metals producing dideuterium.
2 Na + 2 DnO 2NaOD + Do Ca + 2D2O Ca(OD)2 + D2
Sodium deuteroxide Calcium deuteroxide

Na20 + O')© ^ 2NaOD CaO + D2O


for Fre
3. Action of metallic oxides. The oxides of active metals like sodium and calcium react slowly with
heavy water to form their respective deuteroxides.
^ Ca(OD)2
4. Action of non-metalic oxides. Non-nieiallic oxides such as phosphorus pentoxide (P2O5), sulphur
troxide (SO3) etc. readily dissolve in heavy water forming their corresponding deuteroacids.
P2O5 + 3D2O 2 D3PO4 ; SO3 + D2O D2SO4
eBo ks
Your

Deuterophosphoric acid Deuterosulphuric acid


5. Action of metallic carbides. Heavy water reacts with metallic carbides forming deuterohydrocarbons.
AI4C3 + 12DoO ■> 4 AI(0D)3 + 3CD4
ad

Aluminium carbide Deuteromethane


our

CaC2 + 2DoO ^ Ca(OD)2 + DCs CD


Calcium carbide Deuteroacetylene
6. Action with metallic nitrides, phosphides and arsenides. Heavy water reacts with metallic nitrides,
Re

phosphides and arsenides liberating deuteroammonia, deuterophosphine and deuteroarsine re.spectively.


Mg3N2 + 6 D->0 ^ 3Mg(OD)2 + 2ND3
Deuteroammonia
Find Y

Cal. nitride

Ca3P2 + 6 0,0 3Ca(OD)2 + 2PD3


Cal. phosphide Deuterophosphine

NajAs + 3D2O -> 3 NaOD + ASD3


Sod. arsenide Deuteroarsine

7. Formation of deuterates. Like ordinary water, heavy water also combines with many compounds as
heavy water of crystallization. The heavy hydrates thus obtained are called deuterates. For example,
CUSO4.5 D2O ; Na2S04. 10 D2O ; MgS04. 7 DjO, NiCl2- 6 D2O, C0CI2. 6 D2O, etc.
8. Exchange reactions. When treated with heavy water, many compounds exchange their active hydro
gen atoms either partially or completely with deuterium. Some of these exchange reactions are given below :
HYDROGEN 9/29

HCI + DoO V - DCI + HOD : NaOH + D2O NaOD + HOD


CHCI3 + D^O V - CDCI3 + HOD : NH4CI + D2O V ^ NH3DCI + HOD
Chloroform Deuterochloroform

NH3DCI + D2O ^ NH2D2CI + HOD : NH2D2CI + D2O ^ NHD3CI + HOD


NHD3C! + D2O ND4CI + HOD
9. Biological properties. Heavy water is injurious to human beings, plant.s and animals since it slows
down the rates of reactions occurring in them. Thus heavy water does not support life so well as does ordinary
water.

w
9.11.3. Uses of Heavy Water
1. As a moderator. Heavy water is extensively used as a moderator in nuclear reactions since it slows
down the fast moving neutrons and thus helps in controlling the nuclear reactions.

F lo
2. As a tracer compound. Heavy water is widely used as a tracer compound for studying the mechanism
of many reactions. For example, when hypophosphorus acid (H3PO2) is treated witli D2O, only one H-atom
is found to exchange with deuterium. This suggests that H3PO2 has only one ionizable H-atom and hence it is
a monobasic acid.

ee
3. For the preparation of dideuterium. Heavy hydrogen or dideuterium is produced by the electrolysis

Fr
of heavy water or by its decomposition with Na metal.
SUPPLEMENT YOUR
KNOWLEDGE FOR COMPETITIONS

for
1. Deuterium peroxide is prepared by the action of D2SO4 dissolved in D2O on BaO^.
ur
Ba02 + D2SO4 > BaS04 + D2O2
2. Deuterium chloride may be prepared by any one of the following methods :
s
2 AgCl + D2 > 2 Ag + 2 DCI ; 2 NaCl + D2SO4 > Na2S04 + 2 DCI
ok
Yo
SOCI2 + 2 D2O > D2SO3 + 2 DC)
o
eB

9.12. HYDROGEN PEROXIDE

Hydrogen peroxide was discovered by the French Chemist J.L Thenard in 1818. Its molecular formula
is H2O2.
r
ad
ou

9.12.1. Preparation of Hydrogen Peroxide


1. From sodium peroxide (Merck’s method). Calculated amount of sodium peroxide (Na20-,) IS

gradually added to an ice-cold solution of 20% H2SO4.


Y

+ H2SO4 ■> Na2S04 + H2O2


Sodium peroxide
Re

Hydrogen peroxide
nd

Upon cooling, crystals of Na2S04. IOH2O separate out and the resulting solution contains about 30%
H2O2. The solution also contains some dissolved Na2S04 but it does not interfere with the reactions of H2O2.
Fi

A pure sample of H2O2 may, however, be prepared by vacuum distillation.


2. From barium peroxide—Laboratory method of preparation. Hydrogen peroxide is prepared
from barium peroxide by the following methods :
{a) By the action of dilute .sulphuric acid. A thin paste of hydrated barium peroxide (Ba02.8H20) is
prepared in ice-cold water and then added slowly to an ice-cold solution of 20% H2SO4.
Ba02-8H20 (.s) + H2SO4 (aq) BaS04 (.y) + H2O2 {aq) + 8H2O (/)
Hydrated barium peroxide Hydrogen peroxide
The white precipitate of BaS04 is removed by filtration leaving behind a dilute solution (5%) of H7O2.
In this method, anhydrous barium peroxide cannot be used since the precipitated BaSO^ forms a protective
layer around unreacted barium peroxide thereby preventing the further reaction.
9/30 Nt w Course Chemistry (XI)EJSIHII

Limitation. Hydrogen peroxide prepared by this method contains appreciable quantities of Ba^"^ ions
(in the form of dissolved barium persulphate) which catalyse the decomposition of H2O2. Therefore, H2O2
prepared by this method cannot be stored for a long time.
Further, H2SO4 also acts as a catalyst for decomposition of H2O2 therefore, the use of weaker acids
such as CO2 (or H2CO3) and H3PO4 are preferred to H2SO4.
(h) Bv the action of carbon dioxide or carbonic acid. When a rapid stream of CO2 is bubbled through
a thin paste of Ba02 in ice-cold water, H2O2 and BaC03 are produced :
Ba02 + H2O CO2 -> BaC03 'J' + H2O2

low
The insoluble barium carbonate is removed by filtration leaving behind a dilute solution of H2O2.
(c) By the action ofphosphoric acid. Hydrogen peroxide can also be prepared by the action of phosphoric
acid on barium peroxide ;
3 Ba02 + 2H3PO4 ■>
Ba3(P04)2i + 3H2O2
This method has the advantage over Ba02 - H2SO4 method since almost all the heavy metal {e.g. Pb

e
etc.) impurities present in Ba02 and which catalyse the decomposition of H2O2 are removed as insoluble

re
phosphates. As a result, the resulting solution of H2O2 has good keeping properties.
9.12.2. Manufacture of Hydrogen Peroxide

rF
F
1. By electrolysis of 50%. H2SO4. Hydrogen peroxide is manufactured by the electrolysis of a cold
50% solution of H^S04 at high current density in an electrolytic cell using platinum as anode and graphite as

r
cathode. The reactions taking place are :

fo
u 2H2SO4 ^ 2H+ + 2HSO-

2H+ + 2e- H2T


At cathode :
ks
Yo
At anode : 2HSO' H2S2O8 + 2e~
oo
Peroxodisulphuric acid
Peroxodisulphuric acid formed around anode is withdrawn and then distilled with water under reduced
eB

pressure. The low boiling H2O2 distils over along with water leaving behind high boiling H2SO4 which is
recovered and recycled.
H2S2O8 + H2O H2SO5 + H2SO4
ur

Peroxodisulphuric acid Peroxomono-sulphuric acid


ad

H2SO5 + H2O H2SO4 +


H2O2
Yo

Peroxomonosulphuric acid Hydrogen peroxide


Modification. Recently, it has been observed that if instead of 50% H2SO4, an equimolar mixture of
H2SO4 and ammonium sulphate is electrolysed, a more concentrated solution of H2O2 is obtained. The
d

reactions taking place are :


Re
in

(NH4)2S04 + H2SO4 2NH4HSO4


Amm. hydrogen sulphate
F

2NH4HSO4 2H+ + 2 NH4SO4


At cathode : 2H-" + 2 c- -> H2 T
At anode : 2NH4SO- ■> (NH4)2S-50g + 2c
Amm. persulphate
Ammonium persulphate formed around anode is withdrawn and distilled with water to give H2O2.
(NH4)2S20g + 2H2O ●> 2NH4HSO4 + H2O2
This proces.s is now used for the laboratory preparation of D2O2, i.e.,
K2S20g + 2 D2O 2 KDSO4 + D2O2
HYDROGEN 9/31

2. By auto-oxidation of 2-ethylanthraquinol. This is a new method and is widely used in U.S.A. In


this process, air is bubbled through a 10% solution of 2-ethylanthraquinol in benzene and cyclohexane when
2-ethylanlhraquinol is oxidised to 2-ethylanthraquinone and H2O2 is formed according to the following equation.
OH O

.C2H5 ●C2H5
+ H2O2

w
HVPd

OH O
2-Etliylanihraquinol 2-Ethylanthraquinone
The H2O2 thus formed (about 1%) is extracted with water and the aqueous solution is concentrated by

o
distillation under reduced pressure to give 30% (by weight) H,02 solution.

e
2-Ethylanthraquinone formed in the process is reduced with H9 in presence of Pd catalyst to give back
2-ethylanthraquinol which is used again. Thus, in this process, only atmospheric oxygen and hydrogen are

re
rFl
used up which are inexpensive. Therefore, this method is quite cheap and is widely used for the manufacture
of H2O2.

F
Retain in Memory
H2O2 can also be prepared commercially by partial oxidation of 2-propanol. Little H9O2 is added to

r
initiate the reaction
ou
fo
CH CH
Little H2O2+O2
CHOH C = 0 +H,Oo
CH3.
2-Propanol
Under pressure CH3'
ks
Propanone
oo
9.'] 23. Concentration of Hydrogen Peroxide solution
Y
Hydrogen peroxide prepared by any of the above methods is in the form of dilute aqueous solution.
B

Quite often for a large number of reactions, we need a concentrated solution of H9O2. It cannot be concentrated
simply by distillation since it decomposes much below its boiling point to give H2O and 0-)
re

2H2O2 2H2O -I" O2


Further, the process of decomposition is catalysed by presence of heavy metal ion impurities, dust and
ou
Y

rough surfaces. In view of these difficulties, concentration of H2O2 is carried out carefully in a number of
ad

stages as follows:
(i) Evaporation on a water bath. The dilute aqueous solution of hydrogen peroxide is taken in a large
shallow evaporating dish and is heated on a water bath. Slow evaporation of water continues until the solution
d

contains about 50% hydrogen peroxide. Further concentration of hydrogen peroxide by this method is not
in
Re

possible since it tends to decompose, FIGURE 9.10


(ii) Dehydration in a vacuum desiccator. 50% solution of
H2O2 as obtained above is taken in a dish and placed in a vacuum
F

TO VACUUM
desiccator containing cone. H2SO4 as shown in Fig. 9.10. PUMP

Pressure inside the desiccator is reduced by connecting it to a


vacuum pump. As a result of low pressure, water readily evaporates
and the water vapours thus produced are absorbed by cone. H2SO4. DiL.Hp^
In this way, hydrogen peroxide of about 90% concentration is
VACUUM
produced. DESICCATOR
(Hi) Distillation under reduced pressure. The 90% solution
of hydrogen peroxide as obtained above is subjected to distillation CONC. H2SO4
under reduced pressure (10 - 15 mm). During this process, water
distils over between 303 - 313 K leaving behind almost pure (99%) Vacuum Desiccator
hydrogen peroxide.
9/32 'Pfuxde^ 'A New Course Chemistry (XI)S!EISI]

(iv) Removal of last traces of water. The last traces of water in hydrogen peroxide are removed by
freezing it in a freezing mixture consisting of dry ice (solid carbon dioxide) and ether when crystals of
hydrogen peroxide separate out. These crystals are removed, dried and melted to give pure hydrogen peroxide.
Storage of hydrogen peroxide. The following preca itions are taken while storing hydrogen peroxide
solution.

(/) Hydiogen peroxide cannot be stored in glass bottles since the rough surface of glass, alkali oxides
present in it, and exposure to light catalyse its decomposition. Therefore, H2O2 is usually stored in coloured
parafifn wax coated plastic or teflon bottles,
(ii) To further check the decomposition of H^O,, some stabilizer or negative catalyst* such as urea,
glycerine, acetanilide, phosphoric acid, etc. must also be added.

w
9.12.4. Strength of Hydrogen Peroxide solution
The strength of an aqueous solution of hydrogen peroxide is usually expressed in the following two ways :
(a) Percentage strength. It expresses the amount of H2O2 by weight present in 100 mL of the solution.

F lo
For example, a 30% aqueous solution (w/v) of H2O2 implies that 30 grams of H2O2 are present in 100 mL of
the solution.

(/?) Volume strength. The most common method of expressing the strength of an aqueous solution of
hydrogen peroxide is in terms of the volume (in inL) of oxygen liberated at N..TP. by the decomposition of 1

ee
mL of that sample of hydrogen peroxide. Thus, the aqueous solutions of hydrogen peroxide sold in the market

Fr
are labelled as 70 volume, 20 volume, 30 volume, 100 volume, etc. A solution of hydrogen peroxide labelled
as 10 volume actually means that 7 niL of such a solution of hydrogen peroxide on decomposition by heat

produces 10 mL of oxygen at N..TP.

for
Similarly, 1 mL of 20 volume, 30 volume and 100 volume H2O2 solutions produce 20 mL, 30 mL and
100 mL of oxygen at N.T.P. respectively.
ur
Sample Problem □ Calculate the normality of 20 volume
NUMERICAU hydrogen peroxide solution.
s
ook
PROBLEMS
Solution. Step 1. To calculate the strength in g/l of 20 volume H2O2
Yo
BASED
solution.
ON
eB

By definition, 1 litre of 20 volume H2O2 solution on decomposition gives


20 litres of oxygen at N.T.P.
The Strength of
Hydrogen Consider the chemical equation.
Peroxide 2H2O2 ^ 2H2O + O2
our
ad

Solutions 2 X 34 = 68 g 22-4 litres at N.T.P.

Now 22-4 litres of O2 at N.T.P. will be obtained from H202= 68 g


68 X 20
.-. 20 litres of at N.T.P. will be obtained from H2O2 = 60-7 g
Y

- 22-4
Re

Thus, the strength of 20 volume H2O2 solution = 60-7 g/l


nd

Step 2. To calculate the equivalent weight ofH202-


Consider the chemical equation. 2H2O2 4 2H2O + O2
Fi

68 parts by weight 32 parts by weight


From the above equation, 32 parts by wt. of oxygen are obtained from 68 parts by wt. of H2O2
68
8 pails by wt. of oxygen will be obtained from ^ ^ ^ ^t. of H2O2 Eq- wt. of H2O2 = 17
Step 3. To calculate the normality of 20 volume H2O2 solution.
Strength _ 60-7 = 3-57
Now we know that, Normality = 17
Eq. wt.
Hence, the normality of 20 volume H2O2 solution = 3*57 N
*A negative catalyst retards or decreases the rate of a chemical reaction.
HYDROGEN 9/33

^amjgf|5[p‘5ojl|'iem|B Find the volume strength of 1«6 N H2O2 solution.


Solution. We know that strength = Normality x Eq. wt. and Eq. wt. of H2O2 = 17
Strength of 1-6 N H2O2 solution = 1*6 x 17 g//
Now 68 g of H2O2 gives 22400 mL O2 at N.T.P.
22400
1’6 X 17 g of H2O2 will give X 1-6 X 17 = 8960 mL of O2 at N.T.P.

ow
68

But 1-6 X 17 g of H2O2 are present in 1000 mL of H2O2 solution.


Hence, 1000 mL of H2O2 solution gives 8960 mL of O2 at N.T.P
8960
1 mL of H2O2 solution will give = 1000 = 8-96mLof02 at N.T.P.
Hence, the volume strength of 1 -6 N H2O2 solution = 8*96 volume

e
re
Calculate the volume strength of a 3% solution of H2O2.

rFl
Solution. Step 1. To calculate the amount of H2O2 present in one litre of 3% solution.

F
100 mL of H2O2 solution contain H2O2 = 3 g
3
1000 mL of H2O2 solution will contain H2O2 = ~
lUU
X 1000 = 30 g

or
Step 2. To calculate the volume strength
Consider the chemical equation,
ou 2 H2O2 » 2H2O + O2

ksf
2 X 34 = 68 g 224 litres at N.T.P

Now 68 g of H2O2 give O2 at N.T.P = 22-4 litres


224
oo
30 g of H2O2 will give O2 at N.T.P. = 68
x30 = 9-88 litres = 9880 mL.
Y
But 30 g of H2O2 are present in 1000 mL of H2O2.
B

Hence, 1000 mL of H2O2 solution gives O2 at N.T.P. = 9880 mL


9880
re

1 mL of H2O2 solution will give O2 at N.T.P. = 1000


= 9-88 mL.

Hence, the volume strength of 3% H2O2 solution = 9*88


oYu
ad

What is the mass of hydrogen peroxide present in 1 litre of 2M solution ?


Calculate the volume of oii^gen at STP liberated upon complete decomposition of 100 cm^ of the above solution.
Solution. Step 1. To calculate the mass of H2O2 in 1 litre of2M solution.
d

Molecular mass of H2O2 = 2x1+2x16 = 34 u.


in

By definition, 1 litre of 1 M H2O2 contains 34 g of H2O2


Re

1 litre of 2M H2O2 will contain H2O2 = 34 x 2 = 68 g


Step 2. To calculate the volume of O2 liberated at STP from 100 cm^ of 2M solution.
F

1 litre of 2M H2O2 solution contains H2O2 = 68 g


68
100 cm^ of 2M H2O2 solution will contain H2O2 = 1000
X100 = 6-8 g
The equation representing the decomposition of H2O2 is
2H2O2 - + 2H2O + O2
2 X 34 = 68 g 22400 cm% STP
Now
68 g of H2O2 at STP give O2 = 22400 cm^
22400
6*8 g of H2O2 at STP will evolve O2 = x6-8 = 2240 cm^ = 2-24 Utres
68
9/34 New Course Chemistry (XI)CEIHB

Sample Problem 30 mL of a H2O2 solution after acidification required 30 mL of N/10 KMn04


solution for complete oxidation. Calculate the percentage and volume strength of H2O2 solution.
Solution. Step 1. To determine the normality of H2O2 solution. From the given data,
For H2O2, V, = 30 mL, NI = ?
For KMnO^, V9 = 30 mL, N2 = N/10
N.1 =0-lN
Applying normality equation, N[V| = N2V2, i.e., 30 x Nj = 30 x 1/10
Thus, the normality of H2O2 solution = 0*1 N.
Step 2. To determine the percentage strength solution,
We know that. H2O2 ^ 2 + O2 + 2 e- Eq.wt.ofH202 = 34/2= 17
Hence, strength of H^02 solution = Normality x Eq. wt. =0-1 x 17 = 1-7 g/litre

w
1-7x100
% Strength of H2O2 - = 017%.
1000

F lo
Step 3. To determine the volume strength of H2O2 solution.
Consider the chemical equation.
2H2O2 4 2H2O + O,
68 g 22400 mL at N.T.P.

ee
Now 68 g of H20-, give O2 at N.T.P. = 22400 inL

Fr
22400
X1-7 = 560 mL
1 -7 g of H2O2 will give O2 = 68

for
But 1-7 g of H^O-^ are present in 1000 mL of H2O2 solution.
Hence, 1000 mL of H2O2 solution gives 560 mL of O2 at N.T.P.
r
560
You
1 mL of H2O2 solution will give - = 0-56 mL of O2 at N.T.P.
s
1000
ook

Volume strength of H2O2 solution = 0*56.


SUPPLEMENT YOUR
eB

KNOWLEDGE FOR COMPETITIONS

Problems on percentage strength, normality, molarity and volume strength of H2O2 can be easily solved by
the application of the following relations.
our

1. Volume strength = 5*6x Normality


ad

Percentage strength xlO = 5-6x Strength in gL ^


= 5*6 X
Eq. wt. of H2O2 (i.e. 17) Eq. wL of H2O2 17)
2. Volume strength = 11-2 x Molarity
Y

Percentage strength Strength in gL ^


Re

= 11*2 X xlO =5-6x


Mol. wt of H2O2 (i.e. 34) Eq. wt of H202(L«.» 17)
d
Fin

wmm
1. Calculate the concentration in grain/litre of a 20 volume H2O2 solution.
2. Find the volume strength of 2N H2O2 solution.
3. Calculate the normalityof a solution of I litre of hydrogen peroxide labelled 30 volumes.
4. Calculate the amount per litre of 10 mL of a solution of hydrogen peroxide labelled 20 volumes.
5. Calculate the strength of 5 volume H2O2 solution ?
ANSWERS
-1
1. 60-7 g/1 2. 11.2 volume 3. 5-357 N 4. 60*7 g/litre 5. 15-18 gL
HYDROGEN 9/35

9.12,5. Properties of Hydrogen Peroxide


(a) Physical properties. I. Pure hydrogen peroxide is a thick syrupy liquid with pale blue colour.
2. It has a bitter taste.

3. Hydrogen peroxide is more dense (I -44 g/cm^) and more viscous than water. This is due to the reason

that the molecules of H2O2 are even more highly associated through H-bonds than H2O molecules.
4. Its in.p. is 272.4 K. Since it decomposes vigorously on heating, it is not possible to determine its b.p.
at atmospheric pressure. However, its b.p. has been determined to be 423-2 by extrapolation method.

ow
5. It is completely miscible with water, alcohol and ether in all proportions.
6. The dipole moment of H2O2 is little more (2-1 D) than that of H^O (1-84 D).
TABLE 9.5.
Some Physical Characteristics of HjOa
Melting point (K) 272-4 Density (liquid at 298 K) (g cm“^) 1 -4425
Boiling point (extrapolated) (K) 423 Viscosity at 298 K (Centipoise)

e
1-245

Fl
Vapour pressure at 298 K (mm Hg)

re
1-9 Dielectric constant of 298 K 70-7

Density (solid at 268-5) (g cm"^) 1-6434 Electrical conductivity at 298 K (0“* cin“') 15-1 X 10"^

F
(b) Chemical properties. 1. Decomposition. Pure hydrogen peroxide is an unstable liquid and
decomposes into water and oxygen on long standing or heating.
ur
r
2H2O2 (aq) ^ 2H20(/) -f 02(g); AH = -196.0 kJ

fo
It is an example of aulo-oxidation and auto-reduction.
The decomposition is further accelerated by the presence of certain metal ions (e.g. Fe^"^). metal powders
ks
(Co, Au, Ag, Pt etc), and metal oxides (e.g. Mn02). Even charcoal, rough surfaces and light also catalyse its
Yo
decomposition. Therefore, the solutions of H2O2 must be handled with care since they may explode with
oo
traces of organic matter or even specs of dust.
2. Acidic nature. Pure hydrogen peroxide turns blue litmus red but its dilute solution is neutral to
B

litmus. It thus behaves as a weak acid. Its dissociation


constant is T55 x 10'*^ at 293 K which is only slightly
higher than that of water (TO x 10“*^). Thus, hydrogen peroxide is only a slightly stronger acid than water.
re

Since hydrogen peroxide has two ionizable H-atoms, it forms two series of salts, i.e.. hydroperoxides (acidic
salts) and peroxides (normal salts).
u
ad

H2O2 ^ - H'*’ -f HO2 (hydroperoxide ion) ; H2O2 ^ - 2H'*’ + 0| (pero.xide ion)


Yo

The acidic nature of hydrogen peroxide is shown by its neutralization reactions with hydroxides.
NaOH + H2O2 ■)
NaH02 + H2O 2NaOH + H2O2 Na202 + 2H2O
d

Sod. hydroperoxide Sod. peroxide


Re

Ba (0H)2 + H2O2 ^ BaO, + 2H2O


in

Howeve,r since is a weaker acid than carbonic acid (H2CO^), therefore, it does not decompose
carbonates and bicarbonates to evolve CO^
F

3. Oxidising and reducing character. Hydrogen peroxide behaves as an oxidising as well as a reducing
agent in both acidic and alkaline solutions. The oxidation state of oxygen in hydrogen peroxide is -1. It can
be oxidised to O2 (zero oxidation state) or reduced to H2O or OH~ (-2 oxidation state for oxygen). However,
hydrogen peroxide is a powerful oxidising agent but a weak reducing agent.
(a) Oxidising character. Hydrogen peroxide acts as an oxidising agent both in acidic as well as in
alkaline medium.

In acidic medium. H2O2 (aq) + 2H* (aq) + 2e ^ 2H2O (0


In basic medium. H2O9 (aq) + 2e~ ^ 20H-(aq)
9/36 7^>u^dce^'4. New Course Chemistry (XI)E!ZsX9n

Some important reactions in which hydrogen peroxide acts as an oxidising agent are given below :
I. Oxidising action in acidic medium.
(0 It oxidises acidified ferrous sulphate to ferric sulphate
H2O2 ^ H2O + [O]
2FeS04 + H2S04+[0] ^Fe2(S04)3 + H20
2FeS04 + H2S04 + H202 ^Fe2 (S04)3 + 2H20
or
2Fe“+ (aq) + H2O2 (aq) + 2H+ (aq) ■> 2Fe^+ {aq) + 2H2O (/)
(ii) It oxidises acidified potassium ferrocyanide to potassium ferricyanide
H2O2 ^ H2O + [O]
2K4[Fe(CN)6l + H2SO4 + [O] ^ 2K3[ Fe(CN)g] + K2SO4 + H2O

w
2K4 [Fe(CN)g + H2SO4 + H2O2 ^ 2K3[Fe(CN)6] + K2SO4 + 2H2O
or 2[Fe(CN)6l^ (aq) + H2O2 (aq) + 2H+ (aq) 4 2 [Fe(CN)6]3- (aq) + 2H2O (/)

F lo
(Hi) It liberates iodine from acidified potassium iodide solution
H2O2 H2O + [O]
2KI + H2S04+[0] 4K2SO4 + I2 + H2O

ree
2KI + H2SO4 + H2O2 >K2S04 + l2 + 2H20

F
or
21- (aq) + H2O2 (aq) + 2H+ (aq) »l2(s) + 2H20 (/)
(iv) Hydrogen peroxide reacts* with ice-cold acidified potassium dichromate solution (containing ether)

for
to chromium pentoxide which dissolves in ether producing a blue colouration.
H2O2 » H2O + [O] X 4
K2Cr207 + H2S04 + 4[0] » K2SO4 + 2C1O5 + H2O
r
You
^^2^2 ^ K2SO4+ 2Cr05 + 5H2O
oks

Chromium pentoxide
eBo

o. .0

Chromium pentoxide is actually a peroxide having the structure I >r:


O O
O
(v) Hydrogen peroxide oxidises lead sulphide to lead sulphate
ad
our

H2O2 ■> H2O + [O] X 4


PbS + 4[0] -» PbS04
PbS (5) + 4H2O2 (aq) PbS04(^) +4H20(/)
dY
Re

Lead sulphide (black) Lead sulphate (white)


This reaction is used in restoring the white colour of lead paintings which have blackened due to the
formation of lead sulphide by the action of H2S present in the air. On treatment with H2O2, lead sulphide
Fin

(black) changer into lead sulphate (white) and thus the colour of lead paintings is restored.
(vO Hydrogen peroxide oxidises H2S to sulphur and sulphurous acid to sulphuric acid.
H2S(g) + H202(fl^) 2 H2O (/) + S (s)
H2SO3 (g) + H202(fl^) -> H2SO4 (aq) + H2O (/)
(vii) It oxidises mercury to mercuric oxide in acidic medium
DU. H2SO
Hg (0 + H2O2(a<?) U HgO (s) + H2O (0
♦This is a complicated reaction. It cannot be regarded as an oxidation reaction since O.N. of Cr in both
K2Cr207 and CrOg is + 6.
HYDROGEN 9/37

II. Oxidising action in alkaline medium


(/) It oxidises sulphites, nitrites and arsenites to sulphates, nitrates and arsenates respectively in alkaline
medium.

Na2S03 + H2O2 ^ Na2S04 + H2O


Sod. sulphite Sod. sulphate
or
S02- (fl^) + H2O2 (a^) ^ SQ2- (o^) + H20(/)
KNO2 + H2O2 ■4 KNO3 + ^^2^

low
Pot. nitrite Pot. nitrate

or
NO2 + H2O2 - 4 NOJ + H2O
NajAsOj + H2O2 - ^ Na3As04 + H2O
Sod. arsenite Sod. arsenate

or
AsO^- + H2O2 > AsOj" + H2O

ee
rF
(h) It oxidises manganese salts to manganese dioxide in alkaline medium.

Fr
MnS04 +H202 + 2Na0H ^ Na2S04 + Mn02 + 2H2O
Manganese sulphate Manganese dioxide
Mn2+ {aq) + H2O2 {aq) + 2 OH" (aq) > Mn02 (^) + 2 H2O (/)

for
or

{Hi) It oxidises ferrous salts to ferric salts in alkaline medium.


2 FeS04 + 4 NaOH + H2O2 > 2 Fe(OH)3 + 2 Na2S04
u
or
2 Fe2+ {aq) + 4 OH" {aq) + H2O2 {aq) > 2 Fe(OH)3 {s)
s
ook
(/v) It oxidises chromium salts to chromates in alkaline medium.
Yo
Cr2(S04)3 + 3 H2O2 + 10 NaOH ^ 2 Na2Cr04 + 3 Na2S04 + 8 H2O
Chromium sulphate Sod. chromate
eB

or 2 Cr3+ {aq) + 3 H2O2 {aq) + 10 OH" {aq) ■»


2Cr02- (o^) + 8H20(/)
(v) It formaldehyde to formic acid
r

HCHO + H2O2 ^ HCOOH + H2O


ou
ad

Formaldehyde Formic acid

(vz) It oxidises benzene to phenol in alkaline medium


Y

CgHg + H2O2 ^ CgHjOH + H2O


Benzene Phenol
Re
nd

(b) Reducing character. In presence of strong oxidising agents, hydrogen peroxide behaves as a reducing
agent both in acidic as well as in alkaline medium. In all these reactions, molecular oxygen is always produced
Fi

by the combination of H2O2 with the oxygen atom released by the strong oxidising agent:
H2O2 + [O] ■> H2O + O2
from oxidising agent
Acidic medium. ^2^2 (^9) » 2H+(o^) + 02(g) + 2e-
Alkaline medium. H2O2 {aq) + 20H- {aq) > 2 H2O (/) + O2 (g) + 2
Some important reactions in which hydrogen peroxide behaves as a reducing agent are given below :
L Reducing action in acidic medium
(z) It reduces acidified potassium permanganate solution. As a result of this reaction, the pink colour of
KMnO^ solution is discharged
9/38 "pn<idee^'^ New Course Chemistry (X1)^I9D

2KMn04 + 3H2S04 ^ K2SO4 + 2 MnS04 + 3 H2O + 5 [O]


H202 + [0] ^ H2O + O2] X 5
2 KMn04 + 3 H2SO4 + 5 H2O2 ■> K2SO4 + 2 MnS04 + 8 H2O + 5 O2

ow
or
Mn04 {aq) + 8 H+ (a^) + 5 e" {aq) + 4 H2O (01 x 2
H202(o^) ^ 2 H+ {aq) + O2 ig) + 2e~]x5

2 MnO- {aq) + 6 H+ {aq) + 5 H2O2 {aq) > 2 Mn2+ {aq) + 8 H2O (0 + 5 O2 (g)

e
(I’O It reduces acidified potassium dichromate solution. As a result of this reaction, the orange colour of
K2Cr20’j changes to green due to the formation of chromium salt.

re
K2Cr207 + 4 H2SO4 > K2SO4 + Cr2(S04)3 + 4 H2O + 3 [O]
H2O2 + [O] > H2O + O2] X 3

F
Frl
K2Cr207 + 4 H2SO2 + 3 H2O2 ■ ^ K2SO4 + Cr2(S04)3 + 7 H2O + 3 O2
or
CrjO^" {aq) + 14 {aq) + 6 e" -4 2Cr3+(fl^) + 7H2O(0
ou
H202(a^) ->2H+{aq) + 02{g) + 2e-]x3

osr
Cr.p^ {aq) + 8 {aq) + 3 H2O2 {aq) 2Cr3+(fl^) + 7H2O(0 + 3O2 {g)
(/«) It reduces manganese dioxide to manganese sulphate in presence of dil H2SO^.

kf
Mn02 + H2SO4 -
oo ^MnS04 + H20 + [0]
H202+[0] —> H2O + O2
Mn02 + H2SO4 + H2O2 ^ MnS04 + 2 H2O + O2
Y
or
Mn02 {aq) + 2H'*' {aq) + H2O2 {aq) ^ Mn2+ {aq) + 2 H2O (/) + O2 {g)
B

(iv) It reduces ozone to dioxygen


^ 02 + 10]
re
uY

H2O2 + [O] + H2O + O2


H2O2 + O3 + H2O + 2 O2
ad
do

(v) It reduces chlorine and bromine to HCl and HBr respectively.


CI2 + H2O + 2 HCl + [O]
H202 + 10] + H2O + O2
in

H2O2 + CI2 + 2 HCl + O2


Re

and H2O2 + 8^*2 2 HBr + O2


F

Since H2O2 can reduce CI2 to HCl, therefore, it acts as an antichlor in bleaching by removing the
excess unreacted CI2.
(vO It reduces hypohalous acid to halide ion in acidic medium
HOCl + H2O2 > HCl + H2O + O2
or HOCl {aq) + H2O2 {aq) > H3O+ {aq) + Cl" {aq) + O2 {g)
n. Reducing action in alkaline medium
(j) It reduces potassium permanganate to manganese dioxide in alkaline medium.
2 KMn04 {aq) + 3 H2O2 {aq) > 2 Mn02 {s) + 2 KOH {aq) + 3 O2 (g) + 2 H2O (/)
or
2 Mn04 {aq) + 3 H2O2 {aq) ^ 2 Mn02 {s) + 2 OH- {aq) + 3 02(g) + 2 H2O (/)
HYDROGEN
9/39

(ii) It reduces ferric salts to ferrous salts in alkaline medium


^62(804)3 + H2O2 + 2 NaOH - ^ 2 FeS04 + Na2S04 + O2 + 2 H2O
or
2 (aq) + H2O2 (aq) + 2 OH" (aq) 2 Fe2+ (aq) + Oj (g) + 2 H2O (/)
(Hi) It reduces alkaline potassium ferricyanide to potassium ferrocyanide
2 K3[Fe{CN)6] + 2 KOH + H2O2 > 2 K4[Fe(CN)6] + 2 H2O + O2
Pot. ferricyanide Pot. ferrocyanide
or
2 [Fe(CN) (aq) + 2 OH" (aq) + H2O2 (aq) ^ 2 [Fe(CN)g]‘^ (aq) + 2 H2O (0 + O2 (g)
(<v) It reduces metal oxides to metals, i.e., silver oxide to silver in the alkaline medium. Howeve,r lead
dioxide is reduced to lead monoxide.

w
A§20 + H2O2 2Ag + H->0 + O2
Pb02 -f H2O2 PbO + H2O + Ot
Lead dioxide

F lo
Lead monoxide

(v) It reduces halogens to halide ions in basic medium


CI2 + H2O2 + 2 KOH 4 2 KCl + 2 H2O + O^
or
^^2 + H2O2 (aq) + 2 OH" (aq) 2Cl“(fl^) + 2H20‘'(/) + 02(g)

ee
I2 + H2O2 + 2 NaOH 2NaI + 2H2O + 02

Fr
or I2 (ji') + H2O2 (aq) + 2 OH" (aq) ■ >21" (aq) + 2 H2O (/) + O2 (g)
(vi) It reduces hypohalites to halides in alkaline medium

for
NaOBr + H2O2 » NaBr + H2O + O2 or BrO“ + H202 ^ Br" + H2O + O2
Sod. hypobromite
ur
CaOCl2 + H2O2 ^ CaCl2 + H2O + O2 or OCl" + H2O2 ■> Cl + H2O + O2
Bleaching powder
s
(vii) It reduces potassium periodate to potassium iodate is alkaline medium.
ook
Yo
KIO4 + H2O2 ■>
KIO3 + H2O + O2
Pot. periodate Pot. iodate
eB

4. Bleaching action. The bleaching action of hydrogen peroxide is due to the nascent oxygen which it
liberates on decomposition.
H2O2 -> H2O + [O]
The nascent oxygen combines with colouring matter which, in turn, gets oxidised. Thus, the bleaching
our
ad

action 0///2O2 to the oxidation of colouring matter by nascent oxygen. It is used for the bleaching of
delicate materials like ivory, feathers, silk, wool, etc.
Colouring matter + [O] ■> Colourless matter
Y

5. Addition reactions. Hydrogen peroxide reacts with alkenes to form glycols.


Re

CH- CH^OH
nd

II ' + H2O2 ->

CH^ CH2OH
Fi

Ethylene Ethylene glycol


9.12.6. Uses of Hydrogen Peroxide
(0 The most important industrial use of H2O2 is as a bleaching agent for delicate materials like textiles
(silk, wool), paper pulp, straw, leather, ivory, oils and fats.
(ii) In daily life, it is used as a hair bleach and as a mild disinfectant.
(Hi) It is extensively used to manufacture inorganic chemicals like sodium perborate and percarbonate
which are important constituents of good quality detergents,
(iv) It is used in the production of epoxides, proplyene oxide and polyurethanes,
(v) H2O2 is also used in the synthesis of hydroquinone, pharmaceuticals like cephalosoporin and food
products like tartaric acid.
9/40 U New Course Chemistry (XI)S!EIHD

(vi) H->02 is increasingly being used in Environmental (Green) Chemistry to control pollution by
(0 treatment'of domestic and industrial effluents (») oxidation of cyanides and restoration of aerobic conditions
to sewage wastes, etc.
(vi7) It is used as an antiseptic under the name perhydrol for washing wounds, teeth and ears.
{viii) It is used for restoring the colour of lead paintings which have blacke:;ed due to the action of H^S
present in the air on lead paints,
(/.r) it is used in the laboratory for detecting the presence of chromium, titanium and vanadium salts with
which it yields peroxides of characteristic colours,
(xr) 93% H7O2 solution is used as an oxidant for rocket fuel and as a propellant for torpedoes and submarines.
{xi) It is u.s~ed as antichlor (to remove CI2) in textile industry to remove excess of chlorine after bleaching

w
operations.
{xH) A mixture of hydrazine hydrate and hydrogen peroxide with copper (II) catalyst is used as a rocket
propellant.
Cu(n)

F lo
NH2NH2 (/) + 2 H2O2 (/) ^ N2(g)t + 4H20(g)T

(/) H2O2 on treatment with an acidified solution of titanium salt gives a yellow or orange colour due to

ee
the formation of pertitanic acid (112X104)
Xi(S04)2 + H2O2 + 2 H2O > H2T104 + 2 H2SO4

Fr
(i7) It liberates iodine from KI solution which, in turn, gives blue colour with starch solution,
(/(7‘) When an ethereal solution of H2O2 is shaken with acidified solution of K2Cr20y, blue colour

for
appears in the ethereal layer due to the formation of chromium pentoxide (€105)
(iv) When brought in contact with H2O2 solution, a filter paper with black stain of PbS turns white,
ur
(v) It decolourizes acidified KMn04 solution.
s
ook
Yo
dilute acids produce hydrogen peroxide are called
1. Peroxides. Metallic oxides which on treatment with
peroxides. For example, Na202 and Ba02. In these peroxides, the two oxygen atoms are linked by a
eB

single bond and each oxygen atom has an oxidation state of-1. In other words, all peroxides contain a
peroxide ion (O^-) having the structure “ O—0 r . In this structure, all the electrons are paired and
hence all peroxides are diamagnetic.
our
ad

There are certain other oxides like Pb02 and Mn02 which may be mistaken as peroxides. These compounds,
however, do not give H20^ on treatment with dilute acids. As such these compounds do not contain a
peroxide ion (O2") and hence they cannot be called as peroxides. Actually in these compounds the two
oxygen atoms are linked to the metal atom by a double bond and hence are called dioxides, i.e., O = Pb
Y

= 0 (lead dioxide) and O = Mn = O (mangenses dioxide). In dioxides, the oxidation state of each oxygen
Re

atom is -2.
nd

2. Superoxides. Besides peroxides, alkali metals also form higher oxides called superoxides. For example,
potassium superoxide (K02)> rubidium superoxide (Rb02), cesium superoxide (Cs02), etc. All these
Fi

superoxides contain a superoxide ion, i.e., O2 having the structure, ; O—O : . Thus, all superoxides
contain an odd number of electrons {i.e., 13) and hence are paramagnetic.
3. Classification of oxides on the basis of oxygen content.
On the basis of oxygen content, oxides can be classified into the following types :
(i) Normal oxides. Those oxides in which the oxidation number of the element (M) can be deduced from
the empirical formula Mpy by taking the oxidation number of oxygen as -2 are called normal oxides.
For example, H2O, Na,0, MgO AI2O3, CO2, etc. All these oxides contain M—O bonds,
(ii) Polyoxides. These oxides contain more oxygen than would be expected from the oxidation number
of the element (M). These have been further classified into peroxides, and superoxides.
HYDROGEN
9/41

9.12.8. Structure of Hydrogen Peroxide


Hydrogen peroxide is a
FIGURE 9.11
non-planar molecule (Fig. 9.11).
The two oxygen atoms are linked H H

to each other by a single covalent O' Vs


bond (i.e., peroxide bond) and
●o.

J 147.5 pm 111,5° V- 145.8 pm 90.2®


each oxygen is further linked to -—O ^O- O Q
a hydrogen atom by a single 94.8®^ 101.9°
H
covalent bond. The two O—H H

bonds are, however, in different


planes due to repulsions between e o

w
different bonding and anti
bonding orbitals. The dihedral Structure of H2O2 (a) in the gas phase, dihedral angle =111*5°
(interplanar) angle between the
and (b) in the solid phase at 110 K, dihedral angle = 90-2®

F lo
two planes being 111-5° (Fig. 9.1 1 a) in the gas phase but reduces to 90-2° (Fig. 9.1 lb) in the crystalline state
due to hydrogen bonding. The other molecular dimensions of hydrogen peroxide in the gas phase and the
crystalline state are shown in Fig. 9.11.

ee
9.13. HYDROGEN ECONOMY

Fr
The coal and petroleum reserves of the world are limited and are fast dwindling. Therefore, the
fundamental problem which concerns the mankind today is the search for alternative sources of energy.
Nuclear power has recently posed severe health and environmental problems and even if these problems are

for
solved it cannot be used for running cars, ships and flying aeroplanes. One proposed way to meet the need for
new
energy sources is to burn hydrogen as a fuel in industry and power plants and possibly also in homes and
ur
motor vehicles. This proposal is referred to hydrogen economy.
There are several advantages of hydrogen economy. Firstly, dihydrogen releases large quantities of heat
s
on combustion. The energy released by combustion of fuels like dihydrogen, methane,, LPG, etc. as compared
ook
Yo
in terms of the same amounts in mole, mass and volume, is shown in table 9.6.
eB

TABLE 9.6. The energy released by combustion of various


fuels in moles, mass and volume
Energy released Dihydrogen Dihydrogen LPG CH4 gas Octane
our

on combustion (in gaseous (in liquid (in liquid


ad

(in kj) state) state) state)

per mole 286 285 2220 880 5511


per gram 143 142 50 53 47
Y

per litre 12 9968 25590 35


Re

34005
d

From the Table 9.6, it is evident that on mass to mass basis, dihydrogen can release more energy than
Fin

gasoline, i.e., octane number is about three times. Secondly, burning of hydrogen in air or dioxygen not only
liberates large amount of energy but yields water as the only product. In other words, in contrast to burning
coal in power stations, or petrol or diesel in motor engines, burning of hydrogen produces no pollutants like
SO2 that are responsible for acid rain, nor CO2 that is responsible for the greenhouse effect, nor carcinogenic
hydrocarbons, nor lead compounds. The only pollutants will be oxides of dinitrogen (due to the presence of
dinitrogen as impurity with dihydrogen). This can, however, be minimized by injecting small amounts of
water into the cylinder to lower the temperature so that reaction between dinitrogen and dioxygen may not
take place.
However, there are two major barriers in achieving the goal of hydrogen economy. The first is to find
out a cheap method for large scale production of H^. Two methods which have been proposed are: electrolysis
of water and the thermochemical reaction cycle. The first method is not economically viable since the cost of
production of H2 by electrolysis of H2O is so high that almost all H2 is obtained from natural gas which itself
9/42 “pfuxdeefr New Course Chemistry (XI)

is in short supply. The other method involves series of thermochemical reactions in which the only things
consumed are water and heat and the only products are hydrogen and oxygen while all other species are
recycled. One such thermochemical reaction cycle is :
775 K

3 FeCl2 + 4 H2O > Fe304 + 6 HCl + H2] X 2


475 K

2FC3O4+ 12HC1 + 3C12 - 6 FeCl3 + 6 H2O + O2


725 K
2 FeClj 4 2 FeCl2+ Cy X 3

low
2 up > 2 H2 + O2
Whether any of the methods listed above can produce H2 cheaply enough to make its use practicable is
a most distant possibility. The other problem is to fmd out an effective means of storing H2. The gaseous H2,
because of its bulk, is difficult to store, but liquid H2 can be stored relatively easily and safely in cryogenic
tanks (already in use for space programme in U.S.A.). It is also feasible to transport liquid H2 by road or rail
tankers of 20,0(X) US gallons capacity. It can also be stored in underground tanks and transported by pipelines.

ee
However, it may not be convenient to store liquid H2 in a home or in a car since the boiling point of H2 is very

rF
low (20-4 K) In either case (gaseous or liquid form) H2 must be kept out of contact with oxygen or air with

Fr
which it forms explosive mixtures. The only alternative left is to store H2 in a metal or in an alloy (such as
iron-titanium alloy, LaNig, Mg-MgH2, Ti-TiH2, etc.) as interstitial hydride.
The gas can then be released on mild heating. In an automobile, for example, this storage system would

for
replace the gasoline tank. The heat required to release from the metal hydride could come from the
exhaust gases from the engine.
u
It is for the first-time in the history of India that a pilot project using dihydrogen as fuel was launched in
Oct., 2005 for running automobiles. Initially 5% dihydrogen has been mixed with CNG for use in four wheeler
ks
vehicles. The percentage of dihydrogen would be gradually increased to reach the optimum level.
Yo
oo
Now a days, it is also used in fuel cells for generation of electric power. Thus, if a cheap source of large
scale production of hydrogen is discovered in the years to come then hydrogen economy will become the
eB

common source of energy.

9.14. SOME ADDITIONAL USEFUL INFORMATION


ABOUT HYDROGEN (FOR ADVANCED COMPETITIONS)
r

1. Ortho and Para Hydrogen. Dihydrogen has two nuclear spin isomers called ortho and para
ou
ad

dihydrogen.
When two hydrogen atoms combine to fonn a molecule of dihydrogen, the two electrons always spin in
Y

the opposite directions (Pauli’s exclusion principle) as otherwise the molecule will not be stable. However,
the spins of the protons (nuclei) may either be in the same direction or in the opposite direction.
nd
Re

When the spins of the nuclei are in the same


direction (parallel spins), dihydrogen is called
Fi

ortho hydrogen and when the spins are in the


opposite directions (antiparallel spins),
dihydrogen is called para hydrogen.
The two forms of dihydrogen are shown in ORTHO HYDROGEN PARA HYDROGEN
Fig. 9.12. (Parallel nuclear spins ; total {Antiparallel nuclear spins; total
Conversion of one isomer into the other is a nuclear spin = ■*■ y ^^ nuclear spin = + = 0)
slow process. Para isomer with low energy is Ortho and Para dihydrogen
favoured at low temperatures.
At room temperature, ordinary dihydrogen contains 75% of ortho hydrogen and 25% of para hydrogen.
As the temperature is lowered, the percentage of ortho hydrogen in the mixture decreases while that of the
para hydrogen increases. At liquid air temperature (77 K), it is 1 : I and at about 20K, it is pure para hydrogen.
HYDROGEN
9/43

In contrast, when a sample of ordinary dihydrogen is heated say to 400 K or above, the ratio of ortho and para
hydrogen lemains to be the same (3 : 1). Thus, it is possible to obtain pure para hydrogen but it is not
possible to obtain pure ortho hydrogen.
The physical properties of dihydrogen are not greatly affected by nuclear spin isomerism. However,
some of the notable differences are :
(0 Thermal conductivity of P-H2 is 50% greater than that of and
(ii) The melting point of P-H2 is 0-15 K lower than that of hydrogen containing 75% o-H-,.

ow
2. Atomic hydrogen
(a) Preparation. Refer to Ans. to Q. 9.13, page 9/66.
(b) Properties. It is a very reactive form of hydrogen and hence it acts as a powerful reducing agent. It
reduces oxides, chlorides, sulphides and sulphates of heavy metals and many alkali metal salts other than
chlorides and sulphates.
CuO + 2 H
> CU + H2O ; AgCl + H ^ Ag + HCl

e
CU2S + 2 H > 2 Cu + H2S ; BaS04 + 8 H

Fl
BaS + 4H2O

re
It also reduces oxides of non-metals. For example,
CO2 + 2H > HCOOH ; CO + 2H ^ HCHO

F
P4O10 + 32 H > 4PH3+ IOH2O
It combines with O2, Ag and Hg at low temperatures to form their respective hydrides.
ur
r
O2 + 2H > H20.^ ; Ag + H ^ AgH

fo
3. Nascent hydrogen. The hydrogen produced in contact with the substance to be reduced is called
nascent hydrogen. In other words, it is the hydrogen at the moment of its generation. That is why it is also
ks
sometimes called as newly born hydrogen. To differentiate from ordinary hydrogen, nascent hydrogen may
Yo
be represented as H^.
oo
It is believed that a part of chemical energy liberated during the reaction is associated with the hydrogen
molecules making them more reactive than the ordinary hydrogen molecules. Thus, nascent hydrogen is
B

actually molecular hydrogen associated with excess energy. This excess energy is actually responsible for
higher reactivity and higher reducing power of nascent hydrogen over ordinary hydrogen. For example,
re

ordinary hydrogen cannot reduce permanganate, ferric, dichromate and chlorate ions. But, if a small zinc
piece is added to such solutions, these ions are reduced to manganous, ferrous, chromic and chloride ions
respectively.
u
ad
Yo

Zn + H2SO4 ■> ZnS04 ^2 hydrogen)


2 MnO^ + 6 + 5 H2 ^ 2 Mn^+ + 8 H2O ; 2 + h; 2 Fe2+ + 2
d

Cr20“- +sm + 3H* ^ 2 Cr^^ + 7 HoO ;


Re

ClOj + 3H* ^ Cl“ + 3 H2O


in

It has also been reported that nascent hydrogen is liberated in form of tiny bubbles having high internal
pressure. This high internal pressure of the gas bubbles may also be responsible for the hyperactivity of
F

nascent hydrogen.
However, none of the two theories described above are entirely satisfactory to explain the behaviour of
nascent hydrogen.
Some common sources of nascent hydrogen are : (i) a mixture of zinc and dil. H^S04, (n) a mixture of
tin and cone. HCl, {Hi) metallic sodium and absolute
alcohol and (/v) Zinc-copper couple ^d H2O or alcohol.
4. Comparison between Atomic and Nascent hydrogen. Main points of difference are :
(0 Nascent hydrogen can be produced even at room temperature but atomic hydrogen is produced at
elevated temperatures.
(/7) Nascent hydrogen can never be isolated but atomic hydrogencan be isolated.
{Hi) Reducing power of atomic hydrogen is much greater than that of nascent hydrogen.
9/44 “p>u^,de€^’4- New Course Chemistry (XI)EEaCl

In general, the reactivity of the three forms of hydrogen increases in the order :
Molecular hydrogen (^2) < Nascent hydrogen < Atomic hydrogen
5. Heavy hydrogen. It is manufactured by the electrolysis of heavy water containing a little of H2SO4
or NaOH to make it a good conductor of electricity.
Klecirotysis
2D20(/) ■>
202(5) + 02(g)
Heavy water (At cathode) (At anode)

In the laboratory, it can be prepared by the action of heavy water on sodium metal.
2 D2O (/) + 2 Na (.y) ^ 2 NaOD (aq) + (g)

6. The triatomic hydrogen molecule ion was first detected by J.J. Thomson in gas discharges

w
and later fully characterized by mass spectrometry. Its relative atomic mass 3-027 clearly distinguishes
it from tritium (3-016). The observed triangular 3-centre-2-eIec tron structure is more stable than the

F lo
hypothetical linear structure and the comparative stability of the species is shown by the following gas-phase
enthalpies:
-!
H + H + H > ; -AH 855-9 kJ mol

e
-1

Fre
H2 + H+ > ; -AH 423-8 kJ mol

H + H| HJ ; - AH 600-2 kJ mor*
9.15. TYPIGVL PROBLEMS
P. 1. The process 1/2 H2 (g) ^
for
^ H" (g) is endothermic (AH = + 151 kJ mol"^), yet salt like hydrides
r
are known. How do you account for this?
You
oks

Sol. It is true that formation of hydride (H") ion is an endothermic process, yet alkali and alkaline earth
metals form salt like hydrides. This is due to the reason that high lattice energy released (energy released
eBo

during the formation of solid metal hydrides from their corresponding gaseous ions, ie., (g) and
H“ (g)) more than compensates the energy needed for the formation of H ions from H2 gas.
P. 2. Although dihydrogen can be prepared by electrolysis of water but in fertilizer industry, dihydrogen
ad
our

needed for making urea is prepared from natural gas or naphtha. Why so ?
Sol. Manufacturing of H-, by electrolysis of water is a highly expensive process, therefore, in industry, it is
manufactured by partial oxidation of natural gas or naphtha.
1270K
CH4(g) +H,0(g) ^ CO(g)+3H2(g)
Re

Ni
dY

Natural gas Steam


Syngas

1270K
Fin

C H
2n4-2 + nHp Ni ^ /iCO-i-(2/t+l)H2
Naphtha Steam

CO is then removed from syn gas by water gas shift reaction.


P. 3. Biomass gasification rather than coal gasification can be used to control environmental pollution.
Justify ?
Sol. During coal gasification, combustion of coal produces green house gases such as CO2 which causes
environmental pollution (or global warming). But during biomass production as well as biomass
gasification equal quantities of CO2 are used up thereby producing negligible environmental pollution.
P. 4. What is demineralized water ? How is it obtained ?
Sol. Water which does not contain cations and anions is called demineralized water. It is obtained by passing
ordinary water first through cation exchange resin and then through anion exchange resin.
HYDROGEN
9/45

P. 5. Water extinguishes most fires, but it does not extinguish petrol fire. Explain.
Sol. Water extinguishes most fires by lowering down the temperature of the burning materia!. But in case of
petrol fire, petrol being lighter than water, floats over water and hence fire spread.s instead of being
extinguished.

w
P. 6. Calculate the volume of 10 volume H2O2 solution that will react with 200 mL of 2N KMnOj in
acidic medium :

_ Volume strength 10
Sol. Normality of 10 volume H2O2 = 5-6 5-6
N

Applying normality equation, N,V, (H2O2) = N2V2 (KMn04)

o
e
10 2 X 200 X 5-6

re
X V.I = 2 X 200 or V = 224 mL
5-6 ]
10

P. 7. The degree of hardness of a given sample of hard water Is 40 ppm. If the entire hardness in due to

Frl
F
MgS04, how much of MgS04 is present per kg of water ?
Sol. Degree of hardness of H2O is 40 ppm. i.e., 10^ g of water contain CaC03 = 40 g
Since 1 mole of CaC03 = 1 mole of MgS04
ou
r
100 g of CaC03 = 120 g of MgS04

so
40x120
10® g of water contain MgS04 100
= 48g

kf
48x10-^
or 10^ g of water will contain MgS04 X10^ mg
oo
10®
or 1 kg of water will contain MgS04 = 48 mg
Y
B
re
oY

I. Hydrogen or DIhydrogen
u

1. Abundance. Hydrogen is the 9th most abundant element


ad

in the earth’s crust.

2. Isotopes of hydrogen. Hydrogen is the only element which does not have neutrons in its nucleus. It exists
in three isotopes, i.e., protium (jH*) deuterium {|H^
d

or
D) and tritium (jH^ or T). Tritium is, however.
radioactive {ty2 = 12-33 years) and emits P-particles.
in

3. Hydrogen resembles alkali metals and halogens in its properties.


Re

4. The ionization enthalpy of hydrogen is more than those of alkali metals but less than those of halogens.
5. Electrolysis of an aqueous solution of HCI liberates H2 at the cathode while that of fused NaH liberates H2
F

at the anode.

6. Ordinary hydrogen consists of 75% ortho and 25% para hydrogen. The nuclei of ortho hydrogen spin in
the same direction while that of para hydrogen spin in the opposite directions. At room temperature, the
ratio of ortho and para hydrogen is 3 : 1 and remains to be the same when heated to 400 K or above.
However, on cooling, the percentage of ortho hydrogen decreases while that of para hydrogen increases. At
liquid air temperature (77 K) it is 1 : 1 and at 20 K, it is pure para hydrogen.
7. Alkali metals and alkaline earth metals react with cold water producing H2 gas.
8. Mg, Al, Zn decompose boiling water evolving H2 gas while Fe decomposes steam producing H2 gas.
9. Be, Sn, Al and Zn react with boiling NaOH producing H2 gas.
10. Metals like Zn, Fe, Mg, etc. which lie below hydrogen in the electrochemical series react with dilute mineral
acids producing H2 gas.
9/46 'pfux<Uep’'A New Course Chemistry (XI)BEIHI]

11. Electrolysis of heavy water (DjO) produces heavy hydrogen (D2)-


12. Hydrogen forms compounds by losing, gaining or sharing of electrons.

ow
13. Hydrogen shows oxidation states of + 1. 0 and - 1.
14. Dihydrogen acts as a powerfitl reducing agent and thus reduces metal oxides such as CuO, ZnO, PbO and
Fe304 to their respective metals. It, however, does not reduce AI2O3 to Al.
15. It also acts as an oxidising agent, i.e., 2 Na + H2 ^ 2 NaH.

16. A mixture of H, (15%) and He (85%) is used in airships and hot air balloons.
of s, p (except noble gases), d- and /-block elements.

e
17. Dihydrogen forms binajy hydrides with elements
These are of three types :

re
(a) Ionic hydrides. Elements of group 1, (i.e., alkali metals), and group 2 (alkaline earth metals) form ionic
hydrides. BeH2 and MgH2, however, have covalent polymeric structures. LiH reacts with AI2CI6 and B2H6
forming UAIH4 and LiBH4 respectively. Similarly, NaH reacts with B2H6 and forms NaBH4.

F
Frl
{b) Metallic or Interestial hydrides. d-Block elements of groups 3, 4, 5 (Sc, Ti, V, Ya, Zr, Nb, La, Hf, Ta,
Ac, etc.) 10, 11, 12 (Pd, Cu, Zn, etc.) and/-block elements (Ce, Eu, Yb, Th, U, etc.) on heating with H2

ou
under pressure form hydrides. In group 6, only Cr forms the hydride CrH. The metals of groups 7, 8 and 9
do not form hydrides and is called hydride gap. These hydrides are often non-stoichiometric, have metallic

sr
lustre, magnetic properties and conduct electricity. These are used as hydrogen storage media,

kfo
(c) Molecular hydrides. p-Block elements and some 5-block elements (Be and Mg) form molecular or
covalent hydrides. These are usually volatile compounds having low melting and boiling points. These are
of the following three types ; oo
(i) Hydrides of group 13 {i.e., BH3, AIH3, etc.) are electron-deficient hydrides and are polymeric in nature.
(/■/) Hydrides of group 14 {i.e., CH4, SiH4, GeH4, SnH4, PbH4) are electron-exact or electron-precise
Y
hydrides and have tetrahedral structures.
reB

{in) Hydrides of group 15, 16 and 17 {i.e., NH3, PH3, H2O, H^S, HF, HCl, etc.) are electron-rich hydrides.
The complex metal hydrides (LiAlH4, NaBH4) are widely used as reducing agents in organic synthesis.
uY

II. Water

18. In H,0. O is .yp^-hybridized but ZHOH = 104-5° due to greater repulsions between lone pairs as compared
to bond pair-lone pair repulsions. It forms four hydrogen bonds and has a b.p. of 373 K and m.p. of 273 K.
ad

density of
do

19. Ice has open cage like structure with a number of vacant spaces in the crystal lattice. As a result,
ice is lower than that of water. However, density of water is maximum at 277 K.
20. It has amphoteric character and undergoes redox reactions. Towards active metals {i.e., Na, Ca, etc.). It acts
in

as an oxidising agent and itself gets reduced to H2. Towards F2, it acts as a reducing agent and itself gets
oxidised either to O2 or O3.
Re

21. Hard water does not produce lather readily with soap since soap gets precipitated as calcium and magnesium
F

salts of fatty acids. However, hard water produces lather readily with detergents since calcium and magnesium
salts of sulphonic acids are soluble in water.
22. Temporary hardness of water is due to the presence of bicarbonates of calcium and magnesium while
permanent hardness is due to the presence of chlorides and sulphates of calcium and magnesium.
23. Temporary hardness is removed either by simply boiling hard water or by treating it with a calculated
amount of quick lime (Clarke’s method) while permanent hardness is removed by treating hard water with

a calculated amount of Na2C03.


24. Both temporary and permanent hardness of water are removed either by lime-soda process or by ion exchange
process.
25, Zeolite is hydrated sodium aluminium silicate, Na2Al2Si20g-v H2O. It renders hard water soft by exchanging
its Na"^ ions with Ca^'*’ and Mg^"^ ions present in hard water.
HYDROGEN
9/47

26. Demineralised or deionised water is obtained by passing hard water first through a cation exchange
resin (RCOOH or RSO3H) which removes Ca^'*’ and ions from hard water by exchanging them with
H"*" ions and then passing through an anion exchange resin (RNH3OH") which removes Cl" and SOj"
ions present in hard water by exchanging them with OH" ions.
27. Calgon is the trade name for sodium iiexametaphosphate. It .softens hard water by rendering Ca-+ and
Mg~ ions present in hard water ineffective by forming soluble complexes of calcium and magnesium salts.
Thse complexes do not form any precipitate with soap and hence the water treated with calgon readily
produces lather with soap solution.
28. Heavy water (D2O) was discovered by Urey. It is obtained by repeated electrolysis of ordinary water and is
used as a moderator in nuclear reactors.

III. Hydrogen Peroxide


29. H2O2 is prepared by the action of an ice-cold solution of cold dil. H2SO4 011 Na20-> (Merck’s method)

F low
or on Ba02 (laboratory method). Weaker acids such as H2CO3 (H2O + CO2) or H3P54 are preferred over
dil H2SO4 since the latter catalyses the decomposition of H2O2.
30. H2O2 is manufactured by electrolysis of a 50% H2SO4 solution or preferably by electrolysis of an equimolar
mixture of H2SO4 and (NH4)2S04. It has also been manufactured by aerial oxidation of 2-ethylanthraquinol.
31. To check the decomposition of H2O2 by light and alkali metal oxides present in glass, H2O2 is stored in
plastic coated teflon bottles. Further some stabilizers or negative catalysts (such as glycerine, acetanilide,
phosphoric acid, etc. are added to it.

e
for Fr
32. Strength of H2O2 is expressed either as percentage strength {i.e., amount of H2O2 present in 100 ml of the
solution) or volume strength. A 10 volume H2O2 solution implies that 1 ml of such a solution on
decomposition by heat produces 10 ml of O-, at NTP.
33. Normality and volume strength are related, as, volume strength = Normality x 5*6.
34. A 10 volume H2O2 solution is about 3% H2O2 or 1-78 N or 0-89 M H20-, solution.
35. H2O2 is only slightly more acidic than water but less acidic than H2CO3, /.e., CO2 > H2O2 > H2O.
Your
36. It acts as an oxidising agent as well as a reducing
s

agent both in the acidic as well as in the basic medium.


eBo k

37. It oxidises black PbS to white PbS04. This reaction is used for restoring the colour of old lead paintings
which have blackened due to prolonged exposure to H2S present in the atmosphere.
38. H2O2 acts as a bleaching agent for delicate materials like hair, silk, wool, ivory, etc.
ad

39. The bleaching action of H2O2 and O3 is due to oxidation by nascent oxygen and hence is permanent.
our

40. H2O2 is a non-linear molecule having a .structure similar to that of an open book with ZOOH = 94-8”.
41. H2O2 is used as an antiseptic under the name perhydrol for washing wounds, teeth and ears. It is also used
as an oxidant for rocket fuel and as antichlor (to remove excess chlorine) in textile industry.
42. Use of H2 as a fuel in industry, power plants, homes and vehicles is called hydrogen economy.
Re
Y
Find

f
9/48 7^’Kxdeef^ 4 New Course Chemistry (XI)

I
<
V. L&i i

i
*»x

NEET/JEE
SPECIAU

For ultimate preparation of this unit for competitive examinations, students should refer to j
● MCQs in Chemistry for MEET

F low
Pradeep*s Stellar Series.... ● MCQs In Chemistry for JEE (Main)

separately available for these examinations. ,

H Multiple Choice Questions (with one correct Answer)


I. Dihydrogen 5. Dihydrogen of high purity (> 99-95%) is obtained

re
through

for F
1. HCl gas is covalent and NaCl is an ionic
(a) the electrolysis of acidified water using Pt
compound. This is because electrodes
(a) sodium is highly electropositive
(b) the electrolysis of warm Ba(0H)2 solution
(b) hydrogen is a non-metal
using Ni electrodes
(c) HCl is a gas
(c) the electrolysis of brine solution
(d) Electronegativity difference between H and Cl
Your
is less than 2-1. (Karnataka CET 2016) (d) the reaction of Zn with dilute HCl
s
eBook

2. Select the correct statement/s (JEE Main 2020)

(a) H'*’ can exist as HgOj in water 6. 5 g of zinc is treated separately with an excess of
(b) Electrolysis of fused sodium hydride produces (/) dilute hydrochloric acid and (ii) aqueous
H2 at the anode sodium hydroxide
ad
our

(c) Hydride ion is larger that any of the halide The ratio of the volumes of H2 evolved in these
two reactions is;
ions except iodide ion
(d) All are correct (0)1:2 (fc) 2 : 1
3. ^C*^ and jT^ are formed in nature due to the (c) 1 : 4 id)\:\
Re

nuclear reaction of neutron with (JEE Main 2020)


14 13
(a) 7N (^)6C 7. Which of the following has least tendency to
Y

(c) 2He^ (d) (AIIMS 2008) liberate H2 from mineral acids.


Find

4. Which of the following combination will produce (a) Cu (b) Mn


H2 gas ? (c)Ni id) Zn (JEE Main 2022)
(a) Cu metal and cone. HNO3 8. Dihydrogen reacts with CuO to give
ib) Zn metal and NaOH (aq)
(a) CuH2 (b) Cu
(c) Au metal and NaCN (aq) in the presence of air
(c) CU2O (d) Cu(OH)2
(d) Fe metal and cone. HNO3
(JEE Main 2022)
(JEE Advanced 2017)

ANSWERS

!.(</) 2.(d) 3. (a) 4.(b) S.(b) 6. {d) 7. (o) 8. (i»)

1
HYDROGEN 9/49

9. Hydrogen has three isotopes (A), (B) and (C). If 15. When a substance A reacts with water, it produces
the number of neutrons in (A), (B) and (C) a combustible gas B and a solution of substance
respectively are (a:), (y) and (z), the sum of (a:), (y) C In water. When another substance D reacts with
and (z) is
this solution of C. it produces the same gas B on
(a) 4 (b)2 warming but D can produce gas B on reaction with
(c)3 (d) 1 (JEE Main 2020) dilute sulphuric acid at room temperature. A
10. Tritium the radioactive isotope of hydrogen emits imparts a deep golden yellow colour to a smokeless
which of the following particles ? flame of Bunsen burner. A, B, C and D respectively
(a) Neutron (n) (b) Beta (p )
are ;

w
(c) Alpha (a) id) Gamma (y) (a) Na. Ii2, NaOH, Zn (b) K, H2, KOH, A1
(NEET 2021) (c) Ca, H2, Ca(OH)2, Sn
11. Deuterium is different from hydrogen in which (d) CixCj, C2H2, Ca(OH)2, Fe. (AIIMS 2015)

Flo
propeny ? 16. Water gas is produced by
(a) It reacts more vigorously than hydrogen fo) passing steam over red hot coke
(b) It reacts less vigorously than hydrogen (b) passing steam and air over red hot coke

ee
(c) It emits p-particles (c) burning coke in excess of air
id) Its reactivity is same as that of hydrogen

Fr
(d) burning coke in limited supply of air
(NEET 2021) (e) both (a) and (b) (Kerala PET 2017)
12. Hydrogen has three isotopes : protium ('H), 17. The correct order of melting points of hydrides of
deuterium (-H or D) and tritium (% or T). They

for
group 16 elements is
have nearly the same chemical properties but
ur (a) H2S < H2Se < H2Te < H2O
different physical properties. They differ in
(b) H2O < H2S < H2Se < H2Te
(a) number of protons ib) atomic number
(c) H2S < H2Te < H2Se < H2O
ks
(c) electronic configuration
(d) H2SC < HoS < H2Te < H2O (JEE Main 2022)
Yo
id) atomic mass (JEE Main 2022)
18. Which of the following is the correct order of
oo
13. The equation that represents the water-gas shift increasing enthalpy of vaporisation ?
reaction is
eB

(a) NH3 < PH3 < ASH3 (b) ASH3 < PH3 < NH3
1230K

(a)CH4(g) + H20(g) (c) PH3 < ASH3 < NH3 (d) NH3 < AsHg < PH3
^ CO(g)-h3H2(g)
Ni
19. The various types of hydrides and examples of
each type are given below :
r

673K
ib) CO ig) + H20ig)
ou

^ C02(g) + H2(g)
ad

Catalyst Hydride type Compound


1273K (A) Electron-deficient (/) LiH
ic) 2Cis) + O2 ig) + 4 N2 ig) ^ 2 CO ig)
Y

(B) Saline HO CH4


+ 4N2 ig) (C) Electron-precise (m) NH3
1270K
(D) Interstitial iiv) B2H6
Re
nd

id) C is) + H2O ig) ^ CO (g) -I- H2 (g)


(E) Electron-rich (v) CrH
(JEE Main 2020)
Choose the correct matching from the codes given
Fi

14. The hydride ion is a stronger base than below :


hydroxide ion. Which of the following reactions
will occur if sodium hydride (NaH) is dissolved (a) (A) - (//). (B) - iiv), (C) - (v), (D) - (m), (E) - (0
in water ? (b) (A) - iiv), B - (0, (C) - Hi), (D) - (v), (E) - (m)
ia) H- iaq) + H2O (/) H30'^ iaq) ic) (A) - iiv), (B) - HU), (C) - (v), (D) - HO, (E) - iO
(b) H- iag) 4- H2O (/) 4 OH" (aq) + H2 (g) id) (A) - iv), (B) - mo, (C) - (;V). (D) - (i7), (E) - (0
(c) H- + H2O ^ No reaction (e) (A) - (iV), (B) - iv), (C) - (/). (D) - (/V), (E) - (m)
id) None of these. (Kerala PET 2016)

ANSWERS
9. ic) 10. ib) 11. ib) 12. id) 13.ib) 14. (b) 15. ia) 16. («) 17. (a)
18.ic) 19.ib)
9/50 New Course Chemistry (Xl)CZsI9D

20. The hydrides of the first elements in groups {d) Ice formed by heavy water sinks in normal
(JEE Main 2016)
15-17, namely NH3, H2O and HF respectively water

show abnormally high values for melting and 27. Which of the following disproportionates when
boiling points. This is due to treated with water ?

(o) small size of N, O, F (u) SO3 ib) F2


(b) the ability to form extensive intermolecular (c) N, id) CI2
H-bonding 28. Which of the following statement is correct ?
(c) the ability to form extensive intramolecular (a) CI2 oxidises H2O to O2 but Fj does not
H-bonding ib) CU is a stronger oxidising agent than F2

w
(d) effective van der Waals interaction (c) F2 oxidises H2O to O2 but CI2 does not
(West Bengal JEE 2014) id) Fluoride is a good oxidising agent
21. The acidity of diprotic acids in aqueous solutions (Karnataka CET 2019)

F lo
increases in the order
29. Which one of the following reactions does not
(a) H^S < HjSe < H2Te come under hydrolysis type reaction ?
(b) H^Se < HoS < H2Te (a) P40io is) + 6 H2O (/) >4 H3PO4 iaq)

ee
(c) H2Te < HoS < H2Se ib) SiCl4 (/) + 2 H.O (/) > Si02 is)

Fr
id) H2Se < H2Te < HoS + 4 HCl iaq)
22. Hydride ion is a strong ic) is) + 3 H2O (/) ^ NH3 ig)
(a) conjugate acid of H2 (b) conjugate base of H2 + 3 LiOH iaq)

for
(c) conjugate acid of H" (d) conjugate base of H"*" id) 2 P2 ig) + 2 H2O (/) ^ 4 HF iaq) + O2 ig)
ur
23. Consider the following reactions : (NEET 2020)
I: AlH,3 + H- ^ AlH- 30. The one that is not suitable for the removal of
s
II: H2O + H- 4 H-> + OH” permanent hardness of water is
ook
Yo
Select the correct statement from the following : (a) Calgon’s method
ia) H“ is a Lewis acid in I and Lewis base in II ib) Ion-exchangemethod
eB

ib) H" is a Lewis base in I and Bronsted base in II (c) Clark’s method
ic) H~ is a Lewis acid in I and Bronsted base in II id) Treatment with sodium carbonate
id) H" is a Lewis base in I and II (JEE Main 2020)
r

24. Spin isomerism is shown by 31. The metal salts (or products) formed during
ou
ad

id) dichlorobenzene ib) hydrogen treatment of hard water using Clark’s method are
ic) dibasic acid (</) n-butane (a) CaCOj and MgC03
(AIIMS 2008) ib) Ca(OH)2 and Mg(OH)2
Y

25. Para and ortho hydrogen differ in ic) CaC03 and Mg(OH)2
ia) atomic number ib) atomic mass (JEE Main 2022)
id) Ca(OH)2 and MgC03
Re
nd

(c) .spins of protons id) number of neutrons


32. In comparison to the zeolite process for the
II. Water removal of permanent hardness, the synthetic
Fi

resins method is
26. Which one of the following statements about water
(u) less efficient at the resins cannot be
is false ?
regenerated
ia) Water is oxidized to oxygen during photo
ib) less efficient as it exchanges only anions
synthesis
ic) more efficient as it can exchange only cations
(b) Water can act both as an acid and as a base
id) more efficient as it can exchange both cations
(c) There is extensive intramolecular hydrogen as well as anions (JEE Main 2020)
bonding in the condensed phase
ANSWERS
20. ib) 21. («) 22. ib) 23. ib) 24. (6) 25. (c) 26. (c) 27. id) 28. (c) 29. id)
30.(c) 31.(c) 32.(J)
HYDROGEN 9/51

33. Which one of the following methods is most (cl) + HoOo ^ Mn*^+ + 2 OH'
suitable for preparing deionized water ? (JEE Main 2022)
(a) Synthetic resin method 40. The products obtained from a reaction of hydrogen
(h) Calgon method peroxide and acidified potassiumpermanganateare
(c) Clark’s meth(xl (a) Mn**'*', H,0 only (b) Mn^+. H2O only
(d) Permutit method (JEE Main 2021) (c) H2O, O2 only (d) H2O. only
34. The reagent commonly used to detemiine hardness (JEE Main 2022)
of water titrimetrically is 41. 30 volumes H2O2 means
(a) oxalic acid (a) 30% H2O2
(6) disodium salt of EDTA (b) 30 cra^ of the solution contains Ig of H^O->

w
(c) sodium citrate (d) sodium thiosulphate (c) 1 cm^ of the solution liberates 30 cm^ of O2
35. The hardness of water sample containing 0.002 at STP

mole of magnesium sulphate dissolved in a litre {d) 30 cm^ of the solution contain one mole of

F lo
of water is expressed as; H2O2.
(a) 20 ppm (b) 200 ppm 42. The volume of oxygen liberated at STP from
(c) 2000 ppm (d) 120 ppm 15 mL of 20 volume H2O2 is
(e) 240 ppm. (Kerala PET 2007) (a) 100 mL (b) 150 mL

ee
36. The molecular formula of a commercial resin used (c) 200 mL (rf) 250 mL

Fr
for exchanging ions in water softening is (e) 300 mL (Kerala PET 2017)
CgH2S03Na (Mol. wt. of 206). What would be 43. 10 mL of H2O2 solution is treated with KI and

for
the maximum uptake of Ca^"^ ions by the resin titration of liberated Lrequired lOmLof 1 Nhypo.
when expressed in mole per gram resin ?
Thus H2O2 is
(a) 2/309 (b) 1/412
r
(a) 1 N {b) 5-6 volume
(c) 1/103 (d) 1/206
(c)17gL-‘
You
(d) all are correct
(JEE Main 2015)
s
44. The strength of H2O2 (in g/Iitre) in 11-2 volume
ook

37. The hardness of water sample (in terms of solution of H2O2 is


equivalents of CaC03) containing 10“^ M CaS04 (a) 17 (b)5\ (c)34 (d)85
eB

is (molar mass of CaS04 = 136 g mol"') ie) 68 (Kerala PMT 2012)


(a) 10 ppm (b) 100 ppm
45. The volume strength of 1 M H-jOo (molar mass of
(c) 50 ppm (d) 90 ppm
H2O2 = 34 g mol"')
our

(JEE Main 2019)


ad

(a) 224 (b) 16-8


III. Hydrogen Peroxide (c) 5-6 (d) 11-35
(JEE Main 2019)
38. Addition of H2SO4 to Ba02 produces
(a) BaO, SO2 and H2O (b) BaSO^ and O2 46. The strength of 11-2 volume solution of H2O2 is
dY

(Given that molar mass of H = 1 g mol"' and O =


Re

(c) BaS04, and O2 (d) BaS04 and H2O2 16 g mol"')


(JEE Main 2022)
(a) 1-7% (b) 13-6%
Fin

39. Which one of the following reactions indicates the


(c) 3-4% (d) 34%
reducing ability of hydrogen peroxide in basic
medium ? (.iEE Main 2019)

(a) HOCI + H2O2 » H3O+ + Cl" + O2 47. From the following statements regarding H2O2,
choose the incorrect statement.
(b) PbS + 4 H2O2 ^ PbS04 + 4 HoO
(a) It has to be stored in plastic or wax lined glass
(c) 2Mn0" + 3H202 ■> bottles in dark

2 Mn02 + 3 O2 + 2 H2O + 2 OH" (b) It has to be kept away from dust


(c) It can act only as an oxidising agent

33. (u) 34. (b) 35. (b) 36. (/;) 37. {b) 38. (d) 39. (c) 40. [d) 41. (c) 42. (r)
43. (d) 44. (c) 45. [d) 46. (c)
9/52 New Course Chemistry (XI)S!EIHI1

(d) It decomposes on exposure to light 52. In alkaline medium, H2O2 reacts with Fe^"^ and
(JEE Main 2015) Mn^"*" respectively to give
48. In which of the following reactions, H2O2 acts as (a) Fe^+, and Mn"*-" (b) Fe^+and Mn^+
a reducing agent ? (c) Fe2+ and Mn'*+ (d) Fe''+ and Mn^+
(i) H2O2 + 2 H+ + 2 e“ 42H2O (Odisha JEE 2007)
(/i) H2O2 - 2 e“ ^ O2 + 2 53. In the following reaction using isotope *^0 in H2O2.
{in) H2O2 + 2 c' -^2 OH'
2 Mn04 + 3 H20^^ > 2 Mn02 + 3 O2
(,v) H202 + 2 0H--2e-~ ^ O2 + 2 H-5O + 2H2O + 2OH-
(a) (i'O and (iV) {b) (f)and(ii)
Isotopic oxygen goes:
(c) (Hi) and (/v) (d) (0 and (in)

w
(a) both in (b) both in Mn02
(JEE Main 2014)
(c) both in OH“
49. The gaseous product formed when HOCl reacts
{d) One in O^ and one in Mn02

F lo
with H2O2 in acidic medium is
54. H2O2 cannot oxide
(a) H2 ib) CI2
(a) Na2S03 ib) KI
(c) O2 (d) HCIO2
{e) HCIO3 (Kerala PMT 2014)
(c) PbS id) O3

ee
(Karnataka CET 2015)
SO. I. H2O2 + O3 — ^ H2O + 2 O2
55. The correct order in which the O—O bond length

Fr
II. H2O2 + Ag20 ■-> 2 Ag + H2O + O2 increases in the following is
Role of hydrogen peroxide in the above reactions
id) O3 < H2O2 < O2 ib) O2 < O3 < H2O2
is respectively

for
(c) O2 < H2O2 < O3 {d) H2O2 02^ O3
(a) oxidising in (I) and reducing in (U)
(AIIMS 2014, West Bengal JEE 2016)
ur
{b) reducing in (I) and oxidising in (II)
56. The dihedral angle of H2O2 in its solid phase at
(c) reducing in (I) and (II) llOKis
id) oxidising in (I) and (IT) (AIPMT 2014)
s
(a) 104° ib) 111-5°
ook
51. Hydrogen peroxide in its reaction with KIO4 and
Yo
(c) 90-2° id) 111-0°
NH^OH respectively, is acting as a (JEE Main 2022)
id) reducing agent, oxidising agent
eB

2+
Cu
ib) reducing agent, reducing agent 57. The product of NH2—NH2 + H2O2 -> is

(c) oxidising agent, oxidising agent ia) O2 ib) H.


id) oxidising agent, reducing agent (c) NH3 id) N,
our
ad

(JEE Advanced 2014) (Bihar CECE 2015)

H Multiple Choice Questions (with One or More than One Correct Answers)
Y

58. Which of the following will not liberate id) Hydrogen never acts as a cation in ionic salts
Re

dihydrogen ? (NEET Phase-I 2016)


nd

(a) Zn + H2SO4 (dil.) 60. Water can act as


(b) Zn + NaOH {aq)
ia) an acid ib) base
Fi

(c) Cu + H-SO4 (cone.)


(c) reductani id) oxidant
(d) F2 + H2O
59. Which of the following statementsabout hydiogen 61. The reagents used for .softening of the temporary
is incorrect ? hardness of water is/are

(a) Hydronium ion, H30‘‘‘ exists ffeedly in solution (a) Ca3(P04)2 ib) Ca(OH)2
ib) Dihydrogen does not act as a reducing agent (c) Na2C03 id) NaOCl
(c) Hydrogen has three isotopes of which tritium (IIT Paper 1 2010)
is the most common

ANSWERS
47.ic) 48. ia) 49. (c) 50. (c) 51. ia) 52. (c) 53. ia) 54. id)
55. ib) 56. ic) 57. id) 58. ic,d) 59. ib,c) 60. ia.b.c.d) 61. {b,c)
HYDROGEN 9/53

62. When zeolite, which is hydrated sodium (b) Na202 in water


aluminium silicate, is treatd with hard water, the
sodium ions are exchanged with (c) H2O2 in presence of HoS04
(a) ions ih) ions (cl) Na202 in presence of H7SO4
(JEE Advanced 2015)
(c) SO^ ions id) Mg“"^ ions
65. Which of the following statements are correct ?
(e) OH“ ions
63. Which of the following has lower value of D2O («) H2O2 reduces MnO^ both in acidic and basic
than for H2O ? media
(a) Molecular mass (b) H2O2 oxidises Fe^'*' ions both in acidic and
(b) Dielectric constant

w
basic media

(c) Ionization constant (c) H2O2 oxidises Mn-'*' to Mn'^'*’ ions in basic
(d) Viscosity medium

F lo
64. Fe^'*’ is reduced to Fe-"^ by using (d) H2O2 liberates 4 from acidified K1 solution
(a) H2O2 in presence of NaOH and reduces I2 to I" ions in basic medium.

nn^

ee
Multiple Choice Questions (Based on the given Passage/Comprehe nsion)

Fr
The comprehension given below is followed by some multiple choice questions. Each question has one

correct option. Choose the correct option.

for
[compre'hensionlC Hydrogen peroxide can be
(a) 112 mL (b) 336 mL
ur
prepared by the action of dil. H2SO4 or (c) 200 mL (d) 224 mL.
H3PO4 on barium peroxide or by bubbling 68. Hydrolysis of one mole of peroxodisulphuric acid
carbon dioxide through a thin paste of barium produces
s
peroxide. On an industrial scale, it can be
ook
Yo
(a) two moles of sulphuric acid
prepared by hydrolysis of peroxodisulphuric
acid obtained by electrolysis of 50% H2SO4 (b) two moles of peroxomonosulphuric acid
eB

or an equimolar mixture of H2SO4 and (c) one mole of sulphuric acid and one mole of
ammonium sulphate. The strength of H2O2 peroxomonosulphuric acid
solution can be expressed in a number of ways (d) one mole of sulphuric acid, one mole of
namely normality, molarity, percentage
our

peroxomonosulphuric acid and one mole of


ad

strength and volume strength. Volume


hydrogen peroxide.
strength refers to the volume of O2 produced
at N.T.P. by decomposition of 1 mL of H2O2 69. 100 volume hydrogen peroxide solution
solution. H2O2 acts as an oxidising as well a means
Y

reducingagent both in acidicand basic media. (rt) 17-86 N


Re

(b) 30-36% H2O2


nd

66. The correct increasing order of the acidity of CO2, (c) 8-93 M
H2O and H2O2 is
Fi

(d) All are correct


(a) CO2 < H2O2 < H2O
70. Which of the following substances on treatment
ib) H2O < H2O2 < CO2
with H2O2 gives MnO^
(c) H2O < H2O2 > CO2
(a) acidified KMn04
id) H2O2 > CO2 > H2O
{h) alkaline KMn04
67. The volume of 10 volume H-,02 solution that
decolourises 200 mL of 2N KMn04 solution in (c) alkaline MnS04
acidic medium is : (d) both (b) and (c)

ANSWERS
62. (b.d) 63. (b,c) 64. (a.b) 65. (a,b,c,d) 66. (b) 67. id)
68.(c) 69. (d) 70. id)
9/54 'P>utdecfr'^ New Course Chemistry (XDSSSIBD

09 Matching Type Questions


Match the entries of column I with appropriate entries of column II and choose the correct option out
of the four options (a), (b), (c), (d) given at the end of each question.
71. Column I Column II
2+ ;
(A) Syngas (P) Causes sequestration of and Mg ions

(B) Hydride gap (9) Repeated electrolysis of water


(C) Calgon (r) Elerhents of groups 7, 8 and 9
(D) Heavy water (i') CO + H2

w
(a) As, B-r, C-p, D-q (b) A-r, Bs, C-q, D-p (c) A-p, B-r, C-s, G-q (d) A-q> B-p, C-r, D-5
72. Column I (Hydride) Column U (Type of hydride)

F lo
(A) BeH2 (P) Complex
('?) Interstitial
(B) AsHg
(C) (r) Covalent
LaH3

ee
(D) UAIH4 is) Polymeric

Fr
(a) As, B-r, C-p, D-q {b) A-p, Bs, C-q, D-r (c) A-/-, C-s, D-p id) As, B-r, C-q, D-p

Column II (Nature)

for
73. Column I (Hydrides)
(A) MgH, (P) Electron precise
ur
i<l) Electron deficient
(B) GeH4
ir) Electron rich
(C) B.Hg
s
(D) HF is) Ionic
ook
Yo
(fl) As, B-p, C-q, D-r (b) A-r, B-p, C-q, D-5 (c) A-p, B-q, C-s, D-r (d) A-q, B-r, C-s, D-p
eB

p q r s

Matrix-Match Type Questions 1 r

Match the entries of column I with appropriate entries of column II. Each
A
© e
our

I© © O ©
ad

entry in column I may have one or more than one correct option from B '

column II. If the correct matches are A-p, s ; B-r ; C-p, q ; D-s, then the
correctly bubbled 4x4 matrix should be as follows :
c
\0 © o ©
D <
©®[© [©
Y

74. Column I Column II


— —3 11 1
Re

(A) Nascent hydrogen ip) Permutit


nd

(B) Hard water iQ) Used as tracer in the study of reaction mechanism
(C) Hydrogen peroxide ir) Reduces FeCl3 to FeCl2
Fi

(D) Heavy water is) Reduces Cr202~ to Cr^"*"


75. Column I Column II

(A) H2 ip) Complex metal hydride


(B) LiAlH4 iq) Oxidising agent
(C) H2O2 ir) Amphoteric
(D) H2O is) Reducing agent

ANSWERS
71.(a) 72. (cO 73. ill) 74. (A-f;s ,● B-p ; C-,rs ; D-q)
75. {A-q,s ; B-p.s ; C-q,s ; D-q,,rs)
HYDROGEN
9/55

VI.
Inteyor 1 ype Questions A B C D

DIRECTIONS. The answer to each of the following questions is a single digit ©


(0)(0>(0
integer, ranging from 0 to 9. If the correct answers to the question numbers
A, B, C and D (say) are 4, 0, 9 and 2 respectively, then the correct darkening ©000
of bubbles should be as shown on the side :
@ ©@©
76. Number of isotopes of hydrogen are :
©© ©®
77. How many of the following metals dissolve in boiling alkali to produce H-) gas ?
Cu, Ni, Zn, Be, Ag, Fe, Mn, Sn, Al. ®©©®
© ®©©

w
78. Number of electron-rich hydrides among the following are :

CH4, NH3, PH3, H2O, H2S. BH3, HF, AIH3, ASH3. ©®©©
79. Presence of which of the following compounds makes water hard :

F lo
Na2S04, Ca(HC03)2, MgC^, Na2C03. CaS04, KCl, NaHC03, MgS04, CaCl2. ©©©©
80. How many of the following oxides would liberate H^O^ on treatment with dil.
H2SO4 ?
©©©®
Pb02, Na202. Mn02, Ba02, Ti02, CO2. NO2, SnO,, Ag^O. ©® ®,®

ee
Fr
VII.
Numerical Value Type Questions (in Decimal Notation)

for
For the following question, enter the correct numerical value, (in decimal-notation,truncated/roundcd-off
to the second decimal place, e.g., 6-25, 7-00, - 0-33, - -30, 30-27, - 127-30) using the mouse and the on
r
screen virtual numeric keypad in the place designated to enter the answer.
You
81. A 5 0 cm^ solution of H2O2 liberates 0-508 g of iodine from
s
an acidified KI solution. Calculate the strength
ook
of H2O2 solution in terms of volume strength at STP

VIII.
eB

Assertion-Reason Type Questions


TYPE I
our
ad

DIRECTIONS. The questions given below contain statement-1 (Assertion) and statement-2 (Reason).
Each question has four choices (a), (b), (c) and (d) out of which ONLY ONE is correct. Choose the correct
option as under :
(a) Statement-1 is True, Statement-2 is True ; Statement-2 is a correct explanationfor Statement-1.
dY

(^) Statement-1 is True, Statement-2 is True ; Statement-2 is not a correct explanationfor Statement-1,
Re

(c) Statement-1 is True, Statement-2is False. (d) Statement-1is False, Statement-2is TYue.
82. Statement-1. Electiolysis of NaH in the fused state liberates H-, at the anode.
Fin

Statement-2. NaH contains H" ions.

83. Statement-1. HF is an electron-precise hydride.


Statement-2. In HF, F has three lone pairs of electrons.
84. Statement-1. H2O2 decomposes NaHC03 to evolve CO2 gas
Statement-2. H2O2 is a weaker acid than H^C03 acid.
85. Statement-1. The colour of old lead paintings can be restored by wa.shing with a dilute solution of H2O-,.
Statement-2. Hydrogen peroxide oxidises black lead sulphide to while lead sulphate.
ANSWERS
76. (3) 77. (4) 78. (6) 79. (5) 80. (2) 81. (4-48) 82. (n)
83. (rf) 84. (rf). 85. (n)
9/56 New Course Chemistry (Xl)E!SiaD

86. Statement-1. H^02 reduces potassium ferricyanide in alkaline medium.


Statemcnt-2. Whenever H2O2 acts as a reducing agent. O2 is always produced.
87. Statement-1. The amphoteric nature of water is explained by using Lewis acid-base concept.
(JEE Main 2022)
Statement-2. H,0 acts as an acid with NH3 and as a base with HoS.
88. Statement-1. Stannane is an example of molecular hydride.
Statement-2. Stannane is a planar molecule. (JEE Main 2022)
89. Statement-1. Hydrogen peroxide can act as an oxidising agent both in acidic and basic condition.
(JEE Main 2022)
Statement-2. Density of H2O2 at 298 K is lower than that of D^O.
TYPE II

DIRECTIONS. In each of the following questions, a statement of Assertion (A) is given followed by a

w
corresponding statement of Reason (R) just below it. Of the statements, mark the correct answer as
(fl) If both assertion and reason are true, and reason is the true explanation of the assertion.

F lo
{/>) If both assertion and reason are true, but reason is not the true explanation of the assertion,
(c) If assertion is true, but reason is false. id) If both assertion and reason are false
90. Assertion. Chlorine reacts more rapidly with H2 than with D2.
Reason. D-Cl bond is stronger than H-Cl bond. (AIIMS 2015)

e
Fre
91. Assertion. Dihydrogen oxidises sodium to sodium hydride.
Reason. Hydrogen can act only as a reducing agent.
92. Assertion. Electrolysis of molten CaH2 produces hydrogen gas at anode.

for
Reason. In CaH2 hydrogen is present in the form of hydride H”. (AIIMS 2014)
93. Assertion. Beryllium hydride is a covalent hydride.
Reason. The electronegativity difference between Be and H is very high.
r
94. Assertion. H2O is the only hydride of group 16 which is a liquid at ordinary temperature.
You
Reason. In ice, each oxygen atom is surrounded by two covalent bonds and two hydrogen bonds.
oks
(AIIMS 2015)
2+ 10ns.
;
95. Assertion. Calgon is used for removing Ca^'^ and Mg
eBo

2+ :
Reason. Calgon forms precipitate with Ca-'*’ and Mg ions.

96. Assertion. NaCl is less soluble in heavy water than in ordinary water.
Reason. Dielectric constant of ordinary water is more than that of heavy water. (AIIMS 2011)
our

97. Assertion. H2O2 has higher boiling point than water.


ad

Reason. The dipole moment of H2O2 is little more than that of H2O.
98. Assertion. Decomposition of H2O2 is a disproportionation reaction.
Reason. H2O2 molecule simultaneously undergoes oxidation and reduction reactions.
99. Assertion. H2O2 liberates O2 when it reacts with acidified KMn04 solution.
dY
Re

Reason. KMn04 oxidises H2O2 to O2.


100. Assertion. H2O2 can be used as an antichlor in bleaching.
Fin

Reason. It oxidises HCI to CI2.


101. Assertion. The 0-0 bond length in H2O2 is shorter than that of O2F2
Reason. H2O2 is an ionic compound.
102. Assertion. Nascent hydrogen can discharge the pink colour of KMn04 solution.
Reason. Nascent hydrogen is much more reactive than dihydrogen.
103. Assertion. On adding zinc pieces to aqueous FeCl3 solution, colour changes from deep yellow to light green.
Reason. Aqueous FeC^ is acidic and on adding zinc, nascent hydrogen is produced which reduces deep
yellow FCCI3 solution to light green FeCL solution. (AIIMS 2013)
ANSWERS

86. {b) 87.id) 88.(c) 89.(d) 90.(b) 91.(c) 92.(a) 93.(c) 94.(b) 95. (d)
96. (rt) 97. (h) 98. (o) 99. («) 100. (t) 101. {d) 102. (a) 103. (a)
HYDROGEN 9/57

For Difficult Questions

Multiple Choice Questions (with one correct Answer)

ow
1. A compound is said to be covalent if the 12. Due to different masses, the physical properties
electronegativity difference between the of three isotopes of hydrogen are different. This
combining atoms is less than 2-1. is called isotope effect.
2. All statements are correct. Please note the ionic 13. In water gas shift reaction, hydrogen gas is
sizes : H" (208 pm), F" (136 pm), Cr (181 pm), produced economically by the reaction of CO with

e
Br" (196 pm) and T {219 pm) H2O (steam) at 673 K in presence of iron,

re
3. '4n + >„ '“C + chromium and copper catalyst.

Frl
Catalyst
4. Nitric acid being a strong oxidising agent, oxidises CO (g) + H.O (g)

F
673 K > C02(g) + H2(g)
initially Cu to CuO which further reacts with
HNO3 to form Cu(N03)2 with the evolution of 15. 2Na +2H2O ^H2 + 2NaOH
A B C
NO2 gas. However, H2 gas is not produced.
ou
or
and Zn + 2 NaOH ■> Na2Zn02 + H2
Fe becomes passive in cone. HNO3 due to the D C
formation of an impervious layer of iron oxide on B

kfs
its surface which prevents the further action. Zn +H2S04(dil.) ●> ZnS04 +
Au reacts with NaCN in presence of air to form D B
soluble complex of dicyanoaurate (I) Since Na gives golden yellow flame, K
oo
4 Au + 8 NaCN + 2 H2O + gives pink violet flame while Ca gives brick red
4Na[Au(CN)2] + 4 NaOH flame, therefore, option (a) is correct.
Y
Dicyanoaurate (I) 17. Due to H-bonding, H2O has the highest m.p.
B

Only Zn reacts with NaOH solution to evolve H2 (273 K). The melting points of other hydrides
gas increase as their molecular masses increase, i.e.,
re

A H2Te (222 K) > H2Se (208 K) > H2S (188 K).


Zn + 2 NaOH
Na2Zn02 + H2 Thus, option (a) is correct.
Sod. zincate
oYu

19. B2Hg is electron-deficient, saline (salt like) is LiH,


5. Electrolysis of warm Ba(OH)2 .solution using Ni CH4 is electron-precise, CrH is interstitial and NH3
ad

electrodes.
is electron-rich. Thus, option (b) is correct.
6. Zn -t- 2 NaOH Na2Zn02 + H2 21. As the size of the element (X) increases, the length
d

Zn + 2 HCl
ZnCl2 + H2 of H-X bond increases and hence its bond strength
According to stoichiometry, in both the reactions, decreases. As a result, it becomes easier to break
in
Re

equal number of moles of H-) are evolved and thus the H-X bond to liberate H"*" ions and hence the
the ratio of the volume of H2 evolved is 1 : 1. acidity increases in the order: H2S < H->Se < H2Te,
7. Cu does not react with mineral acids to liberate i.e., option {a) is correct.
F

H2. 22. H" ion is a strong conjugate base of H2.


8. CuO + H2 > Cu -I- H2O 23. H" ion is a Lewis base in I and Bronsted base in II.
9. Hydrogen has, no neutron, Deuterium has one 24. ortho and para hydrogens are spin isomers because
and tritium has two. So the total = 0 + 1 + 2 = 3 they differ from each other in the direction of spin
of their nuclei.
10. Tritium is beta particle emitting radioactive isotope
of hydrogen. 26. There is extensive intermolecular H-bonding in
the condensed phase.
?H
1
4 ^He -I- -1
% 0 -I +1
p-panicle
Tritium
27. Clj +H2O 4 HCl + HOCl
11. Due to isotope effect, deuterium reacts less
28. F-j oxidises H^O to 0^ but CI^ does not (refer to
vigorously than hydrogen.
page 9/21).
9/58 "pfuuiee^'^^ New Course Chemistry (Xl)CZsl9D
or 10^ mg hard H^O contains CaS04 = 136 mg
For Difficult Questions
100

29. It is not a hydrolysis type reaction but is a redox


or 10^ mg hard H2O contains CaC03 -136 x 136
reaction in which F2 oxidises H2O to O2 and is = 100 mg
2 F2 (g) + 2 H2O (0 >4 HF (aq) + O2 (g) Degree of hardness = 100 ppm
i self reduced to HF.

30. Clark’s method is used to remove temporary


38. Ba02 + H2SO4 > BaS04 + H2O2

w
hardness from water while all the remaining 40. 2Mn04+6H’^+5H2O2 >
methods are used for removing permanent
hardness of water
2 Mn^+ + 8 H2O + 5 O2
42. 1 mLof20volumeH202liberates02atSTP=20 mL
CaO + H2O > Ca(OH)2

Flo
15 mLof H2O2 will liberate 02atSTP= 15 x 20
Ca(HC03)2 +Ca(OH)2 ■>
2 CaC03 i + 2 HjO = 300 mL
Temporaiy
43. Let the normality of H2O2 = N,

e
hardness

re
31. Mg(OH)2 is formed instead of MgCO^ since N[ X 10 (H2O2) = 1 X 10 (Na2S203)
MgC03 is more soluble in aqueous solution than Normality of H2O2 .solution = 1 N

F
Mg(OH)2- From Supplement Your Knowledge For
33. Demineralized water is obtained by passing water Competitions, page 9/34.
ur
r
-1
successively through a cation exchanger (in the 1 N H2O2 = 5-6 vol. H2O2 = 17 gL

fo
H'*’ form) and an anion exchanger (in the OH" Therefore, all statements are correct.
form) resin.
44. Refer to ‘Supplement Your Knowledge For
ks
35. Amount of MgS04 = 0 002 x 120 x 1000 Competitions’,Page 9/34.
Yo
= 240 mg -1
Strength in gL
oo
Now 120 mg MgS04 = 100 mg CaC03 Volume strength =5-6x
240 mg MgS04 = 200 mg CaC03
Eq.wt.of H.,02
B

I L of water contains = 200 mg of CaC03 Strength in gL


-1

11-2 = 5-6 X
or 10^ mg H-)0 contains = 200 mg of CaC03 or
re

17
Degree of hardness = 200 ppm.
-I _
11-2x17
= 34
or Strength in gL'
36. 2CgH.^SOj Na+ + Ca^+
u

■>
5-6
ad

Imol
Yo

2 mol

(2x206 = 412g) 46. Refer to ‘Supplement your knowledge for


(CgH2SOj)2Ca2+ -+. 2 Na+ competition, page 9/34.
1 mol of Ca^"*" = 412 g of resin 11-2= 11 -2 X Molarity or Molarity = 1 m
uptake of Ca^'*’ ions by the resin
d

or maximum
1000 mL contain H2O2 = 34 g
Re
in

Imol 1
mol/g .-. 100 mL will contain H2O2 = 3-4 g
412g 412
Strength of H2O2 solution = 3-4%
F

37. Mol. miuss of CaS04 = 136 g mol"’ (Given) 47. H2O2 can act both as an oxidising as well as a
Mol. mass of CaC03 = 40 + 12 x 3 x 14 reducing agent.
-1
- 100 g mol
48. O2 is evolved in reactions («) and (iv), i.e., option
Now 136 g of CaS04 = 1 (X) g of CaC03 (a) is correct.
100 49. HOCl being an oxidising agent, oxidises H2O2 to
or 1 mg of CaS04 = mg of CaC03
136 O2 and itself gets reduced to Cl" ion
1 L of hard H2O contains CaS04 = 1(T^ moles +1 -1

= 10-3x136x103 HO-Cl-f H2O2 ^ HP* + Cl" + O2


= 136 mg (Refer to reaction (vi), page 9/38)
HYDROGEN 9/59

Slate. In other words, in alkaline medium. H2O2


For Difficult Questions
reduces Fe-^'*' to Fe^'*' but it oxidises Mn-'*’ to Mn“^.
Thus, option (c) is correct.
50. In both these reactions, H,0-> acts as a reducing
agent since it itself is oxidised to 0-,. 53. H2*^02 {aq) + 2 OH" iaq)
+7 +5 2 H2O (/) + 'SO2 ig) + 2 e-

w
51. KIO4+ H2O2 ■> KI03+H20 + 0^ So, all the '^O label appears in ’^0-)
Reducing agent
54. H2O2 oxidises Na2S03 to Na2S04, KI to I2, PbS
+5 to PbS04 but reduces O3 to O2
NH2OH+ 3H2O2 ^ HNO3+4H2O O3 + H2O2 > 2 O2 + H2O
Oxidising agent 55. Bond length increases as bond order (within

o
52. The maximum O.S. of Fe is + 3, therefore, Fe-^"^ bracketts) decreases, i.e.

e
can only be reduced by H2O2. In contrast, the O2 (2)<03(1-5)<H202(1)

re
minimum stable O.S. of Mn is + 2, therefore, it Cu2+
can be ftmher oxidised by H2O2 to + 4 oxidation

Frl
57. NH2NH2 + 2 H2O2 ^ N2 + 4 H2O

F
m Multiple Choice Questions (with One or More than One Correct Answers)

or
59, Dihydrogen acts a reducing agent. Further, out of the corresponding insoluble carbonates which
ou
the three isotopes of hydrogen, protium is the most upon filtration give soft water.

Ca(HC03)2 + Ca(OH)2 > 2 CaCOg -L + 2 HjO

kfs
common.

61. Ca(OH)2 and Na2C03 react with bicarbonates of Ca{HC03)2 + Na2C03 > CaC03 i
Ca and Mg present in temporary hard water to form
+ 2 NaHC03
oo
DU Multiple Choice Questions (Based on the given Passage/Comprehens ion)
Y
B

66. A stronger acid displaces a weaker acid from its 68. HO—SO2—O—O—SO2—OH + H2O 4

salts.
Peroxodisulphuric acid
re

Ba02 + H2O + CO2 ■> BaC03 HOSO2OH + HOOSO2OH


Weaker acid
oYu

Stronger acid Sulphuric acid Peroxomono-

i.e. CO2 is a stronger acid than H2O2


ad

sulphuric acid
option {b) is correct. 69. Apply the formulae given in ‘Supplement Your
Knowledge For Competitions’ page 9/34. All
d

67. Refer to An.s. to P. 6, page 9/45.


options are correct.
in
Re

VI.
Integer Type Questions
F

76. Three, i.e., H, D and T. 79. Five, i.e., Ca(HC03)2, MgCl2, CaS04, MgS04,
77. Four, i.e., Zn, Be, Sn. Al. CaCl2.
78. Six, i.e., NH3, PH3, H2O, H2S, HF, ASH3. 80. Two, i.e., Na202 and Ba02 contain peroxide
linkage and hence liberate H2O2.
VII.
Numerical Value Type Questions Decimal Notation)
81. 2KI + H2SO4 + H2O2 — ^ K2SO4 -f 2H2O + I2 0-508 g of I2 will be liberated from H2O2
From the above equation, 34

H2O2 = I2 or 34 g of H2O2 = 254 g of I2 254


X 0-508 = 0-068 g
9/60 A New Course Chemistry (XI)

Now 5 0 cm^ of H2O2 solution gives O2


For Difficult Questions
= 224 cm^ at STP

The decomposition of H2O2 occurs as 1 -0 cm^ of H2O2 solution will give O2


2H2O2 2H2O + O')
224
2 X 34 = 68 g 22400 cm^ at NTP = = 448 cm^ at STP
0 068 of H-)02 upon decomposition will give
22400
Thus, volume strength of given H2O2 solution
02 = X 0-068 =224cm^ = 448.
68

w
VIII, Assartion-Reason Type Questions

94. Correct explanation. Due to extensive H-

F lo
83. Correct statement-1. HF is an electron-rich
hydride. bonding, water exists as associated molecules. As
a result, it has high b.p. and thus water is a liquid
84. Correct statement-1. H2O2 does not decompose
at room temperature.
NaHC03 since it is a weaker acid than H2CO3.

ee
95. Correct assertion. Calgon renders Ca^'*’ and Mg^'*’
86. Correct explanation. During the process, H2O2 ions present in hard water ineffective.

Fr
is oxidised to O2. Correct reason. Calgon forms soluble complexes
87. Correct statement-1. The amphoteric nature of with Ca^"*" and Mg^''' ions.

for
water is explained using Bronsted-Lowry concept. 97. Correct explanation. H2O2 is more highly
88. Correct statement-2. Stannane is a non-planar, associated than H2O.
ur
i.e., tetrahedral molecule. 98. Correct explanation. The extent of H-bonding
89. Correct statement-2. Density of H2O2 (1 -44) g in H7O2 is higher than that in H2O.
s
mL"’ is higher than thatofD20 (1-106) g mL~* at 100. Correct reason. H2O2 reduces excess CI2 to Cl"
ook
Yo
ions.
298 K.
101. Correct assertion. The O—O bond length in
90. Correct explanation. Due to isotope effect, rate
eB

H2O2 is much longer than that in O2F2.


of reaction of CI2 with D2 is slower than that with
Correct reason. The lone pairs of electrons on
H2- the 0 atoms are strongly attracted by the
91. Correctreason. Hydrogenacts as oxidising agent electronegative F atoms. As a result, lone pair-
our

towards highly electropositive metals like Na, Ca,


ad

lone pair repulsion of the two oxygen atoms is far


etc.
less in O2F2 than in H2O2. As a result, 0—O bond
93. Correct reason. The electronegativity difference length in O2F2 (1-22 A) is much shorter than in
between Be (1-5) and H (2-1) is small. H2O2 (148 A)
Y
Re
nd
Fi
ow
s-BLOCK ELEMENTS
(ALKALI AND ALKALINE EARTH METALS)

e
Fl
re
10.1. GENERAL INTRODUCTION

F
Elements in which the last electron enters the 5-orbital are called 5-block elements. Since 5-subsheIl
has only one orbital which can accommodate only two electrons, therefore, there are only two groups of
ur
r
5-block elements. Thus, elements of group 1 or lA (alkali metals) and group 2 or IIA (alkaline earth

fo
metals) constitute 5-block elements. The elements of these groups contain one or two electrons in their
outermost 5-orbital while all the inner shells are completely filled. Thus, the electronic configuration of
ks
group 1 (alkali metals) may be represented as 1 Noble gas] ns^ while that of group 2 (alkaline earth
Yo
metals) may be represented as [Noble gas] ns- where n is the number of outermost shell. Although
oo
hydrogen which has one electron in the 5-orbitaI and helium (inert gas) which has two electrons in the s-
orbital may also be regarded as 5-block elements but they are usually discussed separately because of
B

their peculiar behaviour. Therefore, in this unit, we shall discuss the chemistry of alkali and alkaline
earth metals only.
re

Elements of the periodic table in which the last electron enters the 5-orbital are called 5-block
elements.
u
ad

10.1.1. Occurrence and Abundance


Yo

Both alkali and alkaline earth metals are highly reactive and hence do not occur in the^^e state but are
widely distributed in nature in the combined state. Whereas alkali metals mostly occur as halides oxides,
silicates, borates and nitrates ; alkaline earth metals mainly occur as silicates, carbonates, sulphates and
d

phosphates.
Re
in

Although alkali metals have similar chemical properties yet they do not occur together mainly
because their ions are of different sizes. Similarly, alkaline earth metals usually do not occur together
F

except dolomite which is a double carbonate of calcium and magnesium [MgC03 . CaC03]. Likewise
some double salts of alkali and alkaline earth metals such as carnallite (KCl.MgCl2 . 6 H2 O) are also
known to occur in nature.

Some alkali and alkaline earth metals occur abundantly in nature. For example, calcium is the fifth,
magnesium is the sixth, sodium is the seventh and potassium is the eighth, barium is the fourteenth and
strontium is the fifteenth most abundant element by weight in the earth’s crust. Sodium and magnesium are

* Not included in CBSE syllabus. This chapter has been given only for the preparationof competitiveexaminations.

10/1
10/2 New Course Chemistry (XI)KSISI]

also present in relatively large amounts in sea water, brine wells and a few salt lakes. Calcium and potassium
also occur in sea water but to smaller extents.

It is interesting to note that although sodium and potassium are almost equally abundant (2-83% Na and
2-59% K) in the earth’s crust, yet sea water contains 2-8% NaCl but only 0-8% KCl. This is due to the reason
that much of potassium appearing in ground water from dissolved minerals in taken up preferentially by plants
while sodium ion goes to the sea water.

The remaining elements are much less abundant, i.e., rubidium is the twenty-third, caesium is the
forty-sixth and beryllium is the fifty-first most abundant element by weight in the earth’s crust while both
francium (alkali metal) and radium (alkaline earth metal) are extremely scare and are also radioactive. Radium
was isolated by Pierre and Marie Curie by processing many tons of the uranium ore pitchblende. It was used
for radiotherapy treatment of cancer at one time but now ^Co is used. Marie Curie was awarded the Nobel

w
Prize for chemistry in 1911 for isolating and studying radium and polonium.

F lo
The element francium was discovered later in 1939 by Pierre at the Curie Institute, Paris. It is formed
during radioactive decay of actinium.

223 Fr + ^He
227
Ac ■>
89

ree
Its longest lived isotope 2^^Fr has a half-life period of 21 minutes. As a result, not much is known about

for F
its chemistry. Nevertheless, whatever little knowledge we have about this element clearly shows that in its
physical and chemical properties, it closely resembles alkali metals.
10.1.2. Anomalous Behaviour of First Element of a Group
The first element of a group differs considerably from the rest of the elements (heavier elements) of the
same group. This anomalous behaviour is due to (/) small atomic and ionic radii {ii) high electronegativity
Your
and ionization enthalpy, (Hi) high polarising power* of its cation and (iv) absence of d-electrons in its
ks

valence shell. Thus, Li differs from rest of the alkali metals (Na, K, Rb and Cs) and Be differs from rest of the
o
eBo

alkaline earth metals (Mg, Ca. Sr and Ba).

10.1.3. Diagonal Relationship


Some elements of certain groups in the second period resemble with the certain elements of the next
ad
our

higher group in the third period. This is called diagonal relationship. These are usually weaker than the
similarities within a group but are quite pronounced in the following pairs of elements.

Group 1 Group 2 Group 13 Group 14


2nd period: Li Be B C
Re

Si
Y

3rd period: Na Mg A1
Find

*When a cation and an anion approach each other, they attract the electron clouds of each other, but the
effect of a small cation on a large anion is more pronouncedthan that of the large anion on the small cation. This
distortion of the electron cloud of an anion by a cation is called polarization and the capacity of a cation to
polarize an anion is known as its polarizing power or ionic potential. Generally, the polarizing power increases
as the charge on the cation increases and the size decreases. It is expressed in terms of charge/size ratio or (ionic
charge)/(ionic size). Polarising power is also sometimes measured in terms of charge per unit area, i.e.,
ionic charge

j7t (ionic radius)2


S-BLOCK ELEMENTS (ALKALI AND ALKALINE EARTH METALS) 10/3

Thus, lithium resembles magnesium, beryllium resembles aluminium and boron resembles silicon. It
may, howeve,r be noted that these similarities are much weaker than similarities found within the group. The
main reason for the diagonal relationship are : FIGURE 10.1
(/) Similarity in electropositive characte.r Electronegativity increases, Atomic and ionic
We know that electronegativity increases along a sizes decrease, Polarizing power increases
>
period but decreases down a group. Conversely, Period T3 o ED
o 3 <0
electropositive character decreases along a period but P2 In' o-a
u o
increases down a group. Evidently on moving diagonally, =;●
^
3
Q. CO
3

f/./ (O
_.(Q
these two opposing trends partly cancel out. As a result, Q ■D
O
O
3
£1)
3-.
'555
diagonally related elements have similar electropositive %® "» ><§●-
o

w
c
O’ ■D

character (Fig. 10.1) and hence exhibit similar properties. Q. y ‘=“-


S « q
(//■) Similarity in atomic or ionic radii. We know .S' CP 3'

F lo
“ S “
that atomic and ionic radii decrease across a period but
»
<0
V)
CO -

increase down a group. Evidently, on removing CD

Similar electronegativity, Similar atomic and


diagonally, these two trends partly cancel out. As a result, ionic radii. Similar polarizing power
diagonally related elements have similar atomic and ionic

ee
radii (Fig. 10.1) and hence have similar properties. Illustration of diagonal relationship

Fr
ini) Similarity in polarising powe.r On moving across a period, the charge on the ion increases and size
decreases, causing the polarizing power to increase. On moving down a group, the size increases and the

for
polarizing power decreases. On moving diagonally, these two trends partly cancel out. As a result, diagonally
related elements have similar polarizing power and hence have similar properties.
ur
SECTION—I. ALKALI METALS
oks
The group 1 of the periodic table contains six elements, namely lithium (Li), sodium (Na), potassium
Yo
(K), rubidum (Rb), caesium (Cs) and francium (Fr) besides hydrogen. All these elements are typical metals.
o

These are usually referred to as alkali metals since their oxides and hydroxides are soluble in water and form
eB

strong bases or alkalies.

10.2. OCCURRENCE
our

Alkali metals are highly reactive and hence do not occur in the free state but are widely distributed in
ad

nature in the combined state in form of halides, oxides, silicates, borates and nitrates.
Lithium is the thirty-fifth most abundance element by weight in the earth’s crust (lithosphere) and mainly
occurs in form of silicate minerals, i.e. (i) spodumene, LiAl(Si03)2, and (ii) lepidolite, Li2Al2(Si03)
Y

(F, 0H)2- Sodium and potassium are the seventh and eighth most abundant elements by weight in the earth’s
Re

crust. NaCl and KCl occur in large amounts in sea water. The largest source of sodium is rock salt (NaCl).
nd

Other important minerals of sodium are: Na2B40y. 10 H2O iborax), Na2C03.NaHC03.2 H2O (trona), NaNO,
Fi

(chile salt petre) and sodium sulphate (mirabilite).


Potassium mainly occurs as KCl (sylvite), a mixture of KCl and NaCl (sylvinite) and the double salt.
KCl.MgCl2.6 H2O (carnallite) and as K2O.AI2O3.6 Si02 (feldspar).
10.3. ELECTRONIC CONFIGURATION
Alkali metals are s-block elements, because last elecu-on in them enters the 5-orbital. They contain only
one 5-electron outside the noble gas core. Therefore, their general electronic configuration is [noble gas] «5*
where n = 2 to 7. Since all these elements have similar valence shell or outer electronic configuration, they
have similar physical and chemical properties. The electronic configurations of alkali metals are given in
Table 10.1.

i
10/4 New Course Chemistry (XI)EEMD

TABLE 10.1. Electronic configuration of alkali metals


Elements Atomic Electronic Configuration
Number Complete With Inert gas core
1
Lithium (Li) 3 2s' [He] 25
Sodium (Na) 11 3^' [Ne] 35
Potassium (K) 19 l.v“ 2.t-2/35“3/7^4^‘ [At] 45'
Rubidium (Rb) 37 2s^ liP 3^“ 3/7*^ 3d'° 4s~ 4p^ 5s' [Kr] 55'
l.r 2s- 2p^ 3^2 3/' As- Ap^ Ad
10 1
Caesium (Cs) 55 [Xe] 65
5s^ 5/7*’ 6,v'

w
15- 2s- 2p^ 3^ 3p'> 3^/'^ As- Ap^ Ad^°
1
Francium (Fr) 87 [Rn] 75
4/^ 5.S- 5// 5^/’" 6.V- 6p^ 7.?'

F lo
10.4. TREN'J.* IN ATOMIC AND PHYSICAL PROPERTIES
Some important atomic and physical properties of alkali metals are given in Table 10.2.

ee
TABLE 10.2. Some atomic and physical properties
of group 1 elements (alkali metals)

Fr
Elements

for
Property Li Na K Rb Cs Fr

(Radioactive)
r
Atomic number 3 11 19 37 55 87
You
s
Atomic mass (g mol"') 6-94 22-99 3910 85-47 132-91 223
ook

Metallic radius (pm) 152 186 227 248 265 375

Ionic radius (M'^’/pm) 76 102 138 152 167 (180)


eB

Ionization enthalpy A^H j 520 496 419 403 376 *-375

(kJ mol-') A- H2 7298 4562 3051 2633 2230


-1 -276
Hydration enthalpy/kJ mol -506 -406 -330 -310
our
ad

Eletronegativity
(Pauling scale) 0-98 0-93 0-82 0-82 0-79

Density/g ern"^ (at 293 K) 0-53 0-97 0-86 1-53 1-90

Melting point/K 454 371 336 312 302


Y

Boiling point/K 1615 1156 1032 961 944


Re

E°/V at 298 K for


d
Fin

M'*’ (aq) + e~ 4 M (5) - 3-04 - 2-714 - 2-925 - 2-930 - 2-927

Occurrence in lithosphere 227** [.84** 78-12* 2-6*

These properties are briefly discussed below :


1. Large atomic radii. The atomic radii of alkali metals are the largest (after inert gases) in their
respective periods. These increase as we travel down the group from Li to Cs as shown in Table 10.2
Explanation.This is due to the reason that as the atomic number increases, the number of energy shells
increases by one with each succeeding element. As a result, the screening effect of the inner filled shells on
the valence 5-eleclron increases and ihe electron cloud lends to expand. In other words, the distance between
the nucleus and the last shell increases and thus the atomic radius increases with increase in atomic number.

**Perceniage by weight.
*
ppm (parts per million).

i
rw
s-BLOCK ELEMENTS (ALKALI AND ALKALINE EARTH METALS) 10/5

It may, however, be stated here that as the atomic number increases, the nuclear charge also goes up. This
tends to decrease the atomic radii by attracting the electron cloud inward more forcibly. But the screening
effect is so large that it overcomes the contractive effect of the increased nuclear charge. Hence, the increase
in atomic radius as we move down the group from Li to Cs is primarily due to the predominant screening
effect (addition of a new energy shell) of inner iflled shells on the valence s-electron.
2. Large ionic radii. The ionic radii of the cations formed by them are smaller in size than the
corresponding atoms. However, like atomic radii, ionic radii are the largest in their respective periods.

lo
Explanation. When the valence s-eleclron is lost from the alkali metal atom, the monovalent cation
formed has one shell less than the corresponding metal atom. Further, with the loss of one electron, the

F
effective nuclear charge increases and thus the remaining electrons are pulled closer to the nucleus thereby

u
decreasing the size of each individual energy shell. Hence, it can be concludedthat due to contractiveeffect
of the increased nuclear charge and lesser number of energy shells, the radii of cations are much smaller

oF
than the corresponding atoms.
TVend in the group. Like atomic radii, the ionic radii increase we move down the group from Li to Cs

rs
primarily due to the addition of a new energy shell with each succeeding element as shown in Table 10.1.
3. Low ionization enthalpies. (/) The first ionization enthalpies (i.e., A-H,) of the alkali metals are the

ok
lowest as compared to the elements in the other groups.
Explanation. This is mainly because of the fact that noble gas core shields the lone 5-electron in the

o
f
valence shell from the direct attraction of the positive charge on the atomic nucleus. As a result, the valence
5-electron is loosely held by the nucleus and can be easily knocked off by supplying only a small amount of

o
Y
energy.

IVend in the group. The ionization enthalpies of alkali metals decrease progressively as we move
rY
B
down the group from Li to Cs.
Explanation. This is due to the fact that the 5-electron to be removed becomes farther and farther away
from the nucleus because of the addition of a new shell with each succeeding element (Li, 25* ; Na, 35* ; K,
45* ; Rb, 55* ; Cs, 65* and Fr, 75*). Due to an increase in the number of inner shells, the screening effect also
ue

increases.As a result, the attractionof the nucleus for the valence 5-eIectron decreases and hence the ionization
enthalpies decrease as we travel down the group from Li to Cs.
od

(«) The second ionization enthalpies (i.e., A,//2) of alkali metals are very high.
ad

Explanation. When an electron has been removed from the alkali metal, the resulting monovalent
in

cation has the stable noble gas configuration. Since it is difficult to remove an electron from the stable noble
gas configuration, therefore, the second ionization enthalpies of alkali metals are very high. Further, as expected,
like the first ionization enthalpies, second ionization enthalpies decrease as we move down the group due to
increasing size and increasing shielding effect.
Re
F

4. Hydration enthalpy. The hydration enthalpies of alkali metal ions decrease with the increase in
ionic radii, i.e., Li'*’ > Na'*' > K'*’ > Rb'*' > Cs"*"
Because of the smallest size, Li'*' has the maximum degree of hydration and it is because of this reason
that lithium salts are mostly hydrated, e.g., L1C104.3 H2O, Li2C03.3 H2O and LiCl.2 H2O.
5. +1 Oxidation state— Unipositive ions. The alkali metals exhibit oxidation state of +1 in their
compounds and are strongly electropo.sitive in characte.r The electropositive character increasesfrom lithium
down to caesium in the group.
Explanation. As the alkali metals have low ionization enthalpies, they have a strong tendency to lose
the single valence 5- electron to form the unipositive ions. Thus, they show an oxidation state of +1 and are
strongly electropositive.
M M"*" + e
10/6 ^n<idecfi, ^ New Course Chemistry (XI) LV«iwii

The second ionization enthalpies of these elements are fairly high and thus the loss of second electron is
quite difficult. This is due to the fact that the second electron has to be lost from the most stable noble gas
core, i.e., ns^ np^. Thus, we may conclude that the chemistry of alkali metals is essentially the chemistry of
unipositive ions.
As we go down the group, the atomic size increases and the ionisation enthalpy of the elements decreases,
i.e. the tendency of the elements to lose electrons increases. Consequently, the electropositive character
increases.

6. Metallic character. The elements of this group are typical metals which are soft, malleable and
ductile solids. These can be easily cut with a knife. When freshly cut, they are silvery white and on exposure
to air they are tarnished ij.e. become dull.) The metallic character increases down the group.

w
Explanation. The tendency of an element to lose valence electrons to from positive ions determines its
metallic character. As alkali metals have low ionization energies, they have high tendency to lose valence
electrons and are, therefore, typical metals. The metallic character increases down the group because ionization

F lo
energy decreases down the group.
The bigger the kernel or smaller the number of valence electrons, the weaker is the metallic bond.
Since the atoms of alkali metals have bigger kernels and smaller number of valence electrons, the
metallic bonds in them are very weak. Hence they are soft and can be cut with a knife.

ee
7. Melting and boiling points. The melting and boiling points of alkali metals are very low and decrease

Fr
with increase in atomic number.

Explanation. As alkali metals have large size, the intermetallic bonds in them are quite weak. Hence,
they have low melting and boiling points which decrease down the group with the increase in the atomic size.

for
8. Nature of bonds formed. All the alkali metals form ionic (electrovalent) compounds. The ionic
character increases from Li to Cs as we move down the group.
ur
Explanation. This is because alkali metals have low ionization enthalpies which decrease down the
group.
s
ook
9, Density. The densities of alkali metals are quite low as compared to other metals. Li, Na and K are
Yo
even lighter than wate.r These densities increase from Li to C.s.
Explanation. Because of their large size, their atoms are less closely packed. Consequently they have
eB

low densities. Further as we move down the group from Li to Cs, both the atomic size and atomic mass
increase. But the increase in atomic mass more than compensates the bigger atomic size. As a result, the
densities of alkali metals increase from Li to Cs. Potassium is, however, lighter than sodium. This is probably
our

due to an abnormal increase in atomic size of potassium.


ad

10. Flame colouration. All the alkali metals and their salts impart a characteristic colour to the flame.

Metal Li Na K Rb Cs
Y

Colour Crimson Yellow Pale violet Red violet Blue


Re

X/nm 670-8 589-2 766-5 780-0 455-5


nd

Explanation. On heating an alkali metal or its salt especially chloride due to its more volatile nature in
Fi

a flame, the electrons are excited easily to higher energy levels because of absorption of energy. When these
excited electrons return to their ground states, they emit extra energy in form of electromagnetic radiations
which fall in the visible region thereby imparting a characteristic colour to the flame.
The colour actually arises from electronic transitions in short lived species which are formed momentarily
in the flame. The flame is a rich source of electrons, and in case of sodium, the ions are temporarily reduced
to atoms

Na'*’ + e ■> Na

The. yellow flame of sodium commonly called sodium D-line (actually consists of a doublet centred at
589-0 nm and 589-6 nm) arises from the electronic transition 3 s 3 p* in sodium atoms formed in the
flame. It may be noted here that the colours from different elements do not all arise from the same transition,
s-BLOCK ELEMENTS (ALKALI AND ALKALINE EARTH METALS) 10/7

or from the same species. For example, crimson colour of lithium arises from a short lived LiOH species
formed in the flame.

11. Photoelectric effect. Alkali metals (except Li) exhibit photoelectric effect. Photoelectric effect is
the phenomenon of ejection or emission of electrons from the metal surfaces when electromagnetic radiations
are made to .strike against them.
Explanation. Due to low ionization enthalpies, alkali metals, specially potassium and caesium, eject
electrons when exposed to light. It may be noted that lithium which has the highest ionization enthalpy
amongst the alkali metals cannot be used in photoelectric cells because it does not release electrons when
exposed to light.

low
Retain in Memory
Because of strong photoelectric effect, caesium is frequently used in solar cells.

10.5. CHEMICAL PROPERTIES

Some important chemical properties of alkali metals are discussed below :

ee
10.5.1. Reactivity and Electrode Potential

rF
Fr
All the alkali metals arc highly reactive elements since they have a strong tendency to lose the single
valence s'-electron to form unipositive ions having inert gas configuration. This reactivity arises due to their
low ionization enthalpies and high negative values of their standard electrode potentials (Table 10.2). Further,

for
due to their strong tendency to lose the single valence electron, alkali metals also act as strong reducing
agents. In other words, reactivity of an element is measured in terms of its reducing character.
u
Reducing character. Ionization enthalpy is a measure of the tendency of an atom to lose electrons in
s
the gaseous state. Thus, lower the ionization enthalpy, greater is the tendency of an element to lose electrons
ook
Yo
and hence stronger is the reducing character or higher is the reactivity of the element. Since the ionization
enthalpies of alkali metals decrease down the group, therefore, their reducing character or reactivity in the
gaseous .state increasesfrom Li to Cs, i.e., Li < Na < K < Rb < Cs.
eB

However, in aqueous solutions, it has been observed that the reducing character of alkali metals follows
the sequence : Na < K < Rb < Cs < Li. In other words, Li is the strongest while sodium is the least powerful
reducing agent in aqueous solution. This may be explained in terms of electrode potentials (E®) of alkali
r

metals.
ou
ad

Electrode potential is a measure of the tendency of an element to lose electrons in the aqueous solution.
Thus, more negative is the electrode potential, higher is the tendency of the element to lose electrons and
hence stronger is the reducing agent.
Y

Since the standard electrode potentials (E®) of alkali metals become more and more negative as we
move down the group from Na to Cs, therefore, reducing character of these elements increases in the same
Re
nd

order, i.e., Na to Cs. However, standard electrode potential of lithium, is the lowest, i.e., - 3-04 volts. In
other words, lithium is the strongest reducing agent in the aqueous solution. This may be explained as
Fi

follows.

Electrode potential, among other things, depends upon : (/) enthalpy of sublimation, (ii) ionization
enthalpy and (Hi) enthalpy of hydration as shown below :

Sublimation enthalpy Ionization enthalpy


(/) Li (s) ^ U(g) (ii) Li (g) Li'*' (g) + e
(Hi) Li^ (g) + aq 4 Li'*' (aq) + enthalpy of hydration
The sublimation enthalpies of alkali metals are almost similar. Since lithium has the smallest ionic size
among alkali metals, its enthalpy of hydration is the highest. Although ionization enthalpy of lithium is the
highest among alkali metals, it is more than compensated by the large hydration enthalpy released in step
(iii). Thus, among alkali metals, lithium has the most negative standard electrode potential
(E° = -3-04 V) and hence is the strongest reducing agent in the aqueous solution. Since, in general, stronger
10/8 “^uuUeib.'^. New Course Chemistry (XI)S!EIHD

reducing agents have higher reactivity, therefore, Li should also be most reactive in aqueous solution. This is,
however, not correct as discussed below ;

10.5.2. Reactivity towards Water


Alkali metals react with water, liberating H2 and forming their hydroxides. The reaction becomes more
and more violent as we move down the group. Thus, lithium reacts gently, sodium melts on the surface of
water and the molten metal moves around vigorously and may sometimes catch fire, potassium melts and
always catches fire and so are Rb and Cs.
2 Li + 2 H2O ^ 2 LiOH + H2 2 Na + 2 H2O ^ 2 NaOH + H2
2 K + 2 H2O ^ 2 KOH + H2

w
Because of their large negative reduction potentials, alkali metals are better reducing agents than hydrogen.
Hence, they react with compounds containing acidic hydrogen atoms such as water, alcohol and acetylene
liberating hydrogen gas.

F lo
2M + 2H2O 2MOH + H2
(Alkali metals) Alkali metal hydroxide
2M + 2C2HgOH 2 C2H5OM + H2

ee
Ethanol Alkali metal eihoxide

Fr
2M + HC = CH M+ + H2
Acetylene Alkali metal acetylide
Thus, lithium is the least reactive while the reactivity of other alkali metals towards water and other

for
acidic hydrogen containing compounds increases on moving down the group from Na to Cs.
ur
Cause for low reactivity of lithium. The low reactivity of lithium may be explained as follows:
Standard electrode potential (E°) and Gibbs free energy (AG°) are related by the equation,
s
AG'^ = -n FE°
ok
Yo
where n is the number of electrons lost by the metal and F is the faraday constant.
Since E° for the reaction, Li'*' (aq) + e~ > Li (^) has the lowest negative value, i.e. - 3-04 V, therefore,
o

AG° of the reaction has the largest positive value. In other words, this reaction does not occu.r However, the
eB

reverse reaction, Li (s) > Li'*' (aq) + e~ has a large negative value of AG°, so Li liberates more energy than
any other alkali metal when it reacts with H2O. In view of this, at first sight, it appears surprising why Li
reacts with H2O gently, whereas Na, which liberates less energy, reacts more vigorously and the hydrogen
produced catches fire. The explanation for this observation lies in the kinetics (i.e., the rate at which the
r
ou

reaction proceeds) and not in the thermodynamics (i.e., the total amount of energy liberated).
ad

Sodium has a low melting point, and the heat of the reaction is sufficient to make it melt or even
vaporize. The molten metal thus spreads out thereby exposing a larger surface to water. As a result, it reacts
Y

even faster, gets even hotter and catches fire. Lithium, on the other hand, has high melting point. Although the
heat of the reaction is high, it is still not sufficient to melt the metal and the hence the reaction proceeds
gently. Other metals of this group with lower melting points react explosively with water.
Re
nd

From the above discussion, we learn that we should be cautious while predicting the chemical behaviour
of elements on the basis of one single fundamental property of atoms since in many cases it may depend upon
Fi

many other factors.

10.5.3. Reactivity towards Oxygen


The alkali metals tarnish in air due to the formationof an oxide or hydroxide on the surface. When heated in
excess of air, alkali metals form different types of oxides depending upon the nature of the metal, e.g.,
Lithium when heated in oxygen forms lithium monoxide (and some lithium peroxide).
4 Li + O2 ^ 2Li20
Sodium when heated with oxygen at about 575 K forms mainly sodium peroxide (and some sodium
oxide).
575 K . _ _
2 Na + O2 Na202
Sodium peroxide
s-BLOCK ELEMENTS (ALKALI AND ALKALINE EARTH METALS) 10/9

Other alkali metals, i.e., K, Rb and Cs react with oxygen to form superoxides of the general formula.
MO2 where M = K, Rb and Cs, e.g.. Li
K + O2 KO2 > U1(J>
Na b a uj
Potassium superoxide > > »
K
Thus, the reactivity of alkali metals towards oxygen increases down the group.
sp
Rb
Further, the increasing stability of peroxide or superoxide, as the size of the metal
cation increases, is due to the stabilization of larger anions by larger cations through Cs

higher lattice enthalpies.


Explanation. Because of the small size, Li'*’ has a strong po.sitive ifeld around it which attracts the
negative charge so strongly that it does not permit the oxide anion, 0^~ to combine with another oxygen atom

w
toform peroxide ion, o^~ ■ On the other hand, Na'*' ion because of its larger size than Li'*' ion has comparatively
weaker positive field around it which cannot prevent 0-” ion to combine with another oxygen atom to form

F lo
peroxide ion, 0;“.
The larger, K"*", Rb'*' and Cs'*' ions have still weaker positive fields around them which cannot prevent
even peroxide ion, to combine with another oxygen atom to form superoxide, .

ee
1/2 o. O2
Q2- ■>
or ■> 20-

Fr
Oxide Peroxide Superoxide

for
It may be pointed out here that superoxide ion [: O O:) has a th?-ee electron bond, i.e., it has one
unpaired electron which makes it coloured (Li02, Na02 are yellow, KO2 is orange, RbO^ is brown and CSO2
is orange) and paramagnetic. Sodium peroxide is yellow in colour probably due to the presence of a small
r
amount of superoxide in it. The normal oxides of alkali metals are, however, colourless and diamagnetic.
You
s
All alkali metals except Li form ozonides of the formula, MO3
ook

M is) + O3 (g) MO3 (M = Na, K, Rb, Cs)


Metal ozonide
eB

These ozonides are unstable and decompose on standing to form superoxide and oxygen
2 MO3 is) ^ 2 MOt (s) + O2 (g)
10.5.4. Reactivity towards Air and Moisture
our
ad

All the alkali metals on exposure to atmosphere (air and moisture) get converted into oxides, hyd,roxides
and finally to carbonates. Thus alkali metals get tarnished when exposed to air and moisture.
4M + O2 -> 2 M2O ; M2O + H2O ■>2 MOH
dY

2 MOH + CO2 - M')C03 + H2O


Re

It is because of these reactions that alkali metals are stored in inert hydrocarbon solvents like petroleum
ethe,r hexane, benzene and kerosene oil which prevent them from coming in contact with air and moisture.
Fin

10.5.5. Reactivity towards Hydrogen


All the alkali metals react with hydrogen at 673 K (lithium at 1073 K) to form colourless crystalline
ionic hydrides of the general formula. M"* H“ where M stands for the alkali metals
Heat

2M + H2 > 2M-"H- where M = Li, Na, K, Rb or Cs


(i) The order of reactivity of the alkali metals towards hydrogen decreases as we move down the group
from Li to Cs. This is due to the reason that the lattice enthalpies of these hydrides decrease progressively as
the size of the metal cation increases and thus the stability of these hydrides decreases from LiH to CsH.
107'.?K 673 K
2 Li + H2 » 2 LiH 2 Na + H2 ■> 2NaH

Lithium hydride Sodium hydride

I
10/10 T^n^iiUe^’A New Course Chemistry (XI)BEEHD

(it) All the alkali metal hydrides are ionic solids with high melting points.
The ionic character of the hydrides, howeve,r decreases from Li to Cs. The reason being that as the size
of the cation increases, the anion hydride ion can polarize the cation more easily. As a result, the covalent
character increases and hence the ionic character decreases.

(Hi) All the hydrides behave as strong reducing agents and their reducing nature increases down the group,
(iv) Since these hydrides contain the hydride ion (H~), therefore, they liberate hydrogen at the anode on
electroly.'iis.
(v) All these hydrides react with proton donors such as water, alcohols, gaseous ammonia and alkynes
liberating H2 gas.
LiH (s) + H2O (/) ^ LiOH (aq) + H2 (g)

w
NaH (s) + ROH (/) ■»
RONa(i) + ^2(8)
Alcohol Sod. alkoxide

F lo
Heat
NaH (s) + NH3 (g) ^ NaNH2 (s) + H2 (g)
Heat
2 KH (s) + HC = CH (g) KC = CK (s) + 2 H2 (g)
Acetylene Pot. acetylide

ee
Lithium hydride is used as a source of hydrogen for military purposes and for filling metrological

Fr
balloons since it has a low molecular weight and on reacting with H2O, it evolves highest percentage of
hydrogen by weight.

for
10.5.6. Reactivity towards Halogens
Alkali metals react vigorously with halogens to form ionic metal halides of the general formula,
r
X" where M stands for the metal and X for the halogen.
You
2M + X2 ^ 2M+X- where, M = Li, Na, K, Rb or Cs and
s
X = F, Cl, Br or I.
ook

The reactivity of alkali metals towards a particular halogen F2


Li OL
increases as we move down the group from Li to Cs due to a
eB

(- CO
decrease in the ionization enthalpy or increase in the Na ^ ri
r- R Z LU
CO
CI2
^ y UJ CO cc
electropositive character of the metal. For example, potassium K 53
o
UJ
reacts with chlorine more vigorously than sodium. Br2 i= D Q
our
ad

Rb O' X Z < I- <


On the other hand, reactivity of halogens towards a ●2 LU QC LU
5 a: <
particular alkali metal decreases form F2 to I2, i-e., F2 > CI2 > Cs £ S

Br2 > I2
10.5.7. Solutions In Liquid Ammonia
dY
Re

Alt alkali metals dissolve in liquid ammonia (solubility may be as high as5 M) giving highly conducting
deep blue solutions.
Fin

Explanation. These solutions contain ammoniated cations and ammoniated electrons as shown below :
M + (x + y) NH3 ^ (NH3), + e- (NH3)^.
When ordinary light falls on these ammoniated electrons, they get excited to higher energy levels by
absorbing energy corresponding to red region of the visible light. As a result, transmitted light is blue which
imparts blue colour to the solution. Some important properties of blue solutions of alkali metals in liquid
ammonia are as under :
(0 Dilute solutions of alkali metals in liquid ammonia are dark blue in colour but as the concentration
increases above 3 M, the colour changes to copper-bronze and the solutions acquire metallic lustre due to
formation of metal ion clusters.

(ii) The blue coloured solutions are paramagnetic due to the presence of large number of unpaired
electrons but bronze solutions are diamagnetic due to formation of electron clusters in which ammoniated
electrons with opposite spin group together, i.e., 2 e~ (NH3)^ > [T e~ (NH3)j,] [i e~ (NH3)^]
I
s-BLOCK ELEMENTS (ALKALI AND ALKALINE EARTH METALS) 10/11

{in} The solutions of alkali metals in liquid ammonia are good conductors of electricity due to the
presence of ammoniated cations and ammoniated electrons. However, the conductivity decreases as the
concentration increases since the ammoniated metal cations are bound by the free unpaired electrons which
have been described as expanded metals.
(/V) The solutions of alkali metals in liquid ammonia are stronger reducing agents than hydrogen and
hence will react with water to liberate hydrogen. These solutions have also been used for smooth reduction of
aromatic systems to cyclic mono and di-olefins and disubstiluted alkynes to /ran^-alkenes under the name
Birch reduction,

w
(v) In presence of impurities or catalyst such as Fe, the blue coloured solutions decompose to form
metal amides with the liberation of H,.
M"*" {am) + e~ {am) + NH3 (/) - MNH2(«m)+ !/2H2(g)
Na'*' {am) + e~ {am) + NH3 (/) - ^ NaNH2 {am) + 1/2 H-, (g)

Flo
(where 'am' stands for solution in ammonia)
However, under anhydrous conditions and in the absence of catalytic impurities (transition metal ions

e
such as Fe^^, Pt^"^, Zn-'^, etc.), these solutions are stable and can be stored for several days.

re
10.6. GENERAL CHARACTERISTICS OF THE COMPOUNDS OF THE ALKALI METALS

F
All the compounds of alkali metals are predominantly ionic. Some of the general characteristics of these
compounds are discussed below :
ur
or
10.6.1. Oxides and Hydroxides

corresponding hydroxides which are strong alkalies, e.g.,


2Na + 2H2O ^ 2NaOH-hH^
k
Na20 + H2O
sf
All the alkali metals, their oxides, peroxides and superoxides readily dissolve in water to produce

> 2NaOH
Yo
oo
Na202 + 2 H2O ^ 2Na0H + H202 2 KO2 + 2 H2O > 2 KOH + H2O2 + O2
Thus, peroxides and superoxides also act as oxidising agents since they react with H2O forming H20-,
B

and O2 respectively.
The hydroxides of all the alkali metals are white crystalline solids. They are
e

strongest of all bases and readily dissolve in water with the evolution of much heat. A LiOH
I «
number of hydrates of the heavier alkali metal hydroxides, e.g., NaOH. riH^O (where n NaOH
ur

H UJ
cj o ^
= 1,2,3,4,5 and 7) have been prepared from their aqueous solutions but little is known KOH CO Z S
< tu q:
ai
ad

about their structures. ffi (£


Yo

RbOH
CO Z
(/) Basic strength. The basic strength of these hydroxides increases as we move CsOH
down the group from Li to Cs. Thus, NaOH is a stronger alkali than LiOH ; KOH is a
stronger alkali than NaOH and so on.
d

Explanation. The hydroxides of alkali metals behave as .strong bases due to their low ionization
Re
in

enthalpies. Consequently, the M—O bond in M—O—H can easily break giving and OH“ ions.
MOH ^ M+ + OH“ {basic nature)
F

As we move down the group, the ionization enthalpy decreases. As a result, the M—OH bond is more
and more easily cleaved and hence the basic strength increases down the group from LiOH to CsOH.
(//) Solubility and stability. All these hydroxides are highly soluble in water and thermally stable except
lithium hydroxide.
A
2 LiOH LioO + H2O
{Hi) Formation of salts with acids. Alkali metal hydroxides being strongly basic react with all acids
fonning salts.
NaOH + HCI 4 NaCl + H2O 2 NaOH + H2SO4 ^ Na2$04 -K 2H2O
The salts are colourless ionic solids which are soluble in water.
10/12 ‘P’utdeefr'4^ New Course Chemistry (XI)CEIHD

10.6.2. Halides
The alkali metals combine directly with halogens under appropriate conditions forming halides of the
general formula, MX. These halides can also be prepared by the action of aqueous halogen acids (HX) on

ow
metal oxides, hydroxides or carbonates.
M2O + 2 HX ^ 2 MX + H2O ; MOH + HX 4 MX + H2O
M2CO3 + 2 HX ^ 2 MX + CO2 + H2O
(where M = Li, Na, K, Rb or Cs and X = F, Cl, Br or I)
All these halides are colourless, high melting crystalline solids having high negative enthalpies of
formation. However, the AHy? values for fluorides become less and less negative as we move down the group

e
while for chlorides, bromides and iodides, these values become more and more negative (Table 10.3).

re
TABLE 10.3. Standard enthalpies of formation (in kJ mol~^) of alkali metal halides
Element MF MCI MBr MI

F
Frl
Li -612 -398 -350 -271
360 -288
Na

K ou -569
-563
^00
^28 -392 -328

osr
Rb -549 ^23 -389 -329

Cs -531 ^24 -395 -337

kf
From the above table, it is evident that for any given metal, the values decrease in the order: fluoride >
chloride > bromide > iodide. Th\X9,, fluorides are the most stable while iodides are the least stable.
oo
The trends in melting points, boiling points and solubility of alkali metal halides can be understood in
terms of polarization effects, lattice enthalpy and hydration of ions as discussed below :
Y
(fl) Polarization effects—Comparison of ionic and covalent character of alkali metal halides.
B

When a cation approaches an anion, the electron cloud of the


anion is attracted towards the cation and hence gets distorted. This X~
re

effect is called polarization. The power of the cation to polarize


© c£) '
uY

the anion is called its polarizing power and the tendency of the
\
/
anion to get polarized is called its polarizability. The greater the \

polarization produced more is the concentration of electrons between


ad
do

the two atoms thereby decreasing the ionic character or increasing


the covalent character of the bond. POLARIZING POLARIZED
CATION ANION UNPOLARIZED
The covalent character of any compound, in general, depends ANION
in

upon the following factors,


(i) Size of the cation. Smaller the cation, greater is its polarizing power and hence larger is the covalent
Re

character. For example, the covalent character decreases as the size of the cation increases :
F

LiCl > NaCl > KCl > RbCl > CsCl. Thus, LiCl is more covalent than KCl.
(ii) Size of the anion. Larger the anion, greater is its polarizability. This explains why covalent character
of lithium halides is in the order : Lil > LiBr > LiCl > LiF
For the same reason, dipole moment of Lil (6-25 D) is much less than the theoretically expected value
of 11 -5 D if it were 100% ionic.

(Hi) Charge on the ion. Greater the charge on the cation, greater is its polarizing power and hence
larger is the covalent character. That is why the covalent character of some of the halides increases in the
order : Na+ Cl" < Mg^+ (Cr)2 < Ap+ (CL)3.
Similarly, greater the charge on the anion, more easily it gets polarized thereby imparting more covalent
character to the compound formed. For example, covalent character increases in the order : NaCl
< Na^S04 < Na-5P04 as the size of the anion increases and hence larger is the covalent character. Thus, the
covalent character decreases as the size of the anion decreases.
S-BLOCK ELEMENTS (ALKALI AND ALKALINE EARTH METALS) 10/13

(/v) Electronic configuration of the cation. If two cations have the same charge and size, the one with
a pseudo noble gas conifguration, i.e.. having 18 electrons in the outermost shell has greater polarizing
power than a cation with noble gas configuration, i.e., having 8 electrons in the outermost shell. For this
reason, CuCl is more covalent than NaCl although Na'*’ and Cu'*' have same charge, viz. +1 and nearly the
same size, viz. Na"^ (1-02 A) and Cu"^ (0 % A).
These four factors are commonly referred to as Fajan’s rules.
(h) Lattice enthalpies. Lattice enthalpy is defined as the amount of energy required to separate one
mole of solid ionic compound into its gaseous ions. Evidently greater the lattice enthalpy, higher is the
melting point of the alkali metal halide and lower is its (Table 10.4) .solubility in wate.r
TABLE 10.4. Lattice enthalpies, hydration enthalpies (in kJ mol"^), solubilities

w
(in g/100 g H2O)) and melting points (in K) of some alkali met^ halides
Compound Lattice enthalpy Hydration* enthalpy Solubility Melting point

F lo
LiCl -845 -887 63-7 887
NaCl -770 -787 35-7 1084
KCl -703

ee
-711 34-7 1039
RbCl -674 -691 77-0 988

Fr
CsCl -644 -657 162 925
NaF -893 -921 4-22 1261

for
NaCl -770 -787 35-7 1084
NaBr -730 -753 116 1028
ur
Nal -685 -711 184 944
LiF -1005 -1021 0-27 1115
s
Csl -582 -581 440
ook
Yo
(c) Hydration enthalpy. It is the amount of enthalpy released when one mole of gaseous ions combine
with water to form hydrated ions.
eB

M+ (g) + aq M'*’ (aq) + hydration enthalpy


X" (g) + aq - ^ X” ^(aq) + hydration enthalpy
Higher the hydration enthalpy of the ions, greater is the solubility of the salt** in wate.r
r
ad
ou

Further, the extent of hydration depends upon the size of the ion. Smaller the size of the ion, more highly
it is hydrated and hence greater is its hydrated ionic radius and less is its ionic mobility (conductance). Thus,
the order of their radii and mobility are
Y

Ionic radius : Li+ < Na+ < K+ < Rb+ < Cs+
(m pm) 76 102 138 152 167
Re
nd

Hydration number*** : 25-3 16-6 10-5 100 9-9


Hydrated radius : Li+ > Na+ > K+ > Rb+ > Cs+
Fi

{in pm) 340 276 232 228 228


Ionic mobility Li+ < Na+ < < Rb+ < Cs+
(in ohnr^ cm^ mot^):
{at inifnite dilution) 33-5 43-5 64-5 67-5 68

*Suni of the hydration enthalpies of cations and anions. The hydration enthalpies of the alkali metal cations
and halides (in kJ mor') respectively are : Li+ (-506), Na+ (-406), K+ (-330), Rb+ (-310), Cs+ (-276) and
(-515), Cl- (-381), Br- (-347), I" (-305).
**The solubility of a salt in water depends upon : (/) Lattice enthalpy and (i7) Hydration enthalpy.
In general, if hydration energy > lattice energy, .salt dissolves and if hydration enthalpy < lattice enthalpy,
salt does not dissolve.

***The hydration number is the average number of water molecules associated with the metal ion.
10/14 “PncuUe^ U New Course Chemistry (XI)
In the light of above arguments, let us now explain the melting points of alkali metal halides and their
solubility in water or organic solvents. Thus,
(0 A delicate balance between lattice enthalpy and hydration enthalpy determines the ultimate solubility
of a compound in water. For example, LiF (0-27 g/lOO g H2O) is almost insoluble in water due to its high
lattice energy (-1005 kJ mor^). The low solubility of Csl (44 g/100 g H2O), on the other hand, is due to
smaller hydration energy of the two ions [- 276 (Cs*) - 305 (D = - 581 kJ mol~^].

ow
(ii) The solubility of most of the alkali metal halides except those offluorides decreases on descending
the group since the decrease in hydration enthalpy is more than the corresponding decrease in the lattice
enthalpy. For example, difference in lattice enthalpy between NaCl and KCl is 67 kJ mor* but difference in
hydration enthalpy of Na'*’ and K'’’ ion is 76 kJ mol~^ Thus, KCl is less soluble in H2O than NaCl.
(Hi) Becau.se of the small size and higher electronegativity, lithium halides except LiF are predominantly
covalent and hence are soluble in organic solvents such as alcohol, acetone, ethyl acetate ; LiCl is also
soluble in pyridine. In contrast, NaCl being ionic is insoluble in organic solvents.

e
(iv) Due to high hydration enthalpy of Li"^ ion, lithium halides are soluble in water except LiF which is

re
rFl
sparingly soluble due to its high lattice enthalpy. However, as we move down the group, the solubility of
alkali metal fluorides increases regularly as we move from LiF to CsF since the decrease in lattice enthalpy

F
more than compensates the decrease in hydration enthalpy,
(v) For the same alkali metal, the melting points decrease in the order .-fluoride > chloride > bromide
> iodide.

r
Explanation. For the same alkali metal ion, the lattice energies decrease as the size of the halide ion
ou
fo
increases. For example, lattice enthalpies of NaF, NaCl, NaBr and Nal are 893, 770, 730 and 685 kJ
mol”^ As the lattice enthalpies decrease, energy required to break the lattice (melting point) decreases and
hence the melting points of sodium halides decrease from NaF (1261 K)
(1028 K) > Nal (944 K). ks > NaCl (1084 K) > NaBr
oo
(vi) For the same halide ion, the melting points of lithium halides are lower than those of the
corresponding sodium halides and thereafter they decrease as we move down the group from Na to Cs.
Y
Explanation. The low melting point of LiCl (887 K) as compared to that of NaCl (1084 K) is probably
eB

because LiCl is covalent in nature while NaCl is ionic (because Li atom is smaller in size than Na atom).
Thereafter, the melting points decrease as we move from NaCl (1084 K) ^ KCl (1039 K) »RbCl (988 K)
> CsCl (925 K) because the lattice enthalpies decrease as the size of the alkali metal atom increases.
r
ou

10.6.3. Salts of Oxoacids


Y
ad

Since the alkali metals are highly electropositive, therefore, their hydroxides are very strong bases and
hence they form salts with all oxoacids. Oxoacids are those in which the acidic proton is on a hydroxyl
group. Common examples are H2CO3, H3PO4, H2SO4, HNO3, HNO2, etc. They are generally soluble in
water and stable towards heat. Here, we shall discuss only carbonates and bicarbonates.
d

Carbonates. The carbonates (M2CO3) of alkali metals are quite stable towards heat. Li2C03, however,
in
Re

is considerably less stable and decomposes readily on heating. This is due to the reason that small lithium
cation polarizes the oxygen atom of the nearby larger carbonate ion. As a result, the C—O bond weakens and
F

Li—o bond strengthens thereby favouring the formation of Li70 and CO2 from Li2C03. In other words, it is
the higher lattice energy of L12O over that ofLi2CO^ which favours the decomposition of Li2CO^. Further, as
the size of the cation increases from Na'*' to Cs"^, the lattice energy of their corresponding oxides decreases
and hence the stability of their carbonates increases from Na2C03 to CS2CO3.

U2CO3 —> Li^ O + CO2


Bicarbonates. Being strongly basic, alkali metals also form solid bicarbonates. No other metals fonn
solid bicarbonates, though NH4HCO3 also exists as a solid. Lithium, however, does not form solid bicarbonate
though it does exist in solution. All the bicarbonates on gentle heating undergo decomposition to form
carbonates with the evolution of CO->.
A
2 MHCO3 M2CO3 + CO2 + H2O
S-BLOCK ELEMENTS (ALKALI AND ALKALINE EARTH METALS) 10/15

As the basic character of metal hydroxide increases from LiOH to CsOH or the electropositive character
of the metal increases from Li to Cs, the stability of carbonates and bicarbonates increases.
All the carbonates and bicarbonates are soluble in water and their solubilities increase rapidly on
descending the group. This is due to the reason that their lattice energies decrease more rapidly than their
hydration energies on moving down the group.

Curiosity Questions
Q. 1. Alkali metals are usually stored In hydrocarbon solvents to protect them from air and
moisture but lithium cannot. Why ?

w
Ans. Lithium cannot be stored in kerosene oil since it floats over kerosene and other hydrocarbon
solvents because of its very low density. Therefore, lithium is usually kept wrapped in paraffin
wax.

Q. 2. Are sodium cyanide and potassium cyanide equally poisonous ?

F lo
Ans. No. KCN is more poisonous than NaCN.
i

ee
10.7. ANOMALOUS BEHAVIOUR OF LITHIUM

Fr
Lithium— the first element of group I (alkali metals) differs from rest of the elements of this group in
many respects contrary to the expectations. This anomalous behaviour of lithium is due to the following
reasons : (a) very small size of lithium atom and its ion.
ib) higher polarizing power of Lt (i.e. charge/size ratio) resulting in increased covalent character of

for
ur
its compounds which is responsible for their solubility in organic solvents.
(c) comparatively high ionization enthalpy and low electropositive character of lithium as compared to
other alkali metals.
ks
(r/) non-availability ofd-orbitals in its valence shell,
Yo
(e) .strong intermetallic bonding.
oo
Some of the properties in which lithium differs from the other members of its group illustrating its
anomalous behaviour are as follows : (;) Lithium is harder while all other alkali metals are soft.
eB

iii) The melting and boiling points of lithium are comparitively high.
(Hi) Lithium forms monoxide with oxygen while other alkali metals form peroxides (M20^) as well as
superoxides (MO2).
r

(iv) When burnt in air or nitrogen, lithium forms lithium nitride due to its high lattice energy. The other
ou
ad

alkali metals, however, do not fonn their corresponding nitrides because their lattice energies are not only
low but decrease further as their sizes increase from Na to Cs.
Y

6 Li + N2 > 2 Li3N
(v) LiOH is a weak base while the hydroxides of other alkali metals are strong bases,
nd
Re

(v/) The hydride of lithium (i.e., LiH) is more stable than hydrides of other alkali metals.
(vii) Lithium hydroxide on healing decompo.ses to form lithium oxide while other alkali metal hydroxides
do not
Fi

2 LiOH > Li20 + H2O


(viii) Lithium carbonate on heating decomposes to give CO2 while other alkali metal carbonates do not
Li2C03 ) Li20 + CO-5
(«r) Lithium bicarbonate does not form a solid though it exists in solution while all other alkali metals
form solid bicarbonates.

(x^) Lithium does not react with eihyne (acetylene) to form lithium ethynide (acetylide) while all other
alkali metals react to form the corresponding alkynides.
Liq. NH3
2Na + HCsCH
196 K Na^-CsC-Na"- + H2
Acetylene Disodium acetylide
10/16 'PmuUe^'’A New Course Chemistry ('xnpZSTTI

However, when Li is heated with carbon, it forms dilithium acetylide (or lithium carbide) while other
alkali metals do not react with carbon directly
2Li + 2C ^ Li-"-C = C"Li-"
Dilithium acetylide

ow
(xi) Lithium nitrate on healing decomposes to give Li20, NO^ and O2 while other alkali metal nitrates
decompose to give the corresponding nitrite and oxygen
A

4 LiN03 ^ 2 Li20 + 4 NO2 + O2


A
2NaN03 ^ 2NaN02 + 02
(x/i) The oxide, hydroxide, carbonate, phosphate, oxalate and fluoride of lithium are sparingly soluble

e
in water whereas the corresponding salts of other alkali metals are soluble in water

re
(xiii) Becauseof covalentcharacter, halides of lithium (like LiCl) are soluble in organic solvents such as
alcohol, acetone, pyridine, etc. while those of other alkali metals are insoluble,

Frl
(x/v) LiCl is deliquescent and crystallizes as a hydrate, LiC1.2H20, whereas other alkali metal chlorides

F
do not fonn hydrates,
(xv) Lithium ion, being very small in size, is very strongly hydrated in aqueous solution as compared to
other alkali metals ions that is why effective size of Li'*' ion in aqueous solution is the largest and hence its
ou
r
mobility is the lowest, i.e., Li"^ < Na'*’ < < Rb'*' < Cs'*'.

so
(xvi) Li2S04 is the only alkali metal sulphate which does not form an alum.
(xvi7) Lithium when heated with NH3 forms lithium imide, Li2NH while all other alkali metals form

kf
amides, MNH2.
oo
10.8. DIAGONAL RELATIONSHIP OF LITHIUM WITH MAGNESIUM

Lithium resembles magnesium mainly due to the similarity in sizes of their atoms (Li = 152 pm.
Y
Mg = 160 pm) and ions ( Li'*’ = 76 pm, Mg-'^ = 72 pm). The main points of similarity are :
B

(/) Both have almost similar electronegativities (Li = 0-98 and Mg = 1-2).
(/(■) Both Li and Mg are quite hard. They are harder and lighter than other elements in their respective groups.
re

(Hi) Both LiOH and Mg(OH)2 are weak bases,


oY

(/v) Both form ionic nitrides when healed in an atmosphere of nitrogen.


u

A A
ad

6 Li + N2 2 U3N 3 Mg + N2 ■> Mg3N2


(v) The hydroxides of both Li and Mg decompose upon heating.
d

A
2 LiOH ■> Li20 + H^O Mg(OH)2 MgO + H2O
in

(vi) Both lithium and magnesium combine with oxygen to form monooxides while other members of
Re

their respective groups form peroxides and superoxides.


A A
F

4 Li + O2 2 Li20 2 Mg + O2 2 MgO
The Li^O and MgO thus formed do not combine with excess O2 to form peroxides and superoxides.
(vii) The carbonates of these metals decompose on heating to the corresponding oxide with the evolution
of CO2
A
Li2C03 ^ Li20 + CO2 MgCOj MgO + COj
(viii) Both Lithium and magnesium do not form solid bicarbonates.
(ix) Both lithium and magnesium nitrates decompose on heating producing nitrogen dioxide.
A
4 LiN03 2 Li-)0 + 4 NO-> + O2
A
2 Mg(N03)2 ●> 2 MgO + 4 NO2 + O2
S-BLOCK ELEMENTS (ALKALI AND ALKALINE EARTH METALS) 10/17

(jc) The hydroxides, carbonates, oxalates, phosphates and fluorides of both lithium and magnesium are
sparingly soluble in water.
(xi) Because of covalent character, LiCl and MgCl2 are soluble in ethanol.
(xii) Both lithium perchlorate and magnesium perchlorate are highly soluble in ethanol.
(a7/'0 LiCl and MgCl2 are deliquescent and crystallise from aqueous solution as hydrates, LiCl.2 H2O
and MgCl2.6 H^O.

low
1. Monoxides, peroxides and superoxides of alkali metals. All the five alkali metals can be induced to
form normal oxides (i.e., monoxides), peroxides and superoxides by dissolving the metal in liquid ammonia
and bubbling in an appropriate amount of oxygen.
2. Crystal structures of monoxides of alkali metals. Except CS2O which has anti- CdCl2 layer structure,
all other monoxides, i.e., Li^O, Na20, K->0 and Rb20 have anti-fluorite structures.
3. Potassium superoxide (KO2) is used as a source of oxygen in submarines, space shuttles and in emergency
breathing apparatus such as oxygen masks. Such masks are used in rescue work in mines and in other

ee
areas where the air is so deficient in oxygen that an artificial atmosphere must be generated. The moisture

rF
of the breath reacts with superoxide to liberate oxygen, and at the same time the potassium hydroxide

Fr
formed removes carbon dioxide as it is exhaled thereby allowing the atmosphere in the mask to be
continuously regenerated.
4 KO2 (5) + 2 H2O (/) ■> 4 KOH (aq) + 3 O2 ig)

for
2KO2 (s) + 2HoO (0 ^ 2 KOH (oq) + H2O2 (aq) + O, (g)
KOH (oq)+ CO2 ig) ^KHC03(,5)
u
K02also combines directly with CO2 forming K2CO3 and with CO-, and moisture together forming KHCO3.
4 KO2 + 2 CO2 ^2K2C03 + 3 02 ;
ks
4 KO^ + 4 COo + 2 H,0 ^ 4 KHCO3 + 3 O2
Yo
4. Potassium ozonide (KO3) is formed when ozone is passed through KOH.
oo
2 KOH + 50 3 ■> 2 KO3 + 5 O2 + H2O
It is an orange coloured solid and contains the paramagnetic O^ ion.
eB

On standing, it slowly decomposes to give O2 and KO^ but on hydrolysis it directly gives KOH and O2.
Standing
2KO3 2 KO2 + O2 ; 4KO3 + 2H2O + 4 KOH + 5 O2
r

5.
Lithium hydroxide (LiOH) is used to remove CO2 from exhaled air in confined quarters like submarines
ou
ad

and space vehicles.


6. The alkali metals react with halogens and interhalogen compounds forming ionic polyhalide compounds.
Y

Kl + L ^ K [I3] KBr + TCI ^ K [BrICI]


KF + Brp3 ^ K [BrF^J
7. The solution of alkali metals such as Li, Na or K in liquid ammonia is used for reduction of aromatic
nd
Re

systems to cyclic mono and di-olefins and disubstituted alkynes to /ranr-alkenes under the name Birch
reduction.
Fi

8. Cs is the most electropositive element due to its lowest ionization enthalpy.


9. Lithium cannot be used in making photoelectric cells because out of all the alkali metals, it has highest
ionization enthalpy and cannot emit electrons when exposed to light.
10. The compounds of alkali metals are colourless (unless the anion is coloured like permanganate or
dichromate) and diamagnetic. This is because they have noble gas configuration with no unpaired electron.
11. The peroxides of alkali metals are colourless and diamagnetic while superoxides are paramagnetic and
coloured.
12. All alkali metals exist as body-centred cubic lattice with a coordination number of 8.
13. Due to small size, lithium does not form alums.
14. Alkali metals combine with mercury to form compounds known as amalgams. This reaction is highly
exothermic.
10/18 New Course Chemistry (XI)EEHI]

10.9. EXTRACTION OF ALKALI METALS

10.9.1. Difficulties encountered during Extraction of Alkali Metals


Alkali metals, cannot be extracted from their ores by the usual methods of extraction of metals because
of the following difficulties :
(/) Alkali metals are strong reducing agents and hence cannot be extracted by reduction of their oxides
or chlorides,

{ii) Alkali metals being highly electropositive cannot be displacedfrom the aqueous solutions of their

w
salts by other metals.
(Hi) Alkali metals cannot be isolated by electrolysis of the aqueous solution of their salts since hydrogen
is liberated at the cathode instead of the alkali metal because the standard electrode potentials of alkali
metals are much lower (Li = - 3-04 ,V Na = - 2-71 V) than that ofH20 (~ 0-83 V). As a result, H2O is reduced

o
in preference to alkali metals. However, by using mercury as cathode, the alkali metals can be deposited at
the cathode but the alkali metal so deposited readily combines with mercury to form an amalgam from which

e
its recovery is very difficult.

Fl
re
Therefore, in view of the above difficulties, the only successful method is the electrolysis of their molten
(fused) salts usually chlorides.

F
But the melting points of these chlorides are very high which are normally difficult to attain. This
difficulty can, however, be overcome to some extent by lowering the melting points of these chlorides by the
ur
addition of suitable salts such as CaCl^, KF, etc.

r
fo
10.9.2. Extraction of Lithium

Lithium is prepared by electrolysis of a fused mixture of dry lithium chloride (55%) and potassium
chloride (45%) at 723 K. ks
Yo
Potassium chloride is added to increase the conductivity of lithium chloride and also to lower the fusion
oo
temperature. The cell is operated at a temperature of about 723 K and current voltage of 8-9 volts.
As a result of electrolysis, the following reactions take place :
B

LiCl U+ + C1-
At cathode : Li'*' + e ^ Li
e

At anode : 2C\--le- ^Cl2


ur

The metal thus obtained is 99% pure and is stored by keeping it wrapped in paraffin wax. It may be
noted here that lithium being the lightest metal known (density = 0-534 g cm~^) cannot be stored in kerosene
ad

oil since it floats on the surface.


Yo

10.9.3. Uses of Lithium


(/■) It is used in the manufacture of alloys : (a) lithium-lead alloy (0 05% Li) or white metal which is
d

used for making toughened bearings for motor engines and sheets for cables.
Re

(b) Lithium-aluminiumalloy has great tensile strength and elasticity like that of mild steel. It is used for
in

aircraft construction,

(c) Lithium-magnesium alloy (with 14% Li) is extremely tough and corrosion-resistant which is used
F

for armour plate and aerospace components.


(/7) It is used for producing thermonuclear energy required for propelling rockets and guided missiles.
(Hi) Lithium is used to make both primary and secondary batteries.
(iv) Lithium is used as a getter or scavenger since it combines readily with oxygen and nitrogen.
Thus, it is used for removing last traces of oxygen and nitrogen during refining of metals such as copper and
nickel.

(v) Lithium carbonate is used in making a special variety of glass which is very strong and is weather¬
proof.
(w) Lithium chloride is used in air-conditioning plants to regulate the humidity. It is also used in
Ni-Fe accumulators.
s-BLOCK ELEMENTS (ALKALI AND ALKALINE EARTH METALS) 10/19

(vii) Lithium bromide is used in medicine as sedative.


(viii) Lithium stearate is used in making automobile greases.
(ix) Lithium bicarbonate and lithium salicylate have been used for treatment of rheumatism since the
resulting lithium urate is soluble in water.
(x) Lithium hydride is used as source of hydrogen for melerological purposes and for filling of balloons.
(xi) Lithium hydroxide is used for removing CO2 from exhaled air in closed quarters like submarines
and space vehicles.
(xii) Lithium aluminium hydride (LiAlH^) is used as a reducing agent in synthetic organic chemistry.
10.9.4. Extraction of Sodium

w
Sodium is extracted by the electrolysis of fused sodium chloride by a process called Down’s process.
Difficulties encountered. Although sodium chloride is very cheap and is abundantly available yet the
development of Down’s process for the extraction of sodium from fused sodium chloride was delayed becau.se

F lo
of the following reasons :
(/) Sodium chloride melts at 1084 K and it is difficult to attain and maintain this high temperature.
(h) Sodium boils at about 1156 K and hence at the temperatureofelectrolysis, the metal liberatedwill
vapori.se.

ee
(Hi) Molten sodium forms a metallic fog (colloidal .solution) with fused sodium chloride,

Fr
(iv) The products of electrolysis, sodium and
FIGURE 10.2
chlorine, corrode the material of the cell at this
high temperature. CHLORINE

for
In 1924. J.C. Down overcame all these
INLET FOR FRESH SALT DOME
difficulties. He observed that the addition of calcium
r
chloride and potassium fluoride lower the melting KEROSENE
'IL
You
point of sodium chloride to 850 - 875 K.
s
INSULATION
Sodium is now obtained by electrolysis of a / ^MOLTEN
ook

7 SODIUM
fused eutectic mixture of sodium chloride (40%) FUSED^
^ METAL
and calcium chloride (60%) in a Down’s cell (Fig. :f^aCl:+:
eB

-:e^CI5-
10.2) at 873 K using graphite anode and iron fl 0 » 0 vmrfvts
cathode. HOOD
T
IRON

As a result of electrolysis, sodium is liberated FOR- CATHODE


CATHODE
at the cathode and CI2 is evolved at the anode
our
ad

+
GRAPHITE
Dissociation
ANODE
NaCl ^ Na-" + Cr
At cathode : Na"^ + e Na
Down's cell for extraction of Sodium
At anode : cr Cl + e
Y

ci + ci ●>Cl2
Re
d

SUPPLEMENT YOUR
Fin

KNOWLEDGE FOR COMPETITIONS

Extraction of Potassium. Since K metal is more soluble in KCl (than Na metal is soluble in NaCl),
potassium cannot be obtained by electrolysis of a fused solution of KCl. It can, however, be obtained by
the following two methods :
1. By electrolysis of fused KOH. KOH >K++OH
At cathode : K'*' + e" ■> K
At anode: 4 0H- O2 + 2 H2 O + 4 c
2. Reduction of KCl with sodium.

Potassium can also be obtained by reduction of KCl with Na vapours at about 1125 K in a large fractionating
column.
KCl + Na ^ NaCl + K

Potassium thus obtained is about 99-5% pure

t-
10/20 “^uieCec^ j New Course Chemistry (XI) iv»lin

10.9.5. Uses of Sodium


(0 Sodium is used as a reducing ageni in the extraction of boron and silicon.
(//) In the laboratory, it is employed as a reducing agent in form of sodium amalgam and as a reagent in
Wurtz reaction and in the synthesis of many organic compounds.
{Hi) It is also used in Lassaigne’s test for the detection of N. S and halogens in organic compounds.
(;v) About 60% of world production of sodium is used to make tetraethyllead, Pb(C2H5)4 or PbEt4 and
tetramethyllead, Pb(CH3)4 or PbMe4 which are used as ami-knocking agents for gasoline. The use of ^ese
orgatiolead compounds has created serious environmental pollution due to release of poisonous lead compounds
into the atmosphere. Therefore, their use as gasoline additives has now decreased significantly and more

w
vehicles now run on high octane lead free petrol,
(v) Liquid Na or its alloy with potassium is used as a coolant in fast breeder nuclear reactors,
(vt) It is used in the manufacture of number of chemicals such as Na20^, NaCN and NaNH2-

F lo
(vii) Sodium is used in sodium vapour lamps.
(viii) Sodium is largely used in industry for the production of artificial rubber, dyes, drugs, etc.
(/x) Because of its lightness and high thermal conductivity, it is used for filling exhaust valves of aeroplane

ee
engines.

Fr
10.9.6. Uses of other Alkali Metals
Potassium has a vital role in biological systems. KC! is used as a fertilizer while KOH is used in the
manufacture of soaps. It is also used as an excellent absorbent for CO-,.

for
Caesium is used making photoelectric cells (solar cells).
ur
10.10. SOME IMPORTANT COMPOUNDS OF SODIUM

Some industrially important compounds of sodium are sodium carbonate, sodium chloride, sodium
s
ook
hydroxide and sodium hydrogen carbonate. The large scale production of these compounds and their uses are
Yo
described below :
eB

10.10.1. Sodium Carbonate (Washing Soda), NajCOj.lO HjO


Sodium carbonate commonly known as washing soda is manufactured by Solvay-ammonia process.
(a) Principle. The process is based upon the principle that when carbon dioxide is passed through brine
our

(sodium chloride) solution saturated with ammonia, the following ions are present in the solution.
ad

Na+, NH+ , Cl- and HCO“


Out of the various compounds possible, sodium bicarbonate being sparingly soluble, crystallizes out.
This is finally calcined to form sodium carbonate,
Y

(b) Theory. When carbon dioxide is passed through brine solution saturated with ammonia, sodium
Re

bicarbonate is formed :
nd

NH3 + H2O ^ NH3.H2O ^ NHj + OH-


Fi

Na+ + Cl" + NH; + OH- + CO2 ■>


Na^ + HCO3 + NHJ + Cr
Na+ + HCOJ ^ NaHC03 (as precipitate)
NH^ + CL > NH4CI (in solution) ...(0
Because of the common ion effect due to the presence of Na'*’ ions of sodium chloride, sodium bicarbonate
precipitates out. It is filtered and then ignited to get sodium carbonate.
heat

2 NaHC03 Na-,C03 + CO-, + H-,0 ...(»)


Sodium carbonate as obtained above is contaminated with ammonium salts. It is purified by passing
CO2 through its aqueous solution. The sodium bicarbonate thus formed is filtered and heated to give pure
sodium carbonate. Recrystallization from water produces washing soda, Na2CO3.10H2O.
s-BLOCK ELEMENTS (ALKALI AND ALKALINE EARTH METALS) 10/21

Na2C03 + CO2 + H2O -> 2 NaHC03


2 NaHC03 ^ Na2C03 + H2O + CO2
Function of ammonia. CO2 is only slightly soluble in water. As a result, the concentration of H2CO3 is
very low. Further since H2CO3 is a very weak acid, the concentration of HCO^ in the reaction mixture is
extremely low.
CO2 + H2O f ± H2CO3 ...(/)

H2CO3 ^ ± + HCO3 ...(//)

w
To shift the equilibrium in the forward direction, a base such a.s NH3 is used which combines with H'*'
ion to form NH^ ion.
H-" + NH3 ^ NHJ

Flo
Thus the function of NH3 is to produce a sufficient amount of HCO3 ion which enables sparingly

e
soluble NaHC03 to precipitate out of the reaction mixture,

re
(c) Details of the process. The manufacturing details of the Solvay ammonia process are described
below :

rF
(i) Ammoniation tower. A FIGURE 10.3
saturated solution of brine (30%) AMMONIA + LITTLE CARBON DIOXIDE
is introduced into the top of the iron
ur
fo
tower while gaseous ammonia from CO2

ammonia recovery tower is passed 1 X


UJ LIMEKILN
from below (Fig. 10.3). As a result LU
BRINE z
ks 5
O
of this counter current process, X
1-1
\ X
Yo
WATER LiJ
CO
brine gets saturated with ammonia r
z
^ or
oo
G \
O
L_i
IP
(0
LU
while calcium and magnesium O
\ <
O

H X < o>
present as impurities in commercial < HI L_i z X
B

o LU
z5 o 1>
sodium chloride get precipitated as
\
o o 5 L-1
CQ
X
Ca(OH)2 < o
their corresponding insoluble < o
e

/ \

carbonates. <
< L_l o
)^ LU
X
STEAM ^
ur

2 NH3 + CO2 + H2O ■>


NH4CI + A LITTLE NH4HCO3 clci2
im FILTER
ad

(NH4)2C03
NaHCOst' OR IGNITION
FILTER
Yo

CaCl2 + (NH4)2C03
2 NH4CI + CaC03 i Solvay-Ammonia process for the manufacture of
MgCl2 + (NH4)2C03 sodium carbonate

2 NH4CI + MgC03 i
d
Re

The ammoniated brine thus obtained is passed through filters to remove precipitated calcium and
n

magnesium carbonates and then passed through the carbonation tower,


Fi

(ii) Carbonation tower. It is made up of iron and is fitted with a number of perforated horizontal
partitions. The clear ammoniated brine solution is made to trickle down from the top of the tower while
carbon dioxide from lime kiln is introduced from the bottom at a pressure of 1-2 atmospheres. Carbon
dioxide rises through the small holes and reacts with ammoniated brine to form sodium bicarbonate.
NaCl + NH3 + CO2 + H2O > NaHC03 (s) + NH4CI (aq)
Throughout the process, a temperature of 300-31 OK is maintained.
(Hi) Filtration. The solution flowing out of the carbonation tower contains tiny crystals of sodium
bicarbonate. These are filtered by passing through vacuum filters,
(iv) Calcination of sodium bicarbonates. The sodium bicarbonate obtained above is heated strongly
or calcined in a kiln when sodium carbonate is formed.

2 NaHC03 ■> Na^C03 + CO2 + H2O

f
10/22 ^●tcidcefr'^ New Course Chemistry (XI)BS

(v) Ammonia recovery tower. The filtrate from the carbonatioii tower contains ammonium chloride
and a little ammonium bicarbonate. It is made to flow down the ammonia recovery tower while a current of
steam is made to go up. Milk of lime is introduced at a point little above the middle of the tower. Ammonium
bicarbonate is decomposed by steam and ammonium chloride by milk of lime as follows :
Heat
NH4HCO3 - NH3 + CO2 + H2O
2NH4CI + Ca(OH)2 ^2NH3 + CaCl2 + 2H20

ow
This recovered ammonia mixed with little carbon dioxide is used for saturation of brine in the ammoniation
tower as discussed above. Calcium chloride is obtained as a by product,

(vi) Lime kiln. Here, carbon dioxide is generated by heating limestone to about 1273 K
1273 K
CaC03 > Ca0 + C02
The carbon dioxide is brought to the carbonation tower while lime is slaked with water in a tank known

e
as slaker to form milk of lime which is pumped to the ammonia recovery tower.

Fl
re
Raw Materials. The solvay-ammonia process is very economical since except NaCl and CaC03 all
other raw materials used are not consumed in the process. For example,

F
(a) Quick lime (CaO) left in the lime kiln and FIGURE 10.4
ur
ammonium chloride (NH4CI) left in the mother liquor
CaCOg

r
after removal of NaHC03 are reacted to produce

fo
NH3. Therefore, except for small losses, no additional
NH3 is required.
(b) CO2 produced during conversion of ks
Yo
NaHC03 to Na2C03 is reintroduced in the
oo
carbonation lower. H2O
Thus, NaCl and CaC03 are the only raw Ca(0|jl)21 |jsial4|j:| <
eB

materials used which are quite cheap. At the same


time, CaCl2 is produced as a bye product which, of Ca(OH)2 + NH4CI
course, has no large scale industrial applications. > NH3
Thus, solvay ammonia process is economical,
ur

Flow-sheet for Solvay-Anunonia prpc^


continuous and self-contained as shown in Fig. 10.4.
ad

Why potassium carbonate cannot be prepared by Solvay process ? Potassium carbonate cannot be
Yo

prepared by the Solvay-ammonia process. This is due to the reason that unlike sodium bicarbonate (NaHC03)
which is sparingly soluble in water, potassium bicarbonate (KHCO3) is fairly soluble in water. Thus, when
CO2 is passed through ammoniated brine, NaHC03 gets precipitated while KHCO3 precipitated
d

when CO2 is passed through an ammoniated solution of potassium chloride. Hence, potassium carbonate
Re
in

cannot be prepared by the Solvay-ammonia process.


Properties, (i) Sodium carbonate is a white crystalline solid which crystallizes as decahydrate
F

(Na2CO3.10H2O).
(«) It is readily soluble in water.
(Hi) Action of heat On heating below 373 K, it loses 9 molecules of water of crystallization to form
monohydrate (Na2C03.H20). On heating above 373 K, the monohydrate changes to an anhydrous white
powder called soda ash but does not decompose further.
Below 373 K
Na2CO3.10H2O > Na2C03.H20 +9H2O
Above 373 K
Na2C03.H20 > Na2C03 + H2O
(iv) Hydrolysis. Being a salt of a strong base (NaOH) and weak acid (H2CO3), when dissolved in water,
sodium carbonate undergoes hydrolysis to form an alkaline solution.
s-BLOCK ELEMENTS (ALKALI AND ALKALINE EARTH METALS) 10/23

Na2C03 + 2H2O H2CO3 + 2NaOH

{Weakly ionizeil) (Completely ionized)

or
COj~ + H2O HCO3 +OH-
(y) Action of acids. It reacts with dil. mineral acids evolving CO2 gas.
Na2C03 + 2 HCl > 2 NaCI + H2O + CO2 T
(W) Reaction with milk of lime. With hot milk of lime, Ca(OH)2, it reacts to form sodium hydroxide.
Ca(OH)2 + Na2C03 > CaC03 i + 2NaOH
Uses. (/) It is used for softening of hard water, laundary and cleaning.
(ii) It is used in paints and dyes.
(i/7) It is used in the manufacture of soap, glass, borax, caustic soda, sodium phosphate, etc.
(iv) A mixture of Na2C03 and K2CO3 is used as a fusion mixture.

F low
(v) It is used in paper and textile industry and also in petroleum refining and metal refining,
(vi) It is used as a reagent in the laboratory.
10.10.2. Sodium Chloride (Common Salt), NaCI
Preparation. The most abundant source of sodium chloride is sea water which contains 2-l-2-9% by
mass. In tropical countries like India, common salt is generally obtained by evaporation of sea water. In
India, about 50 lakh tons of salt are produced annually by solar evaporation. Crude sodium chloride obtained
by crystallization of brine solution contains impurities of sodium sulphate (Na2S04), calcium sulphate

e
for Fre
(CaS04>, calcium chloride (CaCl2) and magnesium chloride (MgCl2). Since MgCl2 and CaCl2 are
deliquescent (absorb moisture easily from the atmosphere), therefore, impure common salt gets wet in rainy
season. To obtain pure sodium chloride, the crude salt is dissolved in minimum amount of water and filtered to
remove insoluble impurities. The solution is then saturated with hydrogen chloride (HCl) gas when crystals of pure
sodium chloride separate out. The calcium and magnesium chloride being more soluble remain in the solution.
Properties. Pure sodium chloride is not hygroscopic. It melts at 1081 K. Its solubility is 36-0 g per
Your
100 g of H2O at 273 K. However, the solubility does not increase appreciably with rise in temperature.
eBo ks

Uses. (0 It is used as a common salt or table salt for domestic purposes.


(») It is used in the preparation of Na2C03, NaOH and Na202.
Retain in Memory
ad
our

To prevent table salt from absorbing moisture and to make it flow freely in rainy season, Ca3(P04)2
or Ca(H2P04)2 is added to the table salt. The Ca3(P04)2 or Ca(H2P04)2 prevents moisture absorbing
power of other components (MgCl2, CaCl2, MgS04, CaS04, etc.) present in table salt and thus
keeps table salt dry. Further, both Ca and P are needed by the human biological system and hence do
Re

not cause any side effect.


Find Y

10.10.3. Sodium Hydroxide (Caustic Soda), NaOH


This is prepared commercially by the electrolysis of an aqueous solution of sodium chloride using
mercury cathode and graphite anode. The electrolysis is carried out in a specially designed vessel called
Castner and Kellner's cell or Mercury Cathode cell (Fig. 10.5).
It consists of a large rectangular iron tank divided into three compartments by slate partitions which do
not touch the bottom of the tank. They are suspended in mercury placed on the base of the tank. The anodes
are made of stout graphite rods and cathodes are made of iron rods. The mercury serves as an intermediate
electrode, as a cathode in the outer compartment and as an anode in the central compartment by induction.
The brine solution is taken in the side compartments and a very dilute solution of NaOH is taken in the
central compartment. On passing electricity, the electrolysis takes place in the outer and central compartments
as indicated below :
10/24 'P’uideefi- ^ New Course Chemistry (XI) cfviwii

FIGURE 10.5

DILNaOH
SOLUTION
CI2 H2

ow
CI2

BRINE BRINE
DILNaOH
SOLLmON
C
^ SLATE
SLATE PARTITION
PARTITIO

e
:bRINE;

re
IBlIliORP
+ +

Frl
F
MERCURY GROOVE MERCURY

£ ECCENTRIC

ou . WHEEL

sor
Castner and Kellner's cell for the manufacture
of NaOH by electrolytic process

kf
Outer compartments. Sodium chloride undergoes almost complete ionization :
oo
NaCl ^ Na+ + Cl-

H2O being a weak electrolyte ionizes only slightly :


Y
± + OH-
H2O ?
B

Cr and OH" ions migrate towards the graphite anode while Na'*’ and ions are attracted by the
mercury cathode.
re

At the anode, since the discharge potential of Cl" ions is lower than that of OH" ions, so Cl" ions are
oY

discharged at anode and the CI2 evolved passes out through the outlets provided in the outer compartments
u

while OH" ions remain in the solution.


^Cl2T
ad

Cl" ^C\ + e~ ; Cl + Cl

At the cathode, since the discharge potential of Na’*' ions is lower than that of H'*’ ions on the mercury,
d

cathode, so Na"^ ions are discharged while H"^ ions remain in the solution.
Na'*’ + e ^ Na
in

2 Na + .rHg ^ Na2.Hg^.
Re

Sodium amalgam
F

Sodium amalgam thus formed travels to the central compartment due to the rocking motion given to the
cell by the eccentric wheel.
Central compartment. Sodium hydroxide being a strong electrolyte, undergoes complete dissociation
while H2O being a weak electrolyte ionizes only slightly.
NaOH ^ Na-^ + OH- -(0

H2O ^ ^ H'*' + OH'

and Na"*" ions migrate towards the cathode while OH ions travel towards the mercury anode.
At the cathode, since the discharge potential of H'*' ions is lower than that of Na^ ions, H"*" ions are
discharged at the cathode and H2 thus liberated goes out through the outlet provided in the central
compartment.
H^ + e- > H H + H >H2t
s-BLOCK ELEMENTS (ALKALI AND ALKALINE EARTH METALS) 10/25

At the anode. OH ions are attracted towards the mercury anode and after getting discharged, combine
with Na atoms present in sodium amalgam to form NaOH.
OH-- OH + £-

Na2Hg^ + 2 OH ^ 2 NaOH + .r Hg
Since H"^ ions are continuously being discharged at the cathode, the equilibrium (»’) gets shifted towards
right thereby producing more and more OH~ ions. The OH“ ions, in turn, get discharged at the anode dissolving
more and more Na from sodium amalgam thereby forming more and more NaOH. When the concentration of
the NaOH solution increases to about 20%, it is withdrawn, evaporated, fused and finally cast into sticks.
Properties. Sodium hydroxide is a white crystalline solid, m.p. 591 K. It is highly soluble in water to
give a strong alkaline solution which is bitter in taste, corrosive and soapy to touch. Crystals of NaOH are

w
deliquescent. The sodium hydroxide solution formed at the surface reacts with atmospheric CO-, to fonn
crystals of sodium carbonate

F lo
2 NaOH + CO2 Na2C03 + H2O
Uses. (0 It is used in the manufacture of soap, paper, artificial silk (rayon), etc.
{ii) It is used in petroleum refining and in the purification of bauxite.
iiii) It is used in textile industry for mercerising cotton fabrics,

ee
(iv) It is used for the preparation of pure fats and oils,

Fr
(v) It is used in the manufacture of dyes and many other chemicals,
(vi) It is used as a laboratory reagent.

for
10.10.4. Sodium Hydrogen Carbonate (Baking Soda), NaHCO,
Sodium hydrogen carbonate is commonly called Baking soda because on heating it decomposes to
r
evolve bubbles of CO2 (leaving holes in cakes or pastries and making them light and fluffy).
You
Preparation. It is prepared by saturating a solution of sodium carbonate with carbon dioxide. Being
s
ook

less soluble, white crystalline powder of sodium hydrogen carbonate gets separated.
Na2C03 + CO2 + H2O > 2 NaHC03
eB

Uses. (0 It is used in fire extinguishers for generating CO2.


NaHC03 + HCl > NaCI + CO2 + H.>0
(//) It is used in the preparation of baking powder which is a mixture of NaHC03 (30%), starch (40%),
our
ad

calcium dihydrogen phosphate, Ca(H2P04)2 (10%) and sodium aluminium sulphate, NaAi(S04)2. Whereas
starch is used as a filler, NaAl(S04)2 slows down the reaction so that CO2 is given off more slowly.
{Hi) It is a mild antiseptic for skin infections,
(iv) It is used as an antacid in making digestive powders for removing acidity of stomach.
dY
Re

>uiin ^
T jsulphate Na2S203.5H20 (Hypo)*
It is one of the most important salts of thiosulphuric acid and is usually called hvpo, especially by the
Fin

photographers.
Preparation. (1) By heating a solution of sodium sulphite with sulphur. Sodium thiosulphate is
obtained by heating a solution of sodium sulphite with flowers of sulph ur
Na2SC^ + S Na2S.,03
SoH '●ulphite Sod.chiosulphate
The excess of sulphur is removed by filtration and the filtate is concentrated when crystals of
Na2$203.5H20 separate out on cooling.
The solution of sodium sulphite required for the above purpose is prepared very often from sodium
carbonate. A concentrated solution of Na->C03 divided into two equal parts.
*Included in the syllabus of ISCE.
10/26 New Course Chemistry (XI)BS19D

Sulphur dioxide is passed through one part till it is saturated.


Na2C03 + H2O + 2 SO2 > 2 NaHS03 + CO2
Sod. bisulphite
The other part is now added to convert the bisulphite into sodium sulphite.
2NaHS03 +Na2C03 > 2Na2S203 +H2O + CO2
Sod. bisulphite Sod. thiosulphate
(ti) By reaction of iodine on sodium sulphite and sodium sulphide. It is also prepared when calculated
amount of iodine is added to an equimolar mixture of sodium sulphite and sodium sulphide solution.
Na2S + Na2S03 +12 > 2 Nal + Na2S203

w
Sodium thiosulphate crystals separate out first when the resulting solution is subjected to fractional
crystallisation.

F lo
Properties, {a) It forms colourless, monoclinic crystals which melt at 320 K. It is highly soluble in
water.

(0 kction ofheat. On heating it melts at 320 K, loses its water of crystallisation at 488 K and decomposes
above 500 K into sodium sulphate and pentasulphide.

ee
4 Na2S203 ^ 3 Na2S04 + Na2Sg.

Fr
At higher temperatures, pentasulphide loses sulphur.
Na2S3 ^ Na2S + 4 S

for
(ji) Action ofdilute mineral acids. It decomposes with evolution of SO2 and precipitation of sulphur, e.g.,
Na2S203 + 2 HCl > 2 NaCl + H2O + SO2 + S
r
(m) Action of silver nitrate. With a dilute solution of hypo, silver nitrate first gives a white precipitate of
You
silver thiosulphate which gradually changes colour in light from white to yellow to orange and finally black
s
ook

due to the formation of silver sulphide


2 AgN03 + Na2S203 ^
eB

Silver thiosulphate

Ag2S203 + H2O > Ag2S +H2SO4


Silver thiosulphate Silver sulphide
our
ad

(White) (Black)

(b) With a concentrated solution of hypo, no precipitate is formed because silver thiosulphate formed in
the reaction immediately dissolves in the excess of hypo solution giving colourless complex of sodium
argentothiosulphate.
dY
Re

^82^2^3 + 3 Na2S203 > 2Na3[Ag(S203>2]


Silver tUosulphate Sod.silver argentothiosulphate
Fin

(jv) With silver halides. Sodium thiosulphate solution dissolves silver halides (AgCl, AgBr, Agl) forming
a colourless complex—sodium argentothiosulphate, e.g.,
AgCl + 2 Na2S203 > Na3[Ag(S203)2l + NaCl
This reaction is the basis of its use as fixer in photography,
(v) Action of halogens. Sodium thiosulphate is oxidised by chlorine or bromine water and there is
precipitation of sulphur, e.g..
Na2S203 + CI2 "b H2O ^ Na2S04 + 2 HCl + S
But with excess of chlorine, the sulphur is oxidised to sulphuric acid
Na2S203 + 4 CI2 + 5 H2O > 2 NaCl + 6 HCl + 2 H2SO4
It decolourises a solution of iodine giving a quantitative yield of sodium tetrathionate.
s-BLOCK ELEMENTS (ALKALI AND ALKALINE EARTH METALS) 10/27

2 Na2S203 + I2 > Na2S40^ + 2 Nal


Hence this reaction is made use in volumetric estimation of iodine.
(vi) iWth ferric chloride solution. A purple colour due to the formation of ferric thiosulphate is formed.
3 Na2S203 + 2 FeCI 3 ^ Fc2(S203)3 + 6 NaCl
But the colour disappears due to the reduction of ferric ion by the thiosulphate ion.

2Fe^^ +2S.O?-
Z i
> 2Fe2+ +S,0|-
4 O

Uses. (/) It is used in photography due to its property of dissolving silver halides, it is used as fixer in
photography under the name hypo. It removes the excess of silver halides.
(») In textile industry. During bleaching it is used as antichlor for removing excess of chlorine used in

w
bleaching fabrics.
{Hi) In metallurgy. It is used in the extraction of Au and Ag from their ores. The extraction of silver

F lo
depends on the following reactions.
Silver ore + NaCl ^AgCl

AgCl + 2 N32S203 ^ Na3[Ag(S203)2l +NaCI

e
Soluble complex

Fre
Na3[Ag(S203)2] + Cu (Scrap) > Na3[Cu(S203)2] + Agi
(/v) In the laboratory. It is used for the estimation of iodine and as a reagent.

for
2 Na2S203 + I2 ^ +2 Nal
Sodium
r
letrathionale
You
s
10.11. BIOLOGICAL IMPORTANCE OF SODIUM AND POTASSIUM
ook

After calcium, sodium and potassium cations are most common in biological systems. A typical human
being weighing about 70 kg contains about 90 g of Na, 170 g of K, 5 g of Fe and 0 06 g of Cu.
eB

Altliough Na+ and K+ ions have similar chemical properties but surprisingly their biological functions
are quite different. Whereas Na"^ ions are primarily found outside the cells in blood plasma and other
interstitial fluids, K"*" ions are present inside the cells. The Na'*' ions help in transmission of nerve signals, in
our
ad

regulating the flow of water across cell membranes and in the transport of sugars and amino acids into the
cells.

Since K ions are the most abundant cations within the cell fluids, they activate many enzymes which
are
responsible for oxidation of glucose to produce ATP (adenosine triphosphate).
dY

There is a very large variation in the concentration of Na"*- and K+ ions found on the opposite sides of
Re

cell membranes. For example, in blood plasma. Na+ ions are present to the extent of 143 mmol while the
concentration of ions is only 5 mmol L“*. However, within the cells, the concentration of Na+ ions is 10
Fin

mmol L ' and that of ions is 105 mmol L“'. These ionic gradients called the sodium-potassium pump
operate across the cell membranes. The energy needed for transport of these ions is provided by energy irch
ATP molecules.

SECTION—II. ALKALINE EARTH ELEMENTS


The group 2 of the periodic table consists of six metallic elements. These are beryllium (Be), magnesium
(Mg), calcium (Ca), strontium (Sr), barium (Ba) and radium (Ra). These (except beryllium) are commonly
known as alkaline earth metals or simply alkaline earths. The name alkaline earths was initially given only to
magnesium, calcium, barium and strontium since their oxides are alkaline in nature, remain unaffected by
heat or fire and exist in the earth’s crust. The term has now been extended to include all the elements of
group 2.
10/28 “pruidcc^ 'a. New Course Chemistry (XI) 05IHD

10.12. OCCURRENCE
Like alkali metals, alkaline earth metals are also highly reactive and hence do not occur in the free
state but are widely distributed in nature in the combined state as silicates, carbonates, sulphates and
phosphates.

ow
Beryllium is the fifty-first most abundant element by weight found in the earth’s crust. It is found in
small quantities as silicate minerals, heryl Be3Al2 SigOjg and phenacite, Be2Si04.
Magnesium is the sixth most abundant element by weight found in the earth's crust as carbonate,
sulphate and silicate. Important ores of magnesium are : Carnalliie (KCl.MgCl2.6H2O) and magnesite
(MgC03). It also occurs to about 0-13% in sea water as chloride and sulphate.
Calcium is [he fifth most abundant element by weight found in the earth’s crust. It mainly occurs as

e
CaC03 in form of limestone, marble and chalk. Other important minerals of calcium are fluorite or fluorspar

re
CaF2,[fluoropatite, [3 (Ca3(P04)2) ■ Cap2], gypsum CaS04.2 H2O and anhydrite CaS04.
Strontium and baiium are much less abundant. Whereas strontium is mined as celestite SrS04 and
strontianile SrC03 and barium is mined as barytes, BaS04. Radium is extremely scare and is radioactive.

Frl
F
10.13. ELECTRONIC CONFIGURATION
The atoms of all the alkaline earth metals have two
ou ^-electrons in their outermost shell (n^^) as shown in

r
Table 10.5.

so
TABLE 10.5.1 Electronic configuration of alkaline earth metals

kf
Element Atomic Electronic Configuration
Number Complete With Inert gas core
oo
Beryllium (Be) A \PlP [He] 2.r
Y
Magnesium (Mg) 12 \P 2P 2p^ 2P [Ne]
B

Calcium (Ca) 20 l5^ 2p2p^ aP [Arl 4s2


Strontium (Sr) 38 iP 2P 2p^ 35^ 3p^ 3rf"* 4^2 4p^ $P [Kr] 5s^
1 .^2 2/ 3^,2 3^6 3^10 4^2 4^6 4^10 5^ 5^6 ^^2 [Xe]
re

Barium (Ba) 56

\P 2p 2p^ 3p 3/ 3d'^ aP Ap^ As^^Af‘^ [Rn] iP


oY

Radium (Ra) 88
u

5p 5p^ 5(/’^ 6s^ 6p^ Is^


ad

As these elements have similar valence shell electronic configuration , they show similar physical and
chemical properties.
d

Some important atomic and physical properties of alkaline earth metals are given in Table 10.6.
in

TABLE 10.6. Some atomic and physical properties


Re

of group 2 elements (alkaline earth metals)


F

Elements
Property
Ca Sr Ba Ra
Be Mg

Atomic number 4 12 20 38 56 88

9-01 24-31 4008 87-62 137-33 226-03


Atomic mass
112 160 197 215 222
Metallic radius/pm
31 72 100 118 135 148
Ionic radius/pm
899 737 590 549 503 509
Ionisation enthalpy H,
1064 965 979
(kJ mol"') A, H2 1757 1450 1145

-2494 - 1921 - 1577 - 1443 - 1305


Enthalpy of hydration of
M-'*' ions (kJ moP*)
s-BLOCK ELEMENTS (ALKALI AND ALKALINE EARTH METALS) 10/29

Electronegativity
(Pauling scale) 1-57 1-31 100 0-95 0-89 0-9
Density/g mol”* at 293 K 1-84 1-74 1-55 2-63 3-59 (5-5)
Melting point /K 1560 924 1124 1062 1002 973
Boiling point/K 2745 1363 1767 1655 2078 (1973)
(uncertain)
E°/V at 298 K for

mV {aq) + 2e"— ^ M (s) - 1-97 - 2-36 - 2-84 - 2-89 - 2-92 -2-92

low
Occurrence in lithosphere 2* 2-16** 4.6** 384* 390* lO-lO*

10.14. TRENDS IN ATOMIC AND PHYSICAL PROPERTIES


1. Atomic radii. The atomic and ionic radii of alkaline earth metals are fairly large though smaller
than the corresponding alkali metals and these increase down the group.
Explanation. The alkaline earth metals have a higher nuclear charge and, therefore, the electrons arc

ee
attracted more strongly towards the nucleus. As a result, their atomic and ionic radii are smaller than those of

F
the corresponding alkali metals.

Fr
On moving down the group, the atomic and ionic radii increase due to addition of an extra shell of
electrons in each succeeding element and the increasing screening effect.
2. Ionisation enthalpy. The alkaline earth metals have fairly low ionization enthalpies though greater

for
than those of the corresponding elements of group I and these decrease down the group.
ur
Explanation. The low ionisation enthalpy of the alkaline earth metals is because of their strong
tendency to lose electrons due to their smaller nuclear charge and comparatively larger atomic size which
ks
results in weaker forces of attraction between the valence electrons (n5^) and the nucleus. But the values of
the first ionization enthalpy of elements of group 2 are greater than those of the elements of group 1 because
Yo
oo
the atoms of the alkaline earth metals have smaller size and higher nuclear charge than those of alkali
metals.
eB

On moving down the group, ionization enthalpy values go on decreasing because of the increase in
atomic size due to addition of new shells and screening effect of the electrons in the inner shells which
overweigh the effect of increased nuclear charge.
r

It is of interest to note that the second ionization enthalpies Aj H2 of the elements of group I are higher
ou
ad

than those of the elements of group 2. The A,-H, and A,H2 values of sodium (alkali metal) and magnesium
(alkaline earth metals ) are given below :
Y

Element A, H, (kj mol“*) A, (kJ mol-*)


Na (Group 1) 496 4562
Re
nd

Mg (Group 2) 737 1450


Fi

Explanation. The second electron in case of alkali metals (e.g. Na) is to be removed from a cation
(unipositive ion) which has already acquired the stable noble gas configuration whereas in case of alkaline
earth metals (e.g. Mg), the second electron is to be removed from a cation (unipositive ion) which is yet to
acquire the stable noble gas configuration. Therefore, removal of second electron in case of alkaline earth
metals requires much less energy than that in case of alkali metals.
A,H I
Na (g)
—e
Na*- (g) V Na^+ (g)
-e
Is- 2s- 2p^ 3s' l.'^2s^2p^ l.v2 2.r2
Stable configuration

*ppm (parts per million) **


percentage by weight
10/30 New Course Chemistry (XI)EE3MD

A.-H i A,H
Mg (g) Mg"^ (g) ^ Mg^'^ (g)
—e -e

li - 2s~ 2p^ 3s~ Ir 2s^2p^ 3s' \s- 2s~ 2p^


Stable configuration

The A,H3 of Mg will be very high because now the electron has to be removed from the stable noble gas
configuration.
3. Hydration enthalpy. Like alkali metal ions, the hydration enthalpies of alkaline earth metal ions
decrease as the size of the metal ion increases down the group, i.e., Be^"^ > > Ca^’^ > Sr^"*" > Ba +
Due to smaller size of alkaline earth metal ions as compared to alkali metal ions, the hydration enthalpies
of alkaline earth metal ions are larger than those of alkali metal ions. Consequently, the compounds of alkaline

w
of alkali metals. For example, MgCl2 and CaCl2 exist
earth metals are more extensively hydrated than those
as MgCl2.6 H2O and CaCU . 6 H2O while NaCl and KCl do not form such hydrates.
4. Dipositive oxidation state The alkaline earth metals have two electrons more than the nearest

F lo
noble gas configuration. Therefore, they can easily lose these two electrons to form divalent cations. Thus,
alkaline earth metals uniformly show an oxidation state of + 2.
Explanation. In view of lower value of first ionisation enthalpy, it would appear that the alkaline earth
metals should prefer to form +1 (M-*-) ions rather than +2 ions. However, in actual practice, these give

ee
only +2 ions, e.g., Mg^'^, Ca-"^, etc. If ionisation enthalpy were the only factor involved, we would have got

Fr
the monovalent ions, i.e., Mg+, Ca"*", etc. rather than the divalent ions, i.e., Mg^+, Ca~+, etc. This may be
explained as follows :
(i) The divalent cations of alkaline earth metals acquire stable inert gas configuration.

for
(ii) The existence of divalent ions in the solid state is due to the reason that divalent cations due
to smaller size and higher charge form stronger lattices than monovalent (M’*') cations. As a result, a larger
ur
amount of lattice enthalpy is released during the formation of compounds containing M"'*' ions than in the
formation of compounds containing M'*' ions. !t is this greater lattice enthalpy of ions which more
s
enthalpy thereby making ions more stable than A/+
ook
than compensates for the higher .second ionization
Yo
ions.

(Hi) The existence of divalent ions in the aqueous solution is due to greater enthalpy of hydration of
eB

the divalent ions which counterbalances the higher value of second ionisation entlialpy. For example, consider
the energy changes taking place during the formation of MgCl (aq) and MgCl2 (aq). The enthalpy of hydration
of Mg“+, Mg+ and Cl“ions are - 1921,-365 and - 381 kJ moi“' respectively.
Therefore, the enthalpy of hydration of MgCI^ is much higher [- 1921 (Mg^"*”) - 2 x 381 (2 Cl”)
our
ad

= - 2683 kJ mol-'] than that of MgCl [- 365 (Mg+) -381 (CL) = - 746 kJ mor’]. It is this higher enthalpy
of hydration (2683 - 746 = 1937 kJ mol"') which more than compensates for the higher value of A,H2
(1450 kJ mol"'). Consequently, the net energy change accompanying the formation of MgCl2 (aq) is much
Y

higher (i.e. 798 kJ mol"') than that of MgCl (aq) i.e. 85 kJ mol"' as calculated below. Therefore, alkaline
earth metals prefer to form divalent ions rather than monovalent ions.
Re
nd

Comparison of energy changes in the formation of MgCl (aq) and MgCl2 (aq)
Fi

MgCl (aq) A H (kJ mol“l) MgCl2 (aq) A H (kJ mol"')


150
Mg (s) ^ Mg (g) 150 Mg (s) ■> Mg (g)
2187
Mg (g) Mg-^ (g) + e~ 737 Mg (5) ^ Mg--^ (g) + 2 e“
1
^2 Cl (g) 244
->C1 (g) 122 CI2 (g)
2 Cl2(g)
-348 2 Cl (g) + 2e- ^2Cr (g) -696
Cl (g) + e- ^Cl-(g)
Mg"^ (g) + Cl" (g) + aq 746 Mg^"^ (g) + 2 cr (g) + aq -2683

> Mg'*" (aq) + Cr (aq) > Mg-'*’ (aq) + 2 Cl" (aq)


Net energy change = - 85 Net energy change = - 798
S-BLOCK ELEMENTS (ALKALI AND ALKALINE EARTH METALS) 10/31

5. Electropositive or Metallic character. The alkaline earth metals are highly electropositiveor metallic
and their electropositive or metallic character increases down the group. Howeve,r they are less electropositive
or metallic than the alkali metals.

Explanation. On account of their relatively low ionization energies, the alkaline earth metals have a
strong tendency to lose both the valence electrons to form dipositive cations. Thus, these elements show
strong eletropositive or metallic character.
On moving down the group, the atomic radii increase and ionization enthalpies decrease. Consequently,
the electropositive or metallic character increases.
Further, since the atoms of the alkaline earth metals have smaller size and higher ionization enthalpies
as compared to corresponding alkali metals, their tendency to lose valence electrons is lesser than those of

w
alkali metals. Consequently, alkaline earth metals have less electropositive or metallic character as compared
to alkali metals.

Due to smaller size of the cations and greater number of valence electrons, the metallic bonding in

F lo
alkaline earth metals is stronger as compared to alkali metals. Because of this reason, these metals are less
soft (i.e. they are harder) than alkali metals.
6. Melting and boiling points. The alkaline earth metals have higher melting and boiling points as
compared to those of alkali metals. However, down the group, there is no regular trend in their melting and

ee
boiling points.

Fr
Explanation. Because of their smaller size and more close packed crystal lattice as compared to alkali
metals, their melting and boiling points are higher than those of group I elements.
7. Nature of bonds formed. Like alkali metals, alkaline earth metals predominantly form ionic

for
compounds which are, howeve,r less ionic than the corresponding alkali metal compounds. The tendency to
form ionic compounds increases down the group. The first membe.r Be, howeve,r fonns covalent compounds.
ur
Mg also shows some tendency for covalency. All other elements form ionic compounds.
Explanation. They form ionic compounds because they have low ionization enthalpies. Their compounds
s
are less ionic because their ionization enthalpies are higher than those of the corresponding alkali metals. As
ook
Yo
expected, the tendency to form ionic compounds increases down the group because ionization enthalpy
decreases. Beryllium, however, forms covalent compounds because it has smaller size and high ionization
eB

enthalpy.
8. Density. The alkaline earth metals are denser than the alkali metals due to smaller size and better
backing in the cry.stal lattice. Howeve,r the densities of alkaline earth metals do not show any regular trend
with increasing atomic numbe.r The density of these metals first decreases from Be to Ca and then increases
our

from Ca to Ba.
ad

Explanation. Because of their smaller size and hence better packing as compared to alkali metals, they
are denser than alkali metals. The decrease in density from Be to Ca may be due to decrease in the packing of
atoms in their solid lattice.
Y

9. Flame colouration. Like alkali metal salts, alkaline earth metal salts also impart a characteristic
Re

colour to the flame.


nd

Calcium Strontium Barium Radium


Brick red Critthson red Apple green Crimson.
Fi

Explanation. When alkaline earth elements or their compounds are put into a flame, the electrons
absorb energy and are excited to higher levels. When they return to their ground state (normal state), the
absorbed energy is emitted in form of visible light of a particular wavelength.
Beryllium and magnesium atoms are smaller in size and their electrons are strongly held by the nucleus.
They need large amounts of energy for excitation of electrons to higher energy levels which is not available
in the bunsen flame. So they do not impart colour to the flame.

10.15. REACTIVITY AND ELECTRODE POTENTIAL


All the alkaline earth metals are highly reactive elements since they have a strong tendency to lose the
two valence j'-electrons to form the corresponding dipositive ions having inert gas configuration.
10/32 New Course Chemistry (XI)EEIHII

This reactivity arises due to their low ionization enthalpies and high negative values of their standard
electrode potentials (Table 10.6). Further, the chemical reactivity of alkaline earth metals increases on moving
down the group from Be to Ba because the ionization enthalpies decrease and electrode potentials become
more and more negative with increasing atomic number from Be to Ra. Thus, beryllium is the least reactive
while Ba (or Ra) is the most reactive element.
Further since the ionization enthalpies of alkaline earth metals are higher and their electrode potentials
less negative than the corresponding alkali metals, therefore, alkaline earth metals are less reactive than
corresponding alkali metab.
10.15.1. Reducing Character
The alkaline earth metals are weaker reducing agents than the alkali metals. Like alkali metals, their

w
reducing character also increases down the group.
Explanation. Except beryllium, the alkaline earth metals have a fairly strong tendency to lose two
electrons to form dipositive ions (M ^ M“'*‘ + 20 because of their low ionisation enthalpies and high

F lo
negative values of standard electrode potentials. Therefore, they act as reducing agents.
The reducing character of alkaline earth metals, however, increases as we move down the group from
Be to Ba because the ionization enthalpies decrease and electrode potentials become more and more negative

ee
with increasing atomic number from Be to Ba.
Further since the ionization enthalpies of alkaline earth metals are higher and their electrode potentials

Fr
are less negative than the corresponding alkali metals, therefore, alkaline earth metals are weaker reducing
agents than alkali metals.
10.15.2. Reactivity towards Water- Formation of Hydroxides

for
The electrode potential of beryllium (Be^'^/Be = - 1-97 V) is least negative amongst all the alkaline
ur
earth metals. This means that Be is much less electropositive than other alkaline earth metals and hence does
not react with water or steam even at red heat.
s
The electrode potential of Mg (Mg-'^/Mg = - 2-36 V), although more negative
ook
Be
Yo
than that of Be, yet is still less negative than those of alkali metals and hence it does not CO
9 fo
react with cold water but reacts with boiling water or steam. a: oi
< CO
Mg
Mg + H2O > MgO + H2 or Mg + 2 H2O > Mg(OH)2 + 2 H2.
eB

Mg, in fact, forms a protective layer of oxide on its surface, therefore, despite its Ca
favourable electrode potential, it does not react readily with water unless the oxide > o

layer is removed by amalgamating it with mercury. In the formation of oxide film. Mg Sr


b?
r

resembles Al. So
ad
ou

01

Ca, Sr and Ba have more negative electrode potentials similar to those of the Ba
corresponding group 1 alkali metals and hence react with increasing vigour even with
cold water, liberating H2 and forming the corresponding metal hydroxides
Y

Ca + 2 H2O > Ca(OH>2 + H2


Re

This means that reactivity of alkaline earth metals increases as we move down the group. However, the
nd

reaction of alkaline earth metals is less vigorous as compared to alkali metals.


Fi

10.15.3. Reactivity towards Air (Nitrogen and Oxygen)


(a) Formation of oxides and nitrides. The alkaline earth metals being less electropositive than alkali
metals react with air or oxygen slowly upon heating to form oxides, MO. Beryllium and magnesium are
kinetically inert to oxygen because of the formation of a thin film of oxide on their surface. Beryllium in the
massive form does not react with air below 873 K. However, powdered beryllium is more reactive and bums
brilliantly on ignition to give a mixture of beryllium oxide (BeO)* and beryllium nitride (Be3N2)
A A

2 Be + O2 (air) 2 BeO 3 Be + N2 (air) ■> Be3N2

*Please note that except BeO which is made by ignition of the metal, all other metal oxides are usually made
by thermal decomposition of their corresponding carbonates, nitrates or sulphates and by dehydration of their
hydroxides on heating.
s-BLOCK ELEMENTS (ALKALI AND ALKALINE EARTH METALS) 10/33

Magnesium is more electropositive than beryllium and hence bums with dazzling brilliance in air to
form a mixture of magnesium oxide and magnesium nitride
A A
2 Mg + O2 (air) ^ 2 MgO 3 Mg + N2 (air) ^ Mg3N2
Calcium, strontium and barium being even more electropositive react with air readily to form a mixture
of their respective oxides and nitrides.
Thus, reactivity towards oxygen increases as we go down the group. Thus, Ca, Ba and Sr are stored in
paraffin but Be and Mg are not because they form a protective layer of oxide on their surface.
{b) Formation of nitrides. All the alkaline earth metals burn in dinitrogen to fonn ionic nitrides of the

w
formula, M3N2. This is in contrast to alkali metals where only Li forms Li3N.
A

3M + N2 ^ M3N2

F lo
BejN2 being covalent is volatile while the nitrides of all other elements are crystalline solids.
All these nitrides decompose on heating and react with water liberating NH3
A

ee
Be3N2 > 3 Be + N2 Ca3N2 + 6 HoO 4 3 Ca(OH)2 + 2 NH3
(c) Formation of peroxides. Since larger cations stabilize larger anions, therefore, tendency to form

Fr
peroxide increases as the size of the metal ion becomes larger. Thus, barium peroxide (Ba02) is formed by
passing air over heated BaO at 773 K.
773 K 773 K

for
2 BaO + 0-) > 2 BaO-, 2 SrO + 0->
ur high pre.ssure
●> 2Sr02

Sr02 is prepared in a similar way but under high pressure and temperature. Ca02 is not formed this way
but can be prepared as hydrate by treating Ca(OH)2 with H2O2 and then dehydrating the product.
ks
Ca(OH)2 + H2O2 > Ca02.2H2O
Yo
oo
Crude Mg02 has been made using H-,02 but peroxide of beryllium is not known.
All peroxides are white crystalline ionic solids containing the peroxide ion, O2 . Treatment of peroxides
eB

with acids liberates hydrogen peroxide.


Ba02 + 2 HCl ^ BaCl2 + H20t
10.15.4. Reactivity towards Halogens - Formation of Halides
r
ou
ad

All the alkaline earth metals combine with halogens at elevated temperatures forming their halides.
M + X2 > MX2 (X = F, Cl, Br, i)
Y

Thermal decomposition of (NH4)2Bep4 is the best method to prepare Bep2.


Heat
BeO + 2 NH3 + 4 HP > (NH4)2 BeP4 > BeF2 + 2NH4p
nd
Re

BeCl2 is, however, conveniently prepared by heating BeO with CI2 in presence of carbon.
873 K
Fi

BeO + C + CI2 ■> BeCL + CO


Beryllium chloride can also be prepared by heating berylliumoxide with carbon tetrachloride.
973 K
2 BcO + CCI4 2 BeCl2 + CO-)
10.15.5. Reactivity towards Hydrogen - Formation of Hydrides
All the alkaline earth metals except beryllium combine with hydrogen directly on heating to form metal
hydrides of the formula MH^.
Heat

M + H2 4 MH2 (M = Mg, Ca, Sr, Ba)


Metal hydride
10/34 New Course Chemistry (XI)I2EIMI

The hydride of beryllium can, however, be obtained by the reduction of BeCl2 with LiAlH4
2 BeCl2 + LiAlH4 > 2 BeHj + LiCl + AIC13
Both beryllium hydride (BeH-,) and magnesium hydride (MgH2) are covalent compounds.

ow
Since BeH2 or MgH2 (with two nomial covalent bonds) has only four electrons in the valence shell,
therefore, they are electron deifcient molecules. To make up their electron deficiency, each Be or Mg atom
forms four three centre two-electron {3 c - 2 e) bonds or banana bonds as shown below. Thus, it is due to
electron deficiency that BeH2 and MgH^ have polymeric structures.
H H H

.Be Be. .Be Be ^


^ \h/ X/ ^

re
The hydrides of other elements of this group, Le., CaH2, SrH2 and BaH2 are ionic and contain the H~
ions. The stability of alkaline earth metal hydrides decreases down the group from Be to Ba since their lattice

Flr
F
enthalpies decrease progressively as the size of the metal cation increases.
All the hydrides of alkaline earth metals react with water liberating H2 gas and thus act as reducing
agents.

ou
MH2 + 2 H2O ^ M(0H)2 + 2 H2

sr
CaH^, i.s called hydrolith and is used for large scale production of H2 by action of water on it.

fo
10.15.6. Reactivity towards Carbon - Formation of Carbides

k
When beryllium oxide is heated with carbon at 2175 - 2275 K, a brick red coloured carbide of the
formula. Be^C is formed.
2 BeO + 3 C
2175-2275 K oo Bc2C + 2 CO
It is an ionic compound and reacts with water forming methane
Y
Be.C + 4 H2O > 2 Be(OH)2 + CH4
reB

The rest of the alkaline earth metals (Mg, Ca, Sr and Ba) form carbides of the general formula, MC2 either
when the metal is heated with carbon in an electric furnace or when their oxides are heated with carbon.
uY

1373 K 2273 K
Ca -H 2C CaC2 CaO + 3 C C3.C2 CO
All these carbides react with water producing acetylene gas.
CaC2 + 2 H2O ^ HC s CH -1- Ca(OH)2
ad
do

Cal. carbide Acetylene

It may be noted here that MgC-, on heating gives Mg2C3- This carbide contains C^ units and hence
in

reacts with water to form propyne (i.e. methylacetylene)


Mg2C3 + 4 HjO ^ CH3CSCH -1- 2Mg(OH)2
Re

Propyne
F

Calcium carbide is an important chemical intermediate. When CaC2 is heated in an electric furnace with
atmospheric dinitrogen at 1375 K, it forms calcium cyanamide, CaNCN or CaCN2-
1375 K
CaC-> -I* N2 CaNCN + C
Cal. carbide Cal. cyanamide
The mixture of calcium cyanamide and carbon is called nitrolim and is used as a slow acting nitrogenous
fertilizer as it hydrolyses slowly over a period of months evolving NH3 gas
CaNCN -t- 3 H.,0 > CaC03 + 2 NH3
10.15.7. Reactivity towards Acids
All alkaline earth metals react with acids liberating H2.
M-H2HC1 ^ MCI2 + H2 (M = Be, Mg, Ca, Sr, Ba)
s-BLOCK ELEMENTS (ALKALI AND ALKALINE EARTH METALS) 10/35

Be being amphoteric also dissolves is alkalies liberating H2.


Be + 2 NaOH + 2 H2O > Na2[Be{OH )4] + H2
Sod. beryllate
In contrast. Mg, Ca, Sr and Ba being purely basic do not dissolve in NaOH. This shows that the basic
character of these metals increases down the group.
10.15.8. Solutions In Liquid Ammonia

ow
Like alkali metals, all alkaline earth metals dissolve in liquid ammonia. The dilute solutions are bright
blue in colour due to solvated electrons but concentrated solutions are bronze coloured due to the formation
of metal clusters.

These solutions decompose very slowly forming amides and evolving H^, but the reaction is accelerated
by many transition metals and their compounds.

e
M + (j: + 2 y) NH3 > M-+ (NH3)^. + 2e- (NH3)^.

re
1

rFl
e- (NH3>^, > NH; +-H, + (y - 1) NH3

F
Whereas evaporation of ammonia from solutions of alkali metals gives the metal, evaporation of ammonia
from solutions,of alkaline earth metals gives hexammoniatesof the general formula, [M(NH3)g]^''' which
slowly decompose to give the corresponding metal amides.

r
M(NH3>g > M(NH2)2 + 4 NH3 + H2
ou
fo
10.15.9. Tendency to form Complexes

ks
Among alkaline earth elements Be and Mg have the maximum tendency to form complexes. This is due
to their small size and higher charge density. For example, Bep2 combines with F" ions to form the tetrahedral
complex, [BeF^]^”
oo
BeF2 + F2 ^ [BeF4l--
Y
Chlorophyll is an important complex of magnesium.
B

10.16. GENERAL CHARACTERISTICS OF COMPOUNDS OF THE ALKALINE EARTH METALS

As discussed above, due to higher enthalpy of formation in the solid state and due to higher hydration
re

enthalpy in the aqueous solution, alkaline earth metals uniformally form dipositive ions.
Further due to increased nuclear charge and smaller size, alkaline earth metals form compounds which
ou
Y

are less ionic than the corresponding compounds of the alkali metals. Usually the oxides and other salts of Be
ad

and Mg are more covalent than those formed by the heavier and larger members (Ca, Sr, Ba). A general
survey of some of their compounds is presented below.
10.16.1. Oxides and Hydroxides
d

Oxides. The oxides of alkaline earth metals MO, are obtained either by heating the metals in dioxygen
in
Re

or by thermal decomposition of their carbonates.


A

2M + O2 ^ 2MO (M = Be, Mg, Ca)


F

MCO3 MO-HC02 (M = Be, Mg, Ca, Sr, Ba)


Except BeO, all the oxides have rock-salt structures. Further, the enthalpies of formation of these oxides
are quite high and consequently they are very stable.
Metal oxide BeO MgO CaO SrO BaO
(kJ mol"*) 550 590 623 590 545

These have high melting points, have very low vapour pressures, are very good conductors of heal and
are chemically inert. Because of these properties, these oxides are used for lining furnaces and hence are used
as refractory materials. Due to small size of beryllium ion, BeO is covalent (and insoluble in water) but still
has high melting point because of its polymeric nature. Each Be atom is tetriihedrally coordinated to four
other oxygen atoms. Therefore, like other metal oxides, BeO is also used as a refractory.
10/36 ‘P>u\<iee^ 4 New Course Chemistry (XI)

Hydroxides. The hydroxides of Ca, Sr and Ba are obtained either by treating the metal with cold water
or by reacting the corresponding oxides with water. The reaction of these oxides with H2O is also sometimes
called as slaking.
M + 2 H2O^ M(0H)2 + H2 (M = Ca, Sr, Ba)
MO + HoO ^ M(0H)2 (M = Ca, Sr, Ba)
Be(OH)2 and Mg(OH)-> being insoluble are obtained from suitable metal ion solutions by precipitation
with OH“ ions.

BeCl2 + 2 NaOH > Be(OH)2 i + 2 NaCl


MgSO^ + 2 NaOH > Mg(OH)2 i + Na2S04

w
Properties of hydroxides. (/') Basic character. Due to small size and high ionization enthalpy of Be,
Be(OH)2 is amphoteric. It, therefore, dissolves both in acids and bases.
Be(OH>2 + 2 HCl > BeCU + 2 H2O {Basic nature)

F lo
Be(OH)2 + 2 NaOH > Na-,BeOi + 2 H2O {Acidic nature)
Sod. beryllale

ee
or
Be(OH)2 + 2 OH- > [BeCOH)^]-- {Acidic nature)

Fr
Beryllate ion
The hydroxides of Mg, Ca, Sr and Ba are basic. Their basic strength increases as we move down the
group. The basic character of these hydroxides is due to their low ionization enthalpies. Because of low

for
ionization enthalpies, the M—O bond in MOH is weak and thus breaks to give OH“ ions in solution. On
ur
moving down the group, the ionic sizes increase and ionization enthalpies further decrease. As a result,
M—0 bond becomes weaker and weaker down the group and hence the basic character also increases down
the group.
s
However, these hydroxides are less basic than the corresponding
ook
Be(OH)2 r .— >●
Yo
CO O
alkali metal hydroxides because of higher ionization enthalpies, uj a:
CO 52 LU
smaller ionic sizes and greater lattice energies. Mg(OH)2 .a
^
CO ^ LU
2:

< -V
eB

UJ
S o z
(«) Solubility in water. Alkaline earth metal hydroxides are less I-
O CO q: m O
(jO\-
soluble in water as compared to the alkali metal hydroxides.
ui
Ca(OH)2 CO z >■ S
^2 -og
The solubility of the alkaline earth metal hydroxides in water o
b UJ T
r

increases with increase in atomic number down the group. This is due Sr{OH)2 O 2 5 UJ 2
3 UJ <
ou
ad

CO
to the reason that both lattice enthalpy and hydration enthalpy <
CQ

decrease down the group as the size ofthe cation increases but lattice Ba{OH)2
CO

-) o
enthalpy decreases more rapidly than the hydration enthalpy and
Y

hence their solubility increases down the group.


10.16.2. Halides
Re
nd

Preparation. The alkaline earth metals combine directly with halogens at appropriate temperatures
forming halides, MX2.
Fi

These halides can also be prepared by the action of halogen acids (HX) on metals, metal oxides, hydroxides
and carbonates.
M + 2 HX ■> MX, + H, MO + 2 HX > MX2 + H2O
M(0H)2 + 2HX >MX2 + 2H20 ; MCO3 + 2HX >MX2 + C02 + H20
Properties. 1. All beryllium halides are essentially covalent and are soluble in organic solvents. They
are hydroscopic, and fume in air due to hydrolysis. On hydrolysis, they produce acidic solution.
BeCl2 + 2 H2O Be(OH)2 + 2 HCl.
2. The halides of all other alkaline earth metals are ionic. Their ionic character, however, increases as
the size of the metal ion increases.
S-BLOCK ELEMENTS (ALKALI AND ALKALINE EARTH METALS) 10/37

As the ionic character increases or the covalent character decreases, their tendency towards hydrolysis
decreases : BeCl2 > MgCl2 > CaCl^ > SrCU > BaCl^
3. Except BeCl2, all other chlorides of group 2 form hydrates but their tendency to form hydrates
decreases. For example, MgCl2-6 H^O, CaCl2. 6 H^O, SrCl2 . 2 H^O and BaCI->.2 H-,0.
4. The hydrated chlorides, bromides and iodides of Ca, Sr and Ba can be dehydrated on heating but
those of Be and Mg undergo hydrolysis. For example,
A A

MgCl2.6 H2O ^ MgO + 2 HCl + 5 H2O ; CaCl2.6 H2O > CaCl2 + 6 H2O
5. BeF2 is highly soluble in water due to the high hydration enthalpy of the small Be-'*' ion (- 2494 kJ

w
mor’). The other fluorides (MgF^, CaF2, SrF2 and BaF^) are almost insoluble in water. CaF2 is commonly
called fluorspar since it emits light when heated.
The chlorides, bromides and iodides of all other elements, i.e.. Mg, Ca, Sr, Ba are ionic, have lower

F lo
melting points than the fluorides, and are readily soluble in water. The solubility, decreases somewhat with
increasing atomic number.
6. Except BeCl2 and MgCl2, the other chlorides of alkaline earth metals impart characteristic colours to

ee
flame.

CaCl2 SrCl2 BaCU

Fr
Brick red colour Crimson colour Grassy green colour
Structure of BeCl2. In the solid phase, BeCl2 has polymeric chain structure with chlorine bridges as

for
shown below.
ur
The polymeric structure of BeCl2 is due to its electron deifcient nature. Be has only four electrons in the
valence shell and hence can accept two electron pairs (from neighbouring Cl chains forming co-ordinate
bonds) to complete its octet.
s
Thus, each Be atom is tetrahedrally surrounded by four chlorine atoms. Two of the chlorine atoms are
ook
Yo
bonded by two covalent bonds while the other two by co-ordinate bonds.
eB
r
ou
ad

In the vapour phase it tends to form a chlorine bridged dimer which dissociates into the linear triatomic
monomer at high temperatures (at nearly 1200 K).
Y

Cl
Cl—Be Be—Cl Cl—Be—Cl
Re

Cl Monomer
nd

(Dimer)
Fi

Uses. (0 Calcium fluoride or fluorspar (CaF2) is by far the most important of all the fluorides of the
alkaline earth metals since it is the only large scale source of fluorine.
(») CaCl2 is widely used for melting ice on roads, particularly in very cold countries because 30%
eutectic mixture of CaCl2/ice freezes at 218 K (- 55°C) compared to NaCl/ice at 255 K (- 18"C). Anhydrous
CaCl2 is also used as a desiccant (drying agent in the laboratory).
(«0 Anhydrous MgCl2 is used in the electrolytic extraction of magnesium.
10.16.3. Solubility and Thermal Stability of Oxo Salts
The salts containing one or more atoms of oxygen such as oxides, hydroxides, carbonates, bicarbonates,
nitrites, nitrates, sulphates, oxalates and phosphates are called 0x0 salts. Let us now discuss the solubility
and thermal stability of some of the 0x0 salts of alkaline earth metals.
10/38 New Course Chemistry (XI)BS19D

1. Sulphates.
The sulphates of alkaline earth metals (MS04) are prepared by the action of sulphuric acid on metals,
metal oxides, hydroxides and carbonates.
M + H2SO4- ■> MSO4 + H2 MO + H2SO4 ^ MSO4 + H2O
M(0H)2 + H2SO4 ^ MSO4 + 2 H2O MCO3 + H2SO4 MSO4 + CO2 + H2O
Properties. CO

ow
BeSO.}
(0 The sulphates of alkaline earth metals are all white solids. Beryllium,
magnesium and calcium sulphates crystallise in the hydrated form, i.e., MgS04 a:
o Q 2
BeS04.4H-,0, MgS04.7H20, CaS04.2H20 but sulphates of-strontium and
barium crystallise without water of crystallization. CaS04 LU Q
oi

f- CO V
^ =5 O
r
iii) Solubility. The solubility of the sulphates in water decreases down the SrS04 “ O
group, i.e., Be > Mg > > Ca > Sr > Ba. Thus, BeS04 and MgS04 are highly Ml
-j ty z

e
O “ m
soluble, CaS04 is sparingly soluble but the sulphates of Sr, Ba and Raare virtually BaS04 CO “

re
rFl
insoluble.

Explanation. The magnitude of the lattice enthalpy remains almost constant as the sulphate ion is so big

F
that small increase in the size of the cations from Be to Ba does not make any material difference. However
the hydration enthalpy decreases from Be-'*' to Ba-'*' appreciably as the size of the cation increases down the
group. Hence, the solubilities of sulphates of alkaline earth fvetals decrease down the group mainly due to

r
the decreasing hydration enthalpies from Be^* to The high solubility of BeSO^ and MgSO^ is due to
ou
sfo
the high hydration enthalpies because of smaller size of Be-'*' and Mg~'*' ions.
{Hi) Stability. The sulphates of alkaline earth metals decompose on heating giving their corresponding
oxides and SO3. k
A

MSO4 ^ MO + SO3
oo
The temperature of decomposition of these sulphates increases as the electropositive character of the
Y
metal or the basicity of the metal hydroxide increases down the group. For example.
eB

Compound : BeS04 MgS04 CaS04 SrS04


Temp, of decomposition : 773 K 1168 K 1422 K 1647 K

Uses. (0 The almost negligible solubility of BaS04 in water is used in the detection and estimation of
ur

SO^" ions,
ad
Yo

(/i) BaS04 is both insoluble in H^O and opaque to X-rays. Therefore, 'barium meal' is used to obtain a
shadow of the stomach on an X-ray film which is useful in diagnosing stomach ulcers.
2. Carbonates and Bicarbonates.
d

Preparation of carbonates. Alkaline earth metal carbonates are obtained as white precipitates when (/)
Re
in

calculated amount of carbon dioxide is passed through the solution of the alkaline metal hydroxides
M(0H)2 (aq)+ CO2 (g) ■> MCO3 {s) + H2O (/)
F

and (//) sodium or ammonium carbonate is added to the solution of the alkaline earth metal salt such as
CaCU

CaCl2 {aq) + Na2C03 (aq) ^ CaC03 (s) + 2 NaCl {aq)


w
BeC03 CO
UJ
Properties of carbonates. (0 All carbonates are ionic but MgC03
CO
a:
iS
beryllium carbonate is prone to hydrolysis. It contains the hydrated u Q ir
W >● o
q:
o
ion, [Be(H20)4]^'*' rather than Be-"*" and hence is precipitated only in CaC03 O X UJ
^ lij Q
z

b CO >-
an atmosphere of CO->.
SrC03 m < a:
3 O UJ CD
d LU z i<
Absence of CO, O £,ui
BaC03 CO CO

BeC03 -I- 4 H2O < ^ [Be(H20)4]2-^-i- C02-


Presence of CO,
S-BLOCK ELEMENTS (ALKALI AND ALKALINE EARTH METALS) 10/39

(//) Solubility. The carbonates of magnesium and other alkaline earth metals are sparingly soluble in
water and their solubility decreases down the group from Be to Ba. For example, MgC03 is slightly soluble
in water but BaC03 is almost insoluble.
Explanation. As we move down the group, the lattice enthalpies of carbonates remain approximately
the same. The reason being that carbonate ion is so large that relatively small changes in the sizes of the
cations from Be^'*' to Ba^'*’ do not make any material difference. However, the hydration enthalpies of the
metal cations decrease from Be^"*" to Ba^'*’. Hence, the solubilities of carbonates of the alkaline earth metals
decrease down the group mainly due to decreasing hydration enthalpies of the cations from Be^"^ to Ba^'*'.
All the carbonates of alkaline earth metals are, however, more soluble in the presence of CO2 due to the

low
formation of corresponding bicarbonates. For example,
CaC03(s) + C02(g) + H20(g) ^ Ca(HC03)2
(//) Thermal stability. The carbonates of all alkaline earth metals decompose on heating to form the
corresponding metal oxide and CO2. A
MCO3 » MO + CO2
The temperature of decomposition, i.e., thermal stability of these carbonates, howeve,r increases down

ee
the group as the electropositive character of the metal or the basicity of metal hydroxide increases from

F
Fr
Be(OH)2 to Ba(OH)2- For example,
BeC03* MgC03 CaC03 SrC03 BaC03
<373 K 813 K 1173 K 1563 K 1633 K

for
BeC03 is thus unstable and can be kept only in an atmosphere of CO2.
ur
Preparation of bicarbonates. The bicarbonates of alkaline earth metals are prepared by passing CO2
through a suspension of metal carbonates in water.
■> M(HC03)2
ks
{Insoluble) {Soluble)
Yo
oo
All the bicarbonates of alkaline earth metals are stable only in solution and have not been isolated in the
pure state.
eB

Uses. The extremely low solubility of alkaline earth carbonates in water is made use of in qualitative
analysis. For example,
(0 The cations of group V (Ba"*", Sr^'*’, Ca^"^) of qualitative analysis are precipitated as their insoluble
carbonates from the solution of their soluble salts by adding (NH4)2C03 in presence of NH4CI and excess of
r

NH.OH.
ou
ad

(«) The soluble carbonates, i.c., carbonates of alkali metals and NHJ ion are detected by precipitating
Y

them as insoluble magnesium carbonate.


{Hi) CaC03 is used in Solvay-ammonia process for manufacture of Na2C03, in glass making and in
cement manufacture.
Re
nd

3. Nitrates

Alkaline earth metal nitrates are prepared in solution and can be crystallized as hydrated salts by the
Fi

action of HNO3 on oxides, hydroxides and carbonates.


MO + 2 HNO3 » M(N03)2 + H2O
M(0H)2 + 2 HNO3 > M (N03>2 + 2 H2O
MCO3 + 2 HNO3 > M(N03)2 + CO2 + H2O (M = Be, Mg, Ca, Sr or Ba)
Magnesium nitrate crystallizes as Mg(N03)2-6H20 while Ba(N03)2 crystallises as anhydrous salt.
Beryllium nitrate is unusual because it forms basic nitrate, i.e., [Be40(N03>5] in addition to the normal
salt. This again shows that tendency to form hydrates decreases with increasing size and decreasing hydration
enthalpy down the group.

*BeC03 is unstable in air (BeC03 ^ - BeO + CO2), therefore, to avoid decomposition, it is kept in an
atmosphere of CO2.
10/40 “Pteuiee^'^. New Course Chemistry (XI)CEIHD

All nitrates are soluble in water and decompose on heating to give the corresponding oxides with evolution
of a mixture of NO2 and O2.
2 M(N03)2 ^ 2 MO + 4 NO2 + O2 (M = Be, Mg, Ca, Sr or Ba)
Strontium and barium nitrates are used in pyrotechnics for giving red and green flames respectively.
4. Oxalates

ow
The oxalates of Ca, Sr and Ba are sparingly soluble in water but their solubility increases from Ca to Ba.
Beryllium oxalate is, however, soluble in water.
From the above discussion, it follows that calcium, strontium and barium are the three elements of
group 2 whose physical and chemical properties are closely related and change systematically with increasing
atomic size. Thus, Ca, Sr and Ba are highly electropositive, have high negative electrode potentials (i.e. E®)
and show systematic trends in the solubility of their 0x0 salts.

e
re
10.17. ANOMALOUS BEHAVIOUR OF BERYLLIUM

rFl
Beryllium, the first member of alkaline earth metals, shows an anomalous behaviour, i.e., differs from

F
the rest of the members of its family. The main reasons for this difference are as follows:
(a) exceptionally small atomic and ionic size
(b) high ionization enthalpy

r
ou
(c) absence of c/-orbitals in its valence shell.

fo
Some important properties in which beryllium differs from the rest of the members of its group (especially
the next member. Mg) are as follows :
(0 Beryllium is harder than other members of its group, ks
oo
(ii) It has higher melting and boiling points than the other members.
(Hi) Beryllium does not react with water even at high temperatures while other metals do, e.g.,
Y
B

Mg + H2O - ■> MgO + H2


(Boiling)
re

(iv) Beryllium forms covalent compounds (because of high charge density and hence greater polarizing
power) whereas other members form ionic compounds.
ou

Because of covalent character, salts of beryllium are easily hydrolysed. For example,
Y
ad

BeC03 + 4 H2O ^ [Be(H20)4]2+ + COf


(v) Beryllium oxide and hydroxide are amphoteric whereas oxides and hydroxides of other alkaline
d

earth metals are basic.


BeO + 2HC1 ^ BeCl2 + H2O
in
Re

BeO + 2NaOH ^ Na-jBeO-, + H2O


Sod. beryllate
F

(v/) Carbides of Be(Bc2C and BeC2) are covalent (e.g., Be = C = Be) whereas carbides of other members
are ionic e.g., CaC2 is Ca^"*" (C = C)^"
(vii) Beryllium carbide reacts with water to produce methane gas whereas carbides of other alkaline
earth metals give acetylene gas. Thus,
Be2C + 4 H2O ●> 2 Be(OH)2 + CH4
CaC2 + 2 H2O ^ Ca(OH)2 + HC = CH
(viii) Beryllium does not exhibit coordination number of more than four since its valence shell (n = 2)
has only four orbitals ; one s- and three p-. The remaining elements of this group can show a coordination
number of six by making use of rZ-orbitals in addition to s- and y?-orbitals.
S-BLOCK ELEMENTS (ALKALI AND ALKALINE EARTH METALS) 10/41

10.18. DIAGONAL RELATIONSHIP OF BERRYLIUM WITH ALUMINIUM


Just as lithium shows resemblance with its diagonally opposite element magnesium, in a similar way,
beryllium shows resemblance with its diagonally opposite element aluminium. This is due to the reason that
these two elements have the same electronegativity (Be = 1-5, A! = 1-5) and the polarising power
i.e. charge/radius ratio, = 2/31 = 0 064 and AP’*' = 3/50 = 0 060) of their ions are very similar.
Some points of similarity are given below :
(/) Both metals have a tendency to form covalent compounds, e.g., the chlorides of both (i.e. BeCl2 and
AICI3) being covalent are soluble in organic solvents.
(/'O Both BeCl2 and AICI3 act as strong Lewis acids and are used as Friedel-Crafts catalysts.
(Hi) Both BeCl2 chlorine bridged structures in the vapour phase.

w
Cl Cl Cl Cl

F lo
Cl—Be Be—Cl A1 :ai
Cl Cl Cl Cl

(iv) Both Be and A1 are resistant to the action of acids due to the formation of a protective film of the
oxide on their surface. However, both are rendered passive by nitric acid,

e
Fre
(v) Both the metals dissolve in strong alkalies to form soluble complexes: beryllates [Be(OH)4]^“ and
aluminates [Al(OH)4]~
(vi) The oxides of both beryllium (BeO) and aluminium (AI2O3) are hard, high melting insoluble solids,

for
(v/i) The oxides and hydroxides of both Be and A1 are amphoteric and dissolve in sodium hydroxide
solution as well as in hydrochloric acid.
r
BeO + 2 HCl 4 BeCl2 + H20 BeO + 2 NaOH Na-)Be02 + H2O
You
Sod. berrylaie
oks
AI2O3 + 6 HCl 4 2 AICI3 + 3 H2O AI2O3 + 2 NaOH 2NaA102 + H2O
eBo

Sod. meia-aluminate

(viii) Carbides of both the metals react with water liberating methane gas.
Bc2C + 4H2O > 2Be(OH)2 + CH4
AI4C3+ I2H2O ^ 4 A1(0H)3 + 3 CH4
our
ad

(ix) Salts of both these elements form hydrated ions e.g., [Be(H20)4]“'‘’ and [A1(H20)6] 3+ ;
m aqueous
solutions.

(at) Because of similar polarising power both beryllium and aluminiumform complexes. For example,
beryllium forms tetrahedral complexes such as [BeF4]^“ and [Be(C204)2]““ and aluminium forms octahedral
dY

complexes like [AIFg]^" and [A1(C204)3]^“.


Re

SUPPLEMENT YOUR
Fin

KNOWLEDGE FOR COMPETITIONS

1. The compounds of alkaline earth metals in which the metal is divalent are diamagnetic and colourless
unless the anion is coloured. This is because they have noble gas configuration with no unpaired electrons.
2. Reduction of sulphates witit carbon gives sulphides. Therefore, most of the barium compounds are made
from barium sulphate.
A
BaS04 + 4C ■> BaS + 4 CO

3. Magnesium perchlorate, Mg(C104)2 is used as a drying agent under the name anhydrone.
4. Magnesium and calcium metals being strong reducing agents cannot be prepared by reduction of their
oxides with carbon. Instead they are prepared by electrolysis of their fused chlorides. Carnallite ore is
used as a source of MgCl2.

I
10/42 ‘Pruuieefr'4- New Course Chemistry (XI)BZSlSn

10.19. USES

Some important uses of alkaline earth metals are given below :


(/) Beryllium is used in the manufacture of alloys. Copper-beryllium alloys are used in the prepiuration
of high strength springs. Metallic beryllium is used for making windows of X-ray tubes.
(//) Magnesium is used to prepare alloys with aluminium, zinc, manganese and tin. Magnesium-aluminium
alloys (Duralumin, Al = 95%, Cu = 4%, Mn = 0-5%, and Mg = 0-5%) being light in mass are used in
construction of aircrafts. Magnesium (powder or ribbon) is used in flash powders and bulbs, incendiary
bombs and signals. A suspension of magnesium hydroxide in water (called milk of magnesia) is used as
antacid in medicine.

w
Magnesium hydroxide and magnesium carbonate are used in making tooth paste. Magnesium is also
used for ignition of thermite charge in aluminothermy and for preparing Grignard reagents (RMgX) which
are widely used in organic synthesis.

F lo
(///) Calcium is used in the extraction of metals from their oxides which are difficult to reduce with
carbon.

(iv) Calcium and barium metals due to their reactivity towards oxygen and nitrogen at elevated

ee
temperatures have often been used to remove last traces of air from vacuum tubes,
(v) Radium salts are used in radiotherapy, e.g., treatment of cancer.

Fr
10.20. SOME IMPORTANT COMPOUNDS OF MAGNESIUM AND CALCIUM

for
The large scale preparation of some industrially important compounds of magnesium and calcium are
described below :
ur
10.20.1. Magnesium Chloride Hexahydrate*: MgCl2.6 HjO
Preparation. (/) From Carnallite. The main source of magnesium chloride hexahydrate is the mineral
s
ook
Camallite (KCl.MgCl2.6 H2O). The mineral is powdered and then boiled with water. Upon cooling, KCl
Yo
crystallises out while MgCl2 remains in the mother liquor. The mother liquor is concentrated and upon cooling
gives crystals of MgCl2.6 H20-
eB

(//) From sea water Sea water also contains magnesium chloride. Sea water is concentrated and then
treated with lime when magnesium hydroxide gets precipitated.
MgCl2 +Ca(OH)2 > Mg(OH)2 + CaCl2
our
ad

(Sea waler) (.Ppr)

The precipitate is filtered and dissolved in HCl. Upon concentration and cooling, MgCl2-6 H2O separate
out.
Y

(Hi) In the laboratory, magnesium chloride is prepared by the action of HCl on magnesium oxide or
carbonate.
Re
nd

MgO + 2 HCl 4 MgCl2 + H2O


MgCOj + 2 HCl MgCl2 + CO2 + H2O
Fi

The reaction mixture is concentrated and then cooled when crystals of MgCl2.6 H2O separate out.

Properties. (/) It is a colourless deliquescent solid of bitter taste. It is highly soluble in water and melts
at 391 K.

(//) Action of heat. On gentle heating, it undergoes hydrolysis to form basic magnesium chloride which
upon iginition gives magnesium oxide :

MgCl2.6 H2O ^ Mg(OH)Cl + HCl + 5 H2O


Ignition
Mg(OH)Cl » MgO + HCl

*Included in the syllabus of ISCE.

1
S-BLOCK ELEMENTS (ALKALI AND ALKALINE EARTH METALS) 10/43

Anhydrous MgCU can, however, be prepared by heating MgCl2.6 H2O at 443 K in dry HC! gas
DryHClgas
MgCl2.6 H2O 443 K
^ MgCl2 + 6 H2O

Here, excess of HCl prevents the hydrolysis of MgCl2 by its own water of crystallisation.
(Hi) Reaction with MgO. A concentrated solution of MgCl2 on mixing with MgO forms a thick paste

w
known as Sorel’s cement or Magnesia cement with the composition MgCl2.5 MgOj: H2O. It sets into hard
marble like mass.

Uses. (/) Sorel cement is used for making synthetic tiles, filling teeth and repairing porcelain and glass
wares.

(ii) It is used in medicine as a purgative.

o
(Hi) Magnesium metal is obtained by electrolysis of fused MgCl2.

e
re
10.20.2. Calcium Oxide, (Quick Lime), CaO

rFl
Preparation. Calcium oxide is called quick lime. It is prepared by heating limestone in a rotatory kiln

F
at 1070-1270 K.

1070-1270 K
CaCOj V i CaO + CO2 ; AH = + 179-9 kJ

r
Since the reaction is reversible, therefore, the CO2 is removed as soon as it is formed to enable the
ou
fo
reaction to proceed to completion.
Further, the temperature is not allowed to rise above 1270 K otherwise, silica present as impurity in
limestone will react with CaO to form calcium silicate

Above 1270 K ks
oo
CaO + Si02 4
CaSi03
Calcium silicate
Y
Properties, (i) It is a white amorphous solid, m.p. 2870 K.
B

(ii) When heated in oxyhydrogen flame, it emits brilliant white light called limelight.
re

(Hi) When exposed to atmosphere, it absorbs moisture and carbon dioxide forming slaked lime and
calcium carbonate respectively.
CaO + H2O Ca(OH)2 CaO + COt CaC03
ou

■>
Y
ad

moisture slaked lime calcium carbonate

(iv) It is usually obtained in form of hard lumps. The addition of limited amount of water breaks these
lumps. During the process, a hissing sound and a large amount of heat is evolved which converts water into
d

steam. This process is called slaking of lime and the fine powder thus obtained is called slaked lime.
4 Ca(OH)2 ; AH = - 64-5 kJ mol"'
in

CaO + H-)0
Re

(v) Quick lime when slaked with caustic soda gives a solid called soda-lime (CaO + NaOH).
F

(vi) Action of acids and acidic oxides. It is a basic oxide and hence combines with acids and acidic
oxides at high temperature forming salts.
A
CaO + 2HCI CaCl2 + H2O CaO + Si02 ■> CaSi03
A

6 CaO + P4O10 4 2 Ca3(P04)2 CaO -f SOo 4 CaS03


(vii) Reaction with coke. When heated with coke in an electric furnace at 2273 K, it forms calcium
carbide.

2273 K
CaO + 3C CaC2 + CO
10/44 ‘Pn^uieep.’^ New Course Chemistry (XI)EEIHD

iviii) Reaction with ammonium salts. On heating with ammonium salts, it liberates ammonia gas.
A
CaO + 2 NH4CI > CaCl2 + 2 NH3+ H2O
Uses, (i) It is an important primary material and is the cheapest form of alVali.
(//') It is employed in the purification of sugar and in the manufacture of dyestuffs,
(m) As a constituent of mortar, it is used on a very large scale in building constructions.
(iv) In the preparation of cement, glass, calcium carbide and sodium carbonate (from caustic soda),

w
(v) It is used in the preparation of ammonia and soda-lime (CaO 4- NaOH).
ivi) It is used for drying alcohols and non-acidic gases, (vii) It is used as a basic lining in furnaces.
(viii) It is used as a flux in metallurgy.

o
10.20.3. Calcium Hydroxide, (Slaked Lime), CaiOH)^
Preparation. (/) From quick lime. Calcium hydroxide is prepared on a commercial scale by adding

e
Fl
re
water to quick lime. This process is called slaking of lime.
CaO + H2O > Ca(OH)2

F
During the process of slaking, lumps of quick lime crumble to a fine powder,
(ii) From calcium chloride. It is also obtained by treating calcium chloride with caustic soda.
ur
CaCl2 + 2 NaOH > Ca(OH)2 + 2 NaCl

r
fo
Properties. (/) It is a white amorphous powder sparingly soluble in water, the solubility decreasing
further with rise in temperature. The suspension of slaked lime in water is called milk of lime while the
ks
filtered and clear solution is known os lime water. Chemically both are calcium hydroxide.
Yo
(ii) Action of heat. On heating, slaked lime loses water only at temperatures greater than 700 K
oo
>700 K
Ca(OH)2 > CaO + H2O.
eB

(///) Reaction with chlorine. Slaked lime reacts with chlorine to form calcium hypochlorite, a constituent
of bleaching powder
2 Ca(OH)2 + 2 CI2 4 CaCl2 + Ca(OCl)2 + 2 H2O.
ur

Bleaching powder
(iv) Reaction with carbon dioxide. When carbon dioxide is passed through lime water, it turns milky
ad
Yo

due to the formation of insoluble calcium carbonate.

Ca(OH)2 + CO2 4 CaC03>l + H2O


(Milkiness)
d

On passing excess of carbon dioxide, the precipitates of calcium carbonate dissolve to form soluble
Re
in

calcium bicarbonateand hence the milkiness disappears.


CaC03 CO2 + H2O > Ca(HC03)2
F

(Soluble)
If this clear solution of calcium bicarbonate is heated, the solution again turns milky due to the
decomposition of calcium bicarbonate back to calcium carbonate.
. _ _ Heat
Ca(HC03)2 (aq) 4
CaC03 (5) i + CO2 (g) + H2O (/)
(v) Reaction with acids. Slaked lime being a strong base reacts with acids and acidic gases forming
salts.

Ca(OH), + 2 HCl 4 CaCl2 + H2O ^ Ca(OH)2 + SO3 4 CaS04 + H2O


However, Ca(OH)2 does not dissolve in dil. H2SO4 because the calcium sulphate formed is sparingly
soluble in water.
S-BLOCK ELEMENTS (ALKALI AND ALKALINE EARTH METALS) 10/45

Uses. (0 Slaked lime is used as a building material in form of mortar. It is prepared by mixing slaked
lime with 3-4 times its weight of sand. The mixture is made into a thick paste with gradual addition of water.
The paste is called mortar and is used in building construction. It sets into a hard mass by loss of H->0 and
gradual absorption of CO2 from the air. Sometimes coal ash is mixed with lime instead of sand,
(n) It is used in white washing because of its disinfectant properties.
(Hi) It is used in the preparation of ammonia from ammonium chloride and in the softening of hard water,
(/v) It is used in the manufacture of bleaching powder,

w
(v) It is used in glass making, tanning industry and purification of sugar.
(vO It is used for absorbing acidic gases.
(vii) In form of lime water, it is used for detection of CO-,.

10.20.4. Calcium Carbonate, CaCOj

o
Calcium carbonate is also called limestone. It occurs
in nature as chalk, marble, corals, calcite, aragonite,
etc. Mixed with magnesium carbonate, it occurs as dolomite.

e
Fl
re
Preparation. (/) From slaked lime. It can be prepared by passing a calculated quantity of COt through
slaked lime.

F
Ca(OH>2 + CO2 ■> CaCOj 4- + H2O
(ii) From calcium chloride. It can be prepared by adding an aqueous solution of sodium carbonate to
calcium chloride. ur
r
CaCl2 + Na2C03 > CaCO-,3 i + 2 NaCl.

fo
Properties. (/) It is a white solid almost insoluble in water.
(ii) When heated to 1070-1270 K, it decomposes to evolve CO2.
1070-1270 K ks
Yo
CaC03 ^ CaO + CO2
oo
(Hi) It reacts with dilute acids liberating CO2.
CaCO-, + 2 HCl CaCl2 + ^^2 ^ H2O CaCOj + H2SO4 4 CaS04 + CO2 + H2O
eB

Uses. Its main uses are : {/) Calcium carbonate along with magnesium carbonate is used as a flux in the
extraction of metals such as iron.

(ii) Specially precipitated CaC03 is extensively used in the manufacture of high quality paper.
(Hi) It is also used as an antacid, mild abrasive in tooth paste, a constituent of chewing gum and a filler
ur

in cosmetics.
ad

(iv) It is used as a raw material for the manufacture of sodium carbonate in Solvay-ammonia process,
Yo

(v) It is used as a building material in form of marble and in tlie manufacture of quick lime.
10.20.5. Plaster of Paris, CaS04.1/2 H2O or (CaS04)2.H20
d

Plaster of Paris is calcium sulphate hemihydrate, CaS04. 1/2 H2O.


Re
in

Preparation. It is prepared by heating gypsum to 393 K.


393 K
2 CaS04.2 H2O - > 2 CaS04. 1/2 H2O + 3 H2O
F

Gypsum Plaster of Paris

The following conditions are necessary :


(0 The temperature should not be allowed to rise above 393 K because above this temperature, the
whole of water of crystallisation is lost. The resulting anhydrous CaS04 is called dead burnt plaster because
it loses the properties of setting with water.
>393 K
2CaS04. 1/2 H2O 2 CaS04 + H2O
Plaster of Paris Anhyd. calcium sulphate
(Dead burnt plaster)
(ii) The gypsum should not be allowed to come in contact with carbon containing fuel otherwise some
of it will be reduced to calcium sulphite.
10/46 “Pn^neUe^'A New Course Chemistry fxnrogm

Properties. (/) It is a while powder.


(«) It has a remarkable property of setting with water. On mixing with one-third its weight of water, it
forms a plastic mass which sets into a hai’d* mass of interlocking crystals of gypsum within 5 to 15 minutes.
It is due to this reason that it is called plaster. The addition of common salt accelerates the rate of setting,
while a little borax or alum reduces it. TTie setting of Plaster of Paris is believed to be due to rehydralion and
its reconversioninto gypsum.
2 CaS04. 1/2 HoO + 3 H2O 4 2 CaS04. 2 H2O

ow
Plaster of Paris Gypsum
During the process of setting, slight expansion (1%) in volume occurs. As a result, it can take up the
shape and impression of the mould in which it is put.
Uses. (/) Plaster of Paris is used for producing moulds for pottery and ceramics and casts of statues and
busts. It is also u.sed in building industry as well as plasters.
(ii) It is used for making statues, models and other decorative materials.

e
(Hi) It is used in surgical bandages used for plastering broken or fractured bones of the body and for

re
preparing black board chalks,

rFl
(/v) It is also used in dentistry.

F
10.21. CEMENT

Its use was discovered in 1824 by Joseph Aspdin, a mason of Leeds (U.K.). He found that when a
strongly heated mixture of limestone and clay was mixed with water and allowed to stand for some time, it

or
ou
hardened to a stone like mass resembling Portland rock which was a popular building stone in England those
days. He, therefore, named it as portland cement.

ksf
Cement is essentially a finely powdered mixture of calcium silicates and aluminates along with
small quantities of gypsum which sets into a hard stone like mass when treated with water.
oo
10.21.1. Average Composition of Portland Cement
Cement is a product obtained by combining a material rich in lime. CaO with other material such as
Y
B

clay, which contains silica, Si02 along with the oxides of aluminium, iron and magnesium.
The average composition of portland cement is
Lime (CaO) 50 - 60%
re

Magnesium oxide (MgO) 2-3%

Silica (Si02> 20 - 25%


oYu

Ferric oxide {Fe^03) 1-2%


ad

Alumina (A1^03) 5 - 10%

Sulphur trioxide (SO3) 1 -2%


d

For a good quality cement, the ratio of alumina (AI2O3) to silica (Si02) should lie between 2-5 and 4
while that of lime (CaO) to the total of silica (Si02), alumina (AI2O3) and ferric oxide (Fe203) should be as
in
Re

close to 2 as possible.
10.21.2. Raw Materials
F

The essential raw materials required for the manufacture of cement are : (0 limestone CaC03,
(ii) clay which provides both silica and alumina and (Hi) gypsum, CaS04.2H20. Besides these, small atnounis of
magnesia (MgO) and iron oxide (Fe'>03) are also required for imparting suitable colour to cement.
10.21.3. Manufacture of Cement
Portland cement is chiefly manufactured by the following two processes :
(/) Wet Process and (ii) Dry Process.
The wet process is preferred when the raw materials i.e., limestone and clay are soft, climate is humid
and the fuel is cheap. The dry process is preferred when the raw materials are hard.
*The hardening of plaster of Paris is due to transition of orthorhombic form of gypsum into monoclinic form of
gypsum.
s-BLOCK ELEMENTS (ALKALI AND ALKALINE EARTH METALS) 10/47

The main raw materials are lime stone and clay whereas the former supplies lime while the latter is a
source of silica, alumina and iron oxide. The raw materials are first crushed separately in a suitable machine.
They aie then mixed in the required proportions (3 parts lime stone + one part clay) and ground together
finely. This grinding is done either by the diy process or by the wet process.
In the wet process, the clay is washed with water in a wash mill to remove flint and other foreign
substances. It is then mixed with requisite quantity of limestone and pulverised in a special mill. The resulting
pasty mass is then thoroughly homogenised to get ‘raw slurry' containing about 40% water.
In the dry process, the raw materials are dried and mixed in desired proportions. The mixture is then
finely powdered and passed through 300 mesh sieves and homogenised with the help of compressed air. The
homogeneous mass is called ‘raw meal’.
The sluny or the raw meal is

w
FIGURE 10.6
introduced into the upper end of a RAW MATERIAL
rotary kiln (Fig. 10.6) by means of >

screw conveyer. The kiln consists IT7

F lo
COAL DUST
SCREW
of an inclined steel rotating I
CONVEYER
HOPPER

cylinder, 150—200 ft long and O


BURNER
AIR
about 10 ft in diameter lined inside
BLOWER

ee
with firebricks. The charge travels ROTARY KILN
downwards slowly due to the

Fr
rotatory motion given to the kiln
HOTAIR
and is heated by burning coal dust DUST
CHAMBER DUST CHAMBER
which is blown in from the lower

for
end. The charge takes about 2—3 COLD CLINKER DISCHARGE
COOLER
ur
hours to cover the entire journey
in the kiln. In the kiln, the following Manuiacturc of Portland Cement
changes occur:
s
ook
(/) In the upper part of the kiln, the temperature is around 1000—) 100 K. Here, the charge loses all its
Yo
water due to evaporation by hot gases.
(«■) In the middle of the kiln, the temperature is around 1100-1200 K. Here, limestone decomposes to
eB

form calcium oxide and carbon dioxide.

CaCO^ ^ CaO -H CO2


{Hi) The lower part of the kiln, from where coal dust is blown in is the hottest with the temperature
our
ad

ranging between 1770—1870 K. As the charge reaches here, chemical combination takes place between
lime, alumina and silica to form calcium silicates and aluminates :
2CaO + SiOj 2 CaO.Si02
Y

Dicatcium silicate

3 CaO + Si02 3 CaO.Si02


Re
nd

Tricalcium silicate

3 CaO + AI2O3 3 Ca0.Al203


Fi

Tricalcium aluminate

2 CaO + A1-503 2 Ca0.Al203


Dicalcium aluminate

4 CaO -I- AI2O3 -f- Fe^03 4 Ca0.Al203.Fe203


Tetracalcium aluminoferrite

Due to very high temperature in this zone, about 20—30% mass melts and combines with solid mass to
form grey coloured balls (ranging in size from a pea to that of an egg) called cement clinkers.
The hot clinkers are cooled by cold air and are mixed with 2—3% gypsum (to slow down the process of
setting of cement and thereby imparting greater strength to it) and finely powdered to 325 mesh size in
grinding machines. The fine powder called the ‘Portland cement' is sieved and packed in bags.
10/48 New Course Chemistry fXHra?TTO

The flow sheet diagram of the entire manufacturing process is given below :
Calcium silicates
Burnt at
1770-1870 K
(2Ca0.Si02, 3Ca0.Si02) Addition of
Limestone and clay are + Calcium aluininates ► Portland cement
gypsum
powdered and mixed in a rotatory kiln
(3CaO. AI2O3, 2CaO. AI2O3)
{Cement Clinker)

10.21.4. Setting of Cement


The important constituents present in Portland cement are dicalcium silicate (€328104), 26%, tricalcium

w
silicate (€338105), 51% and tricalcium aluminate (Ca3Al20(^), 11%.
When water is added to cement, an exothermic reaction occurs. During this process, cement reacts with
water to form a gelatinous mass which slowly sets into a hard mass having three-dimensional network structure

F lo
involving —Si—O—Si— and 8i—O—A1— chains.
Out of the various constituents of cement, the most important is tricalcium silicate. It sets quickly and
develops considerable strength within a few days. Dicalcium silicate sets slowly and develops appreciable

ee
strength after a month or so. TricaJcium aluminate sets instantaneously in presence of water. The internal
strength acquired by cement is primarily due to the setting of tricalcium aluminate. Tetracalcium aluminoferrite

Fr
also sets rapidly but not as quickly as tricalcium aluminate.
10.21.5. Cement Substitutes

for
In the present industrial age, the demand for cement is increasing day by day. To meet this increasing
ur
demand, efforts have been made to find some substitute of cement. One of these is fly ash which is a waste
product from steel industry and mainly consists of calcium silicate (CaSi03). This can be added to cement to
reduce its cost without affecting the quality. In many countries, rice husk with high silica content has been
s
used to make cement.
ok
Yo
10.21.6. Cement Industry in India
o

India is one of the major cement producing countries in the world. At present more than 60 units, both
eB

in private as well as public sectors, are engaged in the manufacture of cement. Their combined annual production
of cement is about 21 million tons. Of these, Associated Cement Company Ltd, (A.C.C), Dalmia Cement Ltd.,
and Cement Corporation of India (C.C.I—a Govt, undertaking) are among the largest manufacturing groups.
In Haryana, there are two cement factories, one at Surajpur and the other at Charkhi Dadri.
r
ou
ad

10.22. BIOLOGICAL IMPORTANCE OF MAGNESIUM AND CALCIUM

Magnesium and calcium ions play important role in biological processes. An adult body contains about
Y

25 g of Mg and 1200 g of Ca compared with only 5 g of Fe and 0-06 g of Cu. The daily requirement of Ca^"^
and Mg^'*’ ions is 200-300 mg. Mg^"*" ions are concentrated in cells, and Ca^'*’ ions are concentrated in the
Re

body fluids outside the cell, in much the same way as K'^ concentrates inside the cell and Na'*’ outside.
nd

Mg^'*’ ions catalyse a number of enzymatic reactions. We know that energy is stored in form of phosphate
linkages in ATP. The formation of these linkages, i.e., storage of energy is catalysed by Mg^'^ ions. Conversely,
Fi

2+
hydrolysis of phosphate linkages is accompanied by release of energy. This process is also catalysed by Mg-
ions.

Mg^"*" ions are present in chlorophyll-a, the green colouring pigment of plants, which absorbs light and
is essential for photosynthesis.
About 99% of body calcium is present in bones and teeth as hydoxyapatite, Ca5(P04)30H, and the
enamel on teeth as fluoropatite [3 Ca3(P04)2 . CaF2]. Ca-"^ ions are also important in blood clotting and are
required to trigger the contraction of muscles and to maintain the regular beating of the heart.
The concentration of Ca^'*’ ions in blood plasma is about 100 mg L“*. This concentration is maintained
by two hormones called calcitonin and parathyroid. Do you know that bone is not an inert and unchanging
substance ? The calcium ions in bones are continuously dissolving into and redepositing from the blood
s-BLOCK ELEMENTS (ALKALI AND ALKALINE EARTH METALS) 10/49

plasma to the extent of 400 mg per day in man. In normal adult this exchange is in balance, but in elderly
people, especially women, there is sometimes net loss of bone calcium, leading to the disease called
osteoporosis.

10.23. TYPICAL PROBLEMS

P. 1. Find out the oxidation number of K in K2O, K2O2 and KO3. Is it the same or different ? Illustrate.
Sol. In all these compounds, oxidation number (O.N.) of K is the .same, i.e., + 1 as shown below. This is
because O.N. of oxygen is different in all these three compounds.
If A- is the O.N. of K, then
_2

w
(0 In K2 O or 2 A - 2 or a = + 1
X -1

(fV) in K., O2 or 2a-2 or A = + 1

F lo
(O.N. of each oxygen atom in peroxides is - 1).
X -1/2 j
(Hi) In K O2 or A - 2 X —
2
or a = + I

ee
Fr
(the average O.N. of each oxygen atom in superoxide, : O-O is - 1/2.

P. 2. Calcium burns in nitrogen to produce a white powderwhich dissolvesin sufficientwater to produce

for
a gas (A) and an alkaline solution. The solution on exposure to air produces a thin solid layer of (B)
on the surface. Identify the compounds A and B.
ur
Sol. Ca burns in air to form CaO and Cu3N2.
s
A A
2 Ca + O2 ^ 2 CaO 3 Ca + No
ook
Yo
Calcium nitride on hydrolysis with H2O gives ammonia (A)
Ca3N2 + 6 H2O ^ 3 Ca(OH)2 + 2 NH3
eB

The alkaline solution of Ca(OH)2 thus formed reacts with CO2 present in the air to form CaC03 which
being insoluble forms a thin solid layer on the surface.
our

Ca(OH)2 + CO2 CaC03 + H2O


ad

Thus, A = NH3 and B = CaC03 (white powder)


Y
Re
nd
Fi

I. Alkali Metals

1. Members of the group. Except hydrogen, the other six elements of group 1, i.e., lithium (Li), sodium
(Na), potassium (K), rubidium (Rb), caesium (Cs) and francium (Fr) are called alkali metals since their
oxides and hydroxides are soluble in water and form strong alkalies. Francium is, howeve,r radioactive.
General electronic configuration. The general electronic configuration of alkali metals is [noble gas]
n.?’ where n = 2-1. Due to the presence of one unpaired electron, all alkali metals are paramagnetic but
their salts are diamagnetic.
3. Atomic radii. The atomic radii of alkali metals are the largest (after inert gases) in their respective
periods. Both atomic and ionic radii increase down the group from Li to Cs as the number of inner filled
shells and screening effect increases.
10/50 New Course Chemistry (XI)iSSIHD

4. Enthalpy of hydration. The enthalpy of hydration of alkali metal ions decreases with increase is ionic
radii, i.e.. Li"^ > Na"^ > K'^ > Rb"^ > Cs^ In other words, degree of hydration decreases down the group from
Li'*’ to Cs'*’. Consequently, radii of hydrated ions decreases in the order : Li'*’ > Na"*" > K"'’ > Rb'*’ > Cs.
Conversely, the ionic mobility decreases in the opposite order, i.e., Li'*’ < Na'*’ < K'*’ < Rb^ < Cs'*’
Further since Li has the maximum degree of hydration, therefore, several Li salts are hydrated, i.e.,
LiClO^.SH^O, Li2C03.3H20, LiC1.2H20, etc.
5. Ionization enthalpy. The first ionization enthalpy (AjH/) of alkali metals is quite low and decreases

ow
further down the group due to increasing atomic size and shielding effect. Further, due to inert gas
configuration of the uniposilive ions the second ionization enthalpies (A,H2) of alkali metals are very high
and decrease down the group from Li to Cs.
6. Electropositive character. All alkali metals are strongly electropositive due to their low ionization
enthalpies. Further, the electropositive character increases down the group as the ionization enthalpies
decrease. Thus, Fr in the most metallic and electropositive element in the periodic table.

e
Further, due to the presence of a single electron outside the noble gas core, all the alkali metals have a

re
strong tendency to lose this electron to acquire the electronic configuration of the nearest noble gas.

rFl
Consequently, all the alkali metals uniformly show an oxidation state of+J.

F
7. Melting and boiling points. The melting points and boiling points of alkali metals are quite low and
decrease further down the group due to weakening of metallic bonds. Thus, amongst alkali metals, Li has
the highest and Cs has the lowest m.p. and b.p. while Fr is a liquid at room temperature.

or
ou
8. Densities. The densities of alkali metals are quite low ; Li, Na and K are even lighter than water. The
densities, however, increase down the group from Li to Cs due to an increase in their atomic mass which

ksf
more than compensates the bigger atomic size. K, however, is lighter than Na probably due to an abnormal
increase in atomic size of K as compared to its atomic mass.
9. Flame colouration. Due to low ionization enthalpies, all the alkali metals and their salts impart
oo
characteristic colours to the flame. For example.
Metal Li Na K Rb Cs
Y
B

Colour Crimson red Golden yellow Pink violet Red violet Sky blue
Xynm (670-8 nm) (589-2 nm) (766-5 nm) (7800) (455-5 nm)
re

10. Photoelectric effect. Due to low ionization enthalpies, all alkali metals except Li show photoelectric
effect. Due to strong photoelectric effect, K and Cs are commonly used in photoelectric cells.
11. Nature of bonding. Due to low ionization enthalpies, alkali metals form ionic compounds. Further, the
oYu

ionic character increases down the group due to decreasing ionization enthalpies. Lithium, however, because
ad

of its high ionization enthalpy forms covalent compounds, i.e., alkyllithium (R-Li), aryllithium (Ar-Li),
etc.
d

12. Reducing character. Due to large negative electrode potentials, alkali metals are strong reducing agents.
The reducing power increases from Na to Cs but Li is the .stronge.st reducing agent due to the large
in
Re

enthalpy of hydration of the Li'*' ion.


13. Action of water. All alkali metals react with water evolving H2 and forming their corresponding metal
F

hydroxides. 2 M + 2 HoO 2 MOH + H2


Lithium is the least reactive while the reactivity of other alkali metals increases down the group as the
electropositive character of the metal increases : Li < Na < K < Rb < Cs

14. Basie character. Alkali metal hydroxides are strong bases and their solubility and basic character increases
down the group from Li to Cr as the ionization enthalpy of the metal decreases or the electropositive
character of the metal increases. Thus, basicity increases in the order : LiOH < NaOH < KOH < RbOH
< CsOH.

15. Action of oxygen. When heated with oxygen, alkali metals form different types of oxides depending upon
the nature of the metal. The reactivity of alkali metals towards oxygen increases down the group from Li
to Cs. For example, Li forms Li20 (lithium oxide), Na forms mainly Na202 (sodium peroxide) with some
Na^O (sodium oxide) while all other alkali metals form superoxides, i.e., KO2, Rb02 and Cs02-
S-BLOCK ELEMENTS (ALKALI AND ALKALINE EARTH METALS) 10/51

KO2 is used as a source of02 in submarines, space shuttles and oxygen masks : The moisture present in
the breath liberates O2 and KOH thus formed removes the CO2.
4 KO2 + 2 H2O 4 KOH + 3 Oo KOH + CO2 > KHCO 3
16. Action of air and moisture. Alkali metals get tarnished when exposed to air and moisture due to the
formation of first oxides, then hydroxides and finally carbonates. Because of these reactions, alkali metals
are stored in inert hydrocarbon solvents like petroleum ether and kerosene oil which prevent them from
coming in contact with air and moisture.
17. Action of hydrogen. All alkali metals on heating with hydrogen form ionic hydrides of the general formula
M'*' H~. The reactivity of alkali metals towards hydrogen decreasesdown the group from Li to Cs due to
the reason that lattice enthalpies and hence the stability of these hydrides decreases down the group as the

w
size of the metal cation increases, i.e., stability decreases in the order :
LiH > NaH > KH > RbH > CsH

Besides these simple hydrides. Li and Na form complex hydroides, i.e., L1A1H4 and NaBH4 which are

F lo
powerful reducing agents and are widely used in organic chemistry.
18. Action of halogens. Alkali metals combine directly with halogens to form metal halides (MX). With the
exception of lithium halides, all other alkali metal halides are ionic.

ee
The reactivity of alkali metals towards a particular halogen increases in the order. Li < Na < K < Rb < Cs
while that of halogens towards a particular alkali metal decreases in the order ; F2 > C1-, > Br2 > I->.

Fr
(fl) Solubility of alkali metal halides. With the exception of LiF, all other lithium halides are covalent.
Being covalent, LiCI, LiBr and Lil are insoluble in water but are soluble in organic solvents like pyridine,
benzene, alcohols and ethers.

for
(b) Melting points and boiling points. (0 For the same alkali metal, the melting points decrease in the
ur
order: fluorides > chlorides > bromides > iodides due to a corresponding decrease in their lattice enthalpies.
Thus, the m.ps. of sodium halides decrease in the order: NaF (1261 K) > NaCl (1084 K) > NaBr (1028 K)
> Nal (944 K).
s
(//) For the same halide ion, lithium halides being covalent have lower melting points than their
ook
Yo
corresponding sodium halides. Thereafter, the melting points decrease due to a corresponding decrease in
lattice enthalpies as the size of the metal increases. Thus, the m. ps. of alkali metal chlorides decrease in
eB

the order: NaCl (1084 K) > KCl (1039 K) > RbCl (988 K) > CsCI (925 K) > LiCi (887 K).
19. Solubility in liquid ammonia. All the alkali metals dissolve in liquid ammonia to give blue solutions due
to the presence of ammoniated (solvated) electrons in the .solution.
M + (a: + y) NH3 4
(NH3), + ^-(NH3),
r

As a result, the solutionsof alkali metals in liquid ammonia are widely u.sed as reducing agents in organic
ou
ad

chemistryunder the name Birch reduction.


20. Stability of carbonates.The stability of carbonates towards heat increases down the group as the basic
Y

character of the alkali metal hydroxides increases down the group. Li2C03 is, however, less stable and
decomposes on heating
U2CO3 Li20 + CO2
Re
nd

Lithium forms bicarbonate in solution while all other alkali metals form solid bicarbonates. All the
bicarbonates on gentle heating undergo decomposition to form carbonates with the evolution of COt
Fi

2 MHCO3 M2CO3 + CO2 + H2O


All the carbonates and bicarbonates are soluble in water and their solubility increases down the group.
21. Lithium differs from other alkali metals because of its (/) small size, («) high polarising power,
(Hi) high ionization enthalpy and (iV) absence of d-orbitals in the valence shell.
22. Lithium shows diagonal relationship with magnesium mainly due to the similarity in sizes of their
atoms and ions.

23. Alkali metals cannot be extracted (/) by reduction of their oxides or chlorides, (//) by displacement from
aqueous solutions of their salts by other metals or by electrolysis of the aqueous solutions of their salts.
They are actually prepared by electrolysis of their fused salts usually chlorides. For example, .sodium is
extracted by electrolysis of a fused eutectic mixture of sodium chloride (40%) and calcium chloride (60%)
in a Down’s cell at 873 K using graphite anode and iron cathode.
10/52 'P’ta^eUe^'^ New Course Chemistry (XI)BZsISD

24. Sodium carbonate (washing soda), Na^CO^.lO H2O is prepared by Solvay ammonia process. In this
process, CO2 is passed through brine (sodium chloride) solution saturated with ammonia, when sodium
bicarbonate being sparingly soluble separates out. It is filtered and then ignited to get sodium carbonate.
NaCl + H2O + CO2 + NH3 > NaHC03 + NH4CI
2 NaHC03 Na^COg + CO2 + H^O
Potassium carbonate cannot be prepared by this process because KHCO3 is fairly soluble in water. As a
result, when CO2 is passed through ammoniated brine, KHCO3 does not separate out.
25. Pure sodium chloride is not hygroscopic. However, impure sodium chloride gets wet in rainy season due
to the presence of impurities like MgCl2. CaC/2. MgSO^ and CaSO^ all ofwhich are hygroscopic. Therefore,
to prevent it from absorbing moisture, Ca3(P04)2 or Ca(H2P04)2 is added to the table salt.

w
26. Sodium hydroxide is prepared commercially by electrolysis of an aqueous solution of sodium chloride
using mercury cathode and graphite anode in a Castner and Kellner’s cell.

F lo
27. Baking soda is NaHCO^ and is obtained by saturating a solution of Na2C03 with CO2 when NaHC03
being less soluble separates out.
Baking powder is a mixture of NaHC03 (30%), starch (40%), calcium dihydrogen phosphate, Ca(H2P04)2
(10%) and sodium aluminium sulphate, NaAl(S04>2. Whereas starch is used as a filler, NaAl(S04)2 slows

ee
down the reaction so that CO2 is given off more slowly.
28. Biological importance. Na'*’ ions are primarily found outside the cells in blood plasma and other interstitial

Fr
fluids, while K'’’ ions are present inside the cells. Na"^ ions help in the transmission of nerve signals, in
regulating the flow of water across cell membranes and in transport of sugars and amino acids into the

for
cells. K'*’ ions which are more abundant within the cell fluids, activate many enzymes which are responsible
for oxidation of glucose to produce ATP.
r
II. Alkaline Earth Metals
You
Members of the group. Group 2 of the periodic table consists of six elements: beryllium (Be), magnesium
s
(Mg), calcium (Ca), strontium (Sr) and radium (Ra). Radium is, howeve,r radioactive. These elements are
ook

also called alkaline earth elements because the oxides of Mg, Ca, Sr and Ba are found in earth’s crust and
are alkaline in mixture.
eB

29. General electronic configuration. The general electronic configuration of alkaline earth metals is
[noble gas] ns^ where n = 2 - 7.
30. Atomic and ionic radii. The atomic and ionic radii of alkaline earth metals ore fairly large though
smaller than those of the corresponding alkali metals.
our
ad

31. Ionization enthalpy. The first and second ionization enthalpies of alkaline earth metals are fairly low ;
the second ionization enthalpy (AjH2) being almost double of the ifrst ionization enthalpy
32. Hydration enthalpy. Like alkali metal ions, the hydration enthalpies of alkaline earth metal ions decrease as
the size of the ion increases down the group, i.e.. Be^'*' > Mg^'*' > Ca^^ > Sr^'*’ > Ba^^.
Y
Re

33. Oxidation state. All alkaline earth metals uniformly show an oxidation state of +2 both in the solid state
or in the aqueous solutions. In the solid state, the greater lattice enthalpy of the M^"*" ions (relative to M'*'
d

ions) more than compensates the higher second ionization enthalpy while in the aqueous solution, it is
Fin

the greater hydration enthalpy of the M^'*’ ions (relative to M'*' ions) which more that compensates the
higher second ionization enthalpy.
34. Electropositive character. Alkaline earth metals are strongly electropositive since they have a strong
tendency to lose both the valence electrons to form M"'*' ions having noble gas configuration. However,
due to small size and higher ionization enthalpies (A^H] + A,H2), they are less electropositive than alkali
metals. Further, the electropositive character increases down the group (i.e., Be < Mg < Ca < Sr < Ba) as
the ionization enthalpies decrease.
35. Melting and boiling points. The melting points and boiling points of alkaline earth metals are low but
higher than those of alkali metals. However, down the group, the melting points and boiling points do not
show any regular trend.
36. Density. The alkaline earth metals are denser than the alkali metals due to smaller size and better packing
of the atoms in the crystal lattice.
s-BLOCK ELEMENTS (ALKALI AND ALKALINE EARTH METALS) 10/53

37. Flame colouration. Like alkali metal salts, alkaline earth metals and their salts also impart a characteristic
colour to the flame.
Calcium Strontium Barium Radium
Brick red Crimson Apple green Crimson

Be and Mg being smaller in size, their electrons arc strongly held by the nucleus. As a result, they need
large amounts of energy for excitation of electrons to higher energy levels which is not available in the
Bunsen flame. So they do not impart any colour to the flame.
38. Reducing character. The reducing character of alkaline earth metals increases down the group as the
ionization enthalpy of the metals decreases or the electrode potential becomes more and more negative
down the group.

w
39. Action of air. (/) Alkaline earth metals on heating in air form a mixture of oxides and nitrides.
2M + 0-, ^ 2 MO 3 M + Nt 3 M3N2

F lo
Reactivity towards air, however, increases down the group from Be to Ba.
(ii) When heated in O2 ; only oxides are formed and in N2, only nitrides are formed.
(Hi) Ca, Sr and Ba also form peroxides though indirectly.
(jV) The oxides of alkaline earth metals have high melting points, are good conductors of heat and are

ee
chemically inert. Therefore, they are used as refractory materials.

Fr
40. Action of water. The reactivity of alkaline earth metals towards water increases down the group from Mg
to Ba, i.e., Mg < Ca < Sr < Ba. Be does not react with hot water or even steam. Mg reacts with hot water
but Ca, Sr and Ba react vigorously even with cold water evolving H-, gas.

for
M + 2 H2O > 2 M (0H)2 + H2 where M = Mg. Ca, Sr or Ba
ur
Be(OH)2 is amphoteric while all other alkaline earth metal hydroxides are basic in nature ; their solubility
in water and basicity increases down the group from Mg(OH)2 to Ba(OH)2-
41. Action of hydrogen. All the alkaline earth metals except Be combine with hydrogen directly on heating to
s
ook
form metal hydrides, MH->. The stabilities of these hydrides decreases down the group from Be to Ba
Yo
because their lattice enthalpies decrease down the group as the size of the metal cation increases.
Both BeH2 andMgH2 are covalent, electron deifcient molecules. They have polymeric structures involving
eB

hydrogen bridges while all other hydrides (CaH2, SrH2 and BaH2) are ionic and liberate H2 on treatment
with H2O.
CaH2 is called hydrolith and is used for large scale production of H2.
our

42. Formation of carbides. Except Be, all other alkaline earth metals react with carbon when heated in an
ad

electric furnace to form carbides of the general formula MC->. These are called acetylides because on
hydrolysis, they produce acetylene, HC = CH. MgC2 on heating forms Mg2C^ which on hydrolysis gives
propyne, CH^C s CH. BeO when heated with carbon forms BC2C. This on hydrolysis gives methane, CH^.
Y

43. Action of acids. Like alkali metals, alkaline earth metals also react with acids evolving H2 gas. Be, however,
being amphoteric dissolves both in alkalies and acids liberating H2 gas.
Re

44. Birch reduction. Like alkali metals, alkaline earth metals dissolve in liquid ammonia forming blue coloured
nd

solutions due to formation of ammoniated electrons. These blue solutions are widely used as reducing
agents in organic chemistry under the name Birch reduction.
Fi

45. Action of halogens. When heated with halogens (F2, CI2, Br-, and I2). all alkaline earth metals form halides
of the general formula, MX2. (/) Due to small size and high polarising power of Be^'*' ions, beryllium halides
are covalent, soluble in organic solvents, hygroscopic and fume in air due to hydrolysis. The halides of
other alkaline earth metals ai e fairly ionic and their ionic character increases as the size of the metal increases.
(ii) Being electron deficient. BeCl2 has a polymeric .structure (with chlorine bridges) in the solid .state but
exists as a dimer in the vapour .state and as a monomer at 1200 K.
(Hi) BeF2 is very soluble in water due to the high hydration enthalpy of Be^'*' ions. The fluorides of other
alkaline earth metals are insoluble in water.

46. Formation of sulphates. Alkaline earth metals form sulphates of the type MSO4. These are formed by
action of H'>S04 on corresponding oxides, hydroxides or carbonates.
10/54 New Course Chemistry (XI)|35IM!

(0 The solubility of sulphates in water decreases down the group from Be to Ba. This is mainly due to the
reason that as the size of the cation increases, the enthalpy of hydration decreases while the lattice enthalpy
remains abniu the same. Thus, BeS04 and MgS04 are highly soluble, CaS04 is sparingly soluble while
the sulphates of Sr, Ba and Ra are virtually insoluble.
(//) These sulphates decompose on heating forming the corresponding oxides and SO3. The temperature of
decomposition of these sulphates, howeve,r increases as the electropositive character of the metal or the
basicity of the metal hydroxide increases down the group.
47. Carbonates and bicarbonates. (/) The bicarbonates of the alkaline earth metals do not exist in the solid
state but are known only in solution. On heating these bicarbonates decompose forming carbonates with

w
the evolution of CO2.
(//) The solubilities of the carbonates decrease as we move down the group from Be to Ba, i.e., BeCOj >
MgCOrf > CaCOj > SrCOj > BaCO^. This is mainly due to the reason that as the size of the cation
increases, the lattice enthalpy of their carbonates remains almost unchanged (like that of sulphates) but the

Flo
enthalpies of hydration of the cations decrease.
The carbonates of all alkaline earth metals decompose on heating to form the corresponding metal oxide

e
and COt. The temperature of decomposition of these carbonates, however, increases down the group as

re
the electropositivecharacterof the metal or the basicity of metal hydroxide increases from Be(OH)2 to
Ba(OH)2-

F
48. Beryllium differs from re.st of the elements of its family because of (0 exceptionally small atomic and
ionic size, (//) high ionization enthalpy and (Hi) absence of J-orbitals in its valence shell.
Beryllium shows diagonal relationship with aluminium because both these elements have same

r
49.
ur
fo
electronegativity and same polarising power.
50, Calcium oxide (CaO) is also called quick lime. It is obtained when limestone is heated to 1070-1270 K.
On adding water, quick lime gives a hissing sound and forms calcium hydroxide known as slaked lime
ks
while the filtered and clear solution is called lime water. Chemically both are calcium hydroxide.
Yo
51. A mixture of slaked lime with 3-4 times its weight of sand is made into a thick paste. It is called mortar
oo
and is used as a building material.
52. Plaster of Paris is CaS04.1/2 H2O. It is prepared by heating gypsum to 393 K. On mixing with 1/3 of its
B

weight of water, it sets into a hard mass of gypsum (CaS04.2 H2O) with slight expansion (1%) in volume.
It is used for making casts for statues and busts, as decorative material and in dentistry. It is also used in
re

surgical bandages used for plastering broken or fractured bones of the body and for making black board
chalks. When heated above 393 K. it loses its water of crystallization andforms anhydrous calcium sulphate.
It is called dead burnt plaster since it loses the property of setting with water.
u
ad

53. Cement is essentially a finely powdered mixture of calcium silicates and aluminates along with small
Yo

quantities of gypsum which sets into a hard stone like mass when treated with water. The average composition
of Portland cement is :
CaO = 50-60%, MgO = 2-3%. Si02 = 20-25%, Fe203 = 1-2%, AI2O3 = 5-10% and SO3 = 1-2%
The important constituents present in portland cement are dicalcium silicate (Ca2Si04) = 26%, tricalcium
d
Re

silicate (0338105) = 51% and tricalcium aluminaie Ca3Al20^ =11%.


in

When water is added to cement, an exothermic reaction occurs. During this process, cement reacts with
water to form a gelatinous mass which slowly sets into a hard mass having three-dimensional network
F

structure involving —Si—O—Si- and ●Si—O—A1— chains.


54. Fly ash is a waste product of steel industry and mainly consists of calcium silicate (CaSi03). This can be
added to cement to reduce its cost without affecting the quality.
55. Biological importance. Energy is stored in the body due to formation of phosphate bonds in ATP molecules
and energy is released due to hydrolysis of these phosphate linkages. Both these processes are catalysed by
Mg^^ ions.
About 99% of body calcium is present in bones and teeth as apatite, Ca3(P04)2 and the enamel on the
teeth as fluoropatite. Ca^'*' ions are also important in blood clotting and are required to trigger the contraction
of muscles and maintain the regular beating of heart.
In elderly persons, e.specially women, there is sometimes a net loss of bone calcium, leading to the disease
called osteoporosis.
s-BLOCK ELEMENTS (ALKALI AND ALKALINE EARTH METALS) 10/55

[I
I
neEt/jee
SPECIAL

w
For ultimate preparation of this unit for competitive examinations, students should refer to

F lo
* MCQs in Chemistry for NEET
Pradeep's Stellar Series.... * MCQs in Chemistry for JEE (Main) .)
separately available for these examinations. . tnV' ● ● ● / ● ● P- 'n'

Multiple Choice Questions (with one correct Answer)

ee
Fr
5. What is the colour of the flame on healing
I. Group 1 Elemcnts-Alkali Metals
potassium in the flame of a Bunsen burner ?

for
1. Choose the correct order of density of the alkali («3) Golden-yellow (b) Gray-white
metals.
(c) Pale-violet ((/) Orange-red
r
(a) Li < K < Na < Rb < Cs
(J & K GET 2019)
You
{b) Li < Na < K < Rb < Cs
6. Compare the wavelength in flame test for LiCI,
s
ook
(c) Cs < Rb < K < Na < Li NaCl, KCl. RbCl, CsCl
(i/) Li < Na < K < Cs < Rb (JEE Main 2022) (a) NaCl < CsCl < LiCI < RbCl < KCl
eB

2. The correct order of hydration enthalpies of alkali (b) CsCl < NaCl < LiCI < KCl < RbCl
metals is
(c) RbCl < KCl < LiCI < CsCl < NaCl
(Q) Li+> Na+> K+> Rb+> Cs+
(d) CsCl < NaCl < KCl < LiCI < RbCl
(b) Na+ > Li+ > K-*- > Rb-" > Cs+
our
ad

(JEE Main 2021)


(c) Na-" > Li-" > K-" > Cs^" > Rb""
7. The salt of an alkali metal gives violet colour in
(d) Li* > Na* >K*> Cs* > Rh* the flame test. Its aqueous solution gives a white
(JEE Main 2019) precipitate with barium chloride in hydrochloric
dY

3. Ionic mobility of which of the following alkali acid medium. The salt is
Re

metal ions is lowest when aqueous solution of their (a) K2SO4 (b) KCl
salts are put under an electric field?
(c) Na2S04 (d) K2CO3
Fin

(a)K (b) Rb
(e) Li2S04 (Kerala PET 2014)
(c) Li (d) Na (NEET 2017)
8. The salt of an alkali metal gives yellow colour in
4. The ease of absorption of hydrated alkali metal
the flame test. Also its aqueous solution gives an
ions on an ion exchange resins follows the order:
insoluble white precipitate with barium chloride
(a) Li-" < Na-" < K-" < Rb-" in acid medium. The salt is
(b) Rb* < K-" < Na-" < Li*"
(a) NaCl ib) K2SO4
(c) K-" < Na-" < Rb-" < Li-"
(c) N32S04 {d) L12SO4
id) Na* < Li-" < K-" < Rb-"
(e) NaN03 (Kerala PMT 2015)
(AIPMT Prelim 2012)
ANSWERS
1. (a) 2.(0) 3. (c) 4.(0) 5. (c) 6.(b) 7. (o) 8. (c)
10/56 '4- New Course Chemistry (XI)CEIMD

9. The main oxides formed on combustion of Li, Na. 17. The metal nitrate that liberates NO2 on heating is
and K in excess of air are, respectively (a) NaNOj (b) LiN03
(a) L12O, Na20 and KO2 (c) KNO3 (_d) RbNOg
(b) Li02, N3202 and K2O {Karnataka CET 2019)
(c) Li203, Na202 and KO2 18. The alkali metals form salt-like hydrides by the
(d) Li20, Na202 and KO2 direct synthesis at elevated temperature. The
(NEET Phase-I 2016, JEE Main 2022)
thermal stability of these hydrides decreases in
which of the following orders ?
10. Which of the following liberates O2 upon (a) NaH > LiH > KH > RbH > CsH
hydrolysis ?
(h) LiH > NaH > KH > RbH > CsH

w
(a) Pb304 (b) KO2 (c) CsH > RbH > KH > NaH > LiH
(r) N02O2 (d) Li202 (d) KH > NaH > LiH > CsH > RbH
(JEE Advanced 2020)

F lo
(AIPMT 2008)
11. A 5-block element (M) reacts with oxygen to form 19. In the replacement reaction.
an oxide of the formula MO2. The oxide is pale
yellow in colour and paramagnetic. The element
C-I -K MF ^ ^C-F-t-MI

ee
(M) is The reaction will be most favourable if M happens
(a)K to be
ib) Mg

Fr
(c) Na (d) Ca (JEE Main 2021) (a) Na (b)K
(c) Rb id) Li
12. KO2 (potassium superoxide) is used in oxygen

for
cylinders in space and submarines because it (AIPMT Mains 2012)

{a) absorbs CO2 and increases O2 content 20. Which of the following compounds is most
ur
stable ?
{b) eliminates moisture
(a) LiF (b) LiCl
(c) absorbs CO2
s
(c) LiBr (d) Lil
{d) produces ozone (AIIMS 2015)
ook
Yo
13. A metal on combustion in excess air forms X. X 21. The low solubility of LiF and that of Csl in water
upon hydrolysis with water yields H2O2 and O2 are respectively due to which of the properties of
eB

the alkali metal ions ?


along with another product. The metal is
(Cl) Li (b) Rb (a) Higher hydration enthalpy of Li'*', higher lattice
enthalpy of Cs'*'
(c) Mg (d) Na
(h) Smaller hydration enthalpy of Li'*', higher
our

(JEE Main 2019)


ad

lattice enthalpy of Cs'*'


14. Oxidation number of potassium in K2O, K2O', and (c) Smaller lattice enthalpy of Li'*', higher
KO2 respectively, is hydration enthalpy of Cs'*'
(a) -I- 1, -(- 1 and + 1 (b) -t- 2, + 1 and -I- 1/2
(d) Smaller hydration enthalpy of Li'*', smaller
Y

(c) -I- 1, -I- 2 and + 4 (d) + 1, + 4 and + 2 lattice enthalpy of Cs'*’


Re

(JEE Main 2020) (e) Higher lattice enthalpy of Li'*', smaller hydra
nd

15. Among the first group elements which one is used tion enthalpy of Cs’*' (Kerala PET 2016)
in photoelectrodes ? 22. On dissolving moderate amount of sodium metal
Fi

(a) Cesium (b) Lithium in liquid NH3 at low temperature, which one of
(c) Beryllium (d) Magnesium the following does not occur ?
(a) Blue coloured solution is obtained
(JEE Main 2021)
(b) Na'*' ions arc formed in the solution
16. Which of the following acts as reducing as well
as oxidising agent ? (c) Liquid ammonia becomes good conductor of
(a) NaN03 (b) Na20 electricity
(c) Na202 (d) KNO3 (d) Liquid ammonia remains diamagnetic.
ANSWERS
9. (d) 10. (b) 11.(a) 12.(a) 13.(b) 14. (a) 15.(a) 16.(c) 17. (b) 18. (b)
19. (c) 20. (rt) 21. (e) 22. (c)
s-BLOCK ELEMENTS {ALKALI AND ALKALINE EARTH METALS) 10/57

23. Both lithium and magnesium display several (c) Li20 and NaNOo (d) LINOt and Na^O
similar properties due to the diagonal relationship, (JEE Main 2022)
however, the one which is incorrect, is
28. Among baking soda, caustic soda and washing
(a) both form nitrides
soda, carbonate anion is present in
ib) nitrates of both Li and Mg yield NO2 and O2 (fl) Washing soda only
on heating
(c) both form basic carbonates
(b) Washing soda and caustic soda
(d) both form soluble bicarbonates. (c) Washing soda and baking soda only
(JEE Main 2017) (d) Baking soda, caustic soda and washing soda

w
(JEE Main 2022)
24. Among the statements (I) - (IV), the correct ones
are 29. Identify the incorrect statement from the
following :
(I) Lithium has the highest hydration enthalpy
(a) Alkali metals react with water to form their

Flo
among the alkali metals
(II) Lithium chloride is insoluble in pyridine hydroxides

(III) Lithium cannot form ethynide upon its (b) The oxidation number of K in KO2 is + 4
(c) Ionizationenthalpyof alkali metals decreases

ee
reaction with ethyne
(IV) Both lithium and magnesium react slowly from top to bottom in the group

Fr
with H2O (d) Lithium is the strongest reducing agent among
the alkali metals (NEET 2022)
(a) (I), (II) and (IV) only
ib) (n) and (III) only 30. The correct statement!s) among I to III with respect

for
to potassium ions that are abundant within the cell
(c) a) and (IV) only
ur fluids is/are
id) (I), (III) and (IV) only (JEE Main 2020)
I. They activate many enzymes.
25. Which of the following statements are correct ?
IT. They participate in the oxidation of gIuco.se to
ks
I. Both LiCl and MgCl2 are soluble in ethanol. produce ATP.
Yo
II. The oxides Li^O and MgO combine with excess III. Along with sodium ions, they are responsible
oo
of oxygen to form super oxides, for the transmission of nerve cells.
in. LiF is less soluble in water than other alkali
eB

(a) III only {b) I and II only


metal fluorides.
(c) I, II. Ill (d) I and III only
IV. Li20 is more soluble in water than other alkali (JEE Main 2019)
metal oxides.
r

Choose the most appropriate answer from the II. Group 2


ou
ad

options given below ; Elements-Alkaline Earth Metals


(a) (I) and (III) only (b) (I), (III) and (IV) only
31. s-Block element which cannot be qualitatively
Y

(c) (II) and (III) only (d) (I) and (IV) only confirmed by flame test, is
(JEE Main 2022)
(a) Li (b) Na
26. Which of the following statement is incorrect ?
Re
nd

(c) Rb (d) Be (JEE Main 2022)


(a) Low solubility of LiF in water is due to its
32. An element A of group 1 shows similarity with
small hydration energy
Fi

element B belonging to group 2. If A has maximum


(b) KO-) is paramagnetic hydration enthalpy in group 1 than B is
(c) Solution of sodium in liquid ammonia is (a) Mg (b) Be
conducting in nature
(c) Ca (d) Sr (JEE Main 2022)
(d) Sodium metal has higher density than
33. The correct order of density is
potassium metal (JEE Main 2022)
(fl) Be > Mg > Ca > Sr (b) Sr > Ca > Mg > Be
27. Lithium nitrate and sodium nitrate, when heated
separately, respectively, gives; (c) Sr > Be > Mg > Ca (d) Be > Sr > Mg > Ca
(JEE Main 2022)
(a) LiN02 and NaN02 (b) L12O and Na20

23.(c) 24. (d) 25.(a) 26.(a) 27.(c) 28.{a) 29. [h) 30.(c) 31.(d)
32.(«) 33.(c)
10/58 ‘pft^idce^'A New Course Chemistry fXnrosTMn
34. The correct order of melting points is (n) Mg and MgjN2 (b) Na and NaN03
(a) Be > Mg > Ca > Sr (b) Sr > Ca > Mg > Be (c) Mg and Mg(N03)2 (^0 Na and Na3N
(c) Be > Ca > Mg > Sr (d) Be > Ca > Sr > Mg (JEE Main 2020)
(JEE Main 2022) 41. Two elements A and B have similar chemical
35. Match the flame colours of the alkaline earth metal properties. They don’t form solid hydrogen
salts in the Bunsen burner. carbonates, but react with nitrogen to form
(p) Calcium 1. Brick red nitrides. A and B, respectively, are
(if) Strontium 2. Apple green (a) Li and Mg (b) Cs and Ba

w
(r) Barium 3. Crimson (c) Na and Rb (t/) Na and Ca
(a) p-U q-2, r-2 (b)p-3, q-\, r-2 (JEE Main 2020)
(c)p-2, q-2, r-1 (d)p-l, q-2, r-3 42. Arrange the following in increasing order of their

Flo
(West Bengal JEE 2015) covalent character.
36. Identify the correct statement from the following : !. Cap2 II. CaCU III. CaBr2 IV. Cal2
(fl) Lithium chloride is deliquescent and Choosethe correct answer from the options given

ee
crystallizes as a hydrate, LiCl.H20 below :

(b) The order of hydration enthalpy of alkaline

Fr
(a) K<I<1II<IV (fe) I <II<III<IV
earth cations is:
(c) I < II < IV < III (b) I < III < II < IV
Be^+ < Mg--*- < Ca^-*- < Sr^-*- < Ba^-*- (JEE Main 2022)
(c) Lithium and magnesium show some

for
43. The reaction of BeCl2 with LiAlH4 gives ;
ur
similarities in their physical properties as they
I. AICI3 II. BeH2 III. LiH IV. LiCl V. AIH4
are diagonally related in periodic table
(d) Lithium is softer among all alkali metals Choose the correct answer from the options given
ks
below :
(MEET 2020)
Yo
(a) I, IV, V (b) 1,11. IV
37. The hydration energy of Mg^"*- is greater than that of
oo
(c) IV and V (d) II, III, IV
(a) Ap-*- (b) Na-*-
44. Among the following alkaline earth halides, one
(c) Be^-* (d) Mg-^+
eB

38.
which is covalent and soluble in organic solvents
An alkaline earth metal “M’ readily forms water is
soluble sulphate and water insoluble hydroxide.
Its oxide MO is very stable to heat and does not (a) calcium chloride (b) strontium chloride
r

have rock salt structure. M is (c) magnesium chloride (d) beryllium chloride
ou
ad

(a) Ca ib)Mg (NEET 2021)


(c)Sr (d) Be 45. The structure of beryllium chloride in solid state
Y

(JEE Main 2020) and vapour phase are


39. Magnesium reacts with an element (X) to form an (a) chain and dimer, respectively
nd
Re

ionic compound. If the ground state electronic (b) linear in both


configuration of (X) is li“ 2s^ 2p^, the simplest (c) dimer and linear, respectively
formula of this compound is
Fi

(d) chain in both (NEET 2021)


(a) Mg2X3 (b) MgXj 46. HCl was passed through a solution of CaCl2,
(c) Mg2X (d) Mg3X2 MgCU and NaCl. Which of the following
(NEET 2018) compound(s) crystallize(s) ?
40. A metal (A) on heating in nitrogen gas gives (a) Only NaCI
compound (B). (B) on treatment with H2O gives
(b) Only MgCl2
a colourless gas which when passed through
CUSO4 solution gives a dark blue violet coloured (c) NaCl, MgCl2 and CaCl2
solution. (A) and (B) respectively,are : (d) Both MgCl2 and CaCl2 (NEET 2020)
ANSWERS
34. (d) 35. (a) 36. (c) 37. (b) 38. (d) 39. (d) 40. (a) 41. (a) 42. (b) 43. (h)
44.(d) 45.(a) 46.(a)
S-BLOCK ELEMENTS (ALKALI AND ALKALINE EARTH METALS) 10/59

47. Which of the following oxides is most acidic in (c) decreasing lattice energy
nature ?
{d) decreasing hydration enthalpy.
(a) MgO (b) BeO 55. Solubility of the alkaline earth metal sulphates in
(c) BaO (d) CaO (NEET 20!8) water decreases in the sequence.
48. Property of the alkaline earth metals that increases (a) Sr > Ca > Mg > Ba
with their atomic number is
(6) Ba > Mg > Sr > Ca
(a) solubility of their hydroxides in water
(c) Mg > Ca > Sr > Ba
(b) solubility of their sulphates in water
(d) Ca > Sr > Ba > Mg (AIPMT 2015)
(c) ionization energy (d) electronegativity 56. Which of the following alkaline earth metal
(AIPMT Prelim 2010)
sulphates has its hydration enthalpy greater than

w
49. Which of the following is an amphoteric its lattice enthalpy ?
hydroxide ?
(fl) BaS04 ib) SrS04
(a) Sr(OH)2 (b) Ca(OH)2

F lo
(c) CaS04 (d) BeS04
(c) Mg(OH>2 id) Be(OH)2
(JEE Main 2015)
(NEET 2019)
50.
57. A metal M readily forms water soluble sulphate
The decreasing order of basic character of K2O,
MSO4, water insoluble hydroxide M(OH )2 and

ee
BaO, CaO and MgO is
oxide MO which becomes inert on heating. The

Fr
(a) K2O > BaO > CaO > MgO hydroxide is soluble in NaOH. The metal M is
(b) K2O > CaO > BaO > MgO (a) Be (b) Mg
(c) MgO > BaO > CaO > K-,0

for
(c) Ca id) Sr
(d) MgO > CaO > BaO > K2O 58. Which of the following has highest melting point ?
ur
(West Bengal JEE 2015)
51. (a) MgO ib) LiF
Which one of the following order represents the
correct sequence of the increasing basic nature of (c) NaCl id) LiCi
s
ook
the given oxides ? (JEE Main 2021)
Yo
(a) MgO < K2O < AI2O3 < Na20 59. What are the products formed in sequence when
(b) NajO < K2O < MgO < AI2O3 excess of CO2 is pas.sed in slaked lime ?
eB

(c) K2O < Na20 < AI2O3 < MgO ia) CaO, CaC03
(d) AI2O3 < MgO < Na30 < K2O (AIEEE 2011) ib) Ca(HC03)2, CaC03
52. In the following pairs, each member produces the (c) CaC03, Ca(HC03>2
our
ad

same gas when it reacts with water ?


id) CaO, Ca(HC03)2 (JEE Main 2021)
ia) K, KO2 ib) Na, Na202 60. Arrange BaC03, CaC03, SrC03, MgC03
(c) Ca, CaH2 id) Ba, BaO^ according to their decreasing thermal stability ?
(Bihar CECE 2015)
Y

ia) MgC03 > CaC03 > SrC03 > BaC03


53. Among CaH2> BeH2, BaH^, the order of ionic
Re

character is
ib) MgC03 > S1CO3 > CaC03 > BaC03
nd

(c) BaC03 > S1CO3 > CaC03 > MgC03


ia) BeH2 < CaH2 < BaH2
id) BaCOj > SrCOj > MgCOj > CaC03
Fi

ib) CaH2 < BeH2 < BaH2


(JEE Main 2021)
(c) BeH2 < BaH2 < CaH2
61. On heating which of the following will release
(^0 BaH2 < BeH2 < CaH-, (NEET 2018)
CO2 most easily ?
54. The solubility in water of sulphates down the Be
group is : Be > Mg > Ca > Sr > Ba. This is due to (a) Na2C03 ib) MgC03
ia) increase in melting point (c) CaC03 id) K2CO3
ib) increase in ionization energy (RE-AIPMT 2015)
ANSWERS

47. ih) 48. ia) 49. id) 50. (a) 51. id) 52. (c) S3, ia) 54. id) 55. (c) 56. id)
57. ia) 58. (rt) 59. (c) 60. (c) 61. (b)
10/60 A New Course Chemistry (XI) t.-’Mmu

62. Which of the following is least thermally stable ? 69. The suspension of slaked lime in water is known as

(a) MgC03 (b) CaCOg (a) lime water


(c) StCOj (d) BeC03 (b) quick lime
(West Bengal JEE 2018) (c) milk of lime
63. When gypsum is heated to 393 K. it forms (d) aqueous solution of slaked lime.
(fl) Anhydrous CaS04 (b) CaSO4-0-5 H2O (NEET Phase-II 2016)

(c) CaS04’5 HoO (d) Dead burnt plaster 70. Bleaching powder loses its power on keeping for
(JEE Main 2020) a long time because
64. Portland cement contains ‘X’ to enhance the (a) it changes into calcium hypochlorate

w
setting time. What is X (b) it changes into calcium chloride and calcium
hydroxide
(a) CaS04.1/2 H^O ib) CaS04.2 H2O
(c) it absorbs moisture

F lo
(c) CaS04 id) CaC03
{d) it changes into calcium chloride and calcium
(JEE Main 2022) chlorate.
65. Plaster of Paris is represented as 71. The product obtained as a result of a reaction of

ee
(a) CaS04.2 H2O (b) CaS04 . H2O nitrogen with CaC2 is

(0 CaSO,iH,0 (b) Ca2CN

Fr
(a) CaCN3
id) CaS04
(c) CaCN2 id) CaCN
(Karnataka CET 2017) (NEET Phase-I 2016)

for
66. Dead burnt plaster is 72. The element responsible for the neuromuscular
ur
1 function in the body is
(a) CaS04 ib) CaS04.^H20 ia) calcium ib) magnesium
(c) potassium id) sodium
s
(c) CaS04.H20 id) CaS04.2 H2O
ook
ie) manganese (Kerala PMT 2015)
Yo
(Karnataka CET 2018)
73. The function of sodium pump is a biological
67. Be and A1 exhibit diagonal relationship which of process operating in each and every cell of all
eB

the following statements about them is/are not animals. Which of the following biologically
true ?
important ions is also a constituent of the pump ?
(0 Both react with HCl to liberate H2. (a) (/?) Fe2+
r

Hi) They are made passive by HNO3. (c) Ca2+ id) Mg


2+
ad
ou

iiii) Their carbides give acetylene on treatment (AIPMT 2015)


with water.
74. Which of the following statements is false ?
iiv) Their oxides are amphoteric.
Y

ia) Ca^'*' ions are not important in maintaining the


id) iiii) and (iv) ib) (0 and iiii)
regular beating of the heart
Re

(c) (0 only id) iii) and iiii)


nd

ib) Mg^'*’ ions are important in the green parts of


ie) iiii) only the plants
Fi

68. In context with beryllium, which one of the (c) Mg^'*' ions form a complex with ATP
following statements is incorrect ? id) Ca^'*’ ions are important in blood clotting
(a) It is rendered passive by nitric acid (NEET Phase-I 2016)
ib) It forms Bc2C 75. Enzymes that utilize ATP in phosphate transfer
(c) Its salts rarely hydrolyze require an alkaline earth metal (M) as tlie cofactor
id) Its hydride is electron-deficient and polymeric. ia) Sr ib) Be
(NEET Phase-n, 2016) (c) Mg id) Ca (NEET 2019)

62. id) 63. ih) 64.ih) 65.(r) 66. ill) 67.ie) 68.(r) ov. i'-~

70.(d) 71.(c) 72. (u) 73. (</) 74. (iO 75.(c)


s-BLOCK ELEMENTS (ALKALI AND ALKALINE EARTH METALS) 10/61

Multiple Choice Questions (with One or More than One Correct Answers)
76. Which of the following elements form peroxides 78. Flame test is not given by
when heated in excess of air ?
(a) Be .ib)K.
(a)K (h) Na
(c) Sr id) Mg
(c)Ba (d) Ca
79. The alkaline earth metals forming ionic oxides are
77. The pair(s) of reagents that yield paramagnetic
species is/are (a) BeO (b) MgO
(c) CaO (d) SrO
(a) Na and excess of NH3
(b) K and excess of 0-, 80. Which of the following chlorides are soluble in

w
(c) Cu and dilute HNO3 pyridine ?
(d) O2 and 2-ethylanthraquinol (a) LiCl (b) CsCl

F lo
(JEE Advanced 2014) (c) NaCl (d) BeCU

mi Multiple Choice Questions (Based on the given Passage/Comprehens ion)


Each comprehension given below is followed by some multiple choice questions. Each question has one

ee
correct option. Choose the correct option.

Fr
82. Which of the following are arranged in increasing
Qgiiosaaigji^tai) E I Alkali and alkaline earth order of solubilities ?

for
metals along with hydrogen and helium (a) CaC03 < KHCO3 < NaHCOj
constitute s-block elements. They have low (h) NaHC03 < KHCO3 < CaC03
ionization enthalpies and hence exhibit
r
(c) KHCO3 < NaHC03 < CaC03
characteristic flame colourations. They have
(d) CaC03 < NaHC03 < KHCO3.
You
highly negative electrode potentials and hence
s
83. The compound insoluble in acetic acid is
are strong reducing agents. Their solutions
ook

in liquid ammonia are conducting and also (a) calcium oxide (b) calcium carbonate
act as strong reducing agents. Being stronger (c) calcium oxalate (d) calcium hydroxide.
eB

reducing agents than hydrogen, they are 84. The metal that produces red-violet colour in the
usually prepared by electrolysis of their fused non-luminous flame is
chlorides. Their oxides are basic and the basic (a) Ba (b) Rb (c) Mg {d) K
strength increases down the group. The 85. Which of the following process is used in the
our
ad

solubility of carbonates and sulphates of extractive metallurgy of magnesium ?


alkali and alkaline earth metals show opposite (a) fused salt electrolysis (b) self reduction
trends. The carbonates of alkaline earth
(c) aqueous solution electrolysis
metals and lithium carbonate decompose on
(d) thermite reduction
dY

heating while the carbonates of other alkali


Re

metals do not decompose on heating. The


bicarbonates of both alkali and alkaline earth ®inZ{ga(a]D3iEQ5G) n According to Fajan rules,
Fin

metals on heating give carbonates. the percentage of covalent character in an


ionic compound increases if the cation is
81. The basic character of the oxides, MgO, SrO, K2O, highly charged or the cation is small and the
NiO and CS2O increases in the order: anion is large or the cation has pseudo inert
(a) MgO > SrO > K2O > NiO > CS2O gas configuration. As a result of increased
(b) CS2O < K2O < MgO < SrO < NiO covalent character, the melting point
decreases and solubility in less polar solvent
(c) NiO < MgO < SrO < K2O < CS2O increases.
(d) K2O < NiO < MgO < SrO < CS2O.
ANSWERS
76. {h.c.d) 77. (u.Im ) 78. Ui.d) 79. ih.c.d) 80. {(i,d) 81. (£●) 82. id}
83.(c) 84. (h) 85.(«)
10/62 “pn^tdee^'^- New Course Chemistry (XI)SSQISE

86. Which of the following has the lowest melting (b) BeCl2, NaCl, LiCl
point ? (c) NaCl, LiCl, BeC^
(a) LiCl (b) NaCl (d) BeCl2, LiCl, NaCl (AIIMS 2010)

(c) KCI (d) RbCl. 89. Based on lattice energy and other considerations,
which one of the following alkali metal chloride is
87. The correct order of increasing ionic character is
expected to have tlie highest melting point ?
(a) BeCl2 < MgCl2 < CaCl2 < BaCl2 (a) LiCl (b) NaCl
(h) BeCl2 < MgCl2 < BaCl2 < CaCU (c) KCl (d) RbCl (AIIMS 2011)
(c) BeCl2 < BaCl2 < MgCl2 < CaCl2 90. The highest lattice energy corresponds to

w
(d) BaCl2 < CaCU < MgCl2 < BeCl2 (a) MgO (h) CaO
88. The correct order of increasing covalent character is (c) SrO (d) BaO
{a) LiCl, NaCl, BeCl2 (AMU Med. 2010)

F lo
09 Matching Type Questions

ee
Match the entries of column I with appropriate entries of column II and choose the correct option out

Fr
of the four options (a), (b), (c), (d) given at the end of each question.
91. Column 1 (Metal) Column II (Emitted light with

for
wavelength in m)
(A) Li (P) 670-8
ur
(B) Na (q) 589-2
(C) Rb (r) 780-0
s
(D) Cs is) 455-5
ook
Yo
(a) A-r, B-p, C-q, D-s (b) A-p, B-q, C-r, D-j (c) A-j, B-q, C-r, D-^ (d) A-p, B-r, C-5, D-q
eB

(JEE Main 2022)


92. Column I Column II

(A) Strongest reducing agent in aqueous solution ip) Magnesium


our
ad

(B) Does not give flame colouration iq) Caesium

(C) Forms peroxides on heating with excess oxygen ir) Lithium

(D) Used in photoelectric cells is) Sodium


Y

id) As, B-q, C-p, D-r ib) A-q, B-r, C-p, D-i (c) A-r, B-p, Cs, D-q id) A-p, B-5, C-q, D-r
Re
nd

93. Column I Column II

(A) Fly ash ip) Ca3Si05


Fi

(B) Plaster of Paris iq) (CaS04)2 - H2O


(C) Major constituent of Portland cement ir) Na2C03
(D) Solvay’s process (-S) CaSiOg
(a) A-q, B-p, Cs, D-r (b) A-r, B-5, C-q, D-p (c) A-p, B-q, C-r, D-5 id) As, B-q, C-p, D-r

ANSWERS

86. (o) 87. (o) 88. (r) 89. {h) 90. (a) 91. (h) 92. (c) 93. (d)
S-BLOCK ELEMENTS (ALKALI AND ALKALINE EARTH METALS) 10/63

m Matrix-Match Type Questions p q

Match the entries of column I with appropriate entries of column II. Each
entry in column I may have one or more than one correct option from column II. A 0 © 0 ©
If the correct matches are A-p, s ; B-r; C-p, q ; D-.v, then the correctly bubbled
4x4 matrix should be as follows :
B
© © O 0
94. Column I Column II c 0 © o ©
(A) Flame colouration (/^) Be(OH)2 D
© © © ©

low
(B) Amphoteric character (9) K

(C) Soluble in organic solvents (r) BeCl2


(D) Forms superoxide on heating with oxygen (s) LiCl
95. Column I Column II

(A) CaCOg (P) Pink-violei flame colouration

(B) K2CO3 (^) Gives COt on heating

ee
(C) BaCOj (r) Insoluble in water but dissolves in presence of CO2

F
(s) Gives precipitate with dil. H2SO4

Fr
(D) NaHCOj

VI. Integer Type Questions


A B C D

for
ur
DIRECTIONS. The answer to each of the following questions is a single digit
© © 0 0
integer, ranging from 0 to 9. If the correct answers to the question numbers
A, B, C and D (say) are 4, 0, 9 and 2 respectively, then the correct darkening ©O©©
of bubbles should be as shown on the side :
ks
@ ©®©
Yo
96. Out of Li, Na, K, Rb and Cs, how many of them directly form superoxides on
oo
heating with dioxygen ? ®©®©
97. How many of the following 5-block elements do not give characteristic colours in
©©© ©
eB

the flame test ?

Li, Be, Ca, Ba, Sr, Mg, Na, K, Cs


©©©©
98. How many of the following hydroxides is/are amphoteric in character ?
CsOH, LiOH, Ca(OH>2, Be(OH),, Mg(OH2), Ba(OH)2, Sr(OH)2, Ba(OH>2, KOH, ®©®©
r
ou

NaOH.
©©©©
ad

99. How many of the following metals when heated in an atmosphere of N2 gas form
nitrides ? ®©®®
Y

Li, Na, K, Rb, Cs, Mg, Ca, Sr, Ba


100. The sulphates of which of the following metals dissolve in water.
®.0MS>
nd
Re

SrS04, K2SO4, BeS04, Li2S04, MgS04, BaS04, Na2S04, CaS04, Rb2S04

VII,
Fi

Numerical Value Type Questions Decimal Notation)


For the following question, enter the correct numerical value, (in decimal-notation, truncated/rounded-otT
to the second decimal place, e.g., 6*25, 7*(H), - 0*33, - *30, 30‘27, - 127*30) using the mouse and the on
screen virtual numeric keypad in the place designated to enter the answer.
101. The enthalpy of formation of hypothetical CaCl(5) is theoretically found to be - 188 kJ mor^ and AyH° for
CaCl2 (5) is - 795 kJ mol"’. Calculate AyH° (in kJ mol”*) for the disproportionation reaction.
2 CaCl (5) ■> CaCl2 {s) + Ca (5)
ANSWERS
94. {A-q.s ; B-p ; C-.rs : D-q) 95. {A-q,,rs ; B-p ; C-q,r,s ; D-q) 96. (.1) 97. (2)
98.(1) 99.(5) 100.(6) 101. (-419)
10/64 New Course Chemistry (XI)EEEIHD

IVIII, Assertion-Reason Type Questions

TYPE I

DIRECTIONS. The questions given below contains statement-1 (Assertion) and statcment-2 (Reason).
Each question has four choices (a), (b), (c) and (d) out of which ONLY ONE is correct. Choose the correct
option as under :
(a) Statement-1 is True, Statement-2 is True ; Statcment-2 is a correct explanation for Statement-1,
(b) Statement-1 is True, Statement-2 is True ; Statement-2 is not a correct explanation for Statement-1,
(c) Statement-1 is True, Statement-2 is False. (d) Statcment-1 is False, Statemcnt-2 is True.
102. Statemcnt-1. Sodium metal is softer than potassium metal.

w
Statemcnt-2. Metallic bonding in potassium is weaker than in sodium.
103. Statement-1. Sodium reacts with oxygen to form Na^O-, but potassium reacts with oxygen to form K02-

F lo
Statcment-2. Potassium is more reactive than sodium.

104. Statement-1. Alkali metals dissolve in liquid ammonia to give blue solutions.
Statement-2. Alkali metals in liquid ammonia give solvated species of the type [MlNH^),,]'*’ (M = alkali
metals) (IIT Paper II 2007)

ee
105. Statement-1. CuCl is more covalent than NaCl.

Fr
Statement-2. Na"*" ion is more polarising than Cu'*' ion.
106. Statcment-1. Among the alkali metals, caesium salts exhibit the maximum electrical conductance in aqueous
solutions.

for
Statement-2. The radius of hydrated caesium ion is the highest among alkali metals.
ur
107. Statement-1. Be and Mg give characteristic flame colourations.
Statement-2. As compared to other alkaline earth metals, ionization enthalpy of Be and Mg is high.
s
108. Statement-1. Bc{OH)t is soluble both in HCl and NaOH.
ook
Yo
Statement-2. Bc(OH)t is amphoteric in nature.
109. Statement-1. Be forms but A1 forms [AIF^]^.
eB

Statement-2. Be does not have d-orbitals in the valence shell but Al has.

110. Statement-1. LiF is sparingly soluble in water.


Statement-2. The ionic radius of Li"*" ion is smallest among its group members, hence has least hydration
our

enthalpy. (JEE Main 2022)


ad

TYPE II

DIRECTIONS. In each of the following questions, a statement of Assertion (A) is given followed by a
Y

corresponding statement of Reason (R) just below it. Of the statements, mark the correct answer as
(a) If both assertion and reason are true, and reason is the true explanation of the assertion,
Re
nd

(b) If both assertion and reason are true, but reason is not the true explanation of the assertion,
(c) If assertion is true, but reason is false. (d) If both assertion and reason are false.
Fi

111. Assertion. Alkali metals impart colour to the flame.


Reason. Their ionizaton energies are low.
112. Assertion. Sodium reacts with oxygen to form Na202 but potassium reacts with oxygen to form KO2
Reason. Potassium is more reactive than sodium. (AIIMS 2015)
113. Assertion. K, Rb and Cs form supcroxidcs.
Reason. The stability of the superoxides increases from ‘K’ to ‘Cs’ due to decrea.se in lattice energy.
(AIIMS 2011)
ANSWERS
102. Ul) 103. (/;) 104. (/?) 105. (c) 106. (c) 107. (r/) 108. ((/) 109. (a) 110. (c)
111. (<;) 112. (h) 113. (c)
s-BLOCK ELEMENTS (ALKALI AND ALKALINE EARTH METALS) 10/65

114. Assertion. LiCl is predominantly covalent compound.


Reason. Electronegativity difference between Li and Cl is too small. (ARMS 2013)
115. Assertion. The mobility of sodium ion is lower than that of potassium ion.
Reason. The ionic mobilities depend upon the effective radius of the ion. (AIIMS 2013)
116. Assertion. Among the alkali metals, lithium salts exhibit the least electrical conductance in aqueous solutions.
Reason. Smaller the radius of the hydrated cation, lower is the electrical conductance in aqueous solutions.
(AIIMS 2009)
117. Assertion. Li resembles Mg.
2+
Reason. Li'*’ has approximately the same size as Mg
118. Assertion. LLCO^ and Na-)C03 are thermally stable.
Reason. Both the carbonates arc salts of large cations and large anions.

w
119. Assertion. Potassium carbonate cannot be manufactured by a process similar to the Solvay’s soda ammonia
process.
Reason. Potassium hydrogen carbonate is less soluble in water than sodium hydrogen carbonate.

F lo
120. Assertion. In rainy season, common salt becomes damp after some time on keeping in air.
Reason. Common salt is NaCl which is hygroscopic in nature.
121. Assertion. In the electrolysis of aqueous NaCl, Na is preferentially discharged at mercury cathode forming
sodium amalgam.

e
Fre
Reason. It is due to the fact that hydrogen has a high overvoltage at mercury cathode. (ARMS 2007)
122. Assertion. Magnesium keeps on burning in CO2.
Reason. Magnesium reduces CO-, to C.

for
123. Assertion. Calcium and magnesium oxides are not reduced by carbon.
Reason. Calcium and magnesium oxides react with carbon to form their respective carbides.
124. Assertion. Cap2 has been given the name fluorspar.
r
Reason. Solid Cap2 emits light when heated. (ARMS 2014)
You
oks
125. Assertion. Na2S04 is soluble in water while BaS04 is insoluble.
Reason. Lattice energy of barium sulphate exceeds its hydration energy.
eBo

126. Assertion. Bleaching powder is a mixed salt.


Reason. In the presence of C0CI2, bleaching powder decomposes to give CaCl2 and 02- (ARMS 2009)
127. Assertion. Magnesium can be obtained by the electrolysis of aqueous MgCL.
Reason. The electrode potential of Mg^'^ is far higher than that of H2O.
our
ad

128. A.ssertion. Mg is not present in enamel of human teeth.


Reason. Mg is an essential element for biological functions of human.
dY
Re

For Difficult Questions


Fin

Multiple Choice Questions (with one Correct Answer)

I. Due to an abnormal increase in atomic size relative hydrated species is the highest and its mobility in
the electric field is the lowest.
to atomic mass, the density of K is lower than that
of Na. Thus, option (a) is correct. 4. Larger the size of the hydrated cation, smaller is
the ease of absorption. Thus, ease of absorption
3. Li'*' being the smallest alkali metal cation is most increases in the order: Li'*' < Na'*’ < K'*' < Rb'*', i.e.,
highly hydrated. As a result, the mass of the option (a) is correct.

114. (<●) 115.1/7) 116.(0 117. (u) 118. (^/) 119.(0 120.(0 121. («) 122.(0 123.(0
124. (^i) 125.(0) 126. (/7) 127.(0 128. (/7)
10/66 ‘Pxa.Uee^'^ New Course Chemistry (XI)EEHIl

^ -Ml -Ml
I
For Difficult Questions
KO2 or K [ O - O ] or x-2x — = 0 or X = + 1
2
6. The wavelength (in nm) of light observed in the
flame of alkali metal chlorides increases in the
{In superoxides, O.S. of O = - 1/2)
order: 15. The element cesium is used in photoelectrodes.
Thus, option (a) is correct.
CsCl (455-5) < NaCI (589-2) < LiCI (670-8)
Blue Yellow Crison 16. O2 (peroxide ion) can be oxidised to O2 and can
< KCl(766-5) < RbCl(780) be reduced to oxide ion, 0^~. Thus, Na20^ acts
Pale yellow Red violet both a reducing as well as an oxidising agent.
17. 4 LiN03 > 2 Li20 + 2 NO^ + Oo

w
Thus, option (b) is correct.
18. As the size of alkali metal ion increases, lattice
7. K imparts violet colour to the flame and SO^“
ions produce white ppt. of BaS04 when treated enthalpy decreases and hence the stability of the
corresponding metal hydride decreases. Thus,

F lo
K2SO4 + BaCl2 BaS04vt -f 2 KCl
option {b) is correct.
White ppt.
19. 3® Alkyl halides undergo Sj,^! reactions in which a
with an aqueous solution of BaCl2. Thus, option nucleophile attacks a carbocation. Thus, more the
(a) is correct.

ee
ionic character of the M-F bond, more favourable
8. Since the alkali metal salt gives a yellow colour will be the replacement reaction. Since amongst

Fr
in the flame, it must be sodium salt. Further, since Li, Na, K and Rb, Rb has the largest size, therefore,
the salt gives an insoluble white ppt. with BaCl2 Rb-F bond has the highest ionic character and
in acidic medium, it must be sodium sulphate,

for
hence option (c) is correct.
Na2S04. 20. Because of the small size of Li and F, LiF has
ur
Na2S04 -+■ BaClj > BaS04 i + 2 NaCl highest lattice enthalpy and hence most stable.
White ppt. 21. Due to small size, Li'*' has a high lattice energy
s
9. Li forms Li20, Na forms Na202 and K forms KO2. and due to bigger size Cs'*’ has small hydration
ook
Yo
energy.
10. 2 KO2 + 2 H2O > 2 KOH + H2O2 + O2
11. Since 5-block element (M) forms oxide MO^, 22. It is not liquid ammonia but the solution which
eB

therefore, it must be an alkali metal, Na or K. Since becomes good conductor of electricity.


the oxide is pale yellow and paramagnetic, it must 23. Li does not form basic carbonate but Mg does such
contain superoxide (Op ion and thus the element as 3 Mg C03.Mg(0H)2.3 H2O.
24. Statements (I), (III) and (IV) are correct. Lithium
r

is K, i.e., option (a) is correct.


ad
ou

chloride being covalent is soluble in pyridine.


12. 4 KO2 + 2 CO2 > 2 K2CO3 + 3 O2 Thus, option (d) is correct.
Although KO2 also reacts with moisture of the
25. (i) Both LiCI and MgCl2 are covalent in nature
breath but does not completely eliminate it.
due to high polarizingpowerof Li"*" and Mg-"*" ions.
Y

4 KO2+2 H2O > 4 KOH + 3 O2 As a result, they are soluble in ethanol,


Re

or 4 KO2 -t- 4 CO2 2 H2O > 4 KHCO3 + 3 O2 (ii) LiF is least soluble among other alkali metal
nd

For further explanation,refer to page 10/17. fluorides due to its high lattice energy.
13. In excess of air, Rb forms Rb02 (X), while Li
Fi

(Hi) Li20 is least soluble among other alkali metal


forms Li20, Mg forms MgO, and Na forms Na202 oxides due to its high lattice energy.
Thus, option (a) is correct.
Rb + O2 Rb02 ;
26. Low solubility of LiF in water as stated above is
2 Rb02 + 2 H2O » 2 RbO -i- H2O2 + O2
due to its high lattice energy and not due to its
X -1
small hydration energy.
14. K2O orKO or2x-2 = 0orx = -fl
-r -I -I 27. 4UNO3 ^ 2 L12O -I- 4 NO2 -t O2
K2O2or K2[O-0] or X-2 = 0 orx =-P 1 2 NaN03 ^ 2 NaN02 + O2
(In peroxides, O.S. of each oxygen = - 1) Thus, option (c) is correct.
s-BLOCK ELEMENTS (ALKALI AND ALKALINE EARTH METALS) 10/67

H,0
For Difficult Questions 40. 3 Mg +N2 Mg3N2
(S) (B)
28. Washing soda is Na2C03 and hence it contains
Mg(OH)2 + NH3
carbonate (COl") anion. {Colourless gas)

In contrast baking soda is NaHC03 and Caustic CaS04 (fl^) + 4 NH3 > tCu(NH3)4]S04

w
soda is NaOH, {Dark blue violel colour)

29. The oxidation state of K in KO2 is +1 as discussed 41. Li and Mg are diagonally related and hence have
above in Q. 14. similar properties. They react with N2 to form
30. All statements are correct, i.e., option (c) is correct. nitrides and do not form solid hydrogen
carbonates.

o
31. Due to high ionization enthalpy Be does not impart
^ 2L13N ; 3Mg +N2 ^ Mg3Nj

e
any colour to the flame. 6Li +N2
(B)

re
32. Due to small size, Li (element (A) in group 1 and (A)

rFl
its diagonally related Mg (element B) in group 2 Thus, option (a) is correct.
have high hydration enthalpy. 42. From Fajan rule, for a given metal ion, as the size

F
Thus, option (a) is correct. of anion increases, polarizability of anion
33. No regular trend. increases and hence covalent character of the given
34. No regular trend. ionic compound increases. Thus, the increasing

r
ou
35. Calcium ip) - Brick red (1) ; Strontium (g) - order of the covalent character is :

fo
Crimson (3); Barium (r) - Apple green (2). I (CaF2) < II (CaCL) < lU (CaBr2> < IV (Cal2),
Thus, option (a) is correct.
36. {a) LiCl is deliquescent and crystallizes from ks
i.e., option (b) is correct.

43. 2BeCl2 + LiAlH4 > 2BeH2+ LiCl+ AICI3


oo
aqueous solution as hydrate, LiCl.2 H2O. II IV I

(b) The hydration enthalpy of alkaline earth metal


44. Beryllium chloride being covalent is soluble in
Y
ions decrease with increase in ionic size down the
B

group and the order is: organic solvents.


Be^+ > Mg^+ > Ca2+ > Sr^+ > Ba^^ 45. In the solid phase. BeCl-> has polymeric chain
re

(c) Lithium shows diagonal relationship with structure but in the vapour phase, it has dimeric
magnesium hence their physical and chemical structure.

46. Out of CaCl2 (745 g L“^), MgCL (543 g L"') and


ou

properties are almost similar.


Y
ad

(d) Lithium is much harder than other alkali metals. NaCl (360 g L"^), NaCl has the lowest solubility
Thus, options (a), (b) and (d) are wrong. Only in water. Therefore, when HCl gas is passed
option (c) is correct. through a solution of these salts, due to common
d

37. Na''' has +1 unit charge, Mg^'*’ and Be^"*" have +2 ion effect, solubility product of NaCl is reached
unit charge while Mg^"*" and Al^'*' have +3 unit first and it gets crystallized. Thus, option (a) is
in
Re

charge, therefore, hydration energy of Al^"*" and correct.


2+
Mg^"^ are expected to be higher than those of Mg 47. As electropositive character of alkaline earth
F

and which in turn, are expected to be higher metals increases down the group, the basic
than that of Na"*". Further, the hydration energy of character of their oxides increases. Thus, BeO is
Be^'*' is higher than that of Mg^"*" due to its smaller least basic and hence most acidic. Actually, BeO
size. In other words, hydration energy of Mg""^ is is amphoteric in nature.
expected to be higher than that of Na"’’.
48. Solubility of hydroxides of alkaline earth metals
38. Solubility of BeS04 is highest among alkaline increases because both the lattice enthalpy and
earth metal sulphates and Be(OH>2 is insoluble. hydration enthalpy decrease down the group as
BeO does not form rock salt stnicture and is also
the size of the cation increases but lattice enthalpy
very stable to heat. Thus, option id) is correct.
decreases more rapidly than the hydration enthalpy
39. Mg^+ X-"*-, i.e., Mg3%2 and hence the solubility increases down the
Thus, option (d) is correct. group.
10/68 ‘PxaUce^'^i. New Course Chemistry (X1)E!E2MD

For Difficult Quostions

Among sulphates, since the size of SO^ ion is 57. BeS04 is soluble in H2O ; BeO does not dis.solve
very big as compared to the metal cation, therefore, in H2O but being amphoteric dissolves in NaOH.
lattice enthalpy remains almost constant but their 58. Due to high lattice energy of divalent Mg and
hydration enthalpy decreases down the group. divalent oxygen, melting point of MgO is the

ow
Thus, the solubility of sulphates decreases down highest.
the group. 59. First insoluble CaCO^ is formed which dissolves
Of course, electronegativity and ionization on passing excess of CO2 due to the formation of
enthalpy both decrease down the group as the soluble calcium bicarbonate, Ca(HC03)2
atomic size increases.

50. Alkali metal oxides are more basic than their


Ca{OH)2 +CO2 CaC03 i + H2O

e
Slaked lime Insoluble
corresponding alkaline earth metal oxides. Thus,

re
rFl
K2O h the most basic. Further, basic character of CaC03 + CO2 + H2O -> Ca(HC03)2
alkaline earth metal oxides increases down the Soluble

F
group as the electropositive character of the metal
Thus, option (c) is correct.
increases, i.e., BaO > CaO > MgO.
60. As the basicity of the metal hydroxide increases
Combining the two trends, basic character

r
from Mg to Ba, the thermal stability of their
ou
increases in the order: K2O > BaO > CaO > MgO,

sfo
carbonates increases, i.e., BaC03 > SrC03 >
i.e., option (a) is correct.
CaC03 > MgC03.
51. Metallic character decreases across a period in the Thus, option (c) is correct.
sequence as Na, Mg, A1 and increases down the
group in the sequence as Na, K. As metallic
k
61. Alkali metal carbonates (except Li2C03> are more
oo
stable than alkaline earth metal carbonates. Among
character of the element attached to oxygen atom
increases, the basic character of oxides increases. alkaline earth metal carbonates, thennal stability
Y
increases down the group. Thus, MgC03
eB

Hence, the increasing correct order of basic nature


is AI2O3 < MgO < Na20 < K2O. decomposes most easily.

52. Both Ca. CaH2 react with H2O to produce H2 gas. 62. Since Be(OH)2 is least basic, therefore, its
carbonate is least stable, i.e., option (d) is correct.
r

Ca + 2 H2O—^ Ca(OH)2 + H2
63. When gypsum is heated to 393 K, it loses 1-5
ou

CaH2 + 2 H2O > Ca(OH)2 + 2


ad
Y

molecules of water and becomes plaster of Paris.


In contrast, K reacts with H2O to produce H2 but 393 K

KO2 produces O2 (or O-* + H^02) ; Na produces CaS04.2H20 ^CaS04.&5H20+ 1-5 H2O
Plaster of Paris
Gypsum
H2 but Na202 produces O2. Similarly, Ba produces
d

H2 but Ba02 produces O2. 64. Addition of gypsum slows down the setting of
Re
in

53. As the electronegativity of the element decreases cement. Thus, option (a) is correct.
down the group, the ionic character increases, i.e., 67. Their carbides on treatment with H^O give
F

option (a) is correct. methane. (Refer to page 10/41). Aluminium


54. Decreasing hydration enthalpy. Refer to Ans. 28 carbide also reacts with to give methane.
for explanation. AI4C3 + 12 HjO ■> 4 A1(0H)3 + 3 CH4
55. The solubility of alkaline earth metal sulphates 68. Due to small size and high hydration enthalpy Be
decreases down the group due to a corresponding compounds readily hydrolyse.
decrease in hydration enthalpy. 69. Suspension of slaked lime, Ca(OH)2 in water is
56. Among alkaline earth metal sulphates, lattice called milk of lime.

enthalpy remains almost constant but hydration 70. 3 Ca(OCl)2 2 CaCl-) + Ca(C103)2
enthalpy decreases down the group. Thus, BeS04
has higher hydration enthalpy as compared to 71. CaC2 + N2 4 CaCN2 + C
lattice enthalpy. 74. Ca^'*’ maintains the regular beating of the heart.
s-BLOCK ELEMENTS (ALKALI AND ALKALINE EARTH METALS) 10/69

For Difficult Questions

Dl Multiple Choice Questions (with One or More than One Correct Answers)

76. K forms superoxidc, KO-) while others form (c) 3 Cii + 8 HNO3 (dil.) »3 Cu(N03)2
peroxides. + 4 H2O + 2 NO

77. («)Na + (A + y)NH3 ^[Na(NH3]+ + e-(NH3), (paramagnetic)


(d) O2 and 2-ethyIanthraquinol react to produce
Solvated or ammoniated electron which is H-,02 which is not piiramagnetic but is diamagnetic.

w
paramagnetic. 79. BcO is covalent while other oxides are ionic.
(6) K + O2 - ■¥ KO2 (potassium superoxide) 80. Being covalent LiCI and BeCl2 are soluble in
(excess) (paramagnetic) pyridine.

F lo
DU Multiple Choice Questions (Based on the given Passage/Comprehens ion)

e
86. Being predominantly covalent, LiCl has the lowest

Fre
81. Alkali metal oxides are most basic followed by
alkaline earth metal oxides while transition metal m.p.
oxides are least basic. Amongst alkali and alkaline 87. As the size of the cation increases, electro

for
earth metal oxides, basicity increases down the negativity decreases but the electronegativity
group. Thus, C.S2O is more basic than K2O and difference increases and hence ionic character
SrO is more basic than MgO. Therefore,tlie overall increases accordingly, i.e., option (a) is correct.
r
order is : NiO < MgO < SrO < K2O < CS2O. 88. As the size of the cation increases the covalent
You
82. The solubility of bicarbonates of alkali metals
oks
character decreases. Thus option (c) is correct, i.e.,
increases down the group. But alkaline earth metal NaCl < LiCI < BeCL.
carbonates are insoluble in H2O, i.e., option (d) is
eBo

89. LiCI being covalent has the lowest melting point.


correct.
Among the remaining chlorides, because of
83. CaO, CaC03 and Ca(OH)2 are all bases and hence smaller size of Na. NaCl has the highest lattice
must dissolve in acetic acid to form calcium
energy and hence has the highest melting point.
our

acetate. Only calcium oxalate does not dissolve


ad

90. Due to small size, MgO has the highest lattice


in CH3COOH. energy.

VI. Integer Type Questions


dY
Re

96. Three ; K, Rb, Cs. 99. Five ; Li, Mg, Ca, Sr, Ba.
Fin

97. Two ; Be and Mg. 100. Six ; K2SO4, BeS04, Li2S04, MgS04, Na2S04.
98. One ; Be(OH>2. Rl)2S04.

VII.
Numerical Value Type Questions Decimal Notation)
= -795 + 0-2x-188 kJmol-‘
101. AyH° for the given reaction
= -4l9kJ mol-'
= AyH° (products) - AyH° (reactants)
= (CaCl2) + AjrH° (Ca) - 2A^H“ (CaCl)

1
10A70 New Course Chemistry (Xl)CS19n
\ ●:

For Difficult Questions

VIII.
Assertion-Reason Type Questions

102. Correct statement-1. Potassium metal is softer the mass of hydrated Na'*' ions is higher than that
than sodium metal. of K"*" ions and, therefore, their mobility is lower
103. Correct explanation. Due to bigger size of K'*' than that of K"*" ions.
than Na'*', it has a weak positive field around it 116. Correct reason. Among alkali metals, the extent
which cannot prevent peroxide ion (O^") to of hydration decreases as the size of the cation

w
combine with another oxygen atom to form increases from Li'*' to Cs'*'. In other words, Li'*' ions
are most highly hydrated {i.e., have the largest size)
superoxide, .
and hence their ionic mobility, i.e., conductance

F lo
104. Correct explanation. The blue colour of the is the least among alkali metals.
solution is due to ammoniated electrons, i.e.,
e- 118. Correct assertion. L12CO3 is unstable and
decomposes on heating to form Li20 and CO2 but
105. Correct statement-2. Due to pseudo noble gas Na2C03 is thermally stable.
configuration, Cu'*’ ion is more polarising than Na'*'

ee
ion. Correct reason. Li2C03 is thermally unstable

Fr
because it is salt of a small cation and big anion.
106. Correct statement-2. The radius of hydrated
caesium ion is the smallest.
In contrast, Na2C03 is thermally stable because a
bigger cation stabilizes a bigger anion.

for
107. Correct statement-1. Be and Mg do not impart 119. Correct reason. KHCO3 is more soluble in H2O
any colour to the flame. than NaHC03.
ur
110. Correct reason. The low solubility of LiF in water 120. Correct reason. Common salt itself is not hygro
is due to its high lattice energy. scopic but it contains impurities of MgCl2, CaCl2,
oks
112. Correct explanation. Same as in An.s. 88. MgS04, CaS04, etc. which are hygroscopic.
Yo
113. Correct reason. Since for salts of larger anions, 126. Correct explanation. Bleaching powder is a
o

the lattice energy increases with increase in the mixed salt of Ca(OCl)2, CaCl2 and Ca(OH)2.
eB

size of the cation from K"*" to Cs'*’, therefore, the C0CI2 decomposes Ca(OCl)2 to form CaCl2 and
stability of their superoxides increases from K to O2.
Cs.
127. Correct assertion. Mg can be obtained by the
114. Correct reason. Li being small polarizes the large electrolysis of fused MgCl2.
our
ad

2+
Cr ion thereby making LiCl predominantly Correct reason. The electrode potential of Mg
covalent. is far lower (- 2-36 V) than that of H2O (- 0.083 V)
115. Correct explanation. Because of smaller size, the 128. Correct explanation. Human teeth is made up of
extent of hydration of Na"*" ions is higher than that hydroxyapatite, Ca5(P04)30H with enamel made
Y

of K"*" ions. As a result, effective radius and hence up of fluoropatile, 3 Ca3(P04)2.CaF2.


Re
nd
Fi

4
ow
e
Fl
re
SOME p-BLOCK ELEMENTS

F
ur f±MTJ

r
11.1. GENERAL INTRODUCTION

fo
What are p-Block elements ? Elements in which the last electron enters any one of the three p-orbitals
of their respective outermost shells are called p-block elements. Since a /7-subshell has three degenerate
ks
(same energy) /7-orbitals, each of which can accommodate two electrons, therefore, in all, there are six groups
Yo
ofp-block elements, i.e., groups 13, 14, 15, 16, 17 and 18 (excluding helium), each containing six elements.
oo
Thus, in all, there are 36 /7-block elements in the periodic table. (Table 11.1) The atoms of elements of these
groups receive their last electron in 2p, 3p, 4/7, 5/7, 6/7 and Ip- orbitals. Boron, carbon, nitrogen, oxygen,
B

fluorine and neon are the first members of these groups.


re

TABLE 11.1. p-Block Elements


u

18
ad

P-BLOCK ELEMENTS
Yo

Group 1 Group—► 13 14 15 16 17

Group 2 B5 C6 N7 08 F9 Ne10

Groups 3-12 A|13 Si14 p15 S16 C|17 Ari8


d
Re

s-BLOCK
Ga31 Ge32 As33 Se34 Br35 Kr36
in

ELEMENTS
d-BLOCK
●Lt;.' "r'^r In49
ELEMENTS SnSO Sb5i Te52 |53 Xe54
F

T|81 Pb82 Bi83 Po84 At85 Rn86

Uut113 Uuq114 Uiip118 Uijh118 UusH^ uuoiia


f-BLOCK ELEMENTS
LANTHANOIDS

ACTINOIDS

* Not included in CBSE syllabus. This chapter has been given only for the preparation of competitive examinations.

11/1
11/2 ^%€uUe^'4. New Course Chemistry (XI)S!EISI]

11.2. GENERAL CHARACTERISTICS OF P-BLOCK ELEMENTS


1. Electronic configuration and oxidation states. Boron, carbon, nitrogen, oxygen, fluorine and
neon head these groups of p- block elements. Their valence shell electronic configuration is np^~^
where n = 2~l. The inner core of their electronic configuration may, however, differ. This difference in
inner core of electronic configuration of these elements greatly influences their physical properties (such as
atomic and ionic radii, ionization enthalpy, electron gain enthalpy, electronegativity, etc.) as well as chemical
properties. Consequently, lot of variation in properties of elements within a group of p-block elements is
observed. The maximum (or the highest) oxidation state shown by a /;-block element is equal to the sum of
the valence electrons {i.e., the sum of s- and p-electrons) or the group number minus 10. This is called group
oxidation state. Besides, the group oxidation state, p-block elements show a number of other oxidation

w
states. Clearly, as the number of and p-electrons in the valence shell increases, the number of such possible
oxidation stales increases towards the right of the periodic table. These oxidation states usually but not
necessarily, differ from the maximum oxidation state by unit of two. The common oxidation states shown by

F lo
/>-bIock elements are shown in Table 11.2.
TABLE 11.2. General electronic configuration and
common oxidation states of p-block elements

ee
13 14 15 16 17 18
Group No.

Fr
-> 2 2 3 2 4 '> 5 ■’ 6
ns^ np
1
General electronic ns~ np^ ns^ r^r ns^ up ns~ np-^ ns~ np^
configuration

for
+3 +4 +5 +6 +7 +8
Group oxidation state
ur
B C N O F Ne, Ar = 0
+3 +4,^ +5 to -3 -1,-2,+1,+2 -1 Kr
s
ook
Various oxidation state AI Si P, As S, Se, Te Cl, Br, I +2, +4
Yo
+3 +4 +3, +5. -3 -2, +2, -1,+1, Xe
eB

Ga, In, T1 Ge, Sn, Pb Sb, Bi +4, +6 +3, +5, +7 +2, +4,
+3, +1 +4, +2 +5, +3 + 6,+ 8

In boron, carbon and nitrogen families, the group oxidation slate is the most stable for the lighter elements
our

in the group. However, a lower oxidation state which is two units less than the group oxidation slate becomes
ad

progressively more stable for the heavier elements in each group. For example, the group oxidation slate of
group 13 elements is + 3 but + 1 oxidation state is most stable for thallium (Tl). Similarly, the group oxidation
state for group 14 elements is + 4, but + 2 oxidation stale is most stable for lead (Pb).
Y

This trend of occurrence of oxidation state two units less than the group oxidation state is called inert
Re

pair effect and becomes more prominent as we move down the group.
nd

2. General chemical behaviour. It is interesting to note that out of the four blocks (,?-, p-, d- and/-) of
elements in the periodic table, p-block is the only one which contains metals, non-metals and metalloids. The
Fi

common metals among p-block elements are : aluminium, gallium, indium and thallium (group 13); tin and
lead (group 14) and bismuth (group 15). The common metalloids are : silicon, germanium, arsenic, antimony
and tellurium while all the remaining elements are non-metals. In general, the non-metallic character of the
elements decreases down the group. In fact, the heaviest element in each p-block is the most metallic in
nature. This change from non-metallic to metallic character is responsible for diversity in the chemistry of
p-block elements depending on the group to which they belong.
In general, (i) Non-metals have higher ionization enthalpies and higher electronegativities than those of
metals. Therefore, in contrast to metals which readily form cations, non-metals readily form anions.
(ii) The compounds formed by the union of highly reactive metals with non-metals are generally ionic
because of large differences in their electronegativities.
SOME p-BLOCK ELEMENTS 11/3

On the other hand, compounds formed by the union of non-metals themselves are largely covalent in
character due to small ditferences in their electronegativities. The change from non- metallic to metallic
character can be best illustrated by the nature of oxides they form. Whereas oxides of non-metals are either
acidic or neutral, the oxides of metals are always basic in nature.
However, oxides of metalloids are amphoteric. In general, more electropositive the metal, the more

ow
basic is its oxide and more electronegative the non-metal, more acidic is its oxide. Therefore, among
p-block elements, the acidic character of the oxides increases or basic character decreases along a period.
Similarly, the basic character of the oxide increases or the acidic character decreases down the group.
11.2.1 Difference In Chemical Behaviour of First Element of each group
as compared to the subsequent members of the same group
It is interesting to note that first member of each group of p-block elements (particularly groups

e
13-17) differs from its succeeding members (called congeners) of their respeclives groups.

re
The two main reasons for this difference are :

Frl
(/) Size and other properties (such as electronegativity, ionization enthalpy) which depend upon size,

F
(ii) Absence of d-orbitals in their valence shell.
1. Size and other properties which depend upon size. Due to small size, high electronegativity and
high ionization enthalpy, the first element of each group of p-block elements differs from rest of the members
ou
or
of their respective groups in much the same way as the first element of each group of r-block elements, i.e.,
lithium or beryllium differs from rest of the elements of their respective groups.
2. Absence of d>orbitals. The second important difference, which applies only to the p-block elements,

kfs
arises due to the absence of d-orbitals in the elements of second period and the presence of i/-orbitals in the
heavier elements (starting from the third period onwards). Some of these differences are discussed below :
oo
(a) Maximum covalency of four. The first member of each group, (i.e., elements of second period) has
four orbitals (one 2s- and three 2p-orbitals) in the valence shell for bonding and hence can accommodate at
Y
the maximum 4 pairs or 8 electrons. In other words, these elements show a maximum covalency offour. In
B

contrast, elements of third (and higher) periods of p-block elements have vacant 3^/-orbitals lying between
3p- and 4r-levels of energy. Using these z/-orbitals, the elements of third (and higher) periods can accommodate
re

more electrons and hence can expand their covalency beyond four. For example,
(/) boron forms only [BF^]- or [BH4I- ion while A1 gives [AIF^]'’- ion.
oYu

(//) Carbon forms only tetrahalides (CX^, X = F, Cl, Br, I) whereas other members form hexahalides,
ad

i.e., [SiF^]-^ [GeClg]^-, [SnCl^]^-, [PbCI^]^-, etc.


(Hi) Nitrogen forms only NF3 (having an octet of electrons in the valence shell) while phosphorus forms
both trihalides, i.e., PF3, PCI3, etc. and penlalialides, i.e., PFe and PCle (having 10 electrons in the valence
d

shell).
in

(iv) Fluorine does not form FCI3 having 10 valence electrons while chlorine forms CIF3 (having 10
Re

valence electrons).
(b) Reactivity. Due to presence of d-orbitals, the elements of third (and higher) periods are more reactive
F

than elements of second period which do not contain ^/-orbitals. For example, tetrahalides of carbon (i.e.,
CCI4, CBr4, etc.) are not hydrolysed by water while tetrahalides of other elements of group 14 (i.e., SiC^,
S11CI4, etc.) are readily hydrolysed. This hydrolysis involves the nucleophilic attack by water molecules and
the pair of electrons provided b ' H2O is accommodated in the vacant d-orbitals.
(c) Tendency to form n iltiple bonds. The presence of ^/-orbitals also influences the chemistry of
heavier elements in a number of other ways. The combined effect of size and availability of r/-orbitals greatly
affects the ability of these elements to form TC-bonds. Thus, the first member of each group differs from the
heavier elements in its ability to form pTU -p7i-multiple bonds either with itself (e.g., C = C, C = C, N s N) or
with the other elements of the second period (e.g., C = O, C = N, N = O, C = N). This type of 7C-bonding is,
however, not strong in case of heavier p-block elements. The heavier elements also form 7C-bonds but this
involves ^/-orbitals (i.e., dn-j.m or dti - dn).
11/4 "P^uideep. 'a New Course Chemistry (XI)

For example, in SO2, one of the two jc-bonds between FIGURE 11.1

S and O involves dn-pn bonding while the other involves


pn- pn bonding. In contrast in SO3, two of the three 71-
bonds involve dn - /m-bonding while the third one involves
pn - pit bonding. In these dn - />7t-bonds, a half-filled l>d-
orbital of sulphur overlaps with a half-filled 2/>-orbital of HALF-FILLED HALF-FILLED

oxygen as shown in Fig. 11.1. 3d-ORBITAL 2P-ORBITAL

Since tZ-orbitals are of higher energy than p-orbitals, Overlapping of d- and p-orbitals
therefore, they contribute less towards the overall stability

w
to form d7C-p7t double bond
of the molecules as compared to the pn - pn bonding
between elements of second period. But at the same time, such type of bonding may increase the coordination
number in species of heavier elements in the same oxidation state. For example, in + 5 oxidation state, both

F lo
N and P form oxoanions such as NO" (three coordination in which N uses one p-orbital for 7t-bonding) and

PO4" (four coordination in which P uses one ti-orbital to form dn-pn bond with O).

ee
In this unit, we shall discuss the chemistry of groups 13 and 14 elements of the periodic table.

Fr
PART—I. GROUP 13 ELEMENTS : BORON FAMILY

113. INTRODUCTION
The group 13 of the periodic table consists of the elements bomn (B), aluminium (Al), gallium (Ga),

for
ur
indium (In), thallium (Tl) and the newly discovered element, ununtrium (uut) which is radioactive. Except
boron which is a non-metal, all other elements of this group show typical metallic properties. The non-
metaliic character of B is possibly due to its small size, high ionization enthalpy and comparatively high
ks
electronegativity.
Yo
oo
11.4. OCCURRENCE

Boron occurs in two isotopic forms, (19%) and ^^B (81%). Its abundance in the earth s crust is
eB

very low (0-0001% by mass). Boron mainly occurs as


(/) Orthoboric acid, H3BO3
(h) Borax, Na2[B40g{0H)4] . 8H2O or Na2B407. IOH2O
r
ou
ad

(/«■) Kernite, Na2[B405(0H)4] or Na2B407.2H2O


(/V) Colemanite, Ca2[B304(0H)3]2 ■ 2H2O or Ca2BgOu .5H2O
Y

Boron in form of borates is chiefly found in California (U.S.A.) and Turkey. In India, boron occurs as
borax in Puga Valley of Ladakh region of Kashmir and Sambhar Lake in Rajasthan.
Aluminium is the third most abundant element by weight (8-3%) found in the earth’s crust after oxygen
Re
nd

(45-5%) and silicon (27-7%). The important minerals of aluminium are : (/) Bauxite (hydrated aluminium
oxide) : Alo03, 2 H2O. It may also be represented as AlO^(0H)3^2c where 0 < x < 1.
Fi

(n) Cryolite (sodium aluminium fluoride): Na3AlF^.


(Jii) Mica (muscovite) : K2O.3 AI2O3.6 Si02.2 H2O or KAl2(Si3A10jQ) (OH>2.
(/V) Orthoclase (feldspar) ; K2O.AI2O3.6 Si02 or KAlSi30g.
(v) Corundum (anhydrous alumina); AI2O3. (v() Beryl: 3 Be0.Al203.6 SiOj or Be3Al2S%Ojg.
In India, mica is found in Kami, Rewa (Madhya Pradesh), Belgaum (Karnataka), Orissa and Jammu. In
fact, India is the world’s largest supplier of Mica.
Gallium, indium and thallium are less abundant than aluminium. Highest concentration of Ga
(0-1-1 %) is found in the rare mineral Germanite which is a complex sulphide of Zn, Cu, Ge and As.
Traces of indium are found in the sulphide ores of zinc while that of thallium are found in the sulphide
ore of lead.
SOME p-BLOCK ELEMENTS 11/5

11.5. ELECTRONIC CONFIGURATION


The general valence shell electronic configuration of elements of group 13 is ns^ np^ where « = 2-7.
A close look at the electronic configuration suggests that while boron and aluminium have noble gas core,
gallium and indium have noble gas plus 10 i/-electrons and thallium has noble gas plus 14/-plus I0^/-electron
cores. Thus, the electronic configurations of elements of group 13 are more complex than those of groups 1
and 2. This difference in electronic configurations affects the chemistry of all the elements of this group. The
complete electronic configuration of these elements is shown in Table 11.3.
TABLE 11.3.
Electronic configuration of elements of group 13

w
Element Atomic
Electronic Configuration
Number Complete With inert gas core

Flo
Boron (B) 5 15^ 2^2 2p* [He] 2^2 2p'
Aluminium (Al) 13 u2 2s^ 2p6 3i-2 3p^ [Ne] 3^2 3pl
Gallium (Ga) 31 2s2 2p^ 3j2 3p6 3^10 4^2 4^1 [Ar] 3d'0 4j2 4/-'

e
ls2 2j2 2p^ 3j2 3yj6 3jl0 4^2 4^6 4^10 5^2 3^1 [Kr] 4(/'^ 5j2 5p'

re
Indium (In) 49

Thallium (TI) 81 2^2 2^6 3_^.2 3^6 3^10 4^2 4^6 4^10 4^ 14 [Xe]4/*‘‘ 5(/“’6^2 6p‘

rF
5^2 5/ 5rfl0 6^.2 1
Ununtrium (Uut) 113 1^2 2.v2 2p^ 3^2 3p*’ 4,?2 4/ 4/ [Rn]5/‘*^ 6J“^7.s2 7p'
5^2 5p^ 5/ 6^2 752 7p'
ur
fo
11.6. ATOMIC AND PHYSICAL PROPERTIES ks
Some important atomic and physical properties of the elements of group 13 are given in Table 11.4.
Yo
TABLE 11.4.
Some atomic and physical properties of group 13 elements
oo
Elements
Property
B

B Al Ga In Tl
re

Atomic number 5 13 31 49 81
Atomic mass (g mol”') 10-81 26-98 69-72 114-82 204-38
u

*Atomic radius/pm
ad

85 143 135 167 170


Yo

**Ionic radius M^^/pm 27 53-5 62-0 80-0 88-5


Ionic radius M’'''/pm 120 140 150
-1
Ionisation enthalpy/kJ A:f H 1 801 577 579 558 589
nd

A,-H. 2427 1816 1979 1820 1971


Re

A, H3 3659 2744 2962 2704 2877


A,H4 25026 11578
Fi

6200 5200
Electronegativity (Pauling scale) 2-0 1-5 i-6 1-7 1-8
Density /g cm^ 2-35 2-70 5-90 7-31 11-85
Melting point/K 2453 933 303 430 576
Boiling point/K 3923 2740 2676 2353 1730
E°/V at 298 for

M^-*- {aq) + 3e- ^M(^) - 1-66 - 0-56 - 0-34 + 1-26


EW at 298 K for

M'*' {aq) + e ●»M (i) + 0-55 - 0-79 (acid) - 0-18 - 0-34

- 1-39 (alkali)

^Metallic radius. **6-Coordination.


11/6 New Course Chemistry (XI)EZ2lHD

Some of these properties are discussed below.


1. Atomic and Ionic radii. The atomic and ionic radii of group 13 elements are smaller than those of
the corresponding elements of group 2.

ow
Explanation. This is because on moving from left to right, i.e.. from grr ;p 2 to group 13 in a given
period, the nuclear charge increases while the new electron enters the same shell. Further the electrons in the
same shell do not screen each other. Therefore, the effective nuclear charge increases and the outer electrons
are pulled more strongly towards the nucleus. This results in decrease in atomic size. Same is true of ionic
radius.

On moving down the group, both atomic and ionic radii are expected to increase primarily due to

re
addition of a new electron shell with each succeeding element. Howeve,r there are some deviations as we
move from Al to Ga. For example, the ato?nic radius ofGa (135 pm) is slightly lower than that ofAl(l 43 pm).

Flr
Explanation. This is due to the filling of electrons in c/-orbitals. In between Al (Z = 13) and Ga

F
(Z = 31), there are ten elements of the first transition series (Z = 21 to 30) which have electrons in the inner
^/-orbitals. As the J-orbitals are lai-ge in size, these intervening electrons do not shield the nucleus effectively.

ou
Consequently, effective nuclear charge of Ga is greater in magnitude than that of Al. As a result, the electrons
in Ga experience greater force of attraction by the nucleus than in Al and hence atomic radius of Ga is slightly

sr
less than that of Al. In other words, the atomic radii of group 13 elements increase in the order: B < Ga < Al

fo
< In < Tl.

k
The ionic radii, however, follow a regular trend and keep on increasing down the group from B to Tl.
2. Ionisation enthalpy. The ifrst ionization enthalpies (AjH^) of the elements of group 13 are lower
oo
than the corresponding elements of group 2, i.e., alkaline earth metals.
Explanation. This is due to the reason that elements of group 13 have three electrons in the valence
Y
shell ; two of these are present in the A-orbital and one in the /j-orbital. For the first ionization enthalpy
reB

(A,-Hj), the electron has to be removed from the p-orbital in case of group 13 elements whereas in case of
alkaline earth metals (group 2 elements), the ^-electron of the same principal shell has to be removed. Since
an ^-electron is nearer the nucleus (more penetrating towards the nucleus), it is more strongly attracted than
uY

the p-electron of the same principal shell. Hence, the removal of the /7-electron is much easier than the s-
electron and, therefore, the first ionization enthalpies (A,-H|) of the elements of group 13 are lower as compared
to the corresponding values of the alkaline earth metals of group 2.
ad
do

On moving down the group 13 from B to Al, there is a sharp decrease in first ionization enthalpy
(A,- H^) of Al due to an increase in atomic size and screening effect (of the 2s- and 2p- electrons) which
outweigh the effect of increased nuclear charge. Howeve,r the HyofGa is only slightly higher (2 kJ moF^) than
in

that ofAl while that ofTl is much higher than those of Al, Ga and In.
Re

Explanation. This is due to the reason that A! follows immediately after s-block elements while Ga and
In follow after </-block elements and Tl after d- and /-block elements. These d- andf-electrons do not shield
F

the valence elections from the nucleus very effectively than s- and p-electrons (the screening effect decreases
in the order .■ s > p > d > f). As a result, the valence electrons remain fairly tightly held by the nucleus and
hence larger amount of energy is needed for their removal. Now as we move from Al to Ga, due to poor
shielding of the nucleus by 3f/-electrons, the effective nuclear charge acting on Ga is slightly higher than that
on Al. As a result, AjHj ofGa is slightly higher than that ofAl. Further, on moving down the group from Ga

(Z = 31) to In (Z = 49), although the nuclear charge increases by 18 units (49-31) but the total shielding
effect of 3d- and 4t/-electrons outweighs the effect of increased nuclear charge and hence A,W/ of In is lower
than that ofGa. On further moving down the group from In (Z = 49) to Tl (Z = 81), the nuclear charge
increases const derably by 32 units (81-49 = 32) which outweighs the shielding effect all the electrons of the

inner shells including those of 4/- and 5</-electrons. As a result, effective nuclear charge acting on Tl is much

higher than that on In and hence AjH, ofTl is much higher than that ofIn even higher than those ofGa and Al.
SOME p-BLOCK ELEMENTS 11/7

Thus, amongst elements of group 13, B has the highest and In has the lowest first ionization enthalpy. The
overall, AjHj of group 13 elements follows the order :

B > T1 > Ga > AI > In.


Further as expected, the second and third ionization enthalpies (A,//, and A-//3) of these elements (of
group 13) are quite higher than their respective ifrst ionization enthalpies. However, as expected these
successive ionization enthalpies increase in the order: A^Hj < A,H2 < AjH3.
The sum of first three ionization enthalpies for each of the elements is very high. The effect of this will
be apparent when you study chemical properties.

w
3. Electronegativity. The elements of boron family (group 13) are more electronegative than the elements
of alkali metals (group 1) and alkaline earth metals (group 2). The electronegativity first decreases from B to
Al and then increases marginally down the group.

Flo
Explanation. As a we move from B to Al, the atomic size increases considerably, therefore, attraction
of the nucleus for the electrons decreases and hence eieclronegalivily decreases. Thereafter, in spile of the
fact that the size increases (except that of Ga) but the effective nuclear charge increases due to poor shielding

ee
of the inner d- and/-electrons. As a result, force of attraction of the nucleus for the electrons increases and

Fr
hence the electronegativity increases from Al to Tl.
4. Electropositive cliaract-; tallic character. The elements of group 13 are less electropositive or
metallic as compared to alkali metals (group 1) and alkaline earth metals (group 2). On moving down the

for
ur
group, the electropositive character ofthe elements first increases from boron to aluminium and then decreases
from aluminiumto thallium.
Explanation. Amongst the elements of group 13, B has the highest sum of first three ionization
ks
enthalpies, i.e., AyHj + A,H2 + A,-H3. As a result, it has little tendency to lose electrons and hence is least
electropositive amongst group 13 elements. In other words, as expected, it is a non-metal and a poor conductor
Yo
oo
of electricity. However, as we move from B to Al, the sum of A^-H, + A.H2 + A-Hj decreases substantially
(6887 kJ mol * to 5137 kj mol~*) due to increase in the atomic size and hence Al has a high tendency to lose
B

electrons. In other words, Al is highly electropositive. Therefore, Al is a metal and a good conductor of
electricity.
re

The electropositive character of the remaining elements can be more easily explained on the basis of
their respective electrode potentials. Since the electrode potentials for the reaction, (aq) + 3 £●" -> M(.?)
ou

increases from Al to Tl, therefore, their electropositive character decreases, i.e., Al (- 1-66 V) to Ga
ad

(- 0-56 V) to In (- 0-34 V) to Tl (+ 1-26 V) accordingly.


Y

5. Melting points and boiling points. The melting points of group 13 elements do not show a regular
trend as shown by elements of groups 1 and 2. This is probably due to the unusual crystal structures of B and
Ga. Actually, the melting points decrease sharply on moving down the group from B to Ga and then increase
nd
Re

from Ga to Tl. Thus, amongst the elements of group 13, Ga has the lowest melting point (303 K) and could
exist as a liquid at room temperature in summer.
Fi

Boron has a high melting point (2453K) because its crystal structure consists of icosahedral* (Fig. 11.8,
Page 11/13) units with B atoms at all the 12 comers and each boron atom is bonded to five equidistant
neighbours. In contrast, the crystal stmeture of Ga is quite different from that of B. Each Ga metal atom has
one close neighbour at a distance of 243 pm and six more distant neighbours at distances between 270 and
279 pm. This unique structure suggests that Ga consists of almost discrete diatomic molecules and hence its
melting point is exceptionally low. Its high boiling point (2676 K), however, makes it a useful material for
measuring high temperatures, i.e., it is used in high temperature thermometry. Another unusual properly of
Ga is that like Ge and Bi, liquid Ga expands when it changes into solid, i.e., density of solid Ga is less than
that of liquid Ga.

*A solid with 20 faces and 12 comers.


11/8 New Course Chemistry (XDEZSQE]

In contrast, the elements Al, In and T1 have close packed structures. Their melting points decrease from
A1 to In and increases again for Tl.
However, the boiling points of these elements follow a regular trend and decrease regularly on moving
down the group.
6. Density. Due to smaller atomic and ionic radii, the elements of group 13 have higher densities as
compared to elements of group 2.
On moving down the group, the densities increase. This is due to a corresponding increase in the atomic
mass of the element which outweighs the effect of increased atomic size. The densities of boron and aluminium

are, however, quite lower than those of other members.

n.7. CHEMICAL PROPERTIES

w
Some important chemical properties of group 13 elements are described below :

F lo
11.7.1. Oxidation States and Trends in Chemical Reactivity
1. Oxidation states. The elements of group 13 have two electrons in the ^-orbital and one electron in
the p-orbital of the valence shell. So these elements are expected to show a uniform oxidation state of +3.
This is true of boron and aluminium which show an oxidation state of +3 only but gallium, indium and
thallium due to inert pair effect show oxidation .states of both +1 and +5.

ree
Further, as we move down the group, the stability of the +3 oxidation state decreases while that of+1

F
oxidation state progressively increases. In other words, the order of stability of + 1 oxidation state increases

for
in the order: Al < Ga < In < Tl. In fact, in Ga, In and Tl, both +1 and +3 oxidation states are observed. Since
than +1 oxidation state, therefore, both Ga + and In +
in case of Ga and In, +3 oxidation state is more stable
salts undergo disproportionation (self oxidation-reduction) reactions in aqueous solution :
r
+ 1 0 +3
You
oks

3GaX(s) > 2Ga(5) + Ga^'*’(a^) + 3X {aq)


eBo

+1 0 +3
3InX(s) > 2In(^) + In^'^(fl^) + 3X (aq)
For example, thallous compounds such as thallous hydroxide (TlOH) and thallous perchlorate (TICIO4)
are more stable than their corresponding thallic compounds. Further due to lesser stability, TP'*’ salts act as
ad
our

strong oxidising agents. This is supported by its electrode potential data.


r\^ (aq) + 2 e " > TI+ (aq) ; E° = + 1-25 V
Explanation. The reason why the stability of+1 oxidation state increases down the group can be explained
Re

as follows :
dY

As we move down tlie group, the tendency of s-electrons of the valence shell to participate in bond
formation decreases. This reluctance of the ^-electrons to participate in bond formation is called Inert pair
Fin

effect. In other words, the ns'^ electron pair in Ga, In and Tl tends to remain paired. This is due to poor or
ineffective shielding of the ns'^ electrons of the valence shell by intervening d- and f-electrons.
Another reason for the inert pair effect is that as the size of atom increases from Al to Tl, the energy
required to unpair the ns^ electrons is not compensated by the energy released informing the two additional
bonds.

The inert pair effect becomes more predominant as we go down the group because of increased nuclear
charge which outweighs the effect of the corresponding increase in atomic size. The .v-electrons thus become
more tightly held (more penetrating) and, therefore, become more reluctant to participate in bond formation.

Thus down the group, +1 oxidation state becomes more and more stable as compared to +3 oxidation state.
SOME p-BLOCK ELEMENTS 11/9

2. Trends in chemical reactivity. The elements of Group 13 have three electrons in the valence shell.
Except Tl, they normally use these electrons to form three bonds and thus show an oxidation state of +3. In +3
oxidation state, elements of this group are expected to form covalent bonds because of the following three
reasons :

(/) According to Fajan rules, the small size of the ions and their high charge of +3, favour the formation
of covalent bonds.

(«) The sum of the first three ionization enthalpies is very large. This also suggests that bonds will be

ow
largely covalent.
(Hi) The electronegativity values of group 13 elements are higher than those of group 1 and 2. Further,
whentheseelementsreactwithotherelements,the electronegativity difference will be small. This also favours
the formation of covalent bonds.

Boron, because of its small size and high sum of first three ionization enthalpies, does not lose its three
valence electrons to form ions. Therefore, it does not form ionic compounds. Instead, boron always

e
Fl
re
forms covalent compounds by sharing its valence electrons.
But as we move from B to Al, the sum of first three ionization enthalpies considerably decreases and

F
thus aluminium has only little tendency to form ionic compounds. Instead, it has a strong tendency to
form covalent compounds. For example, AIF3 and Al2(S04)3 are ionic while AICI3, AlBr3 and AII3 are
ur
covalent.

or
Like aluminium, many compounds of the other elements such as GaCl3, InCl3 are covalent when

sf
anhydrous. However, Al, Ga, In and Tl all form metal ions in solution. This change from covalent to ionic
nature occurs due to the reason that in aqueous solutions, these ions are hydrated and the amount ofhydration
k
Yo
enthalpy released exceeds the ionization enthalpy. For example, in case of AICI3, 5137 kJ mol“* energy is
required to convert Al to AP+, for Al^+ is - 4665 kJ mol”’ and AH^ydration for Cl” ion is - 381 kJ
oo
mol"’. Thus, the total hydration enthalpy is - 4665 + 3 x (- 381) = - 5808 kJ mol"’. Since this exceeds
ionization enthalpy (5137 kJ mol"’), therefore, AICI3 ionizes in solution.
B

As stated above, other elements of this group, i.e., Ga, In and Tl show two oxidation states of+1 and +3
e

due to inert pair effect. Since the ions in the +1 oxidation state are much larger than the ions in the +3
ur

oxidation state, therefore, these compounds in +7 oxidation states are more ionic than in +3 oxidation state.
Further, in the trivalent state, most of compounds being covalent are hydrolysed by water to form either
ad
Yo

tetrahedral species, [M(OH)4]~ in which the element is 5p^-hybridized or octahedral species, [M(H20)g]^''’ in
which the element is sp^J^-hybridized. For example, BCI3 on hydrolysis forms tetrahedral [B(OH)4]" species
because boron due to the absence of f/-orbitals cannot expand its covalency beyond four. In contrast, AI2CI6
d

on hydrolysis gives octahedral [A1(H20)6]^"'' species because Al due to the presence of 3t/-orbitals can expand
Re

its covalency from four to six.


in

In trivalent state, the number of electrons around the central atom in a molecule (e.g., BF3, AICI3, etc.)
F

of these elements will be only six and thus behave as electron deficient molecules. In other words, they have
a strong tendency to accept a pair of electrons to achieve the stable inert gas configuration and thus behave as
Lewis acids. Thus, BCI3 easily accepts a lone pair of electrons from ammonia to form the adduct, BCI3.NH3

NH3
1
B—Cl + :NH3 B
Cl' Planar
Cl Cl
Cl
Tetrahedral
11/10 “pfuicCee^'^ New Course Chemistry (XI)EZalHI]

The Lewis acid characte,r howeve,r decreases down the group


as the size of the element increases, i.e., BX3 > AIX3 > GaX^ >
InX3 (where X = F, Cl, Br, I, etc.).
It is interesting to note that both boron and aluminium halides
are Lewis acids but only aluminium halides exist as dimers (Fig.
1J.2) whereas boron halides exist only as monomers. This is due to
the reason that boron atom is so small that it cannot accommodate
Structure of aluminium (III) chloride

ow
four large sized halogen atoms around it.

11.7.2. Reactivity towards Dioxygen or Air


All the metals of group 13 react with dioxygen at high temperatures to form trioxides of the formula, M2O3.
High lemp.
4M(s) + SOjig) 2M203(j)

e
Besides, TI2O3, TI also forms some TI2O.

Fl
re
The reactivity of these elements towards dioxygen, however, increases down the group.
Boron is unreactive in the crystalline form. Aluminium does not react with dry air. However, in moist

F
air, its surface gets tarnished (appears dull) due to the formation of a very thin oxide layer on the surface
which protects the metal from further attack. Amorphous boron and aluminium metal on heating in air form
ur
r
boron irioxide (8^03) and aluminium trioxide (AI2O3) respectively.

fo
973 K FIGURE 11.3
4 B (s) + 3 O2 (g) ^ 2 B2O3 (s) ;
Heat
ks
Yo
4 A1 (.9) + 3 (s) » 2 AI2O3 (j) N N N N
oo
With dinitrogen at high temperatures, they form nitrides.
High temp.
B

2 B (^) + N2 (g) > 2 BN (.9)


High temp.
e

2A1(.9) + N2 (g) » 2 AIN (9)


ur

The remaining elements, Ga, In, Tl do not react with


N2 to form the corresponding nitrides.
ad

Structure of boron nitride


Yo

Boron nitride is a white slippery solid. One B and one N


atom together have the same number of valence electrons as
two C atoms. Therefore, boron nitride has almost the same
d

structure as graphite (Fig. 11.16, page 11/31), consisting of sheets


Re

made up of hexagonal rings of alternate B and N atoms joined B2O3 & B(0H)3 Acidic
in

together (Fig. 11.3). {£


lU
AI203 &Ai(OH)3 Amphoteric o w
F

These sheets are stacked one on top of the other, giving a LU

layered structure similar to that of graphite. It is because of the 03303 & Ga{OH)3 Amphoteric is
similarity of structure of boron nitride and graphite that boron
iri203 & in(OH)3 Basic Qz
nitride is also called inorganic graphite. w “
<

Acid-base character of oxides and hydroxides. The TI2O &TIOH Strongly basic
trioxides of group 13 react with water to form their corresponding
hydroxides.
M2O3 + 3 H.O ^ 2M(OH)3
The nature of these oxides and hydroxides varies down the group. B^C>3 and B{OH)^ are weakly acidic.
They dissolve in alkalies forming metal borates.
SOME p-BLOCK ELEMENTS 11/11

B2O3 + 2NaOH 2NaB02 + H2O


Sod. mctaborale

On moving down the group, the acidic character decrea.ses and the basic character increases. This is
because down the group, ionisation enthalpy decreases. Consequently, M-0 bond weakens and is easily
broken resulting in increased basic strength down the group. Thus, the oxides and hydroxides of A1 and Ga
are amphotericwhile those of indiumand thallium are basic. Thallium forms two hydroxides: thallic hydroxide,
T1(0H)3 and thallous hydroxide, TIOH.TKOH)^ is insoluble in H2O but TlOH is soluble and is a strong base
like alkali metal hydroxide.

w
Being amphoteric, alumina and aluminium hydroxide dissolve both in acids as well as in alkalies fomiing
salts.

Fuse

(0 AI2O3 + 2NaOH ■>


2NaA102 + H2O

Flo
Sod. meia-aJuminate

or A1(0H)3 (5) + NaOH {aq) Na+ [A!(0H)4]- iciq)

ee
Sod. toirahydroxoaluminate(III) ion

Fr
(n) 2 A1(0H)3 (5) + 3 H2SO4 {aq) ^ Al2(S04)3 (aq) + 6 H2O (/)
11.73. Reactivity towards Adds and Bases
(a) Action of acids. Boron does not react with non-oxidising acids such as hydrochloric acid. However,

for
it is attacked at high temperatures by strong oxidising acids such as a mixture of hot cone. H2SO4 and HNO3
ur
(2:1) forming boric acid (H3BO3)
Conc.HTS04
ks
B (5) -F 3 HNO3 iaq) F H3B03(fl^) -F 3N02(g)
Yo
Heat
Boric acid
oo
All other elements react with both non-oxidising and oxidising acids. For example, A1 reacts with dilute
eB

acids liberating H2 gas.


2 A1 (j) -F 6 HCl {aq) ■F 2 Al^+ {aq) -F 6 Cl' {aq) -f 3 H2 {g)
With cone. HNO3, A1 becomes passive. This passivity is due to the formation of a thin protective layer
r

of its oxide (A/2O3) on the .surface of the metal which prevents it from further action.
ou
ad

2 A1 + 6 HNO3 {cone.) ^ AI2O3-F 6 NO2 + 3 HjO


(b) Action of alkalies. Boron resists the action of alkalies (NaOH or KOH) upto 773 K but above this
Y

temperature, it reacts forming borates and liberating dihydrogen gas.


>773K
Re
nd

2 B (5) -F 6 KOH (5) F 2 K3BO3 (.9) -F 3 H2 {g)


Pol. boraie
Fi

It also dissolves in fused Na2C03/NaN03 mixture at 1123 K


2 B (5) -F 3 Na2C03 (9) -F 3 NaN03 {s) F 2 Na3B03 (j) -f 3 NaN02 {s) + 3 CO2 (g)
A1 and Ga being amphoteric also react with aqueous alkalies with the evolution of H2 gas
2 A1 (5) + 2 NaOH {aq) -F 6 H2O (/) ■F
2 Na+ [A1(0H)4J- (^7^) -F 3 H2 (g)
Sod. tetrahydroxoaluminate (III)
2 Ga {s) + 2 NaOH {aq) + 6 H2O (/) ^ 2 Na+ [Ga(OH)4]- {aq) -F 3 H2 (g)
Sod. tetrahydroxogaliate (111)
In and Tl, however, do not react with alkalies.
11/12 New Course Chemistry fXlHwii

Curiosity Questions
r Q. 1. What material is used to prepare bullet proof vests ?
Ans. When boron oxide is reduced with carbon in an electric furnace, boron carbide (B4C) is formed.
Electric furnace

2 B20g(s) -h 7 C (s) !873K


» B4C(s) -h 6 CO ig)
Boron carbide is even harder than diamond. Therefore, it is used for making bullet-proof vests.
It Is also used as a shield for radioactive radiations and also as an abrasive.

w
Q. 2. Nitric acid Is highly corrosive and a strong oxidising agent. To avoid accidents caused
by possible breakage of glass containers during transportation, what other unbreakable
container can be used ?

Ans. Nitric acid can be safely transported in aluminium containers. The reason being that Al becomes

Flo
passive due to the formation of oxide layer on its surface. This oxide layer being impervious
does not allow the acid to come in contact with the metal. As a result, further action stops.

2 Al + 6 HNO3 {cone.) > AlgOg -t- 6 NO2 + 3 HgO

e
re
11.7.4. Reactivity towards Halogens

rF
The elements of group 13 react with halogens at high temperatures forming trihalides of the general
formula, MXg.
ur Heat

fo
2 M (5) -h 3 X2 (g) » 2 MX3 (X=F, Cl, Br, I)
Thallium (III) iodide (TII3) is, however, unknown. Here, we shall discuss only trihalides of boron.
ks
FIGURE 11.4
Yo
oo
2s 2p
B EMPTY

(GROUND t EMPTY
B

STATE) ‘ ' '— -ORBITAL


re

B
t t t
X
(EXCITED
STATE) L sp2-HYBRIDIZED
u
ad

BORON
sp2-HYBRIDIZATION
Yo

Structure of boron trihalides


d
Re

Trihalides of Boron. Due to small size and high ionization enthalpy, boron forms covalent trihalides.
in

BF3 is a gas, BCI3 and BBr3, are liquids while BI3 is a solid. All these trihalides are planar molecules in
which B is .9/?--hybridized (Fig. 11.4). The three half filled ;i-orbitals, one on each halogen, overlap along
F

their internuclear axis with three .s/i^-orbitals of boron to form three sp^-p, C—X, a-bonds. The unhybridized
/>-orbital is, however, empty.
Due to the presence of only six electrons in their respective valence shells, FIGURE 11.5
all these trihalides are Lewis acids. Their Lewis acid character, however,
decreases in the order : X

BI3 > BBr3 > BCI3 > BF3 B X


X
Explanation. This order of acidic strength can be easily explained on
the ha.sis of the tendency of the halogen atom to back donate its lone pair of
electrons to the boron atom through pn - pn- bonding (Fig. 11.5). pTl—p7t-Back bonding
in boron trihalides
SOME p-BLOCK ELEMENTS 11/13

Since the size of the vacant 2p-orbital of B and the FIGURE 11.6

2/?-orbital of F containing a lone pair of electrons are


almost identical, therefore, the lone pair of electrons on
F.

A. ●●Q-
F is donated towards the B atom. Fuither due to back
F'
B
► _^B=F
F
donation by three F-atoms, BF3 can be represented as a
resonance hybrid of the following three structures (Fig. F.
11.6). The back bonding in BF3 molecule is supported B F :B F
by the fact that the observed B—F bond length (130 pm)
in BF3 molecule is much less than the sum of their
covalent radii (B = 85 pm and F = 64 pm).

w
As a result ofpK - pn-back donation and resonance, Resonance structures of BF3 molecule
the electron deficiency of B decreases and thus BF3 is involving pTC-p7t back bonding
the weakest Lewis acid. As the size of the halogen atom

F lo
increases from Cl to Br to I, the extent of overlap between
2p orbital of B and a bigger p-orbital of halogen (3/7 in Cl, 4p in Br and 5p in 1) decreases and consequently
tlie electron-deficiency of B increases and thus the Lewis acid character increases accordingly from BF3 to

ee
BCI3 to BBr3 to BI3. Thus, the relative lewis acid strength of the boron trihalides follows the sequence :
BF3 < BCI3 < BBr3 < BI3.

Fr
SUPPLEMENT YOUR
KNOWLEDGE FOR COMPETITIONS

for
All the group 13 trihalides react with Grignard reagents
FIGURE 11.7
and Orgolithium reagents, forming irialkyl or triaryl
ur
compounds. For example, CH3
CH 3
BF3 + 3C2H5MgI ■> B(C2H5)3 + 3 MgIF
s
AICI3+ 3CH3MgI A1(CH3)3 + 3 MglCl Ai Al
ook
Yo
GaCl3 + 3C2H5L1 Ga(C2H5>3 + 3 LiCl
CH3 CH3 CH3
InBr3 + 3 CgHjLi ^ In(C6H5)3 + 3 LiBr
Structure of
eB

The trialkylaluminium compounds are unusual because they


have dimeric structures, and appear to have three-centre two trimethylaluminium dimer
electron bonds involving 5p^-hybrid orbitals of A1 and C in
Al—C—A1 bridges (Fig. 11.7).
r
ou
ad

11.8. PROPERTIES OF BORON

(a) Physical propertle.s. (/) Boron is an extremely hard FIGURE 11.8


solid with high m.p. 2453 K. Its boiling point is 3923 K.
Y

(ii) Elemental boron exists in several different allotropic


forms. At least four allotropes of boron may be obtained under
Re
nd

different conditions. However, transition between different forms


is an extremely slow process. All the four allotropic forms contain
Fi

Bj2 icosahedral units (Fig. 11.8) with boron atoms at all the 12
comers. The different allotropic forms differ from one another
in the way the icosahedra are bonded together,
(b) Chemical properties. Chemical properties of boron
have already been discussed in Art. 11.7.2 to 11.7.4, pages
11/10-11/13.

11,9, ANOMALOUS PROPERTIES OF BORON All allotropic forms of boron consist of


B|2 icosahedral units having 12
Boron, the first member of group 13 elements, shows comers and twenty faces. Each
anomalous behaviour and differs from rest of the members of
corner is represented by
its family. The main reasons for this difference are as follows : a thickened circle
(a) exceptionally small atomic and ionic size.
11/14 New Course Chemistry (XI)nsiaD]

(h) high ionization enthalpy,


(c) absence of ^/-orbitals in its valence shell.
Some important properties in which boron differs from the rest of the members of its group (especially
the next member, Al) are as follows :
(/) Boron being small is harder than the other elements of its group.
(//) It has higher melting and boiling points than those of the other members of its group.
(///■) Boron forms only covalent compounds while all other members form both ionic and covalent
compounds. For example. BF3 is covalent while AIF3 is ionic.

w
(iv) The oxide of boron and its hydroxide are weakly acidic and dissolve in alkalies forming metaborates.
The oxide and hydroxides of Al and Ga are amphoteric while those of In and Tl are basic.
B2O3 + 2 NaOH ^2NaB02 + H20 ; B(0H)3 + NaOH NaB02 + 2 H2O
A1(0H)3 + NaOH ^NaA102 + 2H20 ; A1(0H)3 + 3 HCl ^ AICI3 + 3 H2O

Flo
(v) The trihalides of group 13 being covalent are hydrolysed by water. Whereas boron trihalides, due to
the absence of fr-orbitals, form tetrahedral species [B(OH)4]“, trihalides of Al and other elements, due to the

e
presence of <i-orbitals, form octahedral [Al(H^O)g]^''’ species,

re
(vf) The trihalides of B iire monomeric. The reason being that due to its small size, it cannot accommodate
four hirge sized halogen atoms around it. The monomeric trihalides, being electron-deficient, are strong

F
Lewis acids and thus form complexes with ammonia, alcohols, ethers, F “ ion, etc.
H3N : + BF3 ^ H3N^BF3
ur
r
In contrast, the trihalides of Al and other elements of group 13 have halogen bridged dimeric structures

fo
in which the metal completes its octet by accepting an electron pair from a halogen atom of the other molecule
(Fig. 11.2, Page 11/10). ks
11.10. SOME IMPORTANT COMPOUNDS OF BORON AND ALUMINIUM
Yo
Some useful compounds of boron are borax, orthoboric acid and diborane. Here, we will briefly discuss
oo
their chemistry.
B

11.10.1. BORAX, NA2[B405(0H)J.8 H2O OR NA2B4O7.IOH2O


Borax or sodium tetraborate decahydrate is the most
re

FIGURE 11.9
important compound of boron. OH
In fact, it contains the tetranuclear units, i.e..
u

fB405(0H)4]-~ having two sp~ and two .9/?-^-hybridized boron


ad

8"
Yo

atoms as shown in Fig. 11.9. Therefore, its correct formula is O' O


Na2[B405(0H)4].8 H2O.
HO —b; o B —OH
Preparation (i) From tincai. Naturally occurring borax 0
0.
is called tincai or suhaga. Tincai obtained from dried up lakes
d
Re

contains about 50% borax. It is boiled with water and filtered IB


in

to remove insoluble impurities of clay, sand, etc. The filtrate


is concentrated when crystals of borax separate out. OH
F

(ii) From colemanite. The mineral colemanite,


(Ca2BgOu) is finely powdered and is boiled with sodium Structure of [B405(0H)4] ^ 1 ion

carbonate solution.

Heat
+ 2Na2C03 > Na2B407 +
2NaB02 + 2CaC03 i
Colemanite Borax Sod. metaborate Calcium carbonate

The precipitate of calcium carbonate thus formed is removed by filtration. The filtrate is concentrated
and cooled when crystals of borax .separate out. Sodium metaborate present in the mother liquor can be
converted into borax by passing a current of carbon dioxide throught it.
4NaB02 -I- CO2 Na2B40y +
Na2C03
Sod. metaboraie Sod. tetraborate Sod. carbonate
SOME p-BLOCK ELEMENTS 11/15

3. From boric acid. Borax can also be prepared in small amounts by neutralising boric acid with

w
sodium carbonate.

4 HjBOj + Na2C03 4 Na2B407 + 6 H2O + CO2


On cooling, crystals of borax, i.e., Na2B4O7.10H2O separate out.
Properties. (0 It is a white crystalline solid, less soluble in cold water but more soluble in hot water.
(//) The aqueous solution of borax is alkaline due to hydrolysis. Borax is, therefore, used as a water
softner and cleaning agent.
Na2B407 + 2 H2O ^ 2NaOH + H2B4O7

or
(Strong alkali) (Weak acid)

r
H2B4O7 + 5H2O ^ 4 H3BO3

F
Tetraboric acid Boric acid

(Hi) Action of heat—Borax bead test On heating, borax loses its water of crystallisation and swells up

oF
ul
to form a puffy mass. On further heating, it melts into a clear liquid which solidifies to a transparent glass like
bead which consists of sodium metaborate (NaBOf) and boric anhydride (B20f).
Heat

sr
Na2B4O7.10H2O > Na2B407 + IOH2O
Heat

ko
Na2B407 2NaB02 + B2O3
Sod. metaborate Boric anhydride

of
Transparent glassy bead
The glassy bead is commonly known as borax bead and is employed in qualitative analysis for the
detection of certain coloured basic radicals such as o
Ni^+, Co^'*’, Cr^"^, Cu^"^, Mn^'*', etc. Whenever a coloured
Y
salt containing these cations is heated with borax bead on a platinum wire, the salt decomposes to form the
rB
corresponding metal oxide which then combines with B2O3 present in the glassy bead to form coloured
metaborates. This test is called borax bead test. Thus,
eY

Heat Heat

C0SO4 > CoO + SO3 CoO + B2O3 Co(B02)2


Cobalt sulphate Cobalt oxide Cobalt metaborate (blue)
u

Heat Heat

NiO + B2O3 ■»
Ni(B02)2 Cr203 + 3 B2O3 ■»
2 Cr(B02)3
d
o

Chromium metaborate
ad

Nickel oxide Nickel metaborate Chromium


(brown) trioxide (green)
Heat
in

MnO + B2O3 ■>


Mn(B02)2 > CuO + ®2^3 Cu(B02)2
Manganese oxide Manganese metaborate Cupric oxide Cupric metaborate
(pink violet) (dark blue)
Re

Certain metaborates are reduced to the free metal by the carbon present in the reducing flame of the
F

burner. For example.


2 Cu (B02)2 C 4
2 CUBO2 + B2O3 + CO
Cuprous metaborate (colourless)
2 CUBO2 + C 2 Cu + B2O3 + CO
Metal (red)
(tv) Action of sodium hydroxide. On adding a calculated quantity of sodium hydroxide to borax,
sodium metaborate is formed.

Na2B407 + 2NaOH 4 4NaB02 + H2O


(v) Action of sulphuric acid. On adding a calculated quantity of concentrated sulphuric acid to a hot
concentrated solution of borax, boric acid is produced,
Na2B407 + H2SO4 Na2S04 + H2B4O7 H2B4O7 + 5 H2O ^ 4H3BO3.

A
11/16 New Course Chemistry fxnrosm

(vO Action of ethyl alcohol and sulphuric acid. On heating borax with ethyl alcohol and concentrated
sulphuric acid, vapours of iriethylborate are produced. When ignited these vapours bum with a green edged
flame.
Na2B40-7 + H2SO4 + 5 H2O Na2S04 + 4H3BO3
Conc.H2S04
H3BO3 + 3C2H5OH B(0C2H5>3 + 3 H2O
Ethyl alcohol Triethylborate
This reaction is used as a test for borate ion in qualitative analysis.
Uses of Borax. Borax is used (0 in the manufacture of enamels and glazes for earthen wares i.e., tiles,

w
pottery, etc. The glazed surface is resistant to heat, stains and scratches.
(/7) as a flux in soldering.
(7V) in the preparation of medicinal soaps due to its antiseptic properties

F lo
(/v) in the manufacture of heat resistant {i.e. very low coefficient of thermal expansion) borosilicate
glass (pyrex).
(v) to make peroxoborate [i.e., Na2[(0H)2B(0 - 0)2B(0H)2].6H20]- an important cleansing and
bleaching agent present in washing powders. They also act as optical brightners since they absorb UV light

ee
and emit visible light,

Fr
(w) as a stiffening agent for candle wicks
(vjV) to produce a good finish in laundary

for
(vm) in the laboratory for borax bead test.
{ix) in softening of water.
ur
11.10.2. Orthoborlc add (Boric add), H3BO3 or B(OH)3
Boric acid is the trivial name orthoboric acid.
oks
Preparation (i) From borax. Boric acid is obtained by treating a hot concentrated solution of borax
Yo
with hydrochloric acid or sulphuric acid. The resulting solution on concentration and cooling gives crystals
o

of boric acid.
eB

Na2B407 + 2 HCl + 5 H2O ■> 4 H3BO3 + 2 NaCl


Na2B407 + H2SO4 + 5 H2O 4 H3BO3 + Na2S04
(il) By hydrolysis of boron compounds. Boric acid can also be prepared by the hydrolysis of boron
our
ad

compounds such as halides, hydrides and nitrides.


BCI3 + 3 H2O ^ H3BO3 + 3HC1
B2H6 + 6H2O ^ 2 H3BO3 + 6 H2 BN + 3H2O >H3B03 + NH3
Y

Diborane Boron niuide

(iii) From colemanite. Boric acid is obtained by passing sulphur dioxide through the solution of the
Re
nd

mineral colemanite in boiling water. The resulting solution on concentration and cooling gives crystals of
boric acid while calcium bisulphite being highly soluble in water remains in the mother liquor.
Fi

Ca2B60| I + 11 H2O » 2 Ca(OH)2 + 6 H3BO3


2 Ca(OH)2 + 4 SO2 » 2 Ca(HS03)2
Ca2B60,j + 4SO2+ 11 H2O ^ 2 Ca(HS03)2 + 6 H3BO3
Colemanite Cal. bisulphite Boric acid

Properties. (/) It is a white crystalline solid with a soft soapy touch having a low density of 1-48 g cm“^.
(//) It is sparingly soluble in cold water but fairly soluble in hot water.
(77) Acidic nature. Boric acid behaves as a very weak (K,, = 5-6 x 10“**^) monobasic acid, it does not
act as a proton-donor, i.e., protonic acid. However, due to the small size of B and presence of only six
electrons in its valence shell, B(OH)3 behaves as a Lewis acid, by accepting a pair of electrons from OH “ ion
of water thereby releasing a proton.
SOME p-BLOCK ELEMENTS 11/17

H-^OH + B(0H)3 > [B(0H)4]- + ; /;K„ = 9-25


(/V) Action of heat Boric acid, on heating, loses water in three different stages at different temperatures
ultimately giving boron trioxide.
370 K
H3B03 HBO2 + H2O
Boric acid Metaboric acid

410K Red heat


4HBO2 > H2B40y 2B2O3 + H2O
-H2O

w
Metaboric acid Tetraboric acid Boron trioxide

(v) Reaction with ethyl alcohol. Orthoboric acid reacts with ethyl alcohol in presence of cone. H2SO4
to form triethylborate.

F lo
Conc.H,S04
B(0H)3 + 3C2H5OH B(0C2H5)3 + 3H2O
Orthoboric acid Ethyl alcohol Triethylborate
The vapours of triethylborate when ignited bum with a green-edged flame. This forms the basis for

ee
detecting borates and boric acid in qualitative analysis.

Fr
Uses. Boric acid is used : (0 in the manufacture of heat resistant borosilicate glass.
(I’O as a preservative for milk FIGURE 11.10

for
and food stufft. 2s 2Px 2py 2p^
(i/i) in the manufacture of
(GROUND fi t
ur
enamels and glazes in pottery, A
STATE) LIZ. /
\
\
(/v) The aqueous solution of sp2-HYBRIDIZATION
/ \
s
N
boric acid is used as a mild /
ook
B I
Yo
T
antiseptic especially as eye wash (EXCITED I I
under the name boric lotion. STATE) I Li t t I
+
eB

Structure of boric acid. The


ground state outer electronic FORMS THREE B - O .a- BONDS
configuration of boron is 2s^ 2/?J.. 3- ●
In the excited state, one of the ’ : vTrig^naljjpIahM sttucture ion
our
ad

25-electrons gets promoted to the


vacant 2p^-orbital. The three half-filled atomic orbitals i:-'.
FIGURE 11:11 I
(25, 2p^ and 2p^ thus obtained undergo 5p^-hybridization
o-"'"
Y

to give three 5/7^-hybridized orbitals. Each one of these three


Re

5/7^-orbitals overlaps with 2/7-orbitals of 0“ forming three I


nd

B - O" bonds.

Therefore, BO^“ (borate) ion has trigonal planar


Fi

H
structure as shown in Fig. 11.10.

In boric acid, planar BO^" units are joined by


I
I H
'O
unsymmetrical hydrogen bonds to give a layered structure I
as shown in Fig. 11.11. The adjacent layers in the crystal of -H
boric acid are held together by weak forces of attraction.
The distance between any two successive layers is 318 pm. I I
H H
Because of weak forces of attraction, one layer can slide
over the other. This makes boric acid soft and soapy to touch. S^ct^pfih^ (dotted lines
It is because of this reason that boric acid is used in carom
boards for smooth gliding of pawns.
11/18 New Course Chemistry (XI)S5EIHD

11.10.3. Boron Hydrides


The binary compounds of boron with hydrogen are called boron hydrides. Actually boron does not
combine directly with hydrogen but a number of hydrides of boron are known. These hydrides of boron are
collectively called boranes on analogy with alkanes. These hydrides can be divided into several series of
which the following two are very important:
(/) (called boranes)suchasB2Hg(diborane), B5H9 (pentaborane-9). B^fi^oibexaborane-
10), BgH|2 {octaborane-12), BjqHj4 {decaborane), etc.
(//■) (called arac/tno-boranes) such as (tetraborane), B^Hjj {penlaborane-11), B^H,2
(Jiexaborane-12), BgHj^ {octaborane-14), ByHjj {nonaborane or enneaborane), etc.

w
It may be noted that wherever two or more boranes with the same number of B-atoms but different
number of H-atoms are known (for example, pentaboranes, hexaboranes, octaboranes, etc.), an integer depicting
the total number of hydrogen atoms in the molecule is included in the name so as to distinguish them from one
another.

F lo
Preparation of diborane. It may be prepared by the following methods:
(i) By reduction of boron trifluoride etherate with lithium aluminium hydride {LiAlHf) in diethyl ether.
Diethyl

ee
4 BF3 . Et20 + 3 UAIH4 > 2 B2H6 + 3 LiF + 3 AIF3 + 4 EI2O
ether

Fr
Boron trifluoride etherate Diborane

(//) Laboratory method of preparation. Diborane is prepared in the laboratory by the oxidation of
sodium borohydride (NaBH4) with iodine in diglyme* as solvent

for
Diglymc
2NaBH4 + I2 ^ B2H6 + 2NaI + H2
ur
(Hi) Industrial preparation. On the industrial scale, diborane is obtained by the reduction of BF3 with
sodium hydride.
450 K
s
2BF3 + 6NaH B2H5 + 6NaF
ook
Yo
Preparation of higher boranes. Higher boranes (B4H,q, BjHh, B^Hj2 and BjqH|4) are obtained
when B2H^ is heated at 373-523 K.
eB

Properties. (0 Physical state. Diborane is a colourless, highly toxic gas, b.p. 180 K.
(«) Stability. Diborane is stable only at low temperatures. However, when diborane is heated in a sealed
lube, between 373-523K, a complex reaction occurs and various higher boranes are formed.
r

373-523K _
ad
ou

B2H6 Sealed tube


> B4H10, C5HJJ, BgH,2, etc.
By careful control of temperature, pressure and reaction time, different individual boranes can be obtained.
For example.
Y

353-363K
2B2H6 BaH.o + H2
200 atm, 5/ir
Re

Diborane Tetraborane
nd

(Hi) Combustibility. It catches fire spontaneously upon exposure to air. It burns in oxygen evolving an
enormous amount of heat.
Fi

B2Hg + 3 O2 4 B2O3 + 3 H2O ; H° = - 1976 U moF'


Like diborane, higher boranes also spontaneously bum in air.
(/V) Hydrolysis. Boranes are readily hydroly.sed by water to form boric acid.
B2H6 (g) + 6 H2O (/) >2 H3BO3 (aq) + 6 H2 (g)
With methanol, trimethylborate is formed
+ 6 CH3OH > 2 B(OCH3)3 + 6 H2
(v) Reaction with Lewis bases—Cleavage reaction. Diborane on treatment with Lewis bases first
undergoes cleavage to form borane which then reacts with Lewis bases to form adducts.

*Diethylene glycol dimethyl ether, i.e., CH3OCH2CH2OCH2CH2OCH3 is called diglyme.


SOME p-BLOCK ELEMENTS 11/19

B2Hg + 2 NMcg > 2 BH3 . NMc3 B2H6 + 2 CO > 2 BH3 . CO


(v/) Reaction with ammonia. Diborane combines with ammonia to form an addition product,
B2H5.2NH3 formulated as [BH2(NH3)2]'*’ [BH4]~ which when heated to 473 K decomposes to give a volatile
compound called borazine (or borazole).
Low temp. 473 K
3 B2H6 + 6 NH3 > 3 [BH2(NH3)2f [BHJ- . 2B3N3HJ+ 12 Hj

ow
Diborane Borazine

Borazine is isoelectronic {Le. same number of electrons) and isosteric (i.e. same number of atoms) with
benzene and its structure is similar to that of benzene except that in benzene the jc-electrons are completely
delocalized but in borazine, they are only partially delocalized
H

e
:C H
'^C

re
I

rFl
H /'B C H

F
I I
H H H
Borazine Borazine Benzene

Because of its similarity with benzene, borazine is also called inorganic benzene.

r
ou
(v«) Formation of complex borohydrides or tetrahydridoborates. Several metal hydrides react with

fo
diborane to form tetrahydridoborates commonly known as borohydrides. All these contain the tetrahedral
[BH4] “ ion. For example.
2NaH + B2H6
Diethyl
» 2 Na+ [BH4]- ; ks
2LiH + B2H6
Diethyl
» 2Li+[BH4]-
oo
ether ether
Sod. hydride Sod. borohydride Lithium hydride Lithium borohydride
Both sodium borohydride and lithium borohydride are used as reducing agents in organic synthesis.
Y
eB

They also serve as starting material for many other borohydrides.


(yiii) Reaction with alkalies. Diborane dissolves in strong alkalies (NaOH or KOH) to produce
metaborates and dihydrogen gas.
r

B2H6 + 2 KOH + 2H2O ■»


2 KBO2 + 6 H2
ou

Diborane Pot. metaborate


Y
ad

(ix) Action of halogen acids. Diborane reacts with halogen acids to give halodiboranes evolving H2
gas. The reactivity of halogen acids follows the order: HI > HBr > HCl. Thus, HI reacts at about 323 K in
absence of a catalyst while HBr and HCl react in the presence of their respective aluminium halides as catalyst.
d

323 K
AlBrg
B2H6 + HI B2H^I + H2 \ B2Hg + HBr B2HsBr + H2
in
Re

Diborane lododiborane Diborane Bromodiborane

AICI3
F

B2H6 + HCl B2H5CI + H2


Diborane Chlorodiborane

(x) Action of halogens. Like halogen acids, halogens also react with diborane to form the corresponding
halodiboranes. The reactivity, however, decreases in the order : CI2 > Br2 > l2-
Thus, CI2 reacts explosively at room temperature, Br2 reacts rapidly at 373 K while I2 reacts slowly at
still higher temperatures.
298 K
B2H6 + CI2 B2H5CI + HCl
Explosive
Diborane Chlorodiborane

Uses. (0 Diborane is used for preparing a number of borohydrides such as LiBH4, NaBH4, etc.
(//) It is used as a reducing agent in organic reactions. (Hi) It is used as a fiiel for supersonic rockets.
11/20 ‘P^tuUefo.'4- New Course Chemistry (X1)ESS[9S

Structure of diborane (B2Hg). The structure of diborane is very interesting. In diborane, each boron
atom has three valence electrons for sharing. If we assume that each boron forms three covalent bonds with
three hydrogen atoms, then there are no electrons left with boron atom for sharing with other boron atom.
Therefore, the two boron atoms cannot be linked. This suggests that unlike ethane which has a C-C bond,
diborane does not have a B-B bond. In other words, the structure of diborane is not similar to tliat of ethane
as shown below :
H H H H

h:B ? B:H h:C : C:h

H H H H
No electrons for B-B bond fomialion

w
X-Diffraction studies have shown that the structure of diborane is as shown in Fig. 11.12.
In this structure, there are two types of hydrogen atoms. FIGURE 11.12

F lo
The four hydrogen atoms (two on the left and two on the right
shown by thick lines) are called terminal hydrogens. These four H H
terminal hydrogens and the two boron atoms lie in the same ✓

s
N

plane while the remaining two hydrogen atoms - one lying above
y
s
97°

ee
B B
and one below this plane form bridges and hence are called 2^ y

bridge hydrogens. TTius, there are two types of bonds in B2Hg

Fr
s y

molecules. H H
(/) The four terminal B~H bonds are normal covalent bonds

for
and hence are quite strong. Each bond is formed by sharing a ♦
178 pm
pair of electrons between B and H atoms and hence are also
ur
called two centre electron pair bonds or two centre two- Structure of a diborane molecule
electron bond (2c'2e).
s
(«) The two bridge bonds i.e. B H B are quite different from normal covalent bonds. Each bridge
ook
hydrogen is bonded to two boron atoms by a pair of electrons. In other words, these are three-centre electron
Yo
pair bonds or three centre two electron bonds (3c-2e) and hence are quite weak. Because of their
resemblence to a banana, these three centre electron pair bonds are also called banana bonds.
eB

Explanation of structure of diborane on the basis of hybridisation. The electronic configuration of


boron atom in the excited state is Is^ 2s* 2p^\ 2py\ 2pfi. It undergoes sp^-hybridisation. The two half-filled
hybrid orbitals of each boron atom overlap with the half filled orbitals of hydrogen atoms forming normal
our
ad

covalent bonds whereas the third half-filled hybrid orbital of one boron atom and the vacant hybrid orbital of
the second boron atom (shown by dotted lines) overlap simultaneously with the half-filled orbital of H-atom
(Fig. 11.13). Thus, the electron cloud contains only two electrons but spreads over three atoms (two B atoms
and one H-atom). That is why this bond is called three centre electron pair bond. Because of the shape of
Y

the electron cloud fonned, it is also called banana bond.


Re
nd

FIGURE 11.13

2c - 2e BOND
Fi

2s
sB
ti
H
(GROUND STATE)
B

H
sB
(EXCITED STATE)
t t t
3c-2eBOND
.?;j^-HYBRIDISATION
Structure of diborane involving sp^-hybridisation
of boron
SOME p-BLOCK ELEMENTS 11/21

Molecules like which do not have sufficient number of electrons to form normal covalent bonds
(two centre-electron pair bonds) are called electron deficient molecules.
The structures of boranes were studied in detail by William Lipsocomb who was awarded the 1976
Nobel Prize in Chemistry.
11.10.4. Alums.'

Alums are the double salts of the formula, MiS04.M”^(S0^)j.24H20 where represents a
monovalent cation such as Na"*", K'^, Rb^, NH^, etc. while represents a trivaient cation such as
TP'*', In^'*'. Rh^"^, and Ga^‘*‘. Each molecule of a specific alum contains 24

w
molecules of water of crystallization. Some important alums are :
1. Potash alum, K2S04.AIt(S04)3.24 H->0

F lo
2. Chrome alum, K2S04.Cr2(S04>3.24 H2O
3. Ferric alum, (NH4)2S04-Fe2(S04)3.24 H2O
4. Soda alum, Na2S04.Al2(S04)3.24 H2O

ee
5. Ammonium alum, (NH>2SO4.Al2(S04)3.24 H2O

Fr
Potash alum is prepared in the laboratory by mixing hot solutions of equimolar quantities of K2SO4 and
Al2(S04)3. The solution on concentration and crystallization gives potash alum.

for
Properties. The aqueous solution of alums is acidic due to hydrolysis of A1-)(S04)3, Cr,(S04)3 or
Fe2(S04)3.
ur
Al2(S04)3 + 6 H2O ^ 2 A1(0H)3 -I- 3 H2SO4
On heating, all alums lose water of crystallization and swell up. The anhydrous alum is called burnt
s
alum.
ook
Yo
Uses. Alums are used (0 in purification of water, (/7) for sizing of paper, {Hi) as a syptic to stop bleeding
(/v) as a mordant in dyeing and tanning of leather, (v) in calico printing.
eB

11.11. USES OF BORON, ALUMINIUM AND THEIR COMPOUNDS


Uses of borou' Boron is an extremely hard refractory solid with high melting point. It has low density
our

and low electrical conductivity. Therefore, it finds many applications. Some of these applications are :
ad

(/) Boron fibres have enormous tensile strength and hence are used to make bullet-proof vests and light
composite material for aircrafts,
Y

(/i) Natural boron is 20% boron-10 (^^B) and 80% boron-11 (g* B). Boron-10 has a high cross section
for absorption of low energy (thermal) neutrons. It is because of this special property of B-10 to absorb
Re
nd

neutrons that metal borides are used as protective shields and control rods in atomic reactors. Recently the
possible use of B-10 compounds in cancer chemotherapy is also being investigated.
Fi

{Hi) Boron compounds such as borax and boric acid are used in the manufacture of heat resistant gla.ss
{i.e. pyrex), glass-wool and fibreglass. Borax is also used as a flux in soldering, as a constituent of medicinal
soaps due to its antiseptic properties, in the manufacture of enamels and glazes for earthenwares,i.e., tiles,
pottery, etc. The glazed surface is resistant to heat, stains and scratches. An aqueous solution of onhoboric
acid is also used as a mild antiseptic,
(/v) It is used in steel industry for increasing the hardness of steel. In fact, boron has replaced expensive
metals like Mo, Cr and W in the manufacture of special hard steels,
(v) Boron compounds are being used as rocket fuels because of their high energy/mass ratio.
(W) Boron is also an essential element in plant metabolism.
♦Included only in the syllabus of ISC.
11/22 ‘Pna^deefr New Course Chemistry (XI)

Uses of aluminium. Aluminium is a bright silvery white metal with high tensile strength. Some of its
important applications are listed below :
(/) Being light and good conductor of electricity (on weight to weight basis A1 conducts twice as Cu)
aluminium is used for making transmission cables and for winding the moving oils of dynamos or motors.
(//) It is used for making aluminium paints for protection of iron and zinc. For example, aluminium
powder mixed with linseed oil shines like silver and hence is called silver paint.
(Hi) Aluminium is a cheap metal which resists corrosion. Therefore, it is used for making household

w
utensils, cans for drinks, tubes for toothpaste, picture frames, trays, etc. It is also used in buildings for making
angles for doors, windows, etc.
However, the use of aluminium and its compounds for domestic purposes is now reduced considerably

Flo
due to its toxic nature.

(iv) Aluminium foil is used for wrapping fine articles like photographic films, pharmaceutical products,

e
cigarettes, sweets, etc.

re
(v) Aluminium powder is used as a reducing agent in aluminothermic process for extraction of chromium
and manganese from their ores,

F
(vz) Aluminium powder is used in flash light bulbs for indoor photography,
ur
(vzz) Large amounts of aluininium are converted into alloys. Some important alloys of aluminium are

r
given below ;

fo
Alloy Composition Important properties Uses

(1) Aluminium A1 95%


ks
Light, strong alloy with golden lustre, Coins, utensils, jewellery,
Yo
bronze Cu 5% resistant to corrosion picture frames etc.
oo
(2) Magnalium A1 95% Light, lough and strong Light instruments,
Mg 5% balance beams, pressure
B

cookers etc.

(3) Duralumin A1 95% Light, tough, ductile, resistant to Making aeroplanes,


re

Cu 4% corrosive action. automobile parts, pressure


Mg 0-5% cookers etc.
u

Mn 0-5%
ad
Yo

(viii) A1(0H)3 is widely used as an antacid for treatment of indigestion.


(ix) Anhydrous AICI3 is used as a catalyst in Friedel-Crafts reactions and in cracking of petroleum.
Hydrated AICI3 is used as a mordant in dyeing,
d
Re

(x) Potash alum, K2S04.Al2(S04)3.24H20 is used for purification of water, as styptic for stopping
in

bleeding, in foam type fire extinguishers, as mordant for dyeing and for tanning of leather, in calicoprinting
and sizing of paper.
F

PART—II. GROUP trt ELEMENTS : CARBON FAMILY

11.12. INTRODUCTION

Group 14 includes carbon (C), silicon (Si), germanium (Ge), tin (Sn), lead (Pb) and the recently
discovered element, ununquadium (uuq) which is radioactive. As in the previous groups, there is a discontinuity
in general properties between the first and second row elements followed by a gradual transition towards
more metallic character. Carbon is strictly non-metallic, silicon is essentially a non-metal in its chemical
behaviour, germanium is a semi-metal (metalloid) with pronounced metallic character while tin, lead are
metallic in character.
SOME p-BLOCK ELEMENTS 11/23

11,13. OCCURRENCE

Carbon is the seventeenth most abundant element by weight in the earth’s crust. It occurs in the native
state in form of coal, graphite and diamond. In combined state, it occurs widely as metal carbonates, hydro
carbons (petroleum), carbohydrates and CO2 (0-03%) in air. It is the essential constituent of all living
organisms.

Naturally occurring carbon contains two stable isotopes : '^Cand^^C.In addition to these, third
isotope, ’g C is also present. It is a radioactive isotope with a half life of 5570 years and is used for radiocarbon
dating.
Silicon is the second (27-7%) most abundant element (next only to oxygen) by weight in the earth’s

w
crust. It widely occurs in form of silica, i.e., Si02 (sand and quartz) and in a wide variety of silicates and
clays. Germanium occurs in traces (1-5 ppm) and is mainly recovered from flue dusts arising from the roasting
of zinc ores. The ultrapure form of germanium and silicon are used to make transistors and semiconductor

F lo
devices.

Germanium is transparent to infrared light and is, therefore, used for making prisms, lenses and windows
in infrared spectrometers.
The natural abundances of tin and lead are 2 ppm and 13 ppm respectively. Tin occurs mainly as tinstone

ee
or cassiterite (Sn02). The principal ore of lead is galena (PbS) which is often associated with zinc blende

Fr
(ZnS). Other ores of lead are anglesite (PbS04) and cerussite (PbC03). Small quantities of lead ores occur
in Rajasthan.

for
11.14. ELECTRONIC CONFIGURATION
r
The electronic configuration of group 14 elements are given in Table 11.5.
You
TABLE 11.5. Electronic configuration of elements of group 14
oks
Element Atomic Electronic Configuration
o

Number
Complete
eB

With Inert core

Carbon (C) 6
1^^2.v2 2p‘2p^ [He] 2.2 2/7* 2p>
Silicon (Si) \s~ Is^lp^ 3/7^3pJ, [Ne] 3s^3p\3p^y
our

14
ad

Germanium (Ge) 32 1j2 25^ 3^2 3/ 3^10 [Ar]3i/*0 4.2 4p\4p^y


4s~
dY

Is^ Is^ Ip^ 3/ 3(/io [Kt]4£/*0 5.2 5/7^5/7j,


Re

Tin (Sn) 50

4^,6 4^10 5^2 5p^^5py


Fin

Lead (Pb) 82 \s^ 2j2 2/ 3s2 3p^ [Xe] 4/ *4 6.2 6p* 6pj,
4s^4p^ 4(i'0 4/l4 5_j2 5p6
5f/“^ 65^ 6/7^
Ununquadium 114 \s^ Is^ 2p^ 3p^ 3d^^ 4s^ [Rn] 5/*^ 6t/**^ 7.2 7/7*X 7/7*y
4p6 4^10 4y 14 5^2 5^6 5jl0
5f'‘^6s^6p^6d'^ls^ Ip^lp^X y

The general valence shell electronic configuration of elements of group 14 is n.2 wfjgre n is the
number of the outermost principal shell.
11/24 '4. New Course Chemistry (XI) twii

11.15. ATOMIC AND PHYSICAL PROPERTIES


Some important atomic and physical properties of elements of group 14 are given in Table 11.6.
TABLE 11.6. Some atomic and physical properties of group 14 elements

Elements
Property
C Si Ge Sn Pb

Atomic number 6 14 32 50 82

Atomic mass (g mol"’) 12-01 28-09 72-60 118-71 207-2

w
Covalent radius/pm for M (IV) state 77 118 122 140 146

Ionic radius M^/pm (6 co-ordination) 40 53 69 78

Ionic radius M^'''/pm 73 118 119

F lo
Ionisation enthalpy/kJ mol"’ A,- Hj 1086 786 761 708 715
2352 1577 1537 1411 1450
A,H2
3228 3300 2942 3081
A,H3 4620

ee
6220 4354 4409 3929 4082
A,H4

Fr
2-5 1-8 1-8 1-8 1-9
Electronegativity (Pauling scale)
Density/g cm“^ at 293 K 3-51 2-34 5-32 7-26 11-34

(for diamond)

for
2-22
ur
(for graphite)
4373 1693 1218 505 600
Melting point/K
s
Boiling point/K 3550 3123 2896 2024
ook
Yo
Electrical resistivity/ohm cm at 293 K 10l4_ 10'® 50 50 10-5 2 X 10-5

Some of the important physical properties of elements of group 14 are discussed below :
eB

1. Covalent radii. The covalent radii of group 14 elements are smaller than those of the corresponding
elements of group 13.
Explanation. This is because when we move from group 13 to group 14 within the same period, the
r
ad

effective nuclear charge increases and hence the covalent radius decreases due to stronger attractive influence
ou

of the nucleus on the outer electrons.


Further, the covalent radii of group 14 elements regularly increase as we move down the group.
Y

Explanation. It is primarily due to addition of a new energy shell in each succeeding element. The
increase in covalent radii from Si onwards is, however, small due to ineffective shielding of the valence
Re
nd

electrons by the intervening d-andf- orbitals.


2. Ionisation enthalpy. The first ionization enthalpies of group 14 elements are higher than those of
Fi

the corresponding elements of group 13.


Explanation. This is because of greater nuclear charge and smaller size of the atoms of group 14 elements.
The first ionization enthalpy decreases steadily on moving down the group from carbon to tin. The
decrea.se is very sharp from carbon to silicon while there is a slight increase in the first ionization enthalpy
of lead as compared to that of tin.
The first ionization enthalpies (Aj of the elements of group 14 follow the order :
C > Si > Ge > Sn < Pb

Explanation. The decrease in ionization enthalpy down the group from C to Sn is due to increase in
atomic size and screening effect of the inner electrons which outweigh the effect of increased nuclear change.
However, the small increase in ionization enthalpy from Sn to Pb is due to a considerable increase in nuclear
SOME p-BLOCK ELEMENTS 11/25

chai'ge (82 - 50 = 32 units) which outweighs the shielding effect of all the electrons in the inner shells
including those of 4/- and 5fif-electrons.
3. Electronegativity. The elements of group 14 are more electronegative than group 13 elements because
of smaller size. Electronegativity, however, decreases down from C to Si and remains constant from Si to Sn
and then slightly increases for Pb.
4. Physical properties, (a) Metallic character. All the elements of group 14 are solids. They are less
electropositive and hence less metallic than the group 13 elements because of smaller atomic size and higher
ionization enthalpy. On moving down the group, metallic character, however, increases. For example, carbon

ow
is strictly non-metallic, silicon is essentially a non-metal, germanium is a semi-metal (metalloid) whereas tin
and lead are soft metals with low melting points.
(b) Melting and boiling points. The melting and boiling points of group 14 elements are much higher
than those of the corresponding elements of group 13. This is due to the reason the atoms of group 14 form
four covalent bonds with each other and hence there exist strong binding forces between their atoms both in
the solid as well as in the liquid states. Further, the melting points and boiling points decrease as we move

e
Fl
down the group due to a corresponding decrease in the interatomic forces of attraction. However, the melting

re
point of tin is lower than that of lead.

F
11.16. OXIDATIONSTATES AND TRENDS IN CHEMICALREACTIVITY
1. Oxidation states, {a) Oxidation states of carbon. The general valence shell electronic configuration
ur
of elements of group 14 is ns^ np^ where n is the number of outermost principal shell. Thus these elements

or
can attain inert gas configuration either by losing or gaining four electrons forming or ions.
The ion, however, does not exist ifrstly because it is a highly charged species and secondly its

chemical reactions.
k sf
formation requires a very high ionization enthalpy (14-26 x 10^ kJ mol“') which is usually not available in
Yo
On the other hand, carbon can take up four electrons to form carbide ion, ion. Such a process is also
oo
energetically not favourable since the chemical species is highly charged and thus requires large amount of
energy for adding four electrons. However, carbon forms some carbides such as Be^C , CaC->, SiC, AI4C3 in
eB

which carbon is supposed to be present either as or ions.


(b) Oxidation states of other elements. Like carbon, silicon also shows an oxidation state of +4. The
remaining elements of this group, i.e., Ge, Sn and Pb, however, show two oxidation states of-i-2 and +4 due
ur

to inert pair effect which arises due to ineffective shielding of the valence .v-electrons by the intervening d-
ad

and or/-electrons. Evidently, as the number of d- and or/-electrons increases down the group from Ge to Pb,
Yo

the inert pair effect becomes more and more prominent. As a result, the stability of the +4 oxidation state
decreases while that of the +2 oxidation state increases from Ge to Pb. In other words, the stability of the +2
oxidation state increases markedly in the sequence : Ge < Sn < Pb, i.e., +2 oxidation stale ofPb is the most
d

stable.
Re
in

Further, in the tetravalent state, the number of electrons around the central atom in a molecule (e.g., C in
CCI4) is eight. Being electron-precise molecules, they neither act as electron-acceptors nor electron- donors.
F

2. Trend down the group from covalent to ionic character. Compounds of group 14 elements which
show an oxidation slate of +4 are expected to be covalent because of their extremely .small size and high
charge whereas compounds which show an oxidation stale of +2 are expected to be ionic because of large
size and small charge. For example, SnCl2 is an ionic solid while SnCl4 is a covalent liquid. Further, as we
move down the group, the tendency of the elements to form covalent compounds decreases whereas the
tendency to form ionic compounds increases.
3. TVends in oxidising and reducing properties. Since due to inert pair effect, the elements Ge, Sn
and Pb show two oxidation states of +2 and +4, therefore, these elements in +2 oxidation state can act as
reducing agents while in +4 oxidation states they can act as oxidising agents.
2+
Reducing agent: M + 2 e~

Oxidising agent: + 2 e -
11/26 'P’tauteep^'^ New Course Chemistry (XI)EEIHD

Since +4 oxidation stale of Ge is the most stable followed by Sn and Pb, therefore, in group 14, Ge (II)
salts are the strongest reducing agents followed by Sn (II) salts. But germanium is much less abundant in
nature than tin, therefore. Sn (ll) salts such as SnCl^, is widely used as a reducing agent.
2 FeCl3 + SnCl2 > 2 FeCl, + SnC^ 2 HgClj + SnCl2 ^ Hg2Cl2 + SnCl4
Hg2Cl2 + SnCl2 ^ 2 Hg + SnCl^

w
Further since +2 oxidation slate of lead is the most stable followed by tin and germanium, therefore, Pb (IV)
salts such as lead tetraacetate, Pb{OCOCH^^ and are widely used as oxidising agents. For example,
CH-OH Heat
I ^ + Pb(OCOCH3)4 2 CH2 = O + Pb(OCOCH3)2 + 2 CH3COOH
CH^OH Lead tetraacetate Formaldehyde Lead acetate Acetic acid

o
Etliylene glycol

e
Heat
PbO-) + 4 HCl (cone.) ■> PbCI, +CU +2H,0

re
Lead dioxide

rFl
4. Tendency to form pn -pn bonds. Due to small size and high electronegativity, carbon has a strong

F
tendency to form pK - pn-multiple bonds either with itself (C = C, C = C) or with other atoms of similar size
such as oxygen (C = O) and nitrogen (C = N, C =N).
However, as we move down the group from carbon to lead, this ability to form pn-pn multiple bonds

r
decreases drastically due to a corresponding increase in size and decrease in electronegativity of the atom.
ou
fo
The reluctance of silicon to form such pn - pn bonds to itself is shown by the following facts :
(i) Elemental silicon exists only in the diamond structure and not in the graphite structure and no form
of elemental silicon is comparable to graphite,
(ii) CO2 containing two C = O double bonds is a gas while ks
is a solid. It consists of an infinite
oo
three- dimensional network of Si-0 single bonds (Fig. II.19, page 11/40).
5. Tendency to form dn - /?n-bonds. Carbon does not have d- orbitals and hence it does not form
Y
dn-pK bonds. However, silicon and other heavier elements of this group because of the presence of vacant d-
B

orbitals in them tend to form dn-pn bonds. This tendency is particularly strong in case of silicon linked to
oxygen and nitrogen. For example, in irimethylamine, N{CH3)3, both C and N are 5p^-hybridized. Further,
re

since nitrogen contains a lone pair of electrons but carbon does not, therefore, geometry around nitrogen is
pyramidal while that around carbon is tetrahedral. However, in similar silicon compound, i.e., trisilylamine,
ou

N(SiH3)3, N is 5p^-hybridized. The reason being that a 5/7^-hybridized nitrogen has a lone pair of electrons in
Y
ad

a 2/7-orbital which can overlap with an empty i/-orbital of silicon to form dn-pn bonds (Fig. 11.14). This
additional bonding is responsible for change of hybridization of nitrogen from sp^ to sp^. As a result, N(SiH3)3
has trigonal planar geometry.
d

FIGURE 11.14
in
Re

H3Siwt<
\ '● /
\
s
F

N /

s
N
Si + N s ' dn-pn
s

H3C< >CH3 /
BONDS
\

CH3 SiHs
EMPTY FILLED dn-pn-BOND (PYRAMIDAL) (PLANAR)
d-ORBITAL 2P-ORBITAL
OF SI OF N
N = 1 s2 2s2 2pi 2p} 2p^z N = 1 s2 2s1 2pl 2pj, 2p|
1 > I ——1

sp3 HYBRIDISATION sp2 HYBRIDISATION


^/Tt-pTt'Back bonding between empty d-orbital of Si and /»-orbitaI of N containing the
lone pair of electrons
SOME p-BLOCK ELEMENTS 11/27

Further due to dn ~ pn bonding, the lone pair of electrons is transferred from nitrogen to silicon. As a
result, N is a weaker base than
6. Maximum covalency and tendency to form complexes. Carbon because of the absence of
t/-orbiials, cannot expand its valence shell and hence its maximum covalency or coordination number is four.
However, Si, Ge, Sn and Pb due to the availability of vacant r/-orbitals show a coordination of greater than 4
{i.e. 5 and 6) forming pentacoordinated and liexacoordinated complexes. For example,
[SiFg]- , [SiFj2-, [GeCl^l^- . [Sn(OH)6]2-, [Pb(OH)j2- and IPbClg]^-. etc.
In these complexes, the hybridization of central atom is sp^d~.

w
Further, due to the presence of i/-orbitals, the tetrahalides of Si, Ge, Sn and Pb undergo hydrolysis.
11.16.1. Reactivity towards Oxygen
All the elements of group 14 when healed in oxygen form oxides.

Flo
These are mainly of two types, i.e., monoxides of the formula MO and CO2 Acidic q:
dioxides of the formula MO2. SiO exists only at high temperature where Si02 Less acidic lU

e
it is thought to have been formed by reduction of Si02 with Si, i.e.,
o w
OJ
Ge02 Lesser acidic 2 CO

re
Si02 + Si 2 SiO.
S CK
Besides monoxide and dioxide, lead also forms another oxide called Sn02 O
O

F
O lu
● Amphoteric
trilead tetroxide or red lead or Sindhur (Pb304>. It is obtained by heating Pb02
5 Q
o
PbO (litharge) with excess of air or 0-> at 673 K
ur <

or
673 K
6 PbO + O2 ^ 2Pb304
Properties, (i) Acid-base characte.r The oxides in higher oxidation state of the element are generally

sf
more acidic than those in the lower oxidation stale. Further, as we move down the group, the acidic character
decreases. Thus, among dioxides, CO2 and Si02 are acidic, Ge02 is less acidic than Si02 whereas Sn02 and
k
Yo
Pb02 are amphoteric.
oo
Thus, being acidic CO2, Si02 and Ge02 react only with bases.
CO2 + 2 NaOH > Na2C03 + H2O : CO2 + Ca(0H>2 > CaCO^, + H2O
B

Cal. carbonate

Si02 + 2 NaOH Na2Si03 + H2O i Ge02 + 2Na0H > Na2Ge03 + H2O


re

Sod. silicate Sod. gemianate


In contrast, Sn02 and PbO^ react with both acids and bases.
u
ad

SnOo + 2 NaOH Na2Sn03 + H2O ; SnO, + 4 HCl SnCl^ + 2 H2O


Yo

Sod. stannate
273 K
Pb02 + 2 NaOH Na->Pb03 + H2O ; Pb02 + 4 HCl PbCl4 + 2 H2O
Sod. plumbate
d

Among monoxides, CO is neutral, GeO is distinctly acidic whereas SnO and PbO are amphoteric,
Re
in

(ii) Reducing-oxidising powe.r Since + 4 state of carbon is the most stable, therefore, among the monoxides
of group 14, CO is the strongest reducing agent. Therefore, it is used in the extraction of many metals from
F

their oxides.

A A
Fc203 + 3 CO ^ 2Fe + 3CO, ZnO + CO 4 Zn + CO2
Since due to inert pair effect, +2 oxidation state of Pb is the most stable, therefore, among dioxides of
group 14, Pb02 is a powerful oxidising agent. As such it oxidises HCl to Cl^ and reacts with cone. HNO3 or
H2SO4 to evolve O2 gas.
2 Pb02 + 4 HNO3 2 Pb(N03)2 + 2 H.O + O2
2 Pb02 + 2 H2SO4 2 PbS04 + 2 H2O + O2

Pb02+4HC1 PbCU + CI2 + H2O


11/28 “Pn<uiee^'4. New Course Chemistry (XI)CSISD

11.16.2. Reactivity towards Water


Carbon, silicon and germanium do not decompose water at all. Tin decomposes steam to form tin
dioxide and dihydrogen gas.

Sn (j) + 2 H2O (g) ■> Sn02 (^) + 2 H2 (g)


Lead is not affected by water probably due to the formation of a protective film of lead oxide on its suiface.
11.16.3. Reactivity towards Halogens

ow
The elements of group 14 form halides of the formula MX4 and MX2 (X = F, Cl, Br, I). Except carbon
all other elements react directly with halogen under suitable conditions,
(a) Tetrahalides. (0 All the elements of group 14 form letrahalides of the formula MX4.
(//) Most of the tetrahalides are covalent in nature. The central atom in these halides undergoes sp^~
hybridization and the molecule is tetrahedral in shape. Exceptions are SnF^ and PbF^ which are ionic in
nature.

e
Fl
(in) The thermal stability of these halides decreases with the increasing atomic number or the size of the

re
halogen atom. Thus, PbCl4 is stable, PbBr4 is unstable while Pbl4 is unknown. The non-existence ofPbl^ is

F
probably due to the strong oxidising power ofPh (+ IV) and strong reducing power of I Similarly, instability
of PbBr4 may be due to the strong oxidising power of Pb (+ IV) and weak reducing power of
Br “ ion.
ur
or
The non-existence of Pbl4 may also be explained as follows :
The Pb-1 bond initially formed during the reaction does not release enough energy to unpair 65^ electrons

sf
and excite one of them to 6 p-orbital to have four unpaired electrons around lead atom.
(iv) The tetrachloride of carbon (CCI4) is not hydrolysed by water. However, the tetrachlorides of all the
k
Yo
remaining elements are easily hydrolysed.
oo
SiCl4 + 4H20 ^ Si(OH)4 + 4 HCl SnCl4 + 2 H2O ^ Sn02 + 4 HCl
Explanation. CCI4 is not hydrolysed by water because carbon does not have J-orbitals and hence
eB

cannot expand its coordination number beyond 4. However, silicon can expand its octet (coordination number
beyond four) due to the availability of energetically suitable vacant t/-orbitals in its atom.
CCI4 + H2O No reaction SiCl4 + 4 H2O ^ Si(OH)4 + 4HCl
ur

Silicic acid
ad

The mechanism of hydrolysis of SiCl4 involves the following two steps:


Yo

(a) The ifrst step involves the nucleophilic attack by lone pair of electrons present on the oxygen atom
of water molecule on the metal atom forming a coordinate bond between the metal and oxygen atom of
water.
d

Cl Cl H
Re
in

H o
H
Si + :0 Si
a^/ \Cl ci-7\Cl
F

H
Cl Cl

(b) The second step involves the loss of HCl. During this step, one C! atom of silicon in SiCl4 is
replaced by an OH group. This process continues till all the four Cl atoms are replaced by OH groups yielding
Si(OH)4, i.e.. silicic acid.
Cl H Cl OH
O +3H:.0
H
Si Si Si
Cl^/ \OH HO"^\OH
-HCl -3HC1

Cl Cl HO
Cl
SOME p-BLOCK ELEMENTS 11/29

(v) The tetrahalides of carbon do not fonn complexes because carbon does not have vacant (i-orbitals in
its valence shell and hence cannot increase its coordination
number beyond four. On the other hand, tetrahalides
of other elements form complexes due to the availability of vacant ^/-orbitals in their respective vacant shells.
Therefore, these can increase their coordination number to six. In other words, tetrahalides of Ge, Sn and Pb
behave as Lewis acids but tetrahalide of carbon does not. For example
Sip4 + 2HF H^SiF^ SnCl4 + 2Cr ^ SiiCl^-
Hydrofluorosilicic acid Hexachlorosiannaie ion

(b) Dihalides. All the elements, except carbon and silicon, i.e., Ge. Sn and Pb also form dihalides,
MX2. The stability of these dihalides increases steadily due to inert pair effect as we move down the group
from Ge to Pb, i.e., GeX2 « 80X2 < PbX2. Considering the thermal and chemical stability, GeX4 is more

w
stable than GeX2 whereas PbX2 is more stable than PbX4-

Curiosity Questions

F lo
f Q. 1. What is tin plague ?
Ans. Tin exists in three allotropic forms, i.e., grey, white and rhombic,
291K 434K

e
Grey ■> White t Rhombic

Fre
<■ <■

In cold countries, the conversion of white tin to grey tin occurs. Since grey tin is quite brittle, it
starts crumbling down and hence tin gets corroded. This corroding process of tin is called tin

for
disease or tin pest or tin plague.
Q. 2. Why does water carried through lead pipes becomes poisonous ?
Ans. Lead siowly dissolves in water containing dissolved air forming lead hydroxide which makes
r
water poisonous.
You
2 Pb + O2 + 2 H2O ^ 2 Pb(OH)2
oks

This dissolutionof lead in water is called plumbosolvency.It increases if water contains nitrates,
eBo

organic acids and ammonium salts. However, presence of salts like phosphates, carbonates
and sulphates in water retards plumbosolvency since they form a coating of the corresponding
insoluble lead salts on the surface of the lead pipe which protects it from further action. Since
hard water contains chlorides, sulphates and carbonates of Ca^"^ or Mg^"^ which form the
corresponding insoluble lead salts, therefore, hard water has no action on lead pipes.
J
ad
our

11.17. ANOMALOUS BEHAVIOUR OF CARBON

Carbon, the first member of group 14, shows an anomalous behaviour, i.e., differs from the rest of the
members of its family. The main reasons for this difference are :
Y
Re

(a) exceptionally small atomic size


d
Fin

(b) higher electronegativity


(c) higher ionization enthalpy
id) absence of <i-orbitals in the valence shell.
The main points of difference are :
(0 Carbon in form of diamond is very hard as compared to other members of group 14.
(»■) It has higher melting point and boiling point than other members of the group.
(Hi) Carbon has only four valence orbitals (one 2s and three 2p), therefore, at the maximum, it can
accommodate four pairs of electrons around it. Therefore, the maximum covalcncy of carbon is 4. The other
elements of this group due to the presence of ^/-orbitals can expand their covalency to 6. Thus, carbon does
not form [CFg]^" whereas silicon forms [SiF^l^“.
11/30 “P>taxUe^'A New Course Chemistry (XI)S!EIHD

(iv) Due to small size and high electronegativity, carbon has a strong tendency to form pn ~ pn multiple
bonds either with itself (C = C, C s C) or with other atoms such as oxygen (C = O), nitrogen
(C = N. C s N) and sulphur (C = S). The remaining elements, however, do not form pn - pn bonds because
their atomic orbitals are too large and diffused to have effective overlapping. Instead due to the presence of
J-orbitals, they have a tendency to form dn-pn bonds. Even this tendency to form dn - pn bonds decreases

w
down the group from Si to Pb as the size of the J-orbitals increases,
(v) Catenation. Carbon has the remarkable property of catenation which may be defined as the ability
of like atoms to link with one another through covalent bonds. This is due to smaller size and higher
electronegativity of carbon atom and unique strength of carbon-carbon bonds. The property of catenation
mainly depends upon the strength of element-element bond. Since the bond energy of C-C bond is very large

o
e
(348 kJ mol"'), carbon forms long straight or branched C-C chains or rings of different sizes and shapes.

re
However, as we move down the group, the element-element bond energies decrease rapidly, viz., C-C (348
kJ mor‘), Si-Si (297 kJ mor*), Ge-Ge (260 kJ mol"*), Sn-Sn (240 kJ mor*), Pb-Pb (81 kJ mor^) and,

Frl
therefore, the tendency for catenation decreases in the order

F
C » Si > Ge = Sn » Pb

(vi) Due to the property of catenation and pn - pn bond formation, carbon shows allotropic forms.
ou
or
11.18. CARBON

kfs
11.18.1. Allotropes of Carbon
The phenomenon of existence of an element in two or more forms which have different physical
properties but identical chemical properties is called allotropy and the different forms are called
oo
allotropes.
Y
For example, carbon exists in two allotropic forms : I. Crystalline, and II. Amorphous
eB

I. Crystalline allotropic forms of carbon. Four allotropes of carbon having well defined crystal
structures are :— 1. Diamond, 2. Graphite, 3. Fullerenes and 4. Carbon Nanotubes.
Due to different structures, they have different properties.
ur

1. Diamond
oY

It occurs in nature. It can also be prepared artificially but because of the high cost and poor quality,
ad

diamonds are seldom made artificially.


Structure. In diamond, carbon is .vp^-hybridized. Each carbon FIGURE 11.15
d

is tetrahedrally linked to four neighbouring carbon atoms through


0
four strong C-C, sp^ - sp^, a-bonds. This network extends in three
in

dimensions (Fig. 11.15) and is very rigid.


Re

Properties,(i) Purity. Diamondis the purestfonn of carbon.


(ii) Bond length. Because of .y/7^-hybridizalion. C-C bond
F

lengths in diamond are 1.54 A (154 pm),


(iii) Hardness. Since diamond e.xists as a three-dimensional
network solid, it is the harde.st substance known with high density
and melting point.
(iv) Conductivity. Since all the electrons are firmly held in
C-C, a-bonds, there are no free electrons in a diamond crystal.
Therefore, diamond is a bad conductor of electricity. However, it
has the highest thermal conductivity of any known substance,
Structure of diamond
(v) Transparency. Because of its high refractive index (2.5),
diamond can reflect and refract light. It is. therefore, a transparent
substance.
SOME p-BLOCK ELEMENTS 11/31

Uses. (0 Because of its hardness, diamond is used for cutting glass, making borers for rock drilling and
for making abrasives.
{«) When diamond is cut and polished, brilliant light is refracted from its surfaces. That is why diamond
is used for making precious gems and jewellery.
(Hi) It is used for grinding and polishing of hard materials,
(/v) Diamond is also used for making dies for drawing thin wires from metals.
2. Graphite
It occurs in nature and can also be manufactured artificially by heating coke to 3273-3300 K in an

w
electric furnace.

Structure. In graphite, carbon is ip^-hybridized. Each FIGURE 11.16


carbon is thus linked to three other carbon atoms forming
14T5 pm

Flo
hexagonal rings as shown in Fig. 11.16a. Thus, unlike 4 ¥

diamond, graphite has a two-dimensional sheet like


(layered) structure consisting of a number of benzenerings

ee
fused together (Fig. 11.16h). The various sheets or layers T
I

are held together by weak van der Wools’ forces of I I

Fr
E
attraction. The distance between any two successive layers o. I
o I I
is 3.40 A (340 pm). CO
I I
I I
Properties, (i) Purity. Like diamond, graphite is also I I

for
I I
the purest form of carbon.
ur
(ii) Bond length. Because of sp^-hybridization, the
C-C bond-lengths in graphite are 1.415 A (141-5 pm). o
ks
(iii) Softness. Since any two succe.ssive layers are
Yo
held together by weak forces of attraction, one layer can
oo
slip over the othe.r This makes graphite soft and a good
lubricating agent.
eB

(iv) Conductivity. Since only three electrons of each


carbon are used in making hexagonal rings in graphite,
fourth valence electron of each carbon is free to move. I
r

This makes graphite a good conductor of heal and I


ou
ad

electricity. However, its thermal conductivity is lower than


I

that of diamond. The reason being that thermal conduction


Y

is due to transfer of thermal vibrations from one atom to

the other which is easier in a compact crystal of diamond


than in layered structure of graphite,
nd
Re

(v) Opaqueness. Unlike diamond, graphite is a black o


substance and possesses a metallic lustre.
Fi

Uses. (/) It is used as a reducing agent in steel


manufacturing.
(ii) It is also used in high-strength composite Structure of graphite
materials.

(iii) Graphite is used for making electrodes for dry cells.


(iv) Graphite marks paper black and is, therefore, called black lead. Mixed with desired quantities of
wax or clay, graphite is used for making cores of lead pencils,
(v) It is used in the manufacture of crucibles which can withstand high temperature.
(vi) Graphite is also used as a moderator for fast moving neutrons in atomic reactors.
(vii) It is used as a solid lubricant for heavy machinery.
11/32 ^●uKiee^'A New Course Chemistry (XI)BZ39D

SUPPLEMENT YOUR
KNOWLEDGE FORtCOMPETITION^
1. Graphite is thermodynamically more stable than diamond since its free energy of formation is
i -9 kJ mol"* lower at room temperature and atmospheric pressure. Although the conversion of diamond
into graphite is thermodynamically favourable, yet it normally does not occur because of high energy of
activation for the process. If this energy of activation is made available, the conversion of diamond into
graphite can occur. The reverse process, /.e., conversion of graphite into diamond is thermodynamically
not possible but can be done only under forcing conditions. Thus, graphite can be converted into diamond
at 1873 K under a pressure of 50,000-60,000 atmospheres.
2. Diamond has the highest thermal conductivity of any known substance (about five times that of Cu)

low
although it is a bad conductor of electricity. It is because of its high thermal conductivity, diamond tipped
tools do not overheat and hence are extensively used for drilling and cutting purposes.
3. Kohinoor diamond. The value of diamond is expressed in terms of its weight and purity. The weight of
diamond is expressed in terms of carats (I carat = 200 mg). The famous Kohinoor diamond which decorates
the crown of England’s queen at present weighs 108-93 carats. However, when it was taken from India to
England, it weighed 181 carats. To restore its brilliance and lustre, 43% of its original weight was shed.

ee
3. Fullerenes

rF
Fr
Discovery. Before 1985, only two crystalline allotropes of carbon called diamond and graphite were
known. But in 1985, a third crystalline allotrope of carbon called fullerenes was discovered collectively by
three scientists namely R.E. Smalley and R..F Curl of Rice University, Houston, Texas (U.S.A.) and H.W.

for
Kroto of the University of Sussex, Brighton (U.K.). For this discovery, these scientists shared the 1996 Nobel
prize in chemistry.
Preparation and separation. Fullerenes constitute a new family of carbon allotropes consisting of
u
large spheroidal molecules of composition (where n > 30). These were first prepared by evaporation of
s
graphite using a powerful laser. However, a more practical method for production of fullerenes in macroscopic
ook
Yo
quantities, involves heating of graphite in an electric arc in an inert gas such as helium or argon when a sooty
material is formed by the condensation of C„ small molecules. The sooty material so formed mainly consists
of C50 fullerene with smaller quantity of C70 and traces of other fullerenes consisting of even number of
eB

carbon atoms upto 350 or above. The CgQ and C70 fullerenes can be readily separated from the fitllerene soot
by extraction with benzene or toluene followed by chromatography over alumina.
The complete process of formation and separation of C^q and C70 fullerenes from graphite may be
r

sketched as follows:
ou
ad

Electric arc Condensation


Graphite > Vapourised carbon
He or At
Y

Chromatography
Fullerene soot ^ ^60 ■*■ ^70
Re

over AItOj
nd

(^60 *-70^ Fullerene Fullerene

It is interesting to note that unlike graphite or diamond, the


Fi

fullerenes dissolve in organic solvents. A solution of C^q fullerene


in toluene is purple whereas that of C70 fullerene is orange red. In
fact, fullerenes are the only pure form of carbon because they do
not have dangling edge or surface bonds which attract other atoms
as is the case of graphite or diamond.
Structure. Of all the fullerenes, C^q allotrope is most stable.
as bucky ball.
It looks like a soccer ball and is sometimes called
It contains 20 six membered rings and 12 ifve membered rings. Buckminster fullerene (C^q). Note
Six-membered rings are fused both to other six-membered rings that all carbons arc equivalent and
that no five-membered rings are
and five-membered rings, but the five-membered rings are
adjacent to one another
connected only to six-membered rings (Fig. 11.17).
SOME p-BLOCK ELEMENTS 11/33

All the carbon atoms are equal and they undergo 5/?^-hybridizati on. Each carbon atom forms three a-
bonds with other three carbon atoms. The remaining electron of each carbon is delocalized in molecular
orbitals which, in turn, give aromatic character to the molecule.
This soccer ball shaped molecule has 60 vertices with a carbon atom at each vertex. It contains
both single and double bonds with carbon-carbon distances of 145-3 and 138-3 pm respectively. Noting the
similarity of this molecule to the geodesic domes (having hexagonal and pentagonal patterns) designed and
built by the American architect, Robert Buckminster Fuller, it was named as buckmlnster fiillerene or simply
fullerene.

Because all the carbon atoms are equivalent, the strain caused by the distortion of the bonds from

w
coplanarity is equally distributed among all the carbon atoms. Consequently, the molecule is quite stable.
€70 fullerene, on the other hand, resembles a rugby ball. It consists of 12 five-memhered rings and 25
six- membered rings. As in C5Q fullerene, the pentagons are isolated from each other.

Flo
Properties. (/) Fullerenes being covalent are soluble in organic solvents.
(//) They can be reduced electrochemically and react with group 1 alkali metals, forming solids such as
K3C60- This compound behaves as a superconductor below 18 K which means that it carries electric current

ee
with zero resistance.

Fr
(Hi) It reacts with OSO4 which adds across one of the double bonds in the cage.
(iv) It also forms platinum complexes.
Uses. At present, fullerenes do not have any practical

for
uses. However, it is projected that fullerenes may find many
ur
nanotechnological applications.
4. Carbon nanotubes. A carbon nanotube consists of a

two-dimensional array of hexagonal rings of carbon Just as in a


ks
layer of graphite or a chicken wire. The layer (or the chicken
Yo
wire) is then rolled into a cylinder and capped at each end with
oo
half of a Cgo-fullerene as shown in the Fig. 11.18.
eB

Carbon nanotubes are very tough, about 100 times as


strong as steel. They are electrically conducting only along
the length of the tube.
Carbon nanotubes can be used to produce very strong Carbon nanotube
r
ou

composite materials. They are also being used as probe tips


ad

for analysis of DNA and proteins by atomic force microscopy (AFM). It is also thought that carbon nanotubes
can be used to make molecular size test tubes or capsules for drug delivery.
Y

Retain in Memory
Re
nd

It is interesting to note that of all the crystalline allotropes of carbon, graphite is thermodynamically the most
stable alloirope. Therefore, its standard enthalpy of formation (A^ H“) is taken as zero. The stand^d enthalpy of
fonnation (^H°) of diamond and ^60 fullerene are 1-98 and 38-1 kJ mol * respectively.
Fi

II. Amorphous allotropic forms of carbon


Some important amorphous allotropic forms of carbon are :
1. Coke 2. Charcoal 3. Carbon black. These are all impure forms of graphite or fullerenes.
1. Coke. It is a greyish black hard solid and is obtained by destructive distillation* (strong heating in
absence of air).
2. Charcoal. It is obtained in the following four forms, i.e.,
(i) Wood charcoal. It is obtained by strong heating of wood in a limited supply of air.

*When coal is subjected to destructive distillation for the manufacture of oil gas, coke is left as residue in
the retort. At the same time, a thick deposit is obtained on the roof and sides of the retort. It is called as gas carbon.
11/34 ‘^uuUcp-'4. New Course Chemistry I'XHrosTwn

(ii) Animal charcoal. It is also known as hone black and is obtained by destructive distillation of bones.
It consists of approximately 10% carbon rest being calcium phosphate,
(iii) Sugar charcoal. It is the purest form of amorphous carbon and is obtained by the action of cone.
H2SO4 on sucrose or cane sugar.
Cone.H2SO4
^12^22^11 > 12C(.y) + 11 H20(0
Sugar charcoal
(iv) Activated charcoal. All forms of charcoal are highly porous substances and can adsorb many times

w
their own volume of gases. Their adsorption capacity can be further increased by heating charcoal at 1273 K
in a current of super heated steam. This treatment removes the impurities such as residual hydrocarbons,
oxygen, etc. sticking on the surface and thus blocking the capillary pores. Charcoal thus prepared is called
activated charcoal,

Flo
(v) Carbon black or lamp black. It is an almost pure form of amorphous carbon containing 98-99%
carbon. It is obtained when hydrocarbons such as natural gas and other substances rich in carbon such as
kerosene oil, petroleum, turpentine oil, acetylene, etc. are burnt in a limited supply of air. The soot obtained

ee
is made to stick on wet blankets hung in a chamber. After drying, the soot is removed from the blankets. The
soot thus collected is called the lamp black or carbon black.

Fr
A
CH4 + O2 C + 2H2O
Limited supplyof air
Carbon black

for
ur
All the allotropic forms of carbon, whether crystalline or amorphous, burn in excess of oxygen at different
temperatures to form carbon dioxide. This shows that all the above forms are allotropes of carbon.
11.18.2. Atomic and Physical Properties
ks
(/) As discussed above, carbon exists in various allotropic fonns of which diamond, graphite and fullerene
Yo
are crystalline while coal, wood, charcoal, animal charcoal; lamp black, coke and gas carbon are amorphous.
oo
The X-ray analysis has revealed that so called amorphous carbons also have extremely fine crystals. Therefore,
eB

these allotropes are called microcrystalline carbons.


Some atomic and physical properties of carbon are listed in Table 11.6, page 11/23.
11.18.3. Chemical Properties
r

Some important chemical properties of three important forms of carbon are described below :
ou
ad

(a) Charcoal. (/) Of all the allotropic forms, charcoal is the most reactive. It readily bums in air to form
CO2. It reduces cone. H2SO4 and cone. HNO3 to SO2 and NO2 respectively.
Y

(ii) It combines with many metals to form the corresponding carbides, e.g., CaC2, AI4C3, etc.
(iii) When heated with sulphur, it forms carbon disulphide (CS2).
Re
nd

(b) Diamond. Chemically, diamond is most resistant towards most of the chemical reagents. It bums in
air at 1173 K to form CO2.
Fi

(ii) It is slowly oxidised by a mixture of K2Cf207 + cone. H2SO4 at 473 K to give CO2.
(iii) It is stable in vacuum upto 1773 K but changes into graphite at 2073 K and more rapidly at 2273 K.
(c) Graphite. (/) Graphite, like diamond, is not attacked by dilute acids. It is slowly oxidised by chromic
acid (H2Cr04, i.e., K2Cr207 + cone. H2SO4) to CO2.
(ii) It is oxidised by cone. HNO3 to give graphitic acid, Cj JH4O5 which is an insoluble yellowish green
substance.

(iii) With alkaline KMn04, it is oxidised to oxalic acid and mellitic acid or benzenehexacarboxylic
acid, C6(C00H)6.
11.18.4. Uses of Carbon

(/) Graphite fibre reinforced plastics are prepared by reinforcing or embedding graphite fibres into a
light weight matrix such as epoxy resin, polyester resin or polyamide. These are high strength light weight
SOME p-BLOCK ELEMENTS 11/35

composites and are used for making superior sports goods such as tennis and badminton rackets, fishing rods,
etc., aircrafts, canoes, frames of racing cycles and bodies of racing cars,
(n) Being good conductor, graphite is used for making electrodes for batteries and industrial electrolysis.
Crucibles made up of graphite are inert to the action of dilute acids and alkalies. It is u.sed as a lubricant in
heavy industry. It is also used as a moderator for gas cooled nuclear reactors where it slows down the fast
moving neutrons.
(i7i) Being highly porous, activated charcoal is used for absorbing poisonous gases in gas masks, in
water filters to remove organic contaminalors and in air-conditioning system to control odour. It is also used
to purify and decolourize sugar and as a catalyst for some reactions.

w
(iv) Carbon black is used as black pigment in black ink and as filler in automobile tyres,
(v) Coke is extensively used as a reducing agent in metallurgy and also as a fuel.
(vO Diamond is a precious stone and is used in jewellery. It is measured in carats (I carat = 200 mg).

F lo
Diamond is also used for making drills or as an abrasive for cutting and polishing.
11.19. SOME IMPORTANT COMPOUNDS OF CARBON AND SILICON

ee
11.19.1. Oxides of Carbon

Fr
Carbon forms many oxides. Two of these oxides, viz., carbon monoxide (CO) and carbon dioxide
(CO2) are extremely stable and important. The three less stable oxides are ; carbon suhoxide (C3O2), C5O1
and CJ2O9. Others which are even less stable include graphite oxides, C2O and C2O3.

for
1. Carbon monoxide
ur
Preparation, (i) It is formed by incomplete combustion of carbon and carbon containing fuels.
2C + O2 ^2 CO
s
This type of incomplete combustion occurs during burning of petrol or diesel in automobiles and,
ok
Yo
therefore, CO is always present in automobile exhausts. It is also present in volcanic gases and gases coming
out of furnaces.
o

(ii) In the laboratory, pure carbon monoxide is obtained by dehydration of formic acid with cone. H2SO4
eB

at 373 K.
H2SO4
HCOOH CO + H2O
373 K
r

Formic acid Carbon monoxide


ad
ou

Carbon monoxide can also be prepaied in the laboratory by the action of cone, H2SO4 on potassium
ferrocyanide.
K4 [Fe(CN)6l + 3H2S04- 2 K2SO4 + FeS04 + 6 HCN
Y

HCN + 2 H2O - ^ HCOOH + NH3] X 6


Re

HCOOH HjO + CO] X 6


nd

2 NH3 +H2SO4- (NH4)2S04] X 3


Fi

K4 [FeCCN)^] + 6 H2SO4 + 6 H2O ^ 2 K2SO4 + FeS04 + 3 (NH4)2$04 + 6 CO


Pot. ferrocyanide
Commercial preparation. (/) It is commercially produced by passing steam over red hot coke. This
reaction also produces hydrogen.
473-1273K
C{^) + H20(g) 4 CO(g) + H2(g)
Water gas

A mixture of CO and H2 is called water gas or synthesis gas or simply syngas.


(ii) When air is used instead of steam, a mixture of CO and N2 are produced. This mixture is called
producergas.
11/36 ‘Pxadee^*^ New Course Chemistry (XI)BZ

1273K
2C(s) + 02(g) + 4N2 (g) ^ 2CO(g)+4N^(g)
Air Producer gas
CO present in water gas or producer gas can further undergo combustion forming CO2 with evolution of
heat.

2C0(g) + 02(g) >2C02(g); = -566kJmol"!

low
Thus, water gas and producer gas are two industrially important fuel gases.
Structure. In CO molecule, both C and O atoms are 5p-hybridized. One 5p-hybrid orbital each of C and
O overlap to form a C-O, a-bond while the other sp-orbital on each atom contains the lone pair of electrons.
The two unhybridized p-orbitals of C and O form two pn - pit bonds. Thus, CO is a linear molecule.
+

lone pair in - ► :c=o: < lone pair in


s/7-hybrid orbital s/?-hybrid orbital

e
Due to the presence of a lone pair of electrons on the carbon atom, CO acts a Lewis base or a ligand and

re
rF
can form a coordinate bond with metals (M <— C s o) to form metal carbonyls.

F
The electron dot structure for CO may be represented either by formula I or la.
:c::b: or :c=b:
I la

r
In these structures, C has 6 while O has 8 electrons in the valence sheU. To complete the octet around C,

fo
u
O donates a pair of electrons as shown in structure (Ila or lib or lie).
-:c:::o: or “:c=o: or ks:c^O:
Yo
Ila Ilb lie
oo
However, CO is best represented as a resonance hybrid of the following two structures.
eB

:c^o: <—► -:c=o:

The presence of a triple bond between C and O is supported by the following evidences :
(/) The carbon-oxygen bond length is just 113 pm which corresponds to a carbon-oxygen triple bond.
ur

(«) The dipole moment of CO is very low due to back donation of a pair of electrons from the more
ad

electronegative O to the less electronegative C-atom.


Yo

Properties. Some physical properties of CO are given in Table 11.7, page 11/38.
(/) It is a neutral oxide.
(//) It is a colourless and odourless gas which is only slightly soluble in water.
d

(«0 Poisonous nature. Carbon monoxide is highly poisonous (toxic) in nature. Its toxic nature is due to
Re
in

its ability to form a complex with haemoglobin (carboxyhaemoglob in) which is about
Haemoglobin + CO ■» Carboxyhaemoglobin
F

300 times more stable than the oxygen-haemoglobin (oxyhaemoglobin) complex formed in the lungs.
Haemoglobin + O2 ^ - Oxyhaemoglobin
This prevents haemoglobin present in the red blood cells from carrying oxygen from the lungs to all
parts of the body thereby causing suffocation ultimately leading to death,
(iv) Reducing properties. Since CO can be easily oxidised to CO2, it acts a as powetfitl reducing
agent. As such, it reduces many metal oxides to their respective metals.

ZnO + CO > Zn + CO2 CuO + CO Cu + CO2


I273K

Fe202 + 3 CO ■> 2Fe + 3C02


SOME p-BLOCK ELEMENTS 11/37

It also reduces PdCl2 to Pd and I2O5 to I2.


PdCl2 + CO + H2O > Pd + CO2 + 2 HCl I2O5 + 5 CO ■^12 + 5 CO2
(v) Formation of metal carbonyls. It combines with many transition metals such as iron, cobalt, nickel,
etc. forming metal carbonyls. For example,
330-350K 473 K, 100 atm. pressure
Ni + 4CO Ni(CO)4 Fe + 5 CO ■¥
Fe(CO)s
Nickel carbonyl Iron carbonyl
Nickel carbonyl is volatile. When heated to 440-450 K, it decomposes to form pure nickel.

w
450-470K
Ni(CO)4 > Ni + 4CO

Therefore, nickel carbonyl is used for purification of nickel by Mond’s process.

Flo
(vi) Formation of phosgene. It readily combines with CI2 in presence of sunlight to give carbonyl
chloride or phosgene which is an extremely poisonous gas.
hv

e
CO(g) + C\2 ig) ^ COCl2(g)

e
Phosgene

Fr
(vii) Absorption. Carbon monoxide is readily absorbed by a solution of CuCl in cone. HCl or NH3 due
to the formation of soluble complexes. For example,
CuCl + NH3 + CO 4 [Cu(CO)NH3rcr ; CuCl + HCl + CO ^ H+[Cu(CO)Cl2]~

or
ur
(Soluble complex)
Uses. (/) it is an important constituent of two industrial fuels, i.e., water gas and producer gas.
(Soluble complex)

sf
(if) It is used in Morui’s process for purification of nickel via its nickel carbonyl.
(iif) It is used in the manufacture of methyl alcohol, acetic acid, synthetic petrol, sodium formate, etc.
(iv) In the metallurgy of iron as a reducing agent,
k
Yo
oo
(v) Iron carbonyl is used in the manufacture of magnetic tapes for videos and tape recorders.
2. Carbon dioxide
B

Preparation. (/) It is prepared by burning carbon, fossil fuels and other organic compounds in excess of
air or oxygen.
re

C(s) +02(g) 002(g) ^5^12 ^S) + 8 O2 (g) ^ 5C02(«) + 6H20(g)


(if) In the laboratory, it is prepared by the action of dilute acids on carbonates.
u
ad
Yo

CaC03 + 2 HCl CaCl2 "b CO2 + H2O


(iif) Commercially, CO2 is produced as a by-product during manufacture of ammonia. The hydrogen
needed for the purpose is obtained by passing steam over heated CO or CH4.
nd

CO(g) + H20(g) ^ C02(s) + H2(s) ; CH4(s) + 2H20(s)-^C02(g) + 4H2te)


Re

It is also formed during manufacture of lime or ethyl alcohol by fermentation of glucose or fructose.
Fi

1070-1270K
CaC03 > CaO + CO2
Limestone Lime

Zymase
^6^12^6 Fermentation
■»
2C2H5OH + 2CO2
Glucose or Fructose Ethyl alcohol
Structure. In CO2 molecule, C is ^p-hybridized, it forms two a-bonds with two oxygen atoms and two
pK-pn multiple bonds. As a result, CO2 is a linear, monomeric covalent compound.
The electron dot structure for CO2 may be represented by either formula I or la
● ● ● ●

:o::c::o: or :0=c=o:
I la
11/38 “PnaieUefi^ it New Course Chemistry (XI) Q

This structure predicts that both the carbon-oxygen bond lengths in CO2 should be equal and should
have a typical bond length of 122 pm. However, experimentally, it has been found that carbon-oxygen bond
length in CO2 is only 115 pm. This can be explained, if CO2 is considered to be a resonance hybrid of the
following structures:

A
~:o—C=o: ◄—► :o—C—o: ^
A > :o=c—o

ow
Due to resonance, carbon oxygen bond length acquires some triple bond character and hence the bond
length decreases from 122 pm to 115 pm.
Properties. The physical properties of CO2 are quite different from those of CO. Some important
physical properties of CO and CO2 are given in Table 11.7.
TABLE 11.7.
Ith^ical prpjpeifdes' of GQ

e
Prpjperly CQ ^02 ■■

Fl
re
Melting point (K) 68 2164 at 5-2 atm

F
Boiling point (K) 81-5 194-5 (Sublimes)
Density (gL“^) at 273 K ur 1-250 1-977

C-O bond length (pm) 112 115

r
Heat of formation i.e. Hy (kJ mol“^) -110-5 -393-5

fo
(i) It is a colourless and odourless gas about 1-5 times heavier than air.
ks
(ii) Unlike CO, CO2 is not poisonous. However, it does not support life in animals and human beings.
Yo
However, it can cause suffocation and eventually death due to lack of oxygen,
oo
(iii) Non-combustible nature. Ordinarily, CO2 is neither combustible nor a supporter of combustion.
However, certain active metals such as Na, K, Mg, etc. continue burning in it.
eB

2 Mg + CO2 > 2 MgO + C


(iv) Solubility. It is slightly soluble in water. Its solubility in water, however, increases with increase in
pressure. Soda water and other aerated soft drinks are, in fact, solutions of carbon dioxide in water (containing
ur

sugar, some flavouring and colouring agents) under pressure,


(v) Acidic nature. When CO2 dissolves in water, only some of the molecules react with water to form
ad

carbonic acid (H2CO3) while most of the dissolved, CO2 remains loosely hydrated. Carbonic acid is a weak
Yo

dibasic acid. It forms two series of salts, /.e., the hydrogen carbonates and carbonates derived from the

anions HCOJ and CO^". Thus, CO2 is the anhydride of carbonic acid.
d

C02(g) + H20(/) ;f=^ H2C03(o^)


Re
in

Carbonic acid

H2CO3 (aq) ^ ± H+ (aq) -f HCOJ (aq)


F

HCOJ (aq) ^ ± H+(o9)+ COf- (aq)


Thus, a solution of CO2 in water is actually an equilibrium mixture of CO2, H2CO3, HCOJ and
CO|“. These equilibria are very important in H2CO3/HCOJ buffer system which help to maintain the
pH of the human blood between 7-26 - 7-42. Furthermore, the rate at which CO2 comes in equilibrium with
H2CO3 is slow and this is also important in the physiology of CO2.
(vi) Reaction with lime water. When CO2 is passed through lime water, it turns lime water milky due
to the formation of insoluble calcium carbonate.

Ca(OH)2 + CO2 ■> CaC03 ■¥ H2O


Lime water (Insoluble)
SOME p-BLOCK ELEMENTS 11/39

However, if CO2 is passed for a longer period, the turbidity disappears due to the formation of soluble
calcium bicarbonate.

CaCOj + H2O + CO2 ■>


Ca(HC03)2
(Soluble)
or
Ca(OH)2 + 2CO2 Ca(HC03>2
This reaction is used as a test for CO2, under the name lime-water test,
(vii) Photosynthesis. Carbon dioxide, which is normally present to the extent of ~ 0-03% by volume in
the atmosphere, is absorbed by plants. In presence of chlorophyll (the green colouring pigment of the leaves)
and sunlight, the absorbed CO2 combines with H2O to form glucose and starch (carbohydrates) which are

low
used as food by the plants. This process is called photosynthesis.
Chlorophyll
6CO2 + 12 H2O > CgH|20g + 6 O2 + 6 H2O
sunlight
Glucose

By this process, plants prepare food for themselves as well as for animals and human beings. But the increase
in combustion of fossil fuels and decomposition of limestone for cement manufacture during the past two decades

ee
has considerably increased tlie CO2 content in the atmosphere. This may lead to increase in greenhouse effect and

rF
thus raise the temperature of the atmosphere which might have serious consequences,

Fr
(viii) Action of ammonia. When CO2 is reacted with liquid ammonia at 453-473 K under a pressure of
220 atmospheres, it first forms ammonium carbamate which subsequently rearranges to give urea.
453-473K
2NH3 + CO2 > [NH2COONH4]

for
220 atm.
^ NH2CONH2 + H2O
Amm. carbamate Urea

Uses. Carbon dioxide is used : (/) in the preparation of aerated waters,


u
(n) as a fire extinguisher because it is a non-supporter of combustion.
ks
(Hi) in the manufacture of washing soda by Solvay ammonia process.
Yo
(tV) Solid carbon dioxide is used as a refrigerant under tlie commercial name dri hold (see under dry ice),
oo
(v) Super critical CO2* is used as a solvent to extract organic compounds from their natural sources
such as caffeine from coffee beans and perfumes from flowers.
eB

(vO for artificial respiration (for victims of CO poisoning) as a mixture of 95% O2 and 5% CO2 under
the name carbogen
(vii) for purification of sugar in sugar industry.
r

3. Dry ice
ou
ad

Dry ice is the name given to solid carbon dioxide. It is also called cardice. It is obtained when CO2 is
cooled under pressure (50-60 atm). When solid carbon dioxide is kept in air under one atmospheric pressure,
Y

it sublimes, i.e., it changes directly into the gaseous state without liquefying. As a result, unlike ordinary ice,
it does not wet the surface on which it is kept. Therefore, it is called dry ice.
Dry ice is used for making cold baths in the laboratory by mixing it with volatile organic solvents. For
nd
Re

example, a temperatureof 196 K (-1TC) is obtained by using a mixture of dry ice and acetone and a temperature
of 165 K (-108°C) is obtained by using a mixture of dry ice and ether. It is also used as a coolant for
Fi

preserving perishable articles in food industry. Dry ice is also used for curing local burns and in hospitals for
surgical operation of sores.
11.19.2. Silicon Tetrachloride

Silicon tetrachloride is prepared by heating either silicon or silicon carbide with chlorine.

Si (s) 2 CI2 (g) SiCl4 (0


SiC (s) + 4 CI2 (g) SiCl4 (/) + CCI4 (/)
*A gas can be liquefied by applying pressure if it is below its critical temperature. For CO2, the critical
pressure and temperture are 73 x 1013 x 10* Pa (approx. 72-9 atm) and 304 K respectively. A substance above its
critical pressure is called supercritical fluid. Although at this pressure, CO2 is a gas, yet it is very dense and hence
can behave as a solvent.
11/40 4 New Course Chemistry (XI)CEIHI]

Properties and Uses, (i) Physical state. It is a volatile liquid, b.p. 330-57 K.
(ii) Hydrolysis. Hydrolysis of SiCl4 gives silicic acid, Si(OH)4 which upon further heating undergoes
partial dehydration to give silica gel.

SiCl4(/) + 2H20(/) ^ Si(OH)4(a^) — ■> Si02 . a: HjO (s)


Silicic acid Silica gel

Silica gel is amorphous and very porous. It contains about 4% water. It is used as a catalyst in petroleum
industry and as an adsorbent in column chromatography.
If, however, hydrolysis of SiC^ is carried out at high temperature (in an oxy-hydrogen flame) instead of

w
Si(OH)4, finely powdered silica is obtained
High temp.
SiCl4(g) + 4H20(0 > Si02 (s) + 4 HCl (s)
O2-H2 flame

F lo
The ultrafine silica thus obtained is used as a thixotropic* agent in polyester and epoxy paints and
resins and as an inert filler in silicon rubber,

(iii) Reduction. Reduction of SiCl4 with dihydrogen gives silicon.

ee
SiCl4 (g) + 2 H2 (g) ^ Si (s) + 4 HCl (g)

Fr
This method is u.sed to prepare ultrapure silicon used for making transistors, computer chips and solar
cells. For this purpose, impure silicon is first converted into volatile SiC^ by reacting with chlorine gas.

for
SiCl4 thus formed is separated from impurities by distillation. Subsequent reduction of pure SiC^ with
dihydrogen gives ultrapure silicon,
ur
(iv) Reaction with silicon. Pyrolysis (strong heating) of SiC^ with Si gives a series of perhalosilanes of
the general formula, Si„Cl2„+2 wheren = 2-6.
s
ook
SiCl4 + Si — Si2Clg + higher members of the series upto SigCl 14
Yo
These chains are longer than those formed by hydrides (i.e. silanes). This is due to additional pit- dn
bonding between lone pairs of electrons present on Cl and empty J-orbitals of Si.
eB

11.19.3. Silicon Dioxide, Silica (SIO2)


Silicon dioxide is commonly called silica. 95% of the earth crust is made up of silica and silicates. Silica
r

occurs in several crystallographic forms, namely, quartz, cristobalite and tridymite. These crystalline forms
ad
ou

are interconvertible at suitable temperatures.


Silicon dioxide is a covalent, three-dimensional network solid in FIGURE 11.19

which each silicon atom is bonded to four oxygen atoms which are
Y

Si—O—Si—O—Si — 0—Si—
arranged tetrahedrally around it and each oxygen atom is attached to two
silicon atoms by covalent bonds as shown in Fig. 11.19. Each comer is
Re

O O O O
nd

further shared by another tetrahedron. The entire crystal may thus be


considered as a giant molecule in which eight-membered rings are formed —Si—O—Si—O—Si O—Si—
Fi

with alternate silicon and oxygen atoms (Fig. 11.19). o o O 0


Since Si-0 bonds are very strong (368 kJ mol”’), silica is relatively
_Si—0—Si—0—Si —0—Si-
inert and has a very high melting point. It does not react with halogens
(except F-,), dihydrogen and most of the acids (except HF) and metals
even at high temperatures. Structure of S1O2
However, it is attacked by F2, HF and NaOH.
S1O9 + 2 F2 SiF4 ●+■ O2
Silicon tetrafluoride

Si02 + 4 HF S1F4 + 2 H2O


*The property of showing a temporaryreduction in viscosity when shaken or stirred is called thixotropy.
SOME p-BLOCK ELEMENTS 11/41

Sip4 thus formed dissolves in HF to form liydrofluorosilicic acid. Fuse


Sip4 + 2HF . H^SiF^ SiOo + 2NaOH Na2SiO-^ + H2O
Hydrofluoro- Sod. silicate
.silicic acid

Due to the formation of H2SiFg, SiO-> present in glass dissolves. Therefore, HF cannot be stored in glass
bottles but is stored in containers made up of copper or monel metal alloy (68% Ni + 32% Cu + traces of Fe).
Quartz is extensively used as a piezoelectric material. Through its use, it has been possible to develop
extremely accurate clocks, modem radio and television broadcasting and mobile radio communications.
Amorphous forms of silica. Two important amorphous forms of silica are : silica gel and kieselguhr.
(a) Silica gel. It may be prepared as follows :

w
(t) By the action of water on silicon tetrachloride or tetrafluoride
SiCl4 + 4H2O ^ 4HC1 + Si02.2H20

F lo
Silicon tetrachloride Silica gel
(«) By the action of acid (HCl) on soluble sodium silicate when gelatinous ppt. of silica gel are obtained.
Na2Si03 + 2 HCl + H2O ^ Si(OH)4 or S1O2.2 H2O + 2 NaCl

ee
Silica ge!
(Hi) By dehydrating silicic acid

Fr
Si(OH)4 (aq) ^ Si02.2 H2O
Silicic acid Silica gel

for
Silica gel is amorphous and very porous. It contains about 4% water.
Uses. (/) It is used as a catalyst in petroleum industry and as an adsorbent in column chromatography.
ur
(//) When (NH4)2CoCl4 is added to silica gel, it is used as a humidity detector since it is blue when dry
but turns pink when hydrated. To protect merchandise from moisture during storage, small packets of dry
s
silica gel (blue) are placed in the packing boxes,
ook
Yo
(b) Kieselguhr. It is another form of amorphous silica. It is a porous white powder and is used in
filtration plants, as an abrasive and as an inert support or filler. For example,
eB

(0 Gelignite is a mixture of the explosive nitrobenzene (liquid) and kieselguhr.


(ii) Dynamite is a mixture of glyceryl trinitrate and glyceryl dinilrate absorbed on kieselguhr.
11.19.4. Silicon Carbide, Carborundum, SIC*
our

Preparation. It is produced by heating quartz or silica (sand) with excess of coke in an electric furnace
ad

at 2273-2773 K.

2273-2773K
Si02 + 2C Electric ftimacc
^ Si+ 2 CO
Y

Silica Coke
Re

2273-2773K
Si-hC ■> SiC
nd

Electric furnace
Silicon carbide

or Simply, SiO-> + 3 C
Fi

^ SiC + 2 CO
(ii) Structure. SiC has a thi'ee-dimensional structure of Si and C atoms ; each Si atom is surrounded
tetrahedrally by four C atoms and vice-versa. There are large number of different crystal forms based on
either the diamond or the wurtzite structure.

Properties. (/) Colour. The colour of SiC is often dark purple, black or dark green due to presence of
traces of Fe and other impurities, but pure samples are pale yellow to colourless.
(ii) Hardness. SiC is very hard (9-5 Mohs) though less than diamond (10 0 Mohs).
(in) Inertness. It is extremely inert. It resists the action of all acids (including HF) except H3PO4. But
when fused with alkalies in presence of air, it decomposes to form sodium silicate and sodium carbonate.
*lncluded only in ISC syllabus.
11/42 ‘Pn/netc^p.'A New Course Chemistry (XI)EasnEl

Fuse
SiC + 4 NaOH + 2 O2 ■> Na^C03 + Na-)Si03 + 2 H2O
(iv) Action of CI2. On passing CI2 over SiC at 873 K, it forms SiCl4 and carbon.
873K
SiC + 2 CI2 ^ SiCl4 + C
Uses. (/) It is widely used as an abrasive in form of grinding wheels, knife sharpeners, etc.

ow
(«) It is also used in making metal melting crucibles.
11.19.5. Silicones

Silicones are synthetic organosilicon polymers containing repeated R2SiO units held by
Si—O—Si linkages.
These compounds have the general formula (R2SiO)„ where R = methyl or aryl group. Since their empirical

e
formula, R2S1O, is similar to that of a ketone (R2CO), the name silicone has been given to these materials.

re
Commercial silicone polymers are generally methyl derivatives and to a lesser extent phenyl derivatives.

Fr l
Preparation. When methyl chloride reacts with silicon in the presence of copper as a catalyst at

F
570 K, various types of methyl substituted chlorosilanes of formula CH3SiCl3 (or MeSiCl3), (CH3)2SiCl2 (or
Me2SiCl2), (CH3)3SiCl (or Me3SiCI) along with small amount of (CH3)4Si or Mc4Si are formed. Hydrolysis
of dichlorodimethylsilane followed by polymerization yields straight chain polymers.

or
2 CH3CI
ou + Si
Cu powder
570K
(CH3)2SiCl2

ksf
Methyl chloride Dichlorodimethylsilane

CH3 CH3 'CH3 CH3


oo
Polymerization
Cl—Si
●Cl + 2H2O -2HC1
> HO—Si—OH ; -0--Si—O-^Si—
Y
CH3 CH3 CH3 CH3
B

-'/I

Dichlorodimethylsilane Silicone

The chain length of polymer can be controlled by adding (CH3)3SiCl which blocks the ends as shown
re

below :

CH3 CH3 CH3


oYu

CH3
Polymerization
ad

„HO—Si—OH + HO—Si—CH3 ^ _0-USi—0-hSi —CH,


I -nH,0 I '' ^
CH3 CH3 CH3
d

-‘n

Silicone
in
Re

Properties and Uses. (/) Silicones made up of short chain molecules are oily liquids ; silicones with
medium chains behave as viscous oils, jellies and greases ; those with very long chains behave as rubbery
elastomers and resins,
F

(ii) They are chemically inert, i.e., resistant to oxidation, thermal decomposition or to attack by organic
reagents/chemicals.
(Hi) Silicones being surrounded by non-polar alkyl groups are water repelling in nature.
(iv) They are heal resistant and have high dielectric strength, i.e., have good electrical insulating properties.
Because of these properties they find a wide range of uses as described below :—
(/) For making water proof papers, wool, textiles, wood, etc. by coating them with a thin film of silicone.
(ii) As sealants and electrical insulators.
(Hi) As lubricants at high as well as low temperatures since there is little change in their viscosities with
temperatures.
(iv) Being biocompatible, they are used in surgical and cosmetic implants.
SOME p-BLOCK ELEMENTS 11/43

11.19.6. Silicates

A large number of silicate minerals exist in nature. Some of these important minerals are : feldspars,
i.e.,Albite NaAlSi30g, zeolites, e.g., chahazite Ca2l,(A102)4(Si 02)8.H20], micas (muscovite) [KAl2(Si3A10io)
(0H)2] and asbestos [Mg3(Si205)(OH)4]. The basic structural unit in silicates is the SiO^” tetrahedra. Besides
these natural silicates, some artificial silicates have also been prepared. For example, the two important man
made silicates are glass and cement.

Silicates are the compounds in which the anions present are either discrete SiO^ tetrahedra
or a number of such units joined together through corners.

w
The tetrahedral structure of SiO^ is shown in FIGURE 11.20

Fig. 11.20 (/■). The structure (//) is the projection formula

F lo
of SiOj- in which small open circles represent oxygen O = Oxygen
● = Silicon
atoms and the small closed circle inside represents the
silicon atom.

ee
As stated in the definition above, these discrete

Fr
tetrahedra or a number of tetrahedra linked together ●v V
are present as anions in the silicates. However, when o o
they link together, they do so only by comers and never
Discrete SiO^tetrahedron

for
by edges. Thus, the anions present in the
ur
silicates may be SiO^“ (for discrete tetrahedra), (for two tetrahedra linked through one comer) and
so on.
s
Types of silicates. Silicates are classified into the following different types depending upon the number
ook
Yo
of comers (0, 1, 2, 3 or 4) of SiO^" tetrahedron shared with other tetrahedra through oxygen atoms. The
negative charge on silicate structure is neutralized by positively charged metal ions.
eB

1. Orthosilicates. These are simple silicates containing discrete SiO^" tetrahedra, i.e., there is no
sharing of comers with one another as shown in Fig. 11.20.
Examples : Zircon (ZrSi04), Forestrite
r

FIGURE 11.21
ou
ad

(Mg2Si04), Wellimite (Zn2Si04), Phenacite


(Be2Si04).
Y

2. Pyrosilicates. When two


tetrahedra share one comer (i.e. oxygen atom),
Re
nd

Si,0^- anion is formed (Fig. 11.21). Silicates


containing this anion are called pyrosilicates. The
Fi

Anions, (SI207 ) in pyrosilicates


stmetures possessed by them are also called island
structures.

Examples : Thortveitite, Sc2(Si207), Hemimorphite, Zn3(Si207) Zn(0H)2-H20


3. Cyclic or Ring silicates. If two oxygen atoms per tetrahedron are shared to form closed rings such
that the structures with the general formula (SiO^")^ or (SiOj)^^ are obtained, the silicates containing
2n-

these anions are called cyclic silicates. The typical examples of such anions are Si^O^” and SigOjg", as
shown in Fig. 11.22.
Examples : Wollastonite, Ca3$i309, Beryl, Be3Al2SigOjg. Emerald has the same structure as beryl
except that it contains 1-2% Cr which gives it a strong green colour.
11/44 New Course Chemistry (XI)CZ5IHD

FIGURE 11.22
4. Chain silicates. If two oxygen atoms per
tetrahedron are shared such that a linear single

strand chain of the general formula (SiO^“)^^ or


(SiO^)^" is formed, then the silicates containing
these anions are called chain silicates (Fig. 11.23).
Examples : The above type of structure is
found in the minerals called pyroxenes, e.g.,
Spodumene, LiAl(Si03)’>, Diopside, CaMg(Si03)2.

w
If two chains are cross-linked, the resulting
double stranded (chain) silicates having the formula
[(Si40|j)^“]„ are called amphiboles. Common
Anions, (a) Si30f~and (b) SigOfg

F lo
examples are ;
in cyclic silicates
Tremolite, Ca2Mg5(Si40||)2(0H)2,
Asbestos. CaMg30(Si40]|).

ee
FIGURE 11.23
5. Sheet silicates. The sharing of three

Fr
corners {i.e. three oxygen of each tetrahedron)
results in an infinite two dimensional sheet structure

or (Si20|“)^j as shown
2h-

for
of the formula (Si.,03)^
in Fig. 11.24.
r
Silicates containing these anions are called
You
sheet silicates.
s
Examples : Kaolinite [AI(0H)4Si205], Talc,
ook

Mg3(0H)3Si40||. Clay also belongs to this class


A

Anion (SiO| )„ in chain silicates


containing (Si205)““ anions.
eB

6. Three-dimensional silicates. If ail the four

comers {i.e. all the four oxygen atoms) are shared FIGURE 11.24

with other tetrahedra, three-dimensional network


our
ad

structure is obtained.

Examples : Different forms of .silica such as


quartz, tridymiie and cristobalite.
11.19.7. Zeolites
dY
Re

If some of the silicon atoms in a three


dimensional network silicate are replaced by Ap"^
Fin

ions, the overall structure thus obtained carries a


negative charge and is called an aluminosilicate.
Therefore, to balance the negative charge, some
cations such as Na"*", K'*' or Ca^'*' are incorporated
into the structure. Such three-dimensional

aluminosilicates are called feldspar (KAlSi30g)


and zeolites (NaAlSi20(-,.H20).
Anion (S12O5 )„ in the sheet silicates
Zeolites are widely used as catalysts in
petrochemical industries for cracking of
hydrocarbons and isomerisation. For example, 2SM-5 (a type of zeolite) is used to convert alcohols directly
into gasoline. Hydrated zeolites called permutit are used as ion exchangers in softening of hard water.
SOME p-BLOCK ELEMENTS 11/45

11.20. TYPICAL PROBLEMS

P. 1. Compound X on reduction with L1AIH4 gives a hydride Y containing 21*72% hydrogen along with
other products.The compoundY reacts with air explosivelyresulting in boron trioxide. Identify X
and Y. Give balanced equations involved in the formation of Y and its reaction with air. Draw the
structure of Y.

Sol. Step 1. To determine the molecular fonnula and structure of compound Y.


(;) Since the hydride Y reacts with air forming boron trioxidc, therefore, Y must be an hydride of boron,
(ii) %H = 2\-72% {Given)
%B= 100-21-72 = 78-28%

78-28 21-72
Now, B : H = = 7-12:21-72= I :3 E.F. of Y = BH 3
1

w
Since boron forms two types of hydrides, i.e., ^^{nidoboranes) H H
and B..H {arachnoboranes), therefore, Y must be a nidoborane

F lo
with n = 2. Thus, M.F. of Y = If Y is B2H(^ (diborane), then
its structure must be as follows ;
H ''H-" H
Bridges B H= 134 pm
Terminal B—H = 119 pm

e
Step 2. To determine the structure of the compound X.

Fre
Since compound Y i.e., B->H(', is formed by reduction of compound X with LiAlH4, therefore X must be

either BCI3 or BF3.

for
elher
4 BF3 -f 3 UAJH4 ^ 2 B2H6 + 3 LiAIF4
X Y
r
The equation representing the reaction of Y with O2 may be written as follows :
You
B2H6 + 3 O2 B2O3 + SHjO
oks

Diborane. Y Boron trioxide


eBo

Thus, X = SF3 and Y = ^2^6-


P. 2. Starting from SiCl4, prepare the following in steps not exceeding the number given in parentheses
(give only reactions) :
(fl) Si (1) {b) Linear silicon containing methyl groups (4) (c) Na2Si02 (3).
ad
our

A A

Sol. (a) SiCl4 + 2 Mg Si + 2 MgCl2 or 3 SiCl4 + 4 A1 > 4 AICI3 + 3 Si


Cu powder
{b) (0 S1CI4 + 2 Mg Si + 2MgCl2 {ii) Si + 2 CH3CI 570 K (CH3)2SiCl2
Re
dY

CH3
{Hi) (CH3)2SiCl2 + 2 H2O ^ HO —Si —OH
Fin

CH3
CH3 CH3 CH3
Polymerization
(iv) HO —Si —OH ■> -0-- Si—O -Si —

CH3 CH3 J n CH3


Silicon polymer
(c) (0 SICI4 + 4 H2O ^ Si(OH)4 + 4HCI
A A

{«) Si(OH)4 Si02 + 2 H2O {Hi) SiO, + 2 NaOH ^ Na-5Si03 + H-)0

I
11/46 New Course Chemistry fXIlragrwn

P. 3. For a compound LiAl(Si03)2, what is the charge on the Si03 unit ? What is the arrangement of
oxygen atoms around the silicon atom ?
Sol. Since lithium has one unit +ve charge and aluminium has three units +ve charge, tlierefore, total +ve
charge on cations is 4. Since the mineral LiAl(Si03)2 is electrically neutral, therefore, total - ve charge
on two Si03 units is - 4 or each Si03 unit must carry two units - ve charge and must be represented as
SiO^-.
We know that all silicates consist of SiO^“ tetrahedra. But from the formula of the given silicate, it
appears that there are only 3 oxygen atoms per silicon atom. This can be explained if two of the oxygen

atoms of each SiO^" unit share with each other giving a simple chain structure for the silicate, LiAl(Si03)2
as shown below :

w
F lo
e
Fre
for
r
I. Elements of Group 13
You
ks
1. Group 13 of the periodic table consists of six elements, viz., boron (B), aluminium (Al), gallium (Ga),
o

indium (In), thallium (Tl) and the recently discovered element, ununtrium (Uut) which is radioactive.
eBo

2, The outer electronic configuration of these elements is ns" np'^ where n = l-l.
3. Atomic and ionic radii. The atomic and ionic radii of group 13 elements are smaller than those of the
corresponding elements of group 2 due to increased effective nuclear charge. As expected, on moving
down the group, atomic radii increase. However, the atomic radius of Ga (135 pm) is little smaller than
our
ad

that of Al (143 pm) due to ineffective shielding of the valence electrons by the inten’ening 3d-electrons.
4, Ionization enthalpies. Since /j-electrons are held less strongly than the 5-electrons, therefore, the first
ionization enthalpies (A,-Wy) of the group IS elements are lower than those of the corresponding elements
of group 2.
dY
Re

On moving down the group, from B to Al, there is a sharp decrease in the first ionization enthalpy (A; Hj)
of Al due to an increase in atomic size and screening effect of the 2s and 2p-electrons which outweigh the
effect of the increa.sed nuclear charge. The A, of Ga is. however, slightly higher (2 kJ mot^) than that
Fin

of Al due to ineffective screening of the Sd-electrons. On further moving down from Ga to In, although the
nuclear charge increases by 18 units (49 - 31 = 18), but the screening effect of all the inner electrons
outweighs the effect of increased nuclear charge and hence A^ of In is lower than that ofGa. On further
moving down from In to Tl, the nuclear charge increases considerably by 32 units (81 - 49) which outweighs
the screening effect of all the electrons of the inner shells including those of newly added 4/ and 5d
electrons. As a result, A,- Hj of Tl is much higher than that of In, even higher than those of Ga and Al. Thus,
amongst elements of group 13. B has the highe.st while In has the lowest A,- . The overall. A, of group
13 elements follows the order : B > Tl > Ga > Al > In.
5. Electronegative character. The elements of group 13 arc more electronegative than alkali and alkaline
earth metals. Boron has the highest value of electronegativity (2 0) while Al has the lowest value (1-5).
From Al to Tl, the electronegative values increase slowly.

1
SOME p-BLOCK ELEMENTS 11/47

6. Electropositive character. The elements of group 13 are less electropositive,i.e., less metallicas compared
to alkali and alkaline earth metals. On moving down the group, the metallic character first increases from
B to A1 and then decreases from A1 to Tl. Thus, amongst elements of group 13, B is the least metallic
element, actually it is regarded as non-metallic while Al is the most metallic element.
7. Melting and boiling points. The melting points of group 13 elements do not show a regular trend. They
first decrease from B to Ga and then increase from Ga to Tl. Boron has a high melting point (2453 K)
because of its unique crystal structure which consists of icosahedral units. Gallium has an unusual structure.
It consists of G'd2 molecules. It has a low melting point (303 K) and exists as a liquid upto 2676 K.
Therefore, it is used in high temperature thermometry.
8. Oxidation states. B andAl uniformly show an o.xidation .state of+3 while rest of the elements show two
oxidation states of+1 and +3 due to inert pair effect. Further as we move down the group from Al to Tl,

w
the stability of +3 oxidation states decreases {i.e., Al^'*' > Ga^"'" > In^'*' > Tl^'*') while those of +1 oxidation
states increases {i.e., Al'*’ < Ga"*" < In"*^ < Tr).

F lo
Since in case of Ga and In, +3 oxidation state is more stable than +1 oxidation state, therefore, both Ga'*'
and In'*' salts undergo disproportionation reactions in aqueous solution. Further, since Tl'*' salts are more
stable than Tl^'*’ salts, therefore, Tl^* salts act as strong oxidising agents.
9. Nature of compounds. Due to (i) small size, (I'O high sum of first three ionization enthalpies and {Hi) high

ee
electronegativity values, elements of group 13 mainly form covalent compounds in +3 oxidation state. For
example, (t) B always forms covalent compounds.

Fr
(//) Al mostly forms covalent compounds with little tendency to form ionic compounds.
For example, AICI3, AlBr3 and AII3 are all covalent but AIF3 and Al2(S04)3 are ionic in nature.

for
{Hi) Like Al, many compounds of other elements, i.e., GaCl3, InCl3, etc. are covalent when anhydrous.
However, Al, Ga, In and Tl all form metal ions in aqueous solutions due to their high hydration enthalpies
ur
which exceed their ionization enthalpies.
10. TWoxides. All the elements of group 13 form trioxides of the formula, when heated in dioxygen.
s
Besides, TI2O3, Tl also forms some TI2O.
ook
Yo
The reactivityof these elementstowards dioxygen, however,increases down the group. All these trioxides
react with water forming hydroxides. On moving down the group, the acid-base character of trioxides and
eB

their hydroxides changes gradually from acidic to amphoteric and then to basic as shown :
B2O3, A1^0g,Ga202 In .,03, TI2O,
B(0H)3 Al(6H)3,Ga(OH)3 In (0H)3 TlOH
our
ad

Acidic Amphoteric Basic Strongly basic


u. Formation of nitrides. When heated in dinitrogen at high temperatures, B and Al form nitrides but Ga, In
and Tl do not. The structure of boron nitride is similar to that of graphite and hence it is .sometimes called
as inorganic graphite.
Y

12. Action of acids. Boron does not react with non-oxidising acids such as HCl but reacts with hot oxidising
Re

acids such as a mixture of cone. H7SO4 and cone. HNO3 forming boric acid. All other elements react both
nd

with non-oxidising and oxidising acids liberating H2. However, with cone. HNO3, both Al and Ga become
passive.
Fi

13. Action of alkalies. Boron reacts with alkalies above 773 K but Al and Ga being amphoteric react with
aqueous alkalies on warming evolving H2 gas. In and Tl are, however, not affected by alkalies.
14. Reactionwith halogens,(i) All the elementsof group 13 react with halogensat high temperatureto form
trihalides of the general formula, MX3. All boron halides (BF3, BCI3, BBr3, BI3) are covalent. The fluorides
of Al, Ga, In and Tl are ionic while chlorides, bromides and iodides are largely covalent. The covalent
trihalides being electron deficient act as Lewis acids and form complexes with NH3, amines, alcohols,
ethers, etc. Their Lewis acid characte,r howeve,r decrea.ses down the group as the size of the halogen
increases, i.e., BX3 > AIX3 > GaX3 > 10X3.
(ii) The relative Lewis acid character of boron trihalides decreases in the order :
BI3 > BBr3 > BCI3 > BF3

I
11/48 New Course Chemistry (XI)SSSISS

This is due to pK-pK back bonding which decreases the electron deficiency of the B atom. Since this
tendency is maximum for F and decreases from F to /, therefore, is the strongest and BF^ is the
weakest Lewis acid.

(Hi) Although both B and At triholides act as Lewis acids but only aluminium trihalides (i.e., Al2Clf) exist
as dimers. This is due to the reason that boron atom is so .small that it cannot accommodate four large
sized halogen (except F) atoms around it.
15. Due to (0 small atomic and ionic size. (») high ionization enthalpy and (Hi) absence of t/-orbitals,
B differs from other elements of group 13. For example, due to absence of d-orbitals, boron trihalides
form only tetrahedral [B(OH)^]~ species while trihalides of Al and other elements due to the presence of
.?+
d-orbitals form octahedral species.

w
16. Borax, Na-,B4O7.10 H^O or Na2[B405(0H)4].8 H,0 is obtained from the mineral colemanile (Ca2B50jj)
or boric acid (H3BO3).
(i) Its aqueous solution is alkaline due to hydrolysis (//) On heating, it first swells up due to loss of H2O

F lo
and then melts into a clear liquid which solidifies to form a transparentglassy bead called borax bead. It
con.si.sts of sodium metaborate (NaB02) and boric anhydride (B2OJ). It is used in qualitative analysis for
detection of certain coloured basic radicals such as Ni^"*", Co-'*’, Cr^'*', Mn^'*’, etc. under the

ee
name borax bead test. (Hi) Borax or boric acid on healing with ethyl alcohol and cone. H2SO4, forms
vapours of iriethylboratewhich bum with a green edged flame. This reaction is used as a test for borate

Fr
ion in qualitative analysis.

17. Boric acid (H3BO3) contains triangular BO^” ions in which boron is ip'-hybridized. In solid state,

for
B(0H)3 molecules are H-bonded to fomt a two-dimensional .sheet. It is a weak monobasic acid. It does not
ur
act as a protonic acid but acts as a Lewis acid by accepting a pair of electrons from OH~ ion of water
thereby releasing a proton.
s
ook
H-^OH + B(0H)3 > [B(0H>4] + H''
Yo
18. Boron hydrides. Boron forms a number of covalent hydrides called boranes. These are mainly of two
eB

types:
(/) Nido-boranes, B„H„^4 such as B2H^ (diborane), (pentaborane-9 ), etc. and (ii) Arac/mo-boranes,
such as B4H,q (tetraborane), CgHji (pentaborane-11), etc. All boranes are electron-deficient
compounds.
our
ad

Diborane (B2H^) is formed when boron trifluoride etherate is reduced with LiAlH4, or NaBH4 is treated
with U- BoHg is a non-planar molecule in which each B atom is 5p^-hybridized. B2Hg has four normal
B-H covalent bonds, i.e., two-centre two-electron (2c - 2c) bonds which lie in the same plane and two
Y

bridge bonds, B H, ,B, i.e., three-centre two-electron bonds (3c - 2c) or banana bonds which lie
above and below the plane of the four B—H bonds.
Re
nd

19. Borazine or borazole. B2Hg reacts with NH3 to form an addition product which when heated to 473 K
decomposes to give a volatile compound called borazine or borazole. It is isoelecironic and isosteric with
Fi

benzene. That is why, it is also called inorganic benzene.


II. Elements of Gmup 14
20. Group 14 of the periodic table consists of six elements, i.e., carbon (C), silicon (Si), germanium (Ge), tin
(Sn), lead (Pb) and the recently discovered element, itnunquadium (Uuq) which is radioactive.
21. The outer electronic configuration of the elements of group 14 is ns^ np^ where n = 2- l.
22. The covalent radii of group 14 elements are smaller than those of the corresponding elements of group
13 due to increased ejfective nuclear charge.
23. The first ionization enthalpies of group 14 elements are higher than those of the corresponding elements
of group 13 due to greater nuclear charge. On moving down the group, the A,. Hj decreases from C to Sn.
However, A,H1 ofPb is slightly higherthan ihaiofSn,/.e., A,H| of group 14 elements follows the sequence :
C > Si > Ge > Sn < Pb.

1
SOME p-BLOCK ELEMENTS 11/49

24. Electronegative nature. The elements of group 14 are less electropositive or less metallic than the
correspniuling elements of group 13. On moving down the group, metallic character, however, increases.
For example, C is strictly non-metalUc, silicon is essentially a non-metal, germanium is a metalloid whereas
Sn and Pb are soft metals with low melting points.
25. Oxidation .states. Carbon and silicon both show an oxidation state of +4 while all other elements of this
group, ie„ Ge, Sn and Pb .show two oxidation states of+2 and +4 due to inert pair efTect. Whereas the
stability of +4 oxidation stales decreases from Ge to Pb, i.e., Ge'*'*' > Sn'*'*' > Pb'*'*' while the stability of +2
oxidation states increases from Ge to Pb, i.e., Ge^'*' < Sn^'*' < Pb^'*'. In other words. Pb salts are more stable
in +2 oxidation state than in +4 oxidation state, therefore, Pb (IV) salts such as Pb(OCOCH^ and PbOo

l ow
are widely used as oxidising agents. In contrast, Sn salts are more stable in +4 oxidation state than in +2
oxidation state, therefore, Sn (U) salts such as SnCl2 is widely used as a reducing agent.
26. Due to small size and high electronegativity, carbon formspn~pn multiple (C = C, C = O, C = N, C = C,
C s N. etc.) bonds but other elements do not.
27, Due to absence of ^/-orbitals, carbon does not form dK-pn bonds while silicon and other elements of
group 14 do. For example, due to dn-pK bonding, NlSiH^ )j is a weaker base than N(CH^)j.

ee
Further, due to absence of d-orbitals, carbon cannot expand its covalency beyond 4 while other elements

F
Fr
of this group due to the presence of vacant r/-orbitals can exhibit covalency of 5 and 6 and hence many
complex ions of these elements such as [SiF5]~, [SiF^]^", [GeCl^]^", [Sn(OH)g]^“, [Pb(OH)(^]-“ and
[PbCy^-, etc. are known.
Further, since six large sized Cl atoms cannot be accommodatedaround Si atom and also due to weak

for
ur
interaction of Cl lone pair of electrons with fr-orbitals of Si, [SiCl^]^" is unknown but [SiF^]^" is known.
28. Formation of oxides. All the elements of group 14 when heated in oxygen form monoxides, MO and
dioxides, M02- SiO exists only at high temperatures. Besides, monoxides and dioxides, Pb also forms red
s
lead, Pb304.
ok
Yo
(0 Acid-base character. Among dioxides, COj Si02 are acidic, Ge02 is less acidic than Si02 while
Sn02 and Pb02 are amphoteric. The monoxides, on the other hand, are less acidic than dioxides. For
o

example, CO is neutral, GeO is acidic while SnO and PbO are amphoteric.
eB

(«) Oxidising-reducing properties. Since +4 oxidation state of carbon is the most stable, therefore, among
monoxides of group 14 ; CO is (he strongest reducing agent. Conversely, due to inert pair effect, +2
oxidation state of Pb is the most stable, therefore, among dioxides of group 14, Pb02 is a powerful
r

oxidising agent and thus oxidises HCl to CI2 and reacts with cone. HNO^ or H2 SO^ to evolve O2 gas.
ou
ad

29. Action of water. Carbon, silicon and germanium do not decompose water at all. iTn decomposes steam to
form tin dioxide with evolution of H2 gas. Lead does not react with water probably due to the formation of
Y

a protective iflm of lead oxide on its surface.


30. Formation of halides. All the elements of group 14 form halides of the formula MX4 and MX2 (X = F, Cl,
nd

Br, I).
Re

(«) Tetrahalides. (/) SnF^ and PbF^ are ionic while all other tetrahalides are covalent in nature. They have
tetrahedral structures since the central atom in them is ^p^-hybridized.
Fi

(//) The thermal stability of these tetrahalides decreases as the size of the halogen increases. Thus, PbCl4
is stable, PbBr4 is unstable while Pbl4 is unknown.
(Hi) CCI4 is not hydrolysed by water since carbon does not contain d-orbitals. The tetrahalides of remaining
elements are easily hydrolysed due to the presence of vacant r/-orbitals in their central atoms,
(jv) Due to the absence of rZ-orbitals, the tetrahalides of carbon do not act as Lewis acids while the
tetrahalides of Si, Ge, Sn and Pb behave as Lewis acids due to the presence of vacant </-orbitals in their
central atoms.

(b) Dihalides. Except C and Si, all other elements (Ge, Sn and Pb) form dihalidcs. The stability of these
dihalides increases from Ge to Pb as the inert pair effect becomes more and more prominent down the
group, i.e., thermal stability increases in the order: GeX^ « SnX2 < PbX2.
11/50 'pfuidee^ <1 New Course Chemistry (XI) LVWTWI'I

31. Due to (0 small atomic size, (//) high electronegativity {Hi) high ionization enthalpy and (iv) absence of
J-orbitals, carbon differs from rest of the elements of group 14.
32. Carbon shows catenation to the maximum extent since C - C bond enthalpy is very high (348 kJ mol'*).
However, as we move down the group, the element-element bond enthalpies decrease rapidly, therefore,
their tendency for catenation decreases in the order: C » Si > Ge « Sn » Pb.

33. Allotropic forms. Due to the property of catenation and pn-pn bond formation, carbon shows allotropy
and exists in four allotropic forms.
(0 Diamond is a three-dimensional network solid in which carbon is jp^-hybridized. Each carbon is

w
tetrahedrally linked to four other carbon atoms through strong C - C, sp^-sp^, a-bonds.
(ji) Graphite has a two dimensional sheet-like (layered) structure consisting of a number of benzene rings
fused togethe.r The various sheets or layers are held together by weak van der Waals forces of attraction.
Each carbon in graphite is sp^-hybridized.

Flo
(Hi) Fullerenes constitute a new family of carbon allotropes consisting of spheroidal molecules of
composition. C2„ (where n > 30). Of all the fullerenes, allotrope is the most stable. It looks like a
soccer ball and hence is called bucky ball. It consists of 20 six-membered irngs and 12 five-membered

e
rings. Six membered irngs are fused to both five and six-membered rings but five-membered rings are

re
fused only to six-membered irngs,

F
(iv) Carbon nanotube consists of a graphite layer rolled into a cylinder and capped at each end with half
of a CgQ fullerene. Carbon nanotubes are very tough, about 100 times as strong as steel.
34. Silica. Silicon dioxide is commonly called silica. It is a covalent, three-dimensional network solid in
ur
or
which each silicon atom is bonded to four oxygen atoms which are arranged tetrahedrally around it and
each oxygen atom is attached to two silicon atoms by covalent bonds.

f
35. Silicones are synthetic organosilicon polymers containing repeated R2SiO units held by S-O-Si bonds.
ks
These are prepared by hydrolysis of alkyl or aryl substituted chlorosilanes and their subsequent
polymerization. For example, hydrolysis of dichlorodimethylsilane followed by polymerization yields
Yo
straight chain silicone polymer. The chain length of these polymers, can, however, be controlled by adding
oo
(CH3)3SiCl which blocks the ends. Silicone polymers are stable towards heat and are chemically inert and
are good insulators. Therefore, they are used for making water proof papers, wool, textiles, wood, etc.
B

They are also used as lubricants and in surgical and cosmetic implants.
36. Silicates are the compounds in which the anions present are discrete SiO^ tetrahedra or a number of
re

such units joined together through comers. These are of the following six types :
SiO^" tetrahedron, e.g., zircon [ZrSi04J.
(0 Pyrosilicates - contain single discrete unit of
u
ad

(«) Pyrosilicates - contain Si20^“ anion formed by joining two SiO^’ tetrahedra through one comer
Yo

{i.e., oxygen atom), e.g., thortveitite, 802(81207).


(ki) Cyclic or ring silicates - are obtained when two oxygen atoms per tetrahedron share to form closed

rings with general formula (SiOl")^, e.g., beryl, Be3Al2SigOjg.


d
Re

(rv) Chain silicates - are obtained by sharing two oxygen atoms of each tetrahedron in such a way that
in

linear single strand chain of general formula (SiO^“)^ is formed, e.g., spodumene, LiAI(8i03)2.
F

(v) Sheet silicates - are obtained by sharing three oxygen atoms of each tetrahedron. General formula,
(Si20|“)^, e.g., Kaolinite, [Al(0H)4Si205].
(vi) Three-dimensional silicates - obtained by sharing all the four comers of each tetrahedron. For example,
quartz and tridymite.
37. Zeolites. The three-dimensional silicates in which some of the Si atoms are replaced by Al^'*' ions and the
negative charge is balanced by cations such as Na"*", K'*’, Ca^'*’, etc. are called feldspar (KAlSi30g) and
zeolites (NaAlSi20g.H20).
Zeolites are widely used as catalysts in petrochemical industries for cracking of hydrocarbons and
isomerization. Another zeolite called ZSM-5 is used to convert alcohols directly into gasoline. Hydrated
zeolites called permutit are used as ion exchangers for softening of hard water.
SOME p-BLOCK ELEMENTS 11^1

neet/jee:
SPECIAL

w
Fprultiniate preparation of this unit fpr competitive examinations, students should refer to

F lo
● MCQs in Chemistry for NEET
- C rim
Pradeep's Stellar Series.'... ● MCQs in Chemistry for JEE (Main)
f:
- ^lili^'ii separately available for these examinations.

ee
Multiple Choice Questions correct Answer)

Fr
(fl)Al (b) Ga

for
I. Group 13 Elements (c) Sc (^In
ur
1. Which one of the following elements is unable to (JEE Main 2022)

form MFg“ ion ? 6. BF3 is planar and electron deficient. Hybridization


and number of electrons around the centra! atom,
s
(a) Ga ib) A1
ook
respectively, are :
Yo
(c) B (d) In (NEET 2018)
(a) sp^ and 8 (b) sp^ and 4
2. The increasing order of atomic radii of the (c) sp^ and 6 (d) sp^ and 6
eB

following group 13 elements is


(NEET 2021)
(a) A1 < Ga < In < T1 (b) Ga < A1 < In < Ti
7. Correct statement about B2Hg is
(c) AI < In < Ga < Tl (i/) A1 < Ga < Tl < In
{a) BH3 is a Lewis acid
our
ad

(JEE Advanced 2016)


(b) TerminalH has more p-characlerthan bridge H
3. The correct order of atomic radii in group 13
elements is (c) All B-H bonds are of equal lengths
(a) B < Al < In < Ga < Tl id) Bond angle B-H-B is 120°
Y

(b) B < AI < Ga < In < Tl (JEE Main 2021)


Re

8. Which of the following statement is not correct


nd

(c) B < Ga < Al < Tl < In


about diborane ?
(iA B < Ga < Al < In < Tl (NEET 2018)
Fi

(a) There are two 3-centre-2-electron bonds.


4. The stability of + 1 oxidation state among Al, Ga,
In and Tl increases in the sequence (b) The four terminal B-H bonds are two centre
two electron bonds,
(a) Al < Ga < In < Tl (b) Tl < In < Ga < Al
(c) In < Tl < Ga < Al id) Ga < In < Al < Tl
(c) The terminal hydrogen atoms and the two
boron atoms lie in the same plane.
(AIPMT zU15, JEE Main 2022)
(d) All the boron atoms are .yp^-hybridized.
5. The metal has very low melting point and its
(NEET 2022)
periodic position is close to a metalloid, is

ANSWER!

1. (c) 2. ib) 3. id) 4. ia) 5. ih) 6. id) 7. (a) 8. id)


11/52 New Course Chemistry (X1)B&19D

9. Identify the correct statement for B2Hg from those (c) C2H6 and C2H5Na
given below : (tO CgHg and NaBH4 (Karnataka CET 2015)
I. In B2Hg all B-H bonds are equivalent. 14. The correct statement is
II. In B2Hg there are four 3-centre-2-electron (a) BI3 is the weakest Lewis acid among the boton
bonds. halides

III. BoHg is a Lewis acid. {b) there is minimum pn-pK back bonding in BF3
IV. B2H^ can be synthesized from BF3 and NaBH4- (c) BF3 is the strongest Lewis acid among the other
V. B2Hg is a planar molecule. boron halides

Choose the most appropriate answer from the id) there is maximum pn-pn back bonding in BF3
options given below : (Karnataka CET 2015)

w
(rt) (I) and (V) only ib) ill),(m) and(V)only 15. Which of the following are Lewis acids ?
(c) (III) and (IV) only id) (III) and (V) only ia) PH3 and BCI3 (b) AICI3 and SiCI^

F lo
(JEE Main 2022) (c) PH3 and S1CI4 id) BCl3andAlCl3
10. Borazine, also known as inorganic benzene, can (JEE Main 2018)
be prepared by the action of 3 equivalents of ‘X’ 16. Which of the following is a Lewis acid ?
and 6 equivalents of ‘Y’. ‘X’ and ‘Y’ respectively

ee
are:
ia) NaH ib) NF3
ic) PH3 (^0 B(CH3>3

Fr
ia) B(0H)3 and NH3 ib) BjHg and NH3
[JEE Main (online) 2018]
ic) B2H5 and HN3 id) NH3 and B2O3
17. The bond dissociation energy of B—F in BF3 is

for
(JEE Main 2022)
646 kJ moi“* whereas that of C—F in CF4
11. The reaction of H3N3B3CI3 (A) with LiBH4 in is 515 kJ mol“*. The correct reason for higher
r
tetrahydrofurangives inorganic benzene (B). B—F bond dissociation energy as compared to that
Further, the reaction of (A) with (C) leads to
You
ofC—Fis
s
H3N3B3(Me)3. Compounds (B) and (C)
ia) stronger a-bond between B and F in BF3 as
ook

respectively, are
compared to that between C and F in CF4
ia) boron nitride and MeBr
ib) significantpn-pn interactionbetween B and F
eB

ib) borazine and MeMgBr


in BF3 whereas there is no possibility of such
ic) borazine and MeBr interaction between C and F in CF4
(d) diborane and MeMgBr (JEE Main 2020) (c) lower degree of pn-pn interaction between B
our
ad

12. The geometry around boron in the product ‘B’ and F in BF3 than that between C and F in
formed from the following reaction, is : CF4
450 K id) smaller size of B-atom as compared to that of
BF3 + NaH — ^ A + NaF C-atom (AIEEE 2009)
dY

A + NMc3 ^ B
18. 2 B(0H)3 + 2 NaOH ^ - NaB02
Re

(a) Trigonal planar ib) Tetrahedral


+ Na[B(OH)4] + 2 H2O
(c) Pyramidal id) Square planar
Fin

How can this reaction be made to proceed in


(JEE Main 2022) forward direction ?

13. An alkali metal hydride (NaH) reacts with diborane ia) addition of dj-1, 2-diol
in ‘A’ to give a tetrahedral complex ‘B’ which is ib) addition of borax
extensively used as reducing agent in organic
(c) addition of trans-1, 2-diol
synthesis. The compounds ‘A’ and ‘B’ respectively
are id) addition of Na2HP04
19. In the reaction,
ia) CH3COCH3 and B3N3H6
ib) (C2H5)20 and NaBH4 B(0H)3 2 H2O ■>
[B(0H)4]- H30-^
ISWERS

9. ic) 10. (h) 11. ih) 12.th) 13. ib) 14.id) 15. id) 16.id) 17. ib) 18. ia)
SOME p>BLOCK ELEMENTS 11/53

B(0H)3 functions as soluble in excess of NaOH. Compound ‘X’ when


(a) protonic acid (b) lewis acid heated strongly gives an oxide which is used in
(c) bronsted acid (d) lewis base chromatography as an adsorbent. Then metal ‘M’
IS
(Karnataka CET 2019)
(a) Zn ib) Ca
20. Boric acid is an acid because its molecule
(c) A1 (d) Fe (JEE Main 2018)
(a) contains replacement H'*' ion
28. AIF3 is soluble in HF only in presence of KF. It is
(b) gives up a proton due to the formation of
(c) accepts OH" from water releasing proton
(a) K3[A1F3H3] ib) K3[AIF6]
(d) combines with proton from water molecule
(c) AIH3 (r/) K1IA1F3H]

w
(NEET Phase-ll 2016)
(NEET Phase-II 2016)
21. Addition of mineral acid to an aqueous solution
of borax, the following compound is formed :
29. AI2O3 becomes anhydrous AICI3 upon heating

F lo
(a) with NaCl (b) with dry CI2 and C
(a) boron hydride (b) pyroboric acid
(c) metaboric acid (d) orthoboric acid
(c) with CI2 (d) with dry HCl gas
(Bihar CECE 2015)
(Karnataka CET 2017)
30. The structural formula of lithium tetrahydrido-

ee
22. Which one of the following is called Inorganic aluminate is

Fr
graphite ?
(a)B
(a) Al[LiH4l (b) Al2[LiH4]3
(b) B4C
(c) LiLAlH^J (d) U[AIH4]2
(c) B2H6 {d) BN

for
(AIIMS 2014)
(Bihar CECE 2015)
23. An aqueous solution of which of the following II. Group 14 Elements
ur
boron compounds will be strongly basic its 31. The element that does not show catenation is
nature ?
s
(a) Sn (b) Ge
ook
(a) NaBH4 ib) LiBH4
Yo
(c) Si (d) Pb
(c) B2H6 (d) Na2B40y
(JEE Main 2019)
eB

(JEE Main 2022)


32. Which of the following oxide is amphoteric in
24. When borax is heated with CoO on a platinum nature ?
loop, blue coloured bead formed is largely due to
ia) CO2 (b) Sn02
our

ia) B2O3 ib) Co(B02)2


ad

(c) Si02 (d) GeO-5


(c) C0B4O7 (d) Co[B405(OH)4] (NEET 2020)
(JEE Main 2022)
33. Which of the following groups contains both
25. The green colour produced in the borax bead test acidic oxides.
dY

of a chromium (III) salt is due to


ia) N2O, BaO ib) CaO, Si02
Re

ia) CrB ib) Cr203 (c) B2O3, Si02 id) B2O3, CaO
(c) Cr(B02)3 id) Cr2(B407)3
Fin

(JEE Main 2021)


(JEE Advanced 2019)
34. It is because of inability of ns^ electrons of the
26. Corundum is ... mineral of aluminium valence shell to participate in bonding that
ia) silicate ib) oxide (a) Sn^'*' is oxidising while Pb'*'*’ is reducing
(c) double salt id) sulphate ib) Sn^'*' and Pb^'*' are both oxidising and reducing
(e) nitrate (Kerala PET 2017) (c) Sn'^'^ is reducing while Pb'^'*' is oxidising
27. When metal ‘M' is treated with NaOH, a white id) Sn^'*' is reducing while Pb'*'*’ is oxidising
gelatinous precipitate ‘X’ is obtained which is (NEET 2017)

19.[b) 20. (c) 21. id) 22.id) 23.id) 24. (h) 25.(cj 26.ib) 27.(c) 28. ib)
29. (/;) 30. (c) 31. id) 32. (b) 33. (c) 34. id)
11/54 New Course Chemistry (XI)CEIBI]

35. Which of the following is incorrect statement ? (c) five membered rings are fused only to six-
membered irngs
(fl)
Snp4 is ionic in nature
(b)Pbp4 is a covalent compound (d) six-membered irngs are fused to both six and
five-membered r-ngs. (JEE Main 2021)
(c)SiCl4 is easily hydrolyed
41. How many 0-atoms are shared per Si04 tetrahedra
(d)00X4 (X = F, Cl, Br, 1) is more stable than
(NEET 2019)
in silicate anion of beryl mineral ?
GeX2
(a) 4 (b)3
36. When Pb02 reacts with concentrated HNO3, the
(c)2 id) I
gas evolved is
42. Under hydrolytic conditions, the compounds used
(a) NO2 (b) O2
for preparation of linear polymer and for chain
(c) N2 id) N2O termination respectively are

w
(Karnataka CET 2018)
(a) CH3SiCl3 and Si(CH3>4
37. PbCU is insoluble in cold water. Addition of HCl
(b) (CH3)2SiCl2 and (CH3)3SiCl

F lo
increases its solubility due to
(a) formation of soluble complex anions like (c) (CH3)2SiCl2 and CH3SiCl3
[PbCl3]- (d) SiCl4 and (CH3)3SiCl (JEE Advanced 2015)
ib) oxidation of Pb(II) to Pb (IV) 43. (CH3)3SiCl is used during polymerisation of
organosilicones because

e
2+
(c) formation of [PbfH^O)^]

Fre
(a) the chain length of organosilicon polymers can
id) formadon of polymeric lead complexes
be controlled by adding (CH3)3SiCl
(West Bengal JEE 2017)
(b) (CH3)3SiCl improves the quality and yield of

for
38. The hybridization of C in diamond, graphite and the polymer
ethyne is in the order
(c) (CH3)3SiCl does not block the end terminal
ia) sp^, sp, sp~ ib) sp^, sp^, sp
r
of silicone polymer
(c) sp, sp^, sp^ id) sp^, sp^, sp
You
{d) (CH3)3SiCl acts as a catalyst during
oks
(Karnataka CET 2016) polymerisation (Karnataka CET 2018)
39. In graphite and diamond, the percentage of p- 44. Identify the correct statements from the following :
eBo

characters of the hybrid orbitals in hybridisation


(I) CO2 (g) is used as refrigerant for ice cream and
are respectively frozen food
ia) 33 and 75 ib) 50 and 75
(II) The structure of C^q contains twelve six carbon
(c) 33 and 25 id) 67 and 75 rings and twenty five carbon irngs
our
ad

[JEE Main (online) 2018] (III) ZSM-5, a type of zeolite, is used to convert
40. Incorrect statement regarding the structure of Cgg alcohols into gasoline
IS
(IV) CO is colourless and odourless gas
ia) it contains 12 six-membered rings and 24 five (a) (I) and (III) only ib) (II) and (HI) only
Y
Re

membered irngs (c) (III) and (IV) only id) (I), (II) and (III) only
ib) each carbon atom forms three a-bonds
d

(NEET 2020)
Fin

m Multiple Choice Questions (with One or More than One Correct Answers)

45. Which of the following compounds undergo 46. Which of the following are amphoteric oxides ?
disproportionation in aqueous soludon ?
(a) Sn02 ib) AI2O3
(fl) TICI3 ib) GaCl
(c) Ga203 id) Si02
(c) InCl id) TlCl

35. (/;) 36. (/;) 37. («) 38.ih) 39. id) 40.(n) 41. (c)
42. {/;) 43. (ri) 44.1 45. (h.c) 46. (a.h.c)
SOME p-BLOCK ELEMENTS 11/55

47. The option(s) with only amphoteric oxides is(are) (c) it has a three-dimensional structure due to
(a) Cf203, BeO, SnO, SnO^ hydrogen bonding
(b) ZnO, AI2O3, PbO, Pb02 (d) it is a weak electrolyte in water
(c) NO. B2O3, PbO, Sn02 (JEE Advanced 2014)

((/) Cr203, CrO, SnO, PbO 52. Which of the following species are not known ?
(JEE Advanced 2017) (a) [SiCl^]^- (b) [CFfi]^-
48. Among the following, the correct statement(s) (c) [PbC\ff- (d) [SiFfi]^-
is(are) 53. Which of the following are used as catalyst in

w
Friedel-Crafts reactions ?
(a) A1(CH3)3 has the three-centre two-electron
bonds in its dimeric structure
(a) AICI3 (b) SiCl4
(b) BH3 has the three-centre two-electron bonds (c) BF3 (d) SnCl4

F lo
in its dimeric structure
54. Substances which readily undergo hydrolysis are
(c) the Lewis acidity of BCI3 is greater than that (a) AICI3 (b) BCI3
of AICI3

ee
(c) SiCl 4 (d) PbCl4
(d) AICI3 has the three-centre two-electron bonds
55. With respect to graphite and diamond, which of

Fr
in its dimeric structure.
the statement(s)given is (are) correct ?
(JEE Advanced 2017)
(a) Graphite is harder than diamond

for
49. The crystalline form of borax has
(b) Graphite has higher electrical conductivity
(a) letranuclear [6405(011)4]^" unit
ur
than diamond
(b) all boron atoms in the same plane (c) Graphite has higher thermal conductivity than
(c) equal number of sp^ and sp^ hybridized boron diamond
s
ook
atoms
Yo
(d) Graphite has higher C-C bond order than
(d) one terminal hydroxide per boron atom diamond. (IIT Paper II, 2012)
eB

(JEE Advanced 2016) 56. A tin chloride ‘Q’ undergoes the following
50. Boric acid is used in carom boards for smooth reactions (not balanced)
gliding of pawns because Q -K Cl- ^ X
r

(a) H3BO3 molecules are loosely chemically Q + Mb3N ^ Y


ad
ou

bonded and hence soft


Q -f- CUCI2 ^ Z + CuCl
(b) its low density makes it fluffy
X is mono anion having pyramidal geometry.
(c) it can be powdered to a very small grain size
Y

Both Y and Z are neutral molecules. Choose the


(d) it is chemically inert with the plywood correct option(s)
Re
nd

(e) H-bonding in H3BO3 gives it a layered (a) The central atom in X is 5p^-hybridized
structure. (Kerala PMT 2006)
(b) There is a coordinate bond in Y
Fi

51. The correct statement(s) for orthoboric acid is/are (c) The oxidation state of the central atom in Z is
(a) it behaves as a weak acid in water due to self +2
ionization
(d) The central atom in Z has one lone pair of
(/?) acidity of its aqueous solution increases upon electrons (JEE Advanced 2019)
addition of ethylene glycol

ANSWERS
47. {a.b) 48. ia.b.c) 49. (a.c.d) 50. (e) 51. (h.d) 52. ia.b) 53. (a.c.d)
54. {a.b.c.d) 55. (h,d) 56. (a.b)
11/56 New Course Chemistry (XI)E2SXM1]

DU Multiple Choice Questions (Based on the given Passage/Compreh ension)


Each comprehension given below is followed by some multiple choice questions. Each question has one
correct option. Choose the correct option.
58. The strongest reductant among the following is
IcjorTipTerTension^^ The heavier elements of (a) GeCl2 {b) SnCU
groups 13 and 14 besides the group oxidation (c) PbClj (d) S11CI4
state exhibit another oxidation state which is 59. Elements of group 13
two units lower than the group oxidation state (a) exhibit oxidation state of +3 only
and the stability of this lower oxidation state {b) form M'*' and ions

w
increases down the group. This concept which (c) exhibit oxidation states of + 1 and +3
is commonly called inert pair effect has been {d) form M " and M ions
used to explain many physical and chemical 60. Which of the following statements is incorrect ?

F lo
properties of the elements of these groups.
(a) CO is used as a reducing agent
57. Which among the following is the strongest (b) T1 (III) salts undergo disproportionation
oxidising agent ? (c) CO2 is a greenhouse gas

ee
(d) SiOo is a covalent solid
(o) Sn02 (b) S1O2
61. The oxide which is the strongest acid is

Fr
(c) Ge02 (d) Pb02
(a)Tl203 (/j)Pb02 (c)C02 {</) Sn02

for
m Matching Type Questions
r
Match the entries of column I with appropriate entries of column II and choose the correct option out
of the four options (a), (b), (c), (d) given at the end of each question.
You
s
62. Column I Column II
ook

(A) Cs (P) High temperature thermometer


(B) Ga (<?) Water-repellent sprays
eB

(C) B (r) Photoelectric cells

(D) Si is) Bullet-proof vests


{a) A-r, B-p, C-.9, D~q {b) A-r, C-^, D-p (c) A-q, B-r, C-s, D-p (d) A-p, B-5, C-q, D-r
our
ad

(JEE Main 2022)


63. Column I Column II
(A) Shows inert pair effect ip) InCl
Y

(B) Shows pn - pn back bonding (^) Ga


Re

(C) Shows disproportionation reaction (r) N(SiH3)3


d

(D) Shows pn - dn back bonding (j) BF3


Fin

(a) A-p, B-r, C-q, D-5 (i?) A-r, B-p, C-q, D-5 (c) A-q, B-i, C-p, D-r (d) As, B-q, C-p, D-r
64. Column 1 Column II
(A) Inorganic benzene (p) An allotrope of carbon
(B) Fullerene iq) Orthosilicate
(C) Phenacile (r) An ore of boron
(D) Colemanite is) Borazine (B3N3H^)
(a) As, B-p, C-r, D-^ (b) A-p, B-a‘, C-q, D-r (c) A-q, B-r, C-5, D-p (d) As, B-p, C-q, D-r

-iJ
57. id) 58. (0) 59. (c) 60. (b) 61. (c) 62. ((/) 63. (c) 64. id)
SOME p’BLOCK ELEMENTS 11/57

Matrix-Match Type Questions

Match the entries of column I with appropriate entries of column II. Each entry in column I may have
one or more than one correct option from column II. If the correct matches are \-p, s ; B-r ;
C-p, q ; D-s, then the correctly bubbled 4x4 matrix should be as follows :
p q

A ©@O ©
B
©©o ©

w
c
©o ©
D @©o ©

F lo
65. Column I Column II

(A) Bi^-^ > (BiO)-*- (P) Heat

ee
(B) [AIO2] ■ > A1(0H)3 (*?) Hydrolysis

Fr
(C) SiO^- > Si20^“ (r) Acidification

(D) > [B(0H)3] is) Dilution by water (IIT 2006)

for
ur
66. Column 1 Column II

(A) B2H6 (P) Borax

(B) BF3 Lewis Acid


s
ok
(C) AICI3 (r) pK - pK back bonding
Yo
(D) H3BO3 is) NaBH4
o
eB

A B C D
VI. Integer Type Questions
® © ©©
r

©©00
ad
ou

DIRECTIONS. The answer to each of the following questions is a single


digit integer, ranging from 0 to 9. If the correct answers to the question
numbers A, B, C and D (say) are 4, 0, 9 and 2 respectively, then the ©©®®
Y

correct darkening of bubbles should be as shown on the side : @®®®


67. Number of B-O-B bonds in borax is
0 0 00
Re
nd

68. Three moles of B2Hg are completely reacted with methanol. The number of
moles of boron containing product formed is (JEE Advanced 2015) @000
Fi

69. How many crystalline allotropic forms of carbon arc known ?


0000
70. Amongst the following, the maximum number of compounds which do not
behave as Lewis acids are : ©©©©
SnCl2, H3BO3, AICI3, CF4, Sip4, SiCl4, CCI4, BF3, SnCl4 @000
71. Total number of elements of group 13 and 14 which form basic oxides is
@000
72. How many silicon atoms are present in the anion of a pyrosilicates ?

ANSWERS

65. {A-q ; B-s ; C-p,r ; D-q.r) 66. (/4-.V ,● B-q,r ; C-q ; D-p,q) 67. (5)
68. (6) 69. (4) 70. (3) 71. (1) 72.(2)
11/58 'Pn^tdee^'or New Course Chemistry rxi'^rosrwm

VII.
Numerical Value Type Questions (in Decimal Notation)
For the following question, enter the correct numerical value, (in decimal-notation, truncated/rounded-off
to the second decimal place, e.g., 6-25, 7-00, - 0*33, - *30, 30*27, - 127*30) using the mouse and the on
screen virtual numeric keypad in the place designated to enter the answer.
73. 27 g of AI was treated with NaOH solution when a white gelatinous precipitate was obtained which upon
strong heating gave an oxide. The amount of oxide (in g) is

VIII.
Assertion-Reason Type Questions

w
TYPE I

DIRECTIONS. The questions given below contain Statcnient-1 (Assertion) and Statement-2 (Reason).
Each question has four choices (a), (b), (c), (d) out of which ONLY ONE is correct. Choose the correct

F lo
option as under,
(a) Statement-1 is True, Statement-2 is True ; Statement-2 is a correct explanation for Statement-!,
(b) Statement-! is True, Statement-2 is True ; Statement-2 is not a correct explanation for Statement-!,

ee
(c) Statement-! is True, Statement-2 is False,

Fr
(d) Statement-! is False, Statement-2 is True.
74. Statement 1. Boron always forms covalent bonds.
Statement 2. The small size of favours formation of covalent bond. (IIT Paper U, 2007)

for
75. Statement-!. Aluminium acts as an oxidising agent.
ur
Statement-2. Aluminium has a strong affinity for oxygen.
76. Statement 1. In water, orthoboric acid behaves as a weak monobasic acid.

Statement 2. In water, orthoboric acid acts as a proton donor.


oks
(IIT Paper II, 2007)
77.
Statement-!. (SiH3)3N is a weaker base than (CH3)3N.
Yo
Statement-2. Due \apK-dK back bonding the availability of electrons on the N atom in (SiH3)3N decreases.
o

Statement I. Pb‘^'*’ compounds are stronger oxidizing agents than Sn'*'*’ compounds.
eB

78.

Statement 2. The higher oxidation states for the group 14 elements are more stable for the heavier members
of the group due to inert pair effect. (DT Paper I, 2008)
79. Statement-!. CO2 is a gas but Si02 is a solid at room temperature.
our
ad

Statement-2. CO2 contains C = O bonds but Si02 has a three-directional network structure.
80.
Statement-!. Boron is unable to form BFI".
Y

Statement-2. Size of boron is very small. (JEE Main 2022)


81. Statement-!. Boric acid is a weak acid.
Re
nd

Statement-2. Boric acid is not able to release ion on its own. It receives OH“ from water and releases H'*’
ion. (JEE Main 2022)
Fi

TYPE II

DIRECTIONS. In each of the following questions, a statement of Assertion (A) is given followed by a
corresponding statement of Reason (R) just below it. Of the statements, mark the correct answer as
(a) If both assertion and reason are true, and reason is the true explanation of the assertion,
(b) If both assertion and reason are true, but reason is not the true explanation of the assertion,
(c) If assertion is true, but reason is false,
(d) If both assertion and reason are false.
ANSWERS

73.(51 0) 74. (fl) 75. (</) 76. (c) 77. («) 78. (r) 79. (a) 80. (ft)
81. («)
SOME p-BLOCK ELEMENTS 11/59

82. Assertion. A1 and Ga have nearly same atomic radii.


Reason. AI and Ga show diagonal relationship. (AlIMS 2010)
83. Assertion. Boron always forms covalent compounds.
Reason. The small size of favours formation of covalent bonds. (AIIMS 2014)
84. Assertion. Coloured cations can be identified by borax bead test.
Reason. Transparent bead (NaBO-> + B-)03) forms coloured bead with coloured cation. (AIIMS 2009)
85. Assertion. Borax bead lest is not suitable for Al (III).

Reason. AI2O3 is insoluble in H2O,

w
86. Assertion. SiF^~ is known but SiCl^ is not.
Reason. Size of F is small and its lone pair of electrons interact with J-orbitals of Si strongly.

Flo
87. Assertion. PbCl2 is more stable than PbCl4.
Reason. PbCl4 is a powerful oxidising agent. (AIIMS 2008)

e
88. Assertion. Pbl4 is a stable compound.

re
Reason. Iodide ion stabilizes higher oxidation state.
89. Assertion. Snl^ is an orange solid.

rF
Reason. The colour arises due to charge transfer. (AIIMS 2007)
90. Assertion. Graphite is a good conductor of heat and electricity.
ur
Reason. Graphite has all the electrons firmly held in C-C, a-bonds. (AIIMS 2015)

fo
91. Assertion. C 60 fullerene is an allotrope of carbon.

Reason. In fullerene, five membered rings are isolated from each other.
92. Assertion. Carbon monoxide is highly toxic.
ks
Yo
Reason. Carbon monoxide forms a stable complex with haemoglobin present in red blood cells.
oo
93. Assertion. Silicones are hydrophobic in nature.
(AIIMS 2006)
eB

Reason. Si-O-Si linkages are moisture sensitive.


ur

For Difficult Questions


ad
Yo

Multiple Choice Questions (with one correct Answer)

2. Due to ineffective shielding of 3r/-orbitals, the In other words, stability of + I oxidation state
effective nuclear charge of Ga is greater in increases down the group, i.e., Al < Ga < In < Tl.
nd
Re

magnitude than of Al. As a result, atomic radii of 5. Ga has very low m.p. (303 K) and its periodic
Ga is lower than that of AI. The atomic radii of position is close to a metalloid (Ge, Z = 32).
Fi

remaining elements increase regularly as we move 6. In BF3, B is A7;--hybridized and has 6 (3 x 2)


down the group from In to TI. Thus, atomic radii electrons around it.
of the four elements from Al to Tl increase in the
7. BH3 is a Lewis acid ; BH3 + H“ > [BH4]“
order: Ga < Al < In <TI, i.e., option (h) is correct.
8. Both the B atoms in B2H(^ are ^/j^-hybridized.
3. Refer to Art. 11.6, point 1, “Atomic and ionic Therefore, option (d) is incorrect while all other
radii,” page 11/6. options are correct.
4. As we move down the group from Al to Tl. inert 9. B^H^ is a Lewis acid and it can be synthesized
pair effect becomes more and more pronounced. from BF3 and NaBH4.
ANSWERS
82.(c) 83.(«) 84.(rt) 85.(b) 86.(a) 87. («) 88.(d) 89.(a) 90.(c) 91.(h)
92.(a) 93.(c)
11/60 “PtixeUe^ New Course Chemistry (XI)

17. Due to identical sizes of 2/>-orbitals on B and F,


For Difficult Questions
back donation of a lone pair of electron from the
filled 2p-orbital of F to the empty 2p-orbital of B
450 K
occurs to a considerable extent. As a result, B—F
2Bp3 + 6NaBH4 ^ 4 BoH^ + 6 NaF
bond has some double bond character (refer to Fig.
Thus, option (c) is correct. 11.6, page 11/13) and hence bond dissociation
4 3 [BH2(NH3)2]-^ [BH4J- energy of B—F bond in BF3 is much higher than
10. 3B2H(.+6NH3 that of C—F bond in CF4. The reason being that
X Y
C does not have an empty 2p-orbital and hence
473 K back donation does not occur.
4 2B3N3H^+12H2 18. B(0H)3 reacts with NaOH to form sodium
Borazine metaborate.

w
THF

11. B3N3H3CI3 +UBH4 ^ B3N3Hg +LiH B(0H)3 + NaOH ^ Na-"B0- + 2H20


(A) (B)

F lo
The end point during the titration of H3BO3 with
+ BCI3 NaOH is not sharp since NaB02 undergoes
B3N3H3CI3 + 3 MeMgBr 4 B3N3H3(Me>3 excessive hydrolysis to give back H3BO3 and
NaOH
(A) (C)
+ 3 MgBrCl

ee
NaB02 + 2 H2O ^ 4 B(0H)3 + NaOH
Thus, (B) is borazine and (C) is MeMgBr, i.e.,

Fr
option {b) is correct. However, when certain polyhydroxy compounds
450 K
such as catechol, glycerol, mannitol or sugars or
any other c/5-1.2-diol is added to the titration
12. 2Bp3 + 6NaH 4
B^H^ + 6 NaF

for
solution, the metaborate ion combines with
DiboraiielA)
polyhydroxy compound to form a complex.
ur
NMC3 OH

+ BO2
s
2 H20
^2^6 +2NMe 3 4 2
//\
ook
OH
Yo
A
H H Catechol
H
eB

B
O
Tetrahedral

Geometry around B is tetrahedral. Thus, option Q B


O
(b) is correct.
our
ad

Complex
(C2H5 >20 (A)
13. 2NaH + B2H6 4 Due to the formation of this complex, BO2 ion
Diethyl ether
Sod. hydride does not undergo hydrolysis. As a result, boric acid
Y

2 Na-^ [BH4]- behaves as a strong monobasic acid and the end


Re

Sod. borohydride (B) point can thus be easily detected.


nd

{Tetrahedral complex)
14. There is maximum pn-pn back bonding in BF3 Thus, the above acid-base titration can be made
Fi

due to identical sizes of 2p-orbitals of B and F. to proceed in the forward direction by carrying
15. BCI3 and AICI3 iU'e Lewis acids. out the titration in presence of c«-l, 2-diol, i.e.,
option (a) is correct.
16. B(CH3)3 has only six electrons in the valence shell.
Therefore, it can accept a pair of electrons and 19. Due to small size of B and presence of only six
hence acts as a Lewis acid. Due to presence of a electrons in its valence shell, B(0H)3 accepts a
lone pair of electrons on P, PH3 acts as a Lewis OH" from H2O releasing a proton, i.e., option (b)
is correct.
base, Due to strong -I effect of F, N cannot donate
its pair of electrons and hence it does not act as a
Lewis ba.se. Na"*" H“ is, however, a strong Bronsted H-^OH + B(0H)3 ►H'" + [B(0H)4]"
base.
SOME p-BLOCK ELEMENTS 11/61

31. As the size of the element increases, the element-


For Difficult Questions element bond strength decreases and hence Pb
does not show catenation.
Since it does not release a proton of its own in
32. SnO-) is amphoteric.
water, it does not act as a protonic or Bronsted
acid. 33. B^03 and SiOi both are acidic in nature.
20. As explained above in Ans. 19, boric acid acts as 34. Since inert pair effect becomes more pronounced
a lewis acid by accepting a OH“ ion from water down the group, therefore, +4 oxidation state of
thereby releasing a proton. Sn is more .stable than its +2 oxidation state and

w
hence Sn^'*' loses two electrons and thus acts a
21. Orthoboric acid is formed. Refer to page 11/16.
reducing agent.
Na,B407 + H2SO4 + 5 H2O 4

Sn2+ Sn‘*-^ + 2 e"


4 H3BO3 + Na->S04 Le.ss stable

Flo
More siable
Orthoboric acid

22. BN is called inorganic graphite because its {Reducing agent)


structure resembles with that of graphite. In contrast, +2 oxidation stale of Pb is more stable

ee
23. NaBH4 + 2 HjO NaB02 + 4 H2 than its + 4 oxidation state, therefore, Pb'^'*’ accepts
two electrons and thus acts as an o.xidising agent.

Fr
UBH4 + 2H2O ^ LiBO-> + 4 H2
B2H6 + 6H2O 2 H3BO3 + 6 H2 Pb^-*- +2 6- > Pb 2+
Less stable More stable
Na2B407 + 7 H2O 2 NaOH + 4 H3BO3
{Oxidising agent)

or
Since borax on hydrolysis
ur
produces a weak acid,
H3BO3 and a strong base, NaOH, therefore, Thus, option {d) is con-ecl.
aqueous solution of borax is most basic.

24. Na2B407
A
2 NaBO-> + Bo03
sf
35. F2 being a very strong oxidising agent, it oxidises
Sn and Pb to +4 oxidation and hence Snp4 and
Pbp4 are ionic in nature. Thus, option {b) is
k
Yo
Borax
incorrect.
oo
A
B2O3 + CoO Co(B02)2 36. Refer to page 11/27.
Cobalt inelaborate (blue)
B

37. Addition of Cl" ion to a suspension of PbCU leads


Thus, option (c) is correct. to the formation of soluble complex as shown
re

25. The green coloured bead is due to the formation below :


of chromium metaborate.
A PbClj +C1- [PbCl3]-
ou
ad

Cr203 -t- 3B2O3 2Cr(B02)3 Insoluble Soluble complex

26. Corundum is the oxide mineral (AI2O3) of 38. sp^ (diamond), sp^ (graphite), sp (ethyne).
Y

aluminium.
39. Graphite {sp^, 67% ^-character) and diamond (sp^,
75% p-character).
27. A1 -f 3 NaOH ^ 3 Na+ A1(0H)3
nd

40. Option {a) is wrong because it contains 20 six-


Re

X
NaOH membered rings and 12 five-memberedrings.
A1(0H)3 > NaAl(OH)4
41. Two 0-atoms are shared per Si04 tetrahedra in
Fi

2 A1(0H)3 AI2O3 + 3 H2O silicate anion of beryl mineral. (Referto Pig. 11.22,
page 11/44).
AI2O3 is used as an adsorbent in chromatography.
28. Anhydrous HP is a covalent compound and is 42. (CH3)2SiCl2 for linear polymerization and
strongly H-bonded.Therefore,it does not give F“ (CH3)3SiCl for chain termination.
ions and hence AIP3 does not dissolve in HP. In 43. (CH3)3SiCl is used to control the length of the
contrast, KF being an ionic compound contains polymer chain. Refer to sec. 11.19.5, page 11/42.
P“ ions. Therefore, it combines with AIF3 to form 44. Only statements (III) and (IV) are correct.
soluble complex, K3[AlFg] Statement (I) is wrong since CO2 {g) is not used
Alp3-h3KF ^K3[A1F6] as a refrigerant and statement (II) is wrong since
contains 20 six-membered and 12 five-
Heat
29. AI2O3 -I- 3 CI2 + 3 C > 2 AICI3 -b 3 CO. membered rings.
11/62 ‘PneitCec^'^i, New Course Chemistry (Xl)CSlSS

For Difficult Questions

m Multiple Choice Questions (with One or More than One Correct Answers)
45. The maximum O.S. of T1 is + 3, therefore, TI in H3BO3, however, does not undergo self ionization
TICI3 cannot increase its O.S. beyond + 3. Further, to release a proton.
since + 1 O.S. of TI is much more stable than + 3.
B(0H)3 (H0)2B0“ + H-"

w
therefore, TI in TlCl is not likely to increase its
Further due to H-bonding, H3BO3 is a planar
O.S. to + 3. Thus, both TICI3 and TICl do not molecule (Fig. 11.11, page 11/17). Thus, only
undergo disproportionation. Thus, options {b) and
options (b, cf) are correct.
(c) are correct.

Flo
52. Carbon cannot expand its covalencybeyond 4. Si,
46. Only Si02 is acidic while all others are amphoteric. however, can have a coordination number of 6 but
47. NO is neutral CrO is basic while B2O3 is acidic. due to sleric hindrance of 6 big sized Cl atoms

e
Therefore, all the oxides included in options (a) and weak interaction between lone pairs of Cl and

re
and (b) are amphoteric in nature. Si atom, [SiClg]^“ is not known. Thus, correct

rF
48. In dimeric structure of AICI3, the bridged Cl forms options are (a) and (/?).
a covalent bond with one Cl atom and coordinate 53. Because of big size of Cl atoms and weak
bond with the other, (refer to Fig. 11.2, page 11/ interactions between lone pairs of Cl and Si, Si
ur
10). As such it does have a 3 c - 2 c bond as in

fo
cannot expand its covalency beyond 4 and hence
Al2(CH3)g and 82!!^. Thus option (d) is not correct SiCl4 cannot act as a catalyst.
while all other options (a, b. c) are correct. 54. All the chlorides being Lewis acids accept a pair
49. Refer to structure of borax ion (Fig. 11.9, page
ks
of electrons from H2O molecule hence undergo
Yo
11/14). It contains tetranuclear [3405(011)4]-“ hydrolysis.
oo
units having two .sp^ and two iv^^-hybridized boron 55. Graphite has higher electrical conductivity than
atoms. It also contains one terminal hydroxide per diamond but diamond has higher thermal
B

boron unit. Thus, option (b) which states that all conductivity than graphite. Further, since C—C
boron atoms lie in the same plane is wrong since
e

bond in graphite has double bond character,


all the boron atoms are not ^p^-hybridized. Thus, therefore, its bond order is higher than in diamond.
ur

all the remaining (a,c,d) options are correct. Thus, options (h) and (t/) are correct.
ad

SO. H-bonding in H3BO3 gives it a layered structure 56. Since Q has an oxidation state of +2 and it acts as
Yo

which accounts for its slippery nature. a reducing agent, therefore, compound (Q) is most
51. Orthoboric acid is a weak monobasic Lewis acid.
probably SnCl2 and compound (Z) is SnCl4
It accepts a pair of electrons from OH“ ion of H1O
releasing a proton. SnCU +2CuCI, > SnCl,4 +2CuC\
nd
Re

(0) (Z)

H-^OH + B(0H)3 ■►H'" + [B(OH)4f Since SnCl2 reacts with Cl ion to form a
Fi

monoanion (X) which is 5p^-hybridized and has


On adding ethylene glycol, it acidity increases
pyramidal in shape, therefore, (X) must be SnClJ
CH^OH
[B(0H)4]“ + 2
CH.,OH SnCl^ +cr ^ SnCl3
(Q) (X)

Now SnCl2 has six electrons in the valence shell,


CHjO^ B ^0CH2 therefore, it accepts a lone pair of electrons from
4-4H.,0
Me to form a coordinate bond and complete its
octet.
SOME p-BLOCK ELEMENTS 11/63

For Difficult Questions

DQ Multiple Choice Questions (Based on the given Passage/Comprehens ion)

of Sn, therefore, strongest reductant is GeCl2.


58. +4 oxidation state of Ge is more stable than that

59. Since compounds in + 3 oxidation state are more covalent, therefore, they do not form M^'*’ ions.
Thus, option (b) is wrong. In other words, option (c) is correct.

w
□ Matrix-Match Type Questioi'^
65. A-q

F lo
Hydrolysis
+ H2O ^ BiO-^ + 2 H +

Bismuth oxycaiion

ee
B-s

Fr
Dilution
AIO2 +2H2O wiih water
> AI(0H)3 + OH-
C-p, r

for
(/) Acidification
2SiOj- + 2 H-" ■>
Si20^“ + HjO
ur
(«)A
Orthosilicate Pyrosilicate
Y>-q, r
oks
Acidification
B^O^- +2m
Yo
H2B40y
o

Hydrolysis
eB

H2B4O7 + 5 H2O > 4H3BO3

VI. Integer Type Questions


our
ad

67. Five ; refer to Fig. 11.9, page 11/14.


68. Six; 3B2H6+18MeOH 6 B(OMe)3 + 18 H2
Y

Trimethylborate
Re

69. Four ; diamond, graphite, fullerenes and carbon nanotubes..


nd

70. Three ; CF4, SiC^, CCI4.


71. One : T1 only.
Fi

72. Two ; refer to Fig. 11.21, page 11/43.

VII.
Numerical Value Type Questions Decimal Notation)
NaOH
73. 2A1
^ 2 A(0H)3 AI2O3
54 g 102g

54 g of A1 produce A1203 = 102 g


27 g of A1 will produce AI2O3 = 102/2 = 51’0 g

f
11/64 ‘P%adee^'4. New Course Chemistry (XI)S^iaD

For Difficult Questions

VIII
Assertion-Reason Type Questions

75. Correct statement-1. Aluminium acts as a 82. Correct reason. Due to inert pair effect, A1 and
reducing agent. Ga have nearly same atomic radii.
76. Correct statement-2. In water, orthoboric acid 85. Correct explanation. AI2O3 being colourless does

w
behaves as a weak monobasic Lewis acid. not form a coloured borax bead.

88. Correct assertion. Pbl4 is unstable.


Correctreason.I" being a reducingagent, reduces
H-^OH + B(0H)3 > H"" + ’b(OH)4 Pb^+ to Pb2+ or Pb'^'*’ being an oxidising agent

F lo
78. Correct statement-2. The lower oxidation slates
oxidises I” to I2.
for the group 14 elements are more stable for 90. Correct reason. Each carbon in graphite has one
heavier members due to inert pair effect. free electron which makes graphite a good

ee
80. Correct explanation. Boron belongs to second conductor of electricity.

Fr
period of the periodic table, therefore, it cannot 91. Correctexplanation.Like graphiteand diamond,
have more than 8 electrons (or four bond pairs) in CgQ on burning gives CO2.
93. Correct reason. Si atoms in silicones are
the valence shell. Thus, it can form only BF^ but

for
surrounded by non-polar alkyl groups, which repel
not BFg“.
ur water molecules.
ks
Yo
oo
eB
r
ou
ad
Y
Re
nd
Fi
ow
ORGANIC CHEMISTRY —

e
Fl
re
SOME BASIC PRINCIPLES

F
AND TECHNIQUES
ur
r
fo
PART-1. GENERAL INTRODUCTION TO ORGANIC COMPOUNDS ks
In the previous unit, you have studied that carbon due to its unique property of catenation forms covalent
Yo
bonds with other carbon atoms. It also forms covalent bonds with the atoms of a number of other elements
oo
such as O, N, S, P and halogens. All these compounds are studied under a separate branch of chemistry called
Organic Chemistry.
B

Organic compounds are essential for existence and maintenance of life on earth. These include complex
re

molecules like deoxynucleic acids (DNA), which transmit genetic information from one generation to the
other and proteins which constitute essential components of our blood, muscle and skin. Most of the
u

material which we use daily such as clothing, fuel, polymers, dyes and medicines are made up of organic
ad

compounds.
Yo

Therefore, in this unit, we shall discuss some basic principles and techniques of analysis needed for
understanding the formation and properties of organic compounds.
d
Re

12,1. DISTINCTION BETWEEN ORGANIC AND INORGANIC COMPOUNDS


in

It has been known since times immemorial that minerals, plants and animals are the three major sources
F

of naturally occurring substances. But it was only around the year 1780, that chemists divided these compounds
into two classes, viz-, organic and inorganic mainly on the basis of their source. Thus, the branch of organic
chemistry is about two hundred years old. Compounds like urea, sugars, oils, fats, dyes, proteins, vitamins,
etc., which were isolated directly or indirectly from living organisms, such as animals and plants were called
Organic Compounds and the branch of chemistry which dealt with the study of these compounds was
called Organic Chemistry. On the other hand, compounds like common salt, marble, alums, nitre, blue and
green vitriol.^, etc. which were isolated from non-living sources, such as rocks and minerals, were called
Inorganic Compounds, and the branch of chemistry which dealt with the study of these compounds was
called Inorganic Chemistry.
12/1
12/2 'Pn/tdee^A New Course Chemistry (XI)EEMD

12.1.1. Vital Force Theory


Until early nineteenth century, it was believed that organic compounds cannot be prepared in the
laboratory but can only be isolated from animals and plants. On the basis of this belief, Berzelius, a leading
Swedish chemist in 1815, propounded Vital Force Theory. According to this theory, organic compounds
produced only under the influence of some mysterious force existing in the living organisms. This

w
are

mysterious force was called the Vital Force. Since such a mysterious force cannot be created artificially, it is
impossible to synthesize organic compounds in the laboratory. This theory reigned supreme for a number
of years.
However, in 1828, Wohle,r a German chemist, made an interesting discovery. He accidentally obtained
well known organic compound, present in the urine of man and other mammals by evaporating an

o
urea, a

e
aqueous solution of ammonium cyanate, a typical inorganic compound (obtained by double displacement of

re
ammonium chloride and potassium cyanate):
NH4CI + KCNO NH4CNO + KCl ;

Frl
■>

F
Ammonium Potas.sium Ammonium
chloride cyanate cyanate

O
ou
or
Heat

NH4CNO ^ H2N—C—NH^
Rearranges

kfs
{Inorganic Urea

compound) {Organic compound)


This synthesis gave a death blow to Vital Force Theory and clearly demonstrated that no mysterious
oo
force was required in the formation of organic compounds in the laboratory. The elegant synthesis of acetic
acid by Herman Kolbe in 1845 and that of methane by Berthelol in 1856 from purely inorganic sources
Y
which had previously been obtained from biological materials, drove the last nail in the coffin of Vital Force
eB

Theory.

Electric Dil.H2S04 Oxidation

2C + H2 > HC = CH CH3CHO ^ CH3COOH


ur

arc
HgS04
oY

Acetylene Acetaldehyde Acetic acid

Thereafter, thousands and thousands of organic compounds have been synthesized in the laboratory
ad

thereby showing conclusively that like inorganic compounds, organic compounds can be easily synthesized
in the laboratory.
d

12.1.2. Modern Definition of Organic Compounds


in

With the downfall of iVtal Force Theory, the term organic {pertaining to life) lost its original significance.
Re

However, it was shown that all organic compounds whether natural or synthetic essentially contain carbon
and hydrogen and occasionally a few other elements such as oxygen, nitrogen, sulphur, halogens and
F

phosphorus. Thus, organic chemistry is now defined as the chemistry of carbon compounds containing
usually hydrogen and one or more additional elements like oxygen, nitrogen, sulphu,r halogens, phosphorus,
etc. Inorganic chemistry, on the other hand, is deifned as the chemistry of all elements other than carbon
and their compounds.
It should be mentioned here that some compounds of carbon such as CO, CO^, H2CO3, metal carbonyls,
metal carbonates, carbides, cyanides, etc. which should otherwise have been included in organic chemistry
are still studied under inorganic chemistry because of their greater resemblance with inorganic compounds.

Further, organic compounds made up of only carbon and hydrogen atoms are called hydrocarbons and
all other organic compounds may be regarded to have been derived from these hydrocarbons by replacement
of one or more of their hydrogen atoms by other atoms or groups. Thus,
ORGANIC CHEMISTRY-SOME BASIC PRINCIPLES AND TECHNIQUES 12/3

Organic chemistry may more precisely be defined as the chemistry of hydrocarbons and their
derivatives.

12.2. TETRACOVALENCY OF CARBON


Carbon is the essential element of all organic molecules. Its electronic configuration is 2, 4. In other
words, carbon has four electrons in the valence shell and thus needs four more electrons to complete its octet.
Therefore, carbon is tetravalent. Further, it is energetically very difficult for carbon to either gain or lose four
electrons to achieve the nearest inert gas configuration. Consequently, carbon always combines with other
atoms by mutual sharing of electrons and thus forms covalent bonds. Since ail the organic compounds contain
carbon as the essential element, therefore, the nature of chemical bonding in organic compounds is always

w
covalent. Thus, carbon is always tetracovalent, i.e., it forms four covalent bonds with other atoms as shown
below:

F lo
H Cl H H

H—C—H Cl—C—Cl H—C—C—H


I

e
FIGURE 12.1

Fre
H Cl H H
Methane Carbon Ethane
tetrachloride

for
However, these structures do not reveal as to what
is the spatial orientation of these four bonds whether
r
they are planar, i.e., lie in the same plane or are non-
You
polar, i.e., lie in different planes. The first insight into
oks
the spatial disposition of the four bonds of carbon was
suggested by van't Hoff and Le Bel who in 1874
eBo

predicted that the four bonds of a carbon atom are


directed towards the four corners of a regular (a) A tetrahedron, (b) Vant's Hoff tetrahedral
carbon atom with the four valencies directed
tetrahedron, i.e., the angle between any two adjacent towards the comers of the tetrahedron. The
bonds is 109°-28' {tetrahedral angle) as shown in
our
ad

Fig. 12.1.
wedged line denotes the valency above the plane
of the paper ; the normal lines denote valencies
This tetrahedral concept of carbon has been lying within the plane of the paper while the
established beyond any doubt by electron diffraction, dotted line denotes the valency below the
plane of the paper
X-ray diffraction and spectroscopic studies.
dY
Re

The tetrahedral arrangement of four bonds of carbon laid the foundation of the present day fascinating
field of "Stereochemisny’. It is because of this reason that van’t Hoff was awarded the first Nobel Prize in
Fin

Chemistry in 1901. He is also sometimes called 'The father of Organic Chemistry'.

12.3. SHAPES Or SIMPLE ORGANIC MOLECL r:


One of the most important a.',pecls of organic chemistry is to understand the fundamental concepts of
molecular structuie since on me basis of these structures, we can easily predict the properties of organic
molecules. To explain the structures of organic molecules, the electronic theory of valency and the concept of
hybridization were introduced in unit 4. On the basis of sjo^-hybridization of carbon, the structures of methane
(Fig. 12.2) and ethane (Fig. 12.3) molecules were predicted to be tetrahedral.
Thus, a molecule of methane contains four C—H, a-bonds while that of ethane contains six C H,
CT-bonds and one C—C, a-bond.

1
12/4 7\axUe^'4, New Course Chemistry (XI)EIBD
●'■*-1 'I ● 1' . T ●● FIGURE 12.2
HATOM
H
1s ,

^^109°-28' sp^-s, a bond

● Is
"" H
H

w
HATOM ●, HATOM O
, Is/
H
HATOM

F lo
(a) Orbital picture of methane, (b) Bond angles and bond lengths in methane

FIGURE 12.3

e
Fre
HATOM H H
HATOM

Is
x'H

for
hatomv^A
109°-28'
"C
sp3 109°-28' 154pm “
r
;c: sp^Q sp^-sp^,
You
oks
■ sp3-s. C—C.
o-bond H
Sp3 C—H,
eBo

a-bond
Is o
HATOM HATOM

(a) Orbital picture of ethane, (b) Bond lengths and bond angles in ethane
our
ad

Similarly, structure of elhene (CH2 = CH2) was explained to be planar on the basis of hybridization
(Fig. 12.4) and that of ethyne (HC = HC) to be linear on the basis of 5p-hybridization as shown in Fig. 12.5.
FIGURE 12.4
dY
Re

H-ATOM 2Pz H-ATOM


7t-bond
Is fjs
Fin

r TE-bond
H H
sp2 sp' S

\
1^^52 M sp ■C ^120°
CT-bonds g-bond
O-bonds

Sp2
' \ O-bond Sp2 / H 7C-bond
H

Ms JC-bond
H-ATOM H-ATOM

o O
(a) Orbital picture of ethene. (b) p-electron cloud consists of two equal halves 5 one
lying above and other below the plane of carbon and hydrogen atoms

I
ORGANIC CHEMISTRY-SOME BASIC PRINCIPLES AND TECHNIQUES

rw
12/5

FIGURE 12.5
TT-bond55

K
a-ijond
r
c- - ■H
C o

lo
n

F
7t-bonds

u
(a) Orbital picture of cthyne (acetylene) molecule, (b) Acetylene molecule with its

oF
carbon atoms surrounded by a cylindrical K-electron cloud

The carbon-carbon double bond of ethene consists of a strong carbon-carbon cT-bond and a weak

rs
carbon-carbon 7t-bond. Similarly, the carbon-carbon triple bond of ethyne consists of a strong carbon-

k
carbon CT-bond and two weak carbon-carbon 7t-bonds. The jc-bonds, in turn, are formed by sideways overlap

o
of the p-orbitals.
12.3.1. Some Characteristic Features of n-Bonds

of
For jt-bond formation, it is essential that the/j-orbitals on the adjacent carbon atoms must be parallel for
a proper sideways overlap. Some of its characteristic

o
features are ;
Y
(/) All the atoms directly attached to the carbon atoms of the double bond lie in the same plane. Thus,
B
in H2C = CH2 molecule, all the six atoms, i.e., two carbon and four H-atoms lie in the same plane.
rY
(//) The p-orhitals of a ti-hond are mutually parallel and are perpendicular to the plane of the
molecule.

(Hi) Rotation of one CH2 fragment w.r.t. the other interferes with the maximum overlap of the
ue

p-orbitals. Therefore, such a rotation about carbon-carbon double bond (C = C) is restricted.


(/v) The electron charge cloud of the 7c-bond lies above and below the plane of the bonding atoms. This
od

allows the n-electrons being easily available to the attacking reagents.


ad

In general, the 7C-bonds are the most reactive sites in the molecules containing multiple bonds.
in

12.3.2. Effect of Hybridization on Bond Lengths and Bond Strengths


The bond length and bond strength of any bond depends upon the size of the hybrid orbitals involved.
(0 Bond lengths. Since ap-orbital is much bigger in size than an ^-orbital of the same shell, therefore,
Re
F

as we go from sp^ > sp^ > .yp, the percentage of p-character decreases from 75 —
>66-7 > 50%.
Accordingly, the size of the orbital decreases in the same order; sp^ > sp^ < sp. Since a bigger orbital forms
a
longer bond, therefore, C—C single bond lengths decrease in the order :

C (ip3) — C (,yp3) > c (jp2) — C (5p2) > C{sp) — C{sp)


154 pm 148 pm 138 pm
Further since, the sideways overlap of two p-orbitals (to form 7C-bonds) brings the two carbon atoms
closer to have an optimum degree of overlap, therefore, a single bond is longer than a double bond which, is
turn, is longer than a triple bond. Thus,

C C > ■C=C: > c = c-


154 pm 134 pm 120 pm
12/6 ‘PnaeU^ 'A- New Course Chemistry (XI)E5ISD

Similarly, a C—H bond may be formed by overlap of a or sp-orbital of carbon with an


,9-orbiial of hydrogen, therefore, as the size of the hybrid orbital decreases, the lengths of C—H bond decrease
accordingly.
C(5p-^)_H > 0(5/7^) —H > C{sp)—H
111 pm 110 pm 107-9 pm

{//) Bond strengths. Shorter the bond, greater is its strength. Thus, the a-bond formed by 5/7-hybridized
carbon is the strongest {i.e. m;iximum bond enthalpy) while that formed by sp^-hybridized carbon is the
weakest {i.e., minimum bond enthalpy). For example,
U)C{sp) — H > C(sp^) — n > C{.vp-^) —H
-1 -1
507 kJ mol 443 kJ mol 435 kJ mol

w
(//) C (sp) — C (^p) > C (sp~) — C (ip2) > C (V) — C (sp^)
-1 -i
433 kJ mol 383 kJ mol 348 kJ mol

F lo
As stated above, a 7t-bond is formed by the sideways overlap of two p-orbitals. Since the extent of
overlap in sideways overlap is low, a carbon-carbon 7C-bond is always weaker (251 kJ mol ^) than a carbon-
carbon CT-bond (348 kJ mol“^). A carbon-carbon double bond is, however, stronger (599 kJ mor*) than a
carbon-carbon single bond since it consists of a strong o-bond and a weak Tt-bond. In a similar way, a carbon-
carbon triple bond is still stronger (823 kJ mol"') than a carbon carbon double bond.

e
Fre
{Hi) Electronegativity. The type of hybridization also affects the electronegativity of the atom, i.e.,
more the s-character of the hybrid orbitals more electronegative is the atom. Thus, a ^p-hybridized carbon
atom having hybrid orbitals with 50% .?-character is more electronegative than a 5p“-hybridized carbon with

for
33-33% 5-character and 5p^-hybridized carbon with 25% .5-character. These small differences in
electronegativity are reflected in many physical and chemical properties of the concerned molecules.
r
Curiosity Questions
You
oks
r Q. 1. Why is methane tetrahedral ?
eBo

Ans. Methane has four (3—H bonds. These are inclined at an angle of 109- - 28' which is also called
the tetrahedral angle. The four C—H bonds in methane are held at an angle of 109* - 28'
because this is the only angle in space at which repulsions between the four shared pairs of
electrons is minimum. That is why methane assumes tetrahedral geometry.
Q. 2. How many tetrahedral angles does a molecule of methane have ?
our
ad

Ans. Since methane has tetrahedral geometry, therefore, it has six tetrahedral angles. 1

Sample Problem 12.1 How many o- and 7r-bonds are present in each of the following molecules ?
dY
Re

(a) HC s CCH = CHCH3


(NCERT Solved Example)
(b) CH2 = C = CHCH3
Fin

Solution. Write the complete structural formulae and count the number of carbon-carbon, carbon-hydrogen
sigma-honds and carbon-carbon p(-bonds :
H H

(r^) _^C=C-^C H (b) H-^C = C = C-^C— H


' ^ ol ^ " lo
o
/" o|
H
1(7 al
H H H H H

No. of Oc_c - 4 ; No. of Oc_h = b No. of Oc__c = 3 ; No. of Oc_h - 6


Total no. of a-bonds = 4 + 6 = 10 Total no. of a-bonds = 3 -t- 6 = 9
Total no. of nc = C
bonds = No. of TCc ^ c bonds =1x2 (double bonds) = 2
2 (triple bond) + ! (double bond) = 3
ORGANIC CHEMISTRY-SOME BASIC PRINCIPLES AND TECHNIQUES 12/7

Sample Problem 12.2 What is the type of hybridization of each carbon in the following
compounds ?
(a) CH3CI, (b) (CH3)2C0, (c) CH3CN

ow
(d) HCONH 2’
(e) CH3CH = CHCN (NCERT Solved Example)
Solution. Write the partly condensed formulae of the above compounds and indicate the type of hybridization
on each carbon.
3
sp 3 2 3 3
sp sp sp sp sp
(a) CH3—Cl (h) CH3— C —CH3 (c) CH3—C=N
O

re
2 3 2 2
sp sp sp sp^ sp

{d) H—C—NH2 {e) CH3—CH = CH—C s N

Flr
F
O

Sample Problem 12.3 What is the shape of following compounds ?


(a) HjC^O, (b) CH3F,
ou
(c) HC s N (NCERT Solved Example)
Solution, (a) In H2C = O, C is .sy?--hybridized. hence formaldehyde is trigonal planar,

sr
(b) In CH3 —F, C is .vp^-hybridized, hence methyl fluoride is tetrahedral.

fo
(c) In H—C s N, C is jp-hybridized, hence HCN is a linear molecule.

k
12.4. STRUCTURAL REPRESE^fTATION OF ORGANIC COMPOUNDS
oo
12.4.1. Complete and Condensed Formulae
Y
The structures of organic compounds can be represented in several different ways. The most important
of these is the Lewis structures. But writing these structures is quite time consuming. However, these stmctures
reB

can be simplified if each pair of electrons making a covalent bond is represented by a da.’th (—). Evidently
two dashes (=) will be required to represent a double bond and three dashes (s) to represent a triple bond. The
lone pairs of electrons on the heteroatoms (e.g., oxygen, nitrogen, sulphur, halogens, etc.) may or may not be
uY

shown. Thus, ethane (C2Hg), ethene (C2H4), ethyne (C2H2) and methanol may be represented by the following
structural formulae.

H H
ad
do

H H
H H I
H—C—C—H c = c; H—C = C—H H—C—O—H or H—C—O—H
H H
in

H H Ethene Ethyne H H
Ethane
Re

Methanol

Such structural representations are called complete structural formulae or graphic or displayed
F

formulae.

These structural formulae can be further abbreviated by omitting some or all the dashes representing
covalent bonds and by indicating the number of identical groups attached to an atom by a subscript. For example,
CH3CH3 or C2H6 H2C = CH2 or C2H4 HC = CH or C2H2 CH3—OH or CH3OH
Ethane Ethene Ethyne Methanol

Such structural representations are called condensed structural formulae.


Sometimes these structural formulae can be further condensed by enclosing the repetitive structural unit
within a bracket and placing an integer as a subscript indicating the number of times the structural unit gets
repeated. For example, CH3CH2CH2CH'>CH->CH^CH^CH3 can be further condensed to CH^CCH-iiftCHi
Similarly, CH3CH2CH2CH2CH2CH2CH2COOH ' be“ further condensed to CH3(CH2)6COOH.
can - -0 .5
12/8 ■pru^dce^'A New Course Chemistry (XI)BEiaD

12.4.2. Bond-line Structural Formulae


It is a simple, short and convenient method of representing organic Unsubstituted intersections
molecules. In this method, carbon-carbon bonds are shown by lines drawn in a represent CH2 groups
zig-zag fashion and carbon atoms by line ends and intersections. A single bond
is represented by a single line {—), a double bond by two parallel lines (=) and
a triple bond by three parallel lines (=). Although carbon atoms are not shown

but all atoms other than carbon and hydrogen atoms such as oxygen, nitrogen,
halogen, etc. are shown on the zig-zag line. Further, it is assumed that each

w
carbon on the line end or intersection is attached to required number of hydrogen
atoms, i.e., termini denote CH3 groups and an unsubstituted intersection denotes Termini represent CH 3 groups
a
CH2 group. For example, 3-methyloctane may be represented as

Flo
Likewise 3-bromobutane and octanoic acid may be respectively represented by the following bond line
structural formula.

ee
O

Fr
OH

or
ur
2-Broniobutane Octanoic acid

f
12.4.3. Polygon Formulae
There are many organic compounds, in which the carbon atoms are not joined in a chain but are joined
ks
in a ring. These are called cyclic compounds and are usually represented by polygon without showing carbon
Yo
and hydrogen atoms. The comer of a polygon represents a carbon atom and the sides of a polygon denote a
oo
carbon-carbon bond. If an atom or a group of atoms other than hydrogen is attached to carbon, then that atom
eB

or a group of atoms is shown in the stmcture. For example,


Cl OH
ur
ad
Yo

Cyclopropane Cyclobutane Cyclopentane Cyclohexane Chlorocyclohexanc Cyclohexano!

Sample Problem 12.4 Expand each of the following condensed formulae into their complete
structural formulae
d
Re

(a) CH3CH2COCH2CH3 (b) CH3CH = CH(CH2)3CH3 (NCERT Solved Example)


n

H H H H H
H H
Fi

H H O H H
I
Solution, (a) H—C—C—C—C—C—H ib) H—C C = C—C—C—C—C—H
I I
H H H H H H H H H

Sample Problem T2J For each of the following compounds, write a condensed formula and
also their bond line formulae.
OH

(a) H0CH2CH2CH2CH(CH3)CH(CH3)CH3 (b) N s C—CH—Cs N


(NCERT Solved Example)
ORGANIC CHEMISTRY-SOME BASIC PRINCIPLES AND TECHNIQUES 12/9

Solution. Condensed Formulae (a) HO(CH2)3CH(CH3)CH(CH3)2 (h) HOCH(CN)2


CN

Bond-line Formulae (a) (h)


HO
HO' CN

Sample, Pj^pblem 12.6 Expand each of the following bond-line formulae to show all the atoms

ow
including carbon and hydrogen. (NCERT Solved Example)

(a)
OK (b)

(C) # (d)
X

e
Fl
OH

re
Solution.

F
H H H H
H
V V ur
/H C —H

r
H
(a)
>c C< /I

fo
H

/\
C
/\
c
H >C^H H—C—H ks
H H H H
Yo
H
oo
H H H H H H H H

(b) H—i—c—d:—i—c—c—i —H
B

I I
H H H H H H H H
re

H H
u
ad

H
H H
Yo

C
H H H H
I I I
(c) H—CsC—C—C—C—H (d)
d

H C ■C H
I I I
Re

H O H
in

H /C /C

/ /
F

H H H H
H H

12.5. THREE-DIMENSIONAL (3-D) REPRESENTATION OF ORGANIC MOLECULES


The three-dimensional (3-D) structure of organic molecules can be represented on paper by using certain
conventions. For example, using solid (-^●) and dashed wedge (..Hiiiii|||) formula, 3-D image of a molecule
from a two-dimensional picture can be perceived. In these formulae, the thick solid (or heavy) line or the
solid wedge indicates a bond lying above the plane of the paper and projecting towards the observer while a
dashed wedge is used to represent a bond lying below the plane of the paper and projecting away from the
observer. Wedges are used in such a way that the broad end is towards the observer. The
12/10 "p/uidee^'iX. New Course Chemistry (XI)S!EIHD

FIGURE 12.6
bonds lying in the plane of the paper are depicted by
using a normal or an ordinary line. Dashed wedge
(bond away from observer)
The 3-D representation of methane molecule on Bonds in
the plane
paper is shown in Fig. 12.6. of paper
Such a representation which completely describes
the actual positions of various atoms of a molecule in
Solid wedge
space is called a spatial formula or three-dimensional (bond towards observer)
i.e., 3-D structure.
Wedge-and-dash representation of CH^

low
SUPPLEMENT YOUR
KNOWLEDGE FOR COMPETITIONS

Molecular Models. Molecular models are physical devices that are extensively used for a belter visualization
and perception of 3-D structures of organic molecules. These are made up of wood, plastic or metal and
are commercially available. These arc mainly of three types :
(0 Framework models (li) Ball and stick models and (Hi) Space iflling models as shown below :

ee
F
Fr
%
9

6^0

for
#'
ur
Framework model
a
Ball and stick model space filling model

In the framework models, the atoms themselves are not shown but only the bonds connecting the atoms
s
of a molecule are shown. This type of model is used only to study the pattern of bonds in a molecule while
ok
Yo
ignoring the size of the atoms.
In the ball-and-stick model, both the atoms and bonds are shown. Whereas balls represent the atoms but
Bo

sticks denote the bonds. Unsaturated compounds containing C = C and C s C bonds are better represented
using springs instead of sticks.
In space filling models, bonds are not shown but emphasis is laid on the relative size of each atom
re

dependingupon its van der Waals’ radius. In other words, these models convey the volume occupied by
each atom in the molecule.
ou

In addition to these models, computer graphics can be used for molecular modelling.
ad
Y

PART II. CLASSIFICATION AND NOMENCLATURE OF ORGANIC COMPOUNDS

12.6. CLASSIFICATION OF ORGANIC COMPOUNDS


nd
Re

The simplest organic compounds containing only carbon and hydrogen are called hydrocarbons. They are
considered to be parent organic compounds while all other organic compounds are thought to have been derived
Fi

from them by replacement of one or more of their hydrogen atoms by other atoms or groups of atoms. As the study
of oig;anic chemistry advanced, the number of organic compounds became very large. As a result, it became rather
inconvenient to study the chemical behaviour of these compounds individually. Therefore, in order to simplify and
systematize the study of organic chemistry, all the known organic compounds have been broadly divided into two
categories depending upon the nature of their carbon .skeleton. These are :
/. Acyclic or open chain compounds. II. Cyclic or closed chain compounds.
/. Acyclic or open chain compounds. These compounds contain open chains of carbon atoms in their
molecules. The carbon chains may be either straight chains or branched chains. For example,
Straight chain compounds:
2 1

CH3—CH2—CH2—CH3 CH3—CH =CH—CH3 CH3 ●CH2—CsCH


n-Butane Bul-2-ene But-l-yne
(formerly 2-btiiene) {formerly I-butyne)
ORGANIC CHEMISTRY-SOME BAblC PRINCIPLES AND TECHNIQUES 12/11

O O O

CH3 —CHj—C—CHj —CH3 CH3—C—H CH3—C—OH


Pentan-3-one Ethanal Ethanoic acid
(formerly 3-pentanone) (Acetaldehyde) (Acetic acid)
Branched chain compounds:
CH3 CH3 CH3
2
2I 3I 2 1
CH3—CH—CH3 CH3—C—CH3 CH3—C = CH2 CH3 —CH —C = CH

2-Methylpropene 3-Methylbut-l-yne
CH3 CH3 (Isobutylene)
2-Methylpropane 2, 2-Dimethylpropane
(Isobutane) (Neopentane)

F low
Open chain compounds are also called aliphatic compounds since the earlier compounds of this class
were obtained either from animal or vegetable fats (Greek, aliphatos = fat),
n. Cyclic or closed chain or ring compounds. These compounds contain one or more closed chains
or rings of atoms in their molecules. Depending upon the constitution of the ring, these are further divided
into the following two categories : 7. Homocyclic or carbocyclic compounds 2. Heterocyclic compounds
1. Homocyclic or Carbocyclic compounds. These compounds contain rings which are made up of
only one kind of atoms, i.e., carbon atoms. These are further divided into the following two sub-classes.

e
for Fre
(a) Alicyclic compounds.

i Carbocyclic compounds which resemble aliphatic compounds in most of their properties are
f called alicyclic compounds.

For example,

/"\
CH2 CH2
Your
eBo ks

OR OR

CH2 CH2 CH2 CH2


Cyclopropane Cyclobutane
ad

/CH\CH2
our

CH2
CH2 CH2 OR
OR
CH2 CH2
Re

CH2 CH2
Cyclopentane
\- /
CH2
Find Y

CyclohexOTe

(b) Aromatic compounds. These are special type of cyclic unsaturated compounds. The name aromatic
for this class of cyclic unsaturated compounds has been derived from the Greek word aroma meaning fragrant
smell since most of the compounds (of this class) discovered earlier had pleasant smells. However, the term
aromatic has now lost its original significance because many aromatic compounds are now known to possess
unpleasant odours.
Carbocyclic aromatic compounds are of the following two types :
(i) Benzenoid aromatic compounds. Organic compounds containing one or more fused or isolated
benzene rings and their functionalized derivatives are called benzenoid aromatic compounds.
Depending upon the number of benzene rings fused together, benzenoid compounds may be monocyclic,
bicyclic, tricyclic, etc. aromatic compounds. Examples are :
12/12 'Pfuuleefr'4, N'.iv= Course Chemistry (XI)B!

(0 Monocyclic aromatic compounds :


OH NH2 CHO COOH
CH3 NO2

Benzene Toluene Nitrobenzene Phenol Aniline Benzaldehyde Benzoic acid

(I'O Bicyclic and tricylic aromatic compounds

w
Naphthalene Anthracene
Phenanthrene
(Bicyclic) (Linear tricyclic)

F lo
(Angular tricyclic)
Conqwunds containing two or hkhc benzene rings fused togedier aie also called {M^q^dic aranafic oonqxMmds.
Besides compounds containing fused benzene rings, benzenoid compounds may contain isolated benzene

ee
rings. For example.

Fr
00 000
Biphenyl or Diphenyl Terphenyl

for
(ii) Non-benzenoid aromatic compounds. Aromatic compounds which do not contain a benzene ring
ur
but instead contain other highly unsaturated rings are called non-benzenoid aromatic compounds. For
example, tropone, tropolone and azulene.
s
O
+
ook
Yo
\
eB

Cyclopropenyl Cyclopentadienyl Cycloheptatrienyl Tropone Tropolone Azulene


cation anion cation

2. Heterocyclic compounds.
r

Cyclic compounds containing one or more heteroatoms* in their rings are called heterocyclic k'
ad
ou

u compounds.
The heteroatoms commonly found in these compounds are oxygen, nitrogen and sulphur but occasionally
Y

phosphoms, boron, silicon and some metal atoms like tin, selenium, etc. may also be present. Depending
upon the chemical behaviour, they are further classified into the following two categories :
Re
nd

(i) Alicyclic heterocyclic compounds. Aliphatic cyclic compounds containing one or more heteroatoms
in their rings are called alicyclic heterocyclic compounds. For example,
Fi

O O CH2 CH2 CH2 CH2 CH2

CH2 CH2 CH2 CH2 CH2 CHj CH2 CH2 CH2 CH2
Oxirane or
Oxacyclopropane
Oxetane or
Oxacyclobutane o \s/
(Ethylene oxide) Oxolane H Thiacyclopentane
or Oxacyclopentane Azacyclopentane (Tetrahydrothiophere)
(Tetrahydrojuran or THF) (Tetrahydropyrrole
or Pyrrolidine)
(ii) Aromatic heterocyclic compounds. Aromatic cyclic compounds containing one or more heteroatoms
in their molecules are called aromatic heterocyclic compounds. For example.
*In organic chemistry, atoms other than C and H are called heteroatoms.
ORGANIC CHEMISTRY-SOME BASIC PRINCIPLES AND TECHNIQUES 12/13

CH—CH CH—CH CH—CH


or or or

CH CH CH CH CH CH
w
O
\„/
o \ N/
/
N \s/ \s/
Furan
H
H Thiophene
Pyrrole

ow
N N
Pyridine Quinoline Isoquinoline

The above classification may be summarised as follows :

Organic Compounds

e
Fl
re
Acyclic or Open chain Cyclic or Closed chain or Ring compounds

F
Homocyclic ur Heterocyclic

r
or Carbocyclic

fo
Alicyclic Aromatic ks
Alicyclic Aromatic
Yo
I
oo
Benzenoid Non-benzenoid
eB

12.7. FUNCTIONAL OR CHARACTERISTIC GROUPS

As already stated, hydrocarbons are the parent organic compounds. All other compounds are considered
to have been derived from them by replacing one or more of their hydrogen atoms by some other more
ur

reactive atom or group (G)


-H
R—H ●> R—G
ad

+G
Yo

Hydrocarbon
Thus, each organic molecule consists of two parts, i.e., R and G. The fu’st part, i.e., ‘R’ denotes the
carbon-hydrogen framework of the molecule while the second part i.e., ‘G’ is called the functional group.
d

Thus, a functional group may be defined as an atom or a group of atoms present in a molecule
Re
in

which largely determines its chemical properties.


The remaining part of the molecule {e.g. R) mainly affects the physical properties such as melting point,
F

boiling point, density, solubility, refractive index, etc. For example,-OH {hydroxyl) is the functional group
of alcohols since in almost all the chemical reactions of alcohols, it is the OH group that undergoes a change
while the rest of the molecule {e.g. R) remains unchanged. In a similar way, -COOH is the functional group
of carboxylic acids.
From the above discussion, it follows that the chemical properties of any organic compound are the
properties of its functional group. In other words, all the organic compounds containing the same functional
group show similar chemical reactions. For example, alcohols CH3OH, CH3CH2OH, (CH3)2CHOH, etc. all
produce hydrogen when treated with sodium metal.
2R —O —H + 2Na ^ 2R —0"Na+ + H2
Alcohol Sod. alkoxide

where R = CH3-, CH3CH2-, (CH3)2CH-, etc.


12/14 New Course Chemistry (XI)EZalHD

The reason for this similar chemical behaviour is that C—C and C—H bonds are very strong and hence
do not break easily. In contrast, the O—H bond is weaker and hence reactive. Thus, we conclude that any
organic compound will behave in the .same manner as any other organic compound having the same functional

ow
group no matter what the rest of the molecule is. Howeve.", exceptions are known when the molecule is very
large (e.g. group R is large) or contains two or more functional groups in close proximity to one another. In
such cases, the molecule exhibits some special properties which are typical of neither one nor of the other
functional group.
Some of the common functional groups present in various organic compounds are listed below :

Class of organic compounds Name of the iuiictlonal group Structure of the functional group

re
Alkenes Double bond ;C = C

Alkynes Triple bond -C = C-

Flr
Halogen derivatives Halogen - X (F, Cl, Br, I)

F
Alcohols Hydroxy -OH

Thioalcohol Thiol or Sulphydryl -SH

or Mercaptan
Ethers
ou or Mercapto
Divalent oxygen -O-

sr
Thioethers or Sulphides Divalent sulphur -S-

fo
O

k
Aldehydes Aldehydic (formyl) -C-H

Thioaldehydes
Ketones
Thial

Kelonic (oxo)
oo -CH = S
:C = 0
Y
Thioketones Thione :C = S
reB

0
Carboxylic acids Carboxyl -C-OH
uY

O
Acid chlorides Chlorocarbonyl -C-Cl
ad

0 O
do

Acid anhydrides Anhydride -c-o-c-

o
in

Esters Alkoxycarbonyl -C-OR


Re

O
F

Acid amides Amide


-C-NH2
O

Sulphonic acids Sulphonic acid — S—OH

Primary amines Amino -NH2


Secondary amines Imino ;NH

Tertiary amines Teniary N-atom N —

Alkyl cyanides or Alkanenitriles Cyano or nitrile -C = N


ORGANIC CHEMISTRY-SOME BASIC PRINCIPLES AND TECHNIQUES 12/15

Class of organic compounds Name of the functional group Structure of the functional group
+
—»
Alkyl isocyanides or isoniiriles Isonitrile or isocyano -N=C or -N = Cr

or carbylamines
O + O
Nitroalkanes Nitro
-N^ O or -N
0”

Alkyl nitrites Nitrite -0-N=0

w
12.8. HOMOLOGOUS SERIES

To simplify and to systematize the study of organic chemistry, all the organic compounds have been

F lo
divided into different families or groups depending upon their structure and chemical properties. Each such
family or a group is called a homologous series.

ee
A homologous series is defined as a family or group of structurally similar organic compounds
all the members of which contain the same functional group, show a gradation in physical and

Fr
similarity in chemical properties and any two adjacent (or successive) members of which differ
by a -CH2 group. The individual members of such a series are called homologues and the
phenomenon is called homology.

for
CH4 C2H6
ur
For example, alkanes constitute a homologous series. The first six members of this series are :
^3^8 C4H10 C5H,2 C6H14
Methane Ethane Propane Butane Pentane Hexane
s
These hydrocarbons can be represented by the same general formula, C„H2„+2 where n is the number of
ok
Yo
carbon atoms. Further, any two adjacent members of this class differ by a CH2 group.
If a hydrogen atom from the above hydrocarbons is replaced by a hydroxyl group i.e. -OH, we get a new
Bo

homologous series. This homologous series is called alcohols. The different members of this series can be
represented by the general formula C„H2„+| OH. The first four members of this series are ;
re

Formula
CH3OH CH3CH2OH CH3CH2CH2OH CH3CH2CH2CH2OH
lUPAC names Methanol Ethanol Propan-l-ol Butan-l-ol
ou

(formerly I-propanol) {formerly 1 -butanol)


ad

Some other homologous series are alkcnes, alkynes, alkyl halides, ethers, aldehydes, ketones, amines,
esters, carboxylic acids, etc.
Y

Characteristics of a Homologous Series


(1) Each homologous series can be represented by a general formula. For example, C„H2„+| COOH is
nd
Re

the general formula of carboxylic acids.


(2) All the members of a given homologous series possess the same functional group. For example,
Fi

the functional group of alcohols is the hydroxyl group, i.e.. -OH group. Similarly, the functional group of
aldehydes is the aldehydic group, i.e., -CHO and that of the ketones is the ketonic group, i.e., :C = 0.

(3) The successive members of a homologous series differ by a CH2 group or by 12+2 x 1 = 14 mass
units.

(4) The individual members of a homologous series can be prepared by the general methods of
preparation developedfor that series.
(5) The physical properties such as density, melting point, boiling point of the members of a homologous
series show a regular gradation with rise in molecular mass.
(6) The chemical properties of the members of a homologous series are similar though the ifrst member
may vary considerably from rest of the members.
12/16 “P>uidee^'4- New Course Chemistry (XI)EEMD

12.9. NOMENCLATURE OF ORGANIC COMPOUNDS

The term nomenclature means the system of naming of organic compounds. In case of aliphatic
compounds, two systems of nomenclature generally used are :
(/) Trivial or common system and (//) lUPAC system,
(i) Trivial or common system. In the early stages of the development of organic chemistry, organic
compounds were named after the source from which they were first isolated. For example, urea got its name
since the compound was first obtained from the urine of mammals. Similarly, methyl alcohol was called wood
spirit since it could be obtained by the destructive distillation of wood. Acetic acid got its name from acetum
(Latin : acetum means vinegar) since it is present in vinegar. Similarly, the name formic acid was derived
from formicus (Latin : formicus means red ants) since it could be obtained by the destructive distillation of

w
red ants. Likewise citric acid is named so because it is found in citrus fruits. These names of organic compounds
are called trivial names or common names.

F lo
Common or Trivial names of some organic compounds are given below :

Compound Common name Compound Common name

CH4 Methane CH3COOH Acetic acid

ee
C2H2 Acetylene CH3CONH2 Acetamide

Fr
Acetonitrile
CH3CH2CH2CH3 «-Butane CH3CN
(CH3)2CHCH3 Isobutane C6«6 Benzene

for
(CH3)4C Neopentane C6H5CH3 Toluene
ur
CHC13 Chloroform C^HjOH Phenol

CH3CH2CH20H «-PropyI alcohol C6H5OCH3 Anisole


s
CH3CH20CH3 Ethyl methyl ether C6H5COCH3 Acetophenone
ook
Yo
HCHO Formaldehyde C6H5NH2 Aniline

(CH3)2C0 Acetone C6H5COOH Benzoic acid


eB

C6H5CONH2 Benzamide

Originally these names were given without any systematic basis but later on certain rules were developed
to name the organic compounds by the common system. However, these rules have a number of drawbacks
our
ad

and thus cannot be used to name all organic compounds.


(//) lUPAC system. Because of the unique property of catenation and isomerism, carbon forms a large
number of organic compounds. Even towards the end of nineteenth century, the number of organic compounds
Y

known was so large that it became difficult to remember them by their common or trivial names. In order to
systematize the nomenclature of organic compounds, lUPAC (International Union of Pure and Applied
Re

Chemistry) system of nomenclature was first introduced in 1947. These rules underwent modifications from
nd

time to time and the most exhaustic rules for nomenclature of organic compounds were published in 1979 and
Fi

later revised and updated in 1993. The rules discussed in this chapter are the latest and are based upon 1979
and 1993 editions of the lUPAC nomenclatureof organic compounds.
12.9.1. General Rules for lUPAC Nomenclature of Organic Compounds
The lUPAC system is the most rational and widely used system of nomenclature in organic chemistry.
The most important feature of this system is that any given molecular structure has only one lUPAC name
and any given lUPAC name denotes only one molecular structure.
The lUPAC name of any organic compound essentially consists of three parts, i.e.,
1. Word root 2. Suffix and 3. Prefix.
1. Word root. It is the basic unit of the name. It denotes the number of carbon atoms present in the
parent chain {the longestpossiblecontinuouschain of carbon atoms including the junctional group and the
ORGANIC CHEMISTRY-SOME BASIC PRINCIPLES AND TECHNIQUES 12/17

multiple bonds) of the organic molecule. For chains from one to four carbon atoms, special word roots (based
upon the common names of alkanes) are used but for chains of five or more carbon atoms Greek number roots
are used as given below :

Chain length Word root Chain length Word root

C, Meth-
C7 Hept (a)-
C2 Eth- Oct (fl)-
C3 Prop {ay C, Non («)-
C4 Bul (a)- ^10 Dec (a)-

w
C5 Pent (a)- ^11 Undec (a)-
C6 Hex (ay C12 Dodec (a)-

F lo
Extra ‘a’ given in parenthesis is used only if the primary sulfix (explained later) to be added to the word
root begins with a consonant. In general, the word root for any carbon chain is alk.
2. Suffix. There are two types of suffixes :

ee
(i) Primary sufUx. A primary suffix is always added to the word root to indicate whether the carbon

Fr
chain is saturated or unsaturated. The three basic primary suffixes are given below :
Type of carbon chain Primary suffix General name

for
ur
(a) Saturated (containing single bonds only) -ane Alkane
(b) Unsaturated with one double bond -ene s Alkene

(c) Unsaturated with one triple bond -yne Alkyne


k
Yo
oo
If the parent carbon chain contains two, three, four or more double or triplt; bonds, numerical preifxes
such as di (for two), tri (for three), tetra (for four), etc. are added to the primary suffix. For example.
eB

Type of carbon chain Primary suffix Genera] name

(a) Unsaturated with two double bonds -diene Alkadiene


r

(b) Unsaturated with two triple bonds -diyne Alkadiyne


ou
ad

The following examples illustrate the use of word roots and primary suffixes in naming organic
compounds:
Y

Organic compound Word root Primary suffix lUPAC name


Re
nd

CH3CH2CH2CH3 But ane Butane

CH3CH = CH2 Prop ene


Propene
Fi

CHhCH Eth yne Ethyne


CH2 = CH-CH = CH2 Buta* diene Butadiene
HC s C-C = CH Buta* diyne Butadiyne

(ii) Secondary suffix. A secondary suffix is then added to the primary suffix to indicate the nature of
the functional group present in the organic compound. Secondary suffixes of some important functional
groups are given below :

♦Extra ‘a’ has been added to the word root since the primary suffix, i.e., diene or diyne begins with a
consonant, i.e., ‘d’ instead of a vowel as is the case in the last two examples.
12/18 “PnA<Ucp-'4. New Course Chemistry (XI)CS19D

Cl^s of organic compounds Functional group Secondary suffix

Alcohols -OH -ol

Aldehydes -CHO -al

Ketones >C=0 -one

Carboxylic acids -COOH -oic acid


-amide
Acid amides -CONH2
Acid chlorides -COCl -oyl chloride
Esters -COOR alkyl ....oale

w
Nitriles -CN nitrile

Thiol -SH thiol

Amines -NH2 amine

Flo
It may be noted that while adding the secondary suffix to the primary suffix, the terminal 'e' of the
primary suffix (i.e. ane, ene and yne) is dropped if the secondary suffix begins with a vowel but is retained if

ee
the secondary suffix begins with a consonant.
The following examples illustrate the use of word root, primary suffix and secondary suffix in naming

Fr
organic compounds.

Organic compound Word Primary Secondary lUPAC name

for
root suffix suffix
ur
CH3CH2OH Eth an {e)* ol Ethanol

CH3CH2CH2NH2 Prop an {e)* amine Propanamine


ks
CH3CH2CH2COOH But an(c)
*
oic acid Butanoic acid
Yo
nitrile Propanenitrile
CH3CH2CN Prop ane
oo
CH2 = CHCHO Prop en {e)* al Prop-2-en-l-al
eB

HC = CCOOH Prop yn (e)* oic acid Prop-2-yn-l-oic acid

*The terminal ‘e’ from the primaiy suffix has been dropped because the secondary suffix begins with a vowel.
It may be noted that according to 1993 recommendations of lUPAC nomenclature of Organic Compounds,
r

(/) locants (numericals and/or letters) are placed immediately before the part of the name to which they
ou
ad

relate. For example,


But-2-ene Propan-2-ol
Y

(formerly 2-butene) (formerly 2-propanol)


(ii) the locant I (unity) is often omitted when there is no ambiguity. For example.
nd
Re

CHjCH^CH^NH^ Propan-1-amine is often named as Propanamine


Fi

2 i

CICH2CH2OH 2-Chloroethan-l-ol is often named as 2-Chloroethanol


I

CH3CH2CH2CHO Butan-l-al is often written as Butanal


However, in this book, the numerical locant I is always included when another numerical locant appears
in the same name. For example.
3 2 I 3 2 1

CH2 =CH—CHO HC = C—CO2H


Prop-2-en-1 -al Prop-2-yn-l-oic acid
(formerly 2-propenal) (formerly 2-propynoic acid)
ORGANIC CHEMISTRY-SOME BASIC PRINCIPLES AND TECHNIQUES 12/19

3. Prefix. There are two types of prefixes :


(i) Primary prefix. A primary prefix is used simply to distinguish cyclic fiom acyclic compounds. For example, in
case of carbocyclic compounds, a primary preifx, cyclo is used immediately before the word root. Thus,
CH,
/ ^
CH2 CH2 Cyclo + pent + ane
Cyclopentane
Primary prefix Word root Primary suffix lUPAC name
CH2—CH2
If the prefix cyclo is not used, it simply indicates that the compound is acyclic or open chain,

w
(ii) Secondary prefix. In lUPAC system of nomenclature, certain groups are not considered as functional
groups but instead are treated as substituents. These are called secondary prefixes and are added immediately
before the word root (or the primary preifx in case ofalicyclic compounds) in alphabetical order to denote
the side chains or substituent groups. The secondary prefixes for some groups which are always treated as

Flo
substituent groups (regardless of the fact whether the organic compound is monofunciional or polyfunctional)
are given below :

ee
Substituent group Secondary group Substituent group Secondary prefix

Fr
—F Fluoro OCH3 (OMe) Methoxy
—Cl Chloro
-0C.,H5 (-OEt) Ethoxy
—Br Bromo
-CH3 (-Me) Methyl

for
—I lodo
-C2H5 (-Ft) Ethyl
—NO2 Nitro
ur
-CH2CH2CH3 (n-Pr) /i-Propyl
—NO Nitroso
+
-CH(CH3)2 (-iPr) Isopropyl
Diazo -C(CH3)3 fer/-Bulyl
ks
— N=N
—OR Alkoxy
Yo
oo
Besides these, other functional groups are also treated as substituent groups in case of polyfunctional
compounds as discussed later in this unit.
eB

Thus, the complete lUPAC name of an organic compound consists of the following parts :
Secondary prefix +Primary> preifx + Word root + Primary sufifx + Secondary suffix
The following examples illustrate the use of word root, primary and secondary prefixes and suffixes in
r

naming organic compounds ;


ou
ad

Organic compound Secondary prefix Word root Primary suffix lUPAC name
Y

CH3CH2Br Bromo eth ane Bromocthane

CH3NO2 Nitro meth ane Nitromethane

C2H5OC2H5 Ethoxy eth Elhoxycthane


Re
nd

ane

In case of alicyclic compounds, primary prefixes are also used. For example.
Fi

Br (Substituent)
4-Bromo +
cyclo + hex
CH
4 Secondary Primary Word
5 prefix prefix root
CH2 ^Lri2
+ an(e) = l-ol 4-Brorao-

“CH, Primary Secondary cyclohexan-l-ol


suffix suffix lUPAC name
1
Cl-I

OH (Functional group)
12/20 'pnaxUefr'4. New Course Chemistry (XI)ESsX9D

The common and lUPAC names of some important classes of organic compounds are given below :
12.9.2. Nomenclature of different classes of Organic Compounds
Compounds of carbon and hydrogen are called hydrocarbons. These are further divided into two
classes : saturated and unsaturated hydrocarbons.
1. Saturated hydrocarbons — Alkanes
Hydrocarbons in which all the carbon atoms are linked to one another by only single bonds are called
saturated hydrocarbons. These may be either acyclic (open chain) or cyclic. In the lUPAC system, saturated

w
acyclic hydrocarbons are called alkanes. Earlier, they were also called paraffins (Latin : paruni = little ;
ajfmis = affinity) since they are relatively inert towards most of the chemical reagents.
General formula : C„H2„^2 where n = 1, 2, 3, 4 .... etc. Primary suffix: ane.
The lUPAC names of alkanes are obtained by adding the suffix ane to the word root indicating the

Flo
number of carbon atoms.

For example, names of alkanes containing five or more carbon atoms are obtained by adding prefixes
such as pent (five), hex (six), hept (seven), oct (eight) etc. indicating the number of carbon atoms in the

e
molecule to the suffix 'ane'. However, the first four alkanes (CH4 to C4Hjq) have their special names i.e.

re
methane, ethane, propane and butane. The name of an individual alkane is always written as one word. The
molecular formulae, lUPAC and common names of some alkanes are given below ;

F
n Formula Common name lUPAC name
ur
or
Methane Methane
1. CH4
2. Ethane Ethane
CH3CH3

f
3. CH3CH2CH3 Propane
ks Propane
Butane
4. CH3CH2CH2CH3 Ai-Butane
Yo
Pentane
5. CH3CH2CH2CH2CH3 n-Pentane
oo
6. n-Hexane Hexane
CH3(CH2)4CH3
7. CH3(CH2)5CH3 n-Heptane Heptane
B

8. n-Octane Octane
CH3(CH2)6CH3
re

9. n-Nonane Nonane
CH3(CH2)7CH3
«-Decane Decane
10. CH3(CH2)gCH3
Eicosane
20. CH3(CH2)|8CH3
u
ad

Triacontane
30. CH3(CH2)2sCH3
Yo

Types of alkanes. Depending upon the structure of the carbon chain, alkanes are of the following two types :
(i) Straight chain alkanes. As the name suggests, these alkanes contain straight chains of carbon atoms
in their molecules. In other words, alkanes in which no carbon atom is linked to more than two other carbon
d
Re

atoms are called straight chain alkanes


in

CH3—CH2—CHj—CH3 CH3—CH2—CH2—CH2—CH2—CH3
F

Butane (n-Burane) Hexane (n-Hexane)

The common names of straight chain alkanes are the same as their lUPAC names except that the prefix
n-(normal) is used in the common names (given within brackets),
(ii) Branched chain alkanes. In these alkanes, all the carbon atoms are not arranged in a straight chain.
In other words, alkanes in which at least one carbon atom is linked to three or four other carbon atoms are
called branched chain alkanes. For example,
CH,—CH—CH, CH3—CH—CH2CH3 CH3~CH—CH2CH2CH3
I
CH3 CH3 CH3
2-Mcthylpropane 2-Methylbulane 2-Melhylpenlane
(Isobuiane) (Isopentane) (Isohexane)
ORGANIC CHEMISTRY-SOME BASIC PRINCIPLES AND TECHNIQUES 12/21

CH,
I ^ CH3
CH-—C—CH, CH3—C—CH2CH3
3 I 3
CH3 CH3
2, 2-Dimelhylpropane 2, 2-Dimethylbutane
(Neopeniane) {Neohcxane)
To distinguish the straight chain alkanes from branched chain alkanes of the same molecular formula,
the prefixes iso and neo are used in the common system. The prefix iso is used when the second carbon of the
branched chain alkane carries one methyl group while the prefix neo is used when second carbon of the

w
branched chain alkane carries two methyl groups.
Types of Carbon and Hydrogen atoms in Alkanes
The carbon atoms in an alkane molecule may be classified into four types as primary (1°), secondary

F lo
tertiary (3°) and quaternary (4°) as follows :
(i) A carbon atom attached to one other (or no other) carbon atom is called a primary carbon atom
and is designated as 1® carbon.

ee
(ii) A carbon atom attached to tM>o other carbon atoms is called a secondary carbon atom and is

Fr
designated as 2® carbon.
(Hi) A carbon atom attached to three other carbon atoms is called a tertiary carbon atom and is
designated as 3® carbon,

for
(/v) A carbon atom attached to four other carbon atoms is called a quaternary carbon atom and is
ur
designated as 4® carbon.
The hydrogen atoms attached to 1®. 2° and 3® carbon atoms are r

called primary (1®), secondary (2®) and tertiary (3®) hydrogen atoms. It 1" CH3
s
.J r 3“ 1“
ok
may be noted here that there is nothing like quaternary hydrogen atom CH,—C—CH_—CH—CH,
Yo
since a quartemary carbon does not carry any hydrogen. The following 3 1 2 I 3
o

example clearly illustrates the various types of carbon and hydrogen CH3 1° CH3
eB

atoms. 1°

Alkyl groups. The removal of one hydrogen atom from the molecule of an alkane gives an alkyl group.
Thus, alkyl groups have the general formula, C„H2„+]. These are often represented by the letter R. Their
r

names are derived by replacing the terminal ‘ane’ of the corresponding alkane by the suffix 'yV.
ad
ou

Alkane—ane + yl = Alkyl
In case of propane and higher alkanes, the position of the carbon atom from which the hydrogen is
removed is also prefixed to the name of the alkyl group.
Y

The lUPAC names and structural formulae of some of the alkanes and their alkyl groups are given
below :
Re

General formula, C„H2„+i where n = 1, 2, 3, 4 .... etc.


nd

Name and structural Corresponding alkyl group lUPAC name


Fi

formula of the alkane (common name)

Methane, CH4 CH3- Methyl (Meihyt)


Ethane, CH3—CH3 CH3—CH2- Ethyl (Ethyl)
Removalof 1“H ^ 2 1

I* 2“ ]°
> CH3—CH^—CH2 — 1-Propyl (n-Propyl)
CH3—CH2—CH3
2
Propane Removal of 2®H
■> CH-—CH—CH.,
I j 2-Propyl (Isopropyl)
12/22 New Course Chemistry (XI)SSSIHn

Name and structural Correspondingalkyl group lUPAC name

formula of the alkane (common name)

Rcmovalof 1°H 4 3 2 l
l-Ldtyl (n-Butyl)
^ CH3CH2CH2CH2 —
i 2° r

CH3—CH2—CH2—CH3

ow
2 i
Removal of 2° H
Butane 4
CH3CH2—CH—CH3 2-Butyl (sec-Biityl)

CH3
Removal of 1°H 3 I
2-Methyl-l-propyl {Isobutyl)
-)
1“
> CH3—CH —CH2—
CH3

e
CH3^CH—CH3

re
CH3

rFl
Isobutane 1
Removal of 3° H ^ 0
2-Methyl-2-propyl (tert-Butyl)
->
CH3—C—CH3

F
I

Similarly, removal of different H-atoms in pentane gives the following alkyl groups.

r
ou
CH3CH2CH2CH2CH2- , (CH3)2CHCH2CH2- - (CH3CH2>2CH-, CH3CH2CH(CH3)CH2-

fo
3-MethyIbutyl 3-Pemyl 2-Methylbutyl
1-Pentyl
(n-Pentyl) (Isopentyl)

CH3
ks CH3
oo
1 2 1 2' 2 I ●
CH3—CH—CH2CH2CH3 CH3—C—CH2CH3 CH3—C—CH2 —
Y
I
eB

2-Peniyl 2-Methyl-2-buiyl CH3


(sec-Pentyl) (/erf-Pentyl) 2, 2-Dimethylpropyl
(Neopentyl)
r

2. Unsaturated hydrocarbons. Open chain hydrocarbons which contain carbon-carbon double


ou

(>C = C<) or triple (—C s C—) bonds in their molecules are called unsaturated hydrocarbons. These are
ad
Y

further classified into two types : alkanes and alkynes.


(i) Alkenes. Unsaturated aliphatic hydrocarbons containing a carbon-carbon double bond are called
alkenes. They are also called olefins (Greek : olefiant = oil forming) since the lower members of this class
d

react with chlorine to form oily products. Their general formula and suffix are given below :
Re

Primary suffix : = ene


in

General formula : Cj,H2„ where w = 2, 3, 4 .... etc.


Common names : Alkane-ane + ylene = Alkylene ; lUPAC names : Alkane-ane+ ene = Alkene

The position of the double bond is indicated by the Greek letters a, p, yetc. in the common system
F

while arabic numerals, i.e., 1, 2, 3, 4, etc. are used in the lUPAC system. For example.

n Formula Common name lUPAC name

Ethylene Ethene
2 CH2 = CH2
3 CH3CH = CHj Propylene Propene
P a

4 CHXH-CH=CH- a-Butylene But-l-ene


^ ^2 1
P a
p-Butylene But-2-ene
4
CH3CH =CHCH3
ORGANIC CHEMISTRY-SOME BASIC PRINCIPLES AND TECHNIQUES 12/23

n Formula Common name lUPAC name

l
5
CH3CH2CH2CH = CH2 a-Pemylene Pent-l-ene
2 I
5
CH^CH^CH = CHCH3 P-Pentyiene Pent-2-ene
2 [
6
CH3CH2CH2CH2CH =CH2 a-Hexylene Hex-l-ene
2 I
6
CH3CH2CH2CH = CHCH3 p-Hexylene Hex-2-ene
3

w
6
CH3CH2CH = CHCH2CH3 y-Hexylene Hex-3-ene

(ii) Alkynes. Unsaturated aliphatic hydrocarbons containing a carbon-carbon triple bond are called

F lo
alkynes : In the common system, they are called acetylenes after the name of the first member of this family,
i.e. acetylene.
General formula : C„H2„_2 where n = 2,2, 4,.... etc. ; Primary suffix = yne
Common names : Acetylene and its alkyl derivatives ; lUPAC names : Alkane—ane + yne = Alkyne

ee
The position of the triple bond on the parent chain is designated by the lowest possible arabic numeral.
The common and lUPAC names of a few simple alkynes are given below :

Fr
n Formula Common name lUPAC name

for
2 CHhCH Acetylene Ethyne
3 CH3 = CH Methylacetylene or Allylene Propyne
ur
4
CH3CH2—C^CH Ethylacetylene But-1-yne
s
ook
2 I
Yo
4
CH3—C=C—CH3 Dimethylacetylene Bui-2-yne
2 I
eB

5
CH3CH,CH2—CsCH /i-Propylacetylene Pent-1-yne

5
CH3CH2—C=C —CH3 Ethylmethylacetylene Peni-2-yne
our

2 I
ad

6
CH3CH2CH2CH2 -C =CH n-Butylacetylene Hex-1 -yne
2 ]
6
CH3CH2CH2—C = C—CH3 Methyl n-propylacelylene Hex-2-yne
Y

3 2 1
6
CH3CH2—C = C—CH2CH3 Diethylacetylene Hex-3-yne
Re
nd

3. Haloalkanes. Halogen derivatives of alkanes are called haloalkanes. They are further classified as
mono-, di-, tri- and tetrahaloalkanes. etc. according as they contain one, two, three, fou,r etc. halogen atoms
Fi

respectively in their molecules,


(a) Monohaloaikanes. The monohalogen derivatives of alkanes are called alkyl halides.
General formula : CnHjn+i X where « = 1, 2, 3 .... etc. and X = F, Cl, Br or I.
or R-X where R is any alkyl group
Functional group : X {halogen) ;
Secondary preifx = Halo
Common names. Add the word halide (fluoride, chloride, bromide, iodide) to the name of the alkyl
group, i.e., Alkyl + halide = Alkyl halide
lUPAC names. Add the secondary prefix halo to the name of the corresponding alkane.
i.e., Halo+alkane = Haloalkane
In case of dihalogen derivatives, the positions of both the halogen atoms are indicated.
12/24 New Course Chemistry (XI)S!EIHI1

The lUPAC and common names of some important haloalkanes are given below :
Formula Common name lUPAC name

Methyl chloride Chloromethane


CH3—Cl
Ethyl bromide Bromoelhane
CH3CH2—Br
3 2 1
//-Propyl iodide I-Iodopropane
CH3CH2CH2—1
3 2 1

CH3—CH—CH3 Isopropyl iodide 2-Iodopropane

w
(b) Dihaloalkanes. Alkanes containing two halogen atoms per molecule are called dihaloalkanes.
General formula : CnH2„X2 where n = 1,2, 3....etc.

F lo
Common names. For purpose of naming, dihalogen derivatives of alkanes have been divided into three
categories :
(0 Alkylidene dihalides. Dihalogen derivatives of alkanes in which the two halogen atoms are attached
to the same carbon atom are called alkylidene dihalides or simply alkylidene halides.

ree
Further since the positions on the same carbon atom are called geminal positions, therefore, alkylidene
dihalides are also called geminal dihalides or simply gem-dihalides.

F
(«) Alkylene dihalides. Dihalogen derivatives of alkanes in which the two halogen atoms are attached

for
to adjacent carbon atoms of the chain are called alkylene dihalides or simply alkylene halides. Further since
positions on the adjacent carbon atoms are called vicinal positions, therefore, alkylene dihalides are also
called vicinal dihalides or simply vic-dihalides.
r
(Hi) Polymethylene dihalides. Dihalogen derivatives of alkanes (containing three or more carbon atoms)
in which the two halogen derivatives are present on the terminal carbon atoms, i.e., a, w-positions of the
You
oks

carbon chain are called polyniethylene dihalides.


lUPAC names. In the lUPAC system, all types of dihalides are called dihaloalkanes, the positions of
eBo

the halogen atoms being indicated by lowest possible arable numerals.


The common and lUPAC names of some dihaloalkanes are given below :
Formula Common name lUPAC name
ad
our

Methylene chloride Dichloromethane


CH2CI2
CH3—CHBrj Elhylidene dibromide 1, 1-Dibromoethane

BrCHj—CH2Br Ethylene dibromide 1, 2-Dibromoelhane

CH3CH2CHCI2 Propylidene dichloride 1, l-Dichloropropane


Re
dY

2 1

CH3—CH—CH2 Propylene dichloride 1, 2-Dichloropropane


Fin

Cl Cl
2 I

CH,—C—CH.
3 3 Isopropylidene dichloride 2, 2-Dichloropropane
Cl Cl

3 2 1

Cl—CH2CH2CH2—Cl Trimethylene dichloride 1, 3-Dichloropropane

(c) Tri- and tetrahaloalkanes. The general formula of trihaloalkanes is C„H2„.iX3 while that of
tetrahaloalkanes is C„H 2n-2^4 where n = 1, 2, 3 etc. and X = F, Cl, Br, I.
lUPAC names. In the lUPAC system, these are called trihaloalkanes and tetrahaloalkanes. The
positions of the halogen atoms on the carbon chain being indicated by arabic numerals.
ORGANIC CHEMISTRY-SOME BASIC PRINCIPLES AND TECHNIQUES 12/25

Common or IVivial names. There are no systematic common names for tri- and tetiahaloalkanes. However,
trihalomethanes are best known by their trivial names i.e., haloforms, tetrahalogen derivatives of methane are
called carbon tetra^des. While symmetrical tetrahalogen derivatives of ethane are called acetylene tetrahalides.
The common (or trivial) and lUPAC names of some tri- and tetrahaloalkanes are given below:
Formula Common/lHvial name TUPAC name

CHF3 Fluoroform Trifluoromethane


CHCI3 Chloroform Trichloromethane
CHBr3 Bromoform Tribromomethane
CHI3 Iodoform Triiodomethane

w
CH3—CCI3 1, 1, 1-Trichloroethane
CICH2—CHCI2 1,1, 2-Trichloroethane

F lo
CCI4 Carbon tetrachloride Tetrachloromethane
Br2CH—CHBr2 Acetylene tetrabromide 1, 1, 2, 2-Tetrabromoethane
CICH2—CCI3 1, 1, 1, 2-Tetrachloroethane

ee
4. Alcohols or Alkanols. Alcohols are classified as monohydric, dihydric, trihydric and polyhydric
according as their molecules contain one, two, three and many hydroxyl groups respectively. Since presence

Fr
of two or more hydroxyl groups on the same carbon atom makes the molecule unstable, therefore, in di, tri
and polyhydric alcohols, each hydroxyl group is present on a different carbon atom.

for
(/) Monohydric alcohols.
General formula : C„H2„^.iOH (where n = 1, 2, 3 ...) or R—OH (where R is any alkyl group)
ur
Functional group : - OH {hydroxyl) Secondary suffix: ol
Conunon names. Add the word alcohol to the name of the alkyl group, i.e..
s
ook
Yo
Alkyl + alcohol = Alkyl alcohol
TUPAC names. Replace the terminal ‘ e' from the name of the corresponding alkane by the suffix *ol\
eB

i.e., Alkane -e + ol = Alkanol


Some important examples are :
Forhilila Common Name
our

TUPAC Name
ad

CH3—OH Methyl alcohol Methanol


CH3CH2—OH Ethyl alcohol Ethanol
3 2 1
Y

CH3CH2CH2—OH n-Propyl alcohol Propan-l-ol


Re
nd

3 2 1

CH3—CH—CH3 Isopropyl alcohol Propan-2-ol


I
Fi

OH

(//) Dihydric alcohols:


General formula : C„H2n (OH)2 where n = 2, 3, 4....etc.
Classification. Because of their sweet taste, dihydric alcohols are called glycols. Depending upon the
relative positions of the two hydroxyl groups, they are further classified as a, P, y..... (o glycols, etc. Thus, a-
glycol is 1, 2-glycol, P-glycol is 1, 3-glycol and ©-glycol is one in which the two OH groups are attached to
the terminal carbon atoms of the chain.
Common names. In the common system, OL-glycols (also called vic-glycols) are named by adding the
word glycol to the common name of the alkene from which they have been prepared by direct hydroxylation.
In contrast, p-, y- and ©-glycols are named as the corresponding polymethylene glycols.
12/26 'P'UKUe^'^. New Course Chemistry (Xl)k!isiMD

lUPAC names. Add the suffix 'dioV to the name of the alkane containing the same number of carbon
atoms as the diol.
Alkane + diol = Alkanedio!

The position of the two hydroxyl group is indicated by arabic numerals.


The common and lUPAC names of some glycols are given below :
Formula Common name lUPAC name

ow
1
Ethylene glycol Ethane-1, 2-diol
HOCH^—CH2OH
2 1
Propylene glycol Propane-1, 2-diol
CH3—CHOH—CH2OH
3 2 I
Trimethylcnc glycol Propane-1, 3-diol

e
HOCH^—CH2—CH^OH

Fl
re
(Hi) TVihydric alcohols

F
General formula : (^^^3
Common names. There is no general rule for naming these alcohols.
ur
lUPAC name. Add the suffix "triol' to the name of the alkane containing the same number of carbon

or
atoms as the triol.
Alkane + triol = Alkanetriol

The position of the hydroxyl groups is indicated by arabic numerals.


k sf
Yo
Formula Common name lUPAC name
oo
3 2 1

CH.—CH—CH,
eB

I - I I ^ Glycerol or Glycerine Propane-1, 2, 3- triol


OH OH OH

5. Ethers or Alkoxyalkanes
ur

General formula : R—O—R' where R and R' are same or different alkyl groups. If R = R', ethers are
ad

called simple ethers and if R tS: R', then ethers are called mixed ethers.
Yo

Functional group : - O - Secondary prefix: Alkoxy


Common names. In case of mixed ethers, add the word ether to the names of the alkyl groups arranged
in alphabetical order. In case of simple ethers, the numerical prefix di is added to the name of the alkyl group
d

followed by the word ether.


Re
in

lUPAC names. In the lUPAC system, ethers are called alkoxyalkanes. The smaller alkyl group forms a
part of the alkoxy group while the bigger alkyl group forms a part of the alkane. The names of the ethers are
F

then derived by adding the suffix alkoxy to the name of the alkane, i.e., Alkoxy + alkane = Alkoxyalkane
Some important examples are :
Fonnula Common name lUPAC name

CH3—O—CH3 Dimethyl ether Methoxymethane


CH3—O—CH2CH3 Ethyl methyl ether Methoxyethane
CH3CH2—O—CH2CH3 Diethyl ether Ethoxyethane

6. Monocarboxylic acids or Alkanoic acids


General formula : C„H2„+iCOOH where n = 0, 1,2,3 etc. or R-COOH where R = H or any alkyl
group.
ORGANIC CHEMISTRY-SOME BASIC PRINCIPLES AND TECHNIQUES 12/27

Functional group : —C—OH (carboxyl) Secondary suffix : oic acid


Common names. These are derived from the name of the plant or animal from which they were first
isolated.

lUPAC names. Replace terminal ‘e’ from the name of the corresponding alkane by the suffix oic acid,

w
i.e., Alkane -e ■¥ oic acid = Alkanoic acid
Some important examples are :
Formula Common name lUPAC name

o
H—COOH Formic acid Methanoic acid

CH3—COOH

e
Acetic acid Ethanoic acid

CH3CH2—COOH

re
Propionic acid Propanoic acid

rFl
CH3CH2CH2—COOH n-Butyric acid Butanoic acid

F
CH3CH2CH2CH2—COOH ^-Valeric acid Pentanoic acid

7. Aldehydes or Alkanals
General formula : C„H2„^.j CHO where n = 0, 1, 2, 3

r
etc. or R—CHO where R = H or any alkyl
ou
fo
group.

Functional group : —C—H (aldehyde) ks Secondary suiffix: al


oo
Common names. Replace ic acid from the common name of the corresponding acid which they give
upon oxidation by the word aldehyde. For example. Acetic acid - ic acid + aldehyde = Acetaldehyde
Y
eB

lUPAC names. Replace the terminal 'e' from the name of the corresponding alkane by the suffix al,
i.e.. Alkane -e + al = Alkanal
Some examples are :
r

Formula Common name lUPAC name


ou
Y
ad

H—CHO Formaldehyde Methanal

CH3—CHO Acetaldehyde Ethanal

CH3CH2—CHO Propionaldehyde Propanal


d

CH3CH2CH2—CHO n-Butyraldehyde Butanal


in
Re

8. Ketones or Alkanones

General formula : C„H2„^jCOC„H2,,^j where « = 1, 2, 3 .... etc. or R-CO-R' where R and R', may be
F

same or different alkyl groups. If R = R', ketones are called simple ketones and if R R', ketones are called
mixed ketones.

Functional group : ^ C = O (Ketonic) ;


Secondary suiffix : one
Common names. In case of mixed ketones, name the alkyl groups in alphabetical order and then
add the word ketone. In case of simple ketones, the numerical prefix di is used before the name of the alkyl
group.
lUPAC names. Replace terminal ‘e’ from the name of the corresponding alkane by the suffix one
i.e. Alkane -~e + one = Alkanonc
12/28 “P>uuUefr'A New Course Chemistry (XI)ESI

Some examples are:


Fornuda Common name

CH3—CO—CH3 Dimethyl ketone or Acetone Propanone


Ethyl methyl ketone Butan-2-one
CH3—CO—CH2CH3
1 2 3 4 5
Methyl /i-propyl ketone Pentan-2-one
CH3 —CO—CH2 CH2 CH3
1 2 3 4 5
Diethyl ketone Pentan-3-one
CH3 CH2 —CO—CH2 CH3

w
9. Acid chlorides or Acyl chlorides or Alkanoyl chlorides
General formula : RCOQ where R = H or any alkyl group

F lo
O

Functional group : —C—Cl (Chlorocarbonyl) Secondary suffix : oyl chloride


Common names: Replace ic acid from the common name of the corresponding acid by yl chloride For

ee
example. Acetic acid - ic acid + yl chloride = Acetyl chloride

Fr
lUPAC names : Replace terminal ‘e’ from the name of the corresponding alkane by the suffix oyl
chloride, i.e.. Alkane -e + oyl chloride - Alkanoyl chloride
Some important examples are :

for
Formula Common name JUPACname
ur
H—COCl (unstable) Formyl chloride Methanoyl chloride
oks
CH3—COCl Acetyl chloride Ethanoyl chloride
Yo
CH3CH2—COCl Propionyl chloride Propanoyl chloride
Butanoyl chloride
o

CH3CH2CH2—COCl n-Butyryl chloride


eB

10. Acid anhydrides


General formula: R— CO— O— CO—^R' or (RC0>20 where R or R' may be same or different alkyl
groups.
our
ad

O O

Functional group : —C—O—C— Secondary suffix: anhydride


Common or lUPAC names : Replace the word acid from the common or lUPAC name of the
Y

corresponding acid by the word anhydride.


Re

Symmetrical anhydrides of substituted carboxylic acids are named by adding the prefix bis to the name
nd

to indicate that two identical acyl groups are present. Unsymmetrical anhydrides are named by writing the
names of the two acids alphabetically before the word anhydride.
Fi

Some important examples are :


Foiinula Common name lUFAGhapnie

(CH3C0)20 Acetic anhydride Ethanoic anhydride


(CH3CH2C0)20 Propionic anhydride Propanoic anhydride
(C1CH2C0)20 Bis(chloroacetic anhydride) Bis(chloroethanoic anhydride)
HCO-O-COCH3 Acetic formic anhydride Ethanoic methanoic anhydride

11. Esters

Generalformula: R—COOR' where R = H or any alkyl group while group R' is always an alkyl group.
ORGANIC CHEMISTRY-SOME BASIC PRINCIPLES AND TECHNIQUES 12/29

Functional group : —C—OR' (ester) Secondary prefix: alkyl


Secondary suffix: oate
Common or lUPAC names : Write the name of the alkyl group before the common or lUPAC name of
the parent acid with its terminal ic add replaced by oate.
Some important examples are :
Formula Common name lUPAC name

w
H—COOCH3 Methyl formate Methyl methanoale
H—COOC2H5 Ethyl formate Ethyl nielhanoate
CH3—COOCH3 Methyl acetate

F lo
Methyl ethanoate
CH3--COOC2H5 Ethyl acetate Ethyl ethanoate

ee
12. Acid amides or Alkanamides

General formula : RCONH2 where R = H or any alkyl group

Fr
O

Functional group : —C—NH2 (amide) Secondary suiffx: amide

for
ur
Common names : Replace ic acid from the common name of the corresponding acid by the secondary
suffix amide.

lUPAC names : Replace the terminal V’ from the name of the corresponding alkane by the suffix
s
amide, i.e., Alkane ~ e + amide = Alkanamide
k
Yo
Some important examples are :
oo
Formula Common name lUPAC name
eB

H—CONH2 Formamide Methanamide

CH3—CONH2 Acetamide Ethanamide

CH3—CH2—CONH2 Propionamide Propanamide


r
ou
ad

13. Primary Amines


General formula : R—NH2 where R is any alkyl group
Y

Functional group : —NH2 (amino) ; Secondary suffix: amine


Common names : (/) Add the word amine to the name of the alkyl group, i.e..
nd
Re

Alkyl + amine = Alkylamine


(ii) Attach the prefix amino to the name of the corresponding alkane, i.e..
Fi

Amino + alkane - Aminoalkane


lUPAC names. Replace the terminal 'e' from the name of the corresponding alkane by the secondary
suffix amine, i.e., Alkane - e + amine = Alkanamine
Some important examples are :
Formula Common name lUPAC name

CH3—NH2 Methylamine or Aminomethane Methanamine

CH3CH2—NH2 Ethylamine or Aminoethane Ethanamine

3 2 1

CH3CH2CH2—NH, rt-Propylamine or 1-Aminopropane Propan-l-amine or Propanamine


12/30 7^n4i<Ue^'^ New Course Chemistry (XI)insIHIl

14. Secondary amines


General formula ; R—NH — R' where R and R' may be same or different alkyl groups.
Functional group : / NH (imino) ; Secondaty prefix : N-Alkyl ; Secondary suffix : amine
Common names : {/) Name the alkyl groups in alphabetical order and then add the word amine. In case

ow
the two alkyl groups are the same, the numerical prefix di is used before the name of the alkyl group.
{ii) Add the prefix N-alkyl before the name of the aminoalkane ; the smaller alkyl group forms a part of
the N-alkyl group while the larger alkyl group forms a part of the alkane.
lUPAC names : Add the prefix N-alkyl to the name of the alkanamine corresponding to the larger alkyl
group, i.e., N-Alkyl + alkanamine = N-Alkylalkanamine
Some Important examples are :

e
Formula Common name lUPAC name

re
CH3NHCH3 or (CH3)2NH Diniethylamine or N-Methylaminomethane N-Methylmethanamine

Frl
F
CH3CH2NHCH3 Ethylmethylamine or N-Methylaminoethane N-Methylethanamine
(CH3CH2)2NH Diethylamine or N-Ethylaminoethane N-Eihylethanamine
ou
r
15. Tertiary amines

so
R'

kf
General formula : R— N—R" where R, R', R" may be same or different alkyl groups or two of them
may be same while the third may be different.
oo
‘Secondary preifx : N-alkyl, N-alkyl
Y
Functional group : N— (tertiary nitrogen atom) Secondary suffix : amine
B

Common names. (1) Name the alkyl groups in alphabetical order and add the suffix amine. If two or all
the three alkyl groups are same, the numerical prefixes di and tri are respectively used.
re

(ii) Add the prefixes N-alkyl and N-alkyl (smaller alkyl groups) to the name of the aminoalkane
oY

corresponding to the largest alkyl group.


u

lUPAC names. Add the prefixes N-alkyl and N-alkyl (smaller alkyl groups) to the name of the alkanamine
corresponding to the largest alkyl group.
ad

Some important examples are :


d

Formula Common name lUPAC name


in

(CH3)3N Trimethylamine or N, N-Dimethylaminomethane N, N-Dimethylmethanamine


Re

CH3CH2N{CH3)2 Ethyldimethylamine or N. N-Dimeihylaminoethane N, N-Dimethylethanaminc


(CH3CH7)2NCH3 Diethylmethylamine or N-Ethyl-N-methylethanamine
F

N-Ethyl-N-meihylaminoethane
(CH3CH2)3N Tricthylamine or N, N-Diethylaminoethane N, N-Diethylethanamine

16. NUroalkanes

General formula : R—NO2 where R is any alkyl group.


O O
Functional group : —N or —N (nitro) Secondary prefix: nitro
O"
Common names. There are no comnjon names for nitroalkans.
lUPAC names. Add the secondary prefix nitro to the name of the alkane, i.e.,
Nitro + alkane = Nitroalkane
ORGANIC CHEMISTRY-SOME BASIC PRINCIPLES AND TECHNIQUES 12/31

Some important examples are :


Formula lUPAC name Formula lUPAC name

NO2
CH3—NO2 Nitromethane 2-Nitropropane
CH3—"CH—CH3

ow
1
CH3CH2—NO2 Nitroethaiie
CH3CH2CH2CH2—NO2 1- Nitrobutane

I
NO,
1- Nitropropane I 2 I
CH3CH2CH2—NO2 CH3—CH—CH2CH3
2- Nitrobutane

e
re
17. Alkyl nitrites

rFl
General formula : R—O—N = O where R is any alkyl group

F
Functional group : —O—N = O {nitrite) ; Secondaiy sufifx : nitrite
Common names : Add the secondary suffix nitrite to the name of the alkyl group, i.e..

r
Alkyl + nitrite = Alkyl nitrite
ou
fo
lUPAC names. There are no lUPAC names for alkyl nitrites.
Formula

CH3—O— N = 0
Common name

Methyl nitrite
Formula

CH3CH2CH2—O—N = O
ks Common name

n-Propyl nitrite
oo
O—N = 0

CH3CH2- ■N = 0
Y
Ethyl nitrite Isopropyl nitrite
CH3—CH—CH3
eB

18. Alkyl cyanides or Alkanenitriles


General formula : R—C = N where R is any alkyl group
r

Functional group : — C s N (cyano or nitrite) ;


ou
Y
ad

Secondary sufifx : nitrile


Common names : (/) Add the suffix cyanide to the name of the alkyl group, i.e..
Alkyl + cyanide = Alkyl cyanide
d

{//) Replace ic acid from the common name of the corresponding acid by the suffix onitrile. For example,
Acetic acid - ic acid + onitrile = Acetonitrile
in
Re

However, in case of propionic acid, onic acid is replaced by onitrile. For example,
propionic acid - onic acid + onitrile = Propionitrile.
F

lUPAC names. Add the suffix nitrile to the name of the alkane containing the same number of carbon
atoms as the alkyl cyanide, i.e.. Alkane + nitrile - Alkanenitrile
Some important examples are ;
Formula Common name lUPAC name

CH3. :n Methyl cyanide or Acetonitrile Ethanenitrile

CH3CH2—CN Ethyl cyanide or Propionitrile Propanenitrile


CH3CH2CH2—CN M-Propyl cyanide or n-Butyronitrile Butanenitrile
12/32 4 New Course Chemistry (XI)

19. Isocyanides or Isonitriles or Carbylamines


+
General formula : R—N = C or R N sC where R is any alkyl group

Functional group : —N = C or —NsCr (isocyanidc or isonitrile)


Secondary sufifx : isocyanide or isonitrile
Common names. Add the suffix isocyanide or carbylamine to the name of the alkyl group.
lUPAC names. There are no lUPAC names for isocyanides or isonitriles.
Some important examples are :

w
Formula Common name

CH3—N=C Methyl isocyanide or Methyl carbylamine or Methyl isonitrile

F lo
CH3CH2—N=C Ethyl isocyanide or Ethyl carbylamine or Ethyl isonitrile

12.9.3. Rules for lUPAC Nomenclature for Branched Chain Alkanes

ee
The following rules are used for naming branched chain alkanes.

Fr
1. Longest chain rule. Select the longest continuous chain of carbon atoms. This is called the parent
chain while all other carbon atoms which are not included in the parent chain are called branch chains or side
chains or substituents. The branched chain alkane is then named as a derivative of the parent chain. It may be

for
noted that the longest chain may or may not be straight but it must be continuous. For example,
ur
CH3 CH^CH^j
CH3H-CH -CH—CH->—CH3
s
iCHj—CH2—CH—CH2—CH2—CH3j
: i:;;: :
ook
Yo
I
:CH2—CH2—CH3:
Longest chain contains six carbon atoms and
hence (1) is named as a derivative of hexane II
eB

Longest chain contains seven carbon atoms and


hence (II) is named as a derivative of heptane

2. Rule for larger number of side chains. If two chains of equal lengths are possible, select the one
with the larger number of side chains. For example,
r
ad
ou

|_CH3—CH— CHCH2CH2CH3j CH3—CH“t CHCH2CH2CH3

CH3 CH2CH3 CH3 i CH2CH3 :


Y

Named as hexane with two alkyl Named as hexane with one alkyl
Re

substituents (co/rect) substituent (wron^')


nd

3. Lowest number rule. Number the carbon atoms of the parent chain as 1,2, 3,4.... etc. starting from
that end which gives the lowest possible number to the carbon atom carrying the substituent. For example, in
Fi

structure (I), the numbering can be done in two different ways, t.e. lA and IB. The numbering of the carbon
chain as given in the structure lA is correct since it gives a lower number, i.e. 3 to the substituent, i.e., methyl
group while the numbering as given in structure IB is wrong since it gives a higher number, i.e., 4 to be
substituent.

CH3 CH3
1 2 o I 4 5 6 6 54I 3 2 1

CH3CH.,—CH—CH2CH2CH3 CH3CH2—CH—CH2CH2CH3
lA (correct) IB (wrong)
The number that indicates the position of the substituent on the parent chain is called the positional
number or the locant. Thus, the correct locant for the methyl side chain in structure (I) is 3.
ORGANIC CHEMISTRY-SOME BASIC PRINCIPLES AND TECHNIQUES 12/33

Lowest set of locants rule. When two or more substituents are present, the lowest set of locants rule
is applied. According to this rule, when two or more different sets of locants containing the same number of
terms is possible, then that set of locants is the lowest which when compared term by term with other sets,
each in order of increasing magnitude, has the lowest term at the first point of difference.
For deciding the lowest set of locants, the carbon atoms of the parent chain are numbered from all
possible directions and a locant assigned to each substituent from each direction. The set of locants from each
direction is then compared term by term till the first point of difference is reached. That set of locants is
preferred which has a lower number at the first point of difference. That is why this rule is also soiiietimes
called as first point of difference rule.
Consider, for example, the following alkanes. Each alkane can be numbered in two different ways as
shown in structures I and II.

2 1 6 7
CH.CH, CH.CH,
I ^ ^

F low
I
CH- -^CH—CH—CHXH. CH,—CH—CH—CH,CH,
3 4I 2 3 3 g 4j 2 3
CHXH.CH- CH.CH,CH,
5 6 7 3 ^2
I (correct) II (wrong)
Set of locants = 3,4 Set of locants = 4,5

Out of two sets of locants (3, 4) and (4, 5), the first set is lower and hence preferred because the first

r e
term, i.e., 3 in the first set (3,4) is lower than the first term, i.e., 4 in the second set (4, 5).

for F
CH CH
CH, CH, r**3 T“3
1 2' 3 4I ^ 5 5 41 3 21 1

CH3—C—CH2—CH—CH3
07) CH3—C—CH2—CH—CH3
I
CH3 CH3
Your
Set of locants = 2, 2, 4 (correct) Set of locants = 2, 4, 4 (wrong)
s
eBook

1 2 3 4 5 6 7 8 <— Correct numbering


(«0 CH3—CH—CH2—CH2—CH—CH—CH2—CH3
I
CH3 CH3 CH3
ad

Set of locants = 2,5,6


our

8 7 6 5 4 3 2 1 4— Wrong numbering
CH3—CH—CH2—CH2—CH—CH—CH2—CH3
I
CH3 CH3 CH3
Re

Set of locants = 3,4,7


Here, out of two sets of locants (2,5,6) and (3,4,7), the first set is preferred because the first term, i.e.,
Y

2 in the first set (2, 5, 6) is lower than the first term, i.e., 3 in the second set (3,4, 7). Thus, the correct name
Find

of the alkane is 2, 5, 6-trimethyloctane.


4. Name of the branched chain alkane. Prefix the name of the substituent (i.e., the alkyl groups) to the
name of the parent alkane and indicate its position (on the parent chain) by writing before it the number of
the carbon atom carrying the substituent. The name of the substituent is separated from its locant by a
hyphen (-). The final name of the alkane is always written as one word. Some examples are given below for
illustration.

CH, CH2CH3
1
2J1 ^ 3 4 12 ,1 456
CH3—CH—CH2CH3 CH3CH2—CH —CH2CH2CH3
2-Methylbutane 3-Ethylhexane
12/34 New Course Chemistry (XI)ESEZ

5. Alphabetical order of the side chains. W/ten two or more alkyl groups (side chains) are present on
the parent chain, each alkyl group preifxed by its positional number is arranged in alphabetical order
(irrespective of its positional number) before the name of the parent alkane. For example,
2 1
CH.—CH-
I ^ ^
1 2 3 4 5 6
3' 4
CH3 —CH—CHCH2CH2CH3 CH3—CH —CH —CH2CH3

w
CH3 CH2CH3 CH.—CH.—CH,
5 6 7

3-Ethyl-2-methylhexane 4-Ethyl-3-methyIheptane
It may be noted here that while deciding the alphabetical order of the various alkyl groups, prefixes iso

Flo
and neo are considered to be part of the fundamental name of the alkyl group while the prefixes sec and tert
are not. For example.

e
CH(CH3)2

re
123 4I 5 6789 10
CH3CH2CH2—CH — CH—CH2CH2CH2CH2CH3

F
ur CH3—CH—CH2CH3
5-5ec-Butyl-4-isopropyIdecane

r
6. Numbering of different alkyl groups at equivalent positions. If two different alkyl groups are

fo
present at equivalent positions, the numbering of the parent chain is done in such a way that the alkly group
which comes first in the alphabetical order (written first in the name) gets the lower numbe.r For example.
ks
Yo
1 2 3 4 5 6 7 ^Correctnumbering
CH3 —CH2 —CH—CH2 —CH—CH 2 —CH3
oo
7 5I 4 3I 2 1 ^ Incorrect numbering
CH2CH3 CH3
B

3-Ethyl-5-methylheptane
re

7. Naming same alkyl groups at different positions. When the same alkyl group occurs more than
once on the parent chain at different positions, the positional number of each alkyl group is separated by
u

commas and suitable preifxes such as di (for two), tri (for three), tetra (for four), etc. are attached to the
ad

name of the alkyl group. However, the prefixes di, tri, etc. are not considered while deciding the alphabetical
Yo

order of the alkyl groups.

CH3 CH3 CH, CH,


5 4 3 2I 1 6 5 4I 3 2 11 2I 3I
d

CH3—CH—CH2 -C—CH3 CH3—CH2^—CH2—CH—CH3 CH3 —CH — C—CH.CH,


Re

I 4I
in

CH3 CH3 CH3—CH2 CH3 CH3—CH— CH.—CH,


5 6 4
F

2, 2, 4-Trimeihylpentane 4-Ethyl-2, 4-dimethylhexane 3-Ethyl-2, 3, 4-trimethylhexane


In case the same alkyl group occurs twice on the same carbon atom, its positional number is also
repeated twice. For example.
CH
CH2CH3 CH, CH
12I ^3 12 3I 4 5 1 2I 3 4 5 1^6
CH,—C—CH, CH3 CH2 —C—CH2 CH3 CH, _C—CH—CH. —CH—CH,
3 I 3 3 I 1 2 3
CH3 CH2CH3 H3C CH2CH3
2, 2-Dimethylpropane 3, 3-Diethylpentane 3-Ethyl-2, 2, 5-trimethylhexane
ORGANIC CHEMISTRY-SOME BASIC PRINCIPLES AND TECHNIQUES 12/35

8. Numbering the complex substituent, (a) In case the substituent on the parent chain is complex
(i.e., it has branched chain), it is named as a substituted alkyl group by numbering the carbon atom of this
group attached to the parent chain as 1. The name of such a substituent is always enclosed in brackets to
avoid confusion with the numbers of the parent chain. For example.
r n
CH3
1 2 3 4 5 6 1 2I 3

CH3 CH—CH—CH.—CH,—CH -CH.—CH—CH, ■ -Complex


II 2 2 I■

ow
I ? substituent
CH3 CH3 CH2—CHj
7 8
CH~—CH- {2-methylpropyl)
9 10

2, 3-Dimethyl-6-(2-methylpropyl)decane
r n

I 3CH,
2I

e
I CH,—C—CH, I—Complex substituent

Fl
re
I (2, 2-dimethylpropyt)
I

F
L J
1 2 3 4 5| 6 7 8 9

CH3—CH2—CH2—CH2—CH—CH2 ur CH2—CH2—CH3

r
5-(2, 2-Dimethylpropyl)nonane

fo
(b) If two complex substituents are of equal length, then the complex substituent with larger number of
alkyl groups forms a part of the longest carbon chain while the other one is considered the real complex
substituent. For example, ks
Yo
r “I
CH,—CH,
I I
oo
10 9 8 7 6 5 1 2 I 3 4

CH—I CH,- CH—CH2


CH3 —CH2 —CH2 —CH2 —CH2
I —CH3J
L-1.
eB

Complex substituent
4CH2
(2-ethylbutyl)
CH3—C—CH3
ur

2CH2-CH3
1
ad
Yo

5-(2-Ethylbutyl)-3, 3-dimethyldecane (correct)


5-(2, 2-Dimethylbutyl)-3-ethyIdecane (incorrect)
(c) While deciding the alphabetical order ofthe various substituents, the name ofthe complex substituent
d

is considered to begin with the first letter of the complete name. For example,
Re
in

CH, CH,
3 2I 1 I
CH3—CH—CH <-Note
F

6 7 8 9 10

CH3CH2CH2CH2 —CH—CH—CH2 CH2 CH2 CH3


Note-^ CH2CH3
5-( 1, 2-Dimethylpropyl)-6-ethyldecane
It may be noted here that the complete name of the complex substituent is dimethylpropyl. Since d of
dimethylpropyl group comes first than e of the ethyl group in the alphabetical order, therefore, locant 5 is
given to the complex substituent and 6 to the ethyl group.
(d) If the same complex substituent occurs more than once on the parent chain at different positions,
prefixes bis (for two), tris (for three), tetrakis (forfour), pentakis (forfive), etc. are used before the name of
the complex substituent. For example,
12/36 New Course Chemistry (XI)EEiaD

CH3
CH —CH CH,
3 I I 3
6 5 4 3I 2 I

CH3—CH2—CH2—C — CH—CH3
CH3—CH—CH3
2-MethyI-3, 3-bis( 1 -methylethyl)hexane
r "I
CH3

w
I 3 2 ll
CH_—CH,—C—CH,
L. 1
10 9 8 7 6 5l 4 3 2 1

CH3—CH2—CH2—CH2—CH2—c—CH2—CH2—CH—CH3

F lo
T 2 r\ n

I CH3—CH2—c—CH3! CH3

ee
L
CH3 J

Fr
5, 5-Bis (1, l-dimethylpropyl)-2-methyldecane

Sample Problem 12.7


Structures and lUPAC names of some hydrocarbons arc given below.

for
Explain why the names given in the parenthesis are incorrect. (NCERT Solved Example)
ur
(0 CH3 CH—CH2—CH2—CH—CH—CH2—CH3
I I
CH3 CH3 CH3
ks
2, 5, 6-lVimetliyloctane
Yo
oo
(and not 3, 4, 7*Trimetbyloctane)

(m) CH3—CH2—CH—CHj CH—CHj—CH3


eB

CH3CH2 CH3
3>Ethyl*5>methyiheptane
r

(and not 5>Ethyl-3-niethy]heptane)


ou
ad

Solution. (0 The set of locants (2, 5, 6) is lower than the set of locants (3,5, 7).
{/i) When substituents are at equivalent positions, lowest locant is given to that substituent which comes
first in the alphabetical order. Therefore, lower locant 3 is assigned to ethyl and higher locant 5 is assigned to
Y

methyl group.
Re
nd
Fi

1. Give the lUPAC names of the following alkanes :


H,C
3 C2H5 H C4H9
(0 CH3—CH—CH—CH3 (,7) CH3CH2—C—CH—C2H5 (Hi) CH3—C — C—CH3
C2H3C2H3 CH2CH2CH3 C2H3 CH3
CH3
(,v) (CH3)3CCH2CH2CHCH2CH3 (v) CH3CH2CH2CHCH2CH2CH3
C(CH3)3
ORGANIC CHEMISTRY-SOME BASIC PRINCIPLES AND TECHNIQUES 12/37

CH3 C2H5 CH3


(v/) CH3CH2CHCH2CH2CH—CHj—C—CH3 (v») CH3CH2CH —CH—CH2CH2CH—CH3
CH3 CH—CH3 CH3 CH3—C CH3
CH2—CH3 CH3
CH(CH3)2
(vii7) CH3CH2CH2CH2CHCH2CH2CH2CH3 (u:) CH3CH2CH2—CH —CH—CH2CH2CH2CH2CH3

w
CH3—CH—CH(CH3)2 CH3—CH—CH2CH3
(NCERT)

F lo
2. What is wrong with the following names ? Draw the structures they represent and give their correct names.
(1) 1, 1-Dimethylpentane (ii) 2-Methyl-2-propylhexane (Hi) 3-Dimethylpentane
O'v) 4, 4-Dimethyl-3-ethylpentane (v) 4-(2-Methylethyl)heptane

ee
ANSWERS

1. (/) 3, 4-Dimethylhexane ; (ii) 3, 4-DiethyM-methylheptane : (Hi) 3, 4, 4-Trimethyloctane ; (iv) 2, 2, 5-

Fr
Trimethylheptane ; (v)4-{l, 1-Dimethy!ethyl)heptane ; (vi) 2, 2, 7-Trimethyl-4-(I-methyIpropyl)nonane;
(vii) 5-(i, i-DimethylethyI)-6-ethyl-2-methyloctane ; (vHi) 5-(l, 2-DimethyIpropyl)nonane ; (ix) 5-sec-
Butyl-4-isopropyldecane or 4-(l-Methylethyl)-5-(I-methylpropyl)d ecane

for
CH3 CH3
ur
1 2 3 4 5 6 8 7 6 5 4 '
2. (0 CH3—CH—CH2CH2CH2CH3 (ii) CH3CH2CH2CH,—C—CH3
2-Methylhexane
s
CH.CH^CH,
ook
3 2 ■ 1
Yo
4. 4-Dimethyloctane
CH3 CH,
eB

1 2 3I 4 5 1 2 I ■ 3 4 5

(Hi) CH3CH2—C—CH2CH3
I
(/v) CH,—C—CH—CH,—CH,
3 1 ! 2 3
CH3 H3C CH2CH3
r
ad
ou

3, 3-Dimethylpentane 3-Ethyl-2, 2-dimelhylpentane


CH^CH^CHg
1 2 3 4I " 5“ 6 7
(v) CH3CH2CH2—CH—CH2CH2CH3
Y

4-Propylhepiane
Re
nd

12.9.4. Rules for lUPAC Nomenclature of Unsaturated Hydrocarbons (Alkenes and Alkynes)
Fi

While naming compounds containing multiple (double and triple) bonds, the following additional rules
are followed :
1. The parent chain must contain the multiple bond regardless of the fact whether it also denotes the
longest continuous chain of carbon atoms or not. For example, in structure (I), the parent chain consists of
Jive carbon atoms and not six carbon atoms since the latter does not include the double bond,
""s 4 3 2 1
-| I~6 5 4 71
^H3CH2CH2-C =L_
CH2
C = CH,
-I—'-j
CH2—CH3 CH,
U ^ J J
I I

Parent chain contains five carbon atoms (correct) Parent chain contains six carbon atoms (n’«)ng)
12/38 T^eidee^'^ New Course Chemistry fxmggwrn

2. If both double and triple bonds are present, the numbering ofthe parent chain should always be done
from that end which is nearer to the double or the triple bond, i.e., the lowest set of locants n^e^r the
multiple bonds must be followed. For example,
1 2 3 4 8

CH3—CH =CH—CH2—C = C—CH^—CH3


Set of locants = 2, 5 {correct)
8 7 6 5 1

CH3—CH =CH—CH2—C = C—CH2—CH3


Set of locants = 3,6 (wrong)

w
3. I,f however, there is a choice in numbering, the double bond is always given preference over the
triple bond. For example,
5 4 3 2 1 <— Correct numbering

F lo
CH = C—CH2—CH = CH2
1 2 3 4 5 Wrong numbering
4. If the organic compound contains only one double or the triple bond, its locant or the positional

ee
number is always placed before its suffix in accordance with 1993 recommendations for lUPAC nomenclature
of organic compounds. For example.

Fr
1 2 3 4

CH3—CH = CH—CH3
But + 2-ene = But-2-ene

for
If, however, both double and triple bonds are present, their locants are written before their respective
ur
suffixes, the terminal ‘e’ from the suffix 'ene* is dropped while writing the complete name of the organic
compound. It may be emphasized here that the organic compound is named as derivative of alkyne rather
than alkene. For example.
ks
5 4 3
Yo
CH3—CH = CH—C H CH
oo

Pent + 3-en (/) + 1-yne = Pent-3-en-l-yne


eB

These rules are further illustmted by the following additional examples:


CH, CH3
1 2 I 3 4
1 2 3I 45
r

CH2 = C—CH = CH2 HC = C—CH—C = CH


ou
ad

2-Methylbuta-l, 3-diene 3-Methylpenta-l, 4-diyne


In some cases all the double and triple bonds present in the molecule cannot be included in the longest
Y

chain. In such cases, the following prefixes are used for double and triple bonded groups.
CH2 = CH3CH = CH2 = CH- HCsC-
Methylene Ethylidene Vinyl or ethenyl Ethynyl
Re
nd

CH2
1
Fi

7 3 2 1
For example, CH2 = CH—C—CH=CH2 CH3—C s C—CH—CH2—CH=CH2
I
3-MethyIenepenta-l, 4-diene CH=CH2
4-Ethenylhept-l-en-5-yne or 4-Vinylhept-l-en-5-yne
1 2 3 4 5 7 6 5 4 3
CH-=CH—CH=CH—CH—C s CH CH-,=CH—CH=CH—CH—C s CH
^ I ^ I
CH = CH, CH=CH2
6 1 ^ 2 1

5-Ethynylhepta-l, 3, 6-triene (correct) 3-Ethynylhepta-l, 4,6-triene (wrong)


(double bonds are given preference over the (double bonds are given preference over the
triple bond and lowest set of locants (1, 3, 6) triple bond but a higher set of iocants (1, 4, 6)
is assigned to the three double bonds) is assigned to the three double bonds)
ORGANIC CHEMISTRY-SOME BASIC PRINCIPLES AND TECHNIQUES 12/39

1. Give the lUPAC names of the following compounds :

CH, CH
I ^
3 CH3
(/) CH^CH CH,~C—CH,—C = CH2 (//) CH 3 ●CH—C = CH (///) C(,H5—CH = CH- CH.Cl

w
C2H5
C!

(/V) CH3CH = CH—CH2Br (v) CH, = C—CH =CH,

lo
CH^CH,
I '

e
/
(W) CH., = CH—CH—C = CH,

re
rF
Cl

F
2. Give the condensed and bond-line siruclurai formulae for the following :
{a) 2-Melhylbuta-l, 3-diene {b) Penta-l, 4-diene (t-) Hexa-I, 3, 5-triene (ci) 3-Ethylpenta-l, 3-diene

r
ANSWERS

fo
u
1. (/) 4-Ethyl-2,4-dimethylhepl- 1-ene; (//) 3-Meihylbut-1 -yne; {Hi) 3-Chloro-1-phenylprop-1-ene; (iv) 1 -Bromo-
but-2-ene ; (v) 2-Chlorobuta-l, 3-diene ; (vt) 2-ChIoro-3-ethylp enia-1, 4-diene ; (v«) 5-Propyloct-2-yne
ks
CH3
Yo
2 4
1 2I
oo
1
2. {a) CH,=C—CH=CH,,
3 4 “
3
eB

2 4
i 2 3

(fo) CH2 =CH—CH2—CH=CH2’ 5


ur

3 5

CH = CH—CH =CH2’ ’VA./\


3 2 3 4 5
ad

(c) CHj =CH


Yo

2 4 6

5
d

1
{d) CH3—CH =C—CH =CH2
I
Re
in

C2H5
F

12.9.5. Rules for lUPAC Nomenclature of Compounds containing


one Functional Croup, Multiple Bonds and Substituents
While naming organic compounds containing one functional group, double and triple bonds, and
substituents, the following additional rules are observed.
1. Parent chain. Select the longest possible chain of carbon 4 3 2 :
atoms containing the functional group and the maximum number of CH3— CH-,—CH5— CHo— CH3
1 2 " 3 ; 4 " 5
multiple bonds as the parent chain without caring whether it also 1
denotes the longest possible carbon chain or not. For example, in CH20Hi
compound (1). the parent chain containing the functional group has I
Pareni chain contains four rather
four carbon atoms while the longest possible carbon chain has five than five carbon atoms.
carbon atoms.
12/40 'pfutdeefo’^ New Course Chemistry (XI)iSsI9D
Similarly, in compound (II), the parent chain containing the functional group and the double bond has
six carbon atoms while the longest possible carbon chain has seven carbon atoms.
*5 6
O CH=CH2:
1 2 3 4l i-i 6 7
CH3—c—CH— CHf— CH2CH2CH3
1
CH3
II
Parent chain contains six rather

w
than se ven carbon atoms.

2. Lowest locant rule for the functional group. Number the parent chain in such a way that the
junctional group gets the lowest locant, even if it violates the lowest set of locants rule for substituents.

F lo
For example, in compound (III), the lowest locant for the functional group, i.e., = O is 3 and not 4.
O O
6 5 4 3II 2 1 I 2 3 4II 5 6

CH3 _CH—CH2 —C—CH2 CH3 CH3 CH—CH2—C—CH2CH3

ee
I

Fr
CH3 CH3
III (correct) in (wrong)
(>C = O group gets lowest number 3) (> C = O group gets number 4 which is not lowest)

for
Similarly, for compound (IV), correct numbering gives position 1 to the functional group and not 5
ur
though it violates the lowest set of locants rule.
CH, CH,
5 4I ^3 2I ^3
s
2 1 4
ook
CH,—C—CH,—CH—CH, CH3—C—CH2—CH—CH3
Yo
^ \ ^ ,1 ’ 5'
CH3 *CH20H CH3 CH2OH
eB

rv (correct) IV (wrong)
Set of locants = 2,4,4 Set of locants = 2,2,4
If the parent chain in addition to the functional group also contains a double or a triple bond or both, the
locants for the multiple bond/s is/are automatically fix^ once lowest locant is assigned to the functional
r
ou
ad

group. For example, in compound (V), keto group gets locant 2, double bond gets locant 3 while triple bond
gets locant 6
Y

1 4 5 6 7 8

CH3—C—CH = CH—CH2 C S C—CH3


Re
nd

o
Oct-3-en-6-yn-2-one (V)
Fi

If, however, both double and triple bonds do not form a part of the parent chain, then the lowest locant
is assigned to either the double or the triple bond which ever is nearer to the functional group. For example,
in both the compounds (VI and VII), the lowest locant for the functional group, i.e., keto group is 2. However,
in compounds (VI), double bond gets the lowest locant 4 while in compound (VII), it is the triple bond which
gets the lowest locant 4. Thus,
4 5 6 :i 2 3 :
O CH=CH—CH3 O :CH2CH=CH2; Substituent
CH3—C—CH—;CH2—C=CH J Substituent
II ^1
1 2 3 1 CH3—c—CH—C=C—CH3
1 2 3 4 5 6

3-(Prop-2-ynyl)hex-4-en-2-one (VI) 3-{Prop-2-enyl)hex-4-yn-2-one (VII)


ORGANIC CHEMISTRY-SOME BASIC PRINCIPLES AND TECHNIQUES 12/41

3. Numbering the chain terminating functional groups. When a chain terminating functional group
such as -CHO, -COOH, -COOR, -CONH2, -COCl, -C=N, etc. is present, it is always given number 1 and
number 1 is usually omitted from the final name of the compound when there is no ambiguity. For example,
4 3 2

CH3—CH2 —CH—CH2—CH3
1 COOH

2-Ethylbutan-l-oic acid or simply 2-Ethylbutanoic acid

w
However, in the following examples, the numerical locant 1 is always included when another numerical
locant appears in the same name.

CH,

F lo
5 1 4 3 2I 4 3 2
CH, —CH—C—CH—CH, CH,—CH—CH—CH,
CH3—CH2—C —CH2CH3 3 I I 3
M II I ^
Cl 1iCOOH
CH3O O OC2Hg CHO

ee
1

Fr
3-Ethoxy-4-methoxypentan-3-one 2-Ethyl-2-methylbutan-1 -al 3-Chloro-2-methylbutan-l-oic acid

Br O
4 3 I2 111 4

for
CH3—CH2—CH—C—OC2H5 CH3—CH = CH—CH2OH
ur
Ethyl 2-bromobutan-l-oate But-2-en-l-ol
s
1 2 3 4 5 6 7
5 4 3 2 1
ook
CH,—C = CH—C—CH = C—CH,
CH3—C = C—CH2—CH = O
Yo
3 I. „ I 3
Pent-3-yn-l-al CH3 O
CHj
eB

2, 6-Dimethylhepta-2, 5-dien-4-one
If a compound contains two or more like functional groups, the numerical prefixes di, tri, tetra, etc. are
used and the terminal ‘e' from the primary suffix is retained (not dropped) while writing the lUPAC name.
r

For example.
ou
ad

o o
1 2 1 2 3 1 2 3 1 2II 3 4ll S
Y

CH,—CH,
CH2—CH2—CH2 CH,—CH—CH, CH,—C—CH,—C-^CH,
1^1* I I I 2 I I 2 3 2 3
OH OH OH OH OH OH OH Pentane-2, 4-dione
Re
nd

Ethane-1, 2-diol Propane-1, 3-diol Propane-1, 2, 3-triol


Fi

1 2 1 2 3 4
HOOC—COOH NC—CH = CH—CN HOOC—CH = CH—COOH

Ethane-1, 2-dioic acid But-2-ene-l, 4-dinitrile But-2-ene-l, 4-dioic acid

1 2 3 4 3 4
OHC—CH = CH—CHO H5C2OOC—CH2—CH2—COOC2H5
But-2-ene-l, 4-dial Diethyl butane-1, 4-dioate
12/42 New Course Chemistry (XI)EEIHII

Give the lUPAC names of the following compounds :


0 O

(0 C^Hg—C—CHjOH {ii) CH3—CH = CH—CHO(m)


0 H
CH.,

w
(tv) CH3—CH = CH—COOH (v) (CH3)2C = CHCOCH3 (W) CH^ = CH—CN

CH3

F lo
(v//) OCH—CHO (viii) HOOC—C = C—COOH (a) CH2 =C—COOCH3

CH3 OCH3

ee
(x) CH3CH2— C(C1)—CH2—CONHCH2CH3 ixi) CH3—CH—CHO (xii) [(CH3>2CH ]3COH

Fr
(xiii) CHXH^CHCH^CHO Ufv) CH,CHCH,CH, (XV) CH3CH2CH0HCH2CH2CH(CH3)CH2CH3
■ ■| ‘ 3| - 3
CH3 COOH

for
ur CN
(xvi) CH3CH2COCH2COCH3 (.xvii) (xviii) CH3—CH—CH2~CH—CH—CH3
I
ks OH
CH3 Br
Yo
ANSWERS
oo
(0 2-Ethylprop-2-en-l-ol (ii) Bui-2-en-l-al («7) Ethanoic methanoic anhydride
(vi) Pi'op-2-en-i-nitrile
eB

(iV) Bui-2-en-l-oic acid (v) 4-Methylpent-3-en-2-one


(v/7) Ethane-1, 2-diai (v«7) But-2-yne-l, 4-dioic acid (ix) Methyl 2-methyIprop-2-en-l-oate
(x) 3-Chloro-N-ethyl-3-methylpentan-1 -amide (xi) 2-Methoxypropan-I-al
(xii) 2, 4-Dimethyl-3-(l-methylethyl)pentan-3-ol (xiii) 3'MethylpentanaI
r
ou
ad

(xiv) 2-MethyIbutanoic acid (xv) 6-Methyloctan-3-oI (xvi) Hexane-2, 4-dione


(xvj7) 3-Methylpent-3-ene-l-nilrile (xvi77) 3-Bromo-5-methyIhexan-2-ol
Y

12.9.6. Rules for lUPAC Nomenclature of Polyfunctional Compounds


nd

Organic compounds which contain two or more functional groups are called polyfunctional compounds.
Re

Their lUPAC names are obtained as follows :

1. Principal functional group. When an organic compound contains Two or more different functional
Fi

groups, one of the functional groups is .selectedas the principal functional group while all other groups
(also called the secondaryfunctionalgroups) are treated as substituents. The choice of the principal functional
group is made on the basis of the following order of preference.*
Amine salts > Carboxylic acids > sulphonic acids > anhydrides > esters > acid chlorides > acid
amides > nitriles > isocyanides > aldehydes > ketones > alcohols > phenols > thiols > amines.
All the remaining functional groups such as halo (fluoro, chloro, bromo, iodo), nitroso (-NO),
nitro (-NO2), and alkoxy (-OR), R (alkyl), C5H5, etc. are always treated as substituent groups.
♦According to ‘A Guide to lUPAC Nomenclatre of Organic Compounds—Recommendations 1993 by
R-Panico, W.H. Powell and Jean-Claude Richer. In NCERT book, -COOH group is given preference over -SO3H
which is, however, wrong.
ORGANIC CHEMISTRY-SOME BASIC PRINCIPLES AND TECHNIQUES 12/43

It may be noted that while writing the names of the polyfunciional compounds, the principal functional
group is indicated by adding the secondaiy suffix to the word root while the secondary functional groups are
indicated by adding suitable prefixes to the word root. The prefixes for secondaiy functional groups are listed
below :

Secondary Prefix Secondary Prefix


functional Group functional Group

-X (F, Cl, Br. I) Halo -CHO Formyl


-OH Hydroxy >C=0 Oxo or Keto

w
-SH Sulphanyl* or Mercapto -COOH Carboxy
-OR Alkoxy -COOR Alkoxycarbonyl or
carbalkoxy

F lo
-NH2 Amino -COCl Chlorocarbonyl
-NHR Alkylamino -CN Cyano
●NR2 Dialkylamino -CONH2 Carbamoyl or

ee
Carboxamido

Fr
2. Selecting the principal chain. While selecting the principal chain present in a polyfunctional
compoundcare shouldbe taken that it must contain the principal functional group and the maximum number
of secondary functional groups and multiple (double and triple) bonds, if any.

for
3. Numbering the principal chain. The principal chain present in a polyfunctional compound must be
ur
numbered in such a way that the principal functional group gets the lowest possible number followed by
double bond, triple bond and the substituents, i.e. Principal functional group > double bond > triple
bond > substituents
s
ook
4. Alphabetical order. The prefixes for the secondary functional groups and other substituents should
Yo
be placed in alphabetical order before the word root as explained earlier. If, however, two groups of the same
preference occupy identical positions from either end of the parent chain, the lower number must be given to
eB

the group whose prefix comes first in the alphabetical order. For example,

4 3 2 1 1 2 3 4

ClCH2CH2CH2CH2Br ClCH2CH,CH2CH^Br
r
ad
ou

l-Bromo-4-chlorobutane (correct) 4-Bromo-l-chlorobuiane (wrong)

The rest of the rules for numerical prefixes are the same as explainedearlier.
Y

To illustrate these rules, let us consider the following example.


12.9.7. Rules for lUPAC Nomenclature of Compounds containing
Re
nd

more than two like carbon containing Functional Groups


According to latest convention (1993 recommendations for lUPAC nomenclature), if an unbranched
Fi

carbon chain is directly linked to more than two like functional groups, the organic compound is named as a
derivative ofthe parent alkane which does not include the carbon atoms of thefunctionalgroups. For example,
C =N
1

N = C—CH2 —CH—CH2—C = N
Propane-1. 2, 3-tricarbonitrile
(formerly 3-cyanopemane-l, 5-dinitrile)

*The prefix .sulphanyl is preferred over mercapto.


12/44 New Course Chemistry fXT~>rosT

COOH
2 3I 4 1 5
COOH
HOOC—CH2—CH^—CH — CH^—CH2
Pentane-1, 3. 5-tricarboxylic acid

ow
(formerly 4-ctirboxyheptane-1, 7-dioic acid)
CHO
4 3 2I 1
●CHO
OHC—CH2—CH2—CH—CH2
Butane-1, 2, 4-tricarbaldehyde

e
(formerly 3-formylhexane-I, 6-dial)
Similarly, citric acid may be named as follows :

re
COOH

Frl
F
HOOC—CH2—C —CH2 ●COOH

OH

2-Hydroxypropane-l. 2, 3-tricarboxylic acid


ou
r
(formerly 3-carboxy-3-hydroxypentane-l, 5-dioic acid)

so
I,j howeve,r all the three like groups are not directly linked to the unbranched carbon chain, the two
like groups are included in the parent chain while the third which forms the side chain is considered as a

kf
substituent group. For example, oo
CH2—C = N {Substituent CH2CHO
6 3I'
5 2 1 group) 4 6 5 4 3I 2 1
Y
NsC—CH2—CH2—CH—CH2—C s N OHC —CH2 CH2 CHCH2—CHO
B

3-(Cyanomethyl)hexane-1, 6-dinitrile 3-(FormyImethyl)hexane-l, 6-dial


re

CHXl
CH2CHO
1 1 21 3
oY

OHC—CH2—CH2 —CH—CH2 —CHO Cl—CH2 —c H—C H2CI


u
ad

3-(Formylmethyl)hexane-l, 6-dial 1, 3-Dichloro-2-(chloromethyl)propane

CH2COOH {Substituent group)


d

7 6 5 4 3I 2 1

HOOC—CH2—CH, —CH2 —CH—CH2 —COOH


in
Re

3-(Carboxymethyl)heptane-l, 7-dioic acid


F

Sample Problem fm Write the lUPAC names of the compounds (/- iv) from their given structures
(NCERT Solved Example)
1 2 3 4 5 6 7 8

0) CHj—CH2—CH—CH2—CH2—CH—CH2—CH3
OH CH3
O o o
6 5 4 II 3 2 11 1 6 4 3 2 1

(«) CH3—CH2—c—CH2—C—CH3 {Hi) CH3—C—CH2—CHj—CH2—COOH


6 5 4 3 2 1

(iv) HCsC—CH = CH—CH=CH2


ORGANIC CHEMISTRY-SOME BASIC PRINCIPLES AND TECHNIQUES 12/45

Solution. (0 Step 1. Here, the functional group is alcohol (OH). Hence, its secondary suffix is ol.
Step 2. The longest carbon chain containing the functional group has eight carbon atoms. Therefore, word
root is oct. Further, since the longest carbon chain is saturated, therefore, its primary suffix is ane.
Step 3. Number the carbon atoms of the chain from left to right. It gives lowest locani 3 to the functional
group (OH) and locant 6 to’the substituent methyl group (secondary preifx).
Step 4. Since the secondary suffix 'ol' begins with a vowel ‘o’, therefore, terminal ‘e’ from the primary suffix is
dropped. Therefore, the complete lUPAC name of the compound is: 6-methyl + octane-e + 3-ol = 6-methyIoctan-3-ol.
(//) Step 1. Here, the functional group present is ketone (> C = 0). Therefore, its secondary suffix is one.
Since there are two keto groups, therefore, the numerical prefix di is attached to the secondary suffix one. Hence

w
the complete secondary suffix becomes 'dione'.
Step 2. The longest carbon chain contains 6 carbon atoms, therefore, the word root is he.x. Further, since the
carbon chain is saturated, therefore, the primary suffix is ane.

F lo
Step 3. Number the carbon atoms of the chain from right to left. It gives lowest locants 2 and 4 to the keto
groups.

Step 4. The complete lUPAC name is : hexane -i- 2, 4-dione = hexane-2, 4-dione. Please note that the
terminal ‘e’ from the primary suffix 'ane' has not be dropped here since the secondary suffix dione begins with a

ee
consonant'd' and not with any vowel.

Fr
(Hi) Step 1. In this compound, two functional groups are present, i.e., a ketone and a carboxylic acid. Since
carboxylic acid is the principal functional group, therefore, its secondary suffix is oic acid. Further, since keto
group is the substituent functional group, therefore, its secondary prefix is o.xo or keto.

for
Step 2. The longest carbon chain contains six carbon atoms, therefore, its word root is hex. Further, since the
ur
chain is saturated, therefore, the primary suffix is ane.

Step 3. Number the carbon chain from right to left since the principal functional group, i.e., -COOH is the
chain terminating group, it must have the lowest locant 1. This gives the substituent functional group, i.e., oxo
s
ook
group locant 5.
Yo
Step 4. Drop the terminal 'e' from the primary suffix 'ane' since the secondary suffix, 'oic acid' begins with
a vowel. Therefore, the complete lUPAC name is: 5-oxo + hex + ane-e + oic acid = 5-oxohexanoic acid.
eB

(iV) Step 1. In this compound, there are two C = C and one C = C functional groups. The secondary suffix for
the two C = C groups is 'diene' while the secondary suffix for C = C functional group is 'yne'.
Step 2. The longest carbon chain contains six carbon atoms, therefore, the root word is hex. Funher, since
r

the carbon chain contains two double bond.s and one triple bond, therefore, the complete secondary suffixes are
ad
ou

diene and yne respectively.


Step 3. Whenever there is a choice in numbering, double bond is given preference, therefore, locants for
diene are 1 and 3, and for yne is 5.
Y

Step 4. The terminal 'e' Irom the suffix 'diene' is dropped because the first letter 'y' in the suffix 'yne'
sounds like a vowel. Thus, the complete lUPAC name of the given compound is: hexa + 1, 3-diene-e + yne =
Re
nd

hexa-1, 3-dien-5- yne.


Please note that here extra 'a' has been added to the root word hex since the primary suffix diene begins with
Fi

a consonant rather than a vowel.

1. Give the lUPAC names for the following polyfunctional compounds.

CH3
(/) CH3CH2O—CH,—CHOH—CH3 (ii) CH3—CH—CH2—C—CH3
NO2 OH

I
12/46 “P>utdeefr'4. New Course Chemistry (XI)S23MD

H,C OH
^ I
(in) CH^ =C—CH—CH2—CN (iv) CH,—CH—COOH
^ I
CONH2

(V) HOOCCH2—CH—CH2COOH ('"0 CH3—CH—CH2—CH2


COOH
COCH3 NHj

w
CHO CH3
I
(vii) HOOCCH,—C—CH2COOH (vm) CH3NH ●CH2CH2—C—CH2CH3

Flo
COOH CHO

CH3

ee
(ix) HC=C—CH—CHOH—CH2COCI (x) CH2 = CH—CH2CI

Fr
O O CH3
H
CH3
(xii) CH,—CH—C—COOCH, (xiii) CH3

for
3 I I 3
ur
O Br OH CH3

CH3
ks
(;tiv) CH2 =C—CHOH—CH2—CHO (xv) CH3—CO—CH—CH2—CH2CI
Yo
oo
C2H5
CH2CHO 0
eB

Br

(xv/) [Q
(xvii) 0
(xviii) (xix)
r
ou
ad

ANSWERS

(0 l-Ethoxypropan-2-ol (fi) 2-Methyl-4-nitropentan-2-ol


Y

(Hi) 3-Hydroxy-4-methylpent-4-cne-i-nilrile (iv) 2-Carbamoy!propanoicacid


(v) Propane-1, 2, 3-tricarboxylic acid (vi) 5-Amino-3-methylpentan-2-one
Re
nd

(v/i) 3-Carboxy-3-formylpenlane- l, 5-dioic acid (viii) 2-Ethyl-2-methyl-4-(N-methylamino)butanal


(w) 3-Hydroxy-4-methylhex-5-yn-l-oyl chloride (x) 3-Chloroprop-l -ene
Fi

(xi) 2-Oxopropanal (xti) Methyl 3-bromo-2-hydroxy-2-methylbutan-l- oate


(xiiO 4-Methylhept-5-en-2-one (xiv) 3-Hydroxy-4-methylpent-4-en-l-al
(xv) 5-Chloro-3-elhylpentan-2-one (xvi) 2-(2-Bromophenyl) ethanal
(xvii) 3-Ethyl-4-methyIhept-5-en-2-one (xv/«) 2-Ethyl-3-methylpent-2-en-l-al
(xix) 3, 3, 5-Trimethylhex-l-en-2-oi

12.9.8. Rules for Naming Alicycllc Compounds


The following rules are generally used.
1. The names of alicyclic compounds are obtained by adding the prefix, ‘cyclo’ to the name of the
corresponding straight chain hydrocarbon (alkane, alkene or alkyne).

I
ORGANIC CHEMISTRY-SOME BASIC PRINCIPLES AND TECHNIQUES 12/47

Cyclopropane Cyclobutane Cyclopentane Cyclohexane Cyclopentene Cyclohexene


2. If two or more alkyl groups or other substituent groups are present in the ring, their positions are
indicated by arabic numerals, i.e., 1, 2, 3, 4 .... etc. While numbering the carbon atoms of the ring, the
substituent which comes first in the alphabetical order is given the lowest locant provided it does not violate
the lowest set of locants rule. For example,

low
CH3 CH3
2
H3C CH3
1
CH3
1
.CH2CH3
r^2 2

CH2CH3
1,2-Dimethylcyclopentane l-Ethyl-2-methylcyclohexane 3-Ethyl-1, 1-Methyl-
1 -dimethylcyclohexane 3-propylcyclohexane

ee
{not 1-ethyl-3,3-dimethylcyclohexane)

F
3. (fl) If the ring contains more or equal number of carbon atoms than the alkyl group attached to it, it

Fr
is named as a derivative of cycloalkane and the alkyl group is treated as a substituent group, otherwise
it is named as a derivative of alkane and the cycloalkyl group is considered as a substituent group. For
example,

for
1 2

CH3—CH—CH2—CH3
3 4
ur CH2CH2CH2CH2CH3
1 2
CH3—CH2— CH— CH2—CH3
3 4 5

ks
Yo
o
(2-Butyl) cyclohexane or Pentylcyclopentane 3-Cyclobutylpentane
Bo

(1 -Methylpropyl)cyclohexane

Note that parentheses are used wherever necessary to avoid confusion. Here, in the first example,
re

2-butyl group has been enclosed in parentheses to emphasize that the locant 2 refers to the substituent on the
alkane and not on the ring (i.e., cycloalkane).
(b) If the side than contains a multiple bond or a functional group, the alicyclic ring is treated as the
ou
ad

substituent irrespective of the size of the ring. For example.


Y

4 3 2 1
3 2 1

CH2—CH=CH2 CH=CH—CO—CH3
1
nd
Re
Fi

3-Cyclopropy Iprop-1 -ene 3-Cyclohexylbutan-2-ol 4-Cyclohexylbut-3-en-2-one

(c) If more than one alicyclic ring is attached to a single chain, the compound is named as a derivative
of alkane irrespective of the number of carbon atoms in the ring or the chain. For example.

CH2

Dicyclopropyl methane 1, 3-Dicyclohexylpropane

4. If a multiple (double or triple) bond and some other substituents are present in the ring, the numbering
is done in such a way that the carbon atoms ofthe multiple bond get lowest locants I and 2 and the substituent
groups get the lower locants at the first point of difference. For example.
12/48 New Course Chemistry (XI)ESi

NO2 CH3
3 1

1
3

w
3-Nitrocyciohex-1 -ene 1 -Methylcyclopent-1 -ene 1,5-Dimethylcyclopent-l-ene 1,5-Dimethylcyclohex-l-ene
{not 2,3-dimethylcyclopent-l-ene) {not 2,4-Dimethylcyclohex-l-ene)
5. If the ring contains a multiple bond and the side chain contains a functional group, then the ring is
treated as the substituent and the compound is named as a derivative of the side chain. For example,
3 2 1
CH3—CH—CH2OH

o
e
3 2 I

re
CHj-^ CH2CH2COOH
3 2

Frl
F
2-(Cyclopent-3-en-1 -yl)propan-1 -ol 3-(4-Methylcyclohex-2-en-1 -yl)propanoic acid
6. If the ring as well as the side chain contain functional groups, the compound is named as a derivative
of the side chain or the alicyclic ring according as the side chain or the ring contains the principalfunctional
group. For example.
ou
r
o o o
OH
il

so
6 3

1 3 2 1 5 1
2I1 3

kf
02N- CH=CH—COOH ,
4 2 2;—CH2— CH—CH2—NHCH3
3 2 3
oo
3-(4-Nitrocyclohex-1 -en-1 -yl)prop-2-en-1 -oic acid 3-(6-Oxocyclohex 2-(2-Hydroxy-3-methylaminopropyl)cyclohexan-1 -one
-1 -enyl)propanamide
I,f the alicyclic ring and the side chain contain the same functional group, the compound is named as
Y
B

a derivative of the side chain of the ring according as the side chain or the ring contains higher number of
carbon atoms. For example,
OH
re

OH 7 2-6 1
,CH2—CH—CH2— CH3 OHC CH3—[CH2]—CHO
oY

1
1^2 3^4-^
u

21
ad

2-(2-Hydroxylbut-1 -yI)cyclohexan-1 -ol 7-(3-Formylcyclopent-1 -yl)heptanal


d

7. Ifa compound contains an alicyclic ring directly linked to the benzene ring, it is named as a derivative
of benzene, i.e., the compound having lowest state of hydrogenation. For example.
in
Re

NO2
F

Cyclohexylbenzene ^ ^ CH3 ^ ^
1 -(2-Methylcyclohexy)-4-nitrobenzene
8, If some functional group along with other substituent groups are present in the ring, it is indicated by
some appropriate prefix or suffix and its position is indicated by numbering the carbon atoms of the ring in
such a way that the furwtional group gets the lowest locant. For example.
OH O O
1 1
2 2 ^CH3 2

3
CH3 4 - CH3 OH

3-Methylcyclohexan-1 -ol 5,6-Dimethylcyclohex-2-en-l-one 3-Hydroxycyclohexan-1 -one

A
ORGANIC CHEMISTRY-SOME BASIC PRINCIPLES AND TECHNIQUES 12/49

9. If an alicyclic ring is directly attached to a carbon containing functional group, the carbon atom of
the functional group is not included in the parent name of the alicyclic system. Therefore, for such systems,
the following prefixes and suffixes for the functional groups are commonly used.
Functional group Prefix Suffix

-CHO Formyl Carbaldehyde


-COOH Carboxy Carboxylic acid
-COX (X = F, Cl, Br, 1) Halocarbonyl Carbonyl lialide
-COOR Alkoxycarbonyl or Carbalkoxy Alkyl carboxylate
-CONH2 Carbamoyl Carboxamide
-CN Cyano Carbonitrile

For example,

F low
CN CN CHO O CONH2
1 2 CHO
2
I 2
3

Cyclohcxane- Cyclohex-2-ene- Cyclohcxane- 2-Oxocyclohexane-1 - 2-Methylcyclopcntane-1 -


carbonilrilc 1-carbonitrilc carbaldehydc carbaldehyde carboxamide

O O

re
O O
I 2

for F
A^CH2CH2mCH3
CN 2^ COOH 2, COOC^Hs
l 1 1

2-Oxocyclopentane-1 - 2-(3-Oxobutyl)cyclohexan-l-onc 2'Oxocyclohexane-l - Ethyl (2*oxocyclohexaiio)-l-


carbonitrile carboxylic acid carboxylate
Your
Curiosity Questions
ks

f
eBoo

Q. 1. Isobutyl and sec-butyl represent different alkyl groups. Do Isopropyland sec-propyl also
represent different alkyl groups ? Comment.
Ans. Isobutyl group and sec-butyl group represent different structures since isobutyl group has a
CH3 side chain at position 2 and a free valency at position 1 while sec-butyl group has only a
ad
our

free valency at position 2 as shown below.

2I 1

CH3—CH—CHg— CH3 CH—CH,~CH.,


Isobutyl 1 2 3^4^
Re

sec-Butyl
Since propyl group has three carbon atoms, it cannot have a CH3 side chain but can have only
Y

a free valency at position 2. Therefore, it should be actually called as sec-propyl.


Find

I
CH3—CH—CH.
1 2 3

sec-Propy! or isopropyl
But it is more commonly called as isopropyl group which is a misnomer. Since there is no
ambiguity, therefore, both isopropyl and sec-propyl represent the same alkyl group.
Q. 2. CN group is attached to cyclohexane ring. Should it be called as cyciohexanenitrile or
cyclohexanecarbonitrile ? Explain.
Ans. Since the suffix nitrile does not include the carbon atom of the CN group, therefore, this name
is wrong. The correct name is cyclohexanecarbonitrile since the suffix carbonitrile includes the
carbon atom of the CN group.
J
12/50 "pn^idee^’^ New Course Chemistry (XI)EEIHD

1. Give the lUPAC names of the following compounds :

H3C CH2CHO
(0 (iO m CH= CH—CH— CH2CH3

CH3
o
o
CH3 CH3
COCH3 y\^COOCH3

w
OC2H5
(iv) (V) (Vi)

F lo
C2H5 OH

COOC2H5
(v/i) (vi/i) (ix) (-V) C=0

ee
COOC2H5

Fr
CH3

for
0

COOH ixiii)
{xi) (xii) CONHCftHs
r
You
OHC
s
ook
CO CO

(xiv) CH3—CH CH2CH2CI (XV) NH (xvi) 0


eB

CO CO
Cl

COOH COCI
our

OH
ad

CH3 CHO COOH

(xvii) (xviii) (xix) (XX)


Cl

CH3 CH3 O
dY
Re

ANSWERS

(i) Cyclohexylcyclohexane (ii) 2-(2-Methylcylobul-1 -enyl)ethanal


Fin

(Hi) l-Cyclopropyl-3-methylpent-l-ene (iv) 2-(l- Oxoethyi)cyclohexan-l'One


(v) Methyl (2-oxocyclopentane)-l-carboxylate (vj) 2-Ethoxy-l, 1-dimethylcyclohexane
ivii) 1 -Ethy l-4-methylcyclohexane (viii) Diethyl cyclohexane-1, 2-dicarboxylate
(ix) Cyclohex-2-en-l-ol (x) Cyclohexylidenemethanone
(xi) 4-Formyl-2-oxocyclohexane-l-carboxylic acid (xii) Cyclohexanccarboxanillide
(xiii) 2-Elhenyl-3-methylcyclohexa-l, 3-diene (xiv) l-(l-Chloroethyl)-4-(2-chloroethyl)cyclohexane
(xv) Cyclohexane-1, 2-carboximide (xn^O Cyclohexane-1, 2-dicarboxylic acid anhydride
(xvii) 3,5-DimelhyIcyclohex-l-ene (xviii) 2-Chioro-3-methyicyclohex-3-en-1 -ol
(xix) 2-Formyl-4-oxocyclohexane-1-carboxylic acid (xx) 2-Chlorocarbonylcyclohexane-1-carboxylic acid
ORGANIC CHEMISTRY-SOME BASIC PRINCIPLES AND TECHNIQUES 12/51

12.9.9. Writing Structural Formulae from the lUPAC Name of the Compound
We have discussed above how to write lUPAC name of an organic compound if its structure is given.
Let us now do the reverse, /.e., to write the structure of the organic compound if its name is given. For this
purpose, the procedure used involves the following steps :

w
1. Select the longest carbon chain {parent chain) from the word root of the lUPAC name of the organic
compound. For prop onnect three carbon atoms ; for but onneci four carbon atoms ; for pent —join
five carbon atoms and so on by single bonds in a straight line.
2. Number the carbon chain from either direction.
3. Identify the primary suffix, i.e. ane, ene or yne from the name of the compound. If the compound

o
contains a double bond or a triple bond, identify its position from the name of the organic compound and put

e
the double or the triple bond at its right position along the carbon chain.

re
4. Identify the name and position of the functional group (secondary suffix) from the lUPAC name of
the compound and fix it at its right position on the carbon chain.

Frl
F
5. Identify the names and positions of the other substituents, if any, from the lUPAC name of the compound
and ifx them at their right positions along the carbon chain.
6. Finally, wherever necessary, attach the required number ofhydrogen atoms to sati^ the tetracovalency
ou
of each carbon. Let us illustrate these rules by the following examples :

r
Sample Problem
Derive the structure of (i) 2-chIorohexanc, (ii) pent-4-en-2-ol (iii) 3-nitro-

so
cyclohexene, (iv) cycIohex-2-en-l-ol, (v) 6-hydroxyheptanal. (NCERT Solved Example)

kf
Solution. (/■) Step 1. The word root 'hex’ indicates the presence of six carbon
C—C—(
atoms in the chain and the primary suffix 'ane' suggests that the chain is saturated.
oo
1 2 3 4 5 6
Step 2. Number the carbon atoms of the chain as indicated c—c—c—c—c—c
Y
B

1 2 3 4 5 6
Step 3. The secondary prefix chloro and the numerical prefix 2 before C—C—C—C—c—c
it suggests that there is a chlorine atom at position 2 as shown Cl
re

Step 4. Satisfy the tetracovalency of each carbon with required


CH3—CH—CH2—CH2—CH2—CH3
oY

number of hydrogen atoms. Thus, the structural formula of


u

2-chlorohexane is Cl
ad

{ii) Step 1. The word root 'pent' indicates that the longest carbon chain contains C—(
five carbon atoms.
d

1 2 3 4 5
Step 2. Number the carbon chain as indicated. C—C—C—C—c
in

Step 3. The secondary suffix-o/ and the numerical prefix 2 before it suggests that 1 2 3 4 5
Re

there is a hydroxyl group at position 2. Therefore, fix a hydroxy] group at position 2 as C—C—C—C—C
shown I
F

OH

Step 4. The primary suffix 'en' and the numerical prefix 4 before it suggests that 1 2 3 4 5
C—c—c—c = c
there is a double bond at position 4. Therefore, put a double bond between positions 4 I
and 5 as shown.
OH

Step 5. Satisfy the tetracovalency of each carbon with the required number 1 2 3 4 5

of hydrogen atoms. Thus, the structural formula of pent-4-en-2-ol is CH3—CH—CH2—CH


I
= CH2
OH

{Hi) Step 1. The primary prefix 'cycto' and the word root 'hex' means that the
given compound contains a six-numbered irng. Therefore, draw a cyclohexane irng as
shown :
12/52 ‘pnaUee^ 'a New Course Chemistry (XI)
\

6 2

Step 2. Number the carbon atoms of the cyclohexane ring as indicated. 5 3

Step 3. The primary suffix "en' without any numerical prefix indicates 6i^^2
that there is a double bond at position 1. Therefore, put a double bond between 5 3
positions 1 and 2 as shown ; 4

w
Step 4. The secondary prefix ‘nitro’ and the numerical prefix 3 before 1

it suggests that there is a nitro group at position 3. Therefore, place a nitro


group at position 3.

Flo
This represents complete structure of 3-nitrocyclohexene.

(/V) Step 1. The primary suffix ‘cyc/o’ and the word root hex means that the

ee
given compound contains a six- membered ring. Therefore, draw a cyclohexane irng

Fr
1

6 2

Step 2. Number the carbon atoms of the cyclohexane ring as indicated 5 3

for
4
ur OH

Step 3. The primary suffix 2-e;i and the secondary suffix l-ol indicates that 1
2
there is a double bond at position 2 and an OH group at position 1. Thus, the structure
s
of the compound cyclohex-2-en-l-ol is
ok
Yo
(v) Step 1. The name heptanal is made up of three parts, i.e., kept (word
c—c—c—c
root) + an (primary suffix) + al (secondary suffix). This means that the given <
o
compound is an aldehyde containing 7 carbon atoms including the carbon atom
eB

of the aldehyde group.


7 6 5 4 3 2 1
Step 2. Number the carbon atoms of the carbon chain as follows. c—c—c—c—c—c—c
o
r

Step 3. Since aldehyde group is a chain terminating group, therefore, C—C—C—C—C—C—C—H


ou
ad

convert carbon at position 1 into aldehyde group as shown. 7 6 5 4 3 2 1


OH O
Y

Step 4. The secondary prefix hydroxy and the numerical prefix C—C—C—C—C—C—C—H
7 6 5 4 3 2 1
6 before it indicates that there is an OH group at position 6 as
nd
Re

shown.

Step 5. Satisfy the tetracovalency of each carbon with the required number of hydrogen atoms. Thus, the
Fi

structural formula of 6-hydroxyheptanal is


OH O

CH3—CH—CH2—CH2—CH2—CH2—C—H

1. Draw the structures of the following compounds :


(i) Hex-3-en-l-oic acid (ii) 2-Chloro-2-methylbutan-1 -ol (Hi) 5, 5-Diethylnonan-3-oI
(iV) 1 -Bromo-3-chlorocyc!ohex-1 -ene (v) 1, 3-Dimethylcyclohex-l-ene
ORGANIC CHEMISTRY-SOME BASIC PRINCIPLES AND TECHNIQUES 12/53

2. Write the condensed formulae for each of the following compounds :


(/) Isopropyl alcohol (ii) Methyl r-butyl ether
(Hi) 2-Chloro-l, I, I-trifluoroethane (/v) 2-Methylbuta-l, 3-diene (v) But-2-en-l-ol.
ANSWERS

ow
CH3
I
1. (0 CH3CH2CH = CHCH2COOH iii) CH3CH2—C—CH2OH
Cl
CH.CH,
I ^ ^
m CH3CH^CH2CH,—C—CH2—CH—CH.—CH,

e
J - 2 - I 2 I 2 3

re
CH2CH3 OH

Frl
Cl CH3

F
(/V) (V)
Br CH3
ou
or
2. (/) (CH3)2CH0H (//) CH3—O—C(CH3>3 (iii) F3C—CH2CI

kfs
CH3
(iv) CH2=C—CH =CH2 (v) CH3CH = CHCH2OH
oo
12.9.10. Nomenclature of Simple Aromatic Compounds
Y
Aromatic compounds contain one or more isolated or fused benzene rings. An aromatic compound
B

consists of two parts : (/) Nucleus and {ii) Side chain


(I) Nucleus. The most ideal aromatic compound is benzene. It is represented by a regular hexagon of six
carbon atoms with three alternate single and double bonds. This is called the nucleus. The ring may be
re

represented by any of the following three ways :


oYu

H
ad

H
d

or
Q
in
Re

H
II III
1
F

A circle inside the cyclohexane ring represents six completely delocalised 7i-electrons or three conjugated
double bonds.
R

(ii) Side chain. The alkyl group (R) or any other aliphatic group containing
at least one carbon atom which is attached to the benzene ring is called the side
chain. Q
In view of the above two parts of the aromatic compounds, each family of aromatic compounds consists
of the following two types ol compounds with quite different chemical properties.
(0 Nuclear substituted {ii) Side chain substituted
(i) Nuclear substituted — those in which the functional group is directly attached to the benzene ring.
Most of these compounds are better known by their common and historical names. In the lUPAC system, they
12/54 'P%auUe^’4. New Course Chemistry (XI)hS£sI9n

are named as derivatives of benzene. However, many of their common names have also been adopted by the
lUPAC system. The positions of the substituents in disubstituted benzenes are indicated either by prefixes or
by arabic numerals such as o (ortho) for 1,2 ; m (meta) for 1,3 andp (para) for 1,4.
(ii) Side chain substituted —those in which the junctional group is pre<^ent in the side chain of the
benzene ring. Both in the common and lUPAC systems, these are usually named as phenyl derivatives of the
corresponding aliphatic compounds (except arenes which are named as derivatives of benzene in the lUPAC

ow
system). The positions of the substituents on the side chain including the benzene ring are indicated by Greek
letters i.e., (X, p, y.... in the common system, and by arabic numerals, i.e., 1,2,3 ... etc. in the lUPAC system.
However, many of these compounds are better known by their common names.
The lUPAC and common names (given in brackets) of a few important members of each family are
given below. Wherever only one name is given, it implies that the common name has also been adopted by the
lUPAC system. However, at some places two names are given outside parentheses, it implies that both these

e
names have been adopted by the lUPAC system.

re
rFl
1. Aromatic hydrocarbons (Arenes). Hydrocarbons which contain both aliphatic and aromatic units
are called arenes.

F
CH3 CH3 CH3 CH3 CH2CH3

r
1 1
ou
Q 0.
.0?
"'^^CH3
sfo
Q a 4
Benzene Toluene 1,2-Dimethylbenzene Ethylbenzene
(o-Xylene)
(m-Xylene) k
1,3-Dimethylbenzene
CH3
1,4-Dimethylbenzene
oo
(p-Xylene)
1 2 3
Y
CH3—CH—CH3 CH=CH2 C=CH
eB

A
Q a 0.
r

(2-Propyl)benzene Ethenylbenzene Ethynylbenzene


ou

(Isopropylbenzene or Cumene) (Styrene) (Phenylacetylene)


Y
ad

2. Aryl groups
d

CH2— CH ce CH3
A
in
Re

C6H5- or
Q Q Q a Q
F

Phenyl
Phenyl Benzyl Benzal Benzo o-Tolyl

3. Halogen derivatives
Cl Cl CH3 CH2CI CHCI2 CCI3

JL 1
.Cl 1

Q Q a a a
Chlorobenzene 1,2-Dichlorobenzene 3-ChlorotoIuene Chlorophenyl- Dichlorophenyl- Trichlorophenyl-
(o-Dichlorobenzene) (m-Chlorotoluene) methane methane methane
(Benzyl chloride) (Benzal dichloride) (Benzotrichloride)
ORGANIC CHEMISTRY-SOME BASIC PRINCIPLES AND TECHNIQUES 12/55

4. Hydroxy derivatives. The nuclear hydroxy derivatives are called phenols while the side chain
substituted hydroxy derivatives are called aromatic alcohols.
(/) Phenols

OH OH OH OH
1 1 I
.OH

Q Q QP

w
CH3
Phenol 2-Methylphenol 3-Methylphenol Benzene-1, 2-diol
(o-Cresol) (m-Cresol) CH3 (Catechol)
4-MethylphenoI

Flo
(p-Cresol)

OH OH

ee
1

Fr
3

OH
Benzene-1, 3-diol
(Resorcinol)

for
Benzene-1,4-diol
ur
(Hydroquinone or Quinol)
Benzene-1,2, 3-triol
(Pyrogallol)
Benzene-1,2,4-trioI
(Hydroquinol)

(if) Aromatic alcohols s


ok
Yo
2 1 2 1
CH2OH pCH2—CH2—OH
Bo

Q Q
re

Phenylmethanol 2-Phenylethan-l-ol 1-Phenylethan-l-ol


(Benzyl alcohol) (P-Phenylethyl alcohol) (a-Phenylethyl alcohol)
ou
ad

5. Aromatic ethers
Y

Q Q
nd
Re

Anisole or Methoxybenzene Ethoxybenzene Phenoxybenzene


(Anisole or Methyl phenyl ether)
Fi

(Phenetole or Ethyl phenyl ether) (Diphenyl ether)


6. Aldehydes
2 1 3 2 1
CHO CHO CH2CHO P
CH2CH2CHO
a
1
OH

OP Q Q
Benzaldehyde* or 2-Hydroxybenzaldehyde 2-Phenyiethanal 3-Phenylpropanal
Benzenecarbddehyde* (Salicyladehyde) (Phenylacetaldehyde) (^-Phenylpropionaldehyde)

♦These are also the common names.


12/56 7>nat4:Ue^'^ New Course Chemistiy (XI)^am

7. Ketones
1 2 1 2 3 1 2 3 4

COCH3 COCH2CH3 COCH2CH2CH3 COCgHs

Q Q Q
1 -Phenylethan-1 -one 1 -Pheny Ipropan-1 -one 1 -Phenylbutan-1 -one Diphenylmethanone
or Acetophenone {Propiophenone or {Butyrophenone or Phenyl {Benzophenone or
(Acetophenone or Methyl phenyl ketone) Ethyl phenyl ketone) n-propyl ketone) Diphenyl ketone)
8. Nitro compounds

w
NO2

F lo
Q
Nitrobenzene 1,3-Dinitrobenzene
(m-Dinitrobenzene)

ee
2,4,6-Trinitrophenol

Fr
(Picric acid)

for
9. Amines (/) Arylamines
ur
Aniline or Benzenamine
(Aniline)
s
ook
4-Methylbenzenamine
Yo
or 4-Methylaniline
(p-Toluidine or 4-Methylaniline)
eB

2 1

PCH2-CH2-NH2

(if) Aralkylamines Q
our
ad

Phenylmethanamine 2-Phenylethanamine
(Benzylamine) C^-Phenylethylamine)
10. Carboxylic acids
Y

COOH COOH COOH COOH


Re
nd

Q
Fi

Benzoic acid 2-Methylbenzoic acid 2-Hydroxybenzoic acid 2-Aminobenzoic acid


or Benzenecarboxylic acid (o-Toluic acid (Salicylic acid (Anthranilic acid
(Benzoic acid) or o-Methylbenzoic acid) or o-Hydroxybenzoic acid) or o-Aminobenzoic acid)

COOH COOH
COOH
1
COOH

Q
^"^^COOH
Benzene-1,2-dicarboxylic acid Benzene-1, 3-dicarboxylic acid COOH
(Phthalic acid) (Jsophthalic acid) Benzene-1,4-dicarboxylic acid
(Terephthalic acid)
ORGANIC CHEMISTRY-SOME BASIC PRINCIPLES AND TECHNIQUES 12/57

11. Acid derivatives

COCl CONH2 COOCH3 OOCCH3 COOCgHj

Q Q Q O O
Benzenecarbonyl chloride Benzamide Methyl benzoate Phenyl ethanoatc Phenyl benzoate
{Benzoyl chloride) {Phenyl acetate)

w
NHCOCH3 NHCOCgHj CONHCH3
O

Q Q

Flo
O o=c c=o

N-Phenylethanamide N-Phenylbenzamide N-Methylbenzamide


{N-Phenylacetamide
0 CQ

ee
{N-Phenylhenzamide
or Acetanilide) or Benzanilide)

Fr
Benzoic anhydride

12. Sulphonic adds

for
ur
SO3H SO3H
ks
1
2 3
Yo
Q O 1

<^>^-S03H
oo
CH3
3^S03H
eB

Benzcnesulphonic acid Benzenc-1, 3-disulphonic acid 4-Toiucncsulphonic acid


(m-Benzenedisulphonine acid) {f}-Toluenesulphonic acid)

13. Cyanides and isocyanides


r
ou

C=N
ad

CH2CSN N^C

Q O
Y

Q
Bcnzenecarbonitrile Phenylethanenitrile Phenylisocyanidc or
nd
Re

or Bcnzonitrile {Benzyl cyanide or Phenylcarbylamine


{Benzonitrile or Phenyl acetonitrile)
Phenyl cyanide)
Fi

14. Arenediazonium salts

+ +

N=NCr CH3- N=NHSOi-


Benzenediazonium
4-Toluenediazonium hydrogen sulphate
chloride
{p-Toluenediazonium hydrogen sulphate)

12.9,11. Systematic Nomenclature for Di- and Polyfunctional Aromatic Compounds


(0 When an aromatic compound contains two or more functional groups, it is named as a derivative of
the compound with the principal functional group at position J. For example,
12/58 New Course Chemistry (XI)BS!9D

COOH OH CN

1
JL/NH2
a D?
[4
2-Aininophenoi 3-(l-Oxoethyl)benzenecarbonitrile or
NO2 (OH is the principalJunctional group 3-Acetylbenzonitrile
4-Nitrobenzoic acid while NH2 is the substituent group) (C/V is the principalJunctional group
(COOH is the principalJunctional group while COCH3 is the substiuent group)
while NO2 is the substituent group)

w
OH CO2H CHO
1 1

Ok

F lo
o? Br
4 4

I OH OH

ee
4-Iodo-2-methylphenol 3-Bromo-4-hydroxybenzoic acid 4-Hydroxy-3-methoxybenzaldehyde*
(OH is the principalJunctional group) (-COOH is the principalJunctional group) (-CHO is the principalJunctional group)

Fr
(ii) If all the functional groups present in the benzene ring are such which are normally treated as
substituent groups, the various groups are arranged in alphabetical order with the group named first in the
alphabetical order getting the lowest locant provided it does not violate the lowest locant rule for all the

for
substituents. For example.
ur
Br Cl
1
1 Cl 2^Br
s
ook
Q
Yo
2

02N'^^^2^N02 1 Cl
eB

l-Chloro-2,4-dinitrobenzene 2-Bromo-l, 3-
1 -Bromo-3-chlorobenzene CH3 dichlorobenzene
(not l-ch!oro-3-bromobenzene) (not 4-chloro-1,3-dinitrobenzene) 2-ChIoro-1 -methyl-4-nitrobenzene (not l-bromo-2,6-
(not 3-chloro-4-methyl- dichlorobenzene)
1-nitrobenzene)
r

(Hi) When a substituent is such which when taken together with the benzene ring gives a special name
ou
ad

to the molecule, then it is named as a derivative of that molecule with the substituent at position 1. For example,
NO2 OMe OH NH2
Y

1 1
€1

OP OI3 cQrf
Re
nd

Cl 4
T
Fi

CH3 CH3 CH3 C2H5


2-Chloro-4-nitrotoluene 2-Chloro-4-methyIanisole 3,4-Dimethylphenol 4-Ethyl-2-metbylaniline

(iv) When a benzene ring is attached to an aliphatic chain having afunctional group, it is named as
phenyl derivative of that aliphatic compound. For example,
OH
3I 2 1
—CH—CH2—CHO

2,3-Dibromo-l-phenylpentane 4-Phenyl-3-hydroxybutanal

♦Its common name is vanillin.


ORGANIC CHEMISTRY-SOME 8/ . . PRINCIPLES AND TECHNIQUES 12/59

Sample Protslem
Write t?^e structures of the following compounds : (a) o-Ethylanisole
(b) p-Nitroaniline (c) 4-Ethyl-l-fluoro -2-nitrobenzene. (NCERT Solved Example)
NH2 NO-,
.OMe 1

Solution.
Q
C2H5 4 C2H5
o-Ethylanisole or 4-Ethyl-1-nuoro-2-
NO2
2-Ethylanisole nitrobcnzene

w
/,-Nitroaniline
or 4-Nitroaniline

12.10. ISOMERISM

F lo
One of the most important characteristics ot organic compounds is their ability to show isomerism.
Two or more compounds having the same molecularformula but different chemical and physical
properties are called isomers and the phenomenon is known as isomerism.

ee
It is of two types :

Fr
1. Structural isomerism and 2. Stereoisomerism.

12.10.1. Structural Isomerism

for
Compounds having the same molecular formula but different structures, i.e., different
ur
arrangement of atoms within the molecule are called structural isomers and the phenomenon
is called structural isomerism.
s
It is of the following six types:
ook
Yo
1. Chain or nuclear isomerism.

Compounds having the same molecular formula but different arrangement of carbon chain
eB

within the molecule are called chain or nuclear isomers and the phenomenon is called chain or
nuclear isomerism.

For example, butane and 2-methylpropane are chain isomers.


r
ad
ou

CH3
3 2 ]
CH3—CH,—CH2—CH3 CH3—CH—CH3
Y

Butane (n-Butane) 2-Methylpropane (Isohutane)


Similarly, n-pentane, isopentane and neopentane are chain isomers (For structures, refer to page 13/5).
Re
nd

2. Position isomerism.

Compounds which have the same structure of the carbon chain but differ only in the position of
Fi

the multiple (double or triple) bond or the functional group are called position isomers and the
phenomenon is called position isomerism.
For example, but-l-ene and but-2-ene are position isomers.
4 3 2 1 4 3 2 I
CH3—CHj—CH = CH2 CH3—CH=CH—CH3
But-l-ene But-2-ene

Similarly, propan-l-ol and propan-2-ol are position isomers.


OH
3 2 1 3 2 I

CH3—CH2—CH2—OH CH3—CH—CH3
Propan-1 -ol {n-Propyl alcohol) Propan-2-ol (Isopropyl alcohol)
12/60 ‘P’tadecfr’^ ' Course Chemistry (X1)ESS!9D

3. Functional isomerism.
I?-

Compounds having the same molecular formula but different functional groups are called
functional isomers and the phenomenon is called functional isomerism.
Many classes of organic compounds show functional isomerism among themselves as discussed below :
(/) Alcohols and ethers (C„H2„+2®) show functional isomerism. For example, the molecular formula
C2HgO represents the following two fonctional isomers :
CH3—CH2—OH CH3—O—CH3

w
Ethanol Methoxymethane
(Ethyl alcohol) (Dimethyl ether)
(ii) Carboxylic acids and esters (C„H2„02> show functional isomerism. For example, C2H4O2
represents the following two functional isomers :

Flo
O O

e
CH3—C—OH H—C—OCH3

re
Ethanoic acid Methyl methanoate
(Acetic acid) (Methyl formate)

F
(in) Aldehydes, ketones, unsaturated alcohols and unsaturated ethers (C„H2„0) show functional
isomerism. For example, C3H6O represents the following functional isomers.
ur
O O

r
fo
CH3—C—CH3, CH3CH2—C—H , CH2 = CH—CH2OH and CH2 = CH—O—CH3
Propanone Propanal Prop-2-en-l-ol
ks Methoxyethene
(Acetone) (Propionaldehyde) (Allyl alcohol) (Methyl vinyl ether)
Yo
(iv) Aromatic alcohols, phenols and ethers show functional isomerism. For example, C7HgO represents
oo
the following three functional isomers (i.e., I, II, HI or I, II, IV and I, II, V).
CH2OH OCH3 OH OH
B

a Q Q
e

a or or HO CH3
ur

CH3
Benzyl alcohol (I) Anisole (II) o-Cresol (III) m-Cresol (IV) . p-Cresol (V)
ad

Isomers (HI, IV and V) may also be regarded as position isomers,


Yo

(v) Dienes, allenes and alkynes (C„H2„_2> show functional isomerism. For example, C4H6 represents :
1 2 3 1 2

CH2 =CH—CH = CH2, CH2 = C = CH—CH3 and CH3CH2CSCH or CH3CSCCH3


d
Re

Buta-I, 3-diene Buta-l, 2-diene (An allene) But-I-yne But-2-yne


in

(vi) Nitroalkanes, alkyl nitrites and amino acids (C„H2„+iN02) show functional isomerism.
For example, C2H5NO2 represents, nitroethane, ethyl nitrite and 2-aminoethanoic acid which are
F

functional isomers.
O
+ .0
CH3—^ch2—n; , CH3—CH2- N = 0 H2N—CH2—c—OH
O-
Nitroethane Ethyl nitrite 2-Aminoethanoic acid (Glycine)

(vii) 1®, 2“ and 3®-Amines show functional isomerism. For example.


CH3
C3H9N represents : CH3CH2CH2NH2, CH3CH2—^NH—CH3 and CH3—N—CH3
Propan-1-amine N-Methylethanamine N, N-Dimethylmethanamine
(1° Amine) (2“ Amine) (3" Amine)
ORGANIC CHEMISTRY-SOME BASIC PRINCIPLES AND TECHNIQUES 12/61

(viii) Cyanides and isocyanides (C„H2„_iN) show functional isomerism.


For example, C2H3N represents :
CH3—G = N and CH3—N = C
Ethanenitrile Methyl isocyanide
(tr) Amides and oximes (C„H^^|NO) show functional isomerism. For example, C3H7NO represents :
O

CH3CH2—C—NH2 and CH3CH2CH = N0H

w
Propanamide Propanal oxime
4. Metamerism.

F lo
Compounds having the same molecular formula but different number of carbon atoms (or
alkyl groups) on either side of the functional group (Le., -0-, -S-, -NH-and -CO-) are called
metamers and the phenomenon is called metamerism. Metamerism occurs among the members

ee
of the same homologous family.

Fr
For example.

CH3
(0 CH3CH2—O—CH2CH3 is a metamer of CH3—O—CH2CH2CH3 or CH3O—CH

for
CH3
Ethoxyethane
ur
1 -Methoxypropane
(Diethyl ether) (Methyl n-propyl ether) 2-Methoxypropane
(Isopropyl methyl ether)
s
ook
CH3
Yo
(ii) CH3CH2—S—CH2CH3 is a metamer of CH3-^—CH2CH2CH3 or CH3—S—CH:
Diethyl thioether
CH3
Methyl n-propyl thioether
eB

Isopropyl methyl thioether

CH3
(Hi) CH3CH2—NH—CH2CH3 is a metamer of CH3—NH—CH2CH2CH3 or CH3—NH—CH:
r

CH3
ou
ad

Diethylamine Methyl n-propylamine


Isopropylmethylamine
Y

O o O
1 2 ,11 1 3 ,11 1 2 I
(iv) CH3CH2—C—CH2CH3 is a metamer of CH3 -C—CH,CH,CH, or CH,—C—CH:C
Re

^ ^ ^CHj
nd

Pentan-3-one Pentan-2-one

(Diethyl ketone) (Methyl n-propyl ketone) 3-Methylbutan-2-one


Fi

(Isopropyl methyl ketone)


It may be noted here that metamers may also be position isomers. For example, pentan-2-one and
pentan-3-one may be regarded as position isomers as well as metamers.
The concept of metamerisip can also be extended to include esters. However, here since the two ends of
the ester functional group are different, /.e., C = O and -0-, therefore, the number of carbon atoms on the
either side of the functional group may be same or different. For example.
O o

(i) CH3—C—CX:H2CH3 and CH3 ●CH2—C—OCH3 are metamers.


Ethyl acetate Methyl propionate
12/62 New Course Chemistry (XI)CB29D

O O

(k) CH3CH2—C—OCH2CH3 and CH3—C—OCH2CH2CH3


Ethyl propionate n-Prnpyl acetate
o
/CH3
or CHj—C—O— are metamers.

CH3
5. Tautomerism.

w
It is a special kind offunctional isomerism in which the isomers exist in dynamic equilibrium with each othe.r
It arises due to migration ofa hydrogen atomfrom one polyvalent atom to the other within the same molecule with
necessary rearrangement oflinkages. The isomers thus obtained are called tautomers and phenomenon is called

Flo
tautomerism. Tautomerism is also called desmotropism (desmos - bonds, propos = turn).
Types. Tautomerism is of several types. The two important types are :
(i) Dyad systems («) Triad systems

ee
(0 Dyad system. This type of tautomerism involves 1, 2- migration of a proton from one polyvalent
atom to the other within the same molecule with necessary rearrangement of linkages. For example.

Fr
H—N = C

Hydrogen cyanide Hydrogen isocyanide


Here, hydrogen migrates from carbon to nitrogen. The alkyl derivatives of these tautomers are called

for
ur
cyanides (1) and isocyanides (U).
R—C = N R—N = C
Alkyl cyanide (I) Alkyl isocyanide (U)
ks
It may, however, be noted that although hydrogen cyanide and hydrogen isocyanide are tautomers, alkyl
Yo
cyanides and alkyl isocyanides are not tautomers but are simply functional isomers.
oo
(ii) Triad systems. There are many types of triad systems. In all these systems, hydrogen migrates from
one polyvalent atom to the third polyvalent atom within the same molecule. Keto-enol tautomerism is just
eB

one type of triad system. In this type of tautomerism one form (tautomer) contains the keto group
I
(]i;^C = O) while the other contains the enolic ('^C= C —OH) group. Three simplest examples of keto-
r

enol tautomerism are acetaldehyde, acetone and cyclohexanone.


ou
ad

OH
1 9"
I
'CH3—C—H
Y

ir
—> CH2 =C—H
Acetaldehyde {keto form) Vinyl alcohol (enolic form)
(Negligible amount)
nd
Re

OH
1 2" I
Fi

CH3—C—CH3
<r
CH2 = C—CH3
Acetone (keto form) Prop-l-en-2-ol (enolic form)
(99-7%) (Negligible amount)
O OH

A
Cyclohexanone Cyclohex-l-en-l-ol
(98-8%) (1-2%)
In all the monocarbonyl compounds listed above, the greater stability of the keto-form w.r.t. the enol
form is due to the greater strength of the carbon-oxygen Tt-bond (364 kJ mol“^) as compared to carbon-
carbon 7C-bond (254 kJ mol"*).
ORGANIC CHEMISTRY-SOME BASIC PRINCIPLES AND TECHNIQUES 12/63

Factors affecting the relative amounts of keto and enol forms in keto-enol tautomerism.
The following three factors affect the position of equilibrium in keto-enol tautomerism.
(/) Stability of the enol form. As stated above in simple aldehydes and ketones (i.e. acetaldehyde,
acetone, etc.) the amount of enolic form is negligibly small. However, if the enolic form is stabilized by
intramolecular hydrogen-bonding {chelation) and or resonance, such as in 1, 3-dicarbonyl compounds, the
amount of enolic form is much greater than in acetaldehyde or acetone. For example, in acetoacetic ester, the
amount of enolic form is 7%.
O O .H.
O o' o

w
CH3^ ^0C2H5 ◄ ►

Acetoacetic ester
(/, 3-Dicarbonyl compound) CH3 OC2H5 CH3 OC2H5

F lo
{keto form, 93%) {Enolform, 7%)
Similarly, acetylacetone {i.e., a 1, 3-diketone) also exhibits keto-enol tautomerism but the amount of
enolic form here is much higher (76%) than even in acetoacetic ester. This is due to the reason that keto group

ee
is a much better electron-withdrawing group than the ester group.

Fr
O O H.
o 'O o' o
◄—►

for
ur
Acetylacetone {keto form, 24%)
CH3 CH3 CH3 CH3
{Enol form, 76%)
ks
In contrast, the compound cyclohexa-2, 4-dien-l-one exists totally in its enol form.
Yo
C? OH
oo
H
eB

Cyclohexa-2,4-dien-l-one {keto-form) Phenol {enolform)


The reason being that the enol form is aromatic and is, therefore, stabilized by the resonance energy of
r

the benzene ring (151 kJ mol“^).


ou
ad

From the above discussion it follows, that higher the stability of the enol form, greater is the enol
content. For example, the enol content increases in the order:
Y

CH3COCH3 < CH3COCH2COOC2H5 < C6H5COCH2COOC2H5 < CH3COCH2CHO


< CH3COCH2COCH3 < C6H5COCH2COCH3 < C6H5COCH2COC6H5.
Re
nd

due to H-bonding is of the order of 7 kcal mol“^ (or


Further, in 1,3-diketones, the extent of stabilization
about 29 kJ mol"*), while that due to resonance is 15 kcal mol“* (or about 62-63 kJ mol“*).
Fi

(«) Steric hindrance. Another factor which determines the enol content is the steric factor. To illustrate
this, let us consider the enol content of acetylacetone and a-methylacetylacetone.

o "o o "o

CH3 CH3 CH3 CH3


H CH3
Acetylacetone a-Methylacetylacetone
{91-93% in gas phase) {43-5-44~5% in gas phase)
12/64 New Course Chemistry (XI)^X9D

Although both the enols are stabilized by H-bonding^ the enol form of a-methylacetylacetone is
destabilized to some extent by the steric repulsion due to the presence of the a-methyl group. As a result, a-
methyfacetylacetone has lower enol content (43-5-44-5%) as compared to that of acetylacetone (91-93%).
(«0 Effect of polarity of solvent. Polar protic solvents such as water, methanol, acetic acid, etc. which
form H-bonds with the carbonyl group of the keto- form decrease the enol content. On the other hand, aprotic
solvents such as hexane, benzene, etc. (or even the absence of solvent) increase the enol content. For example,
enol form of acetylacetone is 76% in ethanol but is 92% in hexane.
E^ential conditions. There are two conditions for a molecule to exhibit tautomerism. These are :
(i) The compound must have a electronegative atom (i.e., N, O or S) bonded by a double or a triple
bond, Le., C=0, C = N, N=0,C^NH, C = S, etc.),

w
(ii) The compound must have atleast one acidic a-hydrogen, i.e., a-hydrogen present on a saturated
carbon.. Thus, acetophenone, butan-2-one and propionaldehyde all contain acidic a-hydrogen atoms and
hence show keto-enol tautomerism.

F lo
O O O
a a a a

CgHs—C-CH3 CH3—C—CH2CH3 CH3—CH2—C—H


Acetophenone Butan-2-one Propionaldehyde

ee
However, p-benzoquinone contains a-hydrogens but they are not labile because they are present on a

Fr
double bond. Therefore, it does not show keto-enol tautomerism.
H H

for
O O
ur
'a

H H
s
p-Benzoquinone
ook
Yo
In contrast, benzaldehyde, benzophenone, etc. do not show keto-enol tautomerism because they do not
contain a-hydrogen atoms.
eB

O O

—H
r

Benzaldehyde Benzophenone
ad
ou

Other types of tautomerism involving 1,3-hydrogen shifts are :


(0 Nitro-adnitro tautomerism. 1** and 2** nitroalkanes which contain acidic a-hydrogens show tautomerism.
Y

3 3
O OH
CH3— ± CH, =n::
Re

1 2^ O-
nd

Nitromethane (Nitro-form) (Aci-form)


Fi

(ii) Nitroso-oximino system. For example, nitrosoethane and ethanal oxime are tautomers
1 id 3 2 1

CH3—C^N=0^ CH3—CH =N—O—H


H Ethanal oxime (Oximino)
Nitrosoethane (Nitroso)
(Hi) Imine-enamine system. For example, propan-1-imine and prop-1-en-l-amine are tautomers.

1 2 Q 3 2

CH3—CH =CH—NH2
1

CH3—C^CH = NH
H
Prop-1-en-l-amine (Enamine)
Propan-1-imine (Imine)
ORGANIC CHEMISTRY-SOME BASIC PRINCIPLES AND TECHNIQUES 12/65

(/v) Amido-imidol or Thioamido-imithiol system. For example, urea and thiourea show tautomerism :
3 3
1
i

H—N
1
O—H
I 1 P S—H

C—NH2
N-T-l_
± HN = C—NH2 ; H—N-s-C—NH, - H—N=C—NH2
\J^2
<r <■
2
3 2 3 2
H H

w
{Imidol) (Imilhiol)
Urea (Amido) Thiourea (Thioamido)

SUPPLEMENT.yOUR
KNOWLEDGE FOR COMPETITIONS

Besides 1,3-hydrogen migration, keto-enol tautomerism may involve 1,5 or 1,7-hydrogen migration. For

o
example

:p

e
OH
OH

re
CH3 CH3 CH3

rFl
3
CH3 CH3
CH3 ^ CH3

F
H

5 6
H w
H H
1,5-Hydrogen 1,7-Hydrogen

r
migration migration
ou
sfo
6. Ring-chain isomerism
Compounds having the same molecularformula but possessing open chain and cyclic structures
are called ring chain isomers and the phenomenon is called ring-chain isomerism.
k
oo
For example, (a) Alkenes and cycloalkanes (C„H2n)
Y
B

(0 C3H6 represents : CH3CH = CH2 and which are ring-chain isomers


Cyclopropane
Propene
re

C4Hg represents :
ou
Y
ad

(/i) CH3CH2CH = CH2 and or which are ring-chain isomers.


But-l-ene Cyclobutane Methylcyclopropane
d

Similarly, but-2-ene or 2-methylprop-l-ene are ring chain isomers of cyclobutane and


methylcyclopropane.
in
Re

(b) Alkynes and cycloalkenes (CnH2„.2)


F

C3H4 represents : CH3C = CH and which are also ring-chain isomers.


Cyclopropcne
Propyne

(c) Unsaturated alcohols and cyclic ethers (C„H2nO)


CH.—O
I I
C3HgO represents : CH2 = CH—CH2OH and CH2—CH2 which are ring chain isomers.
Prop-2-en-l-ol Oxetane
12/66 New Course Chemistry (XI) ornm

SUPPLEMENT YOUR
KNOWLEDGE FOR COMPETITIONS

Double bond equivalents. 1. The problems on structural isomerism and structure determination of organic
molecules can be easily solved by the concept of double bond equivalents (D.B.E.) which implies the
.ln{v- 2)
sum of double bonds and rings present in the molecule. It is given by the relation, D.B.E. = + 1
2

v/here n is the number of different kinds of atoms present in the molecule and v is the valency of each
3(4-2) + 60-2)+l{2-2)
atom. For example, D.B.E. of M.F. = +1=0+1=1

w
2

This means that various structural isomers of C^H^O will contain either one double bond (C = C or C = O)
or one ring (homocyclic or heterocyclic) as discussed below under Illustration 1.

F lo
Likewise, 2 D.B.E. implies either two double bonds or one triple bond or two rings or one double bond
+ one ring. Similarly, 4 D.B.E. implies either four double bonds or two triple bonds or two double bonds
+ one triple bond or three double bonds + one ring, i.e., benzene ring or any other combination of rings,
double and triple bonds.

ee
2. A stable compound has a whole number (but not fractional) value of D.B.E.

Fr
Illustration I. Write all the acyclic and cyclic isomers having the molecular formula, €311^0.
(AFMC 2009)

for
1
ur
Solution. D.B.E. = - [3 (4 - 2) + 6 (I - 2) + 1 (2 -2)] + 1 = 1

Since D.B.E. = 1, therefore, €311^0 may contain aC = CorC = 0 double bond or a ring. The following
seven isomers are possible,
s
ook
(a) Acyclic isomers are :
Yo
O OH O
eB

(0 CH3—C—CH3 (t'O CH3—C = CH2 (tautomer of acetone) {Hi) CH3CH2—C—H


Acetone Propionaldehyde
(rv) CH3CH = CH—OH (tautomer of propionaldehyde) (v) CH2 = CH—CH2OH
r

Prop-2-en-l-ol
ou
ad

(W) CH2 = CH—OCH3


Methoxyethene
Y

O
(b) Cyclic isomers are : (v«) (vm)
Re

CH3
nd

Oxetane Methyloxirane Cyclopropanoi


Thus, C3H(50 has nine isomers of which two are tautomers.
Fi

Note. While writing the structural isomers for any given molecular formula, tautomers are neglected unless
otherwise asked for.

Draw .structures of all the acyclic isomeric ethers corresponding to the molecular formula C5H12O.
Write condensed and bond line structural formulae for all the possible isomers of molecular formula C4Hg.
Draw the polygon formulae for all the possible structural isomers having the molecular fonnula CjHjq.
ORGANIC CHEMISTRY-SOME BASIC PRINCIPLES AND TECHNIQUES 12/67

ANSWERS

1. D.B.E.=l/2 [5 (4-2)+ 12(1 -2) + 1 (2-2)] +1 =0.


Since D.B.E. = 0, therefore, C5Hj20 represents only saturated ethers. The following six isomers are
possible.
ICH,
I ^
(/) CH3OCH2CH2CH2CH3 («) CH3O—CHCH2CH3
1-Methoxybutane
2-Melhoxybutane

low
1

CH3 CH3
1 2| 2
(Hi) CH3—O—CH2—CH—CH3 (iv) CH,—O—C—CH,
I
l-Methoxy-2-methylpropane 3CH3

ee
2-Methoxy-2-methylpropane

F
Fr
●CH,
2I
(v) CH3CH2~0~CH2CH2CH3 (vi) CH3CH2—O—CH—CH3

for
1-Ethoxypropane
ur 2-Ethoxypropane

I
2. D.B.E. = - [4 (4 - 2) + 6 (1 - 2)] + 1 = 2 s
ok
Since D.B.E. = 2, therefore, C4Hg has either two double bonds or one triple bond or one double bond and
Yo
a ring or two rings.
Bo

The following nine isomers are possible :

(/) CH2 = CH—CH = CH2 (ii) CH3CHr = CHot /


re

or

Buia-i, 3-diene But-l-yne


ou
ad

(Hi) CHj—C s C—CH3 or (iv) CH2 = C = CHCH3 or


But-2-yne Buta-1, 2-diene
Y

H
(V) (vO CH3 CH2
nd

(v/0 (vHi)
Re

CH3
Cyclobutenc 1-Melhylcyclopropene 3-Methylcycloprop-1 -ene Methylenecyclopropane
Fi

(ix)

Bicyclo(1.1.0] butane

3. By polygon formulae, we mean cyclic formulae. CjHjq has the following six polygon formulae :

Cyclopentane Methylcyclobutane Ethylcyclopropane 1, l-Dimethylcyclopropane 1, 2-Dimethylcyclopropane


12/68 Theuicc^'^ New Course Chemistry fXI^KTZsrm

12.10.2. Stereoisomerism
Isomers which have the same structural formula but have different relative arrangement of atoms or
groups in space are called stereoisomers (Greek : stereo - space, meros = part) and the phenomenon is
called stereoisomerism.

Stereoisomers are of the following two types :


(/■) Conformational isomers. Stereoisomers which differ in the relative position of atoms within the
molecule and which can be interconverted simply by rotation about sigma bonds are called conformational
isomers. In other words, the interconversion of these isomers does not require breaking and remaking of
covalent bonds. For details, refer to unit 13.

w
(»■) Configurational isomers. Stereoisomers which can be interconverted only by breaking and t emaking
of covalent bonds and not simply by rotation about sigma bonds are called configurational isomers. These
iire of two types : (/) Geometrical isomerism and (h) Optical Isomerism. Geometrical Isomerism we will

F lo
discuss in unit 13 under Alkenes but here we will discuss only optical isomerism.

12.10.2.1. Optical Isomerism'


Certain compounds can exist in two stereoisomeric forms which cannot be superimposed on one another.

ee
In spite of this difference in the relative spatial arrangement of atoms, these isomers have identical physical

Fr
properties such as melting point, boiling point, refractive index, density, solubility, etc. They also resemble
one another in most of their chemical reactions. However, they differ only in their behaviour towards plane

for
polarized light; one rotates the plane of polarized light towards right but the other towards left biit to the
same extent. Such a pair of compounds are called optical isomers and the phenomenon is called optical
ur
isomerism.

12.10.2.2. Some General Concepts about Optical Isomerism


s
ook
In order to understand the phenomenon of optical isomerism, we must be familiar with the following
Yo
terms:
eB

1. Plane polarized light. Ordinary light consists of


electromagnetic waves of different wavelengths. FIGURE 12.7

Monochromatic light, on the other hand, consists of waves


of only one wavelength. It can be obtained either by passing
our

>
ad

the ordinary white light through a prism or grating or by


using a source which gives light of only one wavelength.
For example, sodium lamp emits yellow light of about ORDINARY NICOL PLANE
LIGHT PRISM POLARIZED
589-3 nm** wavelength. Whether it is ordinary light or
Y

LIGHT
monochromatic light, it consists of waves having vibrations
Re

in all the planes perpendicular to the line of propagation of Plane polarized light
nd

light. If such a beam of light is passed through a Nicol


Fi

prism (made from a particular crystalline form of CaCOj known as calcite) the light that comes out of the
prism has vibrations only in one plane. (Fig. 12.7).
Such a beam of light which has vibrations only in one plane is called plane polarized light.
The Nicol prism used to obtain plane polarized light is called a polarizer.
2. Angle of rotation. The angle (in degrees) through which the plane of the polarized light gets rotated
on passing through the solution ofan optically active compound is called the angle of rotation. It is denoted
by the Greek letter ‘a’.

*Included only in the syllabus of ISCE.


**The abbreviation nm means nanometer, 1 nm = 10"^ m = 10 x 10“* cm = 10 A.
ORGANIC CHEMISTRY-SOME BASIC PRINCIPLES AND TECHNIQUES 12/69

3. Polarimeter. The instrument used for measuring the angle of rotation of the plane polarized light is
called a polarimeter (Fig. 12.8).

i la

A"
,21
u ^ r ' r EYE
LIGHT FIRST SOL. OF AN SECOND
SOURCE NICOL PRISM OPTICALLY NICOL PRISM

w
(POLARIZER) ACTIVE SUBSTANCE (ANALYSER)

Schematic repi^enitatiohbf^ij^iy^ettt

F lo
It consists of a light source (usually a sodium lamp), two Nicol prisms and a polarimeter tube. The Nicol
prism near the light source is called the 'polarizer* while that near the eye of the observer is called the
analyser. The axis of polarization of the polarizer is fixed while that of Uie analyser can be changed. In

ee
between the polarizer and the analyser is placed a glass tube called the polarimeter tube which contains the

Fr
solution of the optically active substance.
The solid lines represent the plane of polarized light before and broken lines represent after passing
through the solution of an optically active substance.

for
If the axis of polarization of the two Nicol prisms is held parallel (as shown in Fig. 12.8) to each other,
ur
and the polarimeter tube is empty, the plane polarized light from the polarizer passes through the analyser
without any change in intensity. Conversely, if the two Nicol prisms are held with their axes of polarization
perpendicular to each other, no light passes at all.
s
ook
However, when the solution of any optically active compound is placed in between the polarizer and the
Yo
analyser with their axes of polarization parallel to each other, the intensity of the light coming out of the
analyser is much less than that falling on the polarizer. In such cases, in order to have maximum transmission
eB

of the light, the analyser has to be rotated through some angle in one direction or the other. This angle through
which the analyser has to be rotated to get the maximum transmission of light is called the angle of rotation.
The angle of rotation (a) of an optically active compound depends upon the following factors :
r

(/) Nature of the optically active substance


ad
ou

(k) Concentration of the solution in g/mL


(Hi) Length of the solution through which the light passes
Y

(iv) Nature of the solvent


(v) Wavelength of the monochromatic light used
Re
nd

(vi) Teihperature of the sample


4. Specific rotation. In order to compare the rotatory powers of different optically active substances,
Fi

the optical activity is usually expressed in terms of specific rotation. The specific rotation of an optically
active substance is defined as the degrees of rotation observed when the plane polarized light is passed
through a tube having a path length of 1 decimeter (10 cm) and a concentration of 1 g/mL of the compound
at a specified temperature and wavelength. It is denoted by [a] ^ where t is the temperature and D is the
wavelength of light used. Usually D-line of sodium (vapour lamp) having a wavelength of 589-3 nm is used
as the source of the monochromatic light.
Since it is not always possible to dissolve 1 g of the optically active substance in 1 mL of the solvent,
therefore, the measurement of the angle of rotation is usually carried out at some lower concentration. The
specific rotation is then calculated by the application of the following equation.
12/70 ‘P,neteCecf.i''A New Course Chemistry (XI)CEIHD

Observed angieof rotation(a)


[aj D
I X C

where / is the lengtli of the polarimeter tube in decimeters and c is the concentration of the substance in g/mL.
While reporting the specific rotations of optically active substances, it is customary to include the
concentration of the solution used for measurement along with the solvent (used for making the solution) in

w
parentheses after the specific rotation. For example, the specific rotation of cane sugar at 293 K using D-line
of sodium vapour lamp.
= + 66-5° (c = 0-02 g/mL, water)
The +sign before the angle of rotation denotes that cane sugar is dextrorotatory.

o
Just like melting point, boiling point, density, refractive index, etc., the specific rotation is a characteristic

e
property of an optically active compound.

re
rFl
12.10.2.3. Optical Activity-Discovery

F
The phenomenon of optical activity was discovered by the French Chemist, Louis Pasteur in 1848. He
observed that when a concentrated solution of sodium ammonium tartrate was allowed to crystallise, two
types of crystals separated out. Working carefully with a magnifying glass and a pair of tweezers, Pasteur was
able to separate the crystals into two piles, one of right handed and the other of left handed crystals. Each type

r
ou
of crystals when dissolved in water was found to rotate the plane of polarized right.

fo
One solution rotated the plane of polarized light towards right while the other towards left but to the

ks
same extent. Later on, millions of molecules have been discovered which rotate the plane of polarized light.
Such substances which can rotate the plane of polarized light are called optically active substances
oo
and the property of a substance to rotate the plane of polarized light is called optical activity.

12.10.2.4. Molecular Asymmetry and Chirality - Cause of Optical Activity


Y
B

Lious Pasteur attributed this difference in optical activity to the three dimensional anmigement of atoms
[i.e., configurations) in the two types of crystals. Dutch scientist J. van’t Hoff and French scientist
C. Le Bel both independently in the same year (1874)
re

FIGURE 12.9
pointed out that the four valencies of a carbon atom
are directed towards the corners of a regular MIRROR MIRROR
ou
Y

tetrahedron and if all the atoms or groups attached to


ad

p q A A
a carbon atom are different, such a carbon is called
asymmetric carbon or chiral carbon or
stereocentre.
d

A molecule which contains an asymmetric


in

Non-superimposable Superimposable
Re

carbon lacks all elements of symmetry and hence is


(Chiral or dissymmetric) (Achiral or non-dissymmetric)
called an asymmetric molecule and it is this
Mirror Mirror
asymmetry of the molecule which is responsible for
F

optical activity in such organic compounds. n Glass


The symmetry and asymmetry are also observed tumblers

in many day to day objects. For example, alphabet


letters A, button, a sphere, a cube and glass tumblers
(Fig. 12.9) are all identical with their mirror images
and hence are superimposable. However, many Feet
o sphere

objects are not superimposable on their mirror images. Button


For example, alphabet letter P, your left hand and Chiral objects Achiral objects
right hand (Fig. 12.10) look similar but if you put Non-superimposable Superimposable
mirror images mirror images
your left hand on your right hand, they do not
coincide. Similarly, feet, gloves and shoes are not Some common examples of chiral and achiral objects.
superimposable.Thus,
ORGANIC CHEMISTRY-SOME BASIC PRINCIPLES AND TECHNIQUES 12^1

FIGURE 12.10
Objects which are non-superimposable
MIRROR
on their mirror images are called chiral
and the property of non-super’
imposability is called chirality while those
which are superimposable on their
mirror images are called achiral.

Thus, the ultimate test of chirality is the non-


superimposabiliry of an object and its mirror image. IMAGE

Let us now apply this lest of molecular chirality to OBJECT NON-SUPER


organic molecules. To check the non-super- -IMPOSABLE

imposabilty, construct the models or draw the tliree A chiral object cannot be superimposed
dimensional structures of organic molecules and their on its mirror image

F low
mirror images and try to superimpose them. But this
is a difficult test. A simple test of molecular
chirality is the presence of a single asymmetric carbon atom in the molecule. Let us consider two simple
molecules, propan-2-ol and buian-2-ol and their mirror images. As is evident, propan-2-ol does not contain an
asymmetric carbon atom, therefore, the molecule is achiral. Let us now confirm it by the superimposability
test.

The three dimensional structure of FIGURE 12.11

e
for Fre
Mirror
propan-2-ol is A and that of its mirror image
is B. To check the superimposability of B CH3 CH3
over A. rotate B through 180° in such a way % I

that the C—OH bond in the new structure C .'C OH ! OH C. OH


\
projects in the same direction as that in the J \
H
CH3
structure A. Now superimpose it over A, we H
A B
H
C
find, A and C are superimposable (Fig.
Your
Rotate through 180°
eBo ks

12.11). Hence, propan-2-ol is an achiral


molecule. Propan-2-ol is achiral because A and its mirror ^
image B (or C) are superimposable.
Now consider butan-2-ol. It contains
one asymmetric carbon atom. Therefore, it
ad

FIGURE 12.12
should be a chiral molecule. Let us now
our

Mirror
confirm it by the non-superimposability test. I

The three dimensional structure of butan-2-


HsQ-, PH3 (j^ H5C2
ol is D and that of its mirror image is E.
OH I HO OH
Re

Now rotate E through 180° in such a way


that the C—OH bond in the new structure F /
H H H
projects in the same direction as that in the
Find Y

D E F
structure D. Now superimpose F over D, we Rotate through 180°
find, D and F are non-superimposable (Fig. Butan-2-ol is chiral because D and its mirror i
"y

12.12), no matter how we turn and twist the


£ (or F) are non-superimposable.
molecule without breaking the bonds.
Hence, butan-2-ol is a chiral molecule.
From the above discussion, we conclude that the presence of a single asymmetric carbon atom makes
the molecule chiral and hence optically active. That is why, asymmetric carbon is also called a chiral
carbon.

In chiral molecules, the asymmetric carbon or the chiral carbon is denoted by an asterisk (*). Other
examples of chiral and hence optically active molecules are :
12/72 ‘PxiuCee^'A. New Course Chemistry (X1)S!S1SD

I * 3 4 * 1 * 3 1

CH,—CH—CH^CH^ 3CH _CH—CHO Br—CH—I CH,—CH—COOH


3 2|
3 21 2 3 I 2 21 Cl
Cl OH OH Br

2-Chlorobutane 2, 3-Dihydroxypropanal Bromochloroiodomeihane 2-Bromopropanoic acid

w
{Glyceraldehyde)

12.10.2.5. Enantiomers

Optical isomers which are non-superimposab!e mirror FIGURE 12.13

images of each other are called enantiomers and the Mirror

o
phenomenon is called enantiomerism. The enantiomers have CH3 CH3
identical physical and chemical properties but rotate the plane

e
of polarized light in opposite directions but to the same extent.

re
y V

rFl
To understand the phenomenon of enantiomerism, let us COOH ; HOOC
consider the molecule of lactic acid. It contains one
asymmetric

F
HO OH
II
carbon (chiral carbon) atom and hence exists in two
stereoisomeric fontis, I and II, which are non-superimposable Enantiomers of lactic acid
mirror images of each other (Fig. 12.13).

r
ou
Further, each of the two isomers is optically active and rotates the plane of polarized light in the opposite

fo
directions but to the same extent. Thus, I and II represent enantiomers of lactic acid. Similarly, D and E (or D

ks
and F) are enantiomers of butan-2-ol.
The enantiomer which rotates the plane of polarized light towards right is called dextrorotatory and is
represented as 'd' or (+) while the enantiomer which rotates the plane of polarized light towards left is
oo
called laevorotaiory and is represented as 7’ or (-). However, it may be emphasized here that it is not
Y
possible to tell as to which isomer is (+) or (-) just by looking at the structure. It has to be determined
B

experimentally.
12.10.2.6. Racemic mixture
re

A mixture of equal amounts of two enantiomers is called racemic mixture or racemic


modification.
ou
Y
ad

A racemic mixture is always optically inactive because the rotation caused by the molecules of one
enantiomer is exactly cancelled by equal and opposite rotation caused by the same number of molecules of
the other enantiomer. It is represented by prefixing dl or (±) before the name. For example, (±) butan-2-ol.
d

The process of conversion of one enantiomer (+ or -) into a racemic mixture is called


in
Re

racemization.
F

12.10.2.7. DIastereomers

Stereoisomers which are not mirror images of each other are called diastereomers and the phenomenon
is called diastereoisomerism. For example, 2, 3-dichloropentane exists in four stereoisomeric forms, i.e., I,
II, III and IV.
CH3 Mirror CH3 CH3 Mirror CH3
H—C—Cl Cl—C—H H—C—Cl Cl—C—H

Cl—C—H H—C—Cl H—C—Cl Cl—C—H

CH2CH3 CH2CH3 CH2CH3 CH2CH3


I II III IV
ORGANIC CHEMISTRY-SOME BASIC PRINCIPLES AND TECHNIQUES 12/73

I and II are non-superimposable mirror images of each other and hence are enantiomers. Similarly, III
and IV are enantiomers. On the other hand, I and III, I and IV, II and III and II and IV are not mirror images
of each other and hence are diastereomers.
Properties of diastereomers : (/) Since diastereomers are not mirror images of each other, they have
different melting points, boiling points, densities, refractive indices and different solubilities in the given

ow
solvents.

(«') Diastereomers may be either optically active or inactive. For example, geometrical isomers which
are also regarded as diastereomers are optically inactive while others are usually optically active. Even the
optically active diastereomers may have same or opposite signs of rotation.
Number of Stereoisomers. Number of stereoisomers depends upon the number of dissimilar chiral

e
carbon atoms present in a molecule. For compounds containing one chiral carbon atom, there are two

re
stereoisomers {enantiomers). For compounds containing two dissimilar chiral carbon atoms, there are four
stereoisomers. Working on similar lines, we conclude that for compounds containing n dissimilar chiral

Frl
carbon atoms there are 2“ stereoisomers. For example, the following compound has

F
« «

CH2OH - CHOH - CHOH - CHOH - CHOH - CHO


ou
or
four dissimilar chiral carbon atoms and hence has 2^^ = 16 stereoisomers.

12.10.2.8. Mesd compounds

kfs
Now consider tartaric acid.
* *
oo
HOOC - CHOH - CHOH - COOH

Like 2,3-dibhloropentane, it also has two chiral carbons and hence, in principle, should exist in four
Y
stereoisomeric forms, i.e., I, II, III and IV.
eB

MIRROR MIRROR
COOH I
COOH COOH COOH
ur

I
I
oY

HO—C—H I H—C—OH H—C—OH HO—C—H


Plane of
ad

t
V

symmetry
H—C—OH HO—C—H H—C—OH HO—C—H
d

COOH COOH COOH COOH


1
in

II III IV
Re

Non-superimposable Superimposablc mirror images-


mirror images-enantiomers represents only one stereoisomer
F

Stereoisomers, I and II, are mirror images of each other. They are neither superimposablc nor
interconvertible by rotation about carbon-carbon bonds. Thus, they are enantiomers. One of these enantiomers
is dextrorotatory, i.e. (+)-tartaric acid and the other is laevorolatory, i.e., (-)-tartaric acid.
Stereoisomers, III and IV, are also mirror images of each other but become superimposable if one of the
structures is rotated through 180° in the plane of the paper. As a result, stereoisomers III and IV are identical
and represent only one stereoisomer.
This stereoisomer III (or IV) is, however, different from both I and II. Whereas I and II are optically
active. III (or IV) is not. This is due to the reason that this stereoisomer has a plane of symmetry—i.e., one
12/74 New Course Chemistry (XI)BSiai]

half of the molecule is a mirror image of the other hal.f As a result, the stereoisomer III (or IV) is not chiral
or dissymmetric even though it contains tw’o chiral carbon atoms. Consequently, the stereoisomer III (or IV)
is optically inactive. Such an optically inactive compound whose molecule is superirnposable on its mirror
image in spite of the presence of chiral carbon atoms is called a meso compound.
It may be noted that a meso compound is optically inactive because the rotation caused by one half of
the molecule is exactly cancelled by equal and opposite rotation caused by the other half of the molecule. In
other words, the optical inactivity of meso compounds is due to internal compensation (compensation within
the molecule). On the other hand, the optical inactivity of a racemic modification is due to the reason that the
rotation caused by the molecules of one enantiomer is exactly cancelled by equal and opposite rotation

w
caused by the same number of molecules of tlie other enantiomer. In other words, the optical inactivity of the
racemic mixture is due to external compensation (compensation due to the other molecules).
From the above discussion, it follows that tartaric acid exists in three stereoisomeric forms, i.e., (+)-

F lo
tartaric acid, (-)-tartaric acid, and m^.vo-tartaric acid. A racemic modification, i.e., (±)-tartaric acid can be
obtained by mixing equimolar amounts of I and II, i.e., (+)-tartaric acid and (-)-tartaric acid. Since a racemic
modiifcation contains two compounds, it can be resolved. A meso-compound, on the other hand, can’t be

ee
resolved since it consists of molecules of only one .stereoisome.r

Fr
It may be noted that only those compounds (such as 2, 3-dichlorobutane, butane-2, 3-diol, tartaric
acid, etc.) which have two similar chiral carbon atoms exist in three stereoisomers, out ot which, two are

for
enantiomers while the third is the mtw-compound.
ur
PART III. FUNDAMENTAL CONCEPTS OF ORGANIC REACTION MECHANISM

In an organic reaction, the organic compound called the substrate reacts with a suitable attacking
s
ook
species called the reagent to form products. The formation of products may occur either directly from the
Yo
reactants through a transition state or through the formation of one or more intermediates. In addition to the
eB

final products, sometimes byproducts are also formed from the intermediates. The whole sequence of events
is sketched below :

Products (s)
our
ad

Attacking
Organic molecule [Intennediate]
reagent

By products (s)
Y

Substrate is that reactant which supplies carbon for the formation of a new bond while the other reactant
Re

is called the reagent. If both the reactants supply carbon to the new bond, then choice is arbitrary ; usually the
nd

molecule on which attention is focused is called the substrate.


Fi

During an organic reaction, a covalent bond between two carbon atoms or a carbon and some other
atom is broken and a new bond is formed. Thus,

A sequential account of each step, describing details of electron movement, energetics during
bond cleavage and bond formation, and the rates of transformation of reactants into products (kinetics)
is called the reaction mechanism.

A knowledge of reaction mechanism is useful in understanding the reactivity of organic molecules. It


also helps us in planning strategy for their synthesis. Therefore, in this section, we shall study some of the
basic principles which explain how organic reactions take place.
ORGANIC CHEMISTRY-SOME BA PRINCIPLES AND TECHNIQUES 12/75

12.11. ELECTROPHILES AND NUCLEOPHILES


1. Electrophiles are electron loving chemical species. Their attractionfor electrons is due to the presence
of an electron- deifcient atom in them. Electrophiles may be either positively charged or electrically neutral
chemical species, i.e.,

ow
+

(/) Positive electrophiles : H'*’, Cl'^ (chloronium ion), (bromonium ion), I"*" (iodonium ion), NO2
+

(nitronium ion), NO (nitrosonium ion), (carbocation), etc.

(/i) Neutral electrophiles : R- (free radicals), ; CR2 (carbenes), : NR (nitrenes), SO3, BF3, AICI3,
FeCl3, SnCl4, etc.

e
Since both positively charged and neutral electrophiles are short by a pair of electrons (free radicals are,
however, short by one electron), they have a strong tendency to attract electrons from other sources and hence

re
behave as Lewis acids. Electrophiles always attack the substrate molecule at the site of highest electron
density.

Frl
F
Note. Please note that all the positively charged species do not always act as electrophiles. Only those
positively charged species which can accept a pair of electrons can act as electrophiles. Thus, NH^,
Na'*', Ca^'^ , etc. cannot act as electrophiles since
ou all of these already have an octet of electrons in their

r
respective valence shells.

so
2. Nucleophiles are nucleus loving chemical species. Since the nucleus of any atom is positively charged,
therefore, nucleophiles must be electron rich chemical species containing at least one lone pair of electrons.

kf
They may be either negatively charged or neutral chemical species, e.g.,
oo
(0 Negative nucleophiles : H (hydride ion), BH^ (borohydride ion), AlH^ (aluminium hydride ion),
Cr. Br- I- R- (carbanion). RC = C", OH", OR" SR“, NH^, CN" NO2, , RCOO", etc.
Y
eB

iii) Neutral nucleophiles : H2O NH3 , RNHj , R2NH, R3N , ROH,RSH,ROR, R-MgX, R-
Li, etc.
Since both negatively charged and neutral nucleophiles contain at least one unshared pair of electrons,
ur

they have a strong tendency to donate this pair of electrons to electron deficient species and hence behave as
oY

Lewis bases. Nucleophiles always attack the substrate molecule at the site of lowest electron density.
ad

12.12. FISSION OF A COVALENT BOND

1. Homolytic (symmetrical) fission. If a covalent bond breaks in such a way that each atom takes
d

away one electron of the shared pair, it is called homolytic or symmetrical fission. Homolytic fission is
usually indicated by a fish arrow which denotes a one-electron displacement. For example,
in
Re

Homolytic fission
Ails A + B
F

Free radicals

The neutral chemical species (such as A- and B-) which contain an odd or unpaired electron and which
are produced by homolytic ifssion of covalent bonds are called free radicals. Homolyticfission usually
occurs in non-polar bonds and is favoured by high temperature, ultraviolet (UV) radiations and by the
presence of radical initiators such as peroxides.
2. Heterolytic (unsymmetrical) fission. When a covalent bond joining two atoms A and B breaks in
such a way that both the electrons of the covalent bond (i.e., shared pair) are taken away by one of the
bonded atoms, the mode ofbond cleavage is called heterolytic fission. Heterolytic fission is usually indicated
by a curved arrow which denotes a two-electron displacement. For example.
r\ Heterolytic fission f\ Heterolytic Fission
A : B > "A" + :B" ; A : B a:"+6"^
(When B is more electronegative than A) (When A is more electronegative than B)
12/76 ‘P'tadee^'O' New Course Chemistry (XI)SIE]MQ

As shown above, heierolytic fission results in the formation of charged species, x.e., cations and anions.
It usually occurs in polar covalent bonds and is favoured by polar solvents.
Sample Problem 12.11 Using curved arrow notation, show the formation of reactive
intermediates when the following covalent bonds undergo heterolylic cleavage, (a) CH3—SCH3 (ft) CH3—CN
(c) CH3—Cu (NCKRT Solved Example)
Solution, (a) Although both C and S have same electronegativity (2-5) but the cleavage of the C—S bond

ow
occurs towards S atom because S being bigger in size than C can disperse the -ve charge more effectively.

(«) CH3-^SCH3 CH3 + "SCH3 (b) CH3-^CN CH3 + "CN


CN is more electronegative than CH3)
(■.'S being bigger in size than C can disperse the
-ve charge more effectively)

e
(c) CH3-‘- Cu -:ch3 + Cu* (●.● Cu is more electropositive than C)

re
rFl
Sample Problem 12.12 Giving proper justification, categorise the following molecules/ions as

F
+

nucleophile or electrophile : HS , BF3, C2H5O , (CH3)3N Cl^, CH3C- O, H2N NO2.


(NCERT Solved Example)

r
ou
Solution. (0 Nucleophiles : HS", C2H5O", (CH3)3N:, H2N"

fo
All these species have one or more lone pairs of electrons which it can easily donate to an electrophile and
hence behave as nucleophiles.
ks
oo
00 Electrophiles : BF3, Cl-^, CH3C = 0« NO2
All the positively charged species have a sextet of electrons around the positively charged atom and hence
Y
eB

can accept a pair of electrons and thus behave as electrophiles.


Similarly, B has only a sextet of electrons in the valence shell and hence BF3, though neutral, also acts as an
electrophile.
r

12.13. ELECTRONIC DISPLACEMENTS IN A COVALENT BOND


ou
ad
Y

12.13.1. Inductive Effect

Whenever an electron-withdrawing atom such as halogen i.e. -X (or a group such as nitro) is attached
to the end of a carbon chain, the a-electrons of the C—X bond are attracted by or displaced towards the more
d

electronegative halogen atom. As a result, the atom X acquires a small negative charge (i.e. 6") and Cj
acquires a small positive charge (i.e. 6*) as shown below.
Re
in

565-^ 55+ 6+ 6-
C4 —>— C3 — C, ^C, X
F

The small positive charge on Cj, in turn, attracts the a-electrons of the C| - C2 bond towards it. As a
result, C, acquires a small positive charge (i.e., 55+), of course, smaller than that on Cj. Similarly, C3 will
acquire a small positive charge (i.e. 555+) that will still be smaller than that on C2. This type of displacement
of o-electrons along a saturated carbon chain whenever an electron withdrawing (or electron donating)
group is present at the end of the chain is called the inductive effect or the I- Effect. This effect weakens
steadily with increasing distance from the .substituent (electron-withdrawing or electron-donating group)
and actually becomes negligible after three carbon atoms. There are two types of inductive effects, i.e.,
-I-effect and + I-effect.

(/) If the substituent attached to the end of the carbon chain is electron-withdrawing, the effect is
called -I-effect. For example,
ORGANIC CHEMISTRY-SOME BASIC PRINCIPLES AND TECHNIQUES 12/77

556+ 56+ 6+ 6-
C —)— C — C —)— X (electron-withdrawing substituent) (-I-Effect)
The -1-effect of some of the atoms and groups in the decreasing order w.r.t. to hydrogen is:
-NOo > -CN > -COOH > -F > -Cl > -Br > -I > H

{//) If the substituent attached to the end of the carbon chain is electron-donating, the effect is called
+I-effect. For example,
665- 68- 5-
C— C— C CH3 (electron-donating substituent) (+I-eflect)
The +I-effect of some of the atoms or groups in the decreasing order w.r.t. to hydrogen is ;

w
(CH3)3C- > (CH3)2CH- > CH3CH2- > CH3- > D- > H-
r-Butyl Isopropyl Ethyl Methyl

F lo
Inductive effect is a permanent effect operating in the ground state of the organic molecules and hence
is responsible for high melting point, boiling point and dipole moment of polar compounds.
12.13.2. Electromeric Effect

ee
It involves the complete transfer of electrons of a multiple bond (double or triple bond) to one of the
bonded atoms (usually more electronegative) in presence of an attacking reagent. It is called E-effecL

Fr
\ Or Reagent added
C^O C—O-

for
Reagent removed

This effect is temporary and takes place only in the presence of a reagent. As soon as the reagent
ur
is removed, the molecule reverts back to its original position. Electromeric effect is of two types, i.e.,
+ E-effect and -E-effect.
s
If the electrons of the ti-bond are transferred to that atom of the double bond to which the reagent gets
ook
Yo
ifnally attached (i.e., towards the attacking reagent), the effect is called + E-effect. For example, addition of
acids to alkenes.
eB

c=^c: + H*" :C—C (+E-Effect)


our

H
ad

If on the other hand, the electrons of the double bond are transferred to an atom of the double bond
other than the one to which the reagent gets finally attached (i.e., away from the attacking reagent), the
effect is called -E-effect. For example, the addition of cyanide ion to the carbonyl group.
Y
Re

C=fO + CN- > :c—O" (-E-Effect)


T>
nd

CN
Fi

From the above discussion, it is evident that the unsaturated systern shows +E-effect with electrophiles
and -E-effect with nucleophiles.
12.13.3. Resonance or Mesomerism
Sometimes it is not possible to assign a single electronic (Lewis) structure to a molecule which can
satisfactorily explain all its properties. In such a case, it has been found that the molecule can be represented
by two or more electronic structureseach one of which can explain mo.st of the properties but none of them
can explain all the properties of the molecule. The real structure of the molecule lies somewhere in between
all these electronic structures which, however, cannot be represented on paper. Such a molecule is said to
exhibit resonance or mesomerism. Thus, the phenomenon of resonance is said to occur whenever for a
molecule we can write two or more Lewis structures which differ in the position of electrons but not in the
relative position of atoms. The various Lewis structures, none of which is capable of describing all the
12/78 ‘Pn,adeefi> U New Course Chemistry (XI) IM«JWII

known properties of the compound, are called canonical or resonance structures. The actual structure of
the molecule is not represented by any of the resonance structures but is a resonance hybrid of all these
Lewis structures. The various resonance structures are separated by a double headed arrow —>).

Thus, in the light of resonance theory, benzene can be i epresented as a resonance hybrid of the following
two Kekule* (Lewis) structures, I and II.

w
-4-
Q
I II ni

Any of these two Kekule structures, cannot explain all the properties of benzene. According to these
structures, a molecule of benzene should have three carbon-carbon single bonds of 1.54 A length and three

o
carbon-carbon double bonds of 1.34 A length. But actually it has been found that all the six carbon-carbon

e
bonds in benzene are equal (1.39 A). This implies that actual structure of benzene is neither represented by

re
I nor by n but is a resonance hybrid of these two structures. In other words, any two adjacent carbon atoms

Frl
in benzene are neither joined by a pure single bond nor by a pure double bond. As a result, all the carbon-

F
carbon bond lengths are equal, i.e., 1.39 A and lie in between carbon-carbon double bond length of 1.34 A
and carbon- carbon single bond length of 1.54 A. The resonance hybrid or the actual molecule of benzene
is usually represented by the formula HI. The circle inside the ring denotes completely delocalized six n-
electrons.

or
ou
Similarly, carbon dioxide can be written as a resonance hybrid of the following three structures (IV, V
and VI):

kfs
O A
:o=fec=o: ■* ►

-:0—C=0:
● +

^—►
+

:o=c—or
● ●

● ● ● ●
oo
rv V VI

Due to contributions of structures (V and VI), the carbon-oxygen bond in CO2 has some triple bond
Y
character. As a result, the carbon-oxygen bond length (1.15 A) in CO2 lies in between normal carbon-oxygen
B

double bond length of 1.22 A and carbon-oxygen triple bond length of 1.10 A.
Like benzene and CO2 , carboxylic acids may be regarded as a resonance hybrid of the following two
re

structures (Vn and Vni):


5-
oYu

:or O
Cj*
A
ad

+ 8+
R—C^O—H < ► R—C=0—H R_C-^o—H ● ●

vn VIII Resonance hybrid


d

Like carboxylic acids, nitroalkanes may be regarded as a resonance hybrid of two structures
(IX and X):
in
Re

s-
.0:"
R— ◄ ► R—N R—N:
F

o:- o: O
Resonance hybrid
IX X

’'‘Besides the two Kekule structures, the actual benzene molecule has significant contribution from of the
following three Dewar structures.

◄ ► ◄ ►

Thus, benzene may, in fact, be regarded as a resonance hybrid offive structures, i.e., two Kekule structures
and three Dewar structures. The contribution of each
Kekule structure towards the resonance hybrid (real structure)
is about 39% while that of each Dewar structure is about 7%.
ORGANIC CHEMISTRY-SOME BASIC PRINCIPLES AND TECHNIQUES 12/79

Like molecules, ions also show resonance. For example, CO|”, NOj, etc.
(/) Resonance structures of CO|~ ion :
5-

A >
-:o
./tCc—o: <
Oc

-:o^
● ●

-:o o
Contributing structures Resonance hybrid

(n) Resonance structures of NO3 ion.

w
8-

Or ■:o + Oc s-
^N—O:-
► ,r^N=^0: ◄—►

F lo
●Ay'
-:o :o :o O
Contributing structures Resonance hybrid
Resonance energy. A resonance hybrid (or the actual molecule) is always more stable than any of its

ee
canonical structures (hypothetical or imaginary structures). This stability is due to delocalization of electrons
and is measured in terms of resonance energy or delocalization energy. It is defined as the difference in

Fr
internal energy of the resonance hybrid and the most stable canonical structure. Furthe,r more the number
of equivalent resonance structures, greater is the delocalization of electrons, higher is the resonance energy

for
and hence more stable is the compound. In case of benzene, this resonance energy has been determined to be
150-62 kJ (or 36 kcal) mor^
ur
Rules for writing resonance structures. The following rules are followed while writing resonance
structures:
s
(a) The various resonance structures should differ only in the position of electrons and not in the
ook
Yo
position of atoms or nuclei,
ib) All the resonance structures should have the same number of unpaired electrons,
eB

(c) In case of atoms of the second period in the periodic table, such resonance structures which violate
octet rule should not be considered. For example, structure (II) cannot be considered as a resonance

CH2^^CH^NH3 < X > CH2—CH=NH3


our
ad

I II
structure since it violates the octet rule because nitrogen has 10 electrons in the valence shell. But, nitrogen
cannot have more than 8 electrons because it does not have d-orbitals.
Y

(d) As far as possible, all the resonance structures should have nearly the same energy.
The more the number of resonance structures, more stable in the resonance hybrid. In general, the
Re
nd

various resonance structures contribute to the resonance hybrid in proportion of their relative energies. The
following points must be considered while deciding the relative energy of the various resonating structures.
Fi

Relative contributions of Resonance structures.


(0 Structures which are indistinguishable are of equal energy and hence contribute equally towards the
resonance hybrid. For example.

<■ and CH2—CH=^CH2 <■ > CH2=CH—CH2

(«■) Structures with greater number of covalent bonds contribute more towards the resonance hybrid.
For example, 1, 3-butadiene is a resonance hybrid of structures (I, II and HI)
+

CH2=^CH—CH^CH2 <■ ► CH2—CH=CH—CH2 < ► CH2—CH=CH—CH2


I n III
12/80 ‘Prtadee^'^ New Course Chemistry (XI)ESE

Since formation of a bond is accompanied by release of energy, therefore, structure (I) with two 7C-bonds
is more stable than structures (II and IE) which contain one jc-bond each. Hence, structure (I) makes more
contribution towards the resonance hybrid than structures (II and III).
(Hi) Structures which involve separaHon ofpositive and negative charges are of higher energy and hence

w
contribute little towards the resonance hybrid. For example, contribution of stmctures (II and HI) which involve
separation of positive and negative charges is much less than the stracture (I) otwards the resonance hybrid of 1,3-
butadiene. Similarly, the contribution of stmcture (V) which involves separation of positive and negative charges is
much less than the structure (IV) otwards resonance hybrid of carboxylic acids.
:6:-
A

e
o
R—C-^O—H > R—C=0—H

re
IV (more stable) V (less stable)
(iv) Lesser the sepaf-ation ofpositive and negative charges, more stable is the resonance structure.

Frl
For example, out of the resonance structures (VII and VIE) for 1-methoxybuta-l, 3-diene, which have

F
the samenumberof covalentbonds, structure(VEI) is more stable than structure (VE) since it involves less
charge separation.

Ol 1^ A ou +

sor
CH2±CH-tCH±CH—OCHj ► -:CH2—CH:^CH—CH=0—CHj ■* ►
1-Methoxybuta-1,3-diene (VI) VII (less stable)

kf
oo CH2=CH—CH—CH=0—CH3
VIII (more stable)
(v) When atoms of different electronegdtivities are involved, the structure with a negative charge on the
Y
more electronegative atom and positive charge on the less electronegative atom is of lower energy and hence
B

contributes more towards the resonance hybrid than the alternate structure in which the charges are reversed.
For example, structure (XI) with the positive charge on the more electronegative oxygen atom and
R\ Or R
re

^c—or ^^ ^ c—o:
oY

● ●

R R'
u

IX (most important) X (less important) XI (least important)


a negative charge on the less electronegative carbon atom contributes less towards the resonance hybrid of a
ad

ketone (IX) as compared to the alternate structure (X) with the charges reversed.
d

(vi) Structures having like charges on adjacent atoms are highly unstable and hence contribute little
towards the resonance hybrid. For example, structure (XEI) makes only negligible contribution towards the
in

resonance hybrid of diketone (XE).


Re

● ●

:br :b:“
F

I I
R—C C—R -» > R—C C—R
XII XIII
1,2-Diketone (Negligible contribution)
(Most important)
(vii) Structures which help to delocalize the positive charge make important contribution towards the
resonance hybrid regardless of the fact whether the positive charge is on less or more electronegative atom.
For example, structure (XV) makes a significant contribution towards the resonance hybrid of the carbocation
(XTV) since it helps to disperse the positive charge.
-A
(CH3)2C—O—H < > (CH3)2C=0—H
XIV XV
ORGANIC CHEMISTRY-SOME BASIC PRINCIPLES AND TECHNIQUES 12/81

(vm) Resonance structures in which all the atoms have octet of electrons make larger contribution
towards the resonance hybrid than those structures in which one or more of the atoms do not have octet of
electrons.

For example, carbon atom/s in structure (X), XIII and XIV bearing the positive charge have only a
sextet of electrons and hence the contribution of these
structures towards the resonance hybrid of their respective
molecules is small.

12.13.4. Resonance Effect or Mesomerk Effect


In case of conjugated systems (having alternate a-and 7t-bonds), the electrons can flow from one par* of the
system to the other due to resonance. This flow of electrons from one part of the conjugated system to the other

w
creating centres of low and high electron density due to the phenomenon of resonance is called resonance effect
(R-effect) or mesomelic effect (M-effect). It is also called n-n or n-n conjugation. It is of two types :
(/) Groups which donate electrons to the double bond or to a conjugated or aromatic system are said to

F lo
have +R or +M-effect. For example, -OH, -OR -SH, -SR, -NH2, -NHR, -NR^, -Cl, -Br, -I, etc. Thus,
●O A +

CH2=CH^g: < > :cH2—CH=Ci: (+ R-effect)

ee
Vinyl chloride

Fr
(ii) Groups which withdraw electrons from the double bond or from a conjugated system towards
themselves due to resonance are said to have -R or -M-effect. For example, >C = O, -CHO, -COOR,
-CN, -NO9, etc. Thus,

for
+

CH2—CH—CiN: > CH2—CH=^C=Nr (- R-effect)


ur
Acrylonitrile

12.13.5. Hyperconjugation Effect


s
The inductive effect of the alkyl groups on a saturated carbon chain follows the order :
ook
Yo
(CH3)3C- > (CH3>2CH- > CH3CH2- > CH3-
However, when an alkyl group is attached to an unsaturated system such as a double bond or a benzene
eB

ring, the order of inductive effect is actually reversed. This effect is called hyperconjugation effect or
Baker-Nathan effect.
FIGURE 12.14
In fact, hyperconjugation effect is an extension of the tr-TU-CONJUGATION
r

resonance effect. Whereas resonance effect involves


rt-BOND
ou
ad

delocalization of 7C-electrons of two or more conjugated a-BOND


double bonds or 7i-electrons of a double bond and non
bonding, i.e., n-electrons of a hetero-atom, hyper
Y

conjugation involves delocalizationof a-electrons through


overlapping of p-orbitals of a double bond with a-orbital
Re
nd

of the adjacent single bond (i.e. a-Ji-conjugation) as


shown in Fig. 12.14 :
Orbital representation of
Fi

In terms of structures, hyperconjugation may be


represented as follows : hyperconjugation, i.e., <J- 7i-conjugation

H H H

r>
H—C—CH=^CH2 ►H—C=CH—CH2 *♦—►H* C=CH—CHJ 4—► H—C=CH—CHJ
H H H
Propene I II 111

Structures I, II and HI are called hyperconjugative structures. Since there is no bond between carbon
and hydrogen atoms in these structures, hyperconjugation is also called no bond resonance. It may be noted
that although a free proton has been shown in the above structures, it is still bound quite ifrmly to the TZ-cloud
and hence is not free to move.
rw
12/82 New Course Chemistry (XI)ESE

It is evident from the above structures that hyperconjugation occurs through the H-atoms present on the
carbon atom next to the double bond, i.e., a-hydrogen atoms. Naturally more the number of such
G-hydrogen atoms, more are the number of hyperconjugative structures and hence greater is the inductive
effect. Clearly the number of hydrogen atoms is three (maximum) with methyl group, two with the ethyl
group, one with the isopropyl group and none with the tert-butyl group. Thus, the order of hyperconjugation
effect decreases in the order : CH3- > CH3CH2- > (CH3)2CH- > (013)30-
Significance of the hypercoi^ugation effect Although hyperconjugation effect is a much weaker

luo
effect than resonance effect yet it is quite useful in explaining some of the physical and chemical properties of
organic molecules. Some of these are :

F
(i) Directive influence of alkyl groups. The o, p-directive influence of CH3 and other alkyl groups can
be easily explained on the basis of hyperconjugation.

oF
H H H H
I I

rs
H—C H'" H—C H*"
H-C^H
H—C

ok
4- > jP ♦

u-

fo
(+ Six more such structures due to two other a-hydrogen atoms)
As a result of hyperconjugation, the electron density at o- and p-positions w.r.t. the CH3 group increases

o
Y
and hence the electrophilic substitution (discussed in unit 13) reactions in toluene (and other alkylbenzenes)
Y
will occur at o-and p-positions w.r.t. the CH3 (or the alkyl group). Thus, alkyl groups are o, p-directing.
rB
(ii) Shortening of carbon-carbon single bonds adjacent to multiple bonds.
H

H— H2 < H—C=CH—CHJ <■ etc.


ue

3| 2 1 3| 2 1 ^
H H
od

Because of hyperconjugation, C2-C3 single bond in propene acquires some double bond character and
hence is little shorter (149 A) than the normal carbon-carbon single bond length (1-54 A) in propane.
ad
in

Similarly, Cj-C2 bond in acetonitrile is shorter than the normal carbon-carbon single bond length
(1-54 A).
H H
2I
H—(^c^n: <
> H—C=C=N:- ◄—►etc.
Re
F

H
Acetonitrile

(iii) Relative stability of alkenes. Heats of hydrogenation (A^^^) show that greater the number of alkyl
groups attached to the doubly bonded carbon atoms, greater is the stability {i.e., lower is the heat of
hydrogenation) of the alkene. Consider, for example, the following alkenes :
a a a a
CH.CH, CH, CH H
a I ^1 ^ a a a
3\
c=c
CH3 C = C—CH3 CHj—C = CH—CHj H "^CH3
2, 3-Dimethylbut-2-ene (I) 2-Methylbut-2-ene (II) /ra#i5-But-2-ene (HI)

(Twelve a-H ; = - 26-6 kcal mol"^) (Nine a-H ; = - 26*9 kcal mol"*) (Six a-H ; = - 27-6 kcal mol"^)

A
ORGANIC CHEMISTRY-SOME BASIC PRINCIPLES AND TECHNIQUES 12/83

a a

CH
CH3 a
C = C CH2 = CH2
H H CH3—CH =CH2
Propene (V) Ethene (VI)
c/i-But-2-ene (IV)

(Six a-H ; ^h2 = - 28-6 kcal mol *) (Three a-H : =-30-1 kcal mol ') (No a-H ; =-32-8 kcal mol *)
Alkenes 1, II, III (or IV), V and VI have twelve, nine, six, three and none a-hydrogens respecUvely and
hence equal number of hyperconjugatioii structures can be written for each one of them. Since greater the
number of hyperconjugation structures, more stable is the alkene, therefore, the relative stability of these
alkenes follows the sequence : I > II > III (or 1V)>V>VI.

w
trans -But-2-ene (III) is, however, more stable than cw-but-2-ene (IV) in which the two methyl groups
are close together and hence their electronic clouds repel each other. As a result, overall stability of alkenes
follows the order: I > II > III > IV > V > VI.

F lo
(iv) Stability of carbocations and free radicals. The relative stability of carbocations and free radicals
follows the same sequence : tertiary > secondary >primary. This order of stability can be easily explained on
the basis of hyperconjugation. For stability of carbocations refer to page 12/79.

ee
12.14. REACTIVE INTERMEDIATES

Fr
Most of the organic reactions occur through the involvement of certain chemical species. These are
generally short-lived (10“^ seconds to a few seconds) and highly reactive and hence cannot be isolated.

for
These short-lived highly reactive chemical .species through which the majority of the organic reactions occur
are called reactive intermediates. Some important examples of reactive intermediates are : carbocations,
r
carbanions, free-radicals, carbenes and nitrenes.
You
12.14.1. Carbocations
s
ook
Chemical species bearing a positive charge on carbon and carrying six electrons in its valence shell
are called carbocations or carbenium ions. Earlier these were called carbonium ions.

These are fonned by heterolytic cleavage of the covalent bonds in which the leaving group takes away
eB

with it the shared pair of electrons (of the covalent bond). For example.

I.C., (CH3)3C-^C1 >


(CH3)3C + cr
our
ad

?e>7-Butyl chloride fm-Butyl carbocation


Carbocations are also formed during dehydration of alcohols in presence of cone. H2SO4 at 430-440 K.

cone. H2SO4
dY

CH3CH2—OH + H"- > CHjCH2^0H2 > CH3CH2 + H2O


Re

430-440 K
Ethanol Ethyl carbocation
Classification. Carbocations are classified as primary (1 °), secondary(2°) and tertiary (3°) according as
Fin

the positive charge is present on a primary, secondary and a tertiary carbon atom respectively. For example.
CH.
I ●
CH3—CH2 CH3—CH—CH3 CH3—C—CH3
Ethyl carbocation (1®) Isopropyl carbocation (2®) lert. -Butyl carbocation (3®)
Stability. The stability of carbocations follows the order :3°>2°> l°> methyl. This order of stability
can be explained on the basis of the following factors :
(a) Inductive effect. An alkyl group has -i-I-effect (electron-donating inductive effect). When an alkyl
group is attached to a positively charged carbon atom of a carbocation, it tends to release electrons towards
that carbon. In doing so, it reduces the positive charge on the carbon. In other words, the positive charge gets
dispersed and the alkyl group becomes somewhat positively charged. This dispersal of the positive charge
stabilizes the carbocation.
12/84 “PtuuiCee^A New Course Chemistry (X1)BZSI9D

Obviously, more the number of alkyl groups on the carbon atom carrying the +ve charge, greater would
be the dispersal of the charge and hence more stable would be the' carbocation. Thus, the stability of the
carbocations decreases in the order : 3® > 2® > 1“ ; i.e..
R H H H

Rr^C
i > R-^C* > > H C*-
4 \

R R H H

w
3® 2® 1® Methyl carbocation
Further, stability decreases as +I-effect of the alkyl group decreases, i.e.,

(CH3)C > CH3£hCH3 > CH3CH2£h2 > CH3CH+ > CHJ.

Flo
ib) Resonance effect Carbocations in which the +vely charged carbon atom is attached to a double
bond or a benzene ring are stabilized by resonance. For example.

ee
CH2=^ai^CHj ◄ ► CH2—CH=CH2
+

Fr
(Allyl carbocation is stabilized by resonance)

CH2 CH2 CH2 CH2 CH2

for
ur
9 " ^
< ► -4 <■ >

+
ks
(Benzyl carbocation is stabilized by resonance)
Yo
Obviously, more the number of phenyl groups, greater is the stability. For example,
oo

(C6H5)3C^ > (C6H5)2CH+ >


eB

Further, the presence of electron-donating groups such as -CH3, -OCH3, -OH, etc. in the benzene ring
increases the stability while the presence of electron-withdrawi ng groups such as -NO2, -CN,
-COOR, -Cl, etc. decreases the stability of the carbocation.
r
ou

(c) Hyperconjugation effect The relative stability of 1®, 2® and 3® carbocations can also be explained
ad

on the basis of hyperconjugation effect as discussed below :


/ert-Butyl carbocation has nine a-hydrogens and hence nine hyperconjugation structures can be written
Y

for it as shown below;

H H H
Re
nd

I
H—C^C—CBS'*—►H—C=C—CHj-*—► H’' C=C—CH3► H—C=C—CH3
I I I I I I
Fi

H CH3 H CH3 H CH3 H* CH3


tert-Butyl carbocation
(+ six more such structures from the other two methyl groups)
For isopropyl carbocation, six hyperconjugation structures can be written as shown below :
H H H

^ * 1
H— C-^C
C-—CH3 4—►H—C = C—CH3 4—► C=C—CH3<—►H—C=C—CH3
I I I I I I
H H H H H H H"" H
Isopropyl carbocation
(+ three more such structures from the other methyl group)
ORGANIC CHEMISTRY-SOME BASIC PRINCIPLES AND TECHNIQUES 12/85

In a similar way three hyperconjugation structures can be written for ethyl carbocation as shown below :
H H H

H— c—c H H—C = C—H <—► C = C—H ^—► H—C = C—H

H H H H H H H"' H
Ethyl carbocation

However, for CH^ carbocation, no hyperconjugation structure can be written. Thus, the order of stability
of carbocations is : (CHjjjC-*- > (CH3)2CH-^ > CH^CHJ > CH+
In general, the stability of the various carbocations decreases in the order :

w
(0 > (C6H5)2CH+ > CHgCO-^ > (^3)30+ > CgHjCHJ > (^3)201+ > CH2 = CH CHJ >

F lo
CH3CH2CH+ > RC =CH2> CH3CHJ >RCH = CH-"> > CHJ >HC = C+
(1OP-CH3O—C6H4—CHJ >p-CH3—C6H4—CHJ > CgHjCHJ > p-Cl—CgH4—CH| >

P-NO2—C6H4—CH+

ee
Reactivity. Carbocations are highly reactive chemical species

Fr
FIGURE 12.15

since the carbon atom carrying the positive charge has only six EMPTY
electrons in its valence shell and thus has a strong tendency to complete p-ORBITAL

for
its octet. Usually the order of reactivity of any chemical species is
reverse that of its stability. Therefore, the order of reactivity of
r
carbocations follows the sequence \ 1° > 2° > 3°. a R
Orbital structure. The carbocations are planar chemical Sp2-HYBRIDIZED
You
s
species. The carbon atom carrying the positive charge is sp^- CARBON
ook

hybridized. The three 5p^-hybridized orbitals of this carbon form three


CT-bonds with monovalent atoms or groups which lie in a plane and Orbital structure of carbocations
eB

are inclined to one another at an angle of 120°. The unhybridized 2p-


orbital which is perpendicular to the plane of the three a-bonds is,
however, empty (Fig. 12.15).
our

12.13
ad

’Sample Problem Identify the electrophiliccentres in the following :


CH3CH = O, CH3CN, CH3I. (NCERT Solved Example)

. r> ^ .r>
dY

Solution, (a) CH3—CH=i=0'<—► CH3—CH—0“ (b) CH3—C=N^—► CH3—C = N


Re

As a result of resonance, the starred carbon atom in the above molecules carry a +ve charge and hence are
electrophilic centres.
Fin

5+ 8“
(c) CH3—I Since I being much bigger in size than C can dispersethe negativecharge more easily.
therefore, CH3 carries a +ve charge and hence is the electrophiliccentre.
Sample Problem 12.14 Which bond is more polar in the following pairs of molecules :
(a) H3C—H, H3C—Br (b) H3C—NHj, H3C—OH (c) H3C—OH, H3C—SH
(NCERT Solved Example)
Solution, (a) C—Br since Br is more electronegative than H
(b) C—O since 0 is more electronegative than N (c) C—O since O is more electronegative than S.
12/86 ‘P'uuUe^'4^ New Course Chemistry (XI)E!Z5IHD

Sample Problem 12.15 In which C—C bond of CH3CH2CH2Br, the inductive effect is expected
to be the least ? (NCERT Solved Example)
Solution. The magnitude of inductive effect decreases with distance and hence the effect is least in C2—C3
bond.
3 2 1

CH3 CH2 CH2 Br

ow
Sample Problem |12.16 Write resonance structures of CH^COO” and show the movement of
electrons by curved arrows. (NCERT Solved Example)

Solution. CH3—
ft: ► CH3-C^„O:
O:-

r 9*~

e
These are resonance structures since these differ in the position of electrons and not atoms.

Fl
re
Sample Problem 12.17 Write the resonance structures of CH2 = CH—CHO. Indicate relative

F
Stability of the contributing structures order of decreasing stability. (NCERT Solved Example)
+

Solution. CH2—CH—CH=tp: ♦ ur> CH2—CH=CH—o:- ^ > -:CH2—CH=CH—q:

r
I II III

fo
Structure (1) is most stable since both C and O atom has an octet of electrons and none of these atoms carries
any charge. Structures (II and III) both involve separation of charge and hence both are less stable than structure
ks
(I). However, structure (II) is more stable than structure (III) since it carries a -ve charge on the more electronegative
0 atom and +ve charge on the less electronegative C atom while in structure (III), the more electronegative O
Yo
atom carries the +ve charge while the less electronegative C atom carries the -ve charge. Thus, the decreasing
oo
order of stability is : I > II > III
Sample Problem im Explain why the following two structures, I and II cannot be the major
eB

contributors to the real structure of CH3COOCH3. (NCERT Solved Example)


:o:“ :or
ur

CH3—c p—CH3 > CH3—c = p —CH3


ad

I II
Yo

Solution. Both these structures involve separation of charge and hence are of high energy. Therefore, they
do not contribute substantially towards the resonance hybrid. Further, the contribution of structure (I) is lower
than that of structure II since C atom carrying +ve charge has only a sextet of electrons.
d

+ + +
Re

Sample Problem 12.19 Explain why (CH3)3C is more stable than CH3CH2 and CH3 is the
in

least stable cation.


F

Solution. (CH3)3C has nine (page 12/84) a-hydrogens and hence has nine hyperconjugation structures
while CH3CH, has only three a-hydrogens (page 12/85) and hence has only three hyperconjugalion structures.
As a result, (CH3)3C' is more .stable than CH3CH^. In contrast in CH|, the vacant p-orbilal (Fig. 12.15, page
(12/85, R = H) is perpendicular to the plane in which the three C—H bonds lie and hence cannot overlap with it.
Thus, CH^ is not stabilized by hyperconjugation and hence is the least stable of the three cations.
1Z14.2. Carbanlons

Chemical species bearing a negative charge on carbon and possessing eight electrons in its valence
shell are called carbanions.
ORGANIC CHEMISTRY-SOME BASIC PRINCIPLES AND TECHNIQUES 12/87

These are produced by heterolytic cleavage of covalent bonds in which the shared pair of electrons
remains with the carbon atom. For example.
n,
HO" + H-^CH2—CHO >H20 + :ch2—CHO
Hydroxide ion Acetaldehyde Acetaldehyde carbanion

n,
H2N" + H-^C=C—H ■►NH3 + -:c=c—H
Amide ion Acetylene Acetylide ion
Classification. Like carbocations, carbanions are also classified as primary (1®), secondary (2®) and

w
tertiary (3®) according as the negative charge is present on a primary, secondary and a tertiary carbon atom
respectively. For example.

R-CH2 R2CH-

F lo
R3C-
Primary carbanion (1°) Secondary carbanion (2“) Tertiary carbanion (3“)
Stability. Stability of various carbocations can be explained on the basis of the following factors :

ee
(a) Inductive effect The stability of simple alkyl carbanions follows the order : CHj > 1° > 2® > 3®.

Fr
This can be explained on the basis of inductive effect of the alkyl groups. An alkyl group has +I-effect
(electron-donating inductive effect). When an alkyl group is attached to a negatively charged carbon atom of
the carbanion, it tends to release electrons towards that carbon. In doing so, it increases the intensity of the

for
negative charge on the carbon and thus destabilizes the carbanion. Evidently, more the number of alkyl
groups on the carbon atom carrying the negative charge, more would be the intensity of the negative charge
ur
on the carbon atom and hence less stable is the carbanion. In other words, stability decreases in the order:
CHj >1®>2®>3®,/.£.,
s
ook
Yo
H H H R
eB

H—cr > R-^c:" > R-^c:" > R-5-c:"


4 \

H H R R
r

Methyl carbanion 1® 2® 3®
ad
ou

ib) Resonance effect Allyl and benzyl carbanions are stabilized by resonance.
Y

CH2=i=CH—CH2 <■ > "CH2—CH=CH2


{Allyl carbanion is stabilized by resonance)
Re
nd

C^ch3 CH2 CH2 :ch2


Fi

> jp-* > ●

{Benzyl carbanion is stabilized by resonance)


Obviously more the number of phenyl groups, greater is the stability. For example,
(C6H5)3C- > (C6H5)2CH- > CgHsCHj
Further, the presence of electron-withdrawing groups such as -NO2, -CN, -COOR, -Cl, etc. in the
benzene ring tend to disperse the -ve charge and hence increases the stability of the carbanion while the
presence of electron donating groups such as, -CH3 , -OCH3, -OH, etc. intend to intensify the negative
charge and hence decreases the stability of the carbanion.
12/88 ‘P’uxcCcc/b-'^r New Course Chemistry (XI)E2s]aD

(c) A--Character. Stability of the carbanion increases with the increase in 5-character of the carbon
carrying the -ve charge. For example.
R-CsC > R2C = CH > R—CH"
50% s-character 33% ^-character 25% 5-character

In general, the stability of the various carbanions decreases in the order :


' ) HC = C"> (C6H5)3C“ > (C6H5)2CH“ > > C^H^CH" > CH2 = CH—CH* > C^^H" >
CH2 = CH“ > CH“ > CH3CH2 > (CH3>2CH" > (CH3)3C"
(/OP-NO2—C6H4—CH' >p-Cl—CglU—CH” > C^HgCH” >/>-CH3—CgH4—CH”

w
> P-CH3O—€6H4—CH”
Reactivity. The order of reactivity of carbanions is reverse

F lo
FIGURE 12.16
of the order of stability, i.e., 3“ > 2" > T > CH3 .

Orbital structure. The structure of simple alkyl LONE PAIR sp3-OR8ITAL


carbanions is usually pyramidal just like those of ammonia and
amines. The carbon atom carrying the negative charge is sp^-

ee
hybridized. Three of the four 5p^-hybridized orbitals form three R

Fr
o-bonds with monovalent atoms or groups while the fourth sp^-
R
orbital contains the lone pair of electrons. (Fig. 12.16). sp3-HYBR!DIZED
CARBON

for
In contrast, the carbanions which are stabilized by
resonance are plana.r In these carbanions, the carbon atom Orbital structure of carbanions
carrying the -ve charge is sp^-hybridized.
ur
Thus, whereas (CH^):fC ~ is pyramidal, allyl carbanion is planar.
s
12.14.3. Free Radicals
ook
Yo
A free radical may be defined as an atom or a group of atoms having an odd or upaired electron. These
are generally produced by homolytic cleavage of a covalent bond. For example,
eB

hv or A
Cl-^^Cl Cl* + -Cl
Chlorine
Homolytic cleavage Chlorine free ardicals
our
ad

hv or A
CH3^CH3 ♦ -CH3 + -CHj
Chlorine Methyl free radicals
Y

hvorA
R— 0-^0—R *■ RO* + RO-
Re

Peroxide Alkoxy free radicals


nd

Classification. Like carbocations and carbanions, free radicals are also classified as primary (1°),
secondary (2°) and tertiary (3°) according as the carbon atom carrying the unpaired electron is primary,
Fi

secondary and tertiary respectively. For example.


R R
I
R—CH R—C—R
R—CH2
Primary (1®) Secondary (2®) Tertiary (3®)
Stability. The order'of stability of free radicals is the same as that of carbocations, i.e., 3° > 2° > 1°.
This order of stability can be easily explained on the basis of hyperconjugation. Greater the number of alkyl
groups attached to the carbon atom carrying the odd electron, greater is the delocalization of the odd electron
and hence more stable is the alkyl free radical. Thus,
ORGANIC CHEMISTRY-SOME BASIC PRINCIPLES AND TECHNIQUES 12/89

CH3 CH.
i
CH3—C—CH3 >
CH3—CH CH3—CH2 >
CH3
rerf-Butyl free radical (3°) Isopropyl free radical (2°) Eihyl free radical (1“) Methyl free radical
In contrast, ally! and benzyl free radicals are stabilized by resonance effect. For example,

Ol
CH2—CH—CH2 ♦
O > CH2—CH=CH2

qCH2 CH2 CH2


(^CH2 CH2

w
●4 ●4 ●4

F lo
Further, greater the number of phenyl groups more stable is the free radical.
In general, the stability of the various free radicals decreases in the order

ee
(C5H5)3C > (C(5H5)2CH > C^HgCH^ > CH2 =CH—CH2 > (CH3)3C > (CH3)2CH >

Fr
CH3CH2 > CH3 > CH2 = CH > HC = C
Like carbocations and carbanions,/ree radicals are also very short-lived highly reactive chemical species

for
because of the strong tendency of the carbon atom carrying the odd electron to acquire one more electron to
complete its octet.
ur
Orbital structure. Alkyl free radicals like carbocations are FIGURE 12.17

planar chemical species. The only difference being that in


p-ORBITAL
s
carbocations, the unhybridized p-orbital is empty (Fig. 12.15)
ook
while in free radicals, it contains the odd electron (Fig. 12.17).
Yo
UNPAIRED ELECTRON
Please note that like carbanions, free radicals can also assume
eB

pyramidal shape since the energy difference between planar and R

pyramidal (carbon carrying odd electron in 5/?^-hybridized) shapes p2.HYBRIDiZED


is not much. For example, bridgehead free radicals (page 12/126) CARBON

are pyramidal because they cannot as.sume planar geometry due


our

to angle strain. Further, the free radicals in which carbon is


ad

Orbital structure of free radicals


connected to highly electronegative atoms are also pyramidal.For
example, -CF3 has pyramidal shape.
Y

SUPPLEMENT YOUR
KNOWLEDGE FOR COMPETITIONS
Re

1. Carbenes. Neutral divalent carbon species in which the carbon atom is bonded to two monovalent
nd

atoms or groups and also contains two non-bonding electrons are called carbenes. These are generally
produced either by photolysis (irradiation with UV light) or thermolysis or pyrolysis (action of heat) of
Fi

diazoalkanes or ketenes. Thus,

Avor A
CH2—N=Nr <■ > -:ch2-^n=n: :ch2 + N2
Diazomcthane Methylene
(A carbene)

hv or A
CH2 c=o :ch2 + :co
Methylene
Ketene

Like carbocations, carbenes are short-lived highly reactive chemical species since the central carbon
atom has only six electrons in its valence shell and thus has a .strong tendency to complete its octet by
gaining two more electrons. Carbenes, thus, behave as Lewis acids or electrophiles.
12/90 New Course Chemistry (XI)EEMD

Orbital Structure. There are two


FIGURE 12.18
types of carbenes, i.e., singlet and
ONE ELECTRON IN
triplet. In singlet carbenes, the
EACH p-ORBITAL
central carbon atom is sp^- EMPTY p-ORBlTAL
hybridized. Two of the sp^-
hybridized orbitals form two a-
bonds with two monovalent atoms
or groups while the third sp^- R
a

hybridized orbital contains two


non-bonding electrons. The

w
unhybridized /7-orbital is, however, sp2-HYBRIDIZED sp-HYBRlDIZED
CARBON CARBON
empty (Fig. 12.18). Thus, a singlet
carbene has a bent structure. SINGLET CARBENE TRIPLET CARBENE

F lo
In triplet carbenes, the central Orbital structure of singlet and triplet carbenes
carbon is sp- hybridized ; the two
5p-hybridized orbitals fonn two a-bonds with two monovalent atoms or groups while the two unhybridized
/7-orbitals are perpendicular to each other and contain one electron each. Thus, a triplet carbene has a

ee
linear structure and behaves as a diradical (Fig. 12.18). Further, it may be noted that a triplet-cschtut is

Fr
always more stable than a singlet carbene.
2. Nitrenes. Neutral monovalent nitrogen species in which nitrogen has two unshared pairs of electrons
and is bonded to only to one monovalent atom or group are called nitrenes. These are usually produced

for
by thermolysis of azides. For example,
ur
0_* A
R— N=i=N=Nr < > R—n-^n=n: s R—n: + :n=n:

Alkyl azide Alkyl nitrene


ook
Yo
12.15. COMMON TYPES OF ORGANIC REACTIONS

All the organic reactions can be broadly classified into the following eight types :
eB

1. Substitution reactions, 2. Addition reactions,


3. Elimination reactions, 4. Rearrangement reactions,
5. Condensation reactions, 6. Isomerization reactions,
r
ou
ad

7. Pericyclic reactions and 8. Polymerization reactions.


1. Substitution Reactions. A substitution reaction is that which involves the direct replacement
(displacement or substitution) of an atom or a group of atoms in an organic molecule by another atom or
Y

group ofatoms without any change in the remainingpart of the molecule. The product obtained as a result of
substitution is called the substitution product and the new atom or group of atoms which enters the molecule
Re

is called a substituent. Depending upon the nature of the attacking species {nucleophile, electrophile or a
nd

free radical) the substitution reactions are of the following three types:
(/) Nucleophilic .substitution reactions. Substitution reactions which are brought about by nucleophiles
Fi

are called nucleophilic substitution reactions. In all these reactions, a stronger nucleophile usually displaces
a weaker nucleophile. These reactions are typical of alkyl halides. For example, hydrolysis of an alkyl halide
with an aqueous base :

5+P«6-
HO" + R-^X > R—OH + X" (X = Cl, Br or I)
Hydroxide ion Alkyl halide Alcohol Halide ion
(Stronger nucleophile) (Subsiilulion product) (Weaker nucleophile)

Mechanism of nucleophilic substitution reactions


There are two types of nucleophilic substitution reactions. These are :
(a) Sj»j2 {Substitution, nucleophilic, bimolecular). {b) Sj^l {Substitution, nucleophilic, unimolecular).
ORGANIC CHEMISTRY-SOME B/ , k PRINCIPLES AND TECHNIQUES 12/91

(a) Substitution nucleophilic bimolecular S[»j2. The reaction between methyl chloride (CH3CI) and
hydroxide ion to yield methanol (CH3OH) and chloride ion follows second order kinetics, i.e., the rate of the
reaction depends upon the concentration of both the reactants. In other words. Rate =k [CH3CI] [OH“]. This
rate law implies that both the alkyl halide (CH3CI} and the nucleophile (OH“) are taking part simultaneously
in the rate determining step of the reaction.
In other words, when the incoming nucleophile (OH“ ion) approaches the alky! halide (CH3CI)
molecule and starts interacting with it, the carbon-halogen bond starts breaking and a new carbon-OH
bond starts forming. These two processes take place simultaneously in a single step and no intermediate
is formed. Thus, Sj^2 reactions are concerted reactions, i.e., take place in one step. Such reactions

w
occur through a transition state in which both the reactants are partially bonded to each other as shown
below :

Flo
H,
H
HO- +
. C-‘-CI
Of «-
HO c-
6-
Cl ► HO c.., + cr
Incoming
f'i /V i H

ee
nucleophile H Outgoing
H
(Reactant) H H H nucleophile
(Leaving group)

Fr
Methyl chloride Transition state Methanol
(Reactant, alkyl halide) (Product)

In terms of models, the formation of a transition state in a Sjvj2 reaction is depicted in Fig. 12.19.

for
FIGURE 12.19
ur I,..

s O
Leaving
ok
)
Yo
Incoming
Methyl chloride (J Methanol group
nucleophile
(Reactant, alkyl halide) Transition state (Product) (C|- ion)
(OH- ion)
Bo

Formation of transition state in a Sj^2 reaction

In the transition state, the carbon atom is simultaneouslybonded to the incoming nucleophile (OH“ ion)
re

and the outgoing leaving group (Cr ion). In other words, in the transition state, carbon atom is bonded to five
atoms and thus the transition state is unstable and hence cannot the isolated. Being unstable, it ultimately
ou

decomposes to form the product (CH3OH) and the leaving group (Cl" ion).
ad

It is interesting to note that in Sj,j2 reactions, the attack of the nucleophile (i.e., OH" ion) occurs from the
Y

back side) and the leaving group (i.e.. Cl" ion) leaves from the from side.
As a result, V reactions are always accompanied by inversion of configuration just in the same way
as an umbrella turns inside out in a strong wind. This inversion of configuration is also called Walden
nd
Re

inversion.

(b) Substitution nucleophilic unimolecular (Sj^l). Sj,jl reactions are generally carried out in polar
Fi

protic (hydroxylic) solvents such as water, alcohol, acetic acid, etc. The reaction between tert-butyl bromide
and the OH" ion to yield terl-buly\ alcohol andBr" ion foWowa first order kinetics, i.e., the rate of the reaction
depends upon the concentration of tert-b\xiy\ bromide only and is independent of the concentration of OH"
ion. In other words. Rate = k [(CH3)3CBr].
CH3 CH3
CH3—C—Br -H -OH ^ CH3—C—OH -1- Br"
CH3 CH3
fer/-Butyl bromide fert-Butyl alcohol
or 2-Bromo-2-methylpropane or 2-Methylpropan-2-ol
12/92 ‘PnauCee/it,’^ Nfc.w Course Chemistry (XI)ISE

This rate law suggests that the reaction occurs in two steps. In the first step, tert-butyl broinide undergoes
ionization to produce rerf-butyl carbocation and a bromide ion.
The energy needed for the cleavage of the C—^Br bond is obtamed through the solvation of the bromide
ion with the proton of protic solvents.
This step is slow and reversible and hence is the rate-determini ng step of the reaction.
CH3 Ionization, CH3

Stepl. CH3—C-^r
slow
+ Br-
/ \
CH3 H3C CH3
/ert-Butyl bromide fert-Butyl carbocation
In the second step, the carbocation being a reactive chemical species, is immediately attacked by the
nucleophile, i.e., OH“ ion to give the substitution product, i.e., tert-hvXy\ alcohol. This step is fast and hence
does not affect the rate of the reaction.

F low
CH3 CH3
L Fast I
Step n. C "OH - ► CH3—C—OH
/ \ Nucleophile
H3C CH3 CH3
/ert-Butyl carbocation ter/-Butyl alcohol
{Substitution product)

e
(b) Electrophilic substitution reactions. Substitution reactions which are brought about by electrophiles

for Fr
are called electrophilic substitution reactions. These reactions are typical of arenes and other aromatic
compounds. For example, halogenation, nitration, sulphonation and Friedel-Crafts reactions.
Chlorination
—H + Cl'^ (obtained from CI2 + FeCl3) ¥ Cl +
' ' Chloronium ion
Chlorobenzene
Benzene (Electrophile)
Your
(Substitution product)
s
eBo k

+ NO2 (obtained from HNO3 + H2SO4) Nitration^ NO2 +

' ' Nitronium ion


Benzene Nitrobenzene
(Electrophile)
(Substitution product)
ad
our

(Hi) Free radical substitution reactions. Substitution reactions brought about by free radicals are
called free radical substitution reactions. For example, chlorination of methane in presence of heat or diffused
sunlight to give methyl chloride and hydrogen chloride
520-670K
CH3—H + CI2 CH3—Cl + HCl
Re

orhv
Methane Methyl chloride
Y

The reaction occurs by a free radical mechanism which involves the following three steps :
Find

Av or 520-670 K
Initiation : 2C1
Chlorine Homolytic fission Chlorine free radicals

Propagation : CHj-^H + -Cl ¥


●CH3 + H—Cl (i)
Methane Methyl free radical

.(«)
●CH3 + Cl —> CH3—Cl + -Cl
Termination ; ● Cl + ■ Cl »C1 2 ’ ● CH3 -h. CH3 CH3—CH3 and ● CH3 + ■ Cl CH3CI
ORGANIC CHEMISTRY-SOME BASIC PRINCIPLES AND TECHNIQUES 12/93

2. Addition reactions. Reactions which involve combination between two reacting molecules to give a
single molecule of the product are called addition reactions. Such reactions are typical of compounds
containing multiple (double and triple) bonds. Depending upon the nature of the attacking species
(electrophiles, nucleophiles or free radicals), addition reactions are of the following three types :
(i) Nucleophilic addition reactions. Addition reactions brought about by nucleophiles are called

w
nucleophilicaddition reactions. These reactions are typical of aldehydes and ketones. For example, base-
catalysed addition of HCN to aldehydes or ketones.
rv
HO" + H-^CN > H2O + CN"
Nucleophile

o
e
R R
R

re
H 1- CN
X=fO + CN" R—C—O" ♦ R—C—OH
-CN"

Frl
Nucleophile

F
CN CN
Ketone cyanohydrin
(Addilion product)

(ii) Electrophilic addition reactions. Addition reactions brought about by electrophiles are called
ou
or
electrophilic addition reactions. These reactions are typical of alkenes and alkynes. For example, addition of
HBr to propylene in absence of peroxides gives 2-bromopropane.

kfs
Slow +

CH3CH=i=CH2 + H^ > CH3—CH—CH,


oo
Propylene Isopropyl carbocation (2°)
Y
+i Fast
B

CH3—CH—CH3 + Br" > CH3—CH —CH3


Br
re

2-Bromopropane
{Addition product)
(iii) Free radical addition reactions. Addition reactions brought about by free radicals are called free
oYu

radical addition reactions. For example, addition of HBr to alkenes in presence of peroxides :
ad

Peroxides
CH3CH = CH2 + HBr ^ CH3—CH2—CH2Br
d

Propene /t-Propyl bromide


The reaction occurs by the following mechanism :
in
Re

A or hv
Initiation : RO-^^
OR 2RO*
Homolytic fission Alkoxy free radicals
F

Peroxide

RO- + H-^Br
r> > ROH + Br-
Bromine free radical

Slow
Propagation : CH3—CH=^CH2 + Bri > CH3—CH —CH2Br ...(0
Isopropyl free radical (2°)

Fast
CH3—CH—CH2Br + H Br > CH3—CH2—CH2Br + -Br ...(«)
«-Propyl bromide
Tertnination : ● Br + ● Br ^ Br-Br
12/94 “Pnoidee^'A New Course Chemistry (XI)E!&ISD

3. Elimination inactions. An elimination reaction is one that involves the loss of two atoms or groups
of atoms from the same or adjacent atoms of a substance leading to the formation of a multiple (double or
triple) bond. Depending upon the relative positions of the atoms or groups eliminated, these reactions are
classified as a (alpha), P (beta) and y (gamma) elimination reactions,
(i) a-Elimination reactions. In these reactions, the loss of two atoms or groups occurs from the same
atom of the substrate molecule. For example, base-catalysed dehydrohalogenation of chloroform to form
dichlorocarbene :

w
ecu > :CCl2 + H2O + CI"
Dichlorocarbene

Chloroform

F lo
Dichlorocarbene is the reactive intermediate involved in carbylamine reaction and Reimer-Tiemann
reaction.

(ii) p-Elimination reactions are those reactions in which the two groups (usually H and X) are eliminated

ee
from the adjacent carbon atoms : H from the p-and the X from the a-carbon. For example.

Fr
P a

CH3 — CH, — Br + NaOC2Hg ^ CH2 = CH2 + C2H5OH + NaBr


lypes of P-Elimination reactions. Depending upon the structure of the alkyl halide, strength of the

for
base and the polarity of the solvent, p-elimination reactions occur by the following two mechanisms:
ur
(0 E2 and (ii) Ej
(0 E2 (Elimination, bimolecular) reactions. Primary alkyl halides and some secondary alkyl halides
s
undergo elimination or dehydrohalogenation by E2 mechanism. E2 reactions like Sj,j2 reactions are concerted
ook
Yo
reactions, i.e., occur is one step through a transition state.

5-
eB

HO^
HO H H. H H H

H' cr s S-
\c=c/ + HsO + Br"
r

"Br H H
ad

H
ou

Ethyl bromide Transition state Ethylene

Thus, E2 reactions like Sjsj2 reactions show second-order kinetics, i.e., rate oc k [RX] [Nu"].
Y

An important aspect of elimination reactions is that they are always accompanied by substitution reactions
as well. Thus, in the above reaction along with ethylene some ethyl alcohol (substitution product) is also
Re
nd

formed.

(ii) E| (Elimination unimolecular) reaction. Many secondary and tertiary alkyl halides undergo
Fi

elimination reactions when heated in an ionizing solvent. For example, /er/-butyl bromide when heated in
alcohol undergoes dehydrobrominalion to form 2-methylpropene or isobutylene.
CH3 CH3
C2H50H/A
CH3—C —Br 328 K
■>
CH3 —C = CH^ -b HBr
2-Methylpropene
CH3 (Isobutylene)
rc/'f-Butyl bromide
Studies on this reaction have indicated that it follows ifrst-order kinetics, i.e., the rate of the reaction is
proportional to the concentration of the alkyl halide only. Thus,
Rate « [Alkyl halide]
ORGANIC CHEMISTRY-SOME BASIC PRINCIPLES AND TECHNIQUES 12/95

E| reactions like reactions occur in two steps involving the intermediate formation of carbocations
H
y
CH3 CH3 CH2
Ionization \ y Fast CH
CH3—C—Br +c C=CH2
Slow (-BD CH3
2-Mctliylpropene
CH3 CH3

w
(ert-Butyl ?er/-Bulyl carbooalion
bromide (planar)

as the rate-determining step, i.e., reactivity in E| reactions also depends upon the stability of carbocations ;
more stable the carbocation, faster is the reaction. Thus, Ej reactivity follows the order: 3" alkyl halides > 2°
alkyl halides > \° alkyl halide.

o
Like 3° alkyl halides, alcohols (T, 2°, 3°) on treaiment with cone. H2SO4 undergo dehydration to form
alkenes by Ej mechanism. Refer to unit 13, pages 13/32-13/33.

e
Y-Elimination. In these reactions, loss of two atoms or groups occurs from a- and y-positions (i.e. three

Fl
re
bonds away) of the molecule leading to the formation of three-membered rings. For example,
H H

F
\/
c
ur H H

r
Zn dust
c + ZnBi2

fo
y-Elimination H H
Cyclopropane

1, 3-Dibromopropane ks
Yo
This reaction is called Freund reaction and is extensively used for the synthesis of three membered irngs.
oo
4, Condensation reactions. In these reactions, two or more molecules of the same or different reactants
combine to form a product with or without the elimination of simple molecules such as H2O, HCl, NH
ROH, etc. For example, two molecules of acetaldehyde condense in presence of dilute alkali to form
eB

3-hydroxybutanal. Since p-hydroxyaldehydes or ketones are commonly known as aldols, therefore, this reaction
is called aldol condensation.
O OH O
ur

Dil. NaOH 4 3 2 1

CH3—C —H + H CH2—C—H ► CH3—CH—CH2—C—H


ad

a
p
Yo

Ethanal 3-Hydroxybutanal
(Acetaldehyde) (.■1 (i-hydroxyaldehyde or aldol)
Aldols on heating with dilute mineral acids readily undergo dehydration to form a, ^-unsaturated
d

aldehydes or ketones. OH
Re
in

H30^ P a

Thus, CH3—CH—CH2—CHO A CH3—CH = CH—CHO + H2O


F

3-Hydroxybutanal But-2-enaI (Crotomildehyde)


An example of a condensation reaction which occurs with the elimination of a molecule of H2O is the
reaction between benzaldehyde and aniline to fonn benzylideneani line.

C6H5CH=[o_+3]nC(,H5 ^ C^H5CH = NC6Hg + H2O


Benzaldehyde Aniline Benzylideneaniline

5. Rearrangement reactions. Reactions involving the migration of an atom or a group from one atom
to another within the same molecule are called rearrangement reactions. For example,
(/) Dehydration of 2, 2-dimethylpropan-l-ol with cone. H2SO4, occurs through 1, 2-mgiration of the
methyl group to give the rearranged product, i.e., 2-methylbut-2-ene
12/96 ‘PfuuCcc^’^ New Course Chemistry (XI)CQISD

CH3 CH3 CH3


Cone. H2SO4 -H->0 +

CH3—C—CH2—OH > CH3—c—CH2-^0H2


CH3 CH3
2, 2-Dimethylpropan-l-ol
CH3 CH3 l° Carbocation

w
I 2
{less stable)
l.2-Mcthyl
shif) > CH3—C—CH2CH3 ► CH3—C=CHCH3
3° Carbocation 2-Methylbut*2-ene
{more stable)

(») Wohler synthesis of urea from ammonium cyanate (formed by double decomposition of a mixture of

o
NH4CI and sodium cyanate) is also an example of a rearrangement reaction.

e
A
NH4C1 + NaCNO > NH4CNO + NaCl

re
rFl
D;

F
A
NH4CNO ► H3N + HO—C=N
Ammonium cyanate
Proton transfer Tautomerises
HO —C=N"

r
HO —C=NH > 0=C—NH2
ou
fo
■"NH3 NH2 NH2

ks
Urea

(///) Hofmann bromamide reaction involving the conversion of 1“ amides to T amines on treatmentwith
Br2 in presence of KOH.
oo
0 O O O
Y
Br2/KOH OH~
N-(^EBr
B

R—C—NH2 R—C—NHBr ► R—C—


-Br"
Amide
-H2O
re

Rearrangement KOH
> 0=C=N—R ► R—NH2 + K2CO3
{Hydrolysis)
ou

Alkyl isocyanate 1° Amine


Y
ad

In this rearrangement reaction, the group R migrates from carbon to nitrogen to first give an alkyl
isocyanate which upon hydrolysis gives a 1® amine with one carbon atom less than the original amide.
6. Isomerisation reactions. Reactions which involve interconversion of one isomer into another keeping
d

the molecular formulae as well as the carbon skeletons of the reactant and the product intact are called
isomerization reactions. For example, interconversion of frnn5-but-2-ene to cis- but-2-ene and vice versa
in
Re

may be regarded as geometrical isomerization reaction.


CH3 H hv
CH3 /CH3
F

c = c <■ c = c
H
CH3 hv H H
fran.r-But-2-ene cis-But-2-ene

Similarly, 1-bromobutane isomerises to 2-bromobutane in presence of anhydrous AICI3 at 575 K.


Br
Anhyd.AJCl3 I
CH3—CH2—CH2—CH2—Br 575 K > CH3—CH2—CH—CH3
1-Bromobutane 2-Bromobutane

7. Pericyclic reactions. There are a large number of organic reactions which do not involve ionic or
free radical intermediates. Instead these reactions occur in a single step via a cyclic transition state. In these
reactions, bond making and bond breaking occurs simultaneously. These reactions do not require any catalyst
and are initiated either by heat or light. All such reactions are called pericyclic reactions. For example.
ORGANIC CHEMISTRY-SOME BASIC PRINCIPLES AND TECHNIQUES 12/97

CH2 f CH2 CH2—CH2 CH2—CH2


(0
CH2 CHo _CH2- —CH2_ CH2—CH2
Ethene
Four-membcrcd cyclic Cyclobutane
transition state

This addition of one ethene molecule (2 %-eIectron system) to another ethene molecule (2 n-electron
system) is commonly called 2 7i + 2 7t or simply (2 + 2) cycloaddltlon reaction.

w
A
{») +
Diels-Aldcr^
reaction
CN ●CN CN
Buta-l, 3-dienc 2
Acrylonitrile
Six-mcmbered cyclic Cyclohex-3-en-1 -carbonitrilc

F lo
(Diene)
(Dienophile) transition state
The addition of a diene (4 n-electron system) to a dienophile (2 n-electron system) to form a six-
rnembered ring is called (4 + 2)cycIoaddition reaction or Diels-Alder reaction.

ee
8. Polymerization reactions. These reactions involve the union of a large number of small molecules
called monomers to form a molecule with high molecular mass called polymer. For example,

Fr
n CH2 = CH2 -(-CH2 CH2
Ethylene (Monomer) Polythene (Polymer)

for
PART IV. PURIFICiATION OF ORGANIC COMPOUNDS
ur
12.16. GENERAL INTRODUCTION
ks
Compounds isolated from natural sources are seldom pure. They are generally mixed with other substances
which also occur along with them. Similarly, the compounds prepared in the laboratory are also not pure
Yo
oo
since they are generally contaminated with other products that result from the side reactions. It, therefore,
becomes essential to obtain the substance in the purest form in order to characterise it thoroughly. A large
eB

number of methods are available for the purification of substances. The choice of the method, however,
depends upon the nature of the substance (whether solid or liquid) and the type of impurities present in it.
Following are some of the important methods which are commonly employed for the purification of organic
compounds : (/) Filtration, (2) Crystallisation or Recrystallization, (3) Fractional crystallisation, (4)
r

Sublimation, (5) Simple distillation, (6) Fractional distillation, (7) Distillation under reduced pressure, (8)
ou
ad

Steam distillation, (9) Differential extraction, and (JO) Chromatography.


The purity of the organic compound (purified by any one of the above methods) is finally checked by
determining one or more of its physical constants. Melting point, boiling point, refractive index, etc. are used
Y

as the criteria ofpurity of organic compounds. This is due to the reason that each pure compound has definite
values of these physical constants. Besides these, in recent years, the purity of the organic compound is also
Re
nd

checked by more sophisticated methods such as spectroscopic methods and different types of chromatography
(TLC, GLC, HPLC, etc. discussed in sec 12.16.10).
Fi

Let us now discuss the principle and brief procedure of each one of the above methods.
12.16.1. Filtration
FIGURE 12.20
The process offiltration is used to separate insoluble solid component
of a mixture from the soluble component in a given solvent. For example,
this technique can be used to separate a mixture of naphthalene and urea
using water as solvent. Urea dissolves in water while naphthalene remains
insoluble. Upon filtration, naphthalene remains on the filter paper while urea
is recovered from the filtrate by evaporating water.
If the water soluble component of a mixture is appreciably soluble in
hot water but only sparingly soluble in cold water, then to separatethe mixture,
filtration of a hot solution is required. This is carried out through a fluted Fluted filter paper
filter paper (Fig. 12.20) to avoid crystallisation during filtration
12/98 New Course Chemistry (XI)I!EIHB

FIGURE 12.21 FIGURE 12.22

CRYSTALS
HOT BUCHNER
SATURATED FUNNEL
HOT SOLUTION FILTER—<
WATER s/ PAPER
FUNNEL < PERFORATION
I
I
I
I
«
«
«
s
FILTRATION
FLASK I
SUCTION-
PUMP

w
FILTERING THE SOLUTION OF THE FILTRATE
SUBSTANCE USING HOT WATER
FUNNEL TO PREVENT CRYSTALLISATION

F lo
DURING FILTRATION
TO SINK

Quick filtration process using a


Filtration using hot water funnel Buchner funnel and a water suction pump

e
and also to remove suspended impurities. If the solution to be filtered is sufficiently large, then filtration is

Fre
done through a ‘hot water funnel’ (Fig. 12.21) to avoid formation of crystals in the funnel and its stem. The
jacket of the hot water funnel keeps the solution hot in the glass funnel placed in it.
For example, a mixture of anthracene and benzoic acid is separated by dissolving the mixture in hot

for
water and filtering the hot solution. Benzoic acid dissolves in hot water but anthracene does not. Upon
filtration, anthracene remains as a residue on the filter paper while benzoic acid crystallises from the filtrate
r
on cooling.
Sometimes, the filtration is very slow and takes a long time. In such cases, filtration is carried out under
You
oks
reduced pressure using a Buchner funnel and water suction pump are shown in Fig. 12.22.
12.16.2. Crystallisation oi .^ecrystalllsatlon
eBo

Crystals are the purest form of a substance having deifnite geometrical shapes. The process by
which an impure compound is converted into its crystals is known as crystallisation.
This is one of the most commonly used techniques for purification of solid organic compounds. It is
our
ad

based on the difference in the solubilities of thecompound and the impurities in a suitable solvent. The
impure compound is dis.solved in a suitable solvent in which it is sparingly soluble at room temperature but
appreciably soluble at higher temperature. The solution is concentrated to get nearly a saturated solution.
When this saturated solution is cooled, crystals of pure substance will separate out which are removed by
filtration. The filtrate, i.e., mother liquor contains the impurities alongwith small quantity of the compound.
dY
Re

If the compound is highly soluble in one solvent and too little soluble in another solvent, then crystallization
can be carried out in a mixture of these solvents taken in a suitable ratio.
Fin

If the compound and the impiuities have comparable solubilities, then repeated crystallizations may be
necessary to purify the substance.
The main steps of this process are explained below :
(i) Choice of the solvent. A suitable solvent which fulfils the following conditions is selected for
crystallization :
(fl) It should not react chemically with the impure substance,
(b) It should dissolve more of the substance upon heating than at room temperature so that the excess
of the substance is thrown out upon cooling.
(c) Either the impurities should not dissolve at all in the solvent, or if they dissolve, they should be
soluble to such an extent that they remain in the solution, i.e., in the mother liquor upon crystallisation.
The various solvents which are commonly employed for crystallisation are water, alcohol, ether,
chloroform, carbon tetrachloride, benzene, acetone, ethyl acetate, petroleum ether etc.
ORGANIC CHEMISTRY-SOME BASIC PRINCIPLES AND TECHNIQUES 12/99

(ii) Preparation of the solution. The impure substance is finely powdered and then heated in a
conical flask with a small quantity of the solvent which is just sufficient to dissolve whole of the substance on

boiling.
(iii) Filtration of the solution. The hot solution obtained above is then filtered immediately either
through fluted filter paper (Fig. 12.20) or through hot water funnel (Fig. 12.21) to avoid crystallization during
filtration.

(iv) Crystallisation and separation of crystals. The dish containing the solution is then allowed to
cool undisturbed when after some time crystals begin to separate out. If the crystals do not appear even after
a long time, then it may be necessary to scratch the sides of the dish with a glass rod or to ‘'seed' the solution
with a small crystal of the same substance. This provides liny fragments of glass or small particles of the pure

w
substance as nuclei for crystallisation. This process of inducing crystallisation by adding a crystal of the pure
substance into its saturated solution is called seeding.

F lo
When the crystallistaion is complete, the crystals are separated from the mother liquor by filtration
using a Buchner funnel and a water suction pump as shown in Fig. 12.22. The crystals left in the funnel are
washed once or twice with a little of the cold solvent to remove adhering impurities, if any.
(v) Drying of crystals. The crystals are finally dried either in air or by placing them in a steam oven or

ee
an air oven,

Fr
(vi) Removal of colour. Sometimes, the crystals obtained are slightly coloured due to the presence of
certain coloured impurities. In such cases, crystals are redissolved in the same solvent and a small amount of
activated charcoal is added to it. The mixture is boiled for 15-20 minutes. During this treatment, charcoal

for
absorbs all the coloured impurities. The charcoal is then filtered out and the filtrate is allowed to cool when
ur
colourless crystals of the pure substance are obtained.
Examples. For proper crystallisation, the choice of the solvent is very crucial. This is evident form the
following two examples :
s
ook
(/) Crystallisation of sugar. Suppose we have a sample of sugar containing an impurity of common
Yo
salt (sodium chloride). This can be purified by shaking the impure solid with hot ethanol at 348 K. The
sugar will dissolve whereas the common salt remains insoluble. The hot solution is filtered, concentrated
eB

and then allowed to cool when crystals of sugar will separate out. Had water been used as a solvent, the
purification of sugar would not have been possible since both sugar and common salt are readily soluble in
water ?
our

{ii) Crystallisation of benzoic acid. Suppose we have a mixture of benzoic acid and naphthalene. This
ad

mixture can be purified by treating the impure solid witli hot water. Benzoic acid will dissolve while naphthalene
remains insoluble. The hot solution is filtered and then allowed to cool when ciystals of benzoic acid separate
out. The crystals are separated by filtration and dried . Had benzene been used as a solvent, instead of water
Y

in this case, the purification of benzoic acid would not have been possible since both benzoic acid and
Re

naphthalene are quite soluble in benzene.


nd

12.16.3. Fractional Crystallisation


Fi

This method is used to separate and purify two or more compounds which have different solubilities in
the same solvent. The process involves a series of repeated crystallisations as discussed below :
The mixture is dissolved in a suitable solvent in which the two components of the mixture have different
solubilities. When a hot saturated solution of this mixture is allowed to cool, the less soluble substance
crystallises out first while the more soluble substance remains in the solution. The crystals are then septnated
from the mother liquor as discussed above and the mother liquor is again concentrated and the hot solution
again allowed to cool when the crystals of the second {i.e., more soluble) compound are obtained. By repeating
this process, all the components of a mixture can be separated.
The process of separation of different components of a mixture by repeated crystallisations is
known as
fractional crytallisation.
12/100 “pfiadeefi- ’a New Course Chemistry (XI)
12.16.4. Sublimation

It involves the direct conversion of a solid into the gaseous state on heating without passing
through the intervening liquid state and vice versa on cooling.
Only those substances whose vapour pressures become equal to the atmospheric pressure much before
their respective melting points are capable of undergoing sublimation. Such substances are called sublimable.
Since the number of such compounds is expected to be small, therefore, the process of sublimation is not of
general application.
The process of sublimation is very useful in the purification of such solids which sublime on heating and
are associated with non-volatile impurities.
FIGURE 12.23

w
The impure substance is taken in a china dish covered with a
perforated filter paper over which an inverted funnel is placed. The COTTON PLUG

stem of the funnel is plugged with a little cotton (Fig. 12.23). On

F lo
lieating the dish on a sand bath, vapours of the volatile solid rise up, SUBLIMATE

pass through the holes in the filter paper and condense on the cooler PERFORATED
walls of the funnel leaving behind the non-volatile impurities in the FILTER PAPER
dish. CRUDE

ee
Camphor, naphthalene, anthracene, benzoic acid, iodine, etc. ORGANIC

Fr
SUBSTANCE
are purified by this process.
In case of organic compounds which are decomposed by heat,
sublimation is done under reduced pressure.

for
Sublimation
12.16.5. Simple Distillation
r
Liquids are generally purified by simple distillation.
You
Distillation involves conversion of a liquid into vapours by heating followed by condensation of
s
ook
the vapours thus produced by cooling.
The method is commonly used for FIGURE 12.24
eB

those Uquids which are sufficiently stable


at their boiling points and which contain
non-volatile impurities. For example,
simple organic liquids such as benzene, i TO SINK
our
ad

ethanol, acetone, chloroform, carbon .f


ihU
tetrachloride, toluene, xylenes, etc. can
be purified by simple distillation. IMPURE ADAPTER
Procedure. The apparatus used for LIQUID /
Y

simple distillation is shown in Fig. 12.24.


Re

When the flask is heated, the temperature / .PURE


d

FROM TAP LIQUID


rises gradually and the liquid starts
Fin

boiling when its vapour pressure


becomes equal to the atmospheric
pressure. These vapours as they pass Apparatus for simple distillation
through the condenser are condensed.
Only the liquid which distils at a constant temperature is collected in a receiver. This gives us the
pure liquid.
Purification of a mixture of liquids. Simple distillation can also be used for the separation and
purification of a mixture of two or more miscible organic liquids provided their boiling points differ by SO
SO K. The separation is based upon the fact that at the b.p. of the more volatile liquid (low boiling) of the
mixture, the vapours almost exclusively consist of the more volatile liquid. Likewise, at the b.p. of the less
volatile liquid (high boiling), vapours almost entirely consist of the less volatile liquid since the more volatile
liquid has already distilled over. Thus, the separation of the liquid mixture into individual components can be
ORGANIC CHEMISTRY-SOME BASIC PRINCIPLES AND TECHNIQUES 12/101

achieved at their respective boiling points : the more volatile component distils over first while the less
volatile component distils over afterwards. The non-volatile impurities and impurities of liquids having boiling
points much higher than those of the two liquids separated above are, however, left in the distillation flask.
This method can be used to separate :
(a) a mixture of ether (b.p. 308 K) and toluene (b.p. 384 K),
(b) a mixture of hexane (b.p. 342 K) and toluene (b.p. 384 K),
(c) a mixture of benzene (b.p. 353 K) or chlorofom (b.p. 334 K) and aniline (b.p. 457 K) and so on.

w
12.16.6. Fractional Distillation
If the b.p.’s of the two liquids of the mixture are very close to one another i.e. differ by 10 K or so, the
separation cannot be achieved by the simple distillation method as described above. This is due to the reason
that at the b.p. of the more volatile liquid of the mixture there will be sufficient vapours of the less volatile

Flo
liquid as well. As a result, both the liquids of the mixture will distil together and the separation is denied.
The separation of such a liquid mixture into individual

e
components can, however, be achieved by fractional

re
distillation, which involves repeated distillations and
condensations. Fractional distillation is carried out using a

F
fractionating column. It usually consists of a long glass tube
with a wide bore either packed with glass beads, small stones,
porcelain irngs or coke, or blown into a number of spherical
ur
or
or pear-shaped bulbs. The actual purpo.se of the fractionating
column is to increase the cooling surface area and to provide

f
hurdles or obstructions to the ascending vapours and ks
descending liquid. 5ome of the fractionating columns
commonly employed in the labo- ratory are shown in Fig.
Yo
12.25.
oo
The apparatus used for fractional distillation is shown in
B

Fig. 12.26.
Principle. Suppose we have a Various types of fractionating columns
re

mixture of two liquids ‘A’ and ‘B’ of which


FIGURE 12.26
‘A’ is more volatile than ‘B’. When such a
liquid mixture is heated, the temperature
u
ad

rises slowly and mixture starts boiling. The


Yo

vapours afnned mainly consist of the more TO SINK

volatile liquid ‘A' with little of the less


volatile liquid ‘B\ As these vapours travel CONDENSER
d

up the fractionating column, the vapours


Re
in

of the less volatile liquid ‘B’ condense


more readily than those of the more
F

ADAPTER
volatile liquid ‘A’. Therefore, the vapours MIXTURE OF

rising up become richer in ‘A’ and the / LIQUIDS


COLD WATER
liquid flowing down becomes richer in ‘B’. FROM TAP
This process of distillations and RECEIVER

condensations is repeated at eveiy point


in the fractionating column. As a result of DISTILLATE
series of successive distillations, by the
time the vapours reach the top of the
column and escape into the condenser,
they consist mainly of the more volatile
component ‘A’.
Apparatus for fractional distillation using a fractionating column
12/102 New Course Chemistry (XI)EEIHD

Similarly, after a series of successive distillations, the remaining liquid in the distillation flask gets
enriched in higher boiling component. Each successive condensation and vaporization is called theoretical
plate. Commercially, columns with hundreds of theoretical plates are available.
Applications, (a) One of the technological applications of fractional distilhuion is to separate crude oil
in petroleum industry into various useful fractions such as gasoline, kerosene Oil, diesel oil, lubricating oil,
etc.

(b) Fractional distillation has also been used to separate acetone (b.p. 329 K) and methyl alcohol (b.p.
338 K) from pyroligneous acid obtained by destructive distillation of wood.
SUPPLEMENT YOUR

w
KNOWLEDGE FOR COMPETITIONS

Azeotropic distillation. Rectified spirit contains about 95% alcohol (b.p. 351 K) and 5% water (b.p. 373 K)
but alcohol and water cannot be separated from this mixture even though their boiling points differ by

F lo
22 K. This is due to the reason that at this composition, alcohol and water form an azeotrope {i.e. constant
boiling mixture) and thus both the components of the mixture distil together like a pure liquid. To remove
water from such a mixture, a special method called azeotropic distillation is u.sed. Azeotropic distillation means
fractional distillation using a suitable volatile solvent. For example, in the present case, benzene is used.

ee
Rectified spirit is mixed with a suitable amount of benzene and subjected to fractional distillation. The first

Fr
fraction obtained at 331-8 K is a ternary azeotrope consisting of all the water, some alcohol and benzene
(water 7-4%, benzene 74-1%, alcohol 18-5%). The second fraction is a binary azeotrope consisting of the
remaining benzene and some alcohol (benzene 67-7%, alcohol 32-3%) which distils over at 341-2 K. The

for
third fraction which distils at 351 K is absolute alcohol.
ur
12.16.7. Distillation under reduced pressure or Vacuum Distillation
This method is used for the purification of high boiling liquids and liquids which decompose at or below
s
ook
their boiling points.
Yo
Principle. A liquid boils FIGURE 12.27
when its vapour pressure becomes
eB

equal to the external pressure. lij

A
SCREWW MANOMETER
Obviously, the same liquid would CLIP
boil at a lower temperature if the MM
r

TO SINK
pressure acting on it is reduced.
f
ad
ou

Since the liquid now boils at a


lower temperature, its decom f -CAPILLAR't^
ADAPTER
position does not occur. TO REGULATE TO
Y

AIR VACUUM
With the commonly used PUMP
Re

water suction pumps in the IMPURE


nd

/
laboratory, a pressure of 10-20 mm LIQUID
FROM TAP
Hg can be obtained. Under these
Fi

conditions, the boiling points are PURE-


LIQUID
reduced by about 100 degrees.
However, with vacuum pumps,
Distillation under reduced pressure
pressure of the order of 0.1 mm Hg
can be easily obtained.
Apparatus. The apparatus used for vacuum distillation is shown in Fig. 12.27.
Given below are some of the examples where vacuum distillation has been used to purily liquids:
(/) Glycerol which decomposes at its boiling point (563 K) can be distilled without decomposition at
453 K under 12 mm Hg pressure.
(//) Concentration of sugarcane in sugar industry.
ORGANIC CHEMISTRY-SOME BASIC PRINCIPLES AND TECHNIQUES 12/103

1Z16.8. Steam Distillation

This is a convenient method for the separation and purification of organic compounds (solid or liquid)
from non-volatile organic or inorganic impurities. This method is applicable to only those compounds which
are volatile in steam, insoluble in wate,r possess a vapour pressure of about 10-15 mm Hg at 373 K and
contain non-volatile impurities.
Steam distillation is particularly valuable when the substance to be purified boils above 373 K at 760 mm
and decomposes at or below its boiling point. This is due to the fact that steam distillation makes the high-
boiling substances to distil at low temperatures and hence avoids their decomposition. In this respect, ste am

w
distillation is comparable to vacuum distillation even though there is no reduction in the total pressure acting
on the solution during the process of distillation.
Principle. In this method, a mixture of two immiscible liquids, i.e., water and an organic liquid is
heated. Each would exert its own vapour pressure independently of the other and the mixture will begin to

Flo
boil at a temperature when the sum of the vapour pressures of the organic liquid (pj) and that of water (p-y)
becomes equal to the atmospheric pressure (p).

e
P=P\ +Pi

re
Unless the vapour pressure of water or that of the organic liquid is zero, the temperature at which the
mixture boils must be lower than the normal boiling point of both the organic liquid and the water. In other

rF
words, the organic liquid boils at a temperature lower than its normal b.p. and hence the decomposition is
avoided.
ur
Further, we know from Dalton’s law that the partial pressures of different gases in a gaseous mixture

fo
are in the ratio of their relative number of moles. Therefore, the ratio of the number of moles of the organic
liquid and water in the distillate must be in the ratio of their partial vapour pressures (in the boiling mixture).
i.e..
ks
Yo
!h_ = lL
oo
.(0
«2 Pi
where n^ and «2 ^re the no. of moles and, p^ and P2 the partial pressures of the organic liquid and water
B

respectively.
re

Let w, and W2 be the respective masses of the organic liquid and water in the distillate. Then,
n
where M is the molecular mass of the organic liquid
u

1
ad

M
Yo

and
_W2
n
2 “ where 18 is the molecular mass of water.
18

Substituting the above values of nj and «2 in equation (/), we get.


nd
Re

w
I w.
1
xM
— X
M
or X
...(«)
'^2 Pi Wy /?2 X 1 8
Fi

Eq. (ii) gives the relative masses of the organic liquid and water in the distillate.
Consider the following examples for illustration :
(0 A mixture of nitrobenzene {b.p. 483 K) and water {b.p. 373 K) boils at 372.3 K during steam distillation.
At this temperature, the vapour pressure of nitrobenzene (pj) is 20 mm and that of water (^2) is 740 mm. It
follows from the equation (/) that the ratio of the no. of moles of nitrobenzene and water in the distillate must

be 20/740 or 1 : 37 i.e. in the ratio of their respective vapour pressures.


{ii) A mixture of water (b.p. 373 K) and aniline (b.p. 457 K) boils at 371 K at 760 mm pressure in steam
distillation. The vapour pressures of aniline and water at this temperature are 43 mm and 717 mm respectively.
Therefore, the ratio of the no. of moles of aniline and water in the distillate must be 43 : 717 i.e., in the
ratio of their respective vapour pressures.
12/104 New Course Chemistry (X1)DS25D

FIGURE 12.28
Procedure. The apparatus used
for steam distillation is shown in Fig. SAFETY
STEAM + VAPOURS
TUBE
12.28. STEAM OF
STEAM ORGANIC LIQUID
The impure organic compound GENERATOR
TO SINK
mixed with water is taken in a round
bottomed fla.sk and steam is passed.
The r ixture starts boiling when the
co-nbined vapour pressure becomes
z< ADAPTER
equal to the atmospheric pressure. At ^IMPURE PURE
this temperature, steam mixed with ^ S' ORGANIC
ORGANIC
[;|m LIQUID

w
vapours of the compound passes over LIQUID
V.''/ +
/
to the condenser where they are
+
WATER FROM TAP
/ATER
condensed and collected in the

F lo
receiver. The distillate contains the
desired substance and water which
can easily be separated with the help Apparatus for steam distillation
of a separating funnel.

ee
Some of the compounds which can be purified by this process are o-nitroplienol, bromobenzene, aniline,
nitrobenzene, essential oils, turpentine oil, etc.

Fr
12.16.9. Differential Extraction
This method is used to recover organic compounds (solids or liquids) from their aqueous solutions.

for
The process essentially involves the shaking of the aqueous solution of the organic compound in a
ur
separating funnel with a suitable solvent which is immiscible with water but in which the organic compound
is very highly soluble. Ether, benzene, chloroform, carbon tetrachloride are some of the solvents which are
generally employed for extraction.
s
FIGURE 12.29
ook
Procedure. The aqueous solution is mixed with
Yo
Si. ORGANIC
a small quantity of the organic solvent in a separating SOLVENT COMPOUND

funnel (Fig. 12.29). The funnel is stoppered and the LAYER , / IN


eB

SOLVENT
contents are shaken thoroughly for sometimes when ORGANIC LAYER
:h -.i-A:.-,
the organic solvent dissolves the organic compound COMPOUND
-AQUEOUS
present in the aqueous solution. The separating funnel IN
AQUEOUS
LAYER
our

is now allowed to stand for some time when the LAYER


ad

organic solvent and water form two separate layers. ©Before extraction o After extraction
The lower aqueous layer (when the organic solvent
used is benzene or ether) is run out by opening the Differential extraction
Y

tap of the funnel and the organic layer


separated. The aqueous solution is again poured into the funnel, mixed again with a small quantity of the
Re

organic solvent and the process is repeated several times till the entire amount of the organic compound is
nd

extracted.
Fi

The organic layers from all the steps are taken in a distillation flask. The organic solvent is distilled off
leaving the organic compound in the distillation flask.
The efficiency of the process of extraction depends upon the number of times the extraction is repeated.
It has been found that with a given amount of the solvent, larger the number of extractions, greater is the
amount of the material extracted.
This method is normally applicable to non-volatile compounds. For example, benzoic acid can be
extracted from its water solution using benzene.

12.16.10. Chromatography
Chromatography is the most modern and versatile method used for the separation, purification and
testing the purity of organic compounds. This method was first discovered by Tswett, a Russian botanist, in
1906. The name chromatography was originally derived from the Greek word Khroma meaning colour and
ORGANIC CHEMiSTRY>SOME BASIC PRINCIPLES AND TECHNIQUES 12/105

graphy for writing because the method was first used for the separation of coloured substances (plant pigments)
into individual components. Now this method is widely used for separation, puriifcation, identification and
characterisation of the components of a mixture, whether coloured or colourless.
Chromatography is essentially a physical method of separation. It is defined as follows:
The technique of separating the components of a mixture in which separation is achieved by
the differential movement of individual componentsthrough a stationary phase under the
influence of a mobile phase.
Types of chromatography. The stationary phase can be either a solid or tightly bound liquid on a solid
support while the mobile phase can be either a liquid or a gas. Depending upon the nature of the stationary

w
and the mobile phases, the different types of chromatographic techniques commonly used are given in Table
12.1.

F lo
TABLE 12.1. Some common types of chromatography
S.No. Type of chromatography Mobile/stationary phase Uses

ee
1. Adsorption or Column Liquid/Solid Large scale separations
chromatography

Fr
2. Thin layer chromatography (TLC) Liquid/Solid Qualitative analysis (identification and
characterization of organic compounds).
Liquid/Solid Qualitative and quantitative analysis

for
3. High performance liquid
chromatography (HPLC)
ur
4. Gas liquid chromatography (GLC) Gas/Liquid Qualitative and quantitative analysis.
5. Paper or partition chromatography Liquid/Liquid Qualitative and quantitative analysis of
polar organic compounds (sugars, a-
s
ook
amino acids) and inorganic compounds.
Yo
Depending upon the principle involved, chromatography can be divided into the following two categories:
eB

(a) Adsorption chromatography (b) Partition chromatography


(a) Adsorption chromatography
Principle. This method is based upon the differential adsorption of the various components of a mixture
on a suitable adsorbent .luch as silica gel or alumina. Since some compounds are more strongly absorbed
r
ad

than the othe,r they will travel through the column at different rates and thus get separated.
ou

Types of adsorption chromatography


Adsorption chromatography is of the following two types :
Y

(/) Column chromatography (//) Thin layer chromatography.


(i) Column chromatography. Column chromatography is the simplest of all the chromatographic
Re
nd

techniques and is widely used. The whole process is carried out in a long glass column provided with a stop
cock at the bottom. The various steps involved in this process are :
Fi

(i) Preparation of the Column. A plug of cotton or glass wool is placed at the bottom (Fig. 12.30) of a
clean and dry glass column. Above this, a thin layer of acid-washed sand is placed to support the adsorbent.
A suitable adsorbent such as alumina (AI2O3), silica gel, magnesium oxide, starch, charcoal, etc. is made into
slurry with a suitable solvent (preferably non-polar) such as hexane or petroleum ether. The slurry is then
caref^ully packed in the column by gentle tapping so that no air bubble is entraped in the column. This constitutes
the stationary phase.
(/7) Adsorption. The mixture to be separated (or the impure compound to be purified) is dissolved in a
minimum volume of a suitable highly polar solvent* and applied on the top of the column of the adsorbent
*If a less polar or a non-polar solvent is used for dissolving the mixture, it will require a large volume of the
solvent and in that case, it will be difficult to get the mixture adsorbed on the column in a narrow band which is an
essential criterion for optimum separation.
12/106 'P’tadee^'^ New Course Chemistry fxnpPTWTl

FIGURE 12.30

SOLVENT
(MOBILE PHASE) SAND SAND SAND SAND SAND
n
MIXTURE OF" '’ -*(●
UtJ .m
A m A £S:i A
COMPOUNDS ’"'.'.’A IV' i' >

(A+B+C)+SAND B+C B ■it

ADSORBENT- 1
c
(STATIONARY B A
PHASE)

GLASS WOOL 1^

w
:0 :0 :o :0 :0 ;o
0

F lo
'a

:G; :B: ●A-

I II 111 IV V VI

Column chromatography stages I, II, III, IV, V and VI represent the progressive separation of

ee
the mixture into three individual components

Fr
with the help of a dropper or a microsyringe. As the solution travels down, the mixture is adsorbed in a narrow
band. A thin layer of acid-washed sand is again placed at the top of the column followed by a loose plug of

for
cotton or wool. The sand layer prevents the column from being disturbed during the addition of solvent from
time to time.
ur
After the application of the sample, a little amount of the solvent is placed over the sand layer and the
column is allowed to stand for about 15-20 minutes as shown in stage 1 (Fig. 12.30). During this period, the
s
various components of the mixture (say A, B and C) are adsorbed to different extents depending upon their
ook
Yo
polarity (say A > B > C) within a narrow band. Thus, within the narrow band, component A is strongly
adsorbed component B is moderately adsorbed while component C is weakly adsorbed.
eB

(Hi) Elution. It is the process of extraction of the adsorbed components from the adsorbent with the help
of solvents of increasing polarity. The solvents usually employed in the increasing order of polarity are
petroleum ether, carbon tetrachloride, benzene, chloroform, diethyl etlier, ethyl acetate, acetone, alcohol, etc.
our

A solvent or a mixture of solvents which is used to extract the column constitutes the mobile phase and
ad

is usually called an eluent. As the eluent passes down the column, it dissolves the different compounds. The
least strongly adsorbed component of the mixture, i.e., component C is eluted first by the least polar solvent
followed by moderately strongly adsorbed component B by solvent of intermediate polarity while the most
Y

strongly adsorbed component A is eluted last of all by the solvents of higher polarity.
Re

In this way, the various components of the mixture can be separated into different fractions. Distillation
nd

or evaporation of the solvent from different fractions gives the various components of the mixture in pure form.
This technique is being widely used throughout the world for the purification of different substances
Fi

and the separation of mixtures. For example, a mixture of naphthalene (hydrocarbon) and benzophenone
(ketone) can be separated over a column of alumina and by using petroleum ether containing benzene as
eluent. Naphthalene being less polar is weakly adsorbed while benzophenone being more polar is strongly
adsorbed over the column. Elution of the column will first elute naphthalene and then benzophenone.

(ii) Thin layer chromatography (TLC). It is another type of adsorption chromatography in which
separation of the components of a mixture is achieved over a thin layer of an adsorbent. A thin layer (0-2 mm
thick) of an adsorbent such as silica gel or alumina) is spread over a plastic or glass plate of suitable size.
A suitable TLC plate is taken and two pencil lines are drawn across the width of the plate about 1 cm
from each end. The lower pencil line is called the base line while the upper line is called the finish line or
solvent front.
ORGANIC CHEMISTRY-SOME B. k PRINCIPLES AND TECHNIQUES 12/107

A solution of the mixture to be separated is applied as a small FIGURE 12.31

spot with the help of a capillary on the starting line. The plate
is then placed in a closed jar containing a suitable solvent (Fig. JAR

12.31). 4- - -SOLVENT FRONT

As the solvent moves up, tlie components of the mixture also ●' ADSORBENT COATED
move up along the plate to different distances depending upon their ON GLASS PLATE

degree or extent of adsorption. When the solvent front reaches the SAMPLE SPOT
finish line, the plate is removed and then dried in air. - BASE LINE
The spots of coloured components are visible on TLC plate SOLVENT

w
due to their original colour. The spots of the colourless components
which are invisible to the eye can be observed using the following TLC chromatograph being developed
visualization methods.

F lo
(i) Ultraviolet light. Organic compounds which fluoresce can be detected by placing the plate under
UV lamp having light of 254 nm. Since all organic compounds do not produce fluorescence under UV light,
this method is not of general applicability,

ee
(ii) Iodine vapours. This is the most commonly used detection reagent. The developed TLC plate is
placed in a covered Jar containing a few crystals of iodine. Spots of compounds which adsorb iodine will

Fr
show up as brown spots,
(iii) Chemical methods. Sometimes a suitable chemical reagent may be sprayed on the plate. For example,
amino acids can be detected by spraying the plate with ninhydrin solution. Similarly, aldehdyes/ketones can

for
be detected by spraying the plate with the solution of 2, 4-dinitrophehylhydra7,ine.
ur
The various components on the developed TLC plate are identified FIGURE 12.32
through their retention factor, i.e., Rf values (Fig. 12.32). It is defined
s
SOLVENT FRONT
Distance travelled by the compound (X)
ook
Yo
as R
/
Distance travelled by the solvent front (Y) SPOT

Since the solvent front always moves faster on the TLC plate than
eB

the compounds, Revalues are usually expressed as a decimal fraction, X


(b) Partition (or Paper) chromatography. We have discussed
above that column chromatography or TLC is a liquid/solid BASE LINE
r

chromatography i.e., the mobile phase is a liquid while the statiomu^


ad
ou

phase is a solid. In contrast, partition chromatography is a liquid/liquid Developed chromatogram-


chromatography in which both the mobile phase and the stationary phase Mea^urement of Rf value
are liquids.
Y

Paper chromatography is a type of partition chromatography. In paper chromatography, a special quality


paper called chromatographic paper is used. Although paper consists mainly of cellulose, the stationary
Re
nd

phase in paper chromatography is not the cellulose but the water which is adsorbed or chemically bound to
it. The mobile phase is another liquid which is usually a mixture of two or three solvents with water as one
Fi

of the components.
Principle. Paper chromatography works on the principle of partition, i.e.. it is based upon continuous
differential partitioning (or distribution) of the various components of the mixture benveen the stationary
and the mobile phases.
Process. A suitable chromatographic paper is selected and a starling line is drawn across the width of
the paper at about 1 or 2 cm from the bottom. A spot of the mixture of components to be separated is applied
on the starting line with the help of a fine capillary or syringe. The chromatographic paper is then suspended
in a suitable solvent mixture (Fig. I2.33a).
The solvent rises up the paper by capillary action and flows over the spot. The different components of
the mixture travel through different distances depending upon their solubility in or partitioning between the
stationary and the mobile phases. When the solvent reaches the top end of the paper, the paper is taken out
12/108 ‘P'tadee^'a. N'-iv'v Course Chemistry (XI)EZ5IHD

and allowed to dry. The paper strip so developed is called the chromatogram. The spots of the separated
coloured compounds are visible at different heights from the starting line and are identified by their values
as discussed under TLC. The spots of the colourless compounds may, however, be observed either under
ultraviolet light or by the use of an appropriate spray reagent as discussed under TLC.
The type of chro FIGURE 12.33
matography discussed CARD
above is called ascending -- BOARD CARD BOARD

paper chromatography. LID LID

w
Alternatively, the paper can -- JAR
1^
be folded into a cylinder - CHROMATOGRAPHIC
CHROMATOGRAPHIC
and the two ends dipped PAPER
PAPER

together as shown in Fig. -JAR

F lo
\2.33b. This is also
SOLVENT - SOLVENT
sometimes called as
circular chromatography.

ee
There is yet another Apparatus for ascending chromatography
type of paper chromato

Fr
graphy called descending paper chromatography. In this type, the solvent is kept in a trough at the top of
the chamber, spotted end of the paper is dipped in it, and the solvent is allowed to flow down by capillary
action and gravity. In this type, solvent flow is rapid and hence the process is less time consuming than the

for
ascending method.
ur
Uses. Paper chromatography is especially used for separation of sugars and amino acids.
PART-V. QUALITATIVE ANALYSIS s
k
Yo
oo
12.17. GENERAL ANALYSIS

The first step in the structure determination and characterisati on of an organic compound after it has
eB

been obtained in the pure form is to carry out its qualitative analysis, i.e., to detect the various elements
present in it. The elements which commonly occur in organic compounds are carbon (always present) hydrogen
(present in most of the compounds), oxygen and nitrogen. A few other elements such as sulphur, phosphorus,
halogens and metals are also occasionally present.
r
ou
ad

12.17.1. Detection of Carbon and Hydrogen


Principle. The presence of carbon and hydrogen, in an organic compound, is detected by heating the
Y

given compound with dry copper (U) oxide or cupric oxide in a hard glass test tube when carbon present is
oxidised to carbon dioxide and hydrogen is oxidised to water :
nd

A
Re

A
C + 2CuO ^ CO2 + 2Cu 2H -I- CuO H2O + Cu
In general, if the organic compound containing only C and H has the molecularformula then the
Fi

complete combustion equation may be written as


A

C^-Hy + (2 X -I- y/2) CuO ^ X CO2 + y/2 H2O + (2 X -I- y/2)Cu


Carbon dioxide turns lime water milky while water condenses on the cooler parts of the test tube and
turns anhydrous copper sulphate blue.
Ca(OH)2 CO2 ●» CaC03 + H2O ; CUSO4 + 5H2O ^ CUSO4.5H2O
Lime water (Milkiness) (White) (Blue)
Procedure. In actual practice, a .small quantity of the pure and dry compound is mixed with nearly five
to six times its weight of dry and pure cupric oxide powder. The intimate mixture is strongly heated in a hard
glass test tube fitted with a delivery tube having a bulb in the centre as shown in Fig. 12.34. The other end of
the delivery tube is dipped in lime water taken in a test tube.
ORGANIC CHEMISTRY'SOME BASIC PRINCIPLES AND TECHNIQUES 12/109

The bulb in the delivery tube is packed with FIGURE 12.34!

anhydrous copper sulphate supported over glass wool. ORGANIC

On heating, the carbon is oxidised to CO2 which turns COMPOUND +


CUPRIC OXIDE
lime water milky and hydrogen is oxidised to water
which turns anhydrous copper sulphate blue. BULB WITH A
—^TRAP
Modifications, (i) If the organic substance is a

ow
volatile liquid or a gas, the vapours of the compound
are passed through heated cupric oxide taken in a
hard glass test tube and gases evolved are tested for
ANHYDROUS
CO2 and H2O vapours as described above. CuS04
(ii) If the organic compound also contains tP-
l-o
sulphur besides carbon and hydrogen, the method is LIME WATER

e
slightly modified. This is due to the fact that sulphur
is oxidised to sulphur dioxide which also turns lime

re
water milky due to the formation of insoluble calcium

rFl
Detection of Carbon and Hydrogen
sulphite.

F
A
4CuO + S ■> 2CU2O + SO2 \ Ca(OH)2 + SO2 ^ CaS03 + H2O
(Milkiiiess)

In such a case, the outcoming gases are first passed through an acidified solution of potassium dichromate

r
ou
which absorbs sulphur dioxide and turns it green, and then through lime water.

sfo
12.17.2. Detection of Nitrogen
The presence of nitrogen is detected by the following tests ;
k
1. Dry heating test. If the organic compound containing nitrogen is heated strongly, it gives a smell of
oo
burning hair or feather.
Limitation. This test is, however, not reliable since many compounds containing nitrogen do not give
Y
this test.
B

2. Soda lime test. A pinch of the organic compound is heated strongly in a dry test tube with soda-lime
(NaOH + CaO). A smell of ammonia indicates the presence of nitrogen.
re

CaO
NH2CONH2 + 2NaOH 2NH3 + Na2C03
ou

A
Y

Urea
ad

Limitation. This test is also not reliable since many organic compounds containing nitrogen such as
nitro (- NO2). azo (-N = N-) groups, etc. do not give this test.
3. Lassaigne’s test. This is the most reliable test for detecting nitrogen, sulphur and halogens in an
d

organic compound. In this test, the elements present in the organic compound arc convened from covalent
in
Re

form into the ionic form by fusing the compound with sodium metal. This test is carried out as follows:
(1) Preparation of the Lassaigne’s extract. A small piece of freshly cut sodium (of the size of a pea) is
heated gently in a fusion tube till it forms a shining globule. The tube is removed from the flame and a small
F

amount of the organic compound (50—60 mg) is added and the tube heated strongly till it becomes red hot
(2—3 minutes). The hot tube is then plunged into a china dish containing 10—15 mL of distilled water. The
contents of the china dish are boiled for a few minutes, cooled and then filtered. The filtrate is called Lassaigne’s
extract or sodium fusion extract.
(11) Test for nitrogen. The Lassaigne’s extract is usually alkaline since the excess of sodium reacts with
water to produce sodium hydroxide. If not, it is made alkaline by adding a few drops of a dilute solution of
sodium hydroxide. To a part of this alkaline solution is added a few drops of a freshly prepared solution of
ferrous sulphate. The contents are warmed a little, cooled and then acidified with dil. H2SO4. Appearance of
a green or blue colouration indicates the presence of nitrogen. However, appearance of blood red colour
indicates the presence of both nitrogen and sulphur.
12/110 ^nadee^'a^ New Course Chemistry (XI)EEIHIl

Chemistry of the test. During fusion, carbon and nitrogen of the organic compound combine to form
sodium cyanide.
A
Na + C + N -> NaCN

(From organic compound)


On heating the filtrate with ferrous sulphate solution, sodium ferrocyanide, i.e., sodium hexacyanoferrate
(II) is formed and at the same time some ferrous (Fe“'*‘) ions are oxidised to ferric (Fe^'*') ions. These Fe^'*’
ions then react with sodium hexacyanoferrate (II) to produce iron (III) hexacyanoferrate (II) or ferriferrocyanide
which is prussion blue* in colour.
2NaCN + FeS04 ^ Na7S04 + Fe(CN)2
“ Na4lFc(CN)6]

w
Fc(CN)2 + 4NaCN
Sodium hexacyanoferrate (II)
3+
3Nu4 fFe(CN)6l + 4Fe Fe4[Fe(CN)6l3 + 12 Na+

F lo
Iron (III) hexacyanoferrate (II)
(Prussian blue)
If nitrogen and sulphur both are present in the organic compound, they may combine during fusion to
form sodium thiocyanate (sulphocyanide) due to insufficient sodium. This when heated with ferrous sulphate

ee
produces a blood red colouration due to ferric thiocyanate (or sulphocyanide) by reaction with ferric ions

Fr
formed by oxidation of efrrous ions.
A
Na + C + S + N ■> NaSCN

for
Sod. thiocyanate
Fe^+ + 3 NaSCN ■¥
Fe(SCN)3 + 3Na+
ur
Ferric thiocyanate
(Blood red colouration)
s
But the absence of blood red colouration does not necessarily mean that sulphur is absent. This is due to
ok
Yo
the reason that in presence of excess of sodium metal, sodium thiocyanate decomposes to form sodium
cyanide and sodium sulphide.
o
eB

A
2Na + NaSCN ^ Na2S + NaCN
12.17.3. Detection of Halogens
The presence of halogens in an organic compound is detected by the following tests :
r
ad
ou

1. Beilstein test. It is a very simple and sensitive test for the detection of halogens in an organic
compound. In this test, a clean and stout copper wire is heated in the non-luminousflame of the Bunsen
burner until it ceases to impart any green or bluish green colour to the flame. The heated end is then dipped
Y

in the organic compound and again introduced into the Bunsen flame. The appearance of a green or bluish
green flame due to the formation of volatile cupric halides indicates the presence of halogens in the
Re

organic compound.
nd

Limitation.s. (/) Organic compounds like urea, thiourea, etc. which do not contain halogens also give
this test due to the formation of volatile cupric cyanide.
Fi

(/'O It does not tell as to which halogen (chlorine, bromine, or iodine) is actually present in the organic
compound.
2. Lassaigne’s test. It is a very reliable test for the detection of halogens in an organic compound. It
involves the following steps :
(i) Preparation of the La.ssaigne’s extract. A pinch of the organic compound is fused with a small
piece of freshly cut sodium in a fusion tube to prepare the sodium fusion extract in the manner described
*Instead of Fe4(Fe(CN)^]3. prussian blue is also written as NaFe^^^ [Fe^^CN)^] and its formation occurs
according to the following equation :
2 Na4[Fe” (CN)^] + Fe2‘“(S04)3 ^ 2 NaFe”^ [Fe^ICNI^] + 3 Na2S04
ORGANIC CHEMISTRY-SOME BASIC PRINCIPLES AND TECHNIQUES 12/111

under detection of nitrogen. During fusion, the halogens present in the organic compound are converted into
the corresponding sodium halides.
A
Na + X ^ NaX (X = Cl, Br or I)
(From organic compound)
(h) Test for halogens. A part of the Lassaigne’s extract is boiled with dil. HNO3 and cooled. A few
drops of silver nitrate solution are then added,
(fl) A white precipitate soluble in ammonia and insoluble in dil. HNO^ indicates the presence of chlorine,
NaCl + AgN03 AgCl + NaN03

w
(While ppt.)
(b) A pale yellow precipitate partially soluble in ammonia indicates the presence of bromine.
NaBr + AgN03 AgBr + NaN03

Flo
(Pale yellow ppi.)
(c) A yellow precipitate iihsoluble in ammonia indicates the presence of iodine.
Nal + AgN03 Agl + NaNO^

ee
(Yellow ppl.)

Fr
Function of nitric acid. If the organic compound also contains nitrogen or sulphur, the Lassaigne’s
extract on boiling with dil. HNO3 decomposes sodium cyanide or sodium sulphide formed during fusion.
NaCN + HNO3 > NaN03 + HCN T ; Na^S 2HNO3 > 2NaN03 + H^S T

for
If cyanide and sulphide ions are not decomposed, they will react with silver nitrate and hence will
interfere with the test.
ur
NaCN + AgNOj > AgCN + NaNOj
Silver cyanide
ks
(White ppl.)
Yo
N-d2S + 2AgN03 > Ag^S + 2NaN03
oo
Silver sulphide
eB

(Black ppl.)
3. Carbon disulphide test for Bromine and Iodine. A small portion of the Lassaigne’s extract is
boiled with dil. H2SO4 to decompose sodium cyanide and sodium sulphide. The solution is then cooled and
a few mL of freshly prepared chlorine water and carbon disulphide or caibon tetrachloride are added. The
r
ou

solution is vigorously shaken and allowed to stand when :


ad

(/) an orange colour in CSo or CCI4 layer indicates the presence of bromine.
Y

(//) a violet colour in CS2 or CC/4 layer indicates the pre.sence of iodine.
Theory of the test. This is due to the reason that chlorine displaces bromine and iodine from their
corresponding halides. The halogen thus liberated dissolves in CS2 or CCI4 to produce the specific colour.
nd
Re

2NaBr + CL ^ 2NaCl + Br-> (Di.ssol\’es in CS2 or CC/4 orange colour)


2NaI + CI2 > 2NaCl + I2 (Dis.solvcs in CS2 or CC/4 violet colour)
Fi

12.17.4. Detection of Sulphur


The presence of sulphur in the organic compound is delected by the following tests :
1. Lassaigne^s test. If sulphur is present in the organic compound then on fusion with sodium metal,
sodium sulphide is formed. A
2Na + S ^ Na^S
Then the following tests are performed with the Lassaigne’s extract to detect the presence of sulphur :
(0 Sodium nitroprusside test. A small portion of the Lassaigne’s filtrate is treated with a few drops of
sodium nitroprusside solution when a violet colouration is obtained. This colour slowly fades on standing.
NajS + Na2 [Fe(CN)5(NO)J ^ Na4[Fe(CN)5(NOS)]
Sodium nitroprusside (Violet colour)
12/112 "^tadee^ ^ New Course Chemistry (XI) w»iwii

(«) Lead acetate test Another portion of Lassaigne’s filtrate is acidified with dilute acetic acid and a
few drops of lead acetate solution are added to it. Formation of black precipitate of lead sulphide indicates
the presence of sulphur in the given compound.
NajS + (CH3C00)2 Pb ■>
PbS + 2CH3COONa
Lead acetate (Black ppl.)
2. Oxidation test. The given compound is fused with a mixture of potassium nitrate and sodium carbonate.
If sulphur is present, it gets oxidised to sulphate :
A
KNO3 ^ KNO2 + [O]
A
Na2C03 + S + 3 [Ol ^ Na2S04 + CO2
The fused mass is then extracted with water and filtered. The filtrate is first acidified with dilute
hydrochloric acid and then treated with barium chloride solution when a white precipitate insoluble in
hxdrochloric acid is obtained.

F low
Na-)S04 + BaCl-) ^ 2NaCl + BaS04
(While ppt.)

12.17.5. Detection of Phosphorus


Phosphorus is detected by fusing the organic compound with an oxidising agent, i.e., sodium peroxide
when phosphorus is oxidised to sodium phosphate.
A
5Na202 + 2P -> 2Na3P04 + 2Na20
(From organic compound)

re
for F
The fused mass is extracted with water. The aqueous solution is boiled with cone. HNO3 and then
ammonium molybdate solution is added. The appearance of a yellow precipitate or colouration due to the
formation of ammonium phosphomolybdate indicates the presence of phosphorus.
A

Na3P04 + 3HNO3 ^ H3PO4 + SNaNOj


A

H3PO4 + 12 (NH4)2 M0O4 + 21 HNO3


Your
4 (NH4)3P04-12Mo03 + 21 NH4NO3 + I2H2O
ks

Amm. molybdate. Amm, phosphomolybdate


eBoo

(Yellow ppt.)

12.17.6. Detection of Oxygen


There are no direct tests available for the detection of oxygen in an organic compound. However, its
ad

presence can be detected indirectly by any one of the following methods:


our

(i) The compound is tested for the presence of oxygen containing functional group such as -OH,
-CHO, -COOH, -NO2, etc. The presence of any one of these groups in the compound, in turn, confirms the
presence of oxygen in it.
(ii) If the sum of the percentages of various elements present in the given compound, as determined by
Re

known methods, comes out to be less than hundred, the presence of oxygen is indicated and the difference is
taken to be equal to the percentage of oxygen in the given compound.
Y
Find

Curiosity Questions
f Q. 1. Can LI or K metal be used in place of Na during detection of elements by Lassaigne’s
test ?

Ans. Li is not used because it is much less reactive than Na and hence requires longer times to
complete fusion. K is not used because it readily catches fire and hence quite difficult to handle.
Q. 2. Can AgN03 test be used for the detection of fluoride ion in Lassaigne’s extract ?
Ans. No. The reason being that AgF is highly soluble in water and hence F“ ion present in Lassaigne’s
extract does not produce any precipitate of AgF.
NaF + AgNOg ^ No ppt. of AgF
Lassaigne’s extract
1
ORGANIC CHEMISTRY-SOME BASIC PRINCIPLES AND TECHNIQUES 12/113

Q. 3. For testing the presence of sulphur by lead acetate test, the Lassaigne’s extract is acidified with
dilute acetic acid. Can we use dilute hydrochloric or sulphuric acid instead of acetic acid ?
Ans. For testing sulphur, the Lassaigne’s extract is acidified with acetic acid because lead acetate is
soluble and does not interfere with the test. However, if HCI or H2SO4 is used, lead acetate will
react with it forming white ppt. of PbCig or PbS04 and, therefore, it will interfere with the test.
(CH3COO)2Pb + 2 HCI ^ 2CH3COOH + PbClg
Lead acetate Acetic acid (White ppt.)
(CH3COO)2Pb + H2SO4 2 CH3COOH + PbS04
{White ppt.)

PART-VI. QUANTITATIVE ANALYSIS

w
12.18. GENERAL INTRODUCTION
After detecting various elements present in a compound, the next step is to determine the percentage of

F lo
each element. This is called quantitative analysis. We shall now discuss the principles involved in the estimation
(quantitative determination) of various elements usually present in an organic compound.
12.18.1. Estimation of Carbon and Hydrogen

ee
Principle. A known mass of the organic compound is heated strongly with excess of dry copper oxide in
a current of dry air or oxygen {free from carbon dioxide). Under these conditions, carbon present in the

Fr
organic compound is oxidised to carbon dioxide and hydrogen is oxidised to water.
A

for
C (from organic compound) + 2CuO CO2 + 2Cu
A
2H (from organic compound) + CuO ■> H'jO + Cu
our
The water thus produced is absorbed in a U-tube containing anhydrous calcium chloride or anhydrous
magnesium perchlorate while CO2 produced is absorbed in another U-tube containing a strong solution of
s
KOH or ascarite (NaOH + CaO). The tubes are weighed before and after the comliustion. The increase in the
ook

mass of CaCl2 or Mg( €10^)2 U-tube gives the mass of water produced while increase in the ma.ss of KOH or
ascarite U-tube gives the mass of CO2 produced.
Y
eB

The apparatus used for FIGURE 12.35


estimation of carbon and SAMPLE IN CuO
EXCESS
hydrogen is shown in Fig. 12.35. PLATINUM BOAT
OXYGEN
our
ad

Calculations : Let the t-


r
mass of the substance taken I B

V
1
j > B
= w g
PURE
FURNACE
Mass of CO2 formed DRY
dY

OXYGEN
= -vg COMBUSTION
Re

Mass of water formed TUBE


Fin

CaCl2 KOH
Percentage of Carbon TUBE TUBE

One mole of CO2


contains one gram atom of C. Apparatus for estimation of Carbon and Hydrogen
12
i.e. (12 + 2x 16) = 44g CO2 contain carbon = 12 g xg CO2 will contain carbon = ^ g
This is the mass of carbon present in w g of the compound.
12 100
.●. % age of carbon in the compound = —
44
x a: x w

I.e.,
12
Percentage of carbon = — x Mass of CO2 formed xlOO
44 Mass of substance taken

I
12/114 ‘Pxa.ti^'A New Course Chemistry fXIUw^TI

Percentage of Hydrogen
One mole of H2O contains 2 gram atoms of hydrogen.
2
i.e. (2 X 1 + 16) = 18 g of H2O contain hydrogen = 2 g y g H2O will contain hydrogen ~ ^^ ^
This is the mass of hydrogen present in w g of the compound.
2 100
% age of hydrogen in the compound = 18
X y X
w

2 Mass of H2O formed xlOO


i.e.. Percentage of hydrogen = — X
18 Mass of substance taken

w
Modifications under specific conditions. Liebeg’s method as described above is suitable in case of
organic compounds containing C, H and O only. If, however, the organic compound contains one or more of

F lo
tlic elements like N, S and halogens also, the method is modified as under :
(/) Substances containing nitrogen. Under the conditions of combustion, nitrogen, if present, in the
organic compound is oxidised to oxides of nitrogen (NO, NO2, etc.) which are also absorbed in KOH solution.
In such cases, a reduced copper gauze is placed near the exit end of the tube (Fig. 12.27) which reduces

ee
oxides of nitrogen back to N2 gas.

Fr
Heat
2Cu + 2NO » 2CuO + N 2 4Cu + 2NO2 > 4CuO + N2
(«) Substances containing halogens. Halogens, if present, in the organic compound are converted into

for
cupric halides which themselves decompose to form free halogens. These halogens and volatile cupric halides
also get dissolved in KOH solution. In such cases, a roll of bright silver gauze is placed near the exit end of
our
the combustion tube. Silver gauze decomposes volatile cupric halides forming non-volatile silver halides.
Halogens also combine with silver giving silver halides.
s
CuX2 + 2 Ag > 2 AgX + Cu : X, + 2Ag ^ 2AgX
ook
{Hi) Substances containing sulphur or sulphur and halogens. Under the conditions of combustion,
sulphur, if present in the organic compound is oxidised to SO2 which will also be absorbed in KOH solution.
Y
eB

In such cases, either an additional layer of fused lead chromate is placed after the CuO layer or CuO layer is
replaced by fused lead chromate layer. At high temperatures, lead chromate decomposes to give O2 which
oxidises C and H of the organic compound to CO2 and H2O respectively.
our

A
^ 4 PbO + 2 Cr203 + 3 O2
ad

4 PbCr04
SOt and halogens produced during combustion react with lead chromate to form non-volatile lead
sulphate and lead halides which are retained in the combustion tube.
Y

A
4 PbCr04 + 4 SO2 ^ 4 PbS04 + 2 Cr203 + O2
Re

A
nd

4 PbCr04 + 4 X2 ^ 4 PbX2 + 2 Cr203 + 5 O2


Besides, S and halogens. PbCr04 is also useful for phosphorus containing compounds. This is because
Fi

oxides of phosphorus produced during combustion also react with lead chromate to form non-volatile lead
phosphate.

12PbCr04 + 4P205 ^ 4 Pb3(P04)2 + 6 Cr203 + 9 O2


Sample Problem On complete combustion, 0-246 g of an organic compound gave 0*198 g
of carbon dioxide and 0*1014 g of water. Determine the percentage composition of carbon and hydrogen in
the compound. (NCERT Solved Example)
Solution. Here, mass of the organic substance taken = 0.246 g
Mass of CO2 formed = 0.198 g
Mass of H2O formed = 0.1014 g

I
ORGANIC CHEMISTRY-SOME BASIC PRINCIPLES AND TECHNIQUES 12/115

(;) Percentage of Carbon.


One mole of CO2 contains one gram atom of carbon.
i.e. 44 g of CO2 contain carbon = 12 g
^ P
0-198 g CO-y will contain carbon = — x 0-198 g
44

This is the mass of carbon present in 0-246 g of the compound.


12 100
% age of carbon in the compound = — x 0-198 x = 21-95
44 0-246

w
(//) Percentage of Hydrogen
One mole of H2O contains two gram atoms of hydrogen.
2
i.e. 18 g of H2O contain hydrogen = 2 g 0-1014 g H2O will contain hydrogen =—xO-1014

Flo
18

This is the mass of hydrogen present in 0-246 g of the compound.


2 100
%age of hydrogen in the compound = — xO-1014 x = 4-58

ee
18 0-246

Fr
1. 0-6723 g of an organic compound gave on combustion 1-530 g of carbon dioxide and 0-625 g of water,

for
ur
Find the percentage of carbon and hydrogen in the compound.
2. 0-465 g of an organic substance gave on combustion 1-32 g of CO2 and 0-315g of H2O. Calculate the
percentage of carbon and hydrogen in the compound.
ks
3. 0-2475 g of an organic substance gave on combustion 0-495 g of CO-, and 0-2025 g of H2O. Calculate the
Yo
percentage of carbon and hydrogen in it.
oo
4. On complete combustion of 0-492 g of an organic compoundcontainingC. H and 0. 0-7938 g of CO-> and
0-4428 g of H2O was produced. The % composition of oxygen in the compound is (nearest integer).
eB

(JEE Main 2022)


ANSWERS
r

1. C = 62-07%, H= 10-33% 2. C = 77-42%, H = 7-53% 3. C = 54-54%, H = 9-09%


ou
ad

12 a7938
4. %C = —X xl00 = 44-00
44 a492
Y

2 G4428
%H = —X xI00 = 10-00
18 0492
Re
nd

% O = 100 - (44-00 + 10-00) = 46-00


Fi

12.18.2. Estimation of Nitrogen


The two most commonly used methods for the estimation of nitrogen in an organic compound are :
I. Dumas method 2. Kjeldahl’s method.
1. Dumas method. Thi.s method is applicable to all organic compounds containing nitrogen and is
briefly discussed below :
Principle. A known mass of the organic substance is heated with excess of copper (II) oxide in an
atmosphere of CO2. Carbon, hydrogen and .sulphur (if present) are oxidised to CO->, H2O and SO^ respectively
while nitrogen gas is set free. Any oxide of nitrogen that may be formed is reduced back to free nitrogen by
passing over a hot reduced copper gauze.

t
12/116 “Pfieideefr'A New Course Chemistry (Xl)EZsl9D
A A

C + 2CuO > CO2 + 2Cu ; 2H + CuO H2O + CU


Nitrogen + CuO 1^2 + 3 small amount of oxides of nitrogen
A
Oxides of nitrogen + Cu CuO + N2
If the organic compound having nitrogen is assumed to have C^H^^ molecular formula then the

w
complete combustion equation may be written as

C^^N, + (2x + y/2) CuO XCO2 + y/2 H2O + z/2 N2 + (2 X + y/2) Cu


The nitrogen thus formed is collected over cone. KOH solution which absorbs all other gases i.e. CO2,
H2O vapours, SO2 etc. The volume of nitrogen collected is thus noted and from this the percentage of nitrogen

o
can be calculated.

e
Apparatus. The apparatus used for estimation of nitrogen by Duma’s method is shown in Fig. 12.36.

re
It consists of three parts : FIGURE 12.36

rFl
{a) carbon dioxide generator CuO + ORGANIC
COARSE REDUCED KOH

A
CuO

F
COMPOUND COPPER GAUZE SOLUTION
(b) combustion tube and CuO.'
GAUZE . i
/ NITROGEN- -
(c) Schiff’s nitromete.r m- \\
MD. .
(a) Carbon dioxide
n

or
generator (not shown in the Fig.
12.36). CO2 needed for the
ou PURE CO2
FURNACE

purpose is produced by heating

ksf
either sodium bicarbonate or
magnesium carbonate. The gas is MERCURY SEAL
oo
perfectly dried by bubbling SCHIFF’S NITROMETER
through cone. H2SO4 before
Y
passing it through the combustion Apparatus for the estimation of Nitrogen by Dumas Method
B

tube.

(b) Combustion tube. It is a hard glass test tube about 90 cm long and about 2 cm in diameter. It is
re

packed with (/) a roll of oxidised copper gauze which prevents backward diffusion of gases produced during
combustion, {if) an accurately weighed quantity of the substance mixed with excess of cupric oxide,
(Hi) coarse CuO that fdls nearly half of the combustion tube and (iv) a reduced copper gauze which helps to
oYu
ad

reduce any oxides of nitrogen formed during combustion back to nitrogen gas.
(c) Schiff’s nitrometer. It consists of a long graduated tube having a resevoir and a tap at the upper end.
It contains about 40% KOH solution. It also has a mercury seal at the bottom which prevents KOH solution
d

from being sucked back into the combustion tube.


Both CO2 and H2O produced during combustion are absorbed by KOH solution white N2 is collected
in
Re

over it. The volume of N2 is measured after careful levelling (by making the level of KOH in the nitrometer
tube and reservoir the same).
F

Modifications under specific conditions. If the organic compound, in addition to C, H, O and N also
contains S or halogens, the layer of coarse. CuO in the combustion tube is replaced by fused PbCr04 as
discussed under Liebig’s method.
Calculations :

Suppose the mass of the substance taken = w g


Volume of N2 collected = V mL
Atmospheric pressure (read from barometer) = P mm
Room temperature = fC
Aqueous tension at t®C (from tables) = p mm
.-. Pressure of the dry N2 gas = (P—p) mm
ORGANIC CHEMISTRY-SOME BASIC PRINCIPLES AND TECHNIQUES 12/117

Step 1. Conversion of the volume at experimental conditions to the volume at STR


Experimental values At STR

P| = (P-/?) nun ?2 - 760 mm


Vi = VmL V2 = ?
Ti = (273+0 K T2 = 273 K
P1V1P2V2
Applying gas equation, the value of V2 can be calculated.
T2 T2

low
Step 2. Conversion of volume at STR or NTR into mass. According to the definition of Gram Molecular
Volume (GMV),
22400 mL of N2 gas at STP weigh equal to its molecular weight expressed in grams, i.e., = 28 g
28
V2 mL of N2 gas at STP will weigh = xV2g
22400
Step 3. Calculation of% age ofN.

ee
rF
This is the mass of nitrogen present in w g of the substance.

Fr
% age of Nitrogen in the compound =
28
X
V2 xlOO
22400 w

for
Percentage of Nitrogen =
28
X
Volume of N2 at STP XlOO =
Mass of N2 at STP XlOO
22400 Mass of substance taken Mass of substance taken
u
SampIt^iProblern 12,21
ks
In Dumas method for estimation of nitrogen, 0.30 g of an organic
compound gave 50 mL of nitrogen collected at 300 K and 715 mm pressure. Calculate the percentage
Yo
oo
composition of nitrogen in the compound (Aqueous tension at 300 K is 15 mm).
(NCERT Solved Example)
eB

Solution. Here, mass of the substance taken = 0.30 g


Volume of nitrogen collected = 50 mL, Atmospheric pressure = 715 mm Hg
Room temperature = 300 K
r

Aqueous tension at 300 K = 15 mm Actual pressure of the gas (dry gas) = 715 - 15 = 700 mm Hg
ou
ad

Step 1. To convert the volume at experimental conditions to volume at STP.


Experimental values At STP
Y

P| = 700 mm ?2 - 760 mm
V| = 50 mL V2=?
Re
nd

Tj = 300 K T2 = 273 K
Substituting these values in the gas equation,
Fi

^2% ^2^2 700 mmx50 mL _ 760 mmx V2 mL 273x700x50


T. T2
, we get
300 K 273 K or V2 = 300x760
= 41-9 mL

Step 2. To convert the volume at STP into mass.


According to the definition of GMV, 22400 mL of nitrogen at STP weigh = 28 g
28x41-9
.●. 41-9 mL of nitrogen at STP will weigh = g
22400
Step 3. To calculate the percentage of nitrogen.

Percentage of nitrogen =
Mass of N2 at STP XlOO =
28x41-9x100
= 17-46
Mass of the substance taken 22400 X 0-3
12/118 ‘Pxadeefi. ^ New Course Chemistry (XI)

1. 0-2046 g of an organic compound gave 30-4 mL of moist nitrogen measu-od at 288 K and 732-7 mm
pressure. Calculate the percentage of nitrogen in the substance (Aqueous tension at 288 K is 12-7 mm).
2. 0-27 g of an organic compound gave on combustion 0-396 g of CO2 and 0-216 g of H^O. 0-36 g of the same
substance gave 48-88 niL of N, at 290 K and 740 mm pressure. Calculate the percentage composition of the
compound.
3. A sample of 0-125 g of an organic compound when analy.sed by Duma’s method yields 22-78 mL of nitrogen
gas collected over KOH solution at 280 K and 759 mm Hg. The percentage of nitrogen in the given organic

w
compound i.s (Nearest integer). Given the vapour pressure of water of 280 K is 14-2 mm Hg.
b.k. = 0-082 L atm K'' mol"’ (JEE Main 2022)

F lo
ANSWERS

1. 16-68% 2. C = 40%, H = 8-89%, N = 15-56% and 0 = 35-55%


Experimental values At STP

ee
3. P, = 759 - 14-2 = 744-8 mm/Hg ?2 = 760 mm Hg.
T2 = 273 K, V2 = ?

Fr
Vi = 22-78 mL, T, = 280 K
744-8 X 22-78 273
= 21-767 mL
N2 = X

for
280 760
28
Mass of 21 -767 mL of N2 at STP = — X 21-767 g
22400
our
28 21-767
%N = X xlOO =21-767 = 22 (nearest integer)
oks
22400 0-125
o

FIGURE 12.37
2. Kjeldahl’s method.
Y
eB

This is a more convenient and KJELDAHL'S


KJELDAHLIZED NH3-*-S~^EAM
SOU-^NaOH
simpler method for the FLASK'
estimation of nitrogen and is ORGANIC \
KJELDAHL'S
TRAP
largely used for the estimation
our
ad

COMPOUND +

of nitrogen in fertilizers, food CONC. H2SO4


■♦●K2S04 1
stuffs, drugs, etc. The method g,_^0 SINK
is, however, not applicable to
CUSO4 j
\

compounds containing >


I
dY

nitrogen in the ring (e.g.,


Re

pyridine, quinoline, etc.) and TO—►


compounds containing nitrogen TAP
Fin

KNOWN

directly linked to oxygen atom /VOLUME


OF
{e.g. NO2) or another nitrogen STANDARD
atom i.e. azo (-N = N-) V ACID

compounds.
The apparatus used for O Kjeldahl's distillation
the estimation of nitrogen by O Digestion (healing in Kjeldahl’s flask)
Kjeldahl's method is shown in Apparatus for estimation of nitrogen by Kjeldahl's method
Fig. 12.37.
Principle. A known mass of the organic compound is digested (heated) with cone. H2SO4 in presence of
potassium sulphate and a little copper sulphate or mercury in a long-necked flask called Kjeldahl's flask (Fig.
12.37a). Potas.sium sulphate raises the boiling point of H2SO4 and thus ensures complete reaction while
copper sulphate or mercury catalyses the reaction.
ORGANIC CHEMISTRY-SOME BASIC PRINCIPLES AND TECHNIQUES 12/119

As a result of digestion, the nitrogen present in the organic compound is quantitatively converted into
ammonium sulphate. Ammonium sulphate thus obtained is boiled with excess of NaOH solution (Fig. 12.37b)
to liberate ammonia gas which is then absorbed in a known excess of a standard acid such as H2SO4 or HCl.

N (from compound) + Cone. H2SO4 (NH4)2S04


A

(NH4)2S04 + 2NaOH > Na2S04 + 2H2O + 2NH3

w
2NH3 + H2SO4 ^ (NH4) SO4
The volume of the acid left unused is found by titration against a standard alkali solution.
2NaOH + H2SO4 > Na2S04 + 2H2O
From this, the volume of the acid used up (or the volume of ammonia evolved) and hence the percentage

o
of nitrogen in the organic compound can be calculated.
Calculations :

e
Let the mass of organic substance taken = w g

re
rFl
Volume of H2SO4 taken = V mL of Mj molarity

F
and volume of NaOH used for titration of excess of H2SO4 = t; mL of M | molarity
Applying molarity equation,
where and are the acidity, molarity and volume of NaOH used while n^, and are the

r
basicity, molarity and volume of acid used. ^
ou
sfo
1 xuxM, (Na0H) = 2xM,xV^(H2S04) or Vq = vH (basicity of H2SO4, i.e., = 2)
i.e.,
Volume of H2SO4 left unused = u/2 mL of Mj molarity.
Volume of H2SO4 used up by NH3 = (V - u/2) mL of M, H2SO4 k
But
(V - vH) mL of M IH2SO4 = 2 (V - vH) mL of M, NH3
oo
(1 mole of H2SO4 neutralises two moles of NH3)
Y
Now 1(X)0 mL of 1 M NH3 solution contain NH3 = 17gorN=I4g
eB

14
2 (V—u/2) mL of M] NH3 will contain N = 1000 x2(V-u/2)xM,g
This is the mass of nitrogen present in w g of the substance
ur

%age N =
14xM, x2(V-v/2) 100 l-4xMj x2(V-u/2)
ad

iooo ^
Yo

w w

or
% age N =
1 -4 X Molarity of H2SO4 x twice the vol of H2SO^used
Mass of the substance taken
d

1 -4 X Molarity of acid x Basicity of acid x Vol. of the acid used


In general, % age of Nitrogen =
Re
in

Mass of the substance taken


If however, the acid used is monobasic {i.e. basicity = 1), the above equation reduces to
F

1 -4 x Molarity of acid x Vol, of acid used


% age of Nitrogen =
Mass of the substance taken
_ 14 X Normality of acid x Vol, of acid used
Mass of the substance taken (●.' for monobasic acids, molarity = normality)

Sample Problem 12.22During estimation of nitrogen present in an organic compound by


Kjeldhal’s method, the ammonia evolved from 0-5 g of the compound neutralized 10 mL of 1 M H2SO4.
Find the percentage of nitrogen in the compound. (NCERT Solved Example)
Solution. Volume of H2SO4 used = 10 mL of 1 M H2SO4
Now
H2SO4 + 2NH3 ^ (NH4)2S04
Now I mole of H2SO4 reacts with 2 moles of NH3. .-. 10 mL of 1 M H2SO4 s 20 mL of 1 M NH3
12/120 7>>uteiecfi^'A New Course Chemistry (XI)CEIMD
14
x20
Now 1000 mL of I M NH3 contains N = 14 g 20 mL of 1 M NH3 will contain N - 1000

But this amount of nitrogen is present in 0-5 g of the organic compound.


14x20 IW
Percentage of nitrogen - X = 56*0
1000 0-5

l-4xVol. of acid used x Basicity of acid X Molarity of acid 1-4x10x2x1


Ahematively, % age of N - 0-5

ow
Mass of the substance taken
= 56-0

I. 0.2 g of an organic compound on Kjeldahl’s analysis gave enough ammonia to just neutralize 20 mL of
0,1 N H2SO4. Calculate the percentage of nitrogen in the compound.

e
Fl
0.4422 g of an organic compound was Kjeldahlised and ammonia evolved was absorbed in 50 mL of semi-

re
2.
molar (0-5 M) H2SO4. The residual acid required 131 mL of 0.25 M NaOH. Determine the percentage of

F
nitrogen in the compound.
3. Ammonia obtained from 0.4 g of an organic substance by Kjeldahl’s method was absorbed in 30 mL of
0-25 M H-,S04. The excess of the acid was neutralized by the addition of 30 mL of 0-2 M NaOH. Calculate
ur
r
the percentage of nitrogen in the substance.

fo
4. While estimating the nitrogen in an organic compound by Kjeldahl’s method, the ammonia evolved from
0-25 g of the compound neutralized 2-5 mL of 2 M H2SO4. The percentage of nitrogen present in organic
compound is. ks (JEE Main 2022)
Yo
ANSWERS
oo
1. 14% 2. 54.61% 3. 31.5%.

4. Vol. of H2SO4 used = 2-5 mL of 2 M H2SO4 = 5 mL of 1 M H2SO4


B

Now, H2SO4 + 2 NH3 > (NH4>2S04


Now 1 mole of H,S04 reacts with 2 moles of NH3.
re

But 5mLoflMH2SO4=10mLoflMNH3 14
xlO
.'. 10 mL of 1 M NH3 will contain N =
u

Now 1000 mL of 1 M NH3 contains N = 14 g 1000


ad

But this amount of N is present in 0-25 g of the organic compound.


Yo

14x10 100
%ofN = X = 56%
1000 0-25
d

l-4x2-5x2x2
Re

Alternatively, %N = = 56%
in

0-25

Alternatively, applying molarity equation,


F

1-4 X Volume x Basicity x Molarity of acid used 1-4 x 2-5 x 2 x 2 = 56%


%N =
Mass of the substance 0-25

12.183. Estimation of Halogens {Carius method)


Principle. A known mass of the organic substance is heated with fuming nitric acid and a few crystals
of silver nitrate in a sealed hard glass tube (about 2 cm wide 50 cm long) called Carius tube in a furnace,
(Fig. 12.38).
Under the conditions, carbon and hydrogen are oxidised to carbon dioxide and water respectively while
halogen is converted into silver halide. The precipitates of silver halide are filtered, washed, dried and weighed.
Knowing the mass of the substance taken and the mass of the precipitate formed, the percentage of halogen is
calculated as follows:
ORGANIC CHEMISTRY-SOME BASIC PRINCIPLES AND TECHNIQUES 12/121

Calculations: Let the mass of the substance taken FIGURE 12.38

= M’ g FUMING HNO3+ 0.5g AgNOg


and the mass of the silver halide (AgX) formed
= xg

Now. 1 mole of AgX contains 1 gram atom of

w
X(X = C1, Br or I) SMALL TUBE
CONTAINING FURNACE
i.e. (108 + At. mass of X) g of AgX contain X = A KNOWN
(At. mass of X) g WEIGHT OF
CARiUS TUBE MADE IRON
ORGANIC TUBE
X g of AgX contain X COMPOUND
UP OF HARD GLASS

o
At. mass of X A Carius tube for estimation of Halogens

e
108 +At. mass of X

re
This is the mass of the halogen (X) present in w g of the substance.

Frl
At. mass of X 100

F
% age of halogen in the compound = XJCX
108+ At. mass of X w

Thus, (a) For chlorine, as the atomic mass of chlorine = 35-5 u

or
ou
% age of chlorine in the compound =
35-5 X

108 + 35-5
100

w
35-5 X

143-5
X
100

>f

kfs
35-5 X Mass of AgCl formed
or
% age of chlorine = X xlOO
143-5 Mass of substance taken
oo
(b) For bromine, as atomic mass of bromine = 80 u
Y
80x 100 80x 100
B

% age of bromine in the compound = X X


108 + 80 w 188 w

80 Mass of AgBr formed


re

or
% age of bromine = X XlOO
188 Mass of substance taken
oYu

(c) For iodine, as atomic mass of iodine = 127 u


ad

127x 100 127x 100


% age of iodine in the compound = X X
108+127 M’ 235 H'
d

or % age of iodine = 127^ Mass of Agl formed XlOO


in
Re

235 Mass of substance taken

Sample Problem
mm In Carius method of estimation of halogen, 0*15 g of an organic compound
F

gave 0’12 g of AgBr. Find out the percentage of bromine in the compound. (NCERT Solved Example)
Solution. Here, the mass of the substance taken =0-15 g
Mass of AgBr formed = 0-12 g
Now 1 mole of AgBr = 1 g atom of Br or 188 (108 + 88) g of AgBr = 80 g of Br
80
.●. 188 of AgBr contain bromine = 80 g .●. 0-12 g of AgBr will contain bromine = x0-12g
188

But this much amount of bromine is present in 0-15 g of the organic compound.
80 Q-12
Percentage of bromine = XlOO =34-04
188 0-15
12/122 “Pn-tucUe^’^. New Course Chemistry (XI)EEIHD

mam p>P<ASTni^^
1. 0-189 g of an organic substance gave in a Carius determination 0-287 g of silver chloride. What is the
percentage of chlorine in the given compound ?
2. 0-301 g of an organic compound gave 0-282 g of silver bromide by a halogen estimation method. Find the
percentage of bromine in the compound.
3. 0-25 g of an organic compoundcontainingchlorine gave 0-40 g of silver chloride in Carius estimation. The
percentage of chlorine present in the compound (nearest integer). (JEE Main 2022)

low
4. In the estimation of bromine, 0-5 g of an organic compound gave 0-40 g of silver bromide. The percentage
of bromine in the given compound is (nearest integer). (JEE Main 2022)
ANSWERS

1. 37-57% 2. 39-87%

35-5 0-40 80 0-40


3. %C1 = xlOO =39-58 - 40 4. %Br = xl00=34-04=^34
188^0-50

ee
143-5 0-25

F
Fr
12.18.4. Estimation of Sulphur (Carius method)
Principle. A known mass of the substance is heated with sodium peroxide or fuming nitric acid in a
sealed tube (Carius tube). Carbon and hydrogen are oxidised to CO2 and H2O respectively while sulphur

for
present in the compound is oxidised to sulphuric acid which is then precipitated as barium sulphate by adding
ur
excess of barium chloride solution.
A A

C + 20 (from HNO3) ^ CO2 ; 2H + O (from HNO3) ^ H2O


ks
A
Yo
S + H2O + 30 (from HNO3) H2SO4 : H2SO4 + BaCl2 BaS04 + 2HC1
oo
(White ppt.)
eB

The ppt. of BaS04 is filtered, washed, dried and weighed. Knowing the mass of the substance taken and
the mass of BaS04 ppt. formed, the percentage of sulphur can be calculated.
Calculations : Let the mass of the substance taken = w g
r

Mass of BaS04 ppt. formed = x g


ou
ad

Now, 1 mole of BaS04 contains 1 gram atom of S


i.e., (137 + 32 -f 64) = 233 g of BaS04 contain sulphur = 32 g
Y

32
Jt g of BaS04 will contain sulphur = 233
xxg
Re
nd

This is the mass of sulphur present in vv g of the substance.


32
Fi

% age of sulphur present in the compound = X -XlOO


233 w

32 Mass of BaS04formed XlOO


% age of sulphur = X
233 Mass of substance taken

SUPPLEMENT YOUR
KNOWLEDGE FOR COMPETITIONS

Messenger’s method is used for estimation of sulphur. In this method, the organic compound is heated
with alkaline KMn04 solution when sulphur present in the organic compound is oxidised to K2SO4 which
is then estimated as BaS04.
ORGANIC CHEMISTRY-SOME BA . .t PRINCIPLES AND TECHNIQUES 12/123

Sample Problem fm In sulphur estimation. 0-157 g of an organic compound gave 0*4813 g of


barium sulphate. What is the percentage of sulphur in the compound ? {NCERT Solved Example)
Solution. Here, mass of the substance taken = 0-157 g
Mass of BaS04 ppt. fonned = 0-4813 g
Now, 1 mole of BaS04 = 1 g atom of S
or
(137 + 32 -t- 64 ) = 233 g of BaS04 = 32 g of S
i.e. 233 g of BaS04 contain sulphur = 32 g
32
0-4813 g of BaS04 contain sulphur = x0-4813g
233

32 0-4813

w
% age of sulphur in the compound = 233
X
0-157
X100= 42*10

F lo
1. 0.2175 g of the substance gave 0.5825 g of BaS04 by Carius method. Calculate the percentage of sulphur.
2. 0.16 g of an organic substance was heated in Carius tube and the sulphuric acid formed was precipitated

e
as

Fre
BaS04 with BaCl2. The weight of the dry BaS04 wa.s 0.35 g. Find the percentage of sulphur.
3. 0-2595 g of an organic substance, when treated by Carius method, gave 0-25 g of BaS04. Calculate the
percentage of sulphur in the compound.

for
ANSWERS

1. 36.78% 2. 30.04% 3. 13-23%


r
You
12.18.5. Estimation of Phosphorus
oks

Principle. A known mass of the organic compound is heated with fuming nitric acid in a sealed tube
eBo

{Carius tube). Under these conditions, carbon and hydrogen are oxidised to CO-> and H^O respectively while
phosphorus present in the organic compound is oxidised to phosphoric acid which is precipitated as ammonium
phosphomolybdate by heating it with cone. HNO3 and then adding ammonium molybdate
C + 20 (from HNO3) CO2 ; 2H + O (from HNO3) H2O
our
ad

2P -f 50 (from HNO3) P20g ; P20g + SH^O 2H3PO4


Phosphoric acid

H3PO4 -H 12 (NH4)2Mo04 + 21 HNO3 ^ (NH4)3P04.12 M0O3 21 NH4NO3 + 12 H2O


dY
Re

Ammonium Amm. phosphomolybdate


molybdate {yellow ppt.)
Fin

The precipitates of amm. phosphomolybdate thus formed are filtered, washed, dried and weighed.
Alternatively phosphoric acid is precipitated as magnesium ammonium phosphate by adding magnesia
mixture (a solution containing magnesium chloride, ammonium chloride and ammonia).
A
MgCl2 + NH4CI -f H3PO4 ■>
MgNH4P04 + 3HC1
Mag. amm. phosphate
{while ppt.)
The precipitates of magnesium ammonium phosphate are filtered, washed, dried and then ignited.

2MgNH4P04 — ■> ^82^2^7 + 2NH3 + H2O


Mag. pyrophosphate
12/124 ^n^x<Cee^ ]S Course Chemistry (XI) LV»ini

The magnesium pyrophosphate thus formed is weighed. Knowing the mass of the organic compound
taken and the mass of ammonium phosphomolybdate or magnesium pyrophosphate formed, the percentage
of phosphorus can be easily calculated.
Calculations. If phosphorus is estimated as (NH4)3p04.12 M0O3. Let the mass of the organic compound
taken = VI’ g
Mass of ammonium phosphomolybdate formed = a' g
Now I mole of (NH4)3p04.12 M0O3 contains one gram qlom of P i.e.
3x(I4 + 4) + 31 +4x 16+ 12 (96* + 3 x 16)= 1877 g of (NH4)3P04.12 M0O3 contain P = 31 g
31x A

w
.r g of (NH4)3 PO4.I2 M0O3 will contain P = 1877 '
This is the mass of P present in vv g of the compound

F lo
31 31 Mass of amm. phosphomolybdate
% age of P = X
~xi00 or %ageofP = X xiOO
1877 M’ - 1877 Mass of substance taken

Alternatively if phosphorus is estimated as Mg2P207.

ee
Mass of Mg2p^07 formed = a g

Fr
Now. I mole of Mg2P'>Oy contains two gram atoms of P
i.e. (24 x2 + 31 x2 + 16x 7) = 222 g of Mg-,P-,07 contain phosphorus = 62 g

for
62 a
A g of Mg-,P207 will contain phosphorus = g
ur
This is the mass of phosphorus present in w g of the compound.
62
s
i.e., % age of phosphorus present in the compound = X -xlOO
ook
222 w
Yo
62 Mass of Mg2P207 formed X 100
% age of phosphorus = X
eB

222 Mass of substance taken

Sample Problem 12.25 0‘12 g of an organic compound containing phosphorus gave 0*22 g of
usual analysis. Calculate the percentage of phosphorus in the compound.
r
ad

Solution. Here, the mass of the compound taken = 012


ou

Mass of Mg,P^07 formed = 0-22 g


Now 1 mole of Mg2p207 = 2 g atoms of P or (2 x 24 + 2 x 31 + 16 x 7) = 222 g of Mg2P207 = 62 g of P
Y

I.e., 222 g of Mg2P')07 contain phosphorus = 62 g


Re

62
nd

0-22 g of Mg,P207 will contain phosphorus = x0-22g


222
Fi

But this is the amount of phosphorus present in 012 g of the organic compound.
62 0-22
.●. Percentage of phosphorus = xlOO =51-20
222 012

1. 0.092 g of an organic compound containing phosphorus gave 0-111 g of Mg2P207 by usual analysis.
Calculate the percentage of phosphorus in the organic compound.

*Atomic mass of Mo = 96 u
ORGANIC CHEMISTRY-SOME BASIC PRINCIPLES AND TECHNIQUES 12/125

2. 0.40 g of an organic compound containing phosphorus gave 0.555 g of Mg2P20y by usual analysis. Calculate

w
the percentage of phosphorus in the organic compound.
VVERS

1. 33.7% 2. 38.75%

12.18.6. Estimation of Oxygen


The percentage of oxygen in an organic compound is usually calculated by the difference method. For

ro
this, the percentages of all other elements present in the organic compound are added and the sum is subtracted
from 100.

r
Percentage of Oxygen = 100—( Sum of the percentages of all other elements)

F
oF
However, oxygen can be directly estimated as follows :

ul
A known mass of an organic compound is decomposed by heating in a stream of nitrogen gas. The
mixture of gaseous products including oxygen thus formed is passed over red hot coke at 1373 K. When all
the oxygen is converted into carbon monoxide. This mixture is then passed through warm iodine pentoxide

sr
(I2O5) when carbon monoxide is oxidised to carbon dioxide liberating iodine. The various reactions involved

ko
are:

heal
Compound ■) Ot + other gaseous products

of
I373K
2C + 02 ^ 2 CO] X 5
I2O5 + 5CO ■^l2 + 5C02]x2 o
Y
10 C + 5 O2 + 2 I2O5 ^ IOCO2 + 2I2
B
or
5 02= 10CO2 = 2l2
Y

or
O = CO2 = 1/5 I2 ...(/)
er

By knowing the mass of iodine liberated or CO2 formed, the percentage of oxygen in the organic
compound can be calculated easily.
u

Calculations. Let the mass of the organic compound taken = w g and mass of COo formed = x g
From Eq. (i), it is evident that 1 g atom of O present in the organic compound produces one mole of
d
o
ad

CO2, i.e., 44 g of CO2 is formed from O = 16 g


16
in

.●. X g of CO2 will be formed from O = — x ,v g


44

This is the mass of oxygen present in w g of the organic compound.


Re
F

16 X
Percentage of oxygen = —x
44
—xlUU
w

16 Mass of CO., formed


or Percentage of oxygen = X xIOO
44 Mass of substance taken

5x16
X
Mass of 1^ formed xlOO
2x127 Mass of substance taken

These days, the elements carbon, hydrogen and nitrogen present in an organic compound are determined
by an automatic analyser called CHN elemental analyzer. This analyzer requires 1-3 mg of the compound
and displays the percentages of C, H and N on a screen within a few minutes. These results are generally
accurate within ± 0-03% error. A detailed discussion of such methods is beyond the scope of this book.
12/126 New Course Chemistry (XI)SSSIHD

12.19. SOME ADDITIONAL USEFUL INFORMATION ABOUT


ORGANIC CHEMISl RY - SOME BASIC PRINCIPLES AND TECHNIQUES
1. BridgoI.ead carbons and hydrogens. In bicyclic s’. siems, the carbon atoms joining the two rings are
called bridgehead carbons and the hydrogens atoms attached to these carbons are called bridgeheadhydiogens.
For example.
H ^— Bridgehead hydrogen .Third
bridge
^— Bridgehead carbon —
Frist Second bridge — Frist
bridge bridge
— Bridgehead carbon —

w
H ^— [Jridgeiiead hydrogen
Tliird bridge
Bicycio[lTO]butane Bicyclo[2-2-l]heptane

F lo
Other examples of compounds containing bridgehead carbon and hydrogen atoms are :
11 H H

ee
Fr
H H H

Bicyclo[2-2'0]hexane Bicyclo[3-2'0]hcptane Bicyclo[4-40]decane

for
2. Bredt’s rule and Tautomerisni. In small bicyclic ring systems, a double bond at the bridgehead
carbon cannot be formed. This is called Bredt’s rule. For example, norbomene (X) is unstable.
ur oks
Yo
Norbomene (X)
o

The reason being that the carbon atom forming the double bond has to be 5/7^-hybridized. But due to
eB

angle strain (also called the backstrain) it cannot achieve planar geometry, i.e., cannot assume valency angles
of 120° and hence a double bond is not possible at a bridgehead carbon in small bicyclic ring systems.
This rule is not always applicable. It applies only to those bicyclic molecules which have at least one
our

carbon atom in each of the three bridges. For example, it is applicable to norbomene (X) which has 2, 2 and
ad

1 carbon atoms in the three bridges but is not applicable to 9-decalin (Y) because two bridges have 4 carbon
atoms each while the third has no carbon atom

8 I
dY

7 2
Re

6 3
10
Fin

5 4

Norborhenc (X) 9-Decalin (Y)

Since according to Bredt’s rule, a double bond at the bridgehead carbon is not stable, therefore, bridgehead
a-hydrogens of bicyclic ketones cannot participate in keto-enol tautomerism. This is illustrated by the following
examples.

HO-

(1)
ORGANIC CHEMISTRY’SOME BASIC PRINCIPLES AND TECHNIQUES 12/127

Tautomerism is not possible in bicyclic ketone (I) since the d.h. at the bridgehead carbon is not stable

●«—A—

ow
Tautomerism not possible in bicyclic ketone (II) since the d.b. at the bridgehead carbon is not stable

e
>

re
rFl
F
Tautomerism is possible in bicyclic ketone (III) because a-hydrogens which do not lie on the bridgehead
carbon atom can participate in enol formation.

r
ou
fo
3. Geometry of bridgehead carbocations and free radicals. In small bicyclic ring systems, the
carbocations at the bridgehead carbon are unstable because they cannot achieve planar geometry (i.e., valency

ks
angles of 120®) due to angle strain or the backstrain. For example, carbocation (IV) is unstable.
oo
Y
eB

(V)

In contrast, bridgehead free radical (V) is pyramidal because due to angle strain it cannot assume planar
ur

geometry.

4. Instability of bridgehead carbanions.


ad
Yo

Unlike free radicals for which energy difference between planar and pyramidal geometry is very small,
the energy difference between a planar and a pyramidal carbanion is quite large, therefore, stabilization by
resonance favours planar structure. If, due to some reasons, resonance is inhibited, the carbanion is not
d

formed. For example, incase of bridgehead carbanion (VI), resonance is inhibited due to angle strain, therefore,
Re
in

it is unstable and hence not formed.


F
12/128 New Course Chemistry (XI)EEIHD

12.20. TYPICAL PROBLEMS

F. 1. Complete combustion of a hydrocarbon gives 0’66 g of CO2 and 0-36 g of H2O. Find out the
empirical formula of the compound.
Sol. Let the mass of ihe substance taken = w g
12 0-66 2 0-36 4
%C = —X x)00 = — and %H = — X xl00 = -
44 w M’ 18 vv IV

ow
Ratio of atoms. C:H=—x— x- = 3;8
w 12 vv 1

Hence, E.F. = C3Hg


P. 2. An oxygen containing organic compound was found to contain 53% carbon and 13% hydrogen.
Its vapour density is 23. The compound reacts with sodium metal to liberate hydrogen. Identify
the functional isomer of this compound.

e
Sol. % 0 = 100-(53+ 13) = 34

Fl
re
53 13 34
Ratio of atoms, C ; H : 0 = — ● — =4-4: 13;2-125 = 2:6: 1
12 1 16

F
E.F. = C2HgO and E.F. wt = 2 x 12 + 6 x 1 + 1 x 16 = 46
Mol. wt. = 2 X V.D. = 2 X 23 = 46
ur
r
46
n = Mol. wt./E.F. wt. = = 1

fo
46

Thus, M.F. = 1 X E.F. = CjH^O


ks
Since the given organic compound reacts with sodium to liberate H-, gas, therefore, it must be ethanol.
Yo
2 CH3CH2OH + 2 Na > 2 CH3CH20Na + H2
oo
Ethanol

The functional isomer of ethanol is methoxymethane, i.e., CH3OCH3.


eB
ur

I. Hybridization, Classification and Nomenclature


ad

1. Vital force theory. According to this theory, organic compoundsare producedonly under the influence of
Yo

some mysterious force existing in the living organi.sms. This theory was propounded by Berzelius.
2. Wohler’s synthesis. When ammonium cyanate (NH4CNO) or a mixture of NH4CI + KCNO is heated, an
organic compound called urea (NH2CONH2) is formed. It gave a death blow to vital force theory.
d

3. Acetic acid was the first organic compound which was synthesized from its elements, i.e., C and H by
Re

Kolbe in 1845.
in

4. Huge number. Due to the properly of catenation and phenomenon of isomerism carbon forms a huge
number of compounds.
F

5. Characteristics of organic compounds. Some important characteristics of organic compounds are :


(0 organic compounds are covalent while inorganic compounds are electrovalent, (//) organic compounds
being non-polar are usually insoluble in water while inorganic compounds being polar are soluble in
water, (Hi) The reactions of organic compounds are molecular in nature while those of inorganic compounds
are ionic in nature.

6. Cyclic compounds whose rings are made up of only one kind of atoms, i.e., carbon atoms are called
homocyclic or carbocyclic compounds. Aliphatic cyclic compounds are called alicyclic compounds,
e.g., cyclopropane, cyclobulanc, cyclopentane, etc.
7. Organic compounds containing one or more fused or isolated benzene rings and their functionalized
derivativesare called benzenoidsor aromaticcompound.s,e.g., benzene, toluene, naphthalene, anthracene.
etc.
ORGANIC CHEMISTRY-SOME BASIC PRINCIPLES AND TECHNIQUES 12/129

8. Cyclic compounds containing one or more heteroatoms (usually 0, N, S, etc.) are called heterocyclic
compounds.
9. Aliphatic cyclic compounds containing one or more heteroatoms in their rings are called alicyclic
heterocyclic compounds, e.g., ethylene oxide, tetrahydrofuran (THF), pyrrolidine, piperidine, etc.
10. Aromatic compounds containing one or more heteroatoms in their molecules are called aromatic
heterocyclic compounds, e.g., furan, pyrrole, thiophene, etc.
11. The atom or group of atoms present in a molecule which largely determines its chemical properties is
called a functional group, i.e., NH2, OH, NO2, CHO, COOH, etc.
12. A homologousseries is a family of structurallysimilar organic compounds, all members of which ontain

w
the same functional group and whose consecutive members differ by a CH^ group. They are prepared by
the same general methods of preparation and have almost similar chemical properties. For example,
methanol, ethanol, propanol, etc. are the members of a homologous series called alcohols.
13. A lUPAC name consists of three parts : word root, suffix and preifx.

F lo
14. Word root denotes the number of carbon atoms present in the longest continuous chain of carbon atoms
containing the functional group.
15. Suffixes are of two types : (/) Primary suffix denotes whether the chain is saturated or unsaturated. If

ee
saturated, the suffix ane is used and if unsaturated, the suffix ene for double bond and yne for triple bond
is used.

Fr
(ii) A secondary suffix denotes the functional group present in the organic compound.
16. Prefixes are also of two types
(i) A primary prefix cyclo is used before the root word to distinguish cyclic from acyclic compounds.

for
(h) a secondary prefix is used to indicate the nature of secondary functional groups, i.e., functional
our
groups other than the principal functional group.
The complete lUPAC name of the organic compound consists of five parts in the order mentioned below :
s
Secondary + Primary + Word + Primary + Secondary
ook
prefix prefix root sujfix suffix
{s.p.) (P-P-) iyy..r) {p.s.) (s.s.)
Y
eB

For example, the compound Cl OH is named as :

4-chloro + cyclo + hex + an(e) + l-ol = 4-ChIorocyclohexan-l-ol


s.p. p.p. w.r. p.s. s.s.
r
ou
ad

It may be noted that locants for suffixes and prefixes are placed immediately before them.
17. The following order of preferences is used while selecting the principal functional group in polyfunciional
compounds. Amine salts > carboxylic acids > sulphonic acids > anhydrides > e.sters > acid chlorides >
Y

acid amides > nitriles > aldehydes > ketones > alcohols > phenols > thiols > amines.
All the remaining functional groups such as halo, nitro, nitroso, amino, alkoxy, alkyl, phenyl, etc. are
Re

treated as substituent groups.


nd

18. While numbering the carbon atoms of an organic compound the lowest set of locants for substituents is
always preferred. For example, (/) the set of locants (1,4.5) is preferred over the set (2, 3, 5) even though
Fi

both have the same sum of locants, i.e., 10.


(ii) The set of locants (2, 7, 8) is preferred over the set of locants (3, 4, 9) since 2 comes before 3 even
though the sum of locants in the former case is 17 while in the latter case, it is 16.
19. When both double and triple bonds are present in a molecule, the numbering of the parent chain is always
done from that end which is nearer to the double or the triple bond, i.e.,

1 2 3 4 5 67 8

CH3CH=CHCH2C^CH2CH3 (oct-2-en-5-yne)
and the compound is always named as a derivative of alkyne rather than alkene.
20. If. however, there is a choice in numbering, the double bond is always given preference, i.e..
5 4 3 2 1

HCsC —CH2 —CH =CH2 (pent-l-en-4-yne).


12/130 "P,neicUefi-'ti. New Course Chemistry (XI)E2SIHI]

21. When two or more alkyl substituents or other substituents which are usually treated as substituent groups
are present in an alicyclic or aromatic ring, the substituent which comes first in the alphabetical order is
given the lowest locant provided it does not violate the lowest set of loctmts rule. For example,
H3C CH3 Cl
2

4,
^ON I CI-I3
CH2CH3
3-Ethyl-1,1 -dimethylcyclohexane 2-Chloro-l-melhyl-4-nitrobenzcne
[nnt l-ethyI-3,3-dimethylcyclohcxane) {jxoi 3-chloro-4-methyl-1 -nitrobenzene)

w
22. When an alicyclic or aromatic irng is directly attached to a carbon containing functional group, the carbon
atom of the functional group is not included in the name of the alicyclic or aromatic ring. In such cases
suffixes such as carbaldehyde for -CHO group, carbonitrile for -CN group, carboxylic acid for -COOH

F lo
group, etc. are attached to the name of the alicyclic or aromatic ring.
II. Isomerism

23. Compounds having the some molecular formula but different physical and chemical properties are called

ee
isomers and the phenomenon is called isomerism.
24. Structural isomerism. Compounds having the same molecular formula but different structures, i.e., different

Fr
arrangement of atoms within the molecule are called structural isomers. It is of the following six types.
(a) Compounds having the same molecular fomtula but different arrangement of carbon chains are called
chain or nuclear isomers and the phenomenon is called nuclear or chain isomerism. For example,

for
(/) butane and isobutane, (ii) pentane, isopentane and neopentane, (Hi) hexane, 2-methylpentane, 3-
ur
niethylpentane, 2, 3-dimethylbutane and 2, 2-dimethylbutane, etc. are chain isomers.
(b) Compounds having the same structure of the carbon chain but differing in position of multiple (double
or triple) bonds, functional groups or even the substituents in aromatic compounds are called position
s
ok
isomers and the phenomenon is called position isomerism. For example, {/) but-l-ene and but-2-ene
Yo
(ii) but-l-yne and biit-2-yne, (Hi) propan-l-ol and propan-2-ol, (iv) o-, m- and /?-xylenes, etc. are position
o
isomers,
eB

(c) Compounds having the same molecular formula but different functional groups are called functional
isomers and the phenomenon is called functional isomerism. For example, (/) ethanol and methoxymethane,
(ii) acetone, propionaldehyde, allyl alcohol and methyl vinyl ether, (Hi) 1, 3-butadiene, 1,2-butadiene and
1-butyne or 2-butyne, (iv) nitroethane and ethyl nitrite, (v) 1°, 2® and 3° amines, i.e., /t-propylamine,
r

cthylmethylamine and trimethylamine, (vi) cyanides and isocyanides, (vH) benzyl alcohol, anisole, o-cresol,
ou
ad

(wi-cresol or p-cresol) are all functional isomers.


(d) Compounds having the same molecular formula but different number of carbon atoms on either side of
the functional group (O, S or NH) are called metamers. For example, (i) diethyl ether and methyl n-propyl
Y

ether (or isopropyl methyl ether), (ii) diethyl thioether and methyl «-propyI thioether (or isopropyl methyl
thioether), (Hi) diethylamine and methyl n-propylamine (or isopropylmethylamine) are metamers. Please
Re
nd

note that 2-pentanone and 3-pentanone are position isomers and not metamers.
(e) Isomers obtained by 1, 3-migration of a proton from one polyvalent atom to the other within the same
Fi

molecule are called tautomers and the phenomenon is called tautomerism.


If one tai.tomer contains a keto group and the other the enol group, such a tautomersm is called keto-enol
tautomerism. For example, acetaldehyde and vinyl alcohol. The percentage of enol form is negligible in
simple aldehydes and ketones, i.e., acetone, acetaldehyde, etc. However, the percentage of enol form
increases, if it is stabilized by intramolecular H-bonding (chelation). For example, the percentage of enol
form in acetoacetic ester is 7% while that in acetylacetone is 76%.
Only those aldehydes and ketones which contain one or more a-hydrogens on a saturated carbon show
keto-enol tautomerism. For example, acetophenone, butanone, propionaldehyde show keto-enol tautomerism
while benzaldehyde and benzophenone do not.
Similarly, primary and secondary nitroalkanes show tautomerism but tertiary nitroalkanes and nitroarenes
do not.
ORGANIC CHEMISTRY-SOME BASIC PRINCIPLES AND TECHNIQUES 12/131

(f) Compounds having the same molecular formula but possessing open chain and cyclic structures are
called ring chain isomers and the phenomenon is called ring chain isomerism. For example, propene
and cyclopropane.
25. Stereoisomerism. Compounds which have same structures but differ in the relative arrangement of atoms
or groups in space arc called stereoisomers and the phenomenon is called stereoisomerism. These are of
three types :
(/) optical isomerism and (//) conformational isomerism (Hi) cis-trans isomerism or geometrical isomerism.
26. Some Basic Concepts about Optical Isomerism
(0 A plane polarized light is obtained by passing ordinary light through nicol (crystalline variety of
CaC03) prism. It has vibrations only in one plane,
(n) Substances which rotate the plane of polarized light are called optically active. Those which rotate the

w
plane of polarized light towards right are called dextrorotatory and are represented by the prefix (+)
while those which rotate the plane of polarized light towards left are called laevorotatory and are represented
by the prefix (-).

F lo
(///) A molecule is said to be chiral if it is not superimposable on its mirror image.
(iv) A carbon is said to be chiral or asymmetric if it is attached to four different atoms/groups,
(v) The necessary and sufficient condition for a molecule to be optically active is that it should be chiral
or dissymetric, i.e., it should be non-superimposable on its mirror image.

e
Fre
*

(v/) Molecules which contain one chiral carbon atom are always optically active. For example, CHBrClF,
* *

CH3CHOHCH2CH3, CH3CHOHCOOH, etc. But molecules which contain two or more chiral carbon

for
atoms may or may not be optically active.
(vii) Steroisomers which are mirror images of each other are called enantiomers. They have identical
physical {m.p., b.p. solubility, density, etc.) and chemical properties but rotate plane polarized light in
r
opposite directions but to the same extent.
You
(viii) A mixture of equal amounts of two enantiomers is called racemic mixture.
oks
It is always optically inactive due to external compensation because the rotation caused by the molecules
of one enantiomer is exactly cancelled by equal and opposite rotation caused by the same number of
o

molecules of the other enantiomer.


eB

(ix) Stereoisomers which are not mirror images of each other are called diastereomers. They have different
m.ps. b.ps. and solubilities and hence can be separated by fractional distillation and crystallization. They
may or may not be optically active. For example, cis- and trans- isomers of an alkene are diastereomers
our

which are optically inactive.


ad

(x) If a molecule has 'n' different chiral carbon atoms, the number of optical isomers = 2".
(xi) If a molecule contains two similar chiral carbon atoms, the number of stereoisomers is not four. For
example, tartaric acid (HOOCCHOH-CHOHCOOH) has three .stereoisomers, two of which are optically
active (enantiomers) while the third one is a meso stereoisomer. The meso isomer is optically inactive
dY

due to internal compensation because of the presence of a plane of symmetry which divides the molecule
Re

into two equal halves so that one half of the molecule is a mirror image of the other.
III. Fundamental Concepts of Organic Reaction Mechanism
Fin

27. Electrophiles are electron loving chemical species. They may be either positively charged or electrically
neutral chemical species.

(0 Positive electrophiles : H^, Cl'*^, Br^, I'*', N O2, N O , (carbocations), etc.

However, positively charged species such as H30'*’, , Na"^, Ca^"^, etc. cannot act as electrophiles
since they cannot accept a pair of electrons because all of them have already an octet of electrons in their
respective valence shells.

(k) Neutral electrophiles : R* (free radicals), : CR2 (carbenes), :NR (nitrenes), SO3, BF3, AICI3, FeCl3,
SnCl4, etc.
12/132 ^'uidee^'A New Course Chemistry (XI)BSXMI

28. Nucleophiles are nucleus loving chemical species. They may be either negatively charged or neutral chemical
species.
(/) Negative nucleophiles : H“ (hydride ion), Cl~, Br“, I“, R“ (carbanions), OH", OR", SR", NH17,
CN" RCOO", etc.

(u) Neutral nucleophiles ; H^O: , NH^, RNH^, R2NH , R3N, ROH , RSH, ROR, etc.
29. A covalent bond can break in two different ways :
(/) Homolytic fission or homolysis involves cleavage of the covalent bond in such a way that each atom
takes away one electron of the shared pair generating neutral chemical species called free radicals.
Homolytic fission occurs in non-polar bonds and is favoured by high temperature, ultraviolet radiations
and by presence of radical initiators.

w
(«) Heterolytic fission or heterolysis involves cleavage of the covalent bond in such a way that both the
electrons of the covalent bond (i.e., shared pair) are taken away by one of the atoms (/.e., more electronegative

F lo
atom) generating charged species, i.e., cations and anions. It usually occurs in polar covalent bonds and is
favoured by polar solvents.
30. Inductive effect involves displacement of CT-electrons along a saturated carbon chain whenever an electron-
withdrawing or electron donating group is present at the end of the carbon chain. It is a permanent effect
and decreases steadily with increasing distance from the electron withdrawing or the electron donating

ree
group and dies down after three carbon atoms. It is of two types.
(1) -I-e|Tect. If the electron-withdrawinggroup (X) is present at the end of the carbon chain, the effect is

F
called -I-effect.

for
586+ 56+ 5+ 5~

The -I-effect of some of the atoms or groups in decreasing order w..rt. to H is ;


-NO2 > -CN > -COOH > -F > -Cl > -Br > -I > -H
r
(«) +I-effect. If the electron-donating group (Y) is present at the end of the carbon chain, the effect is
You
ks
called -hl-effect.
o

655- 66- 6-
eBo

C-«-C-<-C-^Y
The +I-effect of some of the atoms or groups in decreasing order .wnt. to H is :
(CH3)3C- > (CH3)2CH- > CH3CH2- > CH3- > D- > H-
31. Electromeric effect involves the complete transfer of electron.s of a multiple bond (double or triple bond)
ad
our

to one of the bonded atoms (usually more electronegative) in presence of an attacking reagent.

Reagent added ^
Reagent withdrawn
It is of two types, i.e., + E and - E. Whereas electrophiles show +I-effect, nucleophiles show -E-effect.
Re
dY

32. Resonance. The phenomenon of resonance is said to occur whenever for a molecule we can write two or
more Lewis structures which differ in the position of electrons and not in the relative position of atoms.
The various resonance structures, none of which is capable of describing all the known properties of the
Fin

compound, are called canonical or resonance structures. The actual structure of the molecule is not
represented by any of the resonance structures but is a resonance hybrid of all these Lewis structures. For
example, benzene is a resonance hybrid of two Kekule structures (I and II) while actual molecule called
the resonance hybrid is represented by structure (III).

Q
I II Resonance hybrid (111)
As a result of re.sonance, all the carbon-carbon bonds are equal (139 A) and lie in between carbon-carbon
single bond length of 1-54 A and carbon-carbon double bond length of 1-34 A. The resonance hybrid is
ORGANIC CHEMISTRY-SOME BASIC PRINCIPLES AND TECHNIQUES 12/133

always more stable than any of the canonical structures. The difference in internal energy of the resonance
hybrid and the most stable canonical structure is called the resonance energy or the delocalization energy.
The resonance energy of benzene is 36 kcal mol '* or approx. 151 kJ mor*.
33. Conditions for resonance. All canonical structures must have same atomic skeleton, must be planar,
should involve delocalization of Tt-electrons only, must have same number of paired electrons (covalent
bonds) or unpaired electrons, must not have more than 8 electrons around C, N, O, F, etc. and should have
same or nearly same energy content. Further, resonating structures with more number of covalent bonds
and lesser charge separation are more stable and hence contribute more towards the resonance hybrid.
34. Resonance effect. The flow of 7t-electrons from one part of a conjugated system to the other creating
centres of low and high electron density due to resonance is called resonance effect. It is also called n-K or

w
Tt-TC-conjugation. It of the following two types :
(0 +R- or +M-effect. Groups which donate electrons to a double bond or a conjugated or aromatic system
are said to have +R or +M-effect. For example, -OH, -OR, -SH, -SR, -NH^, -NHR, -NR2, -Cl, -Br,

F lo
-I, etc. Thus,

^
CH2—CH~ci:
P. ":ch2—CH=ci: (+R-effect)

ee
(») -R or -M-effect. Groups which withdraw electrons from the double bond or from a conjugated or
aromatic system towards themselves due to resonance are said to have -R or -M-effect. For example,

Fr
>C = O. -CHO, -COOR. -CN. -NO-,, etc. Thus,

JT\ [} (-R-effect)

for
C1I2—CH—c^n: CH2—ch=c=n:“
35. Hyperconjugation or a-7C-conjugation. It involves release of a-electrons of the C—H bond present on
our
a-carbon of an alkyl group into adjacent double bond or the benzene ring. The hyperconjugation effect
decrea.ses as the number of a-hydrogens on the alkyl group decreases, i.e., it decreases in the order :
oks
CH3- > CH3CH2- > (CH3),CH- > (CH3)3C-
In other words, electron donating ability of the alkyl groups gets reversed when attached to double bond or
a benzene ring. This effect explains the (/) directive influence of alkyl groups in aromatic electrophilic
o
Y
eB

substitution reactions, (//) shortening of carbon-carbon single bond adjacent to a multiple bond, relative
stability of substituted alkenes and stability of carbocations and free radicals.
36. Carbocatlon.s. Carbon species carrying a positive charge are called carbocations. They have only a sextet
of electrons and hence are very reactive. These are planar species since the carbon atom carrying the
our

positive charge is sp^-hybridized. The stability of alkyl carbocations decreases in the order : 3° > 2° > T >
ad

CH3, i.e., (01-13)3C>(CH3)2CH >CH3CH2 >CH3-


This order of stability can be easily explained on the basis of inductive and hyperconjugation effect.
Y

Benzyl and allyl carbocations are much more stable due to resonance. The presence of electron donating
Re

groups (CH3, OCH3, NH2, etc.) in the benzene ring increases the stability while that of electron withdrawing
nd

groups (-NO2, -CN, -COOR, etc.) decreases the stability of carbocations.


37. Carbanions. Carbon species carrying a negative charge are called carbanions. They carry a fomial negative
Fi

charge and hence are very reactive. The stability of simple alkyl carbanions decreases in the order :

CH- > r > 2° > 3“, i.e., CH- > CH3CH- > (CH3)2CH" > (CH3)3C^. This order of stability can be
explained on the basis of -t-I-effect of alkyl groups.
Allyl and benzyl carbanions are more stable due to resonance.

The presence of electron donating groups (CH3, OCH3, NH-,, etc.) decreases the stability while that of
electron withdrawing (-NO2, -CN, -COOR, etc.) groups increases the stability of carbanions.
Tlie structure of simple alkyl carbanions is usually pyramidal like ammonia since the carbon atom carrying
the negative charge is sp^-hybridized. In contrast,the carbanions which are stabilized by resonance are
planar. In these carbanions, the carbon atom carrying the -ve charge is .v/?"-hybridized.
12/134 New Course Chemistry (XI)

38. Free radicals. A free radical is an atom or a group of atoms having an odd or unpaired electron. These are
produced by homolytic fission of a covalent bond. The stability of free radicals follows the same sequence

as that of carbocations : 3‘’>2°> T> CH^ , ‘-e., (€113)30 > (CH3)2CH >CH3CH2 >CH3-
This order of stability can be explained on the basis of inductive effect or hyperconjugation effect of the
alkyl groups. Benzyl and allyl free radicals are more stable due to resonance.
Like carbocations, free radicals are also planar species because the carbon atom carrying the unpaired
electron is 5p^-hybridized.

w
39. Types of organic reactions :
(fl) Substitution reactions involve direct replacement of an atom or a group of atoms in an organic molecule
by another atom or a group of atoms without any change in the remaining part of the molecule. Such
reac tions are typical of saturated compounds. Depending upon the nature of the attacking species, these

Flo
reactions are of three types : (/) nucleophilic substitution reactions. Hi) electrophilic substitution reactions
and (Hi) free radical substitution reactions.
(^) Addition reactions involve the combination of two reacting molecules to give a single product. These

e
reactions are typical of compounds containing double and triple bonds. Depending upon the nature of the

re
attacking species, these are of three types : (/) nucleophilic addition reactions, (//) electrophilic addition
reactions and (Hi) free radical addition reactions,

F
(c) Elimination reactions involve loss of two atoms or groups of atoms from the same or adjacent atoms
of a substrate leading to the formation of a multiple bond. These are mainly of three types : a, ()- and y-
ur
r
elimination reactions. Some important examples are dehydrohalogenation, dehalogenation and dehydration

fo
reactions,

(rf) Rearrangement reactions involve migration of an atom or a group from one atom to another within
the same molecule. Examples are conversion of ammonium cyanate to urea and Hofmann bromamide
reaction.
ks
Yo
(e) Isomerization reactions involve interconversion of one isomer into another keeping the molecular
oo
formulae as well as the carbon skeletons of the reactant and the product intact. For example, isomerization
of cv>but-2-ene into fr£m.v-but-2-ene and vice-versa.
B

IV. Purification of Organic Compounds


40. Crystallization is used to purify organic solids by dissolving them in a suitable solvent followed by
re

filtration of hot solution and keeping the hot filtered solution undisturbed for crystals to grow. For example,
(/) Sugar containing an impurity of common salt can be purified by crystallization from hot ethanol
which dissolves sugar but not the common salt.
u
ad

(//) A mixture of benzoic acid and naphthalene can be separated by treating with hot water which dissolves
Yo

benzoic acid but not the naphthalene.


Sometimes crystal formation can be induced by adding the crystal of the same substance to the saturated
solution. It is called seeding.
d

41. Fractional crystallization can be used to separate two or more compounds which have different solubilities
Re
in

in the same solvent.

42. Sublimation. Substances like camphor, naphthalene, anthracene, benzoic acid, iodine, etc. whose vapour
F

pressures become equal to the atmospheric pressure much before their respective melting points can be
purified by sublimation.
43. Simple distillation is applied for volatile liquids which boil without decomposition at atmospheric pressure
and which contain non-volatile impurities. Mixture of liquids whose boiling points differ by 30-50 K can
also be purified/separated by simple distillation e.g., a mixture of ether (b.p. 308 K) and toluene (b.p.
384 K), benzene (b.p. 353 K) and toluene (b.p. 384 K), etc.
44. Fractional distillation. Mixture of liquids whose boiling points differ by about 10 K can be separated by
fractional distillation, e.g., acetone (b.p. 329 K) and methanol (b.p. 338 K), refining of petroleum, etc.
However, the components of an azeotropic mixture, i.e., constant boiling mixture such as alcohol and
water from rectified spirit cannot be separated by fractional distillation but can be separated by azeotropic
distillation.
ORGANIC CHEMISTRY-SOME BASIC PRINCIPLES AND TECHNIQUES 12/135

45. Steam distillation. Substances which are volatile in steam, insoluble in water ; possess a high vapour
pressure (10-15 mm Hg) and contain non-volatile impurities can be purified by steam distillation. For
example, substances like o-nitrophenol, bromobenzene, aniline, nitrobenzene, essential oils, etc. can be
purified by steam distillation.
Similarly, a mixture of o-nitrophenol (volatile) and /?-nitrophenol (non-volatile), a mixture of o-hydroxy-
acetophenone (volatile) and /7-hydroxyacetophenone (non-volatile), a mixture of salicylaldehyde (volatile)
and p-tiydroxybenzaldehyde(non-volatile) can also be easily separated by steam distillation.
46. High boiling liquids which decompose at or below their boiling points are purified by vacuum distillat'on
or distillation under reduced pressure, c.g., glycerine and concentration of raw juice in sugarcane factories.
47. Solvent extraction. Organic compounds which are usually more soluble in organic solvents than in water
can be easily extracted from their aqueous solutions by solvent extraction.
48. Chromatography is the most modem and versatile method used for the separation, identification and

w
purification of organic compounds even when available in small amounts. The technique was developed
by Tswett, a Russian Botanist in 1906.

F lo
(a) Adsorption or column chromatography. It is a solid/liquid chromatography in which the stationary
phase is a solid while the mobile phase is a liquid. It is based upon the principle of differential adsorption
of the various components of a mixture (mobile phase) on a suitable adsorbent (stationary phase) such as
alumina, silica, etc.
(b) Thin layer chromatography. It is another type of adsorption chromatography in which separation of

e
Fre
the components of a mixture is achieved over a thin layer of an adsorbent (usually silica gel) coaled over
a plastic or a glass plate. The spots on the developed chromatograph are visualized either by UV light or
iodine vapours and then identified by their Ry values,

for
(c) Partition or paper chromatography. It is liquid/liquid chromatographyin which both the stationary
and mobile phases are liquids. The stationary phase in paper chromatography is not the cellulose but the
water which Is adsorbed or chemically bound to it. The mobile phase is another liquid which is usually a
r
mixture of two or three solvents with water as one of the components.
You
Paper chromatography works on the principle of continuous differential partitioning (or distribution) of
ks
the various components of the mixture between the stationary and the mobile phases. This technique is
o
eBo

especially used for separation of sugars and amino acids.


49. Criteria of purity of an organic compound is best determinedby its mixturemelting point which remains
undepressed when the melting point of the mixture of an organic compound with its authentic sample is
determined.

V. Qualitative Analysis
ad
our

50. The presence of C and H, in an organic compound, is delected by heating the given compound with dry
CuO when C is oxidised to CO2 which turns lime-water milky while H is oxidised to H^O which turns
anhyd. CUSO4 blue.
51. Beilstein test is used for detection of halogens (Cl, Br, I but not F). In this test, a blue or bluish green flame
Re
dY

due to the formation of volatile cupric halides is obtained. However, organic compounds like urea, thiourea,
etc. also give postive Beilstein test even though they do not contain halogens.
52. Lassaigne’s test. Nitrogen, sulphur and halogens in an organic compound are detected by Lassaigne’s
Fin

test in which the organic compound is fused with Na metal. During fusion, N is converted into NaCN, S
into No2S (sometimes both N and S react together with Na metal to form NaCNS) and halogens into
sodium halides.

53. The Lassaigne’s extract is heated with a few drops of freshly prepared FeS04 solution, cooled and then
acidified with dil. H2SO4. Appearance of a green or blue colouration due to the formation of ferric
ferrocyanide (Prussian blue), Fe4[Fe(CN)g]3 indicates the presence of nitrogen while appearance of a
blood red colouration due to the formation of Fe(CNS)3 indicates the presence of both N and S. If, however,
blood red colouration is not obtained, it certainly does not indicate the absence of both N and S.
54. If Lassaigne’s extract gives a black ppt. with lead acetate due to the formation of PbS or a violet colouration
with sodium nitroprusside Na4[Fe(CN)5NO] due to the formation of Na4[Fe(CN)5NOS], it indicates the
presence of S.
12/136 New Course Chemistry (XI)CSISQ

55. Compounds which contain N but no carbon do not give positive Lassaigne’s lest for N since NaCN cannot
be formed, e.g.. hydrazine.
56. Before testing for halogens, the Lassaigne’s extract is boiled with dil. HNO3 to decompose Na2S or NaCN
otherwise these will interfere with the tests of halogens.
57. Phosphorus in an organic compound is detected by fusing the organic compound with Na202 (oxidising
agent). The fused mass is extracted with water. The aqueous solution thus obtained is boiled with few
drops of cone. HNO3 and ammonium molybdate solution is then added. Appearance of a yellow ppt. of
ammonium phosphomolybdate, (NH^)^PO ^.12 MoOj indicates the presence of phosphorus.

ow
VI. Quantitative Analysis
58. Leibeg’s method is used for estimation of C and H in an organic compound.
59. CO2 formed during Liebig's method is absorbed in U-tubes containing either cone. KOH solution or ascarile
(NaOH + CaO) and H2O formed is absorbed in U-tubes containing either anhyd. CaCU or Mg(C104)2.

60.
12
%C= —X
MassofC02 XlOO ,%H=—x Mass of H2O xlOO

e
44 Wt. of substance 18 Wt.of substance

Fl
re
61. Duma’s and Kjeldahl’s methods are used for estimation of N in organic compounds. Whereas Duma’s
method is of general applicability but Kjeldahl’s method is used for e.stimation of N in fertilzcrs, drugs,

F
food stuffs, etc.
(a) In Duma’s method, nitrogen of the organic compound is converted into N2 gas which Is collected over
ur
cone. KOH solution in a nitrometer.

r
fo
%N =
28
X
Vol.of N., (in cm^)at NTP XlOO
22400 Wt.of substance
ks
(h) In Kjcldahl’s method, the organic compound is digested with cone. H2SO4 in presence of K2SO4 and
Yo
a little CUSO4 or Hg. Whereas K2SO4 is used to raise the b.p. of H2S04, C11SO4 or Hg catalyses the
oo
reaction.

During digestion, the N of the organic compound is completely converted into (NH4>2S04.
B

The (NH4>.>S04 thus formed is heated with cone. NaOH solution and the NH3 thus evolved is absorbed in
14NxV
re

a known excess of standard dil. H2SO4. % N = where N and V are normality and volume of the
W

acid used respectively and W is the wt. of the sub.stance taken.


u
ad

Kjeldahl’s method is not applicable to compounds containing nitrogen in the ring(i.e., pyridine, quinoline,
Yo

etc.), and compounds containing NO2 and N = N groups since these compounds are not completely
converted into (NH^)2SO^ during digestion.
62. Carlus method is used for estimation of halogens. In this method, a known weight of an organic compound
d

is heated with fuming HNO3 containing a few crystals of AgN03 in a sealed tube called Carius tube. From
Re

the wt. of AgX formed, the percentage of X can be calculated as follows :


in

%C1 = 35-5 ^ Mass of AgCl XlOO, %Br =


80
X
Massof AgBr XlOO
F

143-5 Mass of substance 188 Mass of substance

127 Massof Agl


%1 = X XlOO
235 Mass of sub.stance

Carius method can also be used for estimation of Sand P. During oxidation with HNO3, S is oxidised to
H->S04 which is estimated as BaS04 while P is oxidised to H3PO4 which is estimated as Mg2p207.

%s =
32 Massof BaS04 62 Massof Mg2?.>07 XlOO
X XlOO, %P = X
233 Massof substance 222 Massof substance

63. Percentage of oxygen = 100 - (Sum of percentages of all other elements).


ORGANIC CHEMISTRY-SOME BASIC PRINCIPLES AND TECHNIQUES 12/137

QUESTIONS

Based on NCERT Book

I. Multiple Choice Questions (a) ethane, ethylene, ethyne


(b) methane, methanol, methanal
I. Hybridization, (c) methane, ethane, propane
Classification and Nomenclature (d) 1-hexene. 2-hexene. 3-hexene.

w
8. The members of a homologou.s series have
1. The^tate of hybridization of C->, C3, C5 and of
the hydrocarbon. (a) Different general formulae
CH,
(h) Different molecular weights
CH3

Flo
6l ■ (c) Different methods of preparation
CH3 —C—CH = CH—CH—C = CH
(d) Different chemical properties.
5 4 3 2 1

e
CH3 9. The number of primary, secondary, tertiary and

re
is in the sequence quaternary carbons in neopentane are respectively
(a) 4, 3, 2 and 1 (b) 5, 0, 0 and 1
(a) sp^, sp^, sp^ and sp (b) sp, sp-, sp~ and sp^

F
(c) 4, 0, 0 and I id) 4, 0, 1 and 1
(c) sp, sp~, sp^ and sp^ {d) sp, sp^, sp^ and sp^
2. Considering the state of hybridization of carbon 10. The compound with an isopropyl group is
ur
or
atoms, find out the molecule among the following (a) 2, 2, 3, 3-Tetramethylpentane
which is linear ? (h) 2, 2-Dimethylpentane

f
(a) CH3—CH2—CH2—CH3 (c) 2. 2, 3, Trimethylpentane
ks
(b) CH3—CH = CH—CH3 (d) 2-Methylpentanc.
Yo
(c) CH3—C = C—CH3 11. The structure of isobutyl group in an organic
oo
(d) CH2 = CH—CH2—C = CH compound is
3. The maximum number of carbon atoms arranged (a) CH3—CH2—CH,—CH2—
eB

linearly in the molecule, CH3


CH3 - C = C - CH = CH2 is
(b) CH3—C—
(a) 5 (h)4
ur

(c)3 id) 2. CH3


ad

CH
4. The number of a- and tt-bonds present in 1,3-
Yo

3\
(c) CH—CH^—
butadiene are respectively
id) 9 and 2 ib) 8 and 2 CH3"
(c) 9 and 3 id) 9 and 1 id) CH3—CH—CH2CH3
d
Re

12. Azulene is an example of


5. In hexa-1, 3-dien-5-yne, the number of C-C, c,
in

C-C K and C-H ct bonds respectively are ia) benzenoid aromatic compound
ia) 5, 4 and 6 ib) 6, 3 and 5 ib) non-benzenoid aromatic compound
F

(c) 5, 3 and 6 id) 6, 4 and 5 (c) alicyclic compound


6. Which of the following represents the given mode id) heterocyclic alicyclic compound
of hybridization sp--sp^-sp-sp from left to right ? 13. Which isomer of hexane has only two different
sets of structurally equivalent hydrogen atoms ?
ia) H2C = CH—C = N ib) HC = C—C = CH
ia) 2, 2-dimethylbutane ib) 2-methylpentane
(c) H2C = C = C = CH2
(c) 3-methyIpentane id) 2, 3-dimelhylbutane

id) 14. The compound with molecular formula, C(-,H|4 has


H2C two tertiary carbons. Its lUPAC name is
7. Which of the following represents a homologous ia) n-hexane ib) 2-methylpeniane
series ? (c) 2, 3-dimethylbutane id) 2, 3-dimethylpentane
12/138 New Course Chemistry (XI)EZ

15. The lUPAC name for the hydrocarbon represented {b) 4-methyl-2-pentyne
by the Swastik sign is I (c) isopropylmethyl acetylene
id) 3-methyl-4-pentyne
(a) neononane (b) tetraethylcarbon (e) 2-methyl-3-pentyne
(c) 2-ethylpentane (d) 3, 3-diethylpentane. 22. The lUPAC name of C^HsCOCl is
16. The lUPAC name of (a) chlorobenzyl ketone
(b) benzene chloroketone
(c) benzenecarbonyl chloride

low
(d) chlorophenyl ketone
(a) 3-ethyl-4, 4-dimethylheptane 23. The lUPAC name of the compound X is
(b) 1, 1-diethyl-2, 2-dimethylpentane O CN
(c) 4, 4-dimethyl-5, 5-diethylpentane I

(^0 5, 5-diethyl-4, 4-dimethylpentane X


= CH3/ CH{ \ CH3

ee
17. The correct lUPAC name for the compound
CH3

rF
Fr
(a) 4-cyano-4-methyloxopentane
(b) 2-cyano-2-methyl-4-oxopentane
IS
(c) 2, 2-dimethyl-4-oxopentanenitrile

for
(d) 4-cyano-4-methyl-2-pentanone
(a) 4-ethyl-3-propylhex-l-ene 24. The lUPAC name of the following compound is
u
(b) 3-ethyl-4-ethenylheptane
ks
CH, OH
(c) 3-ethyl-4-propylhex-5-ene I ^ I
Yo
H3C—CH—CH—CH2 —COOH
oo
(d) 3-(l-ethylpropyl) hex-l-ene
18. Consider the following compound, (a) 2-methyl-3-hydroxypentan-5-oic acid
eB

(b) 4, 4-dimethyl-3-hydroxybutanoic acid


(c) 4-methyl-3-hydroxypentanoic acid
(d) 3-hydroxy-4-methylpentanoic acid
r
ou
ad

lUPAC name of this compound is


(a) 5, 6-diethyl-3-methyldecane 25. ThelUPACnameof
Y

(b) 5, 6-diethyl-3-methyldec-4-ene
(c) 3, 5, 6-triethyldec-6-ene
(a) 2, 3-dimethylpentanoyl chloride
Re
nd

(d) 3, 5, 6-trimethyldec-4-ene
(b) 3, 4-dimethylpentanoyl chloride
19. The lUPAC name of the compound having the
(c) 1-chloro-l-0X0-2, 3-dimethylpentane
Fi

formula CH h C—CH = CH2 is


(d) 2-ethyl-3-methylbutanoyl chloride
(a) l-butyne-3-ene (b) but-l-yne-3-ene
(c) l-butene-3-yne (d) 3-butene-1-yne
20. The lUPAC name of the compound,
CH3CH = CHC = CH is 26. The lUPAC name of IS

(a) pent-4-yn-2-ene (b) pent-3-en-1-yne


(c) pent-2-en-4-yne (d) pent-l-yn-3-ene (a) 3-methylcyclohexene
21. The correct lUPAC name for methylisopropyl- (b) l-methylcyclohex-2-ene
acetylene is (c) 6-methylcyclohexene
(a) 2-methyl-4-pentyne (d) l-methylcyclohex-5-ene
ORGANIC CHEMISTRY-SOME BASIC PRINCIPLES AND TECHNIQUES 12/139

27. Give the lUPAC name of the compound 32. The lUPAC name of

,C
CH3 IS
,
(a) 1, 1, 3-Trimelhylcyclohex-2-ene
(b) 1, 3, 3-Triniethylcyclohex-l-ene
(c) 1, 1, 5-Trimethylcyclohex-5-ene (a) acetylcyclohexadiene
(/;) l-cyclohexa-2, 4-dienylethanone

w
(cf) 2, 6, 6-Trimethylcyclohex-l-ene
28. The lUPAC name of the compound shown below is (c) 6-cyclohexa-l, 3-dienylethanone
Cl (d) none of these
33. The lUPAC name of

Flo
CONH IS

Br

ee
(a) 2-bromo-6-chlorocyclohex-1 -ene (a) N-cyclohexylbenzamide

Fr
(b) 6-bromo-2-chlorocyclohexene (b) N-phenyl-N-cyclohexylmethanamide
(c) 3-bromo-l-chlorocyclohexene (c) N-phenyicyclohexanccarboxamide
(d) l-bromo-3-chlorocyclohexene (d) N-cyclohcxyl-N-phenylmethanamide.

for
29. The correct decreasing order of priority for the 34. The lUPAC name of
ur
functional groups of organic compounds in the
lUPAC system of nomenclature is C=0 is
(a) —CONH2. —CHO. —SO3H . —COOH
k s
(a) cyclohexanone
(b) —COOH, —SO3H, —CONH2, —CHO
Yo
oo
(c) —SO3H. —COOH, —CONH2, —CHO (b) cyclohexylmethanone
id) —CHO, —COOH, —SO3H , —CONH2 (c) oxycyclohexene
eB

30. The lUPAC name of the following compound (d) cyclohexylidenemethanone.


OH
II. Isomerism
r

35. Which one of the following is not an isomer of


ou
ad

IS
3-methylbut-l-yne ?
CN (b) Bula-l,3-diene
(a) Pent-l-yne
(c) Pent-2-yne (d) Penta-1,3-diene
Y

Br

(a) 4-bromo-3-cyanophenol (e) 2-Methylbuta-l,3-diene


(b) 2-bromo-5-hydroxybenzonitrile 36. Among the following pairs, the pair that illustrates
Re
nd

(c) 2-cyano-4-hydroxybromobenzene functional isomerism is

(</) 6-bromo-3-hydroxybenzonitrile (a) 1-butanol and 2-butanol


Fi

31. The lUPAC name of the compound (b) dimethyl ether and ethanol
CHO
(c) c/5-2-butene and tra/ii-2-butene
NO2 (d) ethanol and ethanal
IS 37. The type of isomerism observed in urea molecule
IS

(a) chain (/>) position


OCH3
(r) geometrical {d) tautomerism
(a) 4-methoxy-2-nitrobenzaldehyde
38. Identify the compound that exhibits tautomerism
(/)) 4-formyl-3-nitroanisole
(c) 4-methoxy-5-nitrobenzaldehyde (a) lactic acid ib) 2-pentanone
(d) 2-formyl-6-nitrobenzene (c) phenol id) 2-butene
12/140 New Course Chemistry fxnrasTMn

39. The enolic form of acetone contains : 45. Which of the following species is not electrophilic
in nature ?
(a) 9 sigma bonds, 1 pi bond and two lone pairs
+
(b) 8 sigma bonds, 2 pi bonds and two lone pairs
(a) Cl-^ ib) BH3 (c) H3O+ (d) NO2
(c) 10 sigma bonds, 1 pi bond and one lone pair 46. The most unlikely representation of resonance
(d) 9 sigma bonds, 2 pi bonds and one lone pair structure of /?-nitrophenoxide is
40. Maximum enol content is in
“O O
O 0 0

(a) (b)

(a) (b)

w
O O O

O" O
(c) (d)

F lo
o 0 or 0
H H

41. The order of stability of the following tautomeric


compounds is
(c) f] (d)

e
Fre
OH O

CH2=C—CH2—C—CH3 r O" 0"

47. Among the following, the least stable resonance

for
I
structure is
O O
O
r
CH3—C—CH2—C—CH3 (fl) N
You
II 0"
s
ook

OH O 0
+

± CH3—C=CH—C—CH3 (b) N
eB

III
o-

(a) II > I > III (b) I! > III > I o

(c) I > II > III (d) III > II > I (c) N


our

+
ad

o-
42. How many cyclic and acyclic isomers (including
tautomers) can be made by the formula C3HgO ? 0-
(a) 4 (b) 5
(c) 9 (rf) 10
dY

o-
Re

III. Fundamental Concepts 48. Among the following structures, the one which is
not resonating structure of others is
of Organic Reaction Mechanism
Fin

43. The temporary effect in which there is complete


transfer of a shared pair of ^/-electrons to one of
the atomsjoinedby a multiple bond on the demand Me Me
of an attacking reagent is called I II

(a) inductive effect


(b) hyperconjugation effect
(c) positive resonance effect
Me
(d) electromeric effect
III IV
44. : CCI2 is (a) I ib) II
(a) an electrophile (b) a free radical
(c) III (d) IV
(c) a nucleophile (d) none of these
ORGANIC CHEMISTRY-SOME BASIC PRINCIPLES AND TECHNIQUES 12/141

49. Hyperconjugation involves overlap of the (a) iv<m<n<i (/?) m<iv<i<n


following orbitals (c) i<m<iv<ii id) n<iv<ra<i
(a)G-G (b)G-p 56. The order of stability of the following carbocations
(c)p-p (d)n-n IS

50. The arrangement of (CH3)3C-, (CH3)2CH-, + +

CH3CH2- when attached to benzene or unsatu ch2=ch—CH2 ; CH3—CH2—CH2;


rated group in increasing order of inductive effect I II
IS

w
(a) (CH3)3C- < (CH3)2CH- < CH3CH2- CH2
(b) CH3CH2- < (CH3)2CH- < (CH3)3C-
111
(c) (CH3)2CH- < (CH3)3C- < CH3CH2-

Flo
(a) m>i>ii (b) m>n> I
(d) (CH3)3C- < CH3CH2- < (CH3)2CH-
51. The effect that makes 2,3-dimethyl-2-butene more (c) n>m>i (^0 i>n>m

e
stable than 2-butene is 57. The ascending order of stability of the carbanion

re
(a) resonance ib) hyperconjugation CHj(P), C6H5CH2 (Q), (CH3)2CH- (R) and
(c) electromeric effect (d) inductive effect

F
(e) steric effect
CH2 =CHCH2(S) is
52. Hyperconjugation is most useful for stabilizing (a) P<R<S<Q (b) R<P<S<Q
ur
or
which of the following carbocations ? (c) R<P<Q<S (d) P<R<Q<S
(a) neopentyl (b) tert-Butyl 58. The stability of carbanions in the following.

f
(c) isopropyl (d) ethyl ks 1^-
(e) methyl (i)RCsC® (/O
Yo
53. In which of the following compounds, the C-Cl
oo
bond ionisation shall give most stable carbonium 0 0
ion ? (hi) R2C = CH (iv) R3C—CH2
B

H H is in the order of
\
CH—Cl C-—Cl (a) (iv) > (iO > (Hi) > (i)
re

(a) I (b) O2NCH2 H (b) (/)>(i/i)>(i/)>(/v)


(c) (i)>(iO>(/H)>(/v)
u
ad

H
H3C Cl (d) (H)>(m)>(iv)>(i)
Yo

H3C\ 59. Which of the following compounds possesses the


(c) C—Cl (d)
H3C C-H bond with the lowest bond dissociation
energy ?
54. The decreasing order of stability of the carbonium
d

(fl) Toluene (b) Benzene


Re

10ns
in

(c) rt-Pentane (d) 2,2-Dimethylpropane


CgHjCH^ (D,P-(CH30)C6H4CH+ (H),
60. The stablest radical among the following is
F

P-(N02)C6H4CH^ (ffl),
P-(CH3)C6H4CH+ (IV) is (a) CgHg—CH—CH3 (b) CH3—CH—CH3
(a) iv>n>i>m (b) n>IV>ffl>I (c) C6H5CH2CH2 (d) CH3CH2CH2
(c) n>iv>i>in (d) iv>n>m>i
61. Most stable radical is
55. Arrange the following carbocations in order of
increasing stability
(a) CH3 (b) CH2
I. (CH3)3CCHJ n. (CH3)3C+

m. CH3CH2CH+ IV. CH3—CH—CH3 (c) CH2=CHCH2 (d) CH2=CH


12/142 'Pna^Ucfi.'A New Course Chemistry (XI)SSEIHIl

11,0 71. The separation of the constituents of a mixture by


62. The reaction, (CH3)3C—Br ^ (CH^)^C—OU column chromatography depends upon their
IS
{a) different solubilities
(a) elimination reaction (/;) substitution reaction (/?) different boiling points
(c) free radical reaction {d) addition reaction (c) different refractive indices
IV. Methods of Purification (d) differential adsorption.
of Organic Compounds 72. Paper chromatography has following mobile and
stationary phases respectively
63. Camphor is purified by
(a) liquid, solid {b) solid, solid
(a) distillation {b) vacuum distillation

w
(c) gas, liquid (d) liquid, liquid
(c) sublimation id) steam distillation
(e) fractional crystallization V. Qualitative and Quantitative Analysis
64. The best method for the separation of naphthalene

F lo
73. An organic compound which produces a bluish
and benzoic acid from their mixture is
green coloured flame on heating in presence of
(a) sublimation (b) chromatography copper is
(c) crystallisation (d) distillation (a) chlorobenzene (b) benzaldehyde

ee
65. A miscible mixture of C(^Hg+CHCl3 can be (c) aniline (d) benzoic acid

Fr
separated by 74. In the Lassaigne’s test for the detection of nitrogen
(a) sublimation ib) distillation in the organic compound, the appearance of blue
(c) filtration (d) crystallization coloured compound is

for
66. A mixture of t7-nitrophenol and p-nitrophenol can (fl) ferric ferricyanide (b) ferrous ferricyanide
ur
be separated by (c) ferric ferrocyanide (d) ferrous ferrocyanide
(a) sublimation (h) steam distillation 75. The Lassaigne’s extract is boiled with cone. HNO3
(c) fractional crystallization while testing for halogens. By doing so it
s
(a) decomposes Na2S and NaCN, if formed
ook
(d) simple distillation.
Yo
67. Two volatile liquids A and B differ in their boiling (b) helps in the precipitation of AgCl
points by 15K. The process which can be used to (c) increases the solubility product of AgCl
eB

separate them is
(d) increases the concentration of NOJ ions
(a) fractional distillation (b) steam distillation
76. Which of the following compounds will not give
(c) distillation under reduced pressure Lassaigne’s test for nitrogen ?
r

(d) simple distillation. (a) NH2NH2 (b) C6H5NHNH2


ad
ou

68. Glycerol which decomposes at its boiling point (c) PhN = NPh (d) NH2CONH2
can be purified by
77. Which of the following compounds gives blood
ia) steam distillation (b) simple distillation red colouration when its Lassaigne’s extract is
Y

(c) distillation under reduced pressure treated with alkali and ferric chloride ?
Re

(d) fractional di.stillaiion. fa) thiourea (b) diphenyl sulphide


nd

69. In steam distillation of toluene, the pressure of (c) phenylhydrazine (d) benzamide
toluene in vapour is 78. 0-30 g of an organic compound containing C, H
Fi

(a) equal to pressure of barometer and O on combustion yields 0-44 g CO2 and
(b) less than pressure of barometer 0-18 g H2O. If one mole of compound weighs 60,
then molecular formula of the compound is
(c) equal to vapour pressure of toluene in simple
distillation (a) C3HgO ib) C2H4O2
(c) CH2O id) C^H^O
id) more than the vapour pressure of toluene in
simple distillation. 79. In Duma’s method for estimation of nitrogen,
0.25 g of an organic compound gave 40 mL of
70. An organic substance from its aqueous solution
nitrogen collected at 300 K temperature and 725
can be separated by
mm pressure. If the aqueous tension at 300 K is
(a) distillation (/;) steam distillation 25 mm, the percentage of nitrogen in the
(c) solvent extraction (d) fractional distillation. compound is
ORGANIC CHEMISTRY-SOME BASIC PRINCIPLES AND TECHNIQUES 12/143

ia) 16-76 {h) 15-76 85. Assertion. The lUPAC name of the compound
(c) 17-36 id) 18-20 CHo = CH—CH = CH—CH—C = CH is
80. In Kjeldahl’s method, the nitrogen present in the
organic compound is converted into CH =CH2
ia) ga.seous ammonia 5-eihynylhepUi-l. 3, 6-triene
ib) ammonium sulphate Reason. While numbering the carbon chain triple
(c) ammonium phosphate bond is given preference over the double bond.
id) ammonium nitrate. 86. Assertion. Butane and 2-methylbutane arc
81. In Kjeldahl’s method for estimation of nitrogen honiologues.
present in a soil sample, ammonia evolved from Reason. Butane i.s a straight chain alkane while
0-75 g of sample neiitrali zed 10 mL of 1 M H2SO4. 2-melhylbutanc is a branched chain alkane.
The percentageof nitrogen in the soil is 87. Assertion. The lUPAC name of o-hydroxybenzyl
(fl) 37-33 ib) 45-33 chloride is o-chloromclhylphcnol.

F low
(c) 35-33 id) 43-33 Rea.son. The prefix o- is used in the common
82. The ammonia evolved from the treatment of
system but not in the lUPAC system.
0-30 g of an organic compound for the estimation 88. Assertion. CH^CHO and CH2 = CHOH are
of nitrogen was passed in 100 mL of 0-1 M resonance structures.
sulphuric acid. The excess of acid required 20 mL Reason. Tautomers differ both in the position of
of 0-5 M sodium hydroxide solution for complete atoms as well as electrons.
neutralization. The organic compound is
89. Assertion. R-N = C and R - C = N are

re
ia) acetamide ib) benzamide

for F
(c) urea id) thiourea tautomers.

83. In Carius method of estimation of halogens, Reason. They are functional isomers.
250 mg of an organic compound gave 141 mg of 90. Assertion. Enoi form of acetone (CH3COCH3)
AgBr. The percentage of bromine In the compound exists in < 01% quantity. However, the enol form
IS
of acetylacetone exists in approximately 15%
(at. mass Ag = 108 ; Br = 80)
our
quantity.
s

(a) 48 ib) 60
eBook

Reason. Enol form of acetylacetone is stabilized


(c) 24 (d) 36 by intramolecular hydrogen bonding which is not
Y

84. In the estimation of sulphur by Carius method, possible in enol form of acetone.
0-480 g of an organic compound gives 0-699 g of
91. Assertion. While testing for sulphur, the
barium sulphate. The percentage of sulphur in this
ad

compound is (atomic masses : Ba = 137, S = 32


Lassaigne’s extract is acidified dil. H2SO4 before
our

andO= 16) adding lead acetate solution.


ia) 20% ib) 15% Reason. Sulphide ion reacts with lead acetate to
(c) 35% id) 30% give black ppt. of PbS.
92. Assertion. Lassaigne’s test is not shown by
Re

II. Assertion-Reason Type Questions diazonium salts.


Y

Rea.son. Diazonium salts lose on heating much


For questions below, two statements are given
Find

before they have a chance to react with fused sodium.


one labelled Assertion (A) and the other labelled
93. Assertion. Silver nitrate test can be used to detect
Reason (R). Select the correct answer to these
questions from the codes (a), ib), (c) and id) fluorine in organic compound.
given below : Reason. Silver fluoride is soluble in water.
ia) Both A and R are correct and R is the correct
explanation of A. 94. Assertion. R - N = C : acts as an electrophile.
ib) Both A and R are correct but R is not the correct Reason. Due to presence of a lone pair of
explanation of A. electrons on the N atom it acts as a nucleophile.
(c) A is correct but R i.s wrong. 95. Assertion. H2O acts as a nucleophile but not as
id) A is wrong but R is correct. an electrophile.

/
12/144 New Course Chemistry (XI)IZ3E3HD

Reason. H30'^ neither acts as an electrophile nor Reason. Hyperconjugation as well as inductive
as a nucleophile. effect due to additional alkyl groups stabilize
96. Assertion. Homolytic fission of CH3CHiCH3 tertiary carbocations.
+ 100. Assertion. Allyl free radical is more stable than
gives CH3CH2 and CHJ. simple alkyl free radical.
Reason. The allyl free radical is stabilized by
Reason. CHjCHt is more stable than CH^ but resonance.

101. Assertion. Free radicals are always planar.


CH3CH^ is less stable than CH^. Reason. They can achieve sjp- as well as sp^-
hybridisation.
;o:“
102. Assertion. CH^ = CH“ is less stable than
+

97. Assertion. H—C = 0—H is more stable that HCsC".


I Reason. Acetylene is less acidic than ethylene.
:o^ 103. Assertion. In CH2 = C = CH2 all the carbon atoms

F low
H—C — O—H.
are .syj^-hybridized.
Reason. The hydrogen atoms on terminal carbons
II lie in perpendicular planes.
Reason. In structure (1), both the 0 atoms and 104. Assertion. Commercial benzene containing
the carbon atom have an octet of electrons but in thiophene is purified by shaking with cone.
structure (U), oxygen atom has only a sextet of H2SO4.
electrons. Reason. The boiling b.p. of benzene is 353 K

re
for F
98. Assertion. All the carbon atoms of but-2-ene lie while that of thiophene is 357 K.
in one plane. 105. Assertion. Nitrogen present in m-dinitrobenzene
Reason. All the carbon atoms in but-2-ene are sp^- can be estimated by Kjeldahl’s method.
hybridized. Reason. During digestion, nitrogen of m-
99. Assertion. Tertiary carbocations are generally dinitrobenzene is not completely converted into
formed more easily than primary carbocations. (NH4)2S04.
Your
s
eBook

ANSWERS

I. Multiple Choice Questions


ad
our

1. (^0 2. (c) 3. {h) 4. (a) 5. {a) 6. (a) 7. (c) 8. ib) 9. (c) 10. id)
11. (c) 12.ib) 13. id) 14. ic) 15. id) 16.ia) 17. ia) 18. ib) 19. ic) 20. ib)
21. ib) 22. (c) 23.(c) 24. id) 25. (a) 26. ia) 27. ib) 28. (c) 29.ib) 30, ib)
31. ia) 32.ib) 33. (c) 34. id) 35. ib) 36. ib) 37. id) 38. ib) 39. ia) 40. ib)
Re

41. id) 42. (c) 43. {d) 44. (a) 45.(c) 46. (c) 47.ia) 48. id) 49. ib) 50. (n)
Y

51. ib) 52.ib) 53. {d) 54.ic) 55.(c) 56.ia) 57. ib) 58. ic) 59. ia) 60.ia)
Find

61. ia) 62. ib) 63.(c) 64. (c) 65. (b) 66. ib) 67. (a) 68.ic) 69.ib) 70.(c)
71. id) 72. id) 73. (a) 74. ic) 75. ia) 76. ia) 77. ia) 78. ib) 79.ia) 80. ib)
81. ia) 82. (c) 83.(c) 84. ia)

II. Assertion-Reason lype Questions


85. ic) 86. ib) 87. id) 88. id) 89. id) 90. ia) 91. id) 92.ia) 93. id) 94. ic)
95. ib) 96. id) 97. ia) 98.ic) 99. (n) 100.ia) 101. id) 102. (c) 103. id) 104. ib)
105. (d)

\
ORGANIC CHEMISTRY-SOME BASIC PRINCIPLES AND TECHNIQUES 12/145

For Difficult Questions

I. Multiple Choice Questions 4 2


5
1
CH 3
3 CH3 18. 8
5, 6-Diethyl-3-
5 4 3I ' 2 I 10
meihyldec-4'cne
1, CH3—C CH=CH—CH—CsCH

w
7 9
sp^ sp 3 sp
CH3 4 3 2 1

HC = C—CH=CH, l-Butene-3-yne
Thus, option (d) is correct.
(●/ double bond is given preference)

Flo
3. Only four carbon atoms marked as 1, 2, 3 and 4
lie along a line while the fifth one is inclined to 4 3 2 1
this line at angle of 120® as shown : 20. CH3CH=CHCsCH Pent-3-en-l-yne.

e
re
H
1 2 3 4 /

CH3—C=C-j^3CH2
5
2 3 4
/CH3

F
120®
21. CH3—C=C—CHC^
CH3
C—C = C 4-Methyl-2-pentyne
ur
r
TZ

|a oj

fo
H H
22. Benzenecarbonyl chloride.
i
o-Bonds = 9 ; 7C-Bonds = 2 0 CN
Nitrile gets
1 2 3 4 5 6 23.
ks preference over
5/ \3 /\
Yo
5. H—C=C—C=C—CsC—H C = O group
1 I I I
oo
H H H H C-C, o-bonds = 5 CH3 CH2 CH3
C = C, 7C-bonds=4 2, 2-Dimethyl-4-oxopcntanenitriIe
B

Hexa-1, 3-dien-5-yne
C-H, a-bonds - 6
CH. OH
re

CH, CH, 5 4I 3I 2 1
I M ^ 24. H3C—CH—CH—CH2—COOH
13. CH3—CH—CH—CH3 3-Hydroxy-4-inethylpemanoic acid
u
ad

2, 3-Dimethylbutane has two sets of structurally


Yo

equivalent hydrogens (twelve 1° and two 3°)


27. 1,3,3-TrimethyIcyclohex-1 -ene
14. 2,3-Dimethylbutaneas shown in Ans. 12 has two
(double bond is given preference)
3“ carbon atoms.
d

15. I 2
Re

28. 3-Bromo-l-chlorocyclohexene.
in

29. -COOH, -SO3H, -CONH2, -CHO (Refer to page


4 5
12/42)
F

3, 3-Diethylpentane
30. 2-Bromo-5-hydroxybenzonitri!e
OH
(●.■ CN is the principal
3-Ethyl-4,4-dimethyl- functional group while OH
heptane.
I and Br are substituent groups)
1

17. 4-Ethyl-3-propylhex-l-ene 2-Bromo-


4
5-hydroxybenzonitrile
12/146 ‘P>tetde€^ New Course Chemistry (XI) BE

CHO

o o
31. (v CHO is the principal

ow
functional group while NO2 id) <■

and OCH3 are substituent H H


OCH3 groups).
3-Oxobutanal (less stable, double bond
4-Methoxy-
is less highly substituted)
2 'litrobenzaldehyde
O Further, enol of pentane-2,4-dione is more stable

e
2
than that of 3-oxobutanal since it has a more highly
substituted double bond.

re
32. CH3 41. Structure (III) is stabilized by H-bonding
O—H——O
5

F
Frl
l-Cyclohexa-2, 4-dienyIethanone CH3—C CH—C—CH3 and hence it is the
(Functional group is given preference). (III)
33. N-Phenylcyclohexanecarboxamide.
34. Cyclohexylidenemethanone. ou most stable tautomer. Since C = O JC-bond
(364 kJ mol“^) of keto form is more stable than

sr
35. Isomers have same molecular formula but different C = C JC-bond of enol form (254 kJ mol“^),

kfo
structures. Buta-1,3-diene, Le., option {b) has only therefore, structure (II) with two C = O Jt-bonds is
four carbon atoms but all others have five carbon more stable than structure (I) with one C = O and
atoms. Therefore, ib) is not the isomer of the rest one C = C JC-bond. Thus, option {d) is correct.
of the compounds.
oo 42. 9 isomers are possible. Refer to illustration 1,
page 12/66.
37. Urea shows tautomerism. (Refer to page 12/65)
Y
44. Nucleophiles are electron rich species and hence
B
O OH they are Lewis bases.
I
H2N—C—NH2 ^ HN=C—NH2 45. H30'*’ cannot act as an electrophile since its octet
is already complete and hence has no room to
re
uY

a a accept an additional pair of electrons.


38. 2-Pentanone, CH3COCH2CH2CH3 contains a- 46. Nitrogen in option (c) has two double bonds and
hydrogens on a saturated carbon and hence shows one single bond. Therefore, it has 10 (2 x 4 + 1 x 2)
ad

electrons in the valence shell. This is against octet


do

tautomerism.
rule and hence resonance structure (c) is most
H H unlikely.
I
a - Bonds = 9
I
47. Resonance structure (a) in which the positive
in

39. H—C = C—C H Jt-Bonds = 1


I I
Lone pairs = 2 charges are present on the adjacent atoms is
Re

H—O: H
expected to be least stable.
F

Enol form of acetone

40. As compared to simple aldehydes/ketones, 1, 3-


diketones/aldehydes have higher enol content due
to stabilization by H-bonding.

o o d
<■
ib) 4 2
III

Pentane-2,4-dione (more stable, double bond Thus, IV is not the resonating structure of I, II and
is more highly substituted) m.
ORGANIC CHEMISTRY-SOME BASIC PRINCIPLES AND TECHNIQUES 12/147

52. (CH3)3C^ has the maximum number of 9 a- 59. More stable the free radical formed upon
hydrogens and hence most useful in explaining homolytic fission of C-H bond, lesser is the bond
its stability. dissociation energy. Since the stability of the
53. More stable the carbocation, more facile is C-Cl radicals formed from toluene, benzene, n-pentane,
bond ionization. Now the stability of carbocations 2, 2-dimethylpropane follows the order :
decreases in the order :
CgHj CH2 > (CH3)3 C > CH3 CHCH2CH2CH3
(CH3)3C+ > CgHjCHJ > (CH3)2CH+ > CH3CH+

ow
> C6H5,
Thus, option (d) is correct.
therefore, the C-H bond of toluene has the lowest
54. Electron-donating groups (OCH3 > CH3) increase
bond dissociation energy.
while electron-withdrawing groups (NO2)
decrease the stability of substituted carbonium ions 60. Carbocation (a) is stabilized by + I-effect or hyper
w.r.t. parent carbonium ion. Thus, option (c) is conjugation effect of the CH3 group and resonance

e
correct. effect of the CgH5 group.

re
rFl
.CH2
55. (CH3 >3 C (II) is stabilized by nine, CH3 CHCH3 61. (a) ●>—CH3 ib)

F
H
(IV) is stabilized by six, CH3CH2CH+ (III) is Stabilized by resonance Stabilized only by
stabilized by two while (CH3)3CCHj (I) has no and hyperconjugation hyperconjugation

r
hyperconjugation structure. Therefore, II is the effect of the CH3 group effect of one a-H

fo
ou
most stable, followed IV, then by HI while I is the (most stable).
least stable. Thus, option (c) is correct.
56. In benzyl cation (III), +ve charge is dispersed
through resonance over 7 carbon atoms while in
ks
(c) CH2 = CH—CH2 (^0 CH2 =CH
Stabilized by resonance only ; Not stabilized by
oo
allyl cation (I), it is dispersed over only three
no hyperconjugation effect any of these effects
carbon atoms, therefore, cation (HI) is more stable
Y
than (I). The n-propyl carbocation (II) is stabilized Thus, option (a) is correct.
eB

only by the weak + I-effect of the QI3CH2 group 64. Both naphthalene and benzoic acid sublime on
and hence it is the least stable of the three heating and hence cannot be separated by
carbocations. Thus, option (a), i.e., m > I > n. sublimation. Benzoic acid is, however, soluble in
ur

hot water but not naphthalene and hence can be


57. (CH3)2CH-(R) < CHJ (P) < CH2 = CHCH- (S) separated by crystallization.
ad
Yo

<CgH5CH-(Q) 72. The mobile phase in paper chromatography, of


course, is a liquid while the stationary phase is
Refer to page 12/88. Thus, option (b) is correct. also a liquid. It is the water which is adsorbed or
58. The stability of the carbanion decreases as
d

chemically bound to the cellulose, i.e., paper.


the electronegativity of the carbon carrying the Thus, option (d) is correct.
Re
in

-ve charge decreases or the hybridization of 73. Any organic compound which contains chlorine
the carbon carrying the -ve charge changes from when heated in a flame in presence of copper
F

sp - sp^ - sp^. Thus, RC = C“ (/■) is the most stable produces a bluish blue flame due to formation of
while R3C—CH2 (jv) is the least stable carb volatile cupic chloride.
anion. The carbon atom carrying the -ve charge 74. Ferric ferrocyanide, Fe4[Fe(CN)g]3.
in both CgHj and R2C = C 1“ is jp^-hybridized. 75. Cone. HNO3 decomposes Na2S and NaCN, if
formed.
Out of these two, R2C = CH" (Hi) is less stable
76. NH2NH2 does not have carbon to form NaCN
than CgHj (//) due to +I-effect of the two R needed for +ve Lassaigne’s test.
groups. Thus, the overall stability decreases in the 77. Thiourea (NH2CSNH2) contains both N and S and
order ; (/) > (if) > (iif) > (iv), i.e., option (c) is hence gives blood red colouration in Lassaigne’s
correct. test.
12/148 New Course Chemistry fxnmawn

12 0-44
= (14/121) X 100 = 11-57% and in thiourea
78. %C = —X xl00 = 40 (NH2 CSNHj) = (28/76) x 100 = 36-84%

w
44 0-30
Thus, option (c) is correct.
2 0-18
%H=—X xlOO = 6-66 80 0-141
18 0-30
188^0-250 xlOO =24

%0 = 100 - (40-0 6-66) = 53-34


32 0-699
84. %S = xl00 = 20
40
NowC:H:0 = —
6-66 53-34 233^0-480

ro
12 1-0 16

re
= 3-33 : 6-66 : 3-33 n. Asserdon-Reason lype Questions
E.F. = CH20 85. Correct reason. Double bonds are given
preference over triple bonds.

F
E.F. wt. = 12 + 2 -H6 = 30

Fl
But Mol. wt. = 60 86. Correct explanation. They differ by a CH2 group.

u
60 87. CorrectA. The lUPACnameof o-hydroxybenzyl
... M.F. = CH20x— =C2H402 chloride is 2-chloromethylphenol.
88. Correct assertion. CH3CHO and

sr
79. Vi=40mL,V2atNTP = ?
CH9 = CHOH are tautomers.

ko
Pj = 725 - 25 = 7(X) mm, P2 = 760 nun o 89. Correct A. R - N = C and R - C s N are not'
Ti = 300 K,T2 = 273K

of
tautomers but are junctional isomers since their

.-. V2 =_ ^700x40x273 = 33-52 mL properties are different from each other.


PjT, 760x300 91. Correct A. The Lassaigne’s extract is acidified
o
Y
with acetic acid before adding lead acetate.
%N = 28 ^^33-52 XlOO = 16*76
erB

22400^ 0-25 93. Correct assertion. Silver nitrate test cannot be


used to detect fluorine in the organic compound.
14x VolumexMolarityxBasicityof acid
uY

81. %N =
Weight of the substance taken
94. CorrectR.R—N=C: *●
A ►
+
R_N=C:"
1-4x10x1x2
= 37*33 Due to the presence of a lone pair of electrons on
0-75 C, it accepts a proton and hence acts as an
ad
do

82. Let the vol. of acid left unused electrophile.


= umLof0-lMH2SO4 95. Correct explanation. Due to the presence of a
lone pair of electrons on O, it acts as a
in

Applying molarity equation (page 12/118),


we have,
nucleophile. Since O has already 8 electrons it
cannot accept more electrons and hence it does
Re

2 X 0-1 X V = 1 X 0-5 X 20
not act as an electrophile.
F

or u = 50mL
In contrast, due to the presence of a +\e charge
.-.Vol. of acid used = 100 - 50
on O, cannot donate a pair of electrons
= 50mLof0-lMH2SO4
and hence it does not act as a nucleophile. Further
%N =
14xn>l,V, since O has already 8 electrons in the valence shell
wt. of substance taken it cannot expand its valence shell beyond 8 and
hence it caimot act as an electrophile.
1-4x2x0-1x50
= 46*6 96. Correct A. Heterolytic fission of CH3CH2CH3
fr3
gives CH3CH+ and CH".
%N in urea (NH2CONH2) = (28/60) x 100
= 46-6%, in acetamide (CH3CONH2) = (14/59) 98. Correct reason : Only the carbon atoms of the
X 100 = 23-72%, in benzamide (C6H5CONH2) double bond in but-2-ene are sp^-hybridized.

' #
ORGANIC CHEMISTRY-SOME BASIC PRINCIPLES AND TECHNIQUES 12/149

r
101. Correct assertion. Free radicals can be either
planar or pyramidal.
+ H2SO4
102. Correct R. Acetylene is more acidic than ethylene. SO3H + H2O
103. Correct assertion. The central carbon is sp- Thiophene Thiophene-2-sulphonic acid
liybridized while the terminal carbons are sp^-

o
(Dissolves in cone. H2SO4)

lu
hybridized.

F
benzene forms the upper layer. After washing with
104. Correct explanation. When commercial ben
NaHCO^, upper layer of benzene is separated and
zene is shaken with cone. H2SO4 in a .separating finally distilled to get pure benzene.
funnel, thiophene undergoes sulphonation to give
thiophene-2-suIphonic acid which dissolves in 105. Correct A. Nitrogen present in m-dinitrobenzene

F
H2SO4 and forms the lower layer while cannot be estimated by Kjeldahl’s method.

s
o
kr
=1 PROeilEMS

o
I. Hybridization, Classification and Nomenclature

o
1. Write the state of hybridization of all the atoms in CHj = C = CH2 and draw its orbital structure. Do

of
the two atoms on C| lie in the same plane in which hydrogens on C3 lie ? Explain.

Y
Y
Ans. The gross structure of allene (C3H4) is
1 2 3
B
CH2 = C = CH2
The carbon atoms I and 3 are sp^- hybridized since each one of them is joined by a double bond. In
contrast, carbon atom 2 is sp-hybridized since it has two double bonds. Thus, the two 71-bonds in allene like
r
in acetylene are perpendicular to each other as shown below :
d
e
u

2py 2py
7t-bond
7t-bond 2pz
n
o

a-bond o-bond
He
;c c
ad

Hb Hd
i

7t-bond

7t-bond
F
Re

Whereas and lie in the plane of the paper while and lie in a plane perpendicular to the plane of
the paper. Thus, the molecule as a whole is non-planar.
2. Give the lUPAC name for the amine.

CH3
I
CH3—N—C—CHjCHj
I
CH3C2H5
Ans. N, N, 3-Trimethylpentan-3-amine (Alphabets are given preference over numerals).
(Please remember that italic capitals (i.e., N, S, O, etc.) and lower case letters (i.e., n, o, m, p, etc.) have
higher priority of citation than Greek letters, i.e., a, P, y, ...etc.) which, in turn, have higher priority of
citation than arabic numerals (i.e., 1, 2, 3,4, ....etc.). For example, N,cn, 1,2 have higher priority of citation
than 1, 2, 3, 4
12/150 New Course Chemistry (XI)CEIHI]

II. Vsomerism

3, Write metamers of OCH2CH3

An.s. The various melamers are : (I) and (II), (I) and (III), (I) and (IV) and (I) and (V):

OCH2CH3
I

w
CH3 CH3

CH2OCH3 or OCH3 or OCH3 or CII3- OCH3

F lo
II III !V V

Isomers (HI, IV and V) are also position isomers.


4. Write the structures and lUPAC names of the functional isomers having the molecular formula,

ee
C2H5O2N.

Fr
Ans. Three functional isomers and their lUPAC names are :

+ .^0

for
CH3CH2—N CH3CH2—O—N = 0 H2N—CH^—C—OH
ur
0“
Nitroelhane Ethyl nitrite Aminoelhanoic acid
{Glycine)
s
III. Fundamental Concepts of Organic Reaction Mechanism
ok
Yo
5. Write resonance structures of the given compound. (IIT 2000)
o

H3C, .CH2
eB

OH

Ans. The following four re.sonance structures can be written for the given compound.
r
ad
ou

CH2 .CH2 CH2 .CH2


<—► ◄—►

QOH :OH
Y

:OH

o
Re
nd

HN NH (AlPMT Mains 2009)


6. Write down resonating structures and tautomers of
Fi

O O
H

O OH O

+ +

HN NH N N HN NH HN NH
Ans.
+

HO' N OH 0 "O N O"


0 0
H H

V'
Tautomers Resonating structures
ORGANIC CHEMISTRY-SOME BASIC PRINCIPLES AND TECHNIQUES 12/151

7. Explain why exists in keto form but exists in the enol form ?

w
O OH

O OH
Ans.

I II III IV

o
Enol form (II) is not stabilized and hence it exists in the keto form (I)

e
Enol form (IV) is stabilized by resonance energy (151 kJ mol"*) of the benzene ring and hence (HI) exists

re
in the enolic form (TV).

Frl
8. Out of HO“ or HS“ which one is (a) better nucleophile and (A) a better base ?

F
Ans. {a) S is less electronegative than O, therefore, it can donate a pair of electrons more easily than O. Thus,
HS~ is a better nucleophile than HO~.
(b) Since O—H bond is stronger than S—H bond, therefore HO“ has a greater tendency to accept a proton
ou
than HS“ and hence HO~ is a better base than HS~.

or
9. H30'*’ or RN^ neither acts as an electrophUe nor as a nucleophile. Explain why ?

kfs
Ans. HjO^ has a lone pair of electrons but due to the presence of +ve charge, it cannot donate its electron pair
and hence it does not act as a nucleophile. R4N'^, however, does not have a lone pair of electrons, therefore,
it does not act as a nucleophile.
oo
HjO'*' has 8 electrons in the valence shell. It cannot expand its valence shell beyond 8 due to the absence
Y
of fr-orbitals. Therefore, it does not act as an electrophile. Similarly, R^N also has 8 electrons in the
eB

valence shell. Like O, N also cannot expand its valence shell beyond 8 and hence it also does not act as an
electrophile.
ur

Thus, H30'*' or RN^ neither acts as a nucleophile nor as an electrophile.


oY

10. RNC acts only as a nucleophile but not an electrophile. Explain why ?
ad

Ans. R—N = C: . Although N has a +ve charge but it cannot accept a pair of electrons because it has already
8 electrons in the valence shell and cannot expand its valence shell beyond 8 due to the absence of d-
orbitals. Therefore, it cannot act as an electrophile. However, due to the presence of a -ve charge on
d

carbon, it can easily donate a pair of electrons and hence can act as a nucleophile.
in

11. Although C-D bond is stronger than C-H bond, yet (CH3)3C*’ (i) is more stable than (CD3)3C*’
Re

(li) Why so ?
Ans. Both carbocations (/) and (i7) are stabilized by hyperconjugation as shown below :
F

H H^ H
CH3 CH3
(0 H- c—c: < ► H—C=C ◄ ► H-^ C=C' ◄ ►

CH3 CH3 CH3


H H H
I II III

H
CH3
H—C=C etc.

CH3
H+
IV
12^52 New Course Chemistry (XI)Bs]9D

D D*
CD3 CD3 CD3
(iT) D—C^C \ ► D—C=C
\
A ► D* C=C
\
◄ ►

CD3 I CD3 CD3


D D D
V VI vn

D—C=C
? CD3
◄ ► etc.

+D CD3

w
vm

Due to stronger C—bond, contribution of structures (V-Vni) towards stability of carbocation, (003)3(1^ is
less than those of structures (I-IV) for carbocation, (CH3)3C^ therefore, carbocation (0 is more stable than

F lo
carbocation (I’i). This effect is also sometimes called as p- or secondary isotope effect.

12. Although F has high -I-effect, yet F3C^ is more stable than F3C—C^. Explain why is it so ?

ee
Ans. In carbocation, F3C—C the strongly electron withdrawing F3C- group withdraws electrons of the

Fr
C—C bond towards itself thereby intensifying the +ve charge and thus destabilising the carbocation.
Empty
2/M)rt>ital

for
^ FtUed
2p-oibitaI
ur
F
s
F—«-C-<«-C FUlcd <
ook
2p-orbitaI
Yo
F
eB

On the other hand, in carbocation F3C*", the lone pair of electrons on each of the three F-atoms overlap with
the empty 2p-orbital of the carbon atom carrying the +ve charge thereby dispersing the +ve charge and thus
r

+ ^
ad
ou

stabilizing the carbon cation, F3C‘‘ relative to F^C—C .

13. Why less stable carbocations tend to rearrange to more stable carbocations but carbanions and free
radicals do not ? WHhy so ? Explain.
Y

Ans. Less stable carbocations rearrange to the more stable carbocation by either a 1,2-hydride shift or a 1, 2-methyl
shift. For example.
Re
nd

CH3 CH3
+ U-Hydride
Fi

CH3—C—CH—CH3 CH3—C—CHj—CH3

(U
shift
3® carbocation
(more stable)
2® carbocation
{less stable)
CH3
1,2-Methyl I
CH3—C—CH2CH3
shift
3® carbocation
{more stable)
1® carbocation
{less stable)
ORGANIC CHEMISTRY-SOME BASIC PRINCIPLES AND TECHNIQUES 12/153

During 1,2-hydride or methyl shift, H or CH3 ion with its pair of electrons moves to the adjacent carbon of the
carbocation which has an empty p-orbital. Since an orbital can have at the maximum of two electrons according
to Pauli’s exclusion principle, therefore, the rearrangement occurs.
In contrast, in free radicals or the carbanions, the adjacent carbon has already either one or two electrons and
hence cannot accommodate two more electrons donated by H“ or CHJ ion and hence carbanions and free
radicals normally do not rearrange.

w
14. Although carbocations are always planar but carbanions and free radicals can assume either planar
or pyramidal geometry. Why is it so ? Explain.
Ans. The simple alkyl carbanions are pyramidal. For example.

CH3, CH3-CH2, (CH3)3Cr, CH2

o
(Pyramidal carbanions)

e
However, carbanions in which the C-atom carrying the negative charge is adjacent to a double bond or a benzene

re
rFl
ring, are planar due to stabilization by resonance. Thus, allyl and benzyl carbanions are planar.

F
CH2^CH^CH2 <
A- > -:ch2—ch=ch2
sp2
Allyl carbanion {planar)

r
ou
fo
● ●

●o-

CH2 ^ etc.

Benzyl carbanion {planar)


ks
oo
Most of the free radicals are planar but free radicals in which the carbon atom carrying the odd electron is
connected to a bridge head carbon page 12/126 or highly electronegative element are pyramidal. For example,
Y
●CF3 has pyramidal shape.
eB

15. Conversion of 1-bromobutane (I) to 2-bromobutane (U) is called isomerisation reaction. Will the
conversion (I) to l-bromo-2-methylpropane (111) be also called an isomerization reaction ? Justify
your answer:
ur

Br
ad
Yo

CH3CH2—CH—CH3
ai)
CH3CH2CH2CH2—Br CH3
d

(D
CH3—CH—CH2Br
Re
in

(III)
Ans. No. The reason being that the isomerization involves conversion of one isomer into the other keeping the
F

carbon skeleton of the two isomers intact while rearrangement reaction involves migration of one atom or
a group from one atom to the other within the same molecule with simultaneous change of the carbon
skeleton of the molecule.
Thus, the conversion of isomer (I) to isomer (II) is an isomerization reaction while conversion of isomer (I)
to isomer (III) is called a rearrangement reaction.
16. Classify the following transformations according to the reaction type,
(a) H3C—CH = CH—CH3 + Br2 > H3C—CHBr—CHBr—CH3
(b) (H3O2C = C(CH3)2 + Brj > (H3O2C = C(Cnf)CRf^r + HBr
(c) H2C = CH—CH2CH3 > H3C—CH = CH—CH3
(d) CfiHgCHO + CH3COCH3 > C6HsCH(OH)CH2COCH3
12/154 New Course Chemistry (XHcrawi

(e) (CH3)3CC1 + HO- > (CH3)2C = CHj


if) CH3—C = N—0H+ H3O+ > CH3—C= O
Ph NHPh

Ans. {a) Electrophilic addition (b) Free radical substitution


(c) Isomerisation {d) Condensation
(e) P-EIimination reaction (/) Rearrangement.
17. With proper Justification, arrange the following in order of increasing stability

CH3CH- , CH = C", CH, = CH-

w
Ans. In acetylide ion CH = C", the carbon atom carrying the -ve charge is .vp*hybridized and has 50%
^-character ; in CH2 = CH" ion. the carbon atom is sp~- hybridized and has 33-3% .^-character while in

F lo
CH3CH2 ion, the carbon atom bearing the - ve charge is ,9/7-^-hybridizcd and has 25% ^-character.
Since ^-electrons, on the average, are closer to the nucleus than /j-electrons. therefore, a carbon atom with
greater .^-character can accommodate or stabilize the negative charge better than a Cenbon atom with smaller
5-character. In other words, the stability of the carbanion increases as the 5-character of the carbon atom

ee
carrying the negative charge increases. Now since the .v-character of the carbon decreases as we move from

Fr
sp to sp- to ,y/7^-carbon, therefore, the relative stability of the three carbanions follows the sequence:
CH = C“ > CH2 = CH- > CH3CH2 .

for
18. Carbenes are regarded as neutral bivalent carbon species. Explain why ?
Ans. The simplest carbene is methylene (: CH2). Since it has two C—H bonds, therefore, it has two shared pairs of
our
electrons or four electrons and hence is bivalent.

Formal charge on csirbon in : CH, = No. of non-bonding electrons -1-— [No. of electrons forming bonds]
s
ook

= 2+-(4) = 2+2 = 4
2
Y
eB

Since carbon has 4 electrons in the valence shell and carbon in : CH, also has 4 electrons, therefore,: CH2 is
neutral.

IV. Purification and Separation of Organic Compounds


our
ad

19. Suggest a method to purify :


(1) Camphor containing traces of common salt.
(k) Kerosene oil containing water.
Y

{in) A liquid which decomposes at its boiling point.


Re

Ans. (0 Sublimation. Camphor sublimes while common sail remains as residue in the china dish.
nd

(») Since the two liquids are immiscible, the technique of solvent extraction with a separating funnel is
used. The mixture is thoroughly shaken and the separating funnel is allowed to stand. Kerosene being
Fi

lighter than water forms the upper layer while water forms the lower layer. The lower water layer is run off
when kerosene oil is obtained. It is dried over anhydrous CaCl2 or MgS04 and then distilled to give pure
kerosene oil.

(Hi) Distillation under reduced pressure. Since the b.p. of a liquid depends upon the pressure acting on it.
Therefore, a liquid which decomposes at its b.p. can be purified safely at a lower temperature if the pressure
acting on it is reduced.
20. A mixture contains two components A and B. The solubilities of A and B in water near their boiling
point are 10 grams per 100 mL and 2 g per 100 mL respectively. How will you separate A and B from
this mixture ?

Ans. Fractional crystallization. When the saturated hot solution of this mixture is allowed to cool, the less soluble
component B crystallizes out first leaving the more soluble component A in the mother liquor.
ORGANIC CHEMISTRY-SOME PRINCIPLES AND TECHNIQUES 12/155

21. Suggest methods for the separation of the following mixtures,


(i) A mixture of liquid A (b.p. 365 K) and liquid B (b.p. 356 K).
(i7) A mixture of liquid C (b.p. 353 K) and liquid D (b.p. 413 K).
Ans. (0 Fractional distillation because the boiling points of the two liquids differ by just 9°.
(ii) Simple distillation since the boiling points of the two liquids are widely apart.
22. A mixture of X and Y was loaded in the column of silica. It was eluted by alcohol-water mixture.

ow
Compound Y eluted in preference to compound X. Compare the extent of adsorption of X and Y on
column. (CBSE Sample Paper 2016)
Ans. A compound which is more strongly adsorbed on the column is eluted later than the compound which is
weakly adsorbed. Since compound Y is eluted in preference to compound X. therefore, compound X is
more strongly adsorbed than the compound Y.
23. The Revalues of A and B in a mixture determined by TLC in a solvent mixture are 0*65 and 042

e
respectively. If the mixture is separated by column chromatography using the same solvent mixture

re
as a mobile phase, which of the two components,A or B, will elute first ? Explain.

rFl
Ans. Since the value of A is 0 65, therefore, it is less strongly adsorbed as compared to component B with

F
value of 042. Therefore, on extraction of the column. A will elute first.
24. A mixture contains benzoic acid and nitrobenzene. How can this mixture be separated into its
constituents by the technique of extraction using an appropriate chemical reagent ?
Ans. The mixture is shaken with a dilute solution of NaHC03 and extracted with ether or chloroform when

or
ou
nitrobenzene goes into the organic layer. Distillation of the solvent gives nitrobenzene. The filtrate is acidified
with dil. HCl when benzoic acid gets precipitated. The solution is cooled and benzoic acid is obtained by

ksf
filtration.

25. Without using column chromatography, how will you separate a mixture of camphor and benzoic
acid ?
oo
Ans. Sublimation cannot be used since both camphor and benzoic acid sublime on heating. Therefore, a chemical
method using NaHC03 solution is used when benzoic acid dissolves leaving camphor behind. The filtrate
Y
is cooled and then acidified with dil. HCl to get benzoic acid.
B

O
re

26. Out of and


C—CHj which will be eluted first in moderately polar
oYu

solvent and why ? ; \IPMT 2005)


ad

Ans. Q— has significant dipole moment while being symmetrical has zero
d

dipole moment. Therefore, di-tert- butylbenzene is non-polar and hence will not be adsorbed strongly on
in
Re

the adsorbent and hence it will be eluted first in moderately polar solvent.
27. A reaction is carried out using aniline as a reactant as well as a solvent. How will you remove unreacted
F

aniline ?

Ans. The b.p. of aniline (457 K) is very high. If aniline (large excess) is distilled from a small amount of the
product by simple distillation, it may cause decomposition of the product. Therefore, to avoid decomposition,
of the product, aniline is removed either by vacuum distillation or by steam distillation.
28. Commercial benzene obtained from coal-tar distillation contains 3-5% thiophene as impurity. Suggest
a simple method to purify it.
Ans. It is usually not easy to separate thiophene from benzene by fractional distillation because their boiling
points are very close : benzene = 353 K, thiophene = 357 K.
However, thiophene can be removed from commercial benzene by extraction with cone. H2SO4. This
purification is based upon the fact that thiophene undergoes sulphonation much more easily than benzene.
Thus, when commercial benzene is shaken with cone. H2SO4 in a separating funnel, thiophene undergoes
sulphonation to form thiophene-2-sulphonic acid which dissolves in cone. H->S04 while benzene does not
12/156 -iv bourse Chemistry fXIUTZSTTI

Room temp.
+ H2SO4 {cone.) + H2O
S
/ SO3H
s
Thiophene Thiophcne-2-sulphonic acid
(Disxohex in cone. H2SO4)
29. During hearing a court case, the judge suspected that some changes in the original documents had
been deliberately made. He asked the forensic department to check the ink used at two different
places.
After reading the above passage, answer the following questions :
(i) What technique do you think can be used to ascertain whether the ink used at two places was the
same or different ?

w
(«) What is the principle of this technique applied ?
Ans.
(0 Thin layer chromatography (TLC) can be used to identify the componentsin a small sample such as ink.
(«) This technique is based upon the principle of adsorption. To carry out this technique, a glass or a plastic

F lo
slide coated with a suitable adsorbent usually silica gel is taken. Two pencil lines are drawn on this plate—
one little above the bottom and the other near the lop of the plate. With the help of a suitable solvent, the
samples of ink used at two different places are applied on the pencil line near the botom of the TLC plate
little away from each other. The plate is then placed in a closedjar containinga suitable solvent and allowed

ee
to stay till the solvent reaches the top pencil line. The plate is then removed from the jar and dried in air. If
both the samples of ink show a single spot on the plate at the same height from the bottom then the same ink

Fr
is used and if single spots are either seen at different heights or one sample shows a single spot while the
other sample shows two or more spots, then different inks are used.

for
V. Qualitative and Quantitative Analysis
ur
30. Will sodium cyanide give a positive Lassaigne’s test for nitrogen ?
Ans. Yes. During fusion with Na metal, the N of the organic compound is actually converted into NaCN which
is the primary requirement for Lassaigne’s test for nitrogen,
s
ook
31. NH2NH2 contains nitrogen but still it does not give po.sitive Lassaigne’stest ? To get positive test for
Yo
nitrogen in NH2NH, what modification do you suggest ?
eB

Ans. If hydrazine (NH2NH2) is fused with Na metal, it does not form NaCN since it does not contain carbon and
hence will not give +ve test for nitrogen. In order to test the presence of N in such compounds, during
fusion with Na, some charcoal or preferably starch (which contains C but not N, S, halogens,etc.) is added.
Under these conditions. C of starch or charcoal combines with N of the compound to form NaCN which
our

will now give a +ve test for nitrogen.


ad

●Very Short Answer


●Short Answer
Y

● Long Answer
Re
nd

VERY SHORT ANSWER QUESTIONS Carrying 1 mark


Fi

I. Hybridization Classification and Nomenclature


I. Give the common and lUPAC name of an alkane having the lowest molecular mass that contains a
quaternary carbon.
CH,
I
Ans. CH
3 -C—CH3 Common name : Neopentane
lUPAC name : 2, 2-DimethyIpropane
CH3
2. Arrange the following in increasing order of C—C bond length : C2H^, C2H4, C2H2.
Ans. C2H, (I20 pm) < C2H4 (134 pm) < CjHf, (154 pm).
ORGANIC CHEMISTRY-SOME BASIC PRINCIPLES AND TECHNIQUES 12/157

3. Name a functional group whose lUPAC name has only a prefix but no suffix. Give one example.
Ans. Nitro (—NO2) group has only a preifx but no sujfix.
NO2 NO.

CH3—NO2

Nitromcthane Nifrocyclohexanc Nitrobenzene

CH,

4. Write the lUPAC name of (i) CH^—C = CHCOOCH^, (il)

w
Cl

Ans. («) (i) Methyl 3-chlorobut-2-en-l-oate, (it) 3-Methylcycloh exyne

F lo
5. Give the lUPAC names of the following compounds :
CHO ●CHO

(/) (ii) (Hi)

ee
CHO

Ans. (/) 4-Methylpenianal (//) Cyclohexanecarbaldehyde (Hi) Benzenecarbaldehyde.

Fr
6. Give the lUPAC names of the following bifunctionalcompounds
COOH COOH

for
O O
CHO
our
(i) (H) (Hi)
H 'OH
CHO
s
Ans. (1) 5-Oxopentanoic acid (ii) 2-Formylcyclopentane-l-carboxy lic acid (Hi) 3-Formyibenzoic acid.
ook
7. Name the alkyl groups derived from isobutane.
Ans. (0 (CH3)2CHCH2 - 2-Methylpropyl (isobutyl) and (CH3)3C - 1, 1-DimethylethyI (lert- butyl).
Y
eB

II. Isomerism

8. A compound is formed by the substitution of two chlorine atoms for two hydrogen atoms in propane.
What is the number of structural isomers possible ?
r
ad
ou

Ans. Four : 1, I-dichloropropane (CH3CH2CHCI.), 1, 2-dichloropropane (CH3CHCICH2 Cl), 2, 2-dichloropropane


(CH3CC1^CH3) and 1, 3-dichloropropane (CICH2CH2CH2CI).
9. How many metamers of 3-pentanone are possible ? Write their structures and lUPAC names. Can
Y

these be regarded as position isomers as well ?


Ans. Two metamers (I and II) of 3-pentanone are possible.
Re
nd

O O 0 CH3
CH3—C—CH—CH3
Fi

CH3CH2—C—CH2CH3 CH3—C—CH2CH2CH3
3-Pentanone 2-Pentanone (I) 3-Methyl-2-butanone (II)
Both these can also be regarded as position isomers of 3-pentanone because they differ in the position of
the keto group.
10. Write the position isomers of diethyl ether. Write their structures and IIJPAC names. Can these be
also regarded as metamers ?
Ans. Two position isomers (I and II) are possible of diethyl ether.
CH3
CH3CH2—O—CH2CH3 CH3O—CH2CH2CH3 CH3O—CH—CH3
Diethyl ether 1-Meihoxypropane (I) 2-Methoxypropane (II)
12/158 7\<xeUe^'A New Course Chemistry rxnrosmm
Both these can also be regarded as metamers of diethyl ether because these differ in the number of carbon
atoms on either side of the ethereal oxygen.
11. Write the tautomer of acetaldehyde and give its lUPAC name. Which of the two is more stable and
why ?

O ,011
Ans. H CHo = C
H H
Acciaickhyilc Elh-l-en-l-ol
(more slahle) (less srahle)

The tautomer, eth-l-en-l-oi (i.e., vinyl alcohol) is less stable than acetaldehyde because C = O 7i-bond

w
(364 kJ mol"*) is much stronger than C = C 7i-bond (254 kj mol"*).
12. The molecular formula CjH^O represents two isomers containing a carbon-carbon double bond.
Write their structures and give their lUPAC names.

F lo
Ans. The two isomers are :

CH^ =CH—CH.,OH CH2 = CH—0—CH 3


Pri)p-2-cii-a-ol Mcihoxyethenc

ee
13. Write the structure of the tautomer of phenol and give its lUPAC name. Which of the two tautomers
is more stable and why ?

Fr
O H 0
'J-

for
1-1

Ans. H
ur
Phenol Cyclohexa-2,4-dien-l-one
(more stable) (less stable)
s
ook
Phenol is more stable than cyclohexa-2, 4-dien-l-one because phenol being an aromatic compound is
Yo
stabilized by the resonance energy of the benzene ring (151 kJ mol"*).
14. Nitromethane exists in the aci-form but nitrobenzene does not. Explain why ?
eB

Ans. Nitromethane has a-hydrogen and hence it exists in the ad-form.

O"
our
ad

H ^ CH2=N
O" O"
Niiro form .4c/-form

In contra.st, nitrobenzene does not have an a-hydrogen and hence it does not exist in the aci-form.
Y
Re
nd

a
NO. No ac/-form

NO a-hydrogen
Fi

Fundamental concepts of Organic Reaction Mechanism


15. Name two positively charged and two neutral electrophiles.
4*

Ans. Positive electrophiles : Cl'*’ (chloronium ion), NO2 (nitronium ion)


Neutral electrophiles : -R (free radical), : CR. (carbene)
16. Name two negatively charged and two neutral nucleophiles.
Ans. Negative nucleophiles : "CN (cyanide ion). RO" (alkoxide ion).
Neutral nucleophiles : R-NH, (amine). R-O-H (alcohol)
ORGANIC CHEMISTRY-SOME BASIC PRINCIPLES AND TECHNIQUES 12/159

17. How do SO3 and : CCI2 act as electrophiles ?


O

Ans. In SO3, 0 = S+ O , S atom carries a +ve charge and hence acts as an electrophile.

In :CCl2, C atom has six electrons in the valence shell and hence needs two more electrons to complete its
octet. As a result, it acts as an electrophile.
18. How do ROH and RSH act as nucleophiles ?

Ans. In ROH . O has two lone pairs of electrons and similarly in RSH . S has tw’o lone pair ot electrons. Both
of these can donate a pair of electrons and hence act as nucleophiles.

w
19. H2C = O or CH3CN acts as a nucleophile as well as an electrophile. Explain
Ans. Since both O and N are more electronegative than
C, therefore, C carries a small +ve charge and. O

F lo
H \ 5+ .. 0- +8 -6
and N carry a small -ve charge. ^c=o: CH3—c=n:*
In other words, both act as an electrophile due to the H
\ \
presence of a partial +ve charge on C and act as a Eleclrophilc Nucleophile Eieclrophilc Nucleophile
nucleophile due to the presence of a partial -ve

ee
charge on 0 or the N.

Fr
20. CH2 = CH" is a better nucleophile than HC s C”. Explain (All’MT 2008)

Ans. In H2C = CH“, the carbon atom carrying the -ve charge is j/j^-hybridized while in HC = C", the carbon
atom carrying the -ve charge is .?/?-hybridized. Since a .y/7^-hybridized carbon is less electronegative than a

for
5/)-hybridized carbon, therefore, H2C = CH" is a better nucleophile than HC = Cr.
ur
(AIPMT Mains 2009)
21. CHj = CH" is more basic than HC = C". Explain why ?
sp

Ans. CH2= CH" HC = C“


s
Since, 5p-carbon is more electronegative than ^/^“-carbon, therefore, CH =C" is less willing to donate a pair
ook
Yo
of electrons than H2C = CH". In other words, H2C = CH" is more basic than HC = C".
22. Write the resonance structures of aniline and explain which one is the most stable ?
eB

NH2
^NH2
●NH2 NH2 NH7

Ans. ^ <■
r

◄—►
ad
ou

tr
I II III TV V

Resonance structure (I) is neutral because it does not carry any charge but all others carry separation of +ve
Y

and -ve charges. Therefore, structure (I) is the mo.st stable.


23. Arrange the following : (i) - NO2, -COOH, -F, -CN, -I, in decreasing order of -I-effect.
Re
nd

(if) CH3-, D-, (CH3>3C-, (CH3>2CH- CH3CH,-, in increasing order of +1- effect.
Ans. (/) -NO2 > -CN > -COOH > - F > - I (i7) D- < CH3- < CH3CH7- < (CH3),CH- < -C(CH3>3.
Fi

24. Arrange the following :

(i)C^H5CHCH3,C^H5CHCH= CH2’C^H5CH2CH2 C^Hj;C(CH3)2 in order of increasing stabUity.


(ii) CH3CH+, C^H5CHJ,(CH3)3C+, CH2 =CHCH^ in order of decreasing stability.
(«0 HC = C", CH2 = CH", CH3CH2 , CH3 , (CH3)2CH", C^HjCH" in order of increasing stability.
Ans. (0 C^H5CH2CH2<CgHgCHCH3<CgHgC(CH3)3<C^HgCH—CH = CH2-
C^^HgCHj >CH2= CHCH+ > CH3CHJ. (Refer to page 12/85)

(m) (CH3)2CH-< CH3CH2 < CR- <CH2 = CH“< C^HgCH^ < HC = C”. (Refer to page 12/88)
12/160 New Course Chemistry fXIlraswr

IV. Purification of Organic Compounds


25. When do we use a fluted filter paper or hot water funnel for filtration ?
Ans. To avoid cryslallization during filtration, fiuted filter paper is used when the volume of the solution to be
filtered is small and hot water funnel when the volume is large.
26. How will you purify a liquid having non-volatile impurities ?
Ans. Simple distillation will give us the pure liquid while the non- volatile impurities will remain in the flask as
residue. For example, sea water containing non-volatile impurities of chlorides and sulphates of Na, K, Ca
and Mg can be purified by simple distillation.
27. How will you separate a mixture of urea and sodium chloride ?

w
Ans. Both urea and NaCl are soluble in water but urea is soluble in alcohol but NaCl is not. Therefore, this
mixture can be separated by shaking with alcohol when urea goes into solution while NaCl remains
undissolved. The mixture/suspension thus obtained is filtered. Evaporation/distillalion of the filtrate gives

F lo
urea while NaCl remains as a residue on the filter paper.
28. How will you purify essential oils ?
Ans. Essential oils are volatile and are insoluble in water. Therefore, they are purified by steam distillation.
29. How will you separate a mixture of o-nitrophenol and p-nitrophenol ?

ee
Ans. Steam distillation. o-Nitrophenol being steam volatile distils over along with water while /)-nitrophenol

Fr
being non-volatile remains in the flask.
30. Name two methods which can be safely used to purify aniline.

for
Ans. Vacuum distillation and steam distillation.

31. What type of compounds are purified by sublimation ?


ur
Ans. Substances whose vapour pressures become equal to the atmospheric pressure much below their melting
points. For example, naphthalene, benzoic acid, camphor, iodine, etc.
s
32. How will you separate iodine from sodium chloride ?
ook
Yo
Ans. Either by sublimation or by extraction with CCI4 followed by evaporation.
33. How will you separate a mixture of two organic compounds which have different solubilities in the
eB

same solvent ?

Ans. By fractional crystallization.


34. An organic liquid decomposes below its boiling point. How will you purify it ?
Ans. Distillation under reduced pressure, i.e.. vacuum distillation.
our
ad

35. Define the term ‘elution’ as applied to column chromatography.


Ans. It is the process of extraction of different compounds ad.sorbed on the column by means of a suitable
solvent called eluent.
Y

V. Qualitative and Quantitative Analysis


Re
nd

36. Will you get any precipitate if you add silver nitrate solution to chloromethane ? If not, why ?
Ans. No, because chloromethane is a covalent compound and hence doe.s not ionize to give Cl" ions to react with AgNO^
Fi

CH3—Cl -t- AgN03 > No reaction.

37. Name two compounds which do not contain halogen but give positive Beilstein test.
Ans. Urea and thiourea give positive Beilstein test due to the formation of volatile cuprous cyanide.
38. Why is a freshly prepared saturated solution of ferrous sulphate used in the Lassaigne’s test for nitrogen ?
Ans. On keeping aq. FeS04 solution, it undergoes oxidative-hydro lysis to form basic ferric sulphate
4 FeS04 -t- 2 H2O -1- O2 » 4 Fe(0H)S04
The pale yellow colour of Fe^'*' ions interferes with the light green colour usually obtained in Lassaigne’s test.
39. Lassaigne’s test is not shown by diazonium salts. Why ?
Ans. Diazonium salts usually lose N2 on healing much before they have a chance to react with fused sodium
metal. Therefore, diazonium salts do not show positive Lassaigne’s test for nitrogen.
ORGANIC CHEMISTRY-SOME BASIC PRINCIPLES AND TECHNIQUES 12/161

40. Write the molecular formula of iron (III) hexacyanoferrate (II).


Ans. Fe4[Fe(CN)g]3.
41. What conclusion would you draw if during Lassaigne’s test, a blood red colouration is obtained ?
Ans. The formation of blood red colouration during Lassaigne’s test indicates the presence of both N and S.
During fusion, sodium thiocyanate is formed which gives blood red colouration with FeCl^.
3 NaCNS + FeCl3 > Fe(CNS)3 + 3 NaCI
{Blood red)

42. What type of organic compounds cannot be Kjeldahlised ?


Ans. Compounds containing nitrogen atom in the irng and those compoundsin which nitrogen is directly linked

w
cither to oxygen or to another nitrogen atom such as in nitro (-NOi) and azo (-N = N-) compounds.
43. Can we estimate oxygen in an organic compound ?
Ans. Yes. But usually it is indirectly estimated by subtracting the sum of percentagesof all the elements present

Flo
in an organic compound from 100.
44. Find out the percentage of carbon in acetic acid.
2x12

ee
Ans. Mol. mass of acetic acid (CH3COOH) = 60 amu %C = X 100 = 40%
60

Fr
45. How is phosphorus estimated in an organic compound ?
Ans. Either as ammonium phosphomolybdale or as magnesium pyrophosphate.

for
SHORT ANSWER QUESTIONS
ur Carrying 2 or 3 marks

). Hybridization Classification and Nomenclature


ks
1. Explain tetravalency of carbon. [Art. 12.2]
Yo
2. Why does carbon undergo hybridization prior to bond formation ?
oo
[Art. 12.2]
3. Draw the orbital diagram for methane and ethane molecules indicating the hybridization involved.
eB

[Art. 12.2-12.3]
4. Discuss the orbital diagram of ethene indicating the hybridization involved and the nature of the bonds
formed. [Art. 12.3]
5. Describe the orbital diagram of acetylene indicating the hybridizationinvolved and the nature of bonds
r

formed. [Art. 12.3]


ou
ad

6. What is the effect of type of hybridization on (/) bond length (ii) bond strength. [Art. 12.3.2]
7. What are alicyclic compounds. Give two examples with their names. [Art. 12.6]
Y

8. What arc homocyclic and heterocyclic compounds ? Give one example of each type along with its name.
[Art. 12.6]
Re
nd

9. What is a functional group ? Write the functional groups of the following :


(0 Thioalcohol {ii) Isothiocyanate (Hi) Thiocyanate and (/v) Sulphonic acid (v) Sulphones (W) Sulphoxides.
Fi

O
[Ans. (/) -SH (//) -N = C = S (Hi) -S-C s N (/V) - SO3H (v) -S (v/) > S = 0] [Art. 12.7]
●O

10. What is homologous series ? Give its important characteristics. Write the first four homologues of alcohols
and give their lUPAC names. [Art. 12.7]
11. Explain the following terms with one example in each case, (0 word root (ii) primary and secondary suffixes
and prefixes. [Art. 12.9]
12. Pick out the eiror and write the correct lUPAC names for the following.
(0 3-Pentyne (//) 1. 6-Hexadiene (Hi) 2-Ethyl-2-pentene (iv) 1, 2-Dihydroxyethanc (v) Pent- 4-en-l-yne.
[Ans. (/) Pent-2-yne (ii) Hexa-1, 5-diene (Hi) 3-Methylhex-3-ene (iv) Ethane-1,2-diol (v) Pent-1 -en-4-yne.]
12/162 New Course Chemistry (XI)BSZ9I1

II. Isomerism

13. Explain : (/) position isomerism (//) functional isomerism giving one example in each case. [Art. 12.10.1]
14. Write tlie tautomers of (j) ethyl acetoacetate and (;7) acetylacetone. Explain why the latter has higher
percentage of enol than the former. [Art. 12.10.1]
15. Identify chiral and achiral molecules in each of the following pairs of compounds. [Art. 12.10.2]

H H

w
(A) OH Br (B)
H3C H3C CHj
Br Br

(i) (ii) (i) tin

F lo
(C) CH3—CH—CH2CII3 CH3—CHj—CHj—CH2—Br

ee
Br (i) (ii)
[Ans. Chiral : A, (/) Chiral, (//) achiral ; B : (/) chiral, (//) achiral, C : (t) chiral, (ii) achiral]

Fr
III. Fundamental concepts of Organic Reaction Mechanism
16. Explain the following with one example in each case.

for
(0 Homolytic fission
ur
(//) Helerolytic fission of covalent bonds. [Art. 12.11]
17. What are electrophiles and nucleophiles ? Explain with examples. s [Art 12.12]
18. Explain inductive and electromeric effect with examples. [Art. 12.13.1 to 12.13.2]
k
Yo
19. What is resonance ? How does resonance explain that all the carbon-carbon bond lengths in benzene are
oo
equal (139 pm) ? [Art 12.13.3]
20. Spectroscopic measurements indicate that the two oxygen atoms of sodium acetate are equivalent. Both the
eB

C—0 bonds have the same length (126 pm). Explain. [Art 12.13.3]
21. What is resonance effect ? What are its various types ? In what respects, does the resonance effect differ
from inductive effect ? [Art. 12.13.4]
r

22. Comment upon the statement: ‘Usual order of inductive effects of the alkyl groups is often reversed when
ou
ad

attached to a double bond or a benzene ring.’ Name the electronic effect and illustrate your answer with
suiUible examples. [Art. 12.13.5]
Y

23. Explain hyperconjugation effect. [Art. 12.13.5]


24. How does hyperconjugation effect explain the stability of alkenes ? [Art. 12.13.5]
Re
nd

25. What are carbocations ? Discuss their various types. [Art. 12.14.1]
26. Give two methods of preparation of carbocations. (Art. 12.14.1]
Fi

27. Discuss the configuration of carbocations. [Art. 12.14.1]


28. What are reactive intermediates ? How tu-e they generated by bond fission ? [Art. 12.12 and 12.14]
29. How can inductive and hyperconjugation effects explain the stability of primary, secondary and tertiary
carbocations ? [Art. 12.14]
30. What are carbanions ? Discuss their configuration. [Art. 12.14.2]
31. Give two methods of preparation of carbanions. [Art. 12.14.2]
32. Discuss the stability of carbanions on the basis of inductive effects. [Art. 12.14.2]
33. What are free radicals ? Discuss their configuration. [Art. 12.14.3]
34. How can hyperconjugation effect and inductive effect explain the stability of primary, secondary and tertiary
free radicals ? [Art. 12.14.3]
ORGANIC CHEMISTRY-SOME BASIC PRINCIPLES AND TECHNIQUES 12/163

IV. Purification of Organic Compounds


35. Explain the following reactions :
(0 Substitution (//) Addition (Hi) Elimination (iv) Rearrangement (v) Isomerization (v/) Condensation
(vii) Pericyclic. [Art. 12.15]
36. List the different methods used for the purification of organic compounds. [Art. 12.16]
37. How will you separate a mixture of benzoic acid and naphthalene ? [Art. 12.16.2]
38. How will you purify sugar which has impurities of sodium chloride? [Art. 12.16.2]
39. When is the process of fractional crystallisation employed ? [Art. 12.16.3j

w
40. Name and discuss the principle of the method used to separate a mixture of two organic compounds having
different solubilities. [Art. 12.16.3]
41. When is the process of fractional distillation employed ? [Art. 12.16.6]

F lo
42. What is a fractionating column ? How does it help in the separation of miscible liquids ? Give one example.
[Art. 12.11.6]
43. Glycerol decomposes at its boiling point (563 K). Discuss a method which can be used for its purification.

ee
[Art. 12.16.7]

Fr
44. Explain the principle of steam distillation. [Art. 12.16.8]
45. How will you separate two components when :

for
(ij) their boiling points differ by a few degrees [Art. 12.16.6]
(b) they are soluble in the same solvent [Art. 12.16.3]
ur
(c) they are almost immiscible in water but are volatile in steam ? [Art. 12.16.8]
46. What is the basic principle of chromatography ? Explain with one example the use of column chromatography
oks
in the purification of organic compounds. [Art. 12.16.10]
Yo
47. What is the principle of column chromatography? [Art. 12.16.10]
o
eB

V. Qualitative and Quantitative Analysis


48. How will you detect the presence of carbon and hydrogen in an organic compound ? [Art. 12.17.1]
49. Discuss the chemistry of Beilstein test for the detection of halogens. Why is this test not dependable ?
our
ad

[Art. 12.17.3]
50. How will you detect the presence of halogens in a given compound ? What is the function of nitric acid in
this test ? Give chemical equations. [Art. 12.17.3]
51. Describe the chemistry of Lassaigne’s test used for the detection of (i) Nitrogen, (//) Halogens, and (Hi)
Y

Sulphur. [Art. 12.17.2-12.17.4]


Re
nd

52. Write chemical equations involved in the detection of sulphur and phosphorus in organic compounds.
[Art. 12.17.4-12.17.5]
Fi

53. How can carbon and hydrogen be estimated in the organic compound ? Give outlines of the method along
with the method of calculation. [Art. 12.18.1]
54. Discuss the reactions and the principle underlying the estimation of the following :
(0 Carbon and hydrogen [Art. 12.18.1]
(«) Dumas method for estimation of nitrogen [Art. 12.18.2]
(Hi) Kjeldahl’s method for the estimation of nitrogen [Art. 12.18.2]
(iv) Carius method for the estimation of halogens and sulphur [Art. 12.18.3-12.18.4]
(v) Phosphorus. [Art. 12.18.5]

4
12/164 New Course Chemistry (X1)S!EIHD

LONG ANSWER QUESTIONS Carrying 5 or more marks

1. Discuss classification of hydrocarbons into various types and illustrate each class by taking two examples.
[Art. 12.6]
2. Explain the following giving examples :
(/) Functional group .(ii) Homologous series and its characteristics. [Art. 12.7 to 12.8]
3. What is mean by isomerism ? Discuss its various types giving at least one example in each case. [Art. 12.10]
4. Define lautomerism. Discuss briefly keto-enol tautomerism in aldehydes and ketones. Also discuss the
conditions under which enol form predominates. [Art. 12.10]
5. Give a brief account of the various types of electronic effects in a covalent molecule. [Art. 12.13]
6. Define and explain the term resonance with suitable examples. Comment upon the relative contributions of

w
various resonance structures. [Art. 12.13.3]
7. What is hyperconjugation effect ? How does it differ from resonance effect ? Briefly discuss the significance

F lo
of hyperconjugation effect. [Art. 12.13.4 and 12.13.5]
8. How are free radicals, ciu’bocations and carbanions produced ? Discuss their relative stabilities. [Art. 12.14]
9. Give an account of common types of organic reactions with suitable examples. [Art. 12.15]
10. Give a brief description of the principles of the following processes taking an example in each case.

e
(0 Filtration, {ii) Recrystallisation, {Hi) Sublimation, (fv) Distillation under reduced pressure ,

Fre
(v) Steam distillation, (vi) Extraction with a solvent. [Art. 12.16]

for
CASE-BASED VERY SHORT/SHORT QUESTIONS

CASE 1. Lassaigne’s test is used to detect the presence of nitrogen, sulphur and halogens in organic
r
compounds. The test consists of heating a pinch of the organic compound with a pea size cleaned and fleshy cut
You
sodium metal in a fusion tube. After fusion, the red hot fusion tube is plunged into a china dish containing
oks

distilled water. The contents of the china dish are filtered and the filtrate is boiled and cooled. The solution thus
eBo

obtained is called Lassaigne's extract. For testing nitrogen, freshly prepared FeS04 solution is used and for sulphur,
lead acetate is used.

If potassium sulphocyanide is used, sometimes, but not always, both N and S are detected. For halogens,
silver nitrate is used.
ad
our

Based on the above paragraph, answer questions no. 1 to 4 :


1. Lassaigne’s extract is prepared from an organic compound containing nitrogen, sulphur and halogen. What
possible species are expected to be present in the Lassaigne’s extract ?
2. A blue or green colour is obtained in the Lassaigne’s test for nitrogen. Write the chemistry of the test.
Re
dY

3. How is sulphur detected from Lassigne’s extract ? Explain with chemical equation.
4. Write the chemistry of Lassaigne’s test for halogens ? Why is the Lassaigne’s extract boiled with dil. HNO3
Fin

before adding AgN03 solution ?

CASE 2. Electrophiles and nucleohiles are two important reagents in organic reactions. Electrophiles are
electron loving chemical species which may be either positively charged or neutral. In contrast, nucleophile are
nucleus loving chemical species. Like electrophiles, nucleophiles may be either negatively charged or neutral
chemical species. However, some molecules may act both as a nucleophile as well as an electrophile.
Based on the above paragraph, answer questions no. 5 to 8 :
5. In sulphonation of benzene, SO3 acts as an electrophile. Explain how ?
6. In preparation of amines from alkyl halides and ammonia, ammonia acts as a nulcophile. Explain how ?
7. CH3CN acts both as a nucleophile as well as an electrophile. Explain how ?
ORGANIC CHEMISTRY-SOME BASIC PRINCIPLES AND TECHNIQUES 12/165

8. Nucleophiles also act as bases. Out of OH and HS which is a belter base and which is a better nucleophile ?
Comment.

CASE 3.Compounds having the same molecular formula but different physical and chemical properties are
called isomers. Millions of organic compounds show this phenomenon of isomerism. Broadly speaking it is of
four types : (/) structural isomerism, (ii) geometrical isomerism, (Hi) optical isomerism and (/v) conformational
isomerism. Structural isomerism itself is of six different types : (0 chain or nuclear isomerism, (//) position
isomerism, (Hi) functional isomerism, (/v) metamerism, (v) lautomerism and (vi) ring-chain isomerism.
Based on the above paragraph, answer questions no. 9 to 12 :
9. How many ethers are represented by the molecular formula, C4HJQO ? Write the type of isomerism exhibited

w
by them.
O O

F lo
10. Write the tautomers of and
. Which of the two is more stable and why ?

11. What are functional isomers ? Write the functional

ee
isomers of glycine (H2NCH7COOH).
12. Write the cyclic isomers having the molecular formula, C^H^O.

Fr
ANSWERS

for
1. During fusion, N, S and halogens of organic compounds are converted into : CN", S"' and X“ (Cl", Br" and
I") ions. Sometimes both N and S combine to form CNS" ions.
ur
2. 2NaCN + FeS04 > Na2S04 + Fe(CN)2
Lassaigne’s extract
oks
Fe(CN)2 + 4NaCN Na4lFe(CN)^J
Yo
Sod.tiexacyimoferrate(II)
o
eB

3 Na4(Fc(CN)(,J + 4 Fe^^ Fc4 [Fe(CN)^ 13 + 12Na+


Iron (III) hexacyanoferraiedl)
(Prussian blue)

3. During fusion. S of the organic compound is converted into Na2S. The Lassaigne’s extract is acidified with
our
ad

dil. acetic acid and on adding lead acetate, a black ppt. due to the formation of lead sulphide is formed.
NaoS + (CH3COO)2pb ^ PbS + 2CH3COONa
I.ciid Jtcetate (Blck ppt.)
Y

Instead of lead acetate, sodium nitroprusside may be used when a violet colouration is obtained.
Re

Na2S + Na2|Fe(CN)gNO| ^ Na4(Fe(CN)5(NOS)]


nd

Sodium nilroprusside (Violet colouration)


Fi

4. If the organic compound also contains N and S, the Lassaigne’s extract is boiled with dil. HNO3 to decompose
NaCN and Na2S otherwise these will interfere with the test of halogens.
NaCN + HNO3 4 NaN03 + HCN T
Na2S -I- 2HNO3 2 NaN03 + I-I2S T
The Lassaigne’s extract after the removal of Na2S and NaCN is treated with AgN03 solution, a white ppt. of
Agd completely soluble in NH^OH indicates the presence of Cl, a pale yellow ppt. partially soluble in
NHjfOH indicates the presence of Br and a yellow ppt. insoluble in NH^OH indicates the presence of 1.
O

5. In SO3, O = S-0 , S atom carries a +ve charge and hence acts as an electrophile.
12/166 New Course Chemistry (XI)S!EISD

6. In ammonia, NH^ N carries a lone pair of electrons which it can donate to the alkyl halide and hence it
a +

► R—NH3X"
NaOH
► R—NH2
H3N + R—X
Alkyl halide Amine

acts as a nucleophile.
5+ 5-
7. CH3—c=n:
t
Electrophile Nucleophile
Due to the presence of a partial +ve charge on C, it acts as an electrophile and due to the presence of a

w
partial -ve charge on N, it acts os a nucleophile.
8. S is less electronegative than O, therefore, it can donate a pair of electrons more easily than O. Thus, HS~ is
a better nucleophile than HO~.

F lo
Since O-H bond is stronger than S-H bond, therefore, HO" has a greater tendency to accept a proton than
HS~ and hence HCT is a better base than MS'.
9. Three. These are :

ee
CH3

Fr
CH3CH2—O—CH2CH3 CH3—O—CH2CH2CH3 CHj—O—CH—CH3
I II III

for
I and II or I and III are metamers while II and HI are position isomers.
O OH O OH
r
You
10. Tautomer of IS while tautomer of IS
s
ook
O OH H0‘ OH
I II III IV

Tautomer IV being aromatic is more stable than tautomer II.


eB

11. Compounds having same molecular formula but different functional groups are called functional isomers.
The functional isomers of glycine (H2NCH2COOH) are :
0
our

CH3CH2—O—N = 0
ad

CH3—CH2—N
O" Ethyl nitrile
Nitroelhane

12. The cyclic isomers of C3H6O are :


dY

O
CH2 O
Re

CH2 CH2
Fin

CH3
Oxetane Methyloxirane Cyciopropanol

CASE-BASED MCQs AND ASSERTION-REASON QUESTIONS

CASE 1. In an organic reaction, the organic compound called the substrate reacts with a suitable attacking
reagent, which may be either electron-deficient or electron-rich. But most of the substrate molecules as a whole
are electrically neutral. Evidently, the reagent can attack the substrate molecule successfully only if the substrate
possesses oppositively charged centres. This is possible only if the displacement of bonding electrons occurs
either partially or completely creating centres of low and high electron density in the substrate molecule. Such
factors are called electron displacements. These are of the following four types: (j) inductive effect, (i7) electromeric
effect, {Hi) resonance or mesomeric effect and (iv) hyperconjugation effect. These factors play a vital role in
determining the reactivity of any substrate molecule in a given reaction.
ORGANIC CHEMISTRY-SOME BASIC PRINCIPLES AND TECHNIQUES 12/167

Based on the above paragraph, answer questions no. 1 to 4 :


1. The lower stability of ethyl anion compared to methyl anion and higher stability of ethyl radical to methyl,
respectively are due to
(a) +I-effect of methyl group in both cases
(b) +I-effect of methyl group in ethyl anion and conjugation in ethyl radical
(c) -I-effect of methyl group in ethyl anion and c-n* conjugation in ethyl radical
(r/) +)-cffect ot the methyl group in ethyl anion and a-p orbital hyperconjugation in ethyl radical
2. Which of the following correctly represents the +l-effect of the substituents on a saturated carbon chain ?

w
(fi) -C00 >-0->-CR3 ih) -0“>-C00">-CR3
(c) -0~<-C00“<-CR3 id) -COO'<-0-<-CR 3
Clioase the correct option out of the four options given below :

Flo
(a) Both Assertion (A) and Reason (R) are true and Reason (R) is the correct explanation of Assertion (A).
(b) Both Assertion (A) and Reason (R) are true but reason (R) is not the correct explanation of Assertion (A),
(c) Assertion (A) is true but Reason (R) is false.

ee
(d) Assertion (A) is false but Reason (R) is true.

Fr
3. Assertion. and on ionization give the same carbocation.

for
Br
ur
Br

Reason. Rearrangement of carbocations occur through either 1. 2-hydride or 1, 2-alkyl shifts.


4. Assertion. The C-H a-bond length in propene is shortner than that in propane.
ks
Rea.son. C-H a-bond in propene has some double bond character due to hyperconjugation.
Yo
oo
CASE 2. In a reaction, old bonds are broken and the new ones arc formed. Bond cleavage of carbon-carbon
covalent bonds occurs cither homolytically or heterolytically. Whereas homolysis produces free radicals, heterolysi;IS
eB

produces either carbtKations or carbanions. The stability of these reactive chemical sptecies depends upon many
factors such as inductive effect, resonance effect and hyperconjugation effect.
Based on the above paragraph, answer questions no. 5 to 8 :
r

a
ou
ad

5. Consider the following bond cleavages CH3 CI-I3 CH3 SCH3 CH3 Cu
I II HI
Reactive carbon species formed in (1), (II) and (III) respectively are
Y

(a) free radicals, carbocation. carbanion (b) free radicals, carbanion, carbocation
(c) carbocation, carbanion, free radicals (d) carbanion, free radicals, carbocation
nd
Re

6. The stability of the following free radicals decreases in the order ;


Fi

^CH3 CH2 CH2 = CHCH2 CH, =CH


III IV
I II

(fl)I>II>IV<IIl (b) I > II > III > IV (c) III > I > IV > II (d) 1 > III > II > IV
Choose the correct option out of the four options given below :
(a) Both Assertion (A) and Rea.son (R) are true and Reason (R) is the correct explanation of Assertion (A).
(b) Both Assertion (A) and Reason (R) are true but reason (R) is not the correct explanation of Assertion (A),
(c) Assenion (A) is true but Reason (R) is false.
(d) Assertion (A) is false but Reason (R) is true.

7. Assertion. Out of CH3CH2 > CH2 = CH = C'*' the most stable carbocation is CH = C^.
Reason. Stability increases as the ^-character of the hybridized carbon holding the +ve charge decreases.
12/168 New Course Chemistry (XI)EEIHD

8. Assertion. is pyramidal.

Reason. Usually free radicals assume planar geometry.


ANSWERS

l.u/) 2. ib) 3. (h) 4. (a) 5. («) 6. (d) 7. (</) 8. (&)

HINTS/EXPLANATIONS For Difficult Questions

w
2. -O r has three lone pairs on oxygen atom, therefore it has the maximum ability to push electrons into the
single bond attached to it.

F lo
Due to resonance -vely charged oxygen atom has less ability to push electrons into the single bond attached
to it

:or

ee
—c^o:~ <■ >
c=p:

Fr
Due to the weak +R-effect of the three R groups, this has the least ability to push electrons into the single
bond attached to it.
R

for
ur
C-<-R

R
s
Thus, option (b) is correct.
ook
Yo
3. Correct explanation. Compound (I) on ionization gives carbocation (III) which is stabilized by resonance.
CH3
eB

*
-Br
Br
III
r
ou
ad

Compound (II) on ionization gives carbocation (IV) which undergoes I, 2-hydride shift to give the same
more stable carbocation (HI).
Y

Ionization I. 2-Hydride
III
shift
Re

-Br
nd

Br
Fi

II IV

5. Although both C and S have the same electronegativity (2-5) but the cleavage of C-S bond occurs towards
S atom because S being bigger in size can disperse the -ve charge more effectively.
6. I and III are allyl free radicals and hence more stable than free radicals (II and IV). Further, free radical (I) is
more stable than III since it is stabilized by resonance as well as +I-effect of the CH3 group. Out of free
radicals (II and IV), II is more stable since in II, +ve charge is present on a less electronegative .^/^^-hybridized
carbon while in IV, it is present on a more electronegative sp- hybridized carbon. Thus, overall stability
decreases in the order ; 1 > III > II > IV, i.e., option (d) is correct.

7. Correct assertion. Out of CHgCH^. CH2 = CH and HC = C+, the most stable carbocation is CH3CH7 .
8. Correct explanation. Due to angle strain, bridge head carbon cannot assume planar geometry and hence it
is pyramidal.
ORGANIC CHEMISTRY-SOME BASIC PRINCIPLES AND TECHNIQUES 12/169

WITH
ANSWERS

A 1^0

low
Q. 12.1. What are hybridisation states of each carbon atom in the following compounds ?
CHj = C = O, CH3CH = CH2, (CH3)2C0, CH2 = CHCN, CgH^.
O
xp^ 3

ee
xp sp- sp^ xp
Ans.
CH2 = C = O ■ CH3—CH = CH2 ’ CH3—c CH3 ’

rF sp'^

Fr
H

for
sp

CH2= CH—C = N’ Each C is 5/j^-hybridized


u
Q. 12.2. Indicate the a- and -bonds in the following molecules :
ks
CgHg, C6H,2, CHjCIj, CH2 = C = CH2, CH3NO2, HCONHCH3
Yo
oo
H
a
eB

P S' S q.
Ans. Cf jr a
7T
<5-
H a a H
r

a
ou
ad

H
Y

H O H H
H'vCT Cl H O
Jr
qv CT. a Tt CT CT
K K
c c==c==c H—C-g-N
CT a a h-^c-^-n-o-c-o-h
nd

■a a Cf q
H- Cl
Re

H- q

H H
Fi

Q. 12.3. Write bond-line formulas for : Isopropyl alcohol, 2, 3- Dimethylbutanal, Heptan-4-one.

O 0
OH 6 2
3 4

4
Ans. 2 H
7 5 3 1

Isopropyl alcohol
2,3-Dimethylbutanal Hcptan-4-one
12/170 “Ptadet^ ^ New Course Chemistry (XI) IS!

w
Q. 12.4. Give the lUPAC names of the following compounds :

CN
(a) (b) (c)

or
r
(/) CI2CHCH2OH

F
Ans. (a) Propylbenzene (b) 3-Methylpentanenitrile (c) 2, 5-Dimethylheptane (d) 3-Bromo-3-chloroheptane

oF
(e) 3-Chloropropanal (f) 2, 2-Dichloroethanol

ul
Q. 12.5. Which of the following represents the correct lUPAC name for the compounds concerned ?
(a) 2y 2-Dimethylpentane or 2-Dimethylpentane (b) 2, 4, 7-Trimethyloctane or 2, 5, 7-
lYimethyloctane (c) 2-Chloro-4-methylpentane or 4-Chloro-2-methylpentane (d) But>3-yn-l-ol

rs
or But-4-ol-l-yne.

k
Ans. (a) 2, 2-Dimethylpentane. For two alkyl groups on the same carbon its locant is repeated twice, (b) 2,

o
4, 7-Trimethyloctane since 2, 4, 7-locant set is lower than 2, 5, 7. (c) 2-Chloro-4-methylpentane.
Alphabetical order of substituents, (d) But-3-yn-l-ol. Lower locant for the principal function^ group,

of
i.e., alcohol.
Q. 12.6. Draw formulas for the first five members of each homologous series beginning with the following
o
compounds, (a) H-COOH (b) CH3COCH3 (c) H—CH = CHj.
Y
Ans. Refer to Art. 12.9.2, pages (a) 12/27 (b) 12/28 (c) 12/22-12/23.
rB
Q. 12.7. Give condensed and bond line structural formulas and identify the functional group(s) present, if
eY

any, for ; (a) 2, 2, 4-Trimethylpentane (b) 2-Hydroxy-l, 2, 3-propanetricarboxylic acid


(c) Hexanedial
Ans. Condensed formula Bond line formula Functional group/s
u

(a) (CH3)3CCH2CH(CH3)2
od
ad

O
(b) H00CCH2C(0H)(C00H)CH2C00H (carboxyl)
in

—C—OH

and —OH (hydroxyl)


Re
F

(c) 0HC(CH2)4CH0 ●C—H (aldehyde)

Q. 12.8. Identify the functional groups in the following compounds :


CHO NH2

(«)
OMe
(6)
T
(C)
o
CH = CHN02
OH 0^^0CH2CH2N(C2H5)2
ORGANIC CHEMISTRY-SOME BA. ic PRINCIPLES AND TECHNIQUES 12/171

rd t

V
I
I *
I
I Aldehyde
I
I
I
1
^ NH2! 1 ® Amino (aromatic)

Ans. (a) (b)


OMe jPhenolic T
ether 3® Amino
roHi

ow
L^\!
o
I
/'C2H5
Phenolic hydroxyl 01—CH2 —CH2 I
N
Ester
I.

(c) Nitro
r
O

e
I
I CH = CH1-!N

Fl
I

re
Ethylenic |. 0-
double bond

Q. 12.9. Which of the two : 02NCH2CH20^ or CH3CH2O" is expected to be more stable and why ?

F
Ans. O2N—<-CH2 —<-CH2—<-0“ is more stable than CH3 CH2 — O" because NO2 group has
-I-effect and hence it tends to disperse the -ve charge on the 0-atom. In contrast, CH3CH2 has
ur
+I-effect. It, therefore, tends to intensify the -ve charge and hence destabilizes it.

r
fo
Q.12.10. Explain why alkyl groups act as electron donors when attached to a K-system.
Ans. Due to hyperconjugation, alkyl groups act as electron donors when attached to a jc-system as shown
below ;
H
ks
Yo
H+ H H

H—C^CH=ftH2 <—> H—C = CH—CH2


I
oo
<—> C = CH—CH2 <—> H—C = CH—CH^
H H
B

Propene
re

Q.12.11. Draw the resonance structures for the following compounds. Show the electron shift using curved-
arrow notation.
+
u

(a) CgHgOH (b) C6HsN02 (c) CH3CH = CHCHO (d) CgHg—CHO (e) CgHj—CH2
ad
Yo

(f) CH3CH = CHCH2


Ans.

:d-H
d

*d-H +0-H -H :0-H


Q
Re
in

4- 4
-O^ ‘4-
(a)
F

Phenol

A
-■K/
o o o
k./
N N N N

0
o
+

- u
4 4 >

9 "
4
(b)
+
Nitrobenzene
12/172 ^ .. a Course Chemistry (XI)E!SiaD

w
(c) CH3-CH=CH-CH=^; <
+

>CH3-CH=iCH-CH-0: <—► CHj-CH—CH=CH-0:


But-2-en-l-al

V
O

V
o
Vpo: H O:

$*

or
>
id)

r
+
Benzaldehyde

F
oF
CH CH2 CH2 CH2 CH

ul
0
<■ >● *♦
ie)

rs
+
Benzyl carbocation

ko
if) CH3-CH=tCH-*CH2 ◄—► CH3—CH~CH=CH2

of
But-2-en-l-yl carbocation

Q.12.12. What are electrophiles and nucleophiles ? Explain with examples.


o
Y
Ans. Refer to Art. 12.11, page 12/75.
Q.12.13. Identify the reagents shown in bold in the following equations as nucleophiles or electrophiles
YB
(a) CH3COOH + HO- > CH3COO- + HjO
(b) CH3COCH3 + -CN > (CH3)2C(CN)(0H)
er

(c)C6H6+CH3CO ^ C6H5COCH3
u

Ans. Nucleophiles : (a) and (b) and Electrophile : (c).


Q.12.14. Classify the following reactions in one of the reaction type studied in this unit
od
ad

(a) CH3CH2Br + HS" > CH3CH2SH + Br"


(b) (CH3)2C = CH2 + HCl > (CH3)2CC1—CH3
in

(c) CH3CH2Br + HO" > CH2 = CH2 + H2O + Br"


(d) (CH3)3C—CHjOH + HBr > (CH3)2CBrCH2CH3 + HjO
Ans. (a) Nucleophilic substitution
Re
F

(b) Electrophilic addition


(c) Bimolecular elimination
(d) Nucleophilic substitution with rearrangement.
Q.12.15. What is the relationship between the members of following pairs of structures ? Are they structural
or geometrical isomers or resonance contributors ?
O O
D H D
\
(a) (b) C = C
\
C = C^
H D H H

+ OH OH
I
(c) H—C—OH H—C—OH
ORGANIC CHEMISTRY-SOME BASIC PRINCIPLES AND TECHNIQUES 12/173

Ans. (fl) Structural isomers (actually position isomers as well as metamers) (b) geometrical isomers
(c) resonance contributors because they differ in the position of electrons but not atoms.
Q.12.16. For the following bond cleavages, use curved-arrows to show the electron flow and classify each
as homolysis or heterolysis. Identify reactive intermediate produced as free radical, carbocation
and carbanion.

(a) CH3O—OCH3 CH36 + OCH3 (*) \= 0


/
+“OH
_/
=0 + H2O

(c) + Br” (d) + E*^ >


V^Br ^

Ans. («) CHsO^CHs Homolysis■►CH36 + 6CH3

w
Free ardicals

F lo
(6)
\ =0 Heteroiysis^ O + H2O
HO +
■"K H H
Carbanion

ree
Heterolysis ^ + Br

for F
Carbocation

id) + E+ Heterolysis ^
Your
Carbocation
ks

Q.12.17. Explain the terms inductive and electromeric effects. Which electron displacement effect explains
o
eBo

the following correct orders of acidity of the carboxylic acids ?


(a) CI3CCOOH > CljCHCOOH > ClCHjCOOH
(b) CH3CH2COOH > (CH3)2CHC00H > (CH3)3C.C00H
ad

Ans. For definition and explanation of the terms, refer to Art. 12.13.1, pages 12/76-12/77 and Art. 12.13.2,
our

page 12/77.
(a) -I-effect as shown below :
As the number of halogen atoms decreases, the overall -I- effect decreases and the acid strength decreases
accordingly.
Re
dY

O O O
Cl Cl
XCH
Fin

Cl (M-H > C-<-0-$-H > Cl-e-CHj-e-C 0-<-H

Cl X Cl X

(b) +I-e£fect as shown below :


As the number of alkyl groups increases, the +I-effect increases and the acid strength decreases
accordingly.
O O CH3 O
II
CH3CH2^C-^0^H > XH-^C O^H > CH3-^C->-C O^H

CH3
X I
CH3

I
12/174 ^teideefr’^ New Course Chemistry (XI)iSsI9n
Q.12.18. Give a brief description of the principles of the following techniques taking an example in each
case : (a) Crystallisation (b) Distillation (c) Chromatography
Ans. Refer to (a) Art. 12.16.2, pages 12/98-12/99 (b) Art. 12.16.5, pages 12/100-12/101.
(c) Art. 12.16.10, pages 12/104-12/108.
Q.12.19. Describe the method, which can be used to separate two compounds with different solubilities in
a solvent S.
Ans. Two compounds with different solubilities in a solvent S can be separated by factional crystallisation.
When a hot saturated solution of these two compounds is allowed to cool, the less soluble compound
crystallises out first while the more soluble remains in the solution. The crystals are separated from the
mother liquor and the mother liquor is again concentrated and the hot solution again allowed to cool
when the crystals of the second (i.e., more soluble) compound are obtained. These are again filtered
and dried.

w
Q.12.20. What is the difference between distillation, distillation under reduced pressure and steam
distillation ?

F lo
Ans. Distillation involves conversion of a liquid into vapours followed by condensation of the vapours thus
produced by cooling to get the pure liquid while the non-volatile impurities remain in the flask. This
method is commonly used for those liquids which are sufficiently stable at their boiling points and
contain non-volatile impurities.
Distillation under reduced pressure also involves conversion of a liquid into vapours by heating

e
followed by condensation of the vapours thus produced by cooling but the pressure acting on the

Fre
system is not atmospheric but is reduced by using a vacuum pump. Since the boiling point of a liquid
decreases as the pressure acting on it is reduced, therefore, this method is used to purify such liquids

for
which have high boiling liquids or liquids which decompose at or below their boiling points.
Steam distillation is comparable to distillation under reduced pressure (vacuum distillation) even
though there is no reduction in the total pressure acting on the solution. Here, the mixture of organic
liquid and water boils at a temperature when the sum of the vapour pressures of the organic liquid (pj)
r
and that of water (pf) becomes equal to the atmospheric pressure ip), i.e.,p = pj+ P2
You
oks

Since the vapour pressure of water around its boiling point is quite high and that of the liquid is quite
low, therefore, the organic liquid will boil at a temperature much lower than its normal boiling point
eBo

and hence its decomposition is avoided. Steam distillation is used to purify such liquids which are
volatile in steam, insoluble in water, possess a vapour pressure of about 10-5 mm of Hg and contain
non-volatile impurities.
Q.12.21. Discuss the Chemistry of Lassaigne’s test.
ad
our

Ans. Refer to Art. 12.17.2 to 12.17.4, pages 12/109 - 12/112.

Q.12.22. Differentiate between the principle of estimation of nitrogen in an organic compound by (i) Dumas
method (ii) Kjeldahl’s method.
Ans. (0 In Dumas method, a known mass of the organic compound is heated with excess of CuO in an
atmosphere of CO^, when nitrogen of the organic compound is converted into N, gas. The volume of
Re

N2 thus obtained is converted into NTP and the percentage of nitrogen determmed by applying the
dY

28
X
Vol.ofN2atNTP xlOO
equation. %N =
Fin

22400 Mass of the substance taken

(ii) In Kjeldahl’s method, a known mass of the organic substance is digested (heated) with cone.
H2SO4 in presence of K2SO4 (raises the b.p. of H2SO4) and little CU.SO4 or Hg (catalyst) in a long
necked flask called Kjeldahl’s flask when nitrogen present in the organic compound is quantitatively
converted into (NH4)2S04. (NH4)2S04 thus obtained is boiled with excess of NaOH solution to liberate
NH3 gas which is absorbed in a known excess of a standard acid such as H2SO4 or HCl.
The volume of acid unused is found by titration against a standard alkali solution. From the volume of
the acid used, the percentage of nitrogen is determined by applying the equation.
1-4 X Molarity of the acid x Basicity of the acid x Vol. of the acid used
%N =
Mass of the substance taken

I
ORGANIC CHEMISTRY-SOME BASIC PRINCIPLES AND TECHNIQUES 12/175

Q.12.23. Discuss the principle of estimation of halogens, sulphur and phosphorus present in an organic
compound.
Ans. Refer to Art. 12.18.3, page 12/120, 12.18.4, page 12/121 and 12.18.5, pages 12/122-123.
Q.12.24. Explain the principle of paper chromatography.
Ans. Refer to partition chromatography. Art. 12.16.10, pages 12/107 - 12/108.
Q.12.25. Why is nitric acid added to sodium extract before adding silver nitrate for testing halogens ?
Ans. Sodium extract is boiled with nitric acid to decomposeNaCN and Na2S, if present, otherwise these
NaCN + HNOj - NaN03 + HCN t
Na2S +2HNO3 2 NaN03 + H2S t

w
will react with AgN03 and hence will interfere with the test as shown below :
NaCN + AgN03 AgCN + NaN03
Silver cyanide

F lo
(White ppt.)
Na2S + 2AgN03 > Ag2S + 2NaN03
Silver sulphide

ee
(Black ppt.)
Q.12.26. Explain the reason for the fusion of an organic compound with metallic sodium for testing nitrogen,

Fr
snlphur and halogens.
Ans. The organic compound is fused with sodium metal to convert these elements which are present in the
covalent form to ionic form.

for
Q.12.27. Name a suitable technique of separation of the components from a mixture of calcium sulphate
ur
and camphor.
Ans. A mixture of CaS04 and camphor can be separated by the following two methods :
s
(0 Camphor is sublimable but CaSO/^ is not, therefore, sublimation of the mixture gives camphor on
ook
Ae sides of funnel while CaS04 is left in the china dish.
Yo
(it) Camphor is soluble in organic solvents like CHCI3, CCI4 etc. while CaS04 is not. Therefore, when
the mixture is shaken with the solvent, camphor goes into solution while CaS04 remains as residue. It
eB

is filtered and evaporation of solvent gives camphor.


Q.12.28. Explain, why an organic liquid vapourises at a temperature below its boiling point in its steam
distillation ?
our

Ans. In steam distillation, the mixture consisting of the organic liquid and water boils at a temperature when
ad

the sum of the vapour pressure of the liquid (pj) and that of water (p^ becomes equal to the atmospheric
pressure (p), i.e., p = Pi + P2-
Since the vapour pressure of water around the boiling point of the mixture is quite high and that of
Y

liquid is quite low (10 - 15 mm), therefore, the organic liquid distils at a pressure much lower than the
atmospheric pressure. In other words, the organic liquid vapourises at a temperature much lower than
Re
nd

its normal boiling point.


Q.12.29. Will CCI4 give white precipitate of AgCI on heating it with silver nitrate ? Give reason for your
Fi

answer.

Ans. When CCI4 is heated with AgN03 solution, white ppt. of AgCl will not be formed. The reason being
that CC/4 is a covalent compound, therefore, it does not ionize to give Ct~ ions neededfor theformation
of ppt. ofAgCl.
Q.1230. Wliy is a solution of potassium hydroxide used to absorb carbon dioxide evolved during the
estimation of carbon present in an organic compound ?
Ans. CO2 is acidic in nature, therefore, it reacts with the strong base KOH to form K2CO3.
2 KOH + CO2 > K2CO3 + H2O
The increase in the mass of U-tube containing KOH then gives the mass of CO2 produced and from its
mass, the percentage of carbon in the organic compound can be estimated by using the equation.
12/176 7^>uxcCee^ New Cotirse Chemistry (XI) BZ

12
%C = —X
Mass of CO2 fonned xlOO
44 Mass of substance taken

Q.12.31. Why is it necessary to use acetic acid and not sulphuric acid for acidification of sodium extract
for testing sulphur by lead acetate test ?
Ans. For testing sulphur, the sodium extract is acidified with acetic acid because lead acetate is soluble and
does not interfere with the test. If H2SO4 were used, lead acetate itself will react with H2SO4 to form
white ppt. of lead sulphate which will interfere with the test.

low
Pb(OCOCH3)2 + H2SO4 ^ PbS04i + 2CH3COOH
Lead acetate (White ppt.)
Q.1232. An organic compound contains 69% carbon and 4*8% hydrogen, the remainder being oxygen.
Calculate the masses of carbon dioxide and water produced when 0*20 g of this substance is
subjected to complete combustion.
12 Mass of CO2 formed

e
Ans. We know that, %C = — x XlOO
44 Mass of substance taken

re
rF
Substituting the values of % of C and mass of the substance taken, we have,

F
12 mass of CO, formed 69 x 44 x 0-2
69 = — X 2 xlOO or Mass of CO2 formed = 12x100 = 0*506 g
44 0-2g

r
Mass of H2O formed

fo
2
Similarly, % H = — x
ou
XlOO
18 Mass of substance taken
ks
Substituting the values of % of H and mass of the substance taken, we have,
2 Mass of H,0 formed 4-8x18x0-2
oo
4-8 = —x ^ X100 or Mass of H2O formed = = 0*0864 g
18 0-2 2x100
Y
Q.1233. A sample of 0*50 g of an organic compound was treated according to Kjeldahl’s method. The
B

ammonia evolved was absorbed in 50 mL of 0*5 M( H2SO4. The residual acid required 60 mL of
0*5 M solution of NaOH for neutralisation. Find the percentage composition of nitrogen in the
re

compound.
Ans, Step 1. To determine the volume ofH2SO^ used.
u

Volume of acid taken = 50 mL of 0-5 M H2SO4 = 25 mL of 1 M H2SO4


ad
Yo

Volume of alkali used for neutralization of excess acid = 60 mL of 0-5 M NaOH = 30 mL of 1 M NaOH.

Now 1 mole of H2SO4 neutralizes 2 moles of NaOH (i.e. H2SO4 + 2 NaOH ^ Na2S04 + 2 H2O)
30 mL of 1 M NaOH s 15 mL of 1 M H2SO4
d

Volume of acid used by ammonia = 25 -15 = 10 mL


Re

Step 2. To determine percentage of nitrogen.


in

Again 1 mole of H2SO4 neutralizes 2 moles of NH3 10 mL of 1 M H2SO4 s 20 mL of 1 M NH3


F

But 1000 mL of 1 M NH3 contain nitrogen = 14 g


14
20 mL of 1 M NH3 will contain nitrogen = x20g
1000
But this much amount of nitrogen is present in 0-5 g of the organic compound.

Percentage of nitrogen = ^x-^xlOO


10000-5
=56*0.
Alternatively, % of N can be determined by applying the following equation,
%N =
1-4 X Molarity of the acid x Basicity of the acid x Vol. of the acid used
Mass of substance taken
ORGANIC CHEMISTRY>SOME BASIC PRINCIPLES AND TECHNIQUES 12/177

14x1x2x10
Substituting the values of all the items in the above equation, we have, %N - 0-5
= 56-0

Q.12.34. 0*3780 g of an organic chloro compound gave 0*5740 g of silver chloride in Carius estimation.
Calculate the percentage of chlorine present in the compound.
Ans. Here, the mass of the substance taken = 0.3780 g
Mass of AgCl formed = 0-5740 g
Now 1 mole of AgCl s 1 g atom of Cl or (108 35-5) = 143-5 g of AgCl s 35-5 g of Cl

w
35-5 Mass of AgCl formed xlOO
Applying the relation. Percentage of chlorine = 143-5 Mass of substance taken

35-5 0-5740
X X100 = 37*566%.
143-5 0-3780

Flo
Q.12.35. In the estimation of sulphur by Carius method, 0*468 g of an organic sulphur compound afforded
0*668 g of barium sulphate. Find the percentage of sulphur in the given compound.

e
Ans. Here, the mass of the substance taken = 0-468 g

re
Mass of BaS04 formed = 0-668 g
Now 1 mole of BaS04 s 1 g atom of S or (137 -i- 32 -f 4 x 16) = 233 g of BaS04 s 32 g of S

F
32 Mass of BaSO. formed
Z XlOO
Applying the relation, Percentage of sulphur = 233 Mass of substance taken
ur
r
fo
32 0-668
X100 = 19*60.
“ 233 ^ 0-468
Q.12.36. In the organic compound CH2 = CH—CH2—CH2—C s CH, the pair of hybridised orbitals
ks
involved in the formation of C2—C3 bond is ; (a) sp - sp^ (b) sp - sp^ (c) sp^ - sp^ (d) sp^ - sp^
Yo
Ans. When both double and triple bonds are present at equivalent positions, double bond is given preference
oo
1 2 3 4 5 6

while numbering the carbon chain. Thus, CH2= CH—CH2—CH2—C s CH


B

sp^ sp^ sp^ sp^ sp sp


re

.% C2—C3 bond is formed by overlap of sp^ - sp^ orbitals.


Thus, option (c) is correct.
Q.12.37. In the Lassaigne’s test for nitrogen in an organic compound, the Prussian blue colour is obtained
u
ad

due to the formation of:


Yo

(a) Na4[Fe(CN)fi] (b) Fe4[Fe(CN)6l3 (c) Fe2[Fe(CN)6] (d) Fe3[Fe(CN)fi]4


Ans. The prussian blue colour is due to the formation Fe4[Fe(CN)6]3. Thus, option (b) is correct.
Q.12.38. Which of the following carbocation is most stable ?
d
Re

(a) (CH3>3CCH2 (b) (CH3)3C (c) CH3CH2CH2 (d) CH3CHCH2CH3


in

Ans. The order of stability of carbocation is : 3® > 2® > 1®


F

(a) (CH3)3C—CH2 {b) (CH3)3C (c) CH3CH2CH2 (d) CH3CHCH2CH3


1“ Carbocation 3® Carbocation 1® Carbocation 2® Carbocation
Since 3° carbocations are the most stable, therefore,, option (b) is correct.
Q.12.39. The best and latest technique for isolation, purification and separation of organic compounds is
: (a) Crystallisation (b) Distillation (c) Sublimation (d) Chromatography
Ans. Chromatography. Thus, option (d) is correct.
Q.12.40. The reaction : CH3CH2I + KOH (aq) > CH3CH2OH KI is classified as :
(a) electrophilic substitution (b) nucleophilic substitution (c) elimination (d) addition.
Ans. This is an example of nucleophilic substitution reaction since the nucleophile 1“ is replaced by the
nucleophile OH“ ion. Thus, option (b) is correct.
12/178 it New Course Chemistry (XI) or«iwn

1 1
;ify;
,v
WITH ANSWERS,
I HINTS AND SOLUTIONS
me
. SC,

I 3)1
V l¥> r

low
MULTIPLE CHOICE QUEStlONI

1. Which of the following Is the correct lUPAC 5. In which of the following, functional group
name ? isomerism is not possible ?
(a) 3-Ethyl-4, 4-dimethylheptane (a) Alcohols (Z?) Aldehydes

e
(b) 4, 4-Dimethyl-3-ethylheptane (c) Alkyl halides (d) Cyanides

re
(c) 5-Ethyl-4, 4-dimethylheptane 6. The fragrance of flowers is due to the presence

rF
(d) 4, 4-Bis(methyl)-3-ethylheptane of some steam volatile organic compounds

F
2. The lUPAC name for called essential oils. These are generally
O O
insoluble in water at room temperature but are
miscible with water vapour in vapour phase. A

r
suitable method for the extraction of these oils
CH3-C CHj—CHj—C—OH

fo
is.

(a)
u
1-hydroxypentane-l, 4-dione
from the flowers is :
(a) Distillation
ks (b) Crystallisation
(c) Distillation under reduced pressure
(b) 1, 4-dioxopentanol
Yo
(d) Steam distillation
(c) l-carboxybutan-3-one
oo
(d) 4-oxopentanoic acid 7. During hearing of a court case, the judge
3. The lUPAC name for
suspected that some changes in the documents
had been carried out. He asked the forensic
B

Cl department to check the Ink used at two


different places. According to you which
e

NO2 technique can give the best results ?


ur

(a) Column chromatography


(b) Solvent extraction
ad
Yo

CH3 (c) Distillation


(d) Thin layer chromatography
(a) l-Chloro-2-nitro-4-methylbenzene
8. The principle involved in paper chromato
(b) 1 -Chloro-4-methyl-2-nitrobenzene
d

graphy is
(c) 2-ChIoro-1-nitro-5-methylbenzene
Re

(a) Adsorption (Z>) Partition


in

(^0 m-Nitro-p-chlorotoluene
(c) Solubility (d) Volatility
4. Electronegativity of carbon atoms depends 9. What is the correct order of decreasing stability
F

upon their state of hybridisation.In which of of the following cations ?


the following compounds,the carbon marked
with asterisk is most electronegative ?
CH3—CH—CH3 CH3—CH—OCH3
(a) CH3—CH2—*CH2—CH3 I. II.
(b) CH3—*CH = CH—CH3
CH3—CH—CH2—OCH3
(c) CH3—CH2- =CH
III.

(a) II > I > III (b) II > III > I


(J) CH3—CH2—CH =CH2 (c) III > I > II (iO I > II > III
ORGANIC CHEMISTRY-SOME BASIC PRINCIPLES AND TECHNIQUES 12/179

10. Correct lUPAC name for 13. Electrophilic addition reactions proceed in two
H,C—CH—CH—CH, is steps. The first step involves the addition of an
" I I " electrophile. Name the type of intermediate
formed in the first step of the following addition
reaction.
(a) 2-ethyl-3-melhylpeniane
(b) 3, 4-dimethylhexane H3C—HC = CH2 + H ->?

(fl) 2° Carbanion (b) 1° Carbocation


(c) 2-.vec-butylbutane
(c) 2° Carbocation (d) 1° Carbanion
(d) 2, 3-dimethylbutane
14. Covalent bond can undergo fission in two
11. In which of the following compounds the
different ways. The correct representation
carbon marked with asterisk is expected to have
greatest positive charge ? Involving a heterolytic fission of CH3—Br is
©
(a) *CH3—CH2—Cl ©
Br ► CH3 + Br

w
(b) *ch3—CH2—Mg-^cr
(b) CH3-CBr
© 0
(c) *CH3—CH2—Br

F lo
► CH3 + Br
(d) *CH3—CH2—CH3
(c) CH3 nBr
© ©
12. Ionic species are stabilised by the dispersal of ► CH3 + Br
charge. Which of the following carboxylate ion
is the most stable ? noBr
(d) CH3 CH3 + Br
0 15. The addition of HCl to an alkene proceeds in

ree
two steps. The first step is the attack of ion

for F
(a) CH3—C—O- to >C = C< portion which can be shown as
0

(b) Cl—CH2—C—O"
0
Your
(c) F—CH,—C—O"
ks
eBoo

O
(c)H^
(d) CH—C—O
(d) All of these are possible.
ad

MULTIPLE CHOICE QUESTIONS-II


our

In the following questions two or more options CH3 CH3


may be correct.
16. Which of the following compounds contain all the (a) (b)
I ir"
/ Br
Re

carbon atoms in the same hybridisation state ? cr” Br


(a) H—C = C—C s C—H Cl
Y

(b) CH3—C = C—CH3


Find

CH3 Br
(c) CH2 = C = CH2 (d) CH2 = CH—CH = CH2
17. In which of the following representations given (c) id) ,vC
below, spatial arrangement of group/atom is
different from that given in structure ‘A’ ?
H cr
I H

Cl H3C
CH3 18. Electrophiles are electron seeking species.
Which of the following groups contain only
..vC electrophiles ?
cr J H
(a) BF3, NH3, H2O (b) AICI3, SO3, NOJ
Br +

(A) (c)NOJ.CHj, CH3—C = 0

1
12/180 New Course Chemistry (XI)Bd9l]

20. Which of the following pairs are not functional


(d) C2HJ,C2^S’ ^2^5 group isomers ?
Note : Consider the following four compounds (a) II and III (b) II and IV
for answeringquestions 19 and 20. (c) I and IV (d) I and n
O
21. Nucleophile is a species that should have
I. CH3—CH2—CH2—CH2—C—H (a) a pair of electrons to donate
0 (b) positive charge
(c) negative charge
II. CH3—CH2—CH2—C—CH3 (d) electron deficient species
m. CH3—CH2—c—CH2—CH3 22. Hyperconjugation involves delocalisation of

w
0
(a) electrons of carbon-hydrogen a bond of an
IV. CH3—CH—CH2—C—H alkyl group directly attached to an atom of
unsaturated system

F lo
CH3 O
(b) electrons of carbon-hydrogen <r bond of alkyl
19. Which of the following pairs are position group directly attached to the positively
isomers ? charged carbon atom
(a) I and II (b) II and III (c) 7u-electrons of carbon-carbon bond

ee
(c) II and IV (d) m and IV (d) lone pair of electrons

Fr
ANSWERS

Multiple Choice Questions -1

for
1. (a) 2. (d) 3. (b) 4.(c) 5.(c) 6.(^0 7.(d) 8.(b) 9. (a) 10. (fc)
ur
11. (a) 12. (d) 13. (c) 14.(b) 15. (b)
Multiple Choice Questions - II
oks
16. (a, d) 17. (a, c.d) 18. (b,c) 19. (b) 20. (a, c) 21. (a, b) 22. (a, b)
Yo
o
eB

HINTS FOR DIFFICULT MULTIPLE CHOICE QUESTIONS

Multiple Choice Questions -1


our
ad

1. While deciding the alphabetical order of the substituents, numerical prefixes such as di~, iri-, etc. are not con
sidered. Now since ‘e’ of ethyl comes before, ‘m’ of dimethyl, therefore, lower locant 3 is assigned to ethyl.
3. ‘m’ comes before Vi’. Thus, the name l-chloro-4-methyl-2-nitr obenzene is correct.
4. Electronegativity increases as the state of hybridization changes from sp^-sp^-sp. Thus, sp-carbon has the
Y

highest electronegativity, i.e., option (c) is correct.


Re

5. Alcohols and etliers ; aldehydes and ketones ; cyanides and isocyanides are functional group isomers.
nd

Alkyl halides, however, do not show functional group isomerism.


6. Substances (i.e., essential oils) are insoluble in water and have high vapour pressure at 373 K and hence
Fi

can be separated by steam distillation.


7. Thin layer chromatography is used to separate the components present in a small amount of the sample.
8. In paper chromatography, separation of the components of a mixture depends upon their partitioning between
water held in the stationary phase (i.e., paper) and the liquid present in the mobile phase.

9. CH3 CH CH3 CH3—CH —p—CH3 CH3—CH CH2-»-0CH3


I II III
Stabilized by weak Stabilized by strong Destabilized by
+I-effect of tlie +R-effect of the OCH3 groups -1-effect of the
two methyl groups OCH3 groups
Thus, the stability of carbocations decreases in the order : II > I > III, i.e., option (a) is correct.
ORGANIC CHEMISTRY-SOME BASIC PRINCIPLES AND TECHNIQUES 12/181

CH,—CH—CH—CH,
10. 3,4-Dimethylhexane
CH3—CH2 CH2—CH3
6 5 2 1

11. CHj-i-CHz' C1 CH3-«-CH2-*-Mg‘'Cr CH3->-CH2^Br CH3—CH2—CH3


(0 00 m iiv)
Since Cl has the highest electronegativity : Cl (3-0), Mg (1-2), Br (2-8) and C (2-5), therefore, the asterisked

w
carbon in option (a) has the highest positive charge.
O"
O
12. (a) CH3- -4
► ch3-»-c=o = ch3-»-c:^o

Flo
O"
0
► ci-^ch2-<-c=o = ci-<-ch2-^c:^o

e
(b) CI-^CH2-^C—O" ^

re
0“

F
I o
(c) F-^CH2-^C—O" ^ ► f-^ch2-^c=o = f-^ch2-^c:^ o
ur
r
O"

fo
I o
(d) CH ^c—O" < CH C=0
F'^ ks O

In option (a), -ve charge on the carboxylate ion is intensified by + R-effect of the CH3 group but in options
Yo
(b), (c) and (d), it is dispersed by -I-effect of the halogen. Therefore, carboxylate ion (a) is the least stable.
oo
The stability of the other carboxylate ions, however, depends upon the number of halogens atoms and their
electronegativity. Since F has the highest electronegativity and option (d) has two such F atoms, therefore,
dispersal of -ve charge is maximum in option (d).
B

+
re

CH3-»-CH-«-CH3
+ 2° carbocation {more stable)
13. H3C—HC=CH2 + H Thus, option (c) is correct.
u
ad

CH3-»-CH2-»-CH2
Yo

r carbocation {less stable)

14. CH3 Br ► CH3 + Br“


d

Since Br is more electronegative (2-8) than carbon (2-5), therefore, heterolytic fission occurs in such away
Re

that Br gets the negative charge and CH3 the +ve charge. Thus, option {b) is correct.
in

15. Since double bond is a source of electrons and the charge flows from centre of more to less electron
F

density, therefore, 71-electrons of the double bond attack the proton, i.e., representation {b) is correct.

H'" :^c:
Multiple Choice Questions - II
3
sp sp sp sp sp sp sp sp^
16. (a)H—CsC—CsC—H {b) CH3—C=C—^^^3
sp^ sp sp 2 sp
2
sp
2 2
sp^
2
sp^
(c) CH2 = c = CH2 (d) CH2=CH—CH = CH2
12/182 'Pnadee^'4, New Course Chemistry (XI)Q

17. By making two interchanges of atoms/groups, bring H below the plane of the paper and then hnd out the
sequence of the remaining groups in a particular order whether clockwise or anticlockwise starting from
atom with highest atomic number and moving towards atoms with lower atomic numbers.

CH3 CH3
FirstClinterchange
H
^ 2ndBrinterchange
Cl
^ Clockwise
Cl

Br Br Cl
(A)

w
CH3 CH3 CH3
(a)
.xC FirstClinterchange^ xC 2nd Clinterchange^ .xC Anti-

F lo
H Br

cr" y Br Br C1 clockwise
H Cl Br

ee
Fr
Clockwise
H"

for
Cl
ur
CH3 CH3 CH3
FirstBrinterchange ^ 2nd interchange
CI-^Br ^ Anti-clockwise
s
(c) H
ook
Bf" H Br H' Cl
Yo
Cl Cl Br
eB

Br Br Br

First interchange 2nd interchange


id) Anti-clockwise
CH>H ^ xC
CH3->C1 ^
H
h-"7^ Cl CH3
our

cr
ad

H3C CH3 Cl

Thus, structures (a), (c) and (d) have different (anti-clockwise ) spatial arrangement of group/atoms than
Y

that given in structure A (clockwise).


Re

18. (/>) AICI3, SO3, NO2 and (c) NO+, CH3 , CH3—C=0.
nd
Fi

In option (a), NH3 and H2O are nucleophiles whereas BF3 is an electrophile while in option (d) is

a nucleophile while C2 and C2H5 are electrophiles.


19. n and HI are position isomers since they differ in the position of the C = O group.
20. n and m ; I and IV are not functional group isomers, i.e. options {a) and (c) are correct.
21. Nucleophiles should have either a pair of electrons to donate or should have a negative charge, i.e., options
(a) and (c) are correct.
22. Options (a) and (b) are correct.
ORGANIC CHEMISTRY-SOME BASIC PRINCIPLES AND TECHNIQUES 12/183

SHORT ANSWER QUESTIONS

NOTE : Consider structures I to VII and answer the questions 23-26.


CH3
I. CH3—CH2—CH,—CH2—OH II. CH3—CH3—CH—CH3 III. CH3—C—CH3
OH OH

low
IV. CH3—CH—CH2—OH V. CH3—CH2—O—CH2—CH3

CH3
VI. CH3—O—CH2—CH2—CH3 VII. CH,—O—CH—CH,
^ I ^
CH3

e
23. Which of the above compounds form pairs of metamers ?

re
Ans. V and VI or V and VII form a pair of metamers since they differ in the number of carbon atoms on the either

rF
side of the functional group, i.e., 0-atom.

F
24. Identify the pairs of compounds which are functional group isomers.
Ans. I and V, I and VI. I and VII ; II and V, II and VI, II and VII ; HI and V, III and VI ; IH and VII ; IV and

V ; IV and VI and IV and VII are all functional group isomers.

r
25. Identify the pairs of compounds that represent position isomerism.

fo
Ans.

26.
u
I and II. Ill and IV and. VI and VII are position isomers.
Identify the pairs of compounds that represents chain isomerism. ks
Yo
Ans. I and III, I and IV, II and III and II and IV.
oo
27. For testing halogens in an organic compound with AgN03 solution, sodium extract (Lassaigne’s test)
is acidified with dilute HNO3. What will happen if a student acidifies the extract with dilute H2SO4
in place of dilute HNO3 ?
B

Ans. If the Lassaigne’s extract is acidified with dil. H2SO4, instead of dil. HNO3, then the addition of AgN03
re

will produce white ppt. of Ag2S04 which may be mistaken for white ppt. of AgCl due to the presence of
chlorine.

28. What is the hybridisation of each carbon in H2C = C = CH2 ?


u
ad

Ans. The central carbon is ip-hybridized while both the terminal carbon atoms are -hybridized. Refer to Ans.
Yo

to Q. 1, page 12/149.
29. Explain, how is the electronegativity of carbon atoms related to their state of hybridisation in an
organic compound ?
d

Ans. Since .v-electrons are more strongly attracted by the nucleus than p-electrons, therefore, electronegativity
Re
in

increases as the 5-character of the hybridized orbital increases, i.e. in the order: .sp^ < sp^ < sp.
30. Show the polarisation of carbon-magnesium bond in the following structure.
F

CH3—CHj—CHj—CH2—Mg—X
Ans. Since electronegativityof Mg (1-2) is much lower than that of C (2-5), therefore. Mg carries a partial
positive charge while the carbon attached to it carries a partial negative charge, /.e..
-8 +5

CH3 —CH, —CH, —CH, -<-Mg ●X

31. Compounds with same molecular formula but differing in their structures are said to be structural
isomers. What type of structural isomerism is shown by

CH3
CH3—S—CH2—CH2—CH3 and CH3—S—CH
CH3
12/184 "Pfutdeep-'^. New Course Chemistry (XI)QE
Ans. Isomers which differ in the position of the functional group are called position isomers. Thus,
1

CH3
1 2 3 2I 3
CH3—S — CH2—CH2—CH3 and
CH3— Sf-CH—CH3
I II
Methyl n-propyl ether Isopropyl methyl ether
may be regarded as position isomers.

w
They cannot be regarded as metamers since metamers have different number of carbon atoms on either
side of thefunctional group. But here, the number of carbon atoms on either side of sulphur atom (functional
group) is the same, i.e., 1 and 3.
32. Which of the following selected chains is correct to name the given compound according to lUPAC

lo
system.

e
(:h2—CH3 O CH2—CH3 O

re
rF
CH3—CH—CH2—CH—C—OH CH3^CH—CH2—CH-^C—OH

F
I I
4 Carbon chain CH2 I 7 Carbon chain CH2
I I
CH2—OH CH2—OH

r
fo
ou
CH2—CH3

CH3— CH— CH2—CH— C—OH


O
ks CH2—CH3
I
CH3-rCH—CH2—CH—C—OH
O
oo
5 Carbon chain CH2 6 Carbon chain CH2
Y
I
eB

CH2—OH CH2—OH
Ans. The selected carbon chain containing four carbon atoms is correct since this is the longest possible carbon
ur

chain containing both the functional groups, i.e., carboxyl (-COOH) and hydroxyl (-OH).
The remaining three carbon chains containing seven, five and six carbon atoms respectively are incorrect
ad

since none of them contains both the functional groups.


Yo

33. In DNA and RNA, nitrogen atom is present in the ring system. Can I^eldahl method be used for the
estimation of nitrogen present in these ? Give reasons.
d

Ans. In DNA and RNA, nitrogen is present in heterocyclic rings. Since Kjeldahl method cannot be used to
Re

estimate nitrogen present in irngs, azo and nitro groups because nitrogen present in these systems/groups
in

cannot be completely converted into (NH4)2S04 during digestion. Therefore, Kjeldahl method cannot be
used to estimatenitrogenpresent in DNA and RNA.
F

34. If a liquid compound decomposes at its boiling point, which method (s) can you choose for its
purification. It is known that the compound is stable at low pressure, steam volatile and insoluble in
water.

Ans. A liquid which decomposes at its boiling point but is steam volatile, insoluble in water and stable at low
pressure can be purified by steam distillation.
Note : Answer the questions 35 to 38 on the basis of information given below :
^‘Stability of carbocations depends upon the electron releasing inductive effect of groups adjacent to
positively charged carbon atom, involvement of neighbouring groups in hyperconjugation and
99
resonance.
ORGANIC CHEMISTRY-SOME BASIC PRINCIPLES AND TECHNIQUES 12/185

35. Draw the possible resonance structures for CH3—O—CHj and predict which of the structures is
more stable. Give reason for your answer.

r
Ans. The given carbocation has two resonance structures, i.e., I and II.
A-
CH3—p—CH2 < > CH3—p=ch2

o
I II

lu
F
Structure (U) is more stable since both the carbon atoms and the oxygen atom have an octet of electrons.
36. Which of the following ions is more stable ? Use resonance to explain your answer,
e
.CH2

F
◄—►

s
©

(A) (B)

o
r
ok
Ans. Out of the following two ions, carbocation (A) is more stable than carbocation (B).

CH2
p

fo
◄—►

A B

o
Y
TheTeason being that carbocation (A) is more planar and hence is stabilized by resonance. On the other
Y
hand, carbocation (B) is less planar and hence does not undergo resonance. Alternatively, since an endocyclic
(within the ring) double bond is more stable than an exocyclic (outside the ring) double bond, therefore,
B
carbocation (A) is more stable than carbocation (B).
37. The structure of triphenylmethyl cation is given below. This is very stable and some of its salts ran be
stored for months. Explain the cause of high stability of this cation.
r
e
u
d
o
n

Ans. Due to resonance, the +ve charge can move at both the o- and one p-position of each benzene ring. As a
ad

result, three resonance structures (I, II and El) are possible for each benzene ring as shown below. Since
there are three benzene rings, therefore, there are, in all, nine resonance structures. Thus, triphenylmethyl
i

cation is highly stable due to these nine resonance structures.

jO jO
F
Re

+ Six more such


stnictures due to resonance
in the other two benzene
rings
n m
12/186 ‘Pn<^etee^'4^ New Course Chemistry (XI)B*Z

38. Write structures of various carbocations that can be obtained from 2>methylbutane. Arrange these
carbocations in order of increasing stability.
Ans. 2-Methylbutane has four different sets of equivalent hydrogen atoms marked as a, b, c and d.
b d

CH3—CH—CH2—CH3
CH3
a

Removal of one hydrogen form any of these equivalent sets of hydrogens gives four different carbocations,

w
I, II, UI and rV as shown below :
P a

CH3-^CH->-CH2-^CH2 CH3->-CH->-CH-^CH3
I I

Flo
CH3 CH3
1(1") II (2“)

ee
+ a
CH3-»-C-<-CH2—CH3 CH2-^CH-^CH2—CH3

Fr
I
CH3 CH3
m (3“) IV (1”)

r
ur
Now stability of carbocations decreases in the order : 3® > 2® > 1®. Since III being the 3® carbocation, it is

fo
the most stable. This is followed by carbocation (II) which is 2®. Out of the other two 1® carbocations,
carbocation (TV) has an electron-donating CH3 group at a-carbon while carbocation (I) has a CH3 group at
P-carbon. Since +I-effect decreases with distance, therefore, carbocation (TV) is more stable than carbocation
ks
(I). Thus, the overall stability of these four carbocations increases in the order .* I < IV < II < III.
Yo
39. Three students, Manish, Ramesh and Rajni were determining the extra elements present in an organic
oo
compound given by their teacher. They prepared the Lassaigne’s extract (L.E.) independently by the
fusion of the compound with sodium metal. Then they added solid FeS04 and dilute sulphuric acid to
B

a part of Lassaigne’s extract Manish and R^ni obtained prussian blue colour but Ramesh got red
re

colour. Ramesh repeated the test with the same Lassaigne’s extract, but again got red colour only.
They were surprised and went to their teacher and told him about their observation. Teacher asked
them to think over the reason for this. Can you help them by giving the reason for this observation.
u
ad

Also, write the chemical equations to explain the formation of compounds of different colours.
Yo

Ans. If the oiganic compound contains both N and S, then during fusion it may form either sodium thiocyanate
(NaSCN) or a mixture of sodium cyanide (NaCN) and sodium sulphide (Na2S) depending upon the amount
of sodium metal used. If the sodium metal used is less, only NaSCN is produced.
This then reacts with Fe^'*' ions (produced by oxidation of Fe^'*' ions during preparation of Lassaigne’s
nd
Re

extract) to give red colouration due to the formation of ferric thiocyanate.


Fi

Aerial oxidation
Fe2+ Fe3+ ; Fe3+ + 3 NaSCN ■> Fe(SCN)3 + 3Na+
Ferric thiocyanate
{Red colouration)
However, if excess of sodium metal is used, the initially formed sodium thiocyanate decomposes to form a
mixture of sodium cyanide and sodium sulphide.
A
NaSCN + 2Na -> NaCN +
Na2S
Sod. thiocyanate Sod. cyanide Sod. sulphide
NaCN thus formed then reacts with FeS04, more NaCN and Fe^"*" ions to give ultimately prussian blue
colour due to the formation of ferric ferocyanide or iron (IE) hexacyanoferrate (II).
2 NaCN + FeS04 4 Na2S04 + Fe(CN)2
ORGANIC CHEMISTRY-SOME PRINCIPLES AND TECHNIQUES 12/187

Fe(CN)2 + 4NaCN ■»
Na4[Fe(CN)g]
Sod. hexacyanoferrate (II)

ow
3Na4[Fe(CN)6]+4Fe3+ ■>
Fe4[Fe(CN)e]3 + 12Na+
Iron (III) hexacyanoferrate (II)
(Prussian blue)
From the above discussion, it follows that Manish and Rajni used excess sodium and hence formed NaCN
in the Lassaigne *s extract which gave prussian blue colour due to theformation of iron (HI) hexacyanoferrate
(II) while Ramesh used less sodium and hence formed NaSCN in the Lassaigne's extract which gave red
colouration due to the formation offerric thiocyanate.

re
40. Name the compounds whose line fonnulae are given below :
O NO

Flr
F
(0 («)

ou N02

sr
11 , Number the carbon atoms of I Number the carbon atoms
● \5 the longest possible carbon 2
of the irng in such a way

ko
Ans. chain in such a way that the that double bond gets the
^ ^ functional group ; i.e., > C = O 1
lowest possible locants
gets the lowest possible locant

of
7
3-NitrocycIohex-1 -enc followed by the —NO2 group
3-Ethyl-4-methylhept-
5-en-2-one

41. Write structural formulae for compounds named as-


o
Y
(a) 1-Bromoheptane (b) 5-Bromoheptanoic acid
erB

7 6 5 4 3 2 1

Ans. CH3—CH2—CH2—CH2—CH2—CH2—CH2Br
uY

1-Bromoheptane
7 6 5 4 3 2 1

(b) CH3—CH2—CH—CH2—CH2—CH2—C—OH
I
Br O
ad
do

S-Bromoheptanoic acid
42. Draw the resonance structures of the following compounds :
in

(i) CH2 = CH—a: (i*)CH2 = CH—CH = CH2


Re

(m) CH2 = CH—C=0


F

A +

Ans. (OCHjiCH—Ci: ■* 0 ●
► -:ch2—CH=ci:

(«)CH2=^OT^Ch£cH2 ^ ►
+

CH2—CH=CH—CH2 ^ > -:CH2—CH=CH—CH2

+
(Hi) CH 2—CH^C=0 < ► CH2—CH=C—O
I
H H
12/188 ‘P.neuCecfr'A Course Chemistry (XI)EJ20SI]

43. Identify the most stable species in the following set of ions giving reasons :
+ + + + 0 0 0 0

(0 CH3, CHjBr , CHBr2 ’ CBr^ m CH 3 ’ CH2C1, CHCI2, CCI3


+

Ans. (/) CH3 is the most stable species because the -1-effect of Br intensifies the -i-ve charge and hence destabilises
the species. Further, more the number of Br atoms, less stable is the species. Thus, the stability of these
+ + + +

species decreases in the order; CH3 > CHjBr > CHBr, > CBr3 ●
(ii) -I-effect of the Cl atom disperses the -ve charge and thus stabilizes the species. Further, more
the number of Cl atoms, more is the dispersal of the -ve charge and hence more .stable is the species.

w
Thus, “CCI3 is the most stable species. The stability of other species decreases in the order :
XHCI2 > “CH2C1 > -CH3.
44. Give three points of differences between inductive effect and resonance effect.

F lo
Ans. The main points of difference between inductive and resonance effects are given below :
Inductive Effect Resonance Effect

ee
1. It involves displacement of only a-electrons 1. It involves delocalisation of n or n (lone pairs)

Fr
and hence occurs only in saturated of electrons and hence occurs in unsaturated and
compounds. conjugated systems.
2. During inductive effect ; the electron pair is 2. During resonance effect, the electron pair is

for
only slightly displaced towards the more completely transferred and hence full positive
electronegative atom and hence only partial and negative charges appear.
ur
positive and negative charges appear.
3. Inductive effects are transmitted over short 3. The resonanee effects are transmitted all along
the length of the conjugated system without
oks
distances in saturated carbon chains and the
magnitude of the effect decreases rapidly as suffering much change in magnitude. For
Yo
the distance from the heteroatom increases. example, C3 in crotonaldehyde is almost as
o

The effect almost becomes negligible beyond positive as C I-


eB

three carbon atoms from the heteroatom

556+ 65+ 6+ CH3—CH = CH—CH=i=0-<—►


c c c c Cl. Crotonaldehyde
our
ad

CH3—CHiCH^CH—O"
4 3 2 1

<—► CH3— CH—CH = CH—O"


4 3 2 1
Y
Re

45. Which of the following compounds will not exist as resonance hybrid. Give reason for your answer:
nd

(i) CH3OH
(i7) R—CONH,
Fi

(i/i) CH3CH = CHCH2NH2


Ans. (0 CH3—OH does not contain 7t-electrons and hence it cannot be regarded as a resonance hybrid.
(//) Due to the presence of 7t-electrons in C = O bond and /r-electrons on N, amide can be represented as a
resonance hybrid of the following three resonating structures.

0:- o:-

R—C^^
R— ●4 ●4 > R—

NH2 NH2 NH2


I II III
ORGANIC CHEMISTRY-SOME BASIC PRINCIPLES AND TECHNIQUES 12/189

(m) Since the lone pair of electrons on the N atom is not conjugated with the ji-electrons of the double
bond, therefore, resonance is not possible.

CH3—CH = CH—CH,—NH2 Resonancenot possible.


46. Why does SO3 act as an electrophile ?
Ans. Three highly electronegative oxygen atoms are attached to sulphur atom. It makes sulphur atom electron-
deficient. Further, due to resonance, sulphur acquires positive charge. Both these factors, make SO3 an

ow
electrophile.
O O
?v
1:
^ -OAo
.S

O O" O O

e
47. Resonance structures of propenal are given below. Which of these resonating structures is more

re
stable ? Give reason for your answer.

rFl
0 e

F
CH2 = CH—CH = o ^ » CHj—CH = CH—O
I II

Ans. Resonance structures of propenal are shown below :

r
ou
sfo
+

CH2—CH^CH=^0 CH2—CH = CH—O


I II

k
Structure II involves separation of +ve and -ve charges. Furthermore, the terminal carbon has only a sextet
oo
of electrons. Due to these two factors, structure (II) is less stable than structure (I).
48. By mistake, an alcohol (boiling point 97°C) was mixed with a ketone (boiling point 68‘*C). Suggest a
Y
suitable method to separate the two compounds. Explain the reason for your choice.
eB

Ans. If the boiling points of two components of a mixture differ by more than 20°, then these components can be
easily separated by simple distillation since at the boiling point of low boiling liquid, the vapours would
consist entirely of only low boiling liquid without any contamination of vapours of high boiling liquid and
r

vice-versa.
ou
Y
ad

49. Which of the two structures (A) and (B) given below is more stabilised by resonance ? Explain.
e
CH3COOH and CH3COO
d

(A) (B)
in
Re

Ans. Resonance structures of compound (A) and (B) are given below :

.0: /P.:-
F

4 ► R—C
CH3—c:^. ◄ ► CH3—C,. + ;
^o:
A^P—H ^p—H B^or ● ●

I n m IV

Resonance structure (II) carries separation of +ve and -ve charges and hence it is less stable than structure (I).
As a result, contribution of structure (II) towards resonance hybrid of compound A is less than that of
structure (I).
In contrast, both the resonance structures of compound (B), i.e., HI and IV are of equal energy and hence
contribute equally towards the resonance hybrid of structure (B). Thus, structure (B) is more stable than
structure (A).

A
12/190 T^^uteCe^'^ New Course Chemistry (XI)K3SIHD

MATCHING TYPE QUESTIONS

In the following questions, more than one correlation is possible between options of Column I and
Column II. Make as many correlations as you can.
50. Match the type of mixture of compounds in Column I with the technique of separation/puriflcation
given in Column II.
Column 1 Column II

(0 Two solids which have different solubilities in a solvent (a) Steam distillation
and which do not undergo reaction when disoslve in it.
(//) Liquid that decomposes at its boiling point (b) Fractional distillation

w
(Hi) Steam volatile liquid (c) Simple distillation
(iv) Two liquids which have boiling points close to each other (d) Distillation under reduced pressure

F lo
(v) Two liquids with large difference in boiling points. (e) Crystallisation

Ans. (i)^(e); (H)-^(d) ; (Hi)->(a); (iV)^(/)); (v)-^(c).


51. Match the terms mentioned in Column I with the terms In Column II.

ee
Column I Column II

Fr
(0 Carbocation (a) Cyclohexane and 1-hexene
(ii) Nucleophile (b) Conjugation of electrons of C—H a bond with empty
p-orbital present at adjacent positively charged carbon.

for
m Hyperconjugation (c) sp^ Hybridised carbon with empty p-orbital
ur
(iv) Isomers (d) Ethyne
(v) sp Hybridisation (e) Species that can receive a pair of electrons
oks
(Vi) Electrophile (f) Species that can supply a pair of electrons
Yo
Ans. (i)-^(c), (e) Carbocations contain jp^-hybridized carbon with empty p-orbital. They may also be regarded
o

as species that can receive a pair of electrons. Therefore, options (c) and (e) are correct. ;
eB

; (iii)^(b); (iv)^(a); (v)^(d) ; (vi)^(e).


52. Match Column I with Column 11.
Column 1 Column II
our
ad

(/) Dumas method (a) AgN03


(//) Kjeldahl’s method (b) Silica gel
(Hi) Carius method (c) Nitrogen gas
Y

(iv) Chromatography (d) Free radicals


Re

(v) Homolysis (e) Ammonium sulphate


nd

Ans. (O-^(c); (H)^(e) ; (m)->(a) ; (iv)^(b); (v)-^(d).


Fi

53. Match the intermediates given in Column I with their probable structure in Column II.
Column I Column II

(i) Free radical (a) Trigonal planar


(ij) Carbocation (b) Pyramidal
(Hi) Carbanion (c) Linear
Ans. (0“^(a), (b); (H)->(a); (iii)^(a), (b).
Note : Please note that simple free radicals are trigonal planar but bridge head free radicals are pyramidal.
Similarly, simple carbanions are pyramidal while those in which the carbon atom carrying the negative
charge is adjacent to a double bond or a benzene ring are trigonal planar.
ORGANIC CHEMISTRY-SOME BASIC PRINCIPLES AND TRCHNIQUES 12/191

54. Match the ions given in Column 1 with their nature given in Column II.
Column I Column 11

(/) CH3—O—CH—CH3 (a) Stable due to resonance

(n) F3—C@ ib) Destabilised due to inductive effect

CH3
m CH3—C© (c) Stabilised by hyperconjugation

w
CH
3

(/V) CH3—CH CH3 id) A secondary carbocation

Flo
Ans. (b), id);
(b);

ee
Ip Ip
Due to matching sizes of 2/j-orbitals of C and F,

Fr
the lone pair of electrons on F can be donated to F
the empty 2p-orbital of carbon carrying the -t-ve C F
charge. Due to this pn-pn back bonding, F3C'‘ is F

or
stabilized by resonance.
ur
f
Thus, options, (fl) and (h) are correct. ; ks
{iii)^(b);
Yo
(fv)—>(c), (d) It is a secondary carbocation. It can also be stabilized by hyperconjugation. Therefore,
oo
options, (c) and (d) are correct.
eB

ASSERTION AND REASON TYPE QUESTIONS

In the following questions a statement of Assertion (A) followed by a statement of Reason (R) is
ur

given. Choose the correct option out of the choices given below each question.
ad

55. Assertion (A) : Simple distillation can help in separating a mixture of propan-l-ol (boiling point 97°C)
Yo

and propanone (boiling point 56“C).


Reason (R) : Liquids with a difference of more than 20"C in their boiling points can be separated by
simple distillation,
nd

(a) Both A and R are correct and R is the correct explanation of A.


Re

(b) Both A and R are correct but R is not the correct explanation of A.
Fi

(c) Both A and R are not correct. id) A is not correct but R is correct.
Ans. {a) R is the correct explanation of A.
56. Assertion A : Energy of resonance hybrid is equal to the average of energies of all canonical forms.
Reason (R) : Resonance hybrid cannot be represented by a single structure,
(a) Both A and R are correct and R is the correct explanation of A.
(b) Both A and R are correct but R is not the correct explanation of A.
(c) Both A and R are not correct.
(d) A is not correct but R is correct.
Ans. id) Correct A. The energy of the resonance hybrid is equal to the sum of the energies of the various
canonical structures in proportion of their contribution towards the resonance hybrid.

*
12/192 “P^eicUep^'^ New Course Chemistry (XI)SaalHD

57. Assertion (A) : Pent-l-ene and pent-2-ene are position isomers.


Reason (R) ; Position isomers differ in the position of functional group or a substituent.
(«) Both A and R are correct and R is the correct explanation of A.
(b) Both A and R are correct but R is not the correct explanation of A.
(c) Both A and R are not correct. (d) A is not correct but R is correct.
Ans. (a) R is the correct explanation of A.
58. Assertion : All the carbon atoms in H7C = C = CH7 are sp^ hybridised.
Reason (R) : In this molecule all the carbon atoms are attached to each other by double bonds.

w
(a) Both A and R are correct and R is the correct explanation of A.
(b) Both A and R are correct but R is not the correct explanation of A.
(c) A is correct but R is not correct. id) A is not correct but R is correct.

F lo
Ans. (d) Correct A. In CH^ = C = CH, the central carbon is 5/?-hybridized while the terminal carbon atoms are
sp^-hybridized.
59. Assertion (A): Sulphur present in an organic compound can be estimated quantitatively by Carius method.

ee
Reason (R) ; Sulphur is separated easily from other atoms in the molecule and gets precipitated as light
yellow solid,

Fr
(a) Both A and R are correct and R is the correct explanation of A.
(b) Both A and R are correct but R is not the correct explanation of A.

for
(c) Both A and R are not correct. (d) A is correct but R is not correct.
ur
Ans. (d) Correct R. Sulphur is oxidised to H2SO4 in Carius method and then estimated as biu^ium sulphate.
60. Assertion (A) : Components of a mixture of red and blue inks can be separated by distributing the
components between stationary and mobile phases in paper chromatography.
s
ok
Reason (R): The coloured components of inks migrate at different rates because paper selectively retains
Yo
different components according to the difference in their partition between the two phases.
o

(a) Both A and R are correct and R is the correct explanation of A.


eB

(b) Both A and R are correct but R is not the correct explanation of A.
(c) Both A and R are not correct. id) A is not correct but R is correct.
Ans. id) R is the correct explanation of A.
r
ou
ad

LONG ANSWER QUESTIONS

61. What is meant by hybridisation ? Compound CH2 = C = CH2 contains sp or sp^ hybridised carbon
Y

atoms. Will it be a planar molecule ?


Ans. For definition of hybridization, refer to unit 5, and for structure refer to Ans. to Q. 1, page 12/151.
Re
nd

62. Benzoic acid is an organic compound. Its crude sample can be purified by crystallisation from hot
water. What characteristic differences in the properties of benzoic acid and the impurity make this
Fi

process of purification suitable ?


Ans. Benzoic acid can be purified from hot water because of the following characteristics.
(0 Benzoic acid is more soluble in hot water and less soluble in cold water,
(11) Impurities associated with benzoic acid are either insoluble in water or are more soluble in water to such an
extent that when a hot saturated solution of benzoic acid is cooled, the impurities remain in the solution.
Since impure benzoic acid satisfies both these conditions, therefore, benzoic acid can be purified by
crystallization.
63. Two liquids (A) and (B) can be separated by the method of fractional distillation. The boiling point
of liquid (A) is less than boiling point of liquid (B). Which of the liquids do you expect to come out
first in the distillate ? Explain.
ORGANIC CHEMISTRY’SOME BASIC PRINCIPLES AND TECHNIQUES 12/193

Ans. Fractional distillation is used to separate the components of a mixture if their boiling points differ by 20° or
less. In this method, a fractionating column is used in between the flask and the condenser (Fig. 12.25,
page 12/101). The purpose of the fractionating column is to provide hurdles for the ascending vapours and
to provide large surface area for condensing the high boiling liquid. Consequently, the vapours of the low
boiling liquid (A) will move up while those of the high boiling liquid will condense and fall back Into the
flask. As a result of this fractionation, liquid A with lower boiling point will distil first and liquid B with
higher boiling point afterwards.
64. You have a mixture of three liquids A, B and C. There is a large difference in the boiling points of A
and rest of the two liquids i.e., B and C. Boiling point of liquids B and C are quite close. Liquid A
boils at a higher temperature than B and C and boiling point of B is lower than C. How will you

w
separate the components of the mixture. Draw a diagram showing set up of the apparatus for the process.
Ans. Since the boiling point of liquid A is much higher than those of liquids B and C whose boiling points are
quite close, therefore, separate liquid A by simple distillation using the apparatus shown in Fig. 12.24,
page 12/100. Since the boiling points of liquids B and C tue quite close but are much lower than that of

F lo
liquid A, therefore, mixture of liquids B and C will distil together leaving behind liquid A. On further
heating liquid A will distil over.
Now place the mixture of liquids B and C in a flask fitted with a fractionating column as shown in Fig.

ee
12.26, page 12/101. Since the b.p. of liquid B is lower than that of liquid C, therefore, on fractional
distillation, first liquid B will distil over and then liquid C.

Fr
65. Draw a diagram of bubble plate type
fractionating column. When do we FIGURE 12.39
require such type of a column for Condensed

for
separating two liquids. Explain the Vapors —
ur
principle involved in the separation of
components of a mixture of liquids by
using fractionating column. What
industrial applications does this
s
ok
process have ?
Yo
Ans. The bubble plate type fractionating Mixture ^
column is shown in the Fig. 12.39. The
o
—Condenser
of liquids
n
eB

tower is divided into number of


compartments by means of shelves
having openings. The openings are
covered with caps called bubble caps.
r

Each shelf is provided with an overflow Overflow Low boiling liquid


ou
ad

pipe which keeps the liquid to a certain Pipe


Bubble Plate
level and then allows the rest to trickle Stem

down to the lower shelf. Such type of


Y

column is used for continuous separation


of bulk quantities of liquids. For
example, distillation of fermented liquid
Re
nd

for manufacture of rectified spirit. fr

Principle of fractional distillation.


Fi

Refer to Art. 12.6.6, pages 12/101*12/102.


Industrial application. (/) Separation of >High boiling fraction
crude oil in petroleum industry into
various useful fractions such as gasoline, Bubble plate type fractionating column
kerosene oil, diesel oil, lubricating oil.
etc.

(/i) Separation of acetone and methanol from pyroligneous acid obtained by destmctive distillation of
wood.

66. A liquid with high boiling point decomposes on simple distillation but it can be steam distilled for its
purification. Explain how is it possible ?
Ans. Refer to Art. 12.16.8, pages 12/103-12/104.
12/194 ‘P'tadec^'4. New Course Chemistry (XI)SSEIHD

k!k^' m
■f^

NEET/JEE
SPECIAL

w
F^oriultlmate preparation.'oH^^nitJor competitive examinations, students should refer to
● MCQs in Chemistry for NEET

F lo
V
Pradeep's Stellar Series.... ● MCQs in Chemistry for JEE (Main)

,separately.available for these examinations. . SSB


' trnr'i ~i-iivtr>i

ee
Multiple Choice Questions (with one correct Answer)

Fr
1. Hybriurtation, (a) 2 (b)3

for
r'iassification and Nomendature (c)4 (d)5

1. Hybridization order of the carbon atoms from left (West bengal JEE 2015)
ur
to right is: 5. Which of the following molecules represents the
CHj = C = CH — CH3 order of hybridization sp^, sp^, sp, sp from left to
s
right atoms ?
(a) sp^, spy sp^y sp^ ib) sp^, sp^y sp^, sp^
ook
Yo
(a) HC = C-C = CH
(c) sp^y Sp, sp, sp^ {(i) sp, sp, sp^, sp^
(JEE Main 2021) (b) CHt = CH-C s CH
eB

2. How many (/) -hybridized carbon atoms and (c) CH2 = CH-CH = CH2
(/t) 7U-bonds are present in the following (d) CH3-CH = CH-CH3 (NEET 2018)
compound ? 6. In which of the following molecules, all atoms
our
ad

are coplanar ?
C = C - COOCH3

ia) 8,6 (b) 1, 6


Y

(c) 8, 5 id) 1, 5 (NEET 2020) T


io) ib)
Re

3. In allene (C3H4), the type(s) of hybridization of


nd

the carbon atoms is (are)


id) sp and sp^ ib) sp and sp^
Fi

(c) only sp' id) sp^ and sp^ CH3. ●CN

(IIT Paper I, 2012) ic) :c=c id)


4. In the following compound, the number of 'sp’ CH3 ■CN

hybridized carbon is (NEET Phase-II 2016)

CH2 = C = CH -CH-C = CH 7. The number of sigma (o) and pi (7t) bonds in


I
CN
pent-2-en-4-yne is

ANSWERS

1. ia) 2. ib) 3. ib) 4. (c) 5. ib) 6. (a)


ORGANIC CHEMISTRY<SOME BASIC PRINCIPLES AND TECHNIQUES 12/195

{a) 13 (7 bonds and 10 71 bonds {d) 4-ethyl-3-methylhex-2-ene


{b) 10 a bonds and 3 7C bonds (JEE Main (online) 2018)

(c) 8 a bonds and 5 k bonds 13. What is the lUPAC nomenclature of the given
compound ?
(d) 11a bonds and 2 K bonds (NEET 2019)
8. The total number of 7i-bond electrons in the
following structure is (a) 5-Ethynyl-l, 6-heptadiene
H H (b) 3'EthynyM, 6-heptadiene
H3C CH3 (c) 3-Vinyl-hept-6-en-l-yne

w
(d) 5-Vinyl-hept-l-en-6-yne (J & K CET 2018)
CH2 H CH3
14. Which one of the following is j-butylphenylviny!
(a) 12 (b) 16 methane ?

F lo
(c)4 W8 (AIPMT 2015) Ph Me
Me
9. The enolic form of ethyl acetoacetate as shown
below has
(a) (i)

ee
H Me Me
H2 Ph
0

Fr
Me
C c
Ph
OH OC2H5 0
OC2H5

for
Me
(a) 9 sigma bonds and 2 pi-bonds (c) Me
/X/V' (d)
ur
(b) 9 sigma bonds and 1 pi-bond
Me Ph Me
(c) 18 sigma bonds and 2 pi-bonds
(West Bengal JEE 2011)
(d) 16 sigma bonds and 1 pi-bond
s
15. lUPAC name of the compound
ok
(AIPMT 2015)
Yo
10. Which one of the following is a benzenoid H3C H
\ /
o

aromatic compound ? H is
eB

(a) Furan (b) Thiophene H


H V
(c) Pyridine (d) Aniline Br

(e) Cyclopentadienyl anion (Kerala PET 2016) (a) 1-bromobut-2-ene (b) 2-bromo-2-butene
r

11. Tropolone is an example of (c) bromobutane (d) l-bromobut-3-ene


ad
ou

(a) benzenoid aromatic compound (Karnataka CET 2016)


(b) non-benzenoid aromatic compound 16. In the lUPAC system, PhCH2CH2C02H is named
(c) alicyclic compound
Y

as

(d) acyclic compound (a) 3-phenylpropanoic acid


(e) heterocyclic aromatic compound
Re

(b) benzylacetic acid


nd

(Kerala PMT 2014) (c) carboxyethyl benzene


12. The lUPAC name of the following compound is (d) 2-phenylpropanoic acid
Fi

(West Bengal JEE 2017)


17. The lUPAC name of the following compound is
CH.—CH—CH,
I I I
(a) 4-methyl-3-ethyIhex-4-ene CN CN CN
4, 4-diethyl-3-methyIbut-2-ene (fl) 1, 2, 3-tricyanopropane
(c) 3-ethyl-4-methylhex-4-ene
(b) propane-1, 2, 3-trinitri)e
ANSWERS
7. (b) 8.(d) 9. (c) 10.(d) 11. (b) 12.(d) 13.(b) 14. (c) 15.(a) 16.(a)

»
12/196 ‘Pxadee^’^ New Course Chemistry (XI)EZalHD

(c) 3-cyanopentane-l, 5-dinitrile 22. The lUPAC name of the following compound is
id) 1, 3, 5-pentanetrinitrile (AMU Engg. 2014) CHO

18. The correct decreasing order of priority of .CH3


fijnctional groups in naming an organic compound
as per lUPAC system of nomenclature is : H3C
COOH
(fl) - COOH > - CONH2 > - COCi > - CHO
(a) 2, 5-dimethyl-5-carboxyhex-3-enal
{b) - SO3H > - COCI > - CONH2 > - CN
(b) 2, 5-diraethyl-6-carboxyhex-3-enal
(c) - COOR > - COCI > - NH2 > C = 0

low
(c) 2, 5*dimethyl-6-oxohex-3-enoic acid
{d) - COOH > - COOR > - CONH2 > - COCI (d) 6-formyl-2-methylhex-3-enoic acid
(JEE Main 2022) (JEE Main 2020)
19. The correct lUPAC name of the following 23. The lUPAC name of the compound
compound is O
O

ee
H
O2N

rF H IS

Fr
O 0

(a) 4-melhyl-2-notro-5-onohept-3-enal (fl) 5-formylhex-2-en-3-one


(b) 4-methyl-5-oxo-2-nitrohept-3-enal (b) 5-methyl-4-oxohex-2-en-5-al

for
(c) 4-methyl-6-nitro-3-oxohept-4-enal (c) 3-keto-2-methylhex-5-enal
(d) 6-formyl-4-methyl-2-nitrohex-3-enal (d) 3-keto-2-methylhex-4-enal (MEET 2017)
u
(JEE Main 2022) 24. Structure of the compound whose lUPAC name is
ks
3-ethyl-2-hydroxy-4-methylhex-3-en-5-ynoic acid
20. The correct lUPAC name of the following
Yo
is
compound is :
oo
OH
eB

COOH COOH
ia) ib)

OH
(a) 6-bromo-2-chloro-4-methylhexan-4-o 1
r

OH OH
(b) 1 -bromo-4-methyl-5-chlorohexan-3-o 1
ou
ad

(c) 6-bromo-4-methyl-2-chiorohexan-4-o 1
COOH COOH
(d) 1 -bromo-5-chloro-4-methylhexan-3-o 1 ic) (d)
Y

(NEET 2022)
21. The correct structure of 2,6-dimethyldec-4-ene is (AIPMT 2013)
Re
nd

25. Which one of the following structures has the


lUPAC name:
Fi

(a) (b)
3-ethynyl-2-hydroxy-4-methylhex-3-en-5-ynoic
acid ?

OH

HO2C,
ic) id) (a) CO2H .(b) %

OH
(NEET 2021)

ANSWERS

17. (c) 18. (h) 19. (c) 20. (d) 21. (a) 22. (c) 23. (</) 24. (d) 25. (J)

I
ORGANIC CHEMISTRY-SOME BASIC PRINCIPLES AND TECHNIQUES 12/197

OH OH 29. lUPAC name of 4-isopropyl-m-xylene is


(a) 1-Isopropyl-2, 4-dimethylbenzene
(c) CO2H COjH (b) 4-Isopropyl-m-xylene
(c) 1-Isopropyl-3, 5-dimethylbenzene
(d) 4-Isopropyl-3, 5-dimethylbezene.
(JEE Advanced 2020)
30. Amongst the following the one which can exist in
26. The lUPAC name of the following compound is
free state as a stable compound is
CH,
(a) C7H9O {b) CgH,20

low
(c) C^HiiO (d) C10H12O2
(West Bengal JEE 2013)
II. Isomerism

(a) 3-bromo-5-methylcyclopanecarboxylic acid 31. Identify the correct statement in the following :

(b) 3-bromo-5-methylcyclopentanoic acid (fl) Dimethyl ether and ethanol are chain isomers

ee
(c) 5-bromo-5-methylcyclopentanoicacid (b) Ethanoic acid and methyl methanoate are
(d) 4-bromo-2-methyIcyclopentanecarboxylic

rF position isomers

Fr
acid (JEE Main 2020) (c) n-Butane and isobutane are functional isomers.
27. The lUPAC name of the following compound is {d) Propan-l-ol and propan-2-ol arc position
isomers. (Karnataka GET 2017)

for
,OH
32. The correct statement regarding a carbonyl
NH2 compound with a hydrogen atom on its alpha
u
a
carbon is
ks
O^N
(a) a carbonyl compound with a hydrogen atom
Yo
on its alpha-carbon rapidly equilibrates with
oo
CHO
its corresponding enol and this process is
(a) 2-nitro-4-hydroxymethyl-5-aminobenzal- known as carbonylation
eB

dehyde (b) the carbonyl compound with a hydrogen atom


(b) 3-amino-4-hydroxymethyl-5-nitrobenzal- on its alpha-carbon rapidly equilibrates with
dehyde
its corresponding enol and this process is
known as keto-enol tautomerism
r

(c) 5-amino-4-hydroxymethyl-2-nitrobenzal-
ou
ad

dehyde ic) a carbonyl compound with a hydrogen atom


(d) 4-amino-2-formyl-5-hydroxymethylnitro-
on its alpha-carbon never equilibrates with its
corresponding enol
Y

benzene (JEE Main 2020)


(d) a carbonyl compound with a hydrogen atom
28. The lUPAC name for the compound
on its alpha-carbon rapidly equilibrates with
nd
Re

NO2 its corresponding enol and this process is


F. known as aldehyde-ketone equilibrium
Fi

Q IS
(NEET Phase-I 2016)
33, Which of the following compounds will show the
CH3 maximum ‘enoT content ?
(a) l-fluoro-4-methyl-2-nitrobenzene (a) CH3COCH2COCH3
ib) 4-fluoro-1 -methyl-3-niirobenzene
(b) CH3COCH2CONH2
ic) 4-methyl-1 -fluoro-2-nitrobenzene
(C) CH3COCH3
id) 2-fluoro-5-methyl-l-nilrobenzene
(d) CH3COCH2COOC2H5 (JEE Main 2019)
(J & K GET 2010)
ANSWERS

26.id) 27.ic) 28.(a) 29.(a) 30.{b) 31. id) 32. ib) 33.(a)
12/198 New Course Chemistry (Xl)CSlSn

34. The compound which shows metamerism is (a) CH3-C-CH3 (b) CH2 = CH-0H
(a) C4H10O ib) CgHi2 O
(c) C3HgO (d) C3H6O O 0
(NEET 2021)
O O
35. Which of the following has maximum enol
(c) id)
content ?

O
0 o

,0 NH

w
(e) (Kerala PMT 2007)
(a) ib)
0 39. Which of the following does not exhibit

F lo
tautomerism ?
0

{‘J) 0 0 (b) O 0

ic) id) 0 0

ee
o- 0
ic) 0 id) O
/

Fr
(JEE Main 2022)
0 0
36. Given :

for
(Odisha JEE 2008)
H3C CH3 40. The total number of acyclic structural isomers
our
CH3
X \
possible for compound with molecular formula
C^HiqO is
CH3
oks
ia)9 ih)l

O
YO
CH3
ic)5 id) 6
o

(^)8 (Kerala PET 2013,2016)


(I) (II) (lU)
Y
eB

HI. Fundamental Concepts


Which of the following compounds can exhibit
tautomerism ?
of Organic Reaction Mechanism
{a) II and III ib) I, n and UI 41. Which of the following statements is not correct
our
ad

(c) I and II
for a nucleophile ?
id) I and III
(a) Ammonia is a nucleophile
(AIPMT 2015)
{b) Nucleophiles attack low e~ density sites
37. Which among the given molecules can exhibit
(c) Nucleophiles are not electron seeking
Y

tautomerism ?
(d) Nucleophile is a Lewis acid
Re
nd

(RE-AIPMT 2015)
42. Which of the following is the most correct electron
Fi

displacement for a nucleophilic reaction to take


place ?
(ii) III only (b) Both I and III
HC=^C—C—Cl
r\ h2,y
(c) Both I and II id) Both II and III (a) H3C
H
(NEET Phase-I 2016)
38. Which one of the following compounds cannot H r\ H2
show tautomerism ? ib) H3C->-C=!=C^C—Cl
H
ANSWERS

34. (a) 35. ic) 36. ib) 37. (a) 38. id) 39. (a) 40. (/>) 41. (d)
ORGANIC CHEMISTRY-SOME BASIC PRINCIPLES AND TECHNIQUES 12/199

H Hirv
(c) H3C-»-C=C— C-^Cl (a) (b)
H
o

H2pV
id) H3C C= c—Cl (AIPMT 2015) (c) id)
H

ow
43. Which of the following is correct regarding the
-I-effecl of the substituents ? (JEE Main 2017)
(a) -NR2<-0R<-F 48. The molecule/molecules that has/have delocalised

(b) -NR2>-0R<-F lone pair(s) of electrons is/are


(c) -NR2<-0R>-F I. :0:

e
id) -NR2>-0R>-F OCH3

re
44. The correct statement regarding electrophile is H3C CH2"

rFl
ia) electrophile is a negatively charged species
and can form a bond by accepting a pair of

F
II.
electrons from another electrophite
(b) electrophiles are generally neutral species and
H3C CHi
can form a bond by accepting a pair of

r
electrons from a nucleophile
ou
fo
III.
(c) electrophile can be either neutral or positively

ks
charged species and can form a bond by accep
ting a pair of electrons from a nucleophile
id) electrophile is a negatively charged species
oo
IV. CH3CH=CHCH2NHCH3
and can form a bond by accepting a pair of
electrons from a nucleophile. (NEET 2017) ia) I, II, III ib) UUV
Y
B

45. The indicated atom is not a nucleophilic site in (c) I and III id) only III
(West Bengal JEE 2019)
ia) BH- ib) CH^Mgl
re

T T 49. Which of the following carbocations is expected


to be most stable ?
(c) CH3OH id) CH,NH.,
ou

T T
Y

NO2 NO2
ad

(West Bengal JEE 2019)


46. Consider thiol anion (RS®) and alkoxy anion ia) ib)
(RO®). Which of the following statement is
d

correct ?
Y
x: H
XH
Y
in

(a) RS® is less basic and less nucleophilic than


Re

RO® NO2 NO2


ib) RS® is less basic but more nucleophilic than
F

H
RO® /
ic) H\ id)
(c) RS® is more basic and more nucleophilic than
RO® Y

id) RS® is more basic but less nucleophilic than (NEET 2018)
RO® (AIEEE 2011) 50. The correct order of stability for the following
47. Which of the following molecules is least alkoxides is
resonance stabilised ?

ANSWERS

42.(«) 43.(n) 44.(c) 45.ia) 46. {h) 47. {b) 48. {d} 49. in
12/200 "p>uuUe^ <t New Course Chemistry (XI)

0" 0~ 0" (c) <5-p (filled) and c-k electron delocalisations


V/ 02N (t/) p (filled) —> a* and a-7C* electron
delocalisations (JEE Advanced 2013)
NO, NO2
55. The rate of the reaction
(A) (B) (C)

(a) (C) > (B) > (A) (h) (B)>(A)>(C)


R CH2Br + N
(. ) (B)>(C)XA) (d) (0>(A)>{B)
(JEE Main 2020)
51. R- Br-
\\

w
+

CH2
+ + +
is influenced by the hyperconjugation effect of
CH2 CH
CH2 group R. If R sequentially is

F lo
I. CH3- II. CH3—CH2-
(A)
H
(C) (D) CH3
ni. H3C—CH¬

ee
IB) IV. H3C—c-
Among the given species, the resonance stabilized

Fr
carbocations are :
CH3 CH3
(a) (A), (B) and (C) only the increasing order of speed of the above reaction

for
(b) (C) and (D) only (c) (A), (B) and (D) only is

(d) (A) and (B) only (JEE Main 2021) (a) IV, III, II, I ib) I, II, III, IV
ur
52. Which of the carbocations is most stable ? (c) 1. IV, III. II (d) IIUU.IV

OCH3 OCH (e) II, III, I, IV (Kerala PMT 2007)


s
(a) (b) 56. A tertiary butyl carbocation is more stable than a
ook
Yo
+ secondary butyl carbocation because of which of
the following ?
eB

(c) H3CO (d) +


(a) + R-effect of -CH3 groups
H3CO'
+
ib) - R-effect of -CH3 groups
(JEE Main 2022 (c) hyperconjugation
53. Consider the following compounds : id) - I-effect of CH3 groups
our

(NEET 2020)
ad

CH3 Anhyd.AICIj
57. + CH3CH2CH2CI
CH3-C-CH
A
Y

(major product)
CH3 (I) (II) (III)
Re

The stable carbocation formed in the above


nd

Hyperconjugation occurs in reaction is :

id) III only ib) I and II +


Fi

(c) 1 only id) II only


ia) CH3CH2CH2 ib) CH3CH+
(AIPMT 2015) ■^CHCH2CH3
54. The hyperconjugative stabilities of rer/-butyl +
cation and 2-butene, respectively, are due to id) I
(c) CH3-CH-CH3
ia) G-p (empty) and c-k* electron delocalisations
ib) G-G* and G-n electron delocalisations (JEE Main 2022)

ANSWERS

50. (u) 51. id) 52. id) 53. (o) 54. (a) 55. ib) 56. (c) 57. (c)
ORGANIC CHEMISTRY-SOME BASIC PRINCIPLES AND TECHNIQUES 12/201

58. Which of the following carbocation would have (c) HC = C- id) CH3CH-
the greatest stability ?
ie) EtO- (Kerala PET 2019)
(a) CH3-S-CH+ (b) (CH3)2NCH^ 63. The increasing order of basicity for the following
intermediates is (from weak to strong)
(c) CH3-O-CHJ (d) F-CHt
(AlIMS 2013) CH3
0 0
59. The correct stability order for the following species
IS
H3C—C H2C = CH—CH2 HC(in)
= C® CH® CN®
(V)
(«) (iv)
CH3
(/)

w
o (a) (v) < (0 < (iv) < (//) < iiii)
(I) (H) (111) (IV)
(b) (Hi) < (iv) < iii) < (i) < (v)
(a) (II)XIV)>(1)XIII) (c) (v) < {/(/) < iii) < iiv) < (i)

F lo
(b) (I)> (II) XIII) XIV) id) (iii) < (0 < (ii) < (iv) < (v) (JEE Main 2020)
(c) (I1)>(I)>(IV)>(III) 64. The compound having longest C-Cl bond is
id) (I) > (III) > (II) > (IV) (IIT Paper II, 2008)

ee
Cl
60. In the following carbocation, H/CH3 that is

Fr
most likely to migrate to the positively charged Cl
carbon is
(a) (h)
H H

for
1 21 + I4 5 Cl
H,C—C—C—C—CH,
3 I 3| I 3
ur
HO H CH3
(c) id) CH2=CH—Cl
(a) CH3 at C-4
s
(b) Hat C-4
ook
(c) CH3atC-2 (i/) HatC-2
Yo
(IIT Paper II, 2009) NO2
61. The order of the base strength of the compounds
eB

(Karnataka CET 2019)


O 65. Which of the following has the shortest C—Cl
bond ?

(a) Cl—CH = CH—NO2


our

O" 'CHJ
ad

(b) Cl—CH = CH2


(0 00
(c) Cl—CH = CH^H3
O" (rf) Cl—CH = CH—OCH3 (JEE Main 2020)
Y

/iv
66. (C7H502)2 ^ X
^ 2C^H^ + 2C02
Re

NH7
nd

IS
Consider the above reaction and identify the
(Hi) intermediate ‘X’.
(/V)
Fi

o O
(a) (iv) > (ii) > (i) > (iii)
(b) (Hi) > (ii) > (iV) > (i) (a) C^Hj-C* (b) C^H^-C-O"
(c) (ii) > (iii) > (iv) > (i) O O
(d) (ii) > (iii) > (i) > (iv) (AMU Engg. 2010)
62. The strongest base among the following is (c) C^H5-C-0 (d) C^Hg-C-O*
(a) NH- (b) OH- (JEE Main 2022)

ANSWERS

58. (b) 59. (d) 60. (d) 61. (c) 62. (d) 63. (c) 64. (h) 65. (a) 66. (d)
12/202 ‘P’u^eUe^ '4. New Course Chemistry (XI)

67. The order of decreasing ease of abstraction of IV. Methods of Purification


hydrogen atoms in the following molecule is
of Organic Compounds
Me
71. Phenol on reaction with dilute nitric acid gives
H/, two products. Which method will be most efficient
for large scale separation ?
(a) Chromatic separation
(/?) Fractional distillations
(c) Steam distillation
(a) ih) (d) Sublimation (JEE Main 2022)
(c) (d) 72. Which technique among the following is most
(West Bengal JEE 2014) appropriate in separation of a mixture of 100 mg
of p-nitrophenol and picric acid ?
68. The reaction of methyl trichloroacetate
(a) Steam distillation

F low
(CC^COoMe) with sodium meihoxide (NaOMe)
generates (b) 2-5 ft long column of silico gel
(a) carbocation (h) carbene (c) Sublimation

(c) carbanion (d) carbon radical (d) Preparative TLC (This Layer chromatography)
(JEE Main 2022)
(West Bengal JEE 2015)
73. Impure glycerine can be purified by or glycerol
69. For the following reactions :
can be separated from spent lye in soap industries
(A) CH3CH2CH2Br + KOH » CH3CH = CH2 by

e
for Fre
+ KBr + H2O (a) fractional distillation
H3C CH3 H3C CH3 (b) differential extraction
(c) steam distillation
(B) + KOH + KBr

Br OH
(d) distillation under reduced pressure
(JEE Main 2020)
Br
74. A liquid compound can be purified by steam
Your
eBo ks

(C) + Br2 distillation only, if it is


Br (a) steam volatile, immiscible with water
(b) not steam volatile, miscible with water
Which of the following statement is correct ?
(c) steam volatile, miscible with water
(a) (A) is elimination, (B) and (C) are substitution
ad

reactions (d) not steam volatile, immisible with water


our

(NEET 2020)
(b) A is substitution, (B) and (C) are addition
reactions 75. In chromatography technique, the purification of
(c) (A) and (B) are elimination reaction reactions compound is independent of
(a) mobility of flow of solvent system
Re

and (C) is an addition reaction


(d) (A) is elimination, (B) is substitution and (C) (b) mobility of the compound
is addition reaction (NEET Phase-I 2016) (c) physical state of the pure compound
Find Y

70. An alkyne combines with a conjugated diene to (d) length of the column or TLC plate
give an unconjugaled cycloalkadiene. The most (JEE Main 2021)
likely title of this reaction is 76. Paper chromatography is an example of
(a) Schotten Baumann reaction (a) partition chromatography
(h) Hofmann bromamide reaction (h) thin layer chromatography
(c) Pinacol-pinacolone rearrangement (c) column chromatography
(d) Diels-Alder reaction (Odisha JEE 2008) (d) adsorption chromatography (NEET 2020)

-.1

67. ●' 69. (di '/I 71. f 72. 13.(d) 74. (a) 75.(r) 16.(a)
ORGANIC CHEMISTRY-SOME B/ .k PRINCIPLES AND TECHNIQUES 12/203

77. A chromatography column, packed with silica gel 83. Compound added for detection of halogens before
as stationary phase, was used to separate a mixture adding AgN03 is
of compounds consisting of (A) benzanilidc. (B)
(a) HNO3 (b) H2SO4
aniline and (C) acetophenone. When the column
(C-) HCl id) cant be determined
is eluted with a mixture of solvents, hexane : ethyl
acetate (20 : 80), the sequence of obtained {JEE Main 2021)
compounds is 84. During halogen test, sodium fusion extract a boiled
(a) (B), (A) and (C) ib) (A), (B) and (C) with concentrated HNO3 to
(c) (B), (C) and (A) (d) (C), (A) and (B) («) removed unreached sodium
(JEE Main 2020) (h) decompose cyanide or sulphide of sodium
78. The distillation technique most suited for (c) extract halogen from organic compound
separating glycerol from spent-lye in the soap
industry is ({{) maintain the pH of extract (JEE Main 2022)
(a) simple distillation {b) fractional distillation 85. The formula of purple coloured formed in
(c) steam distillation Lassaigne’s lest for sulphur using sodium
nitroprus.side is
(d) distillation under reduced pressure
(a) NaFe [Fe(CN)g]

F low
(JEE Main 2016)
79. The most suitable method of separation of I ; 1 (b) Na[Cr(NH3)2(CNS)4l
mixture of ortho- and para-nitrophenols is (c) Na2[Fe(CN)5NO]
id) chromatography ib) crystallisation id) Na4[Fe(CN)gNOS] (JEE Main 2022)
id) steam distillation id) sublimation 86. An organic compound X having molecular mass
(NEET 2017) 60 is found to contain C = 20%, H = 6-67% and
80. The elution sequence of a mixture of compounds N = 46-67%, while rest is oxygen. On heating, it
containing chlorobenzene, anthracene and gives ammonia along with a solid residue. The
/7-cresol developed on an alumina column using
a solvent system of progressively increasing
polarity is
forFe
solid residue gave violet colour with alkaline
copper sulphate solution. The compound X is
(a) CH3CH2CONH2 ib) CH3NCO
(a) anthracene —> chlorobenzene p-cresol
ib) anthracene -> p-cresol chlorobenzene
ic) CH3CONH2 id) (NH2)2C0
(AHMS 2015)
(c) chlorobenzene —> p-cresol —> anthracene
id) chlorobenzene -> anthracene p-cresol 87. Which of the following compounds will be
eBoks

(e) p-cresol anthracene —> chlorobenzene suitable for Kjeldahl’s method for nitrogen
Your

estimation ?
(Kerala PET 2018)
ad

NH2
V. Qualitative and Quantitative Analysis
{a) {b)
our

81. During the fusion of organic compound with N


sodium metal, nitrogen present in the organic
compound is converted into
NO2 ●Njcr
Re

ic) id)
id) NaN02 ib) NaNH2
N'
(c) NaCN (d) NaNC
(Karnataka CET 2018) (JEE Main 2018)
82. Lassaigne’s test (with silver nitrate) is commonly 88. The Kjeldahl method of nitrogen estimation fails
FindY

used to detect halogens such as chlorine, bromine for which of the following reaction products ?
and iodine but not useful to detect fluorine because
NO,
the product AgF formed is
(a) volatile ib) reactive
Sn/UCi
(c) explosive id) soluble in water (I)

ie) a liquid (Kerala PET 2017)


cEi: 3:13

77. id) 78, (^/) 79. (r) 80. (o) 81. u- 82. (J) 83.(0) 84. (
86. id) 87. ih)
12/204 ‘Ptadee^'^1. K ● Course Chemistry (XI)S!EIHI1

CN (a) C^Hg (b) C2H4


(C) C3H4 (d)
L1AIH4 (JEE Main 2013)
(II)
93. Compound ‘A’ contains 8-7% hydrogen, 74%
carbon and 17-3% nitrogen. The molecular
CH2CN formula of the compound is (Molecular mass of
the compound is 162 mol'^)
(0 SnClj + HCl
(III) (a) C4H6N2 (b) C2H3N
(/OH2O (C) C3H7N id) C,oH,4N2
(JEE Main 2022)
NH2

w
HNO2 94. In carius method of estimation of halogen, 0-45 g
(IV)
HCl of an organic compound gave 0-36 g of AgBr. Find
out the percentage of bromine in the compound,

F lo
(a) (II) and (IV) (b) (ni)and(IV) (a) 34-04% (b) 40-04%
(c) a), (in) and (IV) (d) (H) and (HI) (c) 36-03% (d) 38-04%
(JEE Main 2020) (JEE Main 2022)
89. 29-5 mg of an organic compound containing 95. Two isomers (A) and (B) with molar mass

ee
nitrogen was digested according to Kjeldahl’s 184 g/mol and elemental composition C, 52-2%,

Fr
method and the evolved ammonia was absorbed
11,4-9% and Br 42-9% gave benzoic acid and
in 20 mL of 0-1 M HCl solution. The excess of />-bromobenzoic acid, respectively on oxidation
the acid required 15 mL of 0-1 M NaOH solution with KMn04. Isomer ‘A’ is optically active and

for
for complete neutralization. The percentage of gave a pale yellow predpitals when warmed with
nitrogen in the compound is alcoholic AgN03. Isomers ‘A’ and ‘B’ are
ur
(a) 29-5 (b) 59-0 respectively.
(c) 47-4 id) 23-7
s
90. A sample of 0-5 g of an organic compound was
ook
treated according to Kjeldahl’s method. The
Yo
ammonia evolved was absorbed in 50 mL of (n) H3C—CHBr—C^Hg and
0-5 M H2SO4. The remaining acid after
eB

neutralisation by ammonia consumed 80 mL of CH^CHj


0-5 M NaOH. The percentage of nitrogen in the
CH3
organic compound is
(b) and
our

(a) 14 (b) 28
ad

(c)42 id) 56
ie) 70 (Kerala PMT 2012) Br
91. For the estimation of nitrogen, 1-4 g of an organic
CHoCHg
Y

compound was digested by Kjeldahl method and


the evolved ammonia was absorbed in 60 mL of
Re

M/10 sulphuric acid. The unreacted acid required (c) H3C—CHBr—C^Hg and
nd

20 mL of M/10 sodium hydroxide for complete


neutralization. The percentage of nitrogen in the
Fi

Br
compound is
(a) 5% (b) 6% CH2CH3
(c) 10% (if) 3%
(JEE Main 2014)
id) and H3C—CHBr—CgHg
92. A gaseous hydrocarbon gives upon combustion
0-72 g of water and 3-08 g of CO-,. The empirical
formula of the hydrocarbon is Br

ANSWERS

88. ih) 89. (i/) 90. (/?) 91. (c) 92. ia) 93. id) 94. (a) 95. (c)
ORGANIC CHEMISTRY-SOME BASIC PRINCIPLES AND TECHNIQUES 12/205

in Multiple Choice Questions (with One or More than One Correct Answers)

96. Which of the following compounds contains only


NH3 NH3
5/7^-hybridized carbons ?
(fl) Cycloalkanes *

(b) Straight chain alkanes


(c) Branched chain alkanes I II

(d) Benzene
(a) II is not an acceptable canonical structure
97. The lUPAC name(s) of the following compound because carbonium ions are less stable than
is(are) ammunium ions

ib) II is not an acceptable canonical structure

F low
H3C Cl
because it is non aromatic

(a) l-chloro-4-methylbenzene (c) II is not a acceptable canonical structure


{b) 4-chloroloIuene because nitrogen has 10 valence electrons
(c) l-methyl-4-chlorobenzene id) U is an acceptable canonical structure
(d) 4-methylchlorobenzene 102. Which of the following statements are correct ?
(JEE Advanced 2017) + +
■C = 0; and R C s O: are resonance

re
98. Tautomerism is exhibited by (a) R

for F
structures

(a) (Me3CCO)3CH ib) O


(b) Methyl carbanion is both isostructural and
isoelectronic with ammonia
O
4 3 2 1

(c) In CH^ = CH ■CH = O , nucleophilic centre


Your
(c) O 0 id) O
is position 4 while electrophilic centre is
s
eBook

\ position 1.
0
(d) SnCl4 acts as an electrophile
(West Bengal JEE 2013)
103. Which of the following species is/are planar ?
99. Which of the following behave both as a
(a) /err-Butyl free radical
ad

nucleophile as well as an electrophile ?


our

(a) CH3OH (b) ier/-Butyl carbocation


(b) CH3CI (c) rerr-Butyl carbanion

(c) CH3CN (d) Ally! carbanion.


104. Sodium fusion extract, obtained from aniline,
Re

(d) HCHO on

100. Resonance structures of a molecule should have treatment with iron (II) sulphate and H2SO4 in
Y

(a) identical arrangement of atoms


presence of air gives a prussian blue precipitate.
Find

Hence, the blue colour is due to the formation of


(b) nearly the same energy content
(a) Fe4[Fe(CN)6l3
(c) same number of paired electrons
id) identical bonding ib) Fe3[Fe(CN>6]2
101. Examine the following two structures for the (c) Fe4[Fe(CN)6]2
anilinium ion and choose correct statement from (d) Fe3[Fe(CN)6l3 (UT Paper n, 2007)
the ones given below :

ANSWERS
96. {a,h,c) 97. ia.b) 98. (a.b,d) 99. (c.d) 100. {a,h,c) 101. (fl.c)
102. ia.b.c.d) 103. {a.h.d) 104.ia)

1
12/206 New Course Chemistry (XI)EEIHD

mi Multiple Chi^ice Questions (Based on the given Passage/Comprehen sion)

Each comprehension given below is followed by some multiple choice questions. Each question has one
correct option. Choose the correct option.

[gomprcl?ensi^ f During the detection of The neutral species (free radicals, carbenes,
elements by Lassaigne’s test, the covalent nitrenes, etc.) and positively charged species
compounds are converted into ionic being electron deficient are collectively
compounds by fusion with metallic sodium. called electrophiles while neutral and
The nitrogen, sulphur and halogens present negatively charged species which are electron
in the organic compound are converted into rich are called nucleophiles. An organic

w
cyanides, sulphides and halides respectively reaction usually involves the attack of a
which are then detected by their usual tests. reagent (radicals, positively and negatively
charged species) on the substrate molecule).

F lo
105. An organic compound containing N, S and 0 as The substrate molecule, although as a whole
extra elements is fused with sodium metal and then electrically neutral, has centres of low and
extracted with water. The species which is not high electron density due to displacement of
present in the solution of extract is bonding electrons. These electron
displacements occur through inductive,

ee
{a) CN- (b) CNS"
eicctromeric, resonance and hyperconjuga

Fr
tion effects. Whereas inductive effects involve
(c) NOJ (d) s~-
displacement of a-electrons, resonance effects
106. Which of the following compounds will answer involve transfer of n- and n-electrons along a

for
Lassaigne’s test for nitrogen ? conjugated system. Hyperconjugationeffects,
on the other hand, involve a-7C-conjugation.
(a) NH2NH2 (b) NaCN
ur
Both inductive and hyperconjugation effects
(c) NaN03 {d) NH4CI can be used to explain the stability of
carbocations and free radicals which follow
s
107. Which of the following will give blood red colour
ook
the stability order : 3® > 2® > 1®. The stability
Yo
while doing Lassaigne’s test for nitrogen ?
of carhanions, however, follows opposite

<0>
order.
eB

(a) II2H SO3H


The stability of a molecule can be judged on
the basis of contribution of its resonance
(b) (NH2)2C = 0 structures. Resonance structures have same
(c) CfiHgSO^H position of nuclei and have same number of
our
ad

unpaired electrons. Among resonance


(d) (NH4)2S04 structures, the one which has greater number
108. Sodium niiroprusside reacts with sulphide ion to of covalent bonds, has less separation of
give a purple colour due to the formation of opposite charges, a negative charge on more
dY

electronegative and a positive charge on a


(a) IFe(CN)5NO]-^ (b) [Fe(NO)5CNJ-^
Re

more electropositive atom are more stable


(c) [Fe(CN)5NOS]4- (d) [Fe(CN)5NOS]'-'-. than others.
Fin

(Odisha .lEE 2007)


109. Which of the following series contains only
IKOmprehensiornF An organic reaction electrophiles ?
occurs through making and breaking of (a) H2O, SO3, H3O+ (b) NH3. H2O, AICI3
bonds. The breaking of bonds may occur + +
either homolytically leading to the formation (c) AICI3, SO3, NO2 (d) H2O. Cl , NH3
of radicals or heterolytically generating 110. Out of the following, the one containing only
positively and negatively charged species. nucleophiles is

105. (c) 106. {h) 107. («) 108. (c) 109. (c)
ORGANIC CHEMISTRY-SOME BASIC PRINCIPLES AND TECHNIQUES 12/207

(fl) NH3, CN", CH3OH 114. The most stable carbanion among the following
IS
ib) AICI3, BF3. NH3
CH2—CHJ CHJ
(c) AICI3, NH2 , H2O
id) RNH2,: CX2, H-
111. The C—C bond length in propene is little shorter
(a) o (i) [o
(149 A) than the C—C bond length (1-54 A) in CHJ CHJ
ethane. This is due to

(a) +I-effect of CH3 group


(b) mesomeric effect (c) [Q (</) [Q
(c) clectromeric effect
(d) hyperconjugation effect. OCH3 NO^

112. The most stable free radical among the 115. Which of the following is the most stable cation ?

w
following is (i/) F3C—CHJ (b) (CH3)2CH
+

F lo
(a) CgH5CH2CH2 (b) CgHgCHCHj (c) CHJ (d) CFJ-
(c) CH3CH2 {d) CH3CHCH3 116. For 1-methoxy-l, 3-butadiene, which of the
113. What is the decreasing order of stability of the following resonating stioicture is the least stable ?
+
ions ?
(a) H2C—CH—CH = CH—O—CH3

ree
+

I. CH3—CH—CH3 II. CH3—CH—OCH3 (b) HjC—CH =CH—CH = 6—CHg

for F
III. CH3—CH—COCH3 (c) H2C = CH—CH—CH—O—CH3
(a) I > II > III ib) II > III > I
(c) III > I > II (f/) II > I > HI id) H2C = CH—C H—CH = O—CH3

m
Your
Matching Type Questions
ks
eBoo

Match the entries of column I with appropriate entries of column II and choose the correct option out
of the four options (a), (b), (c), (d) given at the end of each question.
117. Column 1 ● alunt
ad
our

(P) Spiro compound

(9) Aromatic compound


Re
Y

(r) Non-planar heterocyclic compound


Find

is) Bicyclo compound

r (b) As, B-r, C-b, D-q (c) A-r, Bs, C-p, D-q (d) As, B-r, C-q, D-p

(JEE Main 2022)

no. (a) 111. id) 112. ib) 113. {d) 114. (d) 115. (h) 116. (r) 117. (c)

I
12/208 New Course Chemistry (XI)ES19D

118. Column I Column II

(A) Nitrogen of the organic compound is (P) Fe(CNS)3


converted into (NH4)2S04.
(B) The compoundresponsible for blue or green (?) Dumas method

colouration during Lassaigne’s test for nitrogen.


(C) Compound sometimes formed in (r) Fe4[Fe(CN)^]3
Lassaigne’s test for N if S is also present.
(D) Compound responsible for violet (●s) Kjeldahl’s method
colouration in Lassaigne’s test for S.

w
(E) Nitrogen of the organic compound is (0 Na4[Fe(CN)5NOSJ
set free as N2 gas.
(tj) A-r ; B-s'; C~t; D-p ; E-^ (b) As ; B-r ; C-p ; D-r; E- q

F lo
(c) A-p ; B-r; C-q ; D-r ; E- s (d) A-q ; B-5 ; C-r ; D-/>; E-1

119. Column I Column II

ee
A
(A) NH4CNO — ^ NH2CONH2 (P) Condensation reaction

Fr
(B) HO- + CHCI3 —>: CCI2 + H2 o + cr Isomerization reaction

(C) R—X + Y-— R—Y + X- (r) Rearrangement reaction

for
(D) CgHgCH = O + H2NC6H5 ^ C6H5CH = NCgHg + HoO is) Elimination reaction
our
hv
(E) c«-2-bulene f i rron^-2-butene (0 Substitution reaction
oks
(fl) A-r; B-p ; C-r ; D-5 ; E- q (b) A-p ; B-r; C-q ; D-s ; E-1
(c) A-r ; B-5 ; C-r; D-p ; E- q (d) A-q ; B-r; Cs ; D-p ; E- r
o
Y
eB

120. Column I Column II

(A) CH4 + CI2 » CH3CI + HCl (P) Electrophilic addition


our

H2SO4
ad

(B) C6H6 + HNO3 > C6H5NO2 + H2O (?) Nucleophilic substitution


(C) CH3CH2 = CH2 + HBr > CH3CHBtCH3 ir) Free radical substitution

OH
Y

(D) (CH3>2C0 + HCN ^ (CH3)2C (●s) Electrophilic substitution


CN
Re
nd

HBr/Peroxide
(E) CH3CH = CH2 > CH3CH2CH2Br (0 Nucleophilic addition
Fi

(F) R—X + OH-- ●> ROH + X" iu) Free radical addition

(a) A-q ; B-m ; C-r; D-p ; E-r ; F-j (b) A-p ; B-r; C-r; D-^ ; E-1<; F-^
(c) A-u ; B-^ ; C-p; D-r; E- 5 ; F-r (d) A-r; B-5 ; C-p ; D-r; E- « ; F-^

ANSWERS

118.(h) 119.(c) 120. (d)


ORGANIC CHEMISTRY-SOME BASIC PRINCIPLES AND TECHNIQUES 12/209

m Matrix-Match Type Questions


p q r s

iOli® !OI©
Match the entries of column I with appropriate entries of column II. Each
entry in column I may have one or more than one correct option from column
B
®11® ©I ©

ow
II. If the correct matches are A-p, s ; B-r ; C-p, q ; D-s, then the correctly
bubbled 4x4 matrix should be as follows :
C |©| ® 0| ©
D
I® !
11 i 1

121. Column 1 Column II 122. i-^oluni;; 1 Column II


+

e
(A) CH3C = N (P) Resonance (A) (CH3)3C1 -Cl"
> (CH3)3C ip) Electrophile

re
hv
(B) CH2 = C = CH2 (?) Planar
(B) CH3—CH3 2CH3 {q) Heterolytic
fission

Frl
F
(C) (r) Inductive effect hv
(C) CH2N2 > : CHj (r) Nucleophile
-N2
(D) (CH3)3C (s) Non-planar nh;
ou (D) HC s CH > C = CH (s) Homolytic

r
-NH3 fission

so
VI. Integer Type Questions A B C D

kf
©® ® ®
DIRECTIONS. The answer to each of the following questions is a single digit
oo
integer, ranging from 0 to 9. If the correct answers to the question numbers ®®®®
A, B, C and D (say) are 4, 0, 9 and 2 respectively, then the correct darkening
®®®®
Y
of bubbles should be as shown on the side :
B

123. How many of the following are heterocyclic aromatic compounds ?


®®®®
benzene, cyclopentadiene, oxirane, tetrahydrofuran, naphthalene, cyclopentane, @@00
re

tropolone, azulene, diphenyl.


0 @ 00
oY
u

124. Total number of alkyl groups having the molecular formula C4H9 is
125. How many of the following cannot show tautomerism ? 0000
ad

acetophenone, acetaldehyde, cyclohexanone, acetylacetone, benzoquinone, acetone, ©000


benzaldehyde, butanone, ethyl acetoacetate.
d

8) (8) (8 ®
126. The total number of cyclic isomers possible for the hydrocarbon with molecular
,{9 > X9) 19) 1(9
in

formula C4H^ is sWt


Re

127. Amongst the following the total number of electrophiles is :


CN-, H+, NO+ , Ci-^, OH-, H3O+.: CH2 , : NH3, Na+ H3C„^CH2CH3
F

128. Amongst the following the total number of nucleophiles is :


SO3, NH2 , R-, H2O, OR-, ROH, AICI3, H-, BF3.
129. The total number of contributing structures showing hyperconjugation (involving C—H bonds) for the
carbocation given on R.H.S. is (IIT Paper H, 2011)
130. The number of 5/7^-hybridized carbons in an acyclic neutral compound with molecular formula C4H5N is
(JEE Main 2022)

121. (A-p, r, s ; B-.v; C-p, q ; D-r, s) 122. (A-p, q ; U-p, s ; C-p. q ; D-(/, r)
123. (0) 124. (4) 125. (2) 126. (5) 127. (3) 128. (6) 129. (6) 130. (1)

A
12/210 New Course Chemistay (Xl)CZsISD

131. The total number of monobromo derivatives formed by alkanes with molecular formula C5H|2 (excluding
stereoisomers) is

C1I3

132. Number of electrophilic centres in the given compound is (JEE Main 2022)
0

CH-,CN

VII. Numerical Value Type Cvt.estion (In Decimal Notation)

w
For the following question, enter the correct numerical value, (in decimal-notation, truncated/rounded-off
to the second decimal place, e.g., 6*25, 7’00, - 0*33, - *30, 30*27, - 127*30) using the mouse and the on

F lo
screen virtual numeric keypad in the place designated to enter the answer.
133. A mixture of 2-3 g formic acid and 4-5 g oxalic acid is treated with cone. HoS04. The evolved gaseous
mixture is passed through KOH pellets. Weight (in g) of the remaining product at STP will be

ee
(NEET 2018)
134. Number of grams of bromine that will completely react with 5-0 g of pent-l-ene is (JEE Main 2022)

Fr
VE
Assertion-Reason !ype Questions

for
TYPE I
ur
DIRECTIONS. The questions given below contain Statement-1 (Assertion) and Statement-2 (Reason).
Each question has four choices (a), (b), (c), (d) out of which ONLY ONE is correct. Choose the correct
oks
option as under,
Yo
(a) Statement-1 is True, .Statement-2 is True ; Statement-2 is a correct explanation for Statement-1,
o

(b) Statement-1 is True, Statement-2 is True ; Statement-2 is not a correct explanation for Statement-1,
eB

(c) Statement-1 is True, Statement-2 is False,


(d) Statement-1 is False, Statement-2 is True.

135. Statement-1. Simple carbanions are usually pyramidal but allyl carbanion is a planar species.
our

Statement-2. All the carbon atoms in allyl carbanion are jp-'hybridized.


ad

136. Statement-1. /ert-Butyl carbanion is less stable than methyl carbanion.


Statement-2. The +l-effect of the CH3 groups tends to stabilize the rerz-butyl carbanion.
137. Statement-1. Nitrenes cannot be isolated.
Y

Statemeiit-2. Nitrenes are the nitrogen analogues of carbene.


Re
nd

138. Statement-1. Beilstein test can be used to detect fluorine in the organic compound.
Statement-2. CuF is not volatile and hence does not impart any colour to the flame.
Fi

139. Statement' !. Lassaigne’s extract is boiled with dil. HNO3 before testing for halogens by AgN03.
Statement-2.CN“ and S~" ions present in the extract interfere with the test of halide ions by AgN03.
140. Statement-1. Thin layer chromatography is an adsorption chromatography.
Statement-2. A thin layer of silica gel is spread over a glass plate of suitable size in thin layer chromatography
which acts as an adsorbent. (JEE Main 2022)
141. Statement-1. In Lassaigne’s test when both nitrogen and sulphur are present in an organic compound, sodium
thiocyanate is formed.
ANSWERS

131. (8) 132, (3) 133. (2-8) 134. (11.43) 135. {b) 136. (c) 137. (b) 138. (cl)
139. (u) 140. («)
ORGANIC CHEMISTRY-SOME BASIC PRINCIPLES AND TECHNIQUES 12/211

Statement-2. If both nitrogen and sulphur are present in an organic compound, the excess of sodium used in
sodium fusion will decompose the sodium thiocyanate present to give NaCN and Na2S. (JEE Main 2022)
142. Statement-1. A mixture contains benzoic acid and naphthalene. The pure benzoic acid can be separated out
by the use of benzene.
Statement-2. Benzoic acid is soluble in hot water. (JEE Main 2022)

TYPE II

DIRECTIONS. In each of the following questions, a statement of Assertion (A) is given followed by a
corresponding statement of Reason (R) just below it. Of the statements, mark the correct answer as
(a) If both assertion and reason are true, and reason is the true explanation of the assertion,
(b) If both assertion and reason are true, but rea.son is not the true explanation of the assertion,

w
(c) If assertion is true, but reason is false,
(d) If both assertion and reason are false.

F lo
143. Assertion. Bul-l-ene and 2-methylprop-l-ene are position isomers.
Reason. Position isomers have same molecular formula but different arrangement of carbon atoms.

144. Assertion. In CH2 = C = CH2, all the carbon atoms are -hybridized.

ee
Reason. All the hydrogen atoms lie in one plane.

Fr
145. Assertion. A free radical is paramagnetic species.
Reason. A free radical is formed in homolytic fission of covalent bond. (AIIMS 2013)
146. Assertion. Alkyl carbanions like ammonia have pyramidal shape.

for
Reason. The carbon atom carrying negative charge has an octet of electrons.
ur
147. Assertion. /err-Butyl carbanion is more stable than methyl carbtinion.
Reason. The +I-effect of the three methyl groups in tert- butyl carbanion tends to make it more stable than
methyl carbanion.
oks
148. Assertion. A mixture of o-nitrophenol and p- nitrophenol can be separated by steam distillation.
Yo
Reason. p-Nitrophenol is steam volatile while o-nitrophenol is not steam volatile. (AIIMS 2015)
o
eB

149. Assertion. Es.sential oils are purified by steam distillation.


Reason. Essential oils are volatile and are insoluble in water.

150. Assertion. Oils are purified by steam distillation.


our

Reason. The compounds which decompose at their boiling points can be purified by steam distillation.
ad

(AIIMS 2008)
151. Assertion. During test for nitrogen with Lassaigne’s extract on adding FeCl3 solution, sometimes a red
precipitate is obtained.
Y

Reason. Sulphur is also present. (AIIMS 2007)


Re
nd

152. Assertion. NH2 —C—NH2 gives red colour in Lassaigne’s test.


Fi

Reason. Compounds having N along with C give red colour in Lassaigne’s test. (AIIMS 2009)
153. Assertion. Hydrazine contains nitrogen but does not give Lassaigne’s test for nitrogen.
Reason. Hydrazine reacts with fused sodium to give H2 gas.
154. Assertion. Vapour density is the same thing as density.
Reason. Molecular weight is twice density.

141. («) 142. {d) 143. id) 144. (t/) 145. (h) 146. (h) 147. {d) 148. (r) 149. («)
150. id) ISI. (a) 152. tr) 153. {b) 154. (d)
12/212 T^eteCee^ 4. New Course Chemistry (XI) orsiMn

For Difficult Questions

Multiple Choice Questions (with one correct Answer)

■> 2 3 H H
sp~ sp sp^ sp^
0I 0
I. CH2 =C=CH—CH3

w
a
9. H 0

o
ct| 0^C<^
jp2 H o H H
0
ir
n
O—H 0 0
2. .V//2 —C—0CH3

F lo
K ml 0—C—C—H
.sp2 sp2 0I 0

H H
(f) No. of siP' carbons = 6 + 1 = 7 No. of a-bonds = 18 ; No. of K-bonds = 2

ee
(Benzene) (C=0)
10. Aniline, i.e., option {d) is correct since it contains

Fr
(ii) No. of 7i-bonds = 3 + 2 + I =6 a benzene ring.
(Benzene) (CsC) (C=0) 11. Tropolone is an example of non-benzenoid
aromatic compound (Refer to page 12/12).
Thus, option (b) is correct.

for
sp^
sp
ur 1

.1. CH2=C=CH2. 12. 4 2


4-Elhyl-3-methylhex-2-enc
5\
Refer to Q. 1, Page 12/149. 6
ks
2 4 6
Yo
sp
4. Four, i.e., CH., = C = CH -CH-C = CH ■7
oo
13. I 3 5
^ I
CsN 3-EthynyM, 6-heptadiene
eB

sp
14. Option (c) represents ^-butylphenylvinylmethane.
V.2 sp^ sp sp (Ph) Phenyl
3. CH2 =CH-CsCH
r

i
Vinyl
ou
ad

I:
6. Biphenyl, i.e., option (a) is correct since all the
carbon atoms are 4^^-hybridized.
T Butyl
Y

H
4
0
H3C\32/H
7. H—C—C=C—C#C—H 15.
nd
Re

.0 . (j .0 ^ n l-Bromobut-2-ene
H
oj o\ p /a
H H H H Br
Fi

3 2 1
CTc-H ~ 6, Cq^ = 4,7t = 3 Total ct = 10, tc = 3 16. PhCH2CH2COOH
3-Phenylpropanoic acid
H H
II3C, H \ 17. 3-Cyanopentane-l, 5-dinitrile (Refer to page 12/43).
\ C—C
8.
r CH3 18. - SO3H > - COCi > - CONH2 > CN (Refer to
r
n

II3C C—C page 12/42).


■■// \
H CH3
CH2
No. of 7C-bonds = 4. 19. OjN
No. of n-electrons = 4x2 = 8
4-Melhyl-6-nitro-3-oxohept-4-enai
ORGANIC CHEMISTRY-SOME BASIC PRINCIPLES AND TECHNIQUES 12/213

OH
For Difficult Questions
2

20. Number the carbon atoms of the chain from right CO2H
I
since its lowest locant is 3 for the functional group
OH. 6^5
3-Ethynyl-2- hydroxy-
4-methylhex-3-en-5-ynoic acid

Thus, option {d) is correct.


Br

w
The correct lUPAC name is : l-bromo-5-chloro-
4-methylhexan-3-ol.
Thus, option (d) is correct.

F lo
4 2
5
1 Here, —COOH is the functional group and lowest
3
21. 6 10 2,6-Dimethyldec-4-ene locants are : 1,2 and 4.
8

ee
7 9 Thus, the correct name is ; 4-bromo-2-mcthyl-
Thus, option (a) is correct. cyclopentanecarboxylic acid, i.e., option {d) is

Fr
correct.
22. —COOH is the principal functional group while
—CHO is the substituent functional group. .OH

for
Therefore, the numbering of the carbon atoms of
the carbon chain begins with carbon atom of the
our
27.
—COOH group as shown below :
^CHO OjN 1
s
CHO
ook
CH3
Here, —CHO is the principal functional group and
H3C 4
hence carbon atom bearing this group is numbered
Y
eB

1 COOH as 1.
2, 5-Dimelhyl-6-oxohcx-3*enoic acid Following lowest locant rule, the correct locane.
Thus, option (c) is correct. are : 2, 4 and 5 and not 3, 4, 6.
The correct lUPAC name in alphabetical orticr i
r

O
ad
ou

O
5-Amino-4-hydroxymethyl-2-nitrobenzaldchydc,
C
NO2
23. H
F. 2.
Y

5^6 28. 1 -Fluoro-4-methyl-2-nilro-


3-Keto-2-methylhex-4-enal Q benzene {lowest set of liKaiil
Re
nd

CH3 rule is followed).


Aldehyde group gets preference over the keto
group.
Fi

2^
OH

Q
29. 1 1-Isopropyl-
2, 4-dimelhylbenzcne.
24.
^ XOOH 3-Ethyl-2-hydroxy-
i
4-methyl hex-3-en-5- 30. A stable compound has a whole number value of
5 ynoic acid DBE. For example,

25. Here, CO2H is the principal functional group while DBE of CgHj^O
OH is the substituent functional group. Therefore, 8{4-2)-H2(l-2) + l(2-2)
+ 1
numbering of carbon atoms of the carbon chain 2

begins with the carbon atom of the —CO2H group =2+1=3


as shown below :
Thus, option (b) is correct.
12/214 New Course Chemistry (XI)EEIHD

For Difficult Questions

All other formulae have fractional values of DBE, 111.


CH3
i.e., C7H9O, CgHjjO and CjQHjyO-j are 3-5. 3-5 CH3
and 2*5 respectively. OH
31. Propan-l-oland propan-2-ol are position isomers Here, v-H migrates
while all other options are wrong. Thus. I, II and III all show tautomerism, i.e., option
33. 1, 3-Diketone (CH3COCH2COCH3) has higher (b) is correct.

low
(76%) enol content than ketoester 37. Refer to Some Additional Useful Information

(CH3COCH2COOC2H5) when has only 7%. Refer about Organic Chemistry, pages 12/126-12/127.
to page 12/63. 38. The compound listed under option (d) does not
contain an a-hydrogen on a saturated carbon next
34. C4H,oO= CH3CH2-O-CH2CH3 to the keto group and hence cannot show
(I) tautomerism.

39. For tautomerism to occur, at least one a-hydrogen

ee
CH3 should be present on a saturated carbon next to the

rF keto group. Since in option (a), a-hydrogens are

Fr
CH3-O-CH-CH3 CH3-O-CH2CH2CH3 not present on a saturated carbon and hence it does
(U) III
not show tautomerism. While all, other options
I and II or I and III are metamers since they differ contain a-hydrogens on saturated carbons next to

for
in the number of carbon atoms on either side of O the keto group and hence show tautomerism.
atom is different. 40. Seven, out of which four are alcohols :
u
Thus, option (a) is correct. {/) CH3CH2CH2CH2OH, (ii) (CH3)2CHCH20H,
s
O OH
(Hi) CH3CH2CHOHCH3, (/v) (CH3)3C0H
ook
and three are ethers :
Yo
Tautomerism (V) CH3CH2OCH2CH3, (vi) CH3OCH2CH2CH3
35. >
and (viV) CH30CH(CH3)2.
eB

HO' OH
(c) Aromatic 42. In nucleophilicdisplacementreaction, the flow of
electrons occurs in a manner which expels the
Tautomer of option (c) being aromatic has weaker nucleophile, i.e.. Cl” ion.
r

maximum enol content. Thus, option (a) is correct.


ou
ad

36. Keto-enol tautomerism involves migration of a 43. As the electronegativity of the central atom
hydrogen atom either from a-(l,3) or y-(l,5) or increases, its -I-effect increases accordingly, i.e.,
-I-effect increases in the order :
Y

e-(I,7)-position to the oxygen atom of the keto


group. -NR2 < -OR < -F.
Re

45. In BH4 , B does not have a lone pair and hence it


nd

H3C CH3 H3C CH3


X X cannot act as nucleophilic centre.
Fi

I.
46. Since 0—H bond is stronger than S—H bond,
therefore, RO” has a greater tendency to accept a
XX proton than RS” and hence RO” is a stronger base
OH than RS”. Conversely, since S is less electro
Here, a-H migrates negative than 0, therefore, RS” is more willing to
donate a pair of electrons than RO” and hence RS”
is more nucleophilic than RO”. Thus, option (h)
,CH3 is correct.
II.
CH3 47. Aromatic compounds are resonance stabilised.
XX Although compound (b) is also resonance
stabilized but it less resonance stabilized than
OH
Here, a-H migrates aromatic compounds.
ORGANIC CHEMISTRY-SOME BASIC PRINCIPLES AND TECHNIQUES 12/215

Thus, the stability decreases in the order :


For Difficult Questions
(C) > (B) > (A), i.e., option (a) is correct.
+
+ CH2 CH2
<—►

O "O' ~o 51. (A) ◄—► etc.


V

low
p—CH3 CH3 +

48. ^ ► CH2 CH2


●0

Here lone pair of electrons on O are conjugated


(B) H 9 ◄—► H

with the d.b. and hence are delocalized. H H

49. Since -NO2 group is electron-withdrawing,


therefore, presence of a +ve charge on the carbon
+

ee
CH2
atom which is attached to -NO2 group makes the

F
(C)

Fr
carbocation unstable. Thus, carbocations (h)
and {d) are destabilized.
Carbocation (a) is also destabilized because in one not stabilized by resonance

for
of its resonating structures shown below, +ve +

charge is present on the carbon atom to which


-NO2 group is attached. Thus, carbocation (a) is
ur (D)
CH

also destabilized.
s
not stabilized by resonance
ok
Yo
NO2 NO7 Only carbocation (A) and (B) are .stabilized by
resonance, i.e., option (d) is correct.
Bo

Q ◄ ►
52.

vX X OCH3 .0CH3
re

<—►
Ljt
Consequently, carbocation (c) is expected to be ‘+

most stable since in none of its resonating a(i) a («)


ou
ad

structures, +ve charge moves to that carbon which OCH OCH


is attached to -NO2 group.
Y

<—►
+
NO2 NO2 NO2 +

b(i) b (n)
nd
Re

Y > Y. 4 ► Y\ OCH
< ►
Fi

H + H H

Thus, option (c) is correct. b {iii)


50. Higher the delocalization of the negative charge,
more will be the stability of the anion. H3CO
+
► H 3C(0
+

NO,
c(i) c (ii)
4 S

< < "O


4 ► H3COt
O"

(A)
Weakly stable
(B)
Moderately stable
O
0 (C)
Most stable
c (iii)
12/216 New Course Chemistry (XI)BZsZ9S

Now since CH3 has three, ethyl has two, isopropyl


For Difficult Questions
has one and rerr-butyl group has no a-hydrogen,
therefore, hyperconjugation effect decreases and
.+
hence the rate decreases in the same order, i.e.,
H3C0‘ -4 ►
option {b) is correct.
d(i) + 56. Due to 9 hyperconjugalion structures of /err-butyl
carbocation against 5 hyperconjugation structures
H3C0"^ CI> of jec-butyl carbocation, tertiary butyl
d{ii) carbocation is more stable than sec-butyl
carbocation.

57. During EC. alkylation reaction, carbocations are


< ► H3CO' the intermediate. If the initially formed carbocation
d {Hi) is less stable, if undergoes rearrangement to the

w
+
more stable carbocation before reacting. For
4 ► H3CO example,

F lo
d (iv) Anhyd.
AICI3
The carbocation {d) has four resonating structures CH3CH2CH2 Cl ► CH3 CH CH2
and hence the most stable.
H
53. Hyperconjugation in free radicals occurs only

ree
when the a-carbon to the carbon atom carrying l° Carbocation
the odd electron has a hydrogen atom, i.e., option

for F
{less stable)
(a) is correct. 1, 2-Hydride
shift > CH3 CH CH3
2“ Carbocation
{moK stable)

CH3 Thus, option (c) is correct.


Your
HI
58. The lone pair of electrons on S is present in a 3p-
ks

Radicals (I and II) do not have hydrogen/s on the orbital while those an N, O and F are present in a
eBoo

a-carbon/s and hence do not show hyperconjugation. 2p-orbital. Further, the carbon atom bearing the
54. In rert-butyl cation, the carbon bearing positive -i-ve charge has an empty 2/7-orbital. Since effective
charge has an empty /7-orbital and hence hyper resonance stabilization occurs between orbitals of
conjugation here involves c-p (empty) electron similar sizes, therefore, least effective resonance
ad

delocalisation.
will be between S and C atoms. Among N, O and
our

Empty F, since N is least electronegative, therefore, most


7:
H effective resonance will be between N and C and
CH3 CH3 CH3
hence {CH^)2NCH!^ carbocation would be the
^c- c H
Re

H most stable.
CH3 H
Y

H
CH3 CH
Find

/m-Buty! cation 2-Butene


^N^CH2 < ♦
N=CH2
2p
In 2-butene, 7C-bond contains two electrons, one CH3 / 2p
CH3
electron from each of the two carbon atoms , i.e., 59. Carbocation (I) is stabilized by +R-effect of O as
tr-MO contains a pair of electrons, but 7C* MO is well as -i-I-effect of two CH3 groups ; carbocation
empty and hence electrons of the a C—H bond (II) is stabilized by +I-effect of CH3 and
can move only into the 7t* orbital. Thus, option CH2CH(CH3)2 groups ; carbocation (III) is
(a) is correct. stabilizedby +R-effect of O and -t-I-effect of one
55. More the number of H atoms on the a-carbon of CH3 group while carbocation (IV) is stabilized by
the group R, more are the hyperconjugation +I-effect of CH2CHCH(CH3)2 group. Thus,
structures, weaker is the C—Br bond and hence decreasingorder of .stability of these carbocations
higher is the rate of the reaction. IS : I > III > II > IV.
ORGANIC CHEMISTRY-SOME BASIC PRINCIPLES AND TECHNIQUES 12/217

For Difficult Questions CN® (V) < HC s C® m < CH^ = CH—CH? (») ...II
Combining the two trends (I) and (II), the overall
60. In (he given carbocation, H migrates from C-2 to basicity of the five anions increases in the order:
C-3 because
(v) < {Hi) < (//) < {/v) < (i)
(/) the +ve charge generated on C-2 in I is in
conjugation with the ;i-electrons of the O atom Thus, option (c) is correct.
64. In option (6), Cl is present on a saturated (or sp^-
H H hybridized) carbon and hence resonance is not
+ I 3 14 5 possible and hence it has the longest C-Cl bond.
CH 1—C—C—C—CH, ◄—*

w
Vhm I
c:pH H CH3
H H
65. OCH3 and CH3 groups increase the electron den
sity on the carbon atom attached to Cl and hence
I
2 I3 I4 5 increase the C—Cl hand length as shown below :
CH3 C—C—C—C CH3

F lo
li I I II A
+ OH H CH3 id) Cl—CH=CH—0CH3«<—►
©
(i7) It is stabilized by +I-effect of CH3 group.

ee
Cl-e-CH—CH=0CH3
61. The corresponding acids of {/), (it), (Hi) and (<V) +R-Effect ofOCH3 group

Fr
respectively are :
CH3COOH, CH3CH2CH2CH3, NH3 and CgHgOH H

Their acidic character decrea.ses in the order: A ri

for
(c) Cl CH=CH—C
CH3COOH > CgHgOH > NH3 > CH3CH2CH2CH3
ur
Now since a strong acid has a weak conjugate base, H
therefore, their basic character decreases in the
opposite order:
s
ook
©
CH3CH2CH2CH2 (») >NH^ (Hi) > CgHgO (iv)
Yo
Cl-t-CH—CH=C H

>CH^COO-(i)
eB

H
i.e., option (c) is correct. ....Hyperconjugation effect
62. C is less electronegative than O and N hence is
more willing to donate a pair of electrons. Out of In CH2 = CH—Cl (h), due to + R-effect of Cl,
C—Cl bond has some double bond character and
r

HC s C” and CH3CH2 acetylenic carbon is more


ad
ou

hence is shorter
elecu-onegative and hence less willing to donate a
A ©
pair of electrons. Thus, CH3CH2 in the strongest {b) Cl—CH=CH2**—► C1=CH—CH2
Y

base. than in options (c) and (d).


63. Due to + I-effect of the three CH3 groups, tert- In option (a), due to strong -R-effect of the
Re
nd

butylcarbanion (/) is the strongest base followed —^N02 group


by methylcarbanion (iV) with one CH3 group, i.e.,
Fi

(iV)< (;)... (I).


A r\
To decide the basicity of the remaining three
anions, let us consider their acid strength. The O

acids of the correspondinganions are : 0"

H2C = CH—CH3, HC = CH and HCN :C1=CH- CH=N

Their acid strength decreases in the order : 0‘

and + R-effect of the Cl atom, the C—C! bond


HCN > HC = CH > CH2 = CH—CH3
has sufficient double bond character and hence
Since a strong acid has a weak conjugate base, C—Cl bond is the shortest.
therefore, the basic strength of the three anions
increases in the reverse order, i.e.. Thus, option (a) is correct.
12/218 'P'laeCee^’^ New Course Chemistry (XI)EEISD

72. Neither p-niirophcnol nor picric acid is steam


For Difficult Questions
volatile and hence their mixture cannot be
separated by steam distillation. Since amount of
o o
the mixture to be separated is very small (100 mg),
II hv
therefore, column chromatography using 2-5 ft
66. —C—Q-LLo C—C,H5 Homolytic
fission
column of silica gel is not convenient. Thus, only
(C7H502)2 choice is prcpju’ation TLC i.e., option (c) is correct.
o 73. Distillation under reduced pressure.
74. Steam volatile, immiscible with water.
C O' > 2 + 2 CO2

w
X
75. Independent of physical state of the pure
compound.
Thus, option (d) is correct. 76. Paper chromatography is an example of partition

F lo
67. The ease of abstraction of different H atoms chromatography.
depends upon the stability of free radicals formed 77. Since the column is eluted with the solvents having
after the abstraction of the H atom. The relative
more proportion of ethyl acetate, the more polar
stability of the free radicals formed after the compoundwill come out first.

ee
abstraction of and atoms respectively Since the order of polarity of the given

Fr
follows the order :
compounds is : acetophenone (C. }i = 3 05 D),
Me Me H Me benzanilide (A, u = 2-11 D) and aniline (B, |i =
Hz, 1-59 D), therefore, order of elution is : C > A >
Hz,

for
B, i.e.. option (d) is correct.
> >
ur
78. Glycerol decomposes at its b.p., therefore, it is
separated from spent lye by distillation under
H, He reduced pressure.
s
ook
3® allylic 2° allylic 79. The o- and p-nitrophenols are separated by steam
Yo
distillation sine o-isomer is steam-volatile due
Thus, option (/?), i.e., > H^,.
to chelation (intramolecular H-bonding) while
68. A carbene, i.e., dichlorocarbcne is generated
eB

p-isomer is not steam-volatile due to association


according to the following mechanism.
of molecules by intermolecular H-bonding.
OMe 80. Sequence of elution depends upon the polarity of
r

the compound to be eluted ; higher the polarity,


lO
ad
ou

OMe
CI3C—C—OMe ►CI3C—C—OMc later is the sequence of elution. Anthracene being
-Na'*’
b least polar is eluted first, followed by chloro
benzene having little higher polarity while p-cresol
Y

being most polar is eluted last of all. Thus, option


MeO—C—OMe + C^Cl ■HNa* ^ :CCl2 (a) is correct.
\ -NaCI
Re

Cl Dichlorocarbcne
nd

0 81. In sodium fusion test, N of the organic compound


is converted to NaCN.
Fi

69. (A) is elimination, (B) is substitution and (C) is 82. AgF is highly soluble in H2O therefore, NaF
addition reaction. formed in the Lassaigne’sextractdoes not produce
a ppt. with AgN03.
Heat
70.
-I- 86. %0 = 100 - (20 0 -I- 6-67 -t- 46-67) = 26-66
Diels-Alder
reaction
Conjugated Alkyne Unconjugated 20 6-67 46-67 26-66
diene alkadiene C;H;N:0 = —
12 1-0 14 16
71. Phenol on reaction with dilute nitric acid gives a = 1-67 : 6-67 : 3-33 : 1-67 = 1 : 4 : 2 : 1
mixture of o-nitrophenol {steam volatile) and p-
nitrophenol (not steam volatile). The most efficient E.F. = CH4N2O
Now E.F. wt. = 12 + 4 -f 28 -b 16 = 60
method to separate this mixture is steam
distillation, i.e., option (c) is correct. and Mol. wt. = 60 (given)
ORGANIC CHEMISTRY-SOME BASIC PRINCIPLES AND TECHNIQUES 12/219

Volume of the acid used = 50-40


For Difficult Questions
= 10mLof0-5MH2SO4
Mol.wt. 60
M.F. = E.F. X I4x«.M Va
= CH4N2O X —

w
E.F. wt Now %N = a a
60
wt.of substance taken

= CH4N2O
I4x2x0-5xl0
Now out of the given four compounds = 28
0-5
{CH3CH2CONH2 (fl) = C3H7NO : CH3CONH2
(c) = C2H5NO ; CH3NCO (6) = C2H3NO), the 91. Volume of the acid taken = 60 mL of M/10 H2SO4

e
o
MF of (NH2)2C0 (d) matches with that of Let the volume of the add left unused
CH4N2O, there-fore, the given compound is urea,

re
= V mL of M/10 H3SO4
NH2CONH2.
Volume of M/IO NaOH used to neutralize the
Further since urea on heating loses NH3 and gives unreacted acid = 20 mL

Frl
F
a residue called biuret which gives a violet
Applying molarity equation (page 12/118),
colouration with alk. CUSO4 solution, therefore,
the given organic compound is NH2CONH-,. i.e., /i a.M . V
option {d) is correct. ou a

(Acid)
a

(Base)

sor
A

NH2Co|nH2 + H^NHC0NH2 -NH3


where M^ and V^ are the basicity, molarity and
volume of the acid and M^ and V^ are the
Urea (two molecule.s)
acidity, molarity and volume of the base

kf
Alk.CuS04
NH2CONHCONH2 > Violet colouration oo 1 1
or 2x — xi> = lx — x20
Biuret
10 10
87. Kjeldahl’s method for nitrogen estimation is not v= 10 mL
Y
applicable to nitro compounds such as Volume of the acid used = 60-10
B

nitrobenzene and compounds containing nitrogen


in the irng such as pyridine, since during digestion, = 50mL ofM/10H2SO4
Now % of N
there are not completely converted into
re

(NH4)2S04. Furthermore, diazonium salts lose N2 _ 14xMolarityx Volume X Basicity of the acid used
oY

on heating during digestion. Thus, Kjeldahl’s Weight of the substancetaken


u

method for nitrogen estimation is only applicable 14x1x50x2


ad

to aniline, i.e., option (b) is correct. = 100


10x14
89. Weight of substance taken = 29-5 x 10"^ g
d

92. Let the mass of gaseous hydrocarbon taken = w gm


Volume of acid taken = 20 mL of 01 N HCI
12 308 84
in

Now 15 mL of 0-1 N NaOH 2 15 mL of 0-1 N HCI %C = —X xl00 = —


44
Re

Vt' w
Volume of acid used = 20 - 15 = 5 mL of 01 N HCI
2 0-72
F

%H = —x xl00=-
%N =
14 X Volume x Normality 14x5x0d 18 M’ \V

Weight of substance taken 29-5xl0"^ 84 1 8 1


Ratio of C : H atoms = — x — x- = 7:8
= 23-73 vv 12 w 1
Thus, option (d) is correct. Thus, E.F. = C^Hg
90. Volume of acid taken = 50 mL of 0-5 M H^S04 95. Step 1. Molecular formula of isomers (A)
Let the volume of the acid left unused = v mL of and (B).
M/10 H2SO4 52.2 4.90 42.9
C : H : Br =
Applying molarity equation (page 12/118), 12 80
n^M^V^ (acid) = n^M^V^ (base), we have, = 4.35:4.90:50.536
2 X 0-5 X i; = 1 X 0-5 X 80 or u = 40 mL
= 8:9:1
12/220 New Course Chemistry (XI)B&I9D

(«) Since isomer (B) on KMn04, oxidation gives


For Difficult Questions fc-bromobenzoic acid, therefore, isomer (B) is a
p-bromobenzene derivative.
Empirical formula mass =12x8 + 9x1+80 xl In other words, Br can not be in the side chain and
= 185 hence structure (I) for isomer (B) is ruled out.

w
Molecular mass = 184 g mol
-1 Instead it has only structure (II)

.. M.F. = C.F. = CgH^Br CH2Br CH2CH3


Step 2. Structureof isomers (A) and (B) CH 3

(0 Since isomer (A) is optically active and gives


benzoic acid on oxidation with KMn04, therefore,

o
e
1
isomer (A) must be a benzoic acid derivative Br

re
having bromine cantoning two-carbon side chain. II
A structure which satisfies all these facts is Isomer (B)

Frl
Out of the four options given in the question,

F
CHj-CHBr-CgHg
(A) option (c) is correct.

Ql Multiple Choice Questions (with One or More than One Correct Answers)
ou
r
98. Only option (c) does not have a a-H on a oxygen or nitrogen and electrophiles due to the

so
saturated carbon and hence does not exhibit presences of multiple bonds.
103. Allyl carbanion due to resonance is planar, tert-

kf
tautomerism while all others do.

99. CH3C = N and H2C = O both act as nucleophiles Butyl free radical and /erf-butyl carbocation are
due to the presence of lone pair/s of electrons on also planar but tert-butyl carbanion is pyramidal.
oo
QQ
Y
Multiple Choice Questions (Based on the given Passage/Comprehens ion)
B

115. Ctubocations are stabilized by +I-effect of CH3 116. All the atoms in structures (b) and (d) have
groups and destabilized by -I-effect of F atoms. completed octets and hence are more stable than
re

+ structures (a) and (c) in which the positively


Therefore, (CH3)2CH is the most stable charged carbon has only sextet of electrons. Out
oY
u

carbocation. of (fl) and (c), {a) is stabilizedby +R-effectof the


oxygen atom but (c) is not. Thus, (c) is the least
ad

stable structure.

VI. Integer Type Questions


d
in

123. Zero. There are six (3 + 2 + I) a-hydrogens and hence


Re

124. Four, i.e., n-butyi, isobutyl, 5ec-butyl and ten- six hyperconjugation structures are possible.
butyl.
F

130. D.B.E. of C4H5N


125. Two, i.e., benzoquinone and benzaldehyde.
126. Five. Refer to Ans. to Q. 2, page 12/67. 4(4-2) + 5(l-2)+l(3-2)
+ 1
2
127. Three, i.e., H+. NOJ , Cl^.
8-5 + 1
128. Six,/.e., NH2,R“, H2O:. . H". 2
+1=2+1=3

129. ^3^ ^ '


CH2—CH3 Since D.B.E of C4H5N is 3 and further since it is
an a cyclic compound, therefore, it contains either
three double bonds or one double bond and one
triple bond. The structure of the compound which
satisfies these conditions is
ORGANIC CHEMISTRY-SOME BASIC PRINCIPLES AND TECHNIQUES 12/221

For Difficult Questions


Br

s/>^ CH3CH2-CH-CH2CH3
3-Bromopeniane
HC s C-CH2-CH = NH
This contains only one 5y7^-hybridized carbon. n. 2-Methylbutane has four monobromo derivatives

ow
131. C5H|2 has following three structural isomers :
CH3 CH3
I
(0 CH3CH2CH2CH2CH3,
n-Pemane BrCHj-CH-CH^CHj , CH3-C-CH2CH3 ,
I
Br
CH
3\
(M) CH-CH2-CH3 CH.Br
CH3

e
CH3 I ^ i

re
2-Methylbutane CH3-CH-CH-CH3,
] CH3~CH-CH2CH2Br

rFl
CH- m. Neopentane has only one mono bromo derivative.

F
I ^ Thus, in all, there are 3 + 4 + 1 = 8 monobromo
{Hi) CH,-C-CH, derivatives of alkanes having molecule formula,
3 , 3
CH3 ^5^12-

or
Neopentane
ou 132. There are only three electrophilic centres as shown
by arrows in the given compound.
I. Now rt-Pentane has three monobromo derivatives:

CH3CH^CH2CH2CH2Br,
l-Broinopeniane ksf ; .CH3
oo
O
Br
Y
B

CH3-CH-CH2CH2CH3 , CH2—C=N
2-Bromopentane t
re

VII.
Numerical Value Type Questions 0^ Decimal Notation)
oYu
ad

conc.H2SO Now, 90 g of oxalic acid evolve CO = 28 g


133. HCOOH ^ CO -b H,0
Formic add 28 g 4-5 g of oxalic acid will evolve CO
46 g
d

28
Now 46 g of HCOOH evolve CO = 28 g = _x4-5 =l’4g
in

90
Re

28
2-3 g of HCOOH will evolve CO = ~ x 2-3 Total amount of CO evolved = 1-4 + 1-4
46
F

= 2-8g
= l-4g
COOH
conc,H2SO 134. CH3CH2CH2CH = CH2 + Br2 ■>

COOH
CO + CO2 + H2O Pent-l-ene 160g
28g Mol, mass : 70 g mol”'
Oxaiicacld
90g CH3CH2CH2-CHBr-CH2Br
When the gaseous mixture of (CO + CO2) is 70 g of pent-l-ene require Br2 = 160 g
passed through KOH pellets, CO2 is absorbed 5 g of pent-l-ene will require Br2
while CO passes out 160
2 KOH CO2 ■> K2CO3 -I- H2O 70
x5 = 11-428 g = 11-43 g
12/222 ^>tadeef.i'ii. New Course Chemistry (XI)SS2lHD

For Difficult Questions

Vlll. Assertion-Reason Type Questions

135. Correct explanation. Allyl carbanion is stabilized 146. Correct explanation. Due to greater Ip-bp over
by resonance. bp-bp repulsions, carbanions assume pyramidal
136. Correct statement-2. +I-effect of the CH3 groups shape.
tends to destabilize the /tv7-buiyl carbanion. 147. Correct assertion. re/t-Butyl carbanion is less

w
stable than methyl carbanion.
137. Correct explanation. Nitrenes are too reactive to
be isolated. Correct reason. +I-effect of the CH3 groups
138. Correct statement-1. Beilstein lest cannot be used destabilize the tert-butyl carbanion relative to
methyl carbanion.

F lo
to detect Ouorine in the organic compound.
142. Correct statement-1. Benzoic acid cannot be 148. Correct reason. Due to chelation, o-nitrophenol
is steam volatile while p-nitrophenol is not steam
separated from a mixture of benzoic and
volatile.

ee
naphthalene by using benzene since both of them
are soluble in benzene. 150. Correct assertion. Only essential oils are purified

Fr
143. Correct assertion. But-l-ene and 2-methylprop- by steam distillation.
1-ene are chain isomers. Correct reason. Compounds which are volatile
Correct reason. Chain isomers have same in steam and insoluble in water can be purified by

for
steam distillation.
molecular formula but different arrangement of
ur
carbon atoms. 152. Correct reason. Compounds containing N, S and
C give red colour in Lassaigne’s test.
144. Correct assertion. In CH^ = C = CH2, central
carbon atom is ip-hybridizedwhile the terminal 153. Correct explanation. Hydrazine does not contain
s
ones are sp^-hybridized. carbon. Therefore, NaCN cannot be formed which
ook
Yo
Correct reason. The two H-aloms on the first and is essential for positive test for nitrogen.
third carbon atoms lie in perpendicular planes. 154. Correct assertion. Vapour density and density are
eB

different things.
145. Correct explanation. Free radicals have odd
number of electrons in the valence shell and hence Correct reason. Molecular weight is twice vapour
are paramagnetic. density.
r
ou
ad
Y
Re
nd
Fi
ow
HYDROCARBONS

e
Fl
re
13.1. INTRODUCTION

F
The simplest organic compounds containing carbon and hydrogen only are called hydrocarbons. These are
widely distributed in nature in the form of petroleum, natural gas and coal. Thus, hydrocarbons are considered to
ur
be the parent organic compounds while all other compounds are thought to have been derived from them by

r
fo
replacement of one or more of their hydrogen atoms by appropriate functional groups.
Hydrocarbons play a key role in our daily life. You must be familiar with the names LPG (liquefied
ks
petroleum gas) and CNG (compressed natural gas) used as fuels. Another name LNG (Liquefied natural gas)
has also been used in recent times. It is also used as a fuel and is obtained by liquefaction of natural gas.
Yo
oo
Gasoline (petrol), kerosene and diesel are obtained by fractional distillation of petroleum found under the
earth’s crust. Petroleum deposits are usually covered with a gaseous mixture called natural gas. It mainly
B

consists of methane along with decreasing quantities of ethane, propane, butanes and the vapours of low
boiling pentanes and hexanes.
re

Coal gas, obtained by destructive distillation of coal, is also used as a fuel. Kerosene oil is used as a
domestic fuel but it causes some pollution. Automobiles use petrol, diesel and CNG as fuels. Vehicles using
u

petrol and CNG as fuels cause less pollution. All these fuels contain mixtures of hydrocarbons, which are
ad

sources of energy. Hydrocarbons are extensively used for manufacture of polymers such as polythene,
Yo

polypropene, polystyrene, etc. Higher hydrocarbons are used as solvents for paints. They are also used as the
starting materials for manufacture of many dyes and drugs. Thus, hydrocarbons play an important role in our
daily life. Therefore, in this unit, we shall briefly discuss different types of hydrocarbons, their structures,
d
Re

shapes ; physical and chemical properties.


in

13.2. CLASSIFICATION OF HYDROCARBONS


F

On the basis of structure, hydrocarbons can be broadly divided into the following two types :
I. Acyclic or open chain hydrocarbons
2. Cyclic or closed chain hydrocarbons
1. Acyclic or open chain hydrocarbons. These compounds contain open chains of carbon atoms in
their molecules. They are also called aliphatic hydrocarbons. These are further classified into the following
three categories ;
(/) Alkanes, (ii) Alkenes and (Hi) Alkynes

13/1
13/2 “p>uuiee^'A New Course Chemistry (XI)EEISII

This classification is primarily based upon the type of the carbon-carbon bonds present in their molecules.
For example,
An alkane has only carbon-carbon single bonds. For example,
CH3—CH3 CH3—CH.—CH3 CH3—CH2—CH2—CH3
Ethane Propane Butane

w
An alkene has one carbon-carbon double bond. For example,
CH2 = CH2 CH3 CH = CH-, CH3—CH = CH—CH3
Ethene Propene But-2-ene

(formerly 2-buiene)
An alkyne has one carbon-carbon triple bond. For example,

o
e
CH = CH CH3 = CH CH3CH2—C = CH CH3—C s C—CH3
But-2-yne

re
Elhyne Propyne But-l-yne

rFl
(formerly I-bulyne) (formerly 2-butyne)

2. Cyclic or closed chain hydrocarbons. These compounds contain closed chains or rings of carbon

F
atoms in their molecules. They are further divided into the following two classes :
(a) AlicycUc hydrocarbons (b) Aromatic hydrocarbons
(a) Alicycllc hydrocarbons. Hydrocarbons which contain a ring of three or more carbon atoms and

or
ou
have properties similar to those of aliphatic hydrocarbons are called aiicyclic hydrocarbons. These are
further divided into the following three categories ?

ksf
(0 Cycloalkanes («) Cycloalkenes and (Hi) Cycloalkynes
(i) Cycloalkanes. Saturated aiicyc lic hydrocarbons in which all the carbon atoms are joined by single
oo
covalent bonds are called cycloalkanes. For example.
Y
B

Cyclopropane Cyclobutane Cyclopentane Cyclohexane


re

(ii) Cycloalkenes. Unsaiurated aiicyclic hydrocarbons which contain one carbon-carbon double bond
are called cycloalkenes. For example.
oYu
ad

Cyclopentene Cyclohexene
d

Cyclopropene Cyclobutene

(iii) Cycloalkynes. Unsaturated aiicyclic hydrocarbons which contain one carbon-carbon triple bond
in
Re

are called cycloalkynes. For example, cyclopentyne, cyclohexyne, cycloheptyne, cyclooctyne, cyclononyne,
etc. However, cyclopentyne and cyclohexyne are highly strained and unstable, cyclooctyne is strained but
F

somewhat stable while cyclononyne and higher cycloalkynes are unstrained and hence are quite stable.
(6) Aromatic hydrocarbons. As discussed in unit 12, aromatic hydrocarbons are of two types :
(j) Benzenoid aromatic compounds. Hydrocarbons and their alkyl, alkenyl and alkynyl derivatives
which contain one or more benzene rings either fused or isolated in their molecules are called benzenoid
aromatic hydrocarbons. They are also called arenes (aromatic alkenes). For example.
CH3 CH2CH3 CH3
CH3

Benzene Toluene Ethylbenzene o-Xylene


13/3
HYDROCARBONS

Arenas may also contain two or more isolated or fused benzene rings. For example,

Naphthalene Anthracene
Diphenyl

iii) Non-benzenoid aromatic compounds. Aromatic hydrocarbons which do not contain a benzene
ring but instead contain other highly unsaturated rings are called non-benzenoid aromatic compounds. For
example, iropone, tropolonc and azulene. (For structures refer to page 12/12).
The above classification of hydrocarbons is summarized below :
Hydrocarbons

w
Acyclic or Open chain Cyclic

F lo
Saturated Unsaturated Alicyclic Aromatic

(Alkanes)

e
Alkenes Atkynes

Fre
Benzenoids Non-
Saturated Unsaturated (Arenes) benzenoids
(Cycloalkanes)

for
Cycloalkenes Cycloalkynes
r
PART—I. ALKALIES
You
ks
133. ALKANES
o

We have discussed above that saturated hydrocarbons can be either acyclic or cyclic. The acyclic saturated
eBo

hydrocarbons are called paraffins (Latin: parum = little, ajfmis = affinity) since they are relatively unreactive
towards most of the reagents such as acids, bases, oxidising and reducing agents. However, under dra.stic
conditions, i.e., at high temperature and pressure, alkanes do undergo different types of reactions like
halogenation, nitration, sulphonation, pyrolysis, etc.
our
ad

133.1. Nomenclature of Alkanes


In the rUPAC system, they are called alkanes. Their general formula is C^2n+l where = 1, 2, 3,....
etc. The nomenclature of alkanes has already been discussed in unit 12. However, for purpose of recapulation,
the lUPAC names of some straight chain alkanes is given below ;
dY
Re

No. of carbon Formula lUPAC name No. of carbon Formula lUPAC name
atoms atoms
Fin

Undecane
1 CH4 Methane 11 Ci,H24
Dodecane
2 C2H6 Ethane 12 ^12^26
13 C13H28 Tridecane
3 C3H8 Propane
Butane 14 C 14^30 Tetradecane
4 CaHio
Pentane 20 ^20^42 Eicosane
5 CsHi2
Hexane 30 ^30^62 Triacontane
6 C6H.4
10 ^10^22 Decane

The rules for lUPAC nomenclature of branched chain alkanes have already been discussed in unit 12.
13/4
New Course Chemistry (XI)KSSISD

Further Tips about lUPAC Nomenclature of Branched Chain Alkanes


For turther illustration, lUPAC names of some complex alkanes are discussed below ;
●)
CH3 CH^—CH3
4 5 6

ii) CH3—CH—CH2—CH — CH2—CH3


4-Ethyi-2-methylhexane
Remarks : Lowest loctant rule and alphabetical order of substituents is followed here.
I 2 3 4 5 6 7 8 9

(,● , CH3—CH2—CH2—CH2 CH—CH2—CH2—CH2—CH3


'CH2

w
CH3—C—CH3
3]
CH3

F lo
5-(2, 2-Dimeihylpropyl)iionane
Remarks : Numbering in the complex substituent begins with die carbon atom attached to the main chain.

ee
CH,—CH,
8 7 6 5 4 I3 2 I

Fr
(Hi) CH3—CH,—CH,—CH—CH—C—CH,—CH,
1 I I I - ^
CH3—CH CH3 CH2—CH3

for
2CH3
ur
3, 3-Diethyl-5-isopropyl-4-methyloctane or 3, 3-Diethyl-4-methyl -5-(l-methyiethyl)octane
Remarks : The prefix iso is considered to be part of the fundamental name of the alkyl group while
considering the alphabetical order of substituents.
s
ook
1
Yo
CH(CH3)2
i 3 4l 5 6 7 8 9 10

(iv) CH3—CH2—CH2—CH—CH—CH2—CH2—CH2—CH2—CH3
eB

1
H3C—CH—CH2CH3
5-jec-BiUyl-4-isopropyIdecane or 4-( I -Methylethyl)-5-( 1 -methylpropyl)decane
Remarks: The prefix sec is not considered while deciding alphabetical order of substituents and isopropyl
r

is taken as one word.


ou
ad

133.2. Isomerism of Alkanes


Methane, ethane and propane have only one structure since there is only one way in which 1, 2 and 3
Y

carbon atoms can be connected. For butane, two isomers are possible since four carbon atoms can be joined
in two difference ways, i.e.,
Re
nd

CH3—CH-)—CH2—CH3 CH,—CH—CH,
Butane (ii-Biilane) 3 , 3
Fi

(b.p. 273 K) CH3


2-Methylpropane ijsohutane)
(b.p. 261 K)
Now five carbon atoms can be joined in three different ways and hence pentane has three isomers.
These are :

I 2
CH3
1 2I 3
CH3—CH2—CH,—CH,—CH3 CH3—CH—CH2CH3 CH,—C—CH,
Pentane (n-Pentane) 3 j 3
(b.p. 309 K)
CH3 CH3
2-Melhylbutane (Isopentane) 2, 2-Dimethylpropane {Neopentane)
(b.p. 301 K) {b.p. 282-5 K)
HYDROCARBONS 13/5

On the other hand, six carbon atoms in hexane can be arranged in five different ways. Thus, hexane has
five isomers. These are :
I 2 3 4 5

CHj—CH2—CH2—CH2—CH2—CH3 CH3—CH—CH2—CH2—CH3
Hexane (n-Hexane)
(b.p. 342 K)
CH3
2-MethyIpentane (Isohexane)
(b.p. 333 K)

CH3
1 2 3 4 5 1 2 3 4 1 2I 3 4

w
CH3—CH2—CH—CH2—CH3 CH,—CH—CH—CH^ CH3 —c—c CH2—CH3
3 I I 3
CH3 CH3 CH3 CH3

F lo
3-Methylpeniane (b.p. 336 K) 2, 3-Dimethylbutane (b.p. 331 K) 2, 2-Dimethylbutane
(Neohe.xane) (b.p. 323 K)

Similarly, we can show that heptane (CyHjg) has 9, octane (CgHjg) has 18, nonane (C9H20) has 35

ee
while decane (C1QH22) has 75 isomers.
Such structural isomers which differ in the arrangement of carbon chain are called chain or nuclear

Fr
isomers.

13.3.3. Structure of Alkanes

for
Alkanes contain only carbon-carbon and carbon-hydrogen single bonds with average bond lengths of
our
154 pm and 110 pm respectively. Each carbon in alkanes is rp^-hybridized. Since, a 577^-hybridized carbon
has four half-filled rp^-orbitals, therefore, it forms four a-bonds. These four bonds are directed towards the
comers of a regular tetrahedron. In other words, the angle between any two adjacent bonds is 109® 28
s
(tetrahedral angle). Thus, alkanes have tetrahedral structures.
ook
Let us now discuss the structure of the simplest alkane, i.e., methane (CH4). In methane molecule,
the four hydrogens are present at the comers or vertices of
carbon lies at the centre of the tetrahedron while
Y
eB

the regular tetrahedron. Thus, methane has a tetrahedral structure as shown in Fig. 13.1(a).
Three-Dimensional (3-D) Structure of Alkanes
In unit 12, we have already discussed that three-dimensional. (3-D) structures of organic molecules can be
r

represented by wedge and dash formulae. For example, methane may be represented as shown in Fig. 13.1(b).
ou
ad

Another method of representing 3-D stmctures on a 2-D surface was introduced by Emil Fischer, a
German chemist in 1891 and after his name these are called Fischer Projection Formulae.
Y

FIGURE 13.1

H
Re
nd

H
H H
/ \
/ \
Fi

/ \
/ s
\
/ s
/ ●C \
H C H H' H
C
H

\
\ /

\
\ y
y
H
✓ H
H H H

Representation of methane molecule (a) tetrahedral (b) three-dimensional,


i.e., 3-D (c) another 3-D view (d) two-dimensional, i.e., 2-D or Fisher Projection Formula
13/6
^fuidce^'4. New Course Chemistry (XI)E2S0HD

Fischer Projection Formulae. To obtain Fischer Projection Formula, the three dimensional structure
of a molecule is oriented in such a manner that the vertical bonds are projected away from you and horizontal
bonds are directed towards you as shown in Fig. 13.1 (c). The molecule is then projected using a ray of light
in the plane of the paper in form of a cross. The carbon atom lies at the centre of the cross but is not shown.
Thus, Fig. 13.1(d) represents Fischer Projection Formula of methane.
13.3.4. Methods of Preparation of Alkanes
We have discussed above that petroleum is a valuable source for the large scale preparation of a number
of hydrocarbons including alkanes. But many alkanes are not available from petroleum or we might need
only a small amount of a pure alkane for our laboratory use. In such cases, we usually prepare them by

w
laboratory methods of preparation. Some of the laboratory methods of preparation of alkanes are discussed
below :

1. From unsaturated hydrocarbons. The process of addition of hydrogen to an unsaturated compound

F lo
in presence of a catalyst is called hydrogenation or reduction.
Alkenes and alkynes add one and two molecules of hydrogen respectively, in presence of a catalyst such
as Raney* nickel, platinum or palladium to form alkanes.

ee
RaneyNi
CH2 = CH2 + H2 ^ CH3—CH3

Fr
or PtorPd
Ethene Ethane

RaneyNi

for
CH3—CH = CH2 + H2 or PtorPd
^ CH3—CH2—CH3
ur
Propene Propane

Raney Ni
CH3—C = C—H + 2 H2
s
or Pt or Pd > CH3—CH2—CH3
ook
Yo
Propyne Propane
With platinum, palladium and Raney* nickel hydrogenation proceeds smoothly and quantitatively at
ordinary temperatures and pressures. However, with ordinary nickel, relatively higher temperatures {ca. 523-
eB

573 K) are often required. This hydrogenation of unsaturated hydrocarbons using ordinary nickel at a
temperature of about 523-573 K is commonly known as Sabatier and Sendern’s reaction or reduction.
Thus,
r

Ni
ad
ou

CH2 = CH2 + H2 523-573 K > CH3--CH3


Ethene Ethane
{Ethylene)
Y

Ni
CHsCH -I- 2H2 ^ CH3—CH3
Re

523-573K
nd

Ethyne Ethane

{Acetylene)
Fi

This hydrogenation reaction is used in the manufacture of Vanaspati Ghee from edible vegetable oils.
2. From alkyl halides. Alkyl halides can be converted into alkanes by any one of the following
methods :

(i) Through Grignard reagents. Alkyl halides especially, bromides and iodides react with magnesium
metal in presence of dry ethoxyethane (diethyl ether) to form alkylmagnesium halides. The.se are commonly
known as Grignard reagents after the name of their discoverer Victor Grignard who was awarded the Nobel
Prize in Chemistry in 1912 for the discovery of these versatile reagents.

*Raney nickel is an active form of nickel and is obtained by treating Ni-Al alloy with NaOH when A1
dissolves leaving nickel in the finely divided state.

<
HYDROCARBONS 13/7

Dry ether
R —X + Mg 4 R—Mg—X
Alkyl halide Grignard reagent

Dry ether
e.g.. CH3CH2—Br + Mg ^ CH3CH2—^MgBr
Bromoethane Ethylmagnesium
{Ethyl bromide) bromide

Since, carbon is more electronegative (electronegativity = 2.5) than magnesium (electronegativity =


1.2), therefore, C—Mg bond is quite polar. Hence, Grignard reagents readily react with compounds containing

w
active hydrogens such as water, acids, alcohols, ammonia, amines, etc. to form alkanes. For example.
CH3CH2—MgBr + H2O 4 CH3CH3 + Mg(OH)Br

F lo
Ethylmagnesium bromide Ethane

CH3—Mgl + CH3OH ■> CH4 + Mg(OCH3)I


Methylmagnesium iodide Methanol Methane

ee
(ii) Wurtz reaction. When an alkyl halide (preferably bromide or iodide) is treated with metallic sodium
in presence of dry diethyl ether, a synunetrical alkane, containing double the number of carbon atoms present

Fr
in the alkyl group, is formed. This reaction is called Wurtz reaction.
Diy ether
R—[ 2^ L R—R + 2NaX

for
our
Alkyl halide Alkane

Dry ether
e.g.. CH
3—[ ^r jb 2_N^-j^Br_ ^H3 ^ CH3 —CH3 + 2NaBr
Bromomethane
s Ethane
ook
{Methyl bromide)
Dry ether
CH3CH2—[ I_^22{a_+]_ ]-CH2CH3 ^ CH3CH2 —CH2CH3 + 2NaI
Y
eB

lodoethane Butane

{Ethyl iodide)
Thus, Wurtz reaction is a convenient method for the preparation of symmetrical alkanes (R—R), i.e.,
r

alkanes containing even number of carbon atoms.


ou
ad

However, if two different alkyl halides are used to prepare an alkane with odd number of carbon atoms,
a mixture of three alkanes is actually produced. This is due to the reason that the two alkyl halides in addition
Y

to reacting with each other also react amongst themselves giving a mixture of three alkanes as illustrated
below:
Re
nd

Dry ether
CH
3—[I_+_2^a_+ j-CH2CH3 ^ CH3 —CH2CH3 + 2NaI
Fi

lodomethane lodoethane Propane


{Methyl iodide) {Ethyl iodide)

Dry ether
CH3—[ T j- CH3 ^ CH3 —CH3 + 2NaI
Ethane

Dry ether
CH3CH2—[ ]—CH2CH3 ^ CH3CH2 — CH2CH3 + 2 Nal
lodoethane Butane

The boiling points of these alkanes are very close and hence they cannot be separated by fractional
reaction is used for the synthesis of only symmetrical
distillation. It is because of this drawback that Wurtz
13/8 New Course Chemistry (XI)ESS

alkanes and not for the preparation ofunsymmetrical alkanes, i.e., alkanes containing odd number of carbon
atoms.

Mechanism. Two different mechanisms have been suggested for the Wurtz reaction.
(a) Through intermediate formation of an organometallic compound
R—X + 2 Na ■> R"Na+ + NaX

w
Alkylsodium

R-Na'" + R-i-X^ R—R + NaX


Alkyl halide Alkane

o
{b) Through intermediate formation offree radicals

e
R—X + Na R + Na+ X" ; 2 R > R—R

re
Free radical Alkane

rFl
(iii) Corey-House reaction. Wurtz reaction does not give good yields of unsymmetrical alkanes, i.e.,

F
alkanes containing odd munber of carbon atoms. However, Corey-House reaction can be used to prepare
both symmetrical and unsymmetrical alkanes in good yields. In this reaction, the alkyl halide is first treated
with lithium metal in dry ether to form alkyllithium which is then allowed to react with cuprous iodide to yield

r
lithium dialkylcopper (also called GUman reagent). Lithium dialkylcopper thus obtained on subsequent
ou
fo
treatment with a suitable alkyl halide gives the desired alkane.

ks
Dry ether
R—X + 2Li - ^ R—Li + LiX
Alkyl halide Alkyllithium
oo
2R—Li + Cul ■»
[R2Cu]-Li+ + Lil
Y
Lithium dialkylcopper
eB

R>
:Cu Li'" +^R'-^X ^ R—R' + RCu + LiX
r

R-
Alkane AUqrlcopper
ou

(where R and R' may be same or different alkyl groups)


Y
ad

Dry ether
e.g.. CHj— CH2—^Br + 2 Li ^ CH3—CH2—^Li + LiBr
Ethyl bromide Ethyllithium
d

Dry ether
in
Re

2CH3—CH2—Li + Cul ■¥ (CH3—CH2)2CuLi + Lil


Ethyllithium Lithium diethylcopper
F

Dry ether
(CH3—CH2)2CuLi + CH3CH2CTI2—Br ^ CH3CH2CH2CH2CH3 + CH3CH2CU + LiBr
n-Propyl bromide n-Pentane Ethylcopper
(iv) Reduction of alkyl halides. Alkanes can also be prepared by the reduction of alkyl halides preferably
bromides and iodides. This reduction can be carried out in a number of different ways as discussed below :

(a) Reduction by dissolving metals such as Zn and CH^COOH or HCl; Zn and NaOH or Zn—Cu
couple and alcohol.
Zn/HCl
R—X > R—H + HX
Alkyl halide Alkane
HYDROCARBONS 13/9

CH3CI + Zn + H+ CH4 + Zn2+ + Cl"


Chioromethane Methane

CH3CH2CI + Zn + H+ ^ CH3CH3 + Zn2+ + Cl-


Chloroethane Ethane

CH3CH2CH2CI + Zn + H+ ^ CH3CH2CH3 + Zn2+ + Cl"


1-Chloropropane Propane

e.g., 2 CH3CH2—CHBr—CH3 + Zn + 2 H+ 2 CH3CH2—€H2CH3 + ZnBr2


2-Bromobutane Butane

Mechanism. iWth zinc and an acid or with Zn-Cu couple and C2H^OH, the reduction occurs by transfer

w
of electrons and not by nascent hydrogen as originally believed.

F lo
(OZn ^ Zn^+ + 2 e"

(//) R—X + e" -> R * + X-

(m) R- + e > R :

e
(tv) R r + > R—H or R r + C2H5OH > RH + C2H5O-

Fre
However, with Zn/NaOH reduction occurs with nascent hydrogen
Zn + 2 NaOH > Na2Zn02 + 2 [H]

for
Sod. zincate

R—X + 2 [H] > R—H + HX


r
ib) Reduction by chemical reagents such as LiAlH^ NaBH^ and Ph^SnH.
You
oks

H” + R A R—H + X"
eBo

(From L1AIH4) Alkyl halide Alkane

1® and 2® alkyl halides are readily reduced to alkanes by LiAlH4 but 3® alkyl halides mainly undergo
dehydrohalogenation to form alkenes. On the other hand, NaBH4 reduces 2® and 3® alkyl halides but not 1®
whereas triphenyltin hydride (Ph3SnH) reduces all the three types of alkyl halides. The order of reactivity,
ad
our

however, decreases in the order : 1® > 2® > 3®.


(c) Catalytic hydrogenolysis implies cleavage of a sigma bond with H2 in presence of a catalyst. The
best catalyst is Pd—C but Raney Ni has also been effective provided it is used in large excess.
Pd-C
Re
Y

CH3CH2—Br + H2 ^ CH3—CH3 + HBr


Bromoethane Ethane
d
Fin

id) Reduction with HI and red R Alkyl iodides are readily reduced to the corresponding alkanes by
heating with cone. HI in presence of red P at 423 K.
423 K/red P
CH3CH2—I + HI ^ CH3—CH3 + I2
lodoethane Ethane

The purpose of red phosphorus is to remove the iodine liberated in the reaction, otherwise it would react
with the alkane to give back the alkyl iodide.
2 P + 3 I2 ^ 2PI3
It may be noted here that the alkane formed by this method has the same number of carbon atoms as the
alkyl halide.
13/10 New Course Chemistry (XI)ES&I9D

3. From carboxylic acids. Alkanes can be prepared from carboxylic acids by the following two methods :
(0 Decarboxylation, and {i7) Kolbe’s electrolytic method
(i) Decarboxylation.

The process of removal of a molecule of CO2 from an organic compound is called


decarboxylation.

N\hen a carboxylic acid is heated with soda-lime (NaOH -t- CaO in the ratio 3 : I) at about 630 K, a
molecule of CO2 is lost and an alkane with one carbon atom less than the carboxylic acid is formed.
R—COOH -I- NaOH ^ R—COONa -I- H2O
Carboxylic acid
CaO, 630K
R—[ CO(^a_ + 1^0] H > R—H -H Na2C03
Alkane

F low
e.g., CH3COOH + NaOH CH3COONa -H H2O
Ethanoic acid Sod. ethanoate

(Aceiic acid)
CaO. 630K
CH3 ~ r^o] H » CH4 -I- Na2C03
Sod. acetate Methane

CaO

for Fre
Similarly, CH3CH2COOH + 2 NaOH ^ CH3—CH3-f Na2C03-h H2O
630 K
Propanoic acid Ethane

(Propionic acid)
NaOH alone could have been used in the above reaction but soda-lime is preferred because of the
following two reasons:
(/) CaO permits the reaction to be carried out at a relatively higher temperature to ensure complete
decarboxylation.
our
eBo ks

(«) CaO keeps NaOH dry because it is quite hygroscopic (absorbs moisture from air) in nature.
Y

In the laboratory, methane is prepared by heating a mixture of sodium acetate and soda-lime (in the ratio
of 1 :4) in a copper tube (Fig. 13.2) and the methane produced is collected by the downward displacement of
ad

water.
our

FIGURE 13.2

SODIUM ACETATE
SODA-LIME COPPER TUBE
Re

METHANE
Find Y

●9.-

■<Z-

Laboratory preparation of methane

(ii) Kolbe’s electrolytic method. When a concentrated aqueous solution of the sodium or potassium
salt of a mono carboxylic acid is electrolysed, an alkane is produced as shown below :
HYDROCARBONS 13/11

Electrolysis
2RCOONa + 2H2O > R—R + 2 NaOH + + 2 COj
Sod. carboxylate Alkane

Thus, if n is the number of carbon atoms in the salt of the carboxylic acid, the alkane formed has
2(n-l) carbon atoms. Alternatively, if n is the number of carbon atoms in the alkyl group, the alkane formed
has 2 n carbon atoms. For example.
Electrolysis
2CH3COONa + 2H20 > CH3-CH3 + 2 NaOH + H2 + 2 CO2

ow
Sod. etbanoate Ethane

This reaction is believed to occur through the following steps :


Ionization
2 CH3COONa > 2 CH3COO- + 2 Na+
Ionization

2H2O ^ 2 0H- + 2H+

e
Fl
Atanode : 2CH3COO--2e- ^ [2CH3COO] - ^ CH3—CH3 + 2CO2

re
(unstable) Ethane

F
At cathode : Since the electrode potential of H'*’ ions is higher than that of Na'*’ ions, therefore, H'*’ ions
are preferentially discharged to produce H2 while Na‘‘' ions remain in solution. Thus,
ur
2 H+ + 2 e- ^ [2H] ^ H2

or
Kolbe’s reaction laid the foundation for the development of organoelectrochemistry.

sf
Limitation. Like Wurtz reaction, this is also used to prepare only alkanes with even number of carbon
atoms and not alkanes with odd number of carbon atoms. k
Yo
4. Reduction of alcohols, aldehydes, ketones and carboxylic acids. Reduction of alcohols, aldehydes,
oo
ketones and carboxylic acids with HI and red P at 423-523 K gives the corresponding alkanes.
423 K
B

R-OH +2HI
RedP ^ R-H + I2 + HoO
Alcohol Alkane
re

O 423 K
R-C
H
+ 4HI
RedP ^ R-CH3 +2I2 + H2O
Alkane
u

Aldehyde
ad
Yo

o
423 K
R-C-R' + 4HI
RedP ^ R-CH2-R' +2I2 + H2O
Ketone
d

Alkane
Re
in

o
473-523K
R-C-OH + 6HI » R-CH3 + 3 I2 + 2 H2O
F

RedP
Carboxylic acid Alkane

5. By the action of water on beryllium and aluminium carbide. Both these carbides on treatment
with water yield methane.
Be2C + 4H2O ^ CH4 + 2 Be(OH>2
Beryllium carbide Methane

AI4C3 + I2H2O ^ 3 CH4 + 4 A1(0H)3


Aluminium carbide Methane
13/12 New Course Chemistry (XI)sasiMD

13.3.5. Physical Properties of Alkanes


The physical properties of alkanes such as boiling point, melting point, density and solubility depend
upon the intermolecularforces of attraction. Since alkanes are almost non-polar molecules (because of the
covalent nature of C—C and C—H bonds and very little difference of electronegativity between carbon and
hydrogen atoms), therefore, these intennolecular forces of attraction are of van der Waals type which mainly
depend upon the shape and hence the structure of the molecule. The magnitude of these forces of attraction
depend upon the surface area of contact between adjacent molecules. Greater the surface area, stronger are
the van der Waals forces of attraction. In the light of these arguments, let us now discuss the variation in
physical properties of alkanes.
1. Boiling points. Amongst the straight chain alkanes, the first four members (C]-C4) are gases, the

w
next thirteen are liquids (C5-CC7) and the higher members (Cjg onwards) are colourless waxy solids.
The boiling points of straight chain alkanes increase fairly regularly with increase in their molecular
mass. On the average, the boiling point generally increases by 20-30K for the addition of each carbon atom

F lo
or a CH2 group to the chain. The difference in boiling points is, however, greater for the lower homologues
than for the higher homologues.
This regular increase in the boiling points of straight chain alkanes with increase in the carbon content
(Fig. 13.3) is due to a corresponding increase in the molecular size and hence the surface area of the molecules.

ee
As a result, the magnitude of the van der Waals forces of attraction goes up and hence the boiling point

Fr
increases accordingly.
Amongst isomeric alkanes, the branched chain isomer has invariably the lower boiling point than the

for
corresponding n-alkane. This is due to the reason that with branching the shape of the molecule tends to
approach that of a sphere. As a result, the surface area of the branched isomer decreases. Due to lesser
ur
surface area of these molecules, the van der Wools forces of attraction operating between their molecules
become comparatively weaker and hence lesser amount of energy is required to overcome them. As a result,
the boiling points of branched chain isomers are lower than those of the corresponding n-alkaries.
s
ook
Yo
CH3CH2CH2CH2CH3 or
A/\ FIGURE 13.3

H-Peniane (no branching)


eB

600-
(b.p. = 309-1 K, m.p. = 143-3 K)

CH3 500-
our

CH3—CH—CH2CH3
ad

or
400-

Isopeniane (one branch) o


Q.
300-
(&./J. =301 K.m.p. = 113-1 K) O
z

CH3
dY

O 200*
Re

CO

CH3—C—CH3 or
100
80 0
Fin

CH3 4 8 12 16 20
=0
Neopentane (nvo branches) NUMBER OF CARBON ATOMS PER MOLECULE
{b.p. = 282.5 K, m.p. = 256-4 K)
Increase in the boiling points of
Further, it has been observed that greater the n-alkanes with the increase in the number
branching, lower is the boiling point of the branched of carbon atoms per molecule of the
isomer. For example, the boiling point of 2, 2- homologous series
dimethylpropane (neopentane, 282.5 K) with two
branches is lower than those of 2-methylbutane (isopentane, 301 K) with one branch chain and «-pentane
(309-1 K) with no branch chain.
2. Melting points. Like boiling points, the melting points of alkanes also increase with increase in
carbon content but the variation is not regular (Table 13.1).
HYDROCARBONS 13/13

TABLE 13.1. Melting points of some n-alkanes

Alkane C3H8 C4H10 C5H.2 C6H14 C7H,6 ^8^18 CgH20 C10H22


m.p. (K) 85-3 134-6 143-3 178-5 182-4 216-2 222-0 243-3

When the melting points of n-alkanes are plotted against the number of carbon atoms present in them, a
sawtooth pattern (Fig. 13.4) is obtained.
From Table 13.1 and Fig. 13.4, it
is clear that alkanes with even number FIGURE 13.4

w
of carbon atoms have higher melting
340-
points than those with odd number of
carbon atoms. This property is 320-

commonly known as alternation effect A 300-

F lo
and can be explained as follows : 280-
The melting point of a substance 260-
depends not only upon the size and shape 240-

ee
of the molecules but also upon how to
220-
closely the molecules are packed in the

Fr
crystal lattice. But due to sp^- O
Q.
200-

hybridization in alkanes, any two bonds o 180-


of a carbon atom make an angle of 109°-

for
!li 160-
28' with one another. As a result, in n- UJ
ur
140-
alkanes, the carbon atoms are arranged
in a zig-zag chain rather than in a straight 120-

chain as commonly written. Therefore, 100


s
in «-alkanes, containing an even number 80 I I 1 1 1 1 T
ook
Yo
●^ 5 7 9 11 13 15 17 19 21 23
of carbon atoms, the two terminal methyl
NUMBER OF CARBON ATOMS PER MOLECULE=t>
groups lie on the opposite sides of the
eB

zig-zag chain. On the other hand, in case Increase in the melting points of n-alkanes with the increase
of «-alkanes having odd number of in the number of carbon atoms per molecule of the
carbon atoms, the two terminal methyl homologous series
groups will lie on the same side of the
r

zig-zag chain as shown below ;


ad
ou

CH3
CH3 CH2 CH2 CH3 CH2 CH3
Y

«-Hexane «-Pentane
{Even number of carbon atoms, higher melting point) {Odd number of carbon atoms, lower melting point)
Re
nd

Thus, it is clear that n-alkanes with even number of carbon atoms are more symmetrical than those
containing odd number of carbon atoms and hence pack closely in the crystal lattice. In other words, van der
Fi

Waals forces of attraction are much stronger in n-alkanes having even number of carbon atoms than those
having odd number of carbon atoms. Therefore, n-alkanes with even number of carbon atoms have much
higher melting points than the next lower n-alkane with odd number of carbon atoms. For example, n-butane
with four (even) number of carbon atoms melts at 138 K while propane with three (odd) number of carbon
atoms melts at 85.3 K. Similarly, n- hexane melts at 178.5 K while the next lower alkane, i.e., n-pentane melts
at 143.3 K.

It may be noted that such an alternation effect is not observed in boiling points since in the liquid state,
the conformations of the molecules are not fixed but keep on changing as a result of collisions.
3. Solubility. ‘Like dissolves like’ is the general rule of solubility. In the light of this mle, alkanes which
are predominantly non-polar are insoluble in polar solvents such as wate,r alcohol, etc. but are highly
soluble in non-polar solvents such as petroleum ethe,r benzene, carbon tetrachloride, etc.
13/14 T^'utcCee^’A New Course Chemistry (XI)BZ2[HD

It may be noted here that greases are mixtures of higher alkanes and hence are non-polar and
hydrophobic (water repelling) in nature. Similarly, petrol is a mixture of lower alkanes. Thus, both petrol and
greases are non-polar. It is because of this reason that petrol is used for dry cleaning of clothes to remove
grease stains.
4. Density. The densities of alkanes increa.se with increase in the molecular masses till the limiting
value of about 0.8 g cm“^ is reached. This means that all alkanes are lighier than water.

ow
13.3.6. Chemical Reactions of Alkanes

The reactivities of various hydrocarbons are directly related to their structures. We have already discussed
in Unit 12 that alkanes are saturated hydrocarbons. These contain only C—C and C—H, a-bonds. Since
these bonds are quite strong, alkanes are the least reactive of all the hydrocarbons. It is because of this
relative inertness that alkanes are also called paraffins (Latin : parum = little, affmis - affinity or reactivity).

e
Some important chemical reactions of alkanes are discussed below :

re
1. Substitution reactions

rFl
A reaction in which a hydrogen atom of a hydrocarbon is replaced by an atom or a group of

F
atoms is called a substitution reaction.

Alkanes, because of having only C—C and C—H sigma bonds undergo only substitution reactions.
Some important substitution reactions of alkanes are discussed below.

or
ou
1. Halogcnation of Alkanes. Halogenation of an alkane is carried out by treating it with a suitable
halogen in presence of ultraviolet light or by heating the reaction mixture to 520-670 K.

ksf
The order of reactivity of different halogens in these reactions is :
p2 > CI2 > Br2 > I2
(/) Chlorination. During chlorination of methane, all the four hydrogen atoms are replaced one by one
oo
to form a mixture of products. For example,
Y
/ivor
CH4 + CI2 CH3CI -h HC!
B

520-670K
Methane Chloromethane

(Methyl chloride)
re

hv or
CH3CI + CI2 520-670 K
CH2CI2 + HCl
oYu

Chloromethane Dichloromeihane
ad

(Methylene chloride)
hv or
CH2CI2 + CI2 520-670 K
CHCI3 HCl
d

Dichloromeihane Trichioromethane

(Chloroform)
in
Re

hvor
CHCI3 -I- CI2 520-670K
CCI4 + HCl
Trichioromethane Tetrachloromclhane
F

(Carbon tetrachloride)

(ii) Bromination. Bromine reacts with alkanes in a similar manner but less readily.
hVOT
CH3—CH3 Br2 520-670K
» CH3—CH2—Br + HBr
Ethane Bromoelhane

(/i7) lodination. The reaction of iodine with alkanes is reversible because the hydrogen iodide formed
as a by-product is a moderate reducing agent and hence reduces the iodoalkane back to alkane.
CH4 + I2 CH3—I -I- HI
lodomethane
HYDROCARBONS 13/15

Thus, direct iodination of alkanes cannot be brought about. However, the iodination can be carried out
in presence of an oxidising agent such as iodic acid (HIO3) or nitric and (HNO3) which oxidises HI to I2 or
in presence of mercuric oxide (HgO) which eliminates HI as insoluble Hgl2-
5 HI + HIO3 ^ 3 I2 + 3 H2O ; 2 HNO3 + 2 HI ^ 2 H2O + 2 NO2 + I2
HgO + 2HI ^ Hgl2 + H2O
(iv) Fluorination of alkanes is too vigorous to be controlled under ordinary conditions. Furthermore,
fluorination brings about extensive rupture of C—C and C—H bonds leading to a mixture of products. Thus,
fluorination of alkanes with pure fluorine is of little practical use. However, fluorination of alkanes can be
carried out by diluting fluorine with an inert gas such as nitrogen or argon. Alternatively, alkyl fluorides are
more conveniently prepared indirectly by heating suitable chloroalkanes with inorganic lluorides such as

w
ASF3, SbF3, AgF, Hg2F2, etc. For example,
2CH3CH2—Cl + Hg2p2 ^ 2CH3CH2—F + Hg2Cl2
Chloroelhane Mercurou.s fluoride Fluoroethane Mercurous chloride

F lo
This reaction is called Swarts reaction.

SUPPLEMENT YOUR
KNOWLEDGE FOR COMPETITIONS

ee
Even monohalogenation of higher alkanes (propane and higher members) gives a mixture of all the possible
isomeric haloalkanes. For example,

Fr
Cl2,/jv
CH3—CH2—CH3 298 K
CH3—CH2—CH2—Cl + CH3—CHCl—CH3

for
Propane 1-Chloropropane (45%) 2-Chloropropane (55%)
The relative amounts of these isomeric haloalkanes, however, depends upon
ur
(0 nature of halogen (CI2 or Br2) and («) number and type of hydrogens (1 ®, 2° or 3°) being substituted.
In general, the ease of substitution of various hydrogens follows the sequence : 3® > 2® > 1® but their
s
relative rates vary with the nature of the halogen. For example, with CI2, the relative rate of substitution of
ook
Yo
3°, 2® and 1“ hydrogens at 298 K is 5 : 3*8 : 1 while with Br2, it is 1600 : 82 : 1 at 400 K.

Mechanism of halogenatlon. Halogenation of alkanes occurs by a free radical mechanism. Each free
eB

radical reaction consists of three steps :


Let US illustrate the mechanism of halogenation of alkanes by considering the chlorination of methane,
(a) Chain initiation. When a mixture of CH4 and CL is heated to 520-670 K in dark or is subjected to UV
our
ad

light at room temperature, CL absorbs eneigy and undergoes homolytic fission producing chlorine free radicals.

CI-“Cl 520-670 K or UV light ^ 2 Cl


u* Homolytic fission Chlorine free radical
Y

(b) Chain propagation. Each propagation step consists of two reactions. In the first reaction, the ● Cl attacks
Re

the CH4 molecule and abstracts a hydrogen atom forming ● CH3 and a molecule of HCl as shown in reacdon (/). In
nd

the second reaction, ● CH3 thus produced reacts further with a molecule of CL forming a molecule of metliyl
Fi

chloride and another ● Cl as shown in reaction (//). The newly formed ● Cl reacts with another molecule of CH4
(reacdon (i)) to produce another molecule of HCl and another ● CH3. This ● CH3 can again repeat reacdon (ii) and
so on. Thus, the sequence of reacdons depicted in equations (/) and (ii) is repeated over and over again and the
chain gets propagated. In other words, a single photon of light absorbed by CI2 can bring about the conversion of
a large number of CH4 molecules into CH3CI. Such reacdons are called chain reactions.
H H

(0 H—C-^H
In + Cl H—C* F HCl
Chlorine
free radical
H H
Methyl free radical
13/16 7heiciee^'<i, New Course Chemistry (XI)BE

H H

pr I
(//) H—C- + Cl-i-Cl ► H—C—Cl +-C1

H H
Methyl chloride

When sufficient amount of methyl chloride has been formed , the *C1 produced in reaction (ii) has a
greater chance of colliding with a molecule of CH3CI rather than a molecule of CH4. If such a collision
occurs, a new free radical (*€11201) is produced (reaction (Hi)) which may subsequently react with CI2 producing
a molecule of CH2CI2 (reaction (iv)) and another ● Cl. This process continues till ^ the hydrogen atoms of
methane are replaced by halogen atoms (reactions (v), (vi), (v«) and (yiii)).

w
(/●«) CH3CI + ● Cl > ● CH2CI + HCl (/v)CH2Cl + Cl2 CH2CI2+ CI
(v) CH2CI2 + ● Cl > . CHCI2 + HCl (v/)CHCl2 + Cl2 CHCI3+ CI

F lo
(v«) CHCI3 + ● Cl > ● CCI3 + HCl (yiii) ● CCI3 + CI2 ● ^ CCI4+ CI
(c) Chain termination. The chain reactions discussed above may come to a halt if two of the same or
different free radicals combine amongst themselves without producing new free radicals. Some of the possible
chain termination steps are :

ee
C1 + -C1 ^Cl—Cl

Fr
●CH3+CH3
^ CH3—CH3
●CH3+CI
^ CH3—Cl

for
The above mechanism helps us to understand the reason for formation of ethane as a by product during
chlorination of methane.
ur
2. Nitration. The process of replacement of a hydrogen atom by a nitro (-NOf) group is called
nitration. At ordinary temperatures, alkanes do not react with HNO3. However, when a mixture of an
alkane and fuming HNO3 vapours are heated at 423-673K under pressure (vapour phase nitration)^ alkanes
s
ook
undergo nitration giving a mixture of nitroalkanes resulting through cleavage of carbon - carbon bonds. For
Yo
example.
eB

CH3—H + HNO3 - CH3—NO2 + H2O


Methane (Fuming) Nitromethane (low yield)
Fuming HNO3
CH3—CH3 > CH3CH2—NO2 + CH3—NO2
our

673 K
ad

Ethane Nitroethane (80%) Nitromethane (20%)

NO2
I
■ CH3CH2CH2NO2 + CH3 CH—CH3 + CH3CH2—NO2
dY

Fuming HNO3
CH3CH2CH3 1-Nitropropane (25%) 2-Nitropropane (40%) Nitroethane (10%)
Re

673 K
Propane + CH3—^N02 + Other oxidation products (CO2, NO2, H2O, etc.)
Fin

Nitromethane (25%)
The order of reactivity of different hydrogens in this reaction is : 3® > 2° > 1®.
Mechanism. The reaction occurs by the following radical mechanism :
423-673K
HO—NO2 > HO + NO,^
Homolytic fissiont
Nitric acid

R—H + OH 4 R +H2O ; R- + NO2 4 R—NO2


Alkane Nitroalkane

3. Sulphonation. Substitution of a hydrogen atom of an alkane by sulphonic acid group (SO^H) is


called sulphonation. It is carried out by heating an alkane with fuming sulphuric acid (H2SO4 + SO3) at

k
HYDROCARBONS 13/17

675-725 K. Branched chain and higher normal alkanes (containing six or more carbon atoms) undergo
sulphonation to give alkanesulphonic acids. The ease of substitution of hydrogens is 3® > 2® > T.
SO3
CH3(CH2)4CH3 + H2SO4 675 K
> CH3(CH2)4CH2—SO3H + H2O
n-Hexane n-Hexanesulphonic acid
'CH,
2I SO3 2I
CH3—C—H + H2SO4 675 K CH3—C—SO3H + H2O
3CH3 3CH3
2-Methylpropane 2-Methylpropane-2-sulphonic acid
n. Oxidation. Some important oxidation reactions of alkanes are as follows :

w
ia) Complete oxidation or combustion. On heating, alkanes readily bum in air or oxygen producing
CO2 and H2O. This process is called combustion.

F lo
CH4 (g) + 2 O2 (g) > CO2 (g) + 2 H2O (0 ; H® = - 890 kJ mol"!
C4H10 (g) + 13/2 O2 > 4 CO2 (g) + 5 H2O (/); H® = - 2875-84 kJ mol"^
The general combustion equation for any alkane is

ree
r3n + l
^n^2«+2 02 ^ n CO2 + (n+ 1) H2O
I 2

F
Since the process of combustion is accompanied by liberation of large amount of heat, therefore, alkanes

for
which are the constituents of LPG (butane and isobutane), gasoline, kerosene oil, and diesel are widely used
as fuels.

(b) Incomplete combustion, (i) If the combustion of alkanes is carried out in a limited supply of air or
r
oxygen, CO is produced along with unbumt carbon in the form of carbon black or soot.
You
ks

2CH4-b 3O2 ^ 2CO +4H2O ; CH4 + O2 ■» C +2 H2O


o
eBo

{limited) {limited) Carbon black

Carbon black is used in the preparation of black inks, paints, polishes, etc.
(//) Reaction with steam. Methane reacts with steam at 1273 K in presence of nickel as catalyst forming
ad
our

a mixture of CO and H2, also called syngas.


1273K.N1
CH4 + H2O > CO + 3 H2
{Steam)
Syngas
Re

This method is used for industrial preparation of dihydrogen,


dY

(c) Catalytic oxidation. Different products are formed under different conditions.
(/) When a mixture of methane and oxygen (9 : 1 by volume) at a pressure of 100 atmospheres is passed
Fin

through a copper tube at 573 K, methanol is formed.


100 atm, 573 K
2 CH4 -I- O2 Cu tube
> 2 CH3OH
Methane Methanol

{ii) When a mixture of methane and oxygen under pressure is passed over heated molybdenum oxide, it
is oxidised to methanal.
M02O3
CH4 + O2 HCHO + H2O
A, pressure
Methane Methanal

i
13/18 '4^ New Course Chemistry (XI) BZ

(m) Higher alkanes on oxidation in presence of silver oxide give carboxylic acids.

2R—CH3 + 3O2 ■> 2 RCOOH + 2 HjO


Alkane Carboxylic acid

(/v) In general, oxidising agents such as KMn04, K2Cr20y, etc. have no effect on alkanes. However,
alkanes containing a tertiary hydrogen can be oxidised to the corresponding alcohols.
CH
CH,
p-3 Aik. I ^
CH3—C—H + [O] > CH3—C—OH
KMn04
CH3 . C.
CH3
2-Methylpropane 2- Methylpropan-2-ol

w
{Isobutane) (/er/-Butyl alcohol)
nL Isomerization. When n-alkanes are heated with anhydrous aluminium chloride and hydrogen chloride at

F lo
573 K under a pressure of about 35 atmospheres, they are converted into branched chain alkanes. For example,
CH,
AICI3/HCI I ’
(/) CH3—CH2—CH2—CH3 573 K > CH3—CH—CH3

e
n-Butane Isobutane

Fre
CH3 CH3
AICI3/HCI I
(lO CH3—(CH2)4—CH3 > CH3—CH—CH2CH2CH3 + CH3CH2—CH—CH2CH3

for
n-Hexane 2-Methylpentane 3-Methylpentane
The process of isomerization has been of great utility for increasing the octane number of a particular
petroleum fraction.
our
rV. Aromatization. Alkanes containing six to eight carbon atoms when heated to about 773 K under 10-
oks
20 atm pressure in the presence of a catalyst consisting of oxides of chromium, vanadium and molybdenum
supported over alumina, get converted into aromatic hydrocarbons. This process which involves cyclization,
eBo

isomerization and dehydrogenation is called aromatization. For example.


Y

CH3
CH2 CH3 Cr203, V2O5, M02O3/AI2O3 Dehydrogenation
I I
ad

>
our

773 K, 10-20 atm, (-3H2)


CH2 CH2 Cyclization (-H2)
_ Cyclohexane Benzene

n-Hexane

Under similar conditions, n-heptane gives toluene while n-octane gives a mixture of o-, m- and p-xylenes.
Re
dY

CH3
Fin

CH2 CH3 CH3

CH2 CH3 Cf203, V2O5, M02O3/AI2O3


A Dehydrogenation
I I 773 K, 10-20 atm, ^
>
(-3 H2)
CH2 CH2 Cyclization (-H2)

\ch/
_ Methylcyclohexane Toluene

n-Heptane
CH3-(CH2)6-CH3 C6H4(CH3>2
773 K, 10-20atm, cyclization
n-Octane Mixture of o- m- and j?-xylenes
and dehydrogenation (-4H2)
V. Pyrolysis. Decomposition of a compound by application of heat is called pyrolysis.

i
HYDROCARBONS 13/19

Pyrolysis of higher alkanes to give a mixture of lower alkanes, alkenes, etc. is called cracking. It is
usually carried out by heating higher alkanes to high temperatures (773-973 K) under a pressure of 6-7
atmospheres in presence or absence of a catalyst. For example,
773 K
C6Hi4 6-7 atm C6H,2 + Hj + C,Hg + + CaHg + C2H^+CH4
Hexane
Hexene Hydrogen Butene Ethane Propene Ethene Methane
Pyrolysis of alkanes involves breaking of C—C and C—H bonds and occurs by a free radical mechanism.
Preparation of oil gas from kerosene oil and petrol gas from petrol is based upon the process of pyudysis.
For example, dodecane, a constituent of kerosene oil, on heating to 973 K in the presence of Pt, Pd or Ni,
gives a mixture of heptane and pentene along with other products.

w
Pt or Pd or Ni
C,2H26 ^ + CsH.o + Other products
973 K
Dodecane Heptane Pentene

F lo
13.3.7. Uses of Alkanes

(i) Methane in form of natural gas is used for running scooters, cars, buses, etc. LPG (mixture of butane
and isobutane) is used as a fuel in homes as well as in industry.

ee
(//) Methane is used to make carbon black which is used in the manufacture of printing inks, paints and

Fr
automobile tyres.
(Hi) Catalytic oxidation of alkanes gives alcohols, aldehydes and carboxylic acids.
(tV) Higher alkanes in form of gasoline, kerosene oil, diesel, lubricating oils and paraffin wax are widely used,

for
(v) Methane is used for the manufacture halogen containing compounds such as CH2CI2, CHCI3, CCI4,
ur
etc. which are used as solvents both in laboratory and industry.
13.4. STEREOISOMERISM
s
Isomers which have the same structural formula but have different relative arrangement of atoms or
ook
groups in space are called stereoisomers (Greek : stereo = space, meros = part) and the phenomenon is
Yo
called stereoisomerism.

Since each different spatial arrangement of atoms which characterises a particular stereoisomer is called its
eB

configuration, therefore, stereoisomers have the same molecular structure but dijferent configurations.
Stereoisomerism is of the following three types :
(0 Conformational isomerism {ii) Optical isomerism and (Hi) Geometrical isomerism.
r

Let us now discuss conformational isomerism in alkanes.


ad
ou

13.5. CONFORMATIONS OF ALKANES

It has already been stated in Unit 5 that a sigma FtGURE 13.5


Y

(or single covalent) bond between two carbon atoms


is formed by overlap of i'p^-hybrid orbitals of each
Re
nd

carbon along their intemuclear axis. Therefore, the


electron distribution within the molecular orbital
Fi

(MO) thus formed is cylindrically .symmetrical along A cylindrically symmetric MO (of a


the intemuclear axis as shown in Fig. 13.5. Due to single bond) obtained by overlapping of
this cylindrical symmetry of g-MO’s, rotation about sp^-hybrid orbitals of two carbon atoms
carbon-carbon single bond is almost free (as it
requires very little energy for rotation). As a result of this almost free rotation, the molecules of an alkane can
have different shapes, i.e., different relative arrangements of their atoms in space.
The infinite number of momentary arrangements of the atoms in space which result through
rotation about a single bond are called conformations or rotational isomers or simply rotamers.
The field of conformation analysis was developed by the brilliant research carried out by Derek H
Barton and Odd Hassel.

Let us now apply the concept of free rotation to ethane molecule and discuss its conformations.
13/20 New Course Chemistry (XI)CEIHD

13.5.1. Conformations of Ethane

In etiiane (CHJ-CH3) molecule, the two carbon atoms are connected by single covalent bond (i.e.,
a-bond) while the remaining three valencies of each carbon are satisfied by hydrogen atoms. If one of the
methyl gioups in ethane molecule is kept Fixed and the otlicr is rotated about the C-C bond, a large number of
momentary amingements of the hydrogen atoms on one carbon atom with respect to the hydrogen atoms on
the other carbon atom in space are obtained. These infinite number of momentary arrangements of atoms in
space represent conformations of ethane. In all these conformations, the basic structure of ethane molecule
and V nous bond angles and bond lengths remain the same.

w
< *ut ot the infinite number of possible confonnations of ethane, only two extreme conformations, i.e.,
staggered and eclipsed are important. All other conformations lying in between these two conformations are
known as gauche or skew conformations.
Two-dimensional or 2-D-representation of Conformations. We have already discussed that three-

lo
dimensional structures can be represented on a two-dimensional (/>., 2-D) .surface with the help of Fischer
Projections. One serious drawback of these projection formulae is that they represent the molecule in the

e
unfavourable eclipsed conformation. However, while discussing the reactions of a molecule, it is usually

re
desirable to depict the molecule in its actual staggered form rather than in the hypothetical eclipsed form as

rF
shown in the Fischer projection. Therefore, to overcome this difficulty, the conformations of a molecule are

F
usually represented by the following two methods :
(0 Sawhorse Formulae. This is a simple
method of representing three dimensional formulae FIGURE 13.6

r
on paper. The molecule is viewed slightly from above H H

fo
and from the right and projected on the paper. The
ou
bond between the two carbon atoms is drawn
diagonally and is slightly elongated for clarity. The
lower left hand carbon is considered to be towards
the front and the upper right hand carbon towards the
ks
oo
back. The Sawhorse representation for staggered and
eclipsed conformations of ethane are shown in Fig.
Y
eB

13.6.

(») Newman Projection Formulae. Newman H


devised a simple and highly useful method of STAGGERED ECLIPSED
representing three dimensional formulae on paper.
ur

After his name, these are called Newman Pro


Sawhorse representation for Sta^ered and Eclipsed
jections. These projection formulae are obtained by
ad

confonnations of ethane
Yo

viewing the molecule along the bond joining the two


carbon atoms. The carbon atom near the eye is
represented by a point
and the three atoms or FIGURE 13.7
d

groups attached to it by Angle of rotation or


Re
in

three equally spaced torsional angle or


H
H H
(120°) radi i. The carbon dihedral angle
F

atom farther from the

/
.H H
H H
eye is designaU-d by a
circle and the three
atoms or groups
H-
attached to it by three
H
equally spaced radial H H H H H
extensions. The
Newman projections H

for staggered, eclipsed STAGGERED ECLIPSED SKEW


and skew confor
mations of ethane are Newman projection formulae for Staggered, Eclipsed and Skew
conformations of ethane
shown in Fig. 13.7.
HYDROCARBONS 13/21

Relative stability of Staggered and Eclipsed conformations of Ethane. In staggered conformation,


each of the hydrogen atoms on the front carbon lies exactly in between each of the hydrogen atoms on the
back carbon. In other words, in this conformation, any two hydrogen atoms on adjacent carbon atoms (non-
bonded hydrogens) are as far apart as possible (intemuclear H to H distance = 31 A or 310 pm). As a result,
the repulsions between the electron clouds of a-bonds of two non-bonded hydrogen atoms is minimum. On
the other hand, in eclipsed conformation, each of the hydrogen atoms on the back carbon lies exactly behind
each of the hydrogen atoms on the front carbon. In other words, the non-bonded hydrogen atoms are quite
close (2-29 A or 229 pm). As a result, the electron clouds of the o-bonds of two non-bonded hydrogen atoms
repel each other. This raises the energy of the eclipsed conformation relative to staggered conformation. This
repulsive interaction between the electron clouds which affects the stability of a conformation is called
torsional strain. The magnitude of the torsional strain, however, depends upon the angle of rotation about

w
C-C bond. This angle is also called dihedral angle or torsional angle.
The variation of energy of FIGURE 13.8
the conformations of ethane with

F lo
H
H
rotation about the C-C single Eclipsed
bond is shown in Fig. 13.8.
A
Eclipsed H
^J^(less stable) Eclipsed Eclipsed
From the Fig. 13.8, it is

ee
evident that of all the confor >
CD

Fr
mations of ethane, staggered a:
kJ mol
lU
conformation has the minimum z
LU 0 . _ --t--
torsional strain and the eclipsed TT
Staggered
<

for
H
form, the maximum torsional I-

strain. Experimentally, it has


our
LiJ
H H
Staggered
been found that the staggered O
0.
H {more stable)
conformation of ethane is about T
o 0

12-55 kJ mol"^ more stable than 0 60 120 180


s 240 300 360
ook
ANGLE OF ROTATION t>
the eclipsed conformation. Thus,
it may be inferred that rotation Variation of energy during rotation about C—C single bond
about C-C single bond is not
Y
in ethane molecule
eB

completely free.
This energy difference of 12-55 kJ mol“* between the staggered and eclipsed confonmations is, in fact, the
energy barrier to rotation about the C-C single bond in ethane. However, this energy barrier is not large enough
r

to prevent rotation. Even at room temperature, the collisions of the molecules supply sulFicient kinetic energy to
ad
ou

overcome this energy barrier. Thus, the two confonnations are readily interconvertible. A.s' a result, it is not possible
to separate the two conformations of ethane. Howeve,r at any given moment, most of the etltane molecules would
exist in the staggeredconformationdue to its minimumenergy and maximumstability.
Y

PART—II. ALKENES
Re
nd

13.6. ALKENES
Acyclic unsaturated hydrocarbons containing a carbon-carbon double bond are called alkenes. They
Fi

are also called olefins (Greek : oleifant = oil forming) since the lower members of this class such as ethene
(ethylene), propene (propylene), etc. produce oily products on FIGURE 13.9
reaction with halogens such as chlorine and bromine.
H
sp2-s, C-H
fT-bond
13.6.1. Structures of Double Bond
The carbon-carbon double bond in alkenes consists of one

strong carbon-carbon a-bond with a bond dissociation energy


of 348 kJ mol”’ and a weak it-bond with a bond dissociation
sp2-sp2
energy of 251 kJ mol”’. The a-bond is formed by head-on H 'a
C —C,
overlapping of sp^- hybridized orbitals while the xc-bond is cr-bond
formed by lateral or sideways overlapping of the two 2;?-orbitals
Orbital picture of ethene showing
of the two carbon atoms. The orbital diagrams of ethene molecule
a-bonds only
are shown in Fig. 13.9 and 13.10.
13/22 ‘Pn.tsictceft- '4. New Course Chemistry (XI) LWll

FIGURE 13.10

p-p, 7i-bond
7t-cloud
H H
H ^ -H

C c:

H H H H

P-p. -110 pm
7t-bond ^ o o
Orbital picture of ethene showing formation of (a) ;i-bond (b) 7i-cloud and
" (c) bond angles and bond lengths

To have an effective overlap, the p-orbitals move little closer, therefore, a double bond is shorter

F low
(134 pm) than a single bond (154 pm). Further, due to poor overlapping between the two j7-orbitals, a rt-bond
is always weaker than a a-bond. In other words, presence of the it-bond makes alkenes behave as a source of
loosely held electrons. Therefore, alkenes are readily attacked by reagents or compounds which are in search
of electrons. Such reagents are called electrophiles or electrophilic reagents. Further, the presence of 7t-
bond makes alkenes less stable and more reactive than alkanes and hence readily change into compounds
containing single bonds by adding electrophilic reagents.
13.6.2. Nomenclature of Alkenes

re
for F
In the lUPAC system, they are called alkenes. Their general formula is C„H2„ where n = 2, 3, 4 ... etc.
The name of any individual alkene is obtained by replacing terminal ane of the corresponding alkane by
suffix ene. It may be noted that the first member of alkene series is : CH2 (putting n = 1 in C„H2„) known as
methylene in the common system and methene in the lUPAC system. It is highly unstable. Therefore, the first
stable member of alkene series is C2H4 known as ethylene in the common system and ethene in the lUPAC
system. In case of substituted alkenes, the numbering of the parent chain is done from that end which is near
Your
s

to the double bond or which gives lowest number to the carbon atom carrying the double bond. The lUPAC
eBo k

names of some simple and substituted alkenes are given below :


2 1 1 2 3 4

CH3 —CH = CH2 CH3—CH2—CH=CH2 CH3—CH =CH—CH3


ad

Propene But-l-ene But-2-ene


our

H3C CH3
1 2 3 1 2 3 4

CH^ =C—CH, CH2 =CH—CH—CH3 CR^—C = C—CR.


Re

2 I 3 4 3 2 1

CH3 CH3
Y

2-Meihylprop-1 -ene 3-Methylbut-l-ene 2, 3-Dimelhylbut-2-ene


Find

2 J

CH2CH3 H3CH2C CH3 CH, CH3


1 2 II 5 6 5 4I 3 2I 1
CH,CH,CH = C—CH, CH, CH, —C = C—CH, CH, CH,—C C = C —CH,
6 ■’S ^4 3 ^ 3 4

3-Meihylhex-3-ene 3-Ethyl-4-meihylhex-3-ene CH
'“^3| iCH—CH3I
II ^CH31 II
2,4,4-Trimethyl-3-(l- methylethyl)pent-2-ene
HYDROCARBONS 13/23

13.6.3. Isomerism of Alkenes

Alkenes show both structural isomerism and stereoisomerism, i.e., geometrical isomerism,
(a) Structural Isomerism. Ethene and propene can have only one structure but alkenes containing four
or more carbon atoms can show both position and chain isomerism. For example, the formula C4Hg can
represent the following three isomers :
1 2 3 4 1 2 3 4 1 2 3

CH2 =CH—CH2—CH3 CH3 ●CH =CH—CH3 CH2 =C—CH3


I II

But-l-ene But-2-ene CH3

w
III

2-MethyIprop-1 -ene
Where as structures I and II represent position isomers ; structures I and III, and II and HI represent
chain isomers,

F lo
(b) Geometrical Isomerism-Hindered rotation around carbon-carbon double bond
We know that a double bond consists of a a-
FIGURE 13.11

ee
bond and a Tt-bond. Tlie ic-bond is formed by sideways
overlapping of unhybridized /?-orbitals of two carbon

Fr
atoms above and below the plane of carbon atoms. If Rotate ^
now one of the carbon atoms of the double bond is
through 90°
rotated with respect to the other, the /7-orbitals will

for
no longer overlap and the 7t-bond should break (Fig. 7t-bond Overlap of p-orbitals
ur
13.11). But the breaking of a 7C-bond requires 251 kJ not possible because
mol“* of energy which is not provided by the they are perpendicular
to each other
collisions of the molecules at room temperature.
ks
Consequently, the rotation about a carbon-carbon Rotation about a carbon-carbon double
Yo
double bond is not free but is strongly hindered or bond is hindered
oo
restricted.

To further understand the concept of hindered rotation, let us take


eB

FIGURE 13.12
two pieces of strong cardboards and join them by one nail. Now try to
rotate one cardboard against the other, free rotation is possible. Now SSaiSSS,

join them by two nails and try to rotate, rotation is not possible. In Ha Ha
r

other words, rotation around double bond is hindered. 1 2


ou
ad

In a similar way, a 7t-bond prevents free rotation of the carbon Hb Hb


atoms of the double bond with respect to each other. Due to this hindered iSiSiiSlI

rotation, the relative positions of atoms or groups attached to the carbon Restricted rotation about carbon-
Y

atoms of the double bond get fixed. For example, and H/, in Fig. carbon double bond
13.12 cannot exchange their positions by rotation of Cj with respect to
nd
Re

C2 without breaking the 7i-bond.


As a result, many substitutedalkenes can exist in two distinct isomers which differ from each other only
Fi

in the relative positions of atoms or groups in space around the double bond. For example, but-2-ene can exist
in the following two forms (I and II):
CH H
CH3 CH3 3\
C = C
c = c;
H
H
I
H
II
CH3
c/j-But-2-ene (b.p. 277 K) /ra/i^-But-2-ene (b.p. 274 K)

Both these isomers have the same structural formulae but differ in the relative spatial arrangement of
hydrogen atoms and methyl groups around the double bond.
13/24 T^'uidee^'^ New Course Chemistry (XI)E9E

Such isomers which have the same structural formulae but differ in the relative spatial
arrangement of atoms or groups around the double bond are called geometrical isomers and
the phenomenon is called geometrical isomerism. Thus, geometrical isomerism is a type of
space or stereoisomerism. ;l

The isomer I, in which the similar atoms or groups lie on the same side of the double bond is called the
cis-isomer whereas the isomer II, in which the similar atoms or groups lie on the opposite sides of the double
bond is called the trans-isomer. It is because of this reason that geometrical isomerism is also called cis-
trans isomerism.

w
Properties of geometrical isomers
Due to different arrangement of atoms or groups in space, these isomers differ in their physical properties
such as m.p., b.p., dipole moment, solubility, etc.
(i) Melting points. In general, the melting point ofa tram- isomer is higher than that ofthe corresponding

Flo
cis-isome.r In the tram- isomer, the two similar atoms or groups lie on the opposite sides of double bond. Therefore,
these molecules are symmetrical and hence pack well in the crystal lattice. On the other hand, in the cis- isomer, the

e
two similar atoms or groups lie on the same side of the double bond. Therefore, these molecules are unsymmetrical

re
and hence do not pack well in the crystal lattice. As a result, intermolecular forces of attraction holding the molecules
of a trans-isomcT in the crystal lattice are much stronger than those holding the molecules of a c/s-isomer.

F
Consequentiy, the m.p. of a tranj-isomer is much higher than that of the corresponding cis-isomer. For example,
m.p. of cw-but-2-ene is lower than that of /rmts-but-2-ene.
ur
or
CH H
CH3 CH3 C = C
c = c
H
CH3

f
H H
I ks II

c«-But-2-ene (m.p. = 134 K) /ra/i5-But-2-ene (m.p. = 167 K)


Yo
Similarly, the m.p. of cij-l, 2-dichloroethene is lower than that of trans-l, 2-dichloroethene and that of
oo
maleic acid (c/j-isomer)is lower than that of fumaric acid (rrans-isomer).
B

Cl Cl Cl\ c=c H H COOH


:c = c c=c c = c
H H Cl HOOC COOH HOOC H
e

m-1, 2-Dichloroethene trans-l, 2-Dichloroethene Maleic acid (c/s-isomer) Fumaric acid (/ra/ts-isomer)
ur

(m.p. = 193 K) (m.p. = 223 K) (m.p. 403 K) (m.p. 575 K)


ad

(ii) Solubility. In general, the solubility of a cis- isomer is higher than that of the corresponding trans
Yo

isomer. This is due to the reason that the molecules of a cis- isomer being less symmetrical, are weakly held
in the crystal lattice than the corresponding more synunetrical trans-isomer. As a result, lesser amount of
energy is required to break the crystal lattice of the c/s-isomer and consequently the solubility of the cis-
d

isomer is higher as compared to that of the trans-isomer in the same solvent.


Re
in

COOH
c = c c = c
HOOC COOH HOOC H
F

Maleic acid Fumaric acid

(Solubility : 79 g UOO mL H2O) (Solubility: 0 7 g/WO mL H2O)


(iii) Dipole moments. In general, cis-isomer of an alkene is found to be more polar than the trans
isomer. For example, the dipole moment of cis-but-2-ene is 0-33 D while that of trans-b\it-2-ene is zero;
Thus, trans-but-2-ene can be said to be non-polar. This may be explained as follows :
5+ 5+ 5+
CH3 CH3 CH3 H
^C=c^
H H H'
c/j-But-2-ene (p = 0.33 D) lra«j-But-2-ene (p = 0)
HYDROCARBONS 13/25

In case of c/,y-bul-2-ene, the dipole moments of two C-CH3 bonds are inclined at an angle of 60" and
hence have a resultant equal to 0-33 D. However, in case of rra/?.v-but-2-ene, the two dipole moments being
equal and opposite cancel each other and, therefore, dipole moment of /rani-bui-2-ene is zero.
Similarly, cis-\, 2-dichloroethene has higher dipole moment than the corresponding /ra/i.v-isomer.
Cl Cl Cl H

^C=C'^ c=c
H H H Cl
cw-l, 2-Dichloroethene (n = 1.85 D) 2-Dicliloroelhcnc (u = 0)
It may, however, be noted here that a /raui-isomer need not necessarily have a zero dipole moment.
For example, trans- pent-2-ene has a finite dipole nu-iiicat though much smaller than that of the
corresponding c/s-isonier. The reason being that the dipole moments of C-CH3 and C-CH2CH3 bonds are

w
not equal. Although these two dipoles oppose each other, they do not exactly cancel out each other and hence
rrani-penl-2-ene has a small but finite dipole moment. Thus, /ran.s-penl-2-ene is much less polar than r/.v-
pent-2-ene.

F lo
CH3, CH2CH3 CH3 H

C=C c=c
H H H
CH2CH3

ee
c/.y-Pcnt-2-ene (more polar) //●o«.v-Penl-2-ene (less polar)

Fr
(iv) Boiling points. In general, the boiling points of the cis-isomers are higher than those of the
corresponding trans- isomers. The reason being that the boiling points of isomeric compounds depend upon
dipole-dipole interactions. Since the c/.9-isomer.s have higher dipole moments (more polar) than the

for
corresponding rm/u-isomers (less polar), therefore, the boiling points of c/.v-isomers are higher than those of
their corresponding rm^rv-isomers. For example, the boiling point of r/.v-but-2-enc is higher than that of
r
trans- bul-2-ene.
You
CH
3\ CH3 CH3 H
s
c = c c = c
ook

H H H
CH3
c/j-But-2-ene (b.p. = 277 K) //rtni-But-2-ene (b.p. = 274 K)
eB

SUPPLEMENT YOUR
KNOWLEDGE FOR COMPETITIONS

E, Z Nomenclature
our
ad

If three or all the four atoms or groups attached to the carbon atoms of the double bond are different, ds-
trans nomenclature cannot be used. Therefore, E, Z nomenclature has been introduced to name all types
of geometrical isomers. According to this nomenclature, if the atoms or groups of highest priority are on
the same side of the double bond, the isomer is designated as Z {Zusammen in Gentian means together)
dY

and if the two atoms or groups of highest priority are on the opposite sides, the isomer is designated as E
Re

(Entegegan in German means opposite).


In this system of nomenclature, each of the two atoms or groups on each carbon atom of the double bond
Fin

are assigned priority number (I) and (2) on the basis of the following sequence rules given by Cahn,
Ingold and Prelog. These, rules are also called Cahn-Ingold-Prel og rules or simply CIP rules.
Rule 1. The atom of higher atomic number gets higher priority. If the two atoms attached to the double
bond are isotopes, the isotope of higher mass number gets the higher priority. For example, in I-bromo-2-
chIoro-2-fluoro-l-iodoethene, Cj has two atoms viz. Br and I. Since, I (Z = 53) has higher atomic number
than Br (Z = 35), therefore, I is assigned priority (1) while Br is assigned priority (2). Similarly, Cl (Z = 17)
is assigned priority (1) while F (Z = 9) is assigned priority (2).
(2) (2)
Br I 2 F
c=c
Cl
(1) (1)
13/26 T^n^xtUc^'A New Course Chemistry (XI)iasIHD

Rule 2. If two atoms directly attached to the double bond have the same (2) (1)
atomic numbe,r then the relative priority of the groups is determined by a CH.
similar comparison of the atomic numbers ofthe next elements in the groups
^CH2C1
c = c
(and so on, if necessary, working outwards till the first point of difference is '^CH20H
reached). For example, in the following compound, one of the carbon atoms (1) (2)
of the double bond carries CH3 and CH3CH-, groups. Since,
he first atom (i.e., C) attached to the carbon atom of the double bond is the same in CH3 and CH3CH2
groups, compare the atomic numbers of the atoms attached to each of these first atoms. In CH3, these
atoms are H, H, H while in CH3CH2 these are C, H, H. Since, C has higher priority over H, therefore,

w
CH3CH2 group is assigned priority (1) while CH3 group is assigned priority (2).
The second carbon atom of the double bond carries groups CH2CI and CH2OH. Since, the first atom in these
tu'O groups is the same (i.e., C), therefore, compare the atomic numbers of the atoms attached to each of these
in CH2OH, tliese atoms are O, H, H. Since, Cl (Z = 17) has
first atoms. In CH->C1, these atoms are Cl, H, H while

Flo
higher priority over O (Z = 8), therefore, CH2CI is assigned priority (1) while CH2OH is assigned priority (2).
Rule 3. Double bonds and triple bonds are treatedas if they have duplicate or triplicatesingle bonds. For
example,

ee
—CH = CH— is treated as —CH—CH—, C = O is treated as c~o ,

Fr
c c 0 c

.c.

for
H H

and
ur
is treated as C
c 9
C and - C = C - is treated as
C

C—C
C

C C
ks
Yo
For example, consider the following compound.
oo
(2) (1)
CH CHO
3\
c = c
eB

^6^5 '^CH,OH
(1) (2)
One of the carbon atoms of the double bond carries CH3 and C(jH5 groups. Since in C5H5 group, the first
r

carbon is attached to two other carbons one by a double bond and the other by a single bond, therefore,C,
ou

C, CH of phenyl gets higher priority over H, H, H of CH3. Thus, C^H5 is assigned priority (1) and CH3 is
ad

assigned priority (2). The other carbon atom of the double bond carries CH2OH and CHO groups. Since in
CH = O, C is attached to O by a double bond while in CH2OH, C is attached to O by a single bond.
Y

Therefore, O, O, H of CHO gets higher priority over O, H, H of CH2OH group. Thus, CHO is assigned
priority (1) and CH2OH is assigned priority (2).
d
Re

On the basis of relative priorities, E and Z designations are assigned as discussed above. For illustration
consider the following examples :
n

(2)
Fi

(1) (1) (1)


CH. CH
/CH3 C = C
c = c
H ●H H CH3
(2) (2) (2) (1)

(Z)-But-2-ene (E)-Bui-2-ene
(2) (2) (1) (2)
CH.
/CH2CH3 CH
3\ /CH2OH
c = c c = c
Cl ●Br H CHO
(1) (1) (2) (1)

(Z)-3-Bromo-2-ChIoropent-2-ene (E)-2'Hydroxymethylbut-2-en-1 -al


HYDROCARBONS 13/27

Necessary and sufficient condition for geometrical isomerism. It may be mentioned here that all
compounds containing carbon-carbon double bonds do not show geometrical isomerism. The necessary
conditions for a molecule to exhibit geometrical isomerism are
(0 the molecule must contain a double bond, (ii) each of the two carbon atoms of the double bond must
have different substituents which may be same or different. Thus, alkenes of the type abC = Cab and abC =
Cde show geometrical isomerism.
a a a b a d a e

C = C C = C c = c C = C
b b b a b e b ■d

low
c;j-isomer rran5-isomer (Z)-isomer (E)-isomer

(If groups ‘a’ and 'b' have higher priority) (If group.<! 'a' and ‘d’ have higher priority)
However, geometrical isomers are not possible if one or both the doubly bonded carbon atoms carry two
similar substituents. This is because in such cases, the two possible configurations are, in fact, identical as
shown below :

a d a e

ee
C = C C = C
a e

rF a d

Fr
It is because of this reason that terminal alkenes such as propene, but-l-ene, 2-methylprop-l-ene, etc.
and alkenes carrying identical substituents on one of the doubly bonded carbon atoms such as 2-mcthylbut-2-
ene and 2, 3-dimethylbut-2-ene, etc. do not show geometrical isomerism.

for
CH3 H CH3CH2 H CH3 H CH3 CH3
c = c c = c c = c C = C
u
H H H H CH3 H CHj' CH3
ks
Propene But-l-ene 2-Methylprop-1 -ene 2, 3-Dimethylbut-2-ene
Yo
From the above discussion, we conclude that geometrical isomers have the same molecular structure
oo
but differ only in the relative positions of atoms or groups in space, therefore, they are stereoisomers. These
stereoisomers are not optical isomers since their molecules are not chiral.
eB

SUPPLEMENT YOUR
KNOWLEDGE FOR COMPETITIONS

Stability of Alkenes
r

Alkenes add on hydrogen, in presence of finely divided metals such as platinum, palladium or Raney
ou
ad

nickel to form alkanes.

Ptor Pd or Ni I I
c=c + H2 —C—C— + Heat
Y

4
A
I I
H H

This reaction is called hydrogenation. It is an exothermic reaction and the amount of heat evolved when
nd
Re

one mole of an alkene is hydrogenated is called its heat of hydrogenation.


The heat of hydrogenation of some alkenes is given in Table 13.2.
Fi

TABLE 13.2. Heats of hydrogenation of some alkenes

Alkene Heat of hydrogenation Alkene Heat of hydrogenation


(in kJ mol“*) (in kj mol"*)
Ethene 137-2 2-Methylprop-1 -ene 118-8

Propene 125-9 2-Methylbut-l-ene 119-2


But-l-ene 126-7 2-Methylbut-2-ene 112-5
c/j'-But-2-ene 119-6 3-Methylbut-l-ene 126-7
frans-But-2-ene 115-5 2, 3-Dimethylbut-2-ene 112-2
13/28 “Puidee^'4. New Course Chemistry (XI)ESSI9E

But-l-ene, c«5-but-2-ene and /ra/iA'-but-2-ene all on hydrogenation give the same alkane, i.e., n-butane but
evolve different heats of hydrogenation. This means that these alkenes must have different inherent energies
and hence different stabilities. An alkene which has a lower heat of hydrogenation must have less inherent
energy and hence will be more stable than its isomers which have higher heats of hydrogenation. Thus,
heat of hydrogenation of an alkene is the index of its stability. The lower the heat of hydrogenation of an

ow
alkene more .stable it i.s. The following conclusions can be drawn from Table 13.2 given above,
(i) An unsubstituted alkene such a.s eihene has the highest heat of hydrogenation,
(ii) Greater the number of alkyl groups attached to the doubly bonded carbon atoms, more stable is the
alkene. In general, alkenes have the following decreasing order of stability* :
R H
C = C c = c
R2C = CR2 > R2C = CHR > R2C = CH2 > HZ' '^R >RCH = CH2>CH2 = CH2.

e
H

{Hi) Out of geometric isomer.s, the trans-isomer is slightly more stable than the corresponding cis-isome.r

re
Explanation for relative stabilities of alkenes. The relative stabilities of different alkenes can be explained on
the basis of concept of hyperconjugation as discussed in unit 12. Consider, for example, the following alkenes:

Frl
F
a a a a
CH CH
/CH3 CH3
a
3\ 3\
C = C C = CH—CH3 C = C
CH3^ ^^3 CH3 H
CH3
a a
ou a a

r
2, 3-Dimethylbut-2-ene (I) 2-MethyIbut-2-enc (II) frorti-But-2-ene (III)

so
(Twelve a-H ; A = - 112-2 kj mol-') (Nine a-H ; A = - 112-5 kJ mol-') (Six a-H, A H2 =- -115-5 kJ mol-')
«2

kf
a a
CH
/CH3
oo
3\ a
C = C
CH3—CH = CH2 CH2 = CH2
H H
Y
c/5-But-2*ene (IV) Propene (V) Ethene (VI)
eB

(Sixa-H, <^H2 =-119-6 kJ moH) (Threea-H; ^H2 =-125-8 kJ mol'') (No a-C, No a-H ; =-137-2 kJ mol"’)
Alkenes I, II, III (or IV) have twelve, nine, six, three and none a-hydrogens respectively and hence equal
ur

number of hyperconjugation structures can be written for each one of them. Since greater the number of
oY

hyperconjugation structures, more stable is the alkene, therefore, relative stability of these alkenes follows
the sequence : I>II>III>IV>V> VI.
ad

Since /ran^-but-2-ene (III) is more stable than the cu-but-2-ene (IV) in which the two methyl groups lie on
the same side of the double bond and hence cause steric hindrance, therefore, heat of hydrogenation of III
d

is less than that of IV.


in

13.6.4. General Methods of Preparation of Alkenes


Re

Alkenes can be prepared in the laboratory by the following general methods of preparation.
F

1. By partial reduction of alkynes. The catalytic hydrogenation of alkynes to alkenes occurs faster
than that of alkenes to alkanes. Therefore, by using a specific catalyst, it is possible to stop the reduction at
the alkene stage. Further since alkynes show geometrical isomerism, alkynes can be reduced to give cis- or
/r«n5-alkenes depending upon the nature of the catalyst used. For example, catalytic reduction of alkynes in
presence of palladium supported over CaC03 or BaS04 and partially poisoned by addition of PbC03, S or
quinoline (Lindlar’s catalyst) predominantly gives cw-alkenes. However, if alkynes are reduced with sodium
in liquid ammonia (Birch reduction), trani-alkenes are the major products. Thus,

*This order is given in Organic Chemistry by T.W.G. Solomons and C Fryhle but in Organic Chemistry by
R.T. Morrison and R.N. Boyd no distinction has been made on the relative stability of R2C = CH2 and
RCH = CHR.
HYDROCARBONS 13/29

CH. Na/liq.NHj CH H
c = c /CH3 ^ H2/Pd/CaC03+S CH3—C = C—CH3 {Birch reduction)
c = c
{Lindlar's catalyst)
H'
cw-But-2-ene
H H
trans-But-2-ene
CH3
But-2-yne

2. From alkyl halides or haloalkanes. Alkyl halides on heating with a strong base such as sodium ethoxide
or a concentrated alcoholic solution of potassium hydroxide undergo dehydrohalogenation to give alkenes.
H

KOH {ale.)
a A ^ C=C + HX (X = Cl,BrorI)

a
Alkene

low
Alkyl halide
P a 353-363K
e.g.. CH3CH2—I + KOH(a/c.) > CH2 = CH2 + KI + H2O
lodoethane Ethene

{Ethyl iodide) {Ethylene)

ee
F
P a 353-363K

Fr
CH3CH2CH2—Br -f. KOH {ale) > CH3CH = CH2 + KBr + H2O
1-Bromopropane Propene
{n-Propyl bromide) {Propylene)

for
ur
This process of removal of a molecule of a halogen halide (HCly HBr or HI) from a haloalkane
to form an alkene is called dehydrohalogenation.
Dehydrohalogenation is an example of an elimination reaction. Since, in this reaction, a hydrogen is
s
removed from a jJ- carbon and halogen from the a-carbon, therefore, it is called P-elimination reaction.
ok
Yo
The ease of dehydrohalogenation of alkyl halides having the same alkyl group but different halogens is :
o
iodides > bromides > chlorides while for isomeric alkyl halides having the same halogen but different structures
eB

is : tertiary > secondary > primary. Thus, a tertiary alkyl iodide is most reactive.
Saytzeff rule. Depending upon the structure, alkyl halides may give one or more isomeric alkenes. For
example, dehydrohalogenation of 1-chlorobutane gives only one alkene, i.e., but-l-ene since only one type of
P-hydrogen is available on the left side of the molecule.
r
ou
ad

P a A
CHXH.,—CH—CH. + KOH (a/c.) — ■> CH3CH2CH = CH2 + KCl + H2O
3 2 I ,2 But-l-ene
H Cl
Y

1-Chlorobutane

If, however, the structure of the alkyl halide is such that it has a p-hydrogen on either side of the carbon atom
Re
nd

carrying the halogen, it can undergo eliniination in two different ways giving two alkenes. The relative amounts of
these two alkenes is governed by Saytzeff rule. According to this rule, whenever two alkenes are theoretically
Fi

possible during a dehydrohalogenation reaction, it is always the more highly substituted alkene (Le., having lesser
number of hydrogen atoms on the double bond) which predominates. For example,
> CH3—CH = CH—CH3
Br But-2-ene (80%)
KOH(fl/c.)A {More highly substituted
CH3—CH2—CH—CH3 alkene ; more stable)
2-Bromobutane

^ CH3—CH2—CH = CH2
But-l-ene (20%)
{Less highly substituted
alkene ; less stable)
13/30 ^fieuCeefr'4, Nev^.' Course Chemistry (XI)iasi9D

3. From vicinal dihalides or 1,2-dihaloalkanes. Dihalogen derivatives of alkanes in which the two halogen
atoms are present on adjacent carbon atoms are called vicinal or I, 2- dihaloalkanes. Alkenes can be prepared by
heating a suitable vie. or 1,2 dihaloalkane with zinc dust in methanol or ethanol For example,
CH3OH
Br—CH2—CH2—^Br + Zn A CH2 — CH2 + ZnBr2
1,2-Dibromoethane Ethene

(Ethylene dibromide) (Ethylene)

CH3OH
CH,—CH—CH, +Zn CH3^H = CH2 + ZnBr2

w
^1 I ^ Propene
Br Br
(Propylene)
1, 2-Dibromopropane

F lo
(Propylene dibromide)
1
This process of removal of a molecule of halogen (CI2, Bri or I^) from a dihaloalkane to form
an alkene is called dehalogenation :

ee
Even 1, 1-dihaloalkanes or gm-dihalides also undergo dehalogenation when heated with zinc dust in

Fr
methanol.
Br
I CH3OH

for
CH3—CH—Br + Zn —^^ CH2 — CH2 "b ZnBr2
ur
1, 1-Dibromoethane Ethene
(Ethylidene dibromide) s
CH3OH
ok
Yo
CH3CH2CHBr2 Zn A CH3CH = CH2 + ZnBr2
1, l-Dibromopropane Propene
o
eB

If sodium is used instead of zinc, and the reaction is carried out in ether, only a small amount of
propene is formed, the major product being hex-3-ene.

Ether, A
r

2CH3CH2CHBr2 + 4Na » CH3CH2CH = CHCH2CH3 + 4NaBr


ou
ad

1, l-Dibromopropane Hex-3-ene

This method is, however, not useful for the preparation of alkenes because 1, 2-dihaloalkanes are
Y

themselves prepared from alkenes. However, this method is useful for purifying alkenes or for protecting
double bond. For this purpose, Nal may be used instead of zinc dust. For example.
Re
nd

Br
iq
Fi

-2^
Nal Slow
:C=C: —C—C > C=C + IBr
CCI4 -NaBr

Impure alkene I ri
r^Br:
Piire alkene

1“ I
I

4. From monohydric alcohols or alkanols by acidic dehydration. Monohydric alcohols or alkanols


containing a P-hydrogen on heating with a mineral acid such as cone. H2SO4 or H3PO4 or on passing their
vapours over heated alumina at 623-633 K eliminate a molecule of water to form alkenes. Since a water
molecule is eliminated from the alcohol molecule in
the presence of an acid, this reaction is known as acidic
dehydration of alcohols.
HYDROCARBONS 13/31

pi la Cone. H2SO4 or AI2O3, A


c=c
—C—C— + H20
(Elimination of H2O
rL-J.,
lh..oh' .1
or Dehydratiori) Alkene

Alcohol

For example, AI2O3 95%H2S04


CH3CH2OH 623-633K
> CH2 = CH2 + H20 ; CH3CH2OH 433-443 K
^ CH2 = CH2 + H2O
Ethanol Ethene Ethanol Ethene

(Ethyl alcohol) (Ethylene) (r Alcohol)


OH

w
I 60%H2S04
CH3—CH—CH3 373 K
CH3—CH = CH2 + H2O
Propan-2-ol Propene

F lo
(2° Alcohol)
OH

ee
H3PO4.A ^ + H2O

Fr
Cyclohexanol Cyclohexene
(2° Alcohol)

for
CH- CH3
I ’ 30%H2S04
ur
CH3—C—OH 363 K
■>
CH3—C = CH2 + H20
2-Methylpropene
CH3
oks
(Isobutylene)
2-Methylpropan-2-ol
Yo
(i® Alcohol)
o

It is evident from the above reactions, that the order of dehydration of different alcohols is :
eB

> 2® > 7®.

Like alkyl halides, dehydration of alcohols also follows SaytzeCf rule. For example,
> CH3—CH = CH—CH3
our
ad

OH But-2-ene (80%)
Conc.H2S04,A (more stable alkene)
CH3—CH2—CH—CH3
Butan-2-ol ■> CH3CH2—CH = CH2
Y

But-l-ene (20%)
Re

(less stable alkene)


nd

Mechanism. The dehydration of alcohols does not occur in just one step but occurs in a series of steps.
A knowledge of the series of steps by which the reaction occurs is called the mechanism of the reaction. The
Fi

dehydration of alcohols occurs by £; mechanism by the following three steps :


Step 1. An alcohol being a Lewis base accepts a proton from the strong acid to form a molecule of
protonated alcohol.
H2SO4 ^ H+ + HSO4
H
● ● +

CH3CH2—o—H + H r==^ CH3CH2—O


H
Ethanol Protonated ethanol

This Step is usually reversible.


13/32 New Course Chemistry fXniwsTTT

Step 2. The presence of a positive charge on the highly electronegative oxygen atom weakens the
C—O bond and thus it readily eliminates a molecule of water to form areactive chemical species called carbocation.
H
Slow +

CH3CH2-^0 CHj—CH2 + H2O


H
Elhyl carbocation

This step is slow and hence is the rate-determining step of the reaction.

Step 3. The carbocation formed in step 2 immediately loses a proton to form an alkene.

w
+
Fast
H-^CH2—CH2 > CH2=CH2 +
Ethyl carbocation Ethene

F lo
This step is fast and hence does not affect the rate of the reaction.

St ^t.eMENT YOUR

ee
KNOWLE IE FOR COMPETITIONS

Fr
Rearrangement of carbocations during dehydration of alcohols. During dehydration of alcohols,
sometimes unexpected or rearranged alkenes are formed. This is due to the reason that carbocations are
the intermediates. If the initially formed carbocation is less stable, it rearranges either by 1, 2>hydride or

for
1,2-methyl shift to form the more stable carbocation which eventually loses a proton to form the unexpected
or the rearranged alkene. For example, dehydration of 1-butanol or 2-butanol with cone. H2SO4 gives the
ur
same mixture of 1-butene and 2-butene in which the more highly substituted alkene, i.e., but-2-ene
predominates (Saytzeff rule). The formation of this mixture from 1-butanol can be explained by
s
rearrangement of the initially formed less stable T carbocation to the more stable 2° carbocation by 1,2-
ook
Yo
hydride shift which then loses a proton in accordance with Saytzeff rule to afford a mixture of but-2-ene
(major) and but-l-ene (minor)
eB

+H* Slow +

CH3CH2CH2CH2—OH CH3CH2CH2CH2- OH2 ► CH3CH2—CH—CH2


-H* -H2O

ay
I-Butanol
our
ad

1® Carbocation
{less stable)
2 1 2 1
1,2-Hydride +
-H*(Fos/)
Y

shiA ► CH3CH2—CH—CH3 Saytzeff rule


CH3CH=CHCH3 + CH3CH2CH=CH2
2° Carbocation But-2-ene (80%) But-l-ene (20%)
Re

{more stable)
nd

2-Butanol, under these conditions, gives directly the more stable T carbocation which then loses a proton
Fi

to give the same mixture of 2-butene (80%) and 1-butene (20%).

2 + H+ + -H-*-

CH3CH2—CH—CH3 H->o
^ CH3CH2~CH—CH3 Saytzeff rule
^ But-2-ene(80%)
-I-
1° Carbocation
OH
(more stable) But-l-ene (20%)
2-Butanol

Similarly, when 3,3-dimcthylbutan-2-ol is subjected to dehydration with cone. H2SO4, the initially formed
2® carbocation (I) rearranges to the more stable 3° carbocation (II) by 1, 2-methyl shift which subsequently
loses a proton to yield 2, 3-dimethylbut-2-ene as the major product in accordance with Saytzeff rule.
HYDROCARBONS 13/33

CH3 CH3 CH3


4 2 1 + H*
3 +
1,2-Methyl
CH3—C—CH—CH3 ► CH3—C—CH—CH3 shift
> CH3—c—CH—CH3
-H2O

CH3 OH CH3 CH3


3, 3-Dimelhylbutan-2-ol 2° Carbocation (I) 3® Carbocation (10
{less stable) {more stable)

4
CH3
CH3 CH3 2I 3 ^
CH3
-H*

w
SaytzefT rule
-► C=C -H
CH2 ==C—CH.
CH3 CH3 CH3
2, 3-Dimethylbut-2-ene 2, 3-Dimethylbut-l-ene
{major product) {minor product)

F lo
Laboratory method of preparation. In the laboratory elhene is prepared by dehydration of ethyl
alcohol with cone. H2SO4 at 440 K. For this purpose, a mixture of ethanol and cone. H2SO4 in the ratio of

ee
1 : 2 by volume is added from a dropping funnel into a round bottomed flask containing anhydrous
Al2(S04)3 and sand in which Al2(S04)3 catalyses the reaction whereas sand is used to avoid frothing during

Fr
the process.
The reaction mixture is heated to 433-443 K and ethenethus produced is collected over water as shown
in Fig. 13.13. Ethene produced by the above method contains impurities of SO2 (formed by the reduction of

for
H2SO4) and CO2 (formed by the oxidation of ethanol). In order to remove these impurities, impure ethene is
our
passed through a solution of caustic soda (NaOH) which absorbs CO2 and SO2. s FIGURE 13.13
ook
C2H5OH + CONC. H2S04(1;2)
Y
eB

OH
Al2(S04)3 + SAND
+ CONC. H2SO4
r
ad
ou

V ETHENE'
0=1
Y
Re
nd

NaOH SOLUTION
Fi

Laboratory preparation of ethene

2 NaOH -I- COo Na,C03 + H2O 2 NaOH + SO2 4 Na2S03 + H2O


5. From sodium or potassium salts of saturated dicarboxylic acids {Kolbe's electrolytic reaction)
Electrolysis of sodium or potassium salts of saturated dicarboxylic acids gives alkenes. For example,
CH2COOK Electrolysis CH,
+ 2H2O ■> I ' 4- 2 CO2 + H2 + 2 KOH
CH2COOK CH,
Pot. succinate Ethylene

I
13/34 New Course Chemistry (XI)EEIHn

This reaction is called Kolbe’s electrolytic reaction and is believed to occur by the following steps :
CH.COOK CH,COO~
lonization
I -> I + 2K+ ; 2H,0 ^ 2 OH- +2 H-"
CH^COOK CH2COO-
CH.COO"
CH2COO' CH2
At anode : I - 2 e~ + 2CO^
CH2COO- CH2COO CH2
{unstable)

w
At cathode. Since the electrode potential of H'*' ions is much higher than that of ions, therefore,
ions are preferentially reduced to produce H2 while K"*" ions remain in the solution

F lo
2H++2^- ^[2 H] ^ H2
13.6.5. Physical Properties of Alkenes
(i) Physical state, colour, smell, etc. The first three members of the family, i.e., ethene, propene and

ee
butenes are colourless gases ; the next fourteen members (C5-C|g) are liquids while the higher ones are
solids. Except ethene which has a pleasant smell, all other alkenes are colourless and odourless gases,

Fr
(ii) Solubility. They are insoluble in water, but are fairly soluble is non-polar solvents such as benzene,
petroleum ether, etc.

for
(iii) Boiling points. Their boiling points increase regularly with increase in molecular mass. On the
ur
average, the boiling points generally increase by 20-30 K for the addition of each CH2 group to the chain.
Like alkanes, straight chain alkenes have higher boiling points than isomeric branched chain alkenes.
(iv) Density. The densities of alkenes increase with increase in the molecular masses till the limiting
s
value of about 0-8 g cm“^ is reached. This means that all alkenes are lighter than water.
ook
Yo
13.6.6. Why do Alkenes undergo Electrophilic Addition Reactions ?
Alkenes are characterized by the presence of a double bond which consists of a strong C-C, o-bond and
eB

a weak C-C, 7t-bond. The 7i-electrons form an electron cloud which lies above and below the plane of o-
bonded carbon atoms (Fig. 13.10/?, page 13/23). These, n-electrons are, therefore, more exposed and hence
are less tightly held between the two carbon atoms. Since the electrons are negatively charged particles,
r

therefore, the 7t-electrons attract the electrophiles and repel nucleophiles. In other words, alkenes undergo
ou
ad

electrophilic reactions. Now, in principle electrophilic reactions can be of two types : (/) addition and
(ii) substitution.
Y

X
H H H
X*Y" X'^Y"
Re
nd

:C=C :c=c :C—C


-YH H
X H
(Subslitution) Alkene Y
Fi

(Addition)

In electrophilic substitution reactions, one a C-H bond is broken and a new o-bond between one of the
doubly bonded carbon atoms and the electrophile is formed. Since the bond energies of the o C-H bond
broken and the new o C-X bond formed are not much different, therefore, electrophilic substitution reactions
are not accompanied by large energy changes.
On the other hand, in electrophilic addition reactions, one weak TT-bond (251 kJ mol“^) is broken and
two strong a-bonds (2 x 348 = 696 kJ mol"^) are formed. The overall reaction is accompanied by a release of
about 696-251 = 445 kJ mol'^ of energy. In other words, electrophilic addition reactions are energetically
more favourable than electrophilic substitution reactions. Thus, the typical reactions of alkenes are electrophilic
addition reactions and not the electrophilic substitution reactions.
HYDROCARBONS 13/35

13.6.7. Mechanism of Electrophilic Addition Reactions to Alkenes


Let us illustrate the mechanism of electrophilic addition reactions by taking the example of addition of
Br2 to ethylene. The reaction occurs by a two-step ionic mechanism as discussed below :
Step 1. Bromine molecule itself is non-polar but when it comes close to an ethylene molecule, the n-
electrons of the double bond begin to repel the electron pair holding the two bromine atoms together in the
bromine molecule. As a result, bromine molecule gets polarized. The positive end of this bromine dipole
behaves as an electrophile and is attracted by the 7t-electrons of the ethylene molecule to form a 7C-compIex
which subsequently gives the carbocation and the bromide ion. This step is stow and hence is the rate
determining step of the reaction.
Ethylene 8+ 8-
Br—-Br Br—Br

w
Polarized bromine molecule

CH2 8+ 8- CH2 8+ 8-

F lo
Slow -1-
+ Br—Br ^ Br—Br
^ CH^—CH2 + Br-
CH2 CH2
Br
Ethylene 7C-Complex
Bromoelhyl

ee
carbocation

Fr
This step can simply be represented as
a

for
/ ^5+ 5- Slow +

CH2=CH2 + Br —Br > CH2—CH2—Br + Br"


Ethylene Bromoethyl carbocation
r
Step 2. The carbocation formed in step 1 being a reactive chemical species immediately undergoes
You
nucleophilic attack by the bromide ion present in the solution forming the addition product. This step is fast
s
ook
and hence does not affect the rate of the reaction.
Br
eB

Br" + CH2—CH2 Nucleophilic attack ^ CH2—CH2

Br Br
our
ad

Bromoethyl carbocation 1,2*Dibromoetbane

Evidence in support of the above mechanism. If the carbocations are really the intermediates in the
above mechanism, then they should also react with other nucleophiles when added to the reaction mixture
and hence a mixture of products should be formed. This has indeed been found to be so. For example, when
dY

ethylene is bubbled into an aqueous solution of bromine containing sodium chloride, besides 1, 2-
Re

dibromoethane, 1-bromo- 2-chloroethane and 2-bromoethanol are also formed.


Br- 2 1
Fin

^ Br—CH2 —CH2—Br
I, 2- Dibromoethane
Br 2 1
cr
^
CH2 = CH2 -Br- CH,—CH,—Br
2 2 ^ Cl—CH2 —CH2—Br
Ethylene Bromoethyl l-Bromo-2-chioroethane
carbocation
H2O + 2 1
» Br—CH2—CH2—OH2 > Br—CH2 —CH2—OH
-H^
2-Broinoethanol

*In the it-complex, there is no actual bonding between the ethylene and the polarized bromine molecule.
There is only electrostatic attraction.

I
13/36 ‘Px€uUe^ 'a New Course Chemistry (XI) cfu»jiiiii

● ^ UR
, I feTITIONS

1. Limitations of Simple Carbocation Mechanism


of Electrophilic Addition Reactions to Alkenes.
The mechanism of electrophilic addition reactions
Cyclohexene
to alkenes involving simple carbocation inter
mediates does not explain the following points :
"Br Bf2
(1) Stereochemistry of addition of halogens to
alkenes. It has been observed that halogenation of
alkenes always gives rran.r-dihalides and not cis-
dihalides whenever the product of halogen addition
is capable of showing stereoisomerism. For example,

w
addition of Brj to cyclohexene gives only trans-\,
2-dibromocyclohexane. If the simple carbocations
were the intermediates, both cis- and trans-\, 2-

F lo
dibromocyclohexanes should have been formed.
This is due to the reason that carbocations are planar
chemical species and hence the attack of the bromide
ion on the initially formed carbocation (I) from either

e
side of the molecule is equally probable as explained

Fre
in the equations given on RHS.
(ii) Rearrangements of intermediate carbocations
are not observed in halogenation of alkenes. The

for
carbocations formed during the halogenation of
alkenes do not undergo rearrangement.For example,
addition of Br2 to 3,3-dimethyl-1- butene gives only
r
the expected 1,2-dibromo-3, 3-dimethylbutane and trans-1,2- cis-1,2-
You
not the rearranged 1, 3-dibromo-2, 3-dimethylbutane. Dibromocydohexane Dibromocydohexane
oks

CH, CH3
eBo

I Br->
CH.,—C—CH = CH- CH,—C—CH—CH, and not CH,—C—CH—CH.,
3 I 2 ^11 I ^ 3 I I ,2
CH3 H3C Br Br Br CH3 Br

3. 3-Dimethyl-1-butene 1, 2-Dibromo-3, 3-dimethylbulane 1, 3-Dibromo-2, 3-dimethylbutane


our
ad

Cyclic Halonium ion Mechanism


Since, the addition of halogens to alkenes (/) always gives trans-dihalides and (ii) the rearrangement of
intermediate carbocations is not observed, it was suggested that this addition, in fact, occurs through a
cyclic bromonium ion (II) rather than the simple carbocation (I).
dY
Re

I .-'Br:
CH2—Br: CHf' *●
Fin

Carbocation (I) Cyclic bromoniun ion (II)


The nucleophilic attack of Br~ ion on this cyclic bromonium ion can occur only from the back side thereby
giving trans-dihalides because the attack from the front side is hindered by the bulky bromine atom.

Br- CHy Br—CH2


+ .>Br
CH'2-' CH2—Br
Cyclic bromoniun ion (II) /rt//i.v-Dihalide (hypothetical)

Like carbocation, this cyclic bromonium ion is also a very reactive chemical species and hence is readily
attacked by bromide ion (nucleophile) to complete the addition. Howeve,r unlike simple carbocation on
which attack of the bromide ion from either side is equally probable, in cyclic bromonium ion, the attack

fe
HYDROCARBONS 13/37

of the bromide ion can occur only from the backside of the bromine atom (forming the bridge) since the
attack from the front side will be hindered by this bulky bromine atom. This explains why the addition of
halogens to alkenes gives mmi-dihalides.
+
Br-
H Br.
-Br Backside
attack

H H H H Br H

Cyclohexene Cyclic bromonium ion trans-). 2-Dibromocyclohexane

13.6.8. Chemical Reactions of Alkenes


Although alkenes normally undergo ionic electrophilic addition reactions, some reagents also add by

w
free radical mechanism. Under special condition, alkenes also undergo free radical substitution reactions.
Besides addition reactions, alkenes undergo oxidation, ozonolysis and polymerization reactions. These

F lo
are briefly described below :
1. Addition Reactions of Alkenes
1. Addition of dihydrogen. Addition of dihydrogen to unsatiirated hydrocarbons (such as alkenes, alkynes,
arenes, etc.) in presence of a catalyst is called catalytic hydrogenation. Alkenes readily add dihydrogen in
presence of Raney nickel (an active form of nickel), platinum or palladium as catalyst at room temperature or

e
ordinary nickel at 523-573 K to fonn alktines (Sabatier and Senderen ’.v reduction). For example,

Fre
Raney Ni Ni
CH2 = CH2 + H2 ^ CH3—CH3 ; CH3CH = CH2 + H2 ^ CH3—CH2--CH3

for
orPtorPd 523-573 K
Ethene Ethane Propene Propane
2. Addition of halogens. Halogens such as chlorine and bromine readily add to alkenes to form 1, 2-
dihaloalkanes. For example,
r
CCI4 Ether
CH2 = CH2 + Br2 > CH2—CH^ ; CH3CH = CH2 -h CI2
You
^ CH,CH—CHt
oks

Ethene
Propene ^1 I ^
Br Br Cl Cl
(Ethylene)
eBo

(Propylene)
1,2-Dibromoethane 1, 2-Dichloropropane
(Ethylene bromide) (Propylene chloride)
During the addition of bromine to alkenes, the orange red colour of bromine is discharged since the dihromide
formed is colourless. This reaction is, therefore, used as a test for unsaturation in oi^anic compounds.
ad
our

Fluorine reacts with alkenes too rapidly to be controlled in the laboratory while iodine does not react
with alkenes at ordinary temperatures. Thus, the order of reactivity of addition of halogens to alkenes is:
F2 > CI2 > Br2 > I2
3. Addition of halogen halides. Alkenes react with halogen halides (HCl, HBr, HI) to form
monohaloalkanes called alkyl halides.
Re
dY

H
Trans-
Fin

c=c -b HX ¥
C—C
addition CH2 =CH2+HBr ^ CH3—CH2Br
Alkene Ethene Bromoethanc
X
(Ethylene) (Ethyl bromide)
Alkyl halide

The order of reactivity o alogen halides in this reaction is : HI > HBr > HCl. This order of reactivity
can be explained on the basis of bond dissociation energies of the halogen halides ; lower the bond dissociation
energy, more reactive is the halogen halide ;
HI (300 kJ mo|-‘) > HBr (360 kJ moU‘) > HCl (430 kJ mor^)
The actual product formed, however, depends upon whether the alkene is symmetrical or unsymmetrical
as discussed below :
(i) Addition to symmetrical alkenes. When the alkene is symmetrical only one product is theoretically
possible. For example.

I
13/38 New Course Chemistry (XI)DBl9n

CH2 = CH2 + HBr > CH3—CH2—Br ; CH3CH = CHCH3 + HCl ^ CH3—CH—CH2CH3


Ethene Bromoethane But-2-ene
Cl
2-Chlorobutane

Mechanism. Like the addition of halogens, addition of halogen halides to alkenes is also an electrophilic
addition reaction and occurs by the following two steps :

H-ftr Ionization
^ H"" + Br"

Slow
Stepl. CH2=^CH2 + H'" CH2—CH3
Ethene Ethyl carbocation (I)

fNt Nucleophilic attack ^

F low
Stepl. Br“ + CH2—CH3 Fast
Br—CH2—CH3
Ethyl carbocation (I) Bromoethane

(ii) Addition to unsymmetrical alkenes. When the alkene is unsymmetrical, two products are
theoretically possible. For example, the addition of HBr to propene in the dark and in the absence ofperoxides
can, in principle, give two products. But experimentally, it has been found that under these conditions, the
major product is 2-bromopropane and the minor product is 1-bromopropane.
Dark
CH3CH = CH2 + HBr ^ CH3 CH—CH3 + CH3CH2CH2—Br

re
Absence of peroxides I
Propene 1-Bromopropane

for F
Br
(minor product)
2-Bromopropane
(major product)
Markovnikov’s rule. Markovnikov, a Russian chemist, studied a large number of such addition reactions
and postulated an empirical rule in 1869 which is known after his name as Markovnikov’s rule. The rule
states that,
Your
t;'
The addition of unsymmetrical reagents such as HX, H2O, HOX, etc. to unsymmetrical alkenes
s
eBook

' occurs in such a way that the negative part of the agendum (ue., adding molecule) goes to that
; ^ i carbon atom of the double bond which carries lesser number of hydrogen atoms.
CH3 CH3
I
ad

Mark.addn.
CH3 C = CH2 + H+Cl-
For example. CH3—C—CH3
our

2-Methylpropene
Cl
2-Chloro-2-methylpropane
Theoretical explanation of Markovnikov’s rule. The addition of halogen halides to alkenes is an
Re

electrophilic addition reaction. Thus, during the addition of HBr to propene, the first step involves the addition
of a proton. This addition, in principle, can occur in two ways. If the proton adds on the terminal carbon atom
Y

of the double bond, a 2“ carbocation (I) is formed and if the addition occurs on the middle carbon atom, a 1®
Find

carbocation (II) is produced.


+ H'*’, Addition at C] 2 1 H'*', Additionat C2 +

CH3—CH—CH3 « Slow
CH3—CH = CH2 Slow
^ CH3—CH2—CH2
2° Carbocation (I) Propene r Carbocation (II)
(more stable) (less stable)
Fast Br Fast Br

CH3—CH—CH3 CH3—CH2—CH2Br
I
l-Bromopropane
Br
(minor product)
2-Bromopropane
(major product)

I
HYDROCARBONS 13/39

Since, a 2° carbocation (I) is more stable than 1® carbocation (II), therefore, carbocation (I) is
predominantly formed. This carbocation then rapidly undergoes nucleophilic attack by the Br” ion forming
2-bromopropane as the major product. Thus, Markovnikov’s addition occurs through the more stable
carbocation intermediate.

SUPPLEMENT YOUR '


KNOWLEDGE FOR COMPETITIONS

Rearrangement of carbocations during addition of halogen acids. It may be noted that unlike the
addition of Bf2 to alkenes which occurs through a cyclic bromonium ion {II, page 13/37), the addition of
HBr occurs through the intermediate fonnation of .simple carbocations (I, page 13/39). The reason being
that the H-atom does not have a lone pair of electrons to form the cyclic carbonium ion.

w
Since, carbocations are prone to rearrangement, therefore, if the structure of the initially formed carbocation
permits, it may undergo rearrangement either by a 1,2>hydride shift or by a 1,2- methyl shift to form
the more stable carbocation which then undergoes nucleophilic attack by the Br“ ion to form the rearranged

F lo
(unexpected) alkyl halide as the major product along with a small amount of the expected alkyl halide
derived from the less stable carbocation. For example,

CH3 ^
CH3 CH3

ee
1, 2-Hydride
CHj—CH—CH=i=CH2 Slow ^ CH3—C—CH—CH3 > CH3—C—CH2CH3

Fr
shift

0/
3-Methylbut-l-ene
3® Carbocation
(more .ttable)
2® Carbocation

for
(less stable) + Br Fast
our
CH3 CH3
s
ook
CH3—C—CH—CH3 CH3—C—CH2CH3
H Br Br
Y
eB

2-Bromo-3-methylbutane 2-Bromo-2-metylbutanc
(minor product) (major product)
Similarly,
CH3 CH3 CH3
r
ou
ad

+
1,2-Methyl
CH3—C —CH=^CH2 Slow ► CH3—c—CH—CH3 shift > CH3—C—CH—CH3
CH3 CH3
Y

3,3-Dimethylbut-!-ene 3° Carbocation
2° Carbocation (less stable)
(more stable)
Re
nd

+ Br Fast
Fi

CH3 CHt
I
CH3—C—CH—CH3 CH3—C—CH—CH3
I 1
CH3 Br Br CH3
2-Bromo-3, 3-dimethylbulane 2-Bromo-2,3-dimethylbutanc
(minor product) (major product)

Peroxide effect. It may be noted that Markovnikov’s rule is not always followed. In presence of peroxides
such as benzoyl peroxide (CgH5C0-0-0-C0CgHg), the addition of HBr (but not of HCl or HI) to
unsymmetrical alkenes takes place contrary to Markovnikov’s rule. This is known as Peroxide effect or
Kharasch effect. Thus,
13/40 New Course Chemistry (XI)Q

(C6H5C00)2,A
CH3CH = CH2 + HBr CH3CH2—CH2Br
Anti-Mark. addn.
Propene 1-Bromopropane
(Propylene) {n-Propyl bromide)
Mechanism. The addition of HBr to alkenes in presence of peroxides occurs >y a free radical mechanism.
It consists of the following three steps.
(a) Initiation.
o o o

(0 QHj-C-O^-C-QHsBenzoyl peroxide
A

Homolytic fission
> 2C6H5—c—O

w
o

O 11/^- pr
(//) ^ QHs + CO2 (Hi) CgHs + H-^Br QHg + Br

F lo
(b) Propagation. It consists of two steps.
During the first step, a Br* adds to the double bond in such a way so as to give the more stable free
radical. In the second step, the free radical thus produced abstracts a H* from HBr to complete the addition.

e
Fre
(0 CH3—CH=^CH2 + Br >CH3—CH—CH2Br
Propene 2° radical (more stable)

for
r
07) CH3—CH—CH2Br + H Br > CH3—CH2—CH2Br + Br
r
l-Bromopropane
You
(c) Termination.
oks

(0 2Br ■> Br2


eBo

07) CH3—CH—CHjBr-i-Br ^ CH3—CHBr—CH2Br


1, 2-Dibromopropane

CH3^2 3
CH3
ad
our

(Hi) 2 :CH ■» 1 CH—CH


BrCH2 BrCH2 ^CH2Br
1, 4-Dibromo-2, 3-dimethylbutane

Exceptional behaviour
Y
Re

of HBr. To understand why XinHX AH (in kJ/mole)


d

peroxide effect is observed only


(0 X + CHj = CHCH3 (1OXCH2—CHCH3 + HX
Fin

with HBr and not with HF, HCl


or HI, let us consider the AH of —> XCHj—C H—CH3 > XCHj—CH2CH3 + X
the two propagation steps F -209 -hl59
From the above data, it is Cl - 101 -^27

clear that only with HBr, both Br -42 -37


the steps are exothermic and I +12 -104
hence the peroxide effect is
observed. With HCl or HF, the peroxide effect is not observed because the second step involving the reaction of
carbon radical with HCl or HF is endothermic. Further, the peroxide effect is also not observed with HI because the
first step involving the addition of iodine radical to alkenes is endothermic.
HYDROCARBONS 13/41

4. Addition of the elements of hypohalous adds (HOX where X = Cl, Br or I)-Halohydrin formation.
Chlorine and bromine in the presence of water readily add to alkenes to form the corresponding halohydrins.*
X2 + H2O ^ HOX + HX
OH
X2/H2O
C = C:
(or HOX)
C—CC +HX
Alkene
X

Halohydrin

w
CI2/H2O
For example, CH2 = CH2 HO—CHo—CH,—Cl + HCl
^ ^ (orHOQ) 2 2
Ethene 2-Chloroethanol

{Ethylene) {Ethylene chlorohydrin) g_

Flo
The overall reaction involves the addition of the elements of hypohalous acid (HO—X) in accordance
with Markovnikov's rule.
5- 5+ {Mark, addn.)

ee
CH3—CH = CH2 + HO — Br CH,—CH—CH, + HBr
Propene 3 I I 2

Fr
OH Br
{Propylene)
1 -Bromopropan-2-oI
{Propylene bromohydrin)

for
The order of reactivity of different hypohalous acids, i.e.y X2/H2O (HOX) is :
ur
CI2/H2O (HOCl) > Br2/H20 (HOBr) > I2/H2O (HOI)
Mechanism. The reaction actually occurs in two steps. In the first step, halogen adds slowly to the
double bond to form cyclic halonium ion (I). In the second step, rapid nucleophilic attack by H2O on the
k s
cyclic halonium ion (I) followed by loss of a proton gives the corresponding halohydrin.
Yo
oo

( ^6+ 5-
eB

Slow
CH3—CH=CH2 + Br-^Br -Br CH3—CH~_CH2 Fast
► CH3—CH—CH2Br
Propene N II
I
Cl-* fir
6+
O
\
r

H H
(D
ou
ad

► CH3—CH—CH2
Y

I I
OH Br
Re
nd

Propylene bromohydrin
5. Addition of sulphuric add - Indirect hydration of alkenes. Cold cone. H2SO4 adds to alkenes to
form alkyl hydrogen sulphates. In case of unsynunetrical alkenes, addition occurs in accordance with
Fi

Markovnikov's rule.

CH2=^CH2 + H'"-0S020H > CH3—CH2—OSO2OH


Ethylene Sulphuric acid Ethyl hydrogen sulphate

Mark. addn.
CH3CH=!=CH2 + H'^-0S020H ► CH3—CH—CH3
Propylene I
OSO2OH
Isopropyl hydrogen sulphate

’“Compounds containing halogen atom and the hydroxyl group on adjacent carbon atoms are called
halohydrins.
13/42 ‘Prutdee^’^ New Course Chemistiy (XI)Bi
Importance. Alkyl hydrogen sulphates on boiling with water, undergo hydrolysis to produce alcohols.
For example,

CHj—CH—CH3 + HjO CHj—CH—CH3 + H2SO4


OH
OSO3H
Isopropyl hydrogen sulphate Propan-2-ol
Thus, alkenes can be converted into alcohols as follows :

w
(«)Conc.H2S04(coW)
CH2 = CH2 > CH3CH2OH
Ethene
(ii)H20,A Ethanol

This overall two-step conversion of an alkene first into alkyl hydrogen sulphate followed by hydrolysis

F lo
with boiling water to form alcohols is called indirect hydration of alcohols.
6. Addition of water—Direct hydration of alkenes. By hydration we mean addition of water.
Ordinarily, water does not add directly to most of alkenes. However, some reactive alkenes do add water in

ee
presence of mineral acids to form alcohols. The addition occurs in accordance with Markovnikov’s rule. For

Fr
example.
CH-, CH3
5+1 6- 5+ 6- H+ I

CH3—C = CH2 + H—OH CH3—C—CH3

for
MarLaddn. I
ur
2-Methylpropene OH
(Isobutylene) 2-Methylpropan-2-ol
(tert-Butyl alcohol)
ks
n. Oxidation Reactions of Alkenes. Alkenes undergo a number of oxidation reactions to give different
Yo
products.
oo
1. Complete oxidation with oxygen or air - Combustion. Alkenes bum in oxygen or air to form CO2
eB

and H2O. This process is called combustion. All combustion reactions are highly exothermic in nature. For
example, CH2 = CH2 + 3 O2 > 2 CO2 + 2 H2O ; H = - 1411 kJ mor^
2. Controlled oxidation with oxidising agents. Alkenes react with a number of oxidising agents to
give different products:
r

(i) Oxidation with oxygen. Alkenes react with O2 in presence of silver as catalyst to form epoxyalkanes
ou
ad

or epoxides. For example.


Q
Y

1 Ag / \
CH2=CH2 + -02 575 K
CH2—CH2
nd
Re

Ethene Epoxyethane
2 1
1 Ag
Fi

CH3—CH = CH2+-02 575 K


» CH,—CH—CH.
Propene o

1, 2-Epoxypropane
(ii) Oxidation with potassium permanganate. Different products are formed depending upon the
reaction conditions,

(a) With cold dilute neutral or alkaline KMn04. Because of the presence of 7t-bonds, alkenes are
readily oxidised by cold dilute neutral or alkaline KMn04 solution to give vicinal or 1, 2-diols or 1, 2-
glycols while KMn04 is itself reduced to Mn02-
This reaction is called hydroxylation since during this process, two hydroxyl groups are added across
the double bond. For example.
HYDROCARBONS 13/43

2 KMn04 + H2O ^ 2KOH + 2Mn02 + 3 [O]


CH2 298-303K CH,OH
+ HjO+EO] I ^
CHj - CH2OH
FromKMn04
Ethene Ethane-1, 2-diol
(Ethylene) (Ethylene glycol)
298-303 K
3 CH3CH = CHCH3 + 2 KMn04 + 4 H2O » 3CH,CH—CHCH, -i-2MnO, +2KOH
But-2-ene 3| I 3 2

w
OH OH
Butane-2, 3-diol

During this reaction, the pink colour of the KMnO^ solution is discharged and a brown precipitate of
manganese dioxide is obtained. This reaction is, therefore, used as a test for unsaturation under the name

F lo
Baeyer’s test
(b) Oxidation with hot KMn04 solution. When an alkene is heated with hot KMn04 solution, cleavage
of the C=C bond occurs leading to the formation of carboxylic acids, ketones and carbon dioxide depending

ee
upon the nature of the alkene as shown below :

Fr
With terminal* alkenes, one of the products is always methanoic acid (formic acid) which on further
oxidation gives CO2 + H2O. For example.

for
KMn04,K0H [O]
CH2 = CH2 + 4[0]
ur 2H—C—OH ^ 2CO2 + 2H2O
373-383K
Ethene Methanoic acid
(Ethylene) (Formic acid)
O
s
ok
Yo
KMn04,K0H [O]
CH3CH 4 CH2 + 4 [O] CH3—C—OH + [HCOOH] CO2 H2O
o
373-383K
Propehe Ethanoic acid
eB

(Propylene) (Acetic acid)

CH3 CH3
I KMn04,K0H [O]
r

CH3—C ^ ^^2 ^ CH3—C = O + [HCOOH] CO2 + H2O


ou
ad

373-383K
2-MethyIpropene Propanone
(Isobutylene) (Acetone)
Y

With non-terminal alkenes, carboxylic acids or ketones or both of these are obtained depending upon
the nature of the alkene. For example.
Re

KMn04,KOH
nd

CH3CH CHCH3 + 4 [O] 373-383K


» CH3COOH + HOOCCH3
But-2-ene Ethanoic acid (2 molecules)
Fi

(Acetic acid)
KMn04,K0H
CH3CH2CH = CHCH3 + 4 [O] 373-383 K
> CH3CH2COOH + HOOCCH3
Pent-2-ene Propanoic acid Ethanoic acid

(Propionic acid) (Acetic acid)

CH CH
KMn04,K0H CH3 .CH3
^ + 2 [O] 373-383K
:C = 0 + 0 = C
CH3 CH3
2, 3-Dimethylbut-2-ene Propanone (2 molecules)
(Acetone)

*Alkenes in which the double bond is present at the end of the carbon chain are called terminal alkenes.
13/44 “P>uuict^ 'a New Course Chemistry (XI)

(iii) Oxidation with ozone. When ozone is passed through a solution of an alkene in some inert solvent
such as CH2CI2, CHCI3 or CCI4 at a low temperature (196-200 K), it oxidises alkenes to ozonides. Ozonides
are unstable and explosive compounds. Therefore, they are not usually isolated but are reduced, in situ, with
zinc dust and water or H VPd to give aldehydes or ketones or a mixture of these {reductive cleavage) depending
upon the structure of the alkene.
O
CH2 CH2CI2 ^CH2-0, Zn/H-jO
+ O3 > o: ¥ 2H—C—H
-ZnO
’^CH2—O"
196-200 K
CH-,

low
Methanal
(reduclive cleavage)
Eihene Ethene ozonide (Formaldehyde)
(Flhylene) (Ethylene ozonide)

O H H

CH2CI2 Zn/H20
CH3CH=CH2+ O3 ► CH3—CH I t
CH2 -ZnO
► CH3—C=0 + H c=o
196-200 K r I
Propenc I \ Ethanal Methanal

ee
(reduclive
(Propylene) Ot^O cleavage) (Acetaldehyde) (Formaldehyde)

F
Propene ozonide

Fr
However, if the ozonides are decomposed only with water, the H2O2 produced during the reaction
oxidises the initially formed aldehydes to the corresponding acids (oxidative cleavage).

for
This two-step conversion of an alkene into an ozonide followed by its reductive cleavage to yield
carbonyl compounds is called ozonolysis.
ur
Importance. Ozonolysis is a versatile method for locating the position of a double bond in an unknown
ks
alkene since no two different alkenes give the same combination of aldehydes and /or ketones. Therefore, this
Yo
method has been extensively used in the past for structure elucidation of alkenes. For example,
oo
(I) O3/CH2Ci2.196-200K
CH3CH2CH == CH2 CH3CH2CHO + HCHO
B

(ji)Zn/H20 Methanal
Bui-l-ene Propanal
re

(I)03/CH2C12,I96-200K
CH3CH = CHCH3 CH3CHO + CH3CHO
(ii)Zn/H20
Bui-2-ene Ethanal (two molecules)
ou
ad

CH3 CH3
Y

(i)O3/CH2Cl2.I96-200K
CH3—C i CH2 («)Zn/H20
CH3—C = 0 + HCHO
Propanone Methanal
nd
Re

2-Methylpropenc

CH
/'CH3 CH3 CH3
Fi

c = c (0 O3/CH2CI2.196-20OK :C = 0 + 0 = C
CH3 CH3 (lOZn/HjO CH3 CH3
2, 3-Dimeihylbut-2-ene Propanone (two molecules)

SUPPLEMENT YOUR
KNOWLEDGE FOR COMPETITIONS

1. Reduction of ozonides. Instead of Zn/H20 or catalytic hydrogenation (H2/Pd), ozonides can more
conveniently be reduced with dimethyl sulphide, (CH3)2S. A fine jet of (CH3)2S is directly passed through
the ozonolysis mixture, when the ozonide is reduced to the corresponding aldehydes and ketones and
(CH3)2S is itself oxidised to dimethyl sulphoxide (DMSO).
HYDROCARBONS 13/45

0
O
R /-\ .R
\ CH.,\ R R

yZ C\ +
/S c=0 + o=c +
CH3—s—CH3
H H CH3^ H
Aldehydes
H
Dimethyl sulphoxidc
0 O
Dimethyl sulphide
Ozonide

Since DMSO is water soluble and excess of (CH3)2S being volatile simply evaporates, therefore, the
products of ozonolysis, i.e., aldehydes and ketones are obtained almost in pure state.
2. Lemieux reagent is an aqueous solution of sodium periodate (Nal04) and a trace of KMn04 and is used

w
for oxidation of alkenes. In fact, it is a better method than ozonolysis for determining the position of
double bond in an alkene and for preparing carbonyl compounds. With this reagent, the alkene is first
oxidised to c/i'-l, 2-diol which is then cleaved by periodate to aldehydes and/or ketones. Aldehydes thus
produced are further oxidised by KMn04 to acids. The reaction proceeds at room temperature and the

Flo
manganese obtained in the lower oxidation state is reoxidised to permanganate and hence only a trace of
KMn04 is needed. Thus,

ee
KMnOj NaI04 KMnOj
/ “
RCH = CHR > R—CH^CH—R' -> [R—CH = O + 0 = CHR']

Fr
I (1,2-Glycol splitting)
Alkene Aldehydes
OH OH

cis-l, 2-Diol RCOOH + HOOCR'


Carboxylic acid

for
ur
3. AUylic halogenation. When alkenes (except ethylene) are heated with CI2 or Br2 at high temperature or
in presence of light, the hydrogen atom at the allylic carbon {i.e., carbon next to the double bond) is
substituted by a halogen atom forming allyl halides. For example. ks
Heat or light
CH3—CH = CH2 +CI2 ^ Cl—CH2—CH = CH2 + HCl
Yo
Propene 3-Ch1oroprop-l -eiie
oo
{Allyl chloride)

Heat or light
eB

CH3—CH = CH2 +Br2 4 Br—CH2—CH =CH2 + HBr


Propene 3-Bromoprop-1 -ene
Such reactions in which halogenation occurs at the allylic position of an alkene are called allylic
halogenation reactions.
r
ou
ad

m pj 1^ r^p b I em' D A hydrocarbon containing two double bonds on reductive ozonolysis gave
glyoxal, ethanal and propanone. Give the structure of the hydrocarbon along with its lUPAC name.
Y

Solution. Step 1. To write the structure of the products of ozonolysis with their carbonyl groups facing each othe.r
H H H CH,
nd
Re

I I
CH3—C=0 0 = C—C = 0 0=C—CH3
Ethanal Glyoxal Propanone
Fi

Step 2. 7b write the structure of the hydrocarbon.


Remove oxygen atoms from each of the three carbonyl compounds and connect them by double bonds, we have,
H H H CH3
CH,—C = C—C = C—CH,
6 5 4 3 2 1

2-Melhylhcxa-2, 4-dicne
Thus, the given hydrocarbon is 2-methylhexa-2, 4-diene.

Sample;F*rpblem Give the structures of the compounds which on reductive ozonolysis give :
(i) propane*!, 3>dial (ii) glyoxal and formaldehyde (Hi) acetaldehyde, formaldehyde and carbon dioxide.
13/46 “PnaxCeefi. 4 New Course Chemistry (XI) i:*gT!ltn

Solution. (/) Since reductive ozonolysis gives only one product, i.e., propane-1,3-dial, therefore, the compound
must be a cyclic alkene, i.e., cyclopropene.

{/) O3
♦ 0=HC—CH2—CH=0
(//) ZxJUiO
Propane-1,3-dial
Cyclopropene
(//) Since two products, i.e., glyoxal and formaldehyde are obtained, therefore, the compound must be acyclic.
Further, since glyoxal (OCH—CHO) contains two aldehyde groups, therefore, on either side, there must a = CH2
group. Thus, the compound is 1, 3-butadiene.

w
(003
CH2 CH—CH CH2 ■> HCHO + OCH—CHO + HCHO
(«)Zn/H20
1, 3-Butadiene Formaldehyde Glyoxal

F lo
(Hi) Formation of CO-, indicates that on either side of this carbon, there is a double bond. Since CH3CHO
and HCHO are the two aldehydes obtained, therefore, this carbon is attached to CH3CH = group on one side and
CH2 = group on the other side. Therefore, the compound is 1, 2-butadiene.

ee
(003
CH3CH i C =; CH2 > CH3CHO + 0 = C = 0 + HCHO

Fr
<iV)Zn/H20
1. 2-Butadiene Glyoxal Formaldehyde

for
ur
1. An alkene with molecular formula C7H14 gives propanone and butanal on ozonolysis. Write down its
structural formula.
s
2, The reductive ozonolysis of an alkene gave butanone and ethanal. Give the structure and lUPAC name of
k
Yo
the alkene.
oo
3. An unknown alkene on reductive ozonolysis gives two isomeric carbonyl compounds of molecular formula,
C3H^0. Write the structures of the alkene and the two isomeric carbonyl compounds.
eB

ANSWERS

1. CH3CH2CH2CH = C(CH3)2, 2-methyIhex-2-ene 2. CH3CH2(CH3>C = CHCH3, 3-methylpent-2-ene


3, (CH3>2C = CHCH2CH3 (2-methyipent-2-ene), (CH3)2C = O (propanone), CH3CH2CH = O (propanal)
r
ou
ad

in. Polymerization Reactions of Alkenes


Polymerization is a process in which a large number of simple molecules combine to form a big
Y

molecule. The simple molecules are called monomers while the big molecule is called a
macromolecule or a polymer.
nd
Re

Polymers are extremely useful products. These have revolutionized our everyday living. For example,
polymers are u.sed as plastic, textile, rubber and in many other industries.
Fi

Alkenes readily undergo polymerization in the presence of catalysts,


(i) Polymerization of ethene. When ethene is heated to 473 K under a pressure of 1500 atmospheres
and in presence of a trace of oxygen (O-OOl- 0-1 %), it undergoes polymerization to form polythene.
473K,1500atm
n CH2 = CH2 > -^CH2~CH2^
Traces of oxygen
Polythene
It is widely used as a packaging material (in the form of thin plastic films, bags, etc.) and as insulation
making squeeze bottles, refrigerator trays, toys, pipes, radio
for electric wires and cables. It is also used for
and T.V. cabinets, etc.
(ii) Polymerization of propene. Polypropene or polypropylene is prepared either by heating proplyene
in presence of a trace of benzoyl peroxide as radical initiator or by Ziegler-Natta polymerization.
HYDROCARBONS 13/47

CH3
Peroxides, A
^ CH--
n CH3—CH = CH2 or Ziegler-Natta catalyst L i Jn

Propene Polypropene

It is a harder and stronger polymer than polythene and is used for making milk crates, plastic buckets,
automotive mouldings, seat covers, carpet fibres, ropes, etc., containers for oils and gasoline,
(iii) Polymerization of substituted ethenes. A number of polymers can be obtained by using substituted
ethenes as monomers in place of ethene. For example.

w
Polymerisation Polymerisation
nCH2 =CH «CH=CH^ > -^CH—
Cl
Cl
C6H5 C6H5

Flo
Chloroethene Polyvinyl chloride (PVC) Styrene Polystyrene
(iVnyl chloride)
Polymerisation

e
Polymerisation
.CH2—CH ^ ; n CF2 = CF2 > -eCF2-CF2-^
«CH2 =CH >

re
CN
Tetrafluoroethene Polytetrafluoroethene
CN M

rF
(PTFE, Tejlon)
Acrylonitrile Polyacrylonitrile (PAN)
(iVnyl cyanide)
ur
Polyvinyl chloride (PVC) is used for making plastic bottles, syringes, pipes, raincoats, records, etc.

fo
Polyacrylonitrile is used for making Orion and Acrilan fibres used for making clothes, carpets and blankets.
Polystyrene is used for packing and for making toys and household goods. Polytetrafluoroethene or teflon is
resistant to the action of acids and other chemicals. It is used in the manufacture of pipes and surgical tubes.
ks
Because of its great chemical inertness and high thermal stability, teflon is also used for making non-stick
Yo
utensils. For this purpose, a thin layer of teflon is coated on the inner side of the vessel.
oo
13.6.9. Uses of Alkenes
B

(/) Lower members of the family are used as fuels and illuminanls.
(ii) Alkenes and substituted alkenes upon polymerization form a number of useful polymers such as
re

polythene, PVC, teflon, orlon, etc.


(iii) Ethene is employed for the preparation of ethyl alcohol and ethylene glycol (anti-freeze).
u

(iv) Ethylene is used for artificial ripening of green fruits,


ad
Yo

(v) Ethylene is also used in oxygen-ethylene flame for cutting and welding of metals.
(vi) Mustard gas is a war gas and was used in World War-I. It is obtained by the reaction of ethylene
with sulphur monochloride.
nd

CH2 Cl Cl CH2 CH2CI CH2CI


Re

+ i -I- + S
CH2 s—s
CH2 CH2—S—CH2
Fi

Ethylene Sulphur Ethylene Mustard gas


monochloride (p, ^'-Dichlorodielhyl sulphide)
PART—III. ALKYNES

13.7. ALKYNES

Acyclic unsaturated hydrocarbons containing a carbon-carbon triple bond are called alkynes or
acetylenes. Their general formula is C„H2„_2 where n = 2, 3, 4.... etc.
13.7.1. Structure of IVIple bond
Ethyne is the first member of alkyne series. Its structure is shown Fig. 13.14.
Each carbon atom of ethyne is sp-hybridized and hence has two j'p-hybridized orbitals. One jp-hybridized
orbital of each carbon undergoes head on overlap with sp- hybridized orbital of another carbon to form a sp-sp,

I
13/48 New Course Chemistry (XI)EZSI9D

C—C, o-bond. The second i'p-hybridized orbital FIGURE 13.14


of each carbon overlaps along the intemucleiir tixis
sp-s, sp-s,
with l^-orbital of each of the two hydrogen atoms C - H.ct'hond C - H,0’*bond
forming two .s/7-s, C—H, o-bonds. Each carbon
Is, sp sp ,, Is
is now left with two unhybridized p-orbitals {2p^
and 2pp which are perpendicular to each other
as well as to the plane of the C—C sigma bond.
0>c^X>^33
G G

The two 2/?^-orbitals, one on each carbon, sp-sp, C - C


are parallel to each other and hence overlap a-bond
sideways to form a Tt-bond. Similar overlap
Below the plane

w
between 2/?^,-orbitals, one on each carbon, results In the plane of the paper of the paper
in the forma- tion of a second 7i-bond as shown in 2Px 2Px
7t 4
Fig. 13.14(b).
2Py
Since a p-orbital has two lobes, the electron o

F lo
Tt

cloud of a 7C-bond has two halves. If the two halves


of one TZ- bond (formed by overlap of 2/?^-orbitals) C‘

are considered to lie in the plane of the paper, Tt

ee
then one of the two halves of the second 7C-bond n

(formed by overlap of 2/7y-orbitals) would lie 2Py

Fr
above the plane of the paper and the other below
Above the plane
the plane of the paper as shown in Fig. 13.14(b). of the paper
In the plane of the paper
However, the four halves of the electron clouds

for
of two 7c-bonds do not stay as such but merge
ur
together to form a single electron cloud which
© y
has cylindrical symmetry about the intemuclear H C J
H
axis as shown in Fig. 13.14(c).
\
s
w
ook
It is because of this cylindrical symmetry of
Yo
the electron cloud between two carbon atoms that
Orbital picture of ethyne showing (a) sigma overlaps
ethyne is a linear molecule with H—C—C bond
(b) Tt'Overlaps (c) cylindrical nature of electron cloud
eB

angle of 180°.
From the above di.scussion, it follows that a carbon-carbon triple bond
consists of one strong o-bond and two weak n- bonds. The total strength of
120 pm 107-9 pm
C s C bond in ethyne is 823 kJ mol"'. It is stronger than C = C bond of ethene H €■ , C H
r

(599 kJ mol"*) and C—C bond of ethene (348 kJ mol“^). Further, due to the
ad
ou

smaller size of .syj-orbitals (as compared to and sp^~) and sideways overlap
180°
of/?-orbitals, the carbon-carbon bond length in ethyne is shorter (120 pm) than
those of C = C (134 pm) and C—C (154 pm).
Y

In spite of the presence of two K-bonds, alkynes are less reactivethan alkenestowards additionreactions.
Further, alkynes unlike alkenes do not exhibit geometrical isomerismdue to their linear structure.
Re
nd

13.7.2. Nomenclature of Alkynes


In common system, alkynes are named as derivatives of acetylene. In the lUPAC system, they are called
Fi

alkynes. The name of any alkyne is obtained by replacing the terminal ane of the corresponding alkane by the suffix
yne. The position of triple bond is indicated by arabic numbers ; the numbering being started from that end which
is closer to the triple bond. The common and lUPAC names of some alkynes are given Table 13.3.
TABLE 13.3.1 Common and lUPAC names of allies (C„H2q.2)
Value of n Formula Structure Common name lUPAC name

2 C2H2 HC^CH Acetylene Ethyne


3 C3H6 CHy sCH Methylacetylene Propyne
4
C4Hg CH3CH2C s CH Ethylacetylene But-1- yne
C4H« CH3C = CCH, Dimethylacetylene But-2- yne

I
HYDROCARBONS 13/49

For further illustration, names of some substituted alkynes are given below :
CH,
4
3I 2 1 CH2 3 2 I

CH3 = CH C^Hj—C = CH CH—CH2—C s CH


CHv
3-Methylbut-l-yne Elhynylbenzene 3-Cyclopropylprop-1 -yne
(Phenylacetylene)
If both double and triple bonds are present and if there is a choice in numbering, the double bond is
always given preference. For example.
1 2 3 4 5 6

CH2=CH—CH2—CH2—c S CH

w
Hex-l-en-5-yne
However, if there is no choice in numbering, lowest set of locants rule is followed. For example,

F lo
5 4 3 2 I 2 3 4 5

CH3—CH = CH—C s CH CH3—CH = CH—C = CH


Pent-3-en-l-yne ( correct) Peni-2-en-4-yne (wrong)

13.73. Isomerism In Alkynes

ee
Alkynes show four types of isomerism as discussed below :

Fr
(0 Position isomerism. The first two members, i.e., ethyne and propyne exist in one form only. But
butyne and higher alkynes exhibit position isomerism due to different positions of the triple bond on the

for
carbon chain. For example,
CH3—CH2- = CH CH3—C s C—CH3
our
Bui-l-yne But-2-yne
(if) Chain isomerism. Alkynes having five or more carbon atoms show chain isomerism due to different
s
structures of the carbon chain. For example.
ook
CH.
4 3I 2 1
Y
eB

CH3—CH2—CH2—C s CH CH3—CH—C = CH
Pent-1-yne 3-Methylbut-l-yne
(Hi) Functional isomerism. Alkynes are functional isomers of dienes, i.e., compounds containing two
double bonds. For example.
our
ad

4 3 3 1 1 2 3 4 1 4

CH3—CH2—C = CH CH2=CH—CH = CH2 CH2 = C = CH —CH3


But-1-yne Buta-1, 3-dicne Bul-1, 2-diene
Y

(iv) Ring chain isomerism. Alkynes show ring chain isomerism with cycloalkenes. For example,
Re
nd

CH3—C^CH and
are ring chain isomers.
Propyne
Fi

Cyclopropene

13.7.4. Classification of Alkynes


Alkynes are further classified as terminal or non-terminal alkynes according as the triple bond is present
at the end of the carbon chain or within the carbon chain. For example,

Terminal alkynes: CH3C = CH CH3CH2C H CH


Propyne But- 1-yne

Non-terminal alkynes : CH3—C = C—CH3 CH3 s C—CH2CH3


But-2-yne Peni-2-yiie

I
13/50 ^ New Course Chemistry (XI) LVJWtl

13.75. Methods of Preparation of Alkynes


Alkynes are prepared by the following general methods.
1. By the action of water on calcium carbide. Ethync (acetylene) is prepared in the laboratory as well
as on a commercial scale by the action of water on calcium carbide.
CaC2 + 2H2O HC = CH + Ca(OH)2
Calcium carbide Ethyne {Acetylene)
Calcium carbide needed for the purpose is manufactured by heating limestone (calcium carbonate) with
coke in an electric furnace at 2275 K.

w
2275 K 2275 K
CaCOj ■> CaO + CO2 CaO + 3C CaC2 + CO
Procedure. Lumps of calcium carbide are placed on a layer of sand in a conical flask fitted with a
dropping funnel and a delivery tube (Fig. 13.15). The air present in the flask is replaced by oil gas since

Flo
acetylene forms an explosive mixture with air. Water is now dropped from the dropping funnel and the
acetylene gas thus formed is collected over water.

ee
FIGURE 13.15

WATER

Fr
r/

for
C
ur
ACETYLENE-
WZ3T
ks
CALCIUM
Yo
CARBIDE
oo

)
eB

SAND ACIDIFIED BLEACHING POWDER


CUSO4 SOL. SUSPENSION
r

Laboratory preparation of acetylene


ou
ad

Purification. Acetylene gas prepared by the above method contains impurities of hydrogen sulphide
and phosphine due to the contaminations of calcium sulphide and calcium phosphide in calcium carbide.
Y

Hydrogen sulphide is removed by bubbling the gas through an acidified solution of copper sulphate while
phosphine is removed by passing the gas through a suspension of bleaching powder. Pure acetylene is finally
Re
nd

collected over water.

2. By dehydrohalogenation of dihaioalkane.s. Alkynes are prepared by dehydrohalogenation of vicinal-


Fi

dihaloalkanes by heating them with an alcoholic solution of potassium hydroxide. For example,
CH2—Br CH
+ 2KOH(n/c.-.) — + 2KBr + 2H2O
CH2—Br CH

I, 2-Dibromoethane Acetylene
{Ethylene dibromide)
The reaction, in fact, occurs in two steps and each step involves tlie loss of a molecule of HBr as shown below :

B1CH2—CH2Br + KOH (ale.) ^ CH2 = CHBr + KBr + H2O


Ethylene dibromide Vinyl bromide

I
HYDROCARBONS 13/51

A
CH2 = CH—Br + KOH (ale.) » CH = CH + KBr + H2O
Vinyl bromide Acetylene
These two steps occur at different rates as explained below :
In ethylene dibromide, Br is present on a saturated carbon atom. Therefore, like alkyl halides, it is a
reactive molecule. Consequently, on heating with alcoholic KOH, it readily eliminates a molecule of HBr to
form vinyl bromide in good yield. In contrast, due to the presence of Br on a doubly bonded carbon atom,
vinyl bromide is a highly unreactive molecule and hence on heating with alcoholic KOH, it does nui easily
lose a molecule of HBr to form acetylene. Thus, with alcoholic KOH, the yield ofacetylene is low. Therefore,
to obtain acetylene in fairly good yield from vinyl bromide, a much stronger base than alcoholic KOH such as

w
NaNH2 in liquid NH3 is usually used. Thus, dehydrohalogenation of ethylene dibromide to acetylene is
preferably carried out in the following two stages.
H H H

F lo
^ I
H—C^C—H
KOH (ale.), A
H—C^C—H
fi NaNH2/iiq. NH3 ^
H—C=C—H
-HBr
>
-HBr ^
I
Br Br'
rj dBr Acetylene

ee
Ethylene dibromide Vinyl bromide

Fr
Instead of carrying out the above dehydrohalogenation in two steps, the reaction is usually carried out in
one step using NaNH2 in liquid NH3.

for
Liquid NH3
BrCH2—CH2Br + 2NaNH2 1%K
^ CHsCH + 2NaBr + 2NH3
our
1,2 - Dibromoethane Acetylene

Liquid NH3
s
CH,—CH—CH, + 2NaNH2 ^ CH3—C = CH + 2NaBr + 2NH3
3 I ,2
ook
196 K
Propyne
Br Br

1,2-Dibromopropane
Y
eB

Alkynes can also be prepared from gem-dihalides by the action of alcoholic KOH followed by treatment
with NaNH2 in liquid NH3 or preferably by the action of sodamide in liquid NH3. For example.
KOH(ofc),A NaNH2/liq.NH3
r

CH3—CHBr2 ' ^ CH2 = CHBr ^ HC = CH


ad
ou

-HBr 196 K
1, 1-Dibromoethane Vinyl bromide (-HBr) Acetylene ,

Liquid NH3
Y

or CH3—CHBr2 + 2 NaNH2 196 K


^ HCsCH + 2NaBr + 2NH3
1, l-Dibromoethane Acetylene
Re
nd

3. By dehalogenatioii of tetrahalides. Tetrahaloalkanes when heated with zinc dust in methanol undergo
dehalogenation to yield alkynes. For example.
Fi

Br Br
I CH3OH
H—C—C—H + 2Zn
A
H—C = C—H + 2ZnBr2
I I Acetylene
Br Br
1,1,2,2- Tetrabromoethane
4. By dehalogenation of haloforms. Chloroform and iodoform on heating with silver powder undergo
dehalogenation to form ethyne.

CHjCl3_-l_6_Ag_+£l3_jCH ● ^ HC = CH-H6AgCl
r n

Chloroform Chloroform Ethyne

A A
13/52 New Course Chemistry (XI)E!Z

CH[I3J- j^Ag_-|_I3]CH ^ HCsCH + 6AgI


Iodoform Iodoform Ethyne
5. Kolbe’s electrolytic reaction. Acetylene can be prepared by electrolysis of a concentrated solution
of sodium or potassium salt of maleic acid or fumaric acid. Thus,
H COOK H COOK
\/ C
c Electrolysis CH
II
C
or
C + 2H2O CH
+ 2CO2+H2 + 2KOH
/\ / \
Acetylene

w
H COOK KOOC H
Pot. maleate Pot. fumarate
(cw-isomer) (rran5-isomer)

F lo
This reaction is called Kolbe’s electrolytic reaction and is believed to occur by the following steps :
CHCOOK lonization CHCOO-
4 + 2K+ 2H2O V i 20H- + 2H+
CHCOOK

ee
CHCOO-

CHCOO

Fr
CHCOO" CH
At anode: -2 c- ■>
+ 2CO2
CHCOO
CHCOO- CH
{unstable)

for
At cathode : 2H+ + 2C- >[2H\ >H2
ur
6. Synthesis from carbon and hydrogen. Acetylene can be prepared by passing a stream of hydrogen
through an electric arc struck between carbon electrodes.
Electric arc
s
2C + H2 HCsCH
ok
Yo
3270 K
Acetylene
7. Synthesis of higher alkynes from acetylene. Acetylene is first treated with sodium metal at 475 K or
o
eB

with sodamide in liquid ammonia at 196 K to form sodium acetylide. This upon treatment with alkyl halides
gives higher alkynes. For example,
Liq.NH3
HCsCH + NaNH2 ■»
HC = C“Na+ + NH3
r

196 K
ad
ou

Ethyne Sodium acetylide


(Acetylene)
Y

HC^C-Na"" + CH3 Br > HC=C—CH3 + NaBr


Re

Sod. acetylide Bromomethane Propyne


nd

_ c\
Fi

HC=C"Na'" HC=C—CH2CH3 + Nal


Sod. acetylide lodoethane But-l-yne

HC^C-Na"" _ fv > HC=C—CH2CH2CH3 + NaBr


Sod. acetylide 1-Bromopropane Pent-l-yne
(n-Propyl bromide)

2NaNH2 2CH,I
CH=CH ^ Na+-CsC"Na+ ^ CH,—CsC—CH,
-2NH3 -2NaI
Acetylene Disodium acetylide But-2-yne

k n
HYDROCARBONS 13/53

13.7.5.1. Manufacture of Acetylene


Acetylene is manufactured by the following processes.
(/) From calcium carbide.
(ii) From paraffin hydrocarbons (natural gas, LPG, naphtha, fuel oil, even crude oil) by pyrolysis (WulfT
process).
(Hi) From natural gas by partial oxidation (Sachasse process).
Sachasse process. Now a days, acetylene is mainly manufactured by the partial oxidation of natural gas.
1873 K
2 CH.^ ■ OOOl-OOlseconds
■■■ 2 HC = CH + 3 H. ; AH = 333-8 kJ

w
Natural gas (1 mole) and 95% O2 (0-65 mole) are preheated separately to 783 K imd feed into a specially
designed converter.
The converter is a cylindrical unit built in three sections :

F lo
(0 Mixing chamber, (ii) Rame room and (Hi) Quench chamber.
After rapid and thorough mixing of O2 and CH4 in the mixing chamber, the gases are led to flame
chamber. The heat of combustion heats the gases to 1823 K to allow cracking of the excess of methane to

ee
acetylene. The residence time is 0-001 to 0-01 seconds. The decomposition of acetylene is prevented by
rapid quenching of the resulting gases with water to 311 K. The cooled effluent gases on the dry basis contain

Fr
8% acetylene, 54% H2, 26% CO, 5% CH4,4% CO2 and 3% N2 and higher acetylenes. These gases are run to
a filter where using carbon black, acetylene of 99-5% or higher purity is produced.

for
13.7.6. Physical Properties of Alkynes
Hydrocarbon Ethane Ethene Ethyne
ur
(/) Physical state. The first three members
of this family (ethyne, propyne and butyne) are m.p. (K) 101 104 191
colourless gases, the next eight are liquids while
s
b.p. (K) 184-5 171 198
the higher ones are solids.
ook
Yo
(ii) Smell. All the alkynes are odourless. However, acetylene has garlic smell due to the presence of
phosphine as impurity.
eB

(Hi) Melting and boiling points. The melting points and boiling points of alkynes are slightly higher
than those of the corresponding alkenes and alkanes. This is probably due to the reason that because of the
presence of a triple bond, alkynes have linear structures and hence their molecules can be more closely
our

packed in the crystal lattice as compared to those of corresponding alkenes and alkanes.
ad

(/v) Solubility. Alkynes like alkanes and alkenes being non- polar are insoluble in water but readily
dissolve in organic solvents such as petroleum ether, benzene, carbon tetrachloride,etc.
(v) Density. Densities of alkynes like those of alkenes and alkanes increase as the molecular size increases.
Y

However, they are all lighter than water since their densities lie in the range 0-69 - 0-77 g/cm^.
Re
nd

13.7.7. Reactivity of Alkynes versus Alkenes


Although alkynes contain two 7C-bonds, yet they are less reactive than alkenes towards electrophilic
Fi

addition reactions. This is supported by the observation that a majority of the addition reactions of alkynes
are catalysed by heavy metal ions such Hg^'*’, Ba^'*’, etc. while no such catalysts are needed in case of
electrophilic additions to alkenes. This lower reactivity of alkynes as compared to alkenes towards electrophilic
addition reactions is due to the i lowing two reasons :
(/) Due to greater electronegativity of ^p-hybridized carbon atoms of a triple bond than .9p^-hybridizcd
carbon atoms of a double bond, the n-electrons of alkynes are more tightly held by the carbon atoms than k-
electrons of alkenes and hence are less easily availablefor reaction with electrophiles. As a result, alkynes
are less reactive than alkenes towards electrophilic addition reactions,
(ii) Due to cylindrical nature of the 7C-electron cloud of alkynes, the n-electrons of a triple bond are
more delocalized that n-electrons of a double bond. In other words, the n-electrons of a triple bond are less

I
13/54 ‘Pn/tdec^'<i New Course Chemistry (X1)E!2SZSD

readily available for addition reactions than those of the double bond. Consequently alkynes are less reactive
than alkenes.

13.7.8. Chemical Reactions of Alkynes


Alkynes show acidic character, electrophilic and nucleophilic addition reactions, reduction, oxidation
and polymerizationreactions as discussed below ;
1. Acidic character of Alkynes
Unlike alkanes and alkenes, the hydrogen atoms attached to the triple bond of the alkynes, i.e., acetylenic
hydrogens are acidic in nature. This is shown by the following three reactions:

w
(/) Formation of alkali metal acetylides. Ethyne and other terminal alkynes (alkynes in which the triple
bond is at the end of the carbon chain) or 1-alkynes react with strong bases such as sodium metal at 475 K or
sodamide in liquid ammonia at 196 K to form sodium acetylides with evolution of H2 gas.

Flo
475 K
2HC = CH +2Na 4
2CHsC-Na-" + H2
Ethyne Monosodium ethynide

ee
{Acetylene) {Monosodium acetylide)

Fr
Liq.NHj
R—C = CU + NaNH2 196K
^ R—CsC-Na++ NH3
(A terminal alkyne) (/4 sod. alkynide)

for
During these reactions, the acetylenic hydrogen is removed as a proton to form stable carbanions
(acetylide ions).
ur
Sodium acetylide is decomposed by water regenerating acetylene. This shows that water is a stronger
acid than acetylene and thus displaces acetylene from sodium acetylide. s
HC = C“ Na+ + H2O 4 HC = CH + NaOH
k
Yo
Monosodium acetylide Acetylene
oo
(h) Formation of heavy metal acetylides. Acetylenic hydrogens of alkynes can also be replaced by
eB

heavy metal ions such as Ag"*" and Cu'*’ ions. For example, when treated with ammoniacal silver nitrate solution
(Tollens’ reagent), alkynes form white precipitate of silver acetylides.
CH s CH + 2 [Ag(NH3)2]-" OH" ●>
AgCsCAg + 2H2O + 4NH3
r

Ethyne Tollens’ reagent Disilver ethynide


ou
ad

{While ppl.)
R—CsCH + [Ag (NH3)2f OH- ^ R_C = C-Ag + H2O + 2NH3
Y

{Terminal alkyne) Tollens’ reagent Silver alkynide


{White ppl.)
Similarly, with ammoniacal cuprous chloride solution, terminal alkynes form red ppt. of copper acetylides.
Re
nd

HC = CH + 2 [Cu(NH3)2]'' oh 4 CuC = CCu + 2H2O + 4NH3


Dicopper ethynide
Fi

Ethyne
{Red ppt.)

R—CsCH + [Cu(NH3)2r OH“ 4 R—ChC—Cu +H2O + 2NH3


{Terminal alkyne) Monocopper alkynide
{Red ppl.)
Unlike alkali metal acetylides, silver and copper acetylides are not decomposed by water. They can,
however, be decomposed with dilute mineral acids to regenerate the original alkynes.
AgC = CAg + 2HNO3 4 HCsCH + 2AgN03
Disilver acetylide Acetylene
CuC^CCu + 2HC1 ■ 4 HCsCH + 2CuCl

Dicopper acetylide Acetylene

I
HYDROCARBONS 13/55

(in) Formation ofalkynyl Grignard reagents. Acetylene and other terminal alkynes react with Grignard
reagents to fonn the corresponding alkynyl Grignard reagents. For example,
Dry
HC = CH + RMgX ether
^ HC = CMgX + RH
Acetylene Grignard reagent Acetylenic Alkane

Grignard reagent
Dry
R'—CsCH + RMgX R'—CsCMgX + RH
ether
Terminal alkyne Grignard reagent Alkynyl Grignard reagent
Alkynyl Grignard reagents like usual Grignard reagents can be used to prepare a variety of organic

w
compounds.
Importance. The formation of metal acetylides can be used :
(0 for separation and purification of terminal alkynes from non-terminal alkynes, alkanes and alkenes.

F lo
(ii) to distinguish terminal alkynes from non- terminal alkynes or alkenes.
Cause of Acidity of Alkynes. The acidity of alkynes can be explained in terms of the 5p-hybridization
of a triply bonded carbon. We know that an electron in an 5-orbital is more tightly held than in a p-orbital
since s- electrons are closer to the nucleus. Further, since an sp- orbital has more 5-character than an sp^-or an

ee
5/7^-orbilal, therefore, the electrons in an 5/?-orbital are more strongly held by the nucleus than electrons in an

Fr
sp^- or a 5p^-orbital. In other words, an sp- hybridized carbon is more electronegative than sp^- or sp^-
hybridized carbon atom. Due to this greater electronegativity, the electrons of C—H bond are displaced more
towards the carbon than towards the hydrogen atom. In other words, the hydrogen atom is less tightly held by

for
the carbon and hence can be removed as a proton (H'*') by a strong base. Consequently, alkynes behave as
ur
acids.

Further, since the 5-character decreases as we move from sp- to sp^- to 5/?^-carbon atoms, the acidic
character of hydrocarbons decreases in the following order:
s
ook
CHsCH > CH2 = CH2 > CH3—CH3
Yo
K„a 10-25 ^ 10-^5 ^ ^ 10-40
It may be noted here that hydrocarbons are very weak acidsas compared to H2O (K^ = 10“*'^), alcohols
eB

(Kjj = 10“'^ - 10"*^) and carboxylic acids (K^ = 10'^). For example, ethyne (acetylene) is 10^® times and
ethane is about 10^^ limes less acidic than ethanoic acid.
our

SUPPLEMENT YOUR
ad

KNOWLEDGE FOR COMPETITIONS

Relative acidities of water, alcohols, acetylene, ammonia and alkanes can be determined as follows :
(a) NaNH2 reacts with acetylene to form sodium acetylide and NH3. Since a stronger acid displaces a
Y

weaker acid from its salts, therefore, acetylene is a stronger and than NH^
HCsCH + NaNH2 > HC = C-Na+ + NH3
Re
nd

(stronger acid) (weaker acid)


(b) Since lithium alkyls (RLi) react with ammonia to form alkanes, therefore, ammonia is a stronger acid
Fi

than a hydrocarbon (RH)


NH3 + RLi 4 R_H + LiNH2
(stronger acid) (weaker acid)
(c) Since H2O and alcohols decompose sodium acetylide to give back acetylene, therefore, water and
alcohols are stronger acids than acetylene
HCsC'Na+ -I- H2O 4 HCsCH + NaOH

(stronger acid) (weaker acid)


HC=C"Na+ + ROH ^ HC = CH + RONa

(stronger acid) (weaker acid)


13/56 ‘Pn<tcUcp-'A. New Course Chemistry fxnpzsnm

(d) Since H2O decomposes sodium alkoxides to form alcohols, therefore, water ix a stronger acid than
alcohols.
RONa + H2O ■> ROH + NaOH
(stronger acid) (weaker acid)
Combining all the facts stated above, the relative acidities of these compounds follow the order:
H2O > ROH > HC = CH > NH3 > RH
Conversely, the relative basicities follow the order: R > NH2 > HC s C > RO > HO

w
Curiosity Questions
[ Q. 1. Acetylene is said to be acidic ? Does it mean that it turns blue litmus red ?

Flo
Ans. In organic chemistry, the term acidity is used in a wider sense. It implies that a particular hydrogen
can be abstracted by a base which may be a very weak or a very strong base. Acetylene is a
weak acid even weaker than HgO and hence it does not turn blue litmus red.

ee
Q. 2. Does acetylene react with Fehling’s solution ?
Ans. No, because Fehling’s solution contains ions which do not react with acetylene. Acetylene

Fr
reacts with only Cu'*’ ions. That is why it gives red ppt. of copper acetylide with ammoniacal
CuCI solution and not with Fehiing’s solution.
J

for
r
n. Electrophilic Addition Reactions
The electrophilic addition reactions of alkynes take place in two stages as shown below :
ou
ks X X
Trans-addition X
= c— + + X2
X2 c = c
■¥ —c—c—
oo
X
I
X X
Y
eB

By a proper choice of reaction conditions, the addition can be stopped after the addition of one mole of
the reagent. Some important electrophilic addition reactions are discussed below :
1. Addition of halogens. Chlorine and bromine readily add to alkynes first forming 1, 2-dihaIoalkenes
r

and then 1, 1,2, 2-tetrahaloalkanes. For example.


ou
ad

Cl Cl
Cl
Y

CI2 CI2
HC^CH C = C H—C—C—H
CCI4 Cl H CCI4
Ethyne
Cl Cl
nd
Re

irans-l, 2-Dichloroethene I, 1, 2, 2-Tetrachloroeihane


(Wesiron)
Fi

Br Br
I I
Bf2 CH3 Br Bfo
CH,—C—C—H
c = c
CH3 sCH
CCI4
●>
^ I I
CCI4 Br H
Propyne Br Br

/rani-1, 2-Dibromopropene 1, I, 2, 2-Tetrabromopropane

During this reaction, the reddish brown colour ofBr2 is decolourised and hence this reaction is used as
a test for unsaturation, i.e., for double and triple bonds.
The reaction of alkynes with iodine occurs with difficulty. However, if the reaction is carried out in
ethanoUc solution, the reaction stops after addition of one molecule of iodine.
HYDROCARBONS 13/57

C2H5OH H
C = C
HC = CH + I2 ■>
H
Ethyne
trans-1, 2-Iodoethene

Thus, the order of reactivity of halogens is :


CI2 ^ ^^2 ^ l2‘
2. Addition of halogen halides or halogen acids. Halogen halides add to alkynes, their order of reactivity
being HI > HBr > HCl > HF ; HF adds only under pressure. The addition of halogen acids can take place in
the dark but is catalysed by light or metallic halides. These additions occur in accordance with Markovnikov’s

low
rule to form first vinyl halides and then alkylidene halides.
HCl HCl
HC = CH >
CH2 = CHC1 CH3—CHCI2
Mark.addn.
Ethyne l-Chloroethene 1,1- Dichloroethane
(iVnyl chloride) (Ethylidene dichloride)

ee
However, when acetylene is passed through dil. HCl at 338 K in presence of Hg^'*’ ions as catalyst, only
vinyl chloride is formed.

rF
Fr
338 K
HChCH + HCl 2+ ^ CH2 = CHC1
Hg
Acetylene Vinyl chloride

for
With hydrogen bromide, first a vinyl bromide and then an alkylidene dibromide is formed. For
example.
ou
HBr
HBr
HC = CH ^ CH2 = CHBr CH3—CHBr2
ks
Mark.addn.
Ethyne Vinyl bromide 1,1- Dibromoethane
CH Br
oo
HBr CH3 HBr

CH3—C = CH C = CH2 Mark.addn.


CHj-^ "^Br
Y
Mark.addn.
eB

Propyne
2-Bromopropene 2, 2- Dibromopropane

However, in presence of peroxides, anti-Markovnikov’s addition of HBr occurs. For example,


r

HBr HBr
ou
ad

CH3C = CH RCOOR > CH3CH = CHBr RCOOR CH3CH2CHBr2


Propyne 1-Bromopropene 1,1 -Dibromopropane
Mechanism. It is an electrophilic addition reaction and occurs in two stages as discussed below :
Y

+
Slow cr
Re
nd

Stage 1: HC=CH + H"" ► CH2=CH >CH2= CHCl


Fast
Ethyne Vinyl cation l-Chloroethene
Fi

+ Slow +
Slow
Stage n : CH 2 CH2CI < X CH2 = CHCl + H+ ^ CH3—CH—Cl
r Carbocation (II) l-Chloroethene 2° Carbocation (I)
(less stable) (more stable)
Since carbocation (I) is stabilized by + R-effect of the Cl atom while carbocation (II) is destabilized by
-I-effect of Cl, therefore, reaction occurs through the more stable carbocation (I) forming 1,2-dichloroethane.

+ +i
Cl
Fast
CH3—CH = Ci:
(Stabilized by
► CH3—CH—Cl + cr
1
► CH3—CH^ Cl
+R-effect of Cl) 1, 1-Dichloroethane
13/58 ‘Pfuxxieep^'^i. New Course Chemistry (XI)ESE

3. Addition of the elements of hypohalous acids. Like alkenes, alkynes also add chlorine or bromine
in presence of water. The overall reaction occurs in two stages, each stage involving the addition of the
elements of the hypohalous acid to form dihalocarbonyl compound as the final product. For example,
5+ 5-
5+ 5- a-OH
HC = CH + Cl—OH 4 [Cl—CH = CH—OH] >

fithyne Hypochlorous acid 2-Chloroeth-1 -en-1 -ol

Cl—CH—CH—OH ■>
Cl\2CH—CH
1
= 0

w
I I -H2O Cl^
Cl OH 2,2-Dichloroethanal
(unstable) (a, a-Dichloroacetaldehyde)

Flo
8+ 8- 6+ 8- OH
a-OH Cl-OH I
Similarly, CHj—CsCH ■> CH,—C = CH ■»
CH3—C—CHCI2
Markaddn. ^ I I Markaddn. I

ee
Propyne OH Cl OH
(unsable)

Fr
o

CH3—C—CHClj

for
ur -H2O
1, 1-Dichloropropanone
ks (a, a-Dichloroacetone)

4. Addition of H2O—^Hydration of alkynes. Alkynes cannot be hydrated as easily as alkenes because


Yo
of their lower reactivity towards electrophilic addition reactions. Further, dilute H2SO4 has no effect on
oo
carbon-carbon triple bond. However, in presence of HgS04 as catalyst, hydration occurs readily. Under these
conditions, H2O adds to the triple bond to form first an enol which readily tautomerises to the corresponding
eB

carbonyl compound. For example.


O
H2S04.HgS04 Tautomerises
HCsCH + H—OH CH = CH
■»
^ CH3—C—H
r

333 K
ou

Ethyne Ethanal
ad

OH H
(Acetylene) (Acetaldehyde)
Vinyl alcohol
Y

(unstable)

In case of unsymmetrical terminal alkynes, addition occurs in accordance with Markovnikov's rule.
d
Re

For example.
n

HO H O
Fi

5+ 6- 5+ 5- H,S04* HgS04 I I
Tautomerises
CH3—C =CH+H—OH 333 K CH3—C=CH 4 CH3—C—CH3
(Marhaddn.) (unstable)
Propyne Propanone
(Terminal alkyne) (Acetone)

However, if the unsymmetrical alkyne is non-terminal^ a mixture of two isomeric ketones is obtained in
which the methyl ketone predominates. For example.
o o
H2S04/HgS04
CH3CH2—C = C—CH3 333 K
^ CH3CH2CH2—C—CH3 + CH3CH2—C—CH2CH3
Pent-2-yne Pentan-2- one (major) Pentan-3-one (minor)
HYDROCARBONS 13/59

SUPPLEMENT YOUR
KNOWLEDGE FOR COMPETITIONS

Mechanism of hydration of alkynes. Hydration of alkynes is an electrophilic addition reaction and is


believed lo occur in two steps. In the first step, the forms a complex with the triple bond which is
then attacked by water to form an intermediate (I)

2+ Slow
Step I. CH3—C=CH + Hg” ► CH3—C=C—H I
Propyne V I

w
u„2+
Hg
Complex

Flo
1 Fast
> H
0 HO. H

Step 2. CH3—C=_CH + HjO: I


:c=CH :c=c
-H
Hg"

ee
CH3 CH3-
2+ Hg" I
g

Fr
Protonation of intermediate (I) followed by loss of Hg”'*' gives the enol (TI) which subsequently readily
tautomerises to give the corresponding carbonyl compound
HO. H HO. H

or
4-
ur CH{ X->Hg^ Hg“
2+
:c=c

sf
H
CH3'
I II

O
k
Yo
Taulomerises
oo
♦ CH3—C—CH3
Propanone
B

5. Addition of carboxylic acids. When acetylene is passed into warm acetic acid in presence of mercury
re

salts, first vinyl acetate and then ethylidene diacetate is formed.


2+
Hg CH3COOH
HCsCH +CH3COO—H ^ H2C = CH—OCOCH3 ^ CH3—CH(0C0CH3)2
ou
ad

353 K Hg2-
Acetylene Acetic acid Vinyl acetate Ethylidene diacetate
Y

Vinyl acetate is used for manufacture of vinyl resin. Ethylidene diacetate, when heated rapidly to
573 - 673 K, gives acetic anhydride and acetaldehyde.

OCOCH3 CH3—COv,^ O
nd

573-673K
Re

CH3—CH + CH3CHO
OCOCH3 CH3—CO Acetaldehyde
Fi

Ethylidene diacetate Acetic anhydride


6. Addition of hydrogen cyanide. Acetylene adds on hydrogen cyanide in presence of Ba(CN)2 or
CuCl in HCl as catalyst to give vinyl cyanide or acrylonitrile.
BatCN)-,
HCsCH + HCN ■>
CH2 = CH—CN
orCuCl/HCl
Acetylene Vinyl cyanide or Acrylonitrile
Acrylonitrile is widely used for manufacture of Orion ifbres and synthetic rubber such as Buna- N.
III. Nucleophilic Addition Reactions
Because of the greater electronegativity of the sp-hybridized carbons as compared to 5p^-hybridized
carbons, alkynes are more susceptible to nucleophilic addition reactions than alkenes.
13/60 7\<^cCeefo^'4, New Course Chemistry (Xl)SSi

For example, when acetylene is passed into methanol at Ayh-AlZ K in presence of a small amount (1—
2%) of potassium methoxide under pressure, methyl vinyl ether is formed.
CH3O- K+
HCsCH + CH3O—H 433-473K
■> CH2 = CH—OCH3
Acetylene Methanol Methyl vinyl ether
Methyl vinyl ether is used for making polyvinyl ether plastics.
IV. Reduction of Alkynes.
Addition of dihydrogen to alkynes in presence of nickel at 523—^573 gives alkanes.

w
Ni Ni
HC = CH + H2 523-573K
^ CH2 = CH2 523-573K
4
CH3 —CH3
Ethyne Ethene Ethane

The heat of hydrogenation data reveal that the first step has larger AH® than the second step, thereby

Flo
indicating that alkynes are more readily reduced than alkenes.
Catalyst
HCsCH + H2 ^ H2C = CH2 ; AH® = - 176 kJ mol”^

ee
Catalyst
H2C = CH2 + H2 ^ CH3—CH3 ; AH® = - 138 kJ mor^

Fr
Thus, it is possible to stop the reduction at the alkene stage by using specific catalysts such as
Undlar’s catalyst, i.e., Pd supported over CaC03 or BaS04 and partially poisoned by addition of sulphur or

for
quinoline.
ur
In case of non-terminal alkynes, catalytic reduction of alkynes with dihydrogen in presence of Lindlar’s
catalyst always gives cw-alkenes (refer to Ans. to Q. 18, page 13/124 for details) since during hydrogenation,
the addition of dihydrogen occurs on the same face of the alkyne molecule. For example.
ks
Yo
CH
Pd/BaS04+S or quinoline
oo
3\
CH3—C = C—CH3 + H2 c = c
(Lindlar’s catalyst) H H
eB

But-2-yne cw-But-2-ene

In contrast, chemical reduction, i.e., Birch reduction of non-terminal alkynes with Na or Li in liquid
NH3 at 196-200 K gives /rans-alkenes.
CH. H
r

(0 Na/Uq.NHj, 196-200 K
c = c;
ou
ad

CH3—C e C—CH3 ({() Ethanol H


CH3
But-2-yne trfl/w-But-2-ene
Y

Mechanism. Chemical reduction is believed to occur through two electron transfers and two proton
transfers as shown below :
nd
Re

R R R
R—C=C—R + -Na
. ●●-1- H^NH2 ^ C=C
t^R—C=C —R C=C
Fi

-Na -NHJ
Alkyne H R H
ciy-Alkenyl radical rro/iy-Alkenyl radical

a
^^c=c
R H R
H-NH2 ^ c=c
-Na"’ I, /
R H -NHJ R H
/ransr-Alkene

The formation of rran^-product is due to the rapid equilibrium between the cis- and trans alkenyl radicals,
in which the trans-radical being more stable is readily formed.
HYDROCARBONS 13/61

V. Oxidation Reactions of Alkynes


Oxidation of alkynes can be carried out under different conditions to form different products.
1. Oxidation with air or oxygen - Combustion
When heated in air, alkynes undergo combustion to form CO2 and H2O accompanied by release of large
amount of heat and light energy.
2CH = CH + 5O2 4 4 CO2 + 2 H2O ; AH = - 1300 kJ mo\~^
Acetylene

w
Under normal conditions, acetylene bums with a luminous yellow sooty flame due to the presence of
higher carbon content than hydrogen. However, with air or oxygen under high pressure, acetylene bums with
a blue flame (oxyacetylene flame) producing high temperature of the order of 3000 K which is used for
cutting and welding of metals.

Flo
2. Oxidation with cold dilute potassium permanganate
Alkynes are readily oxidised by cold dilute alkaline KMn04 solution to give a-dicarbonyl compounds,

e
i.e., 1, 2-dialdehydes, 1,2-diketones, 2-oxoacids or 1, 2-dioic acids depending upon the nature of alkyne.

re
It may be noted here that in case of terminal alkynes, = CH part is oxidised to -COOH group while in
case of non-terminal alkynes, = CR part is oxidised to R—C = O group. For example.

F
KMn04,H20 [O]
ur
r
CH3—CsCH +3[0] CH3—C—COOH ^ CH3COOH + CO2

fo
298-303K
Propyne Ethanoic acid
o
{Acetic acid)
2-Oxopropanoic acid
ks
Yo
KMn04,H20
CH3—CsC—CH3 + 2[0] CH,—C—C—CH,
oo
But-2-yne
298-303K ^ II II ^
o o
B

Butane-2, 3-dione {Biacetyl)


Acetylene, however, under these conditions gives oxalic acid probably due to further oxidation of the
re

initially formed glyoxal.


CH KMn04,H20 CH = 0 [O] COOH
u
ad

+ 2[0] I
298-303K
Yo

CH CH=0 COOH

Acetylene Ethane-1, 2-dial Ethane-1, 2-dioic acid


{Glyoxal) {Oxalic acid)
d

During this reaction, the pink colour of the KMn04 solution is discharged and a brown precipitate of
Re
in

manganese dioxide is obtained. This reaction is, therefore, used as a test for unsaturation under the name
Baeyer’s test
F

3. Oxidation with hot KMn04 solution.


Like alkenes, alkynes when treated with hot KMn04 solution, undergo cleavage of the CsC
bond leading to the formation of carboxylic acids and carbon dioxide depending upon the nature of the
alkyne. In general, s CH is oxidised to CO^ and H2O and = CR is oxidised to RCOOH
For example, acetylene gives CO2 and H2O while propyne gives acetic acid and CO2.
CH KMn04,KOH COOH [O]
+ 4[0] I 2CO2 "I" H2O
CH 373-383K
COOH
Acetylene Oxalic acid
13/62 “Pnaideefr New Course Chemistry (XI) m«JHl

KMn04,K0H
CH3—C f CH + 4 [O] 373-383K
CH3COOH +CO2
Propyne Ethanoic acid

In contrast, non-terminal alkynes on oxidation with hot KMn04 solution give only carboxylic acids.
For example.

KMn04,K0H
CH3—C*C—CH3 + 4[0] 373-383K
■>
CH3COOH + HOOCCH3
But-2-yne Ethanoic acid (2 molecules)

w
KMn04,K0H
CH3CH2—C ^ C—CH3 + 4 [O] 373-383K
■>
CH3CH2COOH + HOOCCH3
Pent-2-yne Propanoic acid Ethanoic acid

F lo
Thus, by identifying the products formed during alkaline KMnO^ oxidation, it is possible to determine
the position of the triple bond in an alkyne molecule.
4. Oxidation with ozone. Alkynes react with ozone in presence of some inert solvents such as CH2CI2,

ee
CHCI3 or (PCI4 at low temperature (196—2(X) K) to form ozonides. These ozonides on decomposition with

Fr
Zn dust and water or H2/Pd (reductive cleavage) give 1, 2-dicarbonyl compounds. For example.
O
O3/CH2CI2 / \ Z11/H2O
HCsCH

for
» HC CH H—C—C—H
Ethyne
196-200K
ur I
o —o
I -ZnO
{reductive cleavage) o o

Ethyne ozonide s 1, 2- Ethanedial (Glyoxal)


ok
Yo
O3/CH2CI2 Z11/H2O
CH3—C = CH > CH,—C—C—H
> CH3—C H
^ II II
o
196-200K -ZnO
I (reductive cleavage)
eB

Propyne o—o o o
Propyne ozonide 2-Oxopropanal

O
r

3-C^
O3/CH2CI2 ZII/H2O
ou

CH3—C = C—CH3 » CH
—CH3 » CH,—C—C—CH,
ad

196-200K
I I
-ZnO
(reductive cleavage)
Ml II ^
But-2-yne o o
o-—o
Y

But-2-yne ozonide Butane-2, 3-dione


VI. Polymerization Reactions of Alkynes
nd
Re

Like alkenes, alkynes also undergo polymerization reactions.


But alkynes undergo two types of polymerization reactions as discussed below :
Fi

(a) Linear polymerization. As the name suggests in this type of polymerization, two or more molecules
of ethyne combine to form products which have linear structures. For example,
(i) In presence of CUCI/NH4CI, acetylene first gives vinylacetylene and then divinylacetylene. Thus,
CUCI/NH4CI HCsCH
2H—C = C—H » H—C = C—CH = CH2 > H2C = CH—CsC—CH = CH2
Acetylene Vinylacetylene
CUCI/NH4CI
Divinylacetylene
Vinylacetylene is widely used in the manufacture of chloroprene which is the starting material for the
synthetic rubber neoprene.
Cl
Mark, addition
CTl2=CH—CsCH + HCl ^ CH2=CH—C = CH2
Vinylacetylene Chloroprene
HYDROCARBONS 13/63

(k) Under suitable conditions, polymerization of acetylene produces the linear polymer polyacetylene.
It is a high molecular weight conjugated polyene containing the repeating unit (CH = CH—CH = CH)„.
Under proper conditions, this material conducts electricity. Thin films of polyacetylene can be used as electrodes
in batteries. Further, since polyacetylenes have much higher conductance than metal conductors, these can be

ow
used to prepare lighter and cheaper batteries,
(b) Cyclic polymerization. As the name suggests, in this type of polymerization, three or more molecules
of an alkyne combine together to form products which have ring structures. For example,
(0 When ethyne is passed through red hot iron tube at 873 K, it trimerises to give benzene.

e
re
Red hot

Fe tube 873 K

Frl
F
Benzene

(3 molecules)

ou
Similarly, propyne trimerises under similar conditions to form mesitylene (1, 3, 5-trimethylbenzene).

sor
CH3

kf
CH3

Red hot Fe tube


oo
>
873 K
Y
CH3 CH3
B

1,3,5-Trimethylbenzene
Propyne {Mesitylene)
(3 molecules)
re
oY

Under similar conditions, but-2-yne gives hexamethylbenzene.


u

(«) In presence of nickel cyanide as catalyst and under high pressure, four molecules of ethyne combine
to form a tetramer called cycloocta-1, 3, 5, 7-tetraene.
ad

7
d

CH^CH CH=CH 6 5

O' CH 1
in

CH CH 4
Ni(CN)2
Re

or

CH
High pressure ^CH CH 8

\ CH=CH
F

Ol
CH=CH
3
1 2

Ethyne Cycloocta-l, 3, 5,7-tetraene


(4 molecules)

Vn. Isomerization of alkynes.


1-Alkynes can be readily isomerized to the more stable 2-alkynes on treatment with ale. KOH at 473 K.
AIC.KOH
CH3CH2CSCH 473 K
> CHj-C^C-CHg
1-Butyne 2-Butyne
13/64 New Course Chemistry (XI)EZ53MD

The isomerization is believed to occur by the following mechanism.


H Ale, KOH
(-H2O) .A rv ROH (-RO")
CH3—C—C=CH 4 CH3— C-S-ciCH ^ > CH3—C==C=CH'
H H H
l-Butyne
Ak, KOH

..-a r>
CH3—C=C=CH2 ♦ CH3—C^C=CH2 CH3—C=C—CH2

w
H
ROH (-RO“)
CH3—C=C~CH3
2-Butyne

F lo
On the other hand, 2-alkynes can also isomerize to 1-alkynes by action of NaNH2 in an inert solvent
such as kerosene or piiraffin oil at 423 K followed by treatment with H2O.
NaNH2/Paraffin oil H.,0

ee
CH3-C = C-CH3 423K ^ CH3CH2C = C" Na-^ ^ CH3CH2CSCH
2-Butyne

Fr
l-Butyne

13.7.9. Distinction between an Alkane, Alkene and an Aikyne


These three types of hydrocarbons can be distinguished by the following tests.

for
Test Alkane Alkene Aikyne
ur
1, Flame observed on Non-luminous Luminous Smoky
combustion
s
ook
2.
Br2/CCl4 solution No action Orange colour is discharged Orange colour is discharged
Yo
3. Cold aqueous or No action Pink colour is discharged Pink colour is discharged
eB

alkaline KMn04 with the formation of a with the formation of a


solution {Baeyer's glycol. dicarbonyl compound.
reagent)
4. No action
Ammoniacal AgN03 No action White ppt. of silver acetylide
our
ad

solution {Tollens' (given by only terminal


reagent) alkynes).
5. Ammoniacal CuCl No action No action Red ppt. of copper acetylide
Y

solution (given only by terminal


alkynes).
Re
nd

13.7.10. Uses of Alkynes


Fi

(i) Acetylene and its derivatives are widely used in synthetic organic chemistry for synthesis of c/5- and
trfl/15-alkenes, methyl ketones, etc.
(//) Oxyacetylene flame is used for cutting and welding of metals.
(///) Acetylene is used as illuminant in hawker’s lamp and in light houses,
(/v) Acetylene is used for ripening of fruits and vegetables,
(v) Acetylene is used for manufacture of ethyl alcohol, acetaldehyde, acetic acid, vinyl plastics, synthetic
rubbers such as Buna N and synthetic fibres such as Orion.
(v/) Lewisite is a war gas and was used in World War-ll. It is obtained by the action of arsenic trichloride
on acetylene in presence of anhydrous AICI3.
HYDROCARBONS 13/65

CH Cl Anhyd.AlCl3 CHCl
+
II
CH ASCI2 CHASCI2
Acetylene Arsenic trichloride Lewisite

i^-Chlorovinyldichloroarsme)
Lewisite is more poisonous than mustard gas and was used in World War-II. The British chemists
developed an antidote for Lewsite and named it as BAL (British Anti Lewisite).
CH,—CH—CH.
I - I 1 ^
SH SH OH
BAL

PART—IV. ARENES

w
13.8. AROMATIC HYDROCARBONS OR ARENES

F lo
Hydrocarbons and their alkyl, alkenyl and alkynyl derivatives which contain one or more benzene
rings either fused or isolated in their molecules are called aromatic hydrocarbons. They are also called
arenas (aromatic alkenes). Benzene irng is highly unsaturated and in most of the reactions of aromatic
compounds, the unsaturation of benzene ring is retained. However, there are examples of aromatic hydrocarbons

e
which do not contain a benzene irng but instead contain other highly unsaturated irngs. Whereas aromatic

Fre
hydrocarbons containing a benzene irng are called benzenoids, those not containing a benzene irng are
known as non-benzenoids, e.g., azulene, tropolone, cyclopentadienyl anion, cycloheptatrienyl cation, etc.

for
13.8.1. Classification and Nomenclature of Arenes
Arenes are further classified into different seriesdepending upon the number of fused benzene rings
present in their molecules. Their general formula is C„H2„_6,„ where n is the number of carbon atoms and m
our
is the number of irngs. For monocyclic arenes, m = 1 and n = 6, or more. Examples are :
oks
CH3 CH2CH3 CH3 —CH—CH3
eBo
Y

Ethylbenzene Isopropylbenzene
Benzene Methylbenzene
(Cumene)
(Toluene)
our
ad

For bicyclic arenes, n = 10 and m = 2. The first member of this series is the well known naphthalene
which is used in the form of moth balls as a moth repellent for clothes. For tricyclic arenes, n = H and m = 3.
The first member of this series is anthraceneor phenanthrene.
dY
Re
Fin

Naphthalene Anthracene Phenanthrene

Some arenes may contain two or more isolated benzene rings. For example,

CHr c

Diphenyl Diphenylmethane H
or Biphenyl Triphenylmethanc

/
13/66 ^ New Course Chemistry (XI) orsiiii
13.8.2. Isomerism in Arenes

Arenes show position isomerism as discussed below :


Benzene is a symmetrical molecule, i.e., all the six hydrogen atoms of benzene are equivalent. Therefore,
the replacement of one hydrogen atom of benzene by any substituent will give only a single product. Thus,
monosubstitution products ofbenzene do not show isomerism. However, when two hydrogen atoms of benzene
ring are replaced by two same or different substituents, three isomers are possible which differ in the position
of substituents. Thus, disubstitution products of benzene show position isomerism. These three isomers are
called ortho (o-), meta (-) and para {p-} according as the relative positions of the two substituents are I, 2-:
1, 3- and 1,4-respectively. Thus, the three position isomers of dimethylbenzene or xylene are :
CH3 CH3 CH3
1 1 1
CH3

w
2

CH3

F lo
1, 2-Dimethylbcnzene 1, 3-Dimethylbenzene CH3
or o-Dimethylbenzene or /n-Dimethylbenzene
or o-Xylene or m-Xylene ], 4-Dimethylbenzene
or /J-Dimethylbcnzene
or p-Xylene

e
Besides, the three dimethylbenzenes, the fourth isomer is ethylbenzene. For structure, refer to page 13/61.

Fre
Similarly, three position isomers of trimethylbenzene are :
CH3 CH3 CH3
A2/CH3
for
J^ch3
1
r
^ CH3 4 H3C"^5 ^ CH3
You
1,2. 3-Trimethylbenzene CH3 1, 3, 5-Trimelhylbenzene
ks
(Mesilylene)
1, 2,4-Trimethylbenzene
o
eBo

If the number of substituents increases in the benzene


ring, the number of position isomers also goes up.
In case of bicyclic arenes such as naphthalene, even monosubstituted compounds show position
isomerism. For example.
CH3
our
ad

a 1 1

l-Methylnapiithalene 2-Methylnaphthalene
(or (x-Methylnaphthalene) (or ^-Methylnaphlhalene)
dY
Re

With higher arenes, the number of possible position isomers also increases.
13.8.3. Structure of Benzene
Fin

The molecular formula of benzene is which implies that it has eight hydrogen atoms less than the
corresponding saturated alkane, /.c., CgHj4 (hexane). Thus, benzene should be a highly unsaturated compound.
This is supported by the following three observations which suggest that a molecule of benzene contains
three double bonds.

(/) Benzene adds three molecules of hydrogen in presence of Raney nickel or platinum as catalyst at
473—523 K to form cyclohexane.
Raney Ni
CgHe + 3H2 473-523K
Benzene Cyclohexane
(//) In presence of sunlight and in absence of a halogen carrier, benzene adds three molecules of chlorine
to form benzene hexachloride (BHC).

1
HYDROCARBONS 13/67

Sunlight
CfiHe + 3Cl2 ■>
C6H6CI5
Benzene Benzene hexachloride

(Hi) ^^^th O3, it forms a triozonide.


CH2CI2
+ 3O3 196K
C6H6(03>3
Benzene Benzene triozonide

In spite of the presence of three double bonds, benzene behaves like a saturated compound as supported
by the following two facts:
(/) Benzene undei^oes substitution rather than addition reactions. Although benzene is expected to
be highly unsaturated yet it does not undergo addition reactions so typical of unsaturated hydrocarbons such
as alkenes and alkynes. For example, it does not decolourize bromine in carbon tetrachloride solution. Instead,

w
it undergoes substitution reactions which are characteristic of saturated compounds. For example, when
treated with Br2 in presence of a trace of FeBr3, benzene forms monobromobenzene.

F lo
Bf2 Bf2
<r
^ CgHsBr + HBr
Not formed
CCI4 Benzene
FeBr3 Bromobenzene

(Addition product) (Substitution product)

e
Fre
(ii) Benzene is resistant to oxidation. Unlike alkenes and alkynes, benzene does not decalourise alkaline
KMn04. However, with powerful oxidising agents such as chromic acid, potassium permanganate, etc. it is

for
very slowly oxidized to CO2 and H2O. Thus, benzene resists oxidation like saturated compounds.
Kekule structure. The first insight into the structure of benzene was given by Friedrich August Kekule
(1865). He proposed that the six carbon atoms of benzene are joined to each other by alternate single and
r
double bonds to form a hexagonal ring as shown in Fig. 13.16. Each carbon is further connected to a hydrogen
You
atom.
o ks
FIGURE 13.16
eBo

H
I
H
our
ad

Cn..
H

H
dY
Re
Fin

Besides Kekuld, the following other alternative structures were proposed for benzene from time to time
by different scientists, but all these were rejected on one ground or the other.

Dewar (1867) Claus (1867) Baeyer(1892)

Objections to Kekule structure. There are three objections to Kekule structure.


(0 Why is it that benzene shows remarkable stability towards oxidising agents like KMnO^ and undergoes
substitution reactions even though it contains three double bonds ?
13/68 New Course Chemistry (XI)BEIHQ]

(») Kekule formula for benzene contains two kinds of bonds, i.e., single and double bonds. But X-ray
diffraction .studies have shown that the structure of benzene is a regular hexagon with an angle of 120“ and
all the carbon-carbon bond lengths are equal, i.e., 1-39 A or 139 pm which lie in between carbon-carbon
single bond length of J-54 A or 154 pm and carbon-carbon double bond length of 1-34 A or 134 pm.
iiii) Kekule structure would predict only one monosubstitution product and two o-di.substituted products
(a and b) as shown in Fig. 13.17.

ow
In stmcture (a), there is a double bond between the two carbon atoms carrying bromine atoms but in
structure {b) there is a single bond, if Kekule structures were correct, both these isomers should exist and
should have different properties. But only one o-dibromobenzene is known. To account for this, Kekule
proposed a dynamic equilibrium between the two structures as shown in Fig. 13.18.
FIGURE 13.17 FIGURE 13.18

Br Br Br Br

e
re
Br Br Br Br

rFl
A

F
o o o o
Kekule isomers Dynamic equilibrium between two Kekule
of o-dibromobenzene isomers of o-dibromobenzene

r
ou
fo
In other words, positions of single and double bonds are not fixed but oscillate back and forth between
adjacent positions.

ks
Thus, Kekule structure failed to account for the unusual stability of benzene and the equivalence of all
the carbon-carbon bond lengths in benzene. However, these can be easily explained in terms of molecular
oo
orbital and resonance theories of benzene as discussed below :
Y
Molecular orbital structure of benzene.
eB

Since all the bond angles in benzene are I20^


therefore, all the carbon atoms of benzene are
assumed to be 5/?^-hybridized. Each carbon atom
r

forms two C—C, a-bonds with the neighbouring


ou

carbon atoms and one C—H, a-bond with a


Y
ad

hydrogen atom. Thus, in all there are six C—C, a-


bonds and six C—H, o-bonds which all lie in one
plane and the angle between any two adjacent a-
d

bonds is 120°. (Fig. 13.19)


Each carbon is now left with an unhybridized
in
Re

y3-orbital. These six unhybridized p-orbitals have


one electron each and are not only parallel to one
F

another but are also perpendicular to the plane of


a-bonded carbon skeleton. Further, any two p-
orbitals on adjacent carbon atoms are close enough
for a sideways overlap to form 7i-bonds.
Now there are two equally good ways in which
the p- orbitals on adjacent carbon atoms can
a-Boad skeleton of benzene
overlap to form three 7i-bonds as shown in Fig.
13.20 (a) and 13.20 (b).
HYDROCARBONS 13/69

FIGURE 13.20

H' H

H H

\ /

/
H- \ / H or H H
\ /

\ /
\ /
H H

w
\ /

o H' A / H

F lo
/
H' H

/ \
H H
/ \
\
/ A

/ \
/

ee
/ \ or H H
H / \ H

Fr
\,

/ A

H H

for
H' H o
ur
Two possible sideways overlap of six unhybridired p-orbitals to
s
form three 7l-bonds
ook
Yo
In fact, each 2/?-orbital overlaps equally well with 2p-orbitals on adjacent carbon atoms on either side to
form two doughnut shaped 7C-electron clouds ; one of which lies above and other below the plane of carbon
eB

and hydrogen atoms as shown in Fig. 13.21.


our

<
ad

H H

/ \
/ \
V
< [>
Y

/
' / \ >
H- > < H
Re

\ t \
nd

\ t
Fi

\ /
\ /

H' H

Tt-Electron >uds lying above and below the plane of the benzene ting

Thus, two /7-orbitals are not confined to a single Tt- bond between two carbon atoms as was the case with
two structures as shown in Fig. 13.20 but are involved in the formation of more than one 7t-bond. This
participation of n-electrons in more than one bond is called delocalization of electrons. These delocalized
electrons are equally attracted by all the six carbon atoms of the benzene ring. As a result, all the carbon-
carbon bond lengths(139 pm) in benzene are equal, all the carbon-hydrogen bonds (1 !0pm) are equivalent,
its dipole moment is zero and does not show the type of isomerism shown in Fig. 13.17.
13/70 New Course Chemistry (X1)ESS[M]

Further, due to delocalization of electrons, benzene is now a days FIGURE 13.22

shown by a circle within the hexagon (Fig. 13.22). Whereas the hexagon
represents the six carbon atoms, the circle represents the six completely
delocalised 7C-electrons.

It may be noted here that it is this delocalization of Tt-electrons which


makes it more stable than the hypothetical cyclohexatriene molecule {Kekule
structure).
Modem picture of
benzene molecule
13.8 A Resonance

The phenomenon of resonance is said to occur iffor a molecule, we can write two or more than two

w
Lewis structures which differ in the position of electrons and not in the relative position of atoms. The
various Lewis structures are called canonical structures or the resonance structures. The real structure of

F lo
the molecule is not represented by any one of the canonical structures but is a resonance hybrid of the
various resonance structures. The various resonance structures are separated by a double headed (^)

arrow. This arrow should be clearly distinguished from that used for an equilibrium
In the light of resonance theory, benzene can be represented as a resonance hybrid of the following two

ee
Kekule structures (I and II).

Fr
o

for
(I) (II) (III)
ur
Thus, these two structures are the canonical forms of benzene. The actual structure of benzene is neither
represented by T’ nor by TF but is a resonance hybrid of these two structures. Further since these two
oks
canonical forms are of equal energy so they contribute equally (50%) towards the resonance hybrid. In other
Yo
words, structure of the resonance hybrid is in between these two Kekule structures. This means that any two
o

adjacent carbon atoms of the benzene molecule are neither Joined by a pure single bond nor by a pure double
eB

bond. As a result, all the carbon-carbon bond lengths are equal, i.e., 1-39 A (139 pm) and lie in between C =
C bond length of 1-34 A (134 pm) and C—C bond length of 1-54 A (154 pm). The resonance hybrid or the
actual molecule of benzene is usually represented by the formula TIT.
our
ad

Further, a resonance hybrid is always more stable than any of its canonical structures. Thus, the actual
molecule of benzene is more stable than either of the two Kekule structures. The magnitude of stability
conferred on a molecule as a result of resonance is expressed in terms of resonance energy or delocalization
energy. It is deifned as the difference in internal energy of the resonance hybrid and that of the most stable
Y

canonical fonn. In case of benzene, this resonance energy has been determined to be 150-62 kJ (36-0 kcai)
Re

mol“*.
nd

13.8.5. Calculation of Resonance Energy of Benzene


Fi

The resonance energy of benzene can be calculated from heat of hydrogenation data.
Heat of hydrogenation is the amount of heat evolved when one mole of an unsaturated compound is
hydrogenated (in presence of a catalyst).
When benzene is fully hydrogenated, it gives cyclohexane and 208-36 kJ (49-8 kcal) mol"^ of heat is
evolved. On the other hand, cyclohexene having a six- membered irng and one double bond when hydrogenated
gives cyclohexane and 119-66 kJ (28-6 kcal) mol“^ of heat is evolved. If the three double bonds of benzene
did not interact or if there were no resonance (as would be the case in the hypothetical cyclohexatriene
molecule with three non-intreacting double bonds in a six-membered irng) then three times the heal of
hydrogenation of cyclohexene, i.e., 119-66 x 3 = 358-98 kJ (85-8 kcal) mol"^ would have been evolved.
HYDROCARBONS 13/71

Catalyst -1
+
H2 AH=-119-66 kJ mol

Cyciohcxene Cyclohexane

Catalyst -1
+
3H2 AH=-358-98 kJ mol

Cyclohexatricne Cyclohexane

low
{hypothetical)

Catalyst
o + 3H2 > AH=-208-36 kJ mol
-1

Benzene Cyclohexane

Thus, the actual molecule of benzene is 358-98 - 208-36 = 150-62 kJ (36-0 kcal) mol~^ more stable

ee
than the hypothetical cyclohexatricne molecule. In other words, 150-62 kJ (36-0 kcal) mot^ is the resonance

F
energy of benzene.

Fr
13.8.6. Aromaticlty-HUckel Rule
Hiickel, on the basis of molecular orbital theory, has predicted that electrons in cyclic conjugated polyenes
(cyclic polyenes having alternate single and double bonds) containing (4n + 2) 7t-electrons where « = 0,1,2,

for
ur
3...etc. are completely delocalized. This makes these molecules particularly stable. All such compounds are
called aromatic compounds and have chemical properties altogether different from other cyclic unsaturated
compounds which do not contain (4n + 2) 7C- electrons.
Thus, the necessary and sufficient conditions for a molecule to be aromatic are :
s
ok
Yo
(/) It should have a single cyclic cloud of delocalized n-electrons above and below the plane of the
molecule.
Bo

(ii) It should be plana.r This is due to the reason that complete delocalization of 7C-electrons is possible
only if the ring is planar to allow cyclic overlap of p-orbitals.
re

(Hi) It should contain (4/i + 2) %-electrons where n = 0,1, 2, 3...etc.


A molecule which does not satisfy any one or more of the above conditions is said to be non-aromatic.
Applications of Hiickel rule. In the light of the above rules, any planar cyclic system containing 2, 6,
ou
ad

10, 14, 18...etc. K-electrons and having a single cyclic n-electron cloud encompassing all the carbon atoms
in the ring is aromatic. Let us now test the validity of these rules:
Y

(0 Monocyclic systems :

"X"
nd
Re
Fi

\=/
Cyclopropenyl Cyclopropenyl Cyclopentadicnyl Cyclopentadicnyl Cycloheptatriene (non plana.r
cation {plana,r anion {plana.r cation (plana,r anion (plana,r 6n-eleclrons
2n-electrons, An-electron.’;. An-electrons, (sTz-electrons, but not spread over
aromatic) antiaromatic) antiaromatic) aromatic) the entire ring,
non aromatic)
H H
<*■

2e-

\=/ \=/
Cycloheptarienyi cation or Cycloheptarienyi anion Cyclooctateraeme Cyclooctatetraenyl dianion (plana,r
Tropylium cation (plana,r (plana,r %K-electrons, (non-plana,r or tub-shaped, 1 On-electrons .spread
(m-electrons spread over antiaromatic) 8n-electrons, non aromatic) over the entire ring,
the entire ring, aromatic) aromatic)
13/72 New Course Chemistry (XI)sasiHD

Retain in lory
Planar conjugated cyclic systems containing 4nn (n = I, 2, 3, ... etc.) electrons are destabilized by
resonance and hence are antiaromatic. For example.
H

\=/ \=/
Cyclopropenyl Cyclobutadiene Cycloheptarienyl anion Cyclooctatetraene

w
anion {plana,r (planar, {plana,r Sn-elecirons, (plana,r 8n-eleciron.s,
4n-eleclrons, 4K-elecirons, antlaromatic) antiaromatic)
antiaromatic) antiaromatic)

The antiaromatic compounds are even less stable than the corresponding non-planar

Flo
47C-electron systems. That is why planar cycloheptatrienyl anion and planar cyclo-octatetraene adopt
non-planar geometries to become stable, i.e., cyclooctatetraene has non-planar tube shape.

ree
(ii) Polycyclic compounds. Hiickel rule can also explain the aromaticity of polycyclic compounds. For
example, naphthalene (page 13/66) which contains 10 TT-electrons (n = 2), anthracene and phenanthrene
(page 13/66) each containing 14 tr-electrons (n = 3) are all aromatic.

rF
(Hi) Heterocyclic compounds. Heterocyclic compounds such as pyridine, furan, thiophene and pyrrole
are all aromatic since each one of them is planar and has a cyclic system of 6 7t-electrons which is completely
delocalized over the entire ring.
ur
fo
I 1 I 1 I 1
N**/
N o
ks
s N
H
Yo
Pyridine Furan Thiophene Pyrrole
oo
In pyridine, each of the five A/^'-hybridized carbon atoms and the 5p^-hybridized nitrogen atom has a p-
orbital perpendicular to die plane of the ring. Each of these atoms contributes one 7i-electron thereby producing
B

a single cyclic TC-cloud containing 6 rr-electrons. The lone pair of electrons on the nitrogen atom is present in
a A’p^-orbital which being in the plane of the ring does not contribute towards the aromatic sextet.
re

In contrast, in case of furan and thiophene, one of the two lone pairs* of electrons on the 5p^-hybridized
heteroatom (O or S) is present in a p-orbital perpendicular to the plane of the ring. Consequently, it contributes
u
ad

two 7t-electrons while the other four p-orbitals of tlie two double bonds contribute one 7c-electron each thereby
Yo

bringing the total to six 7i-electrons. Pyrrole, on the other hand, has only one lone pair which is present in ap-
orbital perpendicular to the ring. Therefore, it contributes two 7t-electrons while the four p-orbitals of the two
double bonds contribute one 7t-electron each thereby bringing the total to six 7t-electrons. Thus, all the three
ifve-membered heterocycles (furan, thiophene and pyrrole) have a single cyclic electron cloud containing
d
Re

6%-electrons and hence are aromatic in characte.r This theory of six jc-electrons explaining the aromatic
n

character of cyclic compounds is called aromatic sextet theory and the six 7c-electrons are called aromatic
Fi

sextet.

4. Annulenes. Completely conjugated monocyclic polyenes containing an even number ofcarbon atoms
are called annulenes. Their general formula is (CH = CH),, where n = 2, 3, 4...etc.
They are named by indicating the number of carbon atoms in the irng enclosed in square brackets before
the root word annulene. Thus,

/=\
\=/
[4]-Annulene [6]-Annulene [8]-Annulene
(Cyclobutadiene) (Benzene) (Cyclooctatetraene)

*The Other lone pair of electrons on the jp^-hybridized heteroatom lies within the plane of the irng and
hence does not contribute towards the aromatic sextet.
HYDROCARBONS 13/73

Curiosity Questions
f Q. 1. What would have been the shape of benzene molecule had there been no resonance ?
Ans. Due to resonance, delocalization of six n-electrons takes place. As a result, all the
carbon-carbon bonds in benzene are equal (139 pm) and the molecule is symmetrical.
Had there been no resonance, benzene would be an unsymmetrical molecule consisting [[

w
of three carbon-carbon single bonds (154 pm) and three carbon-carbon double bonds ^
(134 pm) as shown ?
Q. 2. Does resonance always lead to stabilization of cyclic conjugated systems ?
Ans. Resonance is a hypothetical concept which has been developed to explain the extra stability of
cyclic conjugated systems containing (4 n + 2) rc-electrons. In case of cyclic systems containing

o
4 n TT-eiectrons, resonance actually destabilizes the molecules and makes them antiaromatic

e
though we can still write resonance structures as shown below :

re
■4- > <—► etc.

Frl
F
13.8.7. Methods of preparation of Arenes
Benzene {jyhene), was first isolated by Michael Faraday (1825) from cylinders of compressed
ou
or
illuminating gas obtained from pyrolysis of whale oil. In 1845, benzene was found in coal-tar by Hofmann.
Arenes are mainly present in the light oil fraction of coal tar distillation from where they are isolated by

kfs
suitable methods. However, it may be prepared by the following methods:
1. From ethyne by cyclic polymerization. Benzene (b.p. 353 K) was first synthesized by Berthelot
(1870) by passing ethyne through red-hot iron tube at 873 K. Under similar conditions, propyne gives mesitylene
oo
(For details refer to pages 13/62-13/63.
2. From sodium benzoate by decarboxylation with soda-lime (Laboratory method)
Y
eB

COONa
CaO.A
Q + NaOH >
Qj + Na2C03
ur

Sodium benzoate Benzene


oY

3. From phenol by reduction with zinc dust


ad

OH
Zn dust

Q Q + ZnO
d

distil

Phenol Benzene
in
Re

4. From chlorobenzene by reduction with Ni-Al alloy/NaOH

.Cl
F

Ni-Al alloy
o + 2[H] NaOH o -H HCl

Chlorobenzene Benzene

5. From benzenediazonium chloride by reduction with hypophosphorous acid (H3PO2) in presence


of Cu'*’ ions
+
N=NCr
Cu"^
Q + H3PO2 + H2O > (f^j + N2 + HCl + H3PO3
Benzenediazonium chloride Benzene
13/74 p>tetdeefr'4. New Course Chemistry (XDESsim
6. From benzenesulphonic acid by heating with superheated steam.
SO3H
423-473 K

Q + H2O
Steam
Q) +H2SO4
Benzenesulphonic acid Benzene

7, From n-hexane and n-heptane. When hexane under a pressure of 10-20 atm is passed over a catalyst
consisting of oxides of chromium, vanadium and molybdenum supported over alumina at 773 K, it gets converted
into benzene. n-Heptane under similar conditions give toluene. For details, refer to page 13/18.
8. By Wurtz-Fittig reaction. Homologues of benzene can be prepared by warming a solution of a

w
suitable haloarene and a haloalkane in presence of metallic sodium in dry ether. For example.

0-5LBF..t*lNa+*B^ Diy ether ^


<0)-CH2CH3 + 2NaBr

F lo
A
Bromomethane
Bromobenzene Ethylbenzene

ee
Biphenyl and n-butane (and some other compounds) are obtained as by-products.
The advantage of Wurtz-Fittig reaction over Friedel-Crafts alkylation are :

Fr
(/) the structure of the product is known,
(I'O long n-side chains can be easily introduced without the danger of any rearrangement. For example.

for
ur
CgHgBr + 2 Na + BrCH2(CH2)2CH3 CgHj—CH2CH2CH2CH3-b2NaBr
Bromobenzene l-Bromobutane
s n-Butylbenzene

9. Friedel-Crafts reaction. Benzene reacts with alkyl halides in presence of a Lewis acid catalyst such
k
Yo
as anhydrous AICI3 to form alkylbenzenes or arenes. For example.
oo
CH3
eB

Anhyd.
Q| + CH3C1 AICI3 ^ Q + HCl
r

Benzene Toluene
ou
ad

Instead of alkyl halides, alcohols can also be used in Friedel-Crafts reaction. Here, the Lewis acid helps
in the generation of a carbocation which then attacks the benzene ring to form the corresponding alkylbenzene.
For example.
Y

H
Re

CH3—CH2^0—AlClJ
nd

CH3CH2OH + AICI3 i CH3CH2 + HO-AICI3"


Ethanol
Ethyl carbocation
Fi

C6H6+CH3CH2 ^ C6H5CH2CH3 +H+


Ethylbenzene
Friedel-Crafts reaction can also be carried out with alkenes in presence of protonic acids such as H2SO4,
H3PO4, HF, etc. For example.
H2SO4 + CsHfi
(CH3)2C = CH2 ■»
(CH3)2C—CH3 -H+
CgHs—C(CH3)3
-HSOJ
2- Methylpropene /ert-Butyl carbocation /ert-Butylbenzene
HYDROCARBONS 13/75

H3PO4
Similarly, + CH3CH = CH2 CgHj—CH(CH3)2
Propenc Cumene

10. From Grignard reagents. Grignard reagents react with alkyl halides in presence of dry ether to
form arenes. For example,

ow
CH3 CH3

MgBr + Br—CH—CH3 Dry ether ^ CH—CH3 + MgBr2


Isopropyl bromide
Phenylmag. bromide Isopropylbenzene or Cumene

13.8.8. Physical Properties of Arenes

e
{/) Physical state, colour, smell, etc. Benzene and its homologues containing upto eight carbon atoms

re
are colourless liquids while the higher ones are solids with characteristic smell. We are all familiar with

Fr l
naphthalene balls which are used in toilets and for preservation of warm clothes because of its unique smell

F
and moth-repellent property.
(«) Combustion. Due to high carbon content, they bum with a sooty flame.
(///) Solubility. Due to large hydrophobic hydrocarbon part, arenes are not soluble in water but are

or
soluble in organic solvents,
ou
(/v) Melting points and boiling points. The boiling points of arenes increase with increase in the

kfs
molecular size due to a con csponding increase in the magnitude of van der Waals forces of attraction. Melting
points, on the other hand, depend upon moleculai" size and symmetry. The more symmetrical the molecule,
better it fits in the crystal lattice and hence higher is its melting point. Amongst isomeric arenes, i.e., o-, m-
oo
and /7-xylenes, the p- isomer which is most symmetrical, has the highest melting point.
13.8.9. Chemical Reactions of Arenes
Y
B

Arenes are highly unsaturated hydrocarbons but behave like saturated hydrocarbons. They are, however,
more reactive than alkanes but are less reactive than alkenes and alkynes. This lower reactivity of arenes
re

over alkanes is because of the extra stability associated with these molecules due to delocalization of k-
electrons.
oYu

Some important reactions of arenes are discussed below :


ad

I. Substitution or replacement reactions


A reaction in which one or more hydrogens of the benzene ring are replaced by other monovalent
atoms or groups is called a substitutionreaction.
d

Arenes undergo a number of substitution reactions as discussed below.


in

1. Halogenation. Benzene reacts with chlorine and bromine in presence of Lewis acids such as AICI3,
Re

FeCl3, FeBr3, etc. as catalyst and in absence of light to form chlorobenzene and bromobenzenc respectively.
F

Cl

Anhyd. AICI3 or FeBr3


Q + CL
310-320 K, dark O + HCl

Benzene Chlorobenzene

Br

Anhyd. FcBr3
Q + Br2
310-320 K, dark Q + HBr

Benzene Bromobenzenc
13/76 ‘pfu^eteep- 4- New Course Chemistry (XL) BSi

CH3 CH3 CH3


Cl
Anhyd. FeCl3
Qj + CI2 dark a +
o
Toluene o-ChlorotoIuene (minor)
Cl

w
p-Chlorotoluene (major)

If excess of electrophilic reagent is used, two or more or even all the hydrogen atoms of the benzene
ring may be successively replaced by the electrophile. For example, benzene on treatment with excess of CI2
in Ae presence of anhydrous AICI3 in dark yields hexachlorobenze ne (CgClg)

o
Cl

e
re
Anhyd. AICI3
+ 6CI2 + 6HC1

rFl
dark, cold
Cl

F
Benzene

Cl
Hexachlorobenzene

r
The function of the Lewis acid is to carry the halogen to the aromatic hydrocarbon. That is why these are
ou
sfo
usually referred to as halogen carriers. In addition to iron and aluminium halides, iodine and iron filings
have also been used as halogen carriers. For example, chlorobenzene is formed when benzene is treated with
chlorine in presence of iron. Under these conditions, iron first reacts with chlorine to form ferric chloride
which then acts as a catalyst. k
oo
2Fe + 3CI2 ^ 2FeCl3
Direct iodination of arenes is not a useful reaction since the HI produced during the reaction reduces the
Y
B

aryl iodide back to the arene and iodine.


re

Q + I2 Q + HI
ou
Y
ad

Benzene lodobenzene

However, in presence of oxidising agents such as nitric acid, iodic acid (HIO3) or mercury oxide, the HI
produced is either oxidised to iodine or is eliminated as mercuric iodide and thus ithe reaction proceeds in the
d

forward direction producing iodobenzene.


5 HI + HIO3 3I2 + 3 H2O
in
Re

2 HI + 2 HNO3 -> I2 + 2NO2 + 2H2O


2 HI + HgO 4 Hgl2 + H20
F

Fluorination of arenes is too vigorous to be of any practical use.


2. Sulphonation. The process of replacement of a hydrogen atom of an arene by a sulphonic acid
group (- SO^fl) is called sulphonation. It is usually carried out by treating an arene with fuming sulphuric
acid or oleum (concentrated sulphuric acid containing dissolved sulphur trioxide) or chlorosulphonic acid.
For example.
SO3H

330 K

Q + H2SO4 0. + H2O
(Fuming)
Benzene Benzenesulphonic acid
HYDROCARBONS 13/77

SO3I-I

Q
330 K
+
CISO3H Q + HCI

Chlorosulphonic acid
Benzene Benzenesulphonic acid
3. Nitration. The process of replacement of a hydrogen atom of an arene by the nitro group (- NOj) is
called nitration. It is usually carried out by treating an arene with a mixture of cone. HNO3 and cone.
H2SO4. {nitrating mixture). For example,

w
NO2

Cone. H2SO4
Q + HNO3 323-333 K
-►
Q + H->0

Flo
{Cone.)
Benzene Nitrobenzene

4. Friedel-Crafts reactions. This is a convenient method for the introduction of an alkyl or an acyl
(RCO -) group into an arene. It is of two types :

ee
(i) Friedel-Craftsalkylation. When benzene or its homologue is treated with an alkyl halide, in presence

Fr
of anhydrous aluminium chloride as catalyst, it forms an alkylbenzene. For example,
CH3

for
Anhyd. AICI3
Q +
ur
CH3—Cl A
>
Q + HCI
Chloromethane
Benzene Toluene
{Methyl chloride)
ks
CH2CH3
Yo
oo
o +
CH3CH2—Br Anhyd.A A1C13 ^ o + HBr
eB

Bromoethane
Benzene Ethylbenzene
{Ethyl bromide)

SUPPLEMENT YOUR
KNOWLEDGE FOR COMPETITIONS
r
ou
ad

Rearrangement of carbocations during Friedel-Crafts alkylation reactions.

Friedel-Crafts reaction of alkyl halides either involves the adduct R^.... ClAlClj" as the electrophile or
Y

the carbocation (R"^) which it gives on ionization. With M-alkyl halides at low temperatures, the reaction
mainly occurs through the adduct but at high temperatures, the reaction may occur through carbocation
intermediate. With 3° alkyl halides, reaction occurs only through the carbocation intermediate. With all
nd
Re

other alkyl halides, the chances of carbocation route increase as the branching in the alkyl halide increases.
Whenever carbocations are the intermediates, and if the initially formed carbocation is less stable, then
Fi

it always rearranges first to the more stable carbocation either by 1, 2-hydride .shift or 1, 2-methyl shift
before attacking the benzene ring to give the rearranged product. For example,
High temp. 1, 2-Hydride +

CH3CH2CH2CI + AICI3 -AICI4


> CH3—CH—CH2 shift ► CH3—CH—CH3
«-Propyl chloride Isopropyl carbocation
H {more stable)

n-Propy! carbocation
{less stable)
CH3
C6H6
-H^
C6H5 —CH
CH3
Isopropylbenzene
13/78 Ncv% Course Chemistry (XI)CS19D

It may be emphasized here that at low temperatures, the major product is n-propylbenzene.
Similarly, isobutyl chloride gives rer/-butylbenzene.

CH3 CHj
AICI3 1, 2-Hydride
CH3—CH—CH2CI -AiClJ shift
CH3—c"
Isobutyl chloride
CH3

CH3
C6H6^ CH3—C
-H*
CH3
/er/-Butylbenzene

F low
With neopentyl bromide, the product is 2-methyl-2-butylbenzene.

CH3 CH3
AICI3 1,2-Methyl
CH3—C—CH2CI -►
CH3—C—CH2CH3
-AlCIJ shift

CH3

e
for Fre
Neopentyl chloride
CH3
C6»6 ^ CH3CH2—C

CH3
2-Methyl-2-butylbenzene
Your
eBo ks

(«) Friedel-Crafts acylation. On treatment with a carboxylic acid chloride or the anhydride in presence
of anhydrous aluminium chloride, benzene forms an acylbenzene. For example,
COCH3
O
ad

Anhyd. AICI3
our

O +
CH3—C—Cl
Ethanoy! chloride
Q + HCl

Benzene
{Acetyl chloride) Acetophenone
Re

COCH3

CH3COs^ O
Find Y

Anhyd. AICI3
O -I-
A Q + CH3COOH
Ethanoic acid
Benzene CH3CO Acetophenone
Ethanoic anhydride
(Acetic anhydride)

II. Addition reactions.

Benzene and its homologues usually do not undergo addition reactions because of the extra stability of
However, under drastic conditions, i.e., high temperature
these molecules due to delocalization of 7C-electrons.
and pressure, they do undergo some addition reactions characteristic of alkenes and alkynes. Two such important
addition reactions are discussed below :
HYDROCARBONS 13/79

(a) Addition ofdihydrogen — Hydrogenation. Benzene adds three molecules of dihydrogen in presence
of Raney nickel or platinum as catalyst at 473—^523 K to form cyclohexane.

Reney Ni
+ 3H2 473-523 K

Benzene Cyclohexane
(b) Addition of halogens. In presence of sunlight and in the absence of halogen carriers (such as AICI3,
FeBr3» etc.), benzene adds three molecules of chlorine to form benzene hexachloride (BHC) or Gammexane.

w
+ 3CI2
/jv, 500 K Cl Wo
W
Absence of halohen carrier

Flo
Benzene

e
Benzene hexachloride (BHC)

re
Similarly, Br2 adds to benzene to form benzene hexabromide.
It may be noted here that in absence of sunlight and in the presence of halogen carriers, benzene

F
undergoes substitution reactions but in presence of sunlight and absence of halogen carriers, it undergoes
addition reactions. ur
nL Oxidation reactions

or
Arenes undergo a number of oxidation reactions as discussed below :

f
1. Complete oxidation with air - combustion. Arenes readily bum on heating in air or oxygen producing
CO2, H2O along with a luminous yellow sooty flame. ks
2 CgHg + 15 O2 » 12 CO2 + 6 H2O ; = - 3000 kJ mor^
Yo
The formation of sooty flame is due to the reason that arenes contain higher carbon content than hydrogen.
oo
This sooty flame test is used as a qualitative test to distinguish aromatic from aliphatic compounds.
2. Controlled oxidation with air. Benzene on oxidation with air at 773 K in presence of V2O5 as
B

catalyst gives maleic anhydride.


re

CHCOOH CHCO

(3 + 502
V205,773K A
0
-2CO2.-H2O CHCOOH -H20
u

CHCO
ad

Benzene
Maleic acid Maleic anhydride
Yo

3. Oxidation with potassium permanganate. Arenes are not oxidised by cold dilute alkaline KMn04
solution (Baeyer’s reagent) and hence this reaction is used to distinguish aromatic compounds from alkenes
and alkynes.
d

Benzene, the first member of arenes, is not oxidised by hot KMn04 solution. But its higher homologous
Re
in

are easily oxidised. During this oxidation, the nucleus remains intact but each of its side chains (irrespective
of its length) is oxidised to a carboxyl (- COOH) group. For example.
F

CH3 COOH

(0 KMn04, KOH,

Oj (3 + H20
373-383 K

(lY) Dil. H2SO4


Toluene Benzoic acid

CH2CH3 COOH

(0 KMn04, KOH,

(3 (3 + CO2 + 2H20
373-383 K

(//)Dil.H2S04
Ethylbenzene Benzoic acid
13/80 'Pxeidecfi.'A New Course Chemistry (XI)S!EIHD

CH3 COOH

CH3 (/) KMn04, KOH, COOH

Q
373-383 K
+ 6[0]
{ii) Dil. H2SO4 Q + 2H2O

o-Xylcnc Phthaiic acid

Similarly. «z-xylene and /?-xylene on oxidation give isophthalic acid and terephthalic acid respectively.
CH3 COOH CH3 COOH

ow
(0 KMn04. KOH. (0 KMn04, KOH,

Q * CQ
373-383 K

^ IQ
373-383 K

(it) Dil. H2SO4 (it) Dil. H2SO4 Q


CH3 COOH
m-Xylene Isophthalic acid
CH3 COOH
p-Xylcne Terephthalic acid

e
Fl
With mild oxidising agents like chromyl chloride (CrO^Cl2). toluene gives benzaldehyde. This reaction

re
is called Etard reaction.

CH3 CHO

F
(i) Ct02Cl2
Q Q
ur
r
{/■/) H2O (Etard reaction)

fo
Toluene Benzaldehyde
4. Oxidation with ozone. Like alkenes and alkynes, arenes also undergo ozonolysis. For example,
ks
benzene reacts with O3 to form a triozonide which upon decomposition with Zn /H2O gives glyoxal.
Yo
O
oo
O
.CH
/
eB

CH
0
O—CH CH=0
03/CH2CI2 Zn/CH20
0 3
196 K CH -ZnO
o CH=0
\o
o—CH (reductive cleavage)
ur

Benzene
Glyoxal
CH
ad
Yo

O
Benzene triozonide

5. Pyrolysis. Biphenyl is obtained when benzene vapours are passed through a red hot tube, preferably
d

packed with pumice at 873-1073 K.


Re
in

Pumice
+ H2
// 873-1073 K
F

Benzene Biphehyl
A more recent method for preparing biphenyl is to mix benzene vapours (preheated to 923 K) with
superheated steam (1273-1373 K) and passing the mixture into steel vessels coated internally with a film of
Fe304.
Unlike alkenes and alkynes, ozonolysis of arenes is not of any practical importance.
13.8.10. Uses of Arenes

(0 Lower arenes such as benzene, toluene, xylenes, etc. are used as solvents for oils, fats, rubbers, etc.
(ii) Arenes are extensively used in the manufacture of dyes, drugs, perfumes, explosives (TNT) and
insecticides (DDT. BHC).
HYDROCARBONS 13/81

(Hi) Benzene is blended with petrol to improve its octane number,


(/v) p-Xylene is used to prepare terephthalic acid and its ester for manufacture of synthetic fibre, terylene.
13.8.11. Mechanism of Electrophilic Substitution Reacti'^ns
Benzene contains two 7C-eleciron clouds-one lying above and the other below the plane of the ring. As
compared to o- electrons, these TC-electrons are loosely held and hence are easily available to electrophiles.In
other words, benzene rin^ acts as a source of electrons (i.e., nucleophile) and attracts electrophiles (or
repels nucleophiles). Thus, the reactions of benzene and other arenes are initiated by electrophiles. Now

low
electrophilic reactions can, in principle, be either substitution or addition. Whereas benzene and other arenes
undergo electrophilic substitution reactions, alkenes and alkynes undergo electrophilic addition reactions.
In fact, ability of a compound to undergo electrophilic substitution reactions is an excellent test of aromaticity.
The mechanism of all the electrophilic substitution reactions is basically the same and proceeds through
the following steps :
Step 1. Generation of an electrophile. First of all, an electrophile is generated as a result of some

e
preliminary reaction.

re
rF
F
E-l-Nu E* :Nu"
Reagent Electrophile Nucleophile

In some cases, an electrophile is not generated directly but a polarized molecule of the reagent serves as

r
the electrophile. For example, in halogenation and Friedel-Crafts alkylation reactions.

fo
u 5+ 6-
ks 8+ 5-

Br CH3-^C1 + AICI3 CH3 CIAICI3


Br + FeBr3 Br BrFeBr3 ;
Yo
Methyl chloride Polarized methyl
Bromine Polarized bromine
chloride molecule
oo
molecule {elecuvphile) {electrophile)

Step 2. Formation of carbocation intermediate. As the electrophile approaches the benzene ring, it is
B

attracted by the jc-electrons to form a 7t-complex.


re

O
u

Tt-Complex
ad
Yo

The 7t-complex then slowly reorganises or rearranges to form a o-complex or a carbocation intermediate
which is stabilized by resonance
d

H H H
Re

Slow
+ E E
in

>

Resonance
F

Resonance stabilization of carbocation intermediate or a-complex hybrid

The carbocation intermediate is also called arenium ion or o-complex.

During the fonnation of o-complex, the aromatic character of the benzene ring is destroyed. Therefore,
this step is slow and hence is the rate determining step of the reaction.
Step 3. Loss of a proton from the carbocation intermediate. The carbocation intermediate formed above,
in principle, can undergo the following two types of reactions.
(/) It can lose a proton to the nucleophile (Nu~) present in the reaction mixture to form a substitution
product

f
13/82 T^^nadeefr’^i, New Course Chemistry (XI)EaEX9D

Fast
+ :Nu" + H—Nu

Substitution product

Since during this step, aromatic character of the benzene ring is restored, therefore, this step is fast and
hence does not affect the rate of the reaction.
(h) Alternatively, the nucleophile (: Nu ) may attack the carbocation intermediate to yield the addition
product.
H Nu

H H
E + :Nu" E

This addition is, however, not observed since during this addition reaction, the aromatic character of the

F low
benzene ring is lost.
Let us now discuss the mechanism of some typical electrophilic substitution reactions.
1. Halogenation. The halogenation of benzene and some other arenes is carried out at low temperatures
(310-320 K), in the absence ofsunlight and in presence ofa Lewis acid such as anhydrous ferric or aluminium
chloride as catalyst. The halogenation proceeds by the following mechanism :
Step 1. Generation of an electrophile.

for Fre
5+ 5-
Cl-*-Cl + AICI3 Cl AICI3
Chlorine Electrophile
Step 2. Formation of d-complex or carbocation intermediate. The +ve end of the polarized chlorine
molecule accepts the jc-electron cloud of the benzene ring to form a o-complex or the carbocation intermediate
which is stabilized by resonance.
Your
eBo ks

+ Cl-^-(CIAICI3 Slow ^ Cl + AICI4


ad

a-Complex
our

H H H H
Cl ◄ Cl ◄ ► Cl + Cl
Re

Resonance
Resonance stabilization of carbocation or o-complex hybrid

This step is slow and hence is the rate-determining step of the reaction.
Find Y

Step 3. Loss of a proton from the carbocation intermediate. The base (AICI4) present in the reaction
mixture then abstracts a proton to form chlorobenzene
H-* Cl
Fast
Cl + AICI4 + AICI3 + HCl
Chlorobenzene

This Step is fast and hence does not affect the rate of the reaction.
2. Nitration. It is carried out by treating benzene and other arenes with a mixture of cone. HNO3 -1-
conc. H2SO4 commonly called the nitrating mixture. The various steps involved are :
HYDROCARBONS 13/83

Step 1. Generation of an electrophile, i.e., NO2 (nitronium ion)


In presence of cone. H2SO4, HNOj acts as the base and accepts a proton to form protonated nitric acid
which then loses a molecule ofH20 to form nitronium ion.
H2SO4 > H+ + HSO4

H"" + H—O—NO2
f ●
H
a
"0-^N02 > H2O + NO2
Nitric acid H Nitronium ion

w
Step 2. Formation of <5-complex or carbocation intermediate. Nitronium ion attacks the benzene ring
to form a carbocation which is stabilized by resonance.
H

F lo
+ NO^
Slow
► IP 1^N02.< ► "^N02-^ ► ^N02 NO2
Resonance
“ Resonance stabilization of carbocation or the a-complex hybrid

ee
This step is slow and hence is the rate-determining step of the reaction.

Fr
Step 3. Loss of a proton from the carbocation intermediate. Finally, the carbocation loses a proton to
the base (HSOp to form nitrobenzene.

for
H-*- NO2
our
● Fast
NO2+ HSO4 > + H2SO4
Nitrobenzene
s
This step is fast and hence does not affect the rate of the reaction.
ook
Evidence in support of the mechanism. This mechanism has been supported by absence of isotope
effect. It has been found that when hexadeuterobenzene is nitrated, the rate of nitration is exactly the same as
Y
eB

that of benzene. Since a C-D bond breaks more slowly than a C-H bond, therefore, the loss of a proton is not
the rate-determining step. Had it been so, the rates of nitration of benzene and hexadeuterobenzene would
have been different ?
r

D NO2
ad
ou

HN03,H2S04 ^
a A Q
Y

D D D D

D
Re

D > Rates are the same


nd

Hexadeuterobenzene

NO2
Fi

HN03,H2S04 ^
JL
a A Q
Benzene J
Thus, electrophilic aromatic substitution is a bimolecular reaction in which the attack ofthe electrophile
and loss ofproton occur in two different steps and theformer is slower and hence is the rate determining step
of the reaction.
3. Sulphonation. Sulphonation is carried out by treating benzene or any other arene with concentrated
sulphuric acid or oleum (cone. H2SO4 + SO3 = H2S2O7). The mechanism for sulphonation involves the
foUowing steps:
13/84 New Course Chemistry (XI)EZ

Step 1. Generation ofan e/ecrro/?/j//^.Sulphur trioxide which acts as an electrophile is first of all generated
by the following acid-base equilibrium between two molecules of sulphuric acid.

2H2SO4 SO3 + HSO4 + H3O+


Step 2. Formation of<s-complex or carbocation intermediate. Sulphur atom of sulphur trioxide molecule
is electron-deficient as shown below.

w
O
?o
o o

Ao ^ *Q^\
*

O O" -o o
Consequently, sulphur trioxide acts as an electrophile and hence attacks the benzene ring to give a

o
carbocation which is stabilised by resonance.

e
re
H H H H
Slow

rFl
AO" " S03<4—► S03 ◄—► "SOJ ^ I + SOJ
-

F
O
Resonance
Resonance stabilization of carbocation or a-complex (I) hybrid
This step is slow and hence is the rate-determining step of the reaction.

or
ou
Step 3, Loss of a proton from the carbocation. The carbocation (I) finally loses a proton to produce
sulphonic acid anion (II). This step is fast and hence does not affect the rate of reaction.

rA: SOJ+
ksf
SOJ
Fast
HSO4 + H2S04
oo
I n

Step 4. Addition ofproton to the benzenesulphonic acid anion (II) gives the final product.
Y
B

+ H3O* ^ + H2O
re

n Benzenesulphonic acid
This equilibrium lies far to the left since sulphonic acid is a strong acid.
oYu

It may be pointed out here that unlike other electrophilic aromatic substitution reactions, sulphonation
ad

is reversible.

Because of the reversibility of this reaction, sulphonic acid group can be easily removed by heating a
sulphonic acid with steam under pressure or by hot dilute mineral acids.
d

o
H^A
in
Re

+ H2O Under pressure + H2SO4


Steam
Benzenesulphonic acid
F

Benzene

This reaction is called desulphonation.


4. Friedel-Crafts alkylation. Alkyl substituted benzenes may be prepared by the reaction between
benzene and a suitable alkyl halide in presence of a Lewis acid catalyst (AICI3, BF3, FeCl3, etc).
R

o
Benzene
+ R—Cl
Alkyl chloride
Anhyd.AlCl3^
A

Alkylbenzene
+ HCl

The reaction is believed to proceed through the following mechanism.


HYDROCARBONS 13/85

Step 1. Generation ofan electrophile. The alkyl halide first reacts with anhydrous AICI3 to form polarized
alkyl halide molecule which acts as the electrophile
§+ §—
R-L-CI + AICI3 > R —CIAICI3
Alkyl halide Electrophile
Step 2. Formation of (5-complex or carbocation intermediate. The +ve end of the polarized alkyl halide
ring to form a a-complex or the carbocation intermediate
molecule attacks the it-electron cloud of the benzene
which is stabilized by resonance.
+
H

+ R-t-ClAlCl3 Slow ^

w
R + AlCl^
a-Complex

F lo
H H H

[fSY R ^ R I
I
+ R

ee
Resonance stabilized carbocation or a-complex Resonance hybrid

Fr
This step is slow and hence is the rate-determining step of the reaction.
Step 3. Loss of a proton from the carbocation.

for
H R
Fast
R + AICI4 + AICI3 + HCl
ur
Alkylbenzene
s
This step is fast and hence does the affect the rate of the reaction.
ook
Yo
5. Friedel-Crafts acylation. Benzene and other arenes react with acid chlorides or anhydrides in presence
of anhydrous AICI3 to form aromatic ketones. For example.
eB

Anhyd.AlCl3 ^
^^^C0CH3
+ CH3—C—Cl A
+ HCl
Acetyl chloride
our

Benzene Acetophenone
ad

+
CH3COv^ o AnhydAlCb ^ ^^C0CH3
A + CH3COOH
CH3CO
Y

Benzene Acetophenone
Acetic anhydride
Re

o o
nd

.c c
Fi

+
Cl
AnhydAlCb
A
^ + HCl

Benzene Benzophenone
Benzoyl chloride
Mechanism. The reaction nvolves the following steps ;
Step 1. Generation ofan electrophile. The acid chloride or anhydride reacts with anhydrous aluminium
+

chloride to form acylium ion, (RC = O) which serves as an electrophile.


O
+

R—C-^-Cl + AICI3 > R—C=0: <■ ► R—c=o:J + AICI4


Acid chloride Acylium ion

I
13/86 ‘Pruidcc^'4. New Course Chemistry (X1)BZSI9D

O
O
R—C
+

O + AICI3 R—C=0: < >R—c^o: + R—C—OAICI3


R— Acylium ion

o
Acid anliydride
Step 2. Formation of G-complex or carbocation intermediate. The acylium ion attacks the benzene irng
resulting in the formation of a carbocation which is stabilized by resonance.

w
H H H
Slow
+ C—R — Rm—► C—R I
I
+ C—R

F lo
0 O o o o
Acylium ion Resonance stabilized carbocation or a-complex Rcsconance hybrid

This step is slow and hence is the rate-determining step of the reaction.

ee
Step 3. Loss of a proton. The carbocation loses a proton to the base (AICI4 or R—COOAICI3)

Fr
present in the reaction mixture to form the final product.

for
—R + HCl +AICI3
Fast
C—R + AICI4
O
r
O Acylbenzene
You
s
This step is fast and hence does not affect the rate of the reaction.
ook

13.8.12. Directive Influence of Substituents and their Effect on Reactivity


eB

All the six hydrogen atoms of benzene irng are equivalent. Therefore, replacement of any one of these
six hydrogen atoms by any substituent always gives a single monosubstituted benzene derivative. However,
when a monosubstituted benzene derivative is converted into a disubstituted benzene derivative, the substituent
already present in the benzene ring determines the position of the incoming group.
our
ad

This ability of a group already present in the benzene ring to direct the incoming group to a particular
position is called the directive influence of groups.
On the basis of the directive influence of groups, the various substituents can be divided into the following
two categories:
dY
Re

1. o, p-Directing groups. The substituents or groups which direct the incoming group to ortho and
para positions are called ortho, para-directing groups. For example, - CH3, - CH2CH3, - CgH5, - Cl,
Fin

- Br, ~ I, - OH, - OCH3, - OCOCH3, - NH2, - NHCH3, - N(CH3)2. - NHCOCH3, etc.

CH3 CH3 CH3

cone. HNOi
—►
+ cone. H2SO4
Toluene
o-Nitrotoluene (minor)
NO2
p-Nitrotoluene (major)

Thus, nitration of toluene gives a mixture of ortho- and />«ra-nitrotoluenes.

I
HYDROCARBONS 13/87

In general, all electron-donating groups are o, p-directing. Further, it may be pointed out here that
although two o- and one p-positions are available for disubstitution yet due to steric hindrance with the
incoming group at o-position, it is usually the p-isomer which predominates in these substitution reactions.
Explanation for directive influence of o, p-directing groups. With the exception of alkyl and phenyl
groups all other groups have atleast one lone pair of electrons on the atom directly attached to the benzene
ring. This lone pair of electrons is involved in resonance with the 7C- electrons of the benzene ring. As a result
of resonance, the electron density increases at all the nuclear positions of the benzene ring but the increase in
electron density is much more at o- and p-positions (structures II-IV) than at m-position as shown below :
● ● ● ● ● ● a ●

:OH +OH +OH :OH


q:

w
> <■ <■ <■

F lo
I
Phenol n III rv V

(Resonance in phenol)

ee
We have discussed above that the typical reactions of benzene are electrophilic substitution reactions in

Fr
which an electrophile attacks the benzene ring. Evidently the electrophile will attack the benzene ring at a
position where the electron density is high. Since the electron density is high at o- and p-positions, than at m-
positions in phenol, therefore, the electrophile will attack preferentially at o- and p-positions. Thus, OH

for
group is o, p-directing.
ur
Similarly, we can explain the o, p-directing influence of the - NH2 group.
Directive influence of the alkyl group. The alkyl group (R) does not have a lone pair of electrons. Its
s
directive influence can be explained on the basis of hyperconjugation effect as shown below :
ook
Yo
H H H H H
I I
eB

H—C H-" H—C H* H—C—H


H-C^H
H—C

M ► ◄ ► ◄ ►
our
ad

Toluene

(Hypeiconjugadon in toluene)
Here again, electron density increases at o- and p- positions and hence directive influence ofCH^ group
Y

is o, p-directing.
Re

Effect of o, p-directing substituents on reactivity. Since o-, p-directing groups increase the electron
nd

density in the benzene ring, therefore, the ring gets activated and the further electrophilic substitution in the
ring becomes easie.r It is because of this reason that all o, p-directing groups except halogens are called
Fi

activating groups. Further, because of the ability of these groups to donate electrons to the benzene ring,
0-, p- directing groups are also called electron-repelling or electron-donating groups (EDG). In the light
of above discussion, two cases arise :
(/) o, p-Directing groups facilitate further electrophilic substitution reaction. For example, nitration of
toluene occurs faster than that of benzene because due to hyperconjugation effect of the CH3 group, electron
density in toluene ring is higher than that in benzene ring,
(/i) Higher the electron-donating ability of the substituent, more facile is the reaction. For example,
bromination of aniline occurs faster than that of phenol because due to lower electronegativity of N over O,
the electron-donating ability of - NH2 group is much higher than that of -OH group. As a result, electron
13/88 New Course Chemistry (X1)ESZS19D

density in the aniline ring is much higher than that in phenol and hence bromination of aniline occurs faster
than that ofphenol.
The electron-donating ability of some substituents follows the order:
-0->NH2>-NR2>-0H>-0CH3,-NHC0CH3>-CH3>-X(-F>-Cl>-Br>-I)
2. m-Directing groups. The substituents or groups which direct the incoming group to the meta-position
are called meta- directing groups. For example.

ow
(CH3)3N- , -NO2, -CN, -CF3, -CHO, -COR, -COOH, -COOR, -SO3H, etc.
Thus, nitration of nitrobenzene mainly gives m-dinitrobenzene.
NO2 NO2

Cone. H2SO4

e
+ HNO3 393 K
+ H2O
tjuming)

re
rFl
Nitrobenzene
m-Dinitrobenzene

F
In general, all electron-withdrawing groups are m-directing.
Explanation for the directive influence of m-directing groups. With the exception of trifluoromethyl,
trichloromethyl (i.e., CF3, CCI3) groups, in all other groups the atom directly attached to benzene ring has

r
one more electronegative atom linked to it by a multiple bond. This more electronegative atom pulls the
ou
fo
electrons of the multiple bond towards it which, in turn, withdraws electrons from all the nuclear positions.
As a result, electron density falls at all the nuclear positions but the decrease is much more at o- and p-
positions than at m-positions as shown below ;
(>
H—C^O
ks H-C-^0-
oo
H—C—O- H—C—O" H—C=0

el
Y
eB

◄ ►
9 ^ b ◄ ► < ►

Benzaldehyde
(Resonance in benzaldehyde)
ur

In Other words, the electron density is comparatively higher at m-positions than at o- and p-positions
and hence the electrophile will attack preferentially at the m- position in benzaldehyde. Thus, - CHO group
ad
Yo

is m- directing.
Similarly, we can explain the meta-directing influence of the - NO2 group. Due to electron-withdrawing
resonance effect (Le., -R-effect) of the - NO2 group, electron density falls more at the o- and p-positions that
d

at m-positions as shown below :


Re
in

&
V
-Q -Q “O O" -Q
\+/ \.y ■V/
F

N N N N

0
◄ —► ◄ —► ◄ —►

I II * m IV V
Nitrobenzene (Resonance in nitrobenzene)
In other words, electron-density is comparatively more at m- positions than at o- and p-positions,
therefore, further substitution in nitrobenzene will occur at m-position.
Effect of m-directing substituents on reactivity. Since m- directing groups decrease the electron-
density in the benzene ring, therefore, the ring gets deactivated and hence further electrophilic substitution
becomes difficult. It is because of this reason that meta-directing groups are also called deactivating groups.
HYDROCARBONS 13/89

Further, because of their ability to withdraw electrons from the benzene ring, meta-directing groups are also
called electron-attracting or electron-withdrawing groups (EWG).
In the light of above discussion, the following two cases arise :
(<) m-Direciing substituents make further electrophilic substitution more difficult. For example, nitration
of benzene occurs faster than that of nitrobenzene. This is due to the reason that nitro group because of its
electron-withdrawing -1- and - R- effect reduces the electron density in nitrobenzene ring while there is no

w
such effect operating in benzene. As a result, electron density in benzene ring is higher than that in nilrob nzene
ring iind hence the electrophile (i.e., NOJ ) will attack benzene faster than nitrobenzene.
(«) Higher the electron-withdrawing ability of the substituent, more difficult is the reaction. For example,
nitration of nitrobenzene occurs much slower than that of benzoic acid since NO2 group is a much more
powerful electron- withdrawing group than carboxylic acid group. As a result, electron density in benzoic

o
acid ring is much higher than that in nitrobenzene ring and hence further electrophilic substitution will occur

e
faster in benzoic acid than in nitrobenzene.

re
Directive influence of halogens. We have discussed above two types of groups : (/) o. p-directing

Frl
groups which activate the benzene ring towards further electrophilic substitution reactions and (ii) /^-directing
groups which deactivate the ring towards further electrophilic substitution reactions. Besides these, there is a

F
third category of groups, i.e., halogens which though deactivating are o, p-directing. This is explained as
follows :
In case of ary halides, halogens are little deactivating because of their strong - I-effect. Therefore,
ou
or
overall electron density on the benzene ring decreases. In other words, halogens are deactivating due to
-I-effect. However, because of the -i-R-effect, i.e., participation of lone pairs of electrons on the halogen atom
with the 7t-electrons of the benzene ring as shown below, the electron density increases more at o- and p-

kfs
positions than at w-positions.
:ci: ■^Ci: ^ci:
(^1: ;ci:
oo
Y
●4 ●4
B

Chlorobenzene

(Resonance in chlorobenzene)
re

As a result, halogens are o, p-directing. The combined result of +R-effect and -I-effect of halogens is
that, halogens are somewhat deactivating but o, p-directing.
oYu
ad

SUPPLEMENT 1CCIUR
KNOWLEDGE FOR C IONS

Besides the charge distribution method discussed above, the direcuve influence of groups can belter be explained
d

on the basis of the stability of the intermediate carbocations, {i.e., a-complexes) involved in the substitution,
(i) Directive influence of electron-withdrawing groups. Let us illustrate this by taking the example of
in

nitration of nitrobenzene. The intermediate carbocations involved in o, p- and /n-substitution respectively are:
Re

NO2 NO2 NO2


F

H H H

0-Attack: NO2 4 ■NO2 -4 NO2


! II III

NO2 NO2 NO2

p-Attack : 4 4

H NO2 H NO2 H NO2


IV V VI
13/90 7^<uUt^ it New Course Chemistry (XI) orsTMn

NO2 NO2 NO2

m-Attack: H ●4 H 4 > H

NO2 NO2 ●NO-,


VII VIII IX

In case of o- and p-substitution, there is one resonance structure (I and V) in which the +ve charge is
present on that carbon which is attached to electron-withdrawing NO2 group. Therefore, this structure is
less stable than the other two. In other words, both the a-complexes resulting from o- and p-substitution

w
are relatively less stable. On the other hand, there is no corresponding resonance structure for the carbocation
resulting from m- substitution. Therefore, the (5-complex resulting from m-substitution is more stable that
those resulting from o- and p-substitution and hence - NO2 group is m-directing.

F lo
(ii) Directive influence of electron-donating groups. Let us illustrate this by taking the example of
nitration of toluene. The intermediate carbocations (only one structure is shown, other structures
corresponding to carbocations, II, III, IV, VI, VIII and IX can be easily drawn) involved in the o, p and m-

ee
substitution respectively are:
0-Attack p-Attack m-Attack

Fr
CH CH3

for
H
ur
I X VII
N02
H NO2
ks
V

Carbocations (I and V) are stabilized by 4-I-effect of the CH3 group. Since, there is no corresponding
Yo
oo
stabilization in the carbocation resulting from m-substitution, Aerefore, CH3 group Is o,p-directing.
(Hi) Directive influence of chlorine atom. Let us consider the following structures :
eB

■"ci: *ci:
^a: ^ci:
:ci:

H H
r

NO2 4 > NO2 : 4 H


ou
ad

I la X X VII
N02
H NO2 H NO2
Y

V Va

For o- and p-substitution, carbocations I and V are stabilized by +R-effect of the Cl atom. Since, this is not
possible for m-substitution (carbocation VII), therefore. Cl is o, p-directing. Further, since -I-effect of Cl
nd
Re

IS stronger than +R-effect, therefore, irng is deactivated and hence rate of electrophilic substitution in
benzene or toluene is higher than in chlorobenzene.
Fi

The advantage of the above method is that it can be used to explain the directive influence of certain
groups such as - CH = CH2 or CgH^ which otherwise cannot be explained by charge distribution method.
(For details, refer to Ans. to Q. 28(0, page 13/123)

13.8.13. Directive Influence of Electron withdrawing


Groups in Nucleophilic Substitution Reactions
Refer to resonance structures (I-V) of nitrobenzene on page 13/88.
The presence of positive charge at o-and p-positions in resonance structure (II-IV) suggests that
nitrobenzene can also undergo nucleophilic substitution reactions at these positions. These reactions, however,
occur with strong nucleophiles under drastic conditions. For example, nitrobenzene when heated with KOH
at 340 K gives a low yield of o-and p-nitrophenols. Thus,
HYDROCARBONS 13/91

NO2 NO-7 NO2


OH
340 K
+ (KOH) +

(solid)
Nitrobenzene o-Nitrophenol
OH
p-NitrophenoI

w
13.8.14. Polynuclear Aromatic Hydrocarbons - Carcinogenicity and Toxicity
Hydrocarbons containing two or more fused or isolated benzene rings are called polynuclear aromatic
hydrocarbons. These may either be linear (naphthalene, anthracene, etc.) or angular (phenanthrene, pyrene, etc.)

Flo
Q
DIQ DIOIQ DIQ

e
re
Naphthalene Anthracene Phenanthrene
Q

rF
Pyrene
The main source of polynuclear aromatic hydrocarbons is coal tar. Naphthalene is the largest single
constituent (6 - 10%) of coal tar. It is obtained by cooling the middle oil fraction (b.p. 443 - 503 K) of coal
ur
fo
tar distillation. Anthracene is obtained from green oil fraction (b.p. 543 - 633 K) of coal tar distillation in
about 1% yield.
Benzene and most of the polynuclear aromatic hydrocarbons are cancer-producing, i.e., are carcinogenic.
ks
Notable among these are 1, 2-benzanthracene, 3-methylcholanthren e, 1, 2-benzpyrene, 9, 10-dimethyl-1, 2-
Yo
benzathracene and 1, 2, 5, 6-dibenzanthracene.
oo

Q DID
B

DIOIQ? DIOIQ
re

1.2-Benzanthraccne
1, 2-Benzpyrenc
u
ad

3-MethychoIanthene
Yo

7
4
IQ
loioioi
d
Re

Q
in

CH3
F

9, lO-Dimelhyl-l, 2-benzanthraene 1. 2. 5, 6-Dibcnzanthracenc

There is no general rule by which one can predict the carcinogenic activity of a polynuclear aromatic
hydroccirbonor its derivative. However, the number and position of cenain groups such as - CH3, - OH,
- CN, - OCH3, etc. have been found to influence carcinogenic activity.
Cancer is primarily an environmental disease. Just as draining of swamps and elimination of mosquitoes
can be used to control malaria and yellow fever, in the same way, elimination of carcinogens from the
environment can reduce the incidence of cancer to a great extent.
In fact, polynuclear aromatic hydrocarbons are the products of incomplete combustion of organic matter
such as coal, petroleum, tobacco, etc. They are widely present in the environment and thus are the major
cause of human cancer.
13/92 New Course Chemistry (Xl)Cd9D

It is believed that when these polynuclear hydrocarbons enter the body of a human being, they are first
converted into their oxides called epoxides and then into dihydroxy epoxides. The dihydroxy epoxides thus
produced react with the purine bases such as guanine present in DNA and RNA of the human cells. The
attachment of this big hydrocarbon part to purine prevents it from fitting into the double helix of DNA. This
damage causes mutations and ultimately leads to cancer.
A simplified view of carcinogenic effect of polynuclear aromatic hydrocarbons (PNH) is shown below :
o2
PNH —> PNH epoxide PNH dihydroxy epoxide

w
DNA or RNA
Mutations ●> Cancer

F lo
13.9. TYPICAL PROBLEMS

P. 1. Two isomeric cyclic dienes X and Y having molecular formula C^Hg on hydrogenation give
cyclohexane as the only product. A mixture of these two dienes on reductive ozonolysis gives
succinaldehyde, propane-1, 3-dial and glyoxal. Deduce the structures of dienes X and Y.

ee
Sol. Since, X and Y on hydrogenation give cyclohexane as the only product, therefore, X and Y must be

Fr
cyclohe.xodienes. The two possible cyclohexadienes are :

for
and
ur
Cyc!ohexa-l, 4-dicnc, X Cydohexa-l, 3-diene, Y

Whereas reductive ozonolysis of X will give two molecules of propane- 1, 3-dial, reductive ozonolysis
s
of Y will give succinaldehyde and glyoxal as shown below :
ok
Yo
6
o

CHO OHC
eB

1 5 (0 O3/CU2CI2, 196 K
CHr -I-
:ch2
2 4 Hi) Z11/H2O CHO OHC
3 Propane-1, 3-dial {two molecules)
Cyclohexa-1,4-diene pC)
r
ad
ou

(0 O3/CH2CI2, 196 K
CHO OHC—CH2
+

(jf) Z11/H2O CHO OHC —CH2


Y

Cyclohexa-l, 3-diene (Y) Glyoxal Succinaldehyde


Re
nd

P. 2. 7-Bromo-l, 3, 5-cycloheptatriene exists as an ion whereas 5-bromo-1, 3-cyclopentadiene does not


form an ion even in presence of Ag'*'. Explain.
Fi

Sol. 7-Bromo-l,3. 5-cyc!oheptatriene.on ionization, gives tropylium ion. Since, tropylium ion contains 6 Ji-
electrons which are completely delocalized, therefore, according to Huckel rule, it is aromatic and hence
stable. Being highly stable, it is easily formed.

Ionization + Br"

Tropylium ion
7-Bromo-l, 3, 5- (6n-electrons, aromatic,
cycloheptatriene highly .stable)
HYDROCARBONS 13/93

I Ag^
+ AgBr

5-Bromo-l, 3-
l, 3-Cyclopenladienyl caiion
{An-eledrons,
cyclopentadiene
aniiaromaiic, kighly tinsUihle)
In contrast, 5-bromo-l, 3-cyclopentadiene, on ionization, will give 1, 3-cyclopentadienyl cation which
contains 4 7t-electrons and hence is antiaromalic. Being antiiiromatic, it is highly unstable and hence is
not formed even in the presence of Ag"*" ion which otherwise facilitates ionization.

w
/A

F lo
I. Alkanes

1. The acyclic saturated hydrocarbons are called alkanes. Their general formula is C„H2„^2 where n = 1, 2,
3...etc.

e
All the carbon atoms in alkanes are j/?^-hybridized.

Fre
2. Hydrogenation of alkenes and alkynes in presence of a catalyst such as Raney Ni, Pt or Pd at room
temperature gives alkanes. However, with ordinary Ni, reduction occurs at higher temperatures
(523-573 K) and is commonly known as Sabatier and Sendern’s reduction. This hydrogenation reaction

for
is used in the manufacture of Vanaspati Ghee from edible vegetable oils.
3. Grignard reagents react with substances containing active hydrogen (Le., H,0, ROH, H2S, RSH, NH3,
RNH2, R2NH, HC = CH, RC s CH, RCOOH, etc.) to give alkanes corresponding to the alkyl group of
r
Grignard reagent.
You
ks
R-MgX + H-OH -> RH + Mg(OH)X
o

With heavy water (D2O), deuterated alkanes are formed.


eBo

R-MgX + D-OD ■> RD + Mg(OD)X


4. Wurtz reaction of alkyl halides with metallic sodium in dry ether gives symmetrical alkanes, i.e., alkanes
containing even number of carbon atoms.
Dry ether
our
ad

R-X + 2Na + X- R > R - R + 2 NaX

5. Corey-House reaction can be used to prepare both symmetrical and unsymmetrical alkanes.
Dry ether
R-X + 2Li > R-Li + LiX
Alkyl halide Alkyllithium
dY
Re

Dry ether
2 R-Li + Cul 4
[R2CU]- Li+ + Lil
Fin

Lithium dialkylcopper
Dry ether
[R2Cu]-Li-^ + R'-X > R - R' + RCu + LiX
(where R and R' may be same or different.)
6. Reduction of alkyl halides with Zn/HCl, Sn/HCl or Zn-Cu couple and alcohol or H2 in presence Pd - C
gives alkanes.
Reduction of 1° and 2“ alkyl halides with LiAlH^ gives alkanes while 3® alkyl halides mainly undergo
dehydrohalogenation to give alkenes. On the other hand, NaBH4 reduces 2® and 3® alkyl halides but not I®
whereas Ph3SnH reduces all the three types of alkyl halides to give alkanes.
7. Reduction of alkyl halides with HI and red P at 423 K gives alkanes.
423K,r«dP
CH3CH2—I + HI ^ CH3—CH3 + I2
13/94 'Pmidee^ ^ New Course Chemistry (Xl)CJSani

8. Decarboxylation of sodium salts of fatty acids with soda-lime (NaOH + CaO) at 630 K gives alkanes
CaO,630K
RCOONa + NaOH > R-H -f- NajCOj

ow
For example, decarboxylation of sodium acetate gives methane and that of sodium propionate gives ethane,
y. Kolbe’s electrolysis of Na or K salts of fatty acids gives alkanes. For example, electrolysis of sod. acetate
gives ethane while that of sod. propionate gives n-butane.
10. Action of water on beryllium and aluminium carbide gives methane.
Be^C + 4 H2O > CH4 -H 2 Be(OH)2 ; AI4C3 + 12 H2O > 3 CH4 + 4 Al(OH)3
11. Boiling points. The boiling points of straight chain alkanes increase regularly with the increase in the
number of carbon atoms. On the average, the b.p. increases by 20-30 K for addition of each carbon atom

re
to the chain.

12. Amongst isomeric alkanes, boiling points decrease with branching. For example, boiling points decrease

Flr
F
in the order : n-pentane > isopentane > neopentane.
13. Melting points. The melting points of n-alkanes with even number of carbon atoms are much higher than
those of the next lower alkanes with odd number of carbon atoms — Alternation effect. Amongst isomeric

ou
pentanes, due to symmetry the m.p. of neopentane is the highest. This is followed by n-pentane while

sr
isopentane has the lowest m.p.
14. Solubility. Being non-polar, all alkanes are insoluble in water but are highly soluble in organic solvents

kfo
such as ether, benzene, carbon tetrachloride, etc.
All alkanes are lighter than water.
15. Chemical Properties of Alkanes. The C-C and C-H a-bonds of alkanes are quite strong which do not
oo
break under ordinary conditions. Therefore, alkanes are inert towards most of the common reagents such
as acids, alkalies, oxidising and reducing agents and hence are called paraffins. They, however, preferably
Y
undergo substitution reactions at high temperatures by free radical chain mechanism in which one or
reB

more H-atoms of alkanes are successively replaced by the reagent. Some free radical chain reactions are
discussed below.

(/) Halogenation. Replacement of one or more hydrogens of alkanes by halogens is called halogenation.
uY

It is carried out by reacting alkanes with halogens either in presence of UV light or at high temperatures
(523 - 673 K). The order of reactivity of halogens is : F2 > CI2 > Br2 > l2- The reaction with F2 is very
violent but the reaction with I2 is very slow and reversible. Therefore, iodination is carried out in presence
ad
do

of an oxidising agent such as HNO3, HIO3, etc.


The ease of substitutionof different hydrogensfollowsthe order: 3° > 2® > 1° but their relative rates vary
with the nature of halogen. With CI2, the relative rates of substitution of 3", 2® and 1® hydrogens at 298 K
in

is 5 : 3-8 : 1 while with Br2, it is 1600 : 82 : 1 at 400 K.


(«) Nitration : Nitration of alkanes is carried out with cone. HNO3 in the vapour phase at 423-673 K
Re

under pressure. Since HNO3 is a strong oxidising agent, therefore, during nitration, cleavage of C-C
F

bonds also occurs. Therefore, nitration of alkanes usually gives a mixture of nitroalkanes.
(Hi) Sulphonation. It is carried out by heating an alkane with fuming sulphuric acid (cone. H2SO4 + SO3)
at 675-725 K. Only branched chain and higher n-alkanes (containing six or more carbon atoms) undergo
sulphonation. For example, n-hexane gives n-hexanesulphonic acid and 2-methylpropane gives 2-
methylpropane-2-sulphonic acid.
16. Combustion. Alkanes readily bum in excess of air or oxygen to form CO2 and H2O with liberation of
large amount of heat and light. For example, LPG which is chiefly a mixture of butane and isobutane is
used for cooking in our kitchens while CNG is used as a fuel for cars and buses in metropolitan cities like
Delhi, Mumbai, Kalkota, Chennai, etc.
17. Isomerization, n-Alkanes when heated with anhyd. AICI3 in presence of HCl at 573 K undergo isomerization
to give branched chain alkanes.
HYDROCARBONS 13/95

J8. Aromatization. «-Alkanes containing 6-8 carbon atoms when heated to about 773 K under 10-20 atm
pressure in the presence of a catalyst consisting of Cr^Oj, V^05, Mo-,03 supported over AICI3 get converted
into aromatic hydrocarbons. For example, n-hexane gives benzene while n-heptane gives toluene.
19. Pyrolysis or cracking of higher alkanes at high temperature (773-973 K) under a pressure of 6-7 atmospheres
in presence or absence of a catalyst gives a mixture of lower alkanes, alkenes, etc.
773 K
C6H,4 6-7 ;ilm
> + CoHft + C3H6 + C2H4 -H CH4.

w
Pyrolysis involves breaking of C-C and C-H bonds and occurs by a free radical mechanism.
20. Conformations of Ethane. The infinite number of momentary arrangement of the atoms in space which
result through rotation about a single bond are called conformations or rotational Isomers. Ethane has
infinite number of conformations, of which, only two, i.e., staggered and eclipsed are important. The

Flo
dihedral angle between the two conformations is 60°.
The distance between non-bonded H-atoms in staggered conformation of ethane is 3-1 A while in eclipsed
conformation, it is 2-29 A.

ee
The staggered conformation of ethane is more stable than the eclipsed conformation by about 3 0 kcal or
12.55 kJ mol"^ Due to small difference in energy, the two conformations are readily interconvertible and

Fr
it is not possible to separate the two conformations of ethane. However, at any given moment, most of the
ethane molecules would exist in the staggered conformation due to its minimum energy and maximum
stability.

for
II. Aikenes
ur
21. The general formula of alkenes is C„H2^j where n = 2, 3,4....etc. All alkenes contain a double bond which
consists of one strong o-bond and one weak 7i-bond. A 7t-bond is formed by sideways overlap of two p-
s
orbitals. The carbon atoms of a double bond are i'p^-hybridized
and all the four atoms directly attached to
ok
Yo
the double bond lie in a plane. The H—C—C angle is 121-7° while H—C—H angle is 116-6. The C = C
bond distance is 134 pm and C—H bond distance is 110 pm.
o

22. Geometrical isomerism. Stereoisomers which have the same structural formula but differ in the spatial
eB

arrangement of atoms around double bond are called geometrical isomers. It arises due to restricted
rotation about carbon-carbon double bond.

(0 The isomer in which the two similar atoms/groups lie on the same side of the double bond is called the
r

cis-isomer while that isomer in which the two similar atoms/groups lie on the opposite sides of the double
ou
ad

bond is called the trans-isomer.

(//) The necessary and sufficient condition for a molecule to exhibit geometrical isomerism is that each of
the two carbon atoms of the double bond must have different atoms/groups which may be same or different.
Y

For example, alkenes of the types abC = Cab and abC = Cde show geometrical isomerism but alkenes of
the types abC = Caa or abC = Cbb or abC = Cdd do not show geometrical isomerism.
Re
nd

(Hi) The molecules of the rra/is-isomer are symmetrical and hence pack well in the crystal lattice. Therefore,
m.p. of a trans-isomer is higher than the corresponding cis-isome.r
Fi

(iv) The solubility of cis-isomer is higher than the trans isomer.


(v) The dipole moment of a cis-isomer is higher than that of the trans-isomer. If, however, the alkene is
symmetrical, the dipole moment of the trans-isomer is zero, i.e., rran.v-2-butene.
Further, due to higher dipole moment, the boiling point of cis-isomer is higher than that of the corresponding
trans-isome,r

(vi) Like compounds containing C = C, compounds containing C = N and N = N also show geometrical
isomerism.

23. Stability of alkenes. Heat of hydrogenation can be used to determine the relative stability of alkenes. The
more highly substituted the alkene, the more stable it is, i.e.,
R2C = CR2 > R2C = CHR > R2C = CH2 > RCH = CHR > RCH = CH2 > CH2 = CH2
This stability order can be explained on the basis of hyperconjugation.
13/96 ‘P'uidccp. ^ New Course Chemistry (XI) orrim

24. Dchydrohalogenation (removal of a molecule of a halogen add) of aikyl halides with hot alcoholic KOH
solution gives alkenes. For example, 1-bromoethane gives ethene while 1-bromopropane gives propene.
When two alkenes are possible, the more highly substituted alkene predominates (Saytzeff rule). For
example, 2-bromobutane on dchydrohalogenation gives 80% of the more stable, but-2-ene and 20% of the
less stable, but-l-ene.

The case of dchydrohalogenation of alkyl halides having the same alkyl group is : iodides > bromides >

w
chlorides > fluorides while the isomeric alkyl halides having the same halogen is : 3“ > 2° > 1°.
Dchydrohalogenation is an example of ^-elimination reaction since hydrogen is removed from the p-
carbon and halogen fivm the .0-carbon.
25. Dehalogenation (removal of a molecule of halogen) of 1, 1- and 1, 2-dibromoethane with Zn dust in

o
boiling methanol or ethanol gives ethene while that of 1, I and 1, 2-dibromopropane gives propene.
26. Dehydration of alcohols with cone. H2SO4 at 433-443 K gives alkenes. The reaction occurs through a

e
carbocation intermediate. For example, CH3CH2OH gives CH2 = CH2.

Fl
re
During dehydrationof alcohols, sometimesrearrangedproductsare formed. This is due to the rearrangement
of the initially formed less stable carbocationto the more stable carbocation either by a 1, 2-hydride or 1,

F
2-methyl shift. For example, dehydration of 1-butanol gives the same mixture of products, i.e., but-2-ene
(80%) and but-l-ene (20%) as obtained from dehydration of 2-butanol under similar conditions.
ur
27. Kolbe’s electrolysis of Na or K salts of saturated dicarboxylic acids gives alkenes. For example, sodium

r
succinate on electrolysis give ethylene.

fo
28. Chemical Properties of Alkenes. (i) Electrophilic addition reactions. The typical reactions of alkenes

ks
are electrophilic addition reactions. In these reactions, an electrophile first attacks the double bond forming
a carbocation intermediate which then undergoes nucleophilic attack to form a 1, 2-addition product
Yo
Some electrophilic addition reactions are :
oo
(ii) Addition of halogens. The order of reactivity is : F2 > CI2 > Br2 > l2- CI2 in CCI4 and Br2 in CCI4 add
readily to alkenes to form 1, 2-haloalkanes. During the addition of Br2, orange red colour of Br2 is
eB

discharged. Therefore, this reaction is used as a test for unsaturation.


(Hi) Addition of halogen acids (HX). The order of reactivity is: HI > HBr > HCl > HF.
Addition of HX to symmetrical alkenes gives only one product but to unsymmetrical alkenes can give two
ur

products. In absence of peroxides, the major product is governed by Markovnikov’s rule which states
that negative part of the unsymmetrical reagent goes to that carbon atom of the double bond which has
ad
Yo

lesser number of H-atoms.


Mark. addn.
CH3—CH = CH2 + HBr ^ CH3—CHBr—CH3
d

Propene 2-Bromopropane
Re

Anti-markovnikov’s addition or peroxide effect or Kharasch effect. In presence of peroxides such as


in

benzoyl peroxide, the addition of HBr (but not of HCl or HI) to unsymmetrical alkenes occurs contrary to
the Markovnikov’s rule. For example.
F

Peroxide
CH3CH = CH2 + HBr CH3CH2CH2—Br
Propene I-Bromopropane
The addition occurs by a free radical mechanism.
(iv) Addition of hypohalous acid (HOX). Addition of HOCl (CI2 in H2O) or HOBr (Br2 in H2O) gives
halohydrins. For example, addition of HOCl to ethylene gives ethylene chlorohydrin (CICH2-CH2OH).
(v) Addition of cold cone. H2SO4 to alkenes followed by hydrolysis with boiling H2O gives alcohols.
Cold.cone. HjO.boil
CH3CH = CHj > CH3—CH (OSO3H)—CH3 ■> CH3—CHOH—CH3
H2SO4
Propene Isopropyl hydrogen Propan-2-ol
sulphate
HYDROCARBONS 13/97

29. Oxidation reactions : (i) Combustion : Alkenes readily bum in air or oxygen to form CO2 and H2O with
liberation of large amount of heat.
(ii) Alkenes react with O2 in presence of silver catalyst at 575 K to give epoxyalkanes. For example, ethene
gives epoxyelhane or ethylene oxide or oxirane.
(h7) With cold dilute neutral or alkaline KMnO^, alkenes give 1. 2-glycols. During this reaction, pink
colour of the KMnO^ solution is discharged and a brown ppt. ofMn02 is formed. Therefore, this reaction
is used as a test for unsaturationunder the name Baeyer’s test and the cold dilute neutral or alkaline
KMn04 solution is called Baeyer’s reagent,
(iv) With hot KMnO^ solution, cleavage of C = C bond occurs leading to the formation of carboxylic acids,

low
ketones and carbon dioxide depending upon the nature of the alkene. For example.

KMn04. KOH
CH3CH = CH2 4
CH3COOH + CO2 + H2O
373-383 K
Propene Ethanoic acid

KMn04.K0H
(CH3)2C = CH2 4
(CH3)2C = 0 + CO2 + H2O

ee
373-383 K
2-Mthylpropene Propanone

rF
Fr
KMn04.K0H
CH3CH2CH = CHCH3 373-383 K
4 CH3CH2COOH + HOOCCH3
Pent-2-ene Propanoic acid Ethanoic acid

(v) When O3 is passed through a solution of an alkene in some inert solvent such as CH2Clo, CHCI3 or

for
CCI4 at low temperature (196-200 K), ozonides are fonned. These on reduction with Zn/H,0 or H,/Pd
give aldehydes or ketones or a mixture of these depending upon the structure of the alkene
ou
(i)O3/CH2Cl2.196-200K
CH3CH = CH2
(/i)Zn/H20
ks
4 CH3CH = 0 + 0 = CH2
Propene Elhunal Methanal

This two step process is called ozonolysis and is used to locate the position of a double bond in an
oo
unknown alkene.
Y
30. Polymerization reactions. Alkenes readily undergo polymerization in presence of suitable catalysts. For
B

example, ethylene gives polythene, vinyl chloride gives PVC, tetrajluoroethylene gives teflon. Because
of its great chemical inertness and thermal stability, teflon is used for making non-stick utensils.
re

III. Alkynes
31. The general formula of alkynes is C„H2„.2 where n = 2, 3, 4....etc. They contain one triple bond which
ou

consists of one strong (T-bond and two weak 7C-bonds. The carbon atoms of a triple bond are jp-hybridized.
ad

Acetylene is a linear molecule with cylindrical electron cloud. The C = C bond length is 120 pm while
C-H bond length is 107.9 pm.
Y

Being linear alkynes do not show geometrical isomerism.


32. Dehydrohalogenation of gem- and vjc-dihalides first with alcoholic KOH followed by treatment with
nd

NaNH2 in liq. NH3 or preferably by the action of NaNH2 in liquid NH3.


Re

KOH (ale) NaNH2/liq.NH3


BiCH2CH2Br CH2 = CHBr
Fi

> HC = CH
A I96K
I, 2-Dibromoethane l-Bromoethene Elhyne
33. (z) Dehalogenation of tetrahalides with zinc du.st in methanol yields alkynes.
(ii) Dehalogenation of chloroform and iodoform by heating with Ag powder gives acetylene.
34. Kolbe’s electrolysis of Na or K salts of maleic or fumaric acid gives acetylene.
35. Reaction of mono- and disodium acetylides with alkyl halides gives both symmetrical and unsymmetrical
alkynes depending upon the nature of the alkyl halide. For example,
HC = C-Ua* -I- CH3Br 4 HC = C—CH3 -t- NaBr
Sod. acetylide Piopyne
Na+-CsC-Na+ 2 CH3Br 4 CH3- sC—CH3 + 2 NaBr
Disodium acetylide But-2-yne
13/98 New Course Chemistry (XI)EEIHI]

36. In the laboratory, acetylene is prepared by the action of H2O on CaC^.


CaC2 + 2 H^O -> HC^CH + Ca(OH)2
Acetylene
37. Acidic nature. Due to j'/^-hybridization of acetylenic carbon, H-atoms of terminal alkynes are acidic in
nature. They react with Na at 475 K or Na in liq. NH3 at 196 K to form sodium alkynides.
(0 With ammoniacal AgN03 solution or Tollens’ reagent, terminal alkynes give white ppt. of silver
alkynides.

low
(//) With ammoniacal CuCl solution, terminal alkynes give red ppt. of copper alkynides.
(m) With Grignard reagents, terminal alkynes give the corresponding alkynyl Grignard reagents,
(tv) The acidic character of ethane, ethene and ethyne follows the order:
CH s CH > CH2 = CH2 > CH3 — CH3
(v) The acidic character of ctuboxylic acids, phenols, alcohols, water, ammonia, acetylene and alkanes
follows the order : RCOOH > C^HgOH > H2O > ROH > HC = CH > NH3 > RH

e
(v/) The basic character, however follows the reverse order :

re
rF
R- > NH- > HC 5 C- > RO- > HO- > CgHjO" > RCOO"

F
38. Electrophilic addition reactions. Like alkenes, alkynes also undergo electrophilic addition reactions.
However, due to (/) greater electronegativity of i/j-hybridized carbon as compared to -hybridized carbon
of ethene, and (i7) cylindrical nature of the 7t-electron cloud, alkynes are less reactive than alkenes towards

or
electrophilic addition reactions. Some of these reactions are discussed below :
(/) Addition of halogens. CI2 and Br2 readily add to alkynes first forming 1, 2-dihaloalkenes and then 1,

f
ou
1, 2, 2-tetrahaloalkanes. The order of reactivity decreases in the order: F2 > C/2 > 5^2 > l2-
ks
(h) Addition of halogen acids. The order of reactivity decreases in the order : HI > HBr > HCl > H.F
In absence of peroxides, addition occurs according to Markovnikov’s rule but in presence of peroxides,
oo
addition of HBr follows anti-Markovnikov’s rule. For example,
Y
HBr HBr
eB

CH3C = CH » CH3C(Br) = CH2 ■> (CH3)2CBr2


{Mark, addn.)
Propyne 2, 2-Dibromopropane
HBr HBr
CH3C = CH »
ur

RCOOR
CH3CH = CHBr RCOOR
■>
CH3CH2CHBr2
Propyne 1-Bromopropene 1, 1-Dibromopropane
ad
Yo

{Hi) Addition of hypohalous acids to alkynes gives dihalocarbonyl compounds. With HOCl, acetylene
gives 1, 2-dichloroethanaI while propyne gives 1, 1-dichloroprop anone.
(iv) Addition of water or hydration of alkynes occurs when treated with dil. H2SO4 in presence of
HgS04 as catalyst to form carbonyl compounds. For example,
d
Re

Dil. H2SO4 Taulomeriscs


in

HC = CH + H—OH ^ IH2C = CHOH] ^ CH3CH = 0


HgS04
Ethyne Vinyl alcohol Ethanal
F

Dil, H2SO4 Tauiomerises


CH3C=CH H—OH ■> [CH3CH(0H) = CH2] CH3COCH3
HgS04
Propyne Propanone
(v) Addition of HCN to acetylene in presence of Ba(CN)2 or CuCl/HCl gives vinyl cyanide or acrylonitrile.
39. Nucleophilic addition reaction. Due to greater electronegativity of 5p-hybridized carbons as compared
to 5p^-hybridized carbons of alkenes, alkynes undergo nucleophilic addition reaction. For example, addition
of CH3OH to acetylene of 433-443 K in presence of CH30Na, gives methyl vinyl ether.
40, Addition of hydrogen. Addition of H-, to alkynes in presence of Ni at 523-573 K first gives alkenes and
then alkanes. The reduction of alkynes can be stopped at the alkene stage. With Lindlar’s catalyst, i.e., Pd
supported over CaC03 or BaS04 and partially poisoned by addition of S or quinoline, alkynes give cis-
alkenes but with Na in liquid NH3 {i.e., Birch reduction), rra/i^-alkenes are formed.
HYDROCARBONS 13/99

41. Oxidation reactions (/) Oxidation of terminal alkynes with cold KMn04 solution gives a mixture of
RCOOH + CO2 while oxidation of non-terminal alkynes gives diketohes. Under these conditions, acetylene,
however, gives oxalic acid.
During this reaction, the pink colour of the KMnO^ solution is discharged and a brown ppt. of Mn02
formed. Therefore, this reaction is used as a test for unsaturation under the name Baeyer’s test.
(/7) Oxidation of non-terminal alkynes with hot KMn04 solution gives only a mixture of dicarboxylic
acids through cleavage of C = C bond.
(///) Reaction with ozone. Alkynes react with O3 to form ozonides which upon reductive ozonolys’s gives
1, 2-dicarbonyl compounds.
42. Polymerization reactions. (0 Acetylene when passed through red hot Fe tube gives benzene while propylene
gives mesitylene.
(ii) Acetylene polymerizes in presence of Ni(CN)2 under high pressure to give cyclooctatetraene.
(Hi) In presence of CUCI/NH4CI, acetylene first gives vinylacetylene and then divinylacetylene.

F low
43. Isomerization reactions. When heated with ale. KOH at 473 K, 1-alkynes, readily isomerize to the more
stable 2-alkynes. On the other hand, 2° alkynes can also isomerize to 1° alkynes by action of NaNH2 in an
inert solvent such as kerosene or paraffin oil at 423 K followed by treatment with H2O.
IV. Arenes

44. Hydrocarbons and their alkyl, alkenyl and alkynyl derivatives which contain one or more benzene rings
either fused or isolated in their molecules are called aromatic hydrocarbons or arenes. Aromatic
hydrocarbons which contain a benzene ring are also called benzenoids while those which do not contain
a benzene ring are called non-benzenoids, e.g., azulene, tropolone, cyclopentadienyl anion,

e
for Fr
cycloheptatricnyl cation, etc.
45. Structure of Benzene, (i) All the C-atoms of benzene are 5/>--hybridized, i.e., all the ZCCC and ZHCH =
120°. It is a planar molecule, i.e., all the six C—C and six C—H bonds lie in a plane.
(I'O Benzene is a resonance hybrid of two Kekule structures and has a resonance energy of 36-0 kcal
mol"’ or 151 kJ mol"’.
Due to resonance, all the carbon-carbon bond lengths are equal (139 pm) and lie in between those of
carbon-carbon single bond lengths of 154 pm and carbon-carbon double bond lengths of 134 pm.
Your
s
eBo k

46. Hiickel rule. Benzene is an ideal aromatic compound. The aromatic character of other cyclic conjugated
polyenes can be determined on the basis of Hiickel rule. The main points of Hiickel rule are :
(0 It should have a single cyclic cloud of delocalized 7t-electrons above and below the plane of the molecule.
(«) It should be planar.
ad

(Hi) It should contain (4n + 2) 7C-electrons where n = 0, 1, 2, 3.... etc.


our

A molecule which does not satisfy any one or more of the above conditions is said to be non-aromatic.
Further, cyclic conjugated polyenes which contain 4n 7U-electrons are destablized by resonance and hence
are called antiaromatic compounds.
47. Methods of Preparation of Arenes. Benzene is obtained by coal tar distillation. Benzene and its
Re

homologues may be prepared by the following methods.


Benzene and its hoinologoues are obtained by (/) cyclic polymerization of alkynes, (//) decarboxylation of
Find Y

aromatic acids with soda-lime, (Hi) reduction of phenols with Zn dust, (/v) reduction of chlorobenzene
with Ni-Al alloy/NaOH, (v) reduction of benzenediazonium chloride with H3P02/ru''', (vi) heating
benzenesulphonic acid with super heated steam, (vH) aromatization of /i-hexane and n-heptane,
(vHi) Wurtz-Fittig reaction, involving heating of an aryl halide with an alkyl halide in presence of metallic
Na in dry ether, (ix) F.C. reaction of benzene with alkyl halides in presence of anhyd. AICI3 and
(x) reaction of arylmagnesiumhalides (Grignard reagents) with alkyl halides in presence of dry ether.
48. Electrophilic substitution reactions. The typical reactions of arenes are electrophilic substitutionreactions
in which one or more H-atoms of benzene ring are replaced by the electrophile. The reaction occurs in two
steps. In the first step, the electrophile attacks the benzene ring to form a resonance-stabilized carbocation.
This step is slow and is the rate-determining step of the reaction. In the second step, the carbocation loses
a proton to form the substitution product. This step is fast and hence does not affect the rate of the reaction.
Some important electrophilic substitution reactions are :
13/100 ^^adccft- U New Course Chemistry (XI) LWll

(/) Chlorination or bromination occurs with CI2 or Br2 in presence of a Lewis acid (also called halogen
carrier) such an anhyd. AICI3, FeBr3, etc. The effective electrophile here is either Cl"^ (chloronium ion) or
Br'*' (bromonium ion).
(h) lodination is carried out with I2 in presence of an oxidising agent such HNO3, HIO3 or HgO.
{Hi) Nitration is carried out with a mixture of cone. HNO3 + cone. H2SO4 (nitrating mixture). The

effective electrophile here is NO^ {nitronium ion),


(iv) Suiphonation is carried out either with cone. H2SO4 or CISO3H (chlorosulphonic acid). The effective
electrophile here is SO3. Unlike, halogenation and nitration, suiphonation is a reversible reaction. That is
why benzenesulphonicacid on heating with super heated steam undergoesdesulphonationto give benzene,
(v) F.C. alkylation is carried out with an alkyl halide in presence of anhyd. AICI3, FeBr3, etc. The effective
electrophile here is R^.
(v/) F.C. acylation is carried out with an acid chloride or anhydride in presence of anhyd. AICI3. The

w
effective electrophile here in the acylium ion (CH3— c=^o: <■

F lo
^ CH3—CsO :)
49. Addition reactions. Benzene normally does not undergo addition reactions but under drastic conditions,
i.e., high temperature and pressure, they do undergo some addition reactions. With H2 in presence of Ni at
473 - 523 K, benzene gives cyclohexane but with CI2 in presence of sunlight and in absence of halogen
carrier, it gives benzene hexachloride (BHC).
50. Oxidation reactions. (/) Oxidation of benzene with O2 in presence of V2O5 at 773 K gives maleic anhydride,

ree
(ii) Reductive ozonolysis of benzene gives glyoxal ; that of o-xylene gives glyoxal, methylglyoxal and

for F
dimethylglyoxal while that of mesitylene gives only methylglyoxal.
{Hi) Benzene itself is not oxidised by hot alk. KMn04 but its higher homologues are easily oxidised.
During this oxidation, the nucleus remains intact but each of its side chain irrespective of the length is
oxidised to a -COOH group. For example, oxidation of toluene or ethylbenzene gives benzoic acid, that of
o-xylene gives phthalic acid and that of p-xylene gives terephthalic acid.
51. Directive influenceof groups. When a monosubstitutedbenzene derivative is converted into a disubstituted
Your
s

derivative, the substituent already present in the benzene ring determines the position of the incoming
k
eBoo

group. This is called directive influence of groups. This is of the following two types :
(0 o, p-Directing groups. The substituents or groups which direct the incoming group to o-and p-positions
are called o, p-directing groups. For example, CH3-, CH3CH2-, Cl- Br-, I-, -OH, -OCH3,
-OCOCH3. -NH2. -NHCH3, -N(CH3)2, -NHCOCH3, etc. ■
ad

{H) m-Directing groups. The substituents or groups which direct the incoming group to the m-position are
our

called m-directing groups. For example, (CH3)3N-, - NO2, - CN, - CF3, - CHO, - COR, - COOH,
- COOR, - SO3H, etc.
52, Effect of substituentson Reactivity,o, p-Directing groups increase the electron density in the benzene
Re

ring and hence activate the nucleus towards further electrophilic substitution reactions. In contrast, m-
directing groups decrease the electron density in the benzene irng and hence deactivate the nucleus towards
Y

further electrophilic substitution reactions.


Find

Please note that although halogens are little deactivating yet they are o, p-directing.
53. Carcinogenicity and toxicity. Benzene and most of the polynuclear aromatic hydrocarbons (PNH) are
cancer producing. In fact, PNH are the products of incomplete combustion of organic matter such as coal,
petroleum, tobacco, etc. They are widely present in the environment and thus are the major source of
human cancer.

These PNH enter the human body. They are first converted into epoxides and then to dihydroxy epoxides.
The dihydroxy epoxides thus produced react with the purine bases such as guanine present in DNA and
RNA of human cells. This causes mutations and ultimately leads to cancer.
Examples of PNH having carcinogenic activity are : 1, 2-benzanthracene, 3-methylcholanthrene, 1, 2-
benzpyrene, 9, 10-dimethyI-l, 2-benzanthracene and 9, 10-dimcthyl-i, 2-dibenzanthracene.
HYDROCARBONS 13/101

V TT \ir QUESTIONS
Based on IMCERT Book

1. Multiple Choice Questions Br Ci

I. Alkanes (a) (6)


1. Pick out the alkane which differs from the other

w
members of the group ic) id)
(a) 2, 2-dimethylpropane
(b) pentane 10. An alkane CyHjg is produced by the reaction of
lithium di(3-pentyl)cuprate with ethyl bromide. The

Flo
(c) 2-methy)butane {d) 2, 2-dimethylbutane
structural Ibnnula of the product is
2. Isopropyl bromide on Wurtz reaction gives
(fl) 3-ethylpentane {b) 2-methylpentane
(a) Hexane (b) Propane

ee
(c) 3-methylhexane {d) 2-methylhexane
(c) 2, 3-Dimethylbutane
11. Which of the following reactions is not correct for

Fr
{d) Neohexane
preparation of alkanes ?
3. An alkyl bromide (X) reacts with sodium in ether LiAIH
to form 4, 5-dielhyloctane, the compound ‘X’ is (a) (CH3)2CHC1 ^ {CH3)2CH2

for
(a) CH3(CH2)3Br (b) CH3(CH2)5Br
ur LiAIH

(c) CH3(CH2)3CH(Br)CH3 ib) (CH3)3CC1 4 (CH3)3CH


NaBH
id) CH3—{CH2)2—CH(Br)—CH2CH3
ic) (CH3)2CHC1 4 (CH3)2CH2
4. The products of the following reactions is(are)
k s NaBH
Yo
dry ether id) (CH3)3CC1 ^ (CH3)3CH
oo
C2H5Br + 2 Na + CHgBr > ?
12. Wurtz reaction of methyl iodide yields an organic
(a) ethane ib) propane compound X. Which of the following reaction also
eB

(c) butane yields X ?


(d) ethane, propane and butane (a) C2H5CI + UAIH4
5. An alkyl halide by formation of its Grignard dry ether
r

reagent and heating with water gives propane. (b) C2H5CI + Mg


ou
ad

What is the original alkyl halide ? (c) C2HgCl + C2H50Na ■>

(a) methyl iodide ib) ethyl iodide


(d) CHCI3 + Ag (powder) ~
Y

(c) ethyl bromide id) propyl bromide


13. Electrolysis of a concentrated aqueous solution of
6. (CH3)3CMgCl on reaction with D2O produces a compound gave C2Hg on anode. The compound
Re
nd

(a) {CH3)3CD ib) (CH3)30D IS

ic) (CD3)3CD id) (CDglgOD (a) CH3COOK ib) CH3CH2COOK


Fi

7. Heating a mixture of sodium benzoate and soda- ic) CH3COOC2H5 id) CH3OCH3
lime gives 14. The correct order of boiling points of 2, 2-dimethyl-
(a) Benzene ib) Methane propane, 2-methylbutane and /i-pentane is
(c) Sodium benzoate id) Talcium benzoate (a) n-pentane > 2, 2-dimethylpropane > 2-methyl-
8. Cycloalkane formed when . 4-dibromopentane is butane
heated with sodium is ib) a-pentane > 2-methylbutane > 2, 2-dimethyl
(a) methylcyclobutane{/?) cyclopentane propane

(c) cyclobutane id) methylcyclopentane (c) 2, 2-dimethylpropane > 2-methylbutane >


9. What will be the product formed when 1-bromo- n-pentane
3-chlorocyclobutanc reacts with two equivalents id) 2-methylbutane > n-pentane > 2, 2-dimethyl
of metallic sodium in ether ? propane
13/102 “P^adee^ 4 New Course Chemistry (XI)

15. Which of the following alkane has lowest boiling (a) 1-hexene (b) 2-hexene
point and highest melting point ? (c) benzene (d) 2-methyIpentane
(a) «-Pentane (b) Isopentane (e) 2, 2-dimethylbutane
(c) Neopentane (d) rt-Hexane 22. n-Octane when heated to 773 K under a pressure
16. On mixing certain alkane with chlorine and of 10-20 atm and in presence of a mixture of Cr203,
irradiating it with ultraviolet light, one forms only V2O5 and M02O3 supported over AI2O3 as catalyst,
one monochloroalkane. The alkane could be gives
(fl) neopentane (b) propane (a) o-xylene (b) m-xylene

w
(c) pentane (d) isopentane (c) p-xylene (d) all the three
17. Methane can be converted into ethane by the 23. Liquid hydrocarbons can be converted to a mixture
reactions of gaseous hydrocarbons by
(a) chlorination followed by the reaction with (fl) hydrolysis (b) oxidation

F lo
alcoholic KOH
(c) cracking
(b) chlorination followed by the reaction with (d) distillation under reduced pressure
aqueous KOH

ee
24. Isomers which can be interconverted through
(c) chlorination followed by Wurtz reaction rotation around a single bond are :

Fr
(d) chlorination followed by decarboxylation (a) conformers (b) diastereomers
18. Of the five isomeric hexanes, the isomer which can
(c) enantiomers (d) positional isomers
give two monochlorinated compounds is
25. Which one of the following is applicable to the

for
(a) n-hexane (b) 2, 3-dimethyIbutane
ur
(c) 2, 2-dimethylbutane
conformations of a hydrocarbon ?
(a) C—C distance changes
(d) 2-methyipentane (b) C—H distance changes
19. 2-Methylbutane on reacting with bromine in the
ks
(c) and C—C—H bond angles change,
presence of sunlight gives mainly
Yo
(d) Only distance between non-bonded H-atoms
oo
(a) 1 -bromo-2-methylbutane
changes.
(b) 2-bromo-2'methylbutane
26. At room temperature, the eclipsed and staggered
eB

(c) 2-bromo-3-methylbutane forms of ethane cannot be isolated because


(d) l-bromo-3-methylbutane (a) they interconvert rapidly
20. Consider the following reaction, (b) both the conformers are equally stable
r

H,C—CH—CH—CH. + Br > ‘X’ + HBr (c) the energy difference between the conformers
ou

’l l ’
ad

is large
D CH3 (d) there is a large energy barrier of rotation about
Y

Identify the structure of the major product ‘X’ the G-bond

27. Increasing order of stability among the three main


(a) CH3—CH—CH—CH2 conformations (i.e., eclipse, anti, gauche) of 2-
nd
Re

fluoroethanol is
D CH3
(fl) eclipse, anti, gauche
Fi

(b) CH3—CH—C—CH3 {b) anti, gauche, eclipse


D CH3 (c) eclipse, gauche, anti
(d) gauche, eclipse, anti
(c) CH.—C—CH—CH,
’ll ’ II. Alkenes
D CH3
28. Which of the following exhibits geometrical
id) CH3—CH—CH—CH3 isomerism ?

CH3 (a) 1, 2-Dibromopropene


{b) 2, 3-Dimethylbut-2-ene
21. n-Hexane on healing to 773 K at 10-20 atmo
spheric pressure in the presence of oxides of (c) 2, 3-Dibromobut-2-ene
vanadium supported over alumina, yields (d) 2-MethyIbut-2-ene
HYDROCARBONS 13/103

29. The lUPAC name of the following compoimd is


/CH(CH3)2
id) c = c
Cl CH2CH3 h/

ow
XI
CH3 36. 3-Phenylpropene on reaction with HBr gives (as a
(a) rra/i5-2-chloro-3-iodo-2-pentene major product)
(b) cw-3-iodo-4-chloro-3-pentene (a) C6H5CM2CH(Br)CH3
(c) /ra/ij-3-iodo-4-chloro-3-pentene (b) C6H5CH(Br)CH2CH3
(d) m-2-chloro-3-iodo-2-pentene (c) C6H5CH2CH2CH2Br
30. Geometrical isomerism is not possible in (d) CgHjCHCBrjCH = CH2

re
(a) 2,4-hexadiene (b) benzaldoxime 37. 3-Phenylpropene reacts with HBr in the presence
(c) but-2-ene (d) 1, 2-dichloroethene of peroxide, the major product formed is

Flr
(a) 2-bromo-1-phenylpropane

F
(e) benzophenone oxime
31. The number of isomers for the compound with (b) 1, 2-dibromo-3-phenylpropane
molecular formula C2BrClFI is (c) 3-(o-bromophenyl)propene
(a) 3 (i>)4

ou (d) l-bromo-3-phenylpropane

sr
(c)5 (d)6 38. The reaction of C6H5CH = CHCH3 with HBr
32. The major product formed when 2-bromo produces

fo
2-methylbutane is refluxed with ethanolic KOH is (<j) CgH3CH2CH2CH2Br

k
-<
(a) 2-methylbut-2-ene (b) 2-methylbutan-l-ol
(b) Br CH = CHCH3
(c) 3-methylbutan-2-ol oo
(d) 2-methylbutan-2-ol
(c) CgH5CHCH2CH3
Y
33. Which of the following organohalogen compounds I
when heated with alcoholic potassium hydroxide Br
reB

does not undergo dehydrohalogenation reaction ?


(a) Secondary butyl chloride (d) c^h^ch2<:hch^
uY

Br
(b) Isopropyl chloride
(c) Neopentyl chloride
39. H3C—CH—CH = CH2 +HBr 4 A
(d) Isobutyl chloride I
(e) Tertiary butyl chloride CH3
ad
do

34. In the reaction below, X is A (predominantly) is


H2SO4 (a) CH,—CH—CH—CH,
Neopentyl alcohol > X
in

I I
(a) 2-methylpentane (b) 2-metyylpent-2-ene Br CH3
Re

(c) 2-methylbut-2-ene (d) neopentane (b) CH-—CH—CH—CH-


M l ’
F

35. The main product of the following reaction is


Conc.H2S04
CH3 Br
CgH5CH2CH(OH)CH(CH3)2 (c) CH3—CH—CH2—CH2Br
I
H5CgCH2CH2>,^ CH3
(a) ^C = CH2
H3C Br
I
H.C H
5^6\ (d) CH3—C—CH2CH3
(t) c=c
HZ' ^H(CH3)2 CH3
40. In which of the following can peroxide effect
(c)
C6HsCH2\ C = C /CH3 operate ?
H
^Hs (a) CH3CH2CH = CH2 + HQ
13/104 7>neieCeefi.'^ New Course Chemistry fXI^PZsnwn

(b) CH3CH2CH = CH2 + HBr 46. Propene on reaction with chlorine water gives
(c) CH3CH = CHCH3 + HBr
(d) CH3CH2CH = CH2+HI
(a) (b)
41. The alkene that will give the same product with

ow
HBr in the absence as well as in the presenceof
peroxidesis
(c) OH (d) Cl
(a) 2-butene (b) 1-butene
47. On ozonolysis, one mole of a hydrocarbon
(c) propene (d) 1-hexene
produces two molecules of ethanal and one
(e) 2-methylpropene molecule of ethanedial. The hydrocarbon could be
42. Observe the following reactions and predict the

e
(a) 1, 3-hexadiene (b) 1, 4-cyclohexadiene
nature of A and B.
(c) 1,4-hexadiene (d) 2, 4-hexadiene

re
HBr HBr 48. An alkene on reductive ozonolysis gives two

Frl
>A
hv molecules of CH2(CH0)2. The alkene is

F
(a) 2, 4-hexadiene
Br
(b) 1, 3-cyclohexadiene
(c) 1,4-cyclohexadiene
ou
or
(a) A and B both are (d) 1-methyl-1, 3-cyclopentadiene
49. Which of the compounds with molecular formula

kfs
C5H10 yields acetone on ozonolysis ?
(b) A and B both are (a) 2-Methyl-1-butene
Br
(b) 2-Methyl-2-butene
oo
Br (c) 3-Methyl-1-butene
(d) Cyclopentane
Y
50. One mole of alkene on ozonolysis gives 2 moles
eB

(c) A is and B is
Br of butanone. The alkene is

(a) 3, 4-dimethylhex-2-ene
(b) 2, 3-dimethylhex-3-ene
ur

(d)Ais and B is
Br (c) 3, 4-dimethylhex-3-ene
oY

Br
Br
(^0 2, 3-dimethylhex-2-ene
43. In presence of peroxide, hydrogen chloride and
ad

hydrogen iodide do not give anti-Markovnikov’s 51. 5-Oxohexanal is obtained by ozonolysis of


addition to alkenes because
d

(a) both are highly ionic


(a) CH2CH3 (/>) CH3

(b) one is oxidising and the other is reducing


in
Re

(c) one of the steps is endothermic in both the cases (c) (d)
CH3 CH3
(d) all the steps are exothermic in both reactions
F

44. 2-Phenylpropene on acidic hydration gives 52. Oxidation of an alkene X gives a diol ; further
(a) 2-phenyl-2-propanol oxidation gives a diketone. Which one of the
following could be X ?
(b) 2-phenyl-1-propanol
ia) (CH3)2C = C (CH3)2
(c) 3-phenyl-1-propanol
ib) CH3CH = C (CH3>2
(d) l-phenyl-2-propanol
(c) (CH3>2CHCH = CH2
45. Among the alkenes which one produces tertiary
butyl alcohol on acid hydration ? (d) CgHjCH = CHCgHs
53. Select the reagent for the following reaction.
(a) CH3CH2CH = CH2
{b) CH3CH = CHCH3 OHC—(CH2)4—CHO
(c) (CH3>2C = CH2 (d) CH3CH = CHj
HYDROCARBONS 13/105

(a) Se02 ib) OyZxdUjO 63. Which of these will not react with acetylene ?
(c) O3, H2O2-CH3COOH (a) NaOH (6) ammoniacal AgN03
{d) PCC (c) Na {d) HCl
54. One mole of a symmetrical alkene on ozonolysis 64. Base strength of
gives two moles of an aldehyde having a molecular
0 0
mass of 44 u. The alkene is
H,CCH^,H^C = CH and H—C(iii)3 C®

ow
{a) ethene (b) propene - -
(i) (ii)
(c) 1-butene (d) 2-butene
is in the order of
55. Ozonolysis of an organic compound gives
(n) (i) > (iii) > (ii) (b) (i)>(ii)>m
formaldehyde as one of the products. This confirms
the presence of (c) (ii) > (i) > (iii) {d) (ill) > (ii) > (i)
(a) a vinyl group (b) an isopropyl group 65. The hydrocarbon which can react with sodium in

e
(c) an acetylene triple bond liquid ammonia is

re
(d) two ethlenic double bonds (a) CH3CH2C 3 CCH2CH3

rFl
56. Mustard gas is (b) CH3CH2CH2C 3 CCH2CH2CH3

F
(a) COCI2 (b) CCI3NO3 (c) CH3CH2C3CH
(c) CHCI2NO2 (d) CH3CH = CHCH3
(d) CICH2CH2SCH2CH2CI 66. The treatment of CH3MgX with

r
CH3C 3 C—H produces
III. Alkyne.s
ou
fo
(^2) CH4 (b) CH3—CH = CH2
57. Which of the following is not an isomer of
3-methylbut-l-yne?
(a) Pent-l-yne (b) Buta-1, 3-diene ks
(c) CH3C3C—CH3 (li) CH3—C = C—CH3
H
I
H
I
oo
(c) Pent-2-yne (d) Penta-1, 3-diene
(e) 2-Methylbuta-l, 3-diene 67. Which of the following reactions will yield, 2, 2-
Y
dibromopropane ?
58. Number of possible alkynes with formula C5Hg is
eB

(a) 3 (b) 5 (a) CH3—CH = CH2 + HBr


(c) 2 (d) 4 (h) CH3- 3CH-h2HBr ■>

59. The number of types of bonds between two carbon (c) CH3CH = CHBr + HBr
r

atoms in calcium carbide is (d) CH = CH + 2 HBr


ou
Y
ad

(a) one sigma, two pi (b) one sigma, one pi 68. Predict the product C obtained in the following
(c) two sigma, one pi (d) two sigma, two pi reaction of butyne-1
HI
60. Which of the following hydrocarbons has the CH3CH2- 3 CH + HCl ^B ^ C
lowest dipole moment ?
d

I
CH
CH3 I
in
Re

(a) C = C
H H
(a) CH3CH2—C—CH3
Cl
F

(b) CH3C 3 CCH3


(c) CH3CH2 C 3 CH (d) CH2 = CH—C 3 CH (b) CH3—CH—CH2CH2I
61. The number of structural and configurational iso
Cl
mers of a bromo compound, C5HgBr formed by
the addition of HBr to 2-pentyne respectively are : I
(a) 1 and 2 (b) 2 and 4
(c)4 and 2 (d) 2 and 1. (c) CH3—CH2—CH2—C—H
62. Among the following compounds, the strongest Cl
acid is

(a) HC 3 CH I
(b)
(c) C2H6 (d) CH3OH. (d) CH3—CH2—CH—CH2CI
13/106 New Course Chemistry (XI)SSEDKD

69. What is the product formed when acetylene reacts (c) NaNH2 and H^O, HgS04, H2SO4
with excess of hypochlorousacid ? id) NaNH2 and KMn04/H-*-
(n) CH3COCI ib) CICH2CHO 76. 2-Hexyne gives /ra/15-2-hexene on treatment with
(c) CloCHCHO (d) CICH2COOH (a) Li/NH3 {b) Pd/BaS04
70. What would be the expected product of the reaction (c) UAIH4 _id) Pt/H2
of propyne with if the mechanism of this 77. Propyne and propene can be distinguishedby
reaction is analogous to that of propene ?
(a) cone. H2SO4 {b) Bf2 in CCI4
{a) 2-Biomopropenol {b) Bromoacetone
(c) alk. KMn04 id) AgNOj in NH3
(c) 2-Bromo-2-propanol
78. Ethylene can be separated from acetylene by

w
{d) Bromoprophenol
passing the mixture through
71. The product(s) obtained via oxymercuration
(a) fuming H2SO4 (b) pyrogallol
(HgS04 + HoS04) of 1-butyne would give

F lo
(c) ammoniacal CU2CI2
O
{d) charcoal powder
(a) CH3CH2—C—CH3 IV. Arenes

ee
ib) CH3CH2CH2—CHO
79. The molecular formula of diphenylmethane,
(c) CH3CH2CHO + HCHO

Fr
C,3Hi2 is
(d) CH3CH2COOH + HCOOH
72. When 2-butyne is treated with dil. H2S04/HgS04, CH2

for
the product formed is
(a) butanol-1 {b) butanol-2 How many structural isomers are possible when
ur
(c) 2-butanone (d) butanal. one of the hydrogens is replaced by a chlorine
atom ?
73. Identify the product in the reaction
s
{b)S
ook
2+ (a) 4
HjO-^.Hg
Yo
PhC s CMe ^ ?
(c)7 (£018
(a) PhCH2CH2CHO (b) PhCOCH2CH3 80. The hydrocarbon which does not decolourise
eB

(c) PhCH2COCH3 (d) PhCOCOMe alkaline KMn04 solution and also does not give
74. In the following reaction : any precipitatewith ammoniacalsilver nitrate is
H2O (a) benzene (b) acetylene
our

C2H2 ^ X ± CH3CHO
ad

HgS04/H2S04 (c) propyne (d) butyne-1


What is X ? 81. Which of the following compounds is not
aromatic ?
(a) CH3CH2OH (b) CH3—O—CH3
Y

(c) CH3CH2CHO (d) CH2 = CHOH


Re

75. Identify the reagents in the following (fl) (*)


nd

transformations :

Br +
Fi

C^CH
(c)
A B N
>
Br
82. Among the following the aromatic compound is
O

(«) (^)

(a) Ale. KOH and H2O, HgS04, H2SO4 (c)


(b) Ale. KOH and KMn04/H+
HYDROCARBONS 13/107

83. Which is a non-aromatic compound ? (a) NO+ (b) NO|


CH3 (c) NO (d) NOJ
(a) I I (b) 92. Nitrobenzene can be prepared from benzene by
using a mixture of cone. HNO3 and cone. H2SO4.
H In the nitrating mixture, nitric acid acts as a
(a) base (b) acid
(c) id) (c) reducing agent id) catalyst
c=/ 93. Benzene reacts with I2 in presence of which of the

w
84. The non-aromatic compound among the following following to give iodobenzene ?
IS (a) HNO3 ib) HI
(c) SO2 id) H2O
ia)

Flo
94. Which one of the following undergoesnitration
most readily ?
(a) Acetophenone ib) Benzonitrile

ee
(c) (c) Benzaldehyde id) Benzoic acid
(e) Benzene

Fr
85. In the reaction, 95. Among the following compounds, the one that is
Oxidation NaOH Soda-lime most reactive towards electrophilic nitration is
C6H5CH3 » A ■» B > C,
(a) benzoic acid (b) nitrobenzene

for
the product C is
id) CgHsOH (b) CgHg
ur (c) toluene (d) benzene
96. Chlorobenzene on treatment with sodium in dry
(c) CgHjCOONa id) CgHjONa. ether gives diphenyl. The name of the reaction is
ks
86. What is the electrophile when RCl + AICI3 are used (a) Fittig reaction
in Friedel-Crafts reaction ?
Yo
(b) Wurtz-Fittig reaction
oo
id) C1+ (b) AICI4 (c) Sandmeyer reaction
(c) id) A1C1+ (d) Wurtz reaction
B

97. Which of the following has the highest melting


87. The treatment of benzene with isobutene in the
re

point ?
presence of sulphuric acid gives
(a) a-Xylene ib) m-Xylene
(a) isobutylbenzene ib) tert-butylbenzene
(c) p-Xylene id) Toluene
ou

(c) n-butylbenzene id) no reaction


ad

98. The reaction of toluene with CI2 in presence of


88. When Friedel-Crafts akylation of benzene is carried
FeCl3 gives predominantly,
Y

out with n-propyl bromide, the major product is


(a) benzyl chloride
(a) n-propylbenzene ib) isopropylbenzene
ib) o- and p-chlorotoluene
(c) 2-ethylbenzene id) none of the above
nd

(c) m-chlorotoluene
Re

89. Conversion of benzene to acetophenone can be


brought by id) benzoyl chloride
Fi

(a) Wurtz reaction .E

ib) Wurtz-Fittig reaction 99. In a compound electrophilic substitution


(c) Friedel-Crafts alkylation
id) Friedel-Crafts acylation has occurred. The substituent-E are methyl,
90. Which of the following compounds will not -CH2CI, -CCI3 and -CHCI2. The correct increa
undergo Friedel-Crafts reaction easily ? sing order towards electrophilicsubstitutionis
(a) Nitrobenzene ib) Toluene (a) -CH3 < -CH2CI < -CHCI2 < -CCI3
(c) Cumene id) Xylene ib) -CH3 < -CHCI2 < -CH2CI < -CCI3
91. The electrophile that participates in nitration of ic) -CCI3 < ^H2C1 < -CHCI2 < -CH3
benzene is id) -CCI3 < -CHCI2 < -CH2CI < -CH3
13/108 ‘Pnaictee^'^ New Course Chemistry (XI)BZ

100. Find the major product in the following reaction NO2


NO
CCI3
(0 [O
CI2
Q FeCl3 ^
id)
ir
CCI3 Br

w [o ri
ib)
Q 106. Among the following compounds, the decreasing
order of reactivity towards electrophilic
substitution is

W [O (d)
Q O a Q

w
Cl

101. Presence of a nitro group in a benzene ring CH3 OCH3 CF3

F lo
(a) deactivates the ring towards electrophilic I II III IV
substitution (a) in>i>ii>iv (b)iv>i>n>m
(b) activates the ring towards electrophilic (c) i>n>m>iv (d) n>i>iii>iv
substitution
107. The order of decreasing reactivity towards an

e
(c) renders the ring basic electrophilic reagent for the following :

Fre
(d) deactivates the ring towards nucleophilic (0 Benzene (ii) Toluene
substitution
(Hi) Chlorobenzoic acid (iV) Phenol

for
102. Which of the following activates the benzene ring would be
most towards electrophilic substitution ?
(a) (iv) > (ii) > (i) > (iif)
(a) -CHO (b) -NR2
(b) (i) > (ii) > (Hi) > (iv)
r
(c) -NHCOCH3 (d) -NO2
(c) (ii) > (iv) > (i) > (Hi)
You
103. Some m^/a-directing substituents in aromatic
oks
(d) (iv) > (Hi) > (ii) > (i)
substitution are given. Which one is most
deactivating ? 108. Which of the following is most reactive towards
eBo

(a) —COOH electrophilic substitution reaction ?


(b) —NO2
(a) Aniline (b) Nitrobenzene
(c) —C = N (d) —SO3H
(c) Benzoic acid (d) Acetanilide
104. Which one of the following is most reactive
towards electrophilic attack ? 109. Which one of the following is most reactive
our
ad

towards electrophilic reagent ?


Cl
,OH CH3

(o) [Q w(Q COT


(b) [O OCH3 OH
dY
Re

.N02
Q
(ST (<0 [0
(C)
Fin

NHCOCH3 CH20H
110. Electrophilic substitution of compound A will be
fastest at position
105. [Q
HNO3 Bf2
■►A 1 2 3 4
H2SO4 FeBr3
Cl CH3
NO2 NO2 I 2 3 4

Br (A)

w [O Ir
(b) [o (a) 1
(c) 3
(b) 2
(d) 4
HYDROCARBONS 13/109

II. Assertion-Reason Type Questions Reason. Water is more nucleophilic than Br ion.

For questions below, two statements are given


120. Assertion. Addition of Br2 to cyclohexene gives
cis-\, 2-dibromocyclohexane.
one labelled Assertion (A) and the other labelled
Reason (R). Select the correct answer to these Reason. The addition occurs through the
questions from the codes (a), {b), (c) and {d) intermediate formation of a cyclic bromonium ion.
given below : 121. Assertion. Acetylene is more reactive than ethylene
(u) Both A and R are correct and R is the correct towards electrophilic addition reactions.
Reason. Acetylene contains two 7C-bonds.

ow
explanation of A.
(b) Both A and R are correct but R is not the correct 122. Assertion. Addition of H2O to acetylene occurs in
explanation of A. presence of dil. H^S04 and HgSOj to give
(c) A is correct but R is wrong. acetaldehyde.
(d) A is wrong but R is correct. Reason. It is an example of electrophilic addition
reaction.
111. Assertion. n-Propylmagnesium bromide and
123. Assertion. Addition of H2O to propyne in presence

e
isopropylmagnesium bromide on treatment with
of dil. H-,S04 to give propanone is catalysed by

Fl
D2O give the same deuterated alkane.

re
Reason. In both the cases, D gets attached to the
HgS04. “
Reason. The addition of H2O to initially formed

F
same carbon that held Mg.
complex between propyne and Hg-'*' ions is
112. Assertion. Melting point of neopentane is higher
followed by loss of Hg""^ ions gives propanone.
than that of n- pentane but the boiling point of n-
ur 124.Assertion. Propyne reacts with CH3MgBr to

r
pentane is higher than that of neopentane.
evolve methane.

fo
Reason. Melting point depends upon packing of
molecules in the crystal lattice while boiling point Reason. CH^MgBr is a good source of methyl
carbanion.
depends upon surface area of the molecule.
113. Assertion. 3-Methylpentane has higher melting
ks
125.Assertion. The reaction of cone. HNO3 and cone.
Yo
point than 2-methylpentane. H-)S04 on nitrobenzene gives m-dinitrobenzene.
oo
Reason. 3-Methylpentane is more symmetrical Reason. The nitro group decreases the electron
than 2-methylpentane and thus fits closely in the density in the benzene ring.
B

crystal lattice. 126. Assertion. Hexadcuterated benzene and benzene

114. Assertion. Conformens are impractical to separate. undergo nitration at the same rate.
re

Reason. Conformers have negligibly small Reason. C-D bond is stronger than C-H bond.
difference in their potential energy. 127. Assertion. lodobenzene is formed when iodine
u

115. Assertion. Propene is more reactive than ethene reacts with benzene in pre.sence of an oxidising
ad

towards electrophilic addition reactions. agent.


Yo

Reason. Hyperconjugation effect of the CH3 group Reason. The oxidising agent oxidises the HI
increases the electron density in the double bond. formed in the reaction to form I2 thereby pushing
the reaction in the forwaid reaction.
116. Assertion. Propene reacts with HBr in presence
d

of organic peroxide to give 1-bromopropane. 128.Asscrtion. Alkylbenzene is not prepared by


Re
in

Friedel-Crafts alkylation of benzene.


Reason. The reaction occurs through carbocation
intermediate. Reason. Alkyl halides are less reactive than acyl
F

halides.
117. Assertion. Addition of HCl to F3C-CH = CH2 is
an electrophilic addition reaction. 129.Assertion. Oxidation of rerr-Butylbenzene with
Reason. Addition follows Markovnikov’s rule. KMn04 gives benzoic acid.
Reason. ferr-Butylbenzene does not contain a
118. Assertion. Cyclohexene on treatment with CI2 in benzylic hydrogen.
presence of UV light gives 3-chlorocyclohex-l-
ene.
130. Assertion. H2C CH^ is an aromatic
Reason. The reaction occurs through intermediate
formation of a stable carbocation. compound.
119. Assertion. Propene on treatment with Br2/H20 Reason. It contains a conjugated system of three
double bonds.
gives 1-bromo-2-propanol.
13/110 Course Chemistry (XI)S!EBEI

ANSWERS

I. Multiple Choice Questions

l.(cf) 2. (c) 3. (d) 4. (tO 5. (iO 6. (a) 7. (a) 8. (a) 9. (rf) 10.(a)
11. (ft) 12.(a) 13. ia) 14. (ft) 15.(c) 16. (a) 17.(c) 18. (ft) 19. (ft) 20. {/.)
21. (c) 22. 23. (c) 24. (a) 25. (d) 26. (a) 27.(a) 28. (a) 29. (a) 30. (e)
31. (d) 32. (a) 33. (c) 34. (c) 35.(b) 36. (fo) 37. ((/) 38.(c) 39. (^/) 40. {/?)
41.(a) 42. (c) 43. (c) 44. (a) 45. (c) 46. (a) 47. (tO 48. (c) 49. (ft) 50. (c)
51. (ft) 52. (iO 53. (b) 54. (rf) 55. (a) 56. (d) 57.(b) 58. (a) 59. (a) 60. (b)
61. (ft) 62. id)

w
63. (a) 64. (ft) 65. (c) 66.(a) 67. (/)) 68. (a) 69. (c) 70. (/.)
71. (a) 72. (c) 73. (/j) 74. ((0 75. (c) 76. (a) 77. 78. (c) 79.(a) 80. (a)
81.(c) 82.(a) 83. (c) 84. (a) 85.(b) 86.(c) 87. (b) 88. (ft) 89. (^0 90. (a)

F lo
91. (ft) 92.(a) 93. (a) 94.(e) 95.(c) 96. (a) 97.(c) 98. (ft) 99. (d) 100. (ft)
101. (a) 102. (ft) 103. (b) 104. (a) 105.(a) 106. (a) 107.(a) 108.(a) 109. (ft) no. (d)

n. Assertion-Reason Type Questions

ee
111. id) 112.(a) 113. (a) 114. (a) 115.(a) 116. (c) 117.(c) 118. (c) 119. (ft) 120. (d)

Fr
121. (d) 122. (a) 123. (a) 124. (ft) 125. (ft) 126. (ft) 127. (a) 128. (ft) 129. (d) 130. (d)

for
ur
s
For Difficult Questions
ook
Yo
I. Multiple Choice Questions Mg/elher
eB

5. CH3CH2CH2Br ^ CH3CH2CH2MgBr
1. (a), (ft) and (c) are all isomers bul (d) is not. Propyl bromide Grignard reagent

CH, HjO.A
CH3 ^ CH3CH2CH3
I I ^
our

-Mg(OH)Br
ad

Propane
2. CH3—CH—Br +2Na+ Br—CH—CH3
6. (CH3)3CMgBr + DO-D
CH3 CH3 (CH3)3CD + Mg(OD)Br
Dry ether I f CaO,630K
dY

> CH.,—CH—CH—CH, +2NaBr


A J J 7. CgHjCOONa +NaOH
Re

2,3-Dimcihylbutane Sod.ben7x>aie

^6^6 + Na2C03
Fin

CH2CH3 Benzene
Dry ether
3. 2CH3—(ch2)2—cn Br +2Na
{Wurtz 2 1
reaction) CH2—CH,—Br Dry ether
8.
CH,CH, CH,CH, CH, —CH—Br {Wurtz reaction)
i 2-3 3| - J 2 6-7 8
3 I
CH3—(CH,),—CH—CH—(CH2)2—CH3
4 5
+ 2NaBr 5CH3
1.4-Dibromopentane
CH,—CH,
4. 5-Dieihyloctane 1^1^
Thus, option (d) is correct. CH2—CH—CH3 +2NaBr
4. Ethane, propane and butane (Refer to page 13/7). Methylcyclobutane
HYDROCARBONS 13/111

9. Since bromides are more reactive than chlorides, 19. 3* H is abstracted most readily, Le.,
therefore, Wurtz reaction occurs on the side of Br
Bt2lhv
atom
CH3—CH—CH^CHj
Br
Wurtz
Na
CH3
reaction
-NaBr ^ -NaCl^ 2-Methylbutane Br
I
cr
CH3—C—CH2CH3
CH3
Bicyclo[I.1.0]butane 2-Brorao-2-methylbutane
20. Most stable free radical (Le., option 'b') is the
product.

w
(CH3CH2)2CH.
10. Cu Li* + CH3CH2-*-Br V,Os/AUO,
773K.10-20atm.^ Benzene
^6*^6

F lo
(CH3CH2)2CH^ 21. C,H„
n-Hexane
Lithium di(3-pentyl)cuprate
22. Mixture of o-, m- and p-xylenes (Refe to page 13/18).
CH2CH3
I 23. Cracking converts liquid hydrocarbons to gaseous
► CH3CH2—CH—CH2CH3 hydrocarbons by cleavage of C—C and C—H

ree
3-Ethylpentane bonds.
(M.F.C7H,6)

for F
27. Because of intramolecular H-bonding, gauche
+ (CH3CH2)2CHCu + LiBr conformation of 2-fluoroethanol is more stable than
anti while eclipsed conformation is the least stable.
11. LLAIH4 reduces T and 2® alkyl halides but brings
Thus, the increasing order of stability is :
about dehydrohalogenation of 3® alkyl halides. In
contrast, NaBH4 reduces both 2® and 3® alkyl
Your
halides (refer to page 13/9). Fqh
ks

12.
eBoo

Dry ether
2CH3I + 2Na ► CH3—CH3 + 2Nal
(fFurtz reaction) Ethane
<

CHJCHz-GcI Dry ether


ad

H" > CH3—CH3


our

Eclipsed
Ethane (Least stable)
(form LiAlH4) (Stable)

UAIH4 reduces 1® alkyl halides to alkanes.


Electrolysis
13. 2 CH3CCX)K + 2 HjO
Re
Y

CH3—CH3 + 2CO2 + 2KOH -I- H2 <


Find

Anode Cathode

15. Due to least surface area, neopentane has the lowest


b.p. but due to high symmetry, it has the highest Gauche

m.p. (Most stable)

CI2 Na.A
Thus, option (a) is correct.
(Wurtz reaction) > CH3—CH3
17. CH4 —^ CH3CI
Ethane 28. Only in 1, 2-dibromopropene (CH3CBr = CHBr)
18. 2, 3-Dimethylbutane has only two types of both the carbon atoms of the double bond have

hydrogens and hence forms only two mono- different substituents and hence shows geometrical
isomerism.
chlorinated compounds.

(
13/112 7^*%adee^ 4. New Course Chemistiy (XI) fSL
2 3 4 5 6
Cohc,H2S04
30. CH3CH=CH—CH=CHCH3 (2, 4-hexadiene) 35. CgHjCHj—CH—CH—CH3 -
shows geometrical isomerism around C3-C4 single I -H2O
bond. OH CH3
H
C = C
C=NOH (benzaldoxime) shows geomet-
H H
'-CH(CH3)2
rical isomerism around C = N bond. Dehydration occurs in such a way that the double
CH3CH = CHCH3 (but-2-ene) and CICH = CHCl bond comes in conjugation with the CgHg group
and further since rran^-alkenes are more stable than
(1, 2-dichloroethene) show geometrical isomerism
around C = C bond
the cw-alkenes, therefore, option (b) is correct.
3 2 1

36. CgHj—CH2—CH=CH2
C6H5-V. 3-Phenylpropene
C=NOH (benzophenone oxime) having

w
C6H5 +

CgHj—CH—CH—CH3 1,2-Hydride ^

F lo
two identical groups on the doubly bonded carbon shift
does not show geometrical isomerism.
31. Six isomers are : dJ
2® Carbocation
/Br F\ I (less stable)
C=C c=c
cK I cK '^Br ’

ree
+

/Cl I CgHj—CH—CH2—CH3

for F
c=c c=c 2® Carbocation
Br^ I Br^ Cl (more stable)
F\ Br CfiHs—CHBr—CH2CH3
/Cl
c=c and :c=c:
'^Br 37. C6H5CH2CH = CH2 + HBr
Cl
Peroxide effect

CH3 > CgH5CH2CH2CH2Br


Your
(Anti-Markovnikov
KOH (ethanolic), A addition) 1 -Bromo-3-phenylpropane
ks

32. CH3—C—CH2CH3
I (Saytzeff elimination) 38. Addition of preferentially occurs on C2 rather
eBoo

Br than on Cj since it generates a carbocation which


2-Bromo-2-methylbutane CH
3\ is stabilized by resonance over the benzene ring.
^C=CH—CH3 +

CH3 CgHg—CH=CHCH3-^ CgHg—CH—CH2CH3


ad

2-MethyIbut-2-ene (Stabilized by resonance


our

CH, over thebenzene irng)


I ^
KOH (ale), A Br“
33. CH,-C-CH,C1 > No P-hydrogen, » CgHg—CH—CH2CH3
3 ipa 2 I
nodehydrohaloge-
CH3 nadon reaction Br
Re

Neopentyl chloride 1 -Bromo-1 -i^enylpropane


Y

CH3 CH3 39.


H
1
Find

34. CH3—C—CH2OH
H+ I
► CH3—C—CH2 H3C—C—CHiCHj-^ CHj—C^CH—CH3
-H2O
CH3
I
CH3 CH3
Neopentyl alcohol 2® Carboration
CH3 (less stable)
1,2-Methyl
shift
► CH3—C—CH2CH3 Br
+
-H^
1,2- +
Br" I
Hydride ► CH3—C—CH2—CH3-^ CH3-C-CH2CH3
CH3 shift
CH3 . CH3 .
CH3—C=CHCH3 3® Carbocation
2-Methylbut-2-enc (more stable)

J
HYDROCARBONS 13/113

40. Only addition of HBr to unsymmetrical alkenes 51.


shows peroxide effect. Thus, option (b) is correct.

5^
4 6
(0O3/CH2Cl2^ 2 CH3
41. Symmetrical alkenes (such as 2-butene) do not CH3 ► CH2—CH
show peroxide effect and thus give only one (//) Z11/H2O c
addition product. 1-Methyl- o
2 1
42. In presence of light, anti-Markovnikov’s addition cyclopentene
CH2—CH=0
occurs while in absence of light, Markovnikov’s 5-Oxohexanal

ow
addition occurs. Therefore, option (c) is correct.
52. Since the diol on further oxidation forms a
43. Refer to table on page 13/40.
diketone, therefore, the diol must be mono-
In case of HCl, 2nd step is endothermic while in
substituted, i.e., it must contain 2° alcoholic groups.
case of HI, the first step is endothermic.
Therefore, option {d) is correct.
(/) H2SO4
44. CH3—C = CH2 »
Cold. aq. alk. KMn04
I (ii) boil with H2O CgHjCH = CHCgHg

e
{Baeyer's reagent)
C6H5 OH (X)

re
2-Phenylpropene I 10]

rFl
CH3—C—CH3
I

F
OH OH
2-Phenyl-2-propanol (Diol contains two 2° alcoholic groups)
H+
45. (CH3)2C = CH2 + H2O 4 C6H5-C-C-C6H5

r
Maik. addn.
ou O O

sfo
(CH3)3C—OH Diketone
rert-Butyl alcohol
54. The aldehyde with molecular mass 44 u is CH3CHO
CI2/H2O
&- 5+ ^
k
(acetaldehyde). Therefore, the symmetrical alkene is
2-butene
oo
46. Cl
AddnofHO-Cl
Propene (i)03/CH2a2.196K
(Markownikov’s HO CH3CH = CHCH3
Y
Addn.)
(»)Zn/H20
eB

2-Butene
47. Writing the structures of the three products of
ozonolysis side by side, we have, 2 CH3CHO
Acetaldehyde
CH3CH=0 -h 0=CH—CH=0 + 0=CHCH3 (Mol. mass = 44 u)
r

Ethanal Ethanedial Ethanal


(/) O3/CH2CI2
ou

Remove the oxygen atoms, and combine the three 55. R2C =CH2
Y
ad

fragments together, the structure of the hydro (») Zn/H20


carbon is Vinyl group

1 2 3 4
R2C = 0 + 0 = CH2
Ozonolysis
Formaldehyde
CH3CH iCH—CH4:CHCH3
d

2,4-Hexadiene 56. Refer to page 13/47.


in
Re

CH3CH = O -1- O = CH—CH = 0 + 0 = CHCH3 57. 1,3-Butadiene (CH2=CHCH = CH2) has only four
carbon atoms while all others have five carbon atoms.
F

48.
58. C5Hg has the following three isomers :
(OO3/CH2CI2 / \ (/●) CH3CH2CH2C s CH (ii) CH3CH2C s CCH3
(«0Znm2O ^ 2 0HC CHO
Propane-1,3-dial (Hi) (CH3)2CH—C s CH.
1,4-Cyclohexadiene
60. CH3C = CCH3 being linear and symmetrical has
CH. CH. lowest (or zero) dipole moment.
I M ^ (0O3/CH2a2 61. Markovnikov’s and anti-Markovnikov’s addition
50. CHjCHj—C=C—CHjCHj (i7)Zn/H20 of HBr gives two structural isomers, i.e..
3,4-DimethyIhex-3-ene
HBr
CH3 CH3C = CCH2CH3 + CH3CH = C(Br)CH2CH3
I
2CH3CH2—C=0 + CH3C(Br) = CHCH2CH3
Butanone
13/114 4^ New Course Chemistry (XDiSsZBD

Each of these two isomers can exist as a pair of 73. Refer to Ans. to Q. 19, Page 13/120.
cis, trans-isomtTS and hence there are four
configurational isomers. H2O
74. HC = CH ^ CH2 = CH0H
62. Alcohols are stronger acids than alkynes. HgS04/H2S04
Acetylene Vinyl alcohol (X)
63. HCsCH + NaOH HC=C-Na+ + H20
Weaker base Stronger base CH3CHO
HC = C“ is a stronger base than NaOH (Refer to Acetaldehyde

w
‘Supplement Your Knowledge For Competitions, 75.
pages 13/55-13/56). Br
64. Base strength decreases as the electronegativity of
.C=CH
the carbon canying the negative chaige increases from
NaNH2
sp^- sp^-sp. TTius, option {b) is correct.
-NaBr,-NH3^

o
Br
65. Only terminal alkynes react with Na in liquid NH3. O

e
Thus, option (c) is correct
HjO, HgS04,

re
Na/liq.NH3

rFl
CH3CH2C = CH CH3CH2C = C- Na+ H2SO4 ^ CH3
66. CH3C = C—H + CH3MgX

F
■>

CH3C = CMgX + CH4 Although ale. KOH can also bring about first
Mark
reaction but NaNH2 gives better yields. Refer to
67. CH3C = CH + 2 HBr ^ CH3C(Br)2—CH3 page 13/51 for details.

r
addition J J
76. Na in liq. NH3 gives rra«5-alkenes.
ou
fo
68. Addition of both HCl and HI occurs in accordance
with Markovnikov’s rule.
77. AgN03 in NH3 reacts with propyne to give white
ppt. of silver propynide while propene does not.
69. Refer to page 13/58, reaction 3.
70. During addition of Br2/H20, first Br'*’ adds follo
wed by nucleophilic attack by H2O. Loss of a H"^
ks
79. Four structural isomers are possible because there
are four types of different hydrogens, t.e.,
oo
hydrogens on CH2, o-, m- and p-hydrogens.
foUowed by tautomerization gives 2-bromoacetone
80. Benzene neither decolourises alkaline KMn04
Y
solution nor gives ppt. with ammoniacal AgN03
eB

+ solution.
CH3—C^CH + Br"" ► CH3—C=CHBr
81. (a) has a cyclic cloud of 2 Tt-electrons and hence is
H2O aromatic, (b) and (d) have a cyclic cloud of six n-
r

> CH3—C=CHBr CH3—C=CHBr electrons and hence are aromatic while (c) has 4
-H* n- electrons and hence is antiaromatic.
ou

■"OH2 OH
Y

82. Only (a) has 2 Ji-electrons and hence is aromatic.


ad

Tautomerises All others have 4 7t electrons and hence are


> CH3—C—CH2Br antiaromatic.

83. Cyclooctatetraene has 8 7C-electrons. It does not


d

O
Bromoacetone follow Hiickel’s rule and is also not planar (has
tub shaped structure), therefore, it is non-aromatic.
in
Re

HgS04 + H2S04 84. Cyclopentadiene, i.e., option (a) contains only 4


71. CH3CH2C s CH + H2O Maik.addn. 7t-electrons and is also non-planar, therefore, it is
F

1-Butyne non aromatic.


Tautomerises
■CH3CH2—C = CH2‘ [O] NaOH
85. C6H5—CH3 CgHj—COOH ■»

OH (A)
Soda-lime
CgH5COONa
CH3CH2—CO—CH3. “ Na2C03
(B) (C)
H2S04/HgS04
72. CH3—C = C—CH3 -I- H2O
2-Butyne
87. (CH3)2C=5=CH2 + H""
Isobutene
*(CH3)jC-^
CH3COCH2CH3. (CH3)3C—C6H5
2-Butanone re/t-Butylbenzene
HYDROCARBONS 13/115

88. CH3CH2CH2Br + AICI3 n. Assertion-Reason Type Questions


-AlCljBr 111. Correct A. n-Propylmagnesium bromide on
n-Propyl bromide
1,2-
■>
treatment D2O gives 1-deuteropropane while
CH3—CH—CH2
Hjdride shift
isopropylmagnesium bromide on treatment with

(V
D2O gives 2-deuteropropane.
116. Correct R. The reaction occurs through free radical
n-Propyl carbocatlon intermediate.
{less stable) 117. Correct R. It follows anti-Markovnikov’s rule.
CH3

low
118. Correct R. The reaction occurs by a free radical
CH3—CH—CH3
Isopropyl caibocation -H ^ CgHs-CH;^ mechanism.
CH3 119. Correct explanation. In the first step Bt2 reacts
{more stable)
Isopropylbenzene with the double bond forming a cyclic bromonium
90. Due to strong deactivation of benzene ring by the ion (I). Since H2O is a better nucleophile that Br“,
powerful electron-withdrawing nitro group, the therefore, in the second step, nucleophilic attack
electron density on the benzene ring decreases to
such an extent that nitrobenzene does not undergo
by H2O occurs on the cyclic bromonium ion (I)

ee
F.C. reaction.
followed by loss of a proton to form l-bromo-2-

rF propanol

Fr
92. During nitration, NOJ is formed from HNO3 as
follows : J-Br
CH3—CH=CH2 +'Br-s-
8+
ft- Slow ^
^ H+ + HSO" -Br"
H2SO4

or
6+
H—O—NO2 + ► H—O—NO2 CH3—CH=CH2

f
ou
'v I
’ *
Base
Acid
I N

H
ks Br^^
>H20 + N0| I {More stable
oo
Thus, nitric acid acts as a base. incipient carbocation)
93. lodination occurs in presence of an oxidising agent CH3—CH—CH2Br ^ CH-—CH—CH.
Y
+ 3 , ,2
B

(HNO3). -H
OH Br
94. Acetophenone, benzonitrile, benzaldehyde and
V\ H
re

1 -Bromo-2-propanol
benzoic acid all contain electron-withdrawing H
groups, therefore, in all these compounds the
electron density over the phenyl ring is lower than 120. Correct A. Addition of Br2 to cyclohexene gives
u

trans-\, 2-dibromocyclohexane.
ad

in benzene. Thus, benzene undergoes electrophilic


121. Correct A. Acetylene is less reactive than ethylene
Yo

nitration most readily.


95. Toluene has electron-donating methyl group (while towards electrophilic addition reactions due to its
others have electron-withdrawing groups) and sp-hybridized carbon atoms and cylindrical nature
of electron clouds, (refer to page 3/53).
hence most reactive towards electrophilic nitration.
nd

124. Correct explanation. CH3MgBr being a stronger


Re

100. —CCI3 is m-directing. So option {b) is correct.


base abstracts a proton from propyne to form
105. NO2 group is m-directing. methane or propyne being a stronger acid displaces
Fi

106. Reactivity towards electrophilic substitution a weaker acid, i.e., methane


decreases as the electron density in the benzene
ring decreases. Thus, option {a) is correct, i.e.,
m>i>n>iv. CH3—C=C—H^
^ a +
5^
CH3—MgBr
5-

109. +R-Effect of OH is greater than that of OCH3 Aropyne Strong base


{Stronger acid)
group.
110. Since -I-effect of -Cl is stronger than its + R- CH3—CsCMgBr + CH4
effect, therefore, it deactivates the ring towards Methane
{Weaker acid)
electrophilic substitution. However, due to
+ I-effect and hyperconjugation of the CH3 group 125.Correct explanation. The nitro group in
at o-positions, electrophilic substitution will be the nitrobenzene mainly decreases the electron density
fastest at position 4. at o- and p-positions and hence it is m-directing.
13/116 New Course Chemistry fXIlwsTwn

126. Correct explanation. Hexadeuterated benzene and 129. Correct assertion. Oxidation of rt-rr-bulylbenzene
benzene undergo nitration at the same rale because with KMn04 does not give benzoic acid.
C-D bond cleavage is not the rate-determining step
of the reaction.

128. Correct explanation. Due to -i-I-effect of the alkyl ISO.Correct A. H2C CH2 is not an
group, electron density increases In alkylbenzene
relative to benzene and hence further substitution aromatic compound since the three conjugated
double bonds are not in the ring.
occurs to form a poly alkyl substituted benzene.

w
1. Alkanes

F lo
1. Devise a scheme for the synthesis of «-butane using CH3I as the only carbon source. Can you employ
the reactionsin your scheme to synthesizepropane in fairly pure state ? Explain.
Na. dry ether (0 CI2//1V Na, dry ether
Ans. 2CH3I ^ CH3—CH3 ^ CH3CH2CI

ee
Wimz reaction Wiirtz reaction
orSO2Cl2/R0OR
Methyl iodide Ethane Ethyl chloride

Fr
CH3CH2—CH2CH3
Butane

for
Propane cannot be synthesized by the above scheme in fairly pure state because Wurlz reaction between
CH3I and C2H3I will give three products, i.e., ethane, propane and butane.
our
2. How will you prepare a pure sample of propane ?
Ans. Propane is an unsymmetrical alkane, it cannot be prepared by Wurtz reaction. However, it can be easily
prepared by Corey-House reaction as shown below :
s
ook
Dry ether
CH3CH,—Br -i-2Li CH3CHt—Li -I-LiBr
Ethyl bromide Ethyliithium
Y
eB

Dry ether
2 CH3CH2—Li + Cul ^ (CH^CH^jjCuLi-H Lil
Lithium diethyicopper
our
ad

CH3CH2
Dry ether
Br > CH3CH2CH3 + CH3CH2C11 + LiBr
CH3CH2 Methyl bromide
Y

Propane
Lithium diethyicopper
Re
nd

3. What happens when butanoic acid is heated with soda-lime at 630 K ? Is there any other acid which
also gives the same product under similar conditions. Write its name and the reaction involved.
Fi

Ans. Butanoic rtcid undergoes decarboxylation to give propane when heated with soda-lime at 630 K.
CaO.630K
CH3CH2CH2COOH + 2NaOH > CH^CH.CH^ -bNa.CO,+ H0O
{Decarboxylation)
Butanoic acid Proptine

2-Methylpropanoic acid is the position isomer of butanoic acid. This also on decarboxylation with soda-
lime gives propane.
CH
3\ CaO.630K
CH—COOH + 2NaOH >\ CHXHXH.
.1 Z .7
-I-Na.CO,-H HoO
CH3-^ (Decarboxylation)
Propane
2-Mcthy]propunoic acid
HYDROCARBONS 13/117

4. Give structure of the product (X) in the following reaction :


(i) Mg, ether
Br CH2CH3 > X
(ii) D2O
Ans.

Mg, ether D2O


Br CH2CH3 CH2CH3 ► D CH2CH3
-Mg(OD)Br

Grignard reagent /)-Deuterioethylbenzene


5. Which of the following has the highest boiling point ?
(i) 2-methylpentane (ii) 2, 3-dimethylbutane {in) 2, 2-dimethylbutane.

w
Ans. As the branching increases, surface area decreases. As a result, magnitude of van der Waals forces of
attraction decreases and hence the boiling point decreases. Now 2- methylpentane has the largest surface
area and hence has the highest boiling point. Further because of the presence of two branches on the same

F lo
carbon, 2, 2-dimcthylbutane has lower surface area and hence has lower boiling point than that of 2, 3-
diniethylbutane. Thus, the overall order of decreasing boiling points is : 2-methylpentane (333 K) > 2, 3-
dimelhylbutane (331 K) > 2, 2-dimethylbutane (323 K).
6. What efl*ect the branching of an alkane has on its melting point ?

ee
Ans. In general, as the branching increases, the packing of the molecules in the crystal lattice becomes less close

Fr
and hence the m.p. decreases accordingly. However, if the branching makes the molecule symmetrical, the
packing of the molecules in the crystal lattice becomes close and hence the m.p. increases. Now since
isopentane is less symmetrical than n-pentane but neopentane is the most symmetrical of the three isomeric

for
pentanes, therefore, the m.p. of isopentane (113 K) is lower than of n- pentane (143 K) but the m.p. of
neopentane (256 K) is much higher than that of isopentane and n-pentane.
ur
7. An organic compound on monochlorination gives a single monochloride. Write the structure
of the hydrocarbon.
s
Ans. Since, the hydrocarbon (CgHjj^) on monochlorination gives a single monochloride. therefore, all the 18 H-
ook
Yo
aloms are equivalent.The only such hydrocarbonis 2, 2, 3, 3-tetramethylbutane, i.e..
CH3 CH3
eB

CH3—C C—CH3
CH3 CH3
our
ad

8. What are the main constituents of LPG ?


Ans. The main constituents of LPG are butane and isobutane. Both these isomers can be easily liquefied and
hence can be conveniently transported in iron cylinders.
Y

9. Why do the C—C bonds rather than C—H bonds break during cracking of alkanes ?
Ans. Since the bond dissociation energy of C—C bonds(348 kJ moL') is lower than bond dissociation energy of
Re

C—H bonds (414 kJ mor*), therefore, during cracking of alkanes, C—C bonds break more easily than
nd

C—H bonds.
Fi

10. Is it possible to isolate pure staggered ethane or pure eclipsed ethane at room temperature ? Explain.
-1
Ans. The energy difference between staggered and eclipsed forms of ethane is just 12-55 kJ mol which is
easily met by collisions of the molecules at room temperature. Therefore, it is not possible to isolate either
pure staggered or pure ec ^>sed ethane at room temperature.
II. Alkenes

11. Dehydration of alcohols to alkenes is always carried out with cone. H2SO4 and not with HCl or
HNO3. Explain.
Ans. Dehydration of alcohols to alkenes occurs through carbocation intermediate. If HCl is used, the chloride
(CL) ion being a good nucleophile brings about substitution reaction to give an alkyl halide.
13/118 “Pfieuie^ '4. New Course Chemistry (XI) ESE

H+
R—CH2—CH2—OH -HjO
» R—CH2—CH2 -H
R—CH = CH2
Alcohol Caitocation Alkene
(Elimination product)
+cr

R—CH2—CH2 —●Cl

w
Alkyl chloride
(Substitution product)

If H2SO4 is used, then the bisulphate (HSOp ion being mn-nucleophilic cannot bring out the substitution
reaction. Instead, the carbocation loses a proton to form an alkene. Cone. HNO3 is not used because it being
a strong oxidising agent, preferentially brings about the oxidation of alcohols irrst to aldehydes or ketones
and then to carboxylic acids. Thus, dehydration ofalcohols toform alkenes is always carried out with such

o
strong acids (e.g., HfSO^ KHSO4) which have non-nucleophilic anion).

e
12. Dehydratioii 1-butanolor 2-butanolwith cone. H2SO4 always gives the same mixture of 2-butene (80%)

re
and 1-butene (20%) but dehydrohalogenatiou of 1-bromobutane with ale. KOH gives 1-butene as the

rFl
n^jor product while that of 2-bromobutane gives 2-butene as the m^jor product Explain i^y ?

F
Ans. During dehydration of alcohols, carbocations are the intermediates. In case of 1-butanol, first 1° carbocation
is formed (refer to page 13/33) which being less stable rearranges to the more stable 2° carbocation before
losing a proton to form a mixture of 2-butene (80%) and 1-butene (20%).

or
2-Butanol, under the same conditions, directly gives the more stable 2** carbocation which then loses a
ou
proton to give the same mixture of 2-butene (80%) and 1-butene (20%).

ksf
1,2-Hydridc ^
+ +

CH3CH2CH2CH20H ► CH3CH2—CH—CH2 shift


CH3CH2—CH—CH3
-H2O
1-Butanol

ay 2° caibocation
(more stable)
oo
r carbocation
(less stable)
Y
B

CH3CH2—CH—CH3 CH3CH2—CH—CH3
-H2O
T carbocation
re

OH (more stable)
2-Butanol
oYu

Since 1-butanol and 2-butanol give the same carbocation intermediate, therefore, on losing a proton, both
ad

give the same mixture of 2-butene (80%) and 1-butene (20%) in accordance with the Saytzeff’s rule.
In case of dehydrohalogenation of alkyl halides with ale. KOH, carbocations are not the intermediates.
Instead the reaction occurs by a concerted mechanism involving a transition state. Further, since in
d

1-bromobutane, there is ^-H only on left side of a-carbon carrying the Br atom, therefore, only 1-butene is
formed as the major pix^uct. However, in 2-bromobutane there are P-H on either side of &e a-carbon
in
Re

carrying the Br atom, therefore, elimination of a proton can occur from either p-H leading to the formation
of a mixtureof alkenes.Since accordingto Saytzeffrule, 2-butene is more stable than 1-butene, therefore,
F

dehydrohalogenation of 2-bromobutane gives 2-butene as the major product.


Br

CH3CH2—CH-^CH2 > CH3CH2—CH=CH2


1-Butene (major product)
HO H
1-Bromobutane

Br
P N p
CH3CH=CHCH3
CH3—C^CH-^H2
CH3CH2— CH= CH2
2-Butene 1-Butene
o_
(major product, 80%) HO H b “OH
fl H (minor product, 20%)
2-Bromobutane

%
HYDROCARBONS 13/119

13. Predict the major product in the folio^ving reaction :

H^SO,
QHg + (CH3)2CHCH20H ●>

Ans. In presence of cone. H2SO4, isobutyl alcohol first gives T carbocation (I) which then rearranges to the
more stable 3“ carbocation(II) by 1,2-hydrideshift. Carbocation (II) then reacts with benzene to form ten-
butylbenzene.
CH3 CH3 CH3

r\n*
● I

CH3—CH—CH2—OH ► CH3—CH—CH 2-^COH2 > CH3—C—CH2


-H2O

low
Isobutyl alcohol
H
I Carbocation (I)
(l&ss stable)
CH3 CH3
1,2-Hydride C6H6
shift
> CH3—CH—CH3 CgHj—C—CH3
-H^

ee
3® Carbocation (11)

rF (more stable) CH3

Fr
rm-Butylbenzene
14. Identify the organic products obtained in the following reaction :
CH3

for
Br
C2ll50Na
C2H5OH
ou
Ans. Dehydrohalogenation of the given alkyl halide can, in principle, yield two alkenes (I) and (II)
ks
CH3 CH3 CH3
oo
Br
C2MsONa
Y
eB

>● +
C2H5OH

I (major) II (minor)
But according to Saytzeff rule, more highly substituted alkene, i.e., (I) being more stable is the major
r

product of dehydrohalogenation. Therefore, in the above reaction, alkene (I) along with a small amount of
ou
ad

alkene (U) is produced.


15. Predict the products of the following reaction
Y

Ph H HBr
c = c; Product
H
CH3
Re
nd

Ans. Electrophilic addition occurs through the more stable carbocation intermediate
Fi

CH—CH2CH3

Stabilized by dispersal of +vc Br


charge over the Ph ring 1 -Bromo-1 -phcnylpropanc

III. Alkyncs
16. Complete the following reactions with appropriate structures of products ?
(ONaNHjO.Ocquiv.)
CgHjCH = CH2 ^ [A] ^ B
(ii)CH3l
13/120 7\<zeCee^’<i, New Course Chemistry (XI)S5E

Bt2 2NaNH2
Ans. CgHjCH = CH2 ■> C,H,—CH—CH, CgHj—C s CH
Styrene 6 5 I I 2 (-2HBr)
Phenylacetylene
Br Br

1, 2-Dibromo-
1-phenylethane (A)

ow
NaNH
^ CgHj—C=C"Na^ (-Nal)
> C6H5—C=C—CH3
(-1/2H2) l-Phenylpropyne (B)

Thus, [A] is 1, 2-dibromo-J-phenylethane and [B] is 1-phenylpropyne.

17. How will you prepare 3-methylbut-l-yne by starting with etbyne ?

e
Na, NH3 (/)

re
Ans. HC = CH 4 HCsC"Na+

rFl
196 K
Ethyne Sod. acetylide

F
CH3 CH3
- _o —

r
4 3

HC=C‘Na* + CHj—CH-'-I > CH3—CH—C=CH + Nal


ou
sfo
Isopropyl iodide 3-Methylbut-1 -yne

18. Predict tbe m^or product in tbe following reaction :

H2
k
oo
R_CsC—R
Lindlar’s catalyst
Y
eB

H
I
H
I R-pC-C^R
I H i i H
I
I
R R
A—i 1—'r
Ans. H2 + Catalyst —► / Catalyst '/
R-CsC-R
> / Catalyst / > / Catalyst / + :c=c:
r

H H
ou

c/s-Alkene
Y
ad

19. Identify tbe product in tbe following reaction.

H30+,Hg2+
d

PbC s CMe

Ans. First of all Hg^"*" ions form a complex (I) with the alkyne. The complex formed here is, however, unsymmetrical
in
Re

in which the +ve charge lies on the carbon attached to the Ph ring since it is stabilized by resonance with the
double bonds of the Ph ring. Later on nucleophilic attack of H2O on this carbon ultimately leads to enol (II)
F

which tautomerisesto give ketone(HI).

OH
I
Ph—C = C—CH3 Hi!:, Ph—C=C—CH3 ^ Ph—C=CH—CH3
-Hg2 II

Cv
I (More stable
incipient carbocation) Tautomerises
> Ph—CO—CH2CH3
lU
HYDROCARBONS 13/121

20. How will you prepare (i) c»-pent-2-ene and ^a/i5-pent-2-ene by starting with ethyne ?
Na, NH3(/) CH3I Na, NHj (/)
Ans. HC = CH HC = C- Na+ ^ CH3—CsCH ■> CH3—C=C-Na+
196 K - Nal 196 K
Ethyne Propyne Sod. propynide
CH3CH2I
-Nal
^ CH3—C = C—CH2CH3
Pent-2-yne

ow
H2/Pd-BaS04
+ S or quinoline Na/liq. NH3, 196 K
(Lindlar’s catalyst) {Birch reduction)

CH
CH
3 \L^ XH0CH3 3\ ^ ^ H
yc = cz ^C = C
H
H CH2CH3

e
cis -Pent-2-ene trans- Pent-2-ene

re
rFl
21. Starting with ethyne, how will you prepare pentan-2-one ?

F
Ans. Prepare pent-2-yne as discussed in Q. 20 above and then convert it to pentan-2-one by hydration.
OH
60%H2SO4.HgSO4 I

r
Tautomerises
CH3—C = C—CH2CH3 + H2O ■>
[CH3 C=CHCH2CH3
ou 333 K

fo
Pent-2-yne
o

ks CH3—C—CH2CH2CH3
Pentan-2-one {major isomer)
oo
22. How will you separate a mixture of ethane, ethylene and acetylene ?
Ans. This mixture can be separated into its constituents by the following steps :
Y
eB

Step 1. Pass the mixture of gases through Tollens’ reagent when acetylene will form white precipitate of
disilver acetylide while ethane and ethylene will pass through.
HC = CH + 2[Ag(NH3)2l+OH- Ag—C=C—Ag + 4NH3 + 2H2O
ur

Acetylene Tollens’ reagent Disilver acetylide


{White ppt.)
ad

Separate the white ppt. by filtration and treat it with dil. HNO3 to regenerate acetylene. Collect it in a
Yo

separate container.
Ag—C = C—Ag + 2HNO3 ^ HCsCH + 2AgN03
d

Step 2. Pass the remaining mixture of ethane and ethylene through cold cone. H2SO4 when ethylene will be
Re

absorbed as ethyl hydrogen sulphate while ethane escapes. The ethanethus obtainedis collectedin a separate
in

container.

CH2 = CH2 + H2SO4 {cone.) ^ CH3CH2OSO2OH


F

Ethylene Ethyl hydrogen sulphate


The ethyl hydrogen sulphate thus obtained is heated with cone. H2SO4 to 433 - 443 K when ethylene is
obtained which is collected in a separate container
433 - 443 K
CH3CH2OSO3H 4 CH2 = CH2+H2S04
23. Give reasons for the following :
CHj = CH" is more basic than HC h C“.
Ans. CH2 = CH" is the conjugate base of the acid H2C = CH2 and HC = C" is the conjugate base of the acid
HC = CH. We know that stronger the acid, weaker is the conjugate base. Since, HC = CH is a stronger acid
than CH2 = CH2, therefore, CH2 = CH" is a stronger base than HC = C".
13/122 ^>uuiUe^‘^ New Course Chemistry (X1)E3S19D

IV. Arenes

24. Predict which of the following systems would be aromatic and why ?

(ii) (iii)

ow
Ans. Compound (/) contains (4 n + 2)n electrons where n = 2, i.e., it contains 5x2=10 7t-electrons and hence
accordingto Hiickel rule, it is aromatic.
Compound (iii) is also aromatic since it contains two benzene rings each one of which contains (4 n + 2) tc-
electrons with n = 1, i.e., 3x2 = 6 electrons.
Compound (//) is a 4 n 7t (/i = 1) electron system and hence is an antiaromatic compound.
Although compound (iv) contains 6 7i-electrons, i.e., (4 ;i + 2) 7i electrons with n = 1, yet the system is not

e
completely conjugated due to the presence of a -hybridized carbon. In other words, this compound is not
planar and hence is not aromatic.

re
rFl
25. Write the ozonolysis products of mesitylene.

F
CH3
O

Ans. (/) O3/CH2CI2. 196


● --(
3CH3—C—CH=0
(/OZ11/H2O

r
H3C ● CH3
ou Methylglyoxal

fo
Mesitylene

ks
26. Write the major product in each case

CH3
oo
Anhyd. AICI3
(a)
QJ + CH3—c—ch2“ Br
Y
B

CH3

o—CO
re

N02
(*) (AIPMT 2006)
ou
Y

Ans. (a) Refer to ‘Supplement Your Knowledge For Competitions’, page 13/78.
ad

(b) Out of-0- and C = O groups, O is electron-donatingand o, p-directing. Therefore, it increases electron
density in the benzene ring directly attached to it. As a result, electrophilic substitution occurs in this ring
predominantly at the less hindered p-position
d

O O
in
Re

O—C O—C
NO2
F

O2N
Phenyl benzoate p-Nitrophenyl benzoate

In contrast, the C = 0 group is electron-withdrawing and m-directing. Since, it decreases the electron-
density in the ring to which it is attached, therefore, substitution does not occur in this irng.
0

O—CO O—C NOo


NO2
HYDROCARBONS 13/123

27. Give reasons for the following :


te/t-Butylbenzene does not give benzoic acid on oxidation with acidic KMn04.
Ans. Alkylbenzenes in which the alkyl groups contain one or more a-hydrogens or benzylic hydrogens on vigorous
oxidation with acidic KMn04 ultimately give the corresponding benzoic acids irrespective of the length of
the carbon chain. For example,
a a

CH3 CH2CH3 CH(CH3)2 COOH

A
.0. or
o or
Q KMnp4/H'*' ^ O.
[O]
Toluenes Ethylbenzene

w
Isopropylbenzene Benzoic acid
[3o-Hs] [2a-Hs] [one a-H]
/e/t-Butylbenzene, on the other hand, does not contain any a-hydrogen or benzylic hydrogen and hence
does not undergo oxidation easily to give benzoic acid.

F lo
COOH
CH3
a I [01
CH3—C—CH3
O

ee
Fr
Q Benzoic acid

tert-Butylbenzene

for
(No a-hydrogen)
28. How will you explain the directive influence of
our
(i)<CH = CH2and (u) - CCI3 group when attached to the benzene ring ?
Ans. (i) Stability of the carbocation intermediate can be used to explain the o, p-directing influence of the
s
- CH = CH2 as follows :
ook

o-Attack:
+

CH=CH2
Y
eB

CH—CH2
E

H > etc.
our
ad

p-Attack:
CH=CH2 —CH2 CH—CH2
A
Y

E^
etc.
Re

X X
nd
Fi

m- Attack;
CH=CH2 CH=CH2

jAs E^ A
kA
<4 <■ etc.

Only carfoocations resulting finom o- and p~ substitution are stabilized by spreading the chaige to the side
chain while such type of stabilization is not possible in case of m-substitution. Thus, —CH = CH2 group is
o, p-directing. Siniilarly, we can explain the directive influence of C5H5 group which is also o, p-diiecting.
13/124 ‘P>uuteeft.’4. New Course Chemistry (XI)BEIMD

(ii) Hyperconjugation effect can be used to explain the directive influence of CCI3 group as discussed
below.

Cl cr cr
q
Cl—c > Cl—c Cl—c

Cl Cl Cl

cr

w
> Cl—C + Six more such resonating
structures for the other two Cl atoms
Cl

Flo
Due to the presence of +ve charge at o- and p- positions, the electron density is comparatively more at m-
positions. TTierefore, -CCI3 is a m-directing group.

e
re
● Very Short Answer
dill
I

rF
● Short Answer
● Long Answer

r Carrying 1 mark

fo
VERY SHORT ANSWER QUESTIONS
ou
I. Alkanes
ks
1. Indicate the primary, secondary, tertiary and quaternary carbon atoms in the following :
oo
(/) 3-Ethyl-2-methylhexane (//) 2, 2, 4-Trimethylpentane.
Y
]o 2° r 1” r
B

CH, CH-CH., CH3 CH3


I 3 , 2 3 1“
e

Ans. (0 CH3—CH—CH—CH2—CH2—CH3 (ii) CH3—C—CH2—CH—CH3


JO 30 30 2“ 2° 1" 20 30 JO
ur

CH3

ad
Yo

2. Give the lUPAC name of an isomer of hexane which has only two different sets of structurally
equivalent hydrogen atoms.

Ans. CH3—CH—CH—CH3 It has 12 equivalent 1° hydrogens and 2 equivalent 3° hydrogens.


d
Re

CH3 CH3
n

2, 3-Dimethylbutane
Fi

3. What is a Grignard reagent ? How is propane prepared from a Grignard reagent ?


Ans. Alkylmagnesium halides are called Grignard reagents. For example, ^-propyl bromide reacts with magnesium
to form n-propylmagnesium bromide. This upon treatment with water gives propane.
Dry ether
CH3CH2CH2Br + Mg -* CH3CH2CH2MgBr (Grignard reagent)
CH3CH2CH2MgBr + H2O > CH3CH2CH3 + Mg(OH)Br
4. w-Propylmagnesium bromide on hydrolysis gives propane. Is there another Grignard reagent which
also gives propane ? If so, give its name, structure and equation for the reaction.
Ans. Isopropylmagnesium bromide, (CH3)2CHMgBr,
(CH3)2CHMgBr + H2O > CH3CH2CH3 + Mg(OH)Br
HYDROCARBONS 13/125

5. How will you convert 2-bromopropane to 2-deuteriopropane ?


CH CH
Mg 3\ DjO
Ans. CH—Br CH—MgBr CH—D +Mg(OD)Br
CH3 dry ether CHj- CH3
2-Bromopropane Isopropylmag.bromide 2-Deuteriopropane

6. How many monobromo derivatives are obtained when 2-methylbutanc is subjected to


photobromination ?
Ans. The number of different monohalogen derivatives obtained from any alkane depends upon the number of
different types of hydrogens it contains. Now 2-methylbutane contains four different sets of equivalent

w
hydrogens (as shown by encircled integers) and hence gives four monobromo derivatives on photobromination.

F lo
CHi— CH— CH,—CH,
@ (D @
7. What does LPG stand for ? Name its important constituents.

ee
Ans. LPG stands for liquefied petroleum gas. It mainly contains butane and isobutane.

Fr
8. Arrange the following in increasing order of their release of energy on combustion.

for
(0 («●) (/V)
our
Ans. More the number of C-atoms having maximum hydrogens, i.e., CH3 groups, greater is the heat of combustion.
Thus, the increasing order of heat of combustion is (Hi) < (iv) < (/) < (//).
s
9. Define cracking ?
ook
Ans. The thermal decomposition of higher hydrocarbons into lower hydrocarbons in presence or absence of a
catalyst is called cracking. For example.
Y
773K
eB

C6H14 6-7 atm ^6^12 ^2 +


C^Hg + + CjHg + C2H4+CH4
Hexane
Hexene Hydrogen Butene Ethane Propene Ethene Methane

II. Alkenes
r
ou
ad

10. Which of the following shows geometrical isomerism ?


(i) CHCI = CHCl (H) CHj = CCl2 (Hi) CCI2 = CHCl
Give the structures of its cis-and /rans-forms.
Y

Ans. Only (0, i.e., CICH = CHCl has two different substituents on each carbon atom of the double bond and
hence shows geometrical isomerism.
Re
nd

●H H
c = c c = c
/Cl
Cl Cl
Fi

Cl H
cis-l, 2-DichIoroethene trans-l, 2-Dichlorocthene
The other two compounds, i.e., (ii) and (Hi) do not show geometrical isomerism because one of the carbon
atoms of the double bond in each case has two identical atoms, i.e.. Cl atoms.
11. Write the structural formulae of all the possible isomers of C2H2CI2 and indicate which of them is
non-polar ?
Ans (0 cis -1, 2-dichloroethene (H) trans-l, 2-dichloroethene and (Hi) 1. 1-dichloroethene. (ii) is non-polar
12. Use Markovnikov’s rule to predict the product of the following reactions :
(i) HCl with CH3CCI = CH2 and
(ii) HCl with CH3CH = C(CH3)2.
Ans. (0 CH3CCI = CH2 + HCl > CH3CCI2CH3
(//) CH3CH = C(CH3)2 + HCl > CH3CH2 - CCI(CH3)2
13/126 New Course Chemistry (XI) CE

13. Give the structure of the alkene (C4Hg) which adds on HBr in the presence and in the absence of
peroxide to give the same product, C4H9Br.
Ans. 2-Butene being symmetrical gives the same product i.e., 2-biomobutane.
HBr
CH3—CH = CH—CH3 orHBr/RCOOR » CH3—CH—CH2CH3
2-Butene I
Br

14. An alkyl halide, X, of formula C5H13Q on treatment with potassium tertiary butoxide gives two

w
isomeric alkenes Y and Z (CgH|2). Both alkenes on hydrogenation give 2,3-dimethylbutane. Predict
the structures of X, Y and Z.

H,C CH, H,C CH, H,C CH,


’1 1 ' (CH3)3C0-K+ , 3| 1 3 3| I 3

Flo
Ans. CH,—C—C—CH, » CH3—C = C—CH3 + CH2=C—CH—CH3
3 I I 3 -HCl

Cl H

e
2-Chloro-2, 2, 3-Dimethylbut-2-ene (Y) 2, 3-Dimethylbut-l-ene (Z)

re
3-dimethylbutane (X)
(Hydrogenation)

F
CH, CH,

r CH3—CH—CH—CH3

r
fo
2,3-Dimethylbutane
ou
15. A hydrocarbon A, adds one mole of hydn^en in presence of platinum catalyst to form n>hexane.
When A is oxidised vigorously with KMn04, a single carboxylic add containing three carbon atoms
ks
is isolated. Give the structure of A and expl^.
Ans. (0 Since, the hydrocarbon A adds one mole of H2 in presence of Pt to form /i-hexane, therefore, A must be
oo
an hexene,
Y
eB

(ii) Since A on vigorous oxidation with KMn04 gives a single carboxylic acid containing three carbon
atoms, therefore, A must be a symmetrical hexene, i.e„ hex-3-ene.
[O]
CH3CH2CH = CHCH2CH3 ^ 2CH3CH2COOH
ur

KMn04
Hex-3-ene (A) Propionic acid
ad

Thus, the given hydrocarbon A is hex-3-ene.


Yo

16. Write the structural isomers of an alkene having M.F. C4Hg. How can you distinguish between them ?
Ans. C4Hg has three structural isomers :
d

CH3\
Re

CH3CH2CH=CH2 CH3CH = CHCH3 ^C = CH2


in

1-Butene 2-Butene CH3'^


2-MethyIpropene
F

These can be easily distinguished on the basis of their reductive ozonolysis products because each of them
gives different products :

(/)03/CH2Cl2,l%K
CH3CH2CH i CH2 (ii)Zn/H20
^ CH3CH2CH = 0 + 0 = CH2
Piopanal Methanal
1-Butene*

(i)03/CH2a2.I96K
CH3CH ^ CHCH3 (/j)Zn/H20
> CH3CH = 0 + 0 = CHCH3
Ethanal Ethanal
2-Butene
HYDROCARBONS 13/127

CH CH
: (i)03/CH2Cl2,196K 3\
C:?CH2 (i7)Zii/H20
c = o +
0 = CH2
CH
3 CH3' Mcthanal

2-Mcthylpropene Propanone
17. Write the structure of the alkene which on reductive ozonolysis gives butanone and ethanal.

CH3 CH3
(i)03/CH2Cl2,196K
Ans.
CH3CH2-C=0 + 0 = CHCH3 (ii)Za^,0
CH3CH2-C 4:CHCH3
Butanone Ethane 3-Methyl-2-pentene

w
I 1. Alkynes
18. Out of ethylene and acetylene which is more reactive towards electrophilic addition reactions ?

F lo
Ans. Acetylene is less reactive than ethylene due to (i) more electronegative 5p-hybridized carbon atoms and
(«) more stable cylindrical nature of Ti-electrons. For details, refer to Art. 13.7.7, page 13/53.
19. Out of ethylene and acetylene which Is more reactive towards nucleophilic addition reactions and why ?

ee
Ans. Acetylene is more reactive than ethylene towards nucleophilic addition reactions due to greater

Fr
electronegativity of the .y/7-hybridized carbon.
20. Out of ethylene and acetylene which is more acidic and why ?
Ans. Acetylene* due to greater electronegativity of the ip-hybridized carbon.

for
21. Name two reagents which can be used to distinguish between ethene and ethyne.
our
Ans. Tollens’ reagent and ammoniacal CuCl solution. For reactions, refer to page 13/54.
22. How will you separate propene from propyne ?
s
Ans. By passing the mixture through ammoniacal AgN03 solution or ammoniacal CuCl solution when propyne
ook
reacts while propene passes over.
23. What is Lindlar’s catalyst ? What Is it used for ?
Y
eB

Ans. Pd deposited over CaC03 or BaS04 and partially poisoned by addition of lead acetate or sulphur or quinoline
is called Lindlar’s catalyst. It is used for partial reduction of alkynes to cis-alkenes.For example,
CH
(/) Pd/BaS04+S or quinoline 3\ CH3
r

c=c
CH3—CsC—CH3+H2
ou
ad

(Lindlar's catalyst) H H
But-2-yne
cw-But-2-cne

24. What is Birch reduction ? What is it used for ?


Y

Ans. Reduction of alkynes with sodium in liquid ammonia is called Birch reduction. It is used for partial reduction
of alkynes to rm«j-alkenes. For example.
Re
nd

CH H
(ONa/Liq.NH3.196-200K 3\
c = c
CH3—C=C—CH3 (ij)Ethanol H/"
Fi

Bul-2-yne CH3
jranj-But-2-ene

25. How will you detect the presence of unsaturation in an organic compound ?
Ans. Either by Baeyer’s reagent (i.e., cold dilute aqueous alkaline KMn04 solution) or by Br2 in CCI4. For
details, refer to pages 13/43 and 13/37 respectively.
26. How will you distinguish between but-l-yne and but-2-yne ?
Ans. But-l-yne is a terminal alkyne. Therefore, it gives white ppt. of silver but-l-ynide with Tollens’ reagent and
red ppt. of copper but-l-ynide with ammoniacal cuprous chloride.
CH3CH2CSCH + [Ag(NH3)2f OH" ^ CH3CH2C s C—Ag + 2 NH3 + H2O
Bul-l-yne Tollens’reagent Silver but-l-ynide
(while ppt.)
13/128 '4. New Course Chemistry (XI) LTAOHH

CH3CH2CSCH+ 2[Cu(NH3)2] OH" ^ CH3CH2C = C—Cu + 2NH3 + H2O


But-l-yne Diamminccoppcr(I) Copper but-l-ynide
hydroxide {red ppt.)

But-2-yne, on the other hand, being a non-terminal alkyne does not give these reactions.
IV. Arencs

ow
27. What is Hiickel rule ?
Ans. Huckel rule states that planar cyclic conjugated systems containing (4 n + 2) 71-electrons are aromatic in
character. Thus, phmar cyclic compounds containing 2, 6, 10, 14, 71-electrons are aromatic in nature.
28. Name two cationic aromatic .systcm.s.
Ads. Cyclopropenyl cation containing 2 ;t-electrons and cycloheptatrienyl cation containing 6 7t-electrons are

e
aromatic in character

re
+

Frl
F
Cyclopropenyl cation Cycloheptatrienyl cation
29. Name one anionic system which is aromatic in nature.
ou
or
Ans. Cyclopentadieny] anion contains 6 7t-electrons and hence is aromatic.

kfs
30. Is cycloheptatriene aromatic ?
Ans. No. Cycloheptatriene contains 6 7t-electrons but is not planar due to the presence of a -hybridized carbon.
oo
Y
Non-planar, not aromatic
B

Cycloheptatriene
31. What is the name given to cyclic conjugated systems containing 4 7t or 8 n-electrons ?
re

Ans. Cyclic conjugated systems containing 4 71 or 8 n-electrons are destabilized by resonance and hence are
called antiaromalic compounds. Examples are ;
oYu
ad


d

Cyclopropenyl anion Cyclopenladienyl cation Planar cyclooctatetraene


contains 4n-electrons, contains 47i-clectrons, contains 8n-electrons,
(unstable) {unstabte) (unstable)
in
Re

Please note that cyclooctatetraene become stable by adopting a tub shape which prevents resonance.
F

Cyclooctatetraene
{Tub shaped, stable)
32. Arrange the following in the increasing order of C—C bond length : C2Hg, C2H4, C2H2, €5!!^.
Ans. C2H2 (120 pm) < C2H4 (134 pm) < C^H^ (139 pm) < C2Hg (154 pm).
33. Although benzene is highly unsaturated, it does not undergo addition reactions.
Ans. Because of extra stability due to delocalization of 7t-electrons, the double bonds of benzene usually do not
undergo addition reactions.
34. How wilt you demonstrate that double bonds of benzene are somewhat different from those of olefines ?
Ans. The double bonds of olefines decolourize Bf2 in CCI4 and discharge the pink colour of Baeyer’s reagent
with simultaneous fonnation of a brown ppt. of Mn02 while double bonds of benzene do not.
HYDROCARBONS 13/129

35. Predict product formed when benzene is treated with iodine chloride in presence of anhydrous
aluminium chloride.
Ans. Since iodine is less electronegative than chlorine, therefore, AICI3 reacts with ICl to form iodonium ion (I"^)
as the electrophile which subsequently reacts with benzene to form iodobenzene.

I-^Cl + ^AlCl3 1* A1C14

C6H6 + I" > CfiHsI +

w
lodobiinzene

SHORT ANSWER QUESTIONS Carrying 2 or 3 marks

Flo
I. Alkanes

e
1. What are alkanes ? Why are they called paraffins ? [Art. 13.3]

re
2. Draw the structure of 2, 2. 4-trimethylhexane and indicate how many each of the 1°, 2°, 3" and 4° carbons
does it contain ? [Art. 13.3.3]

F
3. Comment upon three-dimensional structures of alkanes and their planar representations. [Art. 13.3.3]
4. Write the structural isomers of C5Hj2 and give their lUPAC names.
ur [Art. 13.3.3]

r
5. An alkane has a molecular mass of 72. Give all the possible structural isomers along with their lUPAC

fo
names. [Art. 13.3.3]
Hint. The general formula of alkanes is C„H2„ + 2

12 X « + 1 X (2n + 2) = 72 or 12n + 2n + 2 = 72 or n = 5 ks
Yo
Thus, the molecular formula of the alkane is C5HJ2. For structural isomers and their lUPAC names, refer to
oo
page 13/4,
6. How can alkanes be prepared from (i) an alkene, (ii) an alkyl halide, and (Hi) a carboxylic acid ? [Art. 13.3.4]
B

7. How can the following reactions be used to prepare alkanes. (1) Wurtz reaction, (//) Kolbe’s electrolytic
method, and (Hi) Sabatier and Senderen’s reaction ? [Art. 13.3.4]
re

8. Give two methods for the preparation of alkanes from carboxylic acids. [Art. 13.3.4]
9. Starting from ethanoic acid, how will you prepare (/) methane (j'O ethane ? [Art. 13.3.4]
u

10. What is Wurtz reaction ? How can it be used to prepare butane ? [Art. 13.3.4]
ad
Yo

11. Wurtz reaction is a good method for the preparation of alkanes containing even number of carbon atoms but
not for alkanes containing odd number of carbon atoms. Comment. [Art. 13.3.4]
12. Grignard reagent on hydrolysis gives a hydrocarbon. Give equation suggesting another method by which
the hydrocarbon obtained in the above reaction can be prepared. [Art. 13.3.4]
d
Re

13. What is Wurtz reaction ? What are its limitations ? How have these limitations been overcome ?
in

[Art. 13.3.4]
14. What is Corey-House reaction ? In what way, it is superior to Wurtz reaction ? [Art. 13.3.4]
F

15. Give reasons for the following : (/) The boiling points of alkanes decrease with branching.
(ii) The melting points of alkanes with odd number of carbon atoms are lower than those with even number
of carbon atoms.

(in) Straight chain alkanes possess higher boiling points than the corresponding branched chain isomers.
[Art. 13.3.5]
16. Discuss briefly the mechanism of halogenation of methane. [Art. 13.3.6]
17. Write short notes on : (/) Oxidation reactions of alkanes, (ii) Isomerization of alkanes. [Art. 13.3.6]
18. Why is rotation about carbon-carbon single bond not free ? What type of isomerism does it lead to ?
[Art. 13.5]
13/130 7h<xdet^'4, New Course Chemistry (XI)EEIHI]

19. Draw the Sawhorse and Newman projections for the staggered and eclipsed conformations of ethane. Discuss
their relative stability. Can these conformations be separated ? If not, then why ? [Art. 13.5.1]
20. Discuss the structure of double bond. [Art. 13.6.1]

II. Alkcnes

21. Discuss briefly the isomerism in alkenes. [Art. 13.6.3]


22. A hydrocarbon has the molecular mass 70. Write all the possible structural isomers and give their lUPAC
names. [Art 13.6.3]
[Ans. The hydrocarbon having molecular mass of 70 is C5HJQ. It exists in the following five structural

low
isomers : (0 CH3CH2CH2CH = CH2 (pent-l-ene), (ii) (CH3)2CH—CH = CH2 (3-methylbut-l-ene) ;
(Hi) CH3CH2CH(CH3) = CH2 (2-methylbut-l-ene), (iv) CH3CH2CH = CHCH3 (pent-2-ene) and
(v) (CH3)2C = CHCH3 (2-methyIbut-2-ene)]
23. Explain why rotation about carbon-carbon double bond is hindered ? [Art. 13.6J]
24. What is the origin of geometrical isomerism in alkenes ? [Art 13.63]
25. Which of the following alkenes will show geometrical isomerism ?

ee
(i) Propene (U) But-2-ene (Hi) But-l-ene (iv) 2, 3-Dimethylbut-2-ene.
[Ans. (H) But-2-ene]

rF [Art 13.6.3]

Fr
26. How can alkenes be prepared from (0 alcohol, and (H) an alkyl halide ? [Art 13.6.4]
27. How can ethene be prepared from (/) ethanol and (ii) ethyl iodide ? [Art. 13.6.4]
28. Give the mechanism of dehydration of alcohols. [Art 13.6.4]

for
29. Chloroethane is treated with alcoholic caustic potash ? Give another method by which the hydrocarbon
obtained in the above reaction be prepared. [Art. 13.6.4]
u
30. Explain the following with one example : (a) Dehydrohalogena tion (b) Dehydration. [Art. 13.6.4]
ks
31. Discuss briefly the mechanism of addition of Bt2 to alkenes. [Art 13.6.7]
Yo
32. How will you test the presence of double bond in an alkene ? Give chemical equations for the reactions
oo
involved. [Art. 13.6.8]
33. What is peroxide effect ? Why is it applicable only in case of HBr and not in case of HCl or HI ? [Art. 13.6.8]
eB

34. Give reasons for the following : (/) The melting point of a\s-but-2-ene is lower than that of rrfln5-but-2-ene.
(ii) The solubility of c/j-alkenes is higher than that of the corresponding rran^-alkenes in a particular solvent.
[Art. 13.6.7]
r

35. (a) What is ozonolysis ? How can ozonolysis be used to determine the position of a double bond in an
ou
ad

unknown alkene ? [Art. 13.6.8]


(b) Give the name of the alkene which on ozonolysis gives only propanone. [Art. 13.6.8]
Y

36. Write notes on : (j) Markovnikov’s rule (it) Polymerization of ethene and substituted ethenes. [Art. 13.6.8]

III. Alkynes
Re
nd

37. Discuss briefly structure of a triple bond. [Art. 13.7.1]


[Art. 13.7.3]
Fi

38. Comment upon isomerism in alkynes.


39. Alkynes contain two 7t-bonds but still do not show geometrical isomerism whereas alkenes contain only
one 71- bond but show geometrical isomerism. Comment. [Art. 13.7.3]
40. How is acetylene prepared by (/) Kolbe’s electrolysis (ii) Dehydrohalogenation of 1, 2-dihaloalkanes ?
[Art. 13.7.5]
41. What happens when calcium carbide is treated with water ? Give chemical equation for the reaction.
[Art 13.7.5]
42. Describe with a labelled diagram the laboratory method of preparation of acetylene. How can it be converted
into 1-butyne and 2-butyne ? [Art. 13.7.5]
43. Write the equation for preparation of propyne. [Art. 13.7.5]
13/131
HYDROCARBONS

44. Explain why are alkynes less reactive than alkenes towards electrophilic addition reactions ? [Art. 13.7.7]
45. Comment upon acidic character of terminal alkynes. [Art. 13.7.8]
46. Account for the order of acidity ; acetylene > benzene > hexane. [Art. 13.7.8]

47. Explain why alkynes undergo nucleophilic addition reactions while simple alkenes do not ? [Art. 13.7.8]

ow
48 Discuss the stereochemistry of the reduction products obtained when but-2-yne is reduced with (i) Lindlar’s
catalyst (ii) Na in liquid NH3. [Art. 13.7.8]
49. Sketch the mechanism of addition of water to alkynes. [Art .3.7.8]

50. Discuss the polymerization reactions of alkynes. [Art, 13.7.8]


51. Write short notes on : (i) Oxidation of alkynes (ii) Isomerization of alkynes. [Art. 13.7.8]

e
52. How will you carry out the following conversions ?

re
[Art. 13.7.5]
(i) Br2CH—CHBr2 —> HC s CH
[Art. 13.7.8]
(ii) H3CC = CH — ^ H3C—COCH3

Frl
F
IV. Arenes

53. Draw structures to show position isomerism in arenes. [Art. 13.8.2]

ou
54. Discuss the orbital structure of benzene. [Art 13.8.3]

sor
55. Benzene contains three double bonds yet it behaves like a saturated compound. Explain. [Art. 13.8.3]

56. Why is that the carbon-carbon bond distance in benzene is intermediate between carbon-carbon single
bond and a carbon-carbon double bond. [Art 13.8.3]

kf
57. Define substitution reactions. Why do arenes undergo substitution reactions even though they contain double
bonds ?
oo [Art 13.83]
58. What is meant by (/) delocalization (ii) resonance energy ? [Art. 13.8.4]
59. How is resonance energy of benzene calculated ? Explain. [Art. 13.8.5]
Y
60. Explain the term aromaticity ? How can Huckel rule be used to determine the aromaticity of a compound.
B

[Art 13.8.6]
61. What are the necessary conditions for any compound to show aromaticity ? [Art. 13.8.6]
re

62. What happens when benzene is treated with : (/) Br2 in presence of anhyd. AICI3 (ii) Cone. H2SO4 at
oY

330 K (Hi) Mixture of cone. H2SO4 and cone. HNO3 at 330 K (iv) Chloroethane and anhydrous AICI3
u

and (v) Ethanoyl chloride in presence of anhydrous AICI3. [Art 13.8.9]


ad

63. Discuss the mechanism of nuclear halogenation of arenes ? [Art. 13.8.11]


64. Discuss the role of catalyst in the electrophilic substitution reactions in benzene. [Art. 13.8.11]
d

65. Write down the products and give the mechanism of the following reactions.
in

(0 CgHgOH -I- H2SO4 (cone.)


[Art. 13.8.11]
Re

(li) C6H5CH3 + HNO3 and H2SO4 (cone.)


66. Discuss the directive influence of nitro group and its effect on the reactivity of the compound. [Art, 13.8.12]
F

67. How will you explain that methyl group is o, p- directing ? [Art. 13.8.12]
68. Predict the products of the following reactions :
(1) Aik. KMn04, A [Art. 13.8.9]
(i) Toluene
(ii)H'^/H20
Cone. HNO3
(ii) Nitrobenzene
+ Cone. H2SO4

Anhyd. AICI3
(Hi) Ethylbenzene + acetic anhydride ^. [Art. 13.8.12]

69. Comment upon the toxicity of polynuclear hydrocarbons. [Art. 13.8.13]


13/132
New Course Chemistry fXllinsTTn

LONG ANSWER QUESTIONS Carrying 5 or more marks

1. What are hydrocarbons ? How are they classified ? [Art. 13.1—13.2]


2. Discuss briefly the structure of alkanes. What are the various methods used for their preparation? [Art. 13.3.6]
3. Draw all the structural isomers with the molecular formula, C6H14. Name each one of them by the lUPAC
system. Also indicate primary, secondary, tertiary and quaternary carbons in each one of them.
[Art. 13.3.3—13.3.4]
4. Give the various methods used for the preparation of alkanes. Describe with labelled diagram the laboratory
preparation of methane from .sodium acetate and .soda-lime. [Art. 13J.7]
5. Discuss briefly the various physical properties and chemical reactions of alkanes. [Art. 13.3.8—13.3.9]
6. What do you understand by the term conformation ? Discuss briefly the various methods used for the
representation of conformations of an alkane ? [Art. 13.5]

w
7. What are alkenes ? Discuss briefly the various methods u.sed for the preparation of alkenes. Describe with
a labelled diagram the laboratory preparation of ethene from ethanol. [Art. 13.6 and 13.6.4]

F lo
8. Give an account of physical and chemical reactions of alkenes. [Art. 13.6.6 and 13.6.7]
9. What are alkynes ? How are they prepared in the laboratory ? [Art. 13.7 and 13.7.5]
10. Give an account of physical properties and chemical reactions of alkynes. [Art. 13.7.6—13.7.8]
11. What are arenes ? How are they classified ? Discuss briefly the isomerism and nomenclature of arenes.
[Art 13.8.1—13.8.2]

ree
12. Discuss the structure of benzene laying emphasis on resonance and orbital structure. [Art 13.8.3]

F
13. Justify the statement; Benzene is a highly unsaturated compound but behaves like a saturated compound.

for
[Art 13.8.3]
14. Discuss briefly the mechanism of electrophilic substitution reactions in benzene. [Art. 13.8.11]
15. Explain the directive influence of various substituents and their effect on reactivity of arenes. [Art 13.8.12]
r
You
CASE-BA$ED VERY SHORT/SHORT QUESTIONS
o ks

CASE 1. Alkenes contain a carbon-carbon double bond. These are usually prepared either by dehydration of
eBo

alcohols with cone. H2SO4 or by dehydrohalogenation of alkyl halides with alcoholic KOH. In both these reactions
wherever possible, Saytzejf rule is followed. If, however, a bulky base such as ferf-butoxide is used, instead of
Saylzeff, Hofmann product {i.e. less stable alkene) is obtained. Whereas dehydration of alcohols occurs through
intermediate formation of carbocaiions, dehydrohalogenation occurs by a concerted mechanism. In case the initially
ad
our

formed carbocation is less stable, it first undergoes rearrangement either by 1, 2-hydride or 1,2-alkyl migration, to
the more stable carbocation prior to proton loss to form the alkene. The most characteristic reactions of alkenes
are electrophilic addition reactions. During these addition reactions, an electrophile adds first to form a stable
carbocation which is subsequently attacked by a nucleophile to complete the addition reaction. In case of
unsymmetrical alkenes, addition occurs either in accordance or against Markovnikov’s rule.
Re

Based on the above paragraph, answer questions no.


Y

1 to 4 :
1. Give the product of dehydration of 1-butanol with cone. H2SO4.
d

2. Predict the product of dehydrohalogenation of I-chlorobutane


Fin

with alcoholic KOH ?


3. What happens when ?
(CH3)3C0"K+
CH,—CH—CH—CH,
M l ^
CH3 Br
Give reason for your answer.
4. Consider the addition of HCl to the following two alkenes :
(0 F3C—CH = CH2 («) CH3O—CH = CH2
Which of the two will give Markovnikov’s product and which will give anti-Markovnikov’s product ? Give
reasons.

I
HYDROCARBONS 13/133

CASE 2. Benzene is an ideal aromatic compound. Like alkencs and alkynes, it is an unsaturated compound.
But unlike alkenes and alkynes which undergo electrophilic addition reactions, benzene undergoes electrophilic
substitution reactions like halogenation, nitration, sulphonalion and Friedel-Cratts reactions. Benzene and its
derivatives usually do not undergo addition reactions. However, under suitable conditions, they do undergo
hydrogenation and halogenation. Besides these, benzene and their alkyl derivative undergo oxidation by alk.
KMn04 at 373-383 K, to give the corresponding carboxylic acids. With ozone, benzene and other arenes undergo
ozonolysis.
Based on the above paragraph, answer questions no. 5 to 8 :
5. Benzene on treatment with a mixture of cone. HNO3 and cone. H2SO4 gives nitrobenzene. Write the name
of the active species in this reaction and how is it formed ?
6. Write the product of the reaction of benzene with excess of chlorine in presence of heat and light and in
absence of halogen carrier. What would happen when the reaction of benzene with CI2 is carried out in

w
presence of anhydrous AICI3 ? Give reasons.
7. What are Friedel-Crafts reactions ? Illustrate their types with one example in each case.

F lo
8. Write down the products of ozonolysis of o-xylene. How does the result support Kekule structure of benzene ?
CASE 3. Alkynes contain a carbon-carbon triple bond. These are obtained by a number of methods but one
of the most important methods is dehydrohalogenation of 1, l-and 1,2-dihaloalkanes with alcoholic KOH or
preferably NaNH2 in liquid NH3. The simplest alkyne, i.e., acetylene is conveniently and economically prepared

e
Fre
by action of H2O on calcium carbide. Since due to jp-hybridizati on of acetylenic carbons, C-H bonds of acetylene
are acidic in nature, therefore, acetylene reacts with NaNH2 in liq. NH3 to give sodium acetylide. The reaction of

for
sodium acetylide with a variety of alkyl halides gives us a convenient method for synthesis of higher alkynes.
Like alkenes, alkynes also undergo electrophilic additions although alkenes are more reactive than alkynes
in these reactions. Addition of water to acetylene in presence of dil. H2SO4 and HgS04 as catalyst gives acetaldehyde
our
but with terminal alkynes, it gives methyl ketones.
Depending upon the nature of reducing agent, reduction of non-terminal alkynes can be stopped at the
oks

double bond stage giving c/r-and /rans-alkenes.


eBo

Based on the above paragraph, answer questions no. 9 to 12 :


Y

9. Explain why ethyne is more acidic that ethene. Give one example to support the acidic nature of ethyne.
10. Write the products of addition of excess of HBr to propyne in presence and absence of peroxides.
11. Predict the product of addition of H2O to C5H5C = C-CH3 in presence of dil H2SO4 and catalyst HgS04.
ad
our

12. How will you convert 2-butyne to cis- and trans 2-butene. Give the names of the reagents and their mode of
action.

ANSWERS
Re
Y

1. A mixture of but-2-ene (80%) and but-l-ene (20%) is formed as shown below ;


d
Fin

+ 0"^ Slow

CH3CH2CH2CH2—OH ^ -H+ i CH3CH2CH2CH2—OH2 -HjO


^ CH3CH2 — CH—CH2
1-Butanol
M

r carbocation
(less slahle)

2 2 I
1,2- (Fast)

hydrideshift
CH3CH2—CH—CH3 SaytzelT rule
> CH3CH=CHCH3 -h CH3CH2CH = CH2
2°carbocaiion 8ut-2-cne(80%) But-l-enc (20%)
{inaresuihle)

\
13/134
7\eieCeefr'4, New Course Chemistry (XI)Q

2. 1-Chlorobutane on treatment with ale. KOH undergoes concerted ^-elimination to give 1-butene as the
major product
^C1

CH3CH2—CH-^CH2 + KOH (ale.) CH3CH2CH=CHj + KCl + H2O


1-Butene
H

3.C“3 CH — CHj
CH3 :CH—CH=CH2
J -HBr CHj
4. 3-Metbylbut-l-ene
{Hojmann product)

Because of the bigger size of the base, i.e., (CH3)3CO"K+ and the steric hindrance due to the presence

w
of two CH3 groups at ^-carbon, it does not undergo Saytzeff elimination but instead undergoes Hofinann
elimination to give 3-methylbut-l-ene as the major product.

F lo
4. Due to strong electron-withdrawing effect of F3C-group, addition of H+ occurs at C2 and nucleophilic
attack by Cl“ at C| giving anti-Markovnikov's product (I).
H+ 2 + a-
F3C—CH=CH2 » FjC-e-CH—CHj ■> F3C—CH2—CHjCld)

e
{Anti-MaHcovnikov’s product)
H

Fre
In contrast, due to the strong electron-donating effect of the OCH3 group, addition of H+ occurs at Cj
giving more stable carbocation (II). This is followed by nucleophilicattack by Cl“ to give Maiicovnikov’s
product(m)

for
2 1 ^ ,ri.
CH3O—CH=CH2 +H+ > CH3O CH—CH3 4-4 CH30=CHCH3
our
cr
oks

CH3O—CH—CH3
eBo
Y
Cl
m
(Markovndcov's product)
our
ad

5. The active species involved in nitration of benzene is nitronium ion, NOJ which is formed by the action
of H2SO4 on HNO3 as shown below :
Ionization
H2S04 2H+ + SQ2-
dY
Re

+ +
4
HO—N=0 ^ 0=N=0
-H+ -H2O Nitroniumion
Fin

O'”
O"
Nitric add
{actsasabase)

Av.SOOK
hV'h
4. I + 3Cl2 ► Cl n rH
Cl
Absence of
H
halogen carrier
Benzene {Addition reaction) Cl Cl
Cl H
Benzene hexachloride
(BHC)

A if H
HYDROCARBONS 13/135

In presence of halogen carrier, i.e., AICI3, electrophilic substitution takes place.


Cl

Anhyd. AICI3
+ CI2
(Electrophilic
I + HCl
substitution)
Benzene Chlorobenzene

7. Friedel-Ctafls reaction involves the introduction of an alkyl (R) or an acyl group into benzene irng. It is of

w
two types : (0 F.C. alkylation and («) F.C. acylation.
For details, erfer to page 13/77 for F.C. alkylation and 13/78 for F.C. acylation.
8. Refer to Ans. to Q. 13.17, page 13/148.

Flo
9. In ethyne,. C is ^hybridized while in ethene, it is jp^-hybridized.
sp ' sp^

e
HCsCH H,C = CH,
^Ethene

re
Ethyne

Since a sp-hybridized carbon is more electronegative than a sp^-hybridized carbon, therefore, electrons of

F
C—H bond of ethyne are displaced more towards the carbon than that in ethene. In other words, hydrogen
atom in ethyne is less tightly held by carbon than that in ethene and hence can be ermoved as (H"*") by a
ur
strong base. In other words, ethyne is more acidic than ethene. This is supposed by the reaction of ethyne

r
fo
with NaNH2 which evolves NH3.
HCsCH + NaNHj ^ HCsC"Na+ + NH3
(Stronger acid)
HjCsCHj +NaNH2 ^
ks
No reaction.
(Weaker acid)
Yo
oo
Br

CHj-i-CHj
Maik HBr
B

10. CHj-CsCH +HBr additton 4 CH3—C = CH2 I


Propyne Br Br
re

2.2-DitHomq>ropane

In presence of peroxides, anti-Markownikov's additions occurs.


u
ad

RCOOR HBr
Yo

CH3-CsCH+HBr 4 CH3-CH = CHBr RCCX)R CH3^H2-CHBr2


Propyne l.l-Dibromopropane

11. Refer to Ans. to Q. 19, page 13/120.


d

12. Hydrogenation of 2-butyne in presence of Undlar*s catalyst (H2/Pd-BaS04 + S or quinoline) gives cis-2-
Re

butene but Birch reduction (Na, liq. NH3, 196 K) gives rranj-2-butene.
in

CH3 CH3
F

c=c
H H
cu-2-Butene

CH3—C=C—CH3
2-Butyne

CH3 H
C=C
H
<rans-2-Butene
CH3
13/136
New Course Chemistry fXT1t*75WH

CASE-BASED MCQs AND ASSERTION-REASON QUESTIONS

CASE 1. Although rotation about a carbon-carbon single bond is almost free, that about a carbon-carbon
double bond is highly restricted. The reason being that the TC-bond is formed by sideways overlapping of
unhybridized /7-orbitals of the two carbon atoms. If one of the carbon atoms of the double bond is rotated w..rt. the
other, the /7-orbitals will no longer overlap and the n-bond will break. But the breaking of a 7t-bond requires 251
kJ mor' of energy which cannot be provided by collisions of the molecules at room temperature and hence
rotation about carbon-carbon double bond is highly restricted.
As a result of this restricted rotation, many substituted alkenes exist in two distinct isomers which differ
from each other only in the relative positions of atoms or groups in space around the double bond. These isomers
are called cis- and tm/i.v-isomers or simply geometrical isomers.

w
The geometrical isomers in which the two similar atoms or groups lie on the same side of the double bond
are called cis-isomers and those in which the two similar atoms or groups lie on the opposite sides of the double
bond are called trams-isomers. The cis- and trans-isomers of an alkene have different physical properties such as

F lo
melting points, boiling points, dipole moments, solubility, etc. and some chemical properties as well.
Based on the above paragraph, answer questions no. 1 to 4 :
1. Which one of the following molecule, has a finite dipole moment ?
(<2) rra«5-2-butene (b) trans-l, 2-dichloroethene (c) rran.9-2-pentene (d) trans-3-hexene

ee
2. The geometrical isomerism is shown by

Fr
.CH2 .CH2
(«) (b)
^^CUCl ^-s^CHCl
(c)

for
Choose the correct option out of the four options given below :
our
(a)Both Assertion (A) and Reason (R) are true and Reason (R) is the correct explanation of Assertion (A).
(h)Both Assertion (A) and Reason (R) are true but reason (R) is not the correct explanation of Assertion (A),
s
ook
(c) Assertion (A) is tme but Reason (R) is false.
(d) Assertion (A) is false but Reason (R) is true.
Y
3. Assertion. The boiling point of trans 2-butene is higher than that of the c/^-isomer.
eB

Reason. ctj'-2-Buiene has higher dipole moment than /ran5-2-butene.


4. Assertion. The melting point of fumaric acid is higher than that of maleic acid.
Reason. The molecules of maleic acid being more symmetrical are strongly held in the crystal lattice than
our
ad

those of fumaric acid.

CASE 2. Benzene (C^H^) is an ideal aromatic compound. Like benzene, there are a large number of other
compounds (neutral, cationic or anionic species) which are aromatic in character. A common feature of aromatic
compounds is that they obey Hiickel rule. The most characteristic reactions of aromatic compounds are electrophilic
Y

substitution reactions such as halogenation, nitration, sulphonationand Friedel- Crafts reactions.


Re

Based on the above paragraph, answer questions no. 5 to 8 :


d

5. Which of the following stability order of the pair of compounds is incorrect ?


Fin

{a) <
(b) + >

(c) >
id) >

6. Which of the following statements about electrophilic substitution reactions is not correct ?
(a) Hexadeuteronitrobenzenc and benzene undergo nitration at the same rate.
(b) Sulphonation of benzene is a reversible reaction,
(c) Chlorination of nitrobenzene and nitration of chlorobenzene give the same product.
(d) Nitrobenzene does not undergo Friedel-Crafts reaction.
13/137
HYDROCARBONS

Choose the correct option out of the four options given below :
(a) Both Assertion (A) and Reason (R) are true and Reason (R) is the correct explanation of Assertion (A).
(b) Both Assertion (A) and Reason (R) are true but reason (R) is not the correct explanation of Assertion (A),
(c) Assertion (A) is true but Reason (R) is false.
(d) Assertion (A) is false but Reason (R) is true.
7. Assertion. Friedel-Crafis reaction is used to introduce an alkyl or acyl group in benzene nucleus.
Reason. Benzene is a solvent for the Friedel-Crafts alkylation of bromobenzene.

8. Assertion. CH2 is an aromatic compound.

w
Reason. It contains a conjugated system of three double bonds.

ANSWERS

F lo
l.(c) 2.(J) 3.{d) 4. (c) 5.(d) 6. (c) 7. (c) 8. (d)

HINTS/EXPLANATIONS For Difficult Questions

ee
Fr
CH H
1.
'X

for
H CH2CH3
Although these two dipoles opposite each other, they do not exactly cancel out and hence /rrtn^-2-pentene
ur
has a small but finite dipole moment.
2. Only option (d) has different substituents on each carbon of the double bond and hence it shows geometrical
s
ook
isomerism.
Yo
3. Correct A. The boiling point of cij-2-butene is higher than that of rra«.r-2-butene.
4. Correct R. The molecules of maleic acid being less symmetrical are only weakly held in the crystal lattice
eB

than those of more symmetrical molecules of fumaric acid,

5. (a) being antiaromatic is less stable that


our
ad

(b) + being aromatic is more stable than


Y

(c) - being aromatic is more stable than


Re
nd

(d) + being antiaromatic is less stable than


Fi

6. Since NO2 is w-directing, therefore, chlorination of nitrobenzene gives m-chloronitrobenzene. In contrast.


Cl being o, p-directing, therefore, nitration of chlorobenzene gives a mixture of o- and p-nitrochlorobenzene.
Remaining all statements are correct.
7. Correct reason. Since benzene is more reactive than bromobenzene, therefore, F.C. reaction will occur
preferentially in benzene rather than in bromobenzene and hence benzene cannot be used as a solvent in this
reaction.

8. Correct A. CH2 is not an aromatic compound since the three conjugated double bonds are not in
the ring.
13/138
New Course Chemistry (XI)EZ5ia]]

%
V WITH
■' i;'. “I-
I ANSWERS
t
I 0

low
NCERT INTEXT SOLVED QUESTIONS

Q. 13.1. Write structures of different chain isomers of alkanes corresponding to the molecular formula
bold CfiHj4. Also write their ITJPAC names.
Ans. Refer to Art. 13.3.2, page 13/5.

ee
Q. 13.2. Write structures of different isomeric alkyl groups corresponding to the molecular formula C5H11.

rF
Write lUPACnames of alcohols obtained by attachment of-OHgroup at different carbons of the chain.

Fr
Ans. Structures of CgHj| alkyl group Structures and lUPAC names of the corresponding alcohols
5 4 3 2 1
(,) CH3—CH2—CH2—CH2—CH2 — CH3 —CH2 —CH2—CH2 —CH2 —OH

for
1-Pentyl Pentan-l-ol
1 2 I 2 3 4 5
(«) CH3—CH—CH2—CH2 CH3 CH3—CH—CH2—CH2—CH3
ou
ks
OH
2-Pemyl Pentan-2-ol
oo
1 2 3 1 2 3 4 5

(Hi) CH3—CH2—CH—CH2—CH3 CH3—CH2—CH—CH^—CH3


Y
eB

OH
3-Pentyl Pentan-3-ol

4 3 2 1 4 3 2 1
r

(iv) CH3—CH—CHj—CH2 — CH3 —CH—CH2 —CH2 —OH


ou
ad

I I
CH3 CH3
3-Methyl-l-butyl 3-Methylbutan-1 -ol
Y

2 I 4 3 2 I

(v) CH3—CH2—CH—CH2 — CH3 —CH2 —CH—CH2 —OH


I
Re
nd

CH3 CH3
2-Methyl-1-butyl 2-Methylbutan-1 -ol
Fi

CH3 CH3
1 2I 3 4 I 2I 3 4
{vO CH3—C—CH2—CH3 CH3—C—CH2—CH3
I
OH
2-MethyI-2-buiyl 2-Methylbutan-2-ol
CH, CH,
3 21 1
3 2I
(vii) CH3—C— CH2— CH3—C—CH2OH
CH3 CH3
2, 2-Dimethyl-1-propyl 2, 2-Dimethylpropan-l-ol
HYDROCARBONS 13/139

Q. 133. Write lUPAC names of the following compounds


(i) (CH3)3CCH2C(CH3 )3 (ii) (CH3)2C(C2Hg)2 (in) Tetra-tert-butylmethane.
Ans. The expanded structures of these alkanes are :
2 1
CH, CH.
CH2CH3
1 2 I ^3 4 I ^ 3I 4 5
(0 CH3—C—CH2—C—CH3 (/●/■) CH3—C—CH2CH3
I
CH3 CH3 CH3
2, 2,4,4-Tetramethylpentane 3, 3-Dimethylpentane

CH3

w
1
HX
31
CH,—C—CH,
3 I 3
CH,I 3
1 I 2 3I 4 I 5
(iii) CH,—C- C
-C—CH3

F lo
^ I 1
H3C CH3—C—CH3 CH3
CH3

ee
3, 3-Di-/ert-butyl-2, 2,4,4- teu-amelhylpentane

Fr
or 3, 3-Bis (1,1- dimethylethyl)-2, 2,4,4-tetramethylpentane
Q. 13.4. Write structural formulas of the following compounds,
(i) 3, 4,4,5-Tetramethylheptane

for
(ii) 2, 5-Dimethylhexane.
our
CH3
1 l4^ 5 6
2 7 3

Ans. (i) CH3—CHj—CH—CI CH—CH2—CH3


s
I
ook
H3C CH3 CH3
3, 4, 4, 5-Tetramethylheptane
Y
eB

CH3 CH,
/ ● ●X 1 2 I 3
4 5 I ^ 6
ill) CH3—CH—CH2—CH2—CH—CH3
2, 5-Dimethylhexane
r

Q. 13.5. Write structures for each of the following compounds. Why are the given names incorrect ?
ad
ou

Write correct lUPAC names ;


(i) 2-Ethylpentane
(ii) 5-Ethyl>3>methylheptane.
Y

3 4 5 6

Ans. (i) CH3—CH—CH2CH2CH3 The longest chain here contains six carbon atoms and not five.
Re
nd

2I
Therefore, the correct name is 3-methylhexane.
CH2CH3
Fi

1 6 5 4 3 2 1
Since, ‘e’ comes before ‘m’ in alphabetical
(ii) CH3—CH2—CH—CH2—CH—CH2—CH3 order, therefore, numbering must be done
I from right to left. Therefore, the correct name
CH3 CH2—CH3 is 3-ethyl-5-methylheptane.

Q. 13.6. Sodium salt of which acid will be needed for preparation of propane ? Write chemical equation
for the reaction.

Ans. Since one carbon atom is lost as CO2 during decarboxylation , therefore, the acid needed must contain
one carbon atom more than propane, i.e., butanoic acid or 2-methylpropanoic acid
13/140 New Course Chemistry (XI)EZsl9D
CaO

CH3CH2CH2[c001^
Sodium butanoate
630 K
> CH3CH2CH3 + Na2C03
Propane
CaO
CH-CH [cOONa + NaO^H ^ CH3CH2CH3 + Na2C03
3| I J 630 K
Propane
CH3
Sodium 2-methylpropanoate
Q. 13.7. Write the lUPAC names of the following compounds,

low
(i) (CH3)2CH—CH = CH—CHj—CH = CH—CHCH3 (H)
C2H5
(iii) CH2 = C(CH2CH2CH3)2 (iv) CH3 CHj CH^ CH2 CHXH,
I ^ ^
CH3—CH—CH = C—CHj—CH—CH3
I

e
CH3

re
Ans. (i) The expanded formula is

rF
F
1 2 3 4 5 6 7 8

CH3—CH—CH = CH—CH,—CH = CH—CH—CH3


I' 9I 10

r
CH3 CH2CH3

fo
u 2, 8-Dimethyldeca-3, 6-diene
a-bonds = 33, Jt-bonds = 2
ks
8
Yo
(«) '/\/\/\/
a-bonds= 17
oo
ji-bonds = 4
3 5 7
Octa-l, 3,5,7-tetraene
B

3 4 5

CH2CH2CH3
re

1 2/ 10 9 8 7

m ch2=c:;^ O’v) CH3CH2CH2CH2


6 I 4I
CH2CH3
3
CH2CH2CH3 5 2 1
u
ad

CH3—CH—CH = C—CH2 CH—CH3


Yo

2-Propylpent-1 -ene
a-bonds = 23,7C-bond = 1 CH3
4-Ethyl- 2, 6-dimethyldec-4-ene
a-bonds = 41, tc-bond = 1
d
Re

Q. 13.8. Calculate the number of sigma bonds in the above structures (i> iv).
in

Ans. As shown in Ans. to Q. 13.7 above.


Q. 13.9. Write the structures and lUPAC names of different structural isomers of alkenes corresponding
F

to C5H10.
Ans. The various shnctural isomers and their lUPAC names are given below :
1

(0 CH2= CH—CH2—CH2—CH3 («) CH3—CH = CH—CH2—CH3


Pent-1- ene Pent-2- ene

CH, CH, CH,


..... 1- 2I 3 4 .. . 1 -
2 3I 4 1 2I 3 4

(»0 CH2= C—CH2—CH3 Ov) CH2=CH—CH—CH3 (v) CH3— C = CH—CH3


2-Methylbut-l-ene 3-Methylbut-1 -ene 2-Methylbut-2-ene
HYDROCARBONS 13/141

Q. 13.10. Draw the structures of cis-and trans-isomers of the following compounds. Also write their lUPAC
names, (i) CHCl = CHCl (U) CjHgaCHj) = C(CH3)C2Hs
H H Cl
Ans. (0 C = C c = c
Cl Cl Cl^
CIS-1, 2-DichIoroethene trans-l, 2-Dichloroethene

CH
3 CH3 CH3 5 6

\3 4/
^C = C
\3 4 .CH.CH,
(«) ^C = C^

w
1
2 / \5 6 1 2 ./ \
CH3CH2 CH2CH3 CH3CH2 CH3
cis-3, 4-Dimethylhex-3-ene irans-3, 4-Dimethylhex-3-ene

F lo
Q. 13.11. Which of the following compounds will show cis- trans isomerism ?
(i) (H3O2C = CH—C2H5 (ii) H2C = CBr2 (iii) CgHgCH = CHCH3 (iv) H3C—CH = CCKCH3)
Ans. Alkenes (i) and (i7) have identical atoms or groups on one of the carbon atoms of the double bond and

ee
hence do not show geometrical isomerism. In contrast, alkenes (iii) and (iv) have different atoms or
groups on each carbon atom of the double bond and hence exhibit geometrical isomerism.

Fr
Q. 13.12. Write HJPAC names of the products obtained by addition reactions of HBr to hex-l-ene
(i) in presence of peroxide and (ii) in the absence of peroxide.
Ans. (0 In presence of peroxides, anti-Markovnikov’s addition occurs and 1-bromohexane is formed.

for
ur ^ CH.—CH,—CH,—CH,—CH,—CH,
CH2 = CH—CH2—CH2—CH2—CH3 -I- HBr (i4/ii/-Mark.
Peroxide

I ^ ^ ^ ^ ^ J
Hex-l-ene
addn.) gj. 1-Bromohexane
ks
(ii) In absence of peroxides, Markovnikov’s addition occurs and 2-bromohexane is formed.
Yo
No peroxide
oo
CHj = CH—CH2—CHj—CH2—CH3 + HBr (Mark.addn.) ^ CH3 CH—CH2 —CH2 —CH2 CH3
I
Hex-l-ene 2-Bromohexane
eB

Br
Q. 13.13. Write structures of different isomers corresponding to the 5th member of alkyne series. Also
write HJPAC names of all the isomers. What type of isomerism is exhibited by different pairs of
isomers ?
r

Ans. The molecular formula of the 5th member of the alkyne series is CgHjo- The possible isomers and their
ou
ad

lUPAC names are:

1 2 3 4 5 6 1 2 3 4 5 6
Y

(i) HC = C—CH2—CH2—CH2—CH3 (ii) CH3—CsC—CH2—CH2—CH3


Hex-l-yne Hex-2-yne
nd
Re

1 2 3 4 5 6 4 5

(iii) CH3—CH2—CsC—CH2CH3 (iv) HCsC—CH—CH2—CH3


Fi

Hex-3-yne
CH3
3-Methylpent-1 -yne

CH3
1 2 3 4 5 1 2 3 4 5 1 2 3I 4
(v) HC = C—CH,—CH- CH3 (vi) CH3—C = C—CH— CH,5 (vi7) HC = C—C—CH,
I 3
^ I
CHj CH3 CH3
4-MethyIpent-1 -yne 4-Methylpent-2-yne 3, 3-Dimethylbut-l-yne
Position isomers, (i) and (ii), (i) and (iii), (ii) and (iii), (v) and (v/)-
13/142 ^'utetccfr'4. New Course Chemistry (XI)CSISD
Chain isomers. (/') and (/v), (») and (v), (0 and (vO, (0 and (vU), (U) and (iv), (ii) and (v), (//) and (vi), (ii)
and (vii), (Hi) and {iv), (Hi) and (v), (Hi) and (vi), (Hi) and (w7). (iv) and (v), (v) and (vii).
Q. 13.14. How will you convert ethanoic acid into benzene ?
NaOH(fl<y) Soda-lime C\2,hv
Ans. CH3COOH -HtO
^ CH^COONa 630K
^ CH4 CH3CI
(Chlorination)
Ethanoic acid Sod. ethanoate Methane Chloromethane

Na, dry ether CI2./1V KOH (ale.) Br,,CCl4


(Wtinz reaction)
> CH3CH3 4 CH3CH2CI ●) CH2 = CH2
(Chlorination) A

w
Ethane Chloroethane Ethene

KOH (ale.) NaNHj Red hot Fe tube


BrCHj—CH2Br A ^ CHjsCHBr HCsCH
873 K
inliq.NHj
I, 2- Dibromoethane Bromoethene Ethyne Benzene

Flo
Alternatively, convert ethanoic acid to ethene as follows and then convert ethene to benzene as described
above

e
Li AlH^,ether conc.H2S04

re
CH,COOH ^ CH3CH2OH ^ CH2=CH2
433-443 K
Ethanoic acid Ethanol (Dehydration) Ethene

rF
NCERT EXERCISE
ur
Q. 13.1. How do you account for formation of ethane during chlorination of methane ?

fo
Ans. Chlorination of methane is a free radical reaction which occurs by the following mechanism :

Initiation: Cl-^Cl Homolytic fission. ^ 2 Cl


ks
Yo
Chlorine free radicals
oo
Propagation : CH3~^H + Cl > CH3 + HCl ; CH3 + ci-J-ci CH3—Cl + ‘Cl
B

Termination : (/) CH3 + -CH3 ^ CH3—CH3


re

Ethane

(ii) CH3 + Cl > CH3—Cl (Hi) Cl + Cl > Cl—Cl


u
ad

Methyl chloride
Yo

From the above mechanism, it is evident that during propagation step, CH^ ft’ee radicals are produced
which may undergo three reactions, i.e. (/)—(///). In the chain termination step, the two CH^ free
d

radicals combine together to form ethane (CH3—CH3) molecule.


Re

Q. 13.2. Write lUPAC names of the following compounds :


in

(a) CH3CH = C(CH3)2 (b) CHj = CH—C s C—CH3


F

v\ (d) CHj—CHj—CH=CH2

CH3

(e) OH (0 CH3(CH2)4CH(CH2)3CH3
CH2—CH(CH3>2
(g) CH3—CH=CH—CHj CH = CH—CH—CHj—CH = CH2
I
C2H5
HYDROCARBONS 13/143

CH,
4 3 2I 1 1 2 3 4 5

Ans. (a) CH3—CH=C—CH3 (b) CH2=CH—C = C—CH3


2-Methylbut-2-ene Pen-l-en-3-yne

(d)
^ ' 4-PhenyIbut-l-ene

ow
Buta-1,3-diene

10 6-9 5 2-4 1
(e) OH
(/) CH3—(CH2)4—CH—(012)3—CH3
2-Methylphenol
1CH2-CH-CH3

e
CH3

Fl
re
5-(2-Methylpropyl)decane
10 6 5 4 3 2 1

F
(g) CH3—CH = CH—CH2—CH = CH—CH—CH2—CH = CH2
ur C2H5

r
4-Ethyldeca-l, 5, 8-triene

fo
Q. 13.3. For the following compounds, write structural formulas and lUPAC names for all possible isomers
having the number of double or triple bond as indicated :
(a) C4H8 (one double bond) (b) CsHg (one triple bond) ks
Yo
Ans. (a) Isomers of C4H8 having one double bond are :
oo
4 1
H
2^CH2 CH3
(m) c = c
(0 CH3CH2—CH = CH2 (»*) c=c
B

H CH3
But-l-ene cw-But-2-ene fra/ij-But-2-ene
re

CH
p-3
(fv) CHj C = CH2
u
ad

2-Methy Iprop-1 -ene CH,


Yo

2 1 3I 2 1

(.b) (1) CH3CH2CH2 C = CH (»0 CH3CH2 —C s C—CH3 (Hi) CH3—CH—C = CH


Pent-l-yne Pent-2-yne 3-Methylbut-l-yne
d

Q. 13.4. Write lUPAC names of the products obtained by the ozonolysis of the following compounds :
Re
in

(i) Pent-2-ene (ii) 3, 4-Dbnethylhept-3-ene (iii) 2-Ethylbut-l-ene (iv) 1-Phenylbut-l-ene.


5 4 3 2 1 (i)03/CH2Cl2.196K
F

Ans. (/) CH3—CH2—CH = CH—CH3 ^ CH3—CH2—CH = 0 + 0 = CH—CH3


(ii) Zn/H20 Ethanal
Pent-2-ene Propanal

7 6 5 4 3 2 1 (i)03/CH2Cl2.195K
(«) CH3CH2CH2—C = C—CH2CH3 (u) Znm20
+

H3C CH3
3, 4-Dimethylhept-3-ene
CH3CH2CH2—c = 0 + 0= C—CH2H3
CH3 CH3
Pentan-2-one Butan-2-one
13/144
7^n<zdee^'<i- New Course Chemistry (Xl)EZi

4 3 2 I (i) O3/CH2CI2,196 K
(m) CH3CH2—C = CH2 (ii) Z11/H2O
■>
C = 0 + 0 = CH2
CH3H2 Methanal
CH2CH3 Pentan-3-one
2-Ethylbut-l-ene
4 3 2 1
(i)03.CH2Cl2.196K
(/V) CH3 CH2 —CH = CH—CgHj > CH3CH2CH = 0 + 0 = CH—CgHs
(ii) 211^20

ow
1-Phenylbut-l-ene Propanal Benzaldehyde
Q. 13.5. An alkene ‘A’ on ozonolysis gives a mixture of ethanal and pentan-3-one. Write the structure and
lUPAC name of ‘A’.
Ans. Write the structures of the products of ozonolysis side by side with their oxygen atoms pointing towards
each other. Remove the oxygen atoms and join the two ends by a double bond, the structure of the
alkene ‘A’ is

e
5 4

Fl
CH3CH2^ CH3CH2'.^3 2 1

re
CHjCh/ c =[b (^= CHCH3 ^C=CH—CH3
«■

Ethanal

F
Pentan - 3 - one CH3CH2
ur 3-Ethylpent-2-ene (A)
Q. 13.6. An alkene ‘A’ contains three C—C, eight C—H, G-bonds, and one C—C n-bond. ‘A* on ozonolysis

r
gives two moles of an aldehyde of molar mass 44 u write the lUPAC name of ‘A*.

fo
Ans. (/) An aldehyde with molar mass of 44 u is ethanal, CH3CH = O
(«) Write two moles of ethanal side by side with their oxygen atoms pointing towards each other.
ks
Remove the oxygen atoms and join them by a double bond, the structure of alkene ‘A’ is
Yo
CH3CH =[o o]= CHCH3 «■
CH3-CH = CH-^H3
oo
Ethanal Ethanal But-2-ene

As required, but-2-ene has three C—C, eight C—H a-bonds and one C—C ic-bond.
B

Q. 13.7. Propanal and pentan-3-one are the ozonolysis products of an alkene ? What is the structural
formula of the alkene ?
re

Ans. (i) Write the structures of propanal and pentan-3-one with their oxygen atoms facing each other. Remove
oxygen atoms and join the two fragments by a double bond, the structure of the alkene is
u

2 1
ad

^CH2CH3 6 5 4 3XCH2CH3
CH3CH2CH =[o o]= C
Yo

i-
CH3CH2CH = C
Propanal CH2CH3 \
Pentan-3-one CH2CH3
d

3-Ethylhex-3-ene
Re

Q. 13.8. Write chemical equations for the combustion reaction of the following hydrocarbons,
in

(i) Butane
(ii) Pentene
F

(iii) Hexyne
(iv) Toluene

Ans. (0 C4Hio(g) + 13/2 02(g) 4 CO2 (g) + 5 H2O (g)


Butane

(«) CsHjo (g) + 15/2 O2 (g) 5 CO2 (g) + 5 H2O (g)


Pentene

(m) CgHio (g) + 17/2 O2 (g) 6 CO2 (g) + 5 H2O (g)


Hexyne
HYDROCARBONS 13/145

(/v) CH3(g) or C7HS (g) + 9 O, (g) ^ 7 CO, (g) + 4 H2O (g)


Toluene

Q. 13.9. Draw the cis-and trans-structures for hex-2-ene. Which isomer will have higher b.p. and why ?
Ans. The structures of c/j-and rra/u-isomer of hex-2-ene are :

CH 3 CH2CH2CH3 CH3 H

w
c=c c=c
H H H CH2CH2CH3
c/s-Hex-2-ene frans-Hex-2-ene
(Higher dipole moment, higher h.p.) (Lower dipole me,vent, lover h.p.)
The boiling point of a molecule depends upon dipole-dipole interactions. Since cis-isomer has higher

o
dipole momeni. therefore, it has higher boiling point.
Q. 13.10. Why is benzene extraordinarily stable though it contains three double bunds ?

e
Ans. Resonance or delocalization of electrons usually leads to stability. Since, in benzene all the six 7t-

re
rFl
electrons of the three double bonds are completely delocalized to form one lowest energy molecular
orbital which surrounds all the carbon atoms of the ring, therefore, it is extraordinarily stable.

F
H-
/ \
/ \

r
/ \
ou
fo
s / \ /
H X

\
/ \ / s

/
ks
oo
\ /
\ /
Y
H /
eB

Delocalization of 67r-electrons in benzene

Q. 13.11. What are the necessary conditions for any system to be aromatic ?
ur

Ans. The necessary conditions for a molecule to be aromatic are : (/) It should have a single cyclic cloud of
delocalised n-electrons above and below the plane of the molecule,
ad

(ii) It should be planar. This is because complete delocalization of TC-electrons is possible only if the
Yo

ring is planar to allow cyclic overlap of p-orbitals.


(Hi) It should contain Hiickel number of electrons, i.e., (4n + 2) n-electrons where n = 0, 1,2, 3 etc.

A molecule which does not satisfy any one or more of the above conditions is said to be non-aromatic.
d

Q. 13.12. Explain why the following systems are not aromatic ?


Re
in

/=\
(i) CH2 (ii) (iii)
\=/
F

Ans. (0 sp^ CH2. Due to the presence of a 5p^-hybridized carbon, the system is not planar. It
does contain six Ji-electrons but the system is not fully conjugated since all the six 71-elcctrons do not
form a single cyclic electron cloud which surrounds all the atoms of the ring. Therefore, it is not an
aromatic compound,
Due to the presence of a j'p^-carbon, the system is not planar. Further, it contains only
00 four n electrons, therefore, the system is not aromatic because it does not contain
planar cyclic cloud having (4 n -t- 2) 7i-electrons. This system, however, behaves as a
diene.
13/146 7^'iadee^ 4. New Course Chemistry (XI) BZ

/=\
m I I
\=/
Planar
Tub shaped
Cyclooctatetraene is not planar but is tub-shaped. Further, it has only 871-electrons. Therefore, the system
is not aromatic since it does not contain a planar cyclic cloud having (4n + 2) 7C electrons. The planar
cyclooctatetraene is, however, antiaromatic.
Q. 13.13. How will you convert benzene into

w
(i) /7-nitrobromobenzene (ii) nt-nitrochlorobenzene (Ui) p-nitrotoluene (iv) acetophenone ?
Ans. (i) The two substituents in the benzene ring are present at p-positions. Therefore, the sequence of
reactions should be such that first an o, p-directing group, i.e., Br atom should be introduced in the
benzene ring and this should be followed by nitration. Thus,

Flo
●a
Br2, Anhyd. FeBr3, A Cone. HNO3 + cone. H2SO4, A
Br
(Bmmination) (Nitration)

e
Benzene Bromobenzene

re
NO2

rF
Separate by
O2N Br + Br
fractional distillation > O2N Br

p-Bromonitrobenzene
ur o-Bromonitrobenzene />-Bromonitrobenzene

fo
(major) (minor)

(ii) Here, since the two substituents are at m- position w..rt. each other, therefore, the first substituent in
ks
the benzene ring should be a m-directing group (i.e., NO2) and then the other group (i.e.. Cl) should be
introduced. Therefore, the sequence of reactions is :
Yo
oo

© ●o>
Cone. HNO3 + cone. H2SO4, A Cl2,Anhyd.AlCl3,A
eB

(Nitration) NO2 (Chlorination) NO2


Benzene Nitrobenzene m-Chloronitrobenzene

{Hi) Here, since the two substituents are at p-position w.r.t. each other, therefore, the first substituent in
ur

the benzene ring should be a o, p-directing group {i.e., CH3) and then the other group {i.e., NO2) should
ad

be introduced.
Yo

© <0^'
CH3CI, Anhyd. AICI3, A Dil.HN03 + dil.H2S04,A
>
(EC. alkylation) (Nitration)
d

Benzene Toluene
Re
in

NO2
F

Separate by
O2N CH3 fractional distillation O2N CH3
/7-Nirotoluene o-Nitrotoluene /7-Nitrotoluene
(major) (minor)

(iv) Acetophenone can be prepared by EC. acylation using either acetyl chloride or acetic anhydride
0 O

© a
Anhyd. AICI3. A
+ CH3—C—Cl (EC. acylation) C—CH3 + HCl
Acetyl chloride
Benzene
Acetophenone
HYDROCARBONS 13/147

.0 O
CH3— Anhyd.AlCl3.A
or + o
{EC. acylalion)
c—CH3 + CH3COOH
Etbanoic acid
Benzene
CH3-C. Acetophenone
o
Acetic anhydride
Q. 13.14. In the alkane, CH3CH2—€(^3)2—CHj—CH(CH3>2, identify 1“, 2\ 3“ carbon atoms and give
the number of H-atoms bonded to each one of these.
Ans. The expanded formula of the given compound is

w
1“
H H CH^H CH^ 15 H attached to five T carbon
.cl 20I 40I 90I 3»l 4 H attached to two 2“ carbons
H-Lc-i-C—C—C—C— H

Flo
I I I I I 1 H attached to one 3® carbon
H H CH,H
to J
CH,
to O
1® 1®

e
Q. 13.15. What effect does branching of an alkene chain has on its boiling point ?

re
Ans. As the branching increases, the surface area of an alkane approaches that of a sphere. Since, a sphere
has minimum surface area, therefore, van der Waals forces of attraction are minimum and hence the

rF
boiling point of the alkane decreases with branching.
Q. 13.16. Addition of HBr to propene yields 2-bromopropane, while in presence of benzoyl peroxide, the
same reaction yields 1-bromopropane. Explain and give mechanism.
ur
fo
Ans. Addition of HBr to propene is an ionic electrophilic addition reaction in which the electrophile, i.e.,
H'*' first adds to give a more stable 2® carbocation. In the 2nd second, the carbocation is rapidly attacked
by the nucleophile Br~ ion to give 2-bromopropane
ks
a
Yo
H + Br"
oo
+
Slow
Step 1. CH3—CH=i=CH2 +
B

CH3—CH—CH3
Propene Electophile 2® Carbocation
re

{more stable)

Fast
u

+> CH3—CH—CH3
ad

Step 2. CH3—CH—CH3 + Br"


Yo

Nucleophile
Br
2-Bromopropane

In presence of benzoyl peroxide, the reaction is still electrophilic but the electrophile here is a Br free
nd

radical which is obtained by the action of benzoyl peroxide on HBr


Re

CgHjCO-O^—COCfiHs
A
2 CgHjCOOh +2CO2
Fi

(Homolytic fission)
>
Benzoyl perxide Benzoyl radical Phenyl
{unstable) radical

r>
CfiHs + H-^Br > CfiHg + Br
Benzene

In the first step, Br radical adds to propene in such a way so as to generate a more stable 2® free rascal.
In the second step, the firee radical thus obtained rapidly abstracts a hydrogen atom from HBr to give 1-
bromopropane.
13/148 “Putdce^'^ New Course Chemistry (X1)E!SI9D

Slow
CH3—CHi=CH2 + Br > CH3—CH—CH2Br
Propene 2° Free radical
(more stable)

w
CH3—CH—CHBr + H a Fast
► CH3—CH2—CH2Br+Br
1-Bromopropane

From the above discussion, it is evident that although both reactions {i.e., in presence or absence of
benzoyl peroxide) are electrophilic addition reactions but it is due to different order or sequence of
addition of H and Br atoms which gives different products.

e
o
Q. 13.17. Write down the products of ozonolysis of 1, 2>dimethylbenzene (o-xylene). How does the result

re
support Kekul^ structure of benzene ?
Ans. o-Xylene may be regarded as a resonance hybrid of the following two Kekul^ structures. Ozonolysis of
each one of ftiese gives two products as shown below

Frl
:

F
CH3
CH3
/ .CH3 CH=0
-t ou (OO3.CH2CI2,196K
(/OZ11/H2O ^
+ I

sor
CH=0
0=CH O
I Glyoxal
Methylglyoxal

kf
CH3
CH
(0O3,CH2Cl2,196K
oo CH=0
CH3 ^C CH3 2 I
(ij-)Zn/H20 ^
+
/'W
Y
>~
CH=0
B
Glyoxal
U 0
1,2-Dimethylglyoxal
re

Thus, in all, three products are formed. Since, all the three products cannot be obtained from any one of
oY

the two Kekule structures, this shows that o-xylene is a resonance hybrid of the two Kekul6 structures
u

a and U).
ad

Q. 13.18. Arrange benzene, n-hexane and ethyne in decreasing order of acidic behaviour. Also give reason
for this behaviour.
d

Ans. The hybridization state of carbon in these three compounds is


in

sp

CH3 —(CH2>4—CH3 H—C=C—H


Re

Hexane Ethyne
F

Benzene

Type of orbital; sp^ sp^ sp


s~Character : 33-3% 25% 50%
Since, jr-electrons are closer to the nucleus, therefore, as the ^-character of the orbital making the C—^H bond
increases, the electrons of C—^H bond lie closer and closer to the carbon atom. In other words, the
partial +ve charge on the H- atom increases and hence the acidic character increases as the s- character
of the orbital increases. Thus, the acidic character decreases in the order : Ethyne > Benzene > Hexane
Q. 13.19. Why does benzene undergo electrophilic substitution reactions easily and nucleophilic substitutions
with difficulty ?
Ans. Due to the presence of an electron cloud containing 6 TC-electrons above and below the plane of the
ring, benzene is a rich source of electrons. Consequently, it attracts the electrophiles (electron-deficient)
HYDROCARBONS 13/149

reagents towards it and repels nucleophiles (electron-rich) reagents. As a result, benzene undergoes
electrophilic substitution reactions easily and nucleophilic substitutions with difficulty.
Q. 13.20. How will you convert the following compounds into benzene ?
(i) Ethyne
(ii) Ethene
(iii) Hexane.

Red hot Fe tube


Ans. (0 3 HC=CH 873 K
Ethyne
Benzene

w
(ii) Ethene is first converted into ethyne and then to benzene as shown above.

Br2/CCl4 KOH (ale), A


CH2 = CH2 Br—CH2—CH2—Br 4 CH2 = CHBr

F lo
( Dehydrobromnation )
Ethene 1, 2-Dibromoethane Vinyl bromide

NaNH2/liq NH3 Red hot Fe mbe


■> HCsCH ■>

ee
I96K 873 K
(Dehydrobomination) Ethyne Benzene

Fr
{iii) When vapours of hexane are passed over heated catalyst consisting of Cr203, M02O3 and V2O5
supported over AI2O3 at 773 K under 10-20 atm pressure, cyclization and aromatization occurs
simultaneously to trfford benzene

for
ur
CH3
/
CH2 CH3 Cr203A^2Q5^^°2Q3~^^2Q3 Aromatization
s
I >
ook
Cyclization (-3H2)
Yo
CH2 CH2 (-H2)
cyclohexane Benzene

CHi
eB

Hexane

Q. 13.21. Write structures of all the alkenes which on hydrogenation give 2-methylbutane.
Ans. The basic skeleton of 2-methylbutane is
r
ad
ou

C
1
C—2C—3C—4C
Y

Putting double bonds at various different positions and satisfying the tetracovalency of each carbon,
the structures of various alkenes which give 2-methylbutane on hydrogenation are :
Re

CH3 CH
CH3
nd

4 I
CH3—3cH—CH = CH2
2 I 1 2^ L 4
CH3—2c=^H—CH3
1
CH=C—CH2CH3
2I 3 4
Fi

3-Methylbut-l-ene 2-Methylbut-2-ene 2-Methy Ibut-1 -ene


Q. 13.22. Arrange the following set of compounds in order of their decreasing relative reactivity with an
electrophile, E*^.
(a) Chlorobenzene, 4-dinitrochlorobenzene, p- nitrochlorobenzene
(b) Toluene, p-H3C—C^H^——NO2, P"02N—C^H^—^N02.
Ans. The typical reactions of benzene are electrophilic substitution reactions. Higher the electron-density in
the benzene ring, more reactive is the compound towards these reactions. Since, NO2 is a more powerful
electron-withdrawing group than Cl, therefore, more the number of nitro groups, less reactive is the
compound.Thus, the overall reactivitydecreasesin the order :
Chlorobenzene > p-nitrochlorobenzene > 2, 4-dinitrochlorobenzene

A
13/150 7^'u^decfo. ’<i New Course Chemistry (XI) Igajwii

(b) Here, CH3 group is electron donating but NO2 group is electron-withdrawing. Therefore, the maximum
electron-density will be in toluene, followed by p-nitrotoluene followed by p-dinitrobenzene. Thus, the
overall reactivity decreases in the order:
Toluene > P-H3C—CgH4—^N02 > P-O2N—C5H4—^N02
Q. 13 J3. Out of benzene, m-dinitrobenzene and toluene which will undergo nitration most easily and why ?
Ans. CH3 group is electron-donating while -NO2 group is electron-withdrawing. Therefore, maximum electron

ow
density will be in toluene, followed by in benzene and least in m-dinitrobenzene.Therefore, the ease of
nitration decreases in the order : toluene > benzene > /«-dinitrobenzene.
Q. 13.24. Suggest the name of a Lewis acid other than anhydrous aluminium chloride which can be used
during ethylation of benzene.
Ans. Ethylation means introduction of an ethyl group in the benzene ring. This is usually carried out by
Friedel-Crafts reaction of benzene with ediyl halide (chloride or bromide), ethene or ethanol. The Lewis
acid catalysts other than anhydrous AICI3 used in this reaction are : anhydrous FeC^, SnC^, BF3, etc.

e
Q. 13.25. Why is Wurtz reaction not preferred for preparation of alkanes containing odd number of carbon

re
atoms ? Illustrate your answer by taking one example.

rFl
Ans. For preparation of alkanes containing odd number of carbon atoms, a mixture of two alkyl halides has

F
to be used. Since, two alkyl halides can react in three different ways, therefore, a mixture of three
alkanes instead of the desired alkane would be formed. For example, Wurtz reaction between 1-
bromopropane and 1- bromobutane gives a mixture of three alkanes, i.e., hexane, heptane and octane as
shown below:

r
ou
fo
Diy ether
CM3CH2CH2 ^Br 4_2J^a j_B^—CH2CH2CH3 —-—» CH3CH2CH2CH2CH2CT13 + 2 NaBr
1-Bromopropane 1-Bromopropane

ks
Hexane

Dry ether
CH3CH2CH2—[ir^ TnT+"Brj-CH2CH2CH2CH3
oo
A
1-Broraopropane 1-Bromobutane
Y
CH3CH2CH2CH2CH2CH2CH3 + 2 NaBr
eB

Heptane

Dry ether
CH3CH2CH2CH2— CH2CH2CH2CH3 ■>
r

A
1-Bromobutane 1-Bromobutane
ou

CH3CH2CH2CH2CH2CH2CH2CH3 + 2 NaBr
Y
ad

Octane
d
in
Re
F
HYDROCARBONS 13/151

1 WITH ANSWERS,
HINTS AND SOLUTIONS

EXEMPILAR PI^OBkEIVIS .iT* .1 ● ●


. t

MULTIPLE CHOICE QUESTIONS-I

w
1. Arrange the following in decreasing order of CH3—CHj—CH2—CHj—Br
their boiling points. (C)

Flo
(A) ^-butane (B) 2-methylbutane The mixture consists of

(C) n-pentane (D) 2, 2-dimethylpropane (a) A and B as major and C as minor products
(fl) A > B > C > D (/?) B > C > D > A (b) B as major, A and C as minor products

ee
(c) D > C > B > A ((/)C>B>D>A (c) B as minor, A and C as major products

Fr
2. Arrange the halogens F2, CI2, Br2» l2» in order (J) A and B as minor and C as major products
of their increasing reactivity with alkanes, 6. Which of the following will not show
(a) I2 < Br2 < CI2 < F2 geometrical isomerism ?

for
(b) Br2 < CI2 < F2 < I2 F H

(c) F2 < CI2 < Br2 < I2


ur (fl) C = C
Cl D
{d) Br2 < I2 < CI2 < F2
3. The increasing order of reduction of alkyl F Cl
halides with zinc and dilute HCl is
(b)
k sc = c
Yo
Cl F
(a) R-Cl < R-1 < R-Br
oo
(b) R-Cl < R-Br < R-I
(c) HsC^ C =
eB

(c) R-I < R-Br < R-Cl C

{d) R-Br < R-1 < R-Cl H5C2 CH3


4. The correct lUPAC name of the following CH CH3
alkane is
r

(d) C = C
ou
ad

H3C—CH2—CH—CHj—CH^—CH—CH2—CH3 CH3 C2H5


CH 7. Arrange the following hydrogen halides in
CH2 order of their decreasing reactivity with
Y

/\
CH3 CH3 CH3 propene.
(a) HCl > HBr > HI (h) HBr > HI > HCl
(a) 3, 6-Diethyl-2-methyloctane
Re
nd

(c) HI > HBr > HCl (d) HCl > HI > HBr
{b) 5-Isopropyl-3-ethyloctane
(c) 3-Ethyl-5-isopropyloctane
8. Arrange the following carbanions in order of
Fi

their decreasing stability.


(d) 3-Isopropyl-6-ethyloctane
(A)H3C—C = C- (B)H—C = C“
5. The addition of HBr to 1-butene gives a mixture
of products A, B and C (C) H3C—CH-
Br Br
(<3) A > B > C (6) B > A > C
..C C-... (c) C > B > A {d)C> A>B
CH3 CH3 \-C2H5 9. Arrange the following alkyl halides in
H H decreasing order of the rate of ^-elimination
reaction with alcoholic KOH.
(A) (B)
13/152 'P>t<uUi^ ^ New Course Chemistry (XI)EZ5MD

H 10. Which of the following reactionsof methaneis


I incompletecombustion:
(A) CHj—CI —CH2Br Cu/523K/l()Oalni
(fl) 2CH4 + O2 ^ 2CH3OH
CH3
(B) CHj—CH2—Br ib) CH4 + O2 - > HCHO + H2O
(C) CH3—CH2—CH2—Br (c) CH4 + O2- > C (5) + 2 H2O (/)
(a) A > B > C (fe) C > B > A {d} CH4 + 2O2 CO2 ig) + 2 H2O (/)
(c) B > C > A {d)k>C>h

w
fidULTIPLE C 1 a= QUESTIONS'!!
In the following questions two or more options may be correct.

F lo
11. Some oxidation reactions of methane are given below. Which of them is/are controlled oxidation
reactions ?

(a) CH4 (g) + 2 02 (g) ^C02(g) + 2H20(/) (6)CH4(^) + 02(g) -4 C (s) + 2 H2O (/)

ee
Mo^Oj Cu/523K/100alm
(c) CH4 (g) + O2 (g) ^ > HCHO + H2O (d) 2 CH4 (g) + O2 (g)

Fr
> 2 CH3OH
12. Which of the following alkenes on ozonolysis give a mixture of ketones only ?
(a) CH3—CH = CH—CH3 (b) CH3—CH—CH =CH^

for
I
ur CH3
CH3 /CH3
(c) C
(d) (CH^)^C = C
ks
CH3 CH3
Yo
13. Which are the correct lUPAC names of the following compound ?
oo

HC(CH3)2
eB

I
H3C—CH2—CH2—CH—CH—CH2—CH2—CHj—CH2—CH3
H3C—CH—CH2CH3
r

(a) 5-Butyl-4-isopropyldecane (b) 5-Ethyl-4-propyldecane


ou
ad

(c) 5-jec-Butyl-4-isopropyldecane (d) 4-( 1 -methylethyl)-5-(l -meihylpropyl)decane


14. Which are the correct lUPAC names of the following compound ?
Y

H3C-CH2-CH2-CH2 -CH -CH2-CH2-CH2-CH2-CH3


Re

CH,
nd

I
H3C-C-CH3
Fi

CH3
(a) 5-(2', 2'-Dimethylpropyl)decane ib) 4-Butyl-2, 2-dimethylnonane
(c) 2, 2-Dimethyl-4-pentyloclane (d) 5-NeopentyIdecane
15. For an electrophilic substitution reaction, the presence of a halogen atom in the benzene ring
(a) deactivates the ring by inductive effect
(b) deactivates the irng by resonance
(c) increases the charge density at ortho and para position relative to meta position by resonance
(d) directs the incoming electrophile to meta position by increasing the charge density relative to ortho and
para position.
HYDROCARBONS 13/153

16. In an electrophilic substitution reaction of {d) CH-, = CH® is more stable than
nitrobenzene, the presence of nitro group
{a) deactivates the irng by inductive effect
CH3—CH®
{h) activates the ring by inductive effect 18. Four structures are given in options («) to id).
Examine them and select the aromatic
(c) decreases the charge density at ortho and para
structures.
position of the irng relative to meta position
by resonance
{d) increases the charge density at meta position ih) I

w
relative to the ortho and para positions of the
ring by resonance.
17. Which of the follotving arc correct ?
(c)
{a) CH3—O—CH® is more stable than

Flo
CH3 ■CH® 19. The molecules having dipole moment are
{b) (CH3>2CH® is less stable than

e
re
id) 2, 2-Dimethylprop<uie
CH3—CH^—CH®
ib) trans~Pent-2-ene

F
(c) CH2 = CH—CH® is more stable than (c) c/.5-Hex-3-ene
CH3—CH2 CH® id) 2, 2, 3. 3-Tetramethylbutane
ur
r
fo
ANSWERS

Multiple Choice Questions -1 ks


1. id) 2. ia) 3. (b) 4. (a) 5. (a) 6.(d) 7. (c) 8. ib) 9. id) 10. (c)
Yo
oo
Multiple Choice Questions »II
11. ic, d) 12. ic, d) I3.ic,d) 14. (a, d) 15. (a. c) 16. ia, c) 17, ia, c)
B

18. ia, c) 19. ib, c)


e

HINTS FOR DIFFICULT MULTIPLE CHOICE QUESTIONS


ur

Multiple Choice Questions -1


ad
Yo

1. Boiling points decrease with increase in branching and with decrease in total number of carbon atoms.
Thus, boiling points decrease in the order; C (n-pentane) > B (2-methylbutane) > D (2, 2-dimethylbutane)
> n-butane (A).
2. Reactivity decreases down the group as the electronegativity or the electrode potential of the halogen
d
Re

decreases down the group. Thus, option ia) is correct.


in

3. The reactivity of reduction of alkyl halides with Zn/HCl increases as the strength of the C—X bond decreases,
i.e.,R—CUR—Br<R—I.
F

4. A parent chain with larger number of substituents is preferred.


7 6 5 4 3 2 1 3 4 5 6 7 8

CH3CH2—CH—CH2CH2—CH—CH2CH3 CH3CH2—CH—CH2CH2—CH—CH2CH3
CH
CH2CH3 /2\
CH CH2CH3
CH3 CH 3 1CH3 CH3
3-Ethyl-6-isopropyloctane 3. 6-Diethyl-2-methyloctane
iWrong numbering) iCorrecl numbering)
13/154 7>^t4zcicefi,'^ New Course Chemistry (Xl)BSiail
5. Addition of HBr to 1-butene occurs in accordance with Markovnikov’s rule giving I as the major and C as
the minor product.
HBr
CH3CH2—CH = CH2 Maik.addn. CH3CH2—CH—CH3 H- CH3CH2CH2CH2—Br
1-Butene I C
Br

Since I contains a chiral carbon, it exists in two enantiomers (A and B) which are mirror images of each
other

w
Mirror
Br Br
I

HsCi Hy ^CHj CH,-^ \'C2H5

Flo
H

(A) (B)

ee
Thus, the mixture consists of A and B as major and C as minor product.

Fr
6. For geometrical isomerism, it is essential that each carbon atom of the double bond must have different
substituents. Now option (d) does not show geometrical isomerism since it has two CH3 groups on the
same carbon atom of the double bond.

for
7. Reactivity increases as the strength of H-X bond decreases, i.e., HI > HBr > HCl.
ur
8. sp-Hybridized carbon is more electronegative than a sp^-hybridized carbon and hence can acconunodate
the negative charge better. Therefore, (A) and (B) are more stable than (C). Since CH3 group has +I-effect,
ks
therefore, it intensifies the negative charge and hence destabilizes A relative to B. Thus, the overall stability
decreases in the order: B > A > C.
Yo
oo
9. More the number of ^-substituents, more stable alkene it will give on ^-elimination. Since (A) has two,
(C) has one p-methyl substituent while (B) has no P-methyl substituent, therefore, reactivity towards
eB

P-elimination decreases in the order: A > C > B.


10. Complete combustion of CH4 gives CO2 and H2O. Therefore, reaction (c) in which CH4 is converted into
C and H2O is an incomplete combustion reaction.
r

Multiple Choice Questions - II


ou
ad

11. Reactions (c) and (d) in which CH4 does not undergo complete combustion to give CO2 and H2O or
incomplete combustion to C and H2O are controlled oxidation reactions.
Y

12. Alkenes which have two substituents on each carbon atom of the double bond, give mixture of ketones on
ozonolysis, i.e., options (c) and (d).
nd
Re

.CH3 ■CHj
(C)
>fc CH3
(0 O3/CCI4, 196K ^
(it) Zn/HjO ^ >=o . o=c
Fi

Acetone “CH3
Cyclopentanone

.CH3 ●CH3
(0 O3/CCI4,196K
(d)(CH3)2C4=C (CH3)2C=0 + 0=C
CH3
(it) Zn/HjO ^ Acetone Acetone CH3
On the other hand, alkenes (a) and (b) give a mixture of two aldehydes.
(i)03/CCl4,l%K
(a) CH3CH=j=CH—CH3 (//)Zn/H20
> CH3CH = 0 -I- 0 = CHCH3
Acetaldehyde Acetaldehyde
HYDROCARBONS 13/155

(«)03/CCl4,196K
(b) CH3—CH—CH
I
= CH2 (/OZn/HjO
» CH3—CH—CH=0 + 0 = CH
I Methanal
CH3 CH3
2-Methylpropanal
13. Although lUPAC name for j:ec-butyl and isopropyl groups are 1 -methylpropyl and 1 -methylethyl respectively
yet both these names are also recommendedfor lUPAC nomenclature.

ow
14. The lUPAC name for neopentyl group is 2,2-dimethylpropyl. Therefore, options (a) and (d) are correct.
15. Statements (a), (c) are correct but (b) and (d) are incorrect. Refer to page 13/90 for explanation.
16. Statements (a) and (c) are correct but statements (b) and (d) are wrong. Refer to pages 13/89-13/90 for
explanation.

17. A*

e
CH3—O—CH2 ^ ► CH3—0 = CH2

re
+

rFl
+

CH3OCH2 is stabilized by strong resonance effect while CH3 CH2 is stabilized by only weak +1-

F
effect of the CH3 group. Thus, option (a) is correct.
CH
+ .

CH is stabilized by + I-effect of the two CH3 groups while CH3CH2 CH2 is stabilized by +1-

r
CH3
ou
fo
effect of the ethyl group. Since +I-effect of the ethyl group is weaker than +I-effect of the two CH3 groups,

ks
therefore, (CH3)2CH'*' is more stable than CH3CH2—CH2 group. Thus, option (b) is wrong.
JT\*
oo
CH2=!=CH-*-CH2 ●* CH2—CH=CH2 is stabilized by strong resonance effect while.
Y
CH3CH2-J—CH2 is stabilized by weak +I-effect of the CH2CH3 group. Thus, option (c) is correct.
eB

In CH2 = CH'*', +ve charge is present on the more electronegativ e sp-hybridized carbon while in
+ +
CH3 ●CH2 , +ve charge is present on the less electronegative ap^-hybridized carbon, therefore, CH2 = CH
r

is less stable than CH3 -CH2 . Thus, option (d) is wrong.


ou

18. Cyclopropenyl cation (a) has completely delocalized 2 7t-electrons. Therefore, in accordance with Hilckel
Y
ad

rule, it is aromatic.
Cyclooctatetraene is not planar and also it has 8 7C-clectrons. Therefore, in accordance with HUckel rule, it
is not aromatic,
d

(c) Diphenyl has two benzene rings each one of which is planar and has 6 7C-electrons. Therefore, like
in
Re

benzene, it is also aromatic.


(d) Cyclopropenyl anion although planar has only 4 Ji-electrons. Therefore, in accordance with HUckel
F

rule, it is not aromatic.

CH H
c=c
CH3CH2;!>^ c=c ^CH2CH3
19.
H
'CH2CH3 H H
/ra/is-Pent-2-ene c«-Hex-3-ene

Since the +I-effect of CH3CH2 group is higher than that of CH3 group, therefore, the dipole moments of
C—CH3 and C—CH2CH3 bonds are unequal. Although these two dipoles oppose each other, yet they do
not exactly cancel out each other and hence trans-2-pentene has small but finite dipole moment.
In cw-hex-3-ene, although the dipole moments of the two C—CH2CH3 bonds are equal, they are inclined
to each other at angle of 60“ and hence have a finite resultant.
13/156 4 New Course Chemistry (XI)

Further, 2, 2-dimethylpropane and 2, 2, 3, 3-telramethylbutane have zero dipole moments because of the
following two reasons :
(,) C—C and C—H bonds in alkanes are almost non-polar and have no fixed orientations in space, therefore,
their resultant dipole moments, if any, are almost negligible.
(i7) Both these are symmetrical molecules and hence their dipole moments is zero.

ow
SHORT ANSWER QUESTIONS

20. Why do alkenes prefer to undergo electrophilic addition reactions while arenes prefer electrophilic
substitution reactions ? Explain.
Ans. Due to the presence of a 7C-electron cloud above and below the plane of alkenes and arenes, these are
electron rich molecules and hence attract electrophiles and thus undergo electrophilic reactions.
Alkenes, undergo electrophilic addition rather than substitution reactions because of the following two

e
reasons :

Fl
re
X
H H H
xV X*Y"

F
;c=c :c=c > :c—c
-YH
X H

ur
Substitution Alkene Addition y

r
(i) In electrophilic substitution reactions, one a- bond is broken and another is formed. Since the bond

fo
energies of the a C—H bond broken and new <j C—X bond formed are not much different, therefore,
electrophilic substitution reactions of alkenes are not accompanied by large energy changes and hence are
energetically not favourable, ks
Yo
(ii) In electrophilic addition reactions, one weak 7t-bond is broken and two strong a-bonds (C—X and
C—Y) are formed, therefore, electrophilic addition reactions of alkenes are accompanied by large energy
oo
changes and hence are energetically favourable. In other words, the typical reactions of alkenes are
electrophilic addition reactions and not electrophilic substitution reactions.
B

In contrast, arenes undergo electrophilic substitution reactions rather than electrophilic addition reactions
as explained below :
e

H X
ur

H H X
X*Y~
X*Y~ ^
ad

Y
Yo

-YH

Addition Benzene Substitution


(An arene)
d

Benzene has a large resonance energy of 150-6 kJ mol"‘. During electrophilic addition reactions, two
Re
in

new a-bonds are formed but the aromatic character of the benzene irng is destroyed, i.e., the resonance
energy of the benzene irng is lost. Since the loss in resonance energy is more than the energy released
during formation of two additional a-bonds, therefore, electrophilic addition reaction is not energetically
F

favourable.
In contrast, during electrophilic substitution reactions, one a C—H bond is broken and one new a C—X
bond is formed but the aromatic character of the benzene ring remains intact, i.e., resonance energy of the
benzene ring remains intact. In other words, small energy changes, if any, during making and breaking of
a-bonds is more than compensated by the large resonance energy of the benzene ring. Thus, electrophilic
substitution reactions of arenes are energetically more favourable than electrophilic addition reactions.
Thus, from the above discussion, it follows that alkenes undergo electrophilic addition reactions while
arenes undergo electrophilic substitution reactions.
21. Alkynes on reduction with sodium in liquid ammonia from trans alkenes. Will the butene thus formed
on reduction of 2-butyne show the geometrical isomerism ?
HYDROCARBONS 13/157

Ans. Since 2-butene is capable of showing geometrical isomerism, therefore, reduction of 2-butyne with sodium
in liquid ammonia will give trans-2-butene.
CH
Na, liquid NH3
CH3—C H C—CH3 “C=C
196-200 K H
CH3
2-Butyne tranj-2-Butene
22. Rotation around carbon*carbon single bond of ethane is not completely free. Justify the statement.
Ans. Rotation about C—C single bond is restricted due to repulsions between the electron clouds of C—H
bonds on adjacent carbon atoms. As a result of these repulsions, ethane exists in infinite number of

w
conformations out of which two extreme conformations, Le., staggered and eclipsed are important. Refer
to Fig. 13.6, and Fig. 13.7 page 13/20.
The staggered conformation is, however, more stable than the eclipsed conformation by about 12*55 kJ
mol“^ (Fig. 13.8, page 13/21). This energy difference of 12*55 kJ mol"* between the staggered and the

F lo
eclipsed conformation is, in fact, the energy barrier to rotation about C—C single bond in ethane. However,
this energy barrier is not large enough to prevent rotation. Even at room temperature, collisions of the
molecules supply sufficient kinetic energy to overcome this energy barrier. Thus, the two conformations
are readily interconvertible. As a result, it is not possible to separate the two conformations of ethane.

ee
However, at any given moment, most of the molecules would exist in the staggered conformation due to its

Fr
minimum energy and maximum stability.
23. Draw Newman and Sawhorse projections for the eclipsed and staggered conformations of ethane.
Which of these conformations is more stable and why ?
Ans. Refer to Fig. 13.6 and 13.7, Page 13/20.

for
24. The intermediate carbocation formed in the reactions
of HI, HBr and HCl with propene is the same
ur
and the bond energy of HCl, HBr and HI is 430*5 kJ moH, 363*7 kJ mor^ and 296*8 kJ mol"*
respectively. What will be the order of reactivity of these halogen acids ?
Ans. In the reaction of propene with HCl, HBr or HI, first a H'*' adds to give the same carbocation intermediate
ks
(I) which then undergoes nucleophilic attack by the halide ion (X“) to give the addition product. Here, first
Yo
step is slow and hence is the rate-determining step of the reaction while the second step is fast and hence
oo
does not affect the rate of the reaction.
eB

Slow , Fast
CH3—CH=CH2 + H+ First step ► CH3—CH—CH3 Second step > CH3—CHX —CH3
The rate of the first step depends upon how easily is the proton available. The availability of the proton, in turn,
r

depends upon the bond dissociation energy of the H—X molecule. Since the bond dissociation energy decreases
ou
ad

in the order: HCl (430*5 kJ mol~*) > HBr (363*7 kJ mol“*) > HI (296*8 kJ mol“*), therefore, the reactivity of the
halogen acids decreases in the reverse order: HI > HBr > HCl.
25. What will be the product obtained as a result of the following reaction and why ?
Y

+ H3C—CH2—CH2CI AICI3 ^
Re
nd

Ans. During F.C. reactions, carbocationsare the intermediates. Therefore, AICI3 first reacts with n-propyl chloride
to form n-propyl cation. This being 1® is less stable and hence rearranges to the more stable 2® carbocation,
Fi

i.e., isopropyl carbocation by an hydride shift. The isopropyl carbocation then reacts with benzene to form
isopropylbenzene.
1,2-Hydride +

CH3—CH2—CHj-^Cl + AICI3 -AlCl^ CH3—CH—CH2 shift > CH3—CH—CH3


n-Propyl chloride
0/
n-Propyl carbocation (1®)
Isopropyl carbocation (2°)
{more stable)

{less stable)
A CH3
CH
\
CH3
Isopropylbenzene
13/158 New Course Chemistry (XI) BE

26. How will you convert benzene into


(0 /7-nitrobromobenzene
(ii) m-nitrobromobenzene
Ans. (0 Refer to Ans. to Q. 13.13 (i), page 13/146.
(I’l) Here, since the substituents are at m-position w.r.t. to each other, therefore, the first substituent in the
benzene ring should be m-directing (i.c., NO2) and then the other group (i.e., Br) should be introduced.
Therefore, the sequence of reactions is

Cone. H2SO4

low
0
Br2,Anhyd.AlCl3, A

0 ●4- cone. HNO3. A ^


(Nitration)
Nitrobenzene
NO2
(Bromination)
m-Bromonitrobenzene

27. Arrange the following set of compounds in the order of their decreasing relative reactivity with an
electrophile. Give reason.

ee
OCH3 Cl NO2

t^
rF
Fr
o

for
Ans. The reactivity of arenes towards electrophiles depends upon their relative electron density in the benzene
ring. Higher the electron density, more reactive is the arene.
Now OCH3 group in anisole is strongly electron donating due to its +R-effect. As a result, it increases the
u
electron density in the ring and hence makes anisole more reactive than benzene towards electrophiles
s
ook
Yo
O"
Cl
:6ch ■'OCHs N N

A
eB

◄ —► <—►

AA AA ’ AA A^ Chlorobenzene
r

Anisole, +R-effect Nitrobenzene -1-effect


ou
ad

-Rand-I-Eflfect

On the other hand, -NO2 group in nitrobenzene and Cl atom in chlorobenzene, both are electron-withdrawing
and hence decrease the electron density in the ring. As a result, both nitrobenzene and chlorobenzene are
Y

less reactive than benzene towards electrophiles. But -NO2 group is a strong electron-withdrawing group
due to its strong -R-effect and strong -I-effect In contrast. Cl atom is only a weak electron-withdrawing
Re
nd

group due to its only weak -I-effect. In other words, electron density in chlorobenzene is more than in
nitrobenzene and hence chlorobenzene is more reactive than nitrobenzene towards electrophiles.
Fi

Thus, the overall reactivity of these three compounds towards electrophiles decreases in the order :
anisole > chlorobenzene > nitrobenzene.
28. Despite their -I-efifect, halogens are o- and p-directing in haloarenes. Explain.
Ans. Refer to Art. 3.8.12, page 13/89.
29. Why does presence of a nitro group make the benzene ring less reactive in comparison to the
unsubstitut^ benzene ring. Explain.
Ans. Refer to Ans. to Q. 27 above.
30. Suggest a route for the preparation of nitrobenzene starting from acetylene ?
Ans. Acetylene when passed through red hot iron tube at 873, undergoes cyclic polymerization to give benzene
which upon subsequent nitration gives nitrobenzene.
HYDROCARBONS 13/159

Cone. HNO3 NO2


Red hot + cone. H2SO4,333K
3CHSCH >
Iron tube at 873K (Nitration)
(Cyclic polymerization)
Nitrobenzene

31. Predict the major product(s) df the following reactions and explain their formation.
(Ph-CO-Oj
H3C—CH = CH2 HBr
■»

HBr
H3C—CH = CHj

w
Ans. In presence of peroxides, addition of HBr to propene follows anti-Markovnikov’s rule and hence gives 1-
bromopropane as the major product. The reaction occurs through a free radical mechanism via a more
stable free radical intermediate as shown below :

F lo
O O
I
R— C—<P^—C—R Homolytic
fission 2R + 2CO2
Peroxide

ee
Fr
R + H Br RH + Br

nBr

for
Slow H
CH3—CH^CH2 + Br > CH3—CH—CH2Br Fast ► CH3—CH2—CH2Br + Br
Propene More stable 1-Bromopropane
ur
2° free radical

However, in absence of peroxides, addition of HBr to propene followsMarkovnikov’srule and hence gives
s
2-bromopropane as the major product. The reaction occiurs through ionic mechanism via a more stable
ook
Yo
carbocation intermediate as shown below :
eB

Slow
CH3—CH=^CH2 + H* ♦►CHj—CH—CH3 Fast > CH3—CH—CH3
Propene More stable
Br
our

2° carbocation
ad

2-Bromopropane

32. Nucleophiles and electrophiles are reaction intermediates having electron irch and electron deOcient
centres respectively.Hence, they tend to attackelectrondeficient and electronirch centres respectively.
Classify the following species as electrophiles and nucleophiles.
dY
Re

(OH3CO- («*) H3C—C—O" m a (iV) CljC:


Fin

(VHH3O3C+ (vi) Br (vii) H3COH (viU) R—NH—R


Ans. Nucleophiles are electron-rich species. They may be neutral or negatively charged, i.e..
O

(0 H3CO-, (k) H3C—C—O- , (vi) Br- (vh) H3C—O—H , (viii) R—NH—R


Electrophiles are electron-deficient species. They may be neutral or positively charged, Le.,
m a (iv) CI2C : (v) (H3O3C+.
33. The relative reactivity of 1®, 2®, 3® hydrogen’s towards chlorination is 1 : 3*8 : 5. Calculate the
percentages of all monochlorinated products obtained from 2Tmethylbutane.

A.
13/160 T^nadeefr New Course Chemistry (XI) Q!

Ans. 2-Methylbutane has 4 different sets of equivalent hydrogen atoms marked as ‘o’, ‘fc’, ‘c’ and

I ^
CH3—CH—CH2—CH3 2-MeUiylbutane
and hence gives four different types of monochlorinated products. These are :
CH. CH. Cl
I ' CH
3\ I I
CICH2—CH—CH2 —CH3 C—CH2—CH3 CH3—CH—CH—CH3
ch3-"|Cl

w
A(n C(2“)
B(3»)

CH3

Flo
CH3 ●CH—CH2—CH2CI
D(n

ee
Relative number of equivalent hydrogens, a : b : c: d = 6 : I :2 : 3
Relative amounts of A, B, C and D = No. of hydrogen x relative reactivity

Fr
Relative amounts : A(l“) B(3") C(2») D(n
6x1=6 1x5 = 5 2x3-8 = 7-6 3x1=3

Total amount of monohaloginated compounds = 6 + 5 + 7-6 3 = 21-6.

for
6x100
ur
= 27-72 ; %B = —X100 = 23-15 ;
21-6 21-6
7-6 s
%C = X100 = 35-22 ; %D = — X100 = 13-85.
ok
21-6 21-6
Yo
34. Write the structures and names of products obtained in the reaction of sodium with a mixture of
l-iodo-2-methylpropaneand 2-iodopropane.
o
eB

Ans. Wurtz reaction between l-iodo-2-methylpropane and 2-iodopropane gives the following three products :
CH CH.
I I 1 I A

(0 CH3 —CH—CH2 —L Na JlJ j—CH2 —CH—CH3


r

l-Iodo-2-methylpropane {two molecules)


ou
ad

CH, CH,
1 21 3 4 51 6
Y

CH3—CH—CH2—CH2—CH—CH3 + 2NaI
2, 5-DimethyIhexane
nd
Re

CH3 /CH3 A L./CH3


(«) CH—[lj^2^a^I ]—CH 4

CH3
CH—CH
CH3
+ 2NaI
Fi

CH3 CH3
2-Iodopropane (two molecules) 2, 3-Dimethylbutane

CH, CH3 CH, CH


I I 1 II A 1 2I 3
m CH3 -CH—CH2—I + J—CH—CH3 CH3—CH—CH2—CH—CH3
+ 2NaI

1 -Iodo-2-methylpropane 2-Iodopropane 2, 4-Dimethylpentane


35. Write hydrocarbon radicals that can be formed as intermediates during monochlorination of 2-
methylpropane ? Which of them is more stable ? Give reasons.
Ans. 2-Methylpropane has two sets of equivalent hydrogens marked as ‘a’ and and hence gives two radicals,
1 and n.
HYDROCARBONS 13/161

a
a
CH- CH,
CH3 I ^
a I a
a a
«l ●
CH3—C—CH3 CH3 CH—CH, ^ CH3—CH—CH2
b ^
I 2-Methylpropane n

Radical (I) is more stable than radical (II) because of the following two reasons :
(0 Radical (I) is 3° while radical (II) is 1°.

ow
(«) Radical (I) has nine a-hydrogens and hence is stabilized by nine hyperconjugation structures while
radical (II) has only one a-hydrogen and is stabilized by one hyperconjugation structure as shown below :
H H-

H— c^cC—CH3 ◄ ► H—C = C—CH3 < > etc.


I I
H CH3 H CH3

e
Fl
(I) 9 such stnictures

re
since there are 9 a*hydrogens

F
CH3 CH3
I . I
CH3—(prCHi < > CH3—C=CH2
ur
r
H H*

fo
(U) Only one such structure
since there is only one
a-hydrogen ks
Yo
36. An alkane CgHjg is obtained as the only product on subjecting a primary alkyl halide to Wurtz
reaction. On monobromination this alkane yields a single isomer of a tertiary bromide. Write the
oo
structure of alkane and the tertiary bromide.
Ans. (0 We know that Wurtz reaction of an alkyl halide gives an alkane with double the number of carbon atoms
B

present in the alkyl halide. Since here Wurtz reaction of a primary alkyl halide gives an alkane with M.F.
CgHjg, therefore, the 1® alkyl halide must contain four carbon atoms. Now the two possible primary alkyl
re

halides having four carbon atoms each are I and II


CH,
I 3
u
ad

CH3CH2CH2CH2—X CH3—CH—CH2X
Yo

I n

(ii) Since the alkane CgH]g on monobromination yields a single isomer of a tertiary alkyl halide, therefore,
the alkane CgH^g must contain a tertiary hydrogen. This is possible only if the starting primary alkyl halide
d

has a tertiary hydrogen. Out of I and n, only primary all^l halide (II) has a tertiary hydrogen. In other
Re
in

words, the starting primary alkyl halide is l-hdo-2-methylpropan e (II). If this is so, then the alkane with
M.F. CgHjg must be 2, 5-dimethylhexane (III) and the monobromoderivative must be 2-bromo-2, 5-
dimethylhexane(IV).
F

CH
3\ /CH3
CH—CH2—X + im-iH j
A

CH3 CH3 -2NaX


l-Halo-2-methylpropane (two molecules)
1 6 1 Br 6
3 4 5
/CH3 Br2, Sunlight CH 3\ /CH,
CH—CH2—CH2—CH /C—CH,—CH,—CH
CH3-^2 3
-HBr
CH3 CH3 4 5
CH3
2, S-Dimethylhexane (III) 2-Bromo-2, 5-dimethylhexane (IV)

ft
13/162 “Pnadeefa^ '<t New Course Chemistiy (XI) msiiim

37. The ring systems having following characteristics are aromatic,


(i) Planar ring containing conjugated n-bonds.
(ii) Complete delocalisation of the ^-electrons in ring system, i.e., each atom in the ring has
unhybridised p>orbitaI, and
(ill) Presence of (4r + 2) 7C-electrons in the ring where r is an integer (r = 0,1,2,.......) [Hiickel rule].
Using this information classify the following compounds as aromatic/nonaromatic.

w
is
N

(A) (B) (C) (D)


(F) (G)

Flo
Ans. (A) = All the five carbon atoms and the nitrogen atom of pyridine are sp^-hybridized, the ring is planar, has
delocalised 6 Ji-electrons, follows Huckel rule. It is aromatic.

e
(B) = Has six 7C-electrons, but the delocalization stops at jp^-hybridized CH2 group, the ring is not planar,

re
does not follow Huckel rule. Therefore, it is not aromatic.
(C) = Six delocalised jc-electrons (4 Ji-electrons of the two double bonds +2 unshared electrons on the

F
negatively charged carbon), planar ring, all carbon atoms of the ring are ^p^-hybridized, follows Hiickel
rule. It is aromatic. ur
(D) = Has only 4 delocalised jc-electrons, does not follow Hiickel rule. It is antiaromatic.

r
(E) = Six delocalised 7C-electrons, all carbon atoms are sp^-hybridized, conjugation all over the ring due to

fo
the presence of positively charged carbon, the ring is planar, follows Hiickel rule. It is aromatic.
(F) = All carbon atoms are 5p^-hybridized, ring is planar, has 2 delocalised rc-electrons, follows. HUckel
ks
rule, i.e., (4n + 2) n-electrons where n = 0. It is aromatic.
Yo
(G) = Has 8 jc-electrons, but the delocalisation stops at jp^-hybridized CH2 carbon, does not follow Hiickel,
oo
i.e.y (4r + 2) jc-electrons rule. It not aromatic.
38. Which of the following compounds are aromatic according to Hiickle’s rule ?
B

o
re

(A) (B) (C) H2C CH2

H
u
ad
Yo

(D) (E) (F)


d
Re
in

Ans. (A) = Has 8 Tc-electrons, does not follow Hiickel rule, the orbitals of one double bond are not in conjugation.
It is not aromatic.
F

(B) = Has six delocalized Tc-electrons (4 7C-eIectrons of the two double bonds + two electrons of the lone
pair present on the N atom), all the four carbon atoms and the N atom are sp^-hybridized, the ring is planar,
follows Hiickel rule. It is aromatic.

(C) = Has 6 7C-electrons in conjugation, but not in the ring. Hence, is not aromatic.
(D) = Has 10 delocalized 7i-electrons, all the carbon atom atoms are jp^-hybridized, the ring is planar. It is
aromatic.

(E) = Has 8 7C-electrons ; out of 8 7t-electrons, it has delocalized 6 7C-electrons in one six-membered planar
ring which follows Hiickel rule. Therefore, it is aromatic.
(F) = Has 14 delocalized 71-electrons, all the carbon atoms are jp^-hybridized, ring is planar, follows Hiickel
where n = 3. It is aromatic.
13/163
HYDROCARBONS

39. Suggest a route to prepare ethyl hydrogen sulphate (CH3—CH2—OSO2—OH) starting from ethanol
(C2H5OH).
Ans. When ethanol is heated with cone. H2SO4 at 383 K {ie., 110°C), ethyl hydrogen sulphate is formed.
H2SO4 H+ + 'OSO2OH
Hydrogen sulphate ion

+ H

CH3—CH2—OH + H"" ♦ ► CH3—CH2—p H

low
Ethanol Protonated
ethanol

f\ + H 383 K
HO —SO2—O + CH3CH2-^0 > CH3—CH2—OSO2OH +H2O
H
Hydrogen Ethyl hydrogen sulphate
sulphate ion

e
Temperature should not be allowed to irse above 383 K, otherwise diethyl ether will be obtained at 413 K

re
rF
or ethene at 433-443 K.

F
MATCHING tYPE QUESTIONS
40. Match the reagent from Column I which on reaction with CH3—CH = CH2 gives some product given

or
in Column II as per the codes given below :
ou
Column II

sf
Column I

(i) 03/Zn + H2O (a) Acetic acid and CO2


(ii) KMn04/H-^ (b) Propan-l-ol
k
(i/0 KMn04/0H- (c) Propan-2-ol
oo
(IV) H20/H-*^ id) Acetaldehyde and formaldehyde
Y
(v) B2H(^/NaOH and H2O2 (e) Propane-1, 2-diol
B

(OO3
re

Ans. H)^id); CH3CH = CH2 CH3—CHO + 0 = CH2


(i7)Zn+H20 Fomialdehyde
Propene {Reductive ozonolysis) Acetaldehyde
u

KMn04/H+
ad

(lO-^(fl) ; CH3CH = CH2 » CH3COOH + CO2


Yo

(Oxidation)
Propene Acetic acid

tCMnO^/OH"
(/ii)^(e) ; CH3CH =; CH2 CH3—CHOH—CH2OH
(Hydroxytalion) Propane-1, 2-diol
d

Propene
Re
in

(iv)-Kc); CH3CH = CH2 > CH3—CHOH—CH3


(Mark.addn.)
Propene Propan-2-ol
F

(OB2H6
(v)^(i?) ; CH3CH = CH2 > CH3CH2CH2OH
(»)NaOHAl202
Propene (Hydroboration- Propan-l-ol
oxidation)

41. Match the hydrocarbons in Column I with the boiling points given in Column II.
Column I Column 11

(0 rt-Pentane ia) 282-5 K


{if) Isopentane (b) 309 K
(///) Neopentane (c) 301 K
13/164
New Course Chemistry fxnrosrwm
Ans. Boiling points decrease with branching. Therefore, «-pentane has the highest h.p. (309 K), followed by
isopentane with one branch (301 K) while neopentane with two branches has the lowest b p (282-5 K)
Thus, correladons are : {i)^{b), («)->(c), («i)->(a).
42. Match the following reactants in Column I with the corresponding reaction products in Coiumn II.
Column I Coiumn II

AICI3
(0 Benzene + CI2 (fl) Benzoic acid
AICI3
(«) Benzene + CH3CI ■>
ib) Methyl phenyl ketone
AiCl3

w
(///) Benzene + CH3COCI ■ (c) Toluene
KMn04/Na0H
(/v) Toluene id) Chlorobenzene

F lo
(e) Benzene hexachloride

Ans. (/)->(i/), chlorobenzene ; (h)->(c), toluene (F.C.


1 alkylation) ; (iii)->(b), methyl phenyl ketone (F.C.
acylation); benzoic acid (oxidation).

ee
43. Match the reactions given in Column I with the reaction types in Coiumn II.

Fr
Column I Column II
H+
(0 CH2 = CH2 + H2O

for
> CH3CH2OH (a) Hydrogenation
Pd
(«) CH2 = CHj + H2 - > CH3—CH3 (b) Halogenation
our
(in) CH2=CH2+Cl2- > Cl—CH2—CH2—Cl (c) Polymerisation
Cu tube
s
(/v) 3CHsCH (d) Hydration
ook
Heat

(e) Condensation
Y
eB

Ans. (0->(i/), hydration ; (H)->(fl), hydrogenation ; (i7i)->(i)). halogenation ; (iv)^(c), polymerization.

AS RTION AND REASON TYPE QUESTIONS


our
ad

In the following questions, a statement of assertion (A) followed by a statement of reason (R) is
given. Choose the correct option out of the choices given below :
(a) Both A and R are correct and R is the correct explanation of A.
(b) Both A and R are correct but R is not the correct explanation of A.
Y

(c) Both A and R are not correct.


Re

(d) A is not correct bur R is correct.


nd

44. Assertion (A): The compound cyclooctatetraene has the following structural formula :
Fi

It is cyclic and has conjugated 8 7C-electron system but it is not an aromatic compound.
Reason (R): (4n + 2) ^-electrons rule does not hold good and irng is not planar.
Ans. (a) Both A and R are correct and R is the correct explanation of A.
45. Assertion (A): Toluene on Friedel Crafts methylation gives o- and p-xylene.
Reason (R) : CH3-group bonded to benzene irng increases electron density at o- and p-position.
Ans. (a) Both A and R are correct and R is the correct explanation of A.

I
13/165
HYDROCARBONS

46. Assertion (A): Nitration of benzene with nitric acid requires the use of concentrated sulphunc acid.
Reason (R) ; The mixture of concentrated sulphuric acid and concentrated nitric acid produces the
electrophile, NO!^ .
Ans. (a) Both A and R are correct and R is the correct explanation of A.
47. Assertion (A): Among isomeric pentanes, 2, 2-dimethylpentane has the highest boiling point.
Reason (R) : Branching does not affect the boiling point.
Ans. (c) Correct A. Among isomeric pentanes. 2, 2-dimethylpentan e has the lowest boiling point.
Correct R. Branching decreases the boiling point.

w
LONG ANSWER QUESTIONS
48. An alkyl halide CsHuBr (A) reacts with ethanolic KOH to give an alkene ‘B’, which reacts with Br2

F lo
to give a compound ‘C’, which on dehydrobromination gives an alkyne ‘D’. On treatment with sodium
metal in liquid ammonia, one mole of ‘D’ gives one mole of the sodium salt of ‘D’ and half a mole of
hydrogen gas. Complete hydrogenation of ‘D’ yields a straight chain alkane. Identify A, B, C and D.
Give the reactions involved.

ee
Ans. The outline of reaction scheme involved in the given problem is

Fr
Ale. KOH Brj/CSj Ale. KOH Na-liq.NH_j
CjHiiBr CsHjo C5H,oBr2 -2HBr
-> C5H7Na+ I/2H2
(C) Alkyne (D) Sod. alkynide

for
Alkyl halide (A) Alkene (B)

(i) Since 1 mole of alkyne ‘D’ reacts with 1 mole of Na in liquid NH3 to form half a mole of H2, therefore,
(D) is a terminal alkyne. This means that triple bond is at the end of the carbon chain. The two structures
ur
for alkyne (D) are either (I) or (II)
oks
CH3
Yo
CH3CH2CH2—C = CH CH3—CH—C = CH
o

3-Methylbut-l-yne (II)
eB

1-Pentync (I)
Since alkyne ‘D’ on complete hydrogenation yields a straight chain alkane, therefore, the alkyne (D) is a
straight chain alkyne, i.e., alkyne (D) is 1-pentyne (I).
(ii) Since alkene (B) on reaction with Br2 forms a compound ‘C' which on dehydrohalogenation gives the
our
ad

alkyne, i.e., 1-pentyne (D), therefore, fC) must he }. 2-dibromopentane and alkene (B) must be 1-pentene.
(Hi) Further since alkene (B), i.e., 1-pentene is obtained by dehydrogenation of alkyl halide with M.F.
CjHjiBr, therefore, alkyl halide (A) must be I-bromopentane.
All the reactions involved in this question may now be explained as follows :
Y

1
Br^ in CS , 2
Re

Aic.KOH.A
CH3CH2CH2CH = CH2 ^ CH3CH2CH2—CHBr—CH2Br
nd

CH3CH2CH2CH2CH2Br -HBr

1-Pentene (B) 1, 2-Dibromopentane (C)


1-Bromopeniane (A)
Fi

Aic.KOH.A
Nainliq.NH3
CH3CH2CH2C = CH > CH2CH2CH2C = CNa + 1/2 H2
-2HBr
1-Pentyne (D) Sodium 1-pentynide

Plea.se note that alkyl halide (A) cannot be 2-bromopentane because dehydrobromination of (A) would
have given 2-pentene as the major product in accordance with Markovnikov's rule.
49. 896 mL vapour of a hydrocarbon ‘A’ having carbon 87-80% and hydrogen 12-19% weighs 3-28 g at
STP. Hydrogenation of ‘A’ give.s 2-methylpentane. Also ‘A’ on hydration in the presence of H2SO4
and HgS04 gives ketone ‘B’ having molecular formula CgHj20- The ketone ‘B’ gives a positive
iodoform test. Find the structure of ‘A’ and give the reactions involved.
13/166
T^^n^tdeefi. New Course Chemistiy rxT’^rasnwn
Ans. Step 1. To determine the molecular mass of hydrocarbon (A).
896 mL vapours of hydrocarbon (A) weigh at STP = 3-28 g.
3-28 x 22700
/. 22700 mL vapours of A will weigh at STP = gmoH =83-lg
896
/. Molecular mass of hydrocarbon (A) = 83-1 g mol“^
Step 2. To determine the empirical formula ofhydrocarbon (A).
Element %age Atomic mass Relative ratio Relative no. of atoms Simplest ratio
C 87-8 12 7-31 1 3
H 12-19 1 12-19 1-66 5
Thus, empirical formula of hydrocarbon (A) = C3H5
and empirical formula mass =12x3-t-5xl=41 u

w
Molecular mass 83-1
n = = 2-02 =2
Empirical formula mass 41

F lo
Thus, molecular formula of hydrocarbon (A) = 2 x Empirical formula = 2 x C3H5 =
Step 3. To determine the structures of compounds (A) and (B).
Hydrogenation
CgHio + 2H2 ^CH—CHj—CHj—CH3
CH3

ree
(A) 2-Methylpentane
(0 Since hydrogenation of hydrocarbon (A) requires 2 moles of hydrogen to form 2Tmethylpentane, therefore,

for F
hydrocarbon (A) is an alkyne having five carbon atoms in a straight chain and a methyl substituent at
position 2. Thus, the two possible structures for the alkyne (A) are I and II:
CH3 \ CH
CH—CsCH
C = C—CH3
CH3 CH3
Your
I II
ks

(ii) Since addition of H2O to alkyne (A) in presence of Hg^+ gives a ketone which gives positive iodoform
eBoo

test, therefore, ketone (B) must be a methyl ketone, Le., it must contain a COCH3 group.
Now addition of H2O to alkyne (II) should give a mixture of two ketones in which ketone (B) (which
shows -i-ve iodoform test) predominates.
CH3^
ad

H2O/H+ CH3 CH3


our

^CH—C = C—CH3 Hg2+


CH3' J>CH—CH2—COCH3 CH3'_1
+ ^CH—CO—CH2CH3
CH3
4-Methylpent-2-yne (II) 4-Methylpentan-2-one (B) 2-Methylpentan-3-one
(major) (minor)
Re

In contrast, addition of H2O to alkyne (I) wUl give only one ketone, i.e., 4-methylpentan-2-one which gives
Y

iodoform test.
Find

o
CH3 H2O/H+ CH
^CH—CH,—CsCH
CH3^ Hg
2+
CH3
^^CH—CH2—C—CH3
4-MethyIpent-l-yne (A) 4-Methylpentan-2-one (B)
(gives +ve iodoform test)
Thus, hydrocarbon (A) is 4-methylpent-J-yne.
50. An unsaturated hydrocarbon ‘A’ adds two molecules of Hj and on erductive ozonolysis gives butane-
1, 4-dial, ethanal and propanone. Give the structure of ‘A% write its lUPAC name and explain the
reactions involved.

f Jt
13/167
HYDROCARBONS

Ans. (0 Since the hydrocarbon (A) adds two molecules of Hj, therefore, (A) is either an alkadiene or an alkyne.
(if) On reductive ozonolysis ‘A’, gives three fragments, one of which is a dialdehyde. Therefore, the molecule
has undergone cleavage at two unsaturated sites. In other words, hydrocarbon A'has two double bonds,
le.. A is an alkadiene and not an alkyne.
(Hi) Place the three fragments of reductive cleavage side by side in such a way that the dialdehyde is placed
in the middle while the other two products on either side of the dialdehyde. Thus, we have,
XH3
CH3CH = 0 + 0 = CH-CH2CHj-CH = 0 + 0 =
Ethanal Butane-1, 4-dial
Propanone

w
Now, remove the oxygen atoms, and connect the remaining three fragments by double bonds, the structure
of the alkadiene (A) is

F lo
I
8 7 6 5 4 3 2 /CH3
2-Methylocta-2, 6-diene
CH3—CH =CH—CHj CHj—CH =C^
N:h3

ee
Step 3. To explain reacHons involved in the question.

Fr
CH3 O3/CCI4
●9 .CH3

CH—CH2CH2—^ C\

for
CH3CH= CH—CH2—CH2—C=C 196-200K
> CH3CH
CH3
I
Diozonide
o
ur
o
2-Methylocta-2,6-dicne (A)

CH3
s
CH3CH=0 + 0=CH—CH2CH2—CH=o + 0=cC^CH3
ook
Yo
Ethanal Butane-1,2-dial
Propanone
eB

51. In the presence of peroxide, addition of HBr to propene takes place according to anti-Markovnikov’s
rule but peroxide effect is not seen in the case of HCI and HL Explain.
Ans. In presence of peroxides, addition of HBr to propene takes place according to anti-Markovnikov^s rule.
The reaction occurs by a free radical mechanism as shown below :
our
ad

R—O^—R .^ ^
Homolybc fission
2r6
Peroxide
Y

RO + H-^Br ROH + Br
Re
nd

Slow
First step. CH3—CH=y=CH2 + Br > CH3—CH—CH2Br
Fi

Propene

Fast
Second step. CH3—CH—CH2Br + H-^Br > CH3—CH2—CH2Br + Br
1-Bromopropane

In case of HBr both these steps are exothermic (refer to table on page 13/40) and hence peroxide effect is
observed. However, in case of HCI or HI, either first or the second step is endothermic and hence peroxide
effect is not observed.
13/168
7>nadee^.'A. New Course Chemistry fXnrosTTn

SPECIAL

w
For ultimate preparation of this unit for competitive examinations, students should refer to

Pradeep's Stellar Series.... ● MCQs in Chemistry for NEET

F lo
● MCQs In Chemistry for JEE (Main) ●
separately available for these examinations.

Multiple Choice Questions (with One Correct Answer)

e
Fre
I. Alkanes 4. The number of structural isomers for C^H|4 is
(a) 3 (b)4
1. Which of the following alkane cannot be made in
i

for
(c)5 (d)6
good yield by Wurtz reaction ?
(IIT Paper I, 2007, Kerala PET 2018)
(fl) 2, 3-Dimethylbutane
(b) n-Heptane
r
5. The alkyl halides required to prepare
(c) /i-Butane by Wurtz reaction are
You
oks
(d) n-Hexane (NEET 2020)
(a) and
NaOH + ? Cl Cl
eBo

2. CHjCHjCOO-Na-" Heat Cl

CHj—CHj + Na2C0j (b)


Cl
(a) (b) Red phosphorus
(c) CaO (d) DffiAL-H Cl
ad
our

and
(NEET 2021) (c)
Cl
3. The major product of the following reaction is
id) -|- and (Karnataka CET 2019)
H H H CH, Cl Cl
I I I I ^ M0203
6. The products expected to be formed in the Wurtz
dY
Re

CH,—C—C—C—CH
3
I
Heat
reaction of a mixture of neopentyl bromide and
H H H CH3 isobutyl bromide are
Fin

(/) 2, 2, 4-trimethylpentane
CH3
(fi) 2, 2, 5, 5-tetramethylhexane
(^) (b) (Hi) 2. 2, 4, 4-tetramethylhexane
(/V) 2, 5-dimelhyIhexane
(v) 2, 2, 5-trimethylhexane
(c) (d)
(a) (ii), (Hi) and (v) (b) (H), (/V) and (v)
(c) (0, (iV) and (v) (d) (i), (Hi) and (v)
(JEE Main 2020) (e) (/), (H) and (iv) (Kerala PMT 2015)
ANSWERS
1. (b) 2. (c) 3. (<i) 4. (c) S. (c) 6. (b)
13/169
HYDROCARBONS

7. The best method for the preparation of 2, 2- 12. When «-hexane is heated with anhydrous AICI3
dimethylbutane is via the reaction of and HCl gas, the major product obtained is
(a) 1 -chlorohexane (h) 2-chlorohexane
(a) Me3CBr and MeCH2Br in Na/ether
(c) 3-chlorohexane id) hex-3-ene
(b) (Me3C)2CuLi and MeCH2Br
ie) mixture of 2-methylpentane and
(c) (MeCH2)2CuLi and Me3CBr
3-methylpentanc (Kerala PET 2014)
(d) Mc3CMgI and MeCH2l
(West Bengal JEE 2013) VjOs.SOO-C Clj/Ziv
> A > B
13. n-C7Hig
8. Hydrocarbon (A) reacts with bromine by 10-20alm

substitution to form an alkyl bromide which by What is B in the above reaction ?


Wurtz reaction is converted to gaseous ib) Benzal chloride
(a) Benzyl chloride

w
hydrocarbon containing less than four carbon (c) Hexachlorobenzene
atoms. (A) is
(a) CH^CH ib) CH2 = CH2 id) Benzene hexachloride (AMU Engg. 2014)

F lo
(C) CH3-CH3 (£/) CH4 (NEET 2018) 14. The product formed in the following reaction is
9. Isomers of hexane, based on their branching, can
be divided into three distinct classes as shown in CH CH3
I
3\ H-^
the figure. C = CH2 + H-C-CH3 ^ ?

ee
CH3
CH3

Fr
I. and

CH
3\ /CH3

for
(«) CH-CH2CH2-CHv^
II. and CH3 CH3
r
CH3
You
CH
W\ 3\
s
III.
ib) CH-CH2-C-CH3
ook
CH'
The correct order of their boiling point is
CH3
eB

ia) I>n>III ib) III > II > I


(c) ii>m>i id) III>I>II
/CH3
(JEE Advanced 2014) ic) CH,-CH-CH-CH^
10. A flask contains a mixture of isohexane and 3- M l ^ CH
our
ad

methylpentane. One of the liquids boils at 63°C CH3CH3


while the other boils at 60°C. What is the best way
to separate the two liquids and which one will be CH-CH.
distilled first ?
I ^
id) CH3—C—C—CH3 (JEE Main 2022)
Y

ia) fractional distillation, isohexane


Re

ib) simple distillation, 3-methylpentane CH3CH3


d

(c) fractional distillation, 3-methylpentane 15. With respect to the conformers of ethane, which
Fin

id) simple distillation, isohexane of the following statements is true ?


(JEE Main 2020) (a) Bond angle changes but bond length remains
11. The total number of monohalogenated products same

formed by halogenation of 2,2,4-trimethylhexane ib) Both bond angle and bond length change
IS
(c) Both bond angle and bond length remain same
(a) 5 ib) 7
id) 8
(£0 Bond angle remains same but bond length
ic) 6 (NEET 2017)
changes
ie) 0 (Kerala PET 2016)
ANSWERS

7. ib) 8. i(f) 9. ih) 10. (fl) 11. ic) 12.ic) 13. ia) 14. ih) 15.ic)
13/170
'4. New Course Chemistry (XI)
16. Dihedral angle of least stable conformer of ethane
IS
CH3 H
(a)0" {b) 120“
(c) 80“ (d)60“ (NEET 2020)
17. The dihedral angle between two C—H bonds in
the staggered conformation of ethane is H
CH3
(a) 180“ (b) 0“
(c) 120“ (d) 60“ (JEE Main 2022)
18. The conformation of n-butane, commonly known 21. Newmann projections P. Q, R and S are shown
below :
as eclipsed, gauche and anti-conformations can be
interconverted by

w
(a) rotation around C-H bond of a methyl group
CH3
(b) rotation around C—H bond of a methylene

F lo
group

(c) rotation around C,-C2 linkage C2H5


(d) rotation around C2-C3 linkage

e
(West Bengal JEE 2019)

Fre
19. Which of the following conformer for ethylene
glycol is most stable ?

for
r
You
oks
5
eBo
our
ad

(AIPMT Mains 2010) C2H5


dY
Re

20. In the following structures, which one is having R

staggered conformation with maximum dihedral


CH3
Fin

angle ?

CH3 CH(CH3)2
3

OH
S

16. (a) 17. ui) 18. (d) 19. (r) 20. (v)
HYDROCARBONS 13/171

Which one of the following options represent 26. Give the lUPAC name of the alkene
identical molecules ?

{a) P and Q {b) Q and S


(c) Q and R {d) R and S
(JEE Advanced 2020) {a) Z-3-methyl-4'propyl-3-oclene
11. Alkenes (h) E-3-methyl-4-propyl-3-octene
(c) E-4-butyl-3-methyl-3-heptene
22. The correct order of decreasing H-C-H angle in (rf) E-2-ethyl-3-propyl-2-heptene
the following molecule is (e) Z-2-ethyl-2-propyl-2-heptene
H H 27. Among the following, geometrical isomerism is

X .S'

w
exhibited by
H H
CH. CHCl
1 11 III

F lo
(fl) I > II > III {b) II > I > III
(a) (b)
(c) III > II > I id) I > III > II
(West Bengal JEE 2014) CHj
Cl
23. Which of the following compounds will exhibit

e
Fre
geometrical isomerism ? CHCl CHCI

(a) 2-Phenyl-l-butene
(b) 1, 1-Diphenyl-l-propene

for
ic) id)
(c) 1 -Phenyl-2-butene H3C CH3
id) 3-Phenyl-1-butene (JEE Main 2015)
r
(JEE Main 2020)
24. Which of the following compounds exhibits 28. The correct order of heat of combustion for the
You
geometrical isomerism ?
oks
following alkadienes is
(a) C.H^Br ib) (CH)2(COOH )2
y
o

(c) CH3CHO (d) (CH2)2(C00H)2- y


eB

HI.
25. The Z-isomer among the following is
●y ”y
CH3 .CH2CH3 (a) 1< II <111 (b) II < III < I
(a) :c=c (c) III < 11 < 1 id) I < III < II
our
ad

Cl Br (JEE Main 2020)


29. Acid-catalysed hydration of alkenes except ethene
CH3 CH^OH
leads to the formation of
ib) C=C
H
(a) primary alcohol
CHO
dY

(b) secondary or tertiary alcohols


Re

ic) mixture of primary and secondary alcohols


(d) mixture of secondary and tertiary alcohols
Fin

30. Acid catalysed hydration of alkene is an example


for

(a) free radical substitution


ib) nucleophilic substitution
COOH (c) nucleophilic addition
(AMU Engg. 2011) id) electrophilic addition
H ie) electrophilic substitution (Kerala PMT 2015)
AN3WER1

21. ic) 22. ib) 23.ic) 24. ih) 25.(u) 26. (n) 27. ib) 28.(n) 29. ib) 30. id)
13/172 New Course Chemistry (XI)S!Z5IHD

CH3 Which of these reaction(s) will not produce


H2SO4 Saytzeff product ?
31. CH3CH2—C—CH—CH3 heal
> P
(a) (Hi) only (b) (H) and (/V)
(Major)
CH3OH (c) (0, (Hi) and (iv) (d) (iv) only
What is the major product P in the above reaction ? (JEE Main 2020)
35. When neopentyl alcohol is heated with an acid, it
CH3 is slowly converted into an 85 : 15 mixture of
(a) CH3CH2—CH—CH=CH2 alkenes A and B, respectively. What are these
alkenes ?
CH, CH,
I I CH2
H,C
/ H3C
yCH3
(b) CH3 CH—CH—CH=CH2 (a)
> and
>

w
HjC CH2
CH3 CH3

F lo
(c) CH3CH2—C—CH=CH2 H3C ,CH3

CH3
(b)
H3C
> and H3C

CH2
CH3CH3
CH3
^CH3 CH3
/H2

e
(d) CH3CH2—C = C—CH3
> >
(Aims 2015)

Fre
(c) and

32. Cyclohexene is best prepared from cyclohexanol CH2 CH3


by which of the following

for
H3C CH3 H3C .CHj
/
(a) cone. H3PO4
(c) cone. HCl
(b) cone. HCl/ZnCl2
(d) cone. HBr
id)
H3C
> and

H3'
33. Which of the following is not the product of
r
You
(JEE Main 2020)
oks

dehydration of OH ? 36. The major product in the following reaction is


eBo

H3C CH = CH2

(a) (b)
ad
our

CH3
(a) < CH3
(c) id)
Re

(RE-AIPMT 2015)
dY

H3C CH3
34. Consider the following reactions : X CH3
ic)
Fin

cone. H,SO^
(/) (CH3)3CCH(0H)CH3 ■>

ale. KOH
(JEE Main 2020)
(i7) (CH3)2CHCH(Br)CH3 ■>

37. 2,3-Dimeihyl-2-butene can be prepared by heating


(CHjljCO-K*
(Hi) (CH3)2CHCH(Br)CH3 - ■> which of the following compounds with a strong
acid ?
A

(iv) (CH3)2C—CH2 —CHO (a) (CH3)3C—CH = CH2


OH (b) (CH3)2C = CH—CH2CH3
ANSWERS

31. (d) 32. (a) 33. (a) 34. (a) 35. (h) 36. (/?)
HYDROCARBONS 13/173

(c) (CH3)2CH—CH2—CH = CH^


id) HO CH2-CH-CH3
id) (CH3),CH—CH—CH=CH2
Br
CH3 {JEE Main 2019)
(RE-AIPMT 2015)
42. Elimination reaction of 2-bromopentane to form
38. Addition of HBr to 2-pentene gives
pent-2-ene is
id) 2-broniopentane only
(f) P-eliminalion reaction (ii) follows Zaitsev rule
ib) 3-brompentane only
(m) dehydrohalogenationreaction
(c) 2-bromopentane and 3-bromopentanc
(iv) dehydration reaction
id) 1-bromopentane and 3-broniopenlane

w
(West Bengal JEE 2009) id) (0, (m), (iv) ib) iii), (m), (fv)
39. In the reaction with HCI, an alkene reacts in (c) (0, iii), iiv) id) (0, Hi), HU)
accordance with the Markovnikov’s rule to give a (NEET 2020)

F lo
product 1-chloro-l-methylcyclohexane. The 43. The major product of the following reaction
possible alkene is
CH3
CH2 CH

ee
2 HBr ^
(A) ib) (B)

Fr
ia)

NO2
CH3

for
IS

CH3
ur
ic) (A) and (B) ib)
H3C Br

X Br
s
(RE-AIPMT 2015) (o) ib)
ook
XBr XBr
Yo
40. HBr reacts with CH2 = CH—OCH3 under
anhydrous conditions at room temperature to give O2N 0,N
eB

ia) CH3CHO and CH3Br


CH3
ib) BrCH2CHO and CH3OH Br CH3
ic) BrCH2—CH2—OCH3 Br
X
id) H3C-^HBr—OCH3 ic) id)
our
ad

41. The major product in the following conversion is Br

HBr (excess) NO, NO2


CH3O CH = CH - CH3 Heat ^
(JEE Main 2020)
Y

44. The major product obtained from E2-elimination


Re

ia) CH3O CH-CH2CH3 of 3-bromo-2-fluoropentaneis


nd

Br Br
Fi

(a) CH3CH2 —CH—CH = CHj


ib) HO CH-CH2-CH3
Br
Br I
ib) CH3CH2 —C = CH—CH3
ic) CH3O CH2 - CH - CH3 F

Br ic) CH3 — CH = CH —CH—CH3

37. {a) 38. (c) 39. (c) 40. Uf) 41. (//i 42. (d) 43. (d)
13/174 Nriw Course Chemistry (XI)SS5JHD

id) CH3CH2CH = C—F (JEE Main 2020) 49. The major product obtained by addition reaction
of HBr to 4-methylpent-l-ene in the presence of
CH3 peroxide is
45. The major product formed in the following (a) l-bromo-4-methylpentane
reaction is {b) 4-bromo-2-methyIpentane
HBr (c) 2-bromo-4-methyIpentane
CH3CH = CHCH(CH3)2
{d) 3-bromo-2-methylpentane
(a) Br(CH2)3CH(CH3)2
(e) 2-bromo-2-methylpentane
(b) CH3CH(Br)CH2CH(CH3)2 (Kerala PET 2016)
(c) CH3CH2CH(Br)CH(CH3)2

w
ROOR
id) CH3CH2CH2C(Br)(CH3>2 50. CH3CH2-CH = CH2 + HBr ^ [X] + [Y]
(JEE Main 2020) Major Minor

F lo
[X] and [Y] respectively are
HBr
46. CH3 A (a) BrCH2CH2CH = CH^ and
CCU^ {major product)
C2H5—CHBr—CH3
Product ‘A’ in the above chemical reaction is

ee
(b) C2H5—CH2CH2—Br and
Br Br
Br—CH2CH2—CH = CH2

Fr
(a) CH3 ib)
(c) C2H5—CH2—CH2Br and
C2H5—CHBr—CH3

for
Br
(d) C2HgCHBr—CH3 and C2H5—CH2—CH2Br
ur
Br (AlIMS 2015)
(c)
CH3
CH3 id) 51. The major product of the following chemical
reaction iss
s
CH3 ‘3
ook
Yo
(JEE Main 2021) CH3 (C^HjCO^Oj
47. Which of the following compounds shall not CH —CH = CH2 + HBr ^ ?
CH3
eB

produce propene by reaction with HBr followed


by elimination or direct only elimination reaction ?
CH 3

(a) H3(>^CH2 ia) CH—CH2—CH2—Br


our

CH
C 3
ad

H2
CH
H 3\
ib) CH — CH2 — CH2 — O — COC^Hg
ib) H3C—C—CH2OH CH3’
Y

(c) H2C = C = 0
Re

CH
3\
nd

H2 ic) CH—CH—CH3
(d) H3C —C—CH2Br (NEET Phase II 2016) CHg-
Fi

Br
48. The reaction of propene with HBr in presence of
peroxide proceeds through the intermediate
CH
3\
(a) H3C-(:H-CH3 ib) H3C-CH-CH2Br id) CBr —CH2 — CH3 (NEET 2021)

Br CH3
52. Which one of the following alkenes when treated
(c) H3C-CH-CH2 (d) H3C-CH2-CH2 with HCl yields majorly an anti-Markovnikov’s
(e) none of those (Kerala PET 2017) product ?

44. id) 45. id) 46. ih) 47. (c) 48. (b) 49. («) 50. (c) 51. (n)
13/175
HYDROCARBONS

(fl) CH3O-CH = CH2 ib) Cl-CH = CH2 55. The major product ‘P’ in the reaction
(c) H2N-CH = CH, id) F3C-CH = CH2 Ph Br HBr
♦ P
(JEE Main 2019)
53. Major product ‘B* of the following reaction Br
sequence is Br

Br2 HI (a) Ph Br (b) Ph


CH3-C = CH-CH3 CH3OH
A 4 Br

CH3 B
Br

(Major product)
Br id) Ph Br
(c) Ph
OH Br
I 1
{a) CH3—C—CH—CH3

w
Br Br

CH3

F lo
(JEE Main 2022)

1 Br Ci
I
ib) CH3—C—CH—CH3 A
56. >
CH3

ree
for F
Br OH Cl

(c) CH3—C—CH—CH3 Identify the reagent ‘A’ and conditions(s) for the
above reaction.
CH3 (a) A = HCl, Anhyd. AICI3
our
(b) A = HCl. ZnCl2
Br I
ks

(c) A = CI2, dark, Anhyd. AICI3


eBoo

id) CH3—C—CH—CH3 (JEE Main 2022) (d)A^ CI2. UV light (JEE Main 2021)
I
Y

CH3 57. When ethanol is heated with Cone. H2SO4, a gas


is produced. The compound formed when the gas
54. Among the following the major product of the is treated with cold dilute aqueous solution of
ad

given chemical reaction is Baeyer’s reagent is


our

(a) formal dehyde ib) formic acid


Brj
+ Major Product (c) glycol id) elhanoic acid
CH3OH
O (JEE Main 2022)
Re

Br Br 58. A compound ‘A’ on reaction with ‘X* and ‘Y’


Y

produces the same major product but different by


(«) {b) product ‘a’ and 'b'. Oxidation of 'a' gives a
Find

OCH3 OCH3 substance produced by ants. ‘X’ and ‘Y’


respectivelyare :
.OCHj (a) KMn04/H'*’ and dil. KMn04, 273 K
ssOCHj
ib) KMn04 (dil.) 273 K and KMn04/H+
ic) id)
(c) KMn04/H'*' and O3,
Br O ’'%Br
id) O3, H20/Zn and KMn04/H’^
(JEE Main 2022) (JEE Main 2022)
ANSWERS

SI. id) 53.(6) 54.(0) 55. (c) 56. (J) 57. (c) 58. (rf)
13/176
New Course Chemistry (XI)SZsIMD

59. Two alkenes ‘A’ and ‘B’ on reductive ozonoiysis CH3 CH3
give glyoxai, elhane-I, 2-dicarbaldhyde and 5-
oxohexanal. ‘A’ and ’B’ respectively are : CH3
{■<■■) id)
(a) l-meihylcyciohexa-1, 3-diene and cyclo-
pendene
3
(^7) cycIohexa-1, 3-diene and cyclopentene
(c) l-methyIcyc!ohexa-l. 4-diene and I-methyl- (JEE Main 2015)
cyclopent-l-ene 63. A single compound of the structure,
(i/) cyclohexa-1, 3-diene and i-methylcydopcjit- CH3 CH,
1-ene (JEE Main 2022)
60. Two isomers ‘A’ and ‘B’ with molecular formula OHC C C

w
C4Hg give different products on oxidation with
KMn04/H'*’, ‘A’ results in efferuescence of a gas H2 H2

F lo
and gives a ketone. The compound ‘A’ is is obtainable from ozonoiysis of which of the
(a) But-l-ene (b) c75-But-2-ene following cyclic compounds ?
(c) /ranj'-But-2-ene (d) 2-metliylpropene HjC CH3
(JEE Main 2022)

ee
61. An alkene on ozonoiysis gives methanai as one of

Fr
the products. Its structure is
CHj—CH2 —CH3

for
(a)
r
You
CH2—CH=CH2
s
(AIPMT 2015)
ook

64. Ozonoiysis of an alkene produces only one


(b) dicarbonyl compound. The structure of the alkene
eB

IS

CH2CH2CH3 (a) CH3-CH = CH-CH3


our

(b)
ad

(c)
(c)
(d) CH3-CH = CH-CH = CH2
CH=CH—CH3 (West Bengal JEE 2016)
65. An alkene A on reaction with O3 and Zn-H20
dY

gives propanone and ethanal in equimolar ratio.


Re

(^0 (NEET 2020)


Addition of HCl to alkene A gives B as the major
product. The structure of product B is
Fin

62. Which compound would give 5-keto-2-methyl-


hexanal on ozonoiysis ? /CH3
(«) H3C—CH—
CH3 CH3
Cl

^CH3
(b) CICH2—CH2—CH
CH3

59. id) 60. (cO 61. (b) 62. id) 63. (c) 64. ih)
HYDROCARBONS 13/177

CHXl 111, Alkvncs


1 L . 1

(c) H3C—CH2—CH—CH3 69. The major organic compound formed by the


reaction of 1, I. 1-trichloroethane with silver
CH3 powder is
(fl) 2-butene (b) acetylene
(d) H3C—CH2—C—CH3 (NEET 2019)
(c) ethene (d) 2-butyne
Cl
(JEE Main 2014)
66. The major products obtained during ozonolysis of 70. The reagents for the following conversion
2,3-dimethyl-1-butene and subsequent reduction Br 9

with Zn and H2O are \/\ Br > H H is/are


(a) methanoic acid and 2-methyl-2-butanone
(a) alcoholic KOH
{b) methanal and 3-methyl-2-bulanone
ib) alcoholic KOH followed by NaNH2
(c) methanol and 2,3-dimethyl-3-butanone

F lo w
(c) aqueous KOH followed by NaNH2
id) methanoic acid and 2-methyl-3-butanone
(West Bengal JEE 2016) (d) Z11/CH3OH (IIT Paper 1, 2007)

67. The major products of the following reaction are :


71. The major product of the following reaction is
(i)alc.KOH
CH3 CH3CH2—CH—CH2 (I'ONaNHj in tiq.NHj
<j)KOtBu,a
CH,—CH — CH—CH, Br Br
(ii)03/H202
I
OSO2CH3 ia) CH3CH = C = CH2

re
for F
(b) CH3CH2C = CH
CH 3

(a) + CH3COOH (c) CH3CH2—CH—CH2


CH3 0
NH, NH2
CH3 id) CH3CH = CHCH2NH2 (JEE Main 2019)
Your
(6) + HCHO 72. Which of the following will most readily give the
s
eBo k

CH3 CHO dehydrohalogenation product ?


Br Br
CH3
(a) ib)
ic) + HCOOH
ad

COOH
CH3
our

Br Br

CH3 Ph
id)
ic)
id) + CH,CHO
3
(JEE Main 2020) H
O
CH3
Re

(JEE Main (online) 2018)


73. The correct order of increasing basicity of the
Y

68. Products of the following reaction given conjugate bases (R = CH3) is


Find

(OO3
^ ? are (rt) RCOO- < HC e C“ < NH“ < R-
Mc2C = CHCH3
(n) (CH3)2S
(/>) RCOO- < HC s C- < R- < NH-
ia) CH3CHO + CH3COOH
(c) R- < HC = C“ < RCOO- < NH-
ib) Mc2CO + CH3CHO
(c) Me2CO + CH3COOH id) RCOO- < NH- < HC s C- < R-
id) 2 Me^CO (AIEEE 2010)

ANSWERS

65. id) 66.ib) 67.(c) 68.ib) 69. id) 70.ib) 71.ib) 72.(c) 73. (a)

i
13/178
New Course Chemistry (XI)EJEIHD

74. Which one is the correct order of acidity ? Me—= Me

(a) CH = CH > CH3-C = CH


> CH2 = CH2 > CH3-CH3 O

(b) CH H CH > CH2 = CH2 (a) HgS04, dil. H2SO4 (/?) BH3 ; H20,/Na0H
> CH3-C = CH > CH3 - CH3 (c) OSO4 : HIO4
(c) CH3-CH3 > CH2 = CH2 > CH3-C s CH (d) NaNH2/CH3l : HgS04/dil. H2SO4
>CH=CH (West Bengal JEE 2014)
i_d) CH2 = CH2 > CH3-CH = CH2 79. In the reaction,

> CH3-C = CH > CH s CH (/)NaNH2/liq.NH3


(NEET 2017) H—C = C—H ^ X
(»)CH3CH2Br
75. The correct order for the acid strength of
(ONaNHi/liq.NH,
compounds HC = CH, CH3-C s CH and CH^ = ^ Y

F low
CH2 is as follows : (n)CH3CH2Br
(a) CH3-C = CH > HC = CH > CH2 = CH^ X and Y are

(b) CH3-C = CH > CH2 = CH2 > HC = CH (a) X= 2-butyne, Y = 2-hexyne


(c) HC = CH > CH3-C = CH > CH2 = CH2 (b) X= 1 -butyne, Y = 2-hexyne
(d) HC = CH > CH2 = CH2 > CH3-C = CH (c) X = I-butyne. Y = 3-hexyne
(JEE Main 2019) (d) X= 2-butyne, Y = 3-hexyne
(NEET Phase-I 2016)
76. The compound that will react most readily with

re
gaseous bromine has the formula 80. The most suitable reagent for the following

for F
conversion, is
(«) C3H, (b) C2H2
(c) C4H,o (d) C2H4 H3C\^ /CH3
(NEET Phase-n 2016) H3C-C5C-CH3
H H
77. Predict the correct intermediate and product in the ci's-2-butene
following reaction :
(a) H2O (h) Na/Iiquid NH3
Your
H20.H2SO4
s

(c) H2, Pd/C, quinoline


eBo k

H3C—C = CH
HgS04 {d) Zn/HCl (NEET 2019)
Intermediate Product 81. The rra/i.v-alkenes are formed by the reduction of
(A) (B) alkynes with :
{a) H2, Pd/C, BaS04 {b) NaBH4
ad

(fl) A:H3C—C=CH2 B:H3C—C = CH2 (c) N^liq. NH3


our

I (^0 Sn/HCl
OH SO 4 (JEE Main 2018)
82. The isomerisation of 1-butyne to 2-butyne can be
{b) A:H3C—C—CH3 B : H3C—C B CH achieved by treatment with
Re

0 (fl) hydrochloric acid


{b) ammoniacal silver nitrate
(c) A:H3C—C=CH2 B : H3C—C—CH3
Y

(c) ammoniacal cuprous chloride


Find

OH 0 (</) ethanolic potassium hydroxide


(West Bengal JEE 2017)
{d) A:H3C—C=CH2 B : H3C—C—CH3 83. Identify X in the following sequence of reactions :
I
SO4 O
CH3 - CH - CH -CH2 - CH2 - CH3
(NEET 2017) Br Br
1. NaNH2
78. The reagents to carry out the following conversion 4 X
are 2.Na in liq. NH3

ANSWERS

74. (fl) 75. (c‘) 76.(0) 77. (c) 78. («f) 79. (c) 80. (c) 81. (c) S2. (d)

t
13/179
HYDROCARBONS

O3 Zn/CHjOH
ia) CH3 - CH - CH -CH2CH2CH3 89. CH = CH X ^ Y, Y is
Br NH2 ia) CH2OH—CH2OH (b) CH3COOH
(c) C2H5OH id) CH3—CH3
CH. H
90. Which of the following is a free radical
(h) C = C
substitution reaction ?
^CH2CH2CH3
(a) Acetylene with HBr
CH
3\ ^CH2CH2CH3 ib) Methane with Br2//iv
(C) C = C
H H (c) Propene with HBr (C^H5COO)2

w
id) Benzene with Br2/ArCl3 (NEET 2020)
id) CH3 - CH — CH -CH2CH2CH3
NH2 NH2 H H (ONaNHj
91. :C = C > Major product
{West Bengal JEE 2016)

F lo
Br (r7) Red hot
H3C Fetube,873K
84. Catalyst used in dimerisation of acetylene to
prepare chloroprene is
CH3
(a) HgS04 + H2SO4 ib) CU2CI2

ee
(c) CU2CI2 + NH4OH id) CU2CI2 NH4CI

Fr
(AITVIS 2010) ia) ib)

85. Which of the following reagents will be able to


distinguish between 1-butyne and 2-butyne? CH3 CH3

for
ia) NaNH2 ib) HCl
ur
(c) O2 id) Br2 ic) id)
(AIPMT Mains 2012)
CH3
s
86. 1-Butyne reacts with hot alkaline KMn04 to CH3
ook
CH3
Yo
produce
ia) CH3CH2CH2COOH (JEE Main 2021)
eB

ib) CH3CH2COOH 92. The major product (R) in the following sequence
of reactions is
(c) CH3CH2COOH + CO2
id) CH3CH2COOH + HCOOH (AIIMS 2014) (i)UNH2/ctlicr (.●)HgS04/H,S04
HC = CH ^ [P] ■»
87. A compound X (CgHg) reacts with ammoniacal
r
ad
ou

AgN03 to give a white precipitate, and on an CH-Br

oxidation with hot alkaline KMn04 gives the acid, (CH3 )2CH
(CH3)2CHC00H. Therefore, X is Conc.HjSO^
Y

ia) CH2 = CHCH = CHCH3 [Q1 ^ [R1


A

ib) CH3(CH2)2C = CH
Re
nd

ic) (CH3)2CH—C = CH
ia) C-CH2-CH3
(d) (CH3)2C = C = CH2.
(CH3)2CH
Fi

(«)X
88. CH3 sC—CH3
(») H2O/Z11 HsCs^
ib) C = CH-CH3
CH.—C—C—CH, Xis iCH^)2CH
' I I '
O 0

ia) HNO3 ib) O2 (c) C = CiCHj)^


(c)03 id) KMn04 CH3CH2

ANSWERS

83. ib) 84.id) 85.(«) 86.(c) 87. (c) 88. (c) 89. (ti) 90. ib) 91. (c)
13/180 New Course Chemistry (XI)E2sXX/]

H3CX II [b) I, m
{d)
^CH-CH = CH2 (c) I, IV {d) II, III
(CH3 )2CH (JEE Main 2022)
(JEE Main 2020)
97. Which of the following structures are aromatic in
93. What will be the major product of the following nature ?
sequence of reactions.
(0 n-BuLi I.
n-Bu H Major Product
.Cl
(«)
{Hi) Ltndlar’s catalyst {a) I, II, III and IV {b) Only I and II
/l-Bu /i-Bu
(c) Only I and III id) II, m and IV

w
(a) / {b)
(JEE Main 2022)
HjiCj-w n-Bu rt-Bu
71-C5HJ1 98. Which of the following is most stable ?

F lo
+

n-CsHii
\ (a) ib)
(c) n- Bu
\ n-Bn
V=/

ee
(JEE Main 2022) (c)

Fr
id)
IV. A. ncs
(JEE Main 2022)
94. Which one of the following is an aromatic

for
compound ?
99. Which one of the following compounds is
aromatic in nature ?
ur
(a) Cyclopentadienyl cation
(b) Cycloheptatrienyl cation 3

(c) Cycloheptatrienyl anion


oks
(d) Cycloheptatriene (a) ib)
Yo
(e) Cyclopentadiene (Kerala PMT 2015)
o

95. Which of the following compounds is not


eB

aromatic ?

ic)
ia) ib)
our
ad

+
(JEE Main 2021)
100. The decreasing order of reactivity of the following
ic) H-
H id) organic molecules towards AgN03 solution is :
Cl Cl
Y

(JEE Main 2022)


Re

96. Which of the following is not an example of a


nd

benzenoid compound ?
(A) OMe
Fi

(B)
1. H 11.
CHj-CH-CH3 CH3 -CH-CH2NO2
\=/ Ci Cl
(C) (D)

NH2 (a) (C) > (D) > (A) > (B) (b) (A) > (B) >(D) > (C)
III. IV. (c) (A) > (B) > (C) > (D) (d) (B) > (A) > (C) > (D)
(JEE Main 2020)
ANSWERS

92.(c) 93.(c) 94.(b) 95.(c) 96. (a) 97.(b) 98.(d) 99. (a.d) 100. (d)
13/181
HYDROCARBONS

101. From the following compounds, choose the one


which is not aromatic ?
/=\
(a) ib) I
\=^

(c)

(West Bengal JEE 2016) (a) Zero


102. Which of the following compounds are aromatic ? (West Bengal JEE 2015)

w
107. Given :

H3C. ,CH3 H3C ,CH2

F lo
H
A B c D

(fl) A, B {b) A, B. C CH3 CH3

e
II
(c) B.C (d) B. C, D I

Fre
(e) A, B. D (Kerala PET 2018)
H2C,_^CH2
103. The radical CH2 is aromatic because

for
it has

(a) 7 p-orbitals and seven unpaired electrons CH2


III
(b) 6 p-orbitals and 7 unpaired electrons
r
The enthalpy of hydrogenation of these
(c) 6 p-orbitals and 6 unpaired electrons
You
compounds will be in the order as
oks

(d) 1 p-orbitals and 6 unpaired electrons (a) II > III > I {b) II>I>III
(AIPMT 2013)
eBo

(c) I>II>m id) III > II > I


104. Cycloheptatrienyl cation is (AIPMT 2015)
(a) non-benzenoid and non-aromatic 108. In the given reaction,
ib) non-benzenoid and aromatic HF
ad
our

+ >P
(c) benzenoid and aromatic 0°C

id) benzenoid and non-aromatic


the product P is
(e) non-benzenoid and antiaromatic
(Kerala PET 2017)
ia) ib) ^ F
dY
Re

105. The most likely protonation site in the following


molecule is
Fin

(c) (d)

ia) C-1
(c) C-3 ib) C-6
(West Bengal JEE 2014)
106. The total number of aromatic species generated
(NEET Phase-ll 2016)
in the following reactions is
ANSWERS

101. ib) 102. (fl) 103. (c) 104. ib) 105. (n) 106. (c) 107. id) 108. (c)
13/182
'«t New Course Chemistry (XI) eto au
109. Which of the following reactions would give D
isopropylbenzene as the major product ?
D. D
D
/=
I. II. > (c) D.
AIC13 H2SO4 D

III.
^OH ^C1 D" D
D

IV.
H2SO4 AICI3 D

D
{a) I and IV only
(d) D. H
(b) II and III only
(c) II, m, IV only

w
D D
(d) All of the above (AMU Med. 2013)
D
110. Reaction of benzene with Me3CCOCl in the (West Bengal JEE 2017)

F lo
presence of anhydrous AICI3 gives 112. Aromatic electrophilic substitution reaction that
is reversible is
MejC O
(a) nitration (b) chlorination
CMej
(c) sulphonation
rS
(d) alkylation

e
Fre
(a) (b) (e) acylation (Kerala PET 2018)
113. The compound X in the reaction

for
Anhyd. AICI3
,0 + IC1 > X
(c) MeaC
CMej
r
(a) Cl (b)
O---AICI3
You
oks

id)
Cl
eBo

CMej

(West Bengal JEE 2013) (c) Cl I


id) Cl
111. The major product(s) obtained from the following
reaction of 1 mole of hexadeuteriobenzene is/are
ad

(AIIMS 2007)
our

D 114. Among the following the reaction that proceeds


D.
through an electrophilic substitution is
D
(i) Bf2 (1 mole), Fe
(/OH2O ia) CH2OH + HCI
Re

D
dY

D
CH2CI + H2O
Fin

D D

ia) D, Br ib) Br, Br ib)


+.
N2CI- CU2CI2 ^ CI + N2

D' D D' D AICI3


(c) + CI2 Cl + HCl
D Br

109. (c/) no. (/;) m. (fl) 112. (r) 113. ();)


13/183
HYDROCARBONS

Cl Cl
NO2
H
H
UV light ic) E
Cl id) E
id) + CI2 Cl

Cl Cl (AIEEE 2008)
119. 1,4-DimethyIbenzene on heating with anhydrous
(NEET 2019)
AICI3 and HCl produces
115. The compound which does not lead to benzoic (a) 1, 2-dimethylbenzene
acid by oxidation with KMn04 is (b) 1, 3-dimethylbenzene
(ii) toluene (b) benzyl alcohol
(c) 1, 2, 3-trimethylbenzene
(c) n-butylbenzene (cl) r-buiylbenzene
(d) ethylbenzene (West Bengal JEE 2015)
(e) styrene (Kerala PET 2019)
120. Toluene reacts with halogen in presence of iron

w
116. Sulphonation of benzene with excess sulphuric (III) chloride giving ortho- and para-halo-
acid provides compounds. The reaction is

F lo
(a) benzenesulphonic acid (fl) free radical addition reaction
(b) p-benzenedisulphonic acid {b) electrophilic elimination reaction
(c) o-benzenedisulphonic acid (c) nucleophilic substitution reaction
(d) m-benzenedisulphonic acid id) electrophilic substitution reaction
(e) decomposition of benzene (Karnataka CET 2017)

ree
(Kerala PET 2019)
121. The compound with molecular formula CgH,o

for F
117. The following desulphonation reaction, which will give only two isomers on electrophilic
SO3H substitution with Cl2/FeCl3 or with HNO3/H2SO4
IS

HCl, pressure (a) p-dimethylbenzene (b) m-dimethylbenzene


+ H2O 420-470 K ^ + H2SO4
(c) o-dimethylbenzene (d) ethylbenzene
Your
(DPMT 2010)
takes place through intermediate formation of
ks

122. The major product obtained on monobromination


eBoo

H SOi"
(Br,/FeBr3) of the following compound A is
X H
H
OCH3
ia) ' + ib)
ad
our

SOJ so^-

A
CH3
(c) I 4- ■ H id) OCH3

X
H OCH3
Re

Br
Y

118. The electrophile, attacks the benzene ring to ib)


ict)
Find

generate the intermediate o-complex. Of the CH3


following, which a-complex is of lowest energy ? CH3 Br
NO2
NO2 OCH3 OCH3
Br.
ib)
ia) I + H ic) id)
E
X Br CH3 CH3

114. (c) 115. (r/) 116. {</) 117. {«) 118. (<●) 119. (/>) 120. (r/) 121. (c) 122. (/?)
13/184
New Course Chemistry (XI)BEISD

123. The ease of nitration of the following three 126. The product ‘A’ in the following sequence of
hydrocarbons follows the order reaction is

Bfi/Fc CliM
CH3 CH3 Ethylbenzene
Aic KOH
4 ‘A’

Cl—CH—CH3 Br—CH—CH3
O LQ CH3 CH3

CH3 (o) ib)


(I) (II) (III)

(a) II = 111 = I (b) II > III > I


(c) III > II > I (<f) I sill > II CH—CH2

w
(West Bengal JEE 2011)
124. Arrange the following is increasing order of (c) Cl id) (1

F lo
reactivity towards nitration.
I. />-xylene II. bromobenzene
Br
HI. mesitylene IV. nitrobenzene
(JEE Main 2022)

ree
V. benzene
127. Which among the following pair of structures will
(a) III < IV < V < 1 < II
give different products on ozonolysis ? (Consider

F
(fc) IV < n < V < I < m the double bonds are rigid and not delocalized).

for
(c) IV < III < II < I < II
(^/) m < IV < V < II < I (JEE Main 2022) CH3 CH3
r
125. Consider the given reaction sequence, the major
product ‘C’ is
You
(a) >
s
ook
HNO Bft.A
^ A ^ B
ale. KOH CH3 CH3
4 C
H2SO4
CH3 CH3
eB

NO2 (b) ●4 -►

(a) C=CH2
our
ad

CH3 CH3

(b) O2N CH=CH2 CH3 CH3


Y

CH3 CH3
Re

ic) ●4
d

(c) O2N C=CH2


Fin

N02 CH3 CH3

,N02 id) 4 >

id) CH=CH2 (JEE Main 2022)


(JEE Main 2022)

ANSWERS

123. (/;) 124. (/;) 125. (A) 126. Ul) 127. (c)
HYDROCARBONS 13/185

128. The major product of the following reaction is


OCH3
O2N (0 Na/liq.NHj
(i7) CH3CH2OH

OH
OCH3 OCH3 H3CO H
H H. O2N 0

(a) O2N (6) 02N' (c) id)

w
(JEE Main 2022)

D1 Multiple Choice Questions (with One or More than One Correct Answers)

F lo
129. In the Newman projection for 2,2-dimethylbutane
ic) (d)
H2SO4 BF3-OEt2

ee
H3C CH3
(JEE Advanced 2016)

Fr
132. Which of the following on treatment with warm
dil. H2SO4 in presence of HgS04 will give a
methyl ketone ?

for
(a) Ph—C=C—CH3 (b) CH3 = c—CH3
C=CH
ur
X and Y can respectively be
(c) (d) C=CH
(a) H and H (b) H and C2H5
s
(c) C2HgandH id) CH3andCH3
133. Which of the following molecules, in pure form,
ook
Yo
(IIT 2010)
is(are) unstable at room temperature?
130. The correct statemenl(s) concerning the structures
eB

E, F and G is (are) (a) ib)

O OH O
O
H3C H3C
our

id) I[ \ (IIT 2012)


ad

H3C CH3 H3C CH3


(f) (O

H3C .CH3 134. Among P, Q, R and S, the aromatic compound(s)


is(are)
Y

'OH
H3C (G) Cl
Re
nd

id) E, F and G are resonance structures


AICI3
>
ib) E, F and E, G are tautomers
Fi

(c) F and G are geometrical isomers


id) F and G are diastereomers (IIT Paper 1,2008)
131. Among the following reaction(s) which gives
(give) ferr-butylbenzene as the major product is (NH4)2C03
(are) IIOMIS^C

Y^ci
(a) ib) (a) P ib) Q
NaOC2H5 AICI3
(JEE Advanced 2013)
ANSWERS

128. (u) 129. ib.d) 130. ih.c.d) 131. ih.c.d) 132. (b.c.d)
133. ih.c) 134. ia,b.c.d)
13/186
‘P'uuiceft'A New Course Chemistry fXTIrosTwrn
135. Which of the following reactions produces(s) (C)
Cl Zn, dil. HCl

propane as a major product ? H3C
H3C NaOH, CaO, A Br
(fl) COONa

id) Br Zn
H3C eleclrolysis ^
ib) COONa
+ H20 H3C
(JEE Advanced 2019)

nn Multiple Choice Questions (Based on the given Passage/Comprehens ion)

Each comprehension given below is followed by some multiple choice questions. Each question has one

correct option. Choose the correct option.

;C.onTp^liensib nl|[

w
R R R H
Catalytic hydrogenation \ / \ /
involves addition of hydrogen to a >C = C< (c) (d)

F lo
or -C s C- bond in presence of a metal R H R
/ \H
catalyst such as Ni, Pt, Pd, etc. These are
Lindlar’s Na in
called heterogeneous catalysts. Both H2 and 137. [A] CH3 = C—CH
alken^alkyne get adsorbed on the surface of Catalyst ^ iiq.NH3
the metal ; and syn-addition of hydrogen

e
[B]
atoms takes place. The relative rates of

Fre
hydrogenation follows the order : [A] and [B] are respectively
(a) cis, irans-2-buien& (b) both trans-2-hutenc
-C s C- > >C = C< >C = O > CgH^

for
(c) trans, cij-2-butene {d) both m-2-butene
If sodium or lithium in liquid ammonia is
used, a/if't-addition of H-atoms occurs across 138. The catalytic hydrogenation of which of the
-C = C- bond. These reducing agents can also following alkenes is most exothemiic ?
r
be used to reduce double bonds of benzene
rings and conjugated dienes but not of
You
(«) ib)
ks
isolated dienes.
o
eBo

136. Which of the following alkenes will react most (c) id)
readily with H2 under catalytic hydrogenation
conditions ?
139. Which of the following functional groups is most
R R R R
easily reduced.
ad

\ /
our

\ /
{a) (b) id) >C = O {b) >C = C<
R
/ \R H
/ \H (c)-C = C- (d)

09 Matching Type Questions


Re
dY

Match the entries of column I with appropriate entries of column II and choose the correct option out
of four options (a), (6), (c), (d) given at the end of each question.
Fin

140. Column I Column II


(A) Corey House reaction (P) Alkenes
(B) Fittig reaction Alkyl aryl or diaryl ketones
(C) Friedel-Crafts acylation (r) Alkanes

(D) Dehydrohalogenation is) Diaryls

(a) A‘S ; B-q ; C-p ; D-r (b) A-q ; B-p ; C-r ; D-5 (c) A-p ; B-r ; C-s ; D-q {d) A-r ; B-s ; C-q : D-p

tSMMiF'.
135. (<7.c) 136. ib) 137.(a) 138.id) 139. (c) 140. (rf)
13/187
HYDROCARBONS

141. Column I Column II


HBr
(A) CH3CH = CH2 4 (P) CH3CH2CH2Br
RCOOR

HBr
(B) CH3CH = CH2
4 (.q) CH3—CO—CH3
Dil.H2S04/HgS04
(C) CH3—C = CH ●4 (r) CH, = CH—OCH3
CH3OK, A
(D) HC s CH 4 is) CH3—CHBr—CH3
(a) A-r ; B-^ ; C-p ; D-^ {b) A-p ; B-.9; C-q\ D-r (c) A.-S ; B-p ; C-g ; D-r {d) A-q ; B-5 ; C-p ; D-r

w
Matrix-Match Type Questions p q r s

F lo
Match the entries of column I with appropriate entries of column n. Each A ©@0©
entry in column I may have one or more correct options from column II. If
the correct matches are : A*p, s ; B-r; C-p, q ; D-s then the correctly bubbled
B
©®O©
4x4 matrix should be as follows :
©©

ee
C

Column II

Fr
®i® © ©
142. Column I D

Z11/CH3OH
(A) CH3CHBrCHBrCH3 A
4 ip) CH3CH2CH = CH2

for
conc.H2S04
(B) CH3CH2CH2CH2OH 4 iq) CH3CH = CHCH3
ur
A

Ale. KOH
ir) Carbocation
(C) CH3CH2CH2CH2CI 4
s
A
ook
Yo
Ale. KOII
(D) CH3CH2CHCICH3 4 is) E-) elimination
A
eB

143. Column 1 Column II

(A) Kolbe’s electrolysis ip) Alkanes

(B) Ozonolysis iq) Alkenes

Electrophilic substitution ir) Alkynes


our

(C)
ad

(D) Electrophilic addition is) Arenes A B C D

® ® @®
VI. Integer Type Questions 0000
Y

©@©@
Re

DIRECTIONS. The answer to each of the following questions is a single digit


©@©@
nd

integer, ranging from 0 to 9. If the correct answers to the question numbers


A, B, C and D (say) are 4, 0, 9 and 2 respectively, then the correct darkening ©@© ©
Fi

of bubbles should be as shown on the side :


© © ©©
Cl
144. The total number(s) of stable
coiiformers with non-zero
®®©®
Br.
dipole moment for the
CH3 0 © 0®
Cl
following compound is (are)
Br
® ®0®
CH3 (JEE Advanced 2014) ® ® ®®
ANSWERS

141. (ft) 142. {A-q,5 ; B-p,q.r ; C-p,s ; D-p,q,s) 143. {A-p.q.r ; B-g,.rs ; C-s ; D-q,r)
144. (3)
13/188
New Course Chemistry (XI)EEIHD
145. The number of cis-trans isomers with molecular formula C2BrClFI is
146. Amongst the following, the maximum number of alkenes which show geometrical isomers are

1-pentene, 2-butene, 2-pentene, 1-pentene. 1-butene, propene, 2, 3-dimethyl-2-butene, 3-hexene, I-hexene


147. Which of the following molecules have zero dipole moment ?
c/j-1, 2-dichloroethene, trans-X, 2-dichloroethene. 1, 1-dichloroethene, /ra/jj-2-pentene, c/i-2*p)entene, 1-
butyne, 2-butyne, rra/ij-2-butene, c/5'-2-butene.
148. How many of the following on reductive ozonolysis will give only glyoxal ?
1,3-butadiene, ethylene, acetylene, o-xylene, /u-xylene, p-xylene, benzene, cyclobutadiene, cyclooctateraene.
149. How many of the following species are aromatic in nature ?

w
cyclopenladienyl cation, cyclopentadienyl anion, tropylium cation, azulene, cyclopropenyl cation,
tetrahydrofuran, cyclooctatetraene, furan, cycloheptatriene.
150. Among the following, the number of aromatic compound(s) is

F lo
6

ee
Fr
(JEE Advanced 2017)
151. Total number of hydroxyl groups present in a molecule of the major product P is

for
ur
(0 H2, Pd-BaS04, quinoline
s
{it) dil. KMn04 (excess, 273 K)
ook
Yo
eB

vn.
Numerical Value Type Questions Decimal Notation)
our

For the following question, enter the correct numerical value, (in decimahnotation, truncated/rounded-off
ad

to the second decimal place, c.g., 6-25, 7-00, - 0-33, - -30, 30-27, - 127-30) using the mouse and the on
screen virtual numeric keypad in the place designated to enter the answer.
152. In the following reaction sequence, the amount of D (in g) formed from 10 moles of acetic acid ISi
Y

(The yield (%) corresponding to the product in each step is given in the parenthesis)
Re

NaOH Soda-lime CIj./iv Na, dry ether


nd

CH3COOH ■> A ^ B ^ C ^ D
90% 630 K 70% 80% 70%
Fi

CH3 CHO

153.

5g 92%

In the above reaction. 5 g of toluene is converted into benzaldchyde with 92% yield. The amount of
benzaldehyde produced is (JEE Main 2022)
ANSWEI
145.(6) 146. (3) 147.(3) 148.(4) 149. (5) 150. (5)
151.(0) 152. (52-92) 153. (530 00)
13/189
HYDROCARBONS

Vlll. Assertion-Reason Type Questions

TYPE I

DIRECTIONS. The questions given below contain STATEMENT-1 (Assertion) and STATEMENT-2
(Reason). Each question has four choices (a), (b), (c) and (d) out of which ONLY ONE is correct. Choose
the correct option as under,
(a) Statement-1 is True, Statement-2 is True ; Statement-2 is a correct explanation for Statement-1,
(b) Statement-1 is True, Statement-2 is True ; Statement-2 is not a correct explanation for Statement-1,

w
(c) Statement-1 is True, Statement-2 is False. (d) Statement-1 Is False, Statement-2 is True.
154. Statement-1. The guache conformation of ethylene glycol is more stable than its anti conformation.
Statement-2. The gauche conformation is stabilized by H-bonding.

Flo
155. Statement-1. Dimethyl sulphide is commonly used for the reduction of an ozonide of an alkene to get the
carbonyl compounds.

e
Statement-2. It reduces the ozonide giving water soluble dimethyl sulphoxidc and excess of it evaporates.

re
156. Statement-1. Bromobenzene upon reaction with Br2/Fe gives 1,4-dibromobenzene as the major product.
Statement-2. In bromobenzene, the inductive effect of the bromo group is more dominant than the mesomeric

rF
effect in directing the incoming electrophile. (IIT Paper 1,2008)
157. Statement-1. Toluene on sulphonation gives m-toluenesulphon ic acid.
ur
Statement-2. Sulphonation is a reversible reaction.

fo
158. Statement 1. [6]-Annulene, [8]-Annulene and ci^-[IO]-Annule ne are respectively aromatic, not-aromatic
and aromatic
1 2
10
ks 3

4
Yo
■>
oo
7 6
(6)-Annu!ene (8)-Annulene m-(10)-Annulene
B

Statement 2. Planarity is one of the requirements of aromatic systems.


TYPE II
re

DIRECTIONS. In each of the following questions, a statement of Assertion (A) is given followed by a
corresponding statement of Reason (R) just below it. Of the statements, mark the correct answer as
u
ad

(a) If both assertion and reason are true, and reason is the true explanation of the assertion,
Yo

(b) If both assertion and reason are true, but reason is not the true explanation of the assertion,
(c) If assertion is true, but reason is false. (d) If both assertion and reason are false
159. Assertion. Boiling points of m-isomers are higher than those of trans-isomtrs.
nd
Re

(AIIMS 2008)
Reason. Dipole moments of ci5-isomers are higher than those of rruus-isomers.
160. Assertion. Addition of HI to vinyl chloride produces 1 - chloro-1 - iodoethane.
Fi

Reason. HI adds to vinyl chloride against Markovnikov’s rule.


161. Assertion. Propene reacts with hydrogen chloride in presence of organic peroxide to give 1 -chloropropane.
Reason. Addition of HCl follows Kharasch effect.
162. Assertion. C—H bond in ethyne is shorter than C—H bonds in ethene.
Reason. Carbon atom in ethene is sp hybridised while it is sp^ in ethyne. (AIIMS 2007)

163. Assertion. Calcium carbide on hydrolysis gives ethylene.


Reason. Calcium carbide contains anions. (AIIMS 2010)

154.(a) 155.(a] 156. (r) 157. (t/) 158. (rf) 159, (u) 160. (r) Iftl. id) 162. (c) 163. ((/)
13/190
T^nttdet^'4. N -V Course Chemistry fxnrosnwn
164. Assertion. Acidity of the C—H bond decreases in
the order: HC = CH > H2C = CHt > CH3—CH3
Reason.
increases.
Acidity of the C—H bond increases as the electronegativi ty of the carbon to"which it is attached
165. Assertion. Acetylene reacts with sodamide to evolve H
2 gas-
Reason. Acetylene is a weaker acid than ammonia.
166. Assertion. Acetylene on treating with NaNH2 gives sodium acetylide and ammonia.
Reaction. .s/?-Hybridized carbon atoms of acetylene are considerably electronegative. (AIIMS 2007)
167. Assertion. But-l-yne and but-2-yne can be distinguished by ammoniacal silver nitration solution.
Reason. 1-Butyne forms white ppt. with ammoniacal silver nitrate but 2-butyne does not.

168. Assertion. Alkynes are more reactive towards nucleophilic addition reaction as compared to alkenes.
Reason. Alkynes contain two pi bonds, while alkenes have only one pi bond. (AIIMS 2009)
169. Assertion. 2-Butyne on controlled hydrogenation with Pd/CaCOg in presence of PbO gives aV2-butene.

w
Reason. Hydrogenation occurs at the surfaces of metal containing adsorbed hydrogen. (AIIMS 2014)
170. Assertion. But-2-yne on reduction with Na/NHjC/) gives trans-l-hxxXtne.

F lo
Reason. To minimise interelectronic repulsions, the addition of electrons occurs on the opposite faces of the
triple bond.
171. Assertion. Benzene does not decolourize Br2-water.
Reason. Benzene is stabilized by resonance due to delocalization of 7t-eIectrons.
172. Assertion. Tropylium cation is aromatic in nature.

r ee
for F
+

Reason. The only property that determines its aromatic behaviour is its planar structure. (AIIMS 2008)
173. Assertion. Cyclopentadienyl anion is much more stable than allyl anion.
Reason. Cyclopentadienyl anion is aromatic in character.
Your
1*74. Assertion. Friedel-Craftsreaction is used to introduce an alkyl or acyl group in benzene nucleus.
oks

Reason. Benzene is a solvent for the Friedel-Crafts alkylation of bromobenzene. (AIIMS 2008)
175. Assertion. Friedel-Crafts reaction of benzene with n- propyl chloride on heating gives isopropylbenzene.
eBo

Reason. Benzene undergoes electrophilic substitution readily. (AIIMS 2015)


176. Assertion. Oxidation of toluene as well as ethylbenzene with KMn04 gives benzoic acid.
Reason. Both toluene and ethylbenzene are more reactive than benzene towards electrophilic substitution
reaction.
ad
our

177. Assertion. Rates of nitration of benzene and hexadeuteroben zene are different.
Reason. C—H bond is stronger than C—D bond.
Re
dY

For Difficult Ouestions


Fin

D Multiple Choice Questions (witli One Correct Answer)^


1. Wurtz reaction is suitable for preparation of unsymmetrical alkanes (containing odd number of
symmetrical alkanes (containing even number of carbon atoms) such as u-heptane (7 carbon atoms).
carbon atoms in good yields such as 2, 3-
2. Sodium salts of fatty acids on heating with
dimethylbutane (6 carbons), n-butane (4 carbons) sodalime (NaOH + CaO) undergo decarboxy
and /i-hexane (6 carbions) and not for lation to give alkanes
ANSWERS
164. (n) 165. id) 166. (6) 167. (n) 168. (c) 169. (n) 170. (o) 171. (a) 172. (c) 173. (a)
174. (c) 175.(6) 176.(6) 177. (rf)
13/191
HYDROCARBONS

CH3 CH3
For Difficult Questions

+ CH3—CH CH^—CH^—CH—CH3
NaOH + CaO 2,.‘5-Diinethylhexane(iv)
CH3CH2COO"Na+ Heat
(Ditiierizaiioii product of B)
Sodpropanoate
CH3 CH3
CH3—CH3 +Na2C03
Ethane + CH3 -C—CH^—CH2—CH — CH3
Sodalime is a mixture of NaOH + CaO. therefore, CH3
CaO, i.e., option (c) is correct. 2.2,5 -Trimcihy Ihexane (v)
{Cross product of A and B)
H H H CH3 7.

w
I
3. CH,—C—C—C—CH =
^ I I I I Me3C

F lo
H H H CH3 Dry ether
{Corey-House
CH3 MesC reaction)
CH3
Me
CH CH 1 2I 3 4

e
Me—C—CH2—Me +Me3CCu+LiBr

Fre
CH2 CH3 Cyclization CH2 CH2
Me
H2
CH^ CH3 CH2 CH2 2,2-Dimethylbutane

for
8. (/) Since hydrocarbon (A) reacts with Br2 by
CH2' CH2" substitution to form an alkyl bromide, therefore,
r
CH3 hydrocarbon (A) must be either CH^ or CH3CH3
and the alkyl bromide it forms must be either CH3Br
You
oks
Aromatization
or CH3CH2Br.
hv
-3H2 CH4 + Br2 ^ CH3Br + HBr
eBo

Toluene
hv

Thus, option (a) is correct. CH3-CH3 + Br2 » CH3CH2Br + HBr


4. Five, Refer to page 13/5. (n) Since CH, = CH2 and CH s CH give addition
our

treatment with Br2, therefore.


ad

reactions on
Cl
5. Cl + 2 Na + Dry Aether ^ hydrocarbon (A) cannot be CH2 = CH2 or CH s CH.
(iiO Since alkyl bromide on Wurtz reaction gives a
+ 2 NaCl
hydrocarbon containing less than four carbon
atoms, therefore, alkyl halide must be CH3Br and
dY

the hydrocarbon formed must be ethane


Re

CH, CH3 A
1 2 CH3Br + 2 Na {Wurtz reaaion)
> CH3-CH3 + 2 NaBr
Fin

6. CH3—C—CH,Br
I
+ BrCH2—CH—CH3 Ethane

Isobutyl bromide(B) Please note that Wurtz reaction of ethyl bromide


CH3 would give n-butane which contains 4 carbon
Nec^ntyl bromide (A) atoms. Therefore, hydrocarbon (A) must be CH4,
i.e., option .(d) is correct.
CH3 CH3 9. Boiling points decrease with branching. Alkane
Na/dryether, I^ ^ (III) with no branching has the highest b.p.,
CH,—C—CH,—CH,—C—CH3
{Wurtz reaction) I ^ ^ followed by alkanes (II) with one branch each
CH3 CH3 while alkanes (I) with two branches each has the
2,2,5.5-Tetramelhylhcxane(«) lowest b.p. Thus, the b.ps. decrease in the order :
{Dimerization product of A)
III > II > I, i.e., option {b) is correct.

1
13/192
New Course Chemistry (XI)EESHD

CH
For Difficult Questions
3\±
C-CH
3 + CH3-CH-CH2 ->

CH3
10. CH3
Isohcxane
3-Methylpentanc CH3
b.p, 60"C b.p. 63”C
CH3-CH-CH2-C-CH3
Since isohexane has lower surface area than 3-
meihylpentane. therefore, isohexane has lower b.p. CH3 CH3
(60“C) than 3-methylpenlane (63“C). Further, since Thus, options (b) is correct.

the h.ps. are very close, these two compounds are 15. Both bond angle and bond length remain the same.

Only dihedral angle and distances between the

w
separated by fractional distillation and isohexane
non-bonded hydrogen atoms change.
with lower b.p., distils over ifrst. Thus, option {a)
is correct. 16. Eclipsed conformer is least stable while staggered

F lo
conformer is most stable. In the eclipsed
© © conformer, the dihedral angle is 0®.
CH CH3
© © ©
11. CH3—C—CH2 CH—CH2—CH3

ee
@
CH3

Fr
©
2,2,4-Trimcthylhexane

for
2,2,4-Trimethylhexane has six sets of different
hydrogens and hence gives six monohalogenated
r
products on halogenaiion. Anhyd.AlClj/ Eclipsed, Staggered,
HClgas,A
You
dihedral angle = 0°
12. CH3CH2CH3CH2CH2CH3 4 dihedral angle = 60®
s
(Isomerization)
ook
n-Hexane
19. Gauche conformation (d) of ethylene glycol is
CH3 CH3 most stable due to intramolecular H-bonding as
I
eB

shown :

CH3 -CH -CH2CH2CH3 + CH3CH2 -CH-CH2CH3


2-MethyIpentane 3-Meihylpen(ane
13.
CH3
our
ad

V2Os,5Q0°C
CH3—(CH2)5—CH3 10-20 atm.
n-Heplane
dY

CH^Cl
Re

20. The staggered conformation of ri-butane having


CU/hv
>
maximum dihedral angle is anti conformation with
Fin

dihedral angle of 180®. Thus, option (c) is correct.


Benzyl chloride
21. Converting Newman Projection formulae to
graphic formulae, we have.
CH CH
3\ H+
14.
C = CH^ ■> i
CH, CH3 CH3
CH3' CH3 3 2
= CH3—c C—CH3

CH3—CH—CH2—H
o 4I 5 I
XH2CH3 OH
-H CH3-CH-CH- 2,3,3-Trimethylpcnlan-2-ol
I
CH3 CH3

i
13/193
HYDROCARBONS

Thus, the correct order of decreasing H-C-H bond


For Difficult Questions
angle is : II > I > III, i-e., option (b) is correct.
23. Only in l-phenyl-2-butene (C6H5CH2—CH =
CH—CH3) both the carbon atoms of the double
bond have different substituents and hence shows
CH3 geometrical isomerism.
4 5
1 2 3
24. (CH)2(C00H)2 actually represents
= CH3—C—CH—CH2—CH3
HOOC—CH = CH—COOH, i.e., maleic acid (cis)
OH CH2CH3 and fumaric acid (trans) which show geometrical
3.Eihyl-2-methylpentan-2-ol isomerism.

w
(D @
CH3 CH2CH3
25. C=C Z-isomer

F lo
Cl Br
CH3 CH2CH3
, 3I 45
= CH3—C—CH—CH2CH3
® ® 6 7 8

ee
OH
CH2CH2CH2CH3

Fr
3-Ethyl-2-methylpentan-2-ol 1
26. (a) :
/3
CH3 CH2CH2CH3
© ©

for
Z-3-methyl-4-propyl-3-octene.
ur
CH2CH3 27. Each carbon atom of the double bond has different
CH(CH3)2
3 2 1
substituents and hence it shows geometrical
= CH3CH2 —C—CH—CH3 isomerism.
s
ook
OH CH3
Yo
H Cl Cl\i/H
3-Ethyl-2-methylpenlan-3-ol
eB

and

Thus, options (Q, R) represent identical molecules, CH3 ‘3

i.e., option (c) is correct. as irans


r

22. In cyclopropane, the C-C-C bond angle cannot Thus, option (b) is correct.
ad
ou

be 109°-28' but appears to be only 60°. To account 28. Since generally, a tran^-form is more stable than
for this, it is proposed that cyclopropane has bent the c/s'-form, therefore, stability of the alkadienes
or banana C-C bonds which are formed by orbitals decreases in the order :
Y

with higher p-character {sp^ or sp^). Consequently,


for C-H bonds, carbon does not use pure sp^-
Re

> X
nd

orbitals but uses orbitals with much lower p-


(I) (II) (III)
character, i.e., between sp^ and sp^. As a result, trans, trans trans, cis cis, cis
Fi

H-C-H bond angle in cyclopropane is more than {most stable) (less stable) {least stable)
109°-28' but less than 120°, i.e., approx. 115°. Since heat of combustion is inversely proportional
H
to stability, therefore, heat of combustion
increases in the order :
H - C-Hbondangle is 115°
H I < II < m, i.e., option (a) is correct.
I 29. Ethene gives 1° alcohol (ethanol), propene gives
H
H
2° alcohol while 2-methylpropene gives 3° alcohol.
i°-28'
(0 Cone. H2SO4
H H H
CH2 = CH2 > CH3CH2OH
II III Ethene
(ii) H3O+ Ethanol
13/194
‘Pt<uice^ New Course Chemistry (XI) VOL.III

33.
For Difficult Questions

(I) Cone, H2SO4 OH


OH2 -HiO ^
CH3—CH = CH2 > CH3—CHOH—CH3
Propene (//) H30' 2® Alcohol
P
CH,, a

I ^ Cone. HjSO^ p or

CH3—C=CH2 4 (CH3)3C0H
(») H3O+
2-Methylpropene 3“ Alcohol 3° Carbocation
(h)

w
(most stable, no
Thus, either 2® or 3“ alcohol is formed.
tvairangemcni is
CH.
30. Acid-catalysed hydration is an electrophilic possible)
addition reaction.

F lo
31. or

CH3 (d) (c)

ee
CH3CH2—C CH—CH3 H2SO4
Heat
^ Thus, alkene (a) is not formed.

Fr
CH3 OH CH3
CH3 Cone. H2SO4

for
34. (0 CH3—C—CH—CH3
HjO
ur
CH3CH2—C CH—CH3 -H->0 CH3 OH
CH3 ■"OH2 CH3
s
l.2.Mclhyl ^
ook
CH3 —CH3
Yo
CH3 shin

CH3CH2—C CH—CH3 1,2-Methyl shift ^


eB

CH3 CH3 CH3


2° Carbocation
CH3 CH3 -H+ CH3 CH3
r

{less stable)
ad
ou

H Saytzeff product

CH3 H

:C^ CH—CH3
-H*
CH3
Y

CH3CH2—C—CH—CH3 Saytzeff rule (ii) alcJCOH


CH3 -HBr *
C,Br
Re

CH3
nd

3“ Carbocation
(more stable)
CH3
Fi

CH CH3
CH^CH2^3c=c2/^CH3 4
CH3
\ + CH3-CH= C-CH-CH3 Sayueff product
CH3 CH3
CH3
CH^^^CH^H
CH3 P a

(Hi) :CH (CH3)3CQ-K'^


2, 3-Dimethyl-2-butene (P) 3,4-Dimethyl-2-pentene
(Major product) (Minor product) CH3 -HBr ^
Br
32. The acid used must have a non-nucleophilic anion.
Since CI“ and Br“ ions are good nucleophiles, CH3
therefore, HCl and HBr will give their correspon CH—CH=CH2
ding halides, therefore, H3PO4 is the correct CH3
option. Hofmann product
HYDROCARBONS 13/195

!.“J
For Difficult Questions

Because of the bigger size of the base, i.e., (CH3)3CO and the steric hindrance due to the presence of two
CH3 groups at P-carbon, it does not undergo Saytzeff elimination but instead undergoes Hofmann elimination.
P a A

(iv) (CH3 >2C — CH2 — CHO -HjO


4 (CH3)2C = CH-CH0

ow
Saytzeff product
OH

Thus, option (a) is correct.


CH3 CH3

35. CH3—C—CH2OH ► CH3—C—CH2—OH2 H2O

e
CH3 CH3

re
rFl
Neopentyi alcohol

F
CH3 CH3
1 I 3 4
1,2
CH3 C CH2CH3

r
ou
k sfo
oo
CH3—C =CHCH3 CH3—C—CH2CH3
Y
eB

CH3
H3C ,CH3

H3C
> H3C
r

Saytzeff product CHj


ou

{more stable, 85%) Hofmann product


ad
Y

{less stable, 15%)


Thus, option {b) is correct.
d

37.
a/H CH
H-MD
Re
in

\H
CH3 0 CH3
-H2O
F

CH

H—CH3 2° carbocation
I, 2-Alky] ^ (less stable)
shift
H CH3
CH3 I, 2-Methyl ^
shift -H*
CH3
3® carbocation H
(more stable)
-H
4- CH3 CH3
C=C
'3
CH3 CH3
2, 3-Dimethyl-2-butene
13/196
New Course Chemistry fXTlrosTwm

H
For Difficult Questions
CH3O CH = CH - CH3 >
a P
38. Addition of a proton to 2-pentene can give two
carbocations (T and II)
CH - CH2CH3

CH3CH = CHCH2CH3
n
2-Pentene
Br
+
CH3O CH-CH2CH3
CH3CHCH2CH2CH3 + CH3CH2CHCH2CH3
I II

w
HBr
CH3O CH - CH2CH3
lir^ -CH3Br
Br
CH3CHBrCH2CH2CH3 + CH3CH2CHBrCH2CH3 (D

F lo
2'Bromopentane 3-Bromopentane
HO CH - CH2CH3
which are almost equally stable since carbocation
(I) is stabilized by five (3 + 2) hyperconjugation Br
(II)

ee
structures and carbocation (II) is stabilized by four
P
(2 + 2) hyperconjugation structures. Therefore,
a Elimination

Fr
nucleophilic attack on these gives the corres
42. CH3—CH—CH—CH2CH3 (-HBr)
■>

ponding bromoalkanes in almost equal amounts. Br H


2-Bromopentane

for
Of course, the yield of 2-bromopentane will be
slightly more than that of 3-bromopentane. CH3 -CH = CH—CH2CH3
ur
39. Pent-2-ene

CH3 The above reaction is an example of dehydro-


CH2
s
H3C Cl halogenation reaction since it involves the removal
ok
of a molecule of HBr. It is actually a ^-elimination
Yo
cr ^ ^ ic^cr reaction since hydrogen is lost from ^-carbon and
o

(Mark. (Mark. it follows Zaitsev (or SaytzefT rule) since it gives


eB

Addn.) Addn.)
(A) 1-Chloro-
(B) more highly substituted (or more stable) alkene,
1 -methylcyclohexane i.e., pent-2-ene instead of less stable pent-l-ene.
40. Methyl vinyl ether is a very reactive gas. It Hence option (d) is correct.
readily undergoes hydrolysis with dilute acids to
r

43. At each double bond, addition of a proton first


ad

form methanol and acetaldehyde. However, under


ou

occurs to generate a more stable carbocation (I or


anhydrous conditions at room temperature, it II). Subsequent nucleophilic attack by Br” ion
mainly undergoes addition reaction as shown gives the final product (III).
below :
Y

CH3 CH3 Br CH3


Re
nd

6 2 H Br¬

il) (^^CH—OCH3 >CH3—CH—0CH3


Resonance stabilized
Addition of
Fi

4 4
carbocation
NO2 NO2 NO2
I
◄ ►CH3—CH=0CH3
Br CH3 Br CH3

-Br H+
X Br- X
(ii) CH3—CH—0CH3-^CH3—CHBr —OCH3 Addition at C4 -i-
>

41. Addition of a proton at P-carbon generates a X Br"


carbocation which is stabilized by resonance. H NO2
NO2
Nucleophilic attack by Br" gives intermediate (I) II III

cleavage of ether (I) gives the final product (U). Thus, option (d) is correct.
HYDROCARBONS 13/197

HBr Elimination
i
For Difficult Questions
H2C CH^ ^ CH3CH2CH2B1- (-HBr)
■y

\v n-Propyl bromide
44 Due to greater electronegativity of F over Br, C
C2-H is more acidic than C^-H and due to bigger H2 CH3-CH = CH2
size of Br over F, C-Br bond is weaker than C-F. Propene
Cyclopropane
As a result, during elimination reaction, HBr is
HBr
lost to give alkene (I) as the major product.
(b) CH3-CH2-CH2OH ^ CH3CH2CH2Br
-H2O
iC'
Br «-Propyl alcohol rt-Propyl bromide
1 ●7 4 5 Elimination

CH3— Cj-f — CH2CH3 -HBr 4 CH3-CH = CH2

w
(-HBr)
Propene
H H
F
(d) CH3CH2CH2Br gives CH3CH = CHj as shown

F lo
above. Thus, H2C = C = 0 does not give
CH3—C=CH—CH2CH3 CH3CH = CH2, i.e., option (c) is correct.
(I)
Thus, option (d) is correct. 48. In presence of peroxides, Br radical first adds to
propene to produce a more stable radical, i.e.,

ee
45. Since each carbon of the d.b. has one H,
Homolytic
Markovnikov’s rule is not applicable. The final

Fr
RO OR >2RO*
fission
product is obtained through the formation of a Peroxide

stable carbocation as shown below : RO- + HBr > ROH + Br

for
CH3CH = CHCH(CH3)2
CH3—CH=CH2+Br > CH3—CH~CH2Br
our
+
1,2-Methyl 2“ radical
CH3CH2—CH—
"^CH3 shift ^ {more .'stable)
s
CH3—CH—CH2
ook
2° carbocation
Br
{less stable) 1° radical
Y
eB

+
CH3 Br" {less stable)
CH3CH2CH2 c
5
CH3
3® carbocation
49. CH3v^4 3 2 1

{more stable) CH-CH2-CH=CH2


our
ad

Br CH3-
4-Melhylpenl-l-ene
CH3
CH3CH2CH2—C 5
CH3 Peroxide CH,\ 4 3 2 I
Thus, option (d) is correct. 4
^CH-CH2 CH2-CH2Br
Y

(Anti-Markovnikov's .
Re

addition) 3
CH3 l-Bromo-4-methylpentane
nd

RCOOR
50. CH3CH2-CH = CH2 + HBr 4
Fi

CH3CH2-CH2-CH2Br + CH3CH2-CHBr-CH3
[XI [Y]

= C2H5-CH2CH2Br -b C2H5-CHBr-CH3
1,2-Methyl
shift
^ + Br"
Br [XI [Y1

51. Addition of HBr to alkenes in presence of


Thus, option (b) is correct. peroxides gives anti-Markovnikov's product in
which H adds to that carbon atom of the d.b. which
47. {a) The strained three-niembered ring opens upon
carries less hydrogens and Br adds to that carbon
treatment with HBr to give n-propyl bromide
atom of the d.b. which carries more hydrogens.
which upon elimination of HBr gives propene
For example.

I
13/198 ‘P>uideejlt.'4. New Course Chemistry (Xl)CSlSD

Since MeOH is more nucleophilic than Br~,


For Difficult Questions
therefore, carbocation (I) formed by addition of
CH
Br'*’ undergoes nucleophilic attack by MeOH to
3\ HBr form compound (II) which on treatment with HI
CH—CH = CH2 ■>
undergoes Sjvj2 cleavage of methyl ether by I" ion
(C6H5C0)202
CH3' to form compound(III). Thus, option (c) is correct.
3-Methylbut-l -ene
54.
CH
3\
^CH—CH2—CH^—Br #0CH3
CH3
I-Bromo-3-methylbulane
Thus, option (a) is correct.

w
52. In principle, addition of a proton can occur either (I)

at C| or C2. Addition of a proton at Cj generates a Nucleophilic attack by CH3OH on the top face

F lo
carbocation (I) which is destabilized by the strong gives product (I). Thus, option (a) is correct.
-l-effect of the CF3 group.
2 1 +
H* H
F3C—CH=CH2 F3C-t-CH—CH3 Ph

ee
1 Br
dest^ilized by

Fr
+

-I-effectofCI^
Stabilized by
+
resonance over Ph ring
or F3C—CH2—CH2

for
Br” Ph Br
II
{Nucleophilic
In contrast, addition of a proton at C2 generates a attack)
r
more stable carbocation (II) Br
You
+
cr Thus, option (c) is correct.
s
F3C—CH2—CHj ●y
ook
II 56. In presence of UV light, free-radical substitution
occurs at the more reactive both the allylic
F3C—CH,—CH2—Cl positions.
eB

m
H H H Cl
Subsequent nucleophilic attack by Cl gives the
anti-Markovnikov’s product (III).
X CI2, hv
X
»
our

{Allylic chlorination)
ad

53. CH3—C = CH-CH3


Br2
-Br-
H
XH H
XCl
CH3 Thus, option (d) is correct.
Cone. H2SO4
dY

+ MeOH
CH,—C—CH—CH, 57. CH5-CH2-OH 4
CH2 = CH2
Re

4
A(-HiO)
^11 ^ -H + Ethanol Ethene {gas)
CH3Br
Fin

I Cold dil. aq KMn04 CH,-CH-


» i
{Baeyer's reagene) OH OH
OMe
I HI Ethylene glycol
CH,—C—CH—CH, ■>
Thus, option fc) is correct.
M l ^ Cleavage of ether
(-MeOH)
CH3Br 58. Since ants produce formic acid therefore,
n compound 'b' must be HCCOH. Since ‘a’ an
1 oxidation gives HCOOH, therefore, 'a' must be
CH,—C—CH—CH, formaldehyde. Since reductive ozonolysis (O3, Zn/
3 I I 3 H2O) gives formaldhyde ‘a’ and KMn04/H''‘
CH3Br oxidation gives formic acid, 'b' therefore, option
m (d) is correct.

I
HYDROCARBONS 13/199

63.
For Difficult Questions

H3C CH3
Ozonolysis ^
59. Since conjugated dienes give glyoxal and a 1,2-
dialdehyde, therefore, ‘A’ must be cyclohexa-1,3- CH3 CH3
diene. In contrast 1-methylcyclopent-l-ene (B) on
0=HC CH C
reductive ozonolysis gives 5-oxohexanal.

CH=0 64.
(OO3 ^ I
Ozonolysis

w
(»') Z11/H20
Cyclohexa-l,
CH=0
Glyoxal
CH3—CH=|=CH—CH2 > 2 CH3CH = O
3-dene (A) Ethane-1,2-
-O
dicarbaldehyde

F lo
0 O Ozonolysis ^ + O = CH2
3
(OO3 ^ CHO
07)Zn/H2O, H
Ozonolysis ^

ee
4 2

1 -Methylcyclopent-
5-Oxohaxanal '—CHO

Fr
1-ene (B) Cyclobutene 1,4-Butanedial

I I

Thus, option (d) is correct. I I

CH3—CH=i=CH CH—CH2 CH3CH=0 +

for
60. C4Hg has three isomers, ci5-2-butene, trans-2-
I I
Ethanol

butene and 2-methylpropene. Since (a) on KMn04/ Penta-1,3-diene


our
H'*' oxidation gives CO2 which gives effervescence 0=CH—CH=0 + 0^CH2
amd a ketone, i.e., acetone, it must be 2-methyl- Glyoxal Methanal
s
propene,
ook
Thus, only cyclobutene produces only one
CH3X KMn04/H+ dicarbonyl compound, i.e., option (b) is correct.
C = CH2 4
Y
eB

CH3 CH
3\ (OO3
2-Methylpropeiie 65.
C = 0 -b 0 = CHCH3 < Z11/H2O
CH
CH3'^ Ethanal
3\ Propanone
CO2 -b C = 0
our
ad

CH3' CH-
Aceiene I ^
Thus, option (d) is correct. CH3—C=CHCH3
2-Methyl-2-butene (A)
61. The alkene which has a terminal (or vinyl) = CH2
Y

group upon ozonolysis gives methanal as one of Thus, the structure of the alkene (A) is 2-methyI-
Re

2-butene.
the products.
nd

CH2 —CH4=CH2 CH2—CH = 0 CH3


Fi

HCl

CH3—C=CH—CH3
4
(j) O3/CH2CI2 _ Mark.addition
+ O = CH2
~(//)Zn/H20 * Methanal
(A)

CH3
^CH3 I
CU^—C—CH^CH^
1 I
Ozonolysis o
CH=0 Cl

^ ^CH3
(B)

Thus, option (d) is correct.


5-Keto-2-methylhexanal
13/200 New Course Chemistry fXT^rosm

.Br Br
KOH (ale.), A
For Difficult Questions
Br -HBr H /^y
4 3 2,1 (0 O3 Vinyl bromide
66. CH3-CH-C*CH2 NaNH2.liq.NH3
H—= —H
(ii) Zn/H20 -HBr ^
CH3 CH3 71. Ale. KOH being a weaker base than NaNH2
2,3-Dimelhyl-I-butene
eliminates only one molecule of HBr to form vinyl
halides (A and B). In contrast, NaNH^ in liq. NH3
4 3 2
being a strong base eliminates a molecule of HBr
CH3-CH-C = 0 + 0 = CH2 from vinyl halides (A and B) to form alkyne (I)
1 Methanal

w
CH3 CH3 Ale. KOH
3-Methyl-2-butanone CH3CH2—CH—CH2 -HBr

Br Br

F lo
CH3
4 I 2 1 KO'Bu, a
67. CH3—CH—CCH —CH3 [CH3CH2—CH = CHBr +
-CH3SO2OH A

(Hofmann elimination)
OSO2CH3 CH3CH2—C(Br) = CH2]

ee
B
CH

Fr
3\ (f) O3/H2O2
NaNH2inliq,NH3
CH - CH = CH2 (Oxidative ozonolysis) U CH3CH2—C = CH
CH3' “HBr
I

for
(I)
Thus, option (b) is coirect.
CH
72. Bromobenzene (a) cannot undergo dehydro-
ur
CH-COOH + HCOOH
Formic acid halogenation since it will produce benzyne which
CH3
is a highly unstable species. ! -Bromocyclohexene
s
Isobutyric acid
(h) cannot undergo dehalogenation since it will
ook
Yo
Due to +I-effect of the two CH3 groups, C3-H is produce a triple bond in a small cyclohexane ring.
less acidic than Cj-H. Further, due to bigger size Both options (c) and (d) can undergo
eB

of the base and more acidic Cj-H than C3~H,


dehydrohalogenation.
Hofmann elimination occurs to yield less stable
Br
alkene (I). Oxidative ozonolysis of alkene (I) gives
isobutyric acid and formic acid. Ph
► CH2 =CH—C=C— Ph
our
ad

Thus, option (c) is correct. -HBr


H (Triple bond is in conjugation
68. (CH3>2S is used for decomposition of ozonides to with the d.b. and the Ph
give almost pure products. (c) ring, more stable)
Br
Y

A
69. + 6 Ag + CI3C—CH3
CH3—CCI3 -6AgCl
Re

1.1,1 -T irchloroethane
► CH2=CH —C=C—CH3
nd

CH3 ●C = C—CH3 a HBr


(Triple bond is in conjugation
H
with the d.b. only, less stable)
Fi

2-Butyne
70. Due to resonance, C—Br bond in vinyl bromide (d)
has some double bond character and hence is more Thus, option (c) is correct.
strongly held than in alkyl bromides 73. Refer to ‘Supplement Your Knowledge For

O.
CH2—CH—Br:
0- > -rCHj—CH=Br:
Competitions’, pages 13/55-13/56.
74. +I-Effect of CH3 group increases the electron-
Vinyl bromide density in the C—H bond thereby decreasing its
Therefore, dehydrobromination of alkyl bromides tendency to release a proton. Thus, propyne is less
occurs smoothly with alcoholic KOH but to acidic than ethyne. Thus, the acidity of ethane,
remove HBr from vinyl bromide, stronger base like ethene, ethyne and propyne decreases in the order:
NaNH2 in liq. NH3 is required. option (a) is correct,
HYDROCARBONS 13/201

Alc.KOHA

For Difficult Questions 82. CH^~CH2-C = CH ^


1 -Butync
75. Refer to Ans. 74. Option (c) is correct.
(less stable) CH3-C = C-CH3
2-Butyne
76. Alkenes are more reactive than alkynes towards (more stable)
addition reactions, therefore, both propene (C3H(,) For mechanism, refer to pages 13/63-13/6^.
and ethylene (C2H4) are more reactive than
NaNHj
acetylene (C2H2). Out of propene and ethylene,
83. CH3 - CH - CH - CH2CH2CH3
propene is more reactive because the +I-effcct of -2HBr

Br Br
the CH3 group increases the electron density in
the double bond. CH3—>-CH = CH,. Butane

w
Na in liq. NH3
(C4H1Q) being saturated, however, does not give CH3 - C = C - CH2CH2CH3 (/rans-Addition)
addition reactions at all. 2-Hexyne

F lo
Thus, option (a) is correct. CH3 H
c = c
77. Markovnikov’s addition of H2O accurs. For
details, refer to page 13/58.
H
CH2CH2CH3
tra/i5-2-Hexene

ee
H-OH
CU2CI2 + NH4CI
CH3-C = CH > CH3-C=CH2 84. HCsCH + HC=CH

Fr
HgSO^/H^O I
OH Acetylene
a
Intermediate (A)
HCl

for
Tautomerises H—C = C—CH = CH2 >H2C=C- CH = CH2
^ CH3-C=CH2 Vinylaeeiylene Chloroprene
ur
o 85. 1-Butyne being a terminal alkyne has an acidic
Accion
hydrogen and hence reacts with NaNH2 to evolve
s
NaNHj NH3 gas but 2-butyne does not.
ook
> CH3 —CsC-Na+
Yo
78. Me—C=CH
-NH3 86. Terminal alkynes on oxidation give a mixture of
CO2 and a carboxylic acid with one C-atom less
eB

CHol
3 HgS04/dil.H2S04
^ CH3—CsC—CH3 ■) than that of the starting alkyne.
-Nal
87. Terminal alkyne containing (CH3)2CH group on
O oxidation gives (CH3)2CHCOOH.
our

O3
ad

CH3CH2—c—CH3 - Me—^ Me
89. CH = CH ^ OHC—CHO
Zn/CH30H
(Reduction)
O
HOCH2—CH2OH
NaNH2/iiq. NH3
^ H-CsC'Na+ Ethane-1, 2-diol
Y

79. H—C S c—H


-NH3
Re

hv
CH3CH2Br 90. CH4 + Br2 ^ CH3Br + HBr
nd

-NaBr
^ H-CSC-CH2CH3 is a free radical substitution reaction.
1-Butyne (X)
Fi

H H NaNH->
NaNHj/liq. NH3 91. C = C
^ Na+-C H C-CH2CH3 CH3' Br (Elimination
-NH3 of HBr)

CH3CH2Br CH3
-NaBr
CH3CH2-C = C-CH2CH3 Red hot Fe
3-Hexyne tube873K

80. Catalytic hydrogenation of alkynes with Pd/C CH3—CsCH (Trimerises)


Propyne
partially poisoned by addition of quinoline gives H3C CH3
cw-alkenes.
Mesitylene
81. Na/liq. NH3 or Li/liq. NH3 gives /ran^-products. Thus, option (c) is correct.
13/202 ‘PnacCee^ '4. New Course Chemistry (XI) pwh

For Difficult Questions


96. H The carbon at the junction of the
LiNHj/ether three rings is 5p^-hybridized and
92. HC^CH ■> hence the molecule is non-
-NHj
planar and hence not aromatic.
CH
3\
CH-Br Cycloociatetraene has 8 tt-
(CH3)2CH electrons and also it has non-
HC S C-Li-^ 4
-LiBr planar tub shaped geometry and
hence is not aromatic.

w
CH
HgSO^/HjSOj
CH-C=CH Thus, option (a) is correct.
Addition of H,0
(CH3>,CH 97. Compounds I and II have 6 n-electrons and are
also planar and hence are aromatic. In contrast,

F lo
o
CH III and IV have 8 7i-eIectrons and are also non-
NaBH^
CH —C—CH3 planar and hence are not aromatic.
(CHgKCH Thus, option (b) is correct.

ee
CH 98.
Conc.H2S04

Fr
CH—CH—CH3 ■>

(CH3)2CH/^ I
OH
-HjO +

for
CH3\ +
^C—CH—CH3
ur Aromatic Carbone Antiaromalic 1,3-Cyclohexadiene

(CH3)2CH-^H 4 (27t-^/ecfron)
ks {Electron
deficient)
{4n-Eleciron) {Conjugated diene,
most stable)

},3-Cyclohexadiene is most stable because it is a


2® carbocation
neutral molecule, carbene is also neutral but is
Yo
{less stable)
oo
electron-deficient. The remaining two are
intermediates and hence are less stable.
H3C CH2CH3
eB

Thus, option (d) is correct.


CH3'' I 99. Naphthalene is aromatic, so is acenaphthlene.
H
3° carbocation
Cyclopentadienyl anion is a cyclic planar system
with 6 7C-eiectrons and hence is aromatic in
r

{more stable)
ou

accordance with Huckel’s rule. Thus, option (a)


ad

CH
/CH3 and (d) both are aromatic.
C = C
CH3 CH2CH3 100. Both compounds (A) and (B) on ionization
Y

produce aromatic compounds and Cl“ ion.


Thus, option (c) is correct.
Therefore, they are more reactive towards AgN03
/i-CjHiiCl
nd
Re

h-Bu Li
93. n-Bu -H > n-Bu -Li ■>
than compounds (C) and (D).
-LiCI
Fi

Lindlar'scat'.ilyst
n-Bu C5HU-W
> + Cl
/i-Bu CgHii-n
>
/
\H 2rc-electrons {aromatic)
H
Thus, option (c) is correct.
94. Cycloheptatrienyl cation is an aromatic compound.
95. Due to severe non-bonded interactions between
the internal protons, the molecule is non-planar
and hence not aromatic although it contains
Hiickel number of 10 7t-electrons. 2n:-electrons {aromatic)
Thus, option (c) is correct.
13/203
HYDROCARBONS

Me Me

>
For Difficult Questions

w
Me' Me
H
Tropylium cation
{ammalic. highly stable)
106.

Further, since OMe group in (B) helps in the Cl + SbClj + SbCl^

ro
re
ionization,therefore,(B) is more reactivethan (A). (Completely delocalized
2n-electrons, aromatic)
Amongst compounds (C) and (D), compound {D)
is less reactive since the presence of NO^

F
3 +8 1 -8 2Na
CH3^CH-«-CH3 CHj—2CH->-CH2-^N02

uFl
Cci Cl {Completely delocalized
6n-electruns, aromatic)
(C) (D)

sr
group at C| creates a small +ve charge on C2 which +

slows down the ionization while in compound (C).

ko
o IhO
-i-I-effect of the CH3 groups helps in the ionization Ionization
+ Bf

to produce C!' ion. Thus, compound (C) is more

of
reactive than compound (D). (Completely delocalized
6n-electrons, aromatic)
Combining both the trends, the overall reactivity o
decreases in the order : (B) > (A) > (C) > (D), i.e.,
Y
NH2
option id) is correct. +
erB

101. (a) contains 2jt-electrons and hence is aromatic


HNO,
(c) contains 671-electrons and hence is aromatic -2NnO -N-.
uY

id) contains 67C-electrons (as shown below) and 4jt-Elcctrons

hence is aromatic. {aiitiaromalic)


Thus, option (c) is correct.
107. Out of the three compounds (I, II, III), compound
(I) is aromatic and hence is most .stable. Compound
ad
do

(II) is less stable than compound (I) because it is


only a cyclic conjugated diene while compound
Me.N (III) is least stable since it is neither an aromatic
in

compound nor a cyclic conjugated diene. Thus,


ib) neither contains 4n -1- 2 Ji-elecirons (actually the stability of alkenes decreases in the order : I >
contains 8;t) nor is planar (actually tub-shaped)
Re

II >m.
and hence is not aromatic.
Since enthalpy of hydrogenation is inversely
F

102. A and B are aromatic.


proportional to its stability, therefore, enthalpy of
103. 6 p-orbitals and 6 unpaired electrons form a cyclic hydrogenation decreases in the order :
electron cloud containing 6 7t, i.e., (4 n + 2) 7t- 111 > II > I, i.e., option id) is correct.
electrons which is responsible for aromatic
character. The seventh electron as such has nothing +

to do with the aromatic character of benzyl 108.


from HF
>
radical. Q‘C {Electrophilic
siihslitiition)
104. Cycloheptatrienyl cation is non-benzenoid
(because it does not contain a benzene ring) but is
aromatic.

105. If protonation occurs at Cj, it gives tropylium


H
cation which being aromatic is highly stable.
13/204
New Course Chemistry fxnprzsm

For Difficutt Questions


All other structures cannot have +ve charge on a

carbon atom next to the one carrying SOj group


H2SO4 _ and hence cannot eliminate SO3 to form benzene.
>
109. H2SO4
-HSOf ^
-f-
-H2O 118. Since - NO2 group is a powerful electron-
withdrawing group, therefore, carbocaiions (a),
Cl AICI3
-A\C\4* > + ^ AICI3 Cl
ib), {d) are destabilized. However, in carbocation

ow
n-Proyl chloride
-AlClJ (c) there is no - NO^ group. Thus, a-complex (c)
Isoproyl carbocalion is of lowest energy.
All these reactions first give the same isopropyl 119. 1, 4-Dimethylbenzene when heated with
carbocation intermediate and hence all give anhydrous AICI3 and HCl undergoes isomerization
isopropylbenzene as the final product. to give thermodynamically more stable 1, 3-
Please note that n-propyl chloride gives isopropyl dimethylbenzeneas shown below ;
carbocation after 1, 2-hydride shift as shown

e
below : CH3

re
rFl
Hi 1, 2-Methyl shift .

F
AICI3 1,2-Hydride
-A!Cl4^ shift ► /\
n-Propyl chloride Isoproyl
carbocation CH3 CH3
110.

or
MejCCO-l-Cl + AICI3
ou
0.
>‘Me3C'^C=0+AlCl4
H

CH3

ksf
I
CH3
^-CO '+H*
C5H5 CMC3 MeaC
oo
/erf-Butylbenzene -h"
CH3 CH3
The initially formed acylium ion (I) readily 1, 3-Diinethylbenzene
Y
eliminates a stable molecule of CO to form ten-
B

120. Electrophilic substitution reaction.


butyl carbocation which then attacks the benzene
ring to form rerr-bulylbenzene. 121. Since CH3 groups are o, /5-directing, therefore, o-
dimethylbenzene (i.e., o-xylene) gives only two
re

111. During bromination, one D atom is substituted by


Br atom, therefore, option (a) is correct. isomeric products on chlorination or nitration.
112. Sulphonation is a reversible reaction.
oYu

CH3 CH3 CH3


113. Since, I"*" is the electrophile, therefore, iodobenzene
ad

3 4
is fomted CH3 CH3 CH3
cu

Q FeCl
-I-
QP
d

I-^Cl + AICI3 ► r + Aicij Cl 1

o-Dimethyl- l-Chforo-2,3-
^ CgHsI -h Cl
in
Re

benzcnc dimethylbenzene
l-Chloro-3,4-
114. Chlorination in presence of AICI3 is an example dimethylbenzene
of electrophilic substitution.
F

115. r-Butylbenzene does not have a benzylic hydrogen 122. Both OCH3 and CH3 are o, p- directing groups.
and hence does not undergo oxidation with The possible position of attack which are
KMnO^ to give benzoic acid. facilitated by both OCH3 and CH3 are shown by
116. SO3H group is m-directing, therefore, arrows below:

sulphonation of benzene with excess of H2SO4 OCH3


gives m-benzenedisulphonic acid. OCH3
H sor Bri
4- FeBr3
117. + SO3 3 CH3
t Br
13/205
HYDROCARBONS

126.
- or<'’tSjt)C2X;o©
For Difficult Questions
CH2CH3 CH2CH3
Since, OCH3 group is bulkier than CH3 group,
therefore, due to steric hindrance the reaction does BrVFe CU/A
= ►
not occur at positions 2 and 6 but instead occurs (Nuclear [Side chain
at position 4. halogenalion) henzylic
halogena(ion)
123. In m-xylene, the electron-donating effect of the Br
two CH3 groups reinforce each other and hence
m-xylene is the most reactive. In p-xylene, one CH=CH^
Cl—CH—CH3
CH3 group increases the electron density at o-

w
position by hyperconjugation effect while the other
Ale. KOH.A
CH3 group by its +I-effect. As a result, the electron ¥
density in p-xylene is little more than in toluene (Zehydrohcilogcnation)
and hence p-xylene is expected to be little more

F lo
reactive than toluene. Thus, the overall reactivity Br Br
follows the order : II > III > I, i.e., option (b) is
correct. TI1U.S, option (d) is correct.
124. Electron-deonating groups increase while

ee
127. o-xylene gives different products. Refer to Ans.
electron-withdrawing groups diverse the reactivity toQ. 13.17, page 13/148.

Fr
towards electrophilic subsliluion reaction. Thus,
correct order is Thus, option (c) is correct.
IV (nitrobenzene) < II (bromibenzene) < II 128. Electron-withdrawing groups increase the rate of

for
(benzene) < I (p-xylene) < III (mesitylene) Birch reduction. Addition of electrons occurs at
125. Options suggest that CgH,o is ethylbenzene.
ur
1,4-positions w.r.t. to the electron-withdrawing
CH2CH3 CH2CH3 group to form a 1,4-dianion which subsequently
accepts protons from alcohol to form the addition
s
HN03/Br2S04 Br,, A
product.
ook
f n = ►
Yo
(Nitration) (Side chain
benzylic OCH3 OCH3
Ethylbenzene halogenalion)
eB

(CgHio) NO, 0,N O2N


+ 2c- e
(A)
From Na

Br—CH—CH3 CH=CH2 metal


r

0CH3
ad
ou

Ale. KOH

(Dehydrobromonaliort) H
+ 2H^
¥
0,N H
Y

From
NO2 NO2 CH3CH2OH H
(B) (C)
Re
nd

Thus, option (a) is correct.


Thus, option (b) is correct.
Fi

Multiple Choice Questions (with One or More than One Correct Answers)
4
129. The structure of 2,2-dimethylbutane is
CH3(X)
CH3
1 2I ^ 4

CH3—C—CH2 —CH3
CH3
Taking C3 as the front carbon and looking through *CH3(Y)
C3-C2 bond, the Newman projection of 2,2-
dimeihylbutane is as shown.
13/206
Ntvi Course Chemistry (XI)BZs]sn

' vr-ik'^fuT.-C:iCn©
For Difficult Questions

Therefore, X = CH3 and Y = CH3 and hence option 134.


{d) is correct.
Now look through C1-C2 bond taking C, as the
front carbon, the Newman projection of 2,2- (a)
dimethylbutane is as shown. '3 ±Jaici;
^(Completely delocalized
H(X) 2n-elec(rons, aromatic)

w
H3C^^ 2 .CH3
I

H H Na'H"

Flo
ib) + H,

C2Hs(Y)
QiCompletely delocalized

ree
Therefore, X = H and Y = C2H5 and hence option 6n-electrons, aromatic)
(b) is correct.
130. E, F and F, G are tautomers. Isomers which are

F
not non-superimposable mirror image of each (NH4)2C03
other as called diastereomers. Thus, F and G (i.e., (C)
110°-115'=’C
geometrical isomers) are also diastereomers.

or
Thus, options, (/?), (c) and {d) are correct.
ur O 0
HOl
H
lOH
H
131. r-Butyl bromide on treatment with NaOEt
undergoes elimination to give isobutylene while
NaOC2H5
k sf .."A
N

>
Yo
Br
~ -HBr >
oo
t-Butyl bromide Isobutylene
-2H2O
B

all the remaining reactions give /-butylbenzene as

the major product.


re

132. Only PhC = CCH3 does not give a methyl ketone K(Completely delocalized
(refer to Ans. to Q. 19, page. 13/124) 6n-clcctrons. aromatic)
u
ad

O"
Yo

OH cr
133. (b) < ►
nd
Re

O"
S(Compleiely delocalized
6n-electrons. aromatic)
id) /[ \ ◄ ►
Fi

Thus, all options are correct.

Both (b) and (c) are planar molecules and contain 135. (a) H3C NaOH, CaO, A

cyclic clouds of 47t-electrons and hence are COONa
(Decarboxylation)
antiaromatic. Therefore, these are unstable at room
H3C CH3
temperature. Compound under option (a) also
contains 47i-eIectrons but these are not delocalized Propane
over the entire ring. Therefore, it is non-aromatic. Cl Zn.dil. HCl
Compound under option (d) contains completely (c) H3C
delocalized brc-electrons and is also planar. Therefore, (Reduction) II3C CH3
it is aromatic. Propane
13/207
HYDROCARBONS

carJH;’^
For Difficult Questions

HD Multiple Choice Questions (Based on the given Passage/Compreh ension)


136. During catalytic hydrogenation, the hydrogens are transferred from the catalyst to the same side of die double bond
(refer to figure given in Ans. to Q. 18, page 13/124) thereby giving cri-alkenes. Evidently lesser the number of R
substituents, lesser is the steric hindrance and hence faster is the rate of hydrogenation. Hence, option {b) is correct.
138. Least substituted alkene has the maximum heat of hydrogenation, i.c., option (d) is correct.
VI. Integer ype Questions

w
144. Three ; Convert the given Fisher projection 148. Four ; acetylene, benzene, cyclobutadiene,
formula into the Newmanconformationand rotate cyclooctatetraene.

F lo
each conformationthus obtained through 60°. In 149. Five ; cyclopentadienyl anion, tropylium cation,
this way, we get three stable conformers with non azulene, cyclopropenyl cation, furan.
zero dipole moment as shown below : +
+

Cl 150. Five ;
60°
CH3

e
Br (anticlockwise)

Fre
CH3
27t-electrons 67t'dectrons 6n-electrons
Br Cl

CH3

for
Eclipsed
(unstable)
r
60°
6jt-electrons Phenanthrene
You
(anticlockwise) 147t-electrons
s
¥ (aromatic because
ook
it contains a benzene ring)

151. Six ;
eB

Sragged (stable) Eclipsed


(unstable)

CH3 (0 H2. Pd, BaS04, quinoline


60° 60°
our

Br CH3 (anticlockwise)
ad

(anticlockwise) (Reduction)
¥

Cl Br
Cl
Staggered (stable)
dY
Re

(p^O)

KMn04 (excess), 273 K


Fin

CH3 60° Hydroxylation


(anticlockwise)
¥

Eclipsed Staggered (stable)


(unstable) (PfO)

145. Six ; refer to Ans. to MCQ 31, page 13/115.


146. Three ; 2-butene, 2-pentene, 3-hexene.
147. Three; trans-l, 2-dichloroethene, rranr-2-butene,
Thus, product P has six hydroxyl groups.
2-butyne.

*
I
13/208
New Course Chemistry (XI)E slSU
'■ -L

For Difficult Questions

VII.
Numerical Value Type Questions (in Decimal Notation)
152. The complete sequence of reactions is ; Mol. mass of ethane = 30
NaOH Soda-lime Yield of D (in g)= 1.764x 30
CH3COOH » CH^COONa = 52-92
Acetic acid -H2O 630 K
Sod. acetate fA)
(2molcs) 909b
CH3 CHO
CU, hv
CH4 CH3-CI
Methane (B) Chlorination 153.
Methyl chloride (C) ♦
(70%)

w
80%

Na, dry ether Mol. mass = 92 Mol. mass - 106


Wurtz reaction ●> CH3-CH3 Now 92 g of toluene give benzaldehyde = 106 g

F lo
Ethane (D)
(I mole) 5 g of toluene with give benzaldehyde
From the equations it is evident that 106
2 moles of acetic acid yield ethane = 1 mole x5g
92
10 moles of acetic acid will yield ethane

e
10 But the % age yield = 92%

Fre
= — moles
2 Actual mass of benzaldehyde formed
Yield of D (in moles) 106
— x92

for
X
10 90 70 80 70 92 900
— ^ ^ ^ ^ _ = 1-764
2 100 100 100 100
= 530.00 g
r
VIII.1
Assertion-Reason Type Questions
You
s
156. Correct statenient-2. In bromobenzene,
ook

168. Correct reason. Since 5p-hybridized carbon in


mesomeric effect of the bromo group is more
dominant than its inductive effect in directing the alkynes is more electronegative than sp^-
hybridized carbon in alkenes, therefore,
eB

incoming electrophile.
nucleophiles can attack 5p-hybridized carbon more
157. Correct statement-l. Toluene on sulphonation easily than ^p--carbon in alkenes.
gives a mixture of o- and /?-toluenesulphonic acids.
158. Correct statement-1 [6]-Annulene is aromatic but 172. Correct reason. Planar completely conjugated
our
ad

[8]-annuleneand [lOj-annuleneare not aromatic. cyclic systems containing (4n + 2)n electrons are

aromatic in character.
160. Correct reason. HI adds to vinyl chloride
according to Markovnikov’s rule. 174. Correct reason. Since benzene is more reactive
161. Correct assertion. Propene reacts with HCl in than bromobenzene, therefore. F.C. reaction will
presence or absence of benzoyl peroxide to form occur preferentially in benzene rather than in
dY
Re

2-chloropropane. bromobenzene and hence benzene cannot be used


as a solvent in this reaction.
Correct reason. Addition of HCI does not follow
Kharasch effect or peroxide effect. 175. Correct explanation. The initially formed
Fin

n-

162. Correct reason. Carbon atom in ethene is sp~- propyl cation tirst undergoes rearrangement by
hybridized while it is sp hybridized in ethyne. 1,2-hydride shift to form the more stable isopropyl
163. Correct assertion. Calcium carbide on hydrolysis cation which then attacks the benzene irng to form
gives acetylene. isopropylbenzene.
Correct reason. CaC2 contains C^" anion. 176. Correct explanation. Irrespective of the length
165. Correct assertion. Acetylene reacts with ot the side chain, oxidation occurs at the more

sodamide to evolve NH3, i.e., reactive benzylic carbon leading to the formation
of benzoic acid.
HC = CH + NaNH2 ^ HC = CNa + NH3.
Correct reason. Acetylene is a stronger acid than 177. Correct assertion. Rates of nitration of benzene
ammonia. and hexadeuterobenzene are same.
166. Correct explanation. Acetylene is a stronger acid Correct reason. The C—D or C—H bond is not
than ammonia. involved in rate-determining step of the reaction.
I
*

ow
ENVIRONMENTAL CHEMISTRY

e
Fl
re
SECTION—!. ENVIRONMENT AND ENVIRONMENTAL POLLUTANTS

F
14.1. INTRODUCTION
The term “Environment” literally means “surroundings”.
ur It comprises of the following four major

r
components :

fo
(1) Atmosphere (2) Hydrosphere (3) Lithosphere and (4) Biosphere.
Let us first try to have a brief idea of each of these segments of the environment.
ks
(1) Atmosphere. Atmosphere is a cover of gases that extends to a height of about 1600 km above the
Yo
surface of the earth and protects the life on the earth from the harmful radiations (cosmic rays) coming from
oo
the sun or the outer space.
Functions of the atmosphere, (i) It contains all those gases which are essential for the existence of
B

life on the earth. For example, oxygen present in it is required by humans and animals for respiration and
CO2 is needed for photosynthesis in plants. Nitrogen which is present as a vital component is needed for plants
re

indirectly as it is first converted into water soluble compounds like ammonia and then added into the soil.
{«■) It is an important carrier of water vapour which are needed for all forms of life,
u
ad

(iii) Ozone present in it absorbs the harmful ultraviolet radiations coming from the sun and thus protects
Yo

life on the earth,

(iv) It maintains heat balance of the earth by absorbing infrared radiations coming from the sun and
it controls weather and climatic conditions.
d

those re-emitted from the earth. For the same reason,


Re

Temperature of the atmosphere. Different parts of the atmosphere absorb and reflect different arnounts
in

of heat of the solar radiation. Hence, the temperature of the atmosphere varies from - 56°C to 1200°C (as
F

shown in Table 14.1).


Composition of the Atmosphere or Air. The components of the atmosphere may be divided into three
categories as follows :
Major components: N2, O2, v/ater vapours
Minor components: Ar, CO2
Trace components ; He, Ne, Kr, Xe, CH^, H2, CO, N2O, SO2, NO, NO2, HCHO, NH3, O3.
* Not included in CBSE syllabus. This chapter has been given only for the preparation of competitive examinations,
14/1
14/2
'P'Mde^'A New Course Chemistry rxnronwn
Structure or Regions of the Atmosphere. The atmosphere may be considered to be divided mainly
into four regions above the surface of the earth. These regions are defined by the temperature variation with
height in addition to the decrease in density and chemical composition. The names of these regions, the
altitudes at which they exist, their temperature ranges and their chemical composition are given in Table 14.1 :
TABLE 14.1.
Main regions of the atmosphere
Region Altitude from Temperature range Gascs/Species present
earth’s surface

w
(0 Troposphere O-Il km Decreases from 15 to -56°C ^2’ ^2’ ^^2' ^2^ vapour
(//) Stratosphere 11-50 km Increases from -56 to -2°C N2, O2, O3, 0-atoms
(or ozonosphere)

Flo
(in) Mesosphere 50-85 km Decreases from -2 to -92°C
N2, O2, OJ. NO+
(iv) Thermosphere 85-500 km Increases from -92 to I200"C Oj,0\ NO^ e-

ee
IVoposphere is the most important region of the atmosphere because it is the domain of all living

Fr
orpnisms including animals and plants. This is the region which is greatly affected by air pollution, further,
this is the region which contains water vapour which are essential for the living organisms (Above iO km,
water is present as ice crystals). 80% of the mass of the atmosphere is in thi.s region. The temperature of this

or
region decreases with altitude and is minimum at about 11 km. This point is called tropopau.se. In the next
ur
region, namely stratosphere, the temperature begins to increase. Thus, tropopause is the point of temperature

f
inversion. The ozone present in the stratosphere absorbs the harmful ultraviolet radiation coming from the
sun and thus acts as an umbrella for the ultraviolet radiation for the living organisms on the earth. Due to
ks
presence of ozone layer, this region is also called ozonosphere. The rise in temperature is due to absorption
Yo
of ultraviolet rays which is converted into heat. Mesosphere and thermosphere (collectively called
oo
Ionosphere) contain gases in the ionized form. These ions reflect back the radiowaves to the earth. This
forms the basis of wireless communication.
B

These regions may be represented diagrammatically as shown in Fig. 14.1.


re

FIGURE 14.1
u

Vi ..
ad

110- RE
Yo

85—500 km
100- e

THERMOSPHERE
90 ^
' ’

80H
If <●«.!
i 70- .p.l smoifaeimiK
nd

:’ej
Re

,,,:.4MESQSPHEREiiU-i

iMt&UHMtKt
ai 1

.. iSTRATOPAUSE
:f* § 50-^ i*.-M.tJu;>r;«^STRATbPAUSE
Fi

H . y

STRATOSPHERE
STRATOSPHERE <
30- ^2’^2'^3 (OZONOSPHERE)
■11—50 km O-ATOMS
20-
TROPOPAUSE
loH
ffMKK&u) RE
0-f
LITHOSPHERE
-100® -80° -60° -40° -20° 0° 20°
EARTH TEMPERATURE IN'C

Regions of the atmosphere

*
$
ENVIRONMENTAL CHEMISTRY 14/3

There is one region above the thermosphere. It is called exosphere and is considered to the highest
region of the atmosphere. It lies in the range of 500-1600 km and contains mainly atomic and ionic oxygen,
hydrogen and helium. Beyond exosphere is the unbounded area called interstellar space.
Reasons for fall or rise in temperature in different regions of the atmosphere. In troposphere, the fall
in temperature with altitue is due to fall of pressure and density of air. In stratosphere, the irse in temperature
with altitude is due to absorption of solar ultraviolet radiation by ozone. In mesosphere, decrease in
temperature is due to low absorption of ultraviolet radiations by ozone.

(2) Hydrosphere. It forms that part of the environment which contains water in the form of sea, oceans,

w
rivers, lakes, ponds, etc. About 75 percent of the earth’s surface is covered by hydrosphere. Most of it is in the
oceans and contains about 3-5% of the dissolved salt. Fresh water is present in lakes or rivers or ponds which
flows into them from rain or melting of snow, etc.

F lo
(3) Lithosphere. It is the solid component of the earth consisting of soil, rocks, mountains etc. The
outermost (8-40 km) thick solid part of the earth is called the crust. The uppermost part of the eartli’s crust
contains weatliered rocks as well as organic matter and is called .soil. This is the most important part of

ee
lithosphere because we grow plants on this part. It is also a store-house of minerals.

Fr
(4) Biosphere. It is that part of the lithosphere, hydrosphere and atmosphere where living organisms
interact with these parts and thus live together. For example, green plants during photosynthesis give out
oxygen which is added into the atmosphere, animals inhale oxygen and give out carbon dioxide which is used

for
by plants for photosynthesis.
ur
Lastly, it is interesting to point out that a number of “natural cyclic processes” are taking place in the
environment which keep a balance of the different materials present in the environment. A natural cyclic
process is a process according to which a material of the environment is consumed in one way and then
s
ook
reproduced in another way.
Yo
For example, we have water cycle, oxygen cycle, nitrogen cycle, carbon cycle, phosphorus cycle, sulphur
cycle etc.
eB

Environmental Chemistry is that branch of science which deals with the chemical phenomena
occurring in the environment, i.e., study of origin, transport, reactions, effects and fates of
chemical species in the environment.
r
ou
ad

In fact, it is very wide branch of science which is confined not only to chemistry but also includes other
branches of science like Physics, Botany, Agriculture, Life sciences. Medical sciences etc. Thus, it is a multi
disciplinary science. A more general term for this branch is “environmental science” which deals with the
Y

study of sources of pollution and the methods of monitoring and controlling the pollutants.
Re
nd

Curiosity Question
f
Fi

Q. As we go up above the earth's surface, does temperature decrease continuously ? Explain.


Ans. No, temperature does not decrease continuously. From 0—11 km (called troprosphere),
temperature decreases. From 11—50 km (called stratosphere), temperature increases. From
50—85 km (called mesosphere), temperature again decreases and finally from 85—100 km
(called thermosphere),temperature again increases.
J
14.2. ENVIRONMENTAL POLLUTION AND ENVIRONMENTAL POLLUTANTS
The addition of any undesirable material to ai,r water and soil by a natural source or due to human
activity to such a level of concentration which adversely affects the quality’ of environment is called
“environmental pollution.” The undesirable material thus added to the environment is called a “pollutant.”

I A
14/4
New Course Chemistry (XI)EZ5IHI]

There was a time when our ancestors used to talk of purity in everything that we needed for our growth.
However, with the advancements in technology and industralisatio n, such undesirable materials are being
added into the environment that have disturbed this ecological balance (suitable co-existence of living
organisms, i.e., animals and plants with the non-living components Ike air, water soil, etc). Today everything
is polluted. The air we breathe in is polluted by the exhaust gases due to increase in vehicles, the fruit and
vegetables are polluted due to use of pesticides added into the soil and water is polluted due to toxic effluents
(wastes) being thrown into seas, rivers and lakes by the industries.
1 nvironmenlal pollution affects human beings, animals, plants as well as materials. It does not only
slow down growth of plants or bring illness to human beings but can cause havoc. A few episodes that have

w
occurred in the recent past due to environmental pollution are given below :
(0 In 1952, about 4000 people died in London due to respiratory diseases (London smog).
(ii) During 1953-1960, hundreds of Japanese died due to eating of fish from polluted Minamata Bay.

Flo
(Hi) The use of nuclear weapons in Hiroshima and Nagasaki killed thousands of people and the subsequent
emission of radiations affected hundreds of thousands of people.
(iV) On December 2, 1984, Bhopal gas tragedy (due to leakage of methyl isocyanate vapours) resulted

ee
into loss of about 3000 lives and affected about 50,000 people who are still suffering from respiratory, lung,
throat and eye diseases (see more details on page 14/34).

Fr
(v) In 1972, about 500 Iraqis died after consuming polluted wheat,
(w) In Greece and Italy, the historic marble statues have been damaged by acid rain.

for
(vii) Sulphur dioxide fumes are affecting the whiteness of the marble of Taj Mahal and making it pale.
ur
(viii) In the Ukaraine area of Soviet Union, thousands of people had lost their lives due to the world’s
worst nuclear disaster.

The main causes of pollution are


ks
(0 Fast population growth (H) Rapid urbanisation
Yo
o
(Hi) Excessive industralisation (iv) Use of pesticides in agriculture.
Bo

It is important to mention here that in a number of cases, the effect of the pollutants does not remain
localized. Some of these pollutants are carried by air or by other means so rapidly that their effect can be felt
within days or weaks in almost all parts of the world. For example, if a nuclear test is carried out in any part
re

of the world, its fall out can be detected in any other part of the world within a very short period. Similarly,
pesticides such as DDT have been found to have the adverse effect on animals living thousands of kilometer
away from the place of their actual use.
ou
ad

FIGURE 14.2
The pollution model may thus be represented TRANS
Y

as shown in Fig. 14.2. SOURCE


PORTED
ENTERS ASSIMILATED
OF ► ENVIRO BY ENVIRO
The gradual increase in pollution has now POLLUTANT
BY AIR OR
WATER
NMENT NMENT
reached alarming proportions. Developed countries
nd
Re

started taking steps to check pollution since early


sixties. In 1970, they celebrated, ‘Earth Day’ which BUILDS
. CAUSES
Fi

was organised by United Nations. In 1972, United UP IN


DAMAGE
ENVIRONMENT
Nations held a c(/nference on Human Environment at
Stockholm. Since then the developing countries Origin, transport, assimilation or build up in the
including India have also become concious to protect environment of a pollutant causing damage
their environment. The first step taken by
Government of India was to set up Central Ganga Authority to take necessary steps to purify the water of Ganga
and to make it pollution free. The latest step taken is the use of CNG (Condensed Natural Gas) in place of petrol
and disesel in metropolitan cities.
Difference between a pollutant and a contaminant. A contaminant is a substance which does not
occur in nature but is introduced into the environment
by human activity. It may or may not be harmful to the
living organism or non-living components. On the other hand, a pollutant is a substance produced either by
ENVIRONMENTAL CHEMISTRY 14/5

natural sources or by human activity or by the combined effect of both such that the concentration of that
substance becomes so high in the enviwnment that it has harmful effects on the living organisms or non
living components. Examples of pollulants include CO, SO2, SO3, oxides of nitrogen, dust particles etc. One
well known example of contaminant is that of pyrosulphuric acid (H-,S-)07) that had leaked from a defective
tank in Delhi killing many persons and causing skin and breathing problems to many others. As pyrosulphuric
acid does not occur in the atmosphere, therefore it is a contaminant. Further, as it had dangerous effects, so it
is also pollutant. Generally, highly toxic substances are considered as pollutants.
Threshold Limit Value (TLV). The permissible limit of a pollutant in the atmosphere to which if a
healthy worker Is exposedfor 8 hours a day or 40 hours a week throughout his life, there is no adverse effect
on him is called thre.shold limit value. These studies are made by experimentation on animals. For example,

w
TLV of CO is 40 ppm while that of COt is 5000 ppm. On the other hand, for the poisonous gas, phosgene,
TLV is only 01 ppm.
14.3. TYPES OF POLLUTANTS

F lo
Pollutants can be classified in two different ways as follows :
(1) Primary and Secondary pollutants
Primary pollutants are those which after their formation enter the environment and remain as such.

ee
For example, nitric oxide (NO) formed by bacterial decay or by lightning flashes (resulting into combination

Fr
of N2 and O2 of the air) becomes a pollutant if present in excessive amount. Similarly, SO2, NO2 etc. are also
primary pollutants.

for
Secondary pollutants are those harmful materials which are formed by chemical reactions between
the primarypollutants in the atmosphereor hydrosphere.For example, hydrocarbons and oxides of nitrogen
ur
which are primary pollutants react together in the presence of sunlight to form certain compounds {e.g., PAN,
i.e., Peroxyacyl nitrates, see page 14/19) which are also harmful. The compounds thus formed are called
secondary pollutants.
s
ook
(2) Biodegradable and Non-biodegradable pollutants
Yo
Biodegradable pollutants are materials such as domestic sewage, cow-dung etc. which are easily
eB

decomposed by the microorganisms either by the nature itself or by suitable treatment and thus are not
harmful but if these are present in excess in the environment, they do not undergo degradation completely and
thus become pollutants.
Non-biodegradablepollutants are materials such as mercury, aluminium, DDT etc. which do not
our
ad

undergo degradation or degrade very slowly but their presence even in very small amounts in the environment
is very harmful for the humans as well as plants. They may react with other compounds present in the
environmentand produce even more toxic compounds.
Y

14.4. TYPES OF POLLUTION


Re

Pollution can not only be due to addition of undesirable materials into the environments but can also be
nd

due to factors like noise, electromagneticwaves (like ultraviolet rays) etc. Funher, it can be classified in two
differentways as follows :
Fi

(a) Depending upon the part of the environment polluted. For example (1) Air pollution (2) Water
pollution (3) Soil pollution or Land pollution.
(b) Depending upon the ' ature of the pollutant added into the environment.
A few common example.s of this type tue as follows :
(1) Radioactive pollution (2) Plastic pollution (3) Soap and detergent pollution
(4) Oil pollution (5) Acid Rain pollution (6) Smog pollution
(7) Chemical pollution (8) Slit pollution (9) Metal toxicity pollution
(10) Effluent pollution (11) Noise pollution (12) Thermal pollution
In this unit, we shall discuss briefly the pollution according to first classification.

i I
14/6
'Pn<tdeefr'<x- New Course Chemistry (XI)KEIHD

(—II. AIR OR ATMOSPHERIC POLLUTION

14.5. INTRODUCTION

Air pollution is defined as the addition of undesirable materials into the atmosphere either due
to natural phenomena or due to human activity on the earth which adversely affect the quality
of the air and hence affects the life on the earth.

When we talk of air pollution, we mean the pollution of air upto a height of about 50 km above the
surface of the earth. This part of the air is called homosphere because the different constituents present in it
form a homogeneous mixture.

w
The composition of the pure dry air is as follows : N2 (78-10%), O2 (20-95%), Ar (0-93%),
CO2 (0-032%), Ne (18 ppm). He (5-2 ppm), CH4 (1-3 ppm), Kr (10 ppm), H2 (0-5 ppm), N2O (0-25 ppm),
CO (0-10 ppm), O3 (0-02 ppm), SO2 (0-001 ppm), NO2 (0-001 ppm).

F lo
Ail' has never been pure right from the day the man started burning fuel. The oxides of carbon, nitrogen
and sulphur become pollutants if their amounts exceed a certain limit (called tolerance limit).
In nature, some amount of certain pollutants is taken up by some medium present in the environment.
Such a medium is called a sink for that pollutant. For example, oceans act as sinks for SO2 and CO2 and

ee
micro-organisms act as sink for CO. The living organisms like animals or plants or the materials like metals

Fr
and buildings which are adversely affected by the pollutants are called receptors or targets.
14.6. SOURCES OF AIR/ATMOSPHERIC POLLUTION

for
The main sources of air pollution may be classified into two categories as follows :
(1) Natural sources. A few examples of the natural sources of pollution are as under :
r
(0 Volcanic eruptions emitting poisonous gases like CO, H2S, SO2, etc.
You
s
(//) Forest fires and coal-refuse fires.
ook

(Hi) Vegetation decay.


(iv) Pollen grains of flowers.
eB

(2) Man-made sources, i.e., Sources due to human activity. A few example may be cited as follows :
(/) Burning of fossil fuels (wood, coal, etc.) which produce some poisonous gases as by-products such
as CO, SO2, oxides of nitrogen (NO^), CH4, etc.
our
ad

(ii) Combustion of gasoline in the automobiles, e.g., cars, scooters, buses, trucks, etc. They emit out
poisonous gases like CO, oxides of nitrogen and unbumt hydrocarbons in addition to the particles of lead.
(Hi) Increase in population. This is one of the major causes of pollution. More the population, more
are the needs, greater are the unnatural methods adopted which disturb the balance or equilibrium of the
dY

atmosphere.
Re

(iv) Deforestation. Man has been cutting trees indiscriminately to meet his needs. This has resulted in
Fin

increase in the percentage of CO2 and decrease in the percentage of oxygen in the air (because plants take up
CO2 for photosynthesis and give out O2).
(v) Fast industrialisation. In the last few years, the number of industries in different parts of the world
and their production has increased manifold. These include paper mills, sugar mills, rubber and plastic
industries, metallurgical industries using smelters, leather industries, petroleum refineries, refrigeration, mining,
etc. The smoke coming out of these industries contains not only carbon particles but a number of poisonous
gases like CO, CO2, SO2, H2S, NO, NO2, etc. These industries are responsible for about 20% of the total air
pollution.
(vi) Agricultural activities. The pesticides added to the soil or the sprays done over the crops are
carried by the wind to different parts of the town where they give a foul smell and affect the health of animals
and human beings.

I
ENVIRONMENTAL CHEMISTRY 14/7

(vii) Wars. The nuclear weapons used during wars emit out radiations which adversely affect the health
and prove to be fatal.

w
The study of atmospheric pollution mainly consists of the study of the following two lower parts of the
atmosphere.
(1) Tropospheric pollution, i.e., in the lowest portion of the atmosphere which extends to a height of
about 10 km from sea level.

(2) Stratospheric pollution, i.e., in the region above the troposphere, lying between 10 km to 50 km
above sea level.

The main pollutants of the troposphere are :

ro
re
(a) Gaseous pollutants such as oxides of carbon, sulphur and nitrogen, hydrogen sulphide, etc.
(b) Particulate pollutants such as dust, mist, fumes, smoke, smog, etc.
The stratospheric pollution is mainly due to the formation and breaking down of ozone in this part of the

F
atmosphere. The depletion of ozone layer by chlorofluorocarbons (CFCs) produces ‘ozone hole’ through

Fl
which UV radiation pass into the troposphere thereby affecting the plants and human life adversely.

u
14.7. TROPOSPHERIC POLLUTION

sr
The five major pollutants present in the troposphere are :
(1) Carbon monoxide (CO) (2) Hydrocarbons, (C,H^,) (3) Oxides of nitrogen (NOp

ko
(4) Oxides of sulphur (SO^)
o
(5) Particulates.

of
Now, we shall discuss the sources, hannful effects and the methods to control each of these pollutants
one by one.
(1) Carbon monoxide (CO) as pollutant. It is one of the most serious air pollutant. It is a colourless,
o
Y
odourless and tasteless gas. It is 96-5% as heavy as air.
Sources of CO production
erB

(a) Due to human activity, e.g.,


(0 Incomplete combustion of fossil fuels or hydrocarbon fuels in the automobiles.
uY

1
C + -0., ^CO
2 2
Similarly, the complete combustion of one mole of butane (C4HJ0) requires 6-5 moles of oxygen. If
ad
do

only 6 moles of oxygen are available, one mole of CO and 3 moles of CO2 are produced
C4H10 (g) + 6-5 O2 (g) > 4 CO2 (g) + 5 H2O (/)
C4H10 (g) + 6 O2 (g) >3 CO2 (g) + CO (g) + 5 H2O (/)
in

(i7) Forest fires or agricultural waste burning.


Re

iiii) Reaction of CO2 (present in the air or produced due to complete combustion of carbon containing
compounds) with cai-bon containing materials at high temperature (or with coke, e.g., in a blast furnace)
F

CO2 + C > 2 CO
(iv) Dissociation of CO2 at high temperatures (2000 K)
CO2 CO + O
At one time, it was believed that the major source of CO in the air was due to incomplete combustion of
fuels. But now it has been found that only 7% of CO arises from the man-made sources whereas the remaining
93% comes from the natural sources as given below.
(b) Natural sources, e.g.,
(i) From conversion of methane produced by the bacterial decay of living or organic matter (80%).
(») From synthesis and decay of chlorophyll (3%).
(Hi) From oceans and other undeterminedsources (10%).
14/8
New Course Chemistry (XOQSIBD

Sinks of CO. Although a large amount of CO is being added into the air either due to human activity or
due to natural sources, yet the level of CO in the air does not rise too much. This is because a significant
amount of CO is converted into CO2 by the micro-organisms present in the soil or by reactions in the atmosphere.
In other words, micro-organisms (bacteria) pre.sent in the soil act as a sink for CO.
In the urban areas where there is heavy vehicular traffic, the level of CO in the air is high as soil
available is insufficient to act as a sink. On the other hand, in the rural areas, the level of CO in the air is low
which is within the tolerance limit.

Harmful Effects of CO. Carbon monoxide is poisonous because it combines with haemoglobin of the
red blood corpuscles (R.B.C.) about 200 limes more easily than does oxygen to form carboxyhaemoglobin*
reversibly as : Hb + CO HbCO

w
The normal function of the haemoglobin is to combine with oxygen in the lungs to fonn oxy-haemoglobin
reversibly as : Hb + 02 T HbOo

F lo
The oxyhaemoglobin travels to the different body cells where it gives up oxygen and takes up CO-, for
return to the lungs which is then exhaled out. However, if large amount of CO is present in tlie surrounding
air, it combines more readily with the haemoglobin (as mentioned above). Thus, transport of oxygen to the
different parts of the body is inhibited. In other words, the body becomes oxygen-starved.

ee
It has been found that the maximum permissible concentration of CO in the ambient air (surrounding

Fr
air) is 40 ppm for an exposure of 6-8 hours. At concentrations greater than 100 ppm. difficulty in breathing
starts and there is headache and dizziness. Concentrations of 750 ppm or more lead to acute oxygen-starvation
(called anoxia or asphyxiation) and lead to coma and death.

for
Smokers are more prone to anoxia because they inhale a lot of CO along with the smoke of the cigarette.
In their body, the level of carboxyhaemoglobin easily exceeds 5%. In their ca.ses or in case of heart patients,
ur
even a level of 10 ppm of CO in the atmosphere can prove fatal.
Caution. It is dangerous to sleep in a closed room with a burning coke angithi to warm up the room on
s
ook
a cold night in winter. This is because in the presence of insufficient oxygen, coke may hum to produce a
Yo
lot of carbon monoxide which may cause poisoning and lead to death.
eB

Remedial treatment for CO poisoning :


(/) Carry the patient into the fresh air immediately and do not allow him to walk.
(//) Loosen his clothes and take off his shoes.
our

(Hi) Give artificial respiration if he is not able to breathe properly.


ad

(iv) In the hospital, the patient is kept in a high pressure chamber containing oxygen at 2 to 2-5 atm
pressure. Under pressure, CO of carboxyhaemoglobin is replaced by O2 and thus transport of O^ to different
parts of the body starts.
Y

HbCO + O 2 ± Hb02 + CO
Re

Control of CO pollution. The main source of CO pollution due to human activity is the use of internal
nd

combustion engines in the automobiles. These engines emit a mixture of CO, NO^, hydrocarbons and particulates.
However, any method used to control any one of these pollutants results in the increase of the other pollutants. For
Fi

example, if the air-fuel ratio is increased from 13 to 16, the emission of CO and hydrocarbons is reduced but the
emission of NO^ increases. Thus, to control the CO emission, suitable modificadons have to be done in the internal
combustion engine or in the quality of fuel. A few of these are as follows :
(/) The carburettor is adjusted so as to give a proper air-fuel ratio.
(ii) The exhaust system is developed in such a way that it brings about complete combustion in the fuel
so that negligible CO is fonned.
(Hi) Catalytic convertors can be fitted into the exhaust pipe which may convert the poisonous gases like
CO into harmless products before they are thrown into the air. However, the use of tetraethyl lead as anti-
*The carboxyhaemoglobin is about 300 times more stable than the oxyhaemoglobin complex.
** Moreover, tobacco contains nicotine which is a very hannful chemical for health.
ENVIRONMENTAL CHEMISTRY 14/9

knocking agent in the gasoline creates a problem because the catalyst like platinum get easily poisoned by the
lead. That is why the use of lead-free (unleaded) gasoline has been started.
(iv) Modification of internal combustion engines have been done by certain companies like Honda and
Toyoto of Japan instead of catalytic convertors. They contain an extra combustion chamber so that complete
combustion takes place and exhaust gases have very low amounts of oxides of nitrogen and almost no carbon
monoxide.

(v) Instead of gasoline, the use of CNG (Condensed n iuiral gas) and LNG (Liquefied natural gi.;) lies
been started as they are pollution-free fuels.
SUPPLEMENT YOUR
KNOWLEDGE FOR COMPETITIONS

w
1. CO2 is not a pollutant. CO2 present in norma! concentration is not an air pollutant. Rather, it helps
to maintain the temperature of the eaith required for existence of living organisms. A balance of
CO2 is maintained in the air because CO2 is produced from respiration, burning of fossil fuels and

F lo
decomposition of limestone to produce lime but at the same time, it is consumed by green plants for
photosynthesis. However, when CO-) level increases in the atmosphere due to deforestation and
burning of fossil fuels, it results in global warming (called greenhouse effect) as discus.sed later in
Art. 14.12.

ee
2. How CO links to haemoglobin ? In haemoglobin, Fe(II) atom is coordinated to five groups and the sixth

Fr
site is free. Normally 0-> links to Fe(II) at this sixth coordination site but CO is a stronger ligand than 0-,,
therefore, if CO is present, it occupies the vacant coordination site and not O2.

for
(2) Hydrocarbons as pollutants. Many different hydrocarbons are present in the air. Most of these
hydrocarbons are of low molecular mass (containing upto nearly 10 carbon atoms) and are gases or volatile
ur
liquids at ordinary temperatures. Methane (CH4), the simplest hydrocarbon, is the most abundant hydrocarbon
pollutant. In Los Angeles (in USA) where the vehicular traffic is very high, its amount in the air was found to
s
be 3 ppm in 1965.
ook
Yo
Sources of hydrocarbons :
(a) Natural sources,
eB

(i) Trees emit large amounts of hydrocarbons into the air.


(//) Methane is released into the air in large quantities by the anaerobic decomposition of organic matter
in soil, water and sediments.
Bacteria
our

2CH2O > CH4 + COt


ad

(Hi) Domestic animals contribute a very huge amount of methane into the air every year.
(h) Man>made sources :
(/) Just as in case of CO, the largest man-made source of hydrocarbons pollutants is the internal combustion
Y

engine. About 15% of the total hydrocarbons present in the air is from this .source. Motor fuels are mixtures of
Re

hydrocarbons. They are introduced into the air when unbumt or partially burnt fuel is emitted in the automobile
nd

exhaust. Gasolinealso escapes into the air due to evaporation from the fuel lank and the engine.
{«) Some hydrocarbon pollution also takes place due to burning of stationary fuel, i.e., coal, wood,
Fi

kerosene oil, etc.


(Hi) Evaporation of organic solvents like benzene, toluene, etc. during industrial operations, e.g., in
paint, varnish, lacquer industries, etc. add substantially (about 10%) to the total hydrocarbon pollution.
Hydrocarbon sinks. Hydrocarbons are quite stable. For example, methane has a residence time of
about 3 to 7 years. However, they undergo chemical reactions and photochemical reactions through a series
of steps and ultimately form products like CO2 or other water soluble products which are then washed away
by rain.
Harmful effects. (0 At low concentrations, as they exist in the air at present, there are no harmful
effects of the hydrocarbons on the human beings. However, if their concentration rises to 500-1000 ppm, they
have carcinogenic effect on the lungs and cause cancer.
14/10 “Pn^uice^ New Course Chemistry (XI) tTZ*lwll

(//) Due to their photochemical reactions with oxygen and oxides of nitrogen, they form photochemical
oxidants and photochemical smog (discussed later) which have a strong damaging effect on the human being
■ as well as plants.
Control of hydrocarbon pollution. As automobiles are the main source of hydrocarbon pollution, all
those steps taken to control CO pollution are needed for controlling hydrocarbon pollution. These have
already been discussed.
(3) Oxides of nitrogen (NO^.) as pollutants. A number of oxides of nitrogen such as NO, N2O, NO2,
N2 33 and N2O5 (represented by the general formula NO^) are introduced into the atmosphere due to natural
sources and due to human activity. Out of these only two oxides, i.e., NO and NO2 are considered as pollutants.

w
Nitric oxide (NO) is colourless and odourless gas whereas nitrogen dioxide (NO^) is reddish brown gas
having pungent smell and is suffocating in nature.
Sources of NO^ production :

F lo
(a) Natural sources. Natural bacterial action is the only natural source which discharges NO^ mainly
in the fonn of NO, into the atmosphere in huge amounts. Besides, lightning discharge also results in the
combination of N2 and O2 to form NO.
(b) Man-made sources. (/) Combustion of fossil fuels (coal, oil, natural gas, gasoline, etc.) both in

ee
motor vehicles and stationary sources or power plants produces so high temperatures at which oxidation of

Fr
the atmospheric nitrogen takes place forming NO and NO2 as follows :
1200-1750'’C

N2 + 02 ^ i 2 NO

for
2NO + O0 - 2 NO2
our
The first reaction takes place at a temperature of 1200-1750°C. The reverse reaction is prevented due to
rapid cooling of NO. The second reaction takes place at about 1100°C.
(/7) Chemical industries like sulphuric acid and nitric acid industries produce NO^ as by-products which
s
ok
are discharged into the air.
The natural sources discharge about 5 x 10^ tonnes of NO^ into the air mainly in the form of NO
o
whereas man-made sources release about 5x10^ tonnes of NO^ every year all over the world. Further, the
Y
eB

distribution of NO^. from natural sources is uniform but man-made is not uniform. Its concentration is
10-100 times greater in the urban areas (due to greater industrialisation and automobile population) than in
the rural areas.

Sinks of NO_j. pollutant. The average residence time of NO is 4 days while that of NO2 is 3 days.
r
ad

During this time, they are converted into nitric acid (HNO3) through the following reactions in which ozone
ou

plays an important role.


2 NO + O2 ^2N02
Y

2 NO2 + 2 HjO + O2 ^ 4 HNO3


or
NO2 + O3 - ■> NO3 + O2
Re

NO2 + NO3
nd

N2O5 -I- H26 -> 2 HNO3


Fi

The end product is nitric acid. It comes down from the atmosphere to the surface of the earth in the form
of acid rain. Here, it reacts with the bases such as ammonia, lime, etc. to form nitrates.
Harmful effects of NO^ pollution. (/) Neither NO nor NO-, is a health hazard at the present pollution
level. Out of the two, NO2 is more dangerous as it is a toxic gas and affects respiratory system and damages
the lungs. Its concentration at 100 ppm may be fatal for animals and human beings. Though NO has about
1500 times greater afifnity than CO for haemoglobin but fortunately, it is unable to enter the blood stream
from the atmosphere. But its presence in large amount may result in the increase of NO2 by oxidation and
thus become harmful.

(/■/) Some sensitive or ornamental plants ttre affected by the presence of oxides of nitrogen in the ambient
air (surrounding air) even at low concentrations. Their photosynthetic activity decreases and they start shedding
their leaves and fruits.
14/11
ENVIRONMENTAL CHEMISTRY

(Hi) In the presence of NO^, textiles like cotton, rayon, etc. start fading.
(/V) The acid rain causes the pH of the soil to decrease to 4 or 5 and thus reduces the fertility of the soil.
It also has a corroding effect on marble buildings (like Taj Mahal at Agra) and the metallic structures,
(v) The most serious hazard of nitrogen oxides is in the formation of an unpleasant mixture of gases and
particulates that make up photochemical smog.* It is formed as a result of photochemical reaction taking
place between oxides of nitrogen and hydrocarbons forming ozone, peroxyacylnitrates (PAN) and aldehydes
(RCHO). Los Angeles is famous for this type of smog.
Control of NO_^ pollution. (/) Just as in case of CO, the catalytic convertors are used in the automobile
exhaust which in the first stage convert the oxides of nitrogen to free N2 or to a small amount of NH3.

w
(ii) The flue gases coming from power plants or industrial units and containing NO2 and SO2 are freed
from these gases by scrubbing them with sulphuric acid. The following reactions take place :
1 si step ; NOo + SO2 + H2O > H2SO4 + NO

Flo
2nd step : NO + NO2 ^ N2O3
3rd step : N2O3 + 2H2SO4 >2N0HS04 + H20
The flue gases thus freed from NO, and SO2 are released into the atmosphere. The reaction product

ee
NOHSO4 is decomposed to get H2SO4 which is then used again for scrubbing.
As NO. and SO, are acidic oxides, scrubbing can also be done with alkaline solutions such as Ca(OH)2

Fr
and Mg(OH), .
(4) Oxides ofsulphur as pollutants. The two oxides of sulphur, i.e., SO2 and SO3 (generally represented
by the general formule SO^.) are the most serious pollutants of the atmosphere. Both are colourless gases with

for
pungent smell.
ur
Natural source of release of SO2 into the atmosphere is the volcanic eruptions. It accounts for about
67% of the total amount of SO2 present on the globe. The remaining 33% is discharged into the atmosphere
ks
due to human activity.
Yo
Man-made Sources mainly include the following :
oo
(/) Combustion of sulphur containing coal and fuel oil in the thermal power plants.
eB

(ii) Roasting of sulphide ores in the smelting industries, e.g., iron pyrites (FeS,), copper pyrites (CuFeS2),
copper glance (Cu,S), zinc blende (ZnS), galena (PbS), etc.
e.g.. Cu,S + O2 ^ 2 Cu + SO2
(Hi) A small amount of SO2 undergoes photolytic oxidation in the air to form SO3
r
ou
ad

2 SO, + O2 > 2 SO3.


(iv) In the presence of water vapour, SO3 reacts to form H2SO4 which remains suspended in the air as
Y

droplets or comes down in the form of acid rain.


SO3 + H2O > H2SO4
As in the case of NO,., the sulphur oxides produced from natural sources are uniformly distributed over
nd
Re

the globe but SO^ pollution due to man-made sources is more in the industrial towns or urban areas than in
non-industrial or rural areas.
Fi

Harmful effects of SO_^. (i) SO, and SO3 are both strongly irritating to the respiratory tract SO2 at a
concentration of 5 ppm causes throat and eye irritation (resulting into cough and tears and redness in eyes). It
causes breathlessness and affects larynx, i.e., voice box (upper part of the wind pipe). SO3 is more harmful
even at a concentration of I ppm and causes severe discomfort. Elderly persons and those with heart and lung
diseases are most seriously affected.
(ii) Even a very low concentration of SO2 (0.03 ppm) has a very damaging effect on the plants. If
exposed for a long lime, i.e., a few days or weeks, it slows down the (ormalion of chlorophyll resulting in the
injury to the leaf including loss of green colour. This is called chlorosis.
♦The word “smog” is a combination of “smoke” and “fog”. It is a misnomer here because photochemical
smog is a mixture of a number of unpleasant compounds but not of smoke and fog.
14/12
New Course Chemistry (XI)SEIH

(lii) Sulphur oxides as such or after being converted into H2SO4 damage a number of materials, e.g.,
(a) They damage building materials especially marble. For example, marble of Taj Mahal in Agra is
being damaged due to Mathura refinery, thermal power station and a number of foundaries located nearby.
(h) They corrode metals, particularly iron and steel,
(c) They bring about fading and deterioration of fabrics, leather and paper.
(^0 They affect the colour of the paints.
Sinks of SO^. The sulphur oxides are converted into H2SO4 which combines with the limestone minerals
and thus removed considerably from the atmosphere. Thus, limestone minerals act as sink for sulphur oxides.

w
Control of SO^ pollution. The sulphur oxide pollution can be controlled in a number of ways as follows :
(0 By use of low-sulphur or sulphur-free fuels like natural gas.
iii) By removing sulphur from the fuel (e.g., from fuel oil) before burning.

Flo
(Hi) By making sulphur-free liqueifed gaseous fuel from coal.
But methods (//) and (Hi) are not economical,

e
(/v) By using alternate sources of energy, e.g., hydroelectric power plants and nuclear plants,

re
(v) By removal ofSOJromflue gases. Sulphur oxides can be removed from flue gases by using chemical
scrubbers. These chemical scrubbers contain a slurry of limestone, i.e., CaCO^. The flue gases are passed

F
through this slurry. SO2 combines to form CaS03 whereas SO3 combines to form CaS04 as follows :
CaC03 + SO2 > CaS03 + CO2
ur
or
CaC03 + SO3 ^ CaS04 + CO2
This method is economical but it produces huge amounts of solid CaS03 and CaS04, the disposal of
which is a great problem.

sf
An alternative method is based upon the reaction between HSOJ ions produced from SO2 and citrate
k
Yo
ions (H2Cit ). The flue gases are cooled to about 50°C and then passed through a solution containing citrate
oo
ions. The following reactions take place leading to the formation of a citrate complex :
eB

SO2 + H2O HSO^+H+


HSO3 + H2Cir [HS03.H2Cit]2-
The above solution is then transferred into a closed vessel and H2S gas is passed through it. As a result,
ur

sulphur precipitates out and the citrate ions are regenerated. The sulphur obtained in 99-9% pure and is thus
ad

an excellent by-product.
Yo

(5) Particulates as pollutants. Small solid particles and liquid droplets suspended in air are collectively
called as particulates. Some important characteristics of these particulates are as under :
(i) Size. Their size (diameter) varies from 5 nm to 500,000 nm.
d

(ii) Mass. Their mass varies from 60 )ig to 2000 pg per m^ in urban and industrial areas,
Re
in

(m) Concentration. Their number varies from a few hundred per cm^ in clear air to 10^ per cm^ in
polluted air.
F

(A) Surface area. They have large surface area due to which they have a good sorption for organic as
well as inorganic matter,
(v) Chemical nature. Different types of particulates have different chemical nature.
Sources of particulates :
Natural sources include volcanic eruption, grinding, blowing of dust and soil by the wind, spraying of
salts by seas and oceans, etc.
Man-made sources include a large number of materials suspended in the air as particulates, e.g.,
(0 Soot. These are the smallest particulates (diameter about 5 nm). They are the most common particulates
introduced into the air due to incomplete combustion of fossil fuels such as coal, wood fuel oil, natural gas.
etc.
14/13
ENVIRONMENTAL CHEMISTRY

(ii) Fly ash. These are ash particles coming along with the furnace flue gases and are the biggest
particulates (diameter about 500,000 nm). They are produced due to the combustion of high-ash fossil fuels
as in the thermal power plants or in smelting and mining processes.

(///) Inorganic particulates. A few common inorganic particulates are as follows :


(a) Metallic particles. These are discharged into the air during finishing of metals.
(b) Metal oxides. They are released into the atmosphere during the combustion of fuels containing
metallic compounds. For example, when coal containing iron pyrites is burnt, particulates of Fe304 are
introduced into the air
3 FeS2 + 8 O2 > Fe304 + 6 SO2
(c) Lead halides. Tetraethyl lead, Pb(C2H5)4, is added to gasoline to act as an anti-knocking agent.
During combustion in the engine, it is oxidized to PbO which deposits in different parts of the engine and
many cause damage. To avoid this damage, Pb(C2Hg)4 is mixed with dichloroethane and dibromoethane

w
which convert PbO into PbCl2 and PbBr2 which are volatile and thus come along with the exhaust gases and
introduced into the atmosphere

F lo
Pb(C2H5)4 + O2 + C2H4CI2 + C2H4Br2 ■> CO2 + H2O + PbCl2 + PbBr2
Thus, engine is protected but air gets polluted.
(d) Asbestos dust Asbestos is a fibrous silicate mineral which is introduced into the atmosphere by the
industries manufacturing asbestos sheets, ropes, etc.
(e) Sulphuric acid and nitric acid. These particulates in the form of droplets are produced in the

ree
atmosphere when oxides of sulphur or nitrogen come in contact with the moisture.

F
2 SO2 + O2 + H2O > 2 H2SO4

for
4 NO2 + 2 H2O + O2 > 4 HNO3 or 2 NO2 + O3 + H2O » 2 HNO3 + O2
Their size lies in the range of 500-10(X) nm.
(/v) Organic particulates. Organic particulates include paraffins, olefins and aromatic compounds,
r
particularly polycyclic aromatic hydrocarbons (PAH). These originate from combustion of static fuels or
You
automobile fuels or from petroleum refineries. The polycyclic hydrocarbons are easily adsorbed on the soot
oks

particles and prove to be a great health hazard.


eBo

It may be mentioned that man-made particulates are almost equally contributed by the three sources,
namely stationary fuel combustion, industrial processes and fires such as forest-fires or agriculmral burning
of wastes.
Alternatively, the particulates in the atmosphere may be divided into two categories, i.e., viable and
ad
our

non*viable.
The viable particulates are the small size living organisms such as bacteria, fungi, moulds, algae, etc.
which are dispersed into the air. The fungi cause aUergy in the human beings and diseases in the plants.
The non-viable particulates are formed as a result of the disintegration of large size materials or by
condensation of small size particles or droplets. The atmosphere contains four types of non-viable particulates.
Re
dY

These are mist, smoke, fumes and dust.


Mists are produced from the particles of the spray liquids, e.g., from herbicides and insecticides and the
Fin

condensation of the vapours in the air.


Smoke consists of small soot particles produced as a result of the combustion of orgamc matter, e.g.,
oil, tobacco, carbon smoke, etc.
Fumes are the vapours of certain materials suspended in the air, e.g., metallurgical fumes (fumes of
metals) and alkali fiimes.
Dust denotes fine particles produced during certain industrial processes, e.g., crushing, grinding. It
consists of limestone particles, sand, pulverised coal, cement, fly ash, silica dust, etc.
Harmful effects of particulate pollutants :
Effect on humans, (i) Small sized particulates have greater damaging effect on the humans because
they can pass through the nose and enter the lungs easily whereas entry of big sized particles is checked in
14/14
N-’vv '!!ourse Chemistiy fxnrosrwm

the nose and throat and can be easily removed from there. The small particulates thus accumulated in the
lungs have large surface area and act as excellent sites for adsorption of carcinogenic compounds such as
polynuclear hydrocarbons, asbestos etc. and cause diseases like lung cancer, bronchital asthma, chronic
bronchitis, etc. This disease of the lungs is common among the industrial workers and is called
Pneumoconiosis,
(n) Different types of lung diseases are caused by different types of particulates, e.g., asbestos causes
asbestosis. dust containing free silica {Si02) causes silicosis, co^ miners suffer from black lung disease
whereas textile workers suffer from white lung disease, beryllium compounds such as BeClo, BeSO^, etc
cause beryllosis, etc. ^
(Hi) Lead particulates are highly poisonous and have a serious effect on children’s brain and cause
cancer.

w
Effect on plants. Particulates deposit on the leaves of the plants thereby blocking the stomata and
retarding the transpiration of minerals from the soil. Also it hinders the intake of CO2 from the air thereby
inhibiting photosynhesis.

F lo
Effect on materials. Particulates damage a large number of materials, e.g., soot, dust, fumes, mists etc.
damage paints, clothes, draperies, buildings, soil, sculptures, monuments etc. They also accelerate corrosion
of metals.

Effect on visibility. The light scattered by particulates results in poor visibility of the object in front of

ee
us as it happens at night when we are driving a car or scooter especially on foggy nights. Similarly, the
sunlight scattered by particulates illuminates the air and reduces visibility due to decrease in contrast between

Fr
the object and its background.
Effect on climate. Pailiculates act as nuclei for cloud formation and hence affect the climate. Further,

for
they scatter and reflect back the heat of the sunlight and thus control the wanning up of the earth’s surface due
to increase in CO-,,
2’ i.e., they counteract the greenhouse effect (discussed later).
ur
Control of particulate pollution. To reduce the pollution in the air, the removal of particulates is very
essential. This is done by using the following techniques which are based on settling or washing away of the
s
particulates.
ook
Yo
(/) Gravity settling chamber. It consists of a horizontal rectangular tank. It is so large that the effluent
gases which are allowed to enter into it slow down. As a result, larger particles settle down. The fine particles,
eB

however, cannot be removed by this method.


(ii) Cyclone collector. In this technique, the gas is allowed to flow in a tight circular .spiral. Due to the
centrifugal force, the particulates move towards the wall and start settling down.
(Hi) Wet scrubbers. In this technique, spray towers or chambers are used into which a suitable liquid
our
ad

(usually water) is introduced in the form of a fine spray wliich washes away the particulates.
(iv) Electrostatic precipitator. This is the most effective and efficient method for the removal of
particulates. About 99% of the particulates can be removed by this method. It is based on the principle that
particulates of all size (especially aerosol particles) can be made to acquire negative charge under the influence
Y

of high electric field. These particles are then attracted towards the positive electrode of the electric field
Re

where they start accumulating and then removed.*


d

14.8. CHEMICAL REACTIONS OCCURRING IN THE ATMOSPHERE


Fin

As already explained, atmosphere is divided mainly into four regions, namely, troposphere, stratosphere,
mesosphere and thermosphere. Ail the.se regions of the atmosphere contain gases. In the presence of sunlight,
the.se gases undergo chemical reactions. Thus, the reactions occurring among these gases are photochemical
reactions. However, as the amount of these gases present in the atmosphere is very small, it makes the detection
and analysis of the reaction products extremely difficult. These photochemical reactions also decide the
climatic changes. Now, we shall briefly describe the reactions taking place in different segments of the
atmosphere one by one.
(1) Reactions occurring in the troposphere. This is the lowest region of the atmosphere which lies just
above the surface of the earth (0-11 km) where all biological activity takes place. All visible events like rain, wind
*For details, refer to page 14/34.
14/15
ENVIRONMENTAL CHEMISTRY

storms, climatic or weather changes, heat or cold take place in this region, though the life in this region is affected
by the chemical reactions occurring in the other regions also. The most important reactions occurring in this region
are those involving CO2 and H2O vapour which affect the climate as explained below:

When sunlight enters this region, CO2 molecules present in this region absorb a large part of the energy
and get excited
CO2 + hv » GO2* (excited molecules)
These excited CO2* molecules then collide with the other molecules and the excess energy present in
them is converted into heat. As a result, the temperature of the atmosphere may rise. Similar behaviour is also
shown by the water vapour present in this region. Similarly, the solar energy reaching the earth is absorb^ by

low
it and when the earth cools, the energy is re-emitted as infrared radiation. Some of this re-emitted radiation is
absorbed by tropospheric H2O vapour and CO2 and once again radiated back to the surface of the earth. In
this way, additional heat is kept within the lower atmosphere. Such warming of the earth by absorption and re
emission of solar radiation is called “Greenhouse effect”. With increase in human activity, CO2 is increasing.
As a result, the amount of heat radiated back to the earth increases. Such an increase in temperature may have
disastrous effect on the global’s climate as glaciers and polar ice caps may melt and coastal lands may get

ee
flooded. However green-house effect is being opposed by the increase in dust particles which scatter and

rF
reflect back the sunlight and the amount reaching the earth decreases.

Fr
(2) Reactions occurring in the stratosphere. The main reaction occurring in the stratosphere (11-50
km) is the formation and dissociation of ozone. The amount of ozone present in the stratosphere (10 ppm at
25-30 km) is much more than that present in the troposphere (0 04 ppm). The ozone is formed in the stratosphere

for
in a two-step process. In the first step, the ultraviolet radiation coming from the sun have sufficient energy to
split dioxygen (O2) into two oxygen atoms. In the second step, these oxygen atoms react with more of dioxygen
u
to form ozone. ks
hv
O2 ■>0 + 0 ; O + O2 » O3
Yo
The ozone thus formed absorbs ultraviolet radiations and breaks down into dioxygen and an oxygen
oo
atom. Heat is given off in the reaction which warms up the stratosphere. (That is why the temperature of the
stratosphere increases with temperature as already discussed).
eB

hv
O3 + O2 + o +Heat
In this way, ozone cycle is completed in the stratosphere.
r

The ozone layer thus present in the stratosphere is acting as protective layer for the life on the ear*
ou
ad

because it absorbs the harsh ultraviolet radiation which, if reach the earth, cause skin cancer. However, this
ozone layer is being depleted by the human activity, which leads to discharge of nitrogen oxides and chlorofluoro
Y

carbons (known asfreons) into the atmosphere. For example, in addition to NO produced on the ground, the
engines of the supersonic transport planes release a huge amount of NO in their exhaust gases into the
stratosphere. The NO thus present reacts with the ozone thereby decreasing the amount of ozone. The following
nd
Re

reactions take place:


NO + O3 >N02 + 02 ; NO2 + O > NO + O2
Fi

Oxygen atoms used in the above reaction are available in the stratosphere from the decomposition of
ozone and oxygen.
Freons are introduced into the atmosphere from aerosol sprays in which they function as propellants ^d
from refrigerating equipment, in which they act as coolants. In the stratosphere, they first undergo photochemical
decomposition, giving chlorine atoms
hv hv
CF2CI2 CF2CI + Cl
; CFCI3 CFCI2 + Cl
The reactive chlorine atoms then destroy the ozone through the following sequence of reactions which
are repeated over and again because chlorine atoms are regenerated in the second reaction
C1 + 02 ■> CIO + O2 ; CIO + O >Cl + 02
14/16
7*>ux^:Ue^.'A New Course Chemistry fxnrosrwn
(3) Reactions occurring in the mesosphere and thermosphere. In these regions (collectively called
as ionosphere), free ions and electrons are formed as a result of photochemical reactions. Some of the atom
and ion forming reactions with increasing frequency with altitude are given below ;
hv hv
(0 NO ^ NO+ + £' Hi) ^ O* +e~
O2
hv
(///) N2 +e~
(iV) NJ+0 NO+ + N
hv
hv
(V) O ^ + e~
(W) He ^ He+ + e-
hv hv
(v/7) O2 > 0 + 0 (Wi7) N2 ^ N + N
Free ions and electrons do not stay for very long in the lower part of the mesosphere where pressure and
density are quite high. They immediately collide with other ions, atoms or molecules to form neutral species.
Howevei, such active species can survive for a long time in the upper atmosphere because here density and
pressure are low and they do not encounter with other species with which they could combine.

F low
14.9. OZOfu* LAYER-EARTH'S PROTECTIVE UMBRELLA
14.9.1. Fon le ion of ozone layer
More than three billion years ago, before the evolution of life on the earth, it is believed that the atmosphere
consisted mainly of CH^, N2, CO2, NH3, H2 and H2O vapours and there was no oxygen.When plants strated
growing, they released oxygen during photosynthesis. In the stratosphere (11-50 km), the oxygen started
p^ially being converted into ozone. Now at an altitude of 25-30 km, we have a layer in which the concentration
of ozone is about 10 ppm. It is called ozone layer. This region is, therefore, also called ozonosphere. It does

e
for Fr
not allow the harmful ultraviolet radiations coming from the sun to reach the surface of the earth and thus
protects life on the earth. That is why this ozone layer is called earth’s protective umbrella.
The formation of ozone (a light bluish gas) in the stratosphere takes place in two steps. In the first step,
the ultraviolet radiation coming from the sun have sufficient energy to split the dioxygen into two oxygen
atoms. In the second step, the oxygen atoms react with more dioxygen to form ozone.
hv
O.
>0 + 0 ; O + O2 >03
Your
eBo ks

The ozone thus formed absorbs the ultraviolet radiation and is again broken into dioxygen and an
oxygen atom. Heat is given out in this reaction which warms up the stratosphere. For this reason, stratosphere
is a zone of increasing temperature, as already di.scussed.
hv
ad

O3 ^ O2 + O + Heal
our

In this way, “ozone cycle” is completed in the stratosphere.


As the above reactions are initiated by radiation, these are called “photochemical reactions.
14.9.2. Depletion of ozone layer (Creation of “ozone hole”)
Re

Due to human activity, two types of compounds have been found to be most responsible for depleting
the ozone layer and creating a hole into it. These two ozone-depleting agents are ;
Find Y

(/) Nitric oxide (NO) which may be produced at the ground level due to human activity or natural
sources or is produced in large amounts in the exhaust gases by the engine of supersonic transport planes
and introduced directly into the stratosphere. NO reacts with ozone thereby decreasing the amount of ozone
and forms NO-> which reacts with
oxygen atoms available in the stratosphere due to decomposition of ozone
and oxygen (as already explained) producing back NO. Thus, no NO is consumed but O3 gets depleted.
NO + O3 >N02 + 02 ; NO2 + O >N0 + 02
(/7) Chlorofluorocarbons (CFC), Le., compounds containing chlorine, fluorine and carbon, commonly
known as freons. These are introduced into the atmosphere from aerosol sprays in which they function as
propellants and from refrigerating equipments, in which they act as coolants. They are also used as solvents
and as blowing agents for plastic foams. They are chemically unreactive, non-toxic and odourless. However,
they have a very long life time, i.e., they stay in the atmosphere for years and ultimately reach the upper layer

}
14/17
ENVIRONMENTAL CHEMISTRY

where they start decomposing in the presence of UV radiation coming from the sun. For example, life time of
CF2C12, i.e., CFC-12 is 139 years and that of CFCI3. i.e., CFC-11 is 77 yeans.
In the stratosphere, they first undergo photochemical decomposition to give chlonne atoms or free
radicals
hv
CF2CI2 ^ ● CF^Cl + Cl ●
hv
...(0
CFCI3 > ● CFCI2 + Cl ●
The reactive chlorine atoms then destroy the ozone layer through the following sequence of reactions
which are repeated over and again because chlorine atoms are regenerated in the second reaction, i.e..
^ CIO ● + (K)
Cl ■ + O3 ■
(m)
CIO ■ + O > Cl ● + O2
It has been found that one molecule of CFC can destroy more than one thousand O3 molecules in the

w
stratoshere. Due to depletion of ozone (espeeially by CFCs), it was found in the early ninteen eighties that a
large ozone hole has been created in the ozone layer. A similar hole was found later over the thickly populated

F lo
northern hemisphere. The use of CFCs has, therefore, been completely banned.
Ozone depletion over Antarctica. Although the chain reactions, (0, ('O and {Hi) initiated by
chlorofluorocarbons, leading to depletion of ozone take place in all parts of the stratosphere,
yet the
ozone hole has mainly been observed in the strato.sphere over Antarctica. This is because in other
p^s of

e
the stratosphere, chlorine monoxide radicals combine away with the oxides of nitrogen present in the

Fre
stratosphere and the chlorine free radicals combine away with the methane present in the stratosphere as
follows:

for
^ CIONO2 ...(t'v)
CIO ● + NO2
Chlorine nitrate
...(V)
Cl ● + CH 4 ^ CH3+HCI
r
As a result, the chain reaction stops.
You
In Antarctica, the climatic conditions are quite different. In winters, there are special types of clouds
oks

called Polar Stratospheric Clouds (PSC). These clouds are of two types :
eBo

Type I clouds are those which are formed at about -77°C and contain some solidified nitric acid trihydrate
(HNO3.3 H2O).
Type II clouds are those which are formed at about -85”C and contain some ice. These clouds convert
chlorine nitrate formed in reaction {iv) and HCl formed in reaction (v) into HOCl and CI2 through the
ad
our

following reactions :
Hydrolysis
> HOC1 + HN03 ...(VI)
CIONO2 + H2O -
CIONO^ + HCl > C12 + HN03
Hypochlorous acid (HOCl) and molecular chlorine (Cl^) thus formed are easily converted back into
Re

reactive chlorine atoms even under mild conditions and the chain reaction starts again leading to depletion
dY

of ozone.
It is interesting to emphasize that depletion of ozone layer over Antarctica takes place during spring, i.e.,
Fin

in the months of September and October and is replenished after spring, i.e., in the month of November.
This is because during spring, the sun shines over Antarctica and HOCl and CI2 formed in reactions (vi)
and (vi7) undergo photolysis as follows to form reactive chlorine atoms which destroy the ozone layer as
already explained
HOCl + hv 4 -OH + Cl- ; CI2 + /IV ^ 2 Cl-

Further, due to presence of PSC5, a tight whirlpool of wind is formed in the stratosphere which surrounds
Antarctica. It is called Polar Vortex. It is so rigid that it cuts off Antarctica from the surrounding ozone
rich air of the non-polar regions. As a result, the ozone hole remains unfilled. After the spnng, the intensity
of sunlight increases and the Vortex breaks down. The ozone irch air from surroundings immediately
rushes to fill up the ozone hole.

)
14/18
‘P't4^eUe^’^ New Course Chemistry (XI)S»EIHD
14.9.3. Effects/Consequences of depletion of ozone layer
(/) The most serious effect of the depletion of ozone layer or the development of ozone hole is that the
ultraviolet radiation coming from the sun can pass through the stratosphere and reach the surface of the earth.
This type of radiation is known to be a cancer causing agent. It has been found that with increase in the
exposure to ultraviolet radiation, the chance for occurrence of skin cancer increases.
(//) Exposure of eye to ultraviolet radiation damages the cornea and lens of the eye and may cause
cataract or even blindness.

w
(Hi) Exposure of plants to ultraviolet radiation adversely affects the plant proteins and results in the
reduction of chlorophyll and harmful mutation.
(iv) Ozone depletion has a very strong effect on climate. Ozone layer absorbs the ultraviolet radiation
and heats up the stratosphere, setting up a temperature gradient from -56°C to -2°C as the altitude increases

lo
from 11 to 50 km. Depletion of ozone layer will upset the heat balance of the earth,
(v) Ozone depletion, if not controlled, would cause ecological disturbances, which would adversely

e
affect man and animals.

re
rF
Curiosity Question

F
f Q. Why we should not use freons as coolants in refrigerating equipments ?
Ans. Freons (chlorofluorocarbons) have a very long life. They stay in the atmosphere for years and

r
ultimately reach the upper layer where they undergo photochemical reaction with ozone. Thus,

fo
u
ozone layer is destroyed producing an ozone hole through which ultraviolet radiation from the
sun can pass through and affect life on the earth.
ks J
Yo
SECTION—III. SOME EFFECTS OF AIR POLLUTION
oo
14.10. SMOG AND ITS KINDS

The word smog is a combination of “smoke” and “fog”. The name was so given because for the first
eB

time it was found to be formed due to condensation of some kind of fog on the carbon particles present in the
smoke produced due to combustion of domestic and industrial fuels like coal and petroleum. However, now
It is given a name depending upon the composition or the method of its formation or the place familiar for its
ur

formation. Depending upon these factors, it is mainly of two different types as briefly explained below :
(1) London smog or Sulphurous smog or Classical smog. This type of smog was first observed in
ad

London in December 1952 which killed about 4000 people. That is why it is called “London smog”. Its
Yo

formation is initiated by a mixture of SO2, particulates and high humidity in the atmosphere. Many of the
chemicals present in the particulates catalyse the conversion of SO2 to SO3 which then combines with H2O of
the humidity forming a fog of sulphuric acid droplets. These then condense on the surface of the particulates.
d

During respiration, they are drawn into lungs causing bronchitis and respiratory problems leading to death.
Re
in

The reactions may be represented as follows :


2 SO2 + O2 > 2 SO3 ; SO3 + H2O > H2SO4
F

It has been found that this type of smog produces much more dangerous effects than that produced by
the sum total of sulphur oxides and particulates. Further, as it contains SO2 and carbon, it is reducing in
character. This type of smog is formed in the early morning hours of winter months. However, shortly after
sunrise, it increases due to photochemical oxidation of SO2 to SO3 and subsequent combination with moisture
to form sulphuric acid aerosol.
(2) Photochemical smog or Los Angeles smog. This type of smog was first observed in Los Angeles in
1950 and hence is named as “Los Angeles smog”. It is formed when the air contains NO2 and hydrocarbons
and the mixture is exposed to sunlight. As the reaction takes place in the presence of sunlight to form the
smog, it is called photochemical smog. Further, as strong sunlight is needed, this type of smog is formed in
the months of summer during the day time when NOo and hydrocarbons are present in very large amounts due
to heavy vehicular traffic.

J
14/19
ENVIRONMENTAL CHEMISTRY

Formation of photochemical smog. The mechanism of the formation of photochemical smog may be
explained as follows:
In the presence of sunlight, NO2 undergoes photolysis to form NO and atomic oxygen. Atomic oxygen
then combines with the molecular oxygen in the presence of some molecule M (which acts as a source of
transfer of energy) to form ozone, O3. The ozone thus formed reacts with NO to regenerate NO2 and O2.
Thus, NO2 cycle is completed. The reactions taking place may be represented as follows :
NO2 NO + O O + O2 > O3
O3 + NO >N02 + 02
Thus, NO and O3 produced are used up and no extra NO2 is added into the atmosphere. But the trouble
arises if hydrocarbons are also present in the atmosphere. These hydrocarbons combine with the oxygen atom
produced by the photolysis of NO2 to form highly reactive intermediates called free radicals (which are
reactive species containing unpaired electrons) which may be represented by the general formula RCO* (dot
indicates an unpaired electron). These free radicals initiate a variety of reactions, some of which may be as
follows :

F low
RCO* + O2 RCO* ; RCOj + Hydrocarbons ^ RCHO, R2C = O

RCO* + NO ^RC02 + N02 ; RCO* +O2 -> O3 + RCO2

RCO^ + NO2 RCO3NO2


Peroxyacyinitrates
(PAN)

As a result, concentration of ozone, peroxyacyinitrates


(R7CO) builds up in the atmosphere.
As this type of smog contains O3 and NO2, it is oxidising in character.
Harmful effects of photochemical smog,
for Fre
(PAN) and aldehydes (RCHO) and ketones

(i) All these compounds (particularly ozone and PAN) produce irritation in the eyes and also in the
respiratory system,
eBo ks
Your

(n) They also damage many materials such as metals, stones, building materials, etc.
{Hi) Ozone is particularly destructive to rubber (in which cracks are developed).
(iv) It is also harmful to fabrics, crops and ornamental plants,
ad

(v) NO2 present gives a brown colour to the photochemical smog which reduces visibility. Airplane
our

pilots are familiar with this type of fog hanging over the cities.
The word “smog” is a misnormer here because photochemical smog contains neither smoke nor fog. It
is a mixture of a number of irritation-causing compounds like NO2,03, PAN, aldehydes, ketones, hydrocarbons
Re

and CO.
Control of Photochemical Smog. The formation of photochemical smog can be controlled or suppressed
by adopting the following methods :
(/) By controlling the primary precursors of photochemical smog such as NO2 and hydrocarbons,
Find Y

secondary precursors such as ozone and PAN will automatically be reduced. This can be done by fitting
efficient catalytic converters in the automobiles so that the emission of nitrogen oxides and hydrocarbons by
these automobiles into the atmosphere can be prevented.
(ii) By spraying certain compounds into the atmosphere which generate free radicals that readily combine
with the free radicals that initiate the reactions forming toxic compounds of the photochemical smog.
{Hi) Certain plants such as Pinus, Juniparus, Pyrus, Vitis etc. can metabolise oxides of nitrogen. Hence,
their plantation could be helpful.
Difference between Classical smog (London smog) and Photochemical smog (Los Angeles smog).
The main points of difference are given in the table on the next page :
14/20
New Course Chemistry (XI)CEMD

Classical smog (London smog) Photochemical smog (Los Angeles smog)


I. This type of smog was first observed in London 1. This type of smog was observed in Los Angeles in
in 1952. 1950.

2. It is formed due to presence of SO2 and humidity 2. It is formed due to photochemical reaction taking
in the air which combine to form H2SO4 fog which place when air contains NO2 and hydrocarbons.
deposits on the particulates.
3. It involves smoke and fog. 3. It does not involve any smoke or fog. The word
smog is a misnomer here.

w
4. It is formed in the months of winter particularly 4. It is formed in the months of summer during
in the morning hours when the temperature is low. afternoon when there is bright sunlight so that
photochemical reactions can take place.
5. It causes bronchitis irritation, i.e., problem in the

Flo
5. It causes irritation in the eyes.
lungs.
6. it is a reducing in character. 6. It is oxidizing in character.

e
re
Curiosity Question

F
r Q. Is smog formed only in winter or only in summer or in both ? Explain.
Ans. Classical smog is formed in winter when fog of sulphuric acid droplets condenses on the surface
ur
r
of particulates. Photochemical smog is formed in summer during afternoon when there is bright

fo
sunlight so that photochemical reaction can take place between IMOp and hydrocarbons present
in the air.

J
iCiDRAIN
ks
Yo
oo
id rain is the rain water containing sulphuric acid and nitric acid (along with small amount
of hydrochloric acid) which are formedfrom the oxides of sulphur and nitrogen present in the
eB

air as pollutants and has a pH of 4-5.


Formation of Acid Rain.* Due to natural sources such as bacterial action or volcanic eruptions or due
to human activities mainly involving combustion of fuels like coal, wood, petroleum products etc or from
ur

chemical industries, oxides of nitrogen (NO^ and those of sulphur (SO,) enter into atmosphere as pollutants
The oxides of nitrogen are also formed when rain is accompanied by thunderstorm as high temperature due to
ad

thunderstorm results in the combination of nitrogen and oxygen of the air. The oxides of nitrogen undergo
Yo

oxidation reactions followed by reaction with the water vapours present in the atmosphere to form HNO, as
follows: ^

NO + O3 ^ NO2 + O2 ; NO2 + O3 » NO3 + O2


d

NO2 + NO3 » N2O5 ; N2O5 + H2O > 2 HNO3


Re
in

The nitric acid thus formed comes down to the earth along with rain or reacts with the bases like NHi
and lime to form nitrates.
F

The oxidation of SO2 to SO3 is catalysed by aerosol containing metal ions like Cu (II), Fe (II) Mn (II)
and Ni (II) or by the soot particles or by the presence of NO. The SO. thus fonned reacts with the water
vapour of the air to form H2SO4
NO, soot
2 SO2 + O2 » 2SO3 ; SO.j + HoO ^ H2SO4
. ormetalions _
HNO3 and H2SO4 as formed above combine with HCl present in the air (from natural sources or man-
made sources) to produce acidic precipitation which is commonly known as acid rain.
H2SO4 is the main contributor (60-70%), next is HNO3 (30-40%) and least is HCl.
♦Nornial rain water has a pH of about 5-6 due to dissolution of CO-, into it (H2O + CO2 ^ H2CO3
2 -i- CO|“ ). When pH of rain water falls below 5-6, it becomes acidic.
14/21
ENVIRONMENTAL CHEMISTRY

Harmful effects of Acid Rain. (0 Acid rain causes extensive damage to buildings, statues and sculptural
material, especially those made of marble, limestone, slate, mortar, etc. The reaction with marble takes place
as : CaCOg + H2SO4 > CaS04 + CO2 + H2O
For example. Taj Mahal at Agra, which is a very popular historical monument made of marble is being
attacked by acid rain due to high concentration of oxides of sulphur and nitrogen in the air over Agra because
of a large number of industries in the surrounding areas which are emitting these gases. As a result, marble is
being eaten up causing pitting and discolouring and making it lusti'eless.
(ii) It also damages iron and steel structures.
(Hi) It corrodes water pipes. As a result, heavy metals like iron, lead and copper are leached into drinking
water which have toxic effects.
(iV) Acid rain increases the acidity of water in the lakes which is lethal for the fishes. For this reason,
some of the lakes have become fishless. Thus, it has greatly affected the fish population,
(v) Acid rain damages leaves of trees and plants and retards the growth of forests (as it happened in
Swedish forests). It also retards the growth of certain crops such as peas, beans, potato, raddish, can'ot.

F low
spinach, etc.
Methods to reduce the formation of add rain. Acid rain is due to emission of sulphur dioxide and
nitrogen dioxide in the atmosphere. Therefore, to reduce the formation of acid rain, the emission of these
gases has to be controlled. The following steps may be taken :
(0 By using less vehicles driven by fossil fuels.
(ii) By using less sulphur content fossil fuels for power plants and industries, e.g., by using natural gas
which is a better fuel than coal or using coal with less sulphur content,

e
for Fr
(ill) By using catalytic converters in cars so that NO^ present in the exhaust gases is converted into N2
at 573 K. (The main component of the converter is a ceramic honeycomb coated with precious metals like Pt,
Ft and Rh).
(iv) By adding powdered limestone to neutralize the acidity of the soil.
Action Plan to save Mahal from Acid Rain. To save Taj Mahal from getting disfigured. Government
of India announced an action plan in 1995. According to this plan, it was decided to clear the air in the laj
Your
Trapezium, i.e., area which included the cities of Agra, Firozabad, Mathura and Bharatpur. The action
eBo ks

comprised of the following steps:


(i) All industries (about 2000) in the trapezium would be allowed to use only natural gas or LPG instead
of coal or oil. A pipeline would be laid for this purpose. Mathura refinery has already taken suitable steps
to check the emission of toxic gases,
ad

(ii) People in these areas would also be advised to use LPG instead of coal, wood, kerosene oil, etc.
our

(ill) Vehicles in the nearby area would be encouraged to use low sulphur content tuei.

Curiosity Question
Re

r Q. Rain damages the monuments like Taj Mahal In Agra when Industries are present nearby.
Why?
Find Y

Ans. Industries produce a lot of oxides of nitrogen and sulphur which through reactions produce
compounds which dissolve in rain water to form H2SO4 and HNO3. The rain thus becomes acid
rain. The marble (CaCOs) of the monuments is attacked by these acids. J
14.12. GREENHOUSE EFFECT AND GLOBAL WARMING
CO2 present in normal concentration in the air is not an air pollutant. Rather it helps to maintain the
temperature on the earth required for the existence of living organisms. However, due to human activity such
as burning of fossil fuels like coal, natural gas. petroleum, etc., the amount of CO2 in the air is increasing.
This results in warming of the earth’s surface and thus brings about drastic changes in the climate as explained
below :
14/22
7^^n4idce^'A New Course Chemistry fXT^rosrwm
Out of sun’s energy entering the atmoshere (viz, about 20%) about 34% is scattered and erflected back
by the particulates and the remaining 66% is absorbed by the earth (Fig. 14.3, stage I). The reason for this
energy reaching the earth is that visible and ultraviolet radiation (which have short wavelengths) can pass
through CO2 and water vapour present in the atmosphere. Ultraviolet radiations are absorbed by ozone layer
but visible light reaches the earth and heals it up. However, when the earth cools, the energy is re-emitted
from the earth s surface in the form of infrared radiations (which have longer wavelength and have a heating
effect). These radiations can be absorbed by CO2 and H2O vapours but cannot pass through them (Fig. 14.3,
stage 2). Tlie heat thus absorbed by CO2 and H2O vapour cannot go to the upper atmosphere and is radiated
back to the surface of the earth (Fig. 14.3, stage 3). In this way, additional heat is kept within the lower
atmosphere and warming of the earth occurs.
FIGURE 14.3

w
Solar energy
passing through Atmosphere
contaning CO2 and Infrared radiation {heat)

F lo
H2O vapour re-emitted by earth and
absorbed by CO2 and H2O Heat radiated back to earth
Absorption of vapour in lower atmosphere by CO2 and H2O vapours
energy in the
visible region

e
Fre
EARTH EARTH EARTH

STAGE 1 STAGE 2 \ STAGE 3

for
Stages to show production of greenhouse effect

This warming of the earth or global warming due to re-emission of sun *s energy absorbed by
the earth followed by its absorption by CO2 molecules and H2O vapour present near the earth’s
r
surface and then Us radiation back to the earth is called ‘^greenhouse effect. M
You
oks

The phenomenon has been named as greenhouse effect because it is similar to a greenhouse for plants
(usually needed during winter in the cold countries). A greehouse is a place where plants are grown on the soil
eBo

but there are walls and roof made of glass. The sun’s energy can enter through the glass but cannot go out of
it. The energy is thus absorbed by the soil and the structure of the greenhouse. The energy thus absorbed is re
emitted back into the greenhouse. The same situation is observed inside a car which becomes very hot inside
when it is parked in the sun for some time with its glass windows closed. In the atmosphere, CO2 is playing
our
ad

the role of glass of a greenhouse. Hence, it is called greenhouse effect.


Consequences of Greenhouse effect (Effect on the global climate). (/) If the rate at which solar
radiation are arriving the earth remains constant but the amount of CO2 in the air increases (due to human
activities), the heat radiated back to the earth (by CO2 and H2O molecules) will increase. Consequently, the
temperature of the earth s surface will increase. As a result, evaporation of surface water will increase which
dY
Re

will further help in the irse of temperature. This increase in temperature will have a drastic effect on the
climate and may prove even disastrous. For example, it has been estimated that a 20% increase in the amount
of CO2 would increase the average temperature of the earth by ffC. This may result into the melting of
Fin

glaciers and polar ice caps and the level of sea water may rise thereby flooding the coastal lands and change
the climate radically. The fertile lands may become barren lands and aquatic life will be seriously affected. It
is estimated that the sea level may ri.se by about 0-5 m to 1-5 m in the next .50 to 100 years.
(ii) Due to higher concentration of CO2 in the atmosphere, the plants undergoing photosynthesis will
take up the gases at a faster spead. As a result, the plants in warmer climates and adequate rainfall would grow
faster.

(Hi) Higher global temperature is likely to increase the incidence of infectious diseases like malaria
dengue, yellow fever and sleeping sickness.
It is important to mention that earth is being saved from excessive heating due to certain opposing
factors taking place simultaneously. For example, the increase in the amount of dust scatters away the sunlight
and thus limits its amount reaching the earth.

I
14/23
ENVIRONMENTAL CHEMISTRY

CO2 sinks. Two important sinks of CO2 which help to control the amount of CO2 in the atmosphere are
(/) Oceans which dissolve CO2 into them, (ii) Plants which take up CO2 for photosynthesis.
How to reduce the rate of global warming ? As greenhouse gases are added into the atmosphere due
,j burning of fossil fuels and cutting down of forests and trees, we can follow some simple methods so that
to

the greenhouse gases are added into the atmosphere as minimum as possible. A few of these can be as follows :
(i) By using alternate sources of energy such as solar power, hydropower, wind energy etc. which do not
emit greenhouses gases.
(ii) By preferring walking or using bicycles or going for car pools or taking public transport to reduce
vehicular emission,

(m) By planting more trees as they use up CO2 for photosynthesis ,

w
(iv) By avoiding burning of dry leaves, etc.
smoke in public places and work places,
(v) By not smoking and pursuading others also not to
(v/) By imparting knowledge to those who do not know about the greenhouse effect.

F lo
Other greenhouse gases. Though CO2 is the key gas in the greenhouse effect but there are some other
greenhouse gases also, e.g., methane, chlorofluorocarbons, ozone, nitrous oxide and water vapour. Their relative
contributions are as follows : CO2 (50%), CH4 (19%), CFG (17%), O3 (8%), N2O (4%), H2O (2%).

ee
Curiosity Question

Fr
r Q. It Is expected that time is not far off when coastal land will be flooded. Why ?
Ans. Due to greenhouse effect, temperature of the earth’s surface is expected to rise. This will result

for
into melting of glaciers and polar ice caps. As a result, level of sea water will rise causing floods
ur
in the coastal land.

SECTION-^IV. WATER POLLUTION 1


s
ook
Yo
14.13. INTRODUCTION
Water is one of the basic needs of household, agriculture and industry. For most of the purposes, e.g.,
eB

drinking, washing, etc. we need pure and clean water. However, the natural water (coming from rain, snow,
hail, etc. which is the purest form) gets contaminated or polluted in the following two ways :
(/) By natural processes, e.g., washing away of the decomposed vegetable and animal wastes into the
our

main stream of water.


ad

(//) By human activity (anthropogenic processe.s), e.g., discharge of industrial effluents, domestic wastes,
use of pesticides and fertilizers, etc.
Water pollution is defined as the contamination of water by foreign substances which make it
Y

harmful for health of animals or plants or aquatic life and make it unfit for domestic, industrial
Re

and agricultural use.


nd

Polluted water has any one or more of the following signs:


Fi

(0 It has a bad taste to drink. (ii) It has an offensive odour. (Hi) It has unpleasant colour,
(iv) It has unchecked growth of weeds. (v) It may have oil or grease floating on the surface.

14.14. TYPES OF WATER POLLUTION


Based on the sources and storages of water, water pollution may be classified into five main categories
as follows :

(1) Ground water pollution. Water below the surface of the earth is called ground water. Most of the
water (>90%) is present as ground water. The remaining is present in lakes, rivers, streams, etc. Only about
2% is present as soil moisture above the water table which is needed for the growth of plants. Ground water
collects below the surface of the earth after passing through the pores of the earthy materials which act as a
14/24
New Course Chemistry (XI)CEaB]
filter tor it and is pure. It is for this reason that well water or spring water is used for domestic purposes in
rural areas. However, due to disposal of domestic wastes and industrial effluents and use of fenilizers and
pesticides in agriculture, a number of harmful soluble substances dissolve into the rain water and pass through
the soil and enter into ground water and result in pollution, especially where the water table is high.
Black Foot Disease (BFD). When ground water is contaminated with high concentration of arsenic, it
causes a disease called Black Fool Disease. It is a severe form of peripheral vascular disease (PVC) in which
the blood vessels in the lower limbs are
severely damaged resulting eventually in progressive gangrene. It has

ow
been observed in Taiwan (provice of China).
{2) Surface water pollution. Water present on the surface of earth in any form is called surface water.
As it is an in direct contact with the atmospheric ga.ses, a number of gases like CO,, CO, SO->, H2S, NO , etc.
present in the air as pollutants dissolve into it, thereby polluting it. In addition to these atmospheric gas/s, the
surlace water also gets polluted by industrial and municipal wastes, agricultural wastes including decomposed
plant and animal matter and by radioactive materials.

e
(3) Lake water pollution. Lakes cover a large part of the water area. The water in them gets contaminated

re
in a number of ways as follows :

rFl
(0 Organic wastes from hills and toxic effluents from urban areas flow into them.

F
(//) Industrial effluents are thrown into them.
(Hi) Sewage treatments plants discharge toxic organic matter into them,
(fv) Dumping of huge amounts of sediments, i.e., siltation of lakes.

r
(0 Inorganic nutrients from agriculmral land due to surface run off.
ou
fo
(4) River water pollution. There are 13 major rivers in India flowing through different states. These
include Ganga, Yamuna, Naimada, Brahamputra, Krishna, Cauvery, Damodar, Godavri, Hooghly, Tapti, Gomti,

ks
Sona and Chambal besides many others. There was a time when the water in these rivers was very pure and
clean. Ganga jal was considered to be so pure that it could be preserved for years and a few drops of it used
to be pul in the mouth of relative who was on death bed. But now the water is so polluted that if taken may
oo
speed up the death of the person. Similarly, the water in the Yamuna river is so polluted that not to talk of
drinking, it is unfit even for swimming or taking bath. The main reasons for the pollution of water in these
Y
eB

nvers are
: (i) Industrial discharges including those from paper, textile, rayon, fertilizers, pesticides, detergents,
drug industries and refineries.
(«) Domestic sewage discharge.
The use of polluted water from these rivers can cause water-borne diseases like typhoid, cholera, dysentery,
r

jaundice, etc.
ou
Y
ad

Government of India has, therefore, enacted laws banning the discharge of industrial or domestic waste
into these rivers. Further, it has set up the following plans to clean the water of these rivers :
(/) Ganga Action Plan (i7) Yamuna Action Plan {in) Plan to clean Hooghly water.
d

(5) Sea water pollution. Oceans cover about 70% of the earth’s surface. They are not only a major
source of water but within them lie huge stocks of food, minerals, coal, oil and gas. It is unfit for human
in
Re

consumption and for industrial use because it is very salty. Further, sea water is being mainly polluted by oil
which poses a serious threat to the marines and fisheries.
F

The pollution of sea water due to discharge of wastes from different sources into it thereby
making it harmful for human health and aquatic life like fish, etc. is called marine pollution.
Sources of oil pollution in sea water. Some of the major sources are as follows :
(0 Wreckage of oil tankers in open sea or accidents of ships carrying oil in the sea.
{//) Discharge of oily wastes from tank washings.
ini) Oil leakage from pipelines,
(iv) Shipping operations at the coastal belt,
(v) Oily wastes from oil fields or refineries located near the coast.
(W) Deliberate marine pollution by crude oil, as it happened in the recent Gulf war (Iran-Iraq war in
February 1990).
14/25
ENVIRONMENTAL CHEMISTRY

The spreading of oil into sea is called oil spill and the thick layer of oil on the surface ol sea water is
called oil slick.
In India, an oil spill occurred in Bombay on May 17. 1993 due to rupture of pipeline which damaged the
ecosystem and marine life.
Other sources of pollution of sea water. A few of these are ;
(/) Discharge of urban sewage, silt, plastics.
(ii) Flow of pesticides and other dumped toxic chemicals from land.
(Hi) Discharge of radioactive wastes into sea.
(/v) From beach tourism discharging sanitary and kitchen wastes into sea.
Effects of oil pollution in sea water. {/) Oil spill causes heavy damage to fishery. Oil coating makes them
unable to respire and clogs their gill slits. Aromatic compounds present in them are a poison for the fishes.
(//) Emulsified oil goes deep down into the sea damaging aquatic animals and plants.

w
(in) Oil slick results in reduction of dissolved oxygen (D.O).
(iv) The most affected by oil pollution are the sea-birds. Natural insulating oils and waxes which shield

F lo
the birds from water are broken down by the spilled oil. As result, due to loss of insulation, they start
shivering and are frozen to death, especially in winter.
14.15. SOURCES/CAUSES OF WATER POLLUTION
The main sources responsible for the pollution of water are briefly described below :

e
(1) Sewage and domestic wastes, e.g., human excreta, sewage sludge, soaps and detergents, untreated

Fre
municipal sewages, etc. They produce pathogens which are diseases causing bacteria and result in gastro
intestinal diseases.

for
(2) Indu-strial effluents. These contain toxic chemicals and hazardous compounds including aldehydes,
ketones, phenols, cyanides, oils, greases, dyes, acids, alkalies, toxic metals such as Cd, Pb, Hg, etc. (cadmium
and mercury can damage the kidneys whereas lead poisoning can damage kidneys, liver, brain and central
r
nervous system). Their concentration builds up in the body as they are not excreted.
You
The water flowing from the acid mine contains a lot of sulphuric acid produced by the oxidation of iron
ks

pyrites (FeS2). The acid-polluted water with pH < 3 is fatal for the aquatic life.
o
eBo

(3) Agricultural discharges. These include fertilizers and pesticides, insecticides, etc.
(4) Siltation. It is the process of mixing of soil or rock particles into water. It is a serious problem in hill
streams. The soil particles produce turbidity in water thereby hindering the free movement of aquatic organisms
and hence their growth and productivity.
ad

(5) Thermal pollutants. These are those sources which result in the increase of temperature of the
our

flowing water and hence adversely affect the aquatic life in them. These include thermal power plants, nuclear
plants as well as industries which have cooling requirement. These units discharge their unutilized heat into
the nearby flowing water. Municipal sewage disposal also causes thermal pollution because water containing
sewage has a higher temperature.
Re
dY

(6) Radioactive discharges. They enter into water stream from different sources like nuclear reactors
(nuclear power plants), nuclear tests, dumped nuclear wastes, etc. The radiations emitted by them are highly
Fin

hazardous.

(7) Polychlorinated biphenyls (PCBs). These compounds are used as fluids in transformers and
capacitors. However, they are resistant to oxidation and their release into atmosphere causes skin disorders.
They have also been found to be carcinogenic. They have been added into the list of the water pollutants only
recently.
14.16. CLASSIFICATION OF WATER POLLUTANTS
The various water pollutants may be classified into following categories :
(1) Inorganic pollutants. These include the following : (/) Acids and alkalies from industries
manufacturing acids like H2SO4, HNO3, HCl, H3PO4 and bases like NaOH, KOH, Ca(OH)2, NH3, etc. and
also as effluents from paper, textile, tanning industries. Coal mines also discharge large quantities of H2SO4
and Fe(OH)3 produced due to reaction between iron pyrites (FeS2), air and water.
14/26
New Course Chemistry (XI)man

Acid mine drainage. It refers to water with a high concentration of sulphuric acid (H.SO.) that drains
from mines, mostly coal mines and metal mines.
(«) Soluble sails like carbonates, bicarbonates, fluorides, chlorides, bromides, iodides, acetates nitrates
sulphates, phosphates, etc.
Uii) Insoluble salts, e.g., calcium carbonate, calcium phosphate, etc.
(/v) Polyphosphates entering into water from detergents.

w
(1’) Inorganic pesticides. These contain metals like Cu, Cd, Fe, Mn, Zn, Co etc. and some of them
contain sulphur and arsenic.
(vO Toxic metals, e.g., Pb, Hg, As, Sb, Cd, Fe, Ni, Mn, Zn and Cu which are harmful to humans, animals
and plants. They are introduced into water by industrial discharges, mining processes, burning of fossil fuels
and land run off, etc. Industries especially responsible include paper, textile, rubber, electroplating, galvanizing,

o
battery manufacturing, etc.

e
Body burden. The amounts of heavy metals in our bodies is called body burden. For an average human

re
body, weighing about 70 kg, their values are as under :

Frl
Antimony = 8 mg, Mercury = 13 mg. Arsenic = 18 mg. Cadmium = 30 mg and Lead = 150 mg

F
(2) Organic pollutants. The different types of organic pollutants and their sources are as follows :
(0 Carbohydrates and proteins, e.g., glucose, sucrose, starch, dextrin etc. from breweries, food processing
industries, dairies, sugar mills, slaughter houses, etc.
(ii) Oils from petroleum refineries and drilling.
ou
or
(/70 Aldehydes and phenols from chemical industries,
(iv) Polychlorinated Biphenyls (PCBs) used as fluids in transformers and capacitors and also as lubricants

kfs
and plasticizers,
(v) Polynuclear aromatic hydrocarbons from petroleum refineries which have carcinogenic effects on
oo
humans and animals.

(3) Sediments. These are the soil, rock or mineral particles which flow along with the streams before
Y
entering into water bodies. They also come from soil erosion, agriculture and construction activity. They
B

contain organic as well as inorganic matter including trace amounts of toxic metals like As, Hg, Pb, etc.
(4) Synthetic detergents. Synthetic detergents used as cleaning agent consist of a surfactant, i.e.,
re

surface active agent which are alkyl benzene sulphonates (ABS) and a builder which is usually sodium
phosphate or polyphosphate which is added for removing the hardness producing ions. i.e.. Ca^‘*‘ and
oYu

Mg--^ ions. Besides, some additives are also added which consist of enzymes, perfumes and bleaching
agents.
ad

The surfactants in polluted water create a number of problems as follows:


(0 They do not undergo biodegradation.
d

(//) They inhibit oxidation of organic compounds like phenol because they form an envelope around
them. Hence, they create problem in the treatment of waste water.
in
Re

(Hi) They stabilize the colloidal impurities which do not aggregate to settle down.
(iv) They produce stable foam in rivers.
F

The builders form soluble complexes with Ca^'*' and Mg^"^ ions which act as plant nutrients.
(5) Oxygen-Demanding Wastes. ‘Dissolved oxygen’ is most important for all types of aquatic life.
The growth of fish is hindered if the concentration of dissolved oxygen is below 6 ppm. Oxygen enters into
water through two sources :
(0 Due to direct contact of the surface of water with ai.r Turbulent water takes up more oxygen than still
water as the bubbles get submerged.
(ii) By photosynthe.m of the aquatic green plants. As it takes place in sunlight during the day time, the water
gets supersaturated with oxygen. However, after sunset photosynthesis stops but the plants continue to respire and
consume oxygen, therefore, the amount of oxygen decreases and remains at a reasonable level.
However, the discharge of human sewage and organic wastes from pulp and paper industry, tanneries
and slaughter houses into water creates a problem. Similarly, leaves, grass, trash, etc. are also present in water
ENVIRONMENTAL CHEMISTRY 14/27

as pollutants due to run off. They result in excessive phytoplankton growth. The microorganisms which
decompose this organic waste need oxygen.
CH2O iaq) + O2 iaq) > CO2 {aq) + H->0 (/)
The amount of oxygen consumed by the microorganisms in decomposing the organic waste is proportional
to the amount of waste present*. This is the basis of estimating organic waste present in sewage water before
subjecting it to treatment.
The total amount of oxygen consumed by microorganisms (bacteria) in decomposing the waste
(organic matter) present in a certain volume of a sample of water is called Biochemical Oxygen
Demand (BOD) of the water.
The determination of BOD of a sample of water requires 20-30 days for the complete decomposition of
the waste which is too long a time to wait. Therefore, usually we determine BOD5, i.e., the amount of oxygen

w
consumed in 5 days. The procedure is as follows :
The sample of water is saturated with oxygen and incubated for 5 days at 20°C. The remaining oxygen

F lo
is measured and subtracted from the amount of oxygen originally present to get the BOD5 and is reported in
ppm which is equivalent to milligrams per litre.
Water considered pure has BOD^ of less than 5 ppm whereas highly polluted water has BOD5 value of
more than 17 ppm. The untreated municipal sewage has BOD5 of 100-400 ppm.
The measurement of BOD takes a number of days, i.e., generally 5 days as explained above. Hence,

e
Fre
another quantity generally measured is called Chemical Oxygen Demand (COD). The method consists in
treating the given sample of water with a known amount of an oxidizing agent, generally K2Cr207 in presence
of dilute H2SO4. All the pollutants get oxidized including those which were resistant to microbial oxidation.

for
The amount of K2Cr207 left is found by back titration against a suitable reducing agent such as Mohr salt.
From the amount of K2Cr207 used, the amount of oxygen consumed can be calculated from the following
balanced chemical equation ;
r
K2Cr207 (aq) + 4 H2SO4 (aq) ^ K2SO4 (aq) + Cr2(S04)3 (aq) + 4 H2O (/) + 3 (O) (aq)
You
The amount of oxygen thus consumed by the pollutants is expressed in ppm and is called COD of the
ks
given sample of water.
o
eBo

The decomposition of organic waste by microorganisms produces bad smell and unattractive appearance
making it unfit for recreational use like swimming, boating etc. Further, decrease in the dissolved oxygen
below 6 ppm, as already mentioned may result into the death of the aquatic species includingfishes.
(6) Infectious agents/Disease causing agents. The municipal sewage and the municipal waste water
ad

coming from slums, hotels, restaurants, residential areas and also containing faeces and urine of infected
our

patients, animal extereta etc. contain disease causing microorganisms or bacteria (pathogenic microbes). The
potable water gels contaminated with these microorganisms and causes diseases in humans and animals.
(7) Plant nutrients. The plant nutrients (containing N and P) flow into lakes where they support the
growth of aquatic plants. These plants on decay produce unpleasant odour. Further, the microorganisms in
dY
Re

decomposing these plants consume oxygen. As a result, the amount of dissolvedoxygen in the water decreases
which proves fatal for the aquatic life, i.e., fish, etc.
Fin

Eutrophication. The presence of excessive nutrients in a lake due to inflow (land run-off) of nutrients
from fertilizers results in the increase of phosphate ions in water. As a result, the formation of algae is accelerated
which covers the water surface and hence the concentration of dissolved oxygen decreases. Further, when

*In addition to the consumption of dissolved oxygen by microorganisms, it is also used up for
(1) bio-oxidation of the nitrogenous matter

NHJ (aq) + 2 O2 (aq) >2 (aq) + NOj (aq) + H2O (/)


(ii) the chemical or bio-chcmical oxidation of chemical reducing agents
4 Fe-+ (aq) + O2 (aq) + 10 H2O (/) > 4 Fe (OH)3 (s) + 8 H+ (aq)
2SO^~ (aq) + 0.-,(aq) > (aq)
14/28
'Pn^tdee^'4. New Course Chemistry (XI)EEIHD
short lived algae die or the excessive phytoplankton (organic pollutants such as leaves, grass, trash etc.)
present in water start decomposing (as they are biodegradable), the decomposing bacteria consume dissolved
oxygen and hence deplete dissolved oxygen in water. Consequently, aquatic life is adversely affected. The
fish start perishing. This process is called eutrophication.
(8) Pesticides. The.se are organic compounds which are used to protect plants from pests. These are also
used to stop the growth of weeds (/.c.. unwanted plants which grow along with the main plant). These are mild
poisons. Since weeds are not pests, the chemicals used to stop their growth are better called herbicides. The
general term used for pesticides and herbicides is biocide. However, the term ‘pesticides’ is still used quite
frequently and includes herbicides. Thus, pesticides include insecticides (to kill insects), fungicides (for
fungi or mould), rodenticides (to kill rats and mice), herbicides (to kill weeds), etc.
These How into lakes along with the rain water and cause problems to aquatic as well as human life.

w
(9) Radioactive pollutants. Already discussed.
(10) Thermal pollutants. Already discussed.
(11) Oil. Already discussed.

F lo
14.17. INTERNATIONAL STANDARDS FOR DRINKING WATER
Water used for drinking must be pure as otherwise it can cause serious diseases. International standards

ee
have, therefore, been laid down for the water to be used for drinking. The chemicals that are allowed to be
present and the tolerable limits upto which they are permitted and some other conditions which must be

Fr
satisfied according to the international standards are given below :
Chemical/condition

for
Source Tolerable limit Use/Harms of higher cone.
-3
(0 Fluoride Added externally 1 ppm or 1 mg dm Protects teeth against decay.
ur
Concentration above 2 ppm causes
brown mottling of teeth. High cone.
s
ook
(> 10 ppm) are harmful to bones and
Yo
teeth*

50 ppb or jig dm ^
eB

(ii) Lead Lead pipes used for Lead poisoning (damages kidneys,
transport of water liver, brain and reproductive system)
(Hi) Other metals Zn = 5 ppm
Cu = 3 ppm
our
ad

Fe = 0-2 ppm
A! = 0-2 ppm
Mn = 0 05 ppm
Y

Cd = 0 005 ppm
Re

(iv) Sulphates < 500 ppm Higher cone, has laxative effect
nd

(v) Nitrates 50 ppm Excess causes meihemoglobinemia


(1 ppm = 1 mg dm“^)
Fi

(blue baby syndrome) which results


in a blue colour skin and a congenital
(from birth) heart defect in infants
which laymen often call
‘hole in the heart.’**

(vO pH 5-5 — 9-5

*The F" ions make the enamel on the teeth much htirder
by converting hydroxypatite, [3 Ca3(P04)2.Ca(0H)2],
the enamel on the surface of the teeth, into much harder fluorapatite, [3 Ca3(P04)2.CaF->].
**For details, refer to point 3, page 14/35.
14/29
ENVIRONMENTAL CHEMISTRY

Curiosity Questions
f Q. 1. At one time, hand pump water was considered to be pure and used freely for drinking in
villages but not now. Why ?
Ans. Hand pump water is ground water. It collects below the surface of the earth after passing through
the pores of the earthy materials which act as a filter for it and is pure. Now due to disposal of
domestric wastes and industrial effluents and use of fertilizers and pesticides, a number of
harmful soluble substances dissolve into rain water and pass through the soil and enter into
ground water resulting in pollution.
Q. 2. How fluoride in the tooth paste protects teeth against decay ?
Ans. The fluoride ions make the enamel on the teeth much harder by converting hydroxy patite
(enamel on the surface of the teeth) into much harder fluorapatite.

w
SECTION—V. SOIL/LAND POLLUTION

F lo
14.18. INTRODUCTION

Soil is the uppermost part of the earth's crust and is believed to have been formed as a result of
decomposition and disintegration of surface rocks due to weathering over a very long period of time.

e
It might have been formed at far off distance but is carried by strong winds and rivers to different

Fre
places.
The prosperity of a nation depends upon the quality of its soil as it is the soil which provides nutrients,

for
water and minerals for the growth of the plants. It is, in fact, the heart of biosphere.
However, large scale cutting of trees for human needs due to increase in population (i.e., indiscriminate
deforestation), digging of minerals, increasing brick-kilns, poisonous effluents of the industries and dumping
r
of toxic and harmful wastes, increase in the use of automobiles, etc. are damaging the quality of the soil and
You
oks
resulting into soil pollution. In India, it is sad to point out that backwitrdness of our people is an additional
factor for soil pollution, e.g., using open air lavatories by slum dwellers thereby inviting pig, crows, other
eBo

vultures and flies, throwing of domestic wastes including peelings of vegetables and fruits, plastics, polythene
bags, etc.

14.19. COMPOSITION OF THE SOIL


our
ad

The main components of the soil are as follows :


(1) Mineral matter. It comprises of mineral particles of different sizes which originate from the rock
and are formed by its disintegration. They are present as gravel, coarse sand, fine sand, slit and clay.
(2) Organic matter. This is present in the soil as a result of fallen trees and mixing of their parts such as
dY

roots, leaves etc. and also from the remains of dead animals and their excreta. These materials are decomposed
Re

by microorganisms present in the soil and convert them into humus (process is called humification). Humus
has all those excellent properties which are an urgent need of the plants.
Fin

(3) Biological system. It consists of algae, bacteria, protozoa etc. which help in the decomposition of
organic matter present in the soil.
(4) Soil water. The water present in the soil acts as a solvent for various organic and inorganic materials
needed for the plants and also acts as a transporting agent by carrying nutrients from the soil to different parts
of the plant.
(5) Soil air. It is present in the pores of the soil, i.e., the space between the soil particles. It contains
more of CO2 and moisture than O2. It is needed for soil microorganisms and underground parts of the plants.
The soil containing almost equal amounts of sand, slit and clay along with humus and 34% air and 66%
water is considered to the best soil for most of the crops. It is called loam soil.
14/30 it f ' Course Chemistry fXTtrosrwn

14.20. SOURCES OF SOIL POLLUTION


Some of the main sources of soil pollution are briefly described below :
(1) Industrial wastes. It has been estimated that about 50% of the raw material used by most of the
industries becomes waste product which is either thrown into water or dumped into the soil nearby. This
industrial waste contains huge amounts of toxic and disastrous chemicals many of which are non-biodegradable.
Some of the industries responsible for soil pollution are paper and pulp mills, sugar mills, textiles, chemical
industries, distilleries, metal processing industries, mining, cement and glass industries, petroleum industry, etc.
(2) Urban wastes. Urban waste consists of soil refuse containing garbage and rubbish material like

ow
paper pieces, glass, plastics, used cans, leaves, polythene bags, peelings of vegetables and fruits, food wastes,
etc. These wastes emit out poisonous gases, toxic hydrocarbons and pathogenic microbes (bacteria) which
cause diseases.

(3) Agricultural pollutants. No doubt, the use of fertilizers, pesticides, soil conditioners, fumigants
etc. have increased the yield from the crops but they have polluted the soil. Their entry into food chain has
adversely affected the health of the human beings and led to a number of diseases.

e
Some of the agricultural pollutants are briefly described below :

Fl
re
(i) Fertilizers. Fertilizers act as nutrients for plant but if nitrates and phosphates are present in excess,
they have hazardous effects.

F
(ii) Pesticides. These are the chemicals that are used to kill or stop the growth of unwarned organisms.
However, their entry into food or drinking water adversely affects the health of human beings. These are
ur
further classified into the following different categories,

r
(fl) Insecticides. These are the chemicals that are used to kill the insects which destroy the crop. They

fo
also help to control malaria and yellow fever. The most common insecticides in use since 1950 are the
chlorinated hydrocarbons like DDT (dichlorodiphneyl trichloroethane), BHC (benzene hexachloride), etc.
ks
As they are not much soluble in water, they stay in the soil for long time and show their activity. But their
Yo
disadvantage of this property is that they are absorbed by the soil and contaminate root crops like raddish,
oo
carrot, etc.
FIGURE 14.4
However, their repeated use produces a resistance in the
B

pests and hence they become ineffective. Hence, some other


organic toxins such as Aldrin and Dieldrin were introduced. But Hum^^eing
e

these are non-biodegradable. These highly persistent toxins are


ur

transferred from lower trophic level to higher trophic level


through food chain (Fig. 14.4). At each trophic level, the pollutant
ad

gets 10 times concentrated. Over the lime, the concentration of ■ Uarge'fish' ●


Yo

F:. ●1
toxins in higher animals reaches a level which causes serious
metabolic and physiological disorders.
Aquatic plant
d

Consequently, a new series of products, namely,


Re

organophosphates and carbamates have been introduced. These


in

Molecular level
are more biodegradable. But these chemicals are nei-ve toxins
and hence more harmful to humans and have caused even deaths.
F

At each trophic level, the pollutant


Thus, insecticide industry is engaged in developing new gets 10 times concentrated
insecticides.

{b) Herbicides. These are the compounds used to control weeds. Earlier inorganic compounds, namely,
sodium chlorate (NaClO^) and sodium arsenite (Na3As03) were commonly used as herbicides but arsenic
compounds, being toxic to mammals, are no longer preferred. Instead, organic compounds such as triazines,
are now considered as better herbicides especially for the corn-fields,
(c) Fungicides. These are the chemicals used to stop the growth of fungi and check plant diseases. As
fungi are the plants which do not contain chlorophyll, therefore, they cannot use solar energy for the synthesis
of their food (carbohydrates). Hence, they depend upon other plants and living organisms. As a result, the
growth of the main plant is retarded. Organo-mercury compounds are the most common fungicides. However,
14/31
ENVIRONMENTAL CHEMISTRY

their dissociation in the soil produces mercury which is highly toxic and proves fatal if it enters into grain, as
it happened in Iraq in 1972, resulting into many deaths due to consumption of wheat from fields treated with
methyl mercury as fungicide.
(Hi) Soil conditioners. These are used to protect the soil fertility but contain several toxic metals like
Pb, As, Hg, Cd, Co, etc. which may enter into the food chain.
(iv) Farm wastes. These are one of the biggest sources of soil pollution. In small towns and rural areas,

ow
there has been increase in dairies, poultries and piggery farms. Their washings collect as a wel slurry on the
land. This may seep through the soil and pollute the ground water. If their waste (faecal matter) is dumped, it
may become a breeding place for insects. Moreover, their waste may contain pathogenic bacteria and viruses
which may enter the plant and then pass on to humans.
(4) Radioactive pollutants. Dumping of the nuclear wastes from the nuclear power plants into the soil
has been one of the greatest source of radioactive pollution of the soil. Nuclear tests produce nuclear dust in

e
the atmosphere which ultimately falls into the soil and pollutes it. The use of atomic and hydrogen bombs in

re
the wars produces radioactive nuclides as the by-products. All the radioactive wastes thus produced emit

Frl
radiations which are disastrous for the life on the earth.

F
Biomagnirication/Bionccuinulation. This lenn refere to the accumulation or increase in the concentration
of a substance in the living tissues as it moves through a food chain.
SECTION—VI. STRATEGY FOR CONTROL OF ENVIRONMENTAL POLLUTION
ou
or
14.21. MANAGEMENT OF WASTE
The two main sources for throwing waste into the environment and thus causing pollution are :

kfs
(0 Household waste which includes mainly sewage and municipal garbage.
(//) Industrial waste which may include toxic materials.
oo
14.21.1, Waste management of Household Waste
Y
All the solid household waste should be put in the household garbage box/bin. This should be then put
B

into the community bins so that the municipal workers can take it in their vehicles to the disposable site. Here,
the garbage is separated into biodegradable and non-biodegradabl e materials. The biodegradable waste IS
deposited in the land fills. With the passage of time, it is converted into manure/compost. Remember that if
re

the waste is not collected in the garbage bins, it may find its way into sewers and some may be eaten up by the
cattle. The non-biodegradable waste like polythene bags, metal scrap, etc. choke the sewers. The polythene
oYu

bags, if swallowed by cattle, can result into their death.


ad

The best way to manage domestic waste is to keep two garbage bins, one for the biodegradable (non-
recyclable) and the other for non-biodegradable (recyclable wastes) which can be sold to the vendor/dealer.
d

14.21.2. Management of Industrial Waste


Industrial wastes are also sorted out as biodegradable and non-biodegradable. Biodegradable wastes
in
Re

are generated by cotton mills, paper mills, textile mills and food processing industries. Some of the industries
responsible for generating non-biodegradable waste are as follows :
F

(i) Thermal power plants which produce fly ash.


(i7) Iron and steel plants which produce blast furnace slag.
(ii7) Metal industries like Al, Cu, Zn which produce mud and tailings.
(iv) Fertilizer industries producing gypsum,
(v) Chemicals, drug, pharmaceutical dyes, pesticides, rubber goods, etc. industries producing inflammable
wastes.

However, if the disposal of these wastes is properly managed or they are given a suitable treatment to
make them harmless, the environmental pollution can be considerably reduced. Some of the methods tliat tu'e
usually employed are given below :
(1) Recycling. This is the most useful method for waste disposal as a number of waste materials can be
used as raw materials to manufacture the useful products again, e.g.,
14/32
Pn^dee^A New Course Chemistry rxilroartn
(0 used glass bottles as well as broken pieces of glass
(//) iron scrap for manufacture of steel
(in) plastic wastes and polythene bags
(/v) used newspapers and magazines for making paper envelopes as well as for making paper.
(2) Burning and Incineration. Some combustible waste materials such as dried plant leaves, etc. can
be burnt to get the heat energy.
Many combustible wastes including household wastes, chemical wastes and biological wastes, e.g.,
from hospitals which occupy large volume can be incinerated, /.<?., reduced to ash which can be used as a
land-filling material. However the gases produced must be suitably treated before they escape into the
atmosphere and cause pollution. This is one of the best methods for the disposal of polychlorinated biphenyl

w
(PCBs) as the high temperature produced breaks the C—Cl bonds. However, the disadvantage of this method
is that incomplete combustion of PCBs results in the production of highly toxic chloro compounds. Moreover,
the ash produced consists of very fine particles which can enter into lungs and cause serious problems.

F lo
(3) Sewage treatment. The sewage is treated in a number of steps as follows :
(/) Large size materials are filtered through screens. They are then used to fill low lying land.
(//) It is allowed to stand in tanks. As a result, many solids settle down (called sludge) while oils

ee
and
grease float on the surface from where they can be skinned off.

Fr
(Hi) The organic materials present in it are allowed to undergo microbial oxidation.
(iv) Finally, the waste water is suitably treated for the removal of phosphate followed by coagulation,
filteration and disinfecting it by adding chlorine.

for
(4) Digesting. This method is used to degrade a number of toxic organic wastes. The method consists in
ur
degrading the waste by microorganisms in the absence of oxygen (called anaerobic digestion). The main
products formed are CO2 and CH4.
2 [CH-,0] > CO. (g) + CH4 (g)
s
ook
Methane can be used as a fuel.
Yo
(5) Dumping. Dumping ot the sewage sludge into sea has been very common in all the countries.
However, now a days dumping of the sludge into land is preferred. This is because it contains compounds of
eB

nitrogen and phosphorus which act as a good fertilizer for the soil. However, dumping of urban sewage has to
be controlled carefully because it contains many toxic metals.
Compost, it is the fertilizer obtained by the rottening of vegetable matter dumped into the land.
r

14.21.3. Some Recent Developments About Waste Recycling


ou
ad

Some new researches are being done so that we can get rid of the garbage but at the same time, it is
utilized for some useful purposes. A few of these are given below ;
Y

(1) The fly ash and slag of steel industry is being utilized by the cement industry.
(2) The plant and animal waste (called biowaste) is being converted into fuel which has high octane
Re
nd

number. It does not contain any lead. It is known as green fuel or biofuel.
(3) The plastic waste is being utilized to make clothes by the textile industries.
Fi

(4) A technology is being developed to produce electricity from the garbage. For this purpose, the
ferrous metals, plastics, glass, paper, etc. are removed from the garbage and then mixed with water. It is then
cultured with suitable bacteria. As a result, methane gas is produced, called biogas. This biogas is used for
producing electricity whereas the left out material is used as manure.

14.22. GREEN CHEMISTRY AS AN ALTERNATIVE TOOL FOR REDUCING POLLUTION


14.22.1. Introduction

We have discussed in this unit the hazards of environmental pollution. One of the major causes of this
pollution is the rapid industrialization particularly the development of those industries which either produce
or use toxic chemicals. One way to protect our environment from chemical effluents and wastes is to use
Green chemistry.
14/33
ENVIRONMENTAL CHEMISTRY

By green chemistry, we mean producing the chemicals of our daily needs using such reactions
and chemical processes which neither use toxic chemicals nor emit such chemicals into the
atmosphere.

Although it is a challenging task but some initial efforts have already been made to achieve this goal.
14.22.2. Techniques Used
Green chemistry docs not employ toxic reagents or solvents and severe conditions but uses mild and
environmental friendly reagents, such as sunlight, microwaves, sound waves and enzymes. Each of these is
briefly described below :
(/) Use of sunlight and microwaves. The use of sunlight and ultraviolet light have given birth to
photochemistry. During the past forty years, a number of photochemical reactions have been studied and
their conditions standardized. One major advantage ot these reactions is that sometimes the products of

w
photochemical reactions arc such that they cannot be obtained by usual chemical methods. Recently, many
known reactions have also been carried out using microwave ovens. In these reactions, no toxic solvents are

F lo
used but only the reactants in proper ratios are mixed on a solid support such as alumina and exposed to
microwaves. The reaction is complete within minutes and yields are not only comparable but in some cases,
they are even higher than those obtained under severe conditions using toxic solvents. Thus, the use of
microwaves have not only reduced the time of the reaction but have also increased the yields. Efforts are,
however, needed to develop the solid supports and optimise the dose of microwave radiations to realize the
desired results.

ree
(h) Use of sound waves. Instead of microwaves, sound waves have also been used to carry out certain

for F
known chemical reactions with encouraging results. This branch of chemistry is called sonochemistry. Here
again efforts are needed to develop conditions to achieve the desired goals.
(Hi) Use of enzymes. One of the most thoroughly investigated environmental friendly reagents are
enzymes. These work in aqueous solutions and at ambient temperatures. Using these enzymes, many
biochemical methods have been developed to prepare precursors and intermediates of certain medicines and
Your
antibiotics. For example, semi-synthetic penicillins such as amphicillin and amoxycillin, have been prepared
ks

using this technique.


It is interesting to point out that usually addition reaction are preferred so that the starting materials
eBoo

completely enter into final products and there is no undesirable material produced which may have to be
disposed of or treated. Further, now complete data is available about the list of hazardous and toxic chemicals
so that the task of chemists has been simplified.
ad

14.22.3. Green Chemistry In Day-to-Day Life


our

A few examples of the chemicals used/produced and the techniques employed in green chemistry are
given below :
(1) Dry cleaning of clothes and laundary. Earlier, letrachloroethene (CUC = CCI2) was used as solvent
for dry cleaning. This compound is suspected to be carcinogenic and contaminated the ground water. Its use
Re

has been replaced by liquefied CO-) along with a suitable detergent which is less harmful. Similarly, for
Y

bleaching of clothes in the laundary, hydrogen peroxide (H2O2) is being used which gives better results and
Find

is not harmful. Moreover, it saves a lot of water.


(2) Bleaching of paper. Earlier, chlorine gas was used for bleaching paper which is highly toxic chemical.
Its use has been replaced by hydrogen peroxide along with a suitable catalyst.
(3) In synthesis of chemicals. For example, ethanal (CH3CHO) is now-a-days being prepared on a
commercial scale using environment-friendly chemicals and conditions, i.e., by one step oxidation of ethene
in the presence of ionic catalyst in aqueous medium
Pd(II)/Cu(II)
CH2 = CH2 + O2 in water
) CH3CHO(90%)

The yield is about 90%.


From the above discussion, it is evident that if concerted efforts are made to develop green chemical
products, it will certainly help us to keep our environment pollution free.

I
14/34
‘P'M^ee^’4^ New Course Chemistry (XI)EEIHD

Curiosity Question
Q. How green chemistry has helped in the dry cleaning of clothes and laundary ?
Ans. Refer to application (1), discussed above.

14.23. SOME ADDITIONAL USEFUL INFORMATION ABOUT ENVIRONMENTAL CHEMISTRY


1. How Bhopal Gas Tragedy Occurred ? Methyl isocyanate is produced by reaction between methyl
amine and phosgene which is further used for the manufacture of the insecticide called ccirharyl (or commercial
name-sevm) by reaction with 1-naphthol as follows :
CH3NH2 + COCI2 ^ CH3 — N = C = 0 + 2HC1

w
MIC

F lo
OH 0—C—NHCH3

CH3N=C=0 + [QJT3 DIQ

ee
Fr
The pressure inside one of the tanks containing MIC rose to such a high level that it blasted the release
valve and as a result MIC escaped into the atmosphere. The reason for the sudden rise of pressure is believed

for
to be the entry of some moisture into the tank that must have resulted into the following exothermic hydrolysis
reaction
ur
CH3—N = C = O + H2O ^ CH3NH2 + CO2
s
2. Electrostatic precipitator for
ook
FIGURE 14.5
Yo
precipitation of particulates. This is the most
effective and efficient method for the removal 30000-40000 VOLTS
eB

of particulates from the air. About 99% of the


particulates can be removed by this method. ■o—► PERIPHERAL
POSITIVE .
It is ba.sed on the principle that particulates of ELECTRODE 6~

all size (e.specially aerosol particles) can be


our
ad

. NEGATIVE
made to acquire negative charge under the ELECTRODE
*

influence of high electric field. These particles <—cr


are then attracted towards the positive
electrode of the electric field where they start 4-A
Y

(TOP VIEW)
accumulating and then removed. For this
Re

FLUE GASES
purpose, the air containing the particulates is
nd

(SIDE VIEW)
allowed to enter a tall chamber in which the
An electrostatic precipitator
electrode in the centre is subjected to a
Fi

negative potential of 30,000-40,000 volts whereas peripheral electrode is earthed. The air inside get ionized
into positive ions and electrons. The electrons are adsorbed on the particulates thereby giving them a negative
charge. These are then attracted towards the positive peripheral electrode on which they accumulate and are
removed by vibrating the electrode.
A schematic diagram of an electrostatic precipitator is shown in Fig. 14.5.
Another device commonly used for separation of dust particles or particulates from the air is called
cyclone collector or cyclone separator. Its working is based on use of centrifugal or gravitational forces. No
charging is required. The dust and particle laden air enters the cylindrical chamber tangentially at one or more
points and leaves through a central opening.
14/35
ENVIRONMENTAL CHEMISTRY

3. Methemoglobinemia or Blue baby syndrome. Blue baby syndrome or simply called blue baby is a
disease resulting in blue coloured skin (especially lips, nails and tongue) in the babies. It occurs in rural areas
where ground water used for drinking contains excess of nitrate. In the digestive system, nitrates are reduced
to nitrite ions whicli react with haemoglobin to form methemoglobin. As a result, oxygen carrying capacity of
haemoglobin is reduced and the tissues or heart do not get sufficient oxygen. The blue colouration of the skin
due to lack of oxygen in the blood is called ‘cyanosis’ (‘cyan" in Greek means ‘blue’).
4. Minamata disease, it is a disease caused by eating of polluted fish (mercury poisoning) from po'Vuted
Minamata Bay. The incident occuned during 1935-1960 when hundreds of Japanese died due to this disease.
5. Ecosystem. The co-existence of biotic components (living organisms), i.e., animals, plants and

w
microorganisms and abiotic components (non-living materials) which may be inorganic or organic in the
presence of suitable climatic factors (like temperature, himidity etc.) is called an ecosystem. These biotic and
abiotic components are regarded as linked through nutrient cycles and energy flows. For example, plants
for their growth. Leaves falling from the plants and getting

F lo
(biotic) take up nutrients (abiotic) from the soil
buried into the soil fertilize the soil. Similarly, plants (biotic) take up CO2 (abiotic) from the atmosphere and
carry out photosynthesis in the presence of sunlight and give back oxygen to the atmosphere.

ee
14.24. TYPICAL PROBLEMS

Fr
P. 1. (fl) Which chemical substance is present as enamel on the surface of the teeth ?
(b) Traces of fluoride ions (F^) in drinking water (about 1 ppm) greatly reduce the Incidence of
dental cavities (tooth decay). What is the reason for reduction in cavities ?

for
Sol. (a) The chemical substance present as enamel on the surface of the teeth is 3 Ca3(P04)2-Ca(0H)2
ur
(/>) The fluoride ions (F") convert the enamel to much harder 3 Ca3(P04)2.Cap2. Hence, the incidence of
cavities is greatly reduced.
s
P. 2. How is ozone formed in the atmosphere ? How chlorofluoroca rbons destroy the ozone layer ?
ook
Yo
Sol. Refer to Art. 14.9.1 and 14.9.2.
P. 3. What would be the consequence of presence of following in water ?
eB

(i) Sulphates > 500 ppm


(h) Fluorides between 2 ppm-10 ppm and > 10 ppm
(ni) Nitrates > 50 ppm
r

Sol. (i) Sulphates > 500 ppm have laxative effect.


ad
ou

iii) Fluorides 2 ppm-10 ppm cause brown mottling of teeth


Fluorides > 10 ppm .me harmful to bones and teeth
(<//) Nitrates > 50 ppm cause methemoglobinemia (blue baby syndrome)
Y

P. 4. What is the pH of normal rain water ? What happens to the pH of rain water if it Is accompanied by
Re

a thunderstorm and why ?


nd

Sol. pH of normal rain water is about 5-6. When rain is accompanied by a thunderstorm N2 and O2 of the air
combine to form NO which is further oxidized to NO2 and dissolves in rain water to fonn HNO3. Hence,
Fi

rain water becomes more acidic and hence pH falls below 5-6.
P. 5. One of the products of burning of coal is a toxic gas ‘A’. The gas A combines with the haemoglobin
to form a compound ‘B’. Concentration of A greater than 750 ppm leads to disease called ‘C’ in
human beings. However, the patient suffering from ‘C’ can be treated in a high pressure chamber
containing gas ‘D’. The main product formed on keeping in gas ‘D’ is the product E which helps the
person to recovery. What are A, B, C, D and E ? Write the reactions involved.
Sol. A = Carbon monoxide (CO)
B = Carboxyhaemoglobln (HbCO)
C = Anoxia or Asphyxiation
D = Oxygen (O2)
14/36
4 New Course Chemistry (XI) orsrMD
E = Oxyhaemoglobin (HbO^)

Reaction.
c+^o, ^ CO

Hb + CO HbCO
Carboxyhaemoglobin

HbCO + O2 Hb02
Oxyhaemoglobin

ow
' What are the following diseases and what are they due to ?
(1) Asphyxiation (h) Pneumoconiosis (m) Black foot (iv) Methemoglobinemia
Sol. (0 Asphyxiation. Acute oxygen-starvation in the blood is called asphyxiation or anoxia. It happens when
a person stays for long in the air having a concentration of CO more than 750 ppm.
(ii) Pneumoconiosis. It is a disease of lungs commonly found in industrial workers. It is due to accumulation
of small particulates in the lungs which act as sites for absorption of carcinogenic compounds such as

e
polynuclear hydrocarbons, asbestos etc.

Fl
re
(Hi) Black foot. It is a peripheral vascular disease in which the blood vessels in the lower limbs are
severely damaged resulting eventually in gangrene. It is due to high concentration of arsenic in ground

F
water.

(/V) Methemoglobinemia. It is a blue baby syndrome which results in a blue colour skin and a congenital
(from birth) heart defect in infants which laymen often call ‘hole in the heart’. It occurs when water used
ur
or
for drinking contains nitrates > 50 ppm.

k sf
Yo
Environment. It means surroundings. It consists of four components :
oo
(i) Atmosphere. It is a cover of gases upio a height of about 1600 km from the surface of the earth. Gases
mainly present are O2, N2, O3, CO2 and H2O vapour. It is further divided into four regions :
B

(fl) Troposphere (0—11 km). It is the domain of all living organisms and is greatly affected by air pollution.
Temperature decreases from I5®C to - 56°C.
re

{b) Stratosphere (11 50 km). It contains ozone and hence is called ozonosphere. It protects the living
organisms from harmful ultraviolet radiation. Its temperature increa.ses from - 56*’C to - 2°C.
u
ad

(c) Mesosphere (50—85Toh). It contains gases in the ioinzed form (0+, NO+, etc.) and is called ionosphere.
Yo

Its temperature decreases from - 2“C to - 92®C.


{d) Thermosphere (85—500 km). It is also ionosphere. Its temperature increases from - 92°C to 1200°C.
(ti) Hydrosphere. It is that part which contains water in the the form of ocean, rivers, lakes, etc. It covers
d

about 75% of earth’s surface,


Re
in

(m) Lithosphere. It consists of solid components like soil, rocks, mountains, etc. The upper part consisting
of weathered rocks and organic matter is called soil. The lower 8—40 km thick part is called crust,
F

(iv) Biosphere. It consists of animals and plants.


Environment chemistry. It is that branch of science which deals with the chemical phenomena occurring
in the environment.
Environmental pollution and pollutants. The addition of undesirable material to air, water and .soil by
natural source or due to human activity to such a level of concentration that adversely affects the life on
1 ■
the earth is called environmental pollution. The undesirable materials thus added are called pollutants.
Difference between a pollutant and a contaminant. A pollutant is a substance produced by natural
source or human activity or combined effect of both such that its concentration is so high that it has
hanntul effects on living organisms or non-living components. A contaminant is a substance added into
environment only by human activity and may or may not be harmful to living organisms or non-living
components.
14/37
ENVIRONMENTAL CHEMISTRY

5. Theshold Limit Value (TLV). The permissible limit of a pollutant in the atmosphere to which if a healthy
worker is exposed for 8 hours a day or 40 hours of a week throughout his life, there is no adverse effect on
him is called threshold limit value. TLV of CO is 40 ppm while that of CO2 is 5000 ppm.
6. Types of pollutants :
(i) Primary and secondary pollutants. Primary pollutants are those which after their formation enter
into environment directly, e.g., NO formed from Nj and Oj. Secondary pollutants are those which are
formed from primary pollutants, e.g., peroxyacylnitrate (PAN) from oxides of mtrogen and hydrocarbons.
(i7) Biodegradable and non-biodegradable pollutants. Biodegradable pollutants are those which ^e
decomposed by microorganisms, e.g., cow-dung, domestic sewage, etc. and thus may not be ha^ul.
Non-biodegradable pollutants are those which are not easily decomposed and hence are harmful, e.g.,
mercury, DDT, etc.
7. Air or Atmospheric pollution. It is the addition of undesirable materials into the atmosphere either from

w
natural phenomena or human activity which adversely affects the quality of air and hence life on the earth.
The most important part of the atmosphere is troposphere where we live. The main pollutants of the
troposphere are :

F lo
(0 Carbon monoxide (CO). It enters into atmosphere mainly due to incomplete combustion of fossil
fuels or hydrocarbon fuels of automobiles. It is poisonous because it combines with haemoglobin more
easily than oxygen to form carboxyhaemoglobin (Hb + CO > HbCO) which is much more stable th^
oxyhaemoglobin. Thus, the function of haemoglobin to transport oxygen to different p^ of the body is
hindered. Thus, body becomes oxygen-starved. The disease is called anoxia or asphyxiation. If CO exceeds

e
Fre
100 ppm, person suffers headache and dizziness. If CO exceeds 750 ppm, it leads to coma and death. For
this reason, a burning coal angithi in a closed room in winter should never be used.
(«) Hydrocarbons. Like CO, the main source of hydrocarbon pollutants is the internal combusion engines

for
in which unbumt fuel or partially burnt fuel is emitted in the exhaust gases.
At low concentration, they are not harmful. At concentration > 500 ppm, they are carcinogenic and cause
cancer. On photochemical reaction with O2 and oxides of nitrogen, they form harmful compounds and
r
produce photochemical smog.
You
{Hi) Oxides of nitrogen (NO, NOj, etc). They are produced due to combustion of fossU fuels (coal, oil,
oks

gasoline etc.) both in motor vehicles and power plants where temperature rises so high that H2 and O2 of
> 2 NO, 2 NO + O2 >2 NO2). At present pollution
eBo

air combine to form NO, NO2, etc. (^2 + ^2


level, neither NO nor NO2 is harmful. NO2 is more dangerous as it is a toxic gas and affects respiratory
system and damages lungs. Concentration > 100 ppm is fatal. Though NO has about 1500 times greater
affinity than CO for haemoglobin but fortunately, it is unable to enter the blood stream from atmosphere.
But presence of large amounts may increase NO2 by oxidation and thus become harmful. Further, oxides
our
ad

of nitrogen in air cause acid rain and cause photochemical smog.


(jv) Oxides of sulphur (SOj and SO3). They enter into atmosphere mainly due to volcanic eruptions.
They are also produced due to combustion of sulphur containing coal and fuel oil in thermal power plants
and due to roasting of sulphide ores (FeS2, CuFeS2, CuS, ZnS, PbS, etc.). Both SO2 and SO3 are strongly
iixitating to respiratory tract. SO2 causes throat and eye irritation. It causes breathlessness and affecte
dY
Re

larynx (voice box). It slows down the formation of chlorophyll in plants causing loss of green colour. It is
called Porosis. They also cause acid rain,
Fin

(v) Particulates. These are small solid particles and liquid droplets suspended in the air. They are present
in the air in the form of soot (produced due to incomplete combustion of fossil fuels), ash (coming wi^
the furance flue gases), inorganic particles (metallic, metal oxides, lead halides, asbestos dust, sulphuric
acid and nitric acid) and organic particles (paraffins, olefins, polycyclic aromatic hydrocarbons, i.e.,
etc.). The particulates may also be classified as viable and non-viable. Viable particulates are small size
living organisms such as bacteria, fungi, moulds, algae, etc. Non-viable particulates are formed by
disintegration of large size materials or condensation of small size particles or droplets. These include
mist, smoke, fumes and dust. These particulates pass through the nose easUy and enter the lungs where
they act as sites for adsorption of carcinogenic compounds causing lung cancer and bronchital asthma.
This disease is common in industrial workers and is called pneumoconiosis, {asbestosis due to asbestos,
silicosis due to silica, etc.). They scatter light and hence affect visibility on foggy nights. They counteract
the greenhouse effect as they reflect back heat of sunlight.
14/38
“P^adce^ ^ New Course Chemistry (XI)
8. Ozone
^ layer (Earth’s
^ , , protective
. , umbrella). It exists altitude of 25—30 km (in the stratosphere). It
absorbs the harmful ultraviolet radiation coming from the sun and hence protects life on the earth It is
tormed from O2 and dissociates back into Oj through the following reactions :
O3 and then 03-5^^02 + 0 + Heal. Heal iiberatcd raises the temperature
^+1^ of ozone layer {creation of ozone hole) is taking place due to reaction
with NO (produced from natural sources or human activity or in the exhaust gases of engines of supersonic
planes) or by reaction with chlorotlorocarbons (CFC’s), called (produced from aerosol sprays used
in propellents or from refrigerators where they are used as coolants). The reactions occuring are :
> NO2 + O2, NO2 + O > NO + O2 and CF2 CI2 ■ CF2CI + Cl ■, CFCI, ● CFCU
+ Cl*, Cl ● + O3 > CIO ● + O2, Cl O ● + O > Cl ● + O2). The ozone hole thus produced allows the UV

low
radiations to pass through and reach us, increasing chances of skin cancer.
9. Smog and its kinds. The word ‘smog’ is a combination of ‘smoke' and ‘fog’ because earlier, it was found
to be formed by condensation of fog on carbon particles. Now, it is a misnomer because we also have
photochemical smog which contains neither smoke nor fog. Two types ol smogs are :
(z) London smog/SulpImrous smog/Classical smog. This type of smog was finst observed in London in
1952. It is formed due to presence of SO2 and humidity in the air which combine to form H7SO4 fog

ee
which deposits on particulates. It involves smoke and fog. It is formed in the months of winter particularly

F
in the morning hours when the temperature is low. It causes bronchitis irritation i.e., problem in the lungs.

Fr
As it contains C and SO2, it is reducing in nature.
(z7) Photochemical smog/Los Angeles smog. It was first observed in Los Angeles in 1950. It is formed

for
due to photochemical reactions taking place when air contain NO2 and hydrocarbons [NO9 hv > N0 +
0,0 + 02 ^ ®3* O3 + NO > NO2 + O2, Hydrocarbons + 0 > RCO* (free radicals), RCO* + O2
ur
> RCO3 , RCO3 + Hydrocarbons > RCHO, R2C = 0, RCO3 + NO- > RCO2 + NO2, RCO3 +
O2 + O3 + RCOj, RCO3 + NO2 > RCO3 NO2 (Paroxyacyinitraie. PAN)). Thus, concentration of
s
ook
O3, PAN, RCHO and R2CO builds up in the atmosphere. These compounds produce irritation in the eyes.
Yo
It IS formed in the months of summer during afternoon when there is a bright sunlight so that photochemical
reaction can take place. As it contains O3 and NO2, it is oxidizing in character.
eB

10. Acid rain. It is the rain water containing H2SO4 and HNO3 (along with small amounts of HCl acid) which
are formed from the oxides of sulphur and nitrogen present in the air as pollutants and has a pH of 4 5.
(Remember that normal rain water has a pH of about 5-6). The acids are produced in the air from oxides of
nitrogen and sulphur present as pollutants through the following ractions :
r

(0 NO + O3
ou

^ NO2 + O2, NO, + O3 > NO3 -I O,, NO2 + NO3


ad

^N205,
N2O5 + H2O > 2 HNO3
NO, soot
Y

(z7) 2 SO2 + O2 or metal ions ^ 2 SO3, SO3 + H2O ^ H2SO4


HNO3 and H2SO4 formed above combine with HCl present in air and come down to eartii along with rain.
Re
nd

It damages buildings made of marble like Taj Mahal at Agra, iron and steel structures. Increase of acid in
lakes causes fish to die. It can be reduced by using less vehicles driven by fossil fuels, using less sulphur
content fossil fuels tor power plants and industries and using catalytic convertors in cars so that NO,
Fi

are
converted into N2.
11. Greenhouse effect and Global warming. CO2 in normal concentration in the air is not a pollutant.
However, when its concentration increases due to human activity, it results in the wiu-ming of the earth’s
surface. This happens because visible and ultraviolet radiations can pass through CO2 and H,0 vapour
present in the atmosphere. Ultraviolet radiation are absorbed by ozone layer but visible light reaches the
earth and heats it up. It is re-emitted in the fonn of infrared radiations which heat up CO2 and H,0 vapour
but cannot pass through them and cannot go back to upper atmosphere and is thus radiated back to earth.
The warming of earth due to re-emission of sun's energy absorbed by the eaiih followed by its absorption
by CO2 and H2O vapour presents near the earth's surface and then Us radiation hack to earth is called
greenhouse effect. The phenomenon is similar to greenhouse for plants. Due to increase in temperature of
earth’s surface, melting of glaciers and polar ice caps will occur. As a result, level of sea water may rise
causing floods in the coastal land.
14/39
ENVIRONMENTAL CHEMISTRY

12. Water pollution. It is the comaniinaiion of water by foreign substances which make it harmful for health
of animals or plants or aquatic life and make it unfit for domestic, industrial and agricultural use. Depending
upon the source, we may have following water pollutions :
(i) Ground water pollution. It is water present below the surface of the earth and is pure because it
collects after passing through the soil. However, due to industrial effluents and fertilizers and pesticides,
it gets contaminated. When ground water is contaminated with high concentration of arsenic, it causes a
disease of the foot (gangrene) called ‘black foot disease.’
(i7) Lake water pollution. It occurs due to flow of inorganic nutrients from agricultural land and toxic
materials from urban areas or dumping of industrial effluents into them.
(in) River water pollution. It occurs due to discharge of industrial wastes and domestic sewage into them,
(iv) Sea water pollution. It occurs due to wreckage of oil tankers or oil leakage from pipe lines or refineries

w
located nearby or deliberate marine pollution by crude oil. The spreading of oil into sea is called oil-spill
and layer on the surface is called oil slick. It causes heavy damage to fishery, sea birds and aquatic animals.
13. Sources of water pollution. (/) Sewage and domestic wastes (//) Industrial effluents (Hi) Agricultural

F lo
nutrients (tv) Thermal power plants (causing increase of temperature) (v) Radioactive discharges
(vO Polychlorinated biphenyls (PCBs) used in transformers and capacitators.
The amounts of heavy metals like Sb, Hg, As, Cd and Pb is called ‘body burden.’

ee
14. Biochemical Oxygen Demand (BOD) of water. Aquatic life needs dissolved oxygen. However, human
sewage and organic wastes of industry create a problem. Similarly, leaves, grass trash etc. also enter into

Fr
water due to run off. They result in excessive phytoplankton growth. The microorganisms which decompose
this organic waste need oxygen which depends upon amount of waste present. The total amount of oxygeit
consumed bv microorganisms (bacteria) in decomposing the waste (organic matter) present in a certain

for
volume of a sample of water is called Biochemical Oxygen Demand (BOD) of water. This decomposition
requires a long time (20—30 days). Hence, we determine BODj, i.e., oxygen consumed in 5 days. This is
ur
done by first saturating the water with oxygen and incubating it for 5 days at 20°C. Oxygen remaining after
5 days is determined and oxygen consumed is found by subtraction. Pure water has BODg < 5 ppm and
s
polluted water > 17 ppm.
ok
Yo
15. Chemical Oxygeu Demand (COD). This is a quicker method. It consists in treating the given sample of
water with a known amount of an oxidizing agent, generally K2 Ct2 O7 + dil H2SO4. All pollutants get
o

oxidized. The amount of K2 Cr^O^ left is found by titration against Mohr salt and result is calculateed.
eB

16. Eutrophication. Land run off of nutrients from fertilizers results in the increase of phosphate ions in
water in the lakes. As a result, formation of algae is accelerated which covers the water surface and hence
concentration of dissolved oxygen decreases. Also, presence of phosphates accelerates the growth of
our

plants which consume oxygen. As sa result, aquatic life is affected and fish start dying. This process is
ad

called eutrophication.
17. International standards of drinking water : (/) Fluoride < 1 ppm (1 mg dm“^) - protects teeth against
decay by converting hydroxypatite (teeth enamel) into harder fluorapatite.
Y

(ii) Lead < 50 ppb or pg dm"-^ - Higher concentration causes damage to kidney and brain.
Re

(Hi) Sulphates < 500 ppm - Higher concentration has laxative effect.
nd

(iv) Nitrates < 50 ppm - Excess causes methemoglobinemia (blue baby syndrome),
Fi

(v) pH 5-5 - 9-5.


18. Soil pollution. Soil is the uppermost part of earth’s crust. It consists of (/) mineral matter (gravel, sand,
etc. formed from rocks), (ii) Organic matter (fromed from fallen trees, leaves, etc. and remains of dead
animals and decomposed by microorganisms and converted into humus). (Hi) Biological material (like
algae, bacteria, etc.), (/v) Soil water (which acts as a .solvent for organic and inorganic materials), (v) Soil
air (present in the pores of the soil).
The main pollutants of soil are (/) Industrial wastes (ii) Urban wastes (Hi) Radioactive wastes and
(iv) Agricultural materials viz fertilizers containing excess ol nitrates and phosphates and pesticides.
Pesticides consist of (n) Insecticides (used to kill the insects which destroy the crop, e.g., DDT, BHC, etc.
or now used are Aldrin and Dieldrin) (b) Herbicides (used to control weeds, e.g., NaC103, Na3As03, etc.
or now used are triazines)(r) Fungicides(used to stop growth of fungi, e.g., organo-mercury compounds).
14/40
New Course Chemistry (XI)H siau

19. Control/Management of environmental pollution. Two main sources of environmental pollution are
(0 household waste and («) industrial waste. Household waste can be managed by using separate garbage-
bins for biodegradable and non-biodegradable (re-cyclable) wastes like polythene bags, etc. which choke
the sewers. Industrial waste can
be managed by (z) recycling waste like glass, plastic, etc. (z7) Burning

ow
and incineraton, e.g., plant leaves, etc. (zzz) sewage ireatmem before dispo.^ing it off (zv) Digesting, i.e.,
degradation of toxic organic waste by microorganisms, (v) Dumping of sewage sludge into land.
20. Green chemistry. It deals with producing chemicals of our daily needs using such reactions and chemical
processeswhich neither use toxic chemicalsnor emit such chemicals into the atmosphere. The techniques
generally used in green chemistry are (z) Use of sunlight and microwaves (photochemistry) (zz) Use of
sound waves (sonochemistry) {Hi) Use of enzymes. A few examples of u.se of green chemistry in everyday
life are (z) Dry cleaning of clothes and laundary (using liquefied CO2 along with a suitable detergent

re
instead of tetrachloroethene used earlier) (z7) Bleaching of paper (using H20-> in place of CI2) (zzz) In
synthesis of chemicals using environmental friendly chemicals and conditions.”

Flr
F
ou
sr
fo
k
oo
Y
reB
uY
ad
do
in
Re
F
14/41
ENVIRONMENTAL CHEMISTRY

1^1

P HP NEET/JE
SPECIAL

w
For ultimate preparation of this unit for competitive examinations, students should refer to
● MCQs In Chemistry for NEET .■
Pradeep’s Stellar Series....

F lo
● MCQs In Chemistry for JEE (Main) -

separately available for these examinatior^s.

ee
Multiple Choice Questions (with one correct Answer)

Fr
6. What is the effect of release of CO2 gas on
I. Environment and
atmosphere ?
environmental pollutants

for
(a) Global warming
1. Which of the following is the coldest region ? (/?) Photochemical smog
ur
(o) Troposphere (b) Mesosphere (c) Ozone layer depletion
(c) Stratosphere (d) Thermosphere (d) Tsunami (JEE Main 2019)
oks
7. London smog is found in
II. Atmospheric pollution and its effects (a) Summer during day time
Yo
2. Which oxide of nitrogen is not a common pollutant (b) Summer during morning time
o

introduced into the atmosphere both due to natural


eB

(c) Winter during morning time


and human activity ? (d) Summer during day time
(a) N2O (b) NO2 8. Photochemical smog is formed in
(c) N2O5 id) NO (a) Summer during morning time
our

(AMU Engg. 2015, NEET 2018)


ad

(b) Summer during day time


3. All are primary pollutants except (c) Winter during morning time
ia) SO, (b) H2SO4 (d) Winter during day time
ic) NO2 id) Particulate matter 9. Which one of the following statement is false ?
Y

(IAS Prelim 2010)


(a) Photochemical smog causes irritation in eyes
Re

4. Which of the following is a sink for CO ? (b) London smog is a mixture of smoke and fog
nd

(a) Haemoglobin (c) Photochemical smog results in the formation


ib) Microorganism present in the soil of PAN
Fi

(c) Oceans id) London smog is oxidising in nature


id) Plants (NEET 2017) (AIIMS 2014)
5. The gas leaked from a storage tank of the Union 10. The molecule that has minimum/no role in the
Carbide plant in Bhopal gas tragedy was : formation of photochemical smog is
(a) Phosgene ib) Methyl isocyanate ia) CH2 = O ib)^2
(c) Methylamine id) Ammonia ic) O3 id) NO
(JEE Main 2013) (JEE Main 2019)
ANSWERS

1. ib) 2. (c) 3. (h) 4. (/;) 5. ib) 6. (a) 7. (c) 8. (/>) 9. (rf) 10. (h)
14/42
'Pn^KCce^’^i. New Course Chemistry (XIlrosTTn
11. The correct species responsible for photochemical
smog are
(b) It is due to reaction of SO2, NO2 and CO2
with rain water
(a) CO2, NO2, SO2 and hydrocarbons (c) Causes no damage to monuments like Taj
(b) N2, NO2 and hydrocarbons Mahal
(c) NO, NO2, O3 and hydrocarbons (d) It is harmful for plants (NKET 2020)
(d) No, O2, O3 are hydrocarbons
19. Among the gases (I) to (V), the gases that cause
(JEE Main 2019) greenhouse effect are
It. In Antartica, ozone depletion is due to the
(I) CO2 (II) H2O
formation of the following compound :
(a) Acrolein (III) CFCs (IV) O.
(b) peroxy aceby! nitrate
(c) SO, and SO3 (d) chlorine nitrate (V) O3

w
(e) formaldehyde (a) (I), (III), (IV) and (V)
13. Among the following, the one which is not a (h) (I), (H), (III) and (V)
‘greenhouse gas’ is

F lo
(c) (I), (II), (III) and (IV)
(a) N2O (b) CO2 (d) (I) and (IV) (JEE Main 2020)
(c) CH4 (d) O2 20. The statement that is not tnie about ozone is
(West Bengal JEE 2014)
(a) In the stratosphere, CFCs release chlorine free
14. Among the following, the one that is not a

ee
greenhouse gas is radicals (Cl) which react with O3 to give

Fr
(a) sulphur dioxide (b) nitrous oxide chlorine dioxide radicals
(c) methane (d) ozone (MEET 2019) (b) It is a toxic gas and its reaction with NO gives

for
15. Identify the wrong statement in the following : NO,
(a) Ozone layer does not permit infrared radiation (c) In the atmosphere, it is depleted by CFCs
from the sun to reach the earth
ur
id) In the stratosphere, it fonns a protective shield
(b) Acid rain is mostly because of oxides of against UV radiation (JEE Main 2020)
nitrogen and sulphur
s
21. The presence of ozone in troposphere
ook
(c) Chlorofluorocarbons are responsible for ozone
Yo
layer depletion (a) generates photochemical smog
(d) Green house effect is responsible for global (b) protects us from UV radiation
eB

warming (AIEEE 2008) (c) protects us from X-ray radiation


16. The gas emitted by supersonic jet planes that (d) protects us from greenhouse effect
slowly depletes the ozone layer is
(JEE Main 2021)
our

(fl) CO ib) NO
ad

(c) SO, id) 0,


22. The type of pollution that gets increased during
(e) HF (Kerala PET 2012)
the day time and in the presence of O3 is :
(fl) Global warming (b) Reducing smog
17. Which of the following is not correct about carbon
monoxide ? (c) Acid rain id) Oxidizing smog
dY

(JEE Main 2021)


Re

ia) It reduces oxygen carrying ability of blood


ib) The carboxyhaemoglobin (haemoglobin 23. The greenhouse gas/es is (are)
Fin

bound to CO) is less stable than oxyhaemo- (1) Carbon dioxide (2) Oxygen
globin (3) Water vapour (4) Methane
(c) it is produced due to incomplete combustion Choose the most appropriate answer from the
id) It forms carboxyhaemoglobin (NEET 2020) options given below :
18. Which of the following statement is not true about (a) (i) and (2) only ib) (I), (3) and (4) only
acid rain ?
(r) (1) and (3) only (f/)(!)onIy
ia) Its pH is less than 5-6
(JEE Main 2021)
ANSWERS

II. (c) 12. id) 13. id) 14. (f/) IS. ia) 16. ih) 17. ic) 18. (c) 19. ih) 20. (c) 21. ih)
22.id) 23. ib)
14/43
ENVIRONMENTAL CHEMISTRY

24. Reducing smog is a mixture of 31. The maximum prescribed concentration of


cadmium in drinking water in ppm is
(a) Smoke, fog and N2O3
(a) 0 05 ib) 3
(b) Smoke, fog and O3 id) 5
(c) 2
(c) Smoke, fog and SO2 (Kerala PET 2012)
ie) 0-005
{d) Smoke, fog and CHt = CH-CHO 32. The concentration of fluoride, lead, nitrate and iron
{JEE Main 2021)
in a water sample from an underground lake was
25. Which of the following gases retards the rate of found to the 1000 ppb. 40 ppb, 100 ppm and
photosynthesis ? 0-2 ppm respectively. This water is unsuitable for
(fl) CO ib) NO2 drinking due to high concentration of
ic) CO2 id) CFC (a) fluoride (b) lead

w
(JEE Main 2021) (c) nitrate (d) iron
26. On the surface of polar stratospheric clouds, (JEE Main 2016)
hydrolysis of chlorine nitrate gives A sand B while 33. A water sample has ppm level concentration of

F lo
its reaction with HCl produces B and C. A, B and following anions
C are respectively
(a) HOCl, HNO3, CI2 (h) CI2, HNO3, HOCl
p-= 10; SO^- = 100; NO3 =50
The anion/anions that make/makes the water

ee
(c) HCIO2, HNO2, HOCl
sample unsuitable for drinking is/are

Fr
(d) HOCl. HNO2. CI2O (JEE Main 2022)
27. The acid that is responsible for the damage of Taj (a) only F" (b) only SO|“
Mahal is
(c) only NO3 (d) botli SO4 Jtnd NOj

for
(<7) sulphuric acid (b) hydrofluoric acid (JEE Main 2017)
ur
(c) phosphoric acid id) hydrocloric acid 34. The recommended concentration of fluoride ion
(JEE Main 2022) in drinking water is up to 1 ppm as fluoride ion is
28. The pollution due to oxides of sulphur gets required to make teeth enamel harder by
oks
enhanced due to the presence of converting [3 Ca3(P04)2 ■ Ca(OH)2] to
Yo
(0 particulate matter (») ozone (a) [CaF2]
o

iiii) hydrocarbons (/V) hydrogen peroxide ib) [3 (CaF,). Ca(OH)2l


eB

Choose the most appropriate answer from the (c) [3 Ca3(P04)2 . CaF2l
id) [3 (Ca(OH)2 . Cap2)] (JEE Main 2018)
options given below :
ia) (/), (/V) only ib) (/), (//), (h') only 35. Biochemical oxygen demand (BOD) is the amount
our

of oxygen required (in ppm)


ad

(c) iii), iiii). iiv) only id) ii), iiii), (iv) only
(NEET 2022) ia) for the photochemical breakdown of waste
present in 1 m-^ volume of a water body
III. Water pollution (h) for sustaining life in a water body
Y

29. Water samples with BOD values of 4 ppm and 18 (c) by bacteria to break down organic waste in a
Re

ppm respectively are certain volume of a water sample


nd

(a) Highly polluted and clean id) by anaerobic bacteria to break down inorganic
(b) Highly polluted and highly polluted waste present in a water body
Fi

(c) Clean and highly polluted (JEE Main 2020)


id) Clean and clean (JEE Main 2019)
36. Which of the following statement(s) is (are) the
30. Excess nitrate in drinking water can cause incorrect reason for eutrophication?
ia) methemoglobinemia
(A) excess usage of fertilizers
ib) kidney damage
id) laxative effect
(B) excess usage of detergent
(c) liver damage
ie) leucoderma (Kerala PMT 2011) (C) dense plant population in water bodies
' '
ANSWERS

24. ic) 25. ib) 26. (a) 27. (a) 28. ib) 29, (c) 30.ia) 31. ie) 32.(c) 33.ia) 34.(c)
35. (c)
14/44
7>uuUe^'4. New Course Chemistry rxnrosTTi
(D) lack of nutrients in water bodies that
plant growth
prevent
{d) utilization of existing knowledge base for
reducing the chemical hazards along with
Choose the most appropriate answer from the developmental activities
options given below :
(e) synthesis of chemical compounds using green
(a) (D) only (h) (C) only light (Kerala PET 2014)
(c) (B) and (D) only (d) (A) only 41. Green chemistry used in day-to-day life is for
(JEE Main 2021) (a) cleaning clothes with water
37. The eutrophication of water body results in
(b) using liquid H2O2 in dying clothes
(a) loss of biodiversity (c) using tetrachloroethane is laundries
(h) break down of organic matter (d) using chlorine in bleaching paper
(c) increase in biodiversity (JEE Main 2021)
(d) decrease in BOD (JEE Main 2022) 42. Correct formula of hydroxyapatite is

w
IV. SoilTLand pollution (a) Ca(P04)6. CaF2
(b) 3 [Ca3(P04)2l . Ca(OH)2

F lo
38. Which one of the following statement is not (c) 3 [Ca3(P04)2l . Cap2
correct?
(d) Ca3(P04)2 . Ca(OH)2 (JEE Main 2021)
(a) DDT and BHC are not good insecticides 43. Which one of the following elemental form is not
because they are highly soluble in water present in the enamel of the teeth ?

e
(b) DDT and BHC are not good insecticides

Fre
(a) Ca-^ {b) p3+
because they are absorbed by the soil and (c)F-‘ (d) p5+
contaminate root crops
(JEE Main 2022)

for
(c) Aldrin is not a good insecticide because it is
not biodegradable VI. Miscellaneous
(d) All the above are incorrect
44. Which one of the following statement is not true ?
r
V. Strategy for control of (fl) Oxides of sulphur, nitrogen and carbon are the
You
ks
most widespread air pollutant
environmental pollution
(b) pH of drinking water should be between
o

39. Green chemistry means such reactions which


eBo

5-5-9-5
(a) produce colour during reactions (c) Concentration of DO below 6 ppm is good
ib) reduce the use and production of hazardous for the growth of fish
chemicals
(d) Clean water would have a BOD value of less
(c) are related to the depletion of ozone layer than 5 ppm
our
ad

(AIPMT Prelim 2011)


(d) study the reactions in plants (AIPMT 2008) 45. The process of calcination and roasting in
40. Green chemistry deals with metallurgical industries, respectively, can lead to
(a) study of plant physiology (a) global warming and photochemical smog
(b) study of extraction of natural products from (b) photochemical smog and ozone layer depletion
dY
Re

plants (c) ozone layer depletion and global wrming


(c) detailed study of reactions involved in the (d) global warming and acid rain
Fin

synthesis of chlorophyll (JEE Main 2020)

m Multiple Choice Questions (with One or More than One Correct Answers)
46. The ozone layer is depleted by (b) CO2 can absorb infrared red radiations but
(a) NO (b) SO2 (c) C^H^, (d) CFCs does not allow then to pass through,
47. Choose the wrong statements : (c) NO is more harmful than NO2.
(a) CO2 is responsible for greenhouse effect. (d) Acid rain contains mainly HNO3.
ANSWERS
36. (M) 37. (a) 38. (a) 39. (b) 40. (d) 41. (b) 42. (c) 43.(4.) 44. (c)
45. (d) 46. (a.d) 47. (f.rfi
14/45
ENVIRONMENTAL CHEMISTRY

QQ Matching Type Questions


Match the entries of column I with appropriate entries of column II and choose the correct option out
of the four options (a), (b), (c), (d) given at the end of each question.
48. Column I (Di.sea.se/PhenoinenoH) Column II (Cai/ir')

(A) Anoxia/Asphyxiation (p) SO2


(B) Chlorosis (q) Particulates
(C) Pneumoconiosis (r) CO
is) Plant nutrients
(D) Eutrophication
(a) A-r, B-p, C-q, D-5 (b) A-p, B-q, C-r, D-j

w
(c) As, B-p, C-q, D-r (d) A-r, B-q, C-p, D-5

F lo
49. Column I (Pollutant of oir/water) Column II (Thre.siwld limit)

(A) CO in air (P) 50 ppm


(B) CO2 in air (^) 40 ppm
(C) Fluorides in water (r) 5000 ppm

ee
(D) Nitrates in water is) 1 ppm

Fr
(E) BOD5 of dissolved oxygen of pure water (0 5 ppm

(a) A-p, B-r, C-^, D-r, E-^ (b) A-r, B-q, C-t, D-p, E-5

for
(c) A-q, B-r, C-i’, D-p, E-r (d) As, B-r, C-p, D-q, E-t
r
50. Match List-I with List-II
You
List 1 List II
s
(0 Acid rain
ook

(P) 2SO2 (g) + 02 (g) ^ 2 SO3 (g)


hv
HOCl (g) (ii) Smog
eB

(<?) ^ OH + Cl

ir) CaC03 + H2SO4 ^ G1SO4 + H2O + CO2 iiii) Ozone depletion


hv
is) NO2 (g) NO ig) + 0(g) (iv) Tropospheric pollution
our
ad

Choose the correct answer from the options given below :


ia) ip) - ii), iq) - iii), (r) - (i7i), (s) - (iv) ib) (p) - iii), iq) - iiii), (r) - (iv), (s) - (i)
(c) (p) - iiv), iq) - iiii), ir) - (1), is) - HO id) (p) - m, iq) - (io. (0 - (rv), is) - (i)
dY

(MEET 2021)
Re

51. Match List-I with List-II


Fin

List 1 (Pollntani) List II (Sawrrc)

A. Microorganisms 1. Strip mining


B. Plant nutrients II. Domestic sewage
III. Chemical fertilizer
C. Toxic heavy metals
D. Sediment IV. Chemical factory
Choose the correct answer from the options given below :
(a) A-II, B-m, C-IV, D-I ib) A-n, B-I, C-IV, D-in
id) A-I, B-IV, C-III, D-IT (JEE Main 2022)
(c) A-I, B-IV, C-II, D-III

48. (a) 49. (r) 50. (r) 51. (a)


14/46 ‘a
Course Chemistry fXT^PTsyn
52. Match List-I with List-II
List I (Pollutant) List II (Disease/sickness)
A.
Sulphate (> 500 ppm) I. Methemoglobinemia
B.
Nitrate (> 50 ppm) II.
Brown mottling of teeth
C. Lead (> 50 ppm) III. Laxative effect
D. Fluoride (> 2 ppm) IV.
Kidney damage
Choose the correct answer from the options given below :
(a) A-IV, B-I, c-n, D-m {b) A-ril, B-I, C-IV, D-II
(c) A-II, B-IV, C-I, D-IU (£0 A-II, B-IV, C-III, D-I (JEE Main 2022)

w
09 Matrix-Match Type Questions

F lo
Match the entries of column I with appropriate entries of column II. Each entry in column I may have
one or more than one correct option from column II. If the correct matches are A-p, s ; B-r ; C-p, q ;

ee
D’S, then the correctly bubbled 4x4 matrix should be as follows :

Fr
p q
r

A
©I© o
©0 0:

for
B
ur
c E S. o 0:

D
©© 0
s
ook
Yo
53. Column I (Effeci) Column II (Gases responsible)
(A) Acid rain
ip) CO2
eB

(B) Greenhouse effect iq) SO2


(C) Classical smog (r) NO2
(D) Photochemical smog is) CH4
r
ad
ou

A B C D

19 Integer Type Questions © © ©©


© ©o ©
Y

DIRECTIONS. The answer to each of the following questions is a single digit


integer, ranging from 0 to 9. If the correct answers to the question numbers
® © ©©
Re
nd

A, B, C and D (say) are 4, 0, 9 and 2 respectively, then the correct darkening


of bubbles should be as shown on the side : © ®® ®
Fi

54. CO is a pollutant produced due to incomplete combustion of butane. One mole of © ©© ®


butane requires 6-5 moles of O2 for complete combustion. If 6 moles of oxygen © © ©©
are available, then number of moles of CO produced will be
55. In haemoglobin, CO and not O2 links to Fe (if both CO and O2 are present) because © ® ©©
CO is a stronger ligand than O2. The number of groups to which Fe is coordinated (z)Q(D(D
other than vacant site for CO in haemoglobin is © ©© ©
© ®© ©
ANSWERS
52. (b) S2>. {A-q.r ; B-p,s ; C-q ; D-r) 54.(1) 55.(5)
14/47
ENVIRONMENTAL CHEMISTRY

VI. Assertion-Reason Type Questions


f YPE I

DIRECTIONS. Each question given below contains STATEMENT-1 (Assertion) and STATEMENT-2
(Reason). It has four choices (a), (b), (c) and (d) out of which ONLY ONE is correct. Choose the option as
under:

(a) Statement-1 is True, Statement-2 is True ; Statement-2 is a correct explanation for Statement-1
(b) Statement-1 is True, Statement-2 is True ; Statement-2 is not a correct explanation for Statement-1
(c) Statement-1 is TVue, Statement-2 is False
(d) Statement-1 is False, Statement-2 is True

w
56. Statement-1. Photochemical smog is a combination of photons with smoke and fog.
Statement-2. Photochemical smog is formed in the troposphere.

F lo
57. Statement-1. CO and NO both combine with haemoglobin.
Statement-2. Both have equal affinity for haemoglobin.
TYPE II

e
For the questions given below choose the correct option out of the four options given below :

Fre
(c) Both statement-1 and statement-2 are true, {b) Both statement-1 and statement-2 are false.
(c) Statement 1 is true but statement 2 is false. {d) Statement 1 is false but statement 2 is true.

for
58. Statement-1. The pH of rain water is normally ~ 5-6.
(JEE Main 2021)
Statement-2. If the pH of rain water drops below 5-6, it is called acid rain.
r
59. Statement-1. Non-biodegradable wastes are generated by thermal power plants.
(JEE Main 2021)
You
Statement-2. Biodegradable detergents lead to eutrophication.
s
60. Statement-1. Classical smog occurs in cool humid climate. It is a reducing mixture of smoke, fog and
ook

sulphur dioxide.
Statement-2. Photochemical smog has components ozone, nitric oxide, acrolein, formaldehyde, PAN etc.
eB

(JEE Main 2022)


TYPE III
our

of Assertion (A) is given followed by a


ad

DIRECTIONS. In each of the following questions, a statement


corresponding statement of Reason (R) just below it. Of the statements, mark the correct answer as
(a) If both A and R are true and R Is the correct explanation of A.
(b) If both A and R are true but R is not the correct explanation of A.
Y

(c) If A is true but R is false,


Re

(d) If both A and R are false.


d
Fin

61. Assertion. The temperature in the stratosphere increases with altitude.


Reason. Ozone present absorbs the ultraviolet radiation.
62. Assertion. For greenhouse effect, presence of green plants is essential.
Reason. Chlorophyll of the green plants causes greenhouse effect. (AUMS 2015)
63. Assertion. Photochemical smog does not involve the condensation of any kind of fog on smoke particles
and hence the word ‘smog’ is a misnomer
Reason. Photochemical smog is due to photochemical oxidation of SOt to SO3 which dissolves in moisture
of the air to form sulphurous acid which is injurious to health.
ANSWERS

56. id) 57. (c) 58. (a) 59. (r) 60. {a) 61. {a) 62. (</) 63. (c)
14/48
New Course Chemistry fxnprsrm
64. Assertion. London smog is oxidising in nature.
Reason. London smog contains O3, NO^ and hydrocarbons. (AHMS 2009)
65. Assertion. CO and NO both combine with haemoglobin.
Reason. NO has more affinity than CO towards haemoglobi n.
(ARMS 2010)
66. Assertion. The catalytic converter in the car’s exhaust system converts polluting exhaust gases into non-
toxic gases.
Reason. Catalytjc converter contains a mixture of transition metals and their oxides embedded in the inner
support.
(ARMS 2011)
67. Assertion. Polluted water may have a value of BOD of the order of 17 ppm.
Reason. BOD is a measure of oxygen required to oxidize both the biodegradable and non-biodegradable
organic material in water. 2022)

w
F lo
For Difficult Questions

e
Multiple Choice Questions (with one correct Answer)

Fre
2. N2O5 is not a common pollutant introduced into the

for
atmosphere both due to natural and human activity. Cl(^) + 03(g) ClO(g) + O2 (g)...(2)
Chlorine monoxide
4. Microorganisms present in the soil act as a sink
for CO.
CIO(g) + 0(g) ^ Cl (g) + O2 (g)
r
...(3)
6. CO2 gas causes global warming.
You
Here, chlorine monoxide is formed in 2nd step
oks

9. (d) is false as London smog is reducing in nature. and not chlorine dioxide radicals. Hence, statement
10. N2 has no role in the formation of photochemical
eBo

(a) is not true.


smog.
22. Refer to page 14/19.
11. NO, NO2, O3 and hydrocarbons are the species 26. Refer to page 14/17, reactions (vi) and (vii).
responsible for photochemical smog.
28. Presence of particulate matter in polluted air
ad
our

14. Sulphur dioxide is not a greenhouse gas.


catalyzes the oxidation of SO2 to SO3.
15. (a) is wrong because ozone layer docs not permit Particulate matter
ultraviolet radiation from the sun to reach the earth. 2S02(g) + 02(g) > 2 SO3 (g)
catalyst
17. Correct statement is : The carboxyhaemoglobin is
- 300 times more stable than oxyhaemoglobin. This reaction can also be promoted by O3 and
Re
dY

H2O2 as
18. Acid rain reacts with marble (CaC03) of Taj Mahal
causing damage to it. Promoter
Fin

S02(g) +03(g) SO3 (g) + O2 (g)


CaC03 + H2SO4 ^ CaS04 + CO2 + H2O Promoter
19. CO2, H2O vapours, CFCs and O3 cause SO2 (g) + H2O2 (/) ■>

greenhouse effect. [SO3 (g) + H2O (/)] ^ H2SO4 (ag)


20. In the sirato.sphere, the reactions that occur are 29. Clean water has BOD value less than 5 ppm
uv whereas highly polluted water has BOD value of
CF2CI2 (g) ^ CI(g) + CF,Cl(g) ...(I)
17 ppm or more.

ANSWERl

64. id) 65. (/)) 66. (u) 67. (c)


14/49
ENVIRONMENTAL CHEMISTRY

37. Eutrophication is the process in which nutrient


For Difficult Questions enriched bodies support a dense plant population
which kills animal life by depriving of oxygen and
32. In drinking water, maximum permissible concen results subsequently in loss of biodiversity.
tration (tolerable limits) of different chemicals are
38. DDT and BHC are not much soluble in water.
Fluoride (1 ppm), lead (50 ppb), nitrate (50 ppm)
and iron (0-2 ppm). As the given sample of water 41. is green reagent for dying clothes as it does
contain higher concentration ot nitrate ions not produce any hazardous substance.
(100 ppm) than the permissible limit of 50 ppm, 43. 3 Ca3(P04)2.Cap2 contains Ca^^, and .
therefore, given sample of water is unsuitable for 44. Fish dies in water bodies polluted by sewage or
drinking. Excess of nitrate ions causes a disease other pollutants due to decrease in dissolved
called methemoglobinemia (Refer to page 14/29). oxygen (D.O.) value below 6 ppm.
33. Acceptable level of F“ is upto I ppm. NO^ is 45. Calcination is heating of concentrated ore in the
upto 50 ppm iind soj" ‘s upio 500 ppm absence (or limited supply) of air, mainly used for

w
carbonate ores.
F" concentration in water sample is 10 ppm which

F lo
is higher than the permissible level. Therefore, only Metal carbonates ^ Metal oxide
F“ makes the water sample unsuitable for drinking.
34. Tooth enamel is mostly hydroxyapatite
+
CO2 t
(causes globaJ warming)
[3 Ca3(P04)2. Ca(OH)2l. F" ions convert this into
much harder fluorapatite [3 Ca3(P04)2.CaF2] Roasting is heating of concentrated ore in a regulai

ree
[3 Ca3(P04)2 - Ca(OH)2l + 2 > supply of air, mainly used for sulphide ores.

for F
[3 Ca3(P04>2 . CaF2] + 2 OH- Metal sulphide + O2 Metal oxide
35. The amount of oxygen required by bacteria to
break down the organic matter present in a certain -I-
SO2 T
volume of a sample of water is called biochemical (causes acid rain)

oxygen demand (BOD).


Your
ks

HI Multiple Choice Questions (with One or More than One Correct Answers)
eBoo

46. Ozone layer is depleted by NO as well as CFCs. pungent smell and is suffocating in nature.
(d) is wrong because acid rain contains 60-70%
ad

47. (c) is wrong because NO is colourless and


our

odourless whereas NO-) is reddish brown having H2SO4 whereas HNO3 is 30-40% and least is HCl.

09 Matching Type Questions


Re

51. Microorganisms — Domestic sewage


Y

Plant nutrients > Chemical fertiliz.ers


Find

Toxic heavy metals > Chemical factory


Sediment >: Strip mining (It is the, practice of mining a seam of mineral by first removing a long strip of
overlying soil and rock).

Integer Type Questions

54. C4H{o + 6-5 O2 —> 4 CO2 + 5 H2O


55. In haemoglobin. Fe is coordinated to five groups
and sixth site is free to which CO links if present.
C4H,o + 6 03- ■> 3 COo -I- CO + 5 H-,0

l
14/50
New Course Chemistry (XI)E siau]
nai(^'r^rr^>-! ● r. c^Kz;:!i7rrixs©
For Difficult Questions

VI.
Assertion-Reason Type Questions
56. Correct Statement-1. Photochemical smog is due 63. Correct R. Photochemical smog is due to reaction
to photochemical reaction taking place when air of NO2 with hydrocarbons present in the air.
contains NO2 and hydrocarbons.
57. Correct Statement-2. NO has about 1500 limes 64. Correct A. London smog contains SO2 and carbon
and hence it is reducing in nature.
greater affinity than CO for haemoglobin (but
fortunately, it is unable to enter the blood stream Correct R. London smog contains SO2 and carbon
from the atmosphere) particulates.

59. Correct Statement-2.Biodegrddable detergents do 65. Both A and R are correct but R is not the correct
explanation of A.

F low
not contain phosphate salts which may lead to
eutrophication. 66. R is the correct explanation of A.
62. Correct A. Greenhouse affect is due to presence 67. Correct R. BOD is a measure of amount of oxygen
CO2 and water vapour near the earth’s surface. required by bacteria to break down the organic
Correct R. CO2 and H2O vapour present near the matter present in a certain volume of the sample
earth’s surface cause greenhouse effect. of water.

e
for Fr
Your
eBo ks
ad
our
Re
Find Y

'J
1
APPENDIX

IMPORTANT NAME

ow
REACTIONS AND PROCESSES
(In Alphabetical Order)

e
re
Fl
Q Birch reduction

F
Reduction of unsaturated compounds (alkynes and arenes) with active metals such as Li, Na, K, etc. in
liquid ammonia is called Birch reduction. The product
ur formed, however depends upon the nature of the
starting material. For example.

r
CH H

fo
Na/liq. NH3 ^ C=C
CH3—C = C —CH3 196K
But-2-yne ks H
fram-But-2-ene
CH3
Yo
This reduction is stereoselective and gives only the trans- addition product. In arenes, 1, 4-addition of
oo
hydrogen atoms occurs. For example.
H H
B

Na/liq, NH3 ^
e

196 K
ur

Benzene
H H
ad

Cyclohexa* 1,4-diene
Yo

0 Corey-House reaction
It is an excellent method for the synthesis of both symmetrical and unsymmetrical alkanes (which cannot
d

be prepared by Wurtz reaction) and higher alkanes. In this reaction , an alkyl halide is first reacted with
Re

lithium metal in dry ether to form alkyl lithium which then reacts with cuprous iodide to form lithium
in

dialkylcuprate called the Gilman reagent. The dialkylcuprate on subsequent reaction with the same or different
alkyl halide gives the corresponding symmetrical or unsymmetrical alkane in fairly good yield.
F

R—X + 2Li Dry ether ^ RLi + LiX

Alkyl halide Alkyl lithium


Dry ether
2RLi + Cul R2CuLi + Lil
Lithium dialkylcuprate

Dry ether
R2CuLi + R—X ^ R_R' + RCu H- LiX (R. R' may be same or different.)
Alkane Alkylcopper

A/1
A/2
‘P>teuUefi,'4i. New Course Chemistry (XI)n sOB
For example,

CH CH
CH—Br
Li
CH—Li
Cul (CH3)2CH>^ Cu Li""
Dry ether
CH3‘ CH3 (CH3)2CH
Isopropyl bromide Isopropyllithium
Lithium disopropylciiprate

w
CH
CH3CH2-Br
Dry ether CH—CH2CH3 + (CH3)2CHCu + LiBr
CH3‘ Isopropylcopper
Isopentate

o
Q| Diels-Aider eraction

e
It involves the addition of a conjugated diene {An^electron system) to an unsaturated compound (alkene

re
or alkyne) containing an electron-withdrawing group {In-electron system) usually called dienophile (diene

Frl
loving) to form six-membered cyclic alkenes {usually called Diels Alder adducts). These, reactions are

F
commonly referred to as [4 + 2J cycloaddition reactions since in these reactions, a 4n-electron system adds
to a 2rt-eIectron system via a six-membered cyclic transition state. Diels-Alder reactions normally do not
need any catalysts and generally occur on heating. For example,
ou
or
CH2
.CH2. N.,

kfs
5
CH
CH2 A CH CH2
-I- 4 6
>
CH CH—CHO CH CH\ 3
oo
CHO CHO
Acrolein % 2
CH2 CH2 Cyclohcx-3-ene-l-
Y
Buta-I, 3-diene Six-membered carbaldehydc
B

and cyclic transition


state
re

X ♦C —CO2CH3 CO2CH3
6
C02CH3
+ > 5
oYu

C —CO2CH3 4
ad

Buta-1,3-diene Dimethyl CO2CH3 r - CO2CH3


acetylene dicarboxylate Six-membered
Dimethyl cyclohexa-I, 4-diene-
cyclic transition 1. 2-dicarboxylate
d

state

Q Fittig reaction
in
Re

This rection is a useful variant of Wurtz reaction. It involves the reaction between two molecules of an
aryl hahde with sodium metal in presence of dry ether to form a diaryl. For example,
F

Dry ether
Cl + 2 Na + Cl >
+ 2 NaCl
A

Chlorobenzene {two molecules) Diphenyl or Biphenyl


Friedel-Crafts reaction

This reaction is used for introducing an alkyl or an acyl group into an aromatic compound in presence of
a Uwis acid catalyst. The most commonly used Lewis acid catalyst is anhydrous AlCL while other catalysts
which have been used are BF3, FeCl3, SnC^, etc. ^
{a) Friedei-Crafts alkylation. Benzene and other aromatic compounds react with alkyl halides i in

presence of anhydrous aluminium chloride to form alkylbenzenes. For example.


IMPORTANT NAME REACTIONS AND PROCESSES

CH3

Anhyd. AICI3 ^
Q +
CH3—Cl
Methyl chloride
A Q + HCl

Benzene
Toluene

w
CH2CH3

Anhyd. AICI3
Q +
CH3CH2 —Br
Ethyl bromide
A Q + HBr

Benzene Ethylbenzene

o
It is not always possible to stop the reaction at the monoalkylation stage due to +I-effect of the alkyl

e
groups. In other words, there is always a tendency to over-alkylate, particularly with methyl and ethyl groups.

re
For example,

Frl
CH3 CH3

F
CH3CI CH3CI

Q Anhyd. A!Cl3, ► Q Anhyd. AICI3, ^ O + o- and p-


(s 20% each)
ou CH3

or
A A
Benzene Toluene
m-Xylene
(= 60%)

kfs
It is surprising that although CH3- group is 0, p-directing further alkylation of toluene gives the
thermodynamically more stable m-xylene as the major product.
The Friedel-Crafts alkylation of benzene with olefins and alcohols is usually carried out in presence of
oo
protonic acids such as HF, H2SO4 or H3PO4. For example
Y
CH3
eB

H3PO4 ^
Qj + CH3-CH=CH2
Propene
A
CH

CH3
Benzene
Isopropylbenzene or Cumene
ur

ib) Friedel-Crafts acylation. Benzene and other aromatic compounds react with acid chlorides or
oY

anhydrides in presence of anhyd. AICI3 to form ketones. For example,


ad

Anhyd. AICI3
+
CH3COCI A
COCH3 + HCl
d

Acetyl chloride
Benzene Acetophenone
in

Anhyd. A1C13
Re

{CH3C0)20 A
> COCH3 + CH3COOH
Acetic acid
Acetic anhydride
F

Benzene Acetophenone

Anhyd. AICI3
+ COCl CO + HCl
A

Benzene Benzoyl chloride Benzophenone

Unlike F.C. alkylation, F.C. acylation stops at the monoacylation stage due to electron-withdrawing
effect of the acyl group.
Q Kolbe’s electrolytic reaction
This reaction is used to prepare some alkanes, alkenes and alkynes by electrolysis of aqueous solution
of sodium or potassium salt of suitable acids. For example.
A/4
‘P>tad€€^‘4i. New Course Chemistry (XI)E siau
(a) Ethane is produced when an aqueous solution of potassium acetate is electrolysed.
2 CH3COOK > 2 CH3COO- + 2 K+ {Ionization)
Pot. acetate

2H2O ^ ± 20H- + 2H+


{Ionization)
At anode :
2CH3COO- - 2 e- ^ 2 CH3COO ^ CH3—CH3 + 2CO2
{mstable) Ethane

At cathode : Both K-^ and are present but H-" ions are preferentially reduced due to their higher
electrode potential.
2 H+ + 2 e-
>2H- >H2

w
{h) Ethylene is produced when potassium salt of succinic acid is electrolysed.
CH2COOK CH.COO-
1I ^

F lo
> + 2K-" {Ionization)
CH2COOK CH2COO-
Pot. succinate

ee
2H2O ^ i 20H- + 2H+
{Ionization)

Fr
CHXOO- CH.COO
1 2 CH2
At anode : \ -le- ■> I “ + 2CO2
CH2COO- CH2COO CH2

for
{unstable)
ur
Ethylene
At cathode : H2 is produced as above,
2 + 2 e-
■> H2
s
(c) Acetylene is produced when potassium maleate or fumarate is electrolysed.
ok
Yo
CHCOOK CHCOO"
o

■> + 2K+ {Ionization)


eB

CHCOOK CHCOO”
Pot. maleate

2H2O 20H- + 2H+


{Ionization)
r
ou
ad

CHCOO~ CHCOO CH
At anode : - 2e-
CHCOO
■>
CH
+ 2CO2
CHCOO"
Y

(unsfable) Acetylene
At cathode : H2 is produced as above.
Re
nd

Q Sabatier and Senderens reduction


The reduction of unsaturated hydrocarbons to the corresponding saturated hydrocarbons with hydroge
Fi

in presence of nickel as catalyst at 523 - 573 K is called Sabatier and Senderens reduction. For example,
Ni
CH2 = CH2 + H2 CH3—CH3
523-573 K
Ethylene Ethane
Ni
CH = CH + 2H2 523-573K
^ CH3—CH3
Acetylene Ethane

This catalytic hydrogenation is widely used in the manufacture of Vanaspati Ghee from edible vegetable
oils.
A/5
IMPORTANT NAME REACTIONS AND PROCESSES

It involves the interaction of two molecules of an alkyl halide (preferably bromide or iodide) with
metallic sodium in presence of dry ether to form symmetrical alkanes containing double the number of carbon
atoms present in the alkyl halide. For example.
Dry ether
R—X + 2Na + X—R - ^ R—R + 2NaX

w
Alkyl halide Alkane
Dry ether
e.g.y CH3—Br + 2Na + Br—GH3 ■> CH3—CH3 + 2NaBr
Ethane
Methyl bromide
Dry ether
CH3CH2—I + 2Na + I—CH2CH3 - ^ CH3CH2—CH2CH3 + 2NaI

o
n-Butane
Ethyl iodide

e
Thus, Wurtz reaction is a convenient method for the preparation of symmetrical alkanes (R R), i.e.,

re
rFl
alkanes containing even number of carbon atoms.
However, if two different alkyl halides are used, a mixture of three alkanes is acutally obtained. For

F
example.
Dry ether
CH3—I + 2Na + I—CH2CH3 - ^ CH3—CH2CH3 + 2NaI

r
Methyl iodide Ethyl iodide Propane
ou Dry ether

fo
CH3—I + 2Na +1—CH3 ^ CH3—CH3 + 2NaI
Methyl iodide

CH3CH2—I + 2Na + I—CH2CH3


Dry ether
ks Ethane

^ CH3CH2—CH2CH3 + 2NaI
oo
n-Butane
Ethyl iodide
The boiling points of these alkanes are very close and hence cannot be separated easily by fractional
Y
distillation. That is why Wurtz reaction is only useful for the preparation ofsymmetrical alkanes and not for
eB

the preparation of unsymmetrical alkanes, i.e., alkanes containing odd number of carbon atoms.
r

This reaction is a variant of Wurtz reaction and is used for preparing homologues of benzene by
heating a mixture of an aryl halide and an alkyl halide with metallic sodium in presence of dry ether. For
ou
Y
ad

example.

(0
<o>- Br + 2Na + Br—CH3 Dry ether ^
A
CH3 + 2NaBr
d

Methyl bromide
Bromobenzene Toluene
in
Re

Biphenyl and ethane are obtained as by-products

<o>- <0>“ CH2CH3 + 2NaBr


F

iii) Br + 2Na +Br—CH2CH3


Ethyl bromide
Bromobenzene Ethylbenzene

Biphenyl and n-butane are obtained as by-products.


During Wurtz-Fittig reaction, the rearrangement of the alkyl chain does not occur. Therefore, this reaction
is preferred to Friedel-Crafts alkylation reaction for introducing long n-side chains in the benzene ring. For
example.

CgHgBr + 2Na + BrCH2CH2CH2CH3 A


C6H5CH2CH2CH2CH3 + 2 NaBr
Bromobenzene n-Butyl bromide n-Butylbenzene
2
APPENDIX

w
DISTINCTION BETWEEN
PAIRS OF COMPOUNDS

o
e
re
Q Methane (CH4) and acetylene (CjHj)

rFl
Methane is a saturated hydrocarbon while acetylene is an unsaturated hydrocarbon. These, can be

F
distinguished by the following tests :
(£() Methane being a saturated hydrocarbon does not decolourize Br2 in CCI4 as well as a cold dilute
alkaline solution of KMn04 {Baeyer's reagent) while acetylene being an unsaturated hydrocarbon gives

r
both these tests.
ou
fo
CH4
Methane
BT2/CC14
^ No action CH4
Methane
ks
Cold dil. alk. KM11O4 soln.
(Baeyer's reagent)
No action
oo
Br Br
CCI4 Br2/CCl4
Y
CH = CH + Bt2 - CH = CH H—C—C—H
eB

Acetylene (orange) I I
Br Br Br Br
1, 2-Dibromoethene 1. 1.2, 2-Tetrabromoethane
r

(colourless) (colourless)
ou
Y
ad

OH OH
HjO + O I -2H2O CH = 0
CH H CH + H2O + O CH = CH CH—CH -> I
Acetylene CH = 0
d

(From KMnO^) OH OH OH OH Glyoxa]


in

(unstable)
Re

(colourless)

(b) Acetylene being a terminal alkyne gives a white ppt. of disilver acetylide with ammoniacal silver
F

nitrate solution (Tollens’ reagent) and a red ppt. of dicopper acetylide with an ammoniacal solution of cuprous
chloride while methane does not give these reactions.
HC = CH + 2 [Ag(NH3)2]^OH- Ag—C s C—Ag + 4 NH3 + 2 H2O
Acetyiene Tollens’ reagent Disilver acetylide
(white ppt.)
HC^CH + 2[Cu(NH3)2r OH- ^ Cu—C = C—Cu + 4 NH3 + 2 HjO
Acetylene Dicopper acetylide
(red ppt.)

A/6
A/7
DISTINCTION BETWEEN PAIRS OF COMPOUNDS

Q llUqrleiie amt acetylrae


Both ethylene and acetylene being unsaturated hydrocarbons decolourise Bf2 in CCI4 solution and a
cold dilute alkaline solution of KMn04 {Baeyer’s reagent). These can. however, be distinguished by the
following tests,
(a) Cold cone. H2SO4 test When ethylene is passed through cold cone. H2SO4, it dissolves due to the
formation of ethyl hydrogen sulphate but acetylene does not dissolve.
CH2 = CH2 + H2SO4 (cone.) CH3—CH2—OSO3H

w
Ethylene Ethyl hydrogen sulphate
Cold cone. H2SO4
CH = CH ^ No action

Acetylene

Flo
(b) Ammoniacal silver nitrate test With ammoniacal silver nitrate solution (Tollens' reagentX acetylene
gives a white ppt. of disilver acetylide but ethylene does not

e
HCsCH + 2[Ag(NH3)2rOH- > Ag—C = C—Ag + 4NH3 + 2H2O

re
Acetylene Tol lens’ reagent Disilver acetylide
(white ppt.)

F
Tollens’ reagent
CH2 = CH2 ^ No action
ur
or
Ethylene
(c) Ammoniacal cuprous chloride test With an ammoniacal solution of cuprous chloride, acetylene
gives a red ppt. of dicopper acetylide while ethylene does not.
HCsCH + 2[Cu(NH3)2f OH" ■> Cu—CsC—Cu + 4NH3 + 2H2O
k sf
Yo
Acetylene Dicopper acetylide
oo
(red ppt.)
Ammoniacal cuprous chloride solution
eB

^ No action
CH2 = CH2

Both 1-butyne and 2-butyne are alkynes. Whereas 1-butyne is a terminal alkyne, 2-butyne is a non
ur

terminal alkyne. These can be distinguished by the following two tests :


ad

(a) With ammoniacal silver nitrate solution (Tollens’ reagent^ 1-butyne gives a white ppt of silver
Yo

1-butynide while 2-butyne does not react.


CH3CH2C3CH + [Ag(NH3)2]-"OH- CH3CH2C = CAg + H2O + 2NH3
l-Butyne Tollens’ reagent Silver 1-butynide
d
Re

(Terminal alkyne) (white ppt.)


in

No reaction
CH3—C s C—CH3 + [Ag(NH3)2r OH-
F

2-Butyne Tollens’ reagent


(Non-terminal alkyne)
(b) Similarly, with ammoniacal cuprous chloride solution, 1-butyne gives a red ppt. of copper
1-butynide while 2-butyne does not react.
CH3CH2CSCH + [Cu(NH3)2fOH- ■> CH3CH2CsC—Cu + H2O + 2NH3
l-Butyne Copper 1-butynide
(red. ppt.)
●> No reaction.
CH3—C = C—CH3 + [Cu(NH3)2l+ OH-
2-Butyne
A/8
New Course Chemistry (XI)EEIHH
Q Styrene and phenylacetylene
Phenylacetylene being a terminal aikyne will give white ppt. with ammoniacal AgNO^ solution, (Tollens
reagent) but styrene being an alkene will not
CgHjC^CH + [Ag(NH3)2rOH- 4
CgHjC s CAg + 2 NH3 + H2O
Phenylacetylene Tollens’ reagent Silver phenylacetylide
(white ppt.)
Tollens' reagent
C6H5CH = CH2 ^ No action

w
Styrene

Similarly, with ammoniacal cuprous chloride phenylacetylene gives red ppt. of copper phenylacetylide
but styrene does not.
Cyclohexane and cydohexcne

Flo
Cyclohexene is an alkene, i.e., it contains a double bond. Therefore, it decolourises the orange colour of
Br2 in CCI4 solution by forming the addition product. In contrast, cyclohexane is a saturated compound. As

e
such, it does not give this test since it does not contain a double bond.

re
CCI4 ^

F
+
Br2 H Br
(orange)
Cyclohcxene
Br H
ur
r
2-DibromocycIohexane

fo
(colourless)
Bn/CCL
^ ►
ks
No action
Yo
Cyclohexane
oo
A cold dilute alkaline solution of KMn04 (Baeyer’s reagent) can also be used to distinguish between
these two compounds. Add a few drops of Baeyer’s reagent to each compound and shake. Cyclohexene
B

decolourises the pink colour of KMn04 solution, whereas cyclohexane does not.
re

+ H2O + O > H H

From KMn04
u

Cyclohcxene
ad

OH OH
Yo

m-Cyclohexane-l, 2-diol
(colourless)

KMn04 solution
No reaction
d

(Baeyer's reagent)
Re
in

Cyclohexane

Benzene and cyclohexene


F

Both benzene and cyclohexene are unsaturated hydrocarbons but because of extra stability of benzene
ring due to resonance, it behaves like a saturated compound. Thus, benzene though unsaturated does not

decolourise the orange colour of Br2 in CCl4 solution and a cold alkaline solution of KMn04 {Baeyer's
reagent). But cyclohexene, on the other hand, gives both these tests. For reactions. Refer to distinction 5
given above.
Benzene and ethylene
Both benzene and ethylene are unsaturated hydrocarbons but because of extra stability of the benzene
ring due to resonance, it behaves like a saturated compound. Thus, benzene though highly unsaturated behaves
like a staturated compound and hence does not decolourize the orange colour of Br2 in CCI4 and a pink
colour of a cold alkaline solution of KMnO^ {Baeyer’s reagent) but ethylene gives both these tests.
A/9
DISTINCTION BETWEEN PAIRS OF COMPOUNDS

CCI4
CH2=CH2 + Br2 > Br—CH^—CH2—Br
Ethylene {orange) I, 2-Dibromoethane
(colourless)

CH2=CH2 + H2O + O, > HO —CH2 —CH2 —OH


Ethylene Ethylene glycol
From KM11O4
(colourless)

w
(;) Br2/CCl4
► No action.
or Baeyer's reagent
Benzene

Benzene and acetylene

o
Acetylene gives the following two tests but benzene does not.

e
(i) Ammonical silver nitrate test. Refer to distinction 2, page A/7,

re
(li) Ammonical cuprous chloride test. Refer to distinction 2, page A/7.

rFl
CT Buta-l, 3-diene and but-l-yne

F
But-1 -yne is a terminal alkyne while buta-1, 3-diene is a conjugated diene. These can be distinguished
by the following tests ;
(/) With ammoniacal silver nitrate solution (Tollens’ reagent), but-l-yne gives a white ppt. of silver but-

r
1-ynide while buta-1,3-diene does not
ou
fo
CH3CH2C = CH + [Ag(NH3)2fOH ^ CH3CH2C = CAg + H2O + 2 NH3
But-l-yne
(Terminal alkyne)
Tollens' reagent

CH2 = CH—CH = CH2 + [Ag(NH3)2]+ OH


ks
Silver but- 1-ynide
(white ppt.)
-> No reaction
oo
Buta-l, 3-diene Tollens' reagent
Y
(ii) Similarly, with ammoniacal cuprous chloride solution, but-l-yne gives red ppt. of copper but-1-
eB

ynide while buta-1,3-diene does not.


CH3CH2C = CH -I- [Cu (NH3)2]'-] OH- 4 CH3CH2C s CCu -F H2O -F 2 NH3
But-1- yne Copper but-1-ynide
r

(red ppt.)
ou

CH2 = CH—CH = CH2 -F |Cu(NH3)2rOH" No reaction


Y
ad

Buta-1. 3-diene

PQ Buta-l, 3-diene and butane


d

Buta-l, 3-diene being an unsaturated hydrocarbon decolourizes Br2/CCl4 as well as cold dilute alkaline
KM11O4 solution (Baeyer's reagent) while butane being a saturated hydrocarbon does not respond to these
in
Re

tests.

4 3 2 1
CCI4
F

CH2 = CH—CH = CH, + Br, 1.4-Addition


> Br—CH2—CH=CH—CH2 —Br
Buta-l. 3- diene (orange) I, 4-Dibromobut-2- ene
(colourless)

4 3 2 I
Baeyer’s
CH2 = CH—CH = CH2 + H20-F0 reagent
CH2=CH—CH—CH2
Buta-l, 3- diene (From KMn04) OH OH
But-3-en-l. 2-dio!

Br2/CCl4
■> No reaction
CH3CH2CH2CH3 or Baeyer's reagent
n-Butane
APPENDIX
3

ow
SOME TYPICAL CONVERSIONS

e
re
Q Methane into ethane

rFl
F
cu Na, Dry ether
CH4 ^ CH3—Cl CH3—CH3
Sunlight (Wurtz reaction)
Methane Methyl chloride Ethane

r
Ethane to butane
ou
o
Cl. Na, Dry ether
CH3CH3
Ethane
Sunlight
4
CH3CH2CI
Ethyl chloride ksf
(Wurtz reaction) » CH3CH2—CH2CH3
Butane
oo
1-Chlorobutane to n^octane
Y
2 Na. Dry ether
B

CH3CH2CH2CH2—Cl (-2NaCl), Wurtz reaction CH3(CH2)6CH3


1-Chlorobutane n-Octane
re

Q Ethane to propane
CI2
oYu

Li Cul CH3—I
CH3CH3 ^ CH3CH2CI ^ CH^CH2U > (CH3CH2)2CuLi - ■>
ad

Sunlight Dry ether ( Corey- House reaction)


Ethane Ethyl chloride Ethyllithium Lithium diethylcuprate
CH3CH2CH3 + CH3CH2CU + Lil
d

Propane Ethylcopper
in

Propane to ethane
Re

CI2 KOH (ale.), A


CH3CH2CH3 [ CH3CH2CH2CI + CH3—CHCl—CH3]
F

●» ■>
Sunlight (Dehydrohalogenation)
Propane 1 -Chloropiopane 2-ChIoropropane
HBr, peroxide KOH A
CH3CH = CH2 ^ CH3CH2CH2Br ●> CH3CH2CH2OH
(Anti-Mark, addn.)
Propene 1-Bromopropane Propan-l-ol

K.Cr207/H2S0 NaOH
S CH3CH2COOH
NaOH+ CaO, 630 K
(Oxidation) ^ CH3CH2COONa ^ CH3—CH3
{Decarboxylation)
Propanoic acid Sod. propanoate Ethane

A/10
A/11
SOME TYPICAL CONVERSIONS

^ Ethane to ethylene KOH (u/c.). A


Cl,
CH3CH3 CH3CH2CI ^ CHj = CH2
Sunlight (Dehydrochlorination)
Ethane Ethyl chloride Ethylene

Q n-Propyl chloride to propylene A

CH3CH2CH2CI + KOH(«/c.) CH3CH = CH2 + KCl + H2O


^Dehydrochlorinalion)
H-Propyl chloride Propylene

low
Acetylene to ethylene
Pd. BaS04.S
HC = CH + H2 CH2 = CH2
Limllar’s catalyst
Acetylene (Partial hydrogenation) Ethylene

3 Propene to propane
Ni, 523-573 K

ee
CH3CH = CH2 + H2 - CH3—CH2—CH3
(Catalytic
Propene

rF Propane

Fr
hydrogenation)
10. Acetylene to acetaldehyde
o
Dil, H2SO4 Tautomerises
[CH2 = CHOH] y CH3—C—H

for
HCsCH + HOH ●>
Acetylene HgS04.333K Acetaldehyde
Vinyl alcohol (unstable)
u
11. Ethyne to methane
ks
Dil. H2SO4 K2Cr,07/H2S04 NaOH
Yo
CHsCH > CH3—CHO -y CH3COOH ^ CHjCOONa
oo
HgS04.333K Ethanal
(Oxidation) Ethanoic acid Sod. acetate
Ethyne
NaOH + CaO, 630 K
B

CH4
(Deca rboxylation)
Methane
re

12. Acetylene to glyoxal


O
ou

/ \
ad

03/CH2C12 Td/H^O
HCsCH > HC — CH H—C—C—H
196K -ZnO
Y

Acetylene (Ozonizalion) (Reductive cleavage) o o


0—0
Acetylene ozonide Glyoxal
nd

Ethylene to ethylene glycol


Re

13.

CH, Alkaline, KMn04 CH,OH


II ^ 1 ^
Fi

+
H2O+O ■>

CHj (Baeyer's reagent)


CH2OH
(From alk. KMnO^)
Ethylene Ethylene glycol

14. Ethyl alcohol to ethylene


Cone, H2SO4.440K
CH3—CH2—OH ■> CH2 = CH2 + H2O
(Dehydration)
Ethyl alcohol Ethylene
15. Acetylene to ethane
H,,Ni,523-573 K
Hj, Ni, 523-573 K
HCsCH ^ H2C = CH2 (Reduction)
CH,—CH3
(Reduction) Ethane
Acetylene Ethylene
A/12
Course Chemistry (XI)Q
16.
Acetylene to dicUIoroacetylene
2CI2
HCsCH
NaNH2/liq. NH3
CCI4 CI2CH—CHCI2 -2NaCl
^ Cl—CsC—Cl
Acetylene Tetrachloroethane
Dichloroacetylene
17.
Ethyne (acetylene) to propyne
NaNH2 equiv.) CHj-Br
CH = CH > CH = C“Na+
liq. NH3,196 K -NaBr
^ CH3—CsCH
Ethyne Sod. acetylide Propyne

low
18. I Ethene to ethane
CH2 ●' Ni, 523-573 K CH
+ H2 p^3
CH2 (Reduction)
CH3
Ethene Ethane
19.
Acetylene to but-l-yne

ee
NaNH2.Uq.NH3

rF
Fr
CH = CH CH3CH2Br
> CHsC-Na+
196 K -NaBr
CH3CH2C = CH
Acetylene Sod. acetylide But-l-yne
20. Acetylene to pent-2-yne

for
First convert acetylene to but-l-yne as shown in conversion 19 above and then convert it into pent-2-
yne as follows:
u
NaNH2, liq. NH3 CH,I
ks
CH3CH2CSCH 196 K
^ CH3CH2C = C-Na+ ^ CH3CH2C = CCH3
-Nal
Yo
But-l-yne Sod. but-l-ynide Pent-2-yne
oo
Alternatively
First convert acetylene to propyne as shown in conversion 17 above and then convert into pent-2-yne as
eB

follows.

Na in liq. NH3 CH3CH,Br


CH3CSCH ^ CH3C = C"Na+ ^ > CH3—C = C—CHXH,
196K -NaBr ^ ^ 3
r

Propyne Sod. propynide Pent-2-yne


ou
ad

21. I Etihene to bromoethene


Y

CH. CH.Br . CH.


II 2 Bt2 I 2 KOH(afc.),A
II 2
CH2 CCI4 CH2Br -KBr CHBr
d
Re

Ethene 1,2-Dibromoethane Bromoethene


n

22. Propene to propyne


Fi

Br2,CCl4 NaNH2/Uq. NH3


CH3CH = CH2 CH,—CH—CH, 4 CH3—C = CH
Propene 3 I I 2 (Dehydrohalogenation)
Propyne
Br Br
1, 2- Dibromopropane
23.
Bromomethane to propanone OH
HC H C- Na+ Dil. H2S04,HgS04
CH3—Br ^ CH3—C = CH CH3—C=CH2 Tautomerises
Liq. NH3.I96K 333 K
Bromomethane Propyne (Addition of H2 O) O

CH3—C—CH3
Propanone
A/13
SOME TYPICAL CONVERSIONS

But-l-ene to but-2-ene
HBr
KOH (alc.l A
CH3CH2—CHBr—CH3 ^ CH3CH = CHCH3
CH3CH2CH = CH2 {Mark, addn.) {Saylzeff rule)
i-Bromobutanc But-2-ene {major)
But-I- ene

^3 tert'Butyi bromide to isobutyl bromide


CH3 CH3
CH3

ow
KOH/alcohol. A
HBr/poroxicIc
^ CH3—C = CH, ^ CH3—CH—CH^Br
CH3—C—Br (Dehydrohalogenation)
(Ami-Murk, addn.)
Isobutylene Isobiuyl bromide
CH3
fert-Butyl bromide
3 Acetylene to but-2-yne or CH3I to but-2-yne

e
liq. NH3

re
CH = CH + 2NaNH2 — 196 Na+'C = C-Na+ + 2NH3

rFl
K
Acetylene Disodium acetytide

F
Na'^-C = C'Na+ + 2CH3I ^ CH3—C = C—CH3 + 2NaI
Disodium acetylide (exf’t’.v.s) But-2-ync
27. Bromoetbane to cis-hex-3-ene

r
ou CH3CH2 CH^CH3

fo
H^-Pd/BaS04+S c = c
Na+"C = C"Na+
2CH3CH2—Br -y CH3CH2CSCCH2CH3 H H

Bromoetbane
liq. NH3 (-2NaBr)
3-Hexyne
ks
Lindlar’s catalyst

c/s-Hex-3-ene
oo
j^S Acetylene to cis- or trans-but-2-ene
First convert acetylene (ethyne) to but-2-yne as shown in conversion 26 above and then convert it into
Y
eB

cis- or /ran.y-but-2-enes as follows.


H
Na/liq. NH3 CH3
CH3 CH3 H2-Pd/BaS04+S C=C
c = c <r CH3—C s C—CH3 196 K H CH3
Lindlar’s caialy.st
r

H H But-2-yne
ou

frans-But-2-ene
cij-But-2-ene
Y
ad

Ethyne into butan-2-one


First convert ethyne into but-l-yne as shown in conversion 19, page A/12 and then convert it into
butan-2-one as follows:
d

OH O
in
Re

Dil. H2S04,HgS04 Tautomerises

CH3CH2C S CH 333 K ^ [CH3CH2—C = CH2_ ^ CH3CH2—c—CH3


Butan-2-one
F

But-l-yne {Addition of H2O)

mjg Ethyne to pentan-2-one


First convert ethyne to but-l-yne as shown in conversion 19, page A/12 and then convert it into pentan-
2-one as follows :

Na/liq. NH3
CH,1 Dil. H2S04,HgS04
CH3CH2C = CH 196 K
y CH3CH2C=C-Na-" -Nal
^ CH3CH2C = CCH3 333 K
■y

But-l-yne Peni-2-yne {Addition of H2O)

OH O
Tautomerises

CH3CH2CH = C—CH3_ CH3CH2CH2—c—CH3


Pentan-2-one
A/14
New Course Chemistry (XI)E mn
31.
pthylene to acetylene
CH, Br2 CH.—Br
II ^ — I ' NaNH2/liq.NH3.196K
->
CH

CH2 CCI4 CH2—Br (Liehydrobromination) CH


Ethylene Ethylene dibromide Acetylene
32.
n^Fropyl alcohol to propene
Cone. H2SO4, A
CH3—CH2—CH2—OH — 4 CH3—CH = CH2 + H20
{Dehydration)
n-Propyl alcohol Propene

w
33.
Acetylene to dichloroacetaldehyde H H
CH Cl o
HOCl I I I II
+ HO—Cl - »'CH = CHl HO—C—C—Cl ^ Cl—CH—C—H
CH (orCl2/H20)

F lo
-H2O
(or CI2/H2O) OH Cl HO Cl
Acetylene Hypochlorous (unstable) Dichloroacetaldehyde
acid

ee
34.
! to (^dofaexane

Fr
CH Raney Ni,
Red hot iron tube, 873 K 473-523 K

CH
(Cyclic polymerization) (Catalytic hydrogenation)

for
Acetylene Benzene
Cyclohexane
ur
35.
ll^l^e to m^ilylene
CH3
s
A
ok
Yo
Red hot iron tube, 873 K
3 CH3—CsCH
o

(Cyclic polymerization)
eB

Propyne
CH3"‘ CH3
Mesitylene
36.
r
ad
ou

CH

Red hot iron tube, 873 K


CH CH3
Y

3 CH3—C=C—CH3 (Cyclic polymerization)


But-2-yne
Re
nd

CH3
Hexamethylbenzene
Fi

37.
I^Acetyl^ to oxalic add
Hot alk. KMnO
HC = CH + 4[0] ^ HOOC—COOH
(Oxidation)
Acetylene Oxalic acid
38.

lii
H HCI CH, CH
HCI

CH
II " p-3
CHCl (Mark, addition)
CHCI2
Acetylene Vinyl chloride Ethylidene dichloride
or 1, 1-DichIoroethane

*-
A/15
SOME TYPICAL CONVERSIONS

39. Methane to trichloromethane (chloroform).


CI2//"'
C\2lhv CHCI3
CH2CI2
4
CH4 -HCl
-> CH3CI -HCl -HC!
Dichloromethane Trichloromethane
Methane Chloromethane

Acetic acid to methane


Soda-lime
NaOH (NaOH+CaO). A
CHjCOONa CH4
CH3COOH 4

w
-H^O (-NajCOj) Methane
Acetic acid Sod acetate
(Decarboxylation)

41. Ethane to acetylene


Bf2

Flo
Cl2/*v KOH (ale.). A
^ CH3—CH2CI ^ CH2 = CH2 BrCH2—CH2Br
CH3—CH3 -HCl -KCl,-HjO CCI4
Ethane Ethyl chloride Ethylene Ethylene dibromide

e
NaNH2/liq. NH3

re
^ CH = CH
196 K
Acetylene

F
42. Ethyl iodide to ethane
ur Zn-Cu/alcohol

or
CH3-CH2I or Red P+HI, A
^ CH3—CH3
Ethane
Ethyl iodide

43. Ethylene to ethyl alcohol


k sf
H2O. Boil
Yo
Cold
CH2 = CH2 4 CH3—CH2—OSO2OH CH3—CH2OH
oo
conc.H2S04 -H2SO4 Ethyl alcohol
Ethylene Ethyl hydrogen sulphate
eB

44. I 2-Bromopropane to 1-bromopropane and vice-versa


KOH (ale.). A HBr, peroxide
CH3—CH—CH3 > CH3—CH = CH2 ●> CH3CH2CH2—Br
(Anti-Mark,
ur

I (Dehydrobromination) 1-Bromopropane
Br
Propene addition)
ad
Yo

2-Bromopropane
KOH(fl/c.),A HBr

> CH3—CH = CH2 > CH3 CHBr—CH3


CH3CH2CH2—Br (Dehydrobromination) (Mark .addition)
Propene 2- Bromopropane
1-Bromopropane
d
Re

45. Propan-l-ol to 2-bromopropane


in

Cone. H-,S04.440 K HBr


F

CH3CH2CH2OH 4 CH3- -CH = CH2 (Mark,


^ CH3—CH—CH3
(Dehydration)
Propene addition) Br
Propan-l-ol
2- Bromopropane

^[31 But-l-ene into HCHO and C2H5CHO


O

O3/CH2CI2 / \ Z11/H2O
CH3CH2CH=CH2 ► CH3CH2CH CH2 -ZnO ► CH3CH2CHO + HCHO
196 K Fonnaldehyde
But-1 -ene (Reductive Cleavage) Propionaldehyde
(Ozonization)
o—o
But-l-ene ozonide
A/16
New Course Chemistry (XI)E 19D
47. Benzene to nitrobenzene

NO2

Q + HNO3
(Cone.)
Cone. H2SO4,323-333 K
(Nitration)
>
a + H2O
Benzene
Nitrobenzene
48.
Benzene to benzenesulphonic acid
SO3H

w
Q
330 K

Benzene
+ H2SO4
(Fuming) (Siilphonation) a + H2O

F lo
Benzenesulphonic acid
49.
Benzene to maleic anhydride
CHCO

ee
2 V2OS, 775 K ^
+ 9O2
(Oxidation)
2| + 4CO2 + 4H2O

Fr
(From air) CHCO
Maleic anhydride
50. Benzene to toluene

for
ur
CH3

Anhyd. AICI3, A
a + CH3C1 o
s
+ HCl
(FC. alkylation)
ook
Methyl chloride
Yo
Benzene
Toluene

51.
Benzene to diphenyl
eB

<D
CI2. Anhyd. AICI3 Na, Dry ether, A
310-320 K ^ Cl
Fitting reaction (-2NaCl)
r

(Nuclear halogenation)
ad
ou

Benzene Chlorobenzene
Diphenyl
52.
J^igizene ^ behz^e ^cid
Y

CH3 COOH
Re

(/) Hot alk. KMn04


nd

CH3CI, Anhyd. AICI3. A


Q (EC. alleviation) a
(oxidation)
(«) H-^/H20 ^ Q
Fi

Benzene Toluene Benzoic acid


53. Benzrae to m-nitroben^ic acid

acid conversion 52 and then convert it into m-nitrobenzoic


COOH COOH

Cone. HN03/Conc. H2SO4, A


Q (Nitration)
>
Q
Benzoic acid NO2
/w-Nitrobenzoic acid
A/17
SOME TYPICAL CONVERSIONS

54. Benzene to p-nitrobenzoic acid


COOH
CH3 CH3
Cone. HNO3
+ Cone. H2SO4 (/) tCMn04/QH ,
A o (/7) h'"/h20
{Nitration)
Toluene
NO
NO2
p-Nitrotoluene p-Nitrobenzuic acid
{major product)

w
55. Benzene to acetophenone
COCH3

F lo
Anhyd, AICI3, A ^
Q + CH3COCI {EC. acylation) Q + HCl

Acetyl chloride
Benzene Acetophenone

ee
3 Benzene to cyclohexane

Fr
+ 3H2
Ni, 473-523 K ^
{Hydrogenation)

for
Cyclohexane
ur
Benzene

57. Ethylbenzene to benzene


s
COOH
CH2CH3
ok
Yo
(/) KMn04/0H , A ^ Soda-lime (NaOH + CaO), 630 K ^ o
o

Q Q -CO2
eB

{ii) H^/H20 {Decarboxylation)


{Vigoivus oxidation) Benzoic acid
Benzene
Ethylbenzene
SS Benzene to monodeuteriobenzene
r

D
Br
ad
ou

D2O ^
Q
Br2/Fe, A
{Electrophilic Q
Mg/ether
Q -Mg(OD)Br Q
substitution) Monodeuterio
Phenylmag.
Y

Benzene Bromobenzene
benzene
bromide

^3! Styrene to phenylacetylene


Re
nd

C=CH
CH=CH2 CHBr—CH2Br
Fi

NaNH2/liq. NH3
Br?

Q Q
Q ^
CCI4
196K

Phenylacetylene
Styrene
Benzene to 3,4-dibromonitrobenzene
Br Br

Cone. HNO3 Br?,


Anhyd. FeBr? ^
Q
+ Cone, H2S04^
323-333 K ^ A
1

Benzene Bromobenzene
NO2 NO2
p-Nitrobromobenzene 3,4-Dibromonitrobenzene
APPENDIX
4

w
IDENTIFICATION OF UNKNOWN
ORGANIC COMPOUNDS/REAGENTS

F lo
ee
Q. 1. Complete the following by supplying intermediates

Fr
Mg CH3OH
(i) CH3CH2Br ether
A ^ B + C

NaOH NaOH, CaO Br^

for
(ii) CH3COOH » A ^ B 4 C
ur 630 K hv

Na Br
Na
(iii) CHjBr ether
A ^ B
hv ether
^ C
s
ok
Yo
KMnO^ NaOH + CaO
(iv) CgH5CH2CH3 - SOH, 373 K A ^ B
630 K
o
eB

Li, ether Cut (CH3 )2CHBr


(V) CH3CH2Br [A]- ^ [B] [C]

Na/liq. NH3 EtBr


(vi) MeCH2C s CH
r

^ [A] ^ [B]
ou
ad

Mg, ether CH3OH


Ans. (0 CH3CH2Br ^ CH3CH2MgBr
{Grignard reaction) ^ CH3CH3 + Mg(OCH3)Br
Y

Bromoethane
Ethylmag, Ethane (B) Meihoxymag.
bromide (A) bromide (C)
NaOH
nd
Re

NaOH + CaO, 630 K Bf2, hv


Hi) CH3COOH ^ CH3COONa
.(Decarboxylation)
^ CH4 CHjBr
Ethanoic acid Sodium ethanoate (Bromination)
Methane Bromomethane
Fi

(A) (B) (C)


Na, ether Br,, hv
(Hi) CH3Br » CH3—CH3 ^ CH3CH2—Br
Na, ether
(Wurtz reaction) (Bromination)
¥
Bromomethane Ethane (Wurtz reaction)
Bromoethane
(A) (B)

CH3CH2—CH2CH3
Butane (C)
KMnO4,K0H, 373 K NaOH + CaO, 630 K
(iv) C6H5CH2CH3 ^ CgHjCOOK
(Oxidation) (Decarboxylation)
Ethylbenzene
Pot. benzoate (A) Benzene (B)
A/18
A/19
IDENTIFICATION OF UNKNOWN ORGANIC COMPOUNDS/REAGENTS
CH
3\
CH—Br

Cid
CH3/
(v) CH3CH2Br Li.eth^
A
CH3CH2
^
—Li [(CH3CH2)2Cu]-Li+ {Corey-House reaction)
Bromoethane Ethylithium (A) Lithium diethylcuprate (B)
CH3
CH3—CH—CHj—CH3
2-Methylbutane (C)

low
Na in liq. NH3 EtBr

(yi) MeCH2C = CH ^ MeCH2C = CTNa+ -NaBr


MeCH2CsCEt
(A) Hex-3-yne (B)

Q. 2. Supply the reagents in the foUowing sequence of reactions :


(A) (B) (C)
^ CH3CHBr—CHjBr ^ CH3—CsCH

e
(i) CH3CH2CH2Br ^ CH3CH = CH2

re
O

rF (D)
^ CH3—C—CHj

F
[A] ICl
(ii) CH4 ^ CH3Br C6H5CH3 ^ CgHgCOOH

or
O
/ \
u
f
lAl [Bl
► CH2 CH2 >HCHO
(ui) CH2=CH2
ks
Yo
O O
oo
[A1 [B]
(iv) CH3—CH = CH2 ^ CH3CH2CH2Br ^ C6H5CH2CH2CH3
eB

KOHalc.,A[A] Br2.CCl4 [B]


Ans. (0 CH3CH2CH2Br » CH3CH = CH2 ^ CH3CH2Br—CH2Br
(Dehydrobromination) 1, 2-Dibromopropane
l-Bromopropane Propene
ur

O
DiI.H2S04,HgS04 [D]
ad

KOH ale., A tC]


^ CH3—C—CH3
Yo

CH3—C = CH 330 K
(Dehydrobromination) Propanone
Propyne

C6H6,anhyd.AlCl3[B] (i) KMn04, KOH. 373 K [C]


d

Br2/Av[A]
^ CH3Br ^ C6H5CH3
Re

(ii) CH4 - {F.C. alkylation) {ii) H3O+


in

Toluene
Methane
CgHjCOOH
F

Benzoic acid

P3/CH>^C12[A] / \ Zn,H20[B] ^
H

(Hi) CH2=CH2 CH2 CH2 2 /C=0


19. K ^
Ethene
O O Methanal
Ethene ozonide

HBr/ROOR [A] CfiHsMgBr


(iv) CH3—CH = CH2 ^ CH3CH2CH2Br C6H5CH2CH2CH3
Anti-Mark, {Grignard reaction)
addn. l-Bromopropane «-Propylbenzene
Propene
A/20
New Course Chemistry (XI)BJ
CgHjBr, Na [B]
Alternatively, CH3CH2CH2Br C6H5Ch,ch,ch3
1 -Bromopropane n-Propylbenzene
Q. 3. Complete the following reactions by supplying the products and the reagents.
[A] CH3CI, anhyd. AlCI
(i)CH3CH [C]
^ [B] > CgHgCHO
0^203, AI2O3 Br2/Fe Mg/ether D2O
(ii) CH3(CH2>4CH3 773 K, 10-20 atm
^ [A] ^ [B] > [C] ^ [D]

w
H20,H2S04 C6»6
(m)(CH3)2C = CH2 ^ [A] > [B]
conc.H2S04

F lo
Br Na/liquid NH
(iv)CH3CH = CH2 ^ [A] ^ [B]
Fetttbe
873 K
^ [C]

ee
CH3 CHO
Red hot iron CH3CI, anhyd.

Fr
tube [A] AICI3.A
Ans. (0 HC=CH Ci02Cl2(C)
873 K ^
Acetylene {EC. alkylation) Chromyl chloride ^
(Etard reaction)

for
Benzene
Toluene [B] Benzaldehyde
ur
Br
s
(//) CH3(CH3)4CH3-^^2^ Br2/Fe Mg/ether
ook
Yo
n-Hexane
~~A ^
Benzene [A] Bromobenzene [B]
eB

MgBr D

A D20 A
r
ad
ou

Phenylnoag. bromide [C] Deuteriobenzene (D)


Y

CH3 CH3 CH3


HjO.HjSO^ I CgHg, cone. H2SO4
Re

(Hi) CH3—C=CH2 ► CH3—C—OH


nd

{Addition ofwate,r {EC. reaction) C—CH3


2-Methylpropene
Mark, addn.)
CH3 CH3
Fi

2-Methylpropan-2-ol [A] /ert-Butylbenzene (B)

Bf2 Na/liq. NH3


(/v) CH3—CH=CH2 ■►CH3—CHBr—CH2Br
Propene {Dehydro- ^ CH3—C=CH
1,2-Dibromopropane (A) bromination) Propyne (B)

CH3

Fe tube, 873 K
{Cyclic polymerization) o
CHf CH3
Mesitylene (C)
5
APPENDIX

w
WHAT HAPPENS WHEN ?

o
e
re
0.1. tert-Butyl bromide is treated with sodium metal in dry ether. . , , , ,

rFl
Ans. Tertiary alkyl halides do not undergo Wurtz reaction but instead undergo derhydrohalogenation to

F
yield alkenes. Thus,
CH.
CH,
Na, ether
I ^
CH3—C—Br CH3—C = CH2 + HBr

r
A
ou 2-Melhylpropene

sfo
CH3 (Isobutylene)
rerf-Buiyl bromide
O 2. Benzyl bromide is treated with hydrogen in presence of Pd - C. k
Ans. Hydrogenolysis (cleavage of C—Br bond with hydrogen) occurs to form toluene
oo
Pd-C
+ HBr
C^Hs—CH2—Br + H2
Y
Toluene
Benzyl bromide
eB

O. 3.1-Butanol is heated with concentrated sulphuric acid at 443 K.


Ans. But-2-ene is obtained as the major product. First n- butyl (H carbocation ^
less stable rearranges to a more stable 2“ carbocation which .subsequently loses a proton to form but-2-ene m
accordance with Saytzeff rule.
r

a
ou

+
Y

H2SO4 ^
ad

CH3CH2CH2CH2-L-OH2J > CH3—CH2—CH—CH2


CH3CH2CH2CH2OH -H2O
1-Butanol
H

;i-Butyl carbocation (!°)


d

(less stable)
Re
in

CH—CH3 > CH3CH = CHCH3 + CH3CH2CH = CH2


CH3—CH2 (Saytzeff elimination) Bul-2-ene (80%) Biit-l-ene (20%)
F

2® Butyl carbocation
(more stable)
Q. 4. Propene is treated with BrCCIj in presence of peroxides.
Ans. In presence of peroxides, BrCC^ undergoes homolytic fission to form CCI3 radical which then
attacks the propene molecule at the CH2 group to form a 2“ alkyl free radical. This radical subsequently reacts
with BrCCl3 to complete the-addition.
A
Or
R—O O—R 2RO ; RO + Br-‘-CCl3 > ROBr + *CCl3
Homolytic fission

A/21
A/22
New Course Chemistiy fmro8T»n

r a
CH3—<Q^CH2+^CCl3 >CH3—CH—CH2CCI3 ■-CCI, * CH3—CHBr —CHj—CCI3
Br-lCCIs 3 2 1

Propene , ^ ● 1.
fl ? Uiif 1 j, , - . , . l-Tnchloro-3-bromobutane
\ **“● H2SO4 in presence of HgSO. at 330 K.
Ans. Addition of H2O occurs to form butan-2-one in each case ^
Dil. H2SO4. HgS04
CH3CH2—CSCH + H2O 330 K
■>
CH3CH2—C=CH2'
Tautomerises
■»
But-l-yne
OH
O

w
CH3CH2—C—CH3
Butan-2-one
Dil. H2SO4, HgS04

F lo
CH3—C = C—CH3 + H2O 330 K CH3—C=CHCH3‘
Tautomerises
■»
But-2-yne
OH
CH3—C—CH.CH,
J II 2 3

ee
O

Fr
Q. 6. Acetylene is treated with dilute hydrochloric acid in presence of HcCl,.
n Under these conditions, addition of H2O does not occur across the double bond. Instead Cl" beine

for
V attacks the 7C-complex formed between acetylene and Hg2+ ions This is
followed by addition of a proton to yield vinyl chloride as shown below :
our
Cl Cl
s
H—C=C—H + Hg^-" cr H* I H
ook
>H—C=C—H >H--C=^=CH
Acetylene ► H—C^^CH^
d'2+/
^Hg >H
Hg^ Hg^
Y
eB

Cl H
2+ :c=cH
our

-Hg
ad

H H
Vinyl chloride
?* (U5H5CH - CH2) is treated with HBr in presence of peroxides.
Ans. Addition occurs according to anti-Markovnikov’s rule.
Y

Peroxides
CgH5CH = CH2 + HBr
(Anti-Mark addition) U6H5CH20l2Br
Re

Styrene
Qo n is● treated
^ , with isobutyl alcohol in presence of cone.
l*Bromo-2-phenylethane
nd

. 8. Benzene H^SOj.
Ans. In presence of cone, H2SO4, isobutyl alcohol first generates a r carbocation which then undergoes
Fi

remangement by 1, 2-hydride shift to form


with benzene to form fert-butylbenzene. a more stable tcrr-butyl carbocation. This subsequently reacts
Ans.

CH3 CH3 CH3 CH3 /

CH3-C-CH20Hq^ CH3-C-CH2 1,2-Hydride


shift ► CH3—C^
C6H6 .
-H* * C—CH3
H
Isobutyl alcohol
d/
Isobutyl carbocation
CH3
(EC. alkylation)
CH3
I
/ert-Butyl- tert-Butylbenzene
(less stable) carbocation
(more stable)

You might also like